Sunteți pe pagina 1din 346

Labor Law 1

PEARANDA V BAGANGA PLYWOOD CORP.


PANGANIBAN; May 3, 2006
NATURE
Petition for review assailing the resolutions of the Court of Appeals (CA)
FACTS
- Petitioners Claims
> Petitioner Charlito Pearanda alleges that he was employed by respondent [Baganga]
with a monthly salary of P5,000.00 as Foreman/Boiler Head/Shift Engineer
> His services were terminated without the benefit of due process and valid grounds.
> He was not paid his overtime pay, premium pay for working during holidays/rest days,
night shift differentials and finally claims for payment of damages and attorneys fees
having been forced to litigate the present complaint.
- Respondents Claims
> Respondent [BPC] represented by its General Manager HUDSON CHUA, allege that
complainants separation from service was done pursuant to Art. 283 of the Labor Code. >
BPC was on temporary closure due to repair and general maintenance and it applied for
clearance with the DOLE, Regional Office No. XI to shut down and to dismiss employees.
> Pearanda was not terminated from employment much less illegally. He opted to severe
employment when he insisted payment of his separation benefits.
> Furthermore, being a managerial employee he is not entitled to overtime pay and if ever
he rendered services beyond the normal hours of work, there was no office order/or
authorization for him to do so.
- The labor arbiter ruled that there was no illegal dismissal and that petitioners complaint
was premature because he was still employed by BPC. The temporary closure of BPCs
plant did not terminate his employment.
- Nevertheless, the labor arbiter found petitioner entitled to overtime pay, premium pay for
working on rest days, and attorneys fees in the total amount of P21,257.98.
- NLRC deleted the award of overtime pay and premium pay for working on rest days for
the petitioner was not entitled to these awards because he was a managerial employee.
- CA dismissed Pearandas Petition for Certiorari and held that he failed to: 1) attach
copies of the pleadings submitted before the labor arbiter and NLRC; and 2) explain why
the filing and service of the Petition was not done by personal service.
- In its later Resolution, CA denied reconsideration on the ground that petitioner still failed
to submit the pleadings filed before the NLRC.
ISSUE
WON petitioner is entitled to overtime pay and premium pay for working on rest days
HELD
NO
- Article 82 of the Labor Code exempts managerial employees from the coverage of labor
standards. Labor standards provide the working conditions of employees, including
entitlement to overtime pay and premium pay for working on rest days.
- Under this provision, managerial employees are those whose primary duty consists of
the management of the establishment in which they are employed or of a department or
subdivision.
- The Implementing Rules of the Labor Code state that managerial employees are those
who meet the following conditions:
(1)
Their primary duty consists of the management of the establishment in which
they are employed or of a department or subdivision thereof;
(2)
They customarily and regularly direct the work of two or more
employees therein;
(3)
They have the authority to hire or fire other employees of lower
rank; or their suggestions and recommendations as to the hiring and firing
and as to the promotion or any other change of status of other employees
are given particular weight.
- The Court disagreed with the NLRCs finding that petitioner was a managerial employee.
However, petitioner was a member of the managerial staff, which also takes him out of the
coverage of labor standards. Like managerial employees, officers and members of the
managerial staff are not entitled to the provisions of law on labor standards. The
Implementing Rules of the Labor Code define members of a managerial staff as those with
the following duties and responsibilities:
(1) The primary duty consists of the performance of work directly related to
management policies of the employer;
(2) Customarily and regularly exercise discretion and independent judgment;
(3) (i) Regularly and directly assist a proprietor or a managerial employee whose
primary duty consists of the management of the establishment in which he is
employed or subdivision thereof; or (ii) execute under general supervision work along
specialized or technical lines requiring special training, experience, or knowledge; or
(iii) execute under general supervision special assignments and tasks; and

A2010

-1-

Disini

(4) who do not devote more than 20 percent of their hours worked in a workweek to
activities which are not directly and closely related to the performance of the work
described in paragraphs (1), (2), and (3) above.
- Petitioners duties and responsibilities conform to the definition of a member of a
managerial staff under the Implementing Rules. Petitioner supervised the engineering
section of the steam plant boiler. His work involved overseeing the operation of the
machines and the performance of the workers in the engineering section. This work
necessarily required the use of discretion and independent judgment to ensure the proper
functioning of the steam plant boiler. As supervisor, petitioner is deemed a member of the
managerial staff.
- Noteworthy, even petitioner admitted that he was a supervisor. In his Position Paper, he
stated that he was the foreman responsible for the operation of the boiler. The term
foreman implies that he was the representative of management over the workers and the
operation of the department. Petitioners evidence also showed that he was the
supervisor of the steam plant. His classification as supervisor is further evident from the
manner his salary was paid. He belonged to the 10% of respondents 354 employees
who were paid on a monthly basis; the others were paid only on a daily basis.
Disposition Petition was DENIED

BATONG BUHAY GOLDMINES V DELA SERNA


PURISIMA, August 6, 1999
FACTS
- Employees filed a complaint against Batong Buhay for: unpaid salaries from March 16,
1987 to present, unpaid and ECOLA differentials under Wage Order Nos. 2 and 5, unpaid
13th months pay for 1985 and 1986, and upaid vacation/sick/compensatory leave benefits.
- Labor Standards and Welfare Officers & Regional Director: Batong Buhay must pay Ty
et al. P4,818,746.40
- Regional Director directed Batong Buhay to put up a cash or surety bond otherwise a writ
of execution will be issued.
- The Special Sheriff seized three units of Peterbuilt trucks and then sold the same by
public auction. Various materials and motor vehicles were also seized on different dates
and sold at public auction.
- Batong Buhay finally posted a supersedeas bond and appealed the Order contending
that the Regional Director had no jurisdiction over the case.
- Undersec dela Serna upheld the jurisdiction of the Regional Director and annulled all the
auction sales conducted by Special Sheriff. MR denied.
- Motion for Intervention was filed by MFT Corporation and Salter Holdings Pty., Ltd. For
exclusion from annulment of the properties sold at the auction sale. Granted. MR denied.
ISSUES
1. WON the Regional Director has jurisdiction over the complaint filed by the employees
of BBGMI
2. WON the auction sales conducted by the said Special Sheriff are valid
HELD
1. YES
- The subject labor standards case of the petition arose from the visitorial and enforcement
powers by the Regional Director of DOLE.
- Labor standards refers to the minimum requirements prescribed by existing laws, rules
and regulations relating to wages, hours of work, cost of living allowance and other
monetary and welfare benefits, including occupational, safety and health standards. Labor
standards cases are governed by Article 128(b) of the Labor Code.
- Even in the absence of E.O. 111 , Regional Directors already had enforcement powers
over money claims, effective under P.D. 850, issued on December 16, 1975, which
transferred labor standards cases from the arbitration system to the enforcement system.
- E.O. No. 111 was issued on December 24, 1986 or three(3) months after the
promulgation of the Secretary of Labor's decision upholding private respondents' salary
differentials and ECOLAs on September 24, 1986. The amendment of the visitorial and
enforcement powers of the Regional Director (Article 128(b)) by said E.O. 111 reflects the
intention enunciated in Policy Instructions Nos. 6 and 37 to empower the Regional
Directors to resolve uncontested money claims in cases where an employer-employee
relationship still exists. This intention must be given weight and entitled to great respect.
- The Court would have ruled differently had the petitioner shown that subject labor
standards case is within the purview of the exception clause in Article 128 (b) of the Labor
Code. Said provision requires the concurrence of the following elements in order to divest
the Regional Director or his representatives of jurisdiction, to wit: (a) that the petitioner
(employer) contests the findings of the labor regulations officer and raises issues thereon;
(b) that in order to resolve such issues, there is a need to examine evidentiary matters;
and (c) that such matters are not verifiable in the normal course of inspection.
- Petitioner's refusal to allow the Labor Standards and Welfare Officers to conduct
inspection in the premises of their head office and the failure to file their position paper is
equivalent to a waiver of its right to contest the claims of the employees.

Labor Law 1
- This involves a labor standards case and it is in keeping with the law that "the worker
need not litigate to get what legally belongs to him, for the whole enforcement machinery
of the Department of Labor exists to insure its expeditious delivery to him free of charge.
- The present law, RA 7730, can be considered a curative statute to reinforce the
conclusion that the Regional Director has jurisdiction over the present labor standards
case.
2. NO
- As a general rule, findings of fact and conclusion of law arrived at by quasi-judicial
agencies are not to be disturbed absent any showing of grave abuse of discretion tainting
the same.
- There was grave abuse of discretion when the Undersec, without any evidentiary
support, adjudged such prices as "scandalously low". He merely relied on the self-serving
assertion by the petitioner that the value of the auctioned properties was more than the
price bid.
- The sales are null and void since on the properties of petitioner involved was constituted
a mortgage between petitioner and the Development Bank of the Philippines

CMS ESTATE, INC. V SOCIAL SECURITY SYSTEM


CUEVAS; September 28, 1984
NATURE
Appeal by the CMS Estate, Inc.
FACTS
- Petitioner is a domestic corporation organized primarily for the purpose of engaging in
real estate business. On December 1, 1952, it started doing business with only six (6)
employees.
- January 28, 1957: petitioner entered into a contract of management with one Eufracio D.
Rojas for the operation and exploitation of the forest concession. The logging operation
actually started on April 1, 1957 with four monthly-salaried employees. As of September
1, 1957, petitioner had 89 employees and laborers in the logging operation.
- December 26, 1957: petitioner revoked its contract of management with Mr. Rojas.
- August 1, 1958: petitioner became a member of the Social Security System with
respect to its real estate business. On September 6, 1958, petitioner remitted to the
System the sum of P203.13 representing the initial premium on the monthly salaries
of the employees in its logging business.
- October 9, 1958: petitioner demanded the refund of the said amount.
- On November 10, 1958, petitioner filed a petition with the Social Security Commission
praying for the determination of the effectivity date of the compulsory coverage of
petitioner's logging business.
- January 14, 1960: the instant petition was denied and petitioner was adjudged to be
subject to compulsory coverage as Sept. 1, 1957 and the Social Security System was
directed to effect such coverage of petitioner's employees in its logging and real estate
business conformably to the provisions of Rep. Act No. 1161, as amended.
- Petitioners Claim
CMS Estate, Inc. is not yet subject to compulsory coverage with respect to its logging
business because it does not have the minimum required number of employees (per
company).
- Respondents Comments
The logging business was a mere expansion of petitioner's activities and for purposes of
the Social Security Act, petitioner should be considered a member of the System since
December 1, 1952 when it commenced its real estate business.
ISSUES
1. WON the contributions required of employers and employees under our Social Security
Act of 1954 are obligatory because the said Act was allegedly enacted by Congress in the
exercise of the police power of the State, not of its taxing power
2. WON a contractee-independent contractor relationship existed between petitioner and
Eufracio Rojas. during the time that he was operating its forest concession at Baganga,
Davao
3. WON Section 9 of the Social Security Act on the question of compulsory membership
and employers should be given a liberal interpretation
HELD
1. Ratio The said enactment implements the general welfare mandate of the Constitution
and constitutes a legitimate exercise of the police power of the State.
Reasoning
- The Social Security Law was enacted pursuant to the policy of the government "to
develop, establish gradually and perfect a social security system which shall be
suitable to the needs of the people throughout the Philippines, and shall provide
protection against the hazards of disability, sickness, old age and death" (Sec. 2, RA
1161, as amended).
- Membership in the SSS is not a result of bilateral, concensual agreement where the
rights and obligations of the parties are defined by and subject to their will, RA 1161

A2010

-2-

Disini

requires compulsory coverage of employees and employers under the System. It is


actually a legal imposition on said employers and employees, designed to provide social
security to the workingmen. The principle of non-impairment of the obligation of contract
as provided in the Bill of Rights is not a proper defense, the enactment being a lawful
exercise of the police power of the State.
- The taxing power of the State is exercised for the purpose of raising revenues. However,
under our Social Security Law, the emphasis is more on the promotion of the general
welfare. The Act is not part of out Internal Revenue Code nor are the contributions and
premiums therein dealt with and provided for, collectible by the Bureau of Internal
Revenue. The funds contributed to the System belong to the members who will receive
benefits, as a matter of right, whenever the hazards provided by the law occur.
- Together with the contributions imposed upon employees and the Government, they are
intended for the protection of said employees against the hazards of disability, sickness,
old age and death in line with the constitutional mandate to promote social justice to insure
the well-being and economic security of all the people.
- It is the intention of the law to cover as many persons as possible so as to promote the
constitutional objective of social justice. It is clear that a later law prevails over a prior
statute and moreover the legislative intent must be given effect.
2. Ratio Rojas was not an independent contractor but merely an employee of the
petitioner.
Reasoning
- Rojas was appointed as operations manager of the logging concession; he has no power
to appoint or hire employees; as the term implies, he only manages the employees and it
is petitioner who furnishes him the necessary equipment for use in the logging business;
and he is not free from the control and direction of his employer in matter connected with
the performance of his work. Rojas should be entitled to the compulsory coverage of the
Act.
3. Ratio Because of the broad social purpose of the Social Security Act, all doubts in
construing the Act should favor coverage rather than exemption.
Reasoning
- Prior to its amendment, Sec. 9 of the Act provides that before an employer could be
compelled to become a member of the System, he must have been in operation for at
least two years and has at the time of admission at least six employees. It should be
pointed out that it is the employer, either natural, or judicial person, who is subject
to compulsory coverage and not the business.
- It is the intention of the law to cover as many persons as possible so as to promote the
constitutional objective of social justice. It is axiomatic that a later law prevails over a prior
statute and moreover the legislative in tent must be given effect
Disposition The records show that petitioner started its real estate business on
December 1, 1952 while its logging operation was actually commenced on April 1, 1957.
Applying the provision of Sec. 10 (previously Sec. 9) of the Act, petitioner is subject to
compulsory coverage as of December 1, 1952 with respect to the real estate business and
as of April 1, 1957 with respect to its logging operation. The appeal is dismissed, with
costs against the petitioner.

KASAPIAN NG MALAYANG MANGGAGAWA SA COCACOLA (KASAMMA-CCO) V CA


CHICO-NAZARIO; April 19, 2006
NATURE
Petition for Review on Certiorari assailing the Decision of the Court of Appeals which
affirmed the Decision of public respondent National Labor Relations Commission (NLRC)
dismissing petitioners complaint against private respondent
FACTS
- On 30 June 1998, the CBA for the years 1995-1998 executed between petitioner union
and private respondent company expired. Petitioner submitted its demands to the
company for another round of collective bargaining negotiations. Said negotiations came
to a gridlock as the parties failed to reach a mutually acceptable agreement with respect to
certain economic and non-economic issues.
Thereafter, petitioner filed a notice of strike on 11 November 1998 with the National
Conciliation and Mediation Board on the ground of CBA negotiation deadlock. Several
conciliation conferences were conducted but the parties failed to reach a settlement. On
19 December 1998, petitioner held the strike in private respondents Manila and Antipolo
plants.
- Subsequently, both parties came to an agreement settling the labor dispute. Thus, on 26
December 1998, both parties executed and signed a MOA providing for salary increases
and other economic and non-economic benefits. It likewise contained a provision for the
regularization of contractual, casual and/or agency workers who have been working with
private respondent for more than one year. Said MOA was later incorporated to form part
of the 1998-2001 CBA and was thereafter ratified by the employees of the company.
- Consequently, petitioner demanded the payment of salary and other benefits to the
newly regularized employees retroactive to 1 December 1998, in accord with the MOA.

Labor Law 1
However, the private respondent refused to yield to said demands contending that the
date of effectivity of the regularization of said employees were 1 May 1999 and 1 October
1999. Meanwhile, a certification election was conducted on 17 August 1999 wherein the
KASAMMA-CCO Independent surfaced as the winning union and was then certified by the
DOLE as the sole and exclusive bargaining agent of the rank-and-file employees of private
respondents Manila and Antipolo plants for a period of five years from 1 July 1999 to 30
June 2004. On 23 August 1999, the KASAMMA-CCO Independent demanded the
renegotiation of the CBA which expired on 30 June 1998. Such request was denied by
private respondent as there was already an existing CBA which was negotiated and
concluded between petitioner and private respondent which was yet to expire on 30 June
2001.
- On 9 December 1999, despite the pendency of petitioners complaint before the NLRC,
private respondent closed its Manila and Antipolo plants resulting in the termination of
employment of 646 employees. About 500 workers were given a notice of termination
effective 1 March 2000 on the ground of redundancy. The affected employees were
considered on paid leave from 9 December 1999 to 29 February 2000 and were paid their
corresponding salaries. On 13 December 1999, four days after its closure of the Manila
and Antipolo plants, private respondent served a notice of closure to the DOLE.
- Petitioner contends that respondent violated the MOA by not recognizing the
regularization of the 61 employees as of December 1, 1998 and giving them full benefits
retroactive to that date. Petitioner likewise claims the closure of the plants was in bad faith,
done in order to avoid renegotiations of the CBA, and therefore illegal.
ISSUES
1. WON the regularization of the 61 employees was effective December 1, 1998
2. WON the closure of the plants was legal
HELD
1. YES
Ratio It must be noted that both parties admit the existence of said MOA and that they
have voluntarily entered into said agreement. Furthermore, neither of the parties deny that
the 61 employees have indeed been regularized by private respondent. The MOA, being a
contract freely entered into by the parties, now constitutes as the law between them, and
the interpretation of its contents purely involves an evaluation of the law as applied to the
facts herein. It is the contention of petitioner that the date 1 December 1998 refers to the
effective date of regularization of said employees, while private respondent maintains that
said date is merely the reckoning date from which the one year employment requirement
shall be computed. We agree with petitioner. It is logically absurd that the company will
only begin to extend priority to these employees on a date that has already passed, when
in fact they have already extended priority to these employees by agreeing to the contents
of the MOA and signing said agreement. It is erroneous for the NLRC to conclude that
extending to them the benefits of the MOA would violate the principle of "no-work-no-pay"
as they are actually rendering service to the company even before 1 December 1998, and
continued to do so thereafter. Moreover, under Article 280 of the Labor Code, any
employee who has rendered at least one year of service, shall be considered a regular
employee with respect to the activity in which he is employed and his employment shall
continue while such activity exists. Also, under the law, a casual employee is only casual
for one year, and it is the passage of time that gives him a regular status. Even if we were
to follow private respondents contention that the date 1 December 1998 provided in the
MOA is merely a reckoning date to determine who among the non-regular employees
have rendered one year of service as of said date, all those who have been with the
company for one year by said date must automatically be considered regular employees
by operation of law.
2. YES
Ratio The characterization of the employees service as no longer necessary or
sustainable, and therefore properly terminable, is an exercise of business judgment on the
part of the employer. The wisdom or soundness of such characterizing or decision is not
subject to discretionary review on the part of the Labor Arbiter nor of the NLRC so long, of
course, as violation of law or merely arbitrary and malicious action is not shown. The
private respondents decision to close the plant was a result of a study conducted which
established that the most prudent course of action for the private respondent was to stop
operations in said plants and transfer production to other more modern and technologically
advanced plants of private respondent. The subject closure and the resulting termination
of the 639 employees was due to legitimate business considerations, as evidenced by the
technical study conducted by private respondent.
Disposition The assailed Decisions are hereby AFFIRMED with MODIFICATION. The 61
subject employees are hereby declared regular employees as of 1 December 1998 and
are entitled to the benefits provided for in the Memorandum of Agreement.

DOLE PHILIPPINES INC V PAWIS NG MAKABAYNG


OBRERO
CORONA; (date) 2003

A2010

-3-

Disini

NATURE
Petition for review on certiorari of the decision of the Court of Appeals
FACTS
- The petitioner and the respondent executed a CBA for the period starting February 1996
to February 2001. Under the bonuses and allowances section of the said CBA, a P10
meal allowance shall be given to employees who render at least 2 hrs of overtime work
and free meals shall be given after 3 hours of actual overtime work.
- Pursuant to this provision, some departments of granted free meals after exactly 3 ours
of work. However, other departments granted free meals only after more than 3 hours of
overtime work.
- The respondent filed a complaint against Dole, saying that free meals should be granted
after exactly 3 hrs of overtime work, not after more than 3 hrs. The parties agreed to settle
the dispute to voluntary arbitration. It was decided in favor of the respondent, directing the
petitioner to grant free meals after exactly 3 hrs of overtime work. CA affirmed.
ISSUES
1. WON free meals should be granted after exactly 3 hrs of work
2. WON the petitioner has the right to determine when to grant free meals and its
conditions
HELD
1. YES
- The same meal allowance provision is found in their previous CBAs, the 1985-1988 CBA
and the 1990-1995 CBA. However, it was amended in the 1993-1995 CBA, by changing
the phrase after 3 hrs of overtime work to after more than 3 hrs of overtime work. In the
1996-2001 CBA, the parties had to negotiate the deletion of the said phrase in order to
revert to the old provision. Clearly, both parties had intended that free meals should be
given after exactly 3 hrs of overtime work.
- The disputed provision is clear and unambiguous, hence the literal meaning shall prevail.
No amount of legal semantics can convince the Court that after more than means the
same as after.
2. NO
- The exercise of management prerogative is not unlimited. It is subject to the limitations
provided by law. In this case, there was a CBA, and compliance therewith is mandated by
the express policy of the law.
Disposition Petition denied

DAVAO FRUITS CORPORATION V ASSOCIATED


LABOR UNIONS (ALU)
QUIASON; August 24, 1993
NATURE
This is a petition for certiorari to set aside the resolution of the National Labor Relations
Commission (NLRC)
FACTS
- On December 28, 1982 respondent Associated Labor Unions (ALU), for and in behalf of
all the rank-and-file workers and employees of petitioner, filed a complaint (NLRC Case
No. 1791-MC-XI-82) before the Ministry of Labor and Employment, Regional Arbitration
Branch XI, Davao City, against petitioner, for "Payment of the Thirteenth-Month Pay
Differentials." Respondent ALU sought to recover from petitioner the thirteenth month pay
differential for 1982 of its rank-and-file employees, equivalent to their sick, vacation and
maternity leaves, premium for work done on rest days and special holidays, and pay for
regular holidays which petitioner, allegedly in disregard of company practice since 1975,
excluded from the computation of the thirteenth month pay for 1982.
- In its answer, petitioner claimed that it erroneously included items subject of the
complaint in the computation of the thirteenth month pay for the years prior to 1982,
upon a doubtful and difficult question of law. According to petitioner, this mistake was
discovered only in 1981 after the promulgation of the Supreme Court decision in the case
of San Miguel Corporation v. Inciong (103 SCRA 139).
- A decision was rendered on March 7, 1984 favoring ALU. That ordered Davao Fruits
Corporation to pay the 1982 13th month pay differential to all its rank-and-file
workers/employees herein represented by complainant Union. Petitioner appealed the
decision of the Labor Arbiter to the NLRC, which affirmed the said decision accordingly
dismissed the appeal for lack of merit. Petitioner elevated the matter to the Supreme
Court.
ISSUES
1. WON the computation of the thirteenth month pay given by employers to their
employees under P.D. No. 851, payments for sick, vacation and maternity leaves,
premiums for work done on rest days and special holidays, and pay for regular holidays
may be excluded in the computation and payment thereof, regardless of long-standing
company practice

Labor Law 1
2. WON the petitioner may invoke the principle of solution indebiti
HELD
1. The "Supplementary Rules and Regulations Implementing P.D. No. 851," which put to
rest all doubts in the computation of the thirteenth month pay, was issued by the Secretary
of Labor as early as January 16, 1976, barely one month after the effectivity of P.D. No.
851 and its Implementing Rules. And yet, petitioner computed and paid the thirteenth
month pay, without excluding the subject items therein until 1981. Petitioner continued its
practice in December 1981, after promulgation of the afore-quoted San Miguel decision on
February 24, 1981, when petitioner purportedly "discovered" its mistake. From 1975 to
1981, petitioner had freely, voluntarily and continuously included in the computation
of its employees' thirteenth month pay, the payments for sick, vacation and
maternity leaves, premiums for work done on rest days and special holidays, and
pay for regular holidays. The considerable length of time the questioned items had
been included by petitioner indicates a unilateral and voluntary act on its part,
sufficient in itself to negate any claim of mistake.
- A company practice favorable to the employees had indeed been established and the
payments made pursuant thereto, ripened into benefits enjoyed by them. And any benefit
and supplement being enjoyed by the employees cannot be reduced, diminished,
discontinued or eliminated by the employer, by virtue of Section 10 of the Rules and
Regulations Implementing P.D. No. 851, and Article 100 of the labor of the Philippines,
which prohibit the diminution or elimination by the employer of the employees' existing
benefits (Tiangco v. Leogardo, Jr., 122 SCRA 267, [1983]).
2. Petitioner cannot invoke the principle of solutio indebiti which as a civil law concept that
is not applicable in Labor Law. Besides, in solutio indebiti, the obligee is required to return
to the obligor whatever he received from the latter (Civil Code of the Philippines, Arts.
2154 and 2155). Petitioner in the instant case, does not demand the return of what it paid
respondent ALU from 1975 until 1981; it merely wants to "rectify" the error it made over
these years by excluding unilaterally from the thirteenth month pay in 1982 the items
subject of litigation. Solutio indebiti, therefore, is not applicable to the instant case.
Disposition finding no grave abuse of discretion on the part of the NLRC, the petition is
hereby DISMISSED, and the questioned decision of respondent NLRC is AFFIRMED

SAMAHANG MANGGAGAWA V NLRC


ROMERO; September 7, 1998
NATURE
Petition for Certiorari
FACTS
- Petitioner Samahang Manggagawa sa Top Form Manufacturing United Workers of the
Philippines (SM) was the certified collective bargaining representative of all regular rank
and file employees of private respondent Top Form Manufacturing Philippines, Inc.
- Employer Top Form Manufacturing (TFM) refused to grant across-the-board increases to
its employees in implementing Wage Order No. 01 (granting an increase of P17 per day in
the salary of workers) and Wage Order No. 02 (providing for a P12 daily increase in
salary) of the Regional Tripartite Wages and Productivity Board of the National Capital
Region (RTWPB-NCR). Such refusal was aggravated by the fact that prior to the issuance
of said wage orders, the employer allegedly promised at the collective bargaining
conferences to implement any government-mandated wage increases on an across-theboard basis.
- The union (SM) requested the implementation of said wage orders. But they demanded
that the increase be on an across-the-board basis. Respondent TFM refused to accede to
that demand. Instead, it implemented a scheme of increases purportedly to avoid wage
distortion. TFM granted the P17 increase under WO#01 to workers/employees receiving
salary of P125/day and below. The P12 increase under by WO#02 was granted to those
receiving the salary of P140/day and below. For employees receiving salary higher than
P125 or P140.00/day, TFM granted an escalated increase ranging from P6.99 to P14.30
and from P6.00 to P10.00, respectively.
- SM filed a complaint with the NCR NLRC.
- Petitioners contention: TFM's act of "reneging on its undertaking/promise clearly
constitutes act of unfair labor practice through bargaining in bad faith." It charged TFM
with acts of unfair labor practices or violation of A247 of the Labor Code, as amended,
specifically "bargaining in bad faith," and prayed that it be awarded actual, moral and
exemplary damages. In its position paper, the union added that it was charging private
respondent with "violation of A100 of the Labor Code."
- Respondents contention: In implementing Wage Orders Nos. 01 and 02, it had avoided
"the existence of a wage distortion" that would arise from such implementation.
- There was no agreement to the effect that future wage increases mandated by the
government should be implemented on an across-the-board basis. Otherwise, that
agreement would have been incorporated and expressly stipulated in the CBA. It quoted
the provision of the CBA that reflects the parties' intention to "fully set forth" therein all their
agreements that had been arrived at after negotiations that gave the parties "unlimited

A2010

-4-

Disini

right and opportunity to make demands and proposals with respect to any subject or
matter not removed by law from the area of collective bargaining."
- Labor Arbiter dismissed the complaint for lack of merit. On appeal at the NLRC, same
was dismissed for lack of merit. MFR denied. Hence, this petition.
ISSUES
1. WON private respondent committed an unfair labor practice in its refusal to grant
across-the-board wage increases in implementing Wage Orders Nos. 01 and 02
2. WON there was a significant wage distortion of the wage structure in private respondent
as a result of the manner by which said wage orders were implemented.
HELD
1. NO
Ratio The CBA is the law between the contracting parties. Thus, only provisions embodied
in the CBA should be so interpreted and complied with. Where a proposal or a promise
raised by a contracting party does not find print in the CBA it is not a part thereof and the
proponent has no claim whatsoever to its implementation.
Reasoning
- If there was indeed a promise or undertaking on the part of TFM to obligate itself to grant
an automatic across-the-board wage increase, union SM should have requested or
demanded that such "promise or undertaking" be incorporated in the CBA. After all,
petitioner has the means under the law to compel private respondent to incorporate this
specific economic proposal in the CBA. It could have invoked A252 of the Labor Code
defining "duty to bargain," thus, the duty includes "executing a contract incorporating such
agreements if requested by either party."
- A252 also states that the duty to bargain "does not compel any party to agree to a
proposal or make any concession." Thus, petitioner may not validly claim that the proposal
embodied in the Minutes of the negotiation forms part of the CBA that it finally entered into
with private respondent.
- SMs contention that the Minutes of the collective bargaining negotiation meeting forms
part of the entire agreement is pointless. If indeed private respondent promised to
continue with the practice of granting across-the-board salary increases ordered by the
government, such promise could only be demandable in law if incorporated in the CBA.
(Obiter for our purposes Re: Past Practices) Granted that private respondent TFM had
granted an across-the-board increase pursuant to Republic Act No. 6727, that single
instance may not be considered an established company practice.
2. NO
Ratio The issue of whether or not a wage distortion exists is a question of fact that is
within the jurisdiction of the quasi-judicial tribunals below. Factual findings of
administrative agencies are accorded respect and even finality in this Court if they are
supported by substantial evidence.
Reasoning
- In this case, NLRC unanimously ruled that no wage distortions marred private
respondent's implementation of the wage orders. There was a meaningful implementation
of WO#01 and #02. SMs contention on the issue of wage distortion and the resulting
allegation of discrimination against the TFM's employees are anchored on its dubious
position that TFM's promise to grant an across-the-board increase in governmentmandated salary benefits reflected in the Minutes of the negotiation is an enforceable part
of the CBA.
Disposition NLRC resolutions affirmed. Petition dismissed.

AMERICAN WIRE AND CABLE DAILY RATED


EMPLOYEES UNION V AMERICAN WIRE AND CABLE
CO., INC.
CHICO-NAZARIO: April 29, 2005
FACTS
- American Wire and Cable Co., is a corporation engaged in the manufacture of wires and
cables. On Feb.16, 2001, an original action was filed before the NCMB of the DOLE by the
two unions (American Wire and Cable Daily Rated Employees and American Wire and
Cable Monthly Rated Employees) for voluntary arbitration. They alleged that respondent
company, without valid cause, suddenly and unilaterally withdrew and denied certain
benefits which they have long enjoyed. These are:
a) Service Award
b) 35% premium pay of an employees basic pay for the work rendered during Holy
Monday, Holy Tuesday, Holy Wednesday, December 23, 26, 27, 28 and 29
c) Christmas party
d) Promotional increase.
- A promotional increase was sought by 15 of its members who were given new job
classifications. These new hob classifications according to the union are in the form of a
promotion. Increase was not given.
Petitioners contention

Labor Law 1
- withdrawal of the 35% premium pay for selected days during Holy Week and Christmas
season, the holding of a Christmas party, and its incidental benefits, and the giving of
service awards was a customary practice that can no longer be unilaterally withdrawn by
respondent without consent of the petitioner. The benefits in question were given by
respondent consistently, deliberately and unconditionally since time immemorial. The
benefits given by the respondent cannot be considered as a bonus as they are not
founded on profit. Even assuming that it can be treated as a bonus, the grant of the same,
by reason of its ling and regular concession, may be regarded as part of regular
compensation.
Respondents contention
-The grant of all subject benefits has not ripened into practice that the employees
concerned can claim a demandable right over them. The grant of these benefits was
conditional based upon the financial conditions that existed before have indeed
substantially changed thereby justifying the discontinuance of said grants.
ISSUE
WON respondent is guilty of violating article 100 of the Labor Code, when the
benefits/entitlements given to the members of petitioner union were withdrawn
HELD
*preliminary issue raised by respondent was the error in the mode of appeal by the
petitioners. Respondent contends that petitioner should have raised a petition for review
on certiorari under Rule 45, and not through a special civil action for certiorari under Rule
65 of the Rules on Civil Procedure. Thus, case should be dismissed outright.
NO
- Court ruled that the SC may brush aside the procedural barrier and take cognizance of
the petition as it raises an issue of paramount importance.
- ART. 100. PROHIBITION AGAINST ELIMINATION OR DIMINUTION OF BENEFITS.Nothing in this Book shall be construed to eliminate or in any way diminish supplements,
or other employee benefits being enjoyed at the time of promulgation of this Code.
- a determination must first be made on whether the benefits are in the nature of a bonus
or no, and assuming they are so, whether they are demandable and enforceable
obligations.
- Definition of bonus (Producers Bank of the Philippines v. NLRC)
a bonus is an amount granted and paid to an employee for his industry and loyalty it is
an act of generosity granted by an enlightened employer to spur the employee to greater
efforts the granting of a bonus is a management prerogative thus a bonus is not a
demandable and enforceable obligation except when it is made part of the wage, salary or
compensation of the employee.
- Court ruled that the benefits /entitlements subjects of the instant case are all bonuses
given by respondent out of its generosity and munificence. Benefits/entitlements are all in
excess of what the law requires each employer to give its employees. Since they are
above what is strictly due, the granting of the same was a management prerogative,
which, whenever management sees necessary, may be withdrawn.
- the consequential question therefore that needs to be settled is if the subject benefits,
which are bonuses, are demandable or not.
- the Court does not believe so. For a bonus to be enforceable, it has to be promised by
the employer and expressly agreed upon by the parties or it must have a fixed amount and
had been a long and regular practice on the part of the employer. To be considered
regular practice the giving of the bonus should have been done over a long period of
time and must be shown to have been consistent and deliberate.
- the benefits in question were never part of any express agreement. They were never
even incorporated in the Collective Bargaining Agreement. The Christmas party and its
incidental benefits and the giving of cash incentive together with the service award cannot
be said to have fixed amounts. There was a downtrend in the amount given for service
awards. There was also a downtrend with respect to the holding of Christmas parties as
the locations were changed from paid venues to free ones. -The additional 35% premium
pay for work during Holy Week and Christmas season cannot be held to have ripened into
a company practice that the petitioners have a right to demand. This practice was only
granted for two years and with the express reservation from respondent corporations
owner that it cannot continue the same in view of the companys current financial
condition.

PAG-ASA STEEL WORKS, INC. V CA


CALLEJO, SR; March 31, 2006
NATURE
Petition for review on certiorari of the decision of the Court of Appeals in CA-G.R. SP No.
65171 ordering Pag-Asa Steel Works, Inc. to pay the members of Pag-Asa Steel Workers
Union the wage increase prescribed under Wage Order No. NCR-08.
FACTS

A2010

-5-

Disini

- On September 23, 1999, petitioner and the Union entered into a Collective Bargaining
Agreement (CBA), effective July 1, 1999 until July 1, 2004. Section 1, Article VI (Salaries
and Wage) of said CBA provides:
Section 1. WAGE ADJUSTMENT - The COMPANY agrees to grant all the workers, who
are already regular and covered by this AGREEMENT at the effectivity of this
AGREEMENT, a general wage increase as follows:
July 1, 1999 . . . . . . . . . . . P15.00 per day per employee
July 1, 2000 . . . . . . . . . . . P25.00 per day per employee
July 1, 2001 . . . . . . . . . . . P30.00 per day per employee
- The aforesaid wage increase shall be implemented across the board. Any Wage Order to
be implemented by the Regional Tripartite Wage and Productivity Board shall be in
addition to the wage increase adverted to above. However, if no wage increase is given by
the Wage Board within six (6) months from the signing of this AGREEMENT, the
Management is willing to give the following increases, to wit:
July 1, 1999 . . . . . . . . . . . P20.00 per day per employee
July 1, 2000 . . . . . . . . . . . P25.00 per day per employee
July 1, 2001 . . . . . . . . . . . P30.00 per day per employee
- The difference of the first year adjustment to retroact to July 1, 1999.
- The across-the-board wage increase for the 4th and 5th year of this AGREEMENT shall
be subject for a re-opening or renegotiation as provided for by Republic Act No. 6715.
- On October 14, 1999, Wage Order No. NCR-07 was issued, and on October 26, 1999, its
Implementing Rules and Regulations. It provided for a P25.50 per day increase in the
salary of employees receiving the minimum wage and increased the minimum wage to
P223.50 per day. Petitioner paid the P25.50 per day increase to all of its rank-and-file
employees.
- On July 1, 2000, the rank-and-file employees were granted the second year increase
provided in the CBA in the amount of P25.00 per day.
- On November 1, 2000, Wage Order No. NCR-08 took effect. Section 1 thereof provides:
Section 1. Upon the effectivity of this Wage Order, private sector workers and employees
in the National Capital Region receiving the prescribed daily minimum wage rate of
P223.50 shall receive an increase of TWENTY SIX PESOS and FIFTY CENTAVOS
(P26.50) per day, thereby setting the new minimum wage rate in the National Capital
Region at TWO HUNDRED FIFTY PESOS (P250.00) per day.
- Then Union president Lucenio Brin requested petitioner to implement the increase under
Wage Order No. NCR-08 in favor of the companys rank-and-file employees. Petitioner
rejected the request, claiming that since none of the employees were receiving a daily
salary rate lower than P250.00 and there was no wage distortion, it was not obliged to
grant the wage increase.
- The Union elevated the matter to the National Conciliation and Mediation Board. When
the parties failed to settle, they agreed to refer the case to voluntary arbitration.
- The Union alleged that it has been the companys practice to grant a wage increase
under a government-issued wage order, aside from the yearly wage increases in the CBA.
- Petitioner alleged that there is no such company practice and that it complied with the
previous wage orders (Wage Order Nos. NCR-01-05) because some of its employees
were receiving wages below the minimum prescribed under said orders. As for Wage
Order No. NCR-07, petitioner alleged that its compliance was in accordance with its verbal
commitment to the Union during the CBA negotiations that it would implement any wage
order issued in 1999.
- On June 6, 2001, the VA rendered judgment in favor of the company and ordered the
case dismissed.
- The Union filed a petition for review with the CA. On September 23, 2004, the CA
rendered judgment in favor of the Union and reversed that of the VA. But the findings of
the CA were grounded on the CBA and not on the issue of past practices.
ISSUE
WON the petitioner is obliged to grant wage increase under Wage Order No. NCR-08 as a
matter of practice
HELD
Ratio To ripen into a company practice that is demandable as a matter of right, the giving
of the increase should not be by reason of a strict legal or contractual obligation, but by
reason of an act of liberality on the part of the employer.
Reasoning
- The only instance when petitioner admittedly implemented a wage order despite the fact
that the employees were not receiving salaries below the minimum wage was under Wage
Order No. NCR-07. Petitioner, however, explains that it did so because it was agreed upon
in the CBA that should a wage increase be ordered within six months from its signing,
petitioner would give the increase to the employees in addition to the CBA-mandated
increases. Respondents isolated act could hardly be classified as a "company practice" or
company usage that may be considered an enforceable obligation.
Disposition petition is GRANTED. The Decision of the Court of Appeals in CA-G.R. SP
No. 65171 and Resolution dated January 11, 2005 are REVERSED and SET ASIDE. The
Decision of the Voluntary Arbitrator is REINSTATED.

Labor Law 1
CHINA BANKING CORPORATION V BORROMEO
CALLEJO, SR.; October 19, 2004
NATURE
Certiorari
FACTS
- Borromeo was a Manager of CB assigned at Regional Office in Cebu City. He then had
the rank of Manager Level I. Subsequently, the respondent was laterally transferred to
Cagayan de Oro City as Branch Manager of the petitioner Banks branch thereat.
- He consistently received a "very good" performance rating and was promoted to the
position of Assistant Vice-President, Branch Banking Group for the Mindanao area
effective October 16, 1996.
- Each promotion had the corresponding increase in the respondents salary as well as in
the benefits he received from the petitioner Bank.
- However, prior to his last promotion and then unknown to the China Bank, Borromeo,
without authority from the Executive Committee or Board of Directors, approved several
DAUD/BP accommodations amounting to P2,441,375 in favor of Joel Maniwan, with
Edmundo Ramos as surety. DAUD/BP is the acronym for checks "Drawn Against
Uncollected Deposits/Bills Purchased." Such checks, which are not sufficiently funded by
cash, are generally not honored by banks. Further, a DAUD/BP accommodation is a credit
accommodation granted to a few and select bank clients through the withdrawal of
uncollected or uncleared check deposits from their current account. Under the petitioner
Banks standard operating procedures, DAUD/BP accommodations may be granted only
by a bank officer upon express authority from its Executive Committee or Board of
Directors.
- As a result of the DAUD/BP accommodations in favor of Maniwan, a total of ten out-oftown checks (7 PCIB checks and 3 UCPB checks) of various dates amounting to
P2,441,375 were returned unpaid from September 20, 1996 to October 17, 1996. Each of
the returned checks was stamped with the notation "Payment Stopped/Account Closed."
- On October 8, 1996, the Borromeo wrote a Memorandum to the petitioner Banks senior
management requesting for the grant of a P2.4 million loan to Maniwan.
- The memorandum stated that the loan was "to regularize/liquidate subjects (referring to
Maniwan) DAUD availments."
- It was only then that the petitioner Bank came to know of the DAUD/BP accommodations
in favor of Maniwan. The petitioner Bank further learned that these DAUD/BP
accommodations exceeded the limit granted to clients, were granted without proper prior
approval and already past due.
- Acting on this information, Samuel L. Chiong, the petitioner Banks First Vice- President
and Head-Visayas Mindanao Division, in his Memorandum dated November 19, 1996 for
the respondent, sought clarification from the latter on the following matters:
- May 23, 1997 - Nancy D. Yang, the CBanks Senior VP and Head-Branch Banking
Group, informed the B (through a memorandum) that his approval of the DAUD/BP
accommodations in favor of Maniwan w/o authority and/or approval of higher management
violated the petitioner Banks Code of Ethics. As such, B was directed to restitute the
amount of P1,507,736.79 representing 90% of the total loss of P1,675,263.10 incurred by
the petitioner Bank.
- However, in view of his resignation and considering the years of service in the petitioner
Bank, the management earmarked only P836,637.08 from the respondents total
separation benefits or pay.
- In the Letter dated May 26, 1997 addressed to B, Remedios Cruz, CBanks VP of the HR
Division, again informed him that the management would withhold the sum of P836,637.08
from his separation pay, mid-year bonus and profit sharing.
- The said amount would be released upon recovery of the sums demanded from
Maniwan in Civil Case No. 97174 filed against him by CBank with the RTC in Cagayan de
Oro City.
- Consequently, the B, through counsel, made a demand on the CBank for the payment of
his separation pay and other benefits.
- The CBank maintained its position to withhold the sum of P836,637.08.
- B filed with the NLRC, the complaint for payment of separation pay, mid-year bonus,
profit share and damages against the petitioner Bank.
- The Labor Arbiter (LA)-dismissed the Bs complaint, for B had committed a serious
infraction when, in blatant violation of the banks SOP and policies, he approved the
DAUD/BP accommodations in favor of Maniwan without authorization by senior
management. B even had admitted this breach in the letters that he wrote to the senior
officers of CBank.
- LA- made the finding that B offered to assign or convey a property that he owned to
CBank, as well as proposed the withholding of the benefits due him to answer for the
losses that the petitioner Bank incurred on account of unauthorized DAUD/BP
accommodations.

A2010

-6-

Disini

- LA also held that CBanks act of withholding the benefits due the respondent was
justified under its Code of Ethics and that B as an officer of the CBank, was bound by the
provisions of the said Code.
- B appealed to the NLRC.
- NLRC- affirmed in toto the findings and conclusions of the LA. And ruled that the LA
committed no grave abuse of discretion when he decided the case on the basis of the
position papers submitted by the parties.
- B file a MR but the NLRC denied his motion. So he filed a petition for certiorari with the
CA.
- CA -set aside the decision of the NLRC and ordered that the records of the case be
remanded to the Labor Arbiter for further hearings on the factual issues involved.
- CBank filed a MR but the CA denied as it found no compelling ground to warrant
reconsideration.
ISSUES
Procedural
WON the CA erred in remanding the case to the Labor Arbiter/
WON Bs right to due process was violated by CBank since no administrative investigation
was conducted prior to the withholding of his separation benefits
Substantive
WON B pledged his benefits as guarantee for the losses the bank incurred resulting from
the unauthorized DAUD/BP accommodations in favor of Maniwan/
WON CBank could impose the penalty of restitution against B
HELD
Procedural
YES, CA committed reversible error/
NO, No formal administrative investigation was necessary
Reasoning
- It is settled that administrative bodies like the NLRC, including the Labor Arbiter, are not
bound by the technical niceties of the law and procedure and the rules obtaining in courts
of law.
- Rules of evidence are not strictly observed in proceedings before administrative bodies
like the NLRC, where decisions may be reached on the basis of position papers.
- The holding of a formal hearing or trial is discretionary with the Labor Arbiter and is
something that the parties cannot demand as a matter of right.
- As a corollary, trial-type hearings are not even required as the cases may be decided
based on verified position papers, with supporting documents and their affidavits.
- The assailed CA decisions directive requiring him to conduct further hearings constitutes
undue interference with the Labor Arbiters discretion.
- To require the conduct of hearings would be to negate the rationale and purpose of the
summary nature of the proceedings mandated by the Rules and to make mandatory the
application of the technical rules of evidence.
- As long as the decisions of the LA and the NLRC are devoid of any arbitrariness in the
process of their deduction from the evidence proffered by the parties, all that is left is for
the Court to stamp its affirmation.
- In this case, the factual findings of the Labor Arbiter and those of the NLRC concur on
various points.
- Due process simply demands an opportunity to be heard and this opportunity was not
denied Borromeo as he was, through the memoranda issued to him, given notice of the
charge against him. He was given the opportunity to be heard and considering his
admissions, it became unnecessary to hold any formal investigation.
Substantive
YES to both. LA and NLRC concurred in finding that B indeed pledged his benefits and
CBanks Code of Ethics expressly sanctions the imposition of restitution/forfeiture of
benefits apart from or independent of the other penalties.
Reasoning
- CBanks Code of Ethics-Restitution may be imposed independently or together with any
other penalty in case of loss or damage to the property of the Bank, its employees, clients
or other parties doing business with the Bank. The Bank may recover the amount
involved by means of salary deduction or whatever legal means that will prompt
offenders to pay the amount involved. But restitution shall in no way mitigate the
penalties attached to the violation or infraction.
- Supra-Forfeiture of benefits/privileges may also be effected in cases where
infractions or violations were incurred in connection with or arising from the
application/availment thereof.
- It is well recognized that company policies and regulations are, unless shown to
be grossly oppressive or contrary to law, generally binding and valid on the parties
and must be complied with until finally revised or amended unilaterally or
preferably through negotiation or by competent authority.
- Moreover, management has the prerogative to discipline its employees and to impose
appropriate penalties on erring workers pursuant to company rules and regulations. With
more reason should these truisms apply to the respondent, who, by reason of his
position, was required to act judiciously and to exercise his authority in harmony
with company policies.

Labor Law 1
- Obviously, in view of his voluntary separation from the petitioner Bank, the imposition of
the penalty of reprimand or suspension would be futile. The petitioner Bank was left with
no other recourse but to impose the ancillary penalty of restitution. It was certainly within
the petitioner Banks prerogative to impose on the respondent what it considered the
appropriate penalty under the circumstances pursuant to its company rules and
regulations.
- Significantly, B is not wholly deprived of his separation benefits. The LA stressed in
his decision, "the separation benefits due the complainant (Borromeo) were merely
withheld." The NLRC made the same conclusion.
- B was not just a rank and file employee. At the time of his resignation, he was the Asst.
VP, Branch Banking Group for the Mindanao area of CBank. His position carried authority
for the exercise of independent judgment and discretion, characteristic of sensitive posts
in corporate hierarchy. 41 As such, he was, as earlier intimated, required to act judiciously
and to exercise his authority in harmony with company policies.
Obiter
- CBanks business is essentially imbued with public interest and owes great fidelity to the
public it deals with.
- It is expected to exercise the highest degree of diligence in the selection and supervision
of their employees.
- As a corollary, and like all other business enterprises, its prerogative to discipline its
employees and to impose appropriate penalties on erring workers pursuant to company
rules and regulations must be respected.
- DAYAN v BPI ~ The law, in protecting the rights of labor, authorized neither oppression
nor self-destruction of an employer company which itself is possessed of rights that must
be entitled to recognition and respect
Disposition Petition is GRANTED. CAS DECISION AND RESOLUTION REVERSED
AND SET ASIDE. NLRCS DECISION, affirming that of the Labor Arbiter, is
REINSTATED.

CEBU ROYAL PLANT V DEPUTY MINISTER OF LABOR


CRUZ; August 12, 1987
NATURE
Petitioner faults the Deputy Minister of Labor with grave abuse of discretion
FACTS
- Pilones was dismissed by Cebu Royal Plant (CRP)
- the alleged ground for his removal: pulmonary tuberculosis minimal
- Pilones complained to the Ministry of Labor. The regional director dismissed this
complaint, but the Deputy Minister reversed this and required CRP to reinstate Pilones
and to pay him back wages.
Public respondent maintains:
- that Pilones, the private respondent, was already a permanent employee when he was
dismissed, and thus entitled to security of tenure
- that his pulmonary tuberculosis minimal (PTM) was not certified as incurable within six
months so as to justify his separation
- that the petitioner should have first obtained a clearance for the termination of Pilones
employment
Petitioner maintains:
- Pilones was still on probation at the time of dismissal, and thus had no security of tenure.
- dismissal was necessary for the protection of the public health because he was handling
ingredients in the processing of soft drinks which were being sold to the public
- the findings of the regional director, who had direct access to the facts, should not have
been disturbed on appeal.
- that Pilones was employed on probation on Feb. 16, 1978; that the six-month probation
period ended on Aug. 17, 1978; that he was dismissed on Aug. 21, 4 days after he ceased
to be a probationer, only because the x-ray examination (which showed his PTM) was
made only on Aug. 17, and the results were not immediately available.
- There is, however, proof that PIlones may have been hired in 1977, as shown by a 1977
withholding tax statement issued for him by CRP.
ISSUE
WON Pilones was still a probationary employee when he was dismissed on August 21,
1978
HELD
NO
Ratio When an employee is dismissed due to a disease, the applicable rule is:
Sec. 8, Rule I, Book VI, of the Rules and Regulations implementing the Labor Code:
Disease as a ground for dismissal. Where the employee suffers from a disease and his
continued employment is prohibited by law or prejudicial to his health or to the health of
his co-employees, the employer shall not terminate his employment unless there is a
certification by a competent public health authority that the disease is of such nature or at
such a stage that it cannot be cured within a period of six (6) months even with proper

A2010

-7-

Disini

medical treatment. If the disease or ailment can be cured within the period, the employer
shall not terminate the employee but shall ask the employee to take a leave. The employer
shall reinstate such employee to his former position immediately upon the restoration of
his normal health.
Reasoning
- The records do not show the certification by a competent public health authority that is
required by the above rule, that the disease cannot be cured within a period of 6 months.
It only contained a certificate offered by CRPs own physician, not a public health authority.
The court surmised that if the required certification was not presented, it was because the
disease was not of such a nature that it could not be cured within a period of 6 months. If
so, dismissal was an unlawful sanction.
- Also, the petitioners application for clearance to terminate Pilones was filed only on
August 28, 1978, 7 days after his dismissal. This did not follow the prior clearance rule
which was in force at the time.
-- We agree that there was here an attempt to circumvent the law by separating the
employee after five months' service to prevent him from becoming a regular employee,
and then rehiring him on probation, again without security of tenure. We cannot permit this
subterfuge if we are to be true to the spirit and mandate of social justice. On the other
hand, we have also the health of the public and of the dismissed employee himself to
consider. Hence, although we must rule in favor of his reinstatement, this must be
conditioned on his fitness to resume his work, as certified by competent authority.
Disposition petition is DISMISSED and the temporary restraining order of November 18,
1981, is LIFTED. The Order of the public respondent dated July 14, 1981, is AFFIRMED,
but with the modification that the backwages shall be limited to three years only and the
private respondent shall be reinstated only upon certification by a competent public health
authority that he is fit to return to work. Costs against the petitioner.

LAPANDAY AGRICULTURAL DEVELOPMENT


CORPORATION V CA (and COMMANDO SECURITY
SERVICE AGENCY, INC.)
GONZAGA-REYES; January 31, 2000
NATURE
Petition for Review on Certiorari of the decision of the CA which affirmed the decision of
the RTC.
FACTS
- In June 1986 private respondent and plaintiff entered into a Guard Service Contract.
Respondent provided security guards in defendant's banana plantation. The contract
called for the payment to a guard of P754.28 on a daily 8-hour basis and an additional
P565.72 for a four hour overtime while the shift-in-charge was to be paid P811.40 on a
daily 8-hour basis and P808.60 for the 4-hour overtime.
- Wage Orders increasing the minimum wage in 1983 were complied with by the
defendant. On June 16, 1984, Wage Order No. 5 was promulgated directing an increase
of P3.00 per day on the minimum wage of workers in the private sector and a P5.00
increase on the ECOLA. This was followed on November 1, 1984 by Wage Order No. 6
which further increased said minimum wage by P3.00 on the ECOLA. Both Wage Orders
contain the following provision:
"In the case of contract for construction projects and for security, janitorial and similar
services, the increase in the minimum wage and allowances rates of the workers shall
be borne by the principal or client of the construction/service contractor and the
contracts shall be deemed amended accordingly, subject to the provisions of Sec. 3 (b)
of this order" (Sec. 6 and Sec. 9, Wage Orders No. 5 and 6, respectively).
- Respondent demanded that its Guard Service Contract with defendant be upgraded in
compliance with Wage Order Nos. 5 and 6. Plaintiff refused. Their Contract expired on
June 6, 1986 without the rate adjustment called for Wage Order Nos. 5 and 6 being
implemented. By the time of the filing of respondent's Complaint, the rate adjustment
payable by defendant amounted to P462,346.25. Plaintiff opposed the Complaint.
- The trial court decided in favor of the respondent. Plaintiffs MOR was denied, hence this
petition.
ISSUES
1. WON RTC has jurisdiction over the case
2. WON petitioner is liable to the private respondent for the wage adjustments provided
under Wage Order Nos. 5 and 6 and for attorney's fees
HELD
1. YES
- The enforcement of the written contract does not fall under the jurisdiction of the NLRC
because the money claims involved therein did not arise from employer-employee
relations between the parties and is intrinsically a civil dispute. Thus, jurisdiction lies with
the regular courts. The RTC has jurisdiction over the subject matter of the present case. It
is well settled in law and jurisprudence that where no employer-employee relationship

Labor Law 1
exists between the parties and no issue is involved which may be resolved by reference to
the Labor Code, other labor statutes or any collective bargaining agreement, it is the
Regional Trial Court that has jurisdiction. In its complaint, private respondent is not
seeking any relief under the Labor Code but seeks payment of a sum of money and
damages on account of petitioner's alleged breach of its obligation under their Guard
Service Contract. The action is within the realm of civil law hence jurisdiction over the case
belongs to the regular courts. While the resolution of the issue involves the application of
labor laws, reference to the labor code was only for the determination of the solidary
liability of the petitioner to the respondent where no employer-employee relation exists.
Article 217 of the Labor Code as amended vests upon the labor arbiters exclusive original
jurisdiction only over the following:
1.
Unfair labor practices;
2.
Termination disputes;
3.
If accompanied with a claim for reinstatement, those cases that workers
may file involving wages, rates of pay, hours of work and other terms and
conditions of employment;
4.
Claims for actual, moral exemplary and other form of damages arising
from employer-employee relations;
5.
Cases arising from any violation of Article 264 of this Code, including
questions involving legality of strikes and lockouts; and
6.
Except claims for Employees Compensation, Social Security, Medicare
and maternity benefits, all other claims, arising from employer-employee
relations, including those of persons in domestic or household service, involving
an amount exceeding five thousand pesos (P5,000.00) regardless of whether
accompanied with a claim for reinstatement.
- In all these cases, an employer-employee relationship is an indispensable jurisdictional
requisite; and there is none in this case.
2. Private respondent admits that there is no employer-employee relationship between it
and the petitioner. The private respondent is an independent/job contractor 1 who assigned
security guards at the petitioner's premises for a stipulated amount per guard per month.
The Contract of Security Services expressly stipulated that the security guards are
employees of the Agency and not of the petitioner. Articles 106 and 107 of the Labor Code
provides the rule governing the payment of wages of employees in the event that the
contractor fails to pay such wages1.
- It will be seen from the above provisions that the principal (petitioner) and the contractor
(respondent) are jointly and severally liable to the employees for their wages. This Court
held in Eagle Security, Inc. vs. NLRC and Spartan Security and Detective Agency, Inc. vs.
NLRC that the joint and several liability of the contractor and the principal is mandated by
the Labor Code to assure compliance with the provisions therein including the minimum
wage. The contractor is made liable by virtue of his status as direct employer. The
principal, on the other hand, is made the indirect employer of the contractor's employees
to secure payment of their wages should the contractor be unable to pay them. Even in
the absence of an employer-employee relationship, the law itself establishes one between
the principal and the employees of the agency for a limited purpose i.e. in order to ensure
that the employees are paid the wages due them. In the above-mentioned cases, the
solidary liability of the principal and contractor was held to apply to the aforementioned
Wage Order Nos. 5 and 6. In ruling that under the Wage Orders, existing security guard
services contracts are amended to allow adjustment of the consideration in order to cover
payment of mandated increases, and that the principal is ultimately liable for the said
increases.
- It is clear that it is only when contractor pays the increases mandated that it can claim an
adjustment from the principal to cover the increases payable to the security guards. The
conclusion that the right of the contractor (as principal debtor) to recover from the principal
as solidary co-debtor) arises only if he has paid the amounts for which both of them are
jointly and severally liable is in line with Article 12172 of the Civil Code.
- The right of reimbursement from a co-debtor is recognized in favor of the one who paid.
The liability of the petitioner to reimburse the respondent only arises if and when
respondent actually pays its employees the increases granted by Wage Order Nos. 5 and
6. Payment, which means not only the delivery of money but also the performance, in any
1

Art. 106. Contractor or sub contractor. Whenever an employer enters into a contract with another person for the
performance of the former's work, the employees of the contractor and of the latter's subcontractor, if any, shall be
paid in accordance with the provisions of this Code.
In the event that the contractor or subcontractor fails to pay the wages of his employees in accordance with this
Code, the employer shall be jointly and severally liable with his contractor or subcontractor to such employees to the
extent of the work performed under the contract, in the same manner and extent that he is liable to employees
directly employed by him.
Art. 107. Indirect employer. The provisions of the immediately preceding Article shall likewise apply to any person,
partnership, association or corporation which, not being an employer, contracts with an independent contractor for
the performance of any work, task, job or project.

Art. 1217. Payment made by one of the solidary debtors extinguishes the obligation. If two or more solidary
debtors offer to pay, the creditor may choose which offer to accept.
He who made payment may claim from his codebtors only the share which corresponds to each, with interest for
the payment already made. If the payment is made before the debt is due, no interest for the intervening period
may be demanded. . . .

A2010

-8-

Disini

other manner, of the obligation,is the operative fact which will entitle either of the solidary
debtors to seek reimbursement for the share which corresponds to each of the debtors.
- It is not disputed that the private respondent has not actually paid the security guards the
wage increases granted under the Wage Orders in question. Neither is it alleged that there
is an extant claim for such wage adjustments from the security guards concerned, whose
services have already been terminated by the contractor. Accordingly, private respondent
has no cause of action against petitioner to recover the wage increases. Needless to
stress, the increases in wages are intended for the benefit of the laborers and the
contractor may not assert a claim against the principal for salary wage adjustments that it
has not actually paid. Otherwise, as correctly put by the respondent, the contractor would
be unduly enriching itself by recovering wage increases, for its own benefit.
- Finally, considering that the private respondent has no cause of action against the
petitioner, private respondent is not entitled to attorney's fees.
Disposition Petition GRANTED. The decision of the CA REVERSED and SET ASIDE.
The complaint of private respondent COMMANDO SECURITY SERVICE AGENCY, INC.
is hereby DISMISSED.

VILLAMARIA, JR. V CA
CALLEJO, SR.; April 19, 2006
NATURE
Petition for review on certiorari of the decision of the CA which set aside the Resolution of
the NLRC which in turn affirmed the Decision of the Labor Arbiter dismissing the complaint
filed by respondent Bustamante.
FACTS
- Petitioner Oscar Villamaria, Jr. was the owner of Villamaria Motors, a sole proprietorship
engaged in assembling passenger jeepneys with a public utility franchise to operate along
the Baclaran-Sucat route. By 1995, Villamaria stopped assembling jeepneys and retained
only nine, four of which he operated by employing drivers on a boundary basis. One of
those drivers was respondent. Bustamante remitted P450 a day to Villamaria as boundary
and kept the residue of his daily earnings as compensation for driving the vehicle. In
August 1997, Villamaria verbally agreed to sell the jeepney to Bustamante under the
boundary-hulog scheme, where Bustamante would remit to Villarama P550 a day for a
period of 4 years; Bustamante would then become the owner of the vehicle and continue
to drive the same under Villamarias franchise. It was also agreed that Bustamante would
make a downpayment of P10,000.
- On August 7, 1997, Villamaria executed a contract entitled Kasunduan ng Bilihan ng
Sasakyan sa Pamamagitan ng Boundary-Hulog over the passenger jeepney. The parties
agreed that if Bustamante failed to pay the boundary-hulog for 3 days, Villamaria Motors
would hold on to the vehicle until Bustamante paid his arrears, including a penalty of P50
a day; in case Bustamante failed to remit the daily boundary-hulog for a period of one
week, the Kasunduan would cease to have legal effect and Bustamante would have to
return the vehicle to Villamaria Motors.
- Bustamante continued driving the jeepney under the supervision and control of
Villamaria. As agreed upon, he made daily remittances of P550 in payment of the
purchase price of the vehicle. Bustamante failed to pay for the annual registration fees of
the vehicle, but Villamaria allowed him to continue driving the jeepney.
- In 1999, Bustamante and other drivers who also had the same arrangement with
Villamaria Motors failed to pay their respective boundary-hulog. This prompted Villamaria
to serve a Paalala, reminding them that under the Kasunduan, failure to pay the daily
boundary-hulog for one week, would mean their respective jeepneys would be returned to
him without any complaints. He warned the drivers that the Kasunduan would henceforth
be strictly enforced and urged them to comply with their obligation to avoid litigation. On
July 24, 2000, Villamaria took back the jeepney driven by Bustamante and barred the
latter from driving the vehicle.
- Bustamante filed a Complaint for Illegal Dismissal against Villamaria and his wife
Teresita. He narrated that in July 2000, he informed the Villamaria spouses that the
surplus engine of the jeepney needed to be replaced, and was assured that it would be
done. However, he was later arrested and his drivers license was confiscated because
apparently, the replacement engine that was installed was taken from a stolen vehicle. He
was no longer allowed to drive the vehicle unless he paid them P70,000.
ISSUES
1. WON the existence of a boundary-hulog agreement negates the employer-employee
relationship between the vendor and vendee
2. As a corollary, WON the Labor Arbiter has jurisdiction over a complaint for illegal
dismissal in such a case
HELD
1. NO
Ratio Under the boundary-hulog scheme, a dual juridical relationship is created: that of
employer-employee and vendor-vendee. The Kasunduan did not extinguish the
employer-employee relationship of the parties extant before the execution of said deed.

Labor Law 1
Reasoning
- The boundary system is a scheme by an owner/operator engaged in transporting
passengers as a common carrier to primarily govern the compensation of the driver, that
is, the latters daily earnings are remitted to the owner/operator less the excess of the
boundary which represents the drivers compensation. Under this system, the
owner/operator exercises control and supervision over the driver. It is unlike in lease of
chattels where the lessor loses complete control over the chattel leased but the lessee is
still ultimately responsible for the consequences of its use. The management of the
business is still in the hands of the owner/operator, who, being the holder of the certificate
of public convenience, must see to it that the driver follows the route prescribed by the
franchising and regulatory authority, and the rules promulgated with regard to the
business operations. The fact that the driver does not receive fixed wages but only the
excess of the boundary given to the owner/operator is not sufficient to change the
relationship between them. Indubitably, the driver performs activities which are usually
necessary or desirable in the usual business or trade of the owner/operator.
- Under the Kasunduan, respondent was required to remit P550 daily to petitioner, an
amount which represented the boundary of petitioner as well as respondents partial
payment (hulog) of the purchase price of the jeepney. Thus, the daily remittances also had
a dual purpose: that of petitioners boundary and respondents partial payment (hulog) for
the vehicle. This dual purpose was expressly stated in the Kasunduan. The well-settled
rule is that an obligation is not novated by an instrument that expressly recognizes the old
one, changes only the terms of payment, and adds other obligations not incompatible with
the old provisions or where the new contract merely supplements the previous one. The
two obligations of the respondent to remit to petitioner the boundary-hulog can stand
together.
- The existence of an employment relation is not dependent on how the worker is paid but
on the presence or absence of control over the means and method of the work. The
amount earned in excess of the boundary hulog is equivalent to wages and the fact that
the power of dismissal was not mentioned in the Kasunduan did not mean that private
respondent never exercised such power, or could not exercise such power.
- Neither is such juridical relationship negated by petitioners claim that the terms and
conditions in the Kasunduan relative to respondents behavior and deportment as driver
was for his and respondents benefit: to insure that respondent would be able to pay the
requisite daily installment of P550, and that the vehicle would still be in good condition
despite the lapse of 4 years. What is primordial is that petitioner retained control over the
conduct
of
the
respondent
as
driver
of
the
jeepney.
- As respondents employer, it was the burden of petitioner to prove that respondents
termination from employment was for a lawful or just cause, or, at the very least, that
respondent failed to make his daily remittances of P550 as boundary. However, petitioner
failed to do so. Well-settled is the rule that, the employer has the burden of proving that
the dismissal of an employee is for a just cause. The failure of the employer to discharge
this burden means that the dismissal is not justified and that the employee is entitled to
reinstatement and back wages.
2. YES
Reasoning
- The jurisdiction of Labor Arbiters and the NLRC under Article 217 of the Labor Code is
limited to disputes arising from an employer-employee relationship which can only be
resolved by reference to the Labor Code, other labor statutes or their collective bargaining
agreement.
Disposition Petition is DENIED. Decision of the CA is AFFIRMED.

ANINO V NLRC
PANGANIBAN; May 21, 1998
NATURE
Special Civil Action in the Supreme Court. Certiorari.
FACTS
- Complainants are supervisors of Hinatuan Mining Corporation (HMC) who planned the
formation of a supervisors union. The HINATUAN MINING SUPERVISORY UNION was
formally organized and registered with the DOLE. Complainants Anino, Navarro, Daugdaug and Filoteo were elected officers, while complainants Baladja and Ceredon were
active members of the union.
- On 3 November 1993, HIMSU formally notified the company of its legal existence
through a letter addressed to HMC President Zamora. It informed the company of its
desire for a collective bargaining agreement and submitted its proposals under letter dated
16 November 1993, which again was addressed to Zamora, VP-Operation Ganigan and
VP-Finance Nacorda. However, the company ignored these proposals.
- Union filed an unfair labor practice case against HMC on 13 May 1994.
- HMC dismissed the complainants under letter dated 16 June 1994.
- Labor Arbiter Legaspi held that the services of petitioners were illegally terminated,
ordered their reinstatement and the grant of back wages and attorneys fees equivalent to
10% of monetary award; that there was no positive showing that petitioners were
retrenched purposely to weaken or destroy their union; hence, claim of unfair labor

A2010

-9-

Disini

practice was dismissed. Likewise, claim for damages was denied since no fraud or bad
faith was committed by private respondents in dismissing them.
- NLRC reversed Legaspis ruling, rejected all petitioners claims and questioned
complainants actuations considering that they only challenged 2 months after dismissal
and after receiving separation pay. It also took judicial notice of the economic difficulties
suffered by the mining industry.
Petitioners Claim
- Dismissal was done with malicious intent to cause them and the union damage for their
exercise of the right to self-organization, in defiance of Labor Code Art. 248. Complainants
pray that respondents be: (a) declared guilty of unfair labor practices; (b) ordered to
reinstate complainants to their former positions with backwages and to pay complainants
jointly and severally the amount of P150k, as moral damages and litigation and attorney's
fees, respectively.
Respondents Comments
- Retrenchment was a management prerogative implemented in order to prevent further
losses. It affected rank-and-file, supervisors and managerial staffs and was done with due
notice to take effect 30 days from receipt thereof.
- Complainants had accepted separation pay equivalent to 1 month pay for every year of
service plus other monetary benefits, and complainants executed a waiver and quitclaim
for value received.
- Complaint was an afterthought in order to give semblance of credence to their
position/opposition to conduct a certification election, as manifested by complainants
counsel declaration in open court that they were still filing a new complaint for unfair labor
practice (this case)
ISSUES
1. WON the NLRC committed grave abuse of discretion amounting to lack or excess of
jurisdiction when it absolved respondents from their duty to prove losses as a just ground
for retrenchment
2. WON the NLRC exceeded its jurisdiction in recognizing the waivers/quitclaims executed
by petitioners as an effective bar to this complaint
3. WON the NLRC abused its discretion when it ordered the dismissal of the instant
complaint and totally disregarded the labor arbiters findings of facts and petitioners
motion for execution
HELD
1. YES
Ratio To justify retrenchment, the following requisites must be complied with: (a) the
losses expected should be substantial and not merely de minimis in extent; (b) the
substantial losses apprehended must be reasonably imminent; (c) the retrenchment must
be reasonably necessary and likely to effectively prevent the expected losses; and (d) the
alleged losses, if already incurred, and the expected imminent losses sought to be
forestalled must be proved by sufficient and convincing evidence.
Reasoning
- In termination cases, the burden of proving that the dismissal was for a valid or
authorized cause rests upon the employer.
- HMC merely claimed that retrenchment was undertaken to prevent losses due to the
continuing decline of nickel prices and export volume in the mining industry. Additionally, it
alleged that the reduction of excise taxes on mining from 5% to 1% on a graduated basis,
as provided under RA. 7729, was a clear recognition by the government itself of the
industry's worsening economic difficulties. These bare statements miserably fall short of
the requirements to show the validity of a retrenchment
- Even if, arguendo, the contentions of HMC are accepted at face value, they still fail to
satisfy the jurisprudential requirements that further or expected losses must be substantial
and reasonably imminent; and the dismissal of employees, reasonably necessary and
likely to be effective in preventing the expected losses.
2. YES
Ratio The acceptance of termination pay does not divest a laborer of the right to
prosecute his employer for unfair labor practice acts.
Reasoning
- Quitclaims and/or complete releases are against public policy and, therefore, null and
void.
- Employer and employee do not stand on the same footing. The employer drove the
employee to the wall. The latter must have to get hold of money. Because, out of job, he
had to face the harsh necessities of life. He thus found himself in no position to resist
money proffered. His, then, is a case of adherence, not of choice.
3. YES
Ratio A decision should faithfully comply with Sec. 14, Art. VIII of the Constitution. (No
decision shall be rendered by any court [or quasi-judicial body] without expressing therein
clearly and distinctly the facts of the case and the law on which it is based.)
Reasoning
- The NLRC was definitely wanting in the observance of the constitutional requirement. It
merely raised a doubt on the motive of the complaining employees and took "judicial
notice that in one area of Mindanao, the mining industry suffered economic difficulties."

Labor Law 1
- The factual and legal bases of public respondent's conclusions were bereft of substantial
evidence the quantum of proof in labor cases its disposition is manifestly a violation
of the constitutional mandate and an exercise of grave abuse of discretion. Such decision
is a nullity.
Disposition Petition granted; challenged NLRC Decision set aside. Decision of Legaspi
is reinstated except that Ganigan is not liable for petitioners monetary claims. HMC
ordered to pay separation benefits.
NOTES
- On reinstatement: If reinstatement to former position, or one substantially equivalent
thereto, is not feasible anymore, the employees are entitled to the grant of separation pay
and full back wages. Separation pay shall be equivalent to at least 1 month salary or 1
month salary for every year of service, whichever is higher, a fraction of 6 months being
considered as 1 whole year. It shall be computed from the date the petitioners were
employed by private respondent until this Decision becomes final and executory.
- On liability of Vice President: While the president of the erring company may be held
jointly and severally liable for the obligations of the latter to its dismissed employees, such
solidary liability does not extend to the vice president of the company. Absent any proof of
the extent of the participation of the VP in the formulation and the implementation of
management policies and programs, he cannot be held financially liable for the illegal
dismissal of employees.

EDI STAFF BUILDERS INTERNATIONAL V MAGSINO


MENDOZA; June 20, 2001
NATURE
Petition for review on certiorari
FACTS
- EDI is a recruitment agency. Dominguez is its President and Magsino was, until
dismissal, its supervisor of Processing and Documentation Group.
- The manager of the Processing and Documentation Grp sent a memo to Magsino,
saying that management received reports that Magsino withheld collected premium
payments for workers mandatory repatriation bond. Magsino was required to
clarify/explain.
- Magsino tendered resignation, but was held in abeyance pending result of the
investigation. Respondent was given notice of termination.
- Magsino filed complaint for illegal dismissal, nonpayment of salaries, leave pay, 13 th mo
pay, profit sharing, service award, maternity benefits.
- Labor Arbiter told EDI to reinstate Magsino. NLRC found EDIs claim unsupported by
evidence. CA also held that EDI couldnt present evidence on appeal for the first time, that
EDI failed to prove that Magsino was responsible for the discrepancy to justify her
termination. EDI insists Magsino was terminated for loss of trust and confidence.
ISSUES
1. WON NLRC correctly disregarded evidence on appeal
2. WON considering evidence, Magsino was dismissed for cause
HELD
1. NO
- No undue sympathy is to be accorded to any claim of a procedural misstep in labor
cases. Such must be decided accdg to justice and equity. Petitioners not implausibly
ascribed to the fault of counsel failure to file a position paper with Labor Arbiter. Court
deems it best to admit such evidence.
2. NO
- Court finds evidence insufficient to establish that Magsino was dismissed for loss of trust
and confidence. Petitioners simply alleged that Magsino failed to account for P201,600
without showing how that figure was arrived at.
- CA was right to order separation pay instead of reinstatement because of the strain in the
relationship of the employer and employee. Backwages, from the time of dismissal to the
time of finality of decision, must also be given.

GUSTILO V WYETH PHILIPPINES INC.


SANDOVAL-GUTIERREZ; October 4, 2004
FACTS
- Gustilo was employed by Wyeth Phils Inc. as a pharmaceutical territory manager.
- He was in-charge of the various branches in Metro Bacolod City and Negros Occidental.
- Among his tasks were visiting hospitals, pharmacies, drugstores and physicians;
preparing and submitting his pre-dated itinerary; and submitting periodic reports of his
daily call visits, monthly itinerary and weekly locator and incurred expenses.
- His employment records show that on various dates, Wyeth reprimanded and suspended
him for habitually neglecting to submit his periodic reports.

A2010

- 10 -

Disini

> Nov. 28, 1994- W sent a notice reprimanding G for the late submission of weekly
expense report
> July 5, 1995- late submission of same report so W suspended him for 5 days
> Oct 16 to 20, 23-27, Nov 6-10, 13-17, (all 1995)- late submission of his daily call
reports
> Nov 20-24, 1995- didnt submit his daily call reports so W suspended him for 15 days.
- Wyeth put Gustilo in charge of promoting 4 Lederle (Ws sister company)
pharmaceutical products. G then submitted to W a plan of action where G committed to
make an ave of 18 daily calls to physicians; submit promptly all periodic reports; and
ensure 95% territory program performance for every cycle.
- Gustilo failed to achieve his objectives so W sent him 2 notices charging him with willful
violation of company rules and regulations and directed him to submit a written
explanation.
- G explained that he was overworked and an object of reprisal by his immediate
supervisor, Filemon Verzano Jr.
- Wyeth, upon the recommendation of a review panel, terminated Gustilos services.
- G then filed with the Regional Arbiter Br. No. 6 in Bacolod City a complaint against W for
illegal suspension, illegal dismissal and payment for allowances, other monetary benefits,
damages and attys fees.
- The Labor Arbiter found that G was illegally dismissed from employment and ordered W
and Verzano to pay G jointly and severally Php 991,157.90 representing backwages,
separation pay, car reimbursement, damages and attys fees.
- W appealed to the NLRC in Cebu City
- NLRC- affirmed but modified the Labor Arbiters decision- ordered reinstatement of G, or
in lieu of reinstatement, pay his separation benefits.
- Ws MR was denied so they filed with the CA a petition for Certiorari and TRO and a writ
of preliminary injunction.
- CA- reversed NLRCs decision and dismissed Gs complaint for illegal dismissal (as G
was terminated based on A282 of the LC-gross and habitual neglect by the employee of
his duties) but awarded him separation pay considering the mitigating factors of length of
service, loyalty awards G received and Verzanos grudge against G.
- G filed an MR but was denied.
ISSUE
WON GUSTILO is entitled to his separation pay
HELD
NO, Gustilo isnt entitled to his SP OR to reinstatement as there was a just cause for
dismissal.
Reasoning
- Phil Journalists Inc v Mosqueda- SC ruled that the findings of the CA are conclusive
on the parties and not reviewable by this Court
- Family Planning Org of the Phils Inc v NLRC SC held that it is the employers
prerogative to prescribe reasonable rules and regulations necessary or proper for the
conduct of its business or concern to provide certain disciplinary measures to implement
said rules and to assure that the same be complied with. At the same time, it is one of the
fundamental duties of the employee to yield obedience to all reasonable rules, orders, and
instructions of the employer, and willful or intentional disobedience thereof, as a general
rule, justifies rescission of the contract of service and the preemptory dismissal of the
employee."
- Piedad v Lanao del Norte Electric Cooperative, Inc.- a series of irregularities when
put together may constitute serious misconduct, which under A282 of the LC, as
amended, is a just cause for dismissal.
- The rule embodied in the Omnibus Rules Implementing the Labor Code is that a person
dismissed for cause as defined therein is not entitled to separation pay.
- PLDT v NLRC and Abucay, "x x x henceforth, separation pay shall be allowed as a measure of social justice only
in those instances where the employee is validly dismissed for causes other than
serious misconduct or those reflecting on his moral character. Where the reason for
the valid dismissal is, x x x an offense involving moral turpitude x x x, the employer may
not be required to give the dismissed employee separation pay, or financial assistance, or
whatever other name it is called, on the ground of social justice."
- Telefunken Semiconductors Employees Union-FFW v Court of AppealsWe are of course aware that financial assistance may be allowed as a measure of
social justice in exceptional circumstances and as an equitable concession. We are
likewise mindful that financial assistance is allowed only in those instances where
the employee is validly dismissed for causes other than serious misconduct or
those reflecting on his moral character (Zenco Sales, Inc. vs. National Labor Relations
Commission, 234 SCRA 689). x x x."
- In the case at bar, there is NO exceptional circumstances to warrant the grant of
financial assistance or separation pay to petitioner.
G did not only violate company disciplinary rules and regulations. He falsified his
employment application form by not stating therein that he is the nephew of Mr.
Danao, respondent Wyeths Nutritional Territory Manager.

Labor Law 1
- G manifested his slack of moral principle through his infractions. In simple term,
he is dishonest.
- Philippine Long Distance Telephone vs. NLRC and Abucay- [T]hose who invoke
social justice may do so only if their hands are clean and their motives blameless x x x."
Here, petitioner failed to measure up to such requirement.
Disposition Petition is DENIED
***Wyeth did not interpose an appeal to this Court. Hence, no affirmative relief can be
extended to it. So it has to comply with the CAs decision to grant G his SP.

PHILCOM EMPLOYEES UNION V PHILIPPINE GLOBAL


CORPORATION AND PHILCOM CORPORATION
CARPIO; July 17, 2006
NATURE
Petition for review to annul the decision of the CA affirming Orders by the Secretary of
Labor
FACTS
- Jun 30, 1997 CBA expired
- Jul 1997 Negotions for renewal pending
- Oct 21, 1997 PEU filed notice of Strike; ground: unfair labor practices
- Company suspended CBA negotiations bec. of strike
- Nov 4, 1997 PEU filed 2nd notice of Strike; ground: bargaining deadlock
- Nov 11, 1997 National Conciliatory and Mediation Board (NCMB) proceedings resulting
in agreement to maintain status quo
- Nov 17, 1997 Proceedings ongoing, some PEU officers staged a strike
- Nov 19, 1997 Acting Labor Secretary Trajano assumed jurisdiction; issued Return-towork order
- Nov 28, 1997 Return-to-work order reiterated
- Oct 2, 1998 The Secretary of Labor and Employment issued 1st assailed Order
> Obiter: PEUs manifestation to implead PhilCom granted. (Note: Phil. Global is the
predecessor company of PhilCom resulting in a mere merger, not two separate entities;
thus, PhilCom is to be impleaded).
> PEUs charges of unfair labor practices dismissed. These included:
= misimplementation or non-implementation of employee benefits, non-payment of
overtime and other monetary claims, inadequate transportation allowance, water, and
other facilities
> PEU directed to cease and desist from strike and return to work; PhilCom ordered to
take back workers on strike under the same terms prior to the strike
- Nov 27, 1998 Motions for Reconsideration were filed and denied in its 2 nd assailed
Order
- PEU filed petition for certiorari with SC re: the Secretarys orders; SC referred the case to
the CA
- CA held:
> It is proper for the Secretary to take cognizance of the legality of the strike
> PhilComs acts do not constitute unfair labor practices.
ISSUES
1. WON it is proper for the Secretary of Labor to have taken cognizance of the issue on
the legality of the strike (issue of jurisdiction)
2. WON certain acts committed by PhilCom constitute unfair labor practices as
enumerated in Art. 248 of the Labor Code
3. WON the strike is illegal (a necessary offshoot of the issue of WON a writ of execution
should issue upon PhilCom to permit the PEU officers who participated in the illegal strike
to return to work)
HELD
1. YES
Ratio Since the very reason of the Secretarys assumption of jurisdiction was PEUs
declaration of the strike, any issue regarding the strike is not merely incidental to, but is
essentially involved in, the labor dispute itself.
Reasoning
- Art 263(g) of the Labor Code grants the Secretary of Labor the authority to assume
jurisdiction over a labor dispute causing or likely to cause a strike or lockout in an industry
indispensable to national interest. This includes and extends to all questions and
controversies arising from such labor dispute.
- The power is plenary and discretionary. He is granted great breadth of discretion
characterized as an exercise of police power.
- PEU cannot prevent resolution of the legality of the strike by merely refusing to submit
the issue for resolution. It is also immaterial that this issue, as PEU asserts, was not
properly submitted for resolution of the Secretary
2. NO

A2010

- 11 -

Disini

Ratio Unfair labor practice refers to acts that violate the workers right to self-organization
and to the observance of a CBA. Without that element, the acts, no matter how unfair, are
not unfair labor practices.
Obiter The only exception is Article 248(f), which in any case is not one of the acts
specified in PEUs charge of unfair labor practice
Reasoning
- A review of the acts complained of as unfair labor practices of PhilCom convinces us that
they do not fall under any of the prohibited acts defined and enumerated in Article 248 of
the Labor Code
- The acts assailed are all a matter of implementation or interpretation of the economic
provisions of the CBA between Philcom and PEU subject to the grievance procedure
- The Court has always respected a companys exercise of its prerogative to devise
means to improve its operations. Thus, it has held that management is free to regulate,
according to its own discretion and judgment, all aspects of employment, including hiring,
work assignments, supervision and transfer of employees, working methods, time, place
and manner of work
- This is so because the law on unfair labor practices is not intended to deprive employers
of their fundamental right to prescribe and enforce such rules as they honestly believe to
be necessary to the proper, productive and profitable operation of their business
- Even assuming arguendo that Philcom had violated some provisions in the CBA, there
was no showing that the same was a flagrant or malicious refusal to comply with the
CBAs economic provisions. Such showing is required for the acts to be considered unfair
labor practice
3. YES
Ratio The strike and strike activities were patently illegal for the following reasons:
Reasoning
- PhilCom, being in the communications industry, is engaged in a vital industry protected
from strikes and lockouts by PD 823 as amended by PD 849
- The Secretary had already assumed jurisdiction. Striking employees defied the return-towork order.
- Regardless of their motives, validity of claims, or pending motions, the striking
employees should have ceased and desisted from all acts undermining the authority
granted to the Secretary under Art. 263(g)
- A return-to-work order is immediately effective and executory despite the filing of a
motion for reconsideration. It must be strictly complied with even during the pendency of
any petition questioning its validity
- A return-to-work order imposes a duty that must be discharged more than it confers a
right that may be waived. While the workers may choose not to obey, they do so at the
risk of severing their relationship with their employer as it is valid ground for dismissal. Art.
264(a) governs defiance of such order.
1.
PEU staged the strike using unlawful means.
- PEU posted human barricades at all entrances to and egresses from the
company premises and used coercive methods to prevent company officials
and other personnel from leaving the company premises. Art. 264(a) prohibits
the commission of such acts during a strike and declares any worker or union
officer who knowingly participates to have lost his employment status.
- As PEU never disputed PhilComs assertions of unlawful strike, the former is
deemed to have admitted to such acts.
2.
PEU declared the strike (Nov 17) during the pendency of preventive mediation
proceedings at the NCMB (Nov 11).
Such is a blatant violation of Section 6, Book V, Rule XXII of the Omnibus Rules
Implementing the Labor Code, which explicitly obliges the parties to bargain
collectively in good faith and prohibits them from impeding or disrupting the
proceedings
- Article 264(a) of the Labor Code also considers it a prohibited activity to declare a
strike during the pendency of cases involving the same grounds for the same strike.
3.
PEU staged the strike in utter disregard of the grievance procedure established in
the CBA
- A strike declared on the basis of grievances which have not been submitted to the
grievance committee as stipulated in the CBA of the parties is premature and illegal.
- PEU could have just taken up their grievances in their negotiations for the new CBA
which was already pending
Disposition Petition is dismissed and the decision of the CA is affirmed. The issue of who
participated in the illegal strike, being questions of fact, must be resolved in
appropriate proceedings with the Secretary of Labor

DOLE PHILIPPINES INC V PAWIS NG MAKABAYNG


OBRERO
CORONA; January 13, 2003
[PAGE 3]
VALIAO V CA
QUISUMBING; July 30, 2004

Labor Law 1
NATURE
Petition for review on certiorari of the decision and resolution of the Court of Appeals
FACTS
- Petitioner Valiao was appointed by private respondent West Negros College (WNC) as
Student Affairs Office (SAO) Director, with a starting salary of P2,800 per month.
Subsequently, he was assigned as Acting Director, Alumni Affairs Offfice. He was
transferred to staff position and designated as Records Chief at the Registrars Office but
was again re-assigned as a typist. The latest reassignment was due to his tardiness and
absences, as reflected in the summary of tardiness and absences report, which showed
him to have been absent or late for work from a minimum of seven to maximum of 75
minutes for the period March to October 31, 1991 and to have reported late almost every
day for the period November to December 1991.
- Copies of his tardiness/absences reports were furnished petitioner, along with
memoranda requiring him to explain but his explanations were either unacceptable or
unsatisfactory. Subsequently, reports also showed that he did not change his habits
resulting in tardiness and absences. He was even caught one time manipulating the bundy
clock, thus necessitating another memorandum to him asking him to explain his dishonest
actuations in accomplishing the daily attendance logbook and in using the bundy clock. He
received a suspension order without pay for fifteen days because of dishonesty in
reporting his actual attendance. He reported back to office after serving the suspension
but was another adverse report on tardiness and absences was made against the
petitioner, prompting WNC to send him another memorandum with an attached tardiness
and absences report. Petitioner sent a letter of appeal and explained his side to the new
college president who gave petitioner another chance. The petitioner was appointed as
Information Assistant effective immediately. However, petitioner did not promptly assume
his post, prompting the president to call his attention.
- Subsequently, WNC won a case against the official of the union before the NLRC.
Petitioner was ordered to prepare a media blitz of this victory but the petitioner did not
comply with the order on the ground that such a press release would only worsen the
aggravated situation and strained relations between WNC management and the union
officials. When petitioner reported for work on the first day of January 1993, he was
relieved from his post and transferred to the College of Liberal Arts as Records Evaluator.
Not for long, the Dean of the Liberal Arts sent a letter to the Human Resources Manager
complaining about the petitioners poor performance and habitual absenteeism as shown
in the daily absence reports.
- On January 18, 1993, petitioner was again absent from work without permission or notice
to his immediate superior. It turned out that he went to Bacolod City and on January 28,
1993, the petitioner was one of those arrested during a raid in the house of Toto Ruiz, a
suspected drug pusher and was brought to the Bacolod Police Station along with four
other suspects. The petitioner and other suspects were then charged with violation of the
Dangerous Drugs Act of 1972.
- Petitioner was asked to explain within 24 hours why he should not be terminated as a
result of the raid and charged against him for violation of RA No. 6425 as amended.
Petitioner was dismissed for failure to answer said memorandum. Subsequently, he wrote
to the president of WNC explaining his side and asking for due process. The petitioner
was notified through a memorandum about the grant of his request and that a hearing
would be conducted. He was then placed under preventive suspension and an
investigation committee was organized to conduct the probe. A notice of
hearing/investigation was sent to the petitioner.
- After the investigation attended by the petitioner and his counsel, with proceedings duly
recorded, the investigation committee recommended the dismissal of petitioner. A notice of
termination was then sent to petitioner informing him of his termination from the service for
serious misconduct and gross and habitual neglect of duty. The petitioner received the
notice but did not file a grievance concerning the notice of termination.
- Petitioner filed a complaint against WNC for illegal suspension, illegal dismissal
backwages, salary differential for salary increases and other benefits granted after his
dismissal as well as for moral and exemplary damages and attorneys fees. After due
proceedings, the Labor Arbiter found no justifiable reason to place the petitioner under
preventive suspension as there was no serious or imminent threat to the life or property of
his coworkers. However, the Labor Arbiter found the dismissal of the petitioner to be valid
due to absenteeism and tardiness and after he was accorded the procedural due process
aspect of the law as reflected in the records showing that petitioner was formally
investigated and given the opportunity to refute the alleged findings by the management of
WNC. The Labor Arbiter held that frequent absenteeism and tardiness of the petitioner
constituted not only willful disobedience but also gross and habitual neglect of duties,
which are valid grounds for termination of employment. He stressed that petitioners
frequent absences without proper leave of absence was not only unfair to WNC and the
petitioners co-employees but also set an undesirable example to the employees under his
supervision, considering that the petitioner was not a mere rank-and-file employee but one
who owed more than the usual fealty to the organization.
- On appeal to the NLRC, the latter affirmed the decision of the Labor Arbiter, sustained
the latters findings of facts, and made its own findings of the apprehension of the

A2010

- 12 -

Disini

petitioner for possession of prohibited drugs. Petitioner then filed a Petition for Certiorari
under Rule 65 before the CA but this was dismissed for lack of merit. Petitioner duly filed a
Motion for Reconsideration, which was denied by the CA.
ISSUE
WON petitioner was validly dismissed from employment on the ground of serious
misconduct and gross habitual neglect of duties, including habitual tardiness and
absenteeism

HELD
YES
- So irresponsible an employee like petitioner does not deserve a place in the workplace,
and it is within the managements prerogative of WNC to terminate his employment. Even
as the law is solicitous of the welfare of employees, it must also protect the rights of an
employer to exercise what are clearly management prerogatives. As long as the
companys exercise of those rights and prerogatives is in good faith to advance its interest
and not for the purpose of defeating or circumventing the rights of employees under the
laws or valid agreements, such exercise will be upheld.
Disposition Assailed decision and resolution affirmed with modification.

ACUAV CA
QUISUMBING; May 5, 2006
FACTS
- Petitioners are Filipino overseas workers deployed by private respondent Join
International Corporation (JIC), a licensed recruitment agency, to its principal, 3D PreColor Plastic, Inc., (3D) in Taiwan, Republic of China, under a uniformly-worded
employment contract for a period of two years. Herein private respondent Elizabeth
Alaon is the president of Join International Corporation.
- September 1999 petitioners applied for employment abroad with JIC. They each paid
placement fee of P14,850. They signed a uniformly-worded employment contract which
stipulated that they were to work as machine operators with a monthly salary of
NT$15,840.00, exclusive of overtime, for a period of two years.
- December 9, 1999 they left for Taiwan. Upon arriving at the factory owned by 3D, they
were made to sign another contract which stated that their salary was only NT$11,840.00.
- They were also told that their dormitory was still under construction. In the meantime,
they were brought to a small room where forty women were jampacked and each person
was given a pillow. Since the ladies' comfort room was out of order, they had to ask
permission to use the men's comfort room. Petitioners claim they were made to work
twelve hours a day, from 8:00 p.m. to 8:00 a.m.
- December 16, 1999 due to unbearable working conditions, they informed management
that they were leaving. They booked a flight home, at their own expense. Before they left,
they were made to sign a written waiver. In addition, petitioners were not paid any salary
for work rendered on December 11-15, 1999.
- Immediately upon arrival in the Philippines, petitioners went to JIC, narrated what
happened, and demanded the return of their placement fees and plane fare. JIC refused.
On December 28, 1999, JIC offered a settlement. Mendez received P15,080; Acua and
Ramones received P13,640 and P16,200, respectively. They claim they signed a waiver,
otherwise they would not be refunded.
- January 2000 petitioners invoking Republic Act No. 8042, filed a complaint for illegal
dismissal and non-payment/underpayment of salaries or wages, overtime pay, refund of
transportation fare, payment of salaries/wages for 3 months, moral and exemplary
damages, and refund of placement fee before the National Labor Relations Commission
(NLRC).
- The Labor Arbiter ruled in favor of petitioners, declaring that Myrna Ramones, Juliet
Mendez and Mercedita Acua did not resign voluntarily from their jobs. Thus, private
respondents were ordered to pay jointly and severally, in Philippine Peso, at the rate of
exchange prevailing at the time of payment, the unexpired portion, salary for 4 days, and
overtime pay for 4 hrs in 4 days, refund of placement fee, and moral and exemplary
damages, and attorneys fees (10% of the award).The award which totaled NT$296,880.00
and P285,080.00
- The NLRC ordered that the amounts of P15,080, P13,640 and P16,200 received under
the quitclaim by Mendez, Acua and Ramones, respectively, be deducted from their
respective awards. They were awarded attorney's fees equivalent to ten percent (10%) of
their awarded labor-standards claims for unpaid wages and overtime pays. No moral and
exemplary damages and placement fees were awarded.
ISSUE
WON petitioners were illegally dismissed and are entitled to benefits plus damages
HELD
NO

Labor Law 1
No Constructive Dismissal
- The Labor Arbiter and the NLRC ruled that there was constructive dismissal because of
the unbearable conditions. Constructive dismissal covers the involuntary resignation
resorted to when continued employment becomes impossible, unreasonable or unlikely;
when there is a demotion in rank or a diminution in pay; or when a clear discrimination,
insensibility or disdain by an employer becomes unbearable to an employee. In this case,
the appellate court found that petitioners did not deny that the accommodations were not
as homely as expected. In the petitioners' memorandum, they admitted that they were told
by the principal, upon their arrival, that the dormitory was still under construction and were
requested to bear with the temporary inconvenience and the dormitory would soon be
finished. Likewise, the petitioners did not refute private respondents' assertion that they
had deployed approximately sixty other workers to their principal, and to the best of their
knowledge, no other worker assigned to the same principal has resigned, much less, filed
a case for illegal dismissal. These cited circumstances do not reflect malice by private
respondents nor do they show the principal's intention to subject petitioners to unhealthy
accommodations. Under these facts, there was no constructive dismissal.
Entitled to Overtime Pay
- The claim of overseas workers against foreign employers could not be subjected to same
rules of evidence and procedure easily obtained by complainants whose employers are
locally based. While normally we would require the presentation of payrolls, daily time
records and similar documents before allowing claims for overtime pay, in this case, that
would be requiring the near-impossible. It is private respondents who could have obtained
the records of their principal to refute petitioners' claim for overtime pay. By their failure to
do so, private respondents waived their defense and in effect admitted the allegations of
the petitioners.
- It is a time-honored rule that in controversies between a worker and his employer, doubts
reasonably arising from the evidence, or in the interpretation of agreements and writing
should be resolved in the worker's favor. The policy is to extend the applicability of the
decree to a greater number of employees who can avail of the benefits under the law,
which is in consonance with the avowed policy of the State to give maximum aid and
protection to labor. Accordingly, the private respondents are solidarily liable with the
foreign principal for the overtime pay claims of petitioners.
No Moral and Exemplary Damages
- Moral and exemplary damages are recoverable only where the dismissal of an employee
was attended by bad faith or fraud, or constituted an act oppressive to labor, or was done
in a manner contrary to morals, good customs or public policy. The person claiming moral
damages must prove the existence of bad faith by clear and convincing evidence, for the
law always presumes good faith. Petitioners allege they suffered humiliation, sleepless
nights and mental anguish, thinking how they would pay the money they borrowed for their
placement fees. Even so, they failed to prove bad faith, fraud or ill motive on the part of
private respondents. Moral damages cannot be awarded. Without the award of moral
damages, there can be no award of exemplary damages, nor attorney's fees.
Private Respondents Need Not Pay the Petitioners
- Quitclaims executed by the employees are commonly frowned upon as contrary to public
policy and ineffective to bar claims for the full measure of the workers' legal rights,
considering the economic disadvantage of the employee and the inevitable pressure upon
him by financial necessity. Nonetheless, the so-called "economic difficulties and financial
crises" allegedly confronting the employee is not an acceptable ground to annul the
compromise agreement unless it is accompanied by a gross disparity between the actual
claim and the amount of the settlement.
- The petitioners were not in any way deceived, coerced or intimidated into signing a
quitclaim waiver in the amounts of P13,640, P15,080 and P16,200 respectively. Nor was
there a disparity between the amount of the quitclaim and the amount actually due the
petitioners. Conformably then the petitioners are each entitled to NT$3,959.99 in
Philippine Peso at the rate of exchange prevailing at the time of payment. Since the
prevailing exchange rates on December 1999 was NT$1 to P1.268805, the amount of the
quitclaim paid to petitioners was actually higher than the amount due them.

ORIENTAL SHIP MANAGEMENT CO INC V CA


QUISUMBING; January 25, 2006
NATURE
Petition for review on certiorari
FACTS
- Petitioner Oriental Ship Management Co., Inc. (Oriental, for brevity) is a recruitment
agency duly licensed by the Philippine Overseas Employment Administration (POEA) to
recruit seafarers for employment on board vessels accredited to it. Kara Seal Shipping
Co., Ltd. (Kara Seal, for brevity) is petitioner's foreign-based principal, which owns and
manages M/V Agios Andreas, a vessel accredited to petitioner.
- Respondents Cuesta and Gonzaga were hired as Third Engineers on board the said
vessel for a one-year contract with a monthly salary of $900.

A2010

- 13 -

Disini

- On Nov. 1998, Kara Seal through its vessels Shipmaster signed an Agreement with the
International Transport Workers Federation (ITF for brevity) increasing the monthly salary
from $900 to $1,936.
- On Jan. 1999, an ITF inspector found out that the vessels crew have not been paid
properly. The Shipmaster assured him that the workers will be paid accordingly.
- Upon reaching Port Piombino, however, respondents were ordered repatriated to Manila
and before such repatriation, they were made to sign Letters of Indemnity saying the
contract of employment of the above crewmember is terminated by mutual agreement up
to 23rd January 1999, in the Port of Piombino (Italy).The seamen (sic) hereby
acknowledge has been received all what is due to him, arising from his employment on
board of the mentioned vessel; consequently he declares to have no claim whatever
against the Shipowner.
- Respondents received from Kara Seal payments not in accordance with the Agreement.
As such respondents filed Complaint against Oriental and Kara Seal for illegal dismissal.
- The Labor Arbiter dismissed the complaint saying The voluntariness of their resignation
is confirmed and reflected from the Letter of Indemnity they executed. They were executed
in the presence and with the participation of the ITF. ITF acts as the protector of seamen's
rights against any abuse or shortcomings of ship owners. They will not allow such
eventuality had the complainants been under duress. Besides, there is really no evidence
of threat or intimidation to the complainant's resignation. Accordingly, the validity of their
resignation and repatriation must be upheld.
- The NLRC affirmed the decision of the Labor Arbiter with modification as regards the
vacation pay of Cuesta. The NLRC also denied the Motion for Recognition.
- The CA reversed the NLRC decision saying that the Letters of Indemnity were void. The
CA also denied the MFR filed by Oriental and Kara Seal.
ISSUES
1. WON the quitclaims were valid
2. WON Cuesta and Gonzaga were illegally dismissed
HELD
1. NO
- Pacta privata juri publico derogare non possunct. Private agreements between parties
cannot derogate from public right.
- The law is solicitous of the welfare of employees because they stand on unequal footing
with their employers and are usually left at the mercy of the latter. This is especially true of
Filipino migrant workers who, alone in a foreign country, might have no adequate
alternative resources even for their own personal daily needs.
- Hence, quitclaims signed by our migrant workers, such as the Letters of Indemnity in the
instant case, are viewed with strong disfavor. Public policy dictates that they be presumed
to have been executed at the behest of the employer. It is the employer's duty to prove
that such quitclaims were voluntary. The employee's acknowledgment of his termination
with nary a protest or objection is not enough to satisfy the requirement of voluntariness
on his part.
- Resignation is defined as the voluntary act of an employee who finds himself in a
situation where he believes that personal reasons cannot be sacrificed in favor of the
exigency of the service, and he has no other choice but to disassociate himself from his
employment.
- It would have been illogical for respondents to resign and then claim that they were
illegally terminated. Well-entrenched is the rule that resignation is inconsistent with the
filing of a complaint for illegal dismissal.
- The Court noted that respondents Cuesta and Gonzaga, when repatriated to Manila, had
each been employed for only a little over two (2) months and less than one (1) month,
respectively. Prior to their repatriation, their monthly salaries were even increased from
US$900 to US$1,936. Hence, it is rather strange that they would suddenly resign after
barely beginning service of their twelve (12)-month contract.
- Based on the foregoing disquisition, The Sc is convinced that respondents were forced to
sign the Letters of Indemnity. Thus, said Letters of Indemnity must be deemed void. The
stamp and signature of the ITF representative thereon add nothing to render the letters of
any legal effect, but instead add to the impression of pressure exerted by ITF on the
individual Filipino seamen.
- Settled is the rule that quitclaims are ineffective in barring full recovery of the benefits
due the employee. The acceptance of any monetary benefit, such as repatriation
expenses and accrued wages in this case, would not divest respondents of the right to
fully claim the remainder of what is rightfully due them.
2. YES
- There was no justification for terminating their services and there was no due process as
Oriental did not serve two written notices to respondents prior to their termination from
employment, as required by the Labor Code.
- In this connection, paragraph 5; Section 10 of Republic Act No. 8042 provides:
- In case of termination of overseas employment without just, valid or authorized cause as
defined by law or contract, the worker shall be entitled to the full reimbursement of his
placement fee with interest at twelve percent (12%) per annum, plus his salaries for the
unexpired portion of his employment contract or for three (3) months for every year of the
unexpired term, whichever is less. The SC also noted that there is no evidence on record

Labor Law 1
of payment of placement fee. Hence, it is unable to award reimbursement of the same.
Cuesta is also entitled to vacation leave pay. Lastly, for petitioner's breach of contract and
bad faith, respondents should be awarded P50,000 in moral damages and another
P50,000 as exemplary damages. In addition, they should also be awarded attorney's fees
equivalent to ten percent (10%) of the aggregate monetary awards.
Disposition Petition is DENIED

PERIQUET V NLRC
CRUZ; June 22, 1990
NATURE
Petition to review the decision of the NLRC
FACTS
- The petitioner, Corazon Periquet, was dismissed as toll collector by the Construction
Development Corporation of the Philippines (CDCP), private respondent herein, for willful
breach of trust and unauthorized possession of accountable toll tickets allegedly found in
her purse during a surprise inspection.
- She filed a complaint for illegal dismissal claiming that she was framed
- Said complaint was sustained by the labor arbiter, who ordered her reinstatement within
ten days "without loss of seniority rights and other privileges and with full back wages to
be computed from the date of her actual dismissal up to date of her actual reinstatement.
- On appeal, the order was affirmed by the NLRC on August 29, 1980.
- On March 11, 1989, almost nine years later, the petitioner filed a motion for the
issuance of a writ of execution of the decision, which was granted by the executive labor
arbiter in an order dated June 26, 1989, requiring payment to the petitioner of the sum of
P205,207.42 "by way of implementing the balance of the judgment amount" due from the
private respondent.
- Said amount was garnished by the NLRC sheriff.
- On September 11, 1989, however, the NLRC sustained the appeal of the CDCP and set
aside the order dated June 20, 1989, the corresponding writ of execution of June 26,
1989, and the notice of garnishment.
- In its decision, the public respondent held that the motion for execution was time-barred,
having been filed beyond the five-year period prescribed by both the Rules of Court and
the Labor Code.
- It also rejected the petitioner's claim that she had not been reinstated on time and ruled
as valid the two quitclaims she had signed waiving her right to reinstatement and
acknowledging settlement in full of her back wages and other benefits. (Facts
relating to quitclaims italicized in reasoning)
- The petitioner contends that this decision is tainted with grave abuse of discretion and
asks for its reversal.
ISSUE
WON the NLRC committed grave abuse of discretion amounting to lack or excess of
jurisdiction when it held that the motion for execution was time-barred, and ruled as valid
the two quitclaims petitioner had signed
HELD
NO
On prescription
- Sec. 6, Rule 39 of the Revised Rules of Court, provides: A judgment may be executed on
motion within five (5) years from the date of its entry or from the date it becomes final and
executory. After the lapse of such time, and before it is barred by the statute of limitations,
a judgment may be enforced by action.
- A similar provision is found in Art. 224 of the Labor Code, as amended by RA 6715, viz.
ART. 224. Execution of decision, orders, awards. (a) The Secretary of Labor and
Employment or any Regional Director, the Commission or any Labor Arbiter or MedArbiter, or the Voluntary Arbitrator may, motu propio, or on motion of any interested party,
issue a writ of execution on a judgment within five (5) years from the date it becomes final
and executory, requiring a sheriff or a duly deputized officer to execute or enforce a final
decision, order or award.
- Periquet insists it was the private respondent that delayed and prevented the execution
of the judgment in her favor, but that is not the way the SC sees it.
- The original decision called for her reinstatement within ten days from receipt thereof
following its affirmance by the NLRC on August 29, 1980, but there is no evidence that she
demanded her reinstatement or that she complained when her demand was rejected.
What appears is that she entered into a compromise agreement with CDCP where
she waived her right to reinstatement and received from the CDCP the sum of

A2010

- 14 -

Disini

P14,000.00 representing her back wages from the date of her dismissal to the date
of the agreement
On validity of quitclaims
- After accepting the sum of P14,000.00 from the private respondent and waiving her right
to reinstatement in the compromise agreement, she applied for re-employment with the
CDCP and was on March 16,1987, given the position of xerox machine operator.
- On June 27, 1988; she wrote the new management of the CDCP and asked that the
rights granted her by the decision dated August 29, 1980, be recognized because the
waiver she had signed was invalid
- On September 19, 1988, the Corporate Legal Counsel of the private respondent
recommended the payment to the petitioner of the additional sum of P9,544.00,
representing the balance of her back pay for three years at P654. 00 per month
- On November 10, 1988, the petitioner accepted this additional amount and signed
another Quitclaim and Release
- In her petition she is now disowning both acknowledgments
- Not all waivers and quitclaims are invalid as against public policy. If the agreement
was voluntarily entered into and represents a reasonable settlement, it is binding on
the parties and may not later be disowned simply because of a change of mind. It is
only where there is clear proof that the waiver was wangled from an unsuspecting
or gullible person, or the terms of settlement are unconscionable on its face, that
the law will step in to annul the questionable transaction. But where it is shown that
the person making the waiver did so voluntarily, with full understanding of what he
was doing, and the consideration for the quitclaim is credible and reasonable, the
transaction must be recognized as a valid and binding undertaking.
Disposition Petition denied

EMCO PLYWOOD CORP V ABELGAS


PANGANIBAN; April 14, 2004
NATURE
Petition for review on certiorari of CA decision declaring EMCOs attempted retrenchment
of its employees as legally ineffective
FACTS
- EMCO is a domestic corporation engaged in the business of wood processing,
operating through its sawmill and plymill sections where respondents used to be assigned
as regular workers.
- On Jan 20, 1993 and Mar 2, 1993, EMCO, represented by its Gen Manager Lim,
informed the DOLE of its intention to retrench some of its workers on the ground of
purported financial difficulties.
- EMCO then issued a memorandum, addressed to all its foremen, section heads,
supervisors and department heads, with the instruction of retrenching some workers
based on the ff guidelines:
a) Old Age (58 years and above except positions that are really skilled);
b) Performance (Attitude, Attendance, Quality/ Quantity of Work)
- Per EMCOs notice to the DOLE, 104 workers were proposed for inclusion in its
retrenchment program. EMCO terminated 250 workers.
- Those terminated then received their separation pay of P4,815 each. But deductions
were made by EMCO purportedly for the attorneys fees payable to respondents lawyer,
for his efforts in renegotiating an increase in the wages contained in their CBA.
- Upon receipt of such pay, respondents were made to sign quitclaims, releasing
EMCO and all its officers from all forms of actions/suits, debts, sums of money, etc.
- About 2 years later, they then lodged a complaint, through their labor union, against
EMCO for illegal dismissal, damages and attys fees.
- EMCO interposed the defense of lack of cause of action; respondents had waived
whatever claims they may have against the corporation after signing the quitclaims in
favor of EMCO.
- LABOR ARBITER dismissed the complaint.
- Appeal to the NLRC was also dismissed. It anchored its dismissal on the effect of the
respondents waivers or quitclaims. There is no doubt that the respondents voluntarily
executed their quitclaims/waivers as manifested by the fact that they did not promptly
question their validity within a reasonable time. It took them two (2) years to challenge and
dispute the validity of the waivers by claiming belatedly that they were either forced or
misled into signing the same.
CA Ruling
- EMCO did not comply with one-month prior notice requirement under LC:
a) Memorandum merely provided the guidelines on the conduct of the intended lay-off; this
did not constitute notice.
b) It was not addressed to the workers, but to the foremen, the department supervisors
and the section heads.
c) There was no proper notice to DOLE. EMCO terminated the services of 250 employees
but included only 104 of them in the list it filed with DOLE.
- Before EMCO resorted to retrenchment, it failed to adduce evidence of its losses and
prove that it had undertaken measures to prevent the occurrence of such losses.

Labor Law 1
- EMCO had not paid the legally prescribed separation pay. EMCO violated the LC in
deducting the amount of attorneys fees. 3
- Employees cause of action had not yet prescribed when the case was filed, because an
action for illegal dismissal constituted an injury to their rights. (Art.1146 of NCC is
applicable: 4 yrs prescription period)
ISSUES
1. WON petitioner EMCO had substantially complied with the requisites for a valid
retrenchment
2. WON respondents had voluntarily executed their respective Quitclaims
3. WON the CA may, in a petition for certiorari, correct the evaluation of evidence made by
both the Labor Arbiter and the NLRC
HELD
1. NO
Ratio Retrenchment4 is one of the authorized causes for dismissal of employees, resorted
to by employers to avoid or minimize business losses. It is only "a measure of last resort
when other less drastic means have been tried and found to be inadequate." (Guerrero v
NLRC)
* Standards to justify retrenchment and to avoid abuse:
a) Losses expected should be substantial
b) Losses must be reasonably imminent, as such imminence can be perceived objectively
and in good faith by the employer
c) Retrenchment is reasonably necessary and likely to effectively prevent the expected
losses. Employer should have taken other measures prior or parallel to retrenchment to
forestall losses
d) Alleged losses if already realized, and expected losses sought to be forestalled, must
be proved by sufficient and convincing evidence
Reasoning
- employer bears the burden of proving the existence or imminence of substantial losses
with clear and satisfactory evidence. The evidence submitted by EMCO does not
persuade the SC:
a) Audited financial statements for the years 1991 and 1992. EMCOs net income of
P1.052M for 1991 decreased to P880T in 1992. The F/S also demonstrate that EMCOs
liability then increased from P106.5M to P123M.
* BUT in Somerville Stainless Steel Corp v NLRC, SC held that the presentation of F/S for
a particular year was inadequate to overcome the stringent requirement of the law. Also, in
the analysis of F/S, one particular percentage of relationship may not be too significant in
itself; that is, it may not suffice to point out those unfavorable characteristics of the
company that would require immediate or even drastic action."
b) EMCO undertook preventive measures to prevent the occurrence of imminent losses; it
implemented a work scheme on a rotation basis.
* BUT it did not try other measures, such as cost reduction, lesser investment on raw
materials, adjustment of the work routine to avoid the scheduled power failure, etc.
c) The 146 employees not included in the list submitted to DOLE voluntarily resigned.
* BUT resignation is the voluntary act of employees who are compelled by personal
reasons to dissociate themselves from their employment. It would have been illogical for
respondents to resign and then file a Complaint for illegal dismissal.
2. NO
Ratio The mere fact that the employees were not physically coerced or intimidated does
not necessarily imply that they freely or voluntarily consented to the terms thereof. (Phil
Carpet Employees Assoc v Phil Carpet Manufacturing Corp)
- The corporation, and not its employees, has the burden of proving that the Quitclaims
were voluntarily entered into. (Salonga v NLRC)
- Because the retrenchment was illegal and of no effect, the Quitclaims were therefore not
voluntarily entered into by respondents. Consent was similarly vitiated by mistake or fraud.
(Trendline Employees Assoc-Southern Phil Federation of Labor (TEA-SPFL) v NLRC)
Reasoning

"Article 222. APPEARANCES AND FEES. (b) No attorneys fees, negotiation fees or similar charges of any kind
arising from any collective bargaining negotiations or conclusion of the collective bargaining agreement shall be imposed
on any individual member of the contracting union: Provided, however, That attorneys fees may be charged against
union funds in an amount to be agreed upon by the parties. Any contract, agreement or arrangement of any sort to the
contrary shall be null and void."

ART. 283. CLOSURE OF ESTABLISHMENT AND REDUCTION OF PERSONNEL. The employer may also
terminate the employment of any employee due to the installation of labor saving devices, redundancy, retrenchment to
prevent losses or the closing or cessation of operation of the establishment or undertaking unless the closing is for the
purpose of circumventing the provisions of this Title, by serving a written notice on the worker and the Ministry of Labor
and Employment at least one (1) month before the intended date thereof.
In case of termination due to the installation of labor saving devices or redundancy, the worker affected thereby shall be
entitled to a separation pay equivalent to at least his one (1) month pay or at least one (1) month pay for every year of
service, whichever is higher. In case of retrenchment to prevent losses and in cases of closure or cessation of operations
of establishment or undertaking not due to serious business losses or financial reverses, the separation pay shall be
equivalent to one (1) month pay or at least one-half (1/2) month pay for every year of service, whichever is higher. A
fraction of at least six (6) months shall be considered as one (1) whole year.

A2010

- 15 -

Disini

- EMCO claimed that aside from Eddie de la Cruz, the other respondents did not submit
their respective supporting affidavits detailing how their individual consents had been
obtained. Allegedly, such documents do not constitute the clear and convincing evidence
required under the law to overturn the validity of Quitclaims. But the SC held that the
burden of proof is actually on the part of EMCO.
- As a rule, deeds of release or quitclaim cannot bar employees from demanding benefits
to which they are legally entitled. Acceptance of those benefits would not amount to
estoppel; however, amounts already received are to be deducted from their respective
monetary awards.
3. YES
Ratio The lower tribunals factual findings will not be upheld where there is a showing that
such findings were totally devoid of support, or that the judgment was based on a
misapprehension of facts.
Disposition Petition is DENIED (EMCO is ordered to REINSTATE employees with full
backwages, inclusive of allowances and other benefits)

ANTAMOC GOLDFIELDS MINING COMPANY V CIR


IMPERIAL; June 28, 1940
NATURE
Petition for review by means of certiorari.
FACTS
The National Labor Union, representing the workers of Antamok Goldfield Mining
Company, sent a letter to management demanding higher pay and better working
conditions. Management accepted some of their demands and rejected the others.
Consequently the workers went on strike. The Department of Labor intervened and an
amicable settlement between the parties was entered into. Despite this, another strike was
subsequently held. A stoning incident occurred which resulted in the dismissal of forty-five
workers. The matter was heard in the Court of Industrial Relations (CIR) where witnesses
for both petitioners and respondents testified. The CIR ordered one of its special agents to
proceed to the premises of the mines and to conduct further investigation. " The
investigation disclosed that the precipitate and unwarranted dismissal of the forty-five men
after the incident seems to have been spurred by an over anxious desire on the part of the
company to get rid of these men. It was also found out that more than 400 workers of
different classes among them, mockers, miners, timbermen, trammers and capataces
coming from different mines in the region have been employed by Antamok as fresh
laborers and that almost all, if not all, of these men are not members of the the National
Labor Union, Inc." The CIR ruled that the discharges and indefinite suspensions were
made by Antamok without first securing the consent of the CIR in violation of a previous
order enjoining them from discharging any laborer involved in the dispute without just
cause and without previous authority of the Court. Antamok insists in its right of selecting
the men that it should employ and that in the exercise of this right it should not be
restrained or interfered with by the CIR. Consequently, they assail the validity of
Commonwealth Act No. No. 103, which created the CIR, on the ground that it deprives
them of liberty and property without due process of law.
ISSUE
WON Commonwealth Act No. 103 is unconstitutional
HELD
NO
- In Commonwealth Act No. 103, and by it, our Government no longer performs the role of
a mere mediator or intervenor but that of the supreme arbiter. The policy of laissez faire
has to some extent given way to the assumption by the government of the right of
intervention even in contractual relations affected with public interests. Justice Laurel in
Ang Tibay, and National Workers Brotherhood v Court of Industrial Relations, and National
Labor Union, Inc. states that our Constitution was adopted in the midst of surging unrest
and dissatisfaction resulting from economic and social distress which was threatening the
stability of governments the world over. Embodying the spirit of the present epoch, general
provisions were inserted in the Constitution which are intended to bring about the needed
social and economic equilibrium between component elements of society through the
application of what may be termed as the justitia communis advocated by Grotius and
Leibnits many years ago to be secured through the counterbalancing of economic and
social forces and opportunities which should be regulated, if not controlled, by the State or
placed, as it were, in custodia societatis. 'The promotion of social justice to in sure the
well-being and economic security of all the people' was thus inserted as vital principle in
our Constitution. (Sec. 5, Art. II, Constitution.) And in order that this declaration of principle
may not just be an empty medley of words, the Constitution in various sections thereof has
provided the means towards its realization, For instance, section 6 of Article XIII declares
that the State 'shall afford protection to labor, especially to working women and minors,
and shall regulate the relations between landowner and tenant, and between labor and
capital in industry and in agriculture.' The same section also states that 'the State may
provide for compulsory arbitration.' In extraordinary cases mentioned in section 16, Article

Labor Law 1
VI, of the Constitution, the President of the Philippines may be authorized by law, for a
limited period and subject to such restrictions as the National Assembly may prescribe, to
'promulgate rules and regulations to carry out a declared national policy.' Albeit, almost at
the same time the Congress of the United States approved the National Labor Regulations
Act (49 Stat., 449) on July 5, 1935, commonly known as the Wagner Act, we were in the
Philippines headway towards the adoption of our fundamental law, pursuant to
congressional authority given in the Tydings-McDuffie Independence Act, approved March
24, 1934. In our Bill of Rights we now find the following provision 'The right to form
associations or societies for purposes not contrary to law shall not be abridged.' (Par. 6,
section 1, art. III, Constitution.) What was an agitation in the United States which brought
about the recommendation by the Commission on Industrial Relations created by an Act of
Congress in 1912 for the adoption of a Labor Bill of Rights as an amendment to the United
States Constitution is, in our case, virtually an accepted principle, which may be expanded
and vitalized by legislation to keep pace with the development of time and circumstances.
- By and large, these provisions in our Constitution all evince and express the need of
shifting emphasis to community interest with a view to affirmative enhancement of human
values. In conformity with the constitutional objective and cognizant of the historical fact
that industrial and agricultural disputes had given rise to disquietude, bloodshed and
revolution in our country, the National Assembly enacted Commonwealth Act No. 103,
entitled 'An Act to afford protection of labor by creating a Court of Industrial Relations
empowered to fix minimum wages for laborers and maximum rental to be paid by tenants,
and to enforce compulsory arbitration between employers or landlords, and employees or
tenants, respectively; and by prescribing penalties for the violation of the orders' and, later,
Commonwealth Act No. 213, entitled, 'An Act to define and regulate legitimate labor
organizations.'
- Commonwealth Act No. 213 was enacted in pursuance of what appears to be the
deliberate embodiment of a new social policy, founded on the conception of a society
integrated not by independent individuals at dealing at arms length, but by interdependent
members of a consolidated whole whose interests must be protected against mutual
aggression and warfare among and between divers and diverse units which are impelled
by countervailing and opposite individual and group interests, and this is particularly true in
the relationship between labor and capital. Social and industrial disturbances which fifty
years ago were feudal-like and of isolated importance may now well result in a serious
strain upon the entire economic organism of the nation. Several attempts at meeting and
solving our peculiar social and economic problems have already been made. The system
of voluntary arbitration devised by Act No. 4055 of the defunct Philippine Legislature has
apparently been abandoned by the enactment of the aforementioned Commonwealth Acts
Nos. 103 and 213.

SAROCAM V INTERORIENT MARITIME ENT. INC. AND


DEMACO UNITED LTD.
CALLEJO, SR.; June 27 2006
NATURE
Petition for Review on certiorari under Rule 45 of the ROC of the CA Decision in CA-G.R.
SP No. 84883, which affirmed the February 19, 2004 and April 27, 2004 Resolutions of the
NLRC.
FACTS
- On June 27, 2000 petitioner Benjamin L. Sarocam was hired by Interorient Maritime Ent.,
Inc. and Demaco United Ltd., for a twelve-month contract as bosun on board M/V
Despina.
- While the vessel was navigating to China, petitioner suffered lumbar sprain when he
accidentally fell from a ladder. On Nov.15, 2000, he was examined and found to have
neuromyositis with the waist and diabetes. The examining physician prescribed medicine
and recommended the signing off and hospitalization of petitioner. His employers agreed
to repatriate him on Nov. 30, 2000.
- On Dec. 5, 2000, petitioner was referred to the company-designated physician, Dr.
Pidlaoan. Petitioner was given Alaxan tablet for his back pain and Euglocon for his
elevated blood sugar. He was also advised to return for follow-up evaluation. On Dec. 13,
2000, he returned to the clinic with no more complaints of back pains and his sugar
examination revealed normal results. Petitioner was then declared fit for duty effective on
that
day.
- On March 20, 2001, petitioner executed a release and quitclaim in favor of his employers
where he acknowledged the receipt of US$405.00 as his sickwages and freed his
employers
from
further
liability.
- However, on Nov. 27, 2001, petitioner filed a complaint with the NLRC for disability
benefit, illness allowance/reimbursement of medical expenses, damages and attorneys
fees. To support his claim, he presented medical certificates issued by his 3 personal
doctors, recommending a Grade VIII disability under the POEA schedule of disability
grading.
- On July 11, 2003, Labor Arbiter Macam dismissed the complaint, holding that petitioner
was not entitled to disability benefits because he was declared fit for duty and had

A2010

- 16 -

Disini

previously executed a release and quitclaim in favor of his employers and already
received his sickness allowance. Petitioners claim for moral damages and attorneys fees
were, likewise, not awarded on the Labor Arbiters ruling that there was no evidence of
bad faith and malice on the part of the employers.
- Upon petitioners appeal, the NLRC issued a Resolution affirming the decision of the
Labor Arbiter, with the modification that petitioner was entitled to US$1,350.00 or its peso
equivalent, representing his salary for three (3) months. The petitioners motion for
reconsideration was denied by the NLRC. The Petition for Certiorari filed with the CA was
dismissed. Petitioners MFR was denied by the CA.
- Petitioner avers that the quitclaim he executed is invalid, as the amount he received as
consideration therefor was much lower than what he should have received under the
POEA Standard Employment Contract. He went on to argue that quitclaims are frowned
upon by this Court as they are contrary to public policy.
ISSUES
1. WON the respondents company-designated doctor be considered competent and
reliable enough to declare petitioner as fit to work contrary to the declarations of three (3)
independent physicians similarly finding him otherwise
2. WON the execution by petitioner of a release and quitclaim estop him from claiming
disability benefits under the POEA standard employment contract
HELD
1. YES
- Petitioner did not question the findings of Dr. Pidlaoan and his recommendation. He
questioned the doctors competency and the correctness of his findings only when he filed
the complaint against respondents before the Labor Arbiter, roughly 11 months after
petitioner was examined by the doctor. Petitioner consulted his personal doctors only in
July and August 2001, long after he had been examined by the company-designated
physician.
- Dr. Pidlaoan examined and treated petitioner from the time he was repatriated up to his
recovery and subsequent assessment as fit for duty on December 13, 2000. As in the
German Marine case, the extensive medical attention extended by Dr. Pidlaoan enabled
the latter to acquire familiarity, if not detailed knowledge, of petitioners medical condition.
No doubt such specialized knowledge enabled Dr. Pidlaoan to arrive at a much more
accurate appraisal of petitioners condition, as compared to another physician not privy to
petitioners case from the very beginning. Indeed, the assessment of the three other
personal doctors of petitioner could not have been that reliable considering that they
based their conclusions on the prior findings of Dr. Pidlaoan; moreover, they examined
petitioner 7 or 8 months after he was assessed as fit to work and treated him for only one
day.
- Furthermore and most importantly, petitioner did not question the competency of Dr.
Pidlaoan and his assessment when the latter declared him as fit for duty or fit to work.
- Additionally, petitioner, instead of questioning the assessment of the companydesignated doctor, executed a release and quitclaim in favor of respondents, around three
months after the assessment. In executing the said document, petitioner thus impliedly
admitted the correctness of the assessment of the company-designated physician, and
acknowledged that he could no longer claim for disability benefits.
2. YES
- While petitioner may be correct in stating that quitclaims are frowned upon for being
contrary to public policy, the Court has, likewise, recognized legitimate waivers that
represent a voluntary and reasonable settlement of a workers claim which should be
respected as the law between the parties. Where the person making the waiver has done
so voluntarily, with a full understanding thereof, and the consideration for the quitclaim is
credible and reasonable, the transaction must be recognized as being a valid and binding
undertaking.
- In the instant case, petitioner wrote the release and quitclaim with his own hand. From
the document itself, the element of voluntariness in its execution is evident. Petitioner also
appears to have fully understood the contents of the document he was signing, as the
important provision thereof had been relayed to him in Filipino.
- Not all waivers and quitclaims are invalid as against public policy. If the agreement was
voluntarily entered into and represents a reasonable settlement, it is binding on the parties
and may not later be disowned simply because of a change of mind. It is only where there
is clear proof that the waiver was wangled from an unsuspecting or gullible person, or the
terms of settlement are unconscionable on its face, that the law will step in to annul the
questionable transaction. But where it is shown that the person making the waiver did so
voluntarily, with full understanding of what he was doing, and the consideration for the
quitclaim is credible and reasonable, the transaction must be recognized as a valid and
binding undertaking.
- As a final note, let it be emphasized that the constitutional policy to provide full protection
to labor is not meant to be a sword to oppress employers. The commitment of this Court
to the cause of labor does not prevent us from sustaining the employer when it is in the
right.
Disposition Petition is DENIED for lack of merit. The Decision and Resolution of the CA
are AFFIRMED.

Labor Law 1
PHILIPPINE AIRLINES, INC. V SANTOS
REGALADO; February 4, 1993
NATURE
Petition for certiorari assailng the NLRC decision in favor of the private respondents
(holding that there was illegal suspension, that the respondents be paid their salaries
corresponding to the suspension period, and that disciplinary action from the respondents
service records be deleted)
FACTS
- The private respondents are all Port Stewards in the Catering Sub-Department of the
Passenger Services Department of PAL whose jobs were to prepare meal orders and
checklists, set up standard equipment in accordance with the requirements of the type of
service for each flight; to ski, bin and make an inventory of Commissary supplies and
equipment.
- On several occasions, deductions were made from their salaries allegedly representing
losses of inventoried items charged to them for mishandling of company properties.
- August 21, 1984: The respondents, through the union, made a formal notice of the
deductions to PAL through the Manager for Catering, Mr. Reynaldo Abad. However, no
action was taken by PAL.
- November 4, 1984: Pursuant to the grievance machinery Step 1 of the CBA between PAL
and the union, respondents filed a formal grievance.
- November 21: The said grievance was submitted to the office of Mr. Abad who was on
leave.
- December 5: Mr. Abad was still on leave, and since in the CBA, Mr. Abad (PAL) was
supposed to resolve the grievance within 5 days, the respondents thru the shop steward
wrote a letter addressed to Mr. Abads office expressing their belief that the grievance was
deemed resolved in their favor
- December 7: Mr. Abad (finally) returned and scheduled a meeting on December 12
- the respondents refused to conduct their inventory works thereafter
- December 12: Mr. Abad and the union had the meeting where the former denied the
petition of the respondents, adopting the position that it was the inventory of goods was
the respondents duty and that the deductions in their salaries were due to the losses in
the mishandling of goods
- Due to the respondents refusal to conduct inventory works in early December, Mr. Abad
issued an inter-office memo asking them to explain why no disciplinary action should be
taken against them. The respondents argued that since their grievance in accordance with
the grievance machinery step 1 of their CBA was not resolved within the 5-day period,
they believed that the grievance was resolved in their favor. Mr. Abad found this reasoning
unsatisfactory, THUS suspending the said respondents.
- the union filed another grievance asking for the lifting of the suspension, but PAL denied
the said lifting, only reducing the suspension period for respondent Ramos.
- the union demanded for the reimbursement of the salaries of individual respondents
during the suspension but PAL denied their demand.
- the respondents filed a complaint for ILLEGAL SUSPENSION before the Arbitration
Branch of the NLRC. Complaint dismissed, rule in favor of PAL
- Respondents appealed to NLRC, NLRC ruled in favor of respondents
- PALs petition for reconsideration denied, hence this case
Petitioners claims (PAL)
- The CBA provision on grievance machinery was established both for the union and the
management (PAL), therefore, should NOT be narrowly interpreted; it is the employees
duty to observe status quo (therefore, cant preempt that the decision is resolved in their
favor); the management should be given chance to present their side since before the 5
day prescriptive period begins to run, there should first be the presentment of grievance
and its discussion
ISSUES
1. WON NLRC acted with grave abuse of discretion amounting to lack of jurisdiction in
setting aside the Arbitration Branchs decision in favor of PAL
2. WON Section 2, Article IV of the PAL-PALEA CBA5 should be narrowly interpreted,
THEREFORE favoring the respondents (the prescriptive period runs after the filing of the
grievance)
5

Sec. 2 Processing of Grievances


xxx xxx xxx
STEP 1 Any employee who believes that he has a justifiable grievance shall take the matter up with his shop steward.
If the shop steward feels there is justification for taking the matter up with the Company, he shall record the grievance on
the grievance form heretofore agreed upon by the parties. Two (2) copies of the grievance form properly filled, accepted,
and signed shall then be presented to and discussed by the shop steward with the division head. The division head shall
answer the grievance within five (5) days from the date of presentation by inserting his decision on the grievance form,
signing and dating same, and returning one copy to the shop steward. If the division head fails to act within the five (5)day regl(e)mentary period, the grievance must be resolved in favor of the aggrieved party. If the division head's decision
is not appealed to Step II, the grievance shall be considered settled on the basis of the decision made, and shall not be
eligible for further appeal.

A2010

- 17 -

Disini

HELD
1. NO
- It has not been shown that respondent NLRC has unlawfully neglected the performance
of an act which the law specifically enjoins it to perform as a duty or has otherwise
unlawfully excluded petitioner from the exercise of a right to which it is entitled.
Ratio Judicial review by this Court in labor cases does not go so far as to evaluate the
sufficiency of the evidence upon which the labor officer or office based his or its
determination, but is limited to issues of jurisdiction and grave abuse of discretion
2. YES
- Even if Mr. Abad was on leave when the grievance was filed (and even if the union and
respondents knew that Mr. Abad was on leave), the CBA would still apply since it is hard to
believe that everything under Abad's authority would have to stand still during his absence
from office. To be sure, it is to be expected that someone has to be left to attend to Abad's
duties.
Ratio The sympathy of the Court is on the side of the laboring classes, not only because
the Constitution imposes such sympathy, but because of the one-sided relation between
labor and capital. The constitutional mandate for the promotion of labor is as explicit as it is
demanding. The purpose is to place the workingman on an equal plane with management
with all its power and influence in negotiating for the advancement of his interests
and the defense of his rights. Under the policy of social justice, the law bends over
backward to accommodate the interests of the working class on the humane justification
that those with less privileges in life should have more privileges in law. (in short,
interpretation should be made in favor of the laborers)
Reasoning
- The grievance of employees is not a matter which requires the personal act of Mr. Abad
and thus could not be delegated. Petitioner could at least have assigned an officer-incharge to look into the grievance and possibly make his recommendation to Mr. Abad. It is
of no moment that Mr. Abad immediately looked into the grievance upon returning to work,
for it must be remembered that the grievants are workingmen who suffered salary
deductions and who rely so much on their meager income for their daily subsistence and
survival.
- when the respondents first presented their complaint on August 21, the petitioner
(through Mr. Abad) failed to act on it
- if the provision would be interpreted as to allow the management to act on their laborers
complaints after the acting officer returned from a leave then the causes of the
workingmen would be delayed, thus suffering a great injustice. That could not have been
the intendment of the pertinent provision of the CBA, much less the benevolent policy
underlying our labor laws.
Disposition petition is hereby DENIED and the assailed decision of respondent National
Labor Relations Commission is AFFIRMED. This judgment is immediately executory.

CALALANG V WILLIAMS
LAUREL; December 2, 1940
FACTS
- The Secretary of Public Works and Communications (PWC) approved with modification
the recommendation that originated from the National Traffic Commission (NTC), which
was favorably indorsed by the Director of Public Works (PW), that Rosario Street and
Rizal Avenue be closed to traffic of animal-drawn vehicles, between the points and during
the hours from 7 a.m. to 11 p.m., for a period of one year from the date of the opening of
the Colgante Bridge to traffic; that the Mayor of Manila and the Acting Chief of Police of
Manila have enforced and caused to be enforced the rules and regulations thus adopted;
that as a consequence of such enforcement, all animal drawn vehicles are not allowed to
pass and pick up passengers in the places above-mentioned to the detriment not only of
their owners but of the riding public as well.
- Commonwealth Act No. 548 gives the Director of Public Works, with the approval of the
Secretary of the Public Works and Communications the authority to promulgate rules and
regulations to regulate and control the use of and traffic on national roads.
- Maximo Calang, in his capacity as private citizen and as a taxpayer of Manila, filed a
petition for a writ of prohibition against the Chairman of NTC, Director of PW, Acting
Secretary of PWC, Mayor of Manila and Acting Chielf of Police of Manila.
ISSUES
1. WON Commonwealth Act No. 548 is unconstitutional because it constitutes an undue
delegation of legislative power
2. WON the rules and regulations promulgated constitute an unlawful interference with
legitimate business or trade and abridge the right to personal liberty and freedom of
locomotion
3. WON the rules and regulations complained of infringe the upon the constitutional
precept regarding the promotion of social justice to insure the well-being of all the people
HELD
1. NO

Labor Law 1
- The Legislature cannot delegate power to make law; but it can make a law to delegate a
power to determine some fact or state of things upon which the law makes, or intends to
make, its own action depend.
Reasoning
- adherence to precedent
Rubi vs. Provincial Board of Mindoro, Wayman vs. Southard it was held here that
discretion may be delegated to executive departments or subordinate officials the
execution of certain acts, final on questions of fact.
- textual interpretation of Commonwealth Act No. 548
The provision that .the Director of Public Works, with the approval of the Secretary of
the Public Works and Communications, shall promulgate rules and regulations to
regulate and control the use of and traffic on national roads, is an administrative
function which cannot be directly discharged by the National Assembly.
- practicality
The complexities of modern governments, the multiplication of the subjects of govtl
regulations, and the increased difficulty in administering the law give rise to the
adoption, within certain limits, the delegation of greater powers by the legislative and
vesting a larger amount of discretion in administrative and executive officials, not only
in the execution of the laws, but also in the promulgation of certain rules and
regulations.
2. NO
- The state may enact laws that may interfere with personal liberty, with property, and with
business and occupation if the said laws are intended to promote the welfare of the public.
(police power of the State)
Reasoning
- precedents (US vs. Gomez, Dobbins vs. Los Angeles & People vs. Pomar)
- Paradox - The apparent curtailment of liberty is precisely the very means of insuring its
preservation
- No. Social justice is promoted if the greatest good is brought about to the greatest
number.

PLDT V NLRC
ROMERO; July 23, 1997
NATURE
Petition for certiorari to revoke NLRCs Nov. 16, 1992 decision affirming the resolution of
Labor Arbiter Jose De Vera and denying petitioners motion for reconsideration
FACTS
- Private respondent Lettie Corpuz was employed as traffic operator at the Manila
International Traffic Division (MITD) by PLDT for 10 years 9 months from Sept. 19, 1978
until her dismissal on June 17, 1989. She was tasked with facilitating requests for
incoming and outgoing international calls using a digital switchboard.
- Sometime in Dec. 1987, PLDTs rank-and-file employees went on strike, prompting MITD
to discharge the formers duties to prevent a shutdown of its operations. In the course of
their assignments, 2 supervisors received 2 overseas calls bound for the Middle East, both
callers reporting the same calling number (98-68-16). It was shown that the number had
been permanently disconnected on Sept. 1987 but 439 overseas calls had been made
through it from May to Nov. 1987.
- It was further found that among the 235 telephone operators who handled those calls
(averaging 1.8% calls each), private respondent had handled 12.8% of the total calls.
Some calls, though registered as partly unavailable or busy, yielded unusually long
operator call durations. Private respondent also used said number to make several
personal calls. Based on these finding, MITD Manager Erlinda Kabigting directed
respondent to explain these allegations.
- instead of complying, respondent requested a formal investigation to confront and rebut
the witnesses allegations. On grounds of misconduct and breach of trust, respondent was
terminated.
- In a complaint for illegal dismissal filed by respondent, Labor Arbiter Jose De Vera
rendered a decision ordering the reinstatement of private respondent, later affirmed by
NLRC.
ISSUE
WON the NLRC erred in ordering the reinstatement of private respondent
HELD
NO. Although the power to dismiss is a normal prerogative of the employer, the right to
discharge employees is regulated by the States police power in line with its duty to
preserve its citizens rights.
- Petitioner insists that respondent was guilty of defrauding them by taking several calls
through the disconnected number. However, records show that these calls were neither
unusual nor made in connivance with certain subscribers as other operators shared similar
experiences. Although it is quite certain that there were certain PLDT personnel who

A2010

Disini

- 18 -

tampered with the line, the ultimate blame cannot be set solely on private respondent
based on mere suspicion, but only with concrete and substantial evidence.
- In the instant case, the petitioner failed to establish valid bases of the alleged
misconduct, thus denying private respondent her right to due process. The requirement of
notice and hearing affords the worker ample opportunity to be heard and defend himself.
- Art. 4 of the Labor Code states that all doubts in the implementation and interpretation of
the provisions of the Labor Code including its implementing rules and regulations shall be
resolved in favor of labor, that is, the workers welfare is of paramount importance. The
Constitution furthers that the State shall afford full protection to labor, promote full
employment opportunities for all and guarantee the right to security of tenure.
Disposition instant petition is DISMISSED and the decision dated Nov. 16, 1992 is
AFFIRMED.

AGABON V NLRC
DAYAN V BPI
VITUG; November 20, 2001
NATURE
Review seeking reversal of the decision and resolution of CA reversing the resolution of
the NLRC.
FACTS
- Petitioner Rogelio C. Dayan (Dayan) started his employment on 30 June 1956 with the
Commercial Bank and Trust Company (CBTC). CBTC was eventually absorbed by the
Bank of the Philippine Islands. (BPI) where Dayan was maintained as employee. In 1981,
Dayan was promoted Administrative Assistant by respondent bank in its centralized
accounting office. He held several positions thereafter - Assistant Manager of Internal
Operations in 1983, Assistant Manager of Correspondent Bank in 1988, Assistant
Manager of Branch Operations in 1990, Assistant Manager of the Supplies Inventory in
1991, and then Senior Assistant Manager of the Supplies Inventory in 1991-1992. In
addition to the series of promotions, Dayan was the recipient of various commendations.
- December 1991, the post of Purchasing Officer became vacant. The vacated position
was offered to Dayan which he initially declined but, due to the insistence of his
superiors, he later accepted on a temporary basis in February 1993.
- 10 June 1993, Asst VP Gerlanda E. De Castro of the bank, in a memorandum, placed
petitioner under suspension.
- Dayan is placed under suspension due to matters presented to him in a meeting on the
same morning of the suspension memo.
- It appears that BPI conducted earlier interviews regarding supposed malpractices
committed by Dayan during his term as Purchasing Officer. The report signed ad noted by
Rololfo Bernejo (Mgr) and Victor Guillermo (Sr Mgr) contained alleged misconduct such
as asking for 5% commission on purchase orders, donations totaling 5K for medical bills,
overpricing BPI Family Banks passbook, etc. The report also made negative findings and
observations about his work performance.
- 14 June 1993, petitioner wrote a memorandum to the bank narrating what had transpired
in his meeting with the bank on 10 June 1993 where he denied all the accusations
against him and contested his preventive suspension. His denials and plea for
compassion notwithstanding, petitioner was dismissed by respondent bank via a notice
of termination, dated 25 October 1993, signed by AVP Gerlanda de Castro. In a letter of
confession, dated 28 October 1993, petitioner ultimately admitted his infractions and
instead asked for financial assistance. He, at the same time, executed an undated
"Release Waiver and Quitclaim" acknowledging receipt of P400,000.00 financial
assistance from the bank and thereby releasing and discharging it from any action or claim
arising from his employment with the bank and membership in the retirement plan.
- Subsequently, however, petitioner claimed that the letter and the quitclaim were
signed by him under duress. On 14 February 1994, he filed a case for Illegal Dismissal
and Illegal Suspension, with a prayer for an award of retirement benefits, before the Labor
Arbiter.
- In his decision of 30 June 1995, the Labor Arbiter upheld the validity of the dismissal
of petitioner based on loss of trust and confidence and denied his claim for retirement
benefits and damages.
- On appeal, the NLRC reversed the decision of the labor arbiter and declared the
dismissal to be illegal on the ground that petitioner was denied due process ratiocinating
that a hearing should have been afforded petitioner for a chance to confront the witnesses
against him.
- BPI filed with SC, a petition for certiorari questioning the NLRC decision. The Court
referred the petition to CA. The appellate court reversed the judgment of the NLRC.
- In its petition for review before the SC, petitioner argues that the CA has wrongly relied
on unsworn statements taken by the bank from its contractual employees. Petitioner
believes that the factual conclusions of the NLRC which has acquired expertise on the
matters entrusted to it should have instead been respected by the appellate court.
ISSUES

Labor Law 1
1. WON CA committed an error in granting Dayans dismissal
2. WON there sufficient compliance of notice and hearing
3. WON he should be reinstated in BPI
4. WON the letter and quitclaim were obtained through deception and coercion

HELD
1. NO
Ratio The CA was convinced that Dayans guilty of malfeasance and that the petitioner's
dismissal had been justified under Article 282 of the Labor Code.6
Reasoning
- CA did not commit error in holding to be justifiable the dismissal of Dayan from BPI as
evidence of malpactice is too numerous to be ignored. Contrary to Dayans claim, the
suppliers who complained executed affidavits as part of the records of the case. An
employee under his supervision even narrated other incidents of malpractices. These
charges were even backed up by the audit report of the banks audit team.
- Dayan is not a rank and file employee. His job involves much exercise of independent
judgment and discretion. A bank, being essentially imbued with public interest, cannot be
compelled to continue in its employ a person whom it has lost trust and confidence.
Obiter
- The policy of preventively suspending an employee under investigation for charges
involving dishonesty is an acceptable precautionary measure in order to preserve the
integrity of vital papers and documents that may be material and relevant to the case and
to which he, otherwise, would have access by virtue of his position. It was only after an
exhaustive investigation that respondent bank finally decided to terminate the services of
petitioner on 25 October 1993.
2. NO
Ratio The law requires the employer with 2 written notices before termination can be
legally affected as well as a hearing where the employee can explain his side.
A consultation or conference with the employee is not a substitute for the actual
observance of notice and hearing.
Reasoning
- The first notice apprises the employee of the particular acts or omissions for which the
dismissal is sought. The second notice informs the employee of the employers decision to
dismiss him.
- In the case at bench, Dayan was called to a meeting June 10 where he denied all
charges against him. After which, he was issued a notice of preventive suspension. After
investigation, he was given notice of dismissal. There was failure on the part of BPI to
conform to the notice and hearing requirement. The preliminary meeting is not sufficient
compliance rather it was merely exploratory. Where the employee denies charges against
him, a hearing is necessary to thresh out any doubt.
3. NO
Ratio The 2 notice and hearing rule is indispensable for a dismissal to be validly effected,
but if it is for a just and valid cause, the failure to observe procedural requirements
does not invalidate the dismissal of the employee. Instead, he must be granted
separation pay. Whether reinstated or given separation pay, he should be paid
backwages if he has been laid off without written notice 30 days in advance. For
the omission, an appropriate sanction should be imposed depending on the fact
and gravity of the situation.
Reasoning
- In the case at hand, the purpose of the notice and hearing requirement is not to
comply with due process.
- Art 283 originated from the Spanish Code of Commerece of 1882, which gave either
party to the employer-employee relationship the right to terminate their relationship by
giving notice to the other a month in advance. This was repealed by Art. 2270 of the Civil
Code, then by RA 1052 or Termination Pay Law, and finally by RA1787 providing for the
advance notice or payment of compensation at the rate of month for every year of
service.
- The Termination Pay Law is a regulatory measure to give opportunity for the employer to
look for a replacement or substitute and for the employee to look for another job. The
notice was not required if the dismissal is for just cause. The notice requirement is only
implemented by BP130 amending the Labor Code.
- The employer cannot be expected to be an entirely impartial judge of his own cause.
4. NO
6

"`(c) Fraud or willful breach by the employee of the trust reposed in him by his employer or duly authorized
representative.'

A2010

- 19 -

Disini

Ratio Quitclaims executed by employees are commonly frowned upon as contrary to


public policy and ineffective to bar claims for the full measure of a workers legal
rights. However, if the agreement was voluntarily entered into and represents a
reasonable settlement, it is binding on the parties and may not be disowned unless
there is clear proof that the waiver was wangled from an unsuspecting gullible
person, or the terms are unconscionable on its face.
Reasoning
- Dayan is a managerial employee with vast experience. He cannot be willing to
compromise his future by agreeing to execute a document highly prejudicial to his interest.
Complainant was well aware of the consequences of his acts.
Disposition decision of the Court of Appeals reinstating the decision of the Labor Arbiter
and setting aside the NLRC's decision is AFFIRMED.

MANILA ELECTRIC COMPANY V QUISUMBING


MARTINEZ; January 27, 1999
NATURE
Petition for certiorari, MERALCO seeking to annul the orders of the Sec. of Labor to
execute a collective bargaining agreement (CBA)
FACTS
- Meralco Workers Association (MEWA) is a duly recognized labor organization of the
rank-and-file employees of MERALCO. On Sept. 7, 1995, it informed MERALCO of its
intention to renegotiate the terms and conditions of their existing 1992-97 CBA covering
the remaining period of 2 years starting from December 1, 1995 to November 30, 1997.
- MERALCO signified its willingness to re-negotiate through a letter and formed a CBA
negotiating panel for the purpose. Bargaining negotiations proceeded. However, despite
the series of meetings bet. The negotiating panels, the parties failed to arrive at "terms
and conditions acceptable to both of them.
- On April 23, 1996, MEWA filed a Notice of Strike with the National Capital Region Branch
of the National Conciliation and Mediation Board (NCMB) of the Department of Labor and
Employment (DOLE). on the grounds of bargaining deadlock and unfair labor practices.
The NCMB then conducted a series of conciliation meetings but the parties failed to reach
an amicable settlement. Faced with the imminence of a strike, MERALCO filed an urgent
petition with the DOLE praying the Secretary to assume jurisdiction over the labor dispute
and to enjoin the striking employees to go back to work. The Secretary did so and
conducted conciliation conferences between the parties to bridge their differences.
Thereafter, the parties submitted their respective memoranda and on August 19, 1996, the
Secretary resolved the labor dispute through an Order.
- Dissatisfied, MERALCO filed this petition contending that the Sec. of Labor gravely
abused its discretion in awarding wage increases and other economic benefits (like 2
months xmas bonus, loan for the employees cooperative, signing hours, 40-day union
leave, sick leave, etc.), in expanding the scope of the bargaining unit to all regular rank
and file employees, in exercising discretion in determining the retroactivity of the CBA,
etc.
ISSUES
WON the Secretary failed to properly considered and appreciated the evidence presented
before him, thus committing a grave abuse of discretion
HELD
YES
- the Secretary of Labor disregarded and misappreciated evidence, particularly with
respect to the wage award. The Secretary of Labor apparently also acted arbitrarily and
even whimsically in considering a number of legal points; even the Solicitor General
himself considered that the Secretary gravely abused his discretion on at least three major
points: (a) on the signing bonus issue; (b) on the inclusion of confidential employees in the
rank and file bargaining unit, and (c) in mandating a union security "closed-shop" regime in
the bargaining unit.
- On the wages issue: the Secretary gravely abused his discretion in making this wage
award because he disregarded evidence on record. MERALCO projection had every
reason to be reliable because it was based on actual and undisputed figures the union
projection was based on a speculation of Yuletide consumption that the union failed to
substantiate. The amount of the wage increase would also be an unreasonably high
burden for MERALCO to shoulder.
- On the economic issues:
> Christmas bonus - As a rule, a bonus is not a demandable and enforceable obligation; ,it
may nevertheless be granted on equitable considerations as when the giving of such
bonus has been the company's long and regular practice, To be considered a "regular
practice," the giving of the bonus should have been done over a long period of time, and
must be shown to have been consistent and deliberate. We can not, however, affirm the
Secretary's award of a two-month special Christmas bonus to the employees since there

Labor Law 1
was no recognized company practice of giving a two-month special grant. The two-month
special bonus was given only in 1995 in recognition of the employees' prompt and efficient
response during the calamities. Instead, a one-month special bonus, We believe, is
sufficient, this being merely a generous act on the part of MERALCO.
> RICE SUBSIDY and RETIREMENT BENEFITS for RETIREES
> EMPLOYEES' COOPERATIVE
> GHSIP, HMP
BENEFITS FOR DEPENDENTS and HOUSING EQUITY LOAN
> SIGNING BONUS, RED-CIRCLE-RATE ALLOWANCE
> SICK LEAVE RESERVE OF 15 DAYS, 40-DAY UNION LEAVE
- Non-economic issues
> Scope of the bargaining unit - employees holding a confidential position are prohibited
from joining the union of the rank and file employees
> Security demand - the Secretary cannot rule on the union security demand because this
is not .1 mandatory subject for collective bargaining agreement
> UNION REPRESENTATION IN COMMITTEES
> RETROACTIVITY OF THE CBA
Ratio The Secretary of Labor's statutory power under Art. 263 (g) of the Labor Code to,
assume jurisdiction over a labor dispute in an industry indispensable to the national
interest, and, to render, an award on compulsory arbitration, does not exempt the exercise
of this power from the judicial review. Under this constitutional mandate, every legal
power of the Secretary of Labor under the Labor Code, or, for that matter, any act of the
Executive, that is attended by grave abuse of discretion is subject to review by this Court
in an appropriate proceeding. To be sure, the existence of an executive power alone whether granted by statute or by the Constitution cannot exempt the executive action from
judicial oversight, interference or reversal when grave abuse of discretion is, or is alleged
to be, present. Thus, the actions of the Sec. of Labor is subject to judicial review.
- The extent of judicial review over the Secretary of Labor's arbitral award is not limited to
a determination of grave abuse in the manner of the secretary's exercise of his statutory
powers. This Court is entitled to, and must - in the exercise of its judicial power - review
the substance of the Secretary's award when grave abuse of discretion is alleged to exist
in the award, i.e.; in the appreciation of and the conclusions the Secretary drew from the
evidence presented. The natural and ever present limitation on the Secretary's acts is, of
course, the Constitution. But in this case we believe that the more appropriate and
available standard - and one does not require a constitutional interpretation - is simply the
standard of reasonableness. In layman's terms, reasonableness implies the absence of
arbitrariness
Disposition petition is granted and the orders of public respondent Secretary of Labor
dated August 19, 1996 and December 28, 1996 are set aside to the extent set forth above.
The parties are directed to execute a Collective Bargaining Agreement incorporating the
terms and conditions contained in the unaffected portions of the Secretary of Labor's
orders of August 19, 1996 and December 28, 1996, and the modifications set forth above.
The retirement fund issue is remanded to the Secretary of Labor for reception of evidence
and determination of the legal personality of the MERALCO retirement fund.

AGABON V NLRC
SAROCAM V INTERORIENT MARITIME ENT. INC. AND
DEMACO UNITED LTD.
CALLEJO, SR.; June 27 2006
[PAGE 16]
ASUNCION V NLRC
KAPUNAN; July 31, 2001
FACTS
- Asuncion was employed as an accountant/bookkeeper by the respondent Mabini Medical
Clinic.
- Officials of the DOLE conducted a routine inspection of the company and discovered
upon the disclosure of Asuncion violations of the labor standards law. The company was
made to correct these violations.
- Dr. Juco charged Asuncion with offenses such Chronic Absentism, Habitual tardiness,
Loitering, Disobedience and insubordination and consequently dismissed Asuncion on the
ground of disobedience of lawful orders and for her failure to submit her reply within the
two-day period.
- Labor Arbiter Caday rendered judgment declaring that the petitioner was illegally
dismissed.
- On appeal, NLRC rendered the assailed decision which set aside the Labor Arbiters
ruling.
ISSUE
WON NLRC erred in finding that Asuncion was dismissed by the Company for a just or
authorized cause

A2010

- 20 -

Disini

HELD
YES
- There is lack of evidence to establish the charges of absenteeism and tardiness.
- A workers employment is property in the constitutional sense. He cannot be deprived of
his work without due process. In order for the dismissal to be valid, not only must it be
based on just cause supported by clear and convincing evidence, the employee must also
be given an opportunity to be heard and defend himself. It is the employer who has the
burden of proving that the dismissal was with just or authorized cause. The failure of the
employer to discharge this burden means that the dismissal is not justified and that the
employee is entitled to reinstatement and backwages.
- Company submitted mere handwritten listing and computer print-outs. The handwritten
listing was not signed by the one who made the same. The handwritten listing and
unsigned computer print-outs were unauthenticated and, hence, unreliable.
- Company failed to present a single piece of credible evidence to serve as the basis for
their charges against Asuncion and consequently, failed to fulfill their burden of proving the
facts which constitute the just cause for the dismissal of the petitioner.
- Asuncions letter did not amount to an admission of her alleged absences. Her alleged
absences were incurred on Saturdays. These should not be considered as absences as
there was an arrangement between her and the private respondents that she would not be
required to work on Saturdays.
- Neither had the Company shown by competent evidence that Asuncion was given any
warning or reprimanded for her alleged absences and tardiness.
- The two-day period given to Asuncion to explain and answer the charges against her
was most unreasonable, considering that she was charged with several offenses and
infractions (35 absences, 23 half-days and 108 tardiness), some of which were allegedly
committed almost a year before, the charges leveled against her lacked particularity.
- The law mandates that every opportunity and assistance must be accorded to the
employee by the management to enable him to prepare adequately for his defense.
If doubts exist between the evidence presented by the employer and the employee, the
scales of justice must be tilted in favor of the latter. The employer must affirmatively show
rationally adequate evidence that the dismissal was for a justifiable cause.
- Asuncion has been illegally terminated, she is necessarily entitled to reinstatement to her
former previous position without loss of seniority and the payment of backwages.

EXECUTIVE SECRETARY V CA
CALLEJO, SR.; May 25, 2004
NATURE
Appeal from a decision of the Court of Appeals
FACTS
- The Asian Recruitment Council Philippine Chapter, Inc. (ARCO-Phil) filed on July 17,
1995 a petition for declaratory relief under Rule 63 0f the Rules of Court with the RTC of
Quezon City to declare as unconstitutional portions of RA 8042, otherwise known as the
Migrant Workers and Overseas Filipinos Act of 1995 with a plea for the issuance of a
temporary restraining order and/or a writ of preliminary injunction enjoining the
government from enforcing the said portions of the law. The questioned portions of the
said RA deal with illegal recruitment, penalties for illegal recruitment, and on the venue of
criminal action for illegal recruitment.
- On August 1, 1995, the trial court issued a temporary restraining order on the
implementation or effectivity of the questioned provisions based on the allegations of the
private respondents that they will suffer grave or irreparable damage or injury if the law is
implemented.
- ARCO-Phil was joined in the petition by eleven other corporations which were allegedly
members of the organization when it filed an amended petition. The amended petition also
questioned other sections of the law which dealt with the overseas deployment only of
skilled Filipino workers alleging discrimination against unskilled workers.
- The trial court issued a writ of preliminary injunction on August 21, 1995 upon a bond of
Pesos 50,000.
- Petitioners filed a petition with the court of Appeals assailing the order and the writ with
the court of Appeals on the grounds that respondent, ARCO-Phil, is not the real party-ininterest and that it has not shown any convincing proof that in fact damage or injury would
result in the implementation of the questioned statute. The Court however dismissed the
petition. It subsequently dismissed petitioners motion for reconsideration.
- Hence, the petition for review on certiorari to the Supreme Court.
ISSUES
1. WON private respondents have standing to file suit
2. WON the trial court committed grave abuse of discretion amounting to excess or lack of
jurisdiction in issuing the assailed order and the writ of preliminary injunction on a bond of
only Pesos 50,000

Labor Law 1
3. WON the appellate court erred in affirming the trial courts order and the writ it issued
HELD
1. The SC ruled that the respondents have locus standi citing it earlier ruling in
Telecommunications and Broadcast Attorneys of the Philippines vs Commission of
Elections. It was held in that case that standing jus tertii would be recognized if it can be
shown that the party suing has some substantial relation to the third party, or that the right
of the third party would be diluted unless the party in court is allowed to espouse the third
partys constitutional rights. With regard the portion relating to discrimination against
unskilled workers, the SC ruled that respondents have no standing as they failed to
implead any unskilled worker in their petition.
2. The order and the writ of preliminary injunction issued by the trial court is a grave abuse
of its discretion amounting to excess or lack of jurisdiction. The SC citing jurisprudence
ruled that a law is presumed constitutional until the same is declared unconstitutional by
judicial interpretation. This is so because suspension of the operation of the law is an
interference with the official acts of the duly elected representatives of the people and also
of the highest magistrate of the land. The possible unconstitutionality of a statue, on its
face, does not of itself justify an injunction against good faith attempts to enforce it, unless
there is showing of bad faith, harassment, or any other unusual circumstances that would
call for equitable relief. To be entitled to a preliminary injunction to enjoin the enforcement
of a law assailed to be unconstitutional, the party must establish that it will suffer
irreparable harm in the absence of injunctive relief and must demonstrate that it is likely to
succeed on the merits, or that there are sufficiently serious questions going to the merits
and the balance of hardships tips decidedly in its favor. This higher standard reflects
judicial deference towards legislation or regulations developed through presumptively
reasoned democratic process. In this case, none were shown.
3. The SC also held that the assailed order and writ of preliminary injunction is mooted by
case law. The SC cited various cases it had earlier decided on apply RA 8042. By these
rulings, the SC, in effect, affirmed the validity of the assailed provisions. Hence the
enforcement of the provisions cannot be enjoined unless the SC, by final judgment
declares the provisions to be unconstitutional.

ANG TIBAY, AND NATIONAL WORKERS


BROTHERHOOD V CIR, AND NATIONAL LABOR
UNION, INC.
LAUREL; February 27, 1940
NATURE
The respondent National Labor Union, Inc., prays for the vacation of the judgment
rendered by the majority of this Court and the remanding of the case to the Court of the
Industrial Relations for a new trial.
The petitioner, Ang Tibay, has filed an opposition to the motion for reconsideration of the
respondent National Labor Union, Inc.
FACTS
- CIR created by Commonwealth Act No. 103. Its functions are specifically stated therein
- Nature of the CIR:
> more administrative than part of judicial system
> not a mere receptive organ of Govt, not passive
> active: not just judicial/quasi-judicial in disputes, but also has jurisdiction over the
entire Philippines to consider, investigate, decide, settle any question, matter,
controversy or dispute arising between, and/or affecting employers and employees
or laborers, and regulate the relations between them.
> mingling of executive and judicial functions (a departure from the rigid doctrine of
the separation of governmental powers)
> not narrowly constrained by technical rules of procedure: its required to act
according to justice and equity and substantial merits of the case, without regard to
technicalities or legal forms (Section 20, Commonwealth Act No. 103)
> The fact that the CIR may be said to be free from certain procedural requirements
doe not mean that it can entirely ignore or disregard the fundamental requirements
of due process in trials.
- Primary rights which must be respected even in proceedings of this character:
(1) the right to a hearing, which includes the right of the party interested or affected to
present his own case and submit evidence in support thereof. Morgan v. U.S "the liberty
and property of the citizen shall be protected by the rudimentary requirements of fair play.
(2) Not only must the party be given an opportunity to present his case and to adduce
evidence tending to establish the rights which he asserts but the tribunal must consider
the evidence presented. Edwards vs. McCoy, "the right to adduce evidence, without the
corresponding duty on the part of the board to consider it, is vain. Such right is
conspicuously futile if the person or persons to whom the evidence is presented can thrust
it aside without notice or consideration."
(3) "While the duty to deliberate does not impose the obligation to decide right, it does
imply a necessity which cannot be disregarded, namely, that of having something to

A2010

- 21 -

Disini

support it is a nullity, a place when directly attached." (Edwards vs. McCoy, supra.) Law is
both a grant and a limitation upon power.
(4) Not only must there be some evidence to support a finding but the evidence must be
"substantial." -such relevant evidence as a reasonable mind accept as adequate to
support a conclusion." The statute provides that "the rules of evidence prevailing in courts
of law and equity shall not be controlling.' The obvious purpose of this and similar
provisions is to free administrative boards from the compulsion of technical rules so that
the mere admission of matter which would be deemed incompetent inn judicial
proceedings would not invalidate the administrative order. But this assurance of a
desirable flexibility in administrative procedure does not go far as to justify orders without a
basis in evidence having rational probative force.
(5) The decision must be rendered on the evidence presented at the hearing, or at least
contained in the record and disclosed to the parties affected. -Protect parties in their right
to know and meet the case against them. It should not, however, detract from their duty
actively to see that the law is enforced, and for that purpose, to use the authorized legal
methods of securing evidence and informing itself of facts material and relevant to the
controversy. Boards of inquiry may be appointed for the purpose of investigating and
determining the facts in any given case, but their report and decision are only advisory.
(Section 9, Commonwealth Act No. 103.) The Court of Industrial Relations may refer any
industrial or agricultural dispute or any matter under its consideration or advisement to a
local board of inquiry, a provincial fiscal. a justice of the peace or any public official in any
part of the Philippines for investigation, report and recommendation, and may delegate to
such board or public official such powers and functions as the said Court of Industrial
Relations may deem necessary, but such delegation shall not affect the exercise of the
Court itself of any of its powers.
(6) The Court of Industrial Relations or any of its judges, therefore, must act on its or his
own independent consideration of the law and facts of the controversy, and not simply
accept the views of a subordinate in arriving at a decision. It may be that the volume of
work is such that it is literally Relations personally to decide all controversies coming
before them. In the United States the difficulty is solved with the enactment of statutory
authority authorizing examiners or other subordinates to render final decision, with the
right to appeal to board or commission, but in our case there is no such statutory authority.
(7) The Court of Industrial Relations should, in all controversial questions, render its
decision in such a manner that the parties to the proceeding can know the various issues
involved, and the reasons for the decision rendered. The performance of this duty is
inseparable from the authority conferred upon it.
- The record of the proceedings had before the CIR in this particular case had no
substantial evidence that the exclusion of the 89 laborers was due to their union affiliation.
- The whole transcript of the hearing is just a record of contradictory statements of
opposing counsel, with sporadic conclusion drawn to suit their own views
- these statements have no evidentiary value
Respondents' Comments
1. That Toribio Teodoro's claim that on September 26, 1938, there was shortage of leather
soles in ANG TIBAY making it necessary for him to temporarily lay off the members of the
National Labor Union Inc., is entirely false and unsupported by the records of the Bureau
of Customs and the Books of Accounts of native dealers in leather.
2. That the supposed lack of leather materials claimed by Toribio Teodoro was but a
scheme to systematically prevent the forfeiture of this bond despite the breach of his
CONTRACT with the Philippine Army.
3. That Toribio Teodoro's letter to the Philippine Army dated September 29, 1938, (re
supposed delay of leather soles from the States) was but a scheme to systematically
prevent the forfeiture of this bond despite the breach of his CONTRACT with the Philippine
Army.
4. That the National Worker's Brotherhood of ANG TIBAY is a company or employer union
dominated by Toribio Teodoro, the existence and functions of which are illegal. (281 U.S.,
548, petitioner's printed memorandum, p. 25.)
5. That in the exercise by the laborers of their rights to collective bargaining, majority rule
and elective representation are highly essential and indispensable. (Sections 2 and 5,
Commonwealth Act No. 213.)
6. That the century provisions of the Civil Code which had been (the) principal source of
dissensions and continuous civil war in Spain cannot and should not be made applicable
in interpreting and applying the salutary provisions of a modern labor legislation of
American origin where the industrial peace has always been the rule.
7. That the employer Toribio Teodoro was guilty of unfair labor practice for discriminating
against the National Labor Union, Inc., and unjustly favoring the National Workers'
Brotherhood.
8. That the exhibits hereto attached are so inaccessible to the respondents that even with
the exercise of due diligence they could not be expected to have obtained them and
offered as evidence in the Court of Industrial Relations.
9. That the attached documents and exhibits are of such far-reaching importance and
effect that their admission would necessarily mean the modification and reversal of the
judgment rendered herein.
ISSUE
WON a new trial in the CIR should be granted

Labor Law 1
HELD
YES
Ratio When a hearing before the CIR is conducted and a ruling is arrived at without any
substantial evidence, and there may be more evidence to be heard, a new trial shall be
granted.
Reasoning
- The SC found no substantial evidence that the exclusion of the 89 laborers here was due
to their union affiliation or activity. Although the CIR is a court with special nature- in that it
may be said to be free from technical rules of procedure- it must still respect certain
primary rights, one of which is that its decision must be based on substantial evidence.
The interest of justice would be better served if the movant is given opportunity to present
at the hearing the documents referred to in his motion and such other evidence as may be
relevant to the main issue involved. The legislation which created the Court of Industrial
Relations and under which it acts is new. The failure to grasp the fundamental issue
involved is not entirely attributable to the parties adversely affected by the result.
Disposition The motion for a new trial should be and the same is hereby granted, and
the entire record of this case shall be remanded to the Court of Industrial Relations, with
instruction that it reopen the case, receive all such evidence as may be relevant and
otherwise proceed in accordance with the requirements set forth hereinabove. So ordered.

AIR MANILA, INC. V BALATBAT


REYES; April 29, 1971
NATURE
Petition for certiorari to determine the validity of Resolution No. 139 (68) of the Civil
Aeronautics Board in CAB Case No. 1414, allegedly issued without or in excess of
jurisdiction.
FACTS
- Philippine Airlines Inc (PAL) petitioned the Civil Aeronautics Board (Board), for approval
of a proposed schedule introducing seven flights and the adjustment of the flight schedule
that may thus be affected. On April 15 1968, action on the petition was deferred for further
study.
- On April 22 1968, the Board passed Resolution No. 109 (68), referring PAL's petition to a
hearing examiner for economic justification. PAL moved for reconsideration of Resolution
No. 109 (68). The Board deferred action on this later motion, until PAL shall have resumed
its DC-3 services in certain airports named therein. PAL filed another motion for
reconsideration, on the ground that the new flights which it was proposing to operate will
be serviced by jet-prop or pure jet equipment only, thus, the order for resumption of DC-3
services was improper and should be deleted. In its Resolution No. 131 (68) of May 20
1968, the Board deferred action on this motion for reconsideration.
- On May 15 1968, PAL filed an Urgent Petition for approval of a consolidated schedule of
jet and jet prop flights, with an interim DC-3 schedule to different secondary and feeder
points (DTS-35). On May 28 1968, the Board issued its Resolution No. 139 (68),
approving DTS-35 for a period of 30 days, effective June 1 1968, subject to the conditions
that (a) the flight between Manila and San Fernando, La Union, F210/211 of the same
timetable, be operated daily instead of twice a week as proposed, and (b) that all
schedules under DTS-35, for which no previous approval has been granted by the Board
are to be referred to a hearing examiner for reception of evidence on its economic
justification.
- After the examiner's report, several of the proposed flights were approved for 30 days
from July 31 1968.
- On May 31 1968, Air Manila, Inc., filed the instant petition claiming that the respondent
Board acted without or in excess of jurisdiction and/or with abuse of discretion in issuing
its Resolution No. 139 (68).
- Petitioner alleged that the proposed new schedule, involving an in crease of frequencies,
would not only saturate the routes served also by petitioner, but would also affect its
schedule; that the Board's approval of said Domestic Traffic Schedule without receiving
the evidence of the parties constituted a deprivation of petitioner's light to be heard; and
that such authorization to PAL to operate the proposed schedule without economic
justification amounted to a capricious and whimsical exercise by the Board of its power
amounting to lack of jurisdiction.
ISSUES
1. WON the Board acted without or in excess of jurisdiction and/or with abuse of discretion
in issuing its Resolution No. 139 (68)
2. WON the Board's approval of said Domestic Traffic Schedule without receiving the
evidence of the parties constituted a deprivation of petitioner's light to be heard
HELD
1. NO
- It has been correctly said that administrative proceedings are not exempt from the
operation of certain basic and fundamental procedural principles, such as the due process

A2010

- 22 -

Disini

requirements in investigations and trials. And this administrative due process is recognized
to include (a) the right to notice, be it actual or constructive, of the institution of the
proceedings that may affect a person's legal rights; (b) reasonable opportunity to appear
and defend his rights, introduce witnesses and relevant evidence in his favor, (c) a tribunal
so constituted as to give him reasonable assurance of honesty and impartiality, and one of
competent jurisdiction; and (4) a finding or decision by that tribunal supported by
substantial evidence presented at the hearing, or at least contained in the records or
disclosed to the parties affected.
- However, it can not truthfully be said that the provisional approval by the Board of PAL's
proposed DTS-35 violates the requisites of administrative due process. Admittedly, after
PAL's proposal to introduce new Mercury night flights had been referred to a hearing
examiner for economic justification, PAL submitted a so-called consolidated schedule of
flights, DTS-35, that included the same Mercury night flights, and this was allowed by
Board Resolution No. 139(68). According to respondents, however, the Board's action was
impelled by the circumstance that at the time, the authorizations of certain flight schedules
previously allowed but were incorporated in DTS-35 were about to expire; thus, the
consolidated schedule had to be approved temporarily if the operations of the flights
referred to were not to be suspended. In short, the temporary permit was issued to prevent
the stoppage or cessation of services in the affected areas. This point petitioner has failed
to refute.
- Under the law, the Civil Aeronautics Board is not only empowered to grant certificates of
public convenience and necessity; it can also issue, deny, revise, alter, modify, cancel,
suspend or revoke, in whole or in part, any temporary operating permit, upon petition or
complaint of another or even at its own initiative. The exercise of the power, of course, is
supposed to be conditioned upon the paramount consideration of public convenience and
necessity, and nothing has been presented in this case to prove that the disputed action
by the Board has been prompted by a cause other than the good of the service.
2. NO
- There is no proof that in the hearings conducted by hearing examiner, petitioner was not
notified or given opportunity to adduce evidence in support of its opposition.
Disposition Petition dismissed

CENTURY TEXTILE MILLS INC. V NATIONAL LABOR


RELATIONS COMMISSION
FELICIANO; May 25, 1988
FACTS
- Eduardo Calangi, a machine operator at Century Textile Mills, was terminated because
he was allegedly behind the plot to kill his two supervisors, Melchor Meliton and Antonio
Santos. Marin, another factory worker, noticed that Torrena, a machine operator, put
some substance in a pitcher where Meliton and Santos usually drank from. It was later
found out that the substance was formaldehyde. Torrena confessed that Calangi
personally instructed him to put the substance in the pitcher as an act of revenge against
Meliton and Santos because they repeatedly instigated the termination of the two machine
operators. Torrena and Calangi were preventively suspended and eventually dismissed.
- Calangi filed a complaint against illegal dismissal with the Arbitration Branch, NCR,
MOLE and was dismissed because the evidence was so overwheliming and sufficient
enough against Calangi and he failed inexplicable to deny or controvert any charges.
Calangi filed an appeal in NLRC and the decision of the Arbitration Branch was reversed.
ISSUES
1. WON respondent was illegally dismissed because his termination was not in
accordance with due process
2. If YES, WON respondent can be reinstated in the company with full backwages and
without loss of seniority rights
HELD
1. YES because termination is without notice and hearing. The twin requirements for
notice and hearing constitute essential elements of due process in cases of employee
dismissal: the requirement of notice is intended to inform the employee concerned of the
employers intent and the reason for the proposed dismissal; upon the other hand, the
requirement of hearing affords the employee the opportunity to answer his employers
charges against him and accordingly to defend himself
Reasoning
- A278 Labor Code states that employer should furnish the worker a written notice
containing causes for termination and shall afford ample opportunity to be heard and to
defend himself the burden of proving that the termination was valid rests on the
employer
- Rule XIV Book V of the Rules and Regulations Implementing the Labor Code
Sec 2 Notice of Dismissal: written notice of the particular acts or omission constituting
grounds for dismissal
Sec 5 Answer and Hearing: employer shall afford the worker ample opportunity to be
heard and to defend himself with the assistance of his representatives if he so desires

Labor Law 1
Sec 6 Decision to Dismiss: employer should immediately inform the worker in writing of
the decision to dismiss him stating clearly the reasons
- prior consultation with the labor union is legally insufficient. Right to notice and hearing
are rights personal to an employee. Such consultation or consent is not a substitute for
actual observance of the rights
- nothing in the record that Cainta police interrogated Calangi himself. Basis for the
ground of dismissal is anchored solely on Torrenas sworn statement
2. YES, according to A280 Labor Code, there is security of tenure. No loss of seniority
rights and payment of backwages are the normal consequences when finding an
employee illegally dismissed BUT reinstatement is not for the best interest of the parties
involved. The corporation cannot force to take back an employee who poses a threat to
the lives of other employees. Therefore, separation pay must be paid in lieu of
reinstatement
Disposition Petition for certiorari DISMISSED. TRO and Resolutions WITHDRAWN.
Decision in NLRC case affirmed with modifications on awarding to of three years back
wages and addition of separation pay

LEYTE LAND TRANSPORTATION COMPANY INC V


LEYTE FARMERS' & LABORERS' UNION
PARAS; May 12, 1948
NATURE
Petition to review on certiorari decision of Court of Industrial Relations.
FACTS
- Court of Industrial Relations (CIR) issued order directing Leyte Land Transportation
Company, Inc. (LLTCI) to (among others) grant various employees increase in wages and
salaries (Php5-Php10), grant per diems (Php2/day) to its drivers, conductors, mechanics
and other workers, and grant 15 days vacation with pay and 15 days sick leave with pay to
employees and laborers. LLTCI appealed.
- LLTCIs contentions: CIR made a mistake in conceding salary/ wage increases merely
because such increases would enable employees to meet high cost of living. The
increases, if added to the crippling losses, would only throw the company into bankruptcy.
CIR had no power to order such directives and violated appellants freedom to contract.
ISSUES
1. WON CIR can validly take into account the high cost of living as a factor for
determining reasonableness of salary raise
2. WON CIR can validly determine and fix minimum wages for workers
3. WON appealed decision in effect has deprived LLTCI of its rights to enter into contract
of employment as it and the employee may agree
HELD
1. YES
Ratio The increases ordered are implied in the power/s granted to the CIR by the
Commonwealth Act No. 103.
- Sec.20 of Commonwealth Act no. 103 provides that "in the hearing, investigation and
determination of any question or controversy and in exercising any duties and power
under this Act, the Court shall act according to justice and equity and substantial merits of
the case, without regard to technicalities or legal forms."
- Sec.5 of same act provides, in connection with minimum wages for a given industry or in
a given locality, that the court shall fix the same at a rate that "would give the workingmen
a just compensation for their labor and an adequate income to meet the essential
necessities of civilized life, Laborers' Union and at the same time allow the capital a fair
return on its investment."
2. YES
Ratio The court has already upheld the constitutionality of the power of the CIR to
determine and fix minimum wages for workers (in a long line of cases). In fact, the power
is constitutionally mandated by Art.11,sec,5 ("the promotion of social justice to insure the
well-being and economic security of all the Laborers' Union people should be the concern
of the State"); Art.14,sec. 6 ("the State shall afford protection to labor, especially to
working women and minors, and shall regulate the relations between landowner and
tenant, and between labor and capital in industry and in agriculture" xxx "the State may
provide for compulsory arbitration.")
3. NO
Ratio The fact that both parties are of full age and competent to contract does not
necessarily deprive the State of the power to interfere where the parties do not stand upon
an equality, or where the public health demands that one party to the contract shall be
protected against himself. The State still retains an interest in his welfare, however
reckless he may be. The whole is no greater than the sum of all the parts, and where the
individual health, safety and welfare are sacrificed or neglected, the State must suffer
(citing a US case: West Coast Hotel Company vs. Parrish).

A2010

- 23 -

Disini

Citing Justice Laurel in Ang Tibay v CIR (concurring): The policy of laissez faire has to
some extent given way to the assumption by the government of the right of intervention
even in contractual relations affected with public interest
Obiter regarding the criticism that the additional benefits would benefit the union and well
as non-union members, the court held that as the workers are laborers of the company,
they are entitled to the increase regardless of their affiliation. To make a distinction would
only be an unjust and unwarranted discrimination against non-members.
Disposition Decision Affirmed.

VICTORIANO V ELIZALDE ROPE WORKERS UNION


ZALDIVAR; September 12, 1974
NATURE
Appeal from a decision of the CFI enjoining Elizalde Rope Factory, Inc. from dismissing
Appellee Victoriano
FACTS
- RA 875 provides:
a. General Rule: An employer can make an agreement with a labor organization to require
as a condition of employment membership therein.
- RA 3350 amending RA 875 provides:
b. Exception: But such agreement shall not cover members of any religious sects which
prohibit affiliation of their members in any such labor organization.
- Appellee Victoriano is an employee of the Elizalde Rope Factory, Inc. As such
employee, pursuant to RA 875, he is a member of the Elizalde Rope Workers Union which
had with the Company a Collective Bargaining Agreement containing a Closed Shop
Provision, i.e. membership in the Union is required as a condition of employment.
- Victoriano is a member of Iglesia ni Cristo, a religious sect prohibiting affiliation of its
members with any labor organization. As such, after RA 3350 was enacted, he tendered
his resignation to Appellant Union.
- Company dismissed Victoriano from service. He filed a case with the CFI. The CFI
decided in his favor. Appellant appealed directly to SC.
Petitioners' Claim
1. RA 3350 infringes on the fundamental right to form lawful associations in that it bans all
those belonging to such religious sects from affiliation with any labor organization
2. RA 3350 is unconstitutional for impairing the obligation of contracts in that, while Union
is obliged to comply with CBA:
a. the Act relieves the employer from its reciprocal obligation of maintaining union
membership as a condition for employment
b. impairs the Unions rights to dues from members who, under the act, are relieved from
the obligation to continue as such members.
3. RA 3350 discriminatorily favors religious sects while leaving no rights or protection to
labor organizations.
4. RA 3350 violates the constitutional provision that no religious test shall be required for
the exercise of a civil right,
a. in that the laborers exercise of his civil right to join associations has to be determined
by his affiliation with a religious sect
b. conversely, if a worker has to sever his religious connection with a sect that prohibits
membership in unions in order to join a labor union, the Act would violate religious
freedom
5. RA 3350 violates equal protection of laws, by exempting from the operation of Closed
Shop Agreement the members of the Iglesia ni Cristo, thereby granting said members
undue advantages over their fellow members (i.e. while the Act exempts them from union
obligation, it entitles them to enjoyment of concessions and benefits the union might
secure from the employer)
6. RA 3350 violates the constitutional provision regarding the promotion of social justice.
7. The amendment by RA 3350 in the form of the exception in favor of religious sects
prohibiting union membership is necessary rooted in whether the Closed Shop Provision is
violative of religious freedom.
Respondents' Comments
1. No. The right to join associations includes the right not to join; the Act actually prohibits
compulsion of workers to join labor organization
2. No. The Act formed part of, and was incorporated into, the terms of the Closed Shop
Agreement
3. No. The Act instead accommodated the religious needs of workers and balanced the
collective rights of organized labor with the constitutional right of an individual to freely
exercise his chosen religion.
4. No. The constitutional right of an individual to freely exercise his chosen religion has
primacy over union security measures which are merely contractual.
5 No. The classification of workers depending on their religious tenets is:
a. based on substantial distinction
b. germane to the purpose of the law, and
c. applies to all the members of the given class

Labor Law 1
6. No. The Act was enacted precisely to equalize employment opportunities for all citizens
in the midst of diversities of religious beliefs, a manifestation of social justice.
ISSUES
1 WON RA 3350 infringes on the fundamental right to form lawful associations
2 WON RA 3350 is unconstitutional for impairing the obligation of contracts
3 WON RA 3350 discriminatorily favors religious sects while leaving no rights or protection
to labor organizations.
4 WON RA 3350 violates the constitutional provision that no religious test shall be
required for the exercise of a civil right,
5 WON RA 3350 violates equal protection of laws
6 WON RA 3350 violates the constitutional provision regarding the promotion of social
justice
7. WON the amendment in the form of the exception in favor of religious sects prohibiting
union membership is necessary

HELD
1. NO
- The RA does not prohibit the members of such religious sects from joining unions. Both
RA 3350 and the Constitution recognize the right of freedom of association. A right
comprehends two broad notions:
a. Liberty or freedom absence of legal restraint whereby an employee may act for
himself without being prevented by law
b. Power whereby an employee may join or refrain from joining
Because before RA 3350, if any person, regardless of his religious belief, wishes to be
employed, he must become a member of the Collective Bargaining Union. With the
exception provided in RA 3350 to member of religious organizations, employees who are
members of the same are given the power to join or not to join. They cannot be compelled
to join even when the unions have closed shop agreements with employers.
2. NO
- The prohibition on impairment of obligations by Statute is not unqualified. It prohibits
only unreasonable impairment. In spite of the constitutional prohibition, the State
continues to possess authority to safeguard the vital interests of the people. The
reservation of essential attributes of sovereign power is read into contracts as a postulate
to the preservation of the legal order. The contract clause of the Constitution must
therefore be not only in harmony with, but also in subordination to the reserved power of
the state to safeguard vital interests of the people.
3. NO
-In Aglipay v Ruiz, the Court said that the government should not be precluded from
pursuing valid objectives secular in character even if the incidental result would be
favorable to a religion or sect, as long as it has a secular legislative purpose and a primary
effect that neither advances nor inhibits religion.
- The purpose of RA 3350 is secular: to advance the constitutional right to free exercise of
religion, by averting that certain persons be refused work by reason of their religion and
union security agreements.
- The primary effect of the exemption in favor of members of sects prohibiting union
membership is the protection of said employees against the aggregate force of the CBA,
and relieving certain citizens of a burden on their religious beliefs.
- Although it may benefit some sects in particular, the benefits are merely incidental and
indirect, not primary.
4. NO
- The Act does not require as a qualification for joining any lawful association membership
in any particular religion; neither does the Act require affiliation with a religious sect that
prohibits its members from joining a labor union.
- Joining or withdrawing from a labor union requires a positive act. The Act only exempts
members with such religious affiliation from the coverage of closed shop agreements.
Thus, a religious objector is not required to do a positive act. He is exempted ipso jure.
How can there be a religious test required for the exercise of a right when no right need be
exercised?
5. Equal protection is not a guaranty of equality in the application of laws upon all citizens,
but on persons according to the circumstances surrounding them. It does not forbid
discrimination as to things that are different. All that is required of valid classification is
that it be:
a. reasonable, i.e. based on substantial distinctions which make for real differences
real: based on WON by reasons of their religious belief, cannot sign up with a labor union
b. germane to the purpose of the law the purpose of the law is precisely to avoid those
who cannot, because of their religious belief, join labor unions, from being deprived of their
right to work
c. it must not be limited to existing conditions only
d. apply equally to each member of the class
Every classification allowed by the Constitution by its nature involves inequality.
Whenever it is apparent from the scope of the law that its object is for the benefit of the
public and means by which the benefit is to be obtained are of public character, the law

A2010

Disini

- 24 -

will be upheld even though incidental advantage may occur to individuals beyond those
enjoyed by the general public.
6. NO
- It is not necessary that the entire state be directly benefited. Social justice does not
require social or legal equality. Social justice guarantees equality of opportunity, and this
is precisely what RA 3350 proposes to accomplish it gives laborers, irrespective of their
religious scrupples, equal opportunity for work.
7. A statute which is not necessary is not, for that reason, unconstitutional. Legislatures,
being chosen by the people, are presumed to understand the needs of the people, and it
may change the laws accordingly. For the validity of a statute, the essential basis for the
exercise of power, and not a mere incidental result arising from its exertion, as in its
effects on a particular case, is the criterion.
Disposition appeal is dismissed. The decision of the CFI appealed from is affirmed.

SEPARATE OPINION
FERNANDO [concur]
- Stressing the transcendent character of religious freedom and its primacy even as
against the claims of protection to labor.
- Gerona v Secretary of Education: But between the freedom of belief and the exercise of
said belief, there is quite a stretch of road to travel. If the exercise of said religious belief
clashes with established institutions of society and with the law, then the former must
yield. The specific circumstances of the right curtailed as against religious freedom should
be given consideration on a case to case basis.

ALALAYAN V NPC
FERNANDO; July 29, 1969
NATURE
Appeal from a judgment of the Court of First Instance of Manila. Perez, J.
The facts are stated in the opinion of the Court.
FACTS
- The National Power Corporation was given the power (Sec. 3, R.A. 3043, approved
June 17, 1961, entitled "An Act to Further Amend Commonwealth Act Numbered
One Hundred Twenty, as Amended by Republic Act Numbered Twenty Six Hundred
and Forty-One) to require from franchise holders the conditions that:
a)
they shall not realize a net profit of more than twelve percent annually of its
investments plus two-month operating expenses;
b)
NPC can renew all existing contracts with franchise holders for the supply of
electric power and energy.
- National Power Corporation has for some years now been supplying, distributing,
servicing and selling electric power and energy at fixed rates schedules to the latter who
have for some years now been and still are, legally engaged in re-supplying, redistributing, re-servicing and re-selling the said electric power and energy to individual
customers within the coverage of their respective franchises.
- Reference was made to the particular contracts petitioners entered into with respondent,
the contracts to continue indefinitely unless and until either party would give to the other
two years previous notice in writing of its intention to terminate the same.
- On June 18, 1960, an act authorizing the increase of the capital stock of the National
Power Corporation to P100 million took effect. On June 17, 1961, it was alleged that the
challenged legislation became a law, purportedly to increase further the authorized capital
stock, but including the alleged rider referred to above.
- National Power Corporation approved a rate increase of at least 17.5%, the effectivity of
which, was at first deferred to November 1, 1962, then subsequently to January 15, 1963,
with the threat that in case petitioners would fail to sign the revised contract providing for
the increased rate, 'respondent National Power Corporation would then cease "to supply,
distribute and service electric power and energy to them."
- On March 21, 1963, the lower court, considering that there was "no sufficient ground for
the issuance of the writ of preliminary injunction Petitioners Claims, dismissed the same.
- It was alleged in the facts that Alalayan did purchase and take power and energy as
follows: "Sixty (60) kilowatts and of not less than 140,000 kilowatt-hours in any contract
year at the rate of P120.00 per kilowatt per year" payable in twelve equal monthly
installments, "plus an energy charge of P0.013 per kilowatt hour, payable on the basis of
monthly delivery.
- A letter of June 22, 1962 of respondent National Power Corporation to petitioner
approved his 17.5% rate increase of power so that beginning July 1, 1962, the demand
charge would be P10.00 per kilowatt per month and the energy charge would be P0.02
per kilowatt hour.

Labor Law 1
- The lower court, in a decision of January 30, 1965, sustained the validity and
constitutionality of the challenged provision, hence this appeal.
Petitioners Claims
The challenged provision is a violation of the constitutional requirement that a bill cannot
embrace more than one subject to be expressed in its title.
ISSUES
1. WON the assailed amendment is a violation of the constitutional requirement that a bill
cannot embrace more than one subject to be expressed in its title
2. WON the petitioner is deprived of the liberty to contract without due process of law
HELD
1. Ratio The amendment does not constitute a rider problem. The legislature is not
required to make the title of the act a complete index of its contents.
Reasoning
- The rider provision is aimed against the evils of the so-called omnibus bills as logrolling
legislation as well as surreptitious or unconsidered enactments. Where the subject of a bill
is limited to a particular matter, the lawmakers along with the people should be informed of
the subject of proposed legislative measures. This constitutional provision thus precludes
the insertion of riders in legislation, a rider being a provision not germane to the subject
matter of the bill. The provision merely calls for all parts of an act relating to its subject
finding expression in its title.
- The Constitution does not require Congress to employ in the title of an enactment,
language of such precision as to mirror, fully index or catalogue all the contents and the
minute details therein. It suffices if the title should serve the purpose of the constitutional
demand that it inform the legislators, the persons interested in the subject of the bill, and
the public, of the nature, scope and consequences of the proposed law and its operation.
- If the law amends a section or part of a statute, it suffices if reference be made to the
legislation to be amended, there being no need to state the precise nature of the
amendment.
2. Ratio The petitioner is not deprived the liberty to contract without due process of law.
Reasoning For in the face of a constitutional provision that allows deprivation of liberty,
including liberty of contract, as long as due process is observed, the alleged nullity of a
legislative act of this character can only be shown if in fact there is such a denial.
- The Constitution, when there was the fear expressed in many quarters that a
constitutional democracy, in view of its commitment to the claims of property, would not be
able to cope effectively with the problems of poverty and misery that unfortunately afflict
so many of our people, is not susceptible to the indictment that the government therein
established is impotent to take the necessary remedial measures.
- There is the clause on the promotion of social justice to ensure the wellbeing and
economic security of all the people, as well as the pledge of protection to labor with the
specific authority to regulate the relations between landowners and tenants and between
labor and capital.
- The police power as an attribute to promote the common weal would be diluted
considerably of its reach and effectiveness if on the mere plea that the liberty to contract
would be restricted, the statute complained of may be characterized as a denial of due
process.
- The liberty relied upon is not freedom of the mind, which occupies a preferred position,
nor freedom of the person, but the liberty to contract, associated with business activities,
which, as has been so repeatedly announced, may be subjected, in the interest of the
general welfare under the Police Power, to restriction valid in character and wide ranging
in scope as long as due process is observed.
- There is no controlling and precise definition of due process. It furnishes though a
standard to which governmental action should conform in order that deprivation of life,
liberty or property, in each appropriate case, be valid.
Standard of Due Process: responsiveness to the supremacy of reason, obedience to the
dictates of justice.
- While not explicitly avowed by petitioner, there is the intimation that to apply the
challenged legislation to contracts then in existence would be an infringement of the
constitutional prohibition against any law impairing the obligation of contracts. Statutes
enacted for the regulation of public utilities, being a proper exercise by the state of its
police power, are applicable not only to those public utilities coming into existence after its
passage, but likewise to those already established and in operation.
Disposition The decision of the lower court dismissing the petition is dismissed.

EMPLOYEES CONFEDERATION OF THE PHILIPPINES


V NATIONAL WAGES AND PRODUCTIVITY
COMMISSION
PHILIPPINE AIRLINES, INC. (PAL) V NLRC
MELO; August 13, 1993
NATURE

A2010

- 25 -

Disini

Petition for certiorari from a decision of the NLRC upholding the Labor Arbiters ruling
directing the PAL to allow the latters employees to participate in the formulation of the
Code of Discipline for PAL employees
FACTS
- On March 15, 1985, the Philippine Airlines, Inc. (PAL) completely revised its 1966 Code
of Discipline. The Code was circulated among the employees and was immediately
implemented, and some employees were forthwith subjected to the disciplinary measures
embodied therein.
- Thus, on August 20, 1985, the Philippine Airlines Employees Association (PALEA) filed a
complaint before the National Labor Relations Commission (NLRC) for unfair labor
practice. In its position paper, PALEA contended that PAL, by its unilateral implementation
of the Code, was guilty of unfair labor practice, specifically Paragraphs E and G of
Article 249 and Article 253 of the Labor Code.
- PALEA alleged that copies of the Code had been circulated in limited numbers; that
being penal in nature the Code must conform with the requirements of sufficient
publication, and that the Code was arbitrary, oppressive, and prejudicial to the rights of the
employees. It prayed that implementation of the Code be held in abeyance; that PAL
should discuss the substance of the Code with PALEA; that employees dismissed under
the Code be reinstated and their cases subjected to further hearing; and that PAL be
declared guilty of unfair labor practice and be ordered to pay damages (pp. 7-14, Record.).
- PAL filed a motion to dismiss the complaint, asserting its prerogative as an employer to
prescribe rules and regulations regarding employees' conduct. In its reply to PAL's position
paper, PALEA maintained that Article 249 (E) of the Labor Code was violated when PAL
unilaterally implemented the Code, and cited provisions of Articles IV and I of Chapter II of
the Code as defective for, respectively, running counter to the construction of penal laws
and making punishable any offense within PAL's contemplation. These provisions are the
following:
- Section 2. Non-exclusivity. -- This Code does not contain the entirety of the rules and
regulations of the company. Every employee is bound to comply with all applicable rules,
regulations, policies, procedures and standards, including standards of quality,
productivity, and behaviour, as issued and promulgated by the company through its duly
authorized officials. Any violations thereof shall be punishable with a penalty to be
determined by the gravity and/or frequency of the offense.
- Section 7.
Cumulative Record. -- An employee's record of offenses shall be
cumulative. The penalty for an offense shall be determined on the basis of his past record
of offenses of any nature or the absence thereof. The more habitual an offender has been,
the greater shall be the penalty for the latest offense. Thus, an employee may be
dismissed if the number of his past offenses warrants such penalty in the judgment of
management even if each offense considered separately may not warrant dismissal.
Habitual offenders or recidivists have no place in PAL. On the other hand, due regard shall
be given to the length of time between commission of individual offenses to determine
whether the employee's conduct may indicate occasional lapses (which may nevertheless
require sterner disciplinary action) or a pattern of incorrigibility.
- Labor Arbiter Isabel P. Ortiguerra did not find PAL guilty of unfair labor practice. However,
she said that PAL was not totally faultless and therefore ordered the management to share
decision-making on the code of discipline. NLRC affirmed the Labor Arbiter. Hence, this
case.
ISSUE
WON management may be compelled to share with the union or its employees its
prerogative of formulating a code of discipline
HELD
YES
- The exercise of managerial prerogatives is not unlimited. It is circumscribed by limitations
found in law, a collective bargaining agreement, or the general principles of fair play and
justice (University of Sto. Tomas vs. NLRC, 190 SCRA 758 [1990]). Moreover, as
enunciated in Abbott Laboratories (Phil.), Inc. vs. NLRC (154 SCRA 713 [1987]), it must be
duly established that the prerogative being invoked is clearly a managerial one.
- A close scrutiny of the objectionable provisions of the Code reveals that they are not
purely business-oriented nor do they concern the management aspect of the business of
the company as in the San Miguel case. The provisions of the Code clearly have
repercusions on the employees' right to security of tenure. The implementation of the
provisions may result in the deprivation of an employee's means of livelihood which, as
correctly pointed out by the NLRC, is a property right (Callanta vs. Carnation Philippines,
Inc., 145 SCRA 268 [1986]). In view of these aspects of the case which border on
infringement of constitutional rights, we must uphold the constitutional requirements for the
protection of labor and the promotion of social justice, for these factors, according to
Justice Isagani Cruz, tilt "the scales of justice when there is doubt, in favor of the worker"
(Employees association of the Philippine American Life Insurance Company vs. NLRC,
199 SCRA 628 [1991] 635).
- PAL posits the view that by signing the 1989-1991 collective bargaining agreement, on
June 27, 1990, PALEA in effect recognized PAL's "exclusive right to make and enforce
company rules and regulations to carry out the functions of management without having to

Labor Law 1
discuss the same with PALEA and must less, obtain the conformity thereto" (pp. 11-12,
Petitioner's Memorandum; pp. 180-181, Rollo.)
- Such provision in the collective bargaining agreement may not be interpreted as cession
of employees' rights to participate in the deliberation of matters which may affect their
rights and the formulation of policies relative thereto. And one such matter is the
formulation of a code of discipline.
- Indeed, industrial peace cannot be achieved if the employees are denied their just
participation in the discussion of matters affecting their rights. Thus, even before Article
211 of the Labor Code (P.D. 442) was amended by Republic Act No. 6715, it was already
declared a policy of the State: "(d) To promote the enlightenment of workers concerning
their rights and obligations . . .as employees." This was, of course, amplified by Republic
Act No. 6715 when it decreed the "participation of workers in decision and policy making
processes affecting their rights, duties and welfare." PAL's position that it cannot be
saddled with the "obligation" of sharing management prerogatives as during the
formulation of the Code, Republic Act No. 6715 had not yet been enacted (Petitioner's
Memorandum, p. 44; Rollo, p. 212), cannot thus be sustained. While such "obligation" was
not yet founded in law when the Code was formulated, the attainment of a harmonious
labor-management relationship and the then already existing state policy of enlightening
workers concerning their rights as employees demand no less than the observance of
transparency in managerial moves affecting employees' rights.
Disposition Petition dismissed.

BREW MASTER INTERNATIONAL INC. V NATIONAL


FEDERATION OF LABOR UNIONS (NAFLU)
DAVIDE, JR; April 17, 1997
NATURE
A special civil action for certiorari seeking the reversal of the decision of the National Labor
Relations Commission (NLRC) which modified the decision of the Labor Arbiter by
directing the reinstatement of private respondent Antonio D. Estrada, the complainant,
without loss of seniority rights and benefits.
FACTS
- Private respondent NAFLU, a co-complainant in the labor case, is a labor union of which
complainant is a member.
- Complainant was first employed by Brew Master on 16 September 1991 as route helper
with the latest daily wage of P119.00.
- From 19 April 1993 up to 19 May 1993, for a period of 1 month, complainant went on
absent without permission (AWOP).
- On 20 May 1993, Brew master sent him a Memo: Please explain in writing within 24
hours of your receipt of this memo why no disciplinary action should be taken against you
for the following offense: You were absent since April 19, 1993 up to May 19, 1993.
- In answer to the aforesaid memo, complainant explained:
Sa dahilan po na ako ay hindi nakapagpaalam sainyo dahil inuwi ko ang mga anak ko sa
Samar dahil ang asawa ko ay lumayas at walang mag-aalaga sa mga anak ko. Kaya
naman hindi ako naka long distance or telegrama dahil wala akong pera at ibinili ko ng
gamot ay puro utang pa.
- Finding said explanation unsatisfactory, the company issued a Notice of Termination:
...we regret to inform you that we do not consider it valid. You are aware of the company
Rules and Regulations that absence without permission for 6 consecutive working days is
considered abandonment of work...
- Complainants contend that individual complainants dismissal was done without just
cause; that it was not sufficiently established that individual complainants absence from
April 19, 1993 to June 16, 1993 are unjustified; that the penalty of dismissal for such
violation is too severe; that in imposing such penalty, respondent should have taken into
consideration complainants length of service and as a first offender, a penalty less
punitive will suffice such as suspension for a definite period.
- Upon the other hand, respondent contends that individual complainant was dismissed for
cause allowed by the company Rules and Regulations and the Labor Code; that the act of
complainant in absenting from work for 1 month without official leave is deleterious to the
business of respondent; that it will result to stoppage of production which will not only
destructive to respondents interests but also to the interest of its employees in general;
that the dismissal of complainant from the service is legal.
- The Labor Arbiter dismissed the complaint for lack of merit, citing the principle of
managerial control, which recognizes the employers prerogative to prescribe reasonable
rules and regulations to govern the conduct of his employees. He relied on Shoemart,
Inc. vs. NLRC: ...that individual complainant has indeed abandoned his work... therefore,
under the law and jurisprudence which upholds the right of an employer to discharge an
employee who incurs frequent, prolonged and unexplained absences as being grossly
remiss in his duties to the employer and is therefore, dismissed for cause. An employee is
deemed to have abandoned his position or to have resigned from the same, whenever he
has been absent therefrom without previous permission of the employer for three
consecutive days or more.

A2010

- 26 -

Disini

- the NLRC modified the Labor Arbiter's decision and held that complainants dismissal
was invalid for the following reasons:
Complainant-appellants prolonged absences, although unauthorized, may not amount to
gross neglect or abandonment of work to warrant outright termination of employment.
Dismissal is too severe a penalty...Reliance on the ruling enunciated in the cited case of
Shoemart is quite misplaced because of the obvious dissimilarities-- complainant in the
Shoemart Case was an inveterate absentee who does not deserve reinstatement
compared to herein complainant-appellant who is a first offender
ISSUE
WON the NLRC committed grave abuse of discretion in modifying the decision of the
Labor Arbiter
HELD
NO
Ratio a) Petitioners finding that complainant was guilty of abandonment is misplaced.
Abandonment as a just and valid ground for dismissal requires the deliberate, unjustified
refusal of the employee to resume his employment. Two elements must then be satisfied:
(1) the failure to report for work or absence without valid or justifiable reason; and (2) a
clear intention to sever the employer-employee relationship.
b) Verily, relations between capital and labor are not merely contractual. They are
impressed with public interest and labor contracts must, perforce, yield to the common
good.
While the employer is not precluded from prescribing rules and regulations to govern the
conduct of his employees, these rules and their implementation must be fair, just and
reasonable.
Reasoning
- complainants absence was precipitated by a grave family problem as his wife
unexpectedly deserted him and abandoned the family. Considering that he had a full-time
job, there was no one to whom he could entrust the children and he was thus compelled to
bring them to the province. He was then under emotional, psychological, spiritual and
physical stress and strain. The reason for his absence is, under these circumstances,
justified. While his failure to inform and seek petitioner's approval was an omission which
must be corrected and chastised, he did not merit the severest penalty of dismissal from
the service.
- the elements of abandonment are not present here. First, as held above, complainant's
absence was justified under the circumstances. As to the second requisite, complainant
immediately complied with the memo requiring him to explain his absence, and upon
knowledge of his termination, immediately sued for illegal dismissal. These plainly refuted
any claim that he was no longer interested in returning to work.
- our Constitution looks with compassion on the workingman and protects his rights not
only under a general statement of a state policy, but under the Article on Social Justice
and Human Rights, thus placing labor contracts on a higher plane and with greater
safeguards.
- While we do not decide here the validity of petitioner's Rules and Regulations on
continuous, unauthorized absences, what is plain is that it was wielded with undue haste
resulting in a deprivation of due process, thus not allowing for a determination of just
cause or abandonment. In this light, petitioner's dismissal was illegal. This is not to say
that his absence should go unpunished, as impliedly noted by the NLRC in declining to
award back wages.
Disposition petition is hereby DISMISSED and the decision of the NLRC is hereby
AFFIRMED.

PT&TC V NLRC
REGALADO; May 23, 1997
NATURE
PT&TC seeks relief through certiorari on decision of NLRC
FACTS
- Private respondent Grace De Guzman (GdG) was initially hired by PTTC, on 3 separate
occasions, to relieve 2 of its employees who went on maternity leave. The Reliever
Agreement stated that her employment was to be immediately terminated upon expiration
of the agreed period.
- She was later asked to join the company as a probationary employee with the probation
period covering 150 days. On her application form, she indicated that her civil status was
single although she had contracted marriage 3 months earlier.
- Upon discovery, PTTC, through its Baguio branch supervisor, sent a memorandum to
GdG requiring her to explain the discrepancy and reminding her of the companys policy of
not accepting married women for employment. GdG, in her response dated Jan.17, 92,
explained that she was not aware of such a policy and that she had not deliberately
hidden her true civil status.
- PTTC was unconvinced and dismissed GdG on Jan. 29, 92 which GdG then contested
before the regional arbitration branch of the NLRC in Baguio through a complaint for illegal

Labor Law 1
dismissal. PTTC claimed that the dismissal was due to the fact that she had concealed her
civil status not because of the fact that she was married.
- The Labor Arbiter handed down a decision declaring that GdG, who had already gained
the status of a regular employee, was illegally dismissed by petitioner. Her reinstatement,
plus payment of the corresponding back wages and COLA (cost of living allowances), was
ordered, with the view that the ground relied upon by petitioner in dismissing private
respondent was clearly insufficient, and that it was apparent that she had been
discriminated against on account of her having contracted marriage in violation of
company rules
- PTTC appealed but the NLRC upheld the decision of the labor arbiter with only the
modification that GdG deserved to be suspended for three months in view of the dishonest
nature of her acts. The subsequent MFR by PTTC was likewise rebuffed by NLRC hence
this special civil action.
ISSUE
WON the PTTC erred in dismissing GdG
HELD
1. YES
Ratio An employer is required, as a condition sine qua non prior to severance of the
employment ties of an individual under his employ, to convincingly establish, through
substantial evidence, the existence of a valid and just cause in dispensing with the
services of such employee, ones labor being regarded as constitutionally protected
property.
- On the other hand, an employer is free to regulate, according to his discretion and best
business judgment, all aspects of employment, from hiring to firing, except in cases of
unlawful discrimination or those which may be provided by law
Reasoning
- The petitioners policy of not accepting or considering as disqualified from work any
woman worker who contracts marriage runs afoul of the test of, and the right against,
discrimination, afforded all women workers by our labor laws and by no less than the
Constitution. Contrary to petitioners assertion, the record discloses clearly that her ties
with the company were dissolved principally because of the companys policy that married
women are not qualified for employment in PT&T, and not merely because of her
supposed acts of dishonesty.
- That it was so can easily be seen from the memorandum sent to private respondent by
the branch supervisor of the company, with the reminder that youre fully aware that the
company is not accepting married women employee, as it was verbally instructed to you.
Again, in the termination notice sent to her by the same branch supervisor, private
respondent was made to understand that her severance from the service was not only by
reason of her concealment of her married status but, over and on top of that, was her
violation of the companys policy against marriage (and even told you that married women
employees are not applicable or accepted in our company.) Parenthetically, this seems to
be the curious reason why it was made to appear in the initiatory pleadings that petitioner
was represented in this case only by its said supervisor and not by its highest ranking
officers who would otherwise be solidarily liable with the corporation.
- The government abhors any stipulation or policy in the nature of that adopted by
petitioner PT&T. The Labor Code states, in no uncertain terms, as follows:
ART. 136. Stipulation against marriage. - It shall be unlawful for an employer to
require as a condition of employment or continuation of employment that a woman
shall not get married, or to stipulate expressly or tacitly that upon getting married, a
woman employee shall be deemed resigned or separated, or to actually dismiss,
discharge, discriminate or otherwise prejudice a woman employee merely by reason of
marriage.
- In Zialcita, et al. vs. Philippine Air Lines, a PAL policy requiring that prospective flight
attendants must be single and that they will be automatically separated from the service
once they marry was declared void, it being violative of the clear mandate in Article 136 of
the Labor Code with regard to discrimination against married women. In Gualberto, et al.
vs. Marinduque Mining & Industrial Corporation, the Court of Appeals considered a policy
of the same nature, as repugnant to the Civil Code, Presidential Decree No. 148 and the
Constitution and therefore void and unlawful.
Disposition The petition is dismissed for lack of merit.

GENERAL BANK AND TRUST CO V CA


GUTIERREZ JR; April 9, 1985
FACTS
- plaintiff-appellee was employed with the Cebu Branch of the First National City Bank of
New York for 18 years, where he rose to the position of Chief Clerk, Accounting
Department
- on January 11, 1965, plaintiff-appellee joined the defendant bank in its Cebu branch as
accountant with an annual compensation of P6,000.00

A2010

- 27 -

Disini

- April 26, 1965, the Cebu Branch of defendant bank began operating and doing business
with the public
- January 1, 1966, plaintiff received an increase of P50.00 bringing his monthly salary to
P550.00
- April 11, 1967 defendant bank appointed the plaintiff to the position of Acting Manager of
its Cebu Branch, with the corresponding increase of salary to P700.00 a month
- effective September 1, 1967, defendant bank granted plaintiff a monthly housing
allowance of P200.00 in addition to his monthly salary
- October 3, 1967 defendant bank appointed plaintiff as the regular Manager of its Cebu
Branch effective May 1, 1968
- defendant bank increased plaintiff's salary to P800.00 a month
- May 16, 1969 while the plaintiff was on vacation leave, he happened to visit the bank and
learned that three tellers of defendant bank's branch in Cebu City, namely, Miss Crystal
Enriquez, Miss Yolanda Chu, and Miss Sonia Chiu, had been transferred to the head office
in Manila by defendant Jose D. Santos
- plaintiff went to Manila on May 18, 1969 to make personal representation with the head
office for the retention of the said tellers in Cebu
- May 26, 1969 the plaintiff reported back for duty with defendant bank's branch in Cebu
and reinstated immediately the three tellers to their respective positions in the Cebu
branch of defendant bank
- May 28, 1969 defendant Jose D, Santos submitted a report to defendant Salvador D.
Tenorio alleging that there was excess personnel in the Cebu Branch; that on the same
date defendant Jose D. Santos submitted a supplementary report to defendant Salvador
D. Tenorio charging the plaintiff of over-appraising the real estate offered by Domingo
Chua as collateral for his credit accommodation; that defendant Salvador D. Tenorio
immediately dispatched a letter to the plaintiff dated May 30, 1969 requiring him to explain
within twenty-four hours why no disciplinary action should be taken against him for alleged
repeated violation of defendant bank's policies and directives regarding credit
accommodations and for over-appraisal of the real estate collateral for Domingo Chua's
account, among others
- June 6, 1969, the plaintiff received the said letter of defendant Salvador D. Tenorio but
found it impossible to render the required explanation in 24 hours
- June 19, 1969 defendant Jose D. Santos went to Cebu City and served plaintiff with the
letter of defendant Salvador D. Tenorio, dated June 18, 1969, suspending the plaintiff;
- July 22, 1969 plaintiff was served with the order of his termination signed by defendant
Clarencio S. Yujuico, dated July 18, 1969."
- CFI found the dismissal of plaintiff as without just cause or otherwise illegal arbitrary,
oppressive and malicious, and ordering defendants to pay to the plaintiff, jointly and
severally, the following sums: (a) P1,000.00 a month, as consequential damages for the
loss of his salaries and allowances, from the date of his dismissal until the judgment shall
have become final and executory; (b) P2,500.00 as termination pay; (c) P106.63
representing unpaid salaries from the 16th to 19th of June 1969; (d) P200,000.00 in
concept of moral damages; (e) P50,000.00 as exemplary or corrective damages; (f)
P15,000.00 as attorney's fees; and to pay the costs of the suit."
The Court of Appeals affirmed the decision of the lower court but modified the
judgment by reducing moral damages to P150,000.00 and exemplary damages to
P30,000.00.
ISSUES
1. WON the dismissal of Manuel E. Batucan was justified on the ground that he repeatedly
failed to uphold the interests of the bank thus leading to his employer's loss of confidence
on him
2. WON the award of moral and exemplary damages is proper
HELD
1. NO
- There was no error in the finding of the CA that Mr. Batucan was indeed illegally
dismissed.
- There is no question that managerial employees should enjoy the confidence of top
management. This is especially true in banks where officials handle big sums of money
and engage in confidential or fiduciary transactions. However, loss of confidence should
not be simulated. It should not be used as a subterfuge for causes which are improper,
illegal, or unjustified. Loss of confidence may not be arbitrarily asserted in the face of
overwhelming evidence to the contrary. It must be genuine, not a mere afterthought to
justify earlier action taken in bad faith.
- All the privileges, commendations, and salary increases negate the allegation that the
management had lost confidence in Mr. Batucan. Moreover, there is no evidence that Mr.
Batucan granted unauthorized credit accommodations because after the last three exhibits
were sent, an internal audit examination was conducted on February 11, 1969 by
petitioner Santos together with the Internal Auditor, Mr. Rosauro Macalagay. In this
examination, no unauthorized credit accommodations were found and brought to the
attention of Mr. Batucan. The management's alleged loss of confidence in Mr. Batucan
cannot be reconciled with the latter's commendations for efficient performance, his having
been given an increase in salary and his being asked to speak to other colleagues on
effective banking techniques shortly after the supposed loss of confidence.

Labor Law 1
- The only reason for his dismissal found in the records is his failure to follow topmanagement orders with regards to the transfer of the three tellers. Petitioners alleged it
to be insubordination. Nevertheless, insubordination must be proven to justify dismissal
(St. Luke's Hospital v. Ministry of Labor and Employment, 116 SCRA 240). His earnest
efforts in making representations to retain the three tellers do not warrant his dismissal. A
manager or supervisor must stand up for his subordinates unless the latter are guilty of
wrongdoing or some conduct prejudicial to the employer. Only after his representations
was Mr. Batucan questioned on the several "unauthorized credit accommodations." His
failure to explain within 24 hours which, in the light of the circumstances, was too short,
caused his suspension and later, his dismissal retroactive to the date of suspension.
- There was no valid reason for his dismissal, much less for all the charges and
accusations made against him. The dismissal followed by the efforts to justify it was
tainted by bad faith or malice on the part of the petitioners who wanted Mr. Batucan
removed from his post.
2. YES
- Moral damages may be justly awarded. Moral damages being justified, exemplary
damages may also be awarded.
- Mr. Batucan left a stable job with a reputable bank to join the petitioner bank. He had
been an employee of the First National City Bank of New York for eighteen (18) years.
Undoubtedly, before he accepted petitioner Tenorio's invitation, he must have thought the
matter over several times. And from the time he joined the petitioner bank, the records
show that Mr. Batucan has indeed worked his way up from accountant to permanent
branch manager of the bank. During his term as manager, he was able to increase the
income and resources of the bank. He raised the image of petitioner bank in the business
and banking community and placed its operations on a good and competitive basis. His
peremptory dismissal from the bank was certainly a shock to him and damaged his moral
feelings and personal pride after all the loyalty and hard work he had dedicated to the
bank.
Disposition The decision appealed from is MODIFIED to read as follows:
The petitioners are hereby ordered to pay to the private respondent, jointly and severally,
the following sums TWO THOUSAND FIVE HUNDRED PESOS (P2,500.00)
termination pay; ONE HUNDRED SIX PESOS AND SIXTY THREE CENTAVOS (P106.63)
unpaid salaries; TWELVE THOUSAND PESOS (P12,000.00) in compensatory damages;
TWENTY THOUSAND PESOS (P20,000.00) in moral and exemplary damages; and FIVE
THOUSAND PESOS (P5,000.00) attorney's fees.

STAR PAPER CORP. V SIMBOL


PUNO; April 12, 2006
NATURE
Petition for review on certiorari of a decision of CA
FACTS
- Ronaldo Simbol, Wilfreda Comia and Lorna Estrella were all regular employees of Star
Paper Corp. Simbol and Comia each got married to a co-employee and due to a company
policy banning spouses (and relatives up to the 3rd degree) from working in the same
company, they resigned from their jobs. Estrella was impregnated by a married coemployee. She likewise lost her job (company said she resigned, Estrella said she was
dismissed for immoral conduct).
- Respondents allege that they did not resign voluntarily and they filed a compliant for
unfair labor practices, constructive dismissal, separation pay, and they averred that the
company policy is illegal and contravenes ART 136 of the Labor Code.
- CA declared their dismissal as illegal, ordering the company to reinstate them to their
former positions and to pay attorneys fees and cost of the suit.
ISSUES
1. WON the policy is violative of the constitutional rights towards marriage and the family
of employees and of ART 136 of the Labor Code
2. WON respondents resignations were voluntary
HELD
1. YES
- The questioned policy may not facially violate Article 136 of the Labor Code but it creates
a disproportionate effect and under the disparate impact theory, the only way it could pass
judicial scrutiny is a showing that it is reasonable despite the discriminatory, albeit
disproportionate, effect. The failure of petitioners to prove a legitimate business concern in
imposing the questioned policy cannot prejudice the employees right to be free from
arbitrary discrimination based upon stereotypes of married persons working together in
one company.
-The absence of a statute expressly prohibiting marital discrimination in our jurisdiction
cannot benefit the petitioners. The protection given to labor in our jurisdiction is vast and
extensive that we cannot prudently draw inferences from the legislatures silence that
married persons are not protected under our Constitution and declare valid a policy based
on a prejudice or stereotype. Thus, for failure of petitioners to present undisputed proof of

A2010

- 28 -

Disini

a reasonable business necessity, we rule that the questioned policy is an invalid exercise
of management prerogative.
2. Simbol and Comia MOOT and ACADEMIC
Estrella - NO
- Questioned policy is an invalid exercise of management prerogative. Corollarily, the
issue as to whether respondents Simbol and Comia resigned voluntarily has become moot
and academic.
-We have held that in voluntary resignation, the employee is compelled by personal
reason(s) to dissociate himself from employment. It is done with the intention of
relinquishing an office, accompanied by the act of abandonment. Thus, it is illogical for
Estrella to resign and then file a complaint for illegal dismissal. Given the lack of sufficient
evidence on the part of petitioners that the resignation was voluntary, Estrellas dismissal
is declared illegal.
Disposition Decision of the Court of Appeals in CA-G.R. SP No. 73477 dated August 3,
2004 is AFFIRMED.

FIRESTONE TIRE & RUBBER COMPANY V LARIOSA


FERNAN; February 27, 1987
NATURE
Petition for certiorari to review the decision of the National Labor Relations Commission
FACTS
- Carlos Lariosa worked as a Firestone as a factory worker (started January 3, 1972) and
later as a tire builder.
- July 27, 1983 Lariosa was on his way out of the office so he had to submit himself to
the routine check by security guards (Liao and Olvez) at the gate. In the course of the
inspection, 16 wool flannel swabs were found inside his bag, tucked underneath his soiled
clothes. Said swabs were company property.
- Lariosa was terminated on the ground of stealing company property and loss of trust. A
criminal complaint was also filed against him for theft.
- The Labor Arbiter initially found the dismissal to be justified but on appeal, the NLRC
reversed the decision and ordered Lariosas reinstatement but backwages were not to be
paid. The period when he was out of work should be considered suspension.
Petitioners Claim
- Firestone claims that the NLRC erred in not dismissing Lariosas appeal for being late, in
finding that Lariosa was not accorded due proms and in reversing the Labor Arbiter.
Respondents Comments
- Lariosa filed a suit against Firestone for illegal dismissal, violation of BP Blg. 130 and its
related rules and regulations.
- On reversal upon appeal, the NLRC said that termination was too harsh a penalty.
ISSUE
WON the decision of the NLRC to reinstate Lariosa was rendered with grave abuse of
discretion amounting to lack of jurisdiction
HELD
(1) The particular procedural lapse may be overlooked.
Ratio The shortened period for appeal is principally intended for the employees benefit
rather than that of the employer.
Reasoning
- Lariosas appeal was filed on June 7, 1984 or after the lapse of fourteen days from notice
of the decision of the Labor Arbiter. The Labor Code provide for a reglementary period of
ten calendar days within which to appeal a decision of the Labor Arbiter to the NLRC.
- However the Notice of Decision received by Lariosas lawer advised that an appeal
should be taken to the NLRC within ten working days from receipt of said decision.
(2) Records show that Lariosa was involved in the attempted theft of fannel swabs.
Ratio There is no gainsaying that theft committed by an employee constitutes a valid
reason for his dismissal by the employer. Although as a rule this Court leans over
backwards to help workers and employees continue with their employment or to mitigate
the penalties imposed on them acts of dishonesty in the handling of company property are
a different matter. If there is sufficient evidence that an employee has been guilty of a
breach of trust or that his employer has ample reasons to distrust him, the labor tribunal
cannot justly deny to the employer the authority to dismiss such an employee.
Reasoning
- As a tire builder, Lariosa was entrusted with certain materials for use in his job. On the
day in question, he was given two bundles of wool flannel swabs [ten pieces per bundle]
for cleaning disks.
- He used four swabs from one pack and kept the rest [sixteen pieces].
- If Lariosa, by his own wrongdoing, could no longer be trusted, it would be an act of
oppression to compel the company to retain him, fully aware that such an employee could,
in the long run, endanger its very viability.
- Firestone had valid grounds to dispense with the services of Lariosa and that the NLRC
acted with grave abuse of discretion in ordering his reinstatement. However, considering

Labor Law 1
that Lariosa had worked with the company for eleven years with no known previous bad
record, the ends of social and compassionate justice would be served if he is paid full
separation pay but not reinstatement without backages as decreed by the NLRC.
Disposition petition is granted. The decision of the National Labor Relations Commission
dated December 28, 1984 is reversed and set aside. Petitioner Firestone Tire and Rubber
Company of the Philippines is directed to pay its dismissed worker Carlos Lariosa the
separation pay to which he may be entitled under the law, or any collective bargaining
agreement or company rules or practice, whichever is higher.

STAR PAPER CORP. V SIMBOL


PUNO; April 12, 2006
[PAGE 28]
SARMIENTO V TUICO
CRUZ; June 27, 1988
FACTS
- Petitioner Asian Transmission Corporation terminated the services of Catalino Sarmiento,
vice-president of the Bisig ng Asian Transmission Labor Union (BATU), for allegedly
carrying a deadly weapon in the company premises.
- BATU filed a notice of strike, claiming that the ATC had committed an unfair labor
practice.
- The ATC then filed a petition asking the Ministry of Labor and Employment to assume
jurisdiction over the matter or certify the same to the NLRC for compulsory arbitration.
- MOLE issued an order certifying the labor dispute to the NLRC. At the same time, it
enjoined the management from locking out its employees and the union from declaring a
strike or similar concerted action.
- Proceedings could not continue in the NLRC, however, because of the acceptance by
President Aquino of the resignations of eight of its members, leaving only the vicechairman in office.
- MOLE, set aside the previous orders and directly assumed jurisdiction of the dispute, at
the same time enjoining the company to accept all returning workers.
- This order was later set aside upon motion of both the BATU and the ATC in view of the
appointment of new commissioners in the NLRC. The MOLE then returned the case to the
respondent NLRC and directed it to expeditiously resolve all issues relating to the dispute.
- Conformably, the NLRC issued on January 13, 1987 a resolution, which it affirmed in its
resolution of February 12, 1987, denying the motion for reconsideration.
- Three criminal complaints filed against the petitioning workers, two by the personnel
administrative officer of the ATC and the third by the Philippine Constabulary.
- The first two complaints, were for "Violation of Article 265, par. 1, in relation to Article 273
of the Labor Code of the Philippines." The third, was for coercion. In all three complaints,
the defendants were charged with staging an illegal strike, barricading the gates of the
ATC plant and preventing the workers through intimidation, harassment and force from
reporting for work.
- Judge Orlando Tuico issued a warrant of arrest against the petitioners and committed 72
of them to jail although he later ordered the release of 61 of them to the custody of the
municipal mayor of Calamba, Laguna.
- The petitioners had earlier moved for the lifting of the warrant of arrest and the referral of
the coercion charge to the NLRC and, later, for the dismissal of Criminal Cases on the
ground that they came under the primary jurisdiction of the NLRC.
ISSUES
1. WON a return-to-work order may be validly issued by the National Labor Relations
Commission pending determination of the legality of the strike
2. WON, pending such determination, the criminal prosecution of certain persons involved
in the said strike may be validly restrained
HELD
1. YES
- The question of competence is easily resolved. The authority for the order is found in
Article 264(g) of the Labor Code, as amended by B.P. Blg. 227, which provides as follows:
- When in his opinion there exists a labor dispute causing or likely to cause strikes or
lockouts adversely affecting the national interest, such as may occur in but not limited to
public utilities, companies engaged in the generation or distribution of energy, banks,
hospitals, and export- oriented industries, including those within export processing zones,
the Minister of Labor and Employment shall assume jurisdiction over the dispute and
decide it or certify the same to the Commission for compulsory arbitration. Such
assumption or certification shall have the effect of automatically enjoining the intended or
impending strike or lockout as specified in the assumption order. If one has already taken
place at the time of assumption or certification, all striking or locked out employees shall
immediately return to work and the employer shall immediately resume operations and

A2010

Disini

- 29 -

readmit all workers under the same terms and conditions prevailing before the strike or
lockout. The Minister may seek the assistance of law-enforcement agencies to ensure
compliance with this provision as well as such orders as he may issue to enforce the
same.
- There can be no question that the MOLE acted correctly in certifying the labor dispute to
the NLRC, given the predictable prejudice the strike might cause not only to the parties but
more especially to the national interest. Affirming this fact, we conclude that the return-towork order was equally valid as a statutory part and parcel of the certification order issued
by the MOLE on November 24, 1986. The challenged order of the NLRC was actually only
an implementation of the above provision of the Labor Code and a reiteration of the
directive earlier issued by the MOLE in its own assumption order of September 9, 1986.
- It must be stressed that while one purpose of the return-to-work order is to protect the
workers who might otherwise be locked out by the employer for threatening or waging the
strike, the more important reason is to prevent impairment of the national interest in case
the operations of the company are disrupted by a refusal of the strikers to return to work
as directed. More particularly, it is the national economy that will suffer because of the
resultant reduction in our export earnings and our dollar reserves, not to mention possible
cancellation of the contracts of the company with foreign importers.
- It is also important to emphasize that the return-to-work order not so much confers a right
as it imposes a duty; and while as a right it may be waived, it must be discharged as a
duty even against the worker's will. Returning to work in this situation is not a matter of
option or voluntariness but of obligation
2. YES
- The Court held that while as a general rule the prosecution of criminal offenses is not
subject to injunction, the exception must apply in the case at bar. The suspension of
proceedings in the criminal complaints filed is justified on the ground of prematurity as
there is no question that the acts complained of are connected with the compulsory
arbitration proceedings still pending in the NLRC.
- The three criminal cases should be suspended until the completion of the compulsory
arbitration proceedings in the NLRC, conformably to the policy embodied in Circular No.
15, series of 1982, and Circular No. 9, series of 1986, issued by the Ministry of Justice in
connection with the implementation of B.P. Blg. 227. These circulars, briefly stated,
require fiscals and other government prosecutors to first secure the clearance of the
Ministry of Labor and/or the Office of the President "before taking cognizance of
complaints for preliminary investigation and the filing in court of the corresponding
informations of cases arising out of or related to a labor dispute," including "allegations of
violence, coercion, physical injuries, assault upon a person in authority and other similar
acts of intimidation obstructing the free ingress to and egress from a factory or place of
operation of the machines of such factory, or the employer's premises."
- It does not appear from the record that such clearance was obtained, conformably to the
procedure laid down "to attain the industrial peace which is the primordial objectives of this
law," before the three criminal cases were filed.

PCIB V JACINTO
GANCAYCO; May 6, 1991
FACTS
- Nilda S. Jacinto is a PCI Bank (NAIA branch) customer relation assistant (CRA) who acts
as alternate FX Clerk or Teller.
- 1 May 1984: PCIB discovered the loss of some travelers checks amounting to P
25,325.00 in peso equivalent transacted 30 April 1984. As Jacinto acted as FX clerk on
said day, an investigation was conducted by PCIB allowing Jacinto and other personnel to
explain their side.
- Jacinto was found guilty of gross negligence, meted a 10-day suspension w/o pay (7-20
May 1984), and required to pay the loss by way of salary deductions (P200/month + 50%
of mid-year & Xmas bonus and profit sharing). She was transferred to the Baclaran
branch, 21 May 1984.
- 14 Aug 1986: Jacinto filed a complaint with NLRC questioning her suspension, penalty
and transfer of assignment.
- 19 Feb 1988: The labor arbiter found the 10day suspension and the deductions to be
unjustified and ordered PCIB to erase from Jacinto's 201 file said suspension and to return
to her the amount so far deducted from her salary, bonuses and 13th month pay. PCIB
was further directed to return Jacinto to her former assignment at MIA branch, if she so
prefers. PCIB appealed.
- 23 Oct 1989: NLRC affirmed the appealed decision with the only modification that the
transfer of Jacinto was found to be an appropriate prerogative of management. Hence,
this petition for certiorari with a prayer for the issuance of a writ of preliminary injunction.
ISSUE
[responsibility of a bank employee for the loss of certain funds of the bank]
WON NLRC gravely abused its discretion in holding that gross negligence cannot be
attributed to Jacinto as she was not formally designated to perform the functions of an FX
clerk

Labor Law 1
HELD
1. YES
Ratio Any employee who is entrusted with responsibility by his employer should perform
the task assigned to him with care and dedication. The lack of a written or formal
designation should not be an excuse to disclaim any responsibility for any damage
suffered by the employer due to his negligence. The measure of the responsibility of an
employee is that if he performed his assigned task efficiently and according to the usual
standards, then he may not be held personally liable for any damage arising therefrom.
Failing in this, the employee must suffer the consequences of his negligence if not lack of
due care in the performance of his duties.
- NAIA branch OIC, Mr. Gilberto C. Marquez, verbally requested Jacinto to assume the
duties of the FX Clerk who was on leave (no written memo of assignment). Jacinto
accepted the request; she herself stated that she received the travellers checks, made the
proof sheet thereof, and thereafter pllaced the checks and proof sheet in the FX cash box.
The following day, she reported the loss of said travellers checks from the FX cash box.
- Although she claimed to have prepared the proof sheet, none was found in the box. She
did not microfilm the checks as a matter of course. She did not formally endorse the FX
box to the night shift FX clerk or to the cashier. More so, considering that she knew the
lock of the box was defective. By and large, the finding of PCIB that Jacinto was grossly
negligent is well-taken.
- Jacinto's 10-day suspension w/o pay is a proper penalty in accordance with the
prescribed rules of PCIB. But, since PCIB is guilty of contributory negligence in failing to
have the lock of FX box fixed and to have taken other security measures in the bank
premises, the penalty of reimbursement of the full value of the loss is mitigated by
requiring Jacinto to reimburse the petitioner only 1/2 of the loss by way of salary
deduction.
Disposition Petition GRANTED. NLRC decision reversed and set aside. Jacinto's
complaint dismissed. Pebalty modified: Jacinto is required to indemnify PCIB the amount
of P 12,600.00 through regular payroll deductions.

GTE DIRECTORIES CORP. V SANCHEZ


NARVASA; May 27, 1991
NATURE
Petition for certiorari to review the order of the Department of Labor and Employment.
FACTS
- GTE Directories Corporation (GTE) is a foreign corporation engaged in the Philippines in
the business of publishing the PLDT telephone directories for Metro Manila and several
provinces.
- The practice was for its sales representatives to be given work assignments within
specific territories by the so-called "draw method." These sales territories were so plotted
or mapped out to have an equal number of advertisers as well as revenue. Within these
territories, the sales representatives therein assigned were given quotas.
- Increments were given by the so-called "Grid System," grids within each territory usually
numbering five. Each grid was assigned a fixed closing date. At such closing date, a
salesperson should have achieved a certain amount of the revenue target designated for
his grid; otherwise, he loses the forthcoming grid or forfeits the remaining grids not yet
received.
- June, 1984: GTE realized that competition among media for a share of the advertising
revenue had stepped up. GTE launched an aggressive campaign to get what it considered
to be its rightful share of the advertising budget of its clientele before it could be allocated
to other media. Among the actions taken were:
1.
If the cancelled revenue accounts were not renewed within the assigned
period, said accounts were declaredOPEN TERRITORY to all sales
representatives including the one who reported the cancellation;
2.
If not renewed during said open territory period, said cancelled accounts were
deemed no longer "open territory," and the same could be referred for
handling to contractual salespersons and/or outside agencies.
- A new "Sales Evaluation and Production Policy" was thereafter drawn up. GTE informed
all its sales representatives of the new policy in a Memorandum dated October 12, 1984.
- The new policy did not sit well with the union. The Union demanded that it be given 15
days to raise questions or objections. This, GTE granted, and by letter dated October 26,
1984, the union submitted its proposals for "revisions, corrections and deletions of some
policies incorporated in the Sales Administrative Practices issued on June 14, 1984
including the new policies recently promulgated by Management."
- GTE next formulated a new set of "Sales Administrative Practices," pursuant to which it
issued on July 9, 1985, a memorandum requiring all Premise Sales Representatives
(PSRs) to submit individual reports reflecting target revenues as of deadlines, set at
August 2, 1985. This was superseded by another memorandum dated July 16, 1985,
revising the previous schedules on the basis of "the consensus reached after several
discussions with your DSMs, as well as, most of you," The amount required initially (P30K)
was reduced to P20K

A2010

- 30 -

Disini

- Following this requirement, some Premise Sales Representatives (members of the


union) omitted to submit reports regarding the P20K revenue. GTE again demanded for
the said reports in another Memorandum, but as before the PSR refused to comply.
- August 6, 1985: the union filed in behalf of the sales representatives, a notice of
strike grounded on alleged unfair labor practices of GTE consisting of the following:
1.
Refusal to bargain on unjust sales policies;
2.
Open territory of accounts;
3.
Illegal suspension of Brian Pineda, a union officer;
4.
Non-payment of eight days suspension pay increase.
- On the same day GTE sent another Memorandum to 16 PSRs. No compliance was
made. GTE thereupon suspended its sales representatives "without pay effective August
12, 1985 for five (5) working days" and warned them that their failure to submit the
requisite reports by August 19, 1985 would merit "more drastic disciplinary actions." Still,
no sales representative complied with the requirement to submit the reports. So, by
memorandum of the Marketing Director dated August 19, 1985, all the sales
representatives concerned were suspended anew effective August 20, 1985 until the
submitted the report.
- GTE gave its sales representatives an ultimatum. By memorandum dated August 23,
1985, GTE required them for the last time, to submit the required reports within twentyfour (24) hours from receipt of the memorandum; otherwise they would be terminated "for
cause.
- August 29, 1985: GTE terminated the employment of the recalcitrant sales
representatives (14), with the undertaking to give them "separation pay, upon proper
clearance and submission of company documents, material etc., in . . . (their) possession."
On September 2, 1985, the union declared a strike in which about 60 employees
participated.
Petitioners Claim
GTE should have been commanded: (a) to pay all striking employees their usual salaries,
allowances, commission and other emoluments corresponding to the period of their strike;
(b) to release to its employees the 8-days pay increase unlawfully withheld from them; (c)
to lift the suspension imposed on Brian Pineda and restore to him the pay withheld
corresponding to the suspension period; (d) to pay the sales representatives all their lost
income corresponding to the period of their suspensions, and dismissal, including
commissions that they might have earned corresponding to their one-week forced leave.
Respondents Comment
The termination of the employment of its fourteen (14) premise sales representatives prior
to the strike should have been upheld. It also filed an opposition to the union's motion for
reconsideration.
ISSUE
WON the union's objections to, or request for reconsideration of those regulations or
policies automatically suspend enforcement thereof and excuse the employees' refusal to
comply with the same
HELD
Ratio So long as a company's management prerogatives are exercised in good faith for
the advancement of the employer's interest and not for the purpose of defeating or
circumventing the rights of the employees under special laws or under valid agreements,
this Court will uphold them7
Reasoning
- Even as the law is solicitous of the welfare of the employees, it must also protect the
right of an employer to exercise what are clearly management prerogatives. The free will
of management to conduct its own business affairs to achieve its purpose cannot be
denied.
- Except as limited by special laws, an employer is free to regulate, according to
his own discretion and judgment, all aspects of employment, including hiring, work
assignments, working methods, time, place and manner of work, tools to be used,
processes to be followed, supervision of workers, working regulations, transfer of
employees, work supervision, lay-off of workers and the discipline, dismissal and recall of
work. . . .
- In the case of GTE, it must thus be conceded that its adoption of a new "Sales
Evaluation and Production Policy" was within its management prerogative to regulate,
according to its own discretion and judgment, all aspects of employment, including the
manner, procedure and processes by which particular work activities should be done.
There were, to be sure, objections presented by the union, i.e., that the schedule had not
been "drawn (up) as a result of an agreement of all concerned," that the new policy was
incomprehensible, discriminatory and whimsical, and "would result to further reduction" of
the sales representatives' compensation.
- The Court failed to see how the objections and accusations justify the deliberate and
stubborn refusal of the sales representatives to obey the management's simple
requirement for submission by all PSRs of individual reports or memoranda requiring
reflecting target revenues which it addressed to the employees concerned no less than six
(6) times.
7

LVN, Pictures Workers vs. LVN, 35 SCRA 147

Labor Law 1
- To sanction disregard or disobedience by employees of a rule or order laid down by
management, on the pleaded theory that the rule or order is unreasonable, illegal, or
otherwise irregular for one reason or another, would be disastrous to the discipline and
order that it is in the interest of both the employer and his employees. Deliberate disregard
or disobedience of rules, defiance of management authority cannot be countenanced.
- Minister Sanchez however found GTE to have "acted evidently in bad faith" in firing its 14
salespersons "for alleged violations of the reportorial requirements of its sales policies
which was then the subject of conciliation proceedings between them;" while the company,
in merely implementing its challenged sales policies did not ipso facto commit an unfair
labor practice, it did so when it in mala fide dismissed the fourteen salesmen, all union
members, while conciliation proceedings were being conducted
Disposition Petition is granted, and the order of the public respondent is nullified and set
aside.

GUSTILO V WYETH PHILIPPINES INC.


SANDOVAL-GUTIERREZ; October 4, 2004
[PAGE 10]
LAGNITON V NLRC
CRUZ; February 5, 1993
NATURE
Petition for Review on Certiorari of the Decision of the National Labor Relations
Commission
FACTS
- On August 18, 1986, the private respondents filed with the MOLE a complaint for illegal
dismissal against petitioner Arturo S. Lagniton, Sr., proprietor of a company manufacturing
shoes and other leather products.
- In his defense, Lagniton argued that complainant Generoso Ambrosio was not employed
by the company but a mere sub-contractor. Even if considered a regular employee, he
was nevertheless validly dismissed because of his poor workmanship, which amounted to
serious misconduct or gross and habitual neglect. The other complainants, although
concededly employees, were not dismissed but simply discontinued reporting for work
beginning August 11, 1986, and thus abandoned their employment.
- Labor Arbiter Isabel P. Ortiguerra rendered a decision holding that Ambrosio was an
employee of the company because his work as a sole-stitcher was necessary to its
business and that he worked regular hours under its supervision and control. He and the
other complainants, whose status as regular employees was not questioned, had indeed
not abandoned their work but were in fact illegally dismissed by Lagniton. This decision
was affirmed in toto by the NLRC in a resolution dated September 14, 1988. It is
contended that the NLRC and the Labor Arbiter committed grave abuse of discretion in the
finding that the private respondents were illegally dismissed and were entitled to
separation pay and the ECOLA.
ISSUE
1. WON the complainants were illegally dismissed
2. WON the complainants are entitled to the ECOLA
HELD
1. NO
Ratio The complaint for illegal dismissal was filed only seven days after the complainants
allegedly abandoned their work on August 11, 1986. Such dispatch in protesting their
separation belies the claimed abandonment. We also agree that given the hardship of the
times, the complainants would not simply have left their work unless they were transferring
to other employment offering better terms and conditions. There is no evidence of such
transfer. As it has been established that the workers did not abandon their work, it follows
that their dismissal was illegal for lack of notice and hearing.
2. YES
Ratio The position of the petitioner is that the complainants (who did not have fixed
salaries and were paid by the piece) are not entitled to this benefit because it is available
only to workers earning less than P1,500.00 a month. Since, by the complainants' own
admission, they were earning an average of P1,000.00 a week, they are clearly not
covered by P.D. 1634. Under that decree, only workers earning a monthly salary of not
more than P1,500.00 may claim payment of the ECOLA. However, the figure cited by the
petitioner represents only the peak income of the workers and does not reflect their
monthly pay during the lean seasons, when they did not produce as much and so earned
less, at about P350.00 a week or P55.00 a day. On the average, as the NLRC determined,
the complainants were receiving less than P1,500.00 a month and so came under the
provision of the decree.
Disposition Petition dismissed.

A2010

Disini

- 31 MANEJA V NLRC
MARTINEZ; 1998

NATURE
Petition for certiorari
FACTS
- Petitioner Rosario Maneja. Worked with private respondent Manila Midtown Hotel as a
telephone operator. She was also a member of the National Union of Workers,
Restaurants and Allied Industries (NUWHRAIN) with an existing CBA with the private
respondent.
- ON Feb. 13, 1990, Rowena Loleng, a telephone operator, received a Request for Long
Distance Call (RLDC) form and a deposit of P500 from a pageboy for a Japanese guest
Hirota Ieda. The call was unanswered and the P500 was forwarded to the cashier. Later,
Ieda again made an RLDC and the P500 deposit was collected and given to Loleng. It was
also unanswered
- On feb 15, the cashier inquired about the P1000 deposit made. After a search, the first
one was found in the guest folio while the other in te folder for cancelled calls. Petitioner
saw that the 2nd RLDC form was not time stamped and placed it the machine to stamp it
with the date of Feb 15. But after realizing that the call was made 2 days before, she wrote
on it and changed it to Feb 13.
- On Mar 7, the chief telephone operator asked the petitioner and Loleng to explain the
Feb 15 incident. They submitted their written explanation.
- On Mar 20, a written report was submitted, saying that their actions were covered
violations of the Offenses Subject to Disciplinary Action as 1)forging falsifying official
documents 2)culpable carelessness and negligence or failure to follow established
procedure. On march 23, petitioner was then served notice of dismissal effective on April
1. She refused to sign and wrote therein under protest
- Criminal charges for falsification was charged against her. However, the resolution
recommending the filing of the case was reversed by the 2nd asst. city prosecutor.
- On Oct 2,1990, petitioner filed a complaint for illegal dismissal against the respondent
before the labor arbiter. The Labor arbiter found that the petitioner was illegally dismissed,
stating that even though the case revolves on the matter of implementation and
interpretation of company policies and is thus within the jurisidictional ambit of the
grievance procedure under the CBA, Art 217 of the Labor Code confers original and
exclusive jurisidiction of all termination cases to the Labor Arbiter.
- Respndent appealed the decision to the NLRC. The NLRC dismissed the case for lack of
jurisdiction of the Labor arbiter because the same should have been instead subject to
voluntary arbitration.
ISSUES
1. WON the Labor Arbiter had jurisdiction to decide the case
2. WON the petitioner was illegally dismissed
HELD
1. YES
- The NLRCs interpretation of Art 217c of the Labor Code is erroneous. Even though such
provision provides that labor arbiters have no jurisidiction over cases arising from
interpretation and implementation of CBAs (must be submitted to the grievance machine
or voluntary arbitration), it must be read in conjuction with Art 261 which grants voluntary
arbitrators original and exclusive jurisdiction to hear and decide all unresolved grievances
arising from the interpretation or implementation of the collective bargaining agreement
and those arising from the interpretation or enforcement of company personnel policies.
Note the phrase unresolved grievances. In the case at bar, the termination of petitioner
is not an unresolved grievance.
- According to the Sanyo case, there is the dismissal does not involve an interpretation or
implementation of a Collective Bargaining Agreement or interpretation or enforcement of
company personnel policies but involves termination. Where the dispute is just in the
interpretation, implementation or enforcement stage, it may be referred to the grievance
machinery set up in the Collective Bargaining Agreement or by voluntary arbitration.
Where there was already actual termination, i.e., violation of rights, it is already cognizable
by the Labor Arbiter.
- Also, from article 260, it can be deduced that only disputes involving the union and the
company shall be referred to the grievance machinery or voluntary arbitrators. In the case
at bar, the union does not come into the picture, as the practice in said Hotel in cases of
termination is that they are not referred anymore to the grievance committee; and that
the terminated employee who wishes to question the legality of his termination usually
goes to the Labor Arbiter for arbitration, whether the termination arose from the
interpretation or enforcement of the company personnel policies or otherwise.
- The petitioner also points out that respondent NLRC should have ruled that private
respondent is estopped in questioning the jurisdiction of the Labor Arbiter, since there as
active participation of the private respondent, coupled with his failure to object to the
jurisdiction of the court.
2. NO

Labor Law 1
- The requisites of a valid dismissal are (1) the dismissal must be for any of the causes
expressed in Article 282 of the Labor Code, and (2) the employee must be given an
opportunity to be heard and to defend himself.
- petitioner blames respondent for failure to abide by the established procedure. etitioner,
however, explained that the usual or established procedures are not followed by the
operators and hotel employees when circumstances warrant. For instance, the RLDC
forms and the deposits are brought by the page boy directly to the operators instead of the
cashiers if the latter are busy and cannot attend to the same. Furthermore, she avers that
the telephone operators are not conscious of the serial numbers in the RLDCs and at
times, the used RLDCs are recycled. Even the page boys do not actually check the serial
numbers of all RLDCs in one batch, except for the first and the last.
- On the charge of taking money, it to be noted that the second deposit was found in the
folder for cancelled calls, thus there is no basis for personal appropriation by the petitioner.
- On the tampering of the RLDC form, it was only done to reflect the true date of the
transaction. Also, under the OSDA, infractions of this sort is not without qualifications must
result to loss or damage to company property. There was no proof whatsoever in the case
at bar, except the general allegations made in the companys position paper and other
pleadings. In the same tenor, the respondents charge under OSDA 1.11 on the alleged
falsification of private document is also with a qualification, in that the alleged act of
falsification must have been done in such a way as to mislead the users thereof. Again,
based on the facts of the complained act, there appeared no one to have been misled on
the change of date from RLDC form 15 to 13 February 1990.
- An examination of the record reveals that no hearing was ever conducted by private
respondent before petitioner was dismissed. While it may be true that petitioner submitted
a written explanation, no hearing was actually conducted before her employment was
terminated. She was not accorded the opportunity to fully defend herself. Petitioners right
to due process was clearly violated.
Disposition Decision of the NLRC reversed. Decision of the labor arbiter is reinstated.

SPECIAL STEEL PRODUCTS, INC. V VILLAREAL


SANDOVAL-GUTIERREZ; July 8, 2004
NATURE
Petition for review on certiorari
FACTS
- Special Steel Products, Inc., petitioner, is a domestic corporation engaged in the principal
business of importation, sale, and marketing of BOHLER steel products. Lutgardo C.
Villareal and Frederick G. So, respondents, worked for petitioner as assistant sales
manager and salesman, respectively.
- Sometime in May 1993, respondent Villareal obtained a car loan from the Bank of
Commerce, with petitioner as surety, as shown by a continuing suretyship agreement
and promissory note wherein they jointly and severally agreed to pay the bank
P786,611.60 in 72 monthly installments. On January 15, 1997, respondent Villareal
resigned and thereafter joined Hi-Grade Industrial and Technical Products, Inc. as
executive vice-president.
- Sometime in August 1994, petitioner sponsored respondent Frederick So to attend a
training course in Kapfenberg, Austria conducted by BOHLER, petitioners principal
company. This training was a reward for respondent Sos outstanding sales performance.
When respondent returned nine months thereafter, petitioner directed him to sign a
memorandum providing that BOHLER requires trainees from Kapfenberg to continue
working with petitioner for a period of three (3) years after the training. Otherwise, each
trainee shall refund to BOHLER $6,000.00 (US dollars) by way of set-off or compensation.
On January 16, 1997 or 2 years and 4 months after attending the training, respondent
resigned from petitioner.
- Immediately, petitioner ordered respondents to render an accounting of its various
Christmas giveaways they received. These were intended for distribution to petitioners
customers.
- In protest, respondents demanded from petitioner payment of their separation benefits,
commissions, vacation and sick leave benefits, and proportionate 13 th month pay. But
petitioner refused and instead, withheld their 13th month pay and other benefits.
- On April 16, 1997, respondents filed with the Labor Arbiter a complaint for payment of
their monetary benefits against petitioner and its president, Augusto Pardo, docketed as
NLRC NCR Case No. 04-02820-97.
- In due course, the Labor Arbiter rendered a Decision IN FAVOR OF So and Villareal.
Petitioner filed a motion for reconsideration but was denied Hence, petitioner filed with the
Court of Appeals a petition for certiorari.
- On October 29, 1999, the Court of Appeals rendered a Decision dismissing the petition
and affirming the assailed NLRC Decision. Petitioner filed a motion for reconsideration but
was denied by the Appellate Court in a Resolution dated May 8, 2000.
ISSUES
1. WON Petitioner may legally withhold respondent Villareals monetary benefits as a
preliminary remedy pursuant to Article 2071 of the Civil Code, as amended

A2010

- 32 -

Disini

2. WON Petitioner could withhold his monetary benefits being authorized by the
memorandum he signed, the benefits acting as compensation
HELD
1. NO. It cannot. Article 116 of the Labor Code, as amended, provides:
ART. 116. Withholding of wages and kickbacks prohibited. It shall be unlawful
for any person, directly or indirectly, to withhold any amount from the wages (and
benefits) of a worker or induce him to give up any part of his wages by force,
stealth, intimidation, threat or by any other means whatsoever without the workers
consent.
- The above provision is clear and needs no further elucidation. Indeed, petitioner has no
legal authority to withhold respondents 13 th month pay and other benefits. What an
employee has worked for, his employer must pay. Thus, an employer cannot simply refuse
to pay the wages or benefits of its employee because he has either defaulted in paying a
loan guaranteed by his employer; or violated their memorandum of agreement; or failed to
render an accounting of his employers property.
- Nonetheless, petitioner, relying on Article 2071 (earlier cited), contends that the right to
demand security and obtain release from the guaranty it executed in favor of respondent
Villareal may be exercised even without initiating a separate and distinct action.
- There is no guaranty involved herein and, therefore, the provision of Article 2071 does
not apply.
- A guaranty is distinguished from a surety in that a guarantor is the insurer of the solvency
of the debtor and thus binds himself to pay if the principal is unable to pay, while a surety
is the insurer of the debt, and he obligates himself to pay if the principal does not pay.
- Based on the above distinction, it appears that the contract executed by petitioner and
respondent Villareal (in favor of the Bank of Commerce) is a contract of surety. In fact, it
is denominated as a continuing suretyship agreement. Hence, petitioner could not just
unilaterally withhold respondents wages or benefits as a preliminary remedy under Article
2071. It must file an action against respondent Villareal. Thus, the Appellate Court aptly
ruled that petitioner may only protect its right as surety by instituting an action to demand
a security.
2. NO, compensation may not be used. For legal compensation to take place, the
requirements set forth in Articles 1278 and 1279 of the Civil Code, quoted below, must be
present.
"ARTICLE 1278. Compensation shall take place when two persons, in their own right, are
creditors and debtors of each other.
"ARTICLE 1279.
In order that compensation may be proper, it is necessary:
(1)
That each one of the obligors be bound principally, and that he be at the same
time a principal creditor of the other;
(2)
That both debts consist in a sum of money, or if the things due are
consumable, they be of the same kind, and also of the same quality if the latter has been
stated;
(3)
That the two debts be due;
(4)
That they be liquidated and demandable;
(5)
That over neither of them there be any retention or controversy, commenced
by third persons and communicated in due time to the debtor."
- In the present case, set-off or legal compensation cannot take place between petitioner
and respondent So because they are not mutually creditor and debtor of each other.
- A careful reading of the Memorandum dated August 22, 1994 reveals that the lump sum
compensation of not less than US $6,000.00 will have to be refunded by each trainee to
BOHLER, not to petitioner.
T petitioner has no legal right to withhold respondents 13 th month pay and other benefits
to recompense for whatever amount it paid as security for respondent Villareals car loan;
and for the expenses incurred by respondent So in his training abroad.
Disposition petition is DENIED. The Decision dated October 29, 1999 and Resolution
dated May 8, 2000 of the Court of Appeals in CA-G.R. SP No. 50957 are hereby
AFFIRMED.

INTERNATIONAL SCHOOL ALLIANCE OF EDUCATORS


V QUISUMBING
KAPUNAN; June 1, 2000
NATURE
Petition for Review
FACTS
- Private respondent International School, Inc. (School), pursuant to PD 732, is a domestic
educational institution established primarily for dependents of foreign diplomatic personnel
and other temporary residents. The School hires both foreign and local teachers as
members of its faculty, classifying the same into two: (1) foreign-hires and (2) local-hires.
- The School grants foreign-hires certain benefits not accorded local-hires. These include
housing, transportation, shipping costs, taxes, and home leave travel allowance. Foreignhires are also paid a salary rate 25% more than local-hires. The School justifies the

Labor Law 1
difference on 2 "significant economic disadvantages" foreign-hires have to endure,
namely: (a) the "dislocation factor" and (b) limited tenure.
- At the negotiations for a new CBA, petitioner International School Alliance of Educators,
a legitimate labor union and the collective bargaining representative of all faculty
members of the School, contested the difference in salary rates between foreign and
local-hires. This issue, and the question of whether foreign-hires should be included in the
appropriate bargaining unit, eventually caused a deadlock between the parties.
- Petitioners filed notice of strike. The failure to bring parties to a compromise prompted
the DOLE to assume jurisdiction over the dispute. DOLE issued an Order resolving the
parity and representation issues in favor of the School. Then DOLE Sec. Quisumbing
denied petitioner's MFR. Petitioner now seeks relief in this Court.
- Petitioner claims that the point-of-hire classification employed by the School is
discriminatory to Filipinos and that the grant of higher salaries to foreign-hires constitutes
racial discrimination.
- The School disputes these claims and gives a breakdown of its faculty members,
numbering 38 in all, with nationalities other than Filipino, who have been hired locally and
classified as local hires. Also foreign hires have limited contract of employment unlike the
local hires who enjoy security of tenure
ISSUES
1. WON the International Schools point-of-hire classification for the distinction in salary
rates between foreign-hires and local-hires is discriminatory and an invalid classification
under the law.
2. WON foreign-hires should belong to the same bargaining unit as the local-hires
HELD
1. YES
Ratio The Constitution, Labor Code and the International Covenant on Economic, Social,
and Cultural Rights impregnably institutionalize in this jurisdiction the long honored legal
truism of "equal pay for equal work." Persons who work with substantially equal
qualifications, skill, effort and responsibility, under similar conditions, should be paid
similar salaries. This rule applies to the School, its "international character"
notwithstanding.
Reasoning
- The International Covenant on Economic, Social, and Cultural Rights in Art.7 provides
that: The States Parties to the present Covenant recognize the right of everyone to the
enjoyment of just and favorable conditions of work, which ensure, in particular, fair wages
and equal remuneration for work of equal value without distinction of any kind, in particular
women being guaranteed conditions of work not inferior to those enjoyed by men, with
equal pay for equal work. The Philippines, through its Constitution, has incorporated this
principle as part of its national laws.
- The Constitution specifically provides that labor is entitled to "humane conditions of
work." The Constitution also directs the State to promote "equality of employment
opportunities for all." Similarly, the Labor Code provides that the State shall "ensure equal
work opportunities regardless of sex, race or creed. Discrimination, particularly in terms of
wages, is frowned upon by the Labor Code (Art.135)
- The dislocation factor and limited tenure affecting foreign-hires are adequately
compensated by certain benefits accorded them which are not enjoyed by local-hires,
such as housing, transportation, shipping costs, taxes and home leave travel allowances.
Hence, the "dislocation factor" and the foreign-hires' limited tenure also cannot serve as
valid bases for the distinction in salary rates.
2. NO
- It does not appear that foreign-hires have indicated their intention to be grouped together
with local-hires for purposes of collective bargaining. The collective bargaining history in
the School also shows that these groups were always treated separately. Foreign-hires
have limited tenure; local-hires enjoy security of tenure. Although foreign-hires perform
similar functions under the same working conditions as the local-hires, foreign-hires are
accorded certain benefits not granted to local-hires. These benefits, such as housing,
transportation, shipping costs, taxes, and home leave travel allowance, are reasonably
related to their status as foreign-hires, and justify the exclusion of the former from the
latter.
Disposition Petition GRANTED IN PART. The Orders of the Sec. of Labor and
Employment are REVERSED and SET ASIDE insofar as they uphold the practice of
respondent School of according foreign-hires higher salaries than local-hires.

A2010

- 33 -

Disini

Labor Law 1

A2010

- 34 -

Disini

FERNAN [dissent]
-Providing a rigid mathematical formula for computing the amounts of separation pay
defies the spirit of the constitutional mandate that those who have less in life should have
more in law. These fixed rates would not favor the low-salaried employee as he would
encounter difficulty in finding another job.

GRINO-AQUINO [dissent]
-We should not rationalize compassion.

PLDT V NLRC (ABUCAY)


164 SCRA 671
CRUZ; August 23, 1988
FACTS
-Private respondent Marliyn Abucay, a traffic operator for PLDT, was accused by 2
complainants of having demanded and received from them Php3,800 in consideration of
her promise to facilitate the approval of their applications for phone installations.
Investigated and heard, she was found guilty and dismissed from service.
-Respondent filed a complaint for illegal removal with the Ministry of Labor and
Employment. The complaint was dismissed, but the Labor Arbiter granted her severance
of one month pay for every year of service. Hence this petition, claiming the granting of
financial assistance was made with grave abuse of discretion.
-Petitioner contends that although Art 279 if the Labor Code entitles an employee to
reinstatement and backwages if dismissed without just cause, one dismissed in
accordance with law is not allowed any relief at all as this would be tantamount to
rewarding the dissolute worker.
-Respondent NLRC claims that dismissal was sufficient punishment and that the grant of
financial assistance was intended to help her for the loss of employment after working
faithfully for 10 years. Similarly in the cases of Firestone Co. v Lariosa and Filipino, Inc. v
NLRC, employees who were validly dismissed on grounds of violation of company policies
were still awarded financial assistance on the ground of social and compassionate justice.
These cases ostensibly constitute the exception to Art 279, based on considerations of
equity.
ISSUE
WON it is legal to award financial assistance to an employee who had been dismissed
with just cause
HELD
NO. Separation pay shall only be allowed as a measure of social justice when the
employee is validly dismissed for causes other than serious misconduct and not involving
moral turpitude.
Reasoning
-There is no doubt it is compassionate to give separation pay when the cause is not
iniquitous as when a salesman is dismissed for his inability to fill his quota. His company
cannot be compelled to maintain him at the expense of the efficiency of their operations
but the awarding of financial assistance would be sustainable under the policy of social
justice. However, the same salesman surely does not deserve such generosity if his
offense is misappropriation of the receipts of his sales. Such conduct is not simply inept
but rather depraved and immoral, thus making him undeserving of such assistance.
-Our Constitution is replete with positive commands for the promotion of social justice, and
particularly the protection of the rights of the workers. However, social justice is not
intended to countenance wrongdoing simply because it is committed by the
underprivileged. Compassion for the poor is an imperative in every humane society, but
only when the recipient is not a rascal claiming an undeserved privilege. Hence, it may
only be invoked by those whose hands are clean and whose motives are blameless.
-Applying these considerations, the awarding of financial assistance is unjustified.
Disposition Petition is GRANTED

SEPARATE OPINION
PADILLA [concur]
-I concur. However, in cases were separation pay is awarded, the amount to be granted
should be left to the judgment of the NLRC (rather than the 1 month of pay for each year
of service rule) and should not be disturbed by the Court in absence of evidence of grave
abuse of discretion on the part of the NLRC.

EMPLOYERS CONFEDERATION OF THE PHILIPPINES


V NWPC
201 SCRA 759
SARMIENTO; September 24,1991
NATURE
Appeal
FACTS
- Employers Confederation of the Philippines is questioning the validity of the wage order
issued by the Regional Board of NCR which increased the minimum wage by P17.00 daily
including those already receiving wages above the statutory minimum wage up to P125.00
per day alleging that it was done in excess of the boards authority, and alleges that under
the Republic Act No. 6727, the boards may only prescribe "minimum wages," not
determine "salary ceilings."
- The said order was promulgated pursuant to the authority of Republic Act No. 6727,
which aside from providing new wage rates, also provides, among other things, for various
Regional Tripartite Wages and Productivity Boards in charge of prescribing minimum wage
rates for all workers in the various regions, and for a National Wages and Productivity
Commission to review, among other functions, wage levels determined by the boards.
- The Solicitor General was of the opinion that the Board in prescribing an across-theboard hike did not, in reality, "grant additional or other benefits to workers and employees,
such as the extension of wage increases to employees and workers already receiving
more than minimum wages . . ." but rather, fixed minimum wages according to the "salaryceiling method."
ECOP in its reply insisted that wage-fixing is a legislative function, and Republic Act No.
6727 delegated to the regional boards no more "than the power to grant minimum wage
adjustments" and "in the absence of clear statutory authority," the boards may no more
than adjust "floor wages."
ISSUES
1. WON the regional board of NCR performed an unlawful act of legislation in decreeing
an across the board hike
2. WON RA6727 intended to deregulate the relation between labor and capital
HELD
1. NO
- The National Capital Region, in decreeing an across-the-board hike did not performed an
unlawful act of legislation. It is true that wage-firing, like rate-fixing, constitutes an act
Congress; it is also true, however, that Congress may delegate the power to fix rates
provided that, as in all delegations cases, Congress leaves sufficient standards. As this
Court has indicated, it is impressed that the law provides standards which are sufficient,
and in the light of the floor-wage method's failure, the Court believes that the Commission
correctly upheld the Regional Board of the National Capital Region.
- RA 6727 was intended to rationalize wages, first, by providing for full-time boards to
police wages round-the-clock, and second, by giving the boards enough powers to
achieve this objective. Congress meant the boards to be creative in resolving the annual
question of wages without labor and management knocking on the legislature's door at
every turn.
2. NO
- Apparently, ECOP is of the mistaken impression that Republic Act No. 6727 is meant to
"get the Government out of the industry" and leave labor and management alone in
deciding wages. The Court does not think that the law intended to deregulate the relation
between labor and capital for several reasons: (1 ) The Constitution calls upon the State to
protect the rights of workers and promote their welfare; (2) the Constitution also makes it
a duty of the State "to intervene when the common goal so demands" in regulating
property and property relations; (3) the Charter urges Congress to give priority to the
enactment of measures, among other things, to diffuse the wealth of the nation and to
regulate the use of property; (4) the Charter recognizes the "just share of labor in the

Labor Law 1
fruits of production;" (5) under the Labor Code, the State shall regulate the relations
between labor and management; (6) under Republic Act No. 6727 itself, the State is
interested in seeing that workers receive fair and euitable wages; and (7) the Constitution
is primarily a document of social justice, and although it has recognized the importance of
the private sector, it has not embraced fully the concept of laissez faire or otherwise, relied
on pure market forces to govern the economy; We can not give to the Act a meaning or
intent that will conflict with these basic principles.

AGABON V NATIONAL LABOR RELATIONS


442 SCRA 573
YNARES-SANTIAGO; November 17, 2004
NATURE
Petition for review on certiorari
FACTS
- On January 2, 1992, petitioners Jenny Agabon and Virgilio Agabon were hired as
gypsum board and cornice installers by respondent Riviera Home Improvements, Inc., a
corporation engaged in the business of selling and installing ornamental and construction
materials. Seven (7) years later, on February 23, 1999, their services were terminated on
the ground of abandonment of work. Apparently, petitioners were subcontracting
installation jobs for another company and were frequently absent from work. Thus, when
petitioners reported for work on February 23, 1999, respondent company refused to
reemploy them unless they agree to work on a pakyaw basis. Petitioners demurred
since this would mean losing their benefits. They were given their walking papers without
according them the twin requirements of notice and hearing. Respondent company
stated that they abandon their jobs. Hence, petitioners filed a complaint for illegal
dismissal and payment of money claims against respondent company.
- On December 28, 1999, the Labor Arbiter held that the dismissal of petitioners was illegal
and ordered respondent company to pay them backwages, holidy and service incentive
leave pay, and separation pay in lieu of reinstatement. On appeal, the NLRC reversed the
decision of the Labor Arbiter and ruled that the latter erred in awarding backwages and
separation pay to petitioners who deliberately abandoned their work. On certiorari, the
Court of Appeals affirmed the findings of the NLRC but ordered respondent company to
pay petitioners their money claims.
ISSUES
1. WON petitioners were illegally dismissed from the service
2. WON private respondent should be held liable for non-compliance with the procedural
requirements of due process
HELD
1. NO
Ratio To dismiss an employee, the law requires not only the existence of a just and valid
cause but also enjoins the employer to give the employee the opportunity to be heard and
to defend himself. Article 282 of the Labor Code enumerates the just causes for
termination by the employer: (a) serious misconduct or willful disobedience by the
employee of the lawful orders of his employer or the latters representative in connection
with the employees work; (b) gross and habitual neglect by the employee of his duties; (c)
fraud or willful breach by the employee of the trust reposed in him by his employer or his
duly authorized representative; (d) commission of a crime or offense by the employee
against the person of his employer or any immediate member of his family or his duly
authorized representative; and (e) other causes analogous to the foregoing.
- In this case, Agabon abandoned their job. Abandonment is the deliberate and unjustified
refusal of an employee to resume his employment. It is a form of neglect of duty, hence, a
just cause for termination of employment by the employer. For a valid finding of
abandonment, these two factors should be present: (1) the failure to report for work or
absence without valid or justifiable reason; and (2) a clear intention to sever employeremployee relationship, with the second as the more determinative factor which is
manifested by overt acts from which it may be deduced that the employees has no more
intention to work. The intent to discontinue the employment must be shown by clear proof
that it was deliberate and unjustified.
2. YES
- Where the dismissal is for a just cause, as in the instant case, the lack of statutory due
process should not nullify the dismissal, or render it illegal, or ineffectual. However, the
employer should indemnify the employee for the violation of his statutory rights.
* It is worth noting that this ruling has evolved through times.

A2010

Disini

- 35 -

> Prior to 1989 - the rule was that a dismissal or termination is illegal if the employee was
not given any notice.
> In the 1989 case of Wenphil Corp. v. National Labor Relations Commission - where the
employer had a valid reason to dismiss an employee but did not follow the due process
requirement, the dismissal may be upheld but the employer will be penalized to pay an
indemnity to the employee. This became known as the Wenphil or Belated Due Process
Rule.
> On January 27, 2000, in Serrano - violation by the employer of the notice requirement in
termination for just or authorized causes was not a denial of due process that will nullify
the termination. However, the dismissal is ineffectual and the employer must pay full
backwages from the time of termination until it is judicially declared that the dismissal was
for a just or authorized cause.
Reasoning
a. Constitutional due process is different from statutory due process. The former protects
the individual from the government and assures him of his rights in criminal, civil or
administrative proceedings; while statutory due process found in the Labor Code and
Implementing Rules protects employees from being unjustly terminated without just cause
after notice and hearing.
b. The constitutional policy to provide full protection to labor is not meant to be a sword to
oppress employers. The commitment of this Court to the cause of labor does not prevent
us from sustaining the employer when it is in the right, as in this case.
Disposition DENIED. But the private respondent is ORDERED to pay each of the
petitioners the amount of P30,000.00 as nominal damages for non-compliance with
statutory due process.

SPEARATE OPINION
PUNO [dissent]
- He maintains his view that the workingmans right ot job security and due process
cannot be measured with a reduced price tag. He is unwilling to diminish petitioners right
to procedural due process which is necessary to protect their security of tenure. He
proffers the following precepts:
1. Our Constitution is an ode to social justice.
- Here, Puno defines what social justice is. It is that virtue by which individuals and
groups fulfil their obligations to human society by contributing positively to the complete
well-being of their fellowmen considered as members of that society, and hence their
actions accordingly.
- Then Justice Puno enumerated the litany of constitutional provisions regarding social
justice starting from 1935 Constitution up to the present, giving emphasis that provisions
on protection to labor have long been embedded in all our Constitutions, and thus at the
very least should be respected and protected by our courts.
2. Courts at all times should give meaning and substance to constitutional postulates in
favour of the workingman.
- He emphasized that these substantive rights are not to be weakened by a diminish
procedural right. For in weakening the procedure, we weaken the substantive right.
3. The Constitution puts the employee on equal footing with his employer.
- As between an employee, usually poor and unlettered, and the employer, who has
resources to secure legal advice, the law has reason to demand from the latter stricter
compliance. For, social justice in these cases is not equality but protection.
4. This Court has long extended constitutional due process in labor cases involving
private action.
- Here, Justice Puno enumerated cases that hold that dismissal of employee without
notice constitute an infringement of his constitutional right which must be safeguarded at
all times. It debunks the argument of the majority that it is only statutory due process that
was violated and not the employees constitutional right as the case involves private
individuals and not the State. However, Puno stressed out that the posture that the
constitutional due processs requirement limits government action alone and does not
apply to private action is already pass.
5. An employee who is denied procedural due process is entitled to reinstatement.
Nothing less.
6. Compliance with procedural due process is not a burden on employers.
7. In the hierarchy of rights of an employee, the right to security of tenure is high, if not
the highest.
- All other complementary rights (i.e. right to collective bargaining and negotiations, the
right to strike, etc) are meaningless to an unemployed Juan dela Cruz. It is the policy of
the State to assure the rights of workers to security of tenure. The guarantee is an act of
social justice.
8. Workers need more work than anything else.
- Work is a defining feature of human existence.
9. To simply allow payment of nominal damages for violation of employers right to due
process is to give undue advantage to employers.
- The deprivation of the right to security of tenure and due process is beyond monetary
valuation.
- This Court should protect labor and it should walk the talk.

Labor Law 1
PANGANIBAN [dissent]
- I believe that even if there was just or authorized cause for termination of employment,
but due process was not afforded the employee, the dismissal proceedings must be
declared null and void. Consequently, the employee must be reinstated and given full
back wages and accruing benefits. Depending on the facts of each case, damages as
provided under applicable articles of the Civil Code may additionally be awarded.
- An exception may be entertained if the employer could adequately prove that under the
peculiar circumstances of the case, there was no opportunity to comply with due process
requirements; or doing so would have been impractical or gravely adverse to the
employer, as when the employee was caught in flagrante delicto. Under such
circumstance, dismissal would not be illegal and no award may properly granted.
Nevertheless, as a measure of compassion in this specific instance, the employee may be
given a nominal sum depending on the circumstances, pursuant to Article 2221 of the Civil
Code.

TINGA [concur in the result]


1. The Abandonment Dimension - Agabon abandoned his work. They failed to report
for work or absence without valid or justifiable reason, and a clear intention to sever the
employer-employee relationship.
2. Compliance with Notice Requirement 2 letters sent by private respondent sufficiently
complied with the notice rule. However, the actual violation of the notice requirement by
Riviera Homes lies in its failure to serve on the Agabons the second notice which should
inform them of termination. But this not invalidate the the Agabons dismissal for just
cause.
3. The So-Called Constitutional Law Dimension - The first is that the Due Process Clause
of the Constitution is a limitation on governmental powers. It does not apply to the exercise
of private power, such as the termination of employment under the Labor Code. The
scope and reach of authority of an employer pales in comparison with that of the State.
4. Constitutional Protection of Labor - As manifested by several framers of the 1987
Constitution, the provisions on social justice require legislative enactments for their
enforceability.
5. The Effect of Statutory Violation - Based on reading Section 279 of the Labor Code,
the existence of just cause by itself is sufficient to validate the termination.
Neither the Labor Code nor its implementing rules states that a termination for just cause
is voided because the requirement of notice and hearing was not observed.
6. The Damages Dimensions - Award for Damages Must Have Statutory Basis
- The Impropriety of Award for Separation Pay - separation pay is warranted only for
dismissals for authorized causes, as enumerated in Article 283 and 284 of the Labor
Code.
- The Impropriety of Equity Awards -Social justice should be the aspiration of all that
we do, yet I think it the more mature attitude to consider that it ebbs and flows
within our statutes, rather than view it as an independent source of funding.
- Article 288 of the Labor Code as a Source of Liability
Rule on Damages:

failure to comply with the statutory requirement of notice automatically gives


rise to nominal damages, at the very least, even if the dismissal was
sustained for just cause.

Actual or compensatory damages - employee dismissed for just cause but


denied statutory due process.

pecuniary loss arising - temperate or moderate damages under Article 2224


of the Civil Code.

Moral and exemplary damages - dismissal of the employee was attended by


bad faith, fraud, or was done in a manner contrary to morals, good customs or
public policy, or the employer committed an act oppressive to labor.
Exemplary damages - dismissal was effected in a wanton, oppressive or
malevolent manner.
Appropriate Award of Damages to the Agabons
- the only appropriate award of damages is nominal damages.

A2010

- 36 -

Disini

Labor Law 1

A2010

Disini

- 37 -

- There was no formal investigation of the charges.


- Respondent was not furnished with a copy of the written notice of the penalty imposed
on him.
- For such violation, petitioners are liable for moral damages or for indemnity of P30,000, if
the respondent fails to prove such moral damages. 8 In this case, the respondent failed to
prove such moral damages.
Disposition AFFIRMED with MODIFICATION. Petitioners are ordered to pay P30,000
by way of indemnity. The awards for other damages and attorneys fees are deleted.

UY V BUENO
484 SCRA 628
PUNO; March 14, 2006
DMA SHIPPING PHILS INC V CABILLAR
452 SCRA 551
CALLEJO, SR; February 28, 2005
NATURE
Petition for Review
FACTS
- Henry Cabillar was hired by Monsoon, through DMA Shipping, as Chief Officer of the
M/V Eagle Moon.
- After three (3) months, Cabillar wrote the manager of Monsoon, requesting for an early
repatriation and for his reliever grounded on the failure of DMA Shipping to give the
promised additional allowance. Monsoon approved an increase in Cabillars wage and the
latter withdrew his request for repatriation.
- While the vessel was docked in India, the gantry crane operators refused to work and
demanded for an increase in their allowance. The master of the M/V Eagle Moon
instructed Cabillar to talk to the crew members under his immediate supervision to
convince them not to proceed with the intended strike and have the matter discussed with
the management when the vessel returns to Singapore.
- Instead of talking to the crew members, Cabillar himself joined the strike. Monsoon
expressed its displeasure on Cabillar for joining the strike. Nevertheless, Monsoon agreed
to the demands of the striking crew members to avert any further losses.
- When the vessel arrived at Singapore, officers of Monsoon informed Cabillar that he has
been separated from his employment because of the incident in Calcutta.
- Cabillar filed a complaint with the POEA against DMA and Monsoon seeking payment for
the unexpired portion of his contract.
- The Executive Labor Arbiter rendered a decision in favor of Cabillar declaring his
dismissal as illegal. The NLRC and the Court of Appeals affirmed. Hence this petition.
ISSUES
1. WON the respondent was dismissed by the petitioner Monsoon and
2. If so, WON his dismissal was for a valid cause
3. WON the respondent is entitled to backwages, damages and attorneys fees
HELD
1. Petitioner was dismissed.
Ratio WON the respondent was dismissed or that he resigned as chief officer of the
vessel is a question of fact. The labor arbiter ruled that the respondent was dismissed.
The NLRC and the Court of Appeals affirmed. Petitioners failed to make a clear showing
that the findings were are arbitrary and bereft of any rational basis.
Reasoning
- The entry in the logbook of the vessel shows that the ship captain, for and in behalf of
the petitioners, dismissed the respondent for joining the strike.
- The petitioners failed to adduce documentary evidence to prove their allegation that (1)
they and the respondent agreed that in consideration for the respondents resignation,
they would give him a very good rating (2) they defrayed his plane fare back to the
Philippines (3) they paid for his hotel bills in Singapore.
2. Respondent was dismissed for just cause.
Ratio Under their employment contracts, the crew and officers of the vessel bound
themselves to follow certain procedures for their grievances.
Reasoning
- The crew and the respondent refused to follow the procedure and stop the strike.
- They may have a valid grievance against the petitioners but they are bound to follow the
procedures set forth in their contracts of employment to address said grievances.
3. Petitioners are to pay indemnity.
Ratio The petitioners themselves violated their contracts of employment with the
respondent and the crew because the captain of the vessel failed to comply with the
disciplinary procedures.
Reasoning
- The respondent was not furnished with any written notice of any charges against him.

NATURE
Petition for review on certiorari of a decision of the CA
FACTS
- Amalia Bueno was the Manager of Countrywide Rural Bank of La Carota, Inc. (bank
hereafter) Marbel Branch. She was verbally and summarily dismissed by Atty. Andrea Uy,
interim President and Corporate Secretary of the bank, during a depositors' meeting.
- Bueno filed a case for illegal dismissal and prayed for reinstatement with full backwages
and damages.
ISSUE
WON Uy is an officer of the bank, making her soldarily liable with the corporation for illegal
dismissal
HELD
NO
- The minutes of the depositors' meeting clearly showed that Uy was a mere depositor of
the bank. She was only elected as officer of the Interim Board of Directors craeted by the
association of depositors with the sole task of rehabilitating the bank (which is under
receivership).
- There is no evidence that the association of depositors that elected the interim board
was recognized by BSP. Hence, it had no legal authority to act for the bank.
- The act of dismissing Bueno by Uy cannot be deemed as an act as an officer of the
bank. Consequently, it cannot be held that there existed an employer-employee
relationship between Uy and Bueno.
- The requirement of employer-employee relationship is jurisdictional for the provisions of
the Labor Code on Post-employment to apply. Since such relationship was not
established, the labor arbiter never acquired jurisdiction over Uy.
Disposition CA decision finding Uy solidarily liable with the bank reversed

CABRERA V NLRC (NATIONAL SERVICE


CORPORATION, VILLAMOR)
198 SCRA 573
CRUZ; June 27, 1991
NATURE
Appeal from the decision of the NLRC dismissing the complaint for illegal dismissal by the
petitioners on the ground that it is without jurisdiction.
FACTS
- Dismissed by the National Service Corporation, the petitioners complained to the Ministry
of Labor and Employment on September 17, 1980. After considering the position papers of
the parties, the Labor Arbiter ordered the petitioners' reinstatement without loss of
seniority rights and the payment to them of two years back wages and other benefits. 3
The decision was appealed to and affirmed by the First Division of the NLRC on
December 9, 1985, and in due time, the petitioners moved for the issuance of a writ of
execution. This was opposed by NASECO on the ground that it had not been furnished
with a copy of the decision, but the opposition was rejected and the petition was granted.
Reconsideration of the order having been denied, the NASECO appealed to the NLRC,
which, through its Third Division this time, declared itself without jurisdiction and dismissed
the case on August 18, 1987. 4 Citing the NHA case, the public respondent held that the
NASECO was not covered by the Labor Code but by Civil Service rules and regulations,
being a government-owned or controlled corporation.
ISSUE
WON the National Service Corporation is covered by the Labor Code
8

Agabon v. NLRC, G.R. No. 158693, 442 SCRA 573, Nov. 17, 2004.

Labor Law 1
HELD
YES
- The decision in National Housing Corporation v. Juco was already overturned by the
decision in National Service Corporation v. NLRC. The NLRC erred in dismissing the
petitioners' complaint for lack of jurisdiction because the rule now is that only governmentowned or controlled corporations with original charters come under the Civil Service. The
NASECO having been organized under the Corporation Law and not by virtue of a special
legislative charter, its relations with its personnel are governed by the Labor Code and
come under the jurisdiction of the National Labor Relations Commission.

GAMUGAMO V PNOC SHIPPING AND TRANSPORT


CORP
381 SCRA742
DAVIDE JR; May 7, 2002
NATURE
Special Civil Action in the Supreme Court. Certiorari
FACTS
- On January 23,1963, petitioner Cayo Gamogamo was employed with the Department of
Health as Dental Aide (wherein he was also promoted to the position of Dentist 1). He
remained employed at the DOH for 14 years until he resigned on November 2, 1977.
- On November 9, 1977, petitioner was hired as a company dentist by Luzon Stevedoring
Corporation (LUSTEVECO). Subsequently, respondent PNOC Shipping and Transport
Corp acquired and took over the shipping business of LUSTEVECO. Petitioner was
among those who opted to be absorbed by the respondent. He continued to work as a
company dentist.
- ON June 10,1993, President Fidel V. Ramos issued a memorandum approving the
privatization of PNOC subsidiaries, including respondent pursuant to the provisions of
Section III (B) of the Guidelines and Regulations to implement E.O. No. 37. Respondent
implemented a Manpower Reduction Program to govern employees whose respective
positions have been classified as redundant (respondent decreased its operations and
downsized its organization due to lay up and sale of its vessels.
- Sometime in 1995, petitioner requested to be included in the next retrenchment
schedule. However, his request was turned down because: 1.) he was holding a
permanent position 2.) he was already due for mandatory retirement in April 1995 under
his retirement plan. Eventually petitioner retired after serving respondent for 17 years and
4 mos. He received a retirement pay which is equivalent to one month pay for every year
of service and other benefits (P512,524.15)
- On August 30,1995, respondents president died and was replaced by Nemesio Prudente
who implemented significant cost-saving measures. He ordered that a study on the costeffect of the retrenchment of employees be conducted (upon motion of 2 other employees,
Dr. Rogelio Buena (company doctor) and Mrs. Luz C. Reyes (telephone operator) who
were holding permanent/non-redundant positions. These 2 employees were retrenched
and paid a 2-month separation pay for every year of service under Respondents
Manpower Reduction Program.
- In view of the action taken by respondent in the retrenchment of the said 2 employees,
petitioner filed a complaint at the NLRC for the full payment of his retirement benefits.
Petitioner argued that his service with the DOH should have been included in the
computation of his years of service. Hence, with an accumulated service of 32 years he
should have been paid a 2-month pay for every year of service per the retirement plan
(and thus should have received at least P1,833,920)
- The Labor Arbiter dismissed petitioners complaint. On appeal however, the NLRC
reversed the decision of the Labor Arbiter (considering the 14 years of his service to DOH)
- Respondent filed with the CA a special civil action for certiorari. CA set aside the
judgment of the NLRC. Hence, petitioner filed this petition alleging that 1.) his years of
service with the DOH must be considered as creditable service for the purpose of
computing his retirement pay 2.) he was discriminated against in the application of the
Manpower Reduction Program.
ISSUE
WON, for the purpose of computing an employees retirement pay, prior service rendered
in a government agency can be tacked in and added to the creditable service later
acquired in a government-owned and controlled corporation without original charter.
HELD
NO
- Petition denied and the appealed decision in CA is affirmed (in favor of respondent)
Ratio The Court cannot uphold petitioners contention that his 14 years of service with the
DOH should be considered because his last 2 employers were government-owned and
controlled corporations and fall under the Civil Service Law. Article IX (B), Section 2
paragraph 1 of the 1987 Constitution states: The civil service embraces all branches,

A2010

- 38 -

Disini

subdivisions, instrumentalities, and agencies of the Government, including government


owned or controlled corporations with original charters. While respondent and
LUSTEVECO are government-owned and controlled corporations, they have no original
charters; hence, they are not under the Civil Service Law
Reasoning
- totalization of service credits is only resorted to when the retiree does not qualify for
benefits in either of both systems. In this case, petitioner is qualified to receive benefits
granted by the GSIS, if such right has not yet been exercised.
- It may also be pointed out that upon his receipt of the amount of P512,524.15 from
respondent as retirement benefit pursuant to its retirement scheme, petitioner signed and
delivered to respondent a release and undertaking wherein he waives all actions, causes
of actions, debts, dues, monies and accounts in connection with his employment with
respondent. This quitclaim releases respondent from any other obligation in favor of
petitioner.

LIGHT RAILWAY TRANSIT AUTHORITY V VENUS


485 SCRA 301
PUNO; March 24, 2006
FACTS
- consolidated petitions of Light Rail Transit Authority (LRTA) and Metro Transit
Organization, Inc. (METRO), seeking the reversal of the Decision of the Court of Appeals
directing them to reinstate private respondent workers to their former positions without
loss of seniority and other rights and privileges, and ordering them to jointly and severally
pay the latter their full back wages, benefits, and moral damages. The LRTA and METRO
were also ordered to jointly and severally pay attorneys fees equivalent to ten percent
(10%) of the total money judgment.
- Petitioner LRTA is a government-owned and controlled corporation created by Executive
Order No. 603, Series of 1980, as amended, to construct and maintain a light rail transit
system and provide the commuting public with an efficient, economical, dependable and
safe transportation. Petitioner METRO, formerly Meralco Transit Organization, Inc., was a
qualified transportation corporation duly organized in accordance with the provisions of the
Corporation Code, registered with the Securities and Exchange Commission, and existing
under Philippine laws. petitioner LRTA, after a bidding process, entered into a ten (10)year Agreement for the Management and Operation of the Metro Manila Light Rail Transit
System from June 8, 1984 until June 8, 1994 with petitioner METRO.
- The Agreement provided, among others, that
4. METRO shall be free to employ such employees and officers as it shall deem
necessary in order to carry out the requirements of [the] Agreement. Such employees
and officers shall be the employees of METRO and not of the AUTHORITY [LRTA].
METRO shall prepare a compensation schedule and the corresponding salaries and
fringe benefits of [its] personnel in consultation with the AUTHORITY [LRTA] [par.
3.05];
- On July 25, 2000, the Union filed a Notice of Strike with the National Conciliation and
Mediation Board National Capital Region against petitioner METRO on account of a
deadlock in the collective bargaining negotiation. On the same day, the Union struck.
They completely paralyzed the operations of the entire light rail transit system. As the
strike adversely affected the mobility of the commuting public, then Secretary of Labor
Bienvenido E. Laguesma issued on that same day an assumption of jurisdiction order [3]
directing all the striking employees to return to work immediately upon receipt of this
Order and for the Company to accept them back under the same terms and conditions of
employment prevailing prior to the strike
- Despite the issuance, posting, and publication of the assumption of jurisdiction and
return to work order, the Union officers and members failed to return to work. Thus,
effective July 27, 2000, private respondents, were considered dismissed from employment
- Workers filed a complaint for illegal dismissal. On October 1, 2001, Labor Arbiter Luis D.
Flores rendered a consolidated judgment in favor of the private respondent workers
- On May 29, 2002, on appeal, the NLRC found that the striking workers failed to heed the
return to work order and reversed and set aside the decision of the labor arbiter. The suit
against LRTA was dismissed since LRTA is a government-owned and controlled
corporation created by virtue of Executive Order No. 603 with an original charter and it
ha[d] no participation whatsoever with the termination of complainants employment. In
fine, the cases against the LRTA and METRO were dismissed, respectively, for lack of
jurisdiction and for lack of merit.
- On a petition for certiorari however, the Court of Appeals reversed the NLRC and
reinstated the Decision rendered by the Labor Arbiter. Public respondent appellate court
declared the workers dismissal as illegal, pierced the veil of separate corporate
personality and held the LRTA and METRO as jointly liable for back wages.
ISSUE
WON LRTA should be held liable for the illegal dismissal of employees
HELD

Labor Law 1
NO
- petitioner LRTA argues that it has no employer-employee relationship with private
respondent workers as they were hired by petitioner METRO alone pursuant to its ten
(10)-year Agreement for the Management and Operation of the Metro Manila Light Rail
Transit System with petitioner METRO. Piercing the corporate veil of METRO was
unwarranted, as there was no competent and convincing evidence of any wrongful,
fraudulent or unlawful act on the part of METRO, and, more so, on the part of LRTA.
- Petitioner LRTA further contends that it is a government-owned and controlled
corporation with an original charter, Executive Order No. 603, Series of 1980, as
amended, and thus under the exclusive jurisdiction only of the Civil Service Commission,
not the NLRC.
- We agree with petitioner LRTA. Section 2 (1), Article IX B, 1987 Constitution, expressly
provides that [t]he civil service embraces all branches, subdivisions, instrumentalities, and
agencies of the Government, including government-owned or controlled corporations with
original charters. Corporations with original charters are those which have been created
by special law and not through the general corporation law.
- In Philippine National Oil Company Energy Development Corporation v. Hon.
Leogrado- under the present state of the law, the test in determining whether a
government-owned or controlled corporation is subject to the Civil Service Law is the
manner of its creation such that government corporations created by special charter are
subject to its provisions while those incorporated under the general Corporation Law are
not within its coverage.
- There should be no dispute then that employment in petitioner LRTA should be governed
only by civil service rules, and not the Labor Code and beyond the reach of the
Department of Labor and Employment, since petitioner LRTA is a government-owned and
controlled corporation with an original charter, Executive Order No. 603, Series of 1980
- In contrast, petitioner METRO is covered by the Labor Code despite its later acquisition
by petitioner LRTA. In Lumanta v. National Labor Relations Commission, this Court
ruled that labor law claims against government-owned and controlled corporations without
original charter fall within the jurisdiction of the Department of Labor and Employment and
not the Civil Service Commission
- We therefore hold that the employees of petitioner METRO cannot be considered as
employees of petitioner LRTA. The employees hired by METRO are covered by the Labor
Code and are under the jurisdiction of the Department of Labor and Employment, whereas
the employees of petitioner LRTA, a government-owned and controlled corporation with
original charter, are covered by civil service rules. Herein private respondent workers
cannot have the best of two worlds, e.g., be considered government employees of
petitioner LRTA, yet allowed to strike as private employees under our labor laws.

EBRO III V NLRC (INTERNATIONAL CATHOLIC


MIGRATION COMMISSION)

261 SCRA 399


MENDOZA; September 4, 1996
NATURE
Petition for review on certiorari to set aside the order dated October 13, 1992 and the
resolution dated March 3, 1993 of the National Labor Relations Commission.

FACTS
- Private respondent International Catholic Migration Commission (ICMC) is a non-profit
agency engaged in international humanitarian and voluntary work. It is duly registered with
the United National Economic and Social Council (ECOSOC) and enjoys Consultative
Status, Category II. It was one of the agencies accredited by the Philippine Government to
operate the refugee processing center at Sabang, Morong, Bataan.
- On June 24, 1985, private respondent ICMC employed petitioner Jose G. Ebro III to
teach "English as a Second Language and Cultural Orientation Training Program" at the
refugee processing center. The employment contract provided in pertinent part:
Salary: Your monthly salary for the first 6 months probationary period is P3,155.00
inclusive of cost of living allowance. Upon being made regular after successful
completion of the six (6) months probationary period your monthly salary will be
adjusted to P3,445.00 inclusive of cost of living allowance
If either party wishes to terminate employment, a notice of two (2) weeks should be
given in writing to the party.
- After six months, ICMC notified petitioner that effective December 21, 1985, the latter's
services were terminated for his failure to meet the requirements of "1. classroom
performance . . . up to the standards set in the Guide for Instruction; 2. regular attendance
in the mandated teacher training, and in the schedule team meetings, one-on-one
conferences with the supervisor, etc.; and 3. compliance with ICMC and PRPC policies

A2010

- 39 -

Disini

and procedures."
- On February 4, 1986, petitioner filed a complaint for illegal dismissal, unfair labor
practice, underpayment of wages, accrued leave pay, 14th month pay, damages,
attorney's fees, and expenses of litigation. Petitioner alleged that there was no objective
evaluation of his performance to warrant his dismissal and that he should have been
considered a regular employee from the start because ICMC failed to acquaint him with
the standards under which he must qualify as such. He prayed for reinstatement with
backwages; P3,155.00 for probationary and P3,445.00 for regular salary adjustments;
value of lodging or dormitory privileges; cost of insurance coverage for group life, medical,
death, dismemberment and disability benefits; moral, and exemplary, and nominal
damages plus interest on the above claims with attorney's fees.
- Answering the complaint, ICMC claimed that petitioner failed to quality for regular
employment because he showed no interest in improving his professional performance
both in and out of the classroom after he had been periodically evaluated; that petitioner
was paid his salary up to December 31, 1985, two weeks pay in lieu of notice, and 14th
month pay pro-rata; and that his accrued leave balance already been converted to cash.
- After the parties had formally offered their evidence, private respondents submitted their
memorandum on July 31, 1989 in which, among other things, they invoked ICMC's
diplomatic immunity on the basis of the Memorandum of Agreement signed on July 15,
1988 between the Philippines government and ICMC.
- The Labor Arbiter held that ICMC's legal immunity under the Memorandum could not be
given retroactive effect since "[that would] deprive complainant's property right without due
process and impair the obligation of contract of employment." He also expressed doubt on
the ground that it was provided for by agreement and not through an act of Congress.
Accordingly, the Labor Arbiter ordered ICMC to reinstate petitioner as regular teacher
without loss of seniority rights and to pay him one year backwages, other benefits, and ten
percent attorney's fees for a total sum of P70,944.85.
- Both parties appealed to the NLRC. On August 13, 1990, petitioner moved to dismiss
private respondent's appeal because of the latter's failure to post a cash/surety bond. In its
order of October 13, 1992, however, the NLRC ordered the case dismissed on the ground
that, under the Memorandum of Agreement between the Philippine government and
ICMC, the latter was immune from suit.
ISSUE
WON the Memorandum of Agreement executed on July 15, 1988 granted ICMC immunity
from suit

HELD
YES
Ratio The grant of immunity from local jurisdiction to ICMC . . . is clearly necessitated by
their international character and respective purposes. The objective is to avoid the danger
of partiality and interference by the host country in their internal workings. The exercise of
jurisdiction by the Department of Labor in these instances would defeat the very purpose
of immunity, which is to shield the affairs of international organizations, in accordance with
international practice, from political pressure or control by the host country to the prejudice
of member State of the organization, and to ensure the unhampered performance of their
functions. (International Catholic Migration Commission v. Calleja)
Reasoning
- The grant of immunity to ICMC is in virtue of the Convention on the Privileges and
Immunities of Specialized Agencies of the United Nations, adopted by the UN General
Assembly on November 21, 1947, and concurred in by the Philippine Senate on May 17,
1949. This Convention has the force and effect of law, considering that under the
Constitution, the Philippines adopts the generally accepted principles of international law
as part of the law of the land.

- The scope of immunity of the ICMC contained in the


Convention on the Privileges and Immunities of the
Specialized Agencies of the United Nations is
instructive. Art. III, 4 of the Convention provides for
immunity from "every form of legal process." Thus,

Labor Law 1
even if private respondents had been served summons
and subpoenas prior to the execution of the
Memorandum, they, as officers of ICMC, can claim

A2010

- 40 -

Disini

service may enjoy the privilege and benefits provided by the school, such as free tuition
for the teachers children.
- Ms. Belo, nonetheless, took her leave of absence. In May 1993, she attempted to return
to CKSC and signified her readiness to teach for the coming school year. However, she
was not allowed to return. Hence, she filed a complaint for illegal dismissal, among others,
against CKSC.
- The Labor Arbiter dismissed the complaint but the NLRC disagreed. The Court of
Appeals upheld the NLRCs ruling. Hence, this petition.

immunity under the same in order to prevent

ISSUE
WON private respondent was constructively dismissed

enforcement of an adverse judgment, since a writ of

NATIONAL MINES AND ALLIED WORKERS UNION V


SAN ILDEFONSO COLLEGE

HELD
YES
- Under the Manual of Regulations for Private Schools, for a private school teacher to
acquire a permanent status of employment and, therefore, be entitled to a security
of tenure, the following requisites must concur: (a) the teacher is a full-time teacher;
(b) the teacher must have rendered three consecutive years of service; and (c) such
service must have been satisfactory. Since Ms. Belo has measured up to these
standards, she therefore enjoys security of tenure.
- Constructive dismissal is defined as a cessation from work because continued
employment is rendered impossible, unreasonable, or unlikely; when there is a demotion
in rank or a diminution in pay or both; or when a clear discrimination, insensibility, or
disdain by an employer becomes unbearable to the employee.
- Ms. Belo was constructively dismissed when the petitioners, in implementing their
policies, effectively barred her from teaching for the school year 1993-1994. The three
policies are (1) the non-assurance of a teaching load to a teacher who took a leave of
absence; (2) the hiring of non-permanent teachers in April to whom teaching loads were
already assigned when Ms. Belo signified in May 1993 her intention to teach; and (3) the
non-applicability to children of teachers on leave of the free tuition fee benefits extended to
children of teachers in service.
- Ms. Belo was definitely singled out in the implementation of a future policy (i.e., the policy
that employees not in service are not entitled to any benefit extended by the school). The
petitioners did not take heed of the principle enshrined in our labor laws that policies
should be adequately known to the employees and uniformly implemented to the body of
employees as a whole and not in isolation.
- The continued employment of Ms. Belo was also rendered unlikely by the insistence of
the petitioners in implementing the alleged policy that a teacher who goes on leave for one
year is not assured of a teaching load. While this alleged policy was mentioned in Mr.
Chiens letter of 9 June 1992, it was not included in the schools written statement of
policies dated 13 March 1992. Hence, it was then a non-existent policy. When a nonexistent policy is implemented and, in this case, only to Ms. Belo, it constitutes a clear
case of discrimination.
- Petitioners invocation of the third policy that of giving teaching assignments to
probationary teachers in April to justify their refusal to provide Ms. Belo a teaching load
is a lame excuse that rings of untruth and dishonesty. Patently clear is the illegal manner
by which the petitioners eased out Ms. Belo from the teaching corps.
- Likewise, we do not find merit in petitioners assertion that the Court of Appeals should
not have passed upon the illegality of the school policy of non-assurance of a teaching
load, since the alleged illegality was never raised as an issue before the respondent court
or in the forums below. As pointed out by the private respondent, that policy was part of
the defense invoked by the petitioners in the Arbiter level, in the NLRC, and in the
respondent court to the charge of illegal dismissal; and, hence, it must necessarily be
passed upon and scrutinized. Besides, that policy is intimately intertwined with the main
issue of whether Ms. Belo was illegally dismissed.
- This case is an exception to the general rule that the factual findings and conclusions of
the Labor Arbiter are accorded weight and respect on appeal, and even finality. For one
thing, the findings of the NLRC and the Labor Arbiter are contrary to each other; hence,
the reviewing court may delve into the records and examine for itself the questioned
findings.
Disposition The Petition is DENIED.

CHIANG KAI SHEK COLLEGE V CA (NLRC,


CALAYLAY, AQUINO, GACUTAN, BELO)
437 SCRA 171
DAVIDE, JR; August 24, 2004

AUSTRIA V NLRC (CENTRAL PHIL. UNION MISSION


CORP. OF THE 7TH-DAY ADVENTIST)
312 SCRA 410
KAPUNAN; August 16, 1999

FACTS
- In 1992, Ms. Diana Belo, a teacher of Chiang Kai Shek College since 1977, applied for a
leave of absence for the school year 1992-1993. Upon submitting her application, she was
informed of the school policy that if she takes a leave of absence, she is not assured of a
teaching load upon her return. She was likewise informed that only teachers in active

FACTS
- Pastor Dionisio Austria worked with the Central Philippine Union Mission Corporation of
the Seventh Day Adventists (SDA) for 28 years from 1963 to 1991. He began his work
with the SDA on 15 July 1963 as a literature evangelist, selling literature of the SDA over
the island of Negros. From then on, he worked his way up the ladder and got promoted
several times. In January, 1968, he became the Assistant Publishing Director in the West

execution is "a legal process" within the meaning of


Article III, 4.
- Art III 4 of the Convention on the Privileges and
Immunities of the Specialized Agencies of the United
Nations requires that the waiver of the privilege must
be express. There was no such waiver of immunity in
this case. Nor can ICMC be estopped from claiming
diplomatic immunity since estoppel does not operate
to confer jurisdiction to a tribunal that has none over a
cause of action.
- Finally, neither can it be said that recognition of ICMC's immunity from suit deprives
petitioner of due process. As pointed out in ICMC v. Calleja, petitioner is not exactly
without remedy for whatever violation of rights it may have suffered for the following
reason:
Section 31 of the Convention on the Privileges and Immunities of the Specialized
Agencies of the United Nations provides that "each specialized agency shall make
provision for appropriate modes of settlement of: (a) disputes arising out of contracts or
other disputes of private character to which the specialized agency is a party." Moreover,
pursuant to Article IV of the Memorandum of Agreement between ICMC and the Philippine
Government, whenever there is any abuse of privilege by ICMC, the Government is free to
withdraw the privileges and immunities accorded.
Disposition Petition is DISMISSED for lack of merit.

Labor Law 1
Visayan Mission. In July, 1972, he was elevated to the position of Pastor covering the
island of Panay, and the provinces of Romblon and Guimaras. He held the same position
up to 1988. Finally, in 1989, he was promoted as District Pastor of the Negros Mission of
the SDA and was assigned at Sagay, Balintawak and Toboso, Negros Occidental, with 12
churches under his jurisdiction. In January, 1991, he was transferred to Bacolod City. He
held the position of district pastor until his services were terminated on 31 October 1991.
- On various occasions from August up to October, 1991, Eufronio Ibesate, the treasurer of
the Negros Mission asked him to admit accountability and responsibility for the church
tithes and offerings collected by his wife, Thelma Austria, in his district which amounted to
P15,078.10, and to remit the same to the Negros Mission. Petitioner reasoned out that he
should not be made accountable for the unremitted collections since it was Pastor Gideon
Buhat and Ibesate who authorized his wife to collect the tithes and offerings since he was
very sick to do the collecting at that time.
- On 16 October 1991, petitioner went to the office of Pastor Buhat, the president of the
Negros Mission. During said call, petitioner tried to persuade Pastor Buhat to convene the
Executive Committee for the purpose of settling the dispute between him and Pastor
David Rodrigo. The dispute between David Rodrigo and petitioner arose from an incident
in which petitioner assisted his friend, Danny Diamada, to collect from Pastor Rodrigo the
unpaid balance for the repair of the latter's motor vehicle which he failed to pay to
Diamada. Due to the assistance of petitioner in collecting Pastor Rodrigo's debt, the latter
harbored ill-feelings against petitioner. When news reached petitioner that Pastor Rodrigo
was about to file a complaint against him with the Negros Mission, he immediately
proceeded to the office of Pastor Buhat and asked the latter to convene the Executive
Committee. Pastor Buhat denied the request of petitioner since some committee members
were out of town and there was no quorum. Thereafter, the two exchanged heated
arguments. Petitioner then left the office of Pastor Buhat. While on his way out, petitioner
overheard Pastor Buhat saying "Pastor daw inisog na ina iya (Pastor you are talking
tough)." Irked by such remark, petitioner returned to the office of Pastor Buhat, and tried to
overturn the latter's table, though unsuccessfully, since it was heavy. Thereafter, petitioner
banged the attache case of Pastor Buhat on the table, scattered the books in his office,
and threw the phone. Fortunately, Pastors Yonillo Leopoldo and Claudio Montao were
around and they pacified both.
- On 17 October 1991, petitioner received a letter inviting him and his wife to attend the
Executive Committee meeting. From October 21 to 22, the fact-finding committee
conducted an investigation. Petitioner immediately wrote Pastor Rueben Moralde,
president of the SDA and chairman of the fact-finding committee, requesting that certain
members of the fact-finding committee be excluded in the investigation and resolution of
the case. Out of the 6 members requested to inhibit themselves from the investigation and
decision-making, only 2 were actually excluded: Pastor Buhat and Pastor Rodrigo.
Subsequently, petitioner received a letter of dismissal citing misappropriation of
denominational funds, willful breach of trust, serious misconduct, gross and habitual
neglect of duties, and commission of an offense against the person of employer's duly
authorized representative, as grounds for the termination of his services.

ISSUES
1. WON the Labor Arbiter/NLRC has jurisdiction to try and decide the complaint filed by
petitioner against the SDA
2. WON the termination of the services of petitioner is an ecclesiastical affair, and, as
such, involves the separation of church and state
3. WON such termination is valid

HELD
1. YES and 2. NO [Resolved jointly since they are related]
Ratio An ecclesiastical affair is one that concerns doctrine, creed or form or worship of the
church, or the adoption and enforcement within a religious association of needful laws and
regulations for the government of the membership, and the power of excluding from such
associations those deemed unworthy of membership.
Reasoning
- Based on this definition, an ecclesiastical affair involves the relationship between the
church and its members and relate to matters of faith, religious doctrines, worship and
governance of the congregation. To be concrete, examples of this so-called ecclesiastical
affairs to which the State cannot meddle are proceedings for excommunication,
ordinations of religious ministers, administration of sacraments and other activities which
attached religious significance. The case at bar does not even remotely concern any of
the above cited examples. While the matter at hand relates to the church and its religious
minister it does not ipso facto give the case a religious significance. Simply stated, what is
involved here is the relationship of the church as an employer and the minister as an
employee. It is purely secular and has no relation whatsoever with the practice of faith,
worship or doctrines of the church. In this case, petitioner was not excommunicated or

A2010

- 41 -

Disini

expelled from the membership of the SDA but was terminated from employment.
- Aside from these, SDA admitted in a certification issued by its officer, Ibesate, that
petitioner has been its employee for 28 years. SDA even registered petitioner with the
SSS as its employee. The worker's records of petitioner have been submitted by private
respondents as part of their exhibits. From all of these it is clear that when the SDA
terminated the services of petitioner, it was merely exercising its management prerogative
to fire an employee which it believes to be unfit for the job. As such, the State, through the
Labor Arbiter and the NLRC, has the right to take cognizance of the case.
- Finally, private respondents are estopped from raising the issue of lack of jurisdiction for
the first time on appeal. The active participation of a party coupled with his failure to object
to the jurisdiction of the court or quasi-judicial body is tantamount to an invocation of that
jurisdiction and a willingness to abide by the resolution of the case and will bar said party
from later on impugning the court or body's jurisdiction.
3. NO.
Reasoning
- The issue being the legality of petitioner's dismissal, the same must be measured against
the requisites for a valid dismissal, namely: (a) the employee must be afforded due
process, i.e., he must be given an opportunity to be heard and to defend himself, and; (b)
the dismissal must be for a valid cause as provided in Article 282 of the Labor Code.
Without the concurrence of these twin requirements, the termination would, in the eyes of
the law, be illegal.
As to Due Process
- Article 277(b) of the Labor Code further require the employer to furnish the employee
with 2 written notices, to wit: (a) a written notice served on the employee specifying the
ground or grounds for termination, and giving to said employee reasonable opportunity
within which to explain his side, and, (b) a written notice of termination served on the
employee indicating that upon due consideration of all the circumstances, grounds have
been established to justify his termination.
- The first notice, which may be considered as the proper charge, serves to apprise the
employee of the particular acts or omissions for which his dismissal is sought. The second
notice on the other hand seeks to inform the employee of the employer's decision to
dismiss him. This decision, however, must come only after the employee is given a
reasonable period from receipt of the first notice within which to answer the charge and
ample opportunity to be heard and defend himself with the assistance of a representative,
if he so desires. Non-compliance therewith is fatal because these requirements are
conditions sine quo non before dismissal may be validly effected.
- SDA failed to substantially comply with the above requirements. With regard to the first
notice, the letter dated 17 October 1991, which notified petitioner and his wife to attend the
meeting on 21 October 1991, cannot be construed as the written charge required by law. A
perusal of the said letter reveals that it never categorically stated the particular acts or
omissions on which his impending termination was grounded. In fact, the letter never even
mentioned that he would be subject to investigation. The letter merely mentioned that he
and his wife were invited to a meeting wherein what would be discussed were the alleged
unremitted church tithes and the events that transpired on 16 October 1991. For this
reason, it cannot be said that petitioner was given enough opportunity to properly prepare
for his defense. While admittedly, SDA complied with the second requirement, the notice
of termination, this does not cure the initial defect of lack of the proper written charge
required by law.
As to Just Cause
- Private respondents allege that they have lost their confidence in petitioner for his failure,
despite demands, to remit the tithes and offerings which were collected in his district.
Settled is the rule that under Article 282 (c) of the Labor Code, the breach of trust must be
willful. A breach is willful if it is done intentionally, knowingly and purposely, without
justifiable excuse, as distinguished from an act done carelessly, thoughtlessly, heedlessly
or inadvertently. It must rest on substantial grounds and not on the employer's
arbitrariness, whims, caprices or suspicion, otherwise, the employee would eternally
remain at the mercy of the employer. It should be genuine and not simulated. This ground
has never been intended to afford an occasion for abuse, because of its subjective nature.
The records show that there were only 6 instances when petitioner personally collected
and received from the church treasurers the tithes, collections, and donations for the
church. The testimony of Naomi Geniebla, the Negros Mission Church Auditor and a
witness for private respondents, show that Pastor Austria was able to remit all his
collections to the treasurer of the Negros Mission. Private respondents try to pin on
petitioner the alleged non-remittance of the tithes collected by his wife. In the absence of
conspiracy and collusion, which private respondents failed to demonstrate, between
petitioner and his wife, he cannot be made accountable for the alleged infraction
committed by his wife. After all, they still have separate and distinct personalities. Thus,
the allegation of breach of trust has no leg to stand on.
- Misconduct has been defined as improper or wrong conduct. It is the transgression of
some established and definite rule of action, a forbidden act, a dereliction of duty, willful in
character, and implies wrongful intent and not mere error in judgment. For misconduct to
be considered serious it must be of such grave and aggravated character and not merely
trivial or unimportant. Based on this standard, we believe that the act of petitioner in
banging the attache case on the table, throwing the telephone and scattering the books in
the office of Pastor Buhat, although improper, cannot be considered as grave enough to

Labor Law 1
be considered as serious misconduct. After all, as correctly observed by the Labor Arbiter,
though petitioner committed damage to property, he did not physically assault Pastor
Buhat or any other pastor present during the incident of 16 October 1991. In fact, the
alleged offense committed upon the person of the employer's representatives was never
really established or proven by private respondents. Hence, there is no basis for the
allegation that petitioner's act constituted serious misconduct or that the same was an
offense against the person of the employer's duly authorized representative.
- The final ground alleged by private respondents, gross and habitual neglect of duties,
does not requires an exhaustive discussion. All private respondents had were allegations
but not proof. Aside from merely citing the said ground, private respondents failed to prove
culpability. In fact, the evidence on record shows otherwise. Petitioner's rise from the ranks
proves that he was actually a hard-worker. Private respondents' evidence, which consisted
of petitioner's Worker's Reports, revealed how petitioner travelled to different churches to
attend to the faithful under his care. Indeed, he labored hard for the SDA, but, in return, he
was rewarded with a dismissal from the service for a non-existent cause.
Disposition Finding of the Labor Arbiter that petitioner was terminated from service
without just or lawful cause is SUSTAINED. Petitioner is entitled to reinstatement without
loss of seniority right and the payment of full back-wages without any deduction
corresponding to the period from his illegal dismissal up to the actual reinstatement.
Challenged Resolution of NLRC is NULLIFIED and SET ASIDE.

PEARANDA V BAGANGA PLYWOOD CORP


[PAGE 1]
CBTC EMPLOYEES UNION V CLAVE
141 SCRA 9
DE LA FUENTE; January 7, 1986
NATURE
Petition for certiorari seeking to annul and set aside the decision of the respondent
Presidential Executive Assistant
FACTS
- Commercial Bank and Trust Company Employees' Union lodged a complaint with the
Department of Labor, against Comtrust Bank for non-payment of the holiday pay benefits
provided for under Art 95 of the Labor Code in relation to Rule X, Book III of the Rules and
Regulations Implementing the Labor Code.
- Failing to arrive at an amicable settlement at conciliation level, the parties opted to
submit their dispute for voluntary arbitration. The issue presented was: "Whether the
permanent employees of the Bank within the collective bargaining unit paid on a monthly
basis are entitled to holiday pay effective November 1, 1974, pursuant to Article 95 (now
Article 94) of the Labor Code, as amended and Rule X (now Rule IV), Book III of the Rules
and Regulations Implementing the Labor Code. "
- In addition, the disputants signed a Submission Agreement stipulating as final,
unappealable and executory the decision of the Arbitrator, including subsequent issuances
for clarificatory and/or relief purposes, notwithstanding Article 262 of the Labor Code.
- In the course of the hearing, the Arbitrator apprised the parties of an interpretative
bulletin on "holiday pay" about to be issued by the Department of Labor.
- The Union filed a Manifestation stating that in the event that said Interpretative Bulletin
regarding holiday pay would be adverse to the present claim union respectfully reserves
the right to take such action as may be appropriate to protect its interests, a question of
law being involved. An Interpretative Bulletin which was inexistent at the time the said
commitment was made and which may be contrary to the law itself should not bar the right
of the union to claim for its holiday pay benefits
- Voluntary Arbitrator stated that, there is more reason to believe that, if the Bank has
never made any deduction from its monthly-paid employees for unworked Saturdays,
Sundays, legal and special holidays, it is because there is really nothing to deduct properly
since the monthly, salary never really included pay for such unworked days-and which
give credence to the conclusion that the divisor '250' is the proper one to use in computing
the equivalent daily rate of the monthly-paid employees; that both the decree itself and
the Rules mentioned enumerated the excepted workers. It is a basic rule of statutory
construction that putting an exception limits or modifies the enumeration or meaning made
in the law. It is thus easy to see that a mere reading of the Decree and of the Rules would
show that the monthly-paid employees of the Bank are not expressly included in the
enumeration of the exception.
- Voluntary Arbitrator directed the bank to pay its monthly paid employees their legal
holiday pay.
- The next day, the Department of Labor released Policy Instructions No. 9 which clarifies
controversies on the entitlement of monthly paid employees. The new determining rule is
this: If the monthly paid employee is receiving not less than P 240, the maximum monthly
minimum wage, and his monthly pay is uniform from January to December, he is
presumed to be already paid the ten (10) paid legal holidays. However, if deductions are

A2010

- 42 -

Disini

made from his monthly salary on account of holidays in months where they occur, then he
is still entitled to the ten (10) paid legal holidays.
- Bank appealed to NLRC but appeal was dismissed because it was filed way beyond the
ten-day period for perfecting an appeal and because it contravened the agreement that
the award shall be final and unappealable.
- Acting Secretary of Labor reversed NLRC decision and ruled that the appeal was filed on
time and that a review of the case was inevitable as the money claim exceeded
P100,000.00.
- Presidential Executive Assistant affirmed DOJ ruling, relying heavily on the Manifestation
and Policy Instructions No. 9.
Petitioners Claim
The legal presumption established in Section 2, Rule IV, Book 111 9, of the Rules and
Regulations implementing particularly Article 94 (formerly Article 208) of the Labor Code,
is merely a disputable presumption
Respondents Comments
- The Bank maintains that, since its inception or start of operations in 1954, all monthlypaid employees in the Bank are paid their monthly salaries without any deduction for
unworked Saturdays, Sundays, legals and special holidays. On the other hand, it also
maintains that, as a matter of fact, 'always conscious of its employee who has to work, on
respondent's rest days of Saturdays and Sundays or on a legal holiday, an employee who
works overtime on any of said days is paid one addition regular pay for the day plus 50%
of said regular pay
- The Bank further maintains that the holiday pay is intended only for daily-paid workers.
- The question submitted for arbitration is now moot and academic.
ISSUE
WON the permanent employees of the bank are entitled to holiday pay
HELD
YES
- In excluding the union members of herein petitioner from the benefits of the holiday pay
law, public respondent predicated his ruling on Section 2, Rule IV, Book III of the Rules to
implement Article 94 of the labor Code promulgated by the then Secretary of labor and
Policy Instructions No. 9.
- In Insular Bank of Asia and America Employees' Union (IBAAEU) vs. Inciong, 7 this
Court's Second Division, speaking through former Justice Makasiar, expressed the view
and declared that the aforementioned section and interpretative bulletin are null and void,
having been promulgated by the then Secretary of Labor in excess of his rule-making
authority. It was pointed out, inter alia, that in the guise of clarifying the provisions on
holiday pay, said rule and policy instructions in effect amended the law by enlarging the
scope of the exclusions.
- The questioned Section 2, Rule IV, Book III of the Integrated Rules and the Secretary's
Policy Instruction No. 9 add another excluded group, namely, 'employees who are
uniformly paid by the month'. While the additional exclusion is only in the form of a
presumption that all monthly paid employees have already been paid holiday pay, it
constitutes a taking away or a deprivation which must be in the law if it is to be valid. An
administrative interpretation which diminishes the benefits of labor more than what the
statute delimits or withholds is obviously ultra vires.
Disposition The questioned decision set aside and the award of the arbitrator reinstated.

SONZA V ABS-CBN BROADCASTING CORPORATION


431 SCRA 587
CARPIO; June 10, 2004
NATURE
Petition for review on certiorari
FACTS
- ABS-CBN signed Agreement with Mel and Jay Management and Devt Corp (MJMDC),
which agreed to provide Sonzas services exclusively to the network as talent for radio and
TV.
- Sonza resigned and complained that network didnt pay his salaries, separation pay,
service incentive leave pay, etc. ABS filed Motion to Dismiss because there was no
employer-employee relationship. ABS continued to remit Sonzas monthly talent fees.
- Labor Arbiter dismissed complaint because of lack of jurisdiction. NLRC affirmed
Arbiters decision. Sonza filed certiorari action with CA, which dismissed the case. Hence
this petition.

SECTION 2. Status of employees paid by the month -Employees who are uniformly paid by the month, irrespective of
the number of' working days therein with a salary of not less than the statutory or established minimum wage, shall be
presumed to be paid for all days in the month whether worked or not.
For this purpose, the monthly minimum wage shall not be less than the statutory minimum wage multiplied by 365 days
divided by twelve.

Labor Law 1
ISSUE
WON there was an employer-employee relationship between ABS-CBN and Sonza
HELD
NO
- This is the first Court resolution on nature of relationship between a station and a talent.
- There are 4 elements of employer-employee relationship:
1. Selection of employee
- if Sonza didnt possess his skills, talents and celebrity status, ABS-CBN would not
have entered into agreement with him but would have hired him through personnel
department
2. Payment of wages
- whatever Sonza received arose from the contract and not from the employeremployee relation
- the talent fee is so huge that it indicates more a contractual than an employment
relationship
3. Power to dismiss
- ABS-CBN coulnt retrench Sonza because it is obligated to pay talent fees for
duration of contract
4. Control on employee on means and methods
- also called control test; most impt to determine relationship
- Sonza contends ABS exercised control over means and methods of his work.
Court said ABS merely reserved the right to modify the program format and airtime
schedule. Its sole concern was the quality of the show and the ratings. How Sonza
appeared, sounded, etc. is outside control of ABS.
- Sonza contends that ABS exercised control in providing equipment and crew.
Court said these are not tools needed by Sonza. What he needed were his talent,
skills, costume.
- Sonza contends that ABS subjected him to rules and standards. Court said that
the rules are the TV and Radio Code of the Kapisanan ng Broadcaster sa Pilipinas,
merely adopted by ABS as its code of ethics. It applies to broadcasters, not just to
ABS employees. Besides, these rules are merely guidelines.
- Sonza said his exclusivity is a form of control by ABS. Court said exclusivity is a
widespread practice in entertainment industry, as protection of investment in
building up a talent. Besides, the huge talent fees of an exclusive talent
compensates for exclusivity.
- Arbiter ruled that as talent of MJMDC, Sonza is not an employee of ABS. Sonza insists
that MJMDC is a labor-only contractor and ABS is his employer. In labor-only contract,
there are 3 parties the contractor, employee and the principal (deemed the real
employer). Under this, the contractor is the agent of the principal. If Sonzas argument
was true, then MJMDC turns to be the agent of both Sonza and ABS. Besides, in the
Agreement, there were only two parties mentioned Sonza and ABS, with MJMDC as
Sonzas agent.
- Sonza argues Policy Instruction No. 40 by Minister of Labor said the types of employees
in broadcast are the station and program employees. Court said this instruction is a mere
executive issuance not binding on the Court.
- Court also said that Arbiter can decide a case without a formal trial.
- Sonza argues that treating talents as contractors violates right to security of tenure.
Court said this right exists only if there is an employer-employee relation. Besides, law
also protects rights of talents to contract. Besides, if hosts were employees, managers
can dictate what hosts will say, and this is not conducive to press freedom.
- Difference in tax treatment also showed that theres no employer-employee relation.
- Sonzas claim is based on their agreement. Therefore, action should not be based on
Labor Code but on breach of contract.

RIZAL EMPIRE INSURANCE GROUP V NLRC (RUIZ,


CORIA)
150 SCRA 565
PARAS; May 29, 1987
NATURE
Petition for certiorari
FACTS
- August, 1977- Coria was hired by Rizal Empire Insurance Group(REIG) as a casual
employee with a salary of P10.00 a day.
- January 1, 1978- Coria was made a regular employee, having been appointed as clerktypist, with a monthly salary of P300.00.
- Being a permanent employee, he was furnished a copy of petitioner company's "General
Information, Office Behavior and Other Rules and Regulations."
- In the same year, without change in his position-designation, he was transferred to the
Claims Department and his salary was increased to P450.00 a month.

A2010

- 43 -

Disini

- 1980- he was transferred to the Underwriting Department and his salary was increased
to P580.00 a month plus cost of living allowance, until he was transferred to the Fire
Department as filing clerk.
- July, 1983- he was made an inspector of the Fire Division with a monthly salary of
P685.00 plus allowances and other benefits.
- Oct. 15, 1983- Coria was dismissed from work, on the grounds of tardiness and
unexcused absences.
- Coria filed a complaint with MOLE
- March 14, 1985- LA Ruiz reinstated him to his position with back wages.
- REIG appealed to the NLRC but was dismissed on the ground that the same had been
filed out of time. Hence this petition.
ISSUE
WON it is still within the jurisdiction of the SC
HELD
NO
- Under the provisions of the Revised NLRC Rules, the decision appealed from in this
case has become final and executory and can no longer be subject to appeal.
Ratio Administrative regulations and policies enacted by administrative bodies to interpret
the law which they are entrusted to enforce, have the force of law, and are entitled to great
respect (Espanol v. Philippine Veterans Administration, 137 SCRA 314 [1985])..
Reasoning
- The record shows that REIG received a copy of the decision of the LA on April 1, 1985.
- It filed a Motion for Extension of Time to File Memorandum of Appeal on April 11, 1985
and filed the Memorandum of Appeal on April 22, 1985.
- Rule VIII of the Revised Rules of the NLRC on appeal, provides that decisions or orders
of a LA shall be final and executory unless appealed to the Commission by any or both of
the parties within 10 calendar days from receipt of notice and that no motion or request for
extension of the period within which to perfect an appeal shall be entertained.
NLRC correctly dismissed REIGs appeal pursuant to said rules.
- The NLRC didnt commit GAD amounting to lack of jurisdiction in arbitrarily dismissing
petitioners' appeal on a technicality.
- SC need not interpret the Revised Rules of the NLRC as they are clear and explicit and
leave no room for interpretation.
- Even on the merits, the ruling of the LA appears to be correct; the consistent promotions
in rank and salary of the private respondent indicate he must have been a highly efficient
worker, who should be retained despite occasional lapses in punctuality and attendance.
Perfection cannot after all be demanded.
Disposition Petition DISMISSED.

DUNCAN ASSOCIATION OF DETAILMAN-PTGWO V


GLAXO WELLCOME PHILIPPINES INC
438 SCRA 343
TINGA; September 17, 2004
NATURE
Petition for review on certiorari of the decision and resolution of the Court of Appeals
FACTS
- Petitioner Tecson was hired by respondent Glaxo Wellcome Philippines, Inc. (Glaxo) as
medical representative after Tecson had undergone training and orientation. Thereafter,
Tecson signed a contract of employment which stipulates, among others, that he agrees to
study and abide by existing company rules; to disclose to management any existing or
future relationship by consanguinity or affinity with co-employees or employees of
competing drug companies and should management find that such relationship poses a
possible conflict of interest, to resign from the company.
- The Employee Code of Conduct of Glaxo similarly provides that an employee is
expected to inform management of any existing or future relationship by consanguinity or
affinity with co-employees or employees of competing drug companies. If management
perceives a conflict of interest or a potential conflict between such relationship and the
employees employment with the company, the management and the employee will
explore the possibility of a transfer to another department in a non-counterchecking
position or preparation for employment outside the company after six months.

Labor Law 1
- Tecson was initially assigned to market Glaxos products in the Camarines SurCamarines Norte sales area. Subsequently, Tecson entered into a romantic relationship
with Bettsy, an employee of Astra Pharmaceuticals (Astra), a competitor of Glaxo. Bettsy
was Astras Branch Coordinator in Albay. She supervised the district managers and
medical representatives of her company and prepared marketing strategies for Astra in
that area. Even before they got married, Tecson received several reminders from his
District Manager regarding the conflict of interest which his relationship with Bettsy might
engender. Still, love prevailed, and Tecson married Bettsy.
- Tecsons superiors informed him that his marriage to Bettsy gave rise to a conflict of
interest. Tecsons superiors reminded him that he and Bettsy should decide which one of
them would resign from their jobs, although they told him that they wanted to retain him as
much as possible because he was performing his job well.
- Tecson requested for time to comply with the company policy against entering into a
relationship with an employee of a competitor company. He explained that Astra, Bettsys
employer, was planning to merge with Zeneca, another drug company; and Bettsy was
planning to avail of the redundancy package to be offered by Astra. With Bettsys
separation from her company, the potential conflict of interest would be eliminated. At the
same time, they would be able to avail of the attractive redundancy package from Astra.
- Tecson again requested for more time resolve the problem. Tecson applied for a transfer
in Glaxos milk division, thinking that since Astra did not have a milk division, the potential
conflict of interest would be eliminated. His application was denied in view of Glaxos
least-movement-possible policy. Glaxo transferred Tecson to the Butuan City-Surigao
City-Agusan del Sur sales area. Tecson asked Glaxo to reconsider its decision, but his
request was denied.
- Tecson sought Glaxos reconsideration regarding his transfer and brought the matter to
Glaxos Grievance Committee. Glaxo, however, remained firm in its decision and gave
Tecson time to comply with the transfer order. Tecson defied the transfer order and
continued acting as medical representative in the Camarines Sur-Camarines Norte sales
area.
- During the pendency of the grievance proceedings, Tecson was paid his salary, but was
not issued samples of products which were competing with similar products manufactured
by Astra. He was also not included in product conferences regarding such products.
- Because the parties failed to resolve the issue at the grievance machinery level, they
submitted the matter for voluntary arbitration. Glaxo offered Tecson a separation pay of
one-half month pay for every year of service, or a total of P50,000.00 but he declined the
offer. The National Conciliation and Mediation Board (NCMB) rendered its Decision
declaring as valid Glaxos policy on relationships between its employees and persons
employed with competitor companies, and affirming Glaxos right to transfer Tecson to
another sales territory.
- Aggrieved, Tecson filed a Petition for Review with the Court of Appeals assailing the
NCMB Decision. The Court of Appeals promulgated its Decision denying the Petition for
Review on the ground that the NCMB did not err in rendering its Decision. The appellate
court held that Glaxos policy prohibiting its employees from having personal relationships
with employees of competitor companies is a valid exercise of its management
prerogatives. Tecson filed a Motion for Reconsideration of the appellate courts Decision,
but the motion was denied by the appellate court.
ISSUES
1. WON the Court of Appeals erred in ruling that Glaxos policy against its employees
marrying employees from competitor companies is valid, and in not holding that said policy
violates the equal protection clause of the Constitution
2. WON petitioner was constructively dismissed
HELD
1. NO
- Glaxo has a right to guard its trade secrets, manufacturing formulas, marketing strategies
and other confidential programs and information from competitors, especially so that it and
Astra are rival companies in the highly competitive pharmaceutical industry.
- The prohibition against personal or marital relationships with employees of competitor
companies upon Glaxos employees is reasonable under the circumstances because
relationships of that nature might compromise the interests of the company. In laying
down the assailed company policy, Glaxo only aims to protect its interests against the
possibility that a competitor company will gain access to its secrets and procedures.
- That Glaxo possesses the right to protect its economic interests cannot be denied. No
less than the Constitution recognizes the right of enterprises to adopt and enforce such a
policy to protect its right to reasonable returns on investments and to expansion and
growth. Indeed, while our laws endeavor to give life to the constitutional policy on social
justice and the protection of labor, it does not mean that every labor dispute will be
decided in favor of the workers. The law also recognizes that management has rights
which are also entitled to respect and enforcement in the interest of fair play.
- The challenged company policy does not violate the equal protection clause of the
Constitution as petitioners erroneously suggest. It is a settled principle that the commands
of the equal protection clause are addressed only to the state or those acting under color
of its authority. Corollarily, it has been held in a long array of U.S. Supreme Court
decisions that the equal protection clause erects no shield against merely private conduct,

A2010

- 44 -

Disini

however, discriminatory or wrongful. The only exception occurs when the state in any of its
manifestations or actions has been found to have become entwined or involved in the
wrongful private conduct. Obviously, however, the exception is not present in this case.
Significantly, the company actually enforced the policy after repeated requests to the
employee to comply with the policy. Indeed, the application of the policy was made in an
impartial and even-handed manner, with due regard for the lot of the employee.
- In any event, from the wordings of the contractual provision and the policy in its
employee handbook, it is clear that Glaxo does not impose an absolute prohibition against
relationships between its employees and those of competitor companies. Its employees
are free to cultivate relationships with and marry persons of their own choosing. What the
company merely seeks to avoid is a conflict of interest between the employee and the
company that may arise out of such relationships.
- The Court of Appeals also correctly noted that the assailed company policy which forms
part of respondents Employee Code of Conduct and of its contracts with its employees,
such as that signed by Tecson, was made known to him prior to his employment. Tecson,
therefore, was aware of that restriction when he signed his employment contract and when
he entered into a relationship with Bettsy. Since Tecson knowingly and voluntarily entered
into a contract of employment with Glaxo, the stipulations therein have the force of law
between them and, thus, should be complied with in good faith. He is therefore estopped
from questioning said policy.
2. NO
- Constructive dismissal is defined as a quitting, an involuntary resignation resorted to
when continued employment becomes impossible, unreasonable, or unlikely; when there
is a demotion in rank or diminution in pay; or when a clear discrimination, insensibility or
disdain by an employer becomes unbearable to the employee. None of these conditions
are present in the instant case. The record does not show that Tecson was demoted or
unduly discriminated upon by reason of such transfer. Glaxo properly exercised its
management prerogative in reassigning Tecson to the Butuan City sales area. When the
problem could not be resolved after several years of waiting, Glaxo was constrained to
reassign Tecson to a sales area different from that handled by his wife for Astra. Notably,
Glaxo did not terminate Tecson from employment but only reassigned him to another area
where his home province, Agusan del Sur, was included. In effecting Tecsons transfer,
Glaxo even considered the welfare of Tecsons family. Clearly, the foregoing dispels any
suspicion of unfairness and bad faith on the part of Glaxo.

SALINAS V NLRC (ATLANTIC GULF AND PACIFIC CO)


319 SCRA 54
PURISIMA; November 24, 1999
FACTS
- Petitioners were employed with Atlantic Gulf and Pacific Co. (AG & P):
Salinas: 1983-1988 as carpenter/finishing carpenter
Alejandro: 1982-1989 as bulk cement operator, bulk cement plant/carrier operator &
crane driver
Cortez: 1979-1988 as carpenter/forklift operator
Samulde: 1982-1989 as lubeman/stationary operator
- Complaints (separate but consolidated by the LA): illegal dismissal
- Petitioners Claim: They had been covered by a number of contracts renewed
continuously, with periods ranging from five (5) to nine (9) years, and they performed the
same kind of work through out their employment, and such was usually necessary and
desirable in the trade or business of the respondent corporation; and their work did not
end on a project-to-project basis, although the contrary was made to appear by the
employer through the signing of separate employment contracts.
- LA: Dismissed petitions on the ground that the petitioners are project employees are
project employees whose work contracts with AG & P indicate that they were employed in
such category; that they have been assigned to different work projects, not just to one and
that their work relation with AG & P, relative to termination, is governed by Policy
Instruction No. 20 (rule governing project employees).
- Appeal to NLRC: Affirmed LAs findings
ISSUES
1. WON the petitioners are project employees
Procedural
2. WON this petition for certiorari was proper

Labor Law 1
HELD
1. NO
- The petitioners are regular employees.
- The mandate in Article 281 of the Labor Code, which pertinently prescribes that the
'provisions of written agreement to the contrary notwithstanding and regardless of the oral
agreements of the parties, an employment shall be deemed to be regular where the
employee has been engaged to perform activities which are usually necessary or
desirable in the usual business or trade of the employer' and that any employee who has
rendered at least one year of service, whether such service is continuous or broken shall
be considered a regular employee with respect to the activity in which he is employed and
his employment shall continue while such actually exists,' should apply in the case of
petitioner.
- Failure to report the termination to Public Employment Office is a clear indication that
petitioners were not and are not project employees. (PI No. 20 requires reports of
terminations)
- It is basic and irrefragable rule that in carrying out and interpreting the provisions of the
Labor Code and its implementing regulations, the workingman's welfare should be the
primordial and paramount consideration. The interpretation herein made gives meaning
and substance to the liberal and compassionate spirit of the law enunciated in Article 4 of
Labor Code that "all doubts in the implementation and interpretation of the provisions of
the Labor Code including its implementing rules and regulations shall be resolved in favor
of labor".
- It is beyond cavil that petitioners had been providing the respondent corporation with
continuous and uninterrupted services, except for a day or so gap in their successive
employment contracts. Their contracts had been renewed several times, with the total
length of their services ranging from five (5) to nine (9) years. Throughout the duration of
their contracts, they had been performing the same kinds of work (e.g., as lubeman, bulk
cement operator and carpenter), which were usually necessary and desirable in the
construction business of AG & P, its usual trade or business.
- Undoubtedly, periods in the present case have been imposed to preclude the acquisition
of tenurial security by petitioners, and must be struck down for being contrary to public
policy, morals, good customs or public order.
2. YES
- Anent the issue that the petition should have been brought under Rule 65 and not under
Rule 45 of the Revised Rules of Court, this rule is not inflexible. In the interest of justice,
often the Court has judiciously treated as special civil actions for certiorari petitions
erroneously captioned as petitions for review on certiorari.
- With regard to the issue on non-exhaustion of administrative remedies, the Court hold
that the failure of petitioners to interpose a motion for reconsideration of the NLRC
decision before coming to this Court was not a fatal omission. The exhaustion of
administrative remedies doctrine is not a hard and fast rule and does not apply where the
issue is purely a legal one. A motion for reconsideration as a prerequisite for the bringing
of an action under Rule 65 may be dispensed with where the issue is purely of law, as in
this case. At all events and in the interest of substantial justice, especially in cases
involving the rights of workers, procedural lapses, if any, may be disregarded to enable the
Court to examine and resolve the conflicting rights and responsibilities of the parties. This
liberality is warranted in the case at bar, especially since it has been shown that the
intervention of the Court is necessary for the protection of the herein petitioner(s).
Disposition The questioned Resolution of the NLRC is SET ASIDE and another one is
hereby ENTERED ordering the respondent corporation to reinstate petitioners without loss
of seniority and with full backwages.

ABELLA V NLRC (QUITCO, DIONELE)


152 SCRA 140
PARAS; July 20, 1987
FACTS
- Petitioner Rosalina Perez Abella leased a farm land in Ponteverde, Negros Occidental,
known as Hacienda Danao-Ramona, for a period of ten (10) years, renewable, at her
option, for another ten (10) years. She did renew for another ten years. During the
existence of the lease, she employed private respondents. Private respondent Ricardo
Dionele, Sr. has been a regular farm worker for 33 years while . On the other hand, private
respondent Romeo Quitco started worked for 14 years. Upon the expiration of her
leasehold rights, petitioner dismissed private respondents and turned over the hacienda to
the owners thereof, who continued the management, cultivation and operation of the farm
ISSUE
WON private respondents are entitled to separation pay
HELD
YES
- The closing wasnt due to serious losses or financial reverses. The Court cited Article 284
(this should be 283) which says:

A2010

- 45 -

Disini

"Art. 284.
Closure of establishment and reduction of personnel. The
employer may also terminate the employment of any employee due to the installation
of labor-saving devices, redundancy, retrenchment to prevent losses or the closing or
cessation of operation of the establishment or undertaking unless the closing is for the
purpose of circumventing the provisions of this title, by serving a written notice on the
workers and the Ministry of Labor and Employment at least one (1) month before the
intended date thereof. In case of termination due to the installation of labor-saving
devices or redundancy, the worker affected thereby shall be entitled to a separation pay
equivalent to at least his one (1) month pay or to at least one (1) month pay for every
year of service, whichever is higher. In case of retrenchment to prevent losses and in
cases of closure or cessation of operations of establishment or undertaking not due to
serious business losses or financial reverses, the separation pay shall be equivalent to
one (1) month pay or at least one-half (1/2) month pay for every year of service
whichever is higher. A fraction of at least six (6) months shall be considered one (1)
whole year."
- Petitioner then contends that the aforequoted provision violates the constitutional
guarantee against impairment of obligations and contracts, because when she leased
Hacienda Danao-Ramona, neither she nor the lessor contemplated the creation of the
obligation to pay separation pay to workers at the end of the lease. The Court said that
this contention by petitioner is untenable. The law is clear and to permit such an argument
would mean that the years of service given by the workers will mean nothing since there is
no agreement here that the new management will be the one to shoulder the separation
pay. The old management, pertaining to Abella in this case, should give the payment.
- In any event, it is well-settled that in the implementation and interpretation of the
provisions of the Labor Code and its implementing regulations, the workingman's welfare
should be the primordial and paramount consideration. (Volshel Labor Union v. Bureau of
Labor Relations, 137 SCRA 43 [1985]). It is the kind of interpretation which gives meaning
and substance to the liberal and compassionate spirit of the law as provided for in Article 4
of the New Labor Code which states that `all doubts in the implementation and
interpretation of the provisions of this Code including its implementing rules and
regulations shall be resolved in favor of labor." The policy is to extend the applicability of
the decree to a greater number of employees who can avail of the benefits under the law,
which is in consonance with the avowed policy of the State to give maximum aid and
protection to labor.
Disposition Petition is DISMISSED.

ASIAN TRANSMISSION CORP V CA (BISIG NG ASIAN


TRANSMISSION LABOR UNION)
425 SCRA 478
CARPIO-MORALES; March 15, 2004
NATURE
Petition for certiorari seeking the nullification of the March 28, 2000 Decision of the Court
of Appeals
FACTS
- The Department of Labor and Employment (DOLE), through Undersecretary
Cresenciano B. Trajano, issued an Explanatory Bulletin dated March 11, 1993, wherein it
clarified, that employees are entitled to 200% of their basic wage on April 9, 1993, which,
apart from being Good Friday, and, therefore, a legal holiday, is also Araw ng Kagitingan,
which is also a legal holiday, even if unworked.
- Said bulletin was reproduced on January 23, 1998, when April 9, 1998 was both Maundy
Thursday and Araw ng Kagitingan
- Despite the explanatory bulletin, petitioner Asian Transmission Corporation opted to pay
its daily paid employees only 100% of their basic pay on April 9, 1998.
- Respondent Bisig ng Asian Transmission Labor Union (BATLU) protested.
- In accordance with Step 6 of the grievance procedure of the Collective Bargaining
Agreement (CBA) existing between petitioner and BATLU, the controversy was submitted
for voluntary arbitration.
- On July 31, 1998, the Office of the Voluntary Arbitrator rendered a decision directing
petitioner to pay its covered employees "200% and not just 100% of their regular daily
wages for the unworked April 9, 1998
- Subject of interpretation in the case at bar is Article 94 10 of the Labor which was
amended by Executive Order No. 20311

10

ART. 94. Right to holiday pay. (a) Every worker shall be paid his regular daily wage during regular holidays, except
in retail and service establishments regularly employing less than ten (10) workers; (b) The employer may require an
employee to work on any holiday but such employee shall be paid a compensation equivalent to twice his regular rate;
and (c) As used in this Article, "holiday" includes: New Years Day, Maundy Thursday, Good Friday, the ninth of April, the
first of May, the twelfth of June, the fourth of July, the thirtieth of November, the twenty-fifth and thirtieth of December and
the day designated by law for holding a general election

Labor Law 1
- In deciding in favor of the Bisig ng Asian Transmission Labor Union (BATLU), the
Voluntary Arbitrator held that Article 94 of the Labor Code provides for holiday pay for
every regular holiday, the computation of which is determined by a legal formula which is
not changed by the fact that there are two holidays falling on one day; and that that the
law, as amended, enumerates ten regular holidays for every year, and should not be
interpreted as authorizing a reduction to nine the number of paid regular holidays "just
because April 9 (Araw ng Kagitingan) in certain years, like 1993 and 1998, is also Holy
Friday or Maundy Thursday."
- The Court of Appeals upheld the findings of the Voluntary Arbitrator, holding that the
Collective Bargaining Agreement (CBA) between petitioner and BATLU, the law governing
the relations between them, clearly recognizes their intent to consider Araw ng Kagitingan
and Maundy Thursday, on whatever date they may fall in any calendar year, as paid legal
holidays during the effectivity of the CBA and that "there is no condition, qualification or
exception for any variance from the clear intent that all holidays shall be compensated.
- The Court of Appeals further held that "in the absence of an explicit provision in law
which provides for [a] reduction of holiday pay if two holidays happen to fall on the same
day, any doubt in the interpretation and implementation of the Labor Code
provisions on holiday pay must be resolved in favor of labor."
- Hence, this petition.
ISSUE
WON daily-paid employees are entitled to be paid for two regular holidays which fall on
the same day
HELD
YES
- Holiday pay is a legislated benefit enacted as part of the Constitutional imperative that
the State shall afford protection to labor. Its purpose is not merely "to prevent diminution of
the monthly income of the workers on account of work interruptions. In other words,
although the worker is forced to take a rest, he earns what he should earn, that is, his
holiday pay."8 It is also intended to enable the worker to participate in the national
celebrations held during the days identified as with great historical and cultural
significance.
- Independence Day (June 12), Araw ng Kagitingan (April 9), National Heroes Day (last
Sunday of August), Bonifacio Day (November 30) and Rizal Day (December 30) were
declared national holidays to afford Filipinos with a recurring opportunity to commemorate
the heroism of the Filipino people, promote national identity, and deepen the spirit of
patriotism. Labor Day (May 1) is a day traditionally reserved to celebrate the contributions
of the working class to the development of the nation, while the religious holidays
designated in Executive Order No. 203 allow the worker to celebrate his faith with his
family.
- As reflected above, Art. 94 of the Labor Code, as amended, affords a worker the
enjoyment of ten paid regular holidays. The provision is mandatory, regardless of whether
an employee is paid on a monthly or daily basis. Unlike a bonus, which is a management
prerogative, holiday pay is a statutory benefit demandable under the law. Since a worker
is entitled to the enjoyment of ten paid regular holidays, the fact that two holidays
fall on the same date should not operate to reduce to nine the ten holiday pay
benefits a worker is entitled to receive.
- It is elementary, under the rules of statutory construction, that when the language of the
law is clear and unequivocal, the law must be taken to mean exactly what it says. In
the case at bar, there is nothing in the law which provides or indicates that the
entitlement to ten days of holiday pay shall be reduced to nine when two holidays
fall on the same day.
- In any event, Art. 4 of the Labor Code provides that all doubts in the
implementation and interpretation of its provisions, including its implementing
rules and regulations, shall be resolved in favor of labor. For the working mans
welfare should be the primordial and paramount consideration.
- Moreover, Sec. 11, Rule IV, Book III of the Omnibus Rules to Implement the Labor Code
provides that "Nothing in the law or the rules shall justify an employer in withdrawing or
reducing any benefits, supplements or payments for unworked regular holidays as
provided in existing individual or collective agreement or employer practice or policy.
- From the pertinent provisions of the CBA entered into by the parties, petitioner had
obligated itself to pay for the legal holidays as required by law.
Disposition Petition is dismissed.

CLEMENTE V GSIS
152 SCRA 500
GUTIERREZ, JR; July 31, 1987
11

regular holidays are now:1. New Years Day January 1; 2. Maundy Thursday Movable Date; 3. Good Friday Movable
Date; 4. Araw ng Kagitingan April 9 (Bataan and Corregidor Day); 5. Labor Day May 1; 6. Independence Day June 12; 7.
National Heroes Day Last Sunday of August; 8. Bonifacio Day November 30; 9. Christmas Day December 25; 10. Rizal
Day December 30

A2010

Disini

- 46 -

NATURE
Petition to review decision of the Employees Compensation Commission (ECC) which
affirmed decision of GSIS and denied Clementes claim for death benefits
FACTS
- Carolinas husband, Pedro Clemente was for 10 years a janitor in the DOH Dagupan City
assigned at the Ilocos Norte Skin Clinic.
- He was hospitalized for 12 days due to his ailment of nephritis, and was also found to be
suffering from Hansens Disease (portal cirrhosis and leprosy). He died on Nov 14, 1976.
- Petitioner then filed with GSIS a claim for employees compensation under the Labor
Code. This was denied by GSIS on the ground that such ailments are not occupational
diseases taking into consideration nature of his work. Under Art. 167(L) of the Labor Code
and Sec. 1(b) Rule III of the Amended Rules on Employees Compensation, for the
sickness and the resulting disability or death to be compensable, sickness must be the
result of an occupational disease listed under Annex A of the rules; otherwise proof must
be shown that the risk of contracting the disease is increased by the working conditions.
- Petitioner claimed that the ailments were contracted in the course of employment and
were aggravated by his work since he was in direct contact with persons suffering from
different skin diseases and was exposed to obnoxious dusts and other dirt.
- ECC also dismissed the claim since there was no substantial evidence of causal
connection and there was evidence that deceased had already contracted the Hansens
before employment.
ISSUE
WON petitioner is entitled to the compensation
HELD
YES
Ratio Strict rules of evidence are NOT applicable in claims for compensation. The degree
of proof required is merely substantial evidence, which means such relevant evidence as
a reasonable mind might accept as adequate to support a conclusion. What the law
requires is a reasonable work-connection and not a direct causal relation.
- Doubts should be resolved in favor of the claimant-employee.
Reasoning
- The major ailments of the deceased could be traced to bacterial and viral infections. For
instance, in the case of leprosy, it is known that the source of infection is discharge from
lesions of persons with active cases.
- Petitoners husband worked in a skin clinic and was exposed to different carriers of
diseases. As janitor, he was the employee most exposed to the dangerous concentration
of infected material, and not being a med practitioner, least likely to know how to avoid
them.
- GSISs conservative stand is not consistent with the liberal interpretation of the Labor
Code and the social justice guarantee embodied in the Constitution in favor of workers.
Disposition Decision appealed from is set aside and GSIS is ordered to pay petitioner
P12T as death benefits and P1,200 as attorneys fees.

ACUAV CA
[PAGE 12]
BONIFACIO V GSIS
146 SCRA 276
FERNAN; December 15, 1986
NATURE
Petition for review on certiorari
FACTS
- Lourdes Bonifacio was a classroom teacher assigned to the district of Bagamanoc,
Division of Catanduanes, from August 1965 until she contracted carcinoma of the breast
with metastases to the gastrointestinal tract and lungs which caused her death on Oct. 5,
1978.
- Thereafter a claim for death benefits under P.D. No. 626, as amended, was filed by
petitioner with the GSIS. The same was however denied on the ground that the decedent's
principal ailment, carcinoma of the breast with metastases to gastrointestinal tract and
lungs, is not an occupational disease for her particular work as a teacher, nor is the risk of
contracting said disease increased by her working conditions.
- The Employees Compensation Commission (ECC), on appeal affirmed the decision of
the GSIS.
ISSUES

Labor Law 1
1. WON the GSIS and the ECC erred in denying petitioners claim
2. WON the rule that in case of doubt in the implementation and interpretation of the
provisions of the Labor Code, including its implementing rules and regulations, the same
shall be resolved in favor of the laborer applies in this case
HELD
1. NO
- A compensable sickness means "any illness definitely accepted as an occupational
disease listed by the ECC, or any illness caused by employment subject to proof by the
employee that the risk of contracting the same is increased by working conditions. For this
purpose, the Commission is empowered to determine and approve occupational diseases
and work-related illnesses that may be considered compensable based on peculiar
hazards of employment." [Art. 167(1) Labor Code as amended by P.D. No. 1368, effective
May 1, 1978].
- Thus, for the sickness or the resulting disability or death to be compensable, the
sickness must be the result of an accepted occupational disease listed by the ECC, or any
other sickness caused by employment subject to proof by claimant that the risk of
contracting the same is increased by working conditions. [Sec. 1, Rule 11, Amended Rules
on Employees Compensation]. Carcinoma of the breast with metastases to the
gastrointestinal tract and lungs is not listed by the Commission as an occupational
disease.
- The cancer which affected the deceased not being occupational in her particular
employment, it became incumbent upon petitioner to prove that the decedent's working
conditions increased the risk of her contracting the fatal illness. This onus petitioner failed
to satisfactorily discharge.
- Petitioner's contention that the decision of the ECC totally ignored the SC's
pronouncements on compensation cases is unmeritorious. The petitioner evidently
overlooked that his claim is now within the ambit of the Labor Code and the rulings under
the old law, Act No. 3428, as amended, no longer control.
- The old law as embodied particularly in Section 43 of RA No. 772 amending Act No.
3812, provided for "the presumption of compensability and the rule on aggravation of
illness, which favor the employee," and "paved the way for the latitudinarian or expansive
application of the Workmen's Compensation Law in favor of the employee or worker." The
presumption in essence states that in any proceeding for the enforcement of the claim for
compensation under the Workmen's Compensation Act "it shall be presumed in the
absence of substantial evidence to the contrary that the claim comes within the provisions
of the said Act, that sufficient notice thereof was given, that the injury was not occasioned
by the willful intention of the injured employee to bring about the injury or death of himself
or of another, that the injury did not result solely from the intoxicatiojn of the injured
employee while on duty, and that the contents of verified medical and surgical reports
introduced in evidence by claimants for compensation are correct."
- Thus, under the Workmen's Compensation Law, it is not necessary for the claimant to
carry the burden of proof to establish his case to the point of demonstration It is not
necessary to prove that employment was the sole cause of the death or injury suffered by
the employee. It is sufficient to show that the employment had contributed to the
aggravation or acceleration of such death or ailment. Once the disease had been shown
to have arisen in the course of employment, it is presumed by law, in the absence of
substantial evidence to the contrary, that it arose out of it.
- With this legal presumption in the old law, the burden of proof shifts to the employer and
the employee no longer suffers the burden of showing causation. Under the present Labor
Code, the "latitudinarian or expansive application of the Workmen's Compensation Law in
favor of the employee or worker" no longer prevails as the burden of showing proof of
causation has shifted back to the employee particularly in cases of sickness or injuries
which are not accepted or listed as occupational by the ECC. The Labor Code abolished
the presumption of compensability and the rule on aggravation of illness caused by the
nature of the employment.
2. NO
- While the court does not dispute petitioner's contention that under the law, in case of
doubt in the implementation and interpretation of the provisions of the Labor Code,
including its implementing rules and regulations, the doubt shall be resolved in favor of the
laborer, the court finds that the same has no application in this case since the pertinent
provisions of the Labor Code leave no room for doubt either in their interpretation or
application.
Disposition Petition is dismissed and the decisions of the GSIS and the ECC denying the
claim are affirmed

BRAVO V EMPLOYEES' COMPENSATION


COMMISSION
143 SCRA 101
FERNAN; July 22, 1986

A2010

- 47 -

Disini

PETITION for certiorari to review the decision of the Employees' Compensation


Commission
FACTS
- Evelio Bravo was a supervising cartographer engineer at the Bureau of Coast and
Geodetic Survey. As litho-photo engraving supervisor (another term for a supervising
cartographer engineer?), he was involved in drafting and plate printing, developing and
processing either dry or wet negatives, and supervising the formulation of lightsensitive
lithographic chemicals from reagent of nitric, phosphoric, oleic acids, potassium
ferricynamide, ammonium hydroxide and ammonium dichromate in the kithographic
laboratory.
- sometime in 1979, he complained of irregular bowel movement, constipation and
abdominal pain. In 1980 he was admitted to St. Lukes Hospital and was diagnosed with
"adenocarcinoma sigmoid (colon) Duke's C and chronic periappendicitis". He went
through a series of operations and incurred hospitalization expenses amounting to
P8,650.05.
- He did not return to work and retired at the age of 45 under the provisions of RA 1616.
He received P37,002.31 from GSIS. He filed a claim for disability benefits in the GSIS.
- GSIS: Denied. His diagnosed disease were not occupational diseases in his particular
employment and his working conditions did not increase the risk of contracting them.
-He sought reconsideration, claiming that his work exposed him to chemicals. His MFR
was denied on the ground that his exposure to photographic solutions as litho-photo
engraving supervisor had no causal relationship to the development of his
adenocarcinoma considering that said ailment is traceable to "familial multiple
polyposis, chronic ulcerative colitis, chronic lympho-granuloma venereum, chronic
granuloma inguinale and perhaps adenoma.
- He appealed to the Employees Compensation Commission, but died pending the
appeal. His widow, Angeles, pursued his appeal.
- Commission: affirmed GSIS deci. Bravo's ailments were "too remote to be related
causally to his work and working conditions" at the Bureau of Coast and Geodetic
Survey. His contention that his cancer could be traced to exposure to photographic
solutions was merely supposition and devoid of medical support.
- Petitioners contention
> while the causes of colonic malignancy are as yet undetermined, there is a "probability"
that the fatal ailment of Bravo was work connected as shown by the fact that he was
exposed to various chemicals which are generally considered predisposing factors of
cancer (relying on the decision in Panotes vs. Employees' Compensation Commission
where it was held that the very fact that the cause of a disease is unknown creates the
probability that the working conditions could have increased the risk of contracting the
disease, if not caused by it) ; that the law merely requires reasonable work-connection
because of the liberal interpretation accorded to social legislation; that under the theory of
increased risk, her husbands cancer of the colon is a compensable disease because his
exposure to chemicals and the "stressful demand" of his work increased the risk of
contracting said ailment; and that Commission issued the Resolutions Nos. 2610 and
267712 which provides guidelines for deciding on pending compensation cases regarding
cancer.
- Solicitor Generals reply
> resolutions are just proofs that the Commission is continuously in involved in its task "to
initiate, rationalize, and coordinate policies of the employees' compensation program."
They do not imply that the law merely requires reasonable work-connection because that
requirement which was mandated in the repealed Workmen's Compensation Act is
different from the present requirement of clear medical basis "where before a mere
aggravation or presumption of compensability was sufficient."
ISSUES
WON cancer of the colon and peri-appendicitis which caused the death of a former lithophoto engraving supervisor are compensable diseases under the Labor Code
a. WON cancer of the colon and peri-appendicitis are listed under compensable diseases
under the Labor Code and Rule III, Section IV of the Amended Rules on Employees
Compensation
b. WON petitioner could claim benefits through the increased risk doctrine
HELD
NO
12 Resolution No. 2610 approves the recommendation of the Commission's Technical Committee on Medical Matters that appealed compensation cases "whose subject contingencies concern cancer
diseases shall be held compensable, in line with pertinent Supreme Court Decisions, provided that such diseases shall be duly confirmed by formal reports on biopsies, or opinions of cancer specialists". That
resolution shall be applied prospectively.

Resolution No. 2677 amends Resolution No. 2610 by adding to the pertinent paragraph thereof the phrase "provided that certain predisposing factors that are medically recognized or proven are present." It also

NATURE

approves the modified guidelines on cancer of the breast, liver stomach (gastric), lungs and nasopharynx. As regards "other types of cancer diseases", the guideline states: "An employee's prolonged exposure
to chemicals may predispose him or her to contract and develop other types of cancer diseases". For cancer cases decided by the Supreme Court, the guidelines states: "A claim must be resolved in favor of a
claimant or appellant if facts of his or her case on record indicate reasonable work-connection of the disease, the disease belongs to borderline or 'twilight' cases, and if the cause of the cancer disease is
unknown".

Labor Law 1
Ratio Article 167, paragraph (1) of the Labor Code and Rule III, Section IN of the
Amended Rules on Employees' Compensation provide that for a sickness and the
resulting disability or death to be compensable, the said sickness must be an occupational
disease listed under Annex "A" of said Rules, otherwise, the claimant or employee
concerned must prove that the risk of contracting the disease is increased by the working
conditions (increased risk doctrine)
a. NO
- Both cancer of the colon and peri-appendicitis are not listed as occupational diseases for
Bravo's kind of employment.
b. NO
- Petitioner failed to submit convincing proofs to entitle her to compensation benefits.
Ratio A claimant who depends on the theory of increased risk must present substantial
proof to show that his ailment was contracted during his employment. He or she must also
submit proof that the risk of contracting the ailment was increased by the particular
working conditions.
Reasoning
- On reliance on Panotes case: In the Panotes case and the Cristobal case, both
claimants presented conditions of their employment. In the present case, the petitioner
only enumerated the chemicals to which Bravo was allegedly exposed as a lithophoto engraving supervisor and rely on the "probability" that those chemicals
caused his cancer of the colon.
On interpretation in compensation cases
> Strict rules of evidence are not applied in compensation cases . However, the present
scheme and theory of employees' compensation under the Labor Code requires a clear
medical basis for a claim for benefits to succeed. There are no more presumptions as to
what caused a particular illness because the determination of compensability is medically
and scientifically oriented.

On application of the resolutions by the Commission


> they were issued after the death of Evelio, and are applied prospectively. Even if they
were applied, the petitioner did not submit formal requirements required by said
resolutions.
On liberal interpretation due to social legislation
> We are aware of the mandate that social legislation should be applied in consonance
with the principles of social justice and protection to labor. However, we cannot adopt a
sweeping interpretation of the law in favor of labor lest we engage in judicial legislation.
Disposition decision of the Employees' Compensation Commission is hereby affirmed.

PAL V NLRC (IRINEO)


201 SRCA 687
NARVASA; September 24, 1991
FACTS
- On the basis of the findings and recommendations of a Fact Finding Panel upon
investigation, Irineo and 3 other PAL employees, Damian, Rabasco and Macatol, were
prosecuted and convicted for estafa through falsification of commercial documents
- All 4 filed motions for reconsideration and/or new trial. Only one of them, Macatol, was
absolved for lack of sufficient evidence. 12 years later, Macatol filed a complaint for
illegal dismissal which was dismissed by the Labor Arbiter on the ground of prescription.
The NLRC affirmed, contending that the prescriptive period accrued from the time of his
dismissal and not the termination of the criminal case
- A later appeal with the IAC resulted in the acquittal of Irineo and Rabasco on grounds of
reasonable doubt.
- 17 years after his dismissal, Irineo filed a complaint against PAL for reinstatement and
backwages, claiming the termination was illegal. The Labor Arbiters decision decreed his
reinstatement without loss of seniority rights, payment of backwages and moral damages
of P300k. The Arbiter overruled the defense of prescription and held that since there was a
PAL circular which placed any employee charged with any crime inimical to the companys
interest under preventive suspension, and a standing order by the CIR forbidding the
dismissal of any PAL employee without court authority, Irineos dismissal merely amounted
to suspension. The Arbiter rendered a judgment terminating Irineos suspension with
backwages and moral damages.
- PAL appealed to the NLRC but failed to obtain a reversal of the Arbiters decision. Hence
this petition for certiorari.
ISSUE
WON the NLRC gravely abused discretion amounting to lack or excess of jurisdiction and
arbitrarily exercised power without due regard for the rule of law
HELD
YES

A2010

- 48 -

Disini

- That there should be care and solicitude in the protection and vindication of the rights of
workingmen cannot be gainsaid; but that care and solicitude cannot justify disregard of
relevant facts or eschewal of rationality in the construction of the text of applicable rules in
order to arrive at a disposition in favor of an employee who is perceived as otherwise
deserving of sympathy and commiseration.
- The letter to Irineo from then PAL president Benigno Toda clearly indicated, For being
involved in the irregular refund of tickets in the international service to the damage and
prejudice of the company, you are dismissed from the service effective immediately. For
the Arbiter and the NLRC to construe this as mere suspension would be illogical.
-Their reliance on PAL circular to justify their decision, construing this as a complete
foreclosure of any alternative action on PALs part was unfounded. To further support their
decision they refer said CIR standing order which had been imposed in relation to a
pending labor dispute with the CIR. However, having ended when the parties entered into
a CBA 2 years before Irineos dismissal, the standing order was no longer relevant to the
event.
- Irineos assertion only after 17 years meant he slept on his rightshis claim is thus timebarred.
- Premises considered, it appears that the NLRCs conclusions are flawed by errors
serious as to constitute grave abuse of discretion
Disposition Court GRANTS the petition and issues the writ of certiorari prayed for.

MANNING INTERNATIONAL CORPORATION V NLRC


(BENEDICTO)
195 SCRA 155
NARVASA; March 13, 1991
NATURE
Petition for certiorari to review
FACTS
- Francisco Benedictoa.k.a. Lazaro Benedicto, according to his passportwas hired by
a foreign firm, Abdulasis & Mohamed A. Aljomaih Co., thru its Philippine representative.
Manning International Corporation, as a truck driver for its establishment in Riyadh, Saudi
Arabia. Benedicto was engaged for a stipulated term of two (2) years. He left for Saudi
Arabia on December 1, 1980 to fulfil his employment contract.
- Some months before the expiration of his contract with Abdulasis, etc., Benedicto was
involved in a vehicular accident, was injured, and in consequence, lost both his legs.
From the date of the accident, February 2, 1982, he was confined at a hospital in Saudi
Arabia until sometime when his employment was terminated. He was repatriated to the
Philippines in August 1982.
- Benedicto filed a complaint with POEA for the recovery of his salary for the unexpired
portion of his contract, insurance benefits and projected cost of medical expenses
amounting to P25,000.00
- POEA dismissed Benedictos claim upon finding that complainant was legally terminated
from employment because of his disability. However, Manning and Abdulasis were
ordered to provide compensation benefits for service-connected illness, injuries or death.
- MFR to NLRC affirmed the decision of POEA
- Judgment became final and executory. Benedicto moved for computation of the
amounts due him, and in substantiation, submitted receipts evidencing his actual medical
expenses. His former employers opposed the motion on the ground that the medical
expenses referred to another person, Lazaro Benedicto but the Administrator overruled
the objection and pointed out that the names Lazaro and Francisco Benedicto both
referred to one person, and directed the issuance of an alias writ of execution to enforce
payment of P12,000 as total and permanent disability benefits and P19,450.00 as
hospitalization and medical expenses for 120 days or a total of P31,450.00.
- MFR was filed to NLRC to protest the limitation of the award to him of medical expenses
to a period of 120 days.
- NLRC set aside the the POEA Order and on considerations of equity and social justice
as well as the theory of medical treatment should not be stopped until Benedictos injury
or disability is healed and entered a new judgment increasing the amount to be paid by
employers.
- Petition for certiorari
ISSUES
1. WON the new judgment of the NLRC is void ab initio, insofar as it attempts to vary the
disposition of the final and executory decision of the POE Administrator

Labor Law 1
2. WON the challenged decision of NLRC is without legal basis and unjust
HELD
1. YES
Ratio When a final judgment becomes executory, it thereby becomes immutable and
unalterable, The judgment may no longer be modified in any respect, even if the
modification is meant to correct what is perceived to be an erroneous conclusion of fact or
law, and regardless of whether the modifications is attempted to be made by the Court
rendering it or by the highest Court of the land. The only recognized exceptions are the
correction of clerical errors or the making of so-called nunc pro tunc entries which cause
no prejudice to any party, and, of course, where the judgment is void.
2. YES
Ratio Considerations of equity and social justice cannot prevail over against the
expressed provision of the labor laws allowing dismissal of employees for cause and
without any provision for separation pay.
Disposition Contested Decision ANNULLED AND SET ASIDE and REINSTATING and
AFFIRMING the Order of the POE Administrator.

RELIANCE & INSURANCE CO INC V NLRC (RELIANCE


SURETY & INSURANCE EMPLOYEES UNION)
193 SCRA 365
SARMIENTO; January 25 1991
NATURE
Petition for certiorari whether or not strikers have been found to have staged an illegal
strike may be reinstated to work.
FACTS
- The manager of Reliance Surety Insurance Co (RSIC) effected a change in the seating
arrangement of its personnel to avoid productivity loss due to personal and non-workrelated conversations, calls and visits.
- Isagani Rubio, Rosalinda Macapagal, Glene Molina, and Severa Cansino protested the
transfer of their tables because said change was without prior notice and was just to
harass them as union members. When the manager insisted, a heated discussion ensued,
during which Rubio and companions insulted the manager and supervisors.
- The 4 employees were asked to explain within 48 hours why no disciplinary action
should be taken against them for misconduct, insubordination, and gross disrespect.
Tension rose in the office as Rubio continued to refuse to stay at his designated place, and
Molina and Macapagal still levelled insults to those who testified against them. Hence,
Rubio and companions were placed under preventive suspension on 3 February 1987 and
ultimately dismissed after investigation on 3 March 1987.
- 6 March 1987, the Reliance Surety & Insurance Employees Union (union) filed in behalf
of the dismissed employees with the NLRC, against the RSIC a complaint for illegal
dismissal including the charge of unfair labor practice.
- Unions claim: The company was guilty of unfair labor practice because it effected
transfer and changes in the seating arrangement to pressure or intimidate union members;
because it interfered in the union members' exercise of their right to self-organization by
forcing them to undertake overtime work even on a non-working Saturday and in times
when there were scheduled union meetings to prevent them from attending the same; and
because, thru its manager and assistant managers, it caused the resignation and
withdrawal of union members from the union.
- Pending trial, the union filed with the DOLE a notice of strike predicated on unfair labor
practices by the company. RSIC was given notice of strike and a telegram from DOLE for
initial conciliation conference both to be held on the same date. But even before the initial
conference could take place, the union in the morning of 17 March 1987 struck and
picketed the company premises, which obstructed the free ingress to and egress from its
premises, thereby preventing its officials and employees from doing their usual duties.
- RSIC them filed with the NLRC a petition to declare the strike illegal due to the defiance
of the 30 or 15 day cooling-off-period, disregard of the legal requirement to furnish the
department with the results of the strike vote at least 7 days before the strike and failure to
furnish a written notice of the meeting to declare a strike to the BLR or the Regional Office
- Labor Arbiter found the strike to be illegal. NLRC affirmed with modification upon appeal
holding that although the strike was illegal, dismissal was not the proper penalty. It said
that the strikers should be reinstated without backwages due to the unions belief that the

A2010

- 49 -

Disini

company was committing unfair labor practice. (Ferrer v. Court of Industrial Relations 6
and Almira v. BF Goodrich Philippines, Inc)
- Petitioners claim: NLRC was guilty of grave abuse of discretion.

ISSUE
1. WON the strike was illegal
2. WON the petition should be granted
HELD
1. YES
- The strike in question was illegal, for failure of the striking personnel to observe legal
strike requirements, to wit: (1) as to the fifteen-day notice; (2) as to the two-thirds required
vote to strike done by secret ballot; (3) as to submission of the strike vote to the
Department of Labor at least seven days prior to the strike.
- NLRC also found that certain strikers harassed non-striking employees, called company
officers names, and committed acts of violence (as a result of which, criminal charges
were brought with the fiscal's office.)
- The strike itself was prompted by no actual, existing unfair labor practice committed by
the petitioner. In effecting a change in the seating arrangement, the petitioner merely
exercised a reasonable prerogative employees could not validly question, much less
assail as an act of unfair labor practice. Rearranging furniture cannot justify a four-monthlong strike. As to the private respondent's charges of harassment, the Commission found
none, and as a general rule, the Court is bound by its findings of fact.
2. YES
- The strike that was illegal in more ways than one, the reinstated union officers were
clearly in bad faith, and to reinstate them without loss of seniority rights, is to reward them
for an act public policy does not sanction.
- The Ferrer and Almira cases did not involve illegal strikes. In Ferrer was a defective
strike, one conducted in violation of the thirty-day "cooling-off" period, but one carried out
in good faith "to offset what petitioners were warranted in believing in good faith to be
unfair labor practices [committed by] Management. What Almira on the other hand
declared was that a violent strike alone does not make the action illegal, which would
justify the dismissal of strikers.
- The Court reiterates that good faith is still a valid defense against claims of illegality of a
strike. We do find, however, not a semblance of good faith here, but rather, plain
arrogance, pride, and cynicism of certain workers.
- WRT respondent, Isagani Rubio, what militates against his readmission to the firm is the
fact that he had accepted the sum of P2,448.80 "in full satisfaction of the . . . Decision" (of
the Labor Arbiter).
- The sympathy of the Court is on the side of the laboring classes, not only because the
Constitution imposes sympathy but because of the one-sided relation between labor and
capital. The Court must take care, however, that in the contest between labor and capital,
the results achieved are fair and in conformity with the rules. We will not accomplish that
objective here by approving the act of the National Labor Relations Commission which we
hold to constitute a grave abuse of discretion.
Disposition petition is GRANTED.

PHILIPPINE AIRLINES, INC. (PAL) V NLRC


[PAGE 25]
DUNCAN ASSOCIATION V GLAXO WELLCOME PHILS
[PAGE 43]
UNITED PEPSI-COLA SUPERVISORY UNION (UPSU) V
LAGUESMA
288 SCRA 15
MENDOZA; March 25, 1998
FACTS
- UPSU is a union of supervisory employees. The union filed a petition for certification
election on behalf of the route managers at Pepsi-Cola Products Philippines, Inc.
However, its petition was denied by the med-arbiter and, on appeal, by the Secretary of
Labor and Employment, on the ground that the route managers are managerial employees
and, therefore, ineligible for union membership under the first sentence of Art. 245 of the
Labor Code, which says, Managerial employees are not eligible to join, assist or form any
labor organization. Supervisory employees shall not be eligible for membership in a labor
organization of the rank-and-file employees but may join, assist or form separate labor
organizations of their own.
ISSUE

Labor Law 1
1. WON the route managers at Pepsi-Cola Products Philippines, Inc. are managerial
employees
2. WON the first sentence of Art. 245 of the Labor Code, prohibiting managerial
employees from forming, assisting or joining any labor organization, is constitutional in
light of Art. III, Sec.8 of the Constitution. The right of the people, including those
employed in the public and private sectors, to form unions, associations, or societies for
purposes not contrary to law shall not be abridged.
HELD
1. YES
- Their job descriptions clearly reveal so. They also fall under this category under the
purview of art. 212. The term manager generally refers to anyone who is responsible for
subordinates and other organization resources. As a class, managers constitute three
levels of a pyramid.
- What distinguishes them from the rank-and file employees is that they act in the interest
of the employer in supervising such rank-and-file employees
- Managerial employees may therefore be said to fall into two distinct categories: the
managers per se, who compose the former group described above, and the
supervisors who form the latter group. Whether they belong to the first or second
category, managers, vis--vis employers, are, likewise, employees
2. NO
- As already stated, whether they belong to the first category (managers per se) or the
second category (supervisors), managers are employees. Nonetheless, in the United
States, as Justice Puno's separate opinion notes, supervisors have no right to form
unions. They are excluded from the definition of the term "employee" in 2(3) of the
Labor-Management Relations Act of 1947.
- Commission intended the absolute right to organize of government workers, supervisory
employees, and security guards to be constitutionally guaranteed. By implication, no
similar absolute constitutional right to organize for labor purposes should be deemed to
have been granted to top-level and middle managers. As to them the right of selforganization may be regulated and even abridged conformably to Art. III, 8.
- Types of Managerial Employees:
> FIRST-LINE MANAGERS The lowest level in an organization at which individuals are
responsible for the work of others is called first-line or first-level management. First-line
managers direct operating employees only; they do not supervise other managers.
Example of first-line managers are the foreman or production supervisor in a
manufacturing plant, the technical supervisor in a research department, and the clerical
supervisor in a large office. First-level managers are often called supervisors.
> MIDDLE MANAGERS The term middle management can refer to more than one level
in an organization. Middle managers direct the activities of other managers and
sometimes also those of operating employees. Middle managers principal responsibilities
are to direct the activities that implement their organizations policies and to balance the
demands of their superiors with the capacities of their subordinates. A plant manager in
an electronics firm is an example of a middle manager.
> TOP MANAGERS Composed of a comparatively small group of executives, top
management is responsible for the overall management of the organization. It
establishes operating policies and guides the organizations interactions with its
environment. Typical titles of top managers are chief executive officer, president, and
senior vice-president. Actual titles vary from one organization to another and are not
always a reliable guide to membership in the highest management classification.
Disposition petition is DISMISSED

SONZA V ABS-CBN BROADCASTING CORPORATION


[PAGE 42]
ASIATIC DEVELOPMENT CORP V BROGADA
495 SCRA 166
CORONA; July 14, 2006
NATURE
Petition for review on certiorari
FACTS
- Respondents Wellington and Flordeliza Brogada are the parents of Fermin B. Brogada
who was allegedly employed by petitioner Asiatic Development Corporation from July
1994 up to his death in November 14, 1996.
- Respondents filed with the SSC a petition for social security coverage and payment of
contributions in order to avail of the benefits accruing from the death of Fermin. They
alleged that Fermin worked as survey aide under Engr. Bienvenido Orense, petitioners
geodetic engineer. Fermin was working on a project with Engr. Orense for one of
petitioners clients when he was shot and killed.

A2010

- 50 -

Disini

- Petitioner denied its liability on the ground that there was no employer-employee
relationship between it and Fermin. It claimed that Fermin was the employee of Engr.
Orense.
- SSS held that Fermin was an employee and was subject to the compulsory coverage.
On appeal, the SSC resolution was affirmed by the CA.
ISSUE
WON an employer-employee relationship exists
HELD
- The question of WON an employer-employee relationship exists is a question of fact. In
petitions for review on certiorari under Rule 45, only questions of law may be raised by the
parties and passed upon by this Court. Factual findings of quasi-judicial bodies, when
adopted and confirmed by the CA and if supported by substantial evidence, are accorded
respect and even finality by this Court. While this Court has recognized several
exceptions to this rule, none of these exceptions finds application here.
- Both the SSC and CA found that Fermin was petitioners employee. Thus, petitioner is
liable for unpaid social security contributions.
- Petitioners claims are a mere reiteration of arguments unsuccessfully raised before the
SSC and the CA. No compelling reason whatsoever is shown by petitioner for this Court
to reverse the SSCs findings and conclusions, as affirmed by the CA.
Disposition Petition is DENIED.

VILLAVILLA V CA (SSS, MERCADO, COSUCO)


212 SCRA 488
BELLOSILLO; August 11, 1992
FACTS
- Arturo Villavilla, son of petitioners, was employed as "tripulante" (crew member) of the
fishing boat "F/B Saint Theresa" from 1974 until September 11, 1977, when the boat sank
off Isla Binatikan, Taytay, Palawan. Arturo was not among the known survivors of that
sinking and had been missing since then.
- On November 20, 1979, petitioners Andres Villavilla and Ester Gadiente Villavilla,
parents of Arturo, filed a petition with the Social Security Commission against Reynaldo
Mercado and Marcelino Cosuco, owners of the ill-fated fishing boat, for death
compensation benefits of Arturo whom respondents failed to register as their employee.
- On May 29, 1981, the Social Security System (SSS) filed a petition in intervention
alleging that records from the SSS Production Department showed that "F/B Saint
Theresa", owned by Marcelino Cosuco and operated by Reynaldo Mercado, was a
registered member-employer, and that in the event petitioners succeeded in proving the
employment of Arturo with private respondents, the latter should be held liable in damages
equivalent to the benefits due the petitioners for failure to report Arturo for coverage
pursuant to Sec. 24 (a) of the Social Security Act, as amended.
- Respondent Cosuco filed his answer denying all allegations in the petition and claiming
that he already sold the fishing boat to respondent Mercado on December 10, 1975, and
from then on he did not participate anymore in the operation and management of the boat
nor in the hiring of its crewmembers.
- Meanwhile respondent Mercado was declared in default for failure to file his answer.
- After petitioners had presented their evidence and rested their case, respondent Cosuco
filed a motion to dismiss (demurrer to evidence) on the ground of res judicata and lack of
cause of action.
- Respondent Social Security Commission issued an Order dismissing the petition for lack
of cause of action.
- The Court of Appeals affirmed the questioned Order of respondent Commission there
being no reversible error.
ISSUE
1. WON there was an employer-employee relationship between petitioners' deceased son,
Arturo Villavilla, and herein private respondents
2. WON private respondents are liable for death compensation benefits of Arturo Villavilla
3. WON there was a violation of the Social Security Act, as amended, by private
respondents for not registering Arturo Villavilla with the System as their employee as
mandated by law
HELD
1. NO
- The records disclose that the relationship between Mercado and the crew members of
the ship headed by its skipper, Capt. Pedro Matibag, is one positively showing the
existence of a joint venture. This is clearly revealed in the testimonies of Capt. Pedro
Matibag and Gil Chua, a crew member, both witnesses for petitioners.
- The arrangement between the boat owner and the crew members, one of whom was
petitioners' son, partook of the nature of a joint venture: the crew members did not receive

Labor Law 1
fixed compensation as they only shared in their catch; they ventured to the sea
irrespective of the instructions of the boat owners, i.e., upon their own best judgment as to
when, how long, and where to go fishing; the boat owners did not hire them but simply
joined the fishing expedition upon invitation of the ship master, even without the
knowledge of the boat owner. In short, there was neither right of control nor actual
exercise of such right on the part of the boat owner over his crew members.
- It is clear that there was no employer-employee relationship between petitioner's son
Arturo and private respondent Mercado, much less private respondent Cosuco. As such,
Arturo could not be made subject of compulsory coverage under the Social Security Act;
hence, private respondents cannot be said to have violated said law when they did not
register him with the Social Security System. A fortiori, respondent as well as intervenor
are not answerable to petitioners for any death benefits under the law.
- Culled from the foregoing, the inexorable conclusion is that respondent Court of Appeals
did not err in sustaining the judgment of respondent Social Security Commission.
- It may not be amiss to mention that while petitioners merely raise factual questions which
are not proper under Rule 45 of the Rules of Court, We nevertheless went to great lengths
in dissecting the facts of this case if only to convince Us that petitioners, who are pauper
litigants and seeking claims under a social legislation, have not been denied its benefits.
For, We are not unaware that in this jurisdiction all doubts in the implementation and
interpretation of provisions of social legislations should be resolved in favor of the working
class. But, alas, justice is not fully served by sustaining the contention of the poor simply
because he is poor. Justice is done by properly applying the law regardless of the station
in life of the contending parties.

MIGUEL V JCT GROUP INC


453 SCRA 529
PANGANIBAN; March 16, 2005
NATURE
Petition for Review on Certiorari under Rule 45 of the Rules of Court CA decision on the
ground of grave abuse of discretion because it annulled and set aside decisions of the
labor arbiter and NLRC
FACTS
- 1984 > Glorious Sun Garment Manufacturing Company was a garment exporter until it
folded up and, thereafter, De Soliel Apparel Manufacturing Corporation and American
Inter-Fashion Corporation took over Glorious Suns manufacturing facilities and absorbed
its employees (petitioners Miguel et al)
- 1986 > PCGG sequestered De Soleil and AIFC
- 1989 > JCT Group, Inc. and De Soleil executed a Management and Operating
Agreement for servicing De Soleils export quota to ensure its rehabilitation and preserve
its viability and profitability
- 1990 > De Soleil ceased business operations thus terminating employment
- 1993 > complaints for illegal dismissal and payment of backwages before NLRC against
De Soleil, AIFC, PCGG, Glorious Sun, JCT, Nemesio Co and Vicente Cuevas III. But JCT
and Cuevas filed a motion to dismiss due to lack of jurisdiction because of the absence of
employer-employee relationship between them and petitioners.
- 1995 > Without resolving the motion to dismiss, Labor Arbiter Sampang rendered (1)
Declaring De Soleil, AIFC, PCGG, Glorious Sun, JCT, Nemesio Co and Cuevas jointly and
severally guilty of illegal dismissal and to pay complainants backwages, separation pay,
service incentive leave pay, 13th month pay, unpaid salaries as computed by the
Research and Information Unit (2) Declaring De Soleil et al liable for the payment of
attorneys fees (10% of the total awards or P3,691,743.06)
- Because of the huge amount [monetary award, inclusive of attorneys fees, aggregated
P41,313,094.98 computed by Research and Information Unit], JCT and Cuevas and
Glorious Sun filed separate motions with NLRC for reduction of the appeal bond . NLRC
reduced it to P5,000,000.00 for each respondent. Again, they filed a motion for
reconsideration of said order by way of further reduction of the bond to P500,000.00. CA
and SCA denied their motions
- 1995 > Meanwhile, Glorious Sun and JCT et al et al appealed the labor arbiters decision
to NLRC and petitioners filed a motion to dismiss the appeals bec of failure to post a bond
as required in A223 LC
- 1996 > NLRC absolved Glorious Sun and dismissed JCT et als appeal and sent to CA
for appropriate disposition
- CA: reversed NLRC decision and remanded the case to the labor arbiter bec it found no
factual basis for the ruling that JCT had become the employer of petitioners after the
cessation of operations of Glorious Sun and failed to explain Cuevas liability in solidum
with AIF, De Soleil and JCT; hence this petition by Miguel et al
ISSUE
WON CA committed grave abuse of discretion amounting in ruling to remand the case to
the labor arbiter because of lack of factual findings to prove employer-employee

A2010

- 51 -

Disini

relationship between JCT et al and Miguel et al which would be the basis of the liability of
JCT et al
HELD
NO. Instead, NLRC and the Labor arbiter abused their discretion when they ruled in
favor of the petitioners without determining the existence of an employer-employee
relationship between them and respondents because it was silent on why JCT and
Cuevas were held liable.
Doctrine Grave abuse of discretion implies such capricious and whimsical exercise of
judgment as to be equivalent to lack or excess of jurisdiction. That is, power is arbitrarily
or despotically exercised by reason of passion, prejudice, or personal hostility; and caprice
is so patent or so gross as to amount to an evasion of a positive duty, or to a virtual refusal
to perform the duty enjoined or to act at all in contemplation of law.
Reasoning
- LABOR ARBITER: made no determination whether there was employer-employee
relationship and, if so, whether JCT et al assumed the obligations of Miguel et als
previous employers. There is no dispute that given the nature of their functions and length
of services, were regular employees. But the question is: who was/were their
employer/s?
- MOA: does not appear that JCT became the employer of Miguel et al by virtue of this
- NLRC: silent on JCT being the employer of Miguel et al after Glorious Sun ceased
operations, save for its conclusion that they were absorbed by, or their work continued
under JCT and did not state the reason for liability in solidum of Cuevas. Computation of
the monetary award totaling P37,557,317.08 (exclusive of attorneys fees) covers a period
starting on initial employment (with Glorious Sun) some dating back to 1978
- Saballa v NLRC > This Court has previously held that judges and arbiters should draw
up their decisions and resolutions with due care, and make certain that they truly and
accurately reflect their conclusions and their final dispositions. The same thing goes for
the findings of fact made by the NLRC, as it is a settled rule that such findings are entitled
to great respect and even finality when supported by substantial evidence; otherwise, they
shall be struck down for being whimsical and capricious and arrived at with grave abuse of
discretion. It is a requirement of due process and fair play that the parties to a litigation be
informed of how it was decided, with an explanation of the factual and legal reasons that
led to the conclusions of the court. A decision that does not clearly and distinctly state the
facts and the law on which it is based leaves the parties in the dark as to how it was
reached and is especially prejudicial to the losing party, who is unable to pinpoint the
possible errors of the court for review by a higher tribunal.
Obiter
- employer-employee relationship test:
1) power to select employees
2) who pays for their wages
3) who has the power to dismiss them, and
4) who exercises control in the methods and the results by which the work is
accomplished
*** The last factor, the control test, is the most important.
Disposition Petition is DENIED and the assailed Decision AFFIRMED

WACK-WACK GOLF & COUNTRY CLUB V NLRC


(CAGASAN, DOMINGUEZ, BSMI)
456 SCRA 280
CALLEJO, SR; April 15, 2005
NATURE
Petition for review decision of CA
FACTS
- A large portion of the Wack Wack (WW) clubhouse (including the kitchen) was destroyed
by fire, and because of this, the management had to suspend operations of the Food and
Beverage department, requiring the suspension of 54 employees. The Employees Union
found the suspension as arbitrary and constitutive of union-busting, and went on strike.
The parties soon after entered into an amicable settlement, whereby a special separation
benefit/ retirement package was formulated. The same provides for, among other things,
a 1 month separation pay for every year of service, and be considered on priority basis
for employment by concessionaires and/or contractors, and even by the club, upon full
resumption of operations.
- The package was availed of by 3 employees (Cagasan, Dominguez, and Baluyot), who
received large sums of money as separation pay.
Soon after, WW entered into a Management Contract with Business Staffing and
Management Inc (BSMI), whereby the latter will provide management services for WW.
Cagasan and Dominguez filed their application for employment with BMSI. They, by
reason of the priority given by the separation package, were rehired on probationary
status by BMSI.

Labor Law 1
- WW also engaged other contractors in the operations of the club (like janitorial services,
Finance and accounting services). Because of the various management service contracts,
BMSI made an organizational analysis and manpower evaluation to streamline its
operations. It found the positions of Cagasan and Domiguez redundant, and subsequently
terminated them. Cagasan and Dominguez then filed complaints in the NLRC for illegal
dismissal against WW. NLRC ordered reinstatement
ISSUES
1. WON BMSI is an independent contractor (which will answer the question as WON there
was an employer-employee relationship)
2. WON the employees were illegally dismissed
HELD
1. YES
Reasoning
- An independent contractor is one who undertakes job contracting, i.e., a person who:
(a) carries on an independent business and undertakes the contract work on his own
account under his own responsibility according to his own manner and method, free from
the control and direction of his employer or principal in all matters connected with the
performance of the work except as to the results thereof; and (b) has substantial capital or
investment in the form of tools, equipments, machineries, work premises and other
materials which are necessary in the conduct of the business. Jurisprudence shows that
determining the existence of an independent contractor relationship, several factors may
be considered, such as, but not necessarily confined to, whether or not the contractor is
carrying on an independent business; the nature and extent of the work; the skill required;
the term and duration of the relationship; the right to assign the performance of specified
pieces of work; the control and supervision of the work to another; the employers power
with respect to the hiring, firing, and payment of the contractors workers; the control of the
premises; the duty to supply premises, tools, appliances, materials and labor; and the
mode, manner and terms of payment.
- There is indubitable evidence showing that BSMI is an independent contractor, engaged
in the management of projects, business operations, functions, jobs and other kinds of
business ventures, and has sufficient capital and resources to undertake its principal
business. It had provided management services to various industrial and commercial
business establishments.
- In December 1993, Labor Sec. Laguesma, in a case, recognized BSMI as an
independent contractor. As a legitimate job contractor, there can be no doubt as to the
existence of an employer-employee relationship between the contractor and the workers.
Thus, there is no employer-employee relation between WW and the workers.
2. NO
Ratio As there was no employer-employee relationship between WW and the
complainants, there can be no illegal dismissal.
Reasoning
- the complainants (private respondents herein) were validly terminated upon their option
to take the separation package provided by WW. Thus, the same have no cause of action
against WW.
Disposition Petition granted. CA and NLRC decisions set aside

PHILIPPINE GLOBAL COMMUNICATIONS INC V DE


VERA
459 SCRA 260
GARCIA; June 7, 2005
NATURE
petition for review on certiorari
FACTS
- Petitioner Philippine Global Communications, Inc. (PhilCom), is a corporation engaged in
the business of communication services and allied activities, while respondent Ricardo De
Vera is a physician by profession whom petitioner enlisted to attend to the medical needs
of its employees.
- It appears that on 15 May 1981, De Vera, via a letter dated 15 May 1981, offered his
services to the petitioner, therein proposing his plan of works required of a practitioner in
industrial medicine.
- The parties agreed and formalized respondents proposal in a document denominated as
RETAINERSHIP CONTRACT which will be for a period of one year subject to renewal, it
being made clear therein that respondent will cover the retainership the Company
previously had with Dr. K. Eulau and that respondents retainer fee will be at P4,000.00
a month. Said contract was renewed yearly. The retainership arrangement went on from
1981 to 1994 with changes in the retainers fee. However, for the years 1995 and 1996,
renewal of the contract was only made verbally. On December 1996 Philcom, thru a letter
bearing on the subject boldly written as TERMINATION RETAINERSHIP CONTRACT,
informed De Vera of its decision to discontinue the latters retainers contract with the

A2010

- 52 -

Disini

Company effective at the close of business hours of December 31, 1996 because
management has decided that it would be more practical to provide medical services to its
employees through accredited hospitals near the company premises.
- On 22 January 1997, De Vera filed a complaint for illegal dismissal before the National
Labor Relations Commission (NLRC), alleging that that he had been actually employed by
Philcom as its company physician since 1981 and was dismissed without due process.
He averred that he was designated as a company physician on retainer basis for
reasons allegedly known only to Philcom. He likewise professed that since he was not
conversant with labor laws, he did not give much attention to the designation as anyway
he worked on a full-time basis and was paid a basic monthly salary plus fringe benefits,
like any other regular employees of Philcom.
- On 21 December 1998, Labor Arbiter Ramon Valentin C. Reyes came out with a decision
dismissing De Veras complaint for lack of merit, on the rationale that as a retained
physician under a valid contract mutually agreed upon by the parties, De Vera was an
independent contractor and that he was not dismissed but rather his contract with
[PHILCOM] ended when said contract was not renewed after December 31, 1996.
NLRC reversed (the word used is modified) that of the Labor Arbiter, on a finding that De
Vera is Philcoms regular employee and accordingly directed the company to reinstate
him to his former position without loss of seniority rights and privileges and with full
backwages from the date of his dismissal until actual reinstatement.
- Court of Appeals modified NLRCs decision that of the NLRC by deleting the award of
traveling allowance, and ordering payment of separation pay to De Vera in lieu of
reinstatement.
ISSUES
WON an employer-employee relationship exists between petitioner and respondent
HELD
NO
- De Vera was an independent contractor beinf the retained physician of petitioner
company.
- In a long line of decisions, the Court, in determining the existence of an employeremployee relationship, has invariably adhered to the four-fold test, to wit: the selection and
engagement of the employee; the payment of wages; the power of dismissal; and the
power to control the employees conduct, or the so-called control test, considered to be
the most important element.
- Applying the four-fold test to this case, we initially find that it was respondent himself who
sets the parameters of what his duties would be in offering his services to petitioner in the
letter which he sent to petitioner.
- The letter was substantially the basis of the labor arbiters finding that there existed no
employer-employee relationship between petitioner and respondent, in addition to the
following factual settings:
- The fact that the complainant was not considered an employee was recognized by the
complainant himself in a signed letter, the tenor of which indicated that the complainant
was proposing to extend his time with the respondent and seeking additional
compensation for said extension. This shows that the respondent PHILCOM did not have
control over the schedule of the complainant as it [is] the complainant who is proposing his
own schedule and asking to be paid for the same. This is proof that the complainant
understood that his relationship with the respondent PHILCOM was a retained physician
and not as an employee. If he were an employee he could not negotiate as to his hours of
work.
- De Veras service for the respondent was covered by a retainership contract [which] was
renewed every year from 1982 to 1994. Upon reading the contract dated September 6,
1982, signed by the complainant himself (Annex C of Respondents Position Paper), it
clearly states that is a retainership contract. The retainer fee is indicated thereon and the
duration of the contract for one year is also clearly indicated in paragraph 5 of the
Retainership Contract. The complainant cannot claim that he was unaware that the
contract was good only for one year, as he signed the same without any objections. The
complainant also accepted its renewal every year thereafter until 1994. As a literate
person and educated person, the complainant cannot claim that he does not know what
contract he signed and that it was renewed on a year to year basis.
- The labor arbiter added the indicia, not disputed by respondent, that from the time he
started to work with petitioner, he never was included in its payroll; was never deducted
any contribution for remittance to the Social Security System (SSS); and was in fact
subjected by petitioner to the ten (10%) percent withholding tax for his professional fee, in
accordance with the National Internal Revenue Code, matters which are simply
inconsistent with an employer-employee relationship.
- Clearly, the elements of an employer-employee relationship are wanting in this case. We
may add that the records are replete with evidence showing that respondent had to bill
petitioner for his monthly professional fees It simply runs against the grain of common
experience to imagine that an ordinary employee has yet to bill his employer to receive his
salary.
- We note, too, that the power to terminate the parties relationship was mutually vested on
both. Either may terminate the arrangement at will, with or without cause.Finally,
remarkably absent from the parties arrangement is the element of control, whereby the

Labor Law 1
employer has reserved the right to control the employee not only as to the result of the
work done but also as to the means and methods by which the same is to be
accomplished.
- Here, petitioner had no control over the means and methods by which respondent went
about performing his work at the company premises. He could even embark in the private
practice of his profession, not to mention the fact that respondents work hours and the
additional compensation therefor were negotiated upon by the parties. In fine, the parties
themselves practically agreed on every terms and conditions of respondents engagement,
which thereby negates the element of control in their relationship. For sure, respondent
has never cited even a single instance when petitioner interfered with his work.
Disposition petition is GRANTED and the challenged decision of the Court of Appeals
REVERSED and SET ASIDE. The 21 December 1998 decision of the labor arbiter is
REINSTATED.

SONZA V ABS-CBN
[PAGE 42]
JARDIN V NLRC (PHILJAMA INTL)
326 SCRA 299
QUISUMBING; February 23, 2000
NATURE
Special civil action for certiorari seeks to annul the decision of public respondent
promulgated on October 28, 1994, in NLRC NCR CA No. 003883-92, and its resolution
dated December 13, 1994 which denied petitioners motion for reconsideration.
FACTS
- Petitioners were drivers of private respondent, Philjama International Inc., a domestic
corporation engaged in the operation of "Goodman Taxi." Petitioners used to drive private
respondents taxicabs every other day on a 24-hour work schedule under the boundary
system. Under this arrangement, the petitioners earned an average of P400.00 daily.
Nevertheless, private respondent admittedly regularly deducts from petitioners daily
earnings the amount of P30.00 supposedly for the washing of the taxi units. Believing that
the deduction is illegal, petitioners decided to form a labor union to protect their rights and
interests.
- Upon learning about the plan of petitioners, private respondent refused to let petitioners
drive their taxicabs when they reported for work on August 6, 1991, and on succeeding
days. Petitioners suspected that they were singled out because they were the leaders and
active members of the proposed union. Aggrieved, petitioners filed with the labor arbiter a
complaint against private respondent for unfair labor practice, illegal dismissal and illegal
deduction of washing fees. In a decision dated August 31, 1992, the labor arbiter
dismissed said complaint for lack of merit.
- On appeal, the NLRC (public respondent herein), in a decision dated April 28, 1994,
reversed and set aside the judgment of the labor arbiter. The labor tribunal declared that
petitioners are employees of private respondent, and, as such, their dismissal must be for
just cause and after due process.
- Private respondents first motion for reconsideration was denied. Remaining hopeful,
private respondent filed another motion for reconsideration. This time, public respondent,
in its decision dated October 28, 1994, granted aforesaid second motion for
reconsideration. It ruled that it lacks jurisdiction over the case as petitioners and private
respondent have no employer-employee relationship. It held that the relationship of the
parties is leasehold which is covered by the Civil Code rather than the Labor Code.
ISSUE
WON the NLRC committed grave abuse of discretion in entertaining the motion for
reconsideration and in holding that there is no employer-employee relationship in the
boundary system.
HELD
YES
Ratio Only one motion for reconsideration from the same party is allowed before the
NLRC in line with the policy of assisting the parties in obtaining an expeditious and
inexpensive settlement of labor cases. When the NLRC entertained the second motion for
reconsideration, it therefore committed grave abuse of discretion.
Reasoning
- The phrase "grave abuse of discretion amounting to lack or excess of jurisdiction" has
settled meaning in the jurisprudence of procedure. It means such capricious and
whimsical exercise of judgment by the tribunal exercising judicial or quasi-judicial power
as to amount to lack of power.
- In this case before us, private respondent exhausted administrative remedy available to it
by seeking reconsideration of public respondents decision dated April 28, 1994, which

A2010

- 53 -

Disini

public respondent denied. With this motion for reconsideration, the labor tribunal had
ample opportunity to rectify errors or mistakes it may have committed before resort to
courts of justice can be had. Thus, when private respondent filed a second motion for
reconsideration, public respondent should have forthwith denied it in accordance with Rule
7, Section 14 of its New Rules of Procedure which allows only one motion for
reconsideration from the same party, thus:
"SEC. 14. Motions for Reconsideration. --- Motions for reconsideration of any order,
resolution or decision of the Commission shall not be entertained except when based
on palpable or patent errors, provided that the motion is under oath and filed within ten
(10) calendar days from receipt of the order, resolution or decision with proof of service
that a copy of the same has been furnished within the reglementary period the adverse
party and provided further, that only one such motion from the same party shall be
entertained." [Emphasis supplied]
- The rationale for allowing only one motion for reconsideration from the same party is to
assist the parties in obtaining an expeditious and inexpensive settlement of labor cases.
For obvious reasons, delays cannot be countenanced in the resolution of labor disputes.
The dispute may involve no less than the livelihood of an employee and that of his loved
ones who are dependent upon him for food, shelter, clothing, medicine, and education. It
may as well involve the survival of a business or an industry.
- As correctly pointed out by petitioner, the second motion for reconsideration filed by
private respondent is indubitably a prohibited pleading which should have not been
entertained at all. Public respondent cannot just disregard its own rules on the pretext of
"satisfying the ends of justice", especially when its disposition of a legal controversy ran
afoul with a clear and long standing jurisprudence in this jurisdiction as elucidated in the
subsequent discussion. Clearly, disregarding a settled legal doctrine enunciated by this
Court is not a way of rectifying an error or mistake. In our view, public respondent gravely
abused its discretion in taking cognizance and granting private respondents second
motion for reconsideration as it wrecks the orderly procedure in seeking reliefs in labor
cases.
Obiter
- There is another compelling reason why we cannot leave untouched the flip-flopping
decisions of the public respondent. As mentioned earlier, its October 28, 1994 judgment is
not in accord with the applicable decisions of this Court. The labor tribunal reasoned out
as follows:
- Four-fold test for employer-employee relations:
(1) the selection and engagement of the employee;
(2) the payment of wages;
(3) the power of dismissal; and
(4) the power of control the employees conduct.
- NLRC found that the boundary system is a leasehold system which takes it out of the
ordinary notion of control over employees conduct.
- The SC iterated its ruling that the relationship between jeepney owners/operators on one
hand and jeepney drivers on the other under the boundary system is that of employeremployee and not of lessor-lessee.
- The SC explained that in the lease of chattels, the lessor loses complete control over the
chattel leased although the lessee cannot be reckless in the use thereof, otherwise he
would be responsible for the damages to the lessor. In the case of jeepney
owners/operators and jeepney drivers, the former exercise supervision and control over
the latter. The management of the business is in the owners hands. The owner as holder
of the certificate of public convenience must see to it that the driver follows the route
prescribed by the franchising authority and the rules promulgated as regards its operation.
- As consistently held by this Court, termination of employment must be effected in
accordance with law. The just and authorized causes for termination of employment are
enumerated under Articles 282, 283 and 284 of the Labor Code. The requirement of notice
and hearing is set-out in Article 277 (b) of the said Code. Hence, petitioners, being
employees of private respondent, can be dismissed only for just and authorized cause,
and after affording them notice and hearing prior to termination. In the instant case, private
respondent had no valid cause to terminate the employment of petitioners. Neither were
there two (2) written notices sent by private respondent informing each of the petitioners
that they had been dismissed from work. These lack of valid cause and failure on the part
of private respondent to comply with the twin-notice requirement underscored the illegality
surrounding petitioners dismissal.
- Under the law, an employee who is unjustly dismissed from work shall be entitled to
reinstatement without loss of seniority rights and other privileges and to his full
backwages, inclusive of allowances, and to his other benefits or their monetary equivalent
computed from the time his compensation was withheld from him up to the time of his
actual reinstatement It must be emphasized, though, that recent judicial pronouncements
distinguish between employees illegally dismissed prior to the effectivity of Republic Act
No. 6715 on March 21, 1989, and those whose illegal dismissals were effected after such
date. Thus, employees illegally dismissed prior to March 21, 1989, are entitled to
backwages up to three (3) years without deduction or qualification, while those illegally
dismissed after that date are granted full backwages inclusive of allowances and other
benefits or their monetary equivalent from the time their actual compensation was withheld
from them up to the time of their actual reinstatement. The legislative policy behind
Republic Act No. 6715 points to "full backwages" as meaning exactly that, i.e., without

Labor Law 1
deducting from backwages the earnings derived elsewhere by the concerned employee
during the period of his illegal dismissal. Considering that petitioners were terminated from
work on August 1, 1991, they are entitled to full backwages on the basis of their last daily
earnings.

MANILA GOLF & COUNTRY CLUB INC V IAC (LLAMAR)


337 SCRA 207
NARVASA; September 27, 1994
NATURE
Petition for review
FACTS
- three separate proceedings, all initiated by or on behalf of herein private respondent and
his fellow caddies:
1) filed with the Social Security Commission (SSC) via petition of 17 persons who styled
themselves "Caddies of Manila Golf and Country Club-PTCCEA (Philippine Technical,
Clerical, Commercial Employees Association) for coverage and availment of benefits
under the Social Security Act. It alleged that although the petitioners were employees of
the Manila Golf and Country Club, a domestic corporation, the latter had not registered
them as such with the SSS.
2) a certification election case filed with the Labor Relations Division of the Ministry of
Labor by the PTCCEA on behalf of the same caddies- it was resolved in favor of the
petitioners
3) a compulsory arbitration case initiated before the Arbitration Branch of the Ministry of
Labor by the same labor organization- it was dismissed for lack of merit by Labor Arbiter
on the ground that there was no employer-employee relationship between the petitioning
caddies and the respondent Club
- In the case before the SSC, the Club filed answer praying for the dismissal of the
petition, alleging in substance that the petitioners, caddies by occupation, were allowed
into the Club premises to render services as such to the individual members and guests
playing the Club's golf course and who themselves paid for such services; that as such
caddies, the petitioners were not subject to the direction and control of the Club as regards
the manner in which they performed their work; and hence, they were not the Club's
employees.
- Subsequently, all but two (Fermin Llamar and Raymundo Jomok) of the 17 petitioners of
their own accord withdrew their claim for social security coverage, avowedly coming to
realize that indeed there was no employment relationship between them and the Club.
The Commission dismissed the petition for lack of merit:. . . that the caddy's fees were
paid by the golf players themselves and not by respondent club...While respondent club
promulgates rules and regulations on the assignment, deportment and conduct of caddies
the same are designed to impose personal discipline among the caddies but not to direct
or conduct their actual work. In fact, a golf player is at liberty to choose a caddy of his
preference regardless of the respondent club's group rotation system and has the
discretion on whether or not to pay a caddy...This lends credence to respondent's
assertion that the caddies are never their employees in the absence of two elements,
namely, (1) payment of wages and (2) control or supervision over them.
- From this Resolution appeal was taken to the IAC by the union representing Llamar and
Jomok. The appeal ascribed two errors to the SSC: (1) refusing to suspend the
proceedings to await judgment by the Labor Relations Division of National Capital
Regional Office in the certification election on the precise issue of the existence of
employer-employee relationship between the respondent club and the appellants, it being
contended that said issue was "a function of the proper labor office"; and (2) adjudicating
that self same issue a manner contrary to the ruling of the Director of the Bureau of Labor
Relations, which "has not only become final but (has been) executed or (become) res
adjudicata."
- IAC: declared Fermin Llamar an employee of the Manila
Gold and Country Club,
ordering that he be reported as such for social security coverage and paid any
corresponding benefits, but it conspicuously ignored the issue of res adjudicata raised in
said second assignment.
- The questioned employer-employee relationship between the Club and Fermin Llamar
passed the so-called "control test," establishment in the case i.e., "whether the
employer controls or has reserved the right to control the employee not only as to the
result of the work to be done but also as to the means and methods by which the same is
to be accomplished," the Club's control over the caddies encompassing:
(a) the promulgation of no less than 24 rules and regulations just about every aspect of
the conduct that the caddy must observe, or avoid, when serving as such, any violation

A2010

- 54 -

Disini

of any which could subject him to disciplinary action, which may include suspending or
cutting off his access to the club premises; (b) the devising and enforcement of a group
rotation system whereby a caddy is assigned a number which designates his turn to
serve a player; (c) the club's "suggesting" the rate of fees payable to the caddies.

ISSUE
WON persons rendering caddying services for members of golf clubs and their guests in
said clubs' courses or premises are the employees of such clubs and therefore within the
compulsory coverage of the Social Security System (SSS)
HELD
NO
Ratio The Court does not agree that said facts necessarily or logically point to such a
relationship, and to the exclusion of any form of arrangements, other than of employment,
that would make the respondent's services available to the members and guest of the
petitioner. As long as it is, the list made in the appealed decision detailing the various
matters of conduct, dress, language, etc. covered by the petitioner's regulations, does not,
in the mind of the Court, so circumscribe the actions or judgment of the caddies concerned
as to leave them little or no freedom of choice whatsoever in the manner of carrying out
their services.
Reasoning
- In the very nature of things, caddies must submit to some supervision of their conduct
while enjoying the privilege of pursuing their occupation within the premises and grounds
of whatever club they do their work in. For all that is made to appear, they work for the
club to which they attach themselves on sufference but, on the other hand, also without
having to observe any working hours, free to leave anytime they please, to stay
away for as long they like. It is not pretended that if found remiss in the observance of
said rules, any discipline may be meted them beyond barring them from the
premises which, it may be supposed, the Club may do in any case even absent any
breach of the rules, and without violating any right to work on their part. All these
considerations clash frontally with the concept of employment. The IAC would point to the
fact that the Club suggests the rate of fees payable by the players to the caddies as still
another indication of the latter's status as employees. It seems to the Court, however, that
the intendment of such fact is to the contrary, showing that the Club has not the
measure of control over the incidents of the caddies' work and compensation that
an employer would possess. In the final analysis, petitioner has no was of compelling
the presence of the caddies as they are not required to render a definite number of
hours of work on a single day. Even the group rotation of caddies is not absolute
because a player is at liberty to choose a caddy of his preference regardless of the
caddy's order in the rotation.
Obiter (on issue of res judicata)
- That same issue of res adjudicata, ignored by the IAC beyond bare mention thereof, as
already pointed out, is now among the mainways of the private respondent's defenses to
the petition for review.
- Because the same question of employer-employee relationship has been dragged into
three different fora, willy-nilly and in quick succession, it has birthed controversy as to
which of the resulting adjudications must now be recognized as decisive. On the one
hand, there is the certification case where the decision found for the existence of
employer-employee relationship between the parties; on the other, the compulsory
arbitration case which was dismissed for lack of merit on the ground that there existed no
such relationship between the Club and the private respondent.
- It is well settled that for res adjudicata, or the principle of bar by prior judgment, to apply,
the following essential requisites must concur: (1) there must be a final judgment or order;
(2) said judgment or order must be on the merits; (3) the court rendering the same must
have jurisdiction over the subject matter and the parties; and (4) there must be between
the two cases identity of parties, identity of subject matter and identity of cause of action.
- A certification proceedings is not a "litigation" in the sense in which the term is commonly
understood, but mere investigation of a non-adversary, fact-finding character, in which the
investigating agency plays the part of a disinterested investigator seeking merely to
ascertain the desires of the employees as to the matter of their representation.
- In any case, this Court is not inclined to allow private respondent the benefit of any doubt
as to which of the conflicting ruling just adverted to should be accorded primacy, given the
fact that it was he who actively sought them simultaneously, as it were, from separate fora.
Accordingly, the IAC is not to be faulted for ignoring private respondent's invocation of res
adjudicata; on contrary, it acted correctly in doing so.
Disposition Reversed and set aside, it being hereby declared that the private
respondent, Fermin Llamar, is not an employee of petitioner Manila Golf and Country Club
and that petitioner is under no obligation to report him for compulsory coverage to the
Social Security System.

Labor Law 1
FELIX V BUENASEDA
240 SCRA 139
KAPUNAN; January 17, 1995
NATURE
Petition for review on certiorari
FACTS
- Petitioner Dr. Alfredo Felix joined the National Center for Mental Health (NCMH) as a
resident physician and after only 3 years, was promoted to Senior Resident Physician, a
position he held until the Ministry of Health reorganized the NCMH in Jan. 1988, pursuant
to E.O. 119. Under the said reorganization, Felix was appointed to the position of Sr.
Resident Physician in a temporary capacity immediately after he and other employees
allegedly tendered their courtesy resignations to the Sec. of Health. Felix was later
promoted to the position of Medical Specialist 1 (Temporary Status) which was renewed
the following year.
- In 1988, the Dept. of Health or DoH issued Dept. Order (D.O.) 347, which required board
certification as a prerequisite for renewal of specialist positions in various med. centers,
hospitals and agencies and specifically provided that specialists working in various
branches of DoH be recognized as Fellows of their respective societies and/or
Diplomates of their specialty boards or both, the purpose of which was to upgrade the
quality of specialists in DoH hospitals by requiring them to pass rigorous theoretical and
clinical exams given by recognized specialty boards.
- (Then) Sec. of Health Alfredo Bengzon issued D.O. 478 (amending Sec.4 of D.O. 347)
which provided for an extension of appointments of Medical Specialists in cases where
termination of those who failed to meet the requirement for board certification might result
in disruption of hospital services. The said order provided, among others, that:
xxxxx 2. Medical specialists recommended for extension of appointment shall meet the
following minimum criteria:
a. DOH medical specialist certified
b. Has been in the service of the Department at least three (3) years prior to December
1988
c. Has applied or taken the specialty board examination.
- In 1991, after reviewing petitioners service record and performance, the Medical
Credentials Committee of the NCMH recommended non-renewal of his appointment as
Medical Specialist 1, informing him of its decision on Aug. 22, 1991. He was, however,
allowed to continue in the service, and receive his salary even after being informed of the
termination of his appointment.
- On Nov. 25, 1991, the Chiefs of Service held an emergency meeting to discuss the
petitioners case. In the meeting, the overall consensus among the dept. heads was for
petitioners non-renewal where his poor performance, frequent tardiness and inflexibility
were pointed as among the factors responsible for the recommendation not to renew his
appointment. The matter was referred to the CSC, which ruled that the temporary
appointment can be terminated any time and that any renewal of such appointment is
within the discretion of the appointing authority. Consequently, petitioner was advised by
hospital authorities to vacate his cottage. Refusing to comply, petitioner filed a petition with
the Merit System Protection Board (MSPB) complaining about the alleged harassment and
questioning the non-renewal of his appointment, the MSPB, however, dismissed his
complaint for lack of merit.
- This decision was appealed to the Civil Service Commission (CSC) which dismissed the
same. The MFR was also denied by the CSC hence this appeal.
Petitioners claims
1. CSC erred in holding that by submitting his courtesy resignation and accepting his
temporary appointment, petitioner had effectively divested himself of his security of tenure,
considering the circumstances of such courtesy resignation and acceptance of
appointment.
2. Respondent commission erred in not declaring that the conversion of the permanent
appointment of petitioner to temporary was done in bad faith in the guise of reorganization
and thus invalid, being violative of the petitioners right of security of tenure.
Respondent CSCs claims
1. The petitioners temporary appointments after the reorganization pursuant to E.O. 119
were valid and did not violate his constitutional right to security of tenure.
2. Petitioner is guilty of estoppel or laches, having acquiesced to such temporary
appointments from 1988-1991
3. The respondent CSC did not act with grave abuse of discretion in affirming the
petitioners non-renewal of his appointment to the NCMH.

A2010

- 55 -

Disini

ISSUE
WON petitioners removal from a permanent position (Medical Specialist 1), as a result of
the reorganization of the DoH, was void for being violative of the constitutional provision
on security of tenure
HELD
NO
Reasoning
- A residency or resident physician position in a medical specialty is never a permanent
one. Residency connotes training and temporary status. Promotion to the next postgraduate year is based on merit and performance determined by periodic evaluations and
examinations of knowledge, skills and bedside manner. Under this system, residents,
especially those in university teaching hospitals enjoy their right to security of tenure only
to the extent that they periodically make the grade. While physicians (or consultants) of
specialist rank are not subject to the same stringent evaluation procedures, specialty
societies require continuing education as a requirement for accreditation in good standing,
in addition to peer review processes based on performance, mortality and morbidity
audits, feedback from residents, interns and medical students and research output. The
nature of the contracts of resident physicians meets traditional tests for
determining employer employee relationships, but because the focus of residency
is training, they are neither here nor there. Moreover, stringent standards and
requirements for renewal of specialist rank positions or for promotion to the next
postgraduate residency year are necessary because lives are ultimately at stake.
- From the position of senior resident physician, which he held at the time of the
government reorganization, the next logical step in the stepladder process was obviously
his promotion to the rank of Medical Specialist 1, a position which he apparently accepted.
Such status, however, clearly carried with it certain professional responsibilities including
the responsibility of keeping up with the minimum requirements of specialty rank, the
responsibility of keeping abreast with current knowledge in his specialty and in Medicine in
general, and the responsibility of completing board certification requirements within a
reasonable period of time. The evaluation made by petitioner's peers and superiors clearly
showed that he was deficient in a lot of areas, in addition to the fact that at the time of his
non-renewal, he was not even board-certified.
- As respondent CSC has correctly pointed out, the appointment was for a definite and
renewable period which, when it was not renewed, did not involve a dismissal but an
expiration of the petitioners term.
*On estoppel by laches:
- Public policy and convenience demand that any claim to any position in the civil service,
permanent, temporary or otherwise, or any claim to a violation of the constitutional
provision on security of tenure be made within a reasonable period of time. The failure to
assert a claim or the voluntary acceptance of another position in government, obviously
without reservation, leads to a presumption that the civil servant has either given up his
claim or has already settled into the new position. This is the essence of laches which is
the failure or neglect, for an unreasonable and unexplained length of time to do that
which, by exercising due diligence, could or should have been done earlier; it is the
negligence or omission to assert a right within a reasonable time, warranting a
presumption that the party entitled to assert it either has abandoned it or declined to
assert it.
- Petitioner made no attempt to oppose earlier renewals of his temporary Specialist 1
contracts, clearly demonstrating his acquiescence to - if not his unqualified acceptance of the promotion (albeit of a temporary nature). Whatever objections petitioner had against
the earlier change from the status of permanent senior resident physician to temporary
senior resident physician were neither pursued nor mentioned at or after his designation
as Medical Specialist 1 (Temporary). He is therefore estopped from insisting upon a right
or claim which he had plainly abandoned when he, from all indications, enthusiastically
accepted the promotion. His negligence to assert his claim within a reasonable time,
coupled with his failure to repudiate his promotion to a temporary position, warrants a
presumption that he either abandoned (his claim) or declined to assert it.
Disposition Petition dismissed for lack of merit

R TRANSPORT CORP V EJANDRA


428 SCRA 724
CORONA; May 20, 2004
NATURE
Petition for review of the decision of the CFI of Iloilo
FACTS
- Rogelio Ejandra worked for petitioner bus company as a driver.
- On Jan 31 1996, he was apprehended for obstruction of traffic. His license was
confiscated. He reported this to his manager, Oscar Pasquin, who gave him P500 to
redeem the license. He was able to retrieve the license after a week since the
apprehending officer turned it in only then.

Labor Law 1

A2010

- 56 -

Disini

- On feb 8, 1996, he reported for work. The company said they were reviewing if they were
going to allow him drive again. Also, he was being blamed for damage to the bus. Ejandra
said the bus was damaged during the week he wasnt able to drive.
- Petitioner, on the other hand, claims that Ejandra is a habitual absentee and has
abandoned his job. To belie private respondents allegation that his license had been
confiscated, petitioner asserted that, had it been true, he should have presented an
apprehension report and informed petitioner of his problems with the LTO. But he did not.
Petitioner further argued that private respondent was not an employee because theirs was
a contract of lease and not of employment, with petitioner being paid on commission basis
- The labor arbiter ruled in favor of Ejandra. It was held that he didnt abandon his work,
since there was valid reason for his 1 week absence. He also was not afforded due
process. NLRC and CA affirmed.

respondents claim: Ministry had no jurisdiction over Basiao's claim, asserting that he was
not the Company's employee, but an independent contractor and that the Company had
no obligation to him for unpaid commissions under the terms and conditions of his
contract.
- The Labor Arbiter found for Basiao. He ruled that the underwriting agreement had
established an employer-employee relationship between him and the Company, and this
conferred jurisdiction on the Ministry of Labor to adjudicate his claim. Said official's
decision directed payment of his unpaid commissions ". . . equivalent to the balance of the
first year's premium remaining unpaid, at the time of his termination, of all the insurance
policies solicited by . . . (him) in favor of the respondent company . . ." plus 10% attorney's
fees.
- This decision was, on appeal by the Company, affirmed by the NLRC.

ISSUES
1. WON there was an employee employer relationship
2. WON Ejandra was dismissed for a just cause

ISSUE
WON Basiao had become the Company's employee by virtue of the contract invoked by
him, thereby placing his claim for unpaid commissions within the original and exclusive
jurisdiction of the Labor Arbiter under the provisions of Section 217 of the Labor Code

HELD
1. YES
- Petitioner is barred to negate the existence of an employer-employee relationship. He
has invoked rulings on the right of an employer to dismiss an employee for just cause. The
power to dismiss an employee is one of the indications that there was such relationship.
Also, A97 of the Labor Code says that employees can be paid in form of commissions.
2. NO
- To constitute abandonment, two elements must concur: (1) the failure to report for work
or absence without valid or justifiable reason and (2) a clear intention to sever the
employer-employee relationship. Petitioner did not fulfill the requisites. First, Ejandras
absence was justified since his license wasnt release until after a week. Second, Ejandra
did not want to sever their relationship when he got his license back. Third, labor arbiter
Yulo correctly observed that, if private respondent really abandoned his work, petitioner
should have reported such fact to the nearest Regional Office of the Department of Labor
and Employment in accordance with Section 7, Rule XXIII, Book V of Department Order
No. 9, series of 1997 (Rules Implementing Book V of the Labor Code). Petitioner made no
such report.
- In addition, he wasnt also given due process by not giving him notice and hearing.
Disposition Decision reversed

SONZA V ABS-CBN
[PAGE 42]
INSULAR LIFE V NLRC (BASIAO)
179 SCRA 459
NARVASA; November 15, 1989
NATURE
Petition for certiorari and prohibition to review the resolution of the NLRC.
FACTS
- In 1968, Insular Life Assurance Co., Ltd. (Company) and Melecio T. Basiao entered into
a contract by w/c Basiao was "authorized to solicit w/in the Phils applications for
insurance policies and annuities in accordance with the existing rules and regulations" of
the Company; he would receive "compensation, in the form of commissions . . . ", and
the "rules in Rate Book and its Agent's Manual, as well as all its circulars and those which
may from time to time be promulgated by it . . ." were made part of said contract.
- The contract also contained provisions governing the relations of the parties, the duties
of the Agent, the acts prohibited to him, and the modes of termination of the agreement,
viz.:
"RELATION WITH THE COMPANY. The Agent shall be free to exercise his own
judgment as to time, place and means of soliciting insurance. Nothing herein contained
shall therefore be construed to create the relationship of employee and employer
between the Agent and the Company. However, the Agent shall observe and conform
to all rules and regulations which the Company may from time to time prescribe.
"TERMINATION. The Company may terminate the contract at will, without any
previous notice to the Agent, for or on account of . . . (explicitly specified causes) . . .
- in April 1972, the parties entered into another contract - an Agency Manager's
Contract, while Basiao concurrently fulfilled his commitments under the first contract
with the Company.
- In May 1979, the Company terminated the Agency Manager's Contract. After vainly
seeking a reconsideration, Basiao sued the Company in a civil action and this (he claimed)
prompted the latter to terminate also his engagement under the first contract and to stop
payment of his commissions starting April 1, 1980.
- Basiao filed w/ the Ministry of Labor a complaint against the Company and its president.
The complaint sought to recover commissions allegedly unpaid, plus attorney's fees. The

HELD
NO
- Basiao was not an employee of the petitioner, but a commission agent, an independent
contractor whose claim for unpaid commissions should have been litigated in an ordinary
civil action. The Labor Arbiter erred in taking cognizance of, and adjudicating, said claim,
being without jurisdiction to do so, as did the respondent NLRC in affirming the Arbiter's
decision. This conclusion renders it unnecessary and premature to consider Basiao's
claim for commissions on its merits.
-Control test" (Viana vs. Alejo Al-Lagadan, 1956):
"In determining the existence of employer-employee relationship, the following
elements are generally considered, namely: (1) the selection and engagement of the
employee; (2) the payment of wages; (3) the power of dismissal; and (4) the power to
control the employees' conduct - although the latter is the most important element
(35 Am. Jur. 445). . . ,"
- However, not every form of control that the hiring party reserves to himself over the
conduct of the party hired in relation to the services rendered may be accorded the effect
of establishing an employer-employee relationship between them in the legal or technical
sense of the term.
- Logically, the line should be drawn between rules that merely serve as guidelines
towards the achievement of the mutually desired result without dictating the means or
methods to be employed in attaining it, and those that control or fix the methodology and
bind or restrict the party hired to the use of such means. The first, which aim only to
promote the result, create no employer-employee relationship unlike the second, which
address both the result and the means used to achieve it.
- Rules and regulations governing the conduct of the business are provided for in the
Insurance Code and enforced by the Insurance Commissioner. It is, therefore, usual and
expected for an insurance company to promulgate a set of rules to guide its commission
agents in selling its policies that they may not run afoul of the law and what it requires or
prohibits. Of such a character are the rules which prescribe the qualifications of persons
who may be insured, subject insurance applications to processing and approval by the
Company, and also reserve to the Company the determination of the premiums to be paid
and the schedules of payment. None of these really invades the agent's contractual
prerogative to adopt his own selling methods or to sell insurance at his own time and
convenience, hence cannot justifiably be said to establish an employer-employee
relationship between him and the company.
- Mafinco Trading Corporation v Ople: a person engaged to sell soft drinks for another,
using a truck supplied by the latter, but with the right to employ his own workers, sell
according to his own methods subject only to prearranged routes, observing no working
hours fixed by the other party and obliged to secure his own licenses and defray his own
selling expenses, all in consideration of a peddler's discount given by the other party for at
least 250 cases of soft drinks sold daily, was not an employee but an independent
contractor.
- Investment Planning Corporation of the Philippines v SSS: there was no employeremployee relationship between a commission agent and an investment company, but that
the former was an independent contractor where said agent and others similarly placed
were: (a) paid compensation in the form of commissions based on percentages of their
sales, any balance of commissions earned being payable to their legal representatives in
the event of death or registration; (b) required to put up performance bonds; (c) subject to
a set of rules and regulations governing the performance of their duties under the
agreement with the company and termination of their services for certain causes; (d) not
required to report for work at any time, nor to devote their time exclusively to working for
the company nor to submit a record of their activities, and who, finally, shouldered their
own selling and transportation expenses.
- Sara v NLRC: one who had been engaged by a rice miller to buy and sell rice and palay
without compensation except a certain percentage of what he was able to buy or sell, did
work at his own pleasure without any supervision or control on the part of his principal and

Labor Law 1
relied on his own resources in the performance of his work, was a plain commission agent,
an independent contractor and not an employee.

ALMIREZ V INFINITE LOOP TECHNOLOGY


CORPORATION
481 SCRA 364
CARPIO-MORALES; January 31, 2006
FACTS
- Almirez was hired as a Refinery Senior Process Design Engineer for a specific project by
respondent Infinite Loop Technology Corporation through its General Manager Rubino.
- September 30, 1999 Details were furnished to Almirez regarding her designation in the
company as well as the scope of her services. The scope of the services was to
commence on October 18, 1999 and had a guarantee of 12 continuous months.
> The senior process design engineer was to work together with the Process Design
Consultant in performing the scope of the services which included preparing the
process terms of reference or basis of design for the BPSD Petroleum Refinery, to
review and revise as necessary the existing conceptual process block diagram of the
process flow scheme, implement new process technologies, participate in discussions,
make recommendation reports to the company management team, represent the
company in meetings and perform other related works.
> She was to be paid a professional fee of US$2,000 per month (net of tax) to be 50/50
split in US dollars or equivalent peso every 15 th of the month. She also had other
benefits and bonuses along with equipment such as a laptop computer.
- Almirez received a total of P77,000 the following amounts on the dates indicated:
- 11-23-09 P20,000 (Salary for Nov. 1-15)
12-02-99 P8,000 (Salary for Nov. 15-30)
12-15-99 P2,000 (Full payment for Nov. 15-30 salary)
P10,000 (Salary for Dec. 1-15,)
1-17-00 P12,000 (Salary for January 1-15)
1-16-00 P12,500 (Salary for January 16-31)
1-20-00 P12,500 (Salary for January 1-15)
- Almirez then wrote a letter to the company, expressing her disappointment because she
was receiving less than expected. She hdadexpected the amount to be net of taxes but
she was receiving less because of SSS deductions and tax deductions. She asked that
her SSS dues be not deducted from her salary because she was voluntarily paying such
obligations on her own. She further stated that she was willing to render her services at
Infinite Loop based on the contract and that she was willing to serve as technical
consultant on other relevant projects.
- Rabino said that Almirezs letter was different from what they had previously agreed
upon. According to him, the BPSD project, like any other project, could be deferred and
that since the engineering design for the proposed project was not yet available, it would
be prudent to suspend the professional services of Almirez as Senior Process Design
Engineer effective February 2007.
- Almirez, through counsel, wrote a letter to Rabino, asking that she be properly
compensated with the total amount of her contract because the contract stated that her
tenure would last for 12 months but she had already been suspended by February of
2000. Almirez also noted that she had been paid only P74,229.17 which is way below the
amount promised to her of US$2,000 a month net of tax.
- Rabino responded by explaining to Almirez that the company and its partner corporations
were all experiencing financial difficulties with their projects and asked her to bear with
them.
- December 12, 2000 Almirez filed a complaint against Infinite Loop for breach of
contract of employment. Infinite Loop countered by saying that the NLRC had no
jurisdiction to hear the case because there was no employer-employee relationship and
the contract was one of services, not employment.
- The Labor Arbiter ruled that there was an existence of an employer-employee
relationship and ordered Infinite Loop to pay Almirez US$24,000 in its peso equivalent less
advances of P77,000. Infinite Loop appealed to the NLRC but the appeal was dismissed.
- The Court of Appeals found that the primary cause of Almirezs action was that for a sum
of money on account of an alleged breach of contract to pay a professional fee. It held
that there was no employer-employee relationship so the NLRC and the Labor Arbiter
have no jurisdiction over the said case. Thus Almirezs petition was dismissed.
ISSUE
WON there was an existence of an employer-employee relationship
HELD
1. NO
Ratio Under the control test, an employer-employee relationship exists where the person
for whom the services are performed reserves the right to control not only the end
achieved but also the manner and means to be used in reaching the end.
Reasoning

A2010

- 57 -

Disini

- The Court has consistently held a four tier test to evaluate the existence of an employeremployee relationship which include: 1) manner of selection of engagement, 2) payment
of wages, 3) presence or absence of power of dismissal and 4) presence or absence of
power of control.
- The last test is known as the control test and is regarded as the most crucial and
determinative indicator of the presence of absence of an employer-employee relationship.
- There is no showing of a controlling power of Infinite Loop over Almirez. They only
specified what she needed to achieve but now how she was go on about it.
- The company had hired her based on her expertise but the company naturally had to
appraised of the work progress.
- The deduction for SSS and tax do not bolster Almirezs contention that there was an
employee-employer relationship. However, only one pay slip was issued (Januaryb 16-31,
2000) and the rest were in cash vouchers. As such, the payslip cannot be considered as
proof of an employer-employee relationship.
- The use of the word salary is not determinative of such a relationship either. Salary is
defined as remuneration for services given. The contract details her salary and it serves
between the parties was the law governing them. But the contract, as pointed out earlier,
is bereft of proof of control of Infinite Loop over Almirez.
Disposition Petition is denied for lack of merit with costs against petitioner.

SEVILLA V CA (TOURIST WORLD SERVICES)


160 SCRA 171
SARMIENTO; April 15, 1998
NATURE
Appeal by certiorari
FACTS
- On the strength of a contract, Tourist World Service Inc. (TWS) leased the premises
belonging to Mrs. Segundina Noguera for the formers use as a branch office. Lina Sevilla
bound herself solidarily liable with TWS for the prompt payment of the monthly rentals
thereon.
- When the branch office was opened, it was run by appellant Sevilla payable to TWS by
any airline for any fare brought in on the efforts of Sevilla, 4% was to go to Sevilla and 3%
was to be withheld by TWS.
- TWS appears to have been informed that Sevilla was connected with a rival firm, the
Philippine Travel Bureau, and, since the branch office was anyhow losing, the TWS
considered closing down its office.
- This was firmed up by two resolutions of the TWS board of directors to abolish the office
of the manager and VP of the branch office and authorizing the corporate secretary to
receive the properties in the said branch office.
- The corporate secretary went to the branch office, and finding the premises locked and
being unable to contact Sevilla, padlocked the premises to protect the interests of TWS.
- When neither Sevilla nor her employees could enter the locked premises, she filed a
complaint against TWS with a prayer for the issuance of a mandatory preliminary
injunction.
- The trial court dismissed the case holding that TWS, being the true lessee, was within its
prerogative to terminate the lease and padlock the premises. It likewise found that Sevilla
was a mere employee of TWS and as such, was bound by the acts of her employer.
- The CA affirmed. Hence this petition.
ISSUE
1. WON there was an employer-employee relation between TWS and Sevilla
2. WON the padlocking of the premises by TWS without the knowledge and consent of
Sevilla entitled the latter to the relief of damages prayed for
HELD
1. NO. It was a principal-agent relationship.
Ratio In this jurisdiction, there has been no uniform test to determine the existence of an
employer-employee relation. In general, We have relied on the so-called right of control
test, where the person for whom the services are performed reserves a right to control
not only the end to be achieved but also the means to be used in reaching such end. In
addition, the existing economic conditions prevailing between the parties, like the
inclusion of the employee in the payrolls, are also considered in determining the existence
of an employer-employee relationship.
Reasoning
- Sevilla was not subject to control by TWS either as to the result of the enterprise or as to
the means used in connection therewith.
- Under the contract of lease, Sevilla bound herself in solidum for the rental payments; an
arrangement that would belie the claims of a master-servant relationship for a true
employee cannot be made to part with his own money in pursuance of his employers
business, or otherwise assume liability thereof.

Labor Law 1
- Sevilla was not in the companys payroll. She retained 4% in commissions from airline
bookings, the remaining 3% going to TWS. Unlike an employee who usually earns a fixed
salary, she earned compensation in fluctuating amounts depending on her booking
successes.
- The fact that Sevilla has been designated branch manager does not make her, ergo,
TWS employee. Employment is determined by the right of control test and certain
economic parameters. Titles are weak indicators.
- When Sevilla agreed to man TWS Ermita branch office, she did so pursuant to a
contract of agency. It is the essence of this contract that the agent renders services in
representation or on behalf of another. In the case at bar, Sevilla solicited airline fares,
but she did so for and on behalf of her principal, TWS.
2. YES
Ratio For its unwarranted revocation of the contact of agency, TWS should be sentenced
to pay damages.
Reasoning
- Sevilla had acquired a personal stake in the business itself, and necessarily, in the
equipment pertaining thereto.
- Sevilla was not a stranger to that contract of lease having been explicitly named therein
as third party in charge of rental payments. She could not be ousted from possession
summarily as one would eject an interloper.
- The Court is satisfied with the chronicle of events, there was indeed some malevolent
design to put the petitioner Sevilla in a bad light following the disclosures that she had
worked for a rival firm.
Disposition REVERSED.

INSULAR LIFE ASSURANCE CO LTD V NLRC (DELOS


REYES)
287 SCRA 476
BELLOSILLO; March 12, 1998
NATURE
Petition for review on certiorari of the decision of the NLRC
FACTS
- On 21 August 1992 petitioner entered into an agency contract with respondent Pantaleon
de los Reyes authorizing the latter to solicit within the Philippines applications for life
insurance and annuities for which he would be paid compensation in the form of
commissions. The contract was prepared by petitioner in its entirety and De los Reyes
merely signed his conformity thereto. It contained the stipulation that no employeremployee relationship shall be created between the parties and that the agent shall be
free to exercise his own judgment as to time, place and means of soliciting insurance. De
los Reyes however was prohibited by petitioner from working for any other life insurance
company, and violation of this stipulation was sufficient ground for termination of the
contract. Aside from soliciting insurance for the petitioner, private respondent was required
to submit to the former all completed applications for insurance within ninety (90)
consecutive days, deliver policies, receive and collect initial premiums and balances of
first year premiums, renewal premiums, deposits on applications and payments on
policy loans. Private respondent was also bound to turn over to the company immediately
any and all sums of money collected by him.
- On 1 March 1993 petitioner and private respondent entered into another contract where
the latter was appointed as Acting Unit Manager under its office atthe Cebu DSO V. As
such, the duties and responsibilities of De los Reyes included the recruitment, training,
organization and development within his designated territory of a sufficient number of
qualified, competent and trustworthy underwriters, and to supervise and coordinate the
sales efforts of the underwriters in the active solicitation of new business and in the
furtherance of the agency's assigned goals. It was similarly provided in the management
contract that the relation of the acting unit manager and/or the agents of his unit to the
company shall be that of independent contractor. If the appointment was terminated for
any reason other than for cause, the acting unit manager would be reverted to agent
status and assigned to any unit. As in the previous agency contract, De los Reyes together
with his unit force was granted freedom to exercise judgment as to time, place and means
of soliciting insurance. Aside from being granted override commissions, the acting unit
manager was given production bonus, development allowance and a unit development
financing scheme euphemistically termed "financial assistance" consisting of payment to
him of a free portion of P300.00 per month and a validate portion of P1,200.00. While the
latter amount was deemed as an advance against expected commissions, the former was
not and would be freely given to the unit manager by the company only upon fulfillment by
him of certain manpower and premium quota requirements. The agents and underwriters
recruited and trained by the acting unit manager would be attached to the unit but
petitioner reserved the right to determine if such assignment would be made or, for any
reason, to reassign them elsewhere. Aside from soliciting insurance, De los Reyes was
also expressly obliged to participate in the company's conservation program, i.e.,
preservation and maintenance of existing insurance policies, and to accept moneys duly

A2010

- 58 -

Disini

receipted on agent's receipts provided the same were turned over to the company. As long
as he was unit manager in an acting capacity, De los Reyes was prohibited from working
for other life insurance companies or with the government. He could not also accept a
managerial or supervisory position in any firm doing business in the Philippines without the
written consent of petitioner.
- Private respondent worked concurrently as agent and Acting Unit Manager until he was
notified by petitioner on 18 November 1993 that his services were terminated effective 18
December 1993. He filed a complaint before the Labor Arbiter on the ground that he was
illegally dismissed and that he was not paid his salaries and separation pay.
ISSUE
WON there exist an employer-employee relationship between petitioner and respondent
HELD
YES
- It is axiomatic that the existence of an employer-employee relationship cannot be
negated by expressly repudiating it in the management contract and providing therein that
the "employee" is an independent contractor when the terms of the agreement clearly
show otherwise. For, the employment status of a person is defined and prescribed by law
and not by what the parties say it should be. In determining the status of the management
contract, the "four-fold test" on employment earlier mentioned has to be applied.
(a) selection and engagement of employee
> Petitioner contends that De los Reyes was ever required to go through the preemployment procedures and that the probationary employment status was reserved only
to employees of petitioner. On this score, it insists that the first requirement of selection
and engagement of the employee was not met. A look at the provisions of the contract
shows that private respondent was appointed as Acting Unit Manager only upon
recommendation of the District Manager. This indicates that private respondent was hired
by petitioner because of the favorable endorsement of its duly authorized officer. But, this
approbation could only have been based on the performance of De los Reyes as agent
under the agency contract so that there can be no other conclusion arrived under this
premise than the fact that the agency or underwriter phase of the relationship of De los
Reyes with petitioner was nothing more than a trial or probationary period for his eventual
appointment as Acting Unit Manager of petitioner. Then, again, the very designation of the
appointment of private respondent as "acting" unit manager obviously implies a temporary
employment status which may be made permanent only upon compliance with company
standards such as those enumerated under the management contract.
(b) payment of wages
> Petitioner points out that respondent was compensated strictly on commission basis, the
amount of which was totally dependent on his total output. But, the manager's contract,
speaks differently. It unquestionably demonstrate that the performance requirement
imposed on De los Reyes was applicable quarterly while his entitlement to the free portion
(P300) and the validated portion (P1,200) was monthly starting on the first month of the
twelve (12) months of the appointment. Thus, it has to be admitted that even before the
end of the first quarter and prior to the so-called quarterly performance evaluation, private
respondent was already entitled to be paid both the free and validated portions of the UDF
every month because his production performance could not be determined until after the
lapse of the quarter involved. This indicates quite clearly that the unit manager's quarterly
performance had no bearing at all on his entitlement at least to the free portion of the UDF
which for all intents and purposes comprised the salary regularly paid to him by petitioner.
Thus it cannot be validly claimed that the financial assistance consisting of the free portion
of the UDF was purely dependent on the premium production of the agent. Be that as it
may, it is worth considering that the payment of compensation by way of commission does
not militate against the conclusion that private respondent was an employee of petitioner.
Under Art. 97 of the Labor Code, "wage" shall mean "however designated, capable of
being expressed in terms of money, whether fixed or ascertained on a time, task, price or
commission basis . . . .".
(c) power of dismissal and power of control
> petitioner asserts that its termination of De los Reyes was but an exercise of its inherent
right as principal under the contracts and that the rules and guidelines it set forth in the
contract cannot, by any stretch of the imagination, be deemed as an exercise of control
over the private respondent as these were merely directives that fixed the desired result
without dictating the means or method to be employed in attaining it. The management
contract, however, prescribes reveals that the company practically dictates the manner by
which their jobs are to be carried out particularly exclusivity of service, control of
assignments and removal of agents under private respondent's unit, collection of
premiums, furnishing of company facilities and materials as well as capital described as
Unit Development Fund.
- These are but hallmarks of the management system in which herein private respondent
worked. This obtaining, there is no escaping the conclusion that private respondent
Pantaleon de los Reyes was an employee of herein petitioner.
Disposition Petition denied.

CHAVEZ V NLRC (SUPREME PACKAGING INC, LEE)

Labor Law 1
448 SCRA 478
CALLEJO, SR; January 17, 2005
NATURE
Petition for review on certiorari of the Resolution [1] dated December 15, 2000 of the Court
of Appeals (CA) reversing its Decision dated April 28, 2000 finding private respondents
guilty of illegal dismissal.
FACTS
- The respondent company, Supreme Packaging, Inc. engaged the services of the
petitioner, Pedro Chavez, as truck driver on October 25, 1984. The respondent company
furnished the petitioner with a truck.
- Sometime in 1992, the petitioner expressed to respondent Alvin Lee, respondent
companys plant manager, his (the petitioners) desire to avail himself of the benefits that
the regular employees were receiving such as overtime pay, nightshift differential pay, and
13th month pay, among others. Although he promised to extend these benefits to the
petitioner, respondent Lee failed to actually do so.
- On February 20, 1995, the petitioner filed a complaint for regularization with the Regional
Arbitration Branch No. III of the NLRC in San Fernando, Pampanga. Before the case
could be heard, respondent company terminated the services of the petitioner.
Consequently, on May 25, 1995, the petitioner filed an amended complaint against the
respondents for illegal dismissal, unfair labor practice and non-payment of overtime pay,
nightshift differential pay, 13th month pay, among others. The case was docketed as
NLRC Case No. RAB-III-02-6181-95.
- The respondents, for their part, denied the existence of an employer-employee
relationship between the respondent company and the petitioner. They averred that the
petitioner was an independent contractor as evidenced by the contract of service which he
and the respondent company entered into. The relationship of the respondent company
and the petitioner was allegedly governed by this contract of service.
- The respondents insisted that the petitioner had the sole control over the means and
methods by which his work was accomplished. He paid the wages of his helpers and
exercised control over them. As such, the petitioner was not entitled to regularization
because he was not an employee of the respondent company. The respondents, likewise,
maintained that they did not dismiss the petitioner. Rather, the severance of his
contractual relation with the respondent company was due to his violation of the terms and
conditions of their contract.
ISSUE
WON there existed an employer-employee relationship between the respondent company
and the petitioner.
HELD
YES
- The elements to determine the existence of an employment relationship are: (1) the
selection and engagement of the employee; (2) the payment of wages; (3) the power of
dismissal; and (4) the employers power to control the employees conduct. [11] The most
important element is the employers control of the employees conduct, not only as to the
result of the work to be done, but also as to the means and methods to accomplish it. [12]
All the four elements are present in this case.
- Of the four elements of the employer-employee relationship, the control test is the most
important. Although the respondents denied that they exercised control over the manner
and methods by which the petitioner accomplished his work, a careful review of the
records shows that the latter performed his work as truck driver under the respondents
supervision and control. Their right of control was manifested by the following attendant
circumstances:
1.
The truck driven by the petitioner belonged to respondent company;
2.
There was an express instruction from the respondents that the truck shall be
used exclusively to deliver respondent companys goods; [19]
3.
Respondents directed the petitioner, after completion of each delivery, to park
the truck in either of two specific places only, to wit: at its office in Metro Manila at 2320
Osmea Street, Makati City or at BEPZ, Mariveles, Bataan;[20] and
4.
Respondents determined how, where and when the petitioner would perform his
task by issuing to him gate passes and routing slips. [21]
- These circumstances, to the Courts mind, prove that the respondents exercised control
over the means and methods by which the petitioner accomplished his work as truck
driver of the respondent company.
- The contract of service indubitably established the existence of an employer-employee
relationship between the respondent company and the petitioner. It bears stressing that
the existence of an employer-employee relationship cannot be negated by
expressly repudiating it in a contract and providing therein that the employee is an
independent contractor when, as in this case, the facts clearly show otherwise.
Indeed, the employment status of a person is defined and prescribed by law and not
by what the parties say it should be.[22]

A2010

- 59 -

Disini

- The employer-employee relationship being established, the Court rules that private
respondent is guilty of illegal dismissal.

SAN MIGUEL CORP V ABELLA


461 SCRA 392
CARPIO-MORALES; June 28 2005
NATURE
Special Civil Action in the Supreme Court. Certiorari
FACTS
- Petitioner San Miguel Corporation (SMC) and Sunflower Multi-Purpose Cooperative
(Sunflower), entered into a one-year Contract of Services commencing on January 1,
1993, to be renewed on a month to month basis until terminated by either party. The
pertinent provisions of the contract are:
1. The cooperative agrees and undertakes to perform and/or provide for the company,
on a non-exclusive basis for a period of one year the following services for the Bacolod
Shrimp Processing Plant:
A. Messengerial/Janitorial
B. Shrimp Harvesting/Receiving
C. Sanitation/Washing/Cold Storage
4. There is no employer-employee relationship between the company and the
cooperative, or the cooperative and any of its members, or the company and any
members of the cooperative. The cooperative is an association of self-employed
members, an independent contractor, and an entrepreneur. It is subject to the control
and direction of the company only as to the result to be accomplished by the work or
services herein specified, and not as to the work herein contracted. The cooperative
and its members recognize that it is taking a business risk in accepting a fixed service
fee to provide the services contracted for and its realization of profit or loss from its
undertaking, in relation to all its other undertakings, will depend on how efficiently it
deploys and fields its members and how they perform the work and manage its
operations.
- Pursuant to the contract, Sunflower engaged private respondents to, as they did, render
services at SMCs Bacolod Shrimp Processing Plant at Sta. Fe, Bacolod City. The
contract was deemed renewed by the parties every month after its expiration on January
1, 1994 and private respondents continued to perform their tasks until September 11,
1995. In July 1995, private respondents filed a complaint before the NLRC, Regional
Arbitration Branch No. VI, Bacolod City, praying to be declared as regular employees of
SMC, with claims for recovery of all benefits and privileges enjoyed by SMC rank and file
employees. Private respondents subsequently filed on September 25, 1995 an Amended
Complaint to include illegal dismissal as additional cause of action following SMCs
closure of its Bacolod Shrimp Processing Plant on September 15, 1995which resulted in
the termination of their services. SMC filed a Motion for Leave to File Attached Third Party
Complaint dated November 27, 1995 to implead Sunflower as Third Party Defendant
which was, by Order of December 11, 1995, granted by Labor Arbiter Ray Alan T. Drilon. In
the meantime, on September 30, 1996, SMC filed before the Regional Office at Iloilo City
of the Department of Labor and Employment (DOLE) a Notice of Closure of its
aquaculture operations effective on even date, citing serious business losses. By Decision
of September 23, 1997, Labor Arbiter Drilon dismissed private respondents complaint for
lack of merit.
- Private respondents appealed to the NLRC. By Decision of December 29, 1998, the
NLRC dismissed the appeal for lack of merit, it finding that third party respondent
Sunflower was an independent contractor in light of its observation that [i]n all the
activities of private respondents, they were under the actual direction, control and
supervision of third party respondent Sunflower, as well as the payment of wages, and
power of dismissal. By Decision of February 7, 2001, the appellate court reversed the
NLRC decision and accordingly found for private respondents. Justifying its reversal of the
findings of the labor arbiter and the NLRC, the appellate court reasoned:Although the
terms of the non-exclusive contract of service between SMC and [Sunflower] showed a
clear intent to abstain from establishing an employer-employee relationship between SMC
and [Sunflower] or the latters members, the extent to which the parties successfully
realized this intent in the light of the applicable law is the controlling factor in determining
the real and actual relationship between or among the parties.There being a finding of
labor-only contracting, liability must be shouldered either by SMC or [Sunflower] or
shared by both (See Tabas vs. California Manufacturing, Inc., supra, p. 502). SMC
however should be held solely liable for [Sunflower] became non-existent with the closure
of the aquaculture business of SMC.
ISSUE
1. WON the respondents are employees of SMC
2. WON the retrenchment was valid and consequently, whether the respondents are
entitled to relief

Labor Law 1

A2010

HELD
1. YES
- Since private respondents who were engaged in shrimp processing performed tasks
usually necessary or desirable in the aquaculture business of SMC, they should be
deemed regular employees of the latter and as such are entitled to all the benefits and
rights appurtenant to regular employment. They should thus be awarded differential pay
corresponding to the difference between the wages and benefits given them and those
accorded SMCs other regular employees. Respecting the private respondents who were
tasked with janitorial and messengerial duties, this Court quotes with approval the
appellate courts ruling thereon:
- Those performing janitorial and messengerial services however acquired regular status
only after rendering one-year service pursuant to Article 280 of the Labor Code. Although
janitorial and messengerial services are considered directly related to the aquaculture
business of SMC, they are deemed unnecessary in the conduct of its principal business;
hence, the distinctionThe law of course provides for two kinds of regular employees,
namely: (1) those who are engaged to perform activities which are usually necessary or
desirable in the usual business or trade of the employer; and (2) those who have rendered
at least one year of service, whether continuous or broken, with respect to the activity in
which they are employed.
- The test to determine the existence of independent contractorship is whether one
claiming to be an independent contractor has contracted to do the work according to his
own methods and without being subject to the control of the employer, except only as to
the results of the work. As for those of private respondents who were engaged in janitorial
and messengerial tasks, they fall under the second category and are thus entitled to
differential pay and benefits extended to other SMC regular employees from the day
immediately following their first year of service.
- In legitimate labor contracting, the law creates an employer-employee relationship for a
limited purpose, i.e., to ensure that the employees are paid their wages. The principal
employer becomes jointly and severally liable with the job contractor, only for the payment
of the employees wages whenever the contractor fails to pay the same. Other than that,
the principal employer is not responsible for any claim made by the employees. [50]
- In labor-only contracting, the statute creates an employer-employee relationship for a
comprehensive purpose: to prevent a circumvention of labor laws. The contractor is
considered merely an agent of the principal employer and the latter is responsible to the
employees of the labor-only contractor as if such employees had been directly employed
by the principal employer.[51]
- The Contract of Services between SMC and Sunflower shows that the parties clearly
disavowed the existence of an employer-employee relationship between SMC and private
respondents. The language of a contract is not, however, determinative of the parties
relationship; rather it is the totality of the facts and surrounding circumstances of the case.
[52]
A party cannot dictate, by the mere expedient of a unilateral declaration in a contract,
the character of its business, i.e., whether as labor-only contractor or job contractor, it
being crucial that its character be measured in terms of and determined by the criteria set
by statute
2. SMC has thus proven substantial business reverses justifying retrenchment of its
employees.
- In the case at bar, company losses were duly established by financial documents audited
by Joaquin Cunanan & Co. showing that the aquaculture operations of SMCs
Agribusiness Division accumulated losses amounting to P145,848,172.00 in 1992
resulting in the closure of its Calatrava Aquaculture Center in Negros Occidental,
P11,393,071.00 in 1993 and P80,325,608.00 in 1994 which led to the closure of its San
Fernando Shrimp Processing Plant in Pampanga and the Bacolod Shrimp Processing
Plant in 1995. For termination due to retrenchment to be valid, however, the law requires
that written notices of the intended retrenchment be served by the employer on the worker
and on the DOLE at least one (1) month before the actual date of the retrenchment in
order to give employees some time to prepare for the eventual loss of their jobs, as well as
to give DOLE the opportunity to ascertain the verity of the alleged cause of termination.
Private respondents, however, were merely verbally informed on September 10,
1995 by SMC Prawn Manager Ponciano Capay that effective the following day or on
September 11, 1995, they were no longer to report for work as SMC would be
closing its operations. Where the dismissal is based on an authorized cause under
Article 283 of the Labor Code but the employer failed to comply with the notice
requirement, the sanction should be stiff as the dismissal process was initiated by
the employers exercise of his management prerogative, as opposed to a dismissal
based on a just cause under Article 282 with the same procedural infirmity where
the sanction to be imposed upon the employer should be tempered as the dismissal
process was, in effect, initiated by an act imputable to the employee. In light of the
factual circumstances of the case at bar, the Court awards P50,000.00 to each private
respondent as nominal damages.The grant of separation pay as an incidence of
termination of employment due to retrenchment to prevent losses is a statutory obligation
on the part of the employer and a demandable right on the part of the employee. Private
respondents should thus be awarded separation pay equivalent to at least one (1) month
pay or to at least one-half month pay for every year of service, whichever is higher, as
mandated by Article 283 of the Labor Code or the separation pay awarded by SMC to
other regular SMC employees that were terminated as a result of the retrenchment,

- 60 -

Disini

depending on which is most beneficial to private respondents.Considering that private


respondents were not illegally dismissed, however, no backwages need be awarded. It is
well settled that backwages may be granted only when there is a finding of illegal
dismissal.[80] The appellate court thus erred in awarding backwages to private
respondents. What was involved in that case was one of illegal dismissal

LOPEZ V METROPOLITAN WATERWORKS AND


SEWERAGE SYSTEM
462 SCRA 428
TINGA; June 30, 2005
NATURE
Petition for the review of the decision of the CA
FACTS
- By virtue of an Agreement, petitioners were engaged by the MWSS as collectorscontractors, wherein the former agreed to collect from the concessionaires of MWSS,
charges, fees, assessments of rents for water, sewer and/or plumbing services which the
MWSS bills from time to time.
- In 1997, MWSS entered into a Concession Agreement with Manila Water Service, Inc.
and Benpress-Lyonnaise, wherein the collection of bills was transferred to said private
concessionaires, effectively terminating the contracts of service between petitioners and
MWSS.
- Regular employees of the MWSS were paid their retirement benefits, but not petitioners.
Instead, they were refused said benefits, MWSS relying on a resolution of the CSC that
contract-collectors of the MWSS are not its employees and therefore not entitled to the
benefits due regular government employees.
- Petitioners filed a complaint with the CSC which denied their claims, stating that
petitioners were engaged by MWSS through a contract of service, which explicitly
provides that a bill collector-contractor is not an MWSS employee. Relying on Part V of
CSC Memorandum Circular No. 38, Series of 1993, the CSC stated that contract
services/job orders are not considered government services, which do not have to be
submitted to the CSC for approval, unlike contractual and plantilla appointments.
Moreover, it found that petitioners were unable to show that they have contractual
appointments duly attested by the CSC. In addition, the CSC stated that petitioners, not
being permanent employees of MWSS and not included in the list submitted to the
concessionaire, are not entitled to severance pay. Petitioners claims for retirement
benefits and terminal leave pay were likewise denied.
- Petitioners sought reconsideration of the CSC Resolution, which was however denied
- Petitioners filed a petition for review with the Court of Appeals which affirmed the ruling of
the CSC.
ISSUE
WON petitioners were employees of the MWSS and, consequently, entitled to the benefits
they claim
HELD
YES
- The Court has invariably affirmed that it will not hesitate to tilt the scales of justice to the
labor class for no less than the Constitution dictates that the State . . . shall protect the
rights of workers and promote their welfare. It is committed to this policy and has always
been quick to rise to defense in the rights of labor, as in this case.
- Protection to labor, it has been said, extends to all of laborlocal and overseas,
organized and unorganized, in the public and private sectors. [52] Besides, there is no
reason not to apply this principle in favor of workers in the government. The government,
including government-owned and controlled corporations, as employers, should set the
example in upholding the rights and interests of the working class.
- For purposes of determining the existence of employer-employee relationship, the Court
has consistently adhered to the four-fold test, namely: (1) whether the alleged employer
has the power of selection and engagement of an employee; (2) whether he has control of
the employee with respect to the means and methods by which work is to be
accomplished; (3) whether he has the power to dismiss; and (4) whether the employee
was paid wages. Of the four, the control test is the most important element.
- A review of the circumstances surrounding the case reveals that petitioners are
employees of MWSS. MWSS wielded its power of selection when it contracted with the
individual petitioners, undertaking separate contracts or agreements. The same goes true
for the power to dismiss. Although termed as causes for termination of the Agreement, a
review of the same shows that the grounds indicated therein can similarly be grounds for
termination of employment.
- On the issue of remuneration, MWSS claims that the compensation received by
petitioners does not fall under the definition of wages as provided in Section 2(i) of P.D.
1146. This assertion, however, simply begs the question. The provision is a simple
statement of meaning, operating on the a priori premise or presumption that the recipient

Labor Law 1
is already classified as an employee, and does not lay down any basis or standard for
determining who are employees and who are not.
- On the other hand, relevant and appropriate is the definition of wages in the Labor Code,
namely, that it is the remuneration, however designated, for work done or to be done, or
for services rendered or to be rendered. The commissions due petitioners were based on
the bills collected as per the schedule indicated in the Agreement. Significantly, MWSS
granted petitioners benefits usually given to employees, to wit: COLA, meal, emergency,
and traveling allowances, hazard pay, cash gift, and other bonuses. Petitioners rendered
services to MWSS for which they were paid and given similar benefits due the other
employees of MWSS.
- Now the aspect of control. MWSS makes an issue out of the proviso in the Agreement
that specifically denies the existence of employer-employee relationship between it and
petitioners. It is axiomatic that the existence of an employer-employee relationship cannot
be negated by expressly repudiating it in an agreement and providing therein that the
employee is not an MWSS employee when the terms of the agreement and the
surrounding circumstances show otherwise. The employment status of a person is defined
and prescribed by law and not by what the parties say it should be.
- In addition, the control test merely calls for the existence of the right to control, and not
the exercise thereof. It is not essential for the employer to actually supervise the
performance of duties of the employee, it is enough that the former has a right to wield the
power.
- Other manifestations of control are evident from the records. The power to transfer or
reassign employees is a management prerogative exclusively enjoyed by employers. In
this case, MWSS had free reign over the transfer of bill collectors from one branch to
another. MWSS also monitored the performance of the petitioners and determined their
efficiency ratings.
Disposition Petition was GRANTED IN PART. The Decision of the Court of Appeals in
C.A.G.R. SP No. 55263, as well as the Civil Service Commissions Resolutions Nos.
991384 and 992074, were REVERSED and SET ASIDE. MWSS is ordered to pay
terminal leave pay and separation pay and/or severance pay to each of herein petitioners
on the basis of remunerations/commissions, allowances and bonuses each were actually
receiving at the time of termination of their employment as contract collectors of MWSS.
The case was remanded to the Civil Service Commission for the computation of the above
awards and the appropriate disposition in accordance with the pronouncements in this
Decision.

LAZARO V SSS (LAUDATO)


435 SCRA 472
TINGA; July 30, 2004
NATURE
Petition for Review under ROC Rule 45, assailing the CA Decision, which affirmed two
rulings of the Social Security Commission (SSC)
FACTS
- Private respondent Rosalina M. Laudato filed a petition before the SSC for social security
coverage and remittance of unpaid monthly social security contributions against her three
employers. Among the respondents was herein petitioner Angelito L. Lazaro, proprietor of
Royal Star Marketing, which is engaged in the business of selling home appliances.
Laudato alleged that despite her employment as sales supervisor of the sales agents for
Royal Star from April of 1979 to March of 1986, Lazaro had failed during the said period,
to report her to the SSC for compulsory coverage or remit Laudato's social security
contributions.
- Lazaro denied that Laudato was a sales supervisor of Royal Star, averring instead that
she was a mere sales agent whom he paid purely on commission basis. Lazaro also
maintained that Laudato was not subjected to definite hours and conditions of work. As
such, Laudato could not be deemed an employee of Royal Star.
- SSC ruled in favor of Laudato. Applying the "control test," it held that Laudato was an
employee of Royal Star, and ordered Royal Star to pay the unremitted social security
contributions of Laudato in the amount of P5,007.35, together with the penalties totaling
P22,218.54. In addition, Royal Star was made liable to pay damages to the SSC in the
amount of P15,680.07 for not reporting Laudato for social security coverage, pursuant to
Section 24 of the Social Security Law. Lazaro's MR was denied, prompting him to file a
petition for review with the CA. However, the CA affirmed the finding that Laudato was an
employee of Royal Star, and hence entitled to coverage under the Social Security Law.
- Lazaro's Argument: that Laudato was not qualified for social security coverage, as she
was not an employee of Royal Star, her income dependent on a generation of sales and
based on commissions; that Royal Star had no control over Laudato's activities, and that
under the so-called "control test," Laudato could not be deemed an employee.
ISSUE
WON Laudato is an employee of Royal Star

A2010

- 61 -

Disini

HELD
YES
Doctrine For the purposes of coverage under the Social Security Act, the determination of
employer-employee relationship warrants the application of the " control test," that is,
whether the employer controls or has reserved the right to control the employee, not only
as to the result of the work done, but also as to the means and methods by which the
same is accomplished.
- The fact that Laudato was paid by way of commission does not preclude the
establishment of an employer-employee relationship. In Grepalife v. Judico, the Court
upheld the existence of an employer-employee relationship between the insurance
company and its agents, despite the fact that the compensation that the agents on
commission received was not paid by the company but by the investor or the person
insured.
- Neither does it follow that a person who does not observe normal hours of work cannot
be deemed an employee. In Cosmopolitan Funeral Homes, Inc. v. Maalat, the Supreme
Court declared that there was an employer-employee relationship, noting that "[the]
supervisor, although compensated on commission basis, [is] exempt from the observance
of normal hours of work for his compensation is measured by the number of sales he
makes.
- The determination of an employer-employee relationship depends heavily on the
particular factual circumstances attending the professional interaction of the parties. SC
sees no reversible error in the findings of fact of the courts below. Both SSC and CA found
that Laudato was a sales supervisor and not a mere agent. As such, Laudato oversaw and
supervised the sales agents of the company, and thus was subject to the control of
management as to how she implements its policies and its end results. This is proven by
several documentary evidence.
Disposition Petition is DENIED. CA Decision AFFIRMED. Costs against petitioner.

ALMIREZ V INFINITE LOOP TECHNOLOGY


CORPORATION
[PAGE 57]
LAZARO V SSS (LAUDATO)
[PAGE 61]
DOMASIG V NLRC (CATA GARMENTS)
261 SCRA 779 (96)
PADILLA; September 16, 1996
NATURE
Petition for certiorari under Rule 65 of the Rules of Court to nullify and set aside the
Resolution of respondent National Labor Relations Commission remanding the records of
the case to the arbitration branch of origin for further proceedings.

FACTS
- Complaint was instituted by Eddie Domasig against respondents Cata Garments
Corporation, a company engaged in garments business and its owner/manager Otto Ong
and Catalina Co for illegal dismissal, unpaid commission and other monetary claim(s).
- Complainant alleged that he started working with the respondent on July 6, 1986 as
Salesman; three (3) years ago, because of a complaint against respondent by its workers,
the company changed its name to Cata Garments Corporation; and that on August 29,
1992, he was dismissed when respondent learned that he was being pirated by a rival
corporation which offer he refused.
- The Labor Arbiter held that complainant was illegally dismissed and entitled to
reinstatement and backwages as well as underpayment of salary; 13th month pay; service
incentive leave and legal holiday. The Arbiter also awarded complainant his claim for
unpaid commission in the amount of P143,955.00.
- NLRC remanded the case for further proceedings.
- Petitioners Claim
> Petitioner claims he was an employee, and that he was illegally dismissed.
- Respondents Comments
> Respondents claim that Domasig was a mere commission worker, and not a regular
employee (which would warrant backwages).
ISSUE
WON Domasig is a regular employee (this case is under the topic of proof of
employment)

Labor Law 1
HELD
YES, Domasig is a regular employee.
Ratio Substantial evidence is sufficient as a basis for judgment on the existence of
employer-employee relationship.
Reasoning
- Proof beyond reasonable doubt is not required as a basis for judgment on the legality of
an employers dismissal of an employee, nor even preponderance of evidence for that
matter, substantial evidence being sufficient. Any competent and relevant evidence to
prove the relationship may be admitted.
- Substantial evidence
> relevant evidence as a reasonable mind might accept as adequate to support a
conclusion, and its absence is not shown by stressing that there is contrary evidence on
record, direct or circumstantial, for the appellate court cannot substitute its own judgment
or criterion for that of the trial court in determining wherein lies the weight of evidence or
what evidence is entitled to belief.
> In a business establishment, an identification card is usually provided not only as a
security measure but mainly to identify the holder thereof as a bona fide employee of the
firm that issues it. Together with the cash vouchers covering petitioners salaries for the
months stated therein, these matters constitute substantial evidence adequate to support
a conclusion that petitioner was indeed an employee of private respondent.
> The list presented by private respondents would even support petitioners allegation
that, aside from a monthly salary of P1,500.00, he also received commissions for his work
as a salesman of private respondents.
- Having been in the employ of private respondents continuously for more than one year,
under the law, petitioner is considered a regular employee.
Disposition The decision of the labor arbiter dated 19 May 1993 is REINSTATED and
AFFIRMED.

ABANTE V LAMADRID
430 SCRA 368
YNARES-SANTIAGO; May 28, 2004
NATURE
Petition for review assailing the Decision of the CA which affirmed the Resolution of the
NLRC
FACTS
- Petitioner was employed by respondent company Lamadrid Bearing and Parts
Corporation sometime in June 1985 as a salesman covering the whole area of Mindanao.
His average monthly income was more or less P16,000.00, but later was increased to
approximately P20,269.50. Aside from selling the merchandise of respondent corporation,
he was also tasked to collect payments from his various customers. Sometime in 1998,
petitioner encountered five customers/clients with bad accounts.
- Petitioner was confronted by respondent Lamadrid over the bad accounts and warned
that if he does not issue his own checks to cover the said bad accounts, his commissions
will not be released and he will lose his job. Not contented with the issuance of the
foregoing checks as security for the bad accounts, respondents "tricked" petitioner into
signing two documents, which he later discovered to be a Promissory Note and a Deed of
Real Estate Mortgage.
- Due to financial difficulties, petitioner inquired about his membership with the SSS in
order to apply for a salary loan. To his dismay, he learned that he was not covered by the
SSS and therefore was not entitled to any benefit. While doing his usual rounds as
commission salesman, petitioner was handed by his customers a letter from the
respondent company warning them not to deal with petitioner since it no longer recognized
him as a commission salesman. Petitioner thus filed a complaint for illegal dismissal with
money claims against respondent company and its president, Jose Lamadrid, before the
NLRC.
ISSUE
1. WON an employer-employee relationship exists between plaintiff and respondent
company
2. WON respondent intimidated and tricked plaintiff into providing security for the bad
accounts
HELD
1. NO
Ratio To ascertain the existence of an employer-employee relationship, jurisprudence has
invariably applied the four-fold test, namely: (1) the manner of selection and engagement;
(2) the payment of wages; (3) the presence or absence of the power of dismissal; and (4)
the presence or absence of the power of control. Of these four, the last one is the most
important. Under the control test, an employer-employee relationship exists where the
person for whom the services are performed reserves the right to control not only the end
achieved, but also the manner and means to be used in reaching that end. Where a

A2010

- 62 -

Disini

person who works for another does so more or less at his own pleasure and is not subject
to definite hours or conditions of work, and in turn is compensated according to the result
of his efforts and not the amount thereof, no relationship of employer-employee exists.
Reasoning
- Petitioner Abante was a commission salesman who received 3% commission of his
gross sales. No quota was imposed on him by the respondent. He was not required to
report to the office at any time or submit any periodic written report on his sales
performance and activities. He was not designated by respondent to conduct his sales
activities at any particular or specific place. He pursued his selling activities without
interference or supervision from respondent company and relied on his own resources to
perform his functions. Respondent company did not prescribe the manner of selling the
merchandise; he was left alone to adopt any style or strategy to entice his customers.
Moreover, petitioner was free to offer his services to other companies engaged in similar
or related marketing activities as evidenced by the certifications issued by various
customers.
2. NO
Ratio While petitioner may have been coerced into executing force to issue the said
documents, it may equally be true that petitioner did so in recognition of a valid financial
obligation. He who claims that force or intimidation was employed upon him lies the onus
probandi. He who asserts must prove.
Disposition The decision of the CA is AFFIRMED in toto.

R TRANSPORT CORP V EJANDRA


[PAGE 55]
MANILA ELECTRIC COMPANY V QUISUMBING
[PAGE 19]
MANILA ELECTRIC CO V BENAMIRA
302 SCRA 173
AUSTRIA-MARTINEZ; July 14, 2005
NATURE
Petition for review on certiorari of the Court of Appeals decision
FACTS
- The individual respondents are licensed security guards formerly employed by Peoples
Security, Inc. (PSI) and deployed as such at MERALCOs head office in Ortigas Avenue,
Pasig, Metro Manila. On November 30, 1990, the security service agreement between PSI
and MERALCO was terminated. Immediately thereafter, fifty-six of PSIs security guards,
including herein eight individual respondents, filed a complaint for unpaid monetary
benefits against PSI and MERALCO. Meanwhile, the security service agreement between
respondent Armed Security & Detective Agency, Inc., (ASDAI) and MERALCO took effect
on December 1, 1990. In the agreement, ASDAI was designated as the AGENCY while
MERALCO was designated as the COMPANY.
- Subsequently, the individual respondents were absorbed by ASDAI and retained at
MERALCOs head office.
- Asuncion rendered a decision in NLRC-NCR Case No. 05-02746-90 in favor of the
former PSI security guards, including the individual respondents.
- Less than a month later, or on July 21, 1992, the individual respondents filed another
complaint for unpaid monetary benefits, this time against ASDAI and MERALCO.
- On July 25, 1992, the security service agreement between respondent Advance Forces
Security & Investigation Services, Inc. (AFSISI) and MERALCO took effect, terminating
the previous security service agreement with ASDAI. Except as to the number of security
guards, the amount to be paid the agency, and the effectivity of the agreement, the terms
and conditions were substantially identical with the security service agreement with
ASDAI.
- The individual respondents amended their complaint to implead AFSISI as party
respondent. They again amended their complaint to allege that AFSISI terminated their
services on August 6, 1992 without notice and just cause and therefore guilty of illegal
dismissal.
- The individual respondents alleged that: MERALCO and ASDAI never paid their overtime
pay, service incentive leave pay, premium pay for Sundays and Holidays, P50.00 monthly
uniform allowance and underpaid their 13th month pay; on July 24, 1992, when the
security service agreement of ASDAI was terminated and AFSISI took over the security
functions of the former on July 25, 1992, respondent security guard Benamira was no
longer given any work assignment when AFSISI learned that the former has a pending
case against PSI, in effect, dismissing him from the service without just cause; and, the
rest of the individual respondents were absorbed by AFSISI but were not given any
assignments, thereby dismissing them from the service without just cause.
- ASDAI denied in general terms any liability for the claims of the individual respondents,
claiming that there is nothing due them in connection with their services.

Labor Law 1
- On the other hand, MERALCO denied liability on the ground of lack of employeremployee relationship with individual respondents. It averred that the individual
respondents are the employees of the security agencies it contracted for security services;
and that it has no existing liability for the individual respondents claims since said security
agencies have been fully paid for their services per their respective security service
agreement.
- For its part, AFSISI asserted that: it is not liable for illegal dismissal since it did not
absorb or hire the individual respondents, the latter were merely hold-over guards from
ASDAI; it is not obliged to employ or absorb the security guards of the agency it replaced
since there is no provision in its security service agreement with MERALCO or in law
requiring it to absorb and hire the guards of ASDAI as it has its own guards duly trained to
service its various clients.
- After the submission of their respective evidence and position papers, Labor Arbiter
Pablo C. Espiritu, Jr. rendered a Decision holding ASDAI and MERALCO jointly and
solidarily liable to the monetary claims of individual respondents and dismissing the
complaint against AFSISI. Individual respondents partial appeal assailed solely the Labor
Arbiters declaration that ASDAI is their employer. They insisted that AFSISI is the party
liable for their illegal dismissal and should be the party directed to reinstate them.
For its part, MERALCO attributed grave abuse of discretion on the part of the Labor Arbiter
in failing to consider the absence of employer-employee relationship between MERALCO
and individual respondents.
- On the other hand, ASDAI took exception from the Labor Arbiters finding that it is the
employer of the individual respondents and therefore liable for the latters unpaid
monetary benefits.
- The NLRC affirmed in toto the decision of the Labor Arbiter. The individual respondents
filed a motion for partial reconsideration but it was denied by the NLRC.
The individual respondents filed a petition for certiorari before the SC. They insisted that
they were absorbed by AFSISI and the latter effected their termination without notice and
just cause.
- After the submission of the responsive pleadings and memoranda, we referred the
petition, in accordance with St. Martin Funeral Homes vs. NLRC,[15] to the CA which, on
September 27, 2000, modified the decision of the NLRC by declaring MERALCO as the
direct employer of the individual respondents.
- The CA held that: MERALCO changed the security agency manning its premises three
times while engaging the services of the same people, the individual respondents;
MERALCO employed a scheme of hiring guards through an agency and periodically
entering into service contract with one agency after another in order to evade the security
of tenure of individual respondents; individual respondents are regular employees of
MERALCO since their services as security guards are usually necessary or desirable in
the usual business or trade of MERALCO and they have been in the service of MERALCO
for no less than six years; an employer-employee relationship exists between MERALCO
and the individual respondents because: (a) MERALCO had the final say in the selection
and hiring of the guards, as when its advice was proved to have carried weight in AFSISIs
decision not to absorb the individual respondents into its workforce; (b) MERALCO paid
the wages of individual respondents through ASDAI and AFSISI; (c) MERALCOs
discretion on matters of dismissal of guards was given great weight and even finality since
the record shows that the individual respondents were replaced upon the advice of
MERALCO; and, (d) MERALCO has the right, at any time, to inspect the guards, to require
without explanation the replacement of any guard whose behavior, conduct or appearance
is not satisfactory and ASDAI and AFSISI cannot pull out any security guard from
MERALCO without the latters consent; and, a labor-only contract existed between ASDAI
and AFSISI and MERALCO, such that MERALCO is guilty of illegal dismissal without just
cause and liable for reinstatement of individual respondents to its workforce.
ISSUES
1. WON there existed an employer-employee relationship
2. WON individual respondents cannot be considered as regular employees as the duties
performed by them as security guards are not necessary in the conduct of MERALCOs
principal business which is the distribution of electricity.
3. WON MERALCO has a liability over the dismissed guards
HELD
1. It is a settled rule that in the exercise of the Supreme Courts power of review, the
Court is not a trier of facts and does not normally undertake the re-examination of the
evidence presented by the contending parties during the trial of the case considering that
the findings of facts of the CA are conclusive and binding on the Court. However,
jurisprudence has recognized several exceptions in which factual issues may be resolved
by this Court.
- In the present case, the existence of an employer-employee relationship is a question of
fact which is well within the province of the CA. Nonetheless, given the reality that the
CAs findings are at odds to those of the NLRC, the Court is constrained to look deeper
into the attendant circumstances obtaining in the present case, as appearing on record.
The individual respondents never alleged in their complaint in the Labor Arbiter, in their
appeal in the NLRC and even in their petition for certiorari in the CA that MERALCO was
their employer. They have always advanced the theory that AFSISI is their employer. A

A2010

- 63 -

Disini

perusal of the records shows it was only in their Memorandum in the CA that this thesis
was presented and discussed for the first time. We cannot ignore the fact that this
position of individual respondents runs contrary to their earlier submission in their
pleadings filed in the Labor Arbiter, NLRC and even in the petition for certiorari in the CA
that AFSISI is their employer and liable for their termination. As the object of the pleadings
is to draw the lines of battle, so to speak, between the litigants and to indicate fairly the
nature of the claims or defenses of both parties, a party cannot subsequently take a
position contrary to, or inconsistent, with his pleadings.
Moreover, it is a fundamental rule of procedure that higher courts are precluded from
entertaining matters neither alleged in the pleadings nor raised during the proceedings
below, but ventilated for the first time only in a motion for reconsideration or on appeal.
The individual respondents are bound by their submissions that AFSISI is their employer
and they should not be permitted to change their theory. Such a change of theory cannot
be tolerated on appeal, not due to the strict application of procedural rules but as a matter
of fairness. A change of theory on appeal is objectionable because it is contrary to the
rules of fair play, justice and due process.
- Thus, the CA should not have considered the new theory offered by the individual
respondents in their memorandum.
- The present petition for review on certiorari is far from novel and, in fact, not without
precedence. We have ruled in Social Security System vs. Court of Appeals that:
...The guards or watchmen render their services to private respondent by allowing
themselves to be assigned by said respondent, which furnishes them arms and
ammunition, to guard and protect the properties and interests of private respondent's
clients, thus enabling that respondent to fulfill its contractual obligations. Who the
clients will be, and under what terms and conditions the services will be rendered, are
matters determined not by the guards or watchmen, but by private respondent. On the
other hand, the client companies have no hand in selecting who among the guards or
watchmen shall be assigned to them. It is private respondent that issues assignment
orders and instructions and exercises control and supervision over the guards or
watchmen, so much so that if, for one reason or another, the client is dissatisfied with
the services of a particular guard, the client cannot himself terminate the services of
such guard, but has to notify private respondent, which either substitutes him with
another or metes out to him disciplinary measures. That in the course of a watchman's
assignment the client conceivably issues instructions to him, does not in the least
detract from the fact that private respondent is the employer of said watchman, for in
legal contemplation such instructions carry no more weight than mere requests, the
privity of contract being between the client and private respondent, not between the
client and the guard or watchman. Corollarily, such giving out of instructions inevitably
spring from the client's right predicated on the contract for services entered into by it
with private respondent.
- In the matter of compensation, there can be no question at all that the guards or
watchmen receive compensation from private respondent and not from the companies or
establishments whose premises they are guarding. The fee contracted for to be paid by
the client is admittedly not equal to the salary of a guard or watchman; such fee is arrived
at independently of the salary to which the guard or watchman is entitled under his
arrangements with private respondent.
- Neither does the petitioner have any power to dismiss the security guards. In fact, We
fail to see any evidence in the record that it wielded such a power. It is true that it may
request the agency to change a particular guard. But this, precisely, is proof that the
power lies in the hands of the agency.
- Since the petitioner has to deal with the agency, and not the individual watchmen, on
matters pertaining to the contracted task, it stands to reason that the petitioner does not
exercise any power over the watchmen's conduct. Always, the agency stands between
the petitioner and the watchmen; and it is the agency that is answerable to the petitioner
for the conduct of its guards.
- In this case, the terms and conditions embodied in the security service agreement
between MERALCO and ASDAI expressly recognized ASDAI as the employer of individual
respondents.
- Under the security service agreement, it was ASDAI which (a) selected, engaged or hired
and discharged the security guards; (b) assigned them to MERALCO according to the
number agreed upon; (c) provided the uniform, firearms and ammunition, nightsticks,
flashlights, raincoats and other paraphernalia of the security guards; (d) paid them salaries
or wages; and, (e) disciplined and supervised them or principally controlled their conduct.
The agreement even explicitly provided that [n]othing herein contained shall be
understood to make the security guards under this Agreement, employees of the
COMPANY, it being clearly understood that such security guards shall be considered as
they are, employees of the AGENCY alone. Clearly, the individual respondents are the
employees of ASDAI.
- Needless to stress, for the power of control to be present, the person for whom the
services are rendered must reserve the right to direct not only the end to be achieved but
also the means for reaching such end. Not all rules imposed by the hiring party on the
hired party indicate that the latter is an employee of the former. Rules which serve as
general guidelines towards the achievement of the mutually desired result are not
indicative of the power of control.

Labor Law 1
- Verily, the security service agreements in the present case provided that all specific
instructions by MERALCO relating to the discharge by the security guards of their duties
shall be directed to the agency and not directly to the individual respondents. The
individual respondents failed to show that the rules of MERALCO controlled their
performance.
- Moreover, ASDAI and AFSISI are not labor-only contractors. There is labor only
contract when the person acting as contractor is considered merely as an agent or
intermediary of the principal who is responsible to the workers in the same manner and to
the same extent as if they had been directly employed by him. On the other hand, job
(independent) contracting is present if the following conditions are met: (a) the contractor
carries on an independent business and undertakes the contract work on his own account
under his own responsibility according to his own manner and method, free from the
control and direction of his employer or principal in all matters connected with the
performance of the work except to the result thereof; and (b) the contractor has substantial
capital or investments in the form of tools, equipment, machineries, work premises and
other materials which are necessary in the conduct of his business.[29] Given the above
distinction and the provisions of the security service agreements entered into by petitioner
with ASDAI and AFSISI, we are convinced that ASDAI and AFSISI were engaged in job
contracting.
2. YES
- The individual respondents can not be considered as regular employees of the
MERALCO for, although security services are necessary and desirable to the business of
MERALCO, it is not directly related to its principal business and may even be considered
unnecessary in the conduct of MERALCOs principal business, which is the distribution of
electricity.
- Furthermore, the fact that the individual respondents filed their claim for unpaid monetary
benefits against ASDAI is a clear indication that the individual respondents acknowledge
that ASDAI is their employer.
- We cannot give credence to individual respondents insistence that they were absorbed
by AFSISI when MERALCOs security service agreement with ASDAI was terminated.
The individual respondents failed to present any evidence to confirm that AFSISI absorbed
them into its workforce. Thus, respondent Benamira was not retained in his post at
MERALCO since July 25, 1992 due to the termination of the security service agreement of
MERALCO with ASDAI. As for the rest of the individual respondents, they retained their
post only as hold-over guards until the security guards of AFSISI took over their post on
August 6, 1992.
- In the present case, respondent Benamira has been off-detail for seventeen days while
the rest of the individual respondents have only been off- detail for five days when they
amended their complaint on August 11, 1992 to include the charge of illegal dismissal.
The inclusion of the charge of illegal dismissal then was premature. Nonetheless, bearing
in mind that ASDAI simply stopped giving the individual respondents any assignment and
their inactivity clearly persisted beyond the six-month period allowed by Article 286 of the
Labor Code, the individual respondents were, in effect, constructively dismissed by ASDAI
from employment, hence, they should be reinstated.
3. YES, as an indirect employer.
- The fact that there is no actual and direct employer-employee relationship between
MERALCO and the individual respondents does not exonerate MERALCO from liability as
to the monetary claims of the individual respondents. When MERALCO contracted for
security services with ASDAI as the security agency that hired individual respondents to
work as guards for it, MERALCO became an indirect employer of individual respondents
pursuant to Article 107 of the Labor Code, which reads:
ART. 107. Indirect employer - The provisions of the immediately preceding Article shall
likewise apply to any person, partnership, association or corporation which, not being
an employer, contracts with an independent contractor for the performance of any
work, task, job or project.
- When ASDAI as contractor failed to pay the individual respondents, MERALCO as
principal becomes jointly and severally liable for the individual respondents wages, under
Articles 106 and 109 of the Labor Code, which provide:
ART. 106. Contractor or subcontractor. - Whenever an employer enters into a contract
with another person for the performance of the former[s] work, the employees of the
contractor and of the latter[s] subcontractor, if any, shall be paid in accordance with the
provisions of this Code.
In the event that the contractor or subcontractor fails to pay the wages of his
employees in accordance with this Code, the employer shall be jointly and severally
liable with his contractor or subcontractor to such employees to the extent of the work
performed under the contract, in the same manner and extent that he is liable to
employees directly employed by him.
ART. 109. Solidary liability - The provisions of existing laws to the contrary
notwithstanding, every employer or indirect employer shall be held responsible with his
contractor or subcontractor for any violation of any provision of this Code. For purpose
of determining the extent of their civil liability under this Chapter, they shall be
considered as direct employers.
- ASDAI is held liable by virtue of its status as direct employer, while MERALCO is
deemed the indirect employer of the individual respondents for the purpose of paying their
wages in the event of failure of ASDAI to pay them. This statutory scheme gives the

A2010

- 64 -

Disini

workers the ample protection consonant with labor and social justice provisions of the
1987 Constitution.
- However, as held in Mariveles Shipyard Corp. vs. Court of Appeals, the solidary liability
of MERALCO with that of ASDAI does not preclude the application of Article 1217 of the
Civil Code on the right of reimbursement from his co-debtor by the one who paid, which
provides:
ART. 1217. Payment made by one of the solidary debtors extinguishes the obligation.
If two or more solidary debtors offer to pay, the creditor may choose which offer to
accept.
- He who made the payment may claim from his co-debtors only the share which
corresponds to each, with the interest for the payment already made. If the payment is
made before the debt is due, no interest for the intervening period may be demanded.
When one of the solidary debtors cannot, because of his insolvency, reimburse his share
to the debtor paying the obligation, such share shall be borne by all his co-debtors, in
proportion to the debt of each.
- ASDAI may not seek exculpation by claiming that MERALCOs payments to it were
inadequate for the individual respondents lawful compensation. As an employer, ASDAI is
charged with knowledge of labor laws and the adequacy of the compensation that it
demands for contractual services is its principal concern and not any others.[35]
Disposition present petition is GRANTED. The assailed Decision, dated September 27,
2000, of the CA is REVERSED and SET ASIDE. The Decision of the Labor Arbiter dated
January 3, 1994 and the Resolution of the NLRC dated April 10, 1995 are AFFIRMED with
the MODIFICATION that the joint and solidary liability of ASDAI and MERALCO to pay
individual respondents monetary claims for underpayment of actual regular hours and
overtime hours rendered, and premium pay for holiday and rest day, as well as attorneys
fees, shall be without prejudice to MERALCOs right of reimbursement from ASDAI.

SAN MIGUEL CORP V ABELLA


[PAGE 59]

BIG AA MANUFACTURER V ANTONIO


484 SCRA 392
QUISUMBING; March 3, 2006
NATURE
Petition for review on certiorari of a decision of CA
FACTS
- Petitioner Big AA Manufacturer is a sole proprietorship registered in the name of its
proprietor, Enrico E. Alejo. Respondents filed a complaint for illegal lay-off and illegal
deductions
- Respondents
> That as regular employees, they worked from 8:00 a.m. to 5:00 p.m. at petitioners
premises using petitioners tools and equipment and they received P250 per day. Eutiquio
was employed as carpenter-foreman from 1991-99; Jay as carpenter from 1993-99;
Felicisimo as carpenter from 1994-99; and Leonardo, Sr. also as carpenter from 1997-99;
That they were dismissed without just cause and due process; hence, their prayer for
reinstatement and full backwages.
- Petitioner Big AA Manufacturer
> That it is a sole proprietorship registered in the name of Enrico Alejo and engaged in
manufacturing office furniture, but it denied that respondents were its regular employees. It
claimed that Eutiquio Antonio was one of its independent contractors who used the
services of the other respondents. It said that its independent contractors were paid by
results and were responsible for the salaries of their own workers. Allegedly, there was no
employer-employee relationship between petitioner and respondents. But it allowed
respondents to use its facilities to meet job orders. It also denied that respondents were
laid-off by Big AA Manufacturer, since they were project employees only. It added that
since Eutiquio Antonio had refused a job order of office tables, their contractual
relationship ended.
- Labor Arbiter ruled againstpetitioners. Both appealed to NLRC. Respondents appealed
for not ordering their reinstatement to their former positions. The NLRC modified the Labor
Arbiters decision. It ordered petitioner to reinstate respondents to their former positions or
to pay them separation pay in case reinstatement was no longer feasible, with full
backwages in either case. The NLRC ruled that respondents were regular employees, not
independent contractors. It further held that petitioner failed to justify its reason for

Labor Law 1
terminating respondents and its failure to comply with the due process requirements. CA
affirmed NLRC ruling.
ISSUES
1. WON respondents were regular employees
2. WON respondents were illegally dismissed
HELD
1. YES
- Respondents were employed for more than 1 year and their work as carpenters was
necessary or desirable in petitioners usual trade or business of manufacturing office
furniture. Under Art. 280 of the Labor Code, the applicable test to determine whether an
employment should be considered regular or non-regular is the reasonable connection
between the particular activity performed by the employee in relation to the usual business
or trade of the employer.
- True, certain forms of employment require the performance of usual or desirable
functions and exceed 1 year but do not necessarily result to regular employment under
Art. 280 of the Labor Code. Some specific exceptions include project or seasonal
employment. Yet, in this case, respondents cannot be considered project employees.
Petitioner had neither shown that respondents were hired for a specific project the
duration of which was determined at the time of their hiring nor identified the specific
project or phase thereof for which respondents were hired.
Obiter on Requirements for an Independent contractor: a) he carries a distinct and
independent business, b) possesses substantial capital or investment in tools, equipment,
machinery or work premises, c) he does not work within another employer/companys
premises using the latters tools and materials, and d) he is not under the control and
supervision of an employer or company
2. YES
- The consistent rule is that the employer must affirmatively show rationally adequate
evidence that the dismissal was for a justifiable cause, failing in which would make the
termination illegal, as in this case.
- Contrary to petitioners claim of abandonment as a valid just cause for termination,
herein respondents did not abandon their work. Petitioner failed to prove that (1) not only
of respondents failure to report for work or absence without valid reason, but (2) also of
respondents clear intention to sever employer-employee relations as manifested by some
overt acts.
- By filing the complaint for illegal dismissal within two days of their dismissal and by
seeking reinstatement in their position paper, respondents manifested their intention
against severing their employment relationship with petitioner and abandoning their jobs. It
is settled that an employee who forthwith protests his layoff cannot be said to have
abandoned his work
Disposition Petition denied.

COCA-COLA OTTLERS PHILS INC V NLRC


(CANONICATO)
307 SCRA 131
BELLOSILLO; May 17, 1999

A2010

- 65 -

Disini

holding, NLRC applied Art.280 of the Labor Code and declared that Canonito was a
regular employee of Coca-Cola. Its motion for reconsideration having been denied, Coca
Cola filed this petition.
ISSUE
WON Canonito was a regular employee of Coca-cola and thus malking Coca-Cola liable
for illegal dismissal
HELD
NO
- In Kimberly Independent Labor Union v. Drilon where the Court took judicial notice of the
practice adopted in several government and private institutions and industries of hiring
janitorial services on an "independent contractor basis." In this respect, although janitorial
services may be considered directly related to the principal business of an employer, as
with every business, we deemed them unnecessary in the conduct of the employer's
principal business.
- This judicial notice, of course, rests on the assumption that the independent contractor is
a legitimate job contractor so that there can be no doubt as to the existence of an
employer-employee relationship between contractor and the worker. In this situation, the
only pertinent question that may arise will no longer deal with whether there exists an
employment bond but whether the employee may be considered regular or casual as to
deserve the application of Art. 280 of the Labor Code.
- It was error therefore for the NLRC to apply Art. 280 of the Labor Code in determining the
existence of an employment relationship of the parties herein, especially in light of our
explicit holding in Singer Sewing Machine Company v. Drion that The Court agrees with the petitioner's argument that Article 280 is not the yardstick for
determining the existence of an employment relationship because it merely distinguishes
between two kinds of employees, i.e., regular employees and casual employees, for
purposes of determining the right of an employee to certain benefits, to join or form a
union, or to security of tenure. Article 280 does not apply where the existence of an
employment relationship is in dispute
- In determining the existence of an employer-employee relationship it is necessary to
determine whether the following factors are present: (a) the selection and engagement of
the employee; (b) the payment of wages; (c) the power to dismiss; and, (d) the power to
control the employee's conduct. Notably, these are all found in the relationship between
BJS and Canonicato and not between Canonicato and petitioner COCA COLA. As the
Solicitor-General manifested
- BJS satisfied all the requirements of a job-contractor under the law, namely, (a) the ability
to carry on an independent business and undertake the contract work on its own account
under its own responsibility according to its manner and method, free from the control and
direction of its principal or client in all matters connected with the performance of the work
except as to the results thereof; and, (b) the substantial capital or investment in the form of
tools, equipment, machinery, work premises, and other materials which are necessary in
the conduct of its business.
- All told, there being no employer-employee relationship between Canonicato and COCA
COLA, the latter cannot be validly ordered to reinstate the former and pay him back
wages.

PAL V NLRC (STELLAR INDUSTRIAL SERVICES INC)


FACTS
- On April 7, 1986 Coca-Cola entered into a contract of janitorial services with Bacolod
Janitorial Services (BJS) as an independent contractor.
- Private respondent Ramon Canonicato was hired as a janitor by the Bacolod Janitorial
Services (BJS). He was assigned at the Coca Cola Bottlers, Inc. considering his familiarity
with its premises, having been previous casual employee there.
- Goaded by information that COCA COLA employed previous BJS employees who filed a
complaint against the company for regularization pursuant to a compromise agreement,
Canonicato submitted a similar complaint against COCA COLA to the Labor Arbiter on 8
June 1993 and consequently did not report for work.
- On September 28,1993, BJS sent him a letter advising him to report to work within 3
days from receipt, otherwise he would be terminated.
- (there was no express mention of a termination but based on the fact I assume he did
not return to work and was terminated)
- On July 23, 1993, respondent filed with the Labor Arbiter a complaint for illegal dismissal
and underpayment of wages. He included BJS therein as a co-respondent. The Labor
Arbiter dismissed the complaint and ruled that a) there was no employer-employee
relationship between Canonicato and Coca Cola (b) BJS was a legitimate job contractor,
hence, any liability of COCA COLA as to Canonicato's salary or wage differentials was
solidary with BJS in accordance with pars. 1 and 2 of Art. 106, Labor Code; (c) COCA
COLA and BJS must jointly and severally pay Canonicato his wage differentials amounting
to P2,776.80 and his 13th month salary of P1,068.00, including ten (10%) percent
attorney's fees in the sum of P384.48.
- The NLRC rejected the decision of the Labor Arbiter on the ground that the janitorial
services of Canonicato were found to be necessary in the usual trade of Coca Cola. In so

298 SCRA 430


PANGANIBAN; November 9, 1998
NATURE

Special civil action for certiorari, seeking to nullify


the July 13, 1994 Decision and the June 27, 1996
Resolution of the National Labor Relations
Commission, which held Philippines Airlines, Inc.
liable for separation pay.
FACTS
- Sometime in 1977, PAL, a local air carrier, entered into a service agreement with

Labor Law 1
STELLAR, a domestic corporation engaged, among others, in the business of job
contracting janitorial services. Pursuant to their service agreement, which was impliedly
renewed year after year, STELLAR hired workers to perform janitorial and maintenance
services for PAL. The employees were assigned at PAL's various premises under the
supervision of STELLAR's supervisors/foremen and timekeepers. The workers were also
furnished by STELLAR with janitorial supplies, such as vacuum cleaner and polisher.
- On December 31, 1990, the service agreement between PAL and STELLAR expired.
PAL then called for [the] bidding of its janitorial requirements. This notwithstanding,
STELLAR exerted efforts to maintain its janitorial contract with PAL which, in the
meantime, allowed Manuel Parenas and others to work at the PAL's premises.
- Subsequently, in a letter dated October 31, 1990, PAL formally informed STELLAR that
the service agreement would no longer be renewed effective November 16, 1991, since
PAL's janitorial requirements were bidded to three other job contractors. Alleging that they
were illegally dismissed, the aforenamed individual private respondents filed, from
January to June 1992, five complaints against PAL and STELLAR for illegal dismissal and
for payment of separation pay.
- Labor Arbiter Manuel P. Asuncion rendered on October 29, 1993 a Decision which held
PAL liable for the separation pay of terminated individual respondents.
- In its Decision affirming the ruling of the labor arbiter, Respondent Commission held
petitioner, as an indirect employer, jointly and severally liable with STELLAR for separation
pay. First, the individual private respondent's work, although not directly related to the
business of petitioner, was necessary and desirable for the maintenance of the petitioner's
premises and airplanes. Second, the individual private respondents were retained for
thirteen long years, despite the fact that the contract, which petitioner had entered into
STELLAR in 1977, was only for one year.
On reconsideration, the NLRC modified its earlier Decision by absolving STELLAR of
liability, thereby making PAL solely responsible for the award decreed by the labor arbiter.
It held that, first, petitioner was the employer of the individual private respondents, for it
engaged in labor-only contracting with STELLAR. This was shown by the failure of
petitioner to refute the factual finding that it continued to employ the individual private
respondents after the expiration of the service contract on December 31, 1990. Second,
the individual private respondents' admission in their Complaint that they were employees
of STELLAR was not conclusive, as the existence of an employer-employee relation was a
question of law that could not be the subject of stipulation. Respondent Commission
concluded that their dismissal was without just and valid cause. Because they were no
longer seeking reinstatement, petitioner was liable for separation pay.
ISSUES
1. WON the individual private respondents are regular employees of PAL
2. WON petitioner is liable to them for separation pay

HELD
1. No employer-employee relation between
complainants and petitioner.
Ratio a) Janitorial service agreement is not labor-only contacting AND
b) Extension of service contract is not a source of employer-employee relation.
Reasoning
a) Prohibited labor-only contracting is defined in Article 106 of the Labor Code as follows:

Disini

A2010 - 66 such cases, the person or intermediary shall be

considered merely as an agent of the employer who


shall be responsible to the workers in the same
manner and extent as if the latter were directly
employed by him.
- This definition covers any person who undertakes to supply workers to an employer,
where such person:
(1) Does not have substantial capital or investment in the form of tools, equipment,
[machinery], work premises and other materials; and
(2) The workers recruited and placed by such person are performing activities which are
directly related to the principal business or operations of the employer in which workers
are habitually employed.
- On the other hand, permissible job contracting requires the following conditions:
(1) The contractor carries on an independent business and undertakes the contract work
on his own account under his own responsibility according to his own manner and method,
free from the control and direction of his employer or principal in all matters connected
with the performance of the work except as to the results thereof; and
(2) The contractor has substantial capital or investment in the form of tools, equipment,
[machinery], work premises, and other materials which are necessary in the conduct of his
business.

- The employee-employer relation existed between the


individual private respondents and STELLAR, not
PAL. STELLAR possessed these earmarks of an
employer:
(1) the power of selection and engagement of
employees
(2) the payment of wages
(3) the power of dismissal, and
(4) the power to control the employee's conduct
- A contract of employment existed between
STELLAR and the individual private respondents,

There is "labor-only" contracting where the person


proving that it was said corporation which hired them.
supplying workers to an employer does not have
It was also STELLAR which dismissed them, as
substantial capital or investment in the form of tools,
evidenced by Complainant Parenas' termination letter,
equipment, machineries, work premises, among
which was signed by Carlos P. Callanga, vice
others, and the workers recruited and placed by such
president for operations and comptroller of
persons are performing activities which are directly
STELLAR. Likewise, they worked under STELLAR's
related to the principal business of such employer. In

Labor Law 1
own supervisors, Rodel Pagsulingan, Napoleon
Parungao and Renato Topacio. STELLAR even had
its own collective bargaining agreement with its
employees, including the individual private
respondents. Moreover, PAL had no power of control
and dismissal over them.
- In fact, STELLAR claims that it falls under the definition of an independent job contractor.
Thus, it alleges that it has sufficient capital in the form of tools and equipment, like vacuum
cleaners and polishers, and substantial capitalization as proven by its financial statements.
Further, STELLAR has clients other than petitioner, like San Miguel Corporation,
Hongkong and Shanghai Bank, Eveready, Benguet Management Corporation and Japan
Airlines.
- All these circumstances establish that STELLAR undertook said contract on its account,
under its own responsibility, according to its own manner and method, and free from the
control and direction of the petitioner. Where the control of the principal is limited only to
the result of the work, independent job contracting exists. The janitorial service agreement
between petitioner and STELLAR is definitely a case of permissible job contracting.
b) What actually happened was that PAL and STELLAR impliedly renewed, as they had
previously done before, their service agreement until PAL's janitorial requirements were
bidded to other job contractors. This explains why the individual private respondents
remained working at PAL's premises even after December 31, 1990.
- It is evident that petitioner was engaged in permissible job contracting and that the
individual private respondents, for the entire duration of their employ, were employees not
of petitioner but of STELLAR. In legitimate job contracting, no employer-employee relation
exists between the principal and the job contractor's employees. The principal is
responsible to the job contractor's employees only for the proper payment of wages. But in
labor-only contracting, an employer-employee relation is created by law between the
principal and the labor-only contractor's employees, such that the former is responsible to
such employees, as if he or she had directly employed them. Besides, the Court has
already taken judicial notice of the general practice adopted in several government and
private institutions of securing janitorial services on an independent contractor basis.
2. NO, STELLAR is the one liable for separation pay.
Ratio Despite the protestations of STELLAR, the service agreement was not a project
because its duration was not determined or determinable.
Reasoning
- In order to avoid liability for separation pay, STELLAR argues that it terminated the
services of the individual private respondents for a just and valid cause: the completion of
a specific project. Thus, they are not entitled to separation pay.
- The Court is not convinced. The position of STELLAR that individual private respondents
were its project employees is totally unfounded. A regular employee is distinguished from
a project employee by the fact that the latter is employed to carry out a specific project or
undertaking, the duration or scope of which was specified at the time the employees were
engaged. A "project" has reference to a particular job or undertaking that may or may not
be within the regular or usual business of the employer. In either case, the project must
be distinct, separate and identifiable from the main business of the employer, and its
duration must be determined or determinable.
- While the service agreement may have had a specific term, STELLAR disregarded it,
repeatedly renewed the service agreement, and continued hiring the individual private
respondents for thirteen consecutive years. Had STELLAR won the bidding, the alleged
"project" would have never ended. In any event, the aforesaid stipulations in the
employment contract are not included in Articles 282 and 283 of the Labor Code as valid
causes for the dismissal of employees.
Again, we must emphasize that the main business of STELLAR is the supply of manpower
to perform janitorial services for its clients, and the individual private respondents were
janitors engaged to perform activities that were necessary and desirable to STELLAR's
enterprise. In this case, we hold that the individual private respondents were STELLAR's
regular employees, and there was no valid cause for their dismissal.
Disposition petition is hereby GRANTED. The assailed Decision and Resolution are
SET ASIDE insofar as they held PAL liable for separation pay. The July 13, 1994 Decision
is however reinstated insofar as it ORDERED STELLAR liable for such award.

MERCURY DRUG CORPORATION V LIBUNAO

A2010

- 67 -

Disini

434 SCRA 404


CALLEJO, SR; July 14, 2004
NATURE
Petition for review on certiorari of a CA decision which modified an RTC decision, and the
Resolution of the CA denying the petitioners motion for reconsideration
FACTS
According to the plaintiff.
> Libunao and his friend bought some items at Mercury. He paid for his purchase and
placed his receipt in his pocket. As they exited, they were accosted by Sido, the security
guard. Sido was armed with a service gun, and was 20 pounds heavier than Libunao. He
held Libunaos upper right arm and demanded to see the receipt. Libunao searched but it
took time because Sido was holding his right arm. Sido then said Wala yatang resibo
yan! Libunao finally found it, and asked Sido, Satisfied ka na? Sido reacted by lunging
at him and saying Putang ina mo! Sido was able to hit lubnao on the face, nose, chin,
and mouth. He then pointed his revolver at Libunao and said Putang ina mo, pag hindi
kayo lumabas ditto papuputukin ko to sa iyo! Libunao eventually filed a criminal complaint
against Sido. He was traumatized by the event, he had to consult a psychiatrist, and was
found to be suffering from post-traumatic depression syndrome.
According to the defendants
> Sido, the security guard at Mercury, noticed Libunao exiting the store with a plastic bag,
and that no receipt was stapled to it. He asked for the receipt, but was given the plastic
bag. He found no receipt, and when Libunao finally found the receipt and shoved it in his
face, he just explained he was doing his duty. Libunao said Baka hindi mo ako kilala,
security guard ka lang! Ano ba talaga ang problema mo? A violent argument ensued.
- The court rendered judgment in favor of the plaintiff, that the defendants Sido, Mercurly
Drug Corporation, and Store Manager Vilma Santos, pay the plaintiff moral and exemplary
damages, to discourage disrespect of the public by such acts as were committed by
defendants
ISSUE
WON the remedy of the petitioner is proper (that Mercury Drug be liable for Sidos actions)
HELD
NO
Ratio The petitioner was not Sidos employer; hence, CC A 2180 should not be applied
against petitioner.
Reasoning
- The respondent was burdened to prove that the petitioner was the employer of Sido but
failed to discharge this burden.
- The respondents counsel admitted Sido was not employed by the petitioner
- Store manager Santos testified that Sido was not an employee of the petitioner, but of
BSSC, Black Shield Agency.
- The petitioner adduced in evidence its contract with the BSSC, which contained the
following provisions: 1. THE AGENCY shall provide the CLIENT with the necessary
number of armed, uniformed and qualified security guards properly licensed by the Chief
of Philippine Constabulary; who shall provide security services to the CLIENT at its
establishment at
These security guards during the life of the Agreement shall be assigned in accordance
with arrangements to be made between the CLIENT and the AGENCY.
...
6. The AGENCY assumes full responsibility for any claim or cause of action which may
accrue in favor of any security guard by reason of employment with the AGENCY, it
being understood that security guards are employees of the AGENCY and not of the
CLIENT.
- Therefore, the respondent had no cause of action against the petitioner for damages for
Sidos illegal and harmful acts. The respondent should have sued Sido and the BSSC for
damages, conformable to A2180.
- In Soliman, Jr. v. Tuazon the court held that where the security agency recruits, hires and
assigns the works of its watchmen or security guards to a client, the employer of such
guards or watchmen is such agency, and not the client, since the latter has no hand in
selecting the security guards. Thus, the duty to observe the diligence of a good father of a
family cannot be demanded from the said client
- The petitioner had assigned Sido to help the management open and close the door of
the drug store; inspect the bags of customers as they enter the store; and, check the
receipts issued by the cashier to said customers for their purchases. Such circumstances
do not automatically make the security guard the employee of the petitioner, and, as such,
liable for the guard's tortious acts. The fact that a client company may give instructions or
directions to the security guards assigned to it, does not, by itself, render the client
responsible as an employer of the security guards concerned and liable for their wrongful
acts or omissions.
Disposition petition is hereby GRANTED. The Decision dated June 9, 2000 and the
Resolution dated August 9, 2000 of the Court of Appeals in CA-G.R. CV No. 59754 are

Labor Law 1
hereby REVERSED and SET ASIDE. The complaint filed by the respondent against
petitioner Mercury Drug Corporation in Civil Case No. Q-92-14114 is DISMISSED. The
counterclaims of the latter are also DISMISSED. No costs.

MARIVELESSHIPYARD V CA
415 SCRA573
QUISUMBING; November 11, 2003
FACTS
- In October 1993, petitioner Mariveles Shipyard Corporation engaged the services of
Longest Force Investigation and Security Agency, Inc. to render security services at its
premises. Pursuant to their agreement, Longest Force deployed its security guards, the
private respondents herein, at the petitioners shipyard in Mariveles, Bataan.
- According to petitioner, it found the services being rendered by the assigned guards
unsatisfactory and inadequate, causing it to terminate its contract with Longest Force on
April 1995. Longest Force, in turn, terminated the employment of the security guards it had
deployed at petitioners shipyard.
- Private respondents filed a case for illegal dismissal and underpayment of wages, among
others. In turn, Longest Force filed a cross-claim against Mariveles Shipyard, alleging that
the service fee paid by the latter to it was way below the PNPSOSIA and PADPAO rate.
- The petitioner denied any liability on account of the alleged illegal dismissal, stressing
that no employer-employee relationship existed between it and the security guards.
Petitioner likewise prayed that Longest Forces cross-claim be dismissed for lack of merit.
Petitioner averred that Longest Force had benefited from the contract, it was now
estopped from questioning said agreement on the ground that it had made a bad deal.
- The Labor Arbiter found Mariveles and Longest Force jointly and severally liable for
private respondents money claims and attorneys fees. Longest Force was likewise
ordered to reinstate private respondents without loss of seniority rights and privileges with
full backwages. The NLRC affirmed the Labor Arbiters decision.
- The Court of Appeals refused to give due course to Mariveles Shipyards appeal for
failure to comply with procedural requirements.
ISSUES
1. WON the Court of Appeals dismissal of the petition was in order despite petitioners
subsequent compliance with the procedural requirements
2. WON petitioner was denied due process of law by the NLRC
3. WON petitioner is jointly and severally liable with Longest Force for private
respondents money claims
HELD
1. NO
- The requirement in the Rules that the certification of non-forum shopping should be
executed and signed by the plaintiff or the principal means that counsel cannot sign said
certification unless clothed with special authority to do so. The reason for this is that the
plaintiff or principal knows better than anyone else whether a petition has previously been
filed involving the same case or substantially the same issues. Hence, a certification
signed by counsel alone is defective and constitutes a valid cause for dismissal of the
petition. In the case of the corporations, the physical act of signing may be performed, on
behalf of the corporate entity, only by specifically authorized individuals for the simple
reason that corporations, as artificial persons, cannot personally do the task themselves.
In this case, not only was the originally appended certification signed by counsel, but in its
motion for reconsideration, still petitioner utterly failed to show that Ms. Rosanna Ignacio,
its Personnel Manager who signed the verification and certification of non-forum shopping
attached thereto, was duly authorized for this purpose.
2. NO
- The essence of due process is simply an opportunity to be heard, or, as applied to
administrative proceedings, an opportunity to explain ones side or an opportunity to seek
a reconsideration of the action or ruling complained of. Not all cases require a trial-type
hearing. The requirement of due process in labor cases before a Labor Arbiter is satisfied
when the parties are given the opportunity to submit their position papers to which they
are supposed to attach all the supporting documents or documentary evidence that would
prove their respective claims, in the event the Labor Arbiter determines that no formal
hearing would be conducted or that such hearing was not necessary. In any event,
petitioner was given ample opportunity to present its side in several hearings conducted
before the Labor Arbiter and in the position papers and other supporting documents that it
had submitted. Such opportunity more than satisfies the requirement of due process in
labor cases.
3. YES
- Petitioners liability is joint and several with that of Longest Force, pursuant to Articles
106, 107 and 109 of the Labor Code. In this case, when petitioner contracted for security
services with Longest Force as the security agency that hired private respondents to work
as guards for the shipyard corporation, petitioner became an indirect employer of private
respondents pursuant to Article 107. Following Article 106, when the agency as contractor
failed to pay the guards, the corporation as principal becomes jointly and severally liable

A2010

- 68 -

Disini

for the guards wages. This is mandated by the Labor Code to ensure compliance with its
provisions, including payment of statutory minimum wage. The security agency is held
liable by virtue of its status as direct employer, while the corporation is deemed the indirect
employer of the guards for the purpose of paying their wages in the event of failure of the
agency to pay them. This statutory scheme gives the workers the ample protection
consonant with labor and social justice provisions of the 1987 Constitution.
- Petitioner cannot evade its liability by claiming that it had religiously paid the
compensation of guards as stipulated under the contract with the security agency. Labor
standards are enacted by the legislature to alleviate the plight of workers whose wages
barely meet the spiraling costs of their basic needs. Labor laws are considered written in
every contract. Stipulations in violation thereof are considered null. Similarly, legislated
wage increases are deemed amendments to the contract.
- However, we must emphasize that the joint and several liability imposed on petitioner is
without prejudice to a claim for reimbursement by petitioner against the security agency for
such amounts as petitioner may have to pay to complainants, the private respondents
herein. The security agency may not seek exculpation by claiming that the principals
payments to it were inadequate for the guards lawful compensation. As an employer, the
security agency is charged with knowledge of labor laws; and the adequacy of the
compensation that it demands for contractual services is its principal concern and not any
others.
- On the issue of the propriety of the award of overtime pay despite the alleged lack of
proof thereof, suffice it to state that such involves a determination and evaluation of facts
which cannot be done in a petition for review.
- Upon review of the award of backwages and attorneys fees, we discovered certain
errors that happened in the addition of the amount of individual backwages that resulted in
the erroneous total amount of backwages and attorneys fees. These errors ought to be
properly rectified now. Thus, the correct sum of individual backwages should be
P126,648.40 instead of P126,684.40, while the correct sum of total backwages awarded
and attorneys fees should be P3,926,100.40 and P392,610.04, instead of P3,927,216.40
and P392,721.64, respectively.
Disposition The Court of Appeals Resolution is AFFIRMED with MODIFICATION.

NEW GOLDEN CITY BUILDERS V CA (GALLO ET AL)


418 SCRA 411
YNARES-SANTIAGO; December 11, 2003
FACTS
- New Golden City Builders and Development Corporation, a corporation engaged in the
construction business, entered into a construction contract with Prince David Development
Corporation for the construction of a 17-storey office and residential condominium building
along Katipunan Road, Loyola Heights, Quezon City, Metro Manila.
- Petitioner engaged the services of Nilo Layno Builders to do the specialized concrete
works, form works and steel rebar works, for a total contract price of P5 Million. Nilo
Layno Builders hired private respondents to perform work at the project. After the
completion of the phase for which Nilo Layno Builders was contracted sometime in 1996,
private respondents filed a complaint case against petitioner and its president, Manuel Sy,
with the Arbitration Branch of the NLRC for unfair labor practice, non-payment of 13th
month pay, non-payment of 5 days service incentive leave, illegal dismissal and severance
pay in lieu of reinstatement.
- The Labor Arbiter found that Nilo Layno Builders was a labor-only-contractor; thus,
private respondents were deemed employees of New Golden City. Both parties appealed
the decision of the Labor Arbiter to the NLRC. Petitioner maintained that Nilo Layno
Builders was an independent contractor and that private respondents were not its
employees. On the other hand, private respondents claimed that the Labor Arbiter erred in
finding that they were not illegally dismissed and not entitled to recover monetary claims
like premium pay for rest days, regular holidays and special holiday. The NLRC affirmed
with modification the Labor Arbiters decision. As modified, the NLRC held that private
respondents were illegally dismissed and ordered petitioner to reinstate them and to pay
their full back wages. CA affirmed.

ISSUES
1. WON Nilo Layno Builders was an independent contractor and not a labor-only
contractor
2. WON an employer-employee relationship existed between petitioner and private
respondents

HELD
1. YES

Labor Law 1
Ratio The test to determine the existence of independent contractorship is whether one
claiming to be an independent contractor has contracted to do the work according to his
own methods and without being subject to the control of the employer, except only to the
results of the work.
Reasoning
- Under Section 8, Rule VIII, Book III, of the Omnibus Rules Implementing the Labor Code,
an independent contractor is one who undertakes job contracting, i.e., a person who: (a)
carries on an independent business and undertakes the contract work on his own account
under his own responsibility according to his own manner and method, free from the
control and direction of his employer or principal in all matters connected with the
performance of the work except as to the results thereof; and (b) has substantial capital or
investment in the form of tools, equipments, machineries, work premises, and other
materials which are necessary in the conduct of the business.
- Nilo Layno Builders hired its own employees, the private respondents, to do specialized
work in the Prince David Project of the petitioner. The means and methods adopted by
the private respondents were directed by Nilo Layno Builders except that, from time to
time, the engineers of the petitioner visited the site to check whether the work was in
accord with the plans and specifications of the principal. As admitted by Nilo G. Layno, he
undertook the contract work on his own account and responsibility, free from interference
from any other persons, except as to the results; that he was the one paying the salaries
of private respondents; and that as employer of the private respondents, he had the power
to terminate or dismiss them for just and valid cause.
- As a licensed labor contractor, Nilo Layno Builders complied with the conditions set forth
in Section 5, Rule VII-A, Book III, Rules to Implement the Labor Code, among others,
proof of financial capability and list of equipment, tools, machineries and implements to be
used in the business.
2. YES. [But for a limited purpose only]
Ratio In legitimate job contracting, the law creates an employer-employee relationship for
a limited purpose, i.e., to ensure that the employees are paid their wages. The principal
employer becomes jointly and severally liable with the job contractor only for the payment
of the employees wages whenever the contractor fails to pay the same. Other than that,
the principal employer is not responsible for any claim made by the employees.
Reasoning
- The petitioner did not, as it could not, illegally dismissed the private complainants.
Hence, it could not be held liable for back wages and separation pay. Nevertheless, it is
jointly and severally liable with Nilo Layno Builders for the private complainants wages, in
the same manner and extent that it is liable to its direct employees. The pertinent
provisions of the Labor Code read:
ART. 106. Contractor or subcontractor. Whenever an employer enters into a
contract with another person for the performance of the formers work, the employees
of the contractor and of the latters subcontractor, if any, shall be paid in accordance
with the provisions of this Code. XXX In the event that the contractor or subcontractor
fails to pay the wages of his employees in accordance with this Code, the employer
shall be jointly and severally liable with his contractor or subcontractor to such
employees to the extent of the work performed under the contract, in the same manner
and extent that he is liable to employees directly employed by him.
ART. 107. Indirect employer. The provisions of the immediately preceding Article
shall likewise apply to any person, partnership, association or corporation which, not
being an employer, contracts with an independent contractor for the performance of
any work, task, job or project.
- This liability covers the payment of service incentive leave and 13th month pay of the
private complainants during the time they were working at petitioners Prince David
Project. So long as the work, task, job or project has been performed for petitioners
benefit or on its behalf, the liability accrues for such period even if, later on, the employees
are eventually transferred or reassigned elsewhere.
Disposition Petition PARTLY GRANTED. Decision of the CA MODIFIED. Petitioner
ABSOLVED from liability for back wages. However, he is ORDERED to pay, jointly and
severally with Nilo Layno Builders, private complainants Service Incentive Leave Pay and
13th Month Pay.

VINOYA V NLRC (REGENT FOOD CORP)


324 SCRA 469
KAPUNAN; February 2, 2000
NATURE
Petition for certiorari under Rule 65 seeking to annul NLRC decision
FACTS
- Petition seeks to annul and set aside the decision of the NLRC which reversed the
decision of the Labor Arbiter, ordering RFC to reinstate Alexander Vinoya to his former
position and pay him backwages.
Petitioners Claim

A2010

- 69 -

Disini

> Vinoya applied and was accepted by RFC as sales representative. RFC issued him an
identification card.
- He reported daily to the RFC office to take the van for the delivery of products. He was
assigned to various supermarkets and grocery stores where he booked sales orders and
collected payments for RFC. He was required by RFC to put up a monthly bond of P200
as security deposit to guarantee his work performance.
- After more than a year, he was transferred by RFC to Peninsula Manpower Company,
Inc., an agency which provides RFC with additional contractual workers pursuant to a
contract for the supply of manpower services. After this, petitioner was reassigned to RFC
as sales rep.
- 5 months later, he was informed by the personnel manager of RFC that his services were
terminated and he was asked to surrender his ID card. Petitioner was told that his
dismissal was due to the expiration of the Contract of Service between RFC and PMCI.
Petitioner claims that he was dismissed from employment despite the absence of any
notice or investigation.
- He filed a case against RFC before the Labor Arbiter for illegal dismissal and nonpayment of 13th month pay.
Respondents Comments
> RFC maintains that there is no employer-employee relationship. Petitioner is actually an
employee of PMCI, an independent contractor, which had a Contract of Service with RFC.
RFC presented an Employment Contract signed by petitioner on 1 July 1991, wherein
PMCI appears as his employer. RFC denies that petitioner was ever employed by it prior
to 1 July 1991. Petitioner was issued an ID card so that its clients and customers would
recognize him as a duly authorized representative of RFC. With regard to the P200
monthly bond, RFC asserts that it was required in order to guarantee the turnover of his
collection since he handled funds of RFC. While RFC admits that it had control and
supervision over petitioner, it argues that such was exercised in coordination with PMCI.
Finally, RFC contends that the termination of its relationship with petitioner was brought
about by the expiration of the Contract of Service between itself and PMCI.
ISSUES
1. WON PMCI is a labor-only contractor or an independent contractor
2. WON petitioner was an employee of RFC or PMCI
3. WON petitioner was lawfully dismissed
HELD
1. PMCI is a labor-only contractor.
Ratio In determining the existence of an independent contractor relationship, several
factors might be considered such as, but not necessarily confined to, whether the
contractor is carrying on an independent business; the nature and extent of the work; the
skill required; the term and duration of the relationship; the right to assign the performance
of specified pieces of work; the control and supervision of the workers; the power of the
employer with respect to the hiring, firing and payment of the workers of the contractor; the
control of the premises; the duty to supply premises, tools, appliances, materials and
labor; and the mode, manner and terms of payment.
Reasoning
- Labor-only contracting, a prohibited act, is an arrangement where the contractor or
subcontractor merely recruits, supplies or places workers to perform a job, work or service
for a principal. The following elements are present: (a) The contractor or subcontractor
does not have substantial capital or investment to actually perform the job, work or service
under its own account and responsibility; (b) The employees recruited, supplied or placed
by such contractor or subcontractor are performing activities which are directly related to
the main business of the principal.
- Permissible job contracting or subcontracting refers to an arrangement whereby a
principal agrees to put out or farm out with a contractor or subcontractor the performance
or completion of a specific job, work or service within a definite or predetermined period,
regardless of whether such job, work or service is to be performed or completed within or
outside the premises of the principal. A person is considered engaged in legitimate job
contracting or subcontracting if the following conditions concur: (a) The contractor or
subcontractor carries on a distinct and independent business and undertakes to perform
the job, work or service on its own account and under its own responsibility according to
its own manner and method, and free from the control and direction of the principal in all
matters connected with the performance of the work except as to the results thereof; (b)
The contractor or subcontractor has substantial capital or investment; and (c) The
agreement between the principal and contractor or subcontractor assures the contractual
employees entitlement to all labor and occupational safety and health standards, free
exercise of the right to self-organization, security of tenure, and social and welfare
benefits.
- First of all, PMCI does not have substantial capitalization or investment in the form of
tools, equipment, machineries, work premises, among others, to qualify as an independent
contractor.
-Second, PMCI did not carry on an independent business nor did it undertake the
performance of its contract according to its own manner and method, free from the control
and supervision of its principal, RFC. The evidence at hand shows that the workers
assigned by PMCI to RFC were under the control and supervision of the latter.

Labor Law 1
-Third, PMCI was not engaged to perform a specific and special job or service.. As stated
in the Contract of Service, the sole undertaking of PMCI was to provide RFC with a
temporary workforce able to carry out whatever service may be required by it. Apart from
that, no other particular job, work or service was required from PMCI. Obviously, with such
an arrangement, PMCI merely acted as a recruitment agency for RFC.
- Lastly, in labor-only contracting, the employees recruited, supplied or placed by the
contractor perform activities which are directly related to the main business of its principal.
In this case, the work of petitioner as sales representative is directly related to the
business of RFC. Being in the business of food manufacturing and sales, it is necessary
for RFC to hire a sales representative like petitioner to take charge of booking its sales
orders and collecting payments for such. Thus, the work of petitioner as sales
representative in RFC can only be categorized as clearly related to, and in the pursuit of
the latters business. Logically, when petitioner was assigned by PMCI to RFC, PMCI
acted merely as a labor-only contractor.
2. Petitioner was an employee of RFC
Ratio In determining the existence of employer-employee relationship the following
elements of the "four-fold test" are generally considered, namely: (1) the selection and
engagement of the employee or the power to hire; (2) the payment of wages; (3) the
power to dismiss; and (4) the power to control the employee. Of these four, the "control
test" is the most important.
- No particular form of proof is required to prove the existence of an employer-employee
relationship. Any competent and relevant evidence may show the relationship.
Reasoning
- PMC I as a labor-only contractor, cannot be considered as the employer of petitioner
- Even granting that PMCI is an independent contractor, still, a finding of the same will not
save the day for RFC. A perusal of the Contract of Service entered into between RFC and
PMCI reveals that petitioner is actually not included in the enumeration of the workers to
be assigned to RFC. This only shows that petitioner was never intended to be a part of
those to be contracted out.
-With regard to the first element, ID card is enough proof that petitioner was previously
hired by RFC prior to his transfer as agency worker to PMCI. ID card issued by RFC to
petitioner was dated more than one year before the Employment Contract was signed by
petitioner in favor of PMCI. While the Employment Contract indicates the word "renewal,"
presumably an attempt to show that petitioner had previously signed a similar contract
with PMCI, no evidence of a prior contract entered into between petitioner and PMCI was
ever presented by RFC. It follows that it was RFC who actually hired and engaged
petitioner to be its employee
- With respect to the payment of wages, the Court takes judicial notice of the practice of
employers who, in order to evade the liabilities under the Labor Code, do not issue
payslips directly to their employees. Even though the wages were coursed through PMCI,
we note that the funds actually came from the pockets of RFC. Thus, in the end, RFC is
still the one who paid the wages of petitioner albeit indirectly.
- As to the third element, the power to dismiss, the Contract of Service gave RFC the right
to terminate the workers assigned to it by PMCI without the latters approval. The
dismissal of petitioner was indeed made under the instruction of RFC to PMCI.
-The power of control refers to the authority of the employer to control the employee not
only with regard to the result of work to be done but also to the means and methods by
which the work is to be accomplished. The "control test" calls merely for the existence of
the right to control the manner of doing the work, and not necessarily to the actual
exercise of the right. The Labor Arbiter found that petitioner was under the direct control
and supervision of the personnel of RFC and not PMCI.
3. YES
Ratio The requirements for the lawful dismissal of an employee are two-fold, the
substantive and the procedural aspects. Not only must the dismissal be for a valid or
authorized cause, the rudimentary requirements of due process - notice and hearing
must, likewise, be observed before an employee may be dismissed.
Reasoning
- Since petitioner, due to his length of service, already attained the status of a regular
employee, he is entitled to the security of tenure provided under the labor laws. Hence, he
may only be validly terminated from service upon compliance with the legal requisites for
dismissal.
- RFC never pointed to any valid or authorized cause under the Labor Code which allowed
it to terminate the services of petitioner. Its lone allegation that the dismissal was due to
the expiration or completion of contract is not even one of the grounds for termination
allowed by law.
- Neither did RFC show that petitioner was given ample opportunity to contest the legality
of his dismissal. In fact, no notice of such impending termination was ever given him.
- An employee who has been illegally dismissed is entitled to reinstatement to his former
position without loss of seniority rights and to payment of full backwages corresponding to
the period from his illegal dismissal up to actual reinstatement.
Disposition Petition granted.

MANILA WATER COMPANY V PENA


434 SCRA 52

A2010

- 70 -

Disini

YNARES-SANTIAGO; July 8, 2004


NATURE
Petition for review on certiorari
FACTS
Manila Water is one of the 2 concessionaires contracted by MWSS to manage water
distribution. Pursuant to RA No. 8041, petitioner undertook to absorb ex-employees of
MWSS whose names were in the list, and those not in the list were terminated.
Respondents are contractual collectors, not listed, but petitioner still engaged their
services. They signed a 3 month contract.
121 collectors incorporated the Association Collectors Group Inc. or ACGI. Petitioner
continued to transact with ACGI, but eventually terminated its contract. Respondents filed
complaint for illegal dismissal, saying they were petitioners employees. Petitioner asserts
respondents were employees of ACGI, an independent contractor.
Arbiter found dismissal illegal. NLRC reversed. Respondents filed certiorari petition with
CA, which reversed NLRC decision and reinstated with modification the Arbiter decision.
Hence this petition.
ISSUES
1. WON theres employer-employee relationship
2. WON respondents were illegally dismissed
HELD
1. YES
- We must resolve WON ACGI is an independent contractor or a labor-only contractor.
- Labor-only contracting refers to arrangement where contractor merely recruits and
places workers for a principal. Elements
- contractor doesnt have substantial capital
- contractor doesnt control performance of contractual employee
- Arbiter correctly ruled that ACGI was not an independent contractor. ACGI doesnt have
substantial capital. It has no office. The work of the respondents was directly related to
business of petitioner. And ACGI did not carry on an independent business according to
its own manner.
- ACGI was a labor-only contractor, an agent of the petitioner.
- Then the workers are employees of the petitioner.
- Even the four-fold test (selection, payment of wages, dismissal power, control of conduct)
indicate the relationship.
2. YES
- The term fixed in the subsequent contract was used to defeat the tenurial security.
- Dismissal was illegal.

GRANDSPAN DEVELOPMENT CORP V


BERNARDO
470 SCRA 461
SANDOVAL-GUTIERREZ; September 21, 2005
NATURE
Certiorari under Rule 45
FACTS
- The instant controversy stemmed from a complaint for illegal dismissal and non-payment
of benefits filed with the LA by Ricardo Bernardo, Antonino Ceidoza and Edgar Del
Prado, against Grandspan and/or its warehouse manager, Manuel G. Lee
- In their complaint, respondents alleged that sometime in 1990, they were employed as
truck scale monitors by Grandspan with a daily salary of P104.00 each. Eventually, they
were assigned at its Truck Scale Section of the Warehouse/Materials Department. They
were issued identification cards signed.
- Oct 28, 1992-Grandspan sent them a notice terminating their services effective October
29, 1992 for using profane or offensive language, in violation of Article VI (2) (a) of the
companys Rules and Regulations.
- Grandspan denied the allegations and claimed that respondents are employees of J.
Narag Construction.
- Sometime in the 3rd quarter of 1992, Canad Japan Co., Ltd. engaged Grandspans
services for fabrication works of several round and rectangular steel tanks needed for the
HCMG or Sogo project due for completion in September, 1992. As a consequence,
Grandspan subcontracted the services of J. Narag Construction which, in turn, assigned
its 3 helpers (herein respondents) to work for its project.
- Sometime in October, 1992, Manuel G. Lee, manager of Grandspans Warehouse
Department received a report from supervisor Robert Ong that respondents vandalized
the companys log book and chairs.
- This prompted petitioner to send J. Narag Construction a memorandum terminating the
services of respondents for violation of the companys Rules and Regulations.

Labor Law 1
- June 30, 1994- LA dismissed respondents complaint; concluded that respondents were
validly dismissed from employment; held too that respondent were project employees
whose services were terminated upon completion of the project for which they were hired.
- NLRC- remanded the case to the LA for appropriate proceedings to determine whether
there is an employer-employee relationship between the parties.
- Both parties filed MRs but were denied by the NLRC in separate Resolutions
- Meantime, Del Prado died and was substituted by his surviving parent, Edgardo Del
Prado.
- Sept 17, 1999- CA set aside the NLRCs Resolutions and ordered Grandspan (1) to
reinstate respondents Bernardo and Ceidoza to their former positions and pay, jointly and
severally with J. Narag Construction, their backwages and other benefits, and (2) to pay
respondent Del Prado his separation pay.
- CA found that respondents are employees of petitioner; that they were non-project
workers; and that they were denied due process, thus:
They(Respondents Bernardo et al) worked in Grandspans premises using the
materials, supplies and equipment of Grandspan. They were under the supervision of
Grandspan as to the manner and results of their work, and performed services directly
connected to the usual business of respondent Grandspan for the fabrication of heavy
structural components
- Oct 8, 1999- Grandspan filed MR. Respondents also filed a motion for reconsideration
and/or clarification praying that the Appellate Courts Decision be modified by awarding
respondent Del Prado his backwages.
- Jan 6, 2000- CA denied Grandspans MR but order Grandspan and J. Narag
Construction to pay respondent Del Prado his separation pay and backwages.
ISSUE
WON the CA erred in holding that respondents are employees of Grandspan
HELD
YES
Grandspans Argument:
> it has no employer-employee relationship with respondents since they are employees of
J. Narag Construction, an independent contractor.
- Miguel vs. JCT Group - The test for determining an employer-employee relationship
hinges on resolving who has the power to select employees, who pays for their wages,
who has the power to dismiss them, and who exercises control in the methods and the
results by which the work is accomplished.
- SC agrees with CA when it found that J. Narag Construction assigned respondents to
perform activities directly related to the main business of petitioner. . These
circumstances confirm the existence of an employer-employee relationship between
petitioner and respondents.
> They worked in petitioners premises, using its equipment, materials and supplies. J.
Narag Constructions payroll worksheets covering the period from December 21, 1990
to July 31, 1991 show that the payment of their salaries was approved by petitioner.
> The manager and supervisor of petitioners Warehouse Department supervised the
manner and results of their work.
> It was petitioner who terminated their services after finding them guilty of using
profane or offensive language in violation of Article VI (2) (a) of the companys Rules
and Regulations
- SC also agrees with the CA that J. Narag Construction is a labor-only contractor.
> A106 LC as amended, provides that there is labor-only contracting where the
person supplying workers to an employer does not have substantial capital or
investment in the form of tools, equipment, machineries, work premises, among others,
and the workers recruited and placed by such person are performing activities which
are directly related to the principal business of such employer. x x x.
- J. Narag Construction is indeed a labor-only contractor. These are the reasons:
(1) it is not registered as a building contractor with the SEC;
(2) it has no contract with petitioner; and
(3) there is no proof of its financial capability and has no list of equipment, tools,
machineries and implements used in the business.
- Kiamco vs. NLRC: The principal test for determining whether particular employees are
properly characterized as project employees, as distinguished from regular employees,
is whether or not the project employees were assigned to carry out a specific project or
undertaking, the duration and scope of which were specified at the time the employees
were engaged for that project. As defined, project employees are those workers hired (1)
for a specific project or undertaking, and (2) the completion or termination of such project
or undertaking has been determined at the time of engagement of the employee.
- Grandspan could not present employment contracts signed by respondents showing that
their employment was for the duration of the HCMG or Sogo project and failed to present
any report terminating the services of respondents when its projects were actually finished
pursuant to Sec2.2 (e) of the Labor Department Order No. 19
SC: The failure of the employer to file termination reports after every project completion
with the nearest public employment office is an indication that respondents were not
project employees. TF: respondents are Grandspans regular employees. As such, they

A2010

- 71 -

Disini

are entitled to security of tenure and can only be dismissed for a just or authorized cause,
as provided by Article 279 of the Labor Code.
- Bolinao Security and Investigation Service, Inc. vs. Toston- it is incumbent upon the
employer to prove by the quantum of evidence required by law that the dismissal of an
employee is not illegal, otherwise, the dismissal would be unjustified.
- SC: Termination is ILLEGAL. Grandspan violated respondents right (both substantive
and procedural) to due process as records show that respondents were not served by
notices of any kind nor were asked to explain the misconduct imputed to them.
> Loadstar Shipping Co., Inc. vs. Mesano: The law requires that an employee sought
to be dismissed must be served two written notices before termination of his
employment. The first notice is to apprise the employee of the particular acts or
omissions by reason of which his dismissal has been decided upon; and the second
notice is to inform the employee of the employers decision to dismiss him. Failure to
comply with the requirement of two notices makes the dismissal illegal. The procedure
is mandatory. Non-observance thereof renders the dismissal of an employee illegal
and void.
- SC: they are entitled to reinstatement without loss of seniority rights, full backwages,
inclusive of allowances, and other benefits or their monetary equivalent computed from
the time their compensation was withheld from them up to the time of their actual
reinstatement.
Disposition CAs decision AFFIRMED with modification. Reinstatement in this case is N/A
because of antagonism. Respondents are entitled to a separation pay of P4,992.00
plus their respective full backwages, and other privileges and benefits, or their monetary
equivalent, during the period of their dismissal up to their supposed actual reinstatement.

PHILIPPINE AIRLINES INC V NLRC (VILLENA,


NATIONAL ORGANIZATION OF WORKING MEN,
VILLACRUZ)
296 SCRA 214
QUISUMBING; September 25, 1998
NATURE
Special Civil Action in the Supreme Court. Certiorari.
FACTS
- Petitioner Philippine Airlines Inc. (PAL) is a domestic corporation principally engaged in
the air transportation industry for both domestic and foreign markets. Private respondent
National Organization of the Workingmen (NOWM) is a labor union, while the other private
respondents are members of respondent union and complainants in aforementioned
cases.
- PAL contracted the services G. C. Services Enterprises, to undertake specific projects.
Accordingly, G. C. Services recruited and hired carpenters, painters, and electricians and
assigned them to different PAL shops, namely: Carpentry Shop, Electrical Shop, Technical
Center Shop and Inflight Center Shop, all under PALs Construction and Corporate
Services Department.
- PAL terminated its contract with G. C. Services. As a result, all G.C. employees
assigned as PAL project workers were notified by G.C. Services not to report anymore to
PAL. Later, PAL decided to give G.C. Services employees the opportunity to apply as
regular employees, in accordance with its practice of giving employment priority to
qualified persons who had been connected with PAL. Due to lack of vacant positions and
also due to alleged unsatisfactory work performance records of some, not all G.C.
Services employees were hired. Those who were not hired instituted the instant complaint
for illegal dismissal. The complainants were represented in their case by the NOWM.
- Initially, there were 36 complainants in these three consolidated cases. In the course of
the proceedings, PAL agreed to employ 23 qualified complainants. Only 12 complainants
were left.
- The rest of the complainants alleged that they applied for employment with G.C.
Services; that after they were accepted they were made to work at PAL Maintenance
Department where each of them worked as carpenters, welders, or electricians; that they
were not considered employees of PAL but that of G.C. Services; that their work are
necessary and directly related to PALs principal business. In pointing at PAL as their real
employer, they averred that G.C. Services is only an agent of PAL because it does not
have substantial capital in the form of cash investments, tools, equipment or work
premises; that it merely supplied workers to PAL and these workers were supervised,
directed and controlled by PAL regular employees; that PAL actually decided when, where
and what to work; that PAL decided how many of them were to be taken in, when they
would start, and when they would not. Complainants, thus, argued that G.C. Services
being a mere agent, the real employer was PAL pursuant to Art.106 of the Labor Code
which prohibits the employment of persons through labor only contracting agencies, like
the G.C. Services Enterprise.
- In claiming that they were illegally dismissed, complainants alleged that they were
dismissed from employment without just cause and without due process and without any

Labor Law 1
prior notice. They thus prayed for reinstatement with full backwages from the date of their
dismissal on March 31, 1990 up to the date of their actual reinstatement.
- Respondent PAL denied the existence of employer-employee relationship between it and
the complainants. It averred that G.C. Services Enterprises, as a duly licensed
independent contractor, contracted on its own account under its own responsibility; that
the contractor has substantial capital or investment in the form of tools, equipment and
other materials necessary in the conduct of its business; that complainants were being
paid their wages by G.C. Services and not PAL; and that they were terminated by G.C.
Services. PAL further argued that even granting arguendo that complainants are entitled to
be regularized, it is not obliged to employ all the complainants; and that there are no more
positions or substantially equivalent positions within its organization for which they maybe
(sic) qualified.
- The Labor Arbiter ruled that G.C. Services Enterprises is a labor-only contractor and
mere agent of PAL (petitioner herein), thus, the private respondents are deemed
employees of petitioner. The Labor Arbiter then declared the termination of private
respondents services illegal, and held petitioner and G.C. Services Enterprises jointly and
severally liable to pay private respondents their separation pay, backwages as well as
attorneys fees.
- Both parties appealed to the NLRC, which, affirmed the Labor Arbiters decision with
modification as to the computation of the monetary award.
- Its motion for reconsideration having been denied, petitioner filed the instant petition.
ISSUES
1. WON the public respondents committed grave abuse of discretion in declaring the
dismissal of private respondents illegal despite the finding of redundancy
2. WON private respondents are entitled to separation pay as well as backwages
3. WON petitioner should be held jointly and severally liable
HELD
1. YES
- The petitioner regularized and/or re-employed 23 original complainants as there were
vacant positions to which they could qualify. However, the remaining 12 complainants
(private respondents herein) could no longer be absorbed into petitioners regular
workforce as there were no longer vacant positions as evidenced by the Table of
Organization of PAL Construction and Corporate Services Department. Simply put, the
services of private respondents were already in excess of what is reasonably demanded
by the actual manpower requirement of petitioner. It is settled that where there is need for
reduction of workforce, management has the right to choose whom to layoff, depending on
the work still required to be done and the qualities of the workers to be retained.
- Under Article 203 (must be 283) of the Labor Code, the employer may terminate an
employee due to redundancy or retrenchment.
- In Wilshire (sic) File Co., Inc. v. NLRC, 193 SCRA 672 the Supreme Court aptly ruled:
Redundancy, for purposes of our Labor code, exists where the services of an
employee are in excess of what is reasonably demanded by the actual requirements of
the enterprise. Succinctly put, a position is redundant where it is superfluous and the
superfluity of a position or positions may be the outcome of a number of factors, such
as over-hiring of workers xxx. The employer has no legal obligation to keep in its
payroll more employees than are necessary for the operation of its business.
(underscoring supplied)
- Clearly, the Labor Arbiter recognized the existence of redundancy. Despite said findings
the Labor Arbiter ruled as follows:
xxx In consonance therefore under Art. 280 of the Labor Code of the Philippines,
herein complainants are regular employees. For being so, they are protected by the
Security of Tenure provision of law (Art. 279, Labor Code) the complainant dismissal
being not in contemplation with Art. 282 of the Labor Code it is therefore illegal. xxx
- The reference to Article 282 is misplaced. Article 282 enumerates the causes for
termination by reason of some blameworthy act or omission on the part of the employee.
- In the instant case, the cause of termination is redundancy which is an authorized cause
for termination under Article 283. In any event, it is absurd for the Labor Arbiter to declare
a finding of redundancy, on one hand, and to conclude, on the other, that the termination
of private respondents services is illegal. There being redundancy, the dismissal of private
respondents is valid
2. NO
- Since private respondents were validly dismissed under Art. 283, they are not entitled to
backwages. Apparently, public respondents awarded backwages to private respondents
to penalize PAL for engaging in a labor-only scheme. However, the law does not give
public respondents such authority. The only effect of labor-only contracting is that the
person or intermediary shall be considered merely as an agent of the employer who shall
be responsible to the workers in the same manner and extent as if the latter were directly
employed by him (Art. 106, Labor Code).
- Thus, private respondents are entitled to separation pay only. The award of backwages
to them has no basis in law.
3. YES
- Petitioner and G.C. Services Enterprises are jointly and severally liable to the private
respondents for the latters monetary claims. The reason is that G.C. Services Enterprises,

A2010

- 72 -

Disini

being a labor only contractor, is merely an agent of the petitioner (the employer); the
resultant liability must be shouldered by either one or shared by both. Hence, petitioner
cannot avoid liability by invoking its Service Agreement with G.C. Services Enterprises
considering that here the liability is imposed by law.

SAN MIGUEL V MAERC INTEGRATED SERVICES


405 SCRA 579
BELLOSILLO; July 10, 2003
FACTS
- TWO HUNDRED NINETY-ONE (291) workers filed their complaints (nine [9] complaints
in all) against San Miguel Corporation (petitioner herein) and Maerc Integrated Services,
Inc. (respondent herein), for illegal dismissal, underpayment of wages, non-payment of
service incentive leave pays and other labor standards benefits, and for separation pays
from 25 June to 24 October 1991. The complainants alleged that they were hired by San
Miguel Corporation (SMC) through its agent or intermediary Maerc Integrated Services,
Inc. (MAERC) to work in two (2) designated workplaces in Mandaue City: one, inside the
SMC premises at the Mandaue Container Services, and another, in the Philphos
Warehouse owned by MAERC. They washed and segregated various kinds of empty
bottles used by SMC to sell and distribute its beer beverages to the consuming public.
They were paid on a per piece or pakiao basis except for a few who worked as checkers
and were paid on daily wage basis. Complainants alleged that long before SMC
contracted the services of MAERC a majority of them had already been working for SMC
under the guise of being employees of another contractor, Jopard Services, until the
services of the latter were terminated on 31 January 1988. SMC informed MAERC of the
termination of their service contract by the end of June 1991. SMC cited its plans to phase
out its segregation activities starting 1 June 1991 due to the installation of labor and costsaving devices. When the service contract was terminated, complainants claimed that
SMC stopped them from performing their jobs; that this was tantamount to their being
illegally dismissed by SMC who was their real employer as their activities were directly
related, necessary and desirable to the main business of SMC; and, that MAERC was
merely made a tool or a shield by SMC to avoid its liability under the Labor Code
- MAERC for its part admitted that it recruited the complainants and placed them in the
bottle segregation project of SMC but maintained that it was only conveniently used by
SMC as an intermediary in operating the project or work directly related to the primary
business concern of the latter with the end in view of avoiding its obligations and
responsibilities towards the complaining workers.
-The Labor Arbiter rendered a decision holding that MAERC was an independent
contractor. He dismissed the complaints for illegal dismissal but ordered MAERC to pay
complainants' separation benefits in the total amount of P2,334,150.00. MAERC and SMC
were also ordered to jointly and severally pay complainants their wage differentials in the
amount of P845,117.00 and to pay attorney's fees in the amount of P317,926.70.
- The National Labor Relations Commission (NLRC) ruled that MAERC was a labor-only
contractor and that complainants were employees of SMC. The NLRC also held that
whether MAERC was a job contractor or a labor-only contractor, SMC was still solidarily
liable with MAERC for the latter's unpaid obligations, citing Art. 109 4 of the Labor Code.
Thus, the NLRC modified the judgment of the Labor Arbiter and held SMC jointly and
severally liable with MAERC for complainants' separation benefits. In addition, both
respondents were ordered to pay jointly and severally an indemnity fee of P2,000.00 to
each complainant.
- SMC filed petition for certiorari
ISSUE
WON the complainants are employees of petitioner SMC or of respondent MAERC
HELD
Employees of SMC
- In ascertaining an employer-employee relationship, the following factors are considered:
(a) the selection and engagement of employee; (b) the payment of wages; (c) the power of
dismissal; and, (d) the power to control an employee's conduct, the last being the most
important. Application of the aforesaid criteria clearly indicates an employer-employee
relationship between petitioner and the complainants.
- Evidence discloses that petitioner played a large and indispensable part in the hiring of
MAERC's workers. It also appears that majority of the complainants had already been
working for SMC long before the signing of the service contract between SMC and
MAERC.
- The incorporators of MAERC admitted having supplied and recruited workers for SMC
even before MAERC was created. The NLRC also found that when MAERC was
organized into a corporation, the complainants who were then already working for SMC
were made to go through the motion of applying for work with Ms. Olga Ouano, President
and General Manager of MAERC, upon the instruction of SMC through its supervisors to
make it appear that complainants were hired by MAERC. This was testified to by two (2) of
the workers who were segregator and forklift operator assigned to the Beer Marketing

Labor Law 1
Division at the SMC compound and who had been working with SMC under a purported
contractor Jopard Services since March 1979 and March 1981, respectively. Both
witnesses also testified that together with other complainants they continued working for
SMC without break from Jopard Services to MAERC.
- As for the payment of workers' wages, it is conceded that MAERC was paid in lump sum
but records suggest that the remuneration was not computed merely according to the
result or the volume of work performed. The memoranda of the labor rates bearing the
signature of a Vice-President and General Manager for the Vismin Beer Operations as
well as a director of SMC appended to the contract of service reveal that SMC assumed
the responsibility of paying for the mandated overtime, holiday and rest day pays of the
MAERC workers. SMC also paid the employer's share of the SSS and Medicare
contributions, the 13th month pay, incentive leave pay and maternity benefits. In the lump
sum received, MAERC earned a marginal amount representing the contractor's share.
These lend credence to the complaining workers' assertion that while MAERC paid the
wages of the complainants, it merely acted as an agent of SMC.
- Petitioner insists that the most significant determinant of an employer-employee
relationship, i.e., the right to control, is absent. The contract of services between MAERC
and SMC provided that MAERC was an independent contractor and that the workers hired
by it "shall not, in any manner and under any circumstances, be considered employees of
the Company, and that the Company has no control or supervision whatsoever over the
conduct of the Contractor or any of its workers in respect to how they accomplish their
work or perform the Contractor's obligations under the Contract."
- In deciding the question of control, the language of the contract is not determinative of
the parties' relationship; rather, it is the totality of the facts and surrounding circumstances
of each case.
- Despite SMCs disclaimer, there are indicia that it actively supervised the complainants.
SMC maintained a constant presence in the workplace through its own checkers. Its
asseveration that the checkers were there only to check the end result was belied by the
testimony of Carlito R. Singson, head of the Mandaue Container Service of SMC, that the
checkers were also tasked to report on the identity of the workers whose performance or
quality of work was not according to the rules and standards set by SMC. According to
Singson, "it (was) necessary to identify the names of those concerned so that the
management [referring to MAERC] could call the attention to make these people improve
the quality of work."
- Other instances attesting to SMC's supervision of the workers are found in the minutes of
the meeting held by the SMC officers on 5 December 1988. Among those matters
discussed were the calling of SMC contractors to have workers assigned to segregation to
undergo and pass eye examination to be done by SMC EENT company doctor and a
review of compensation/incentive system for segregators to improve the segregation
activities.
- But the most telling evidence is a letter by Mr. Antonio Ouano, Vice-President of MAERC
dated 27 May 1991 addressed to Francisco Eizmendi, SMC President and Chief
Executive Officer, asking the latter to reconsider the phasing out of SMC's segregation
activities in Mandaue City. The letter was not denied but in fact used by SMC to advance
its own arguments. Briefly, the letter exposed the actual state of affairs under which
MAERC was formed and engaged to handle the segregation project of SMC. It provided
an account of how in 1987 Eizmendi approached the would-be incorporators of MAERC
and offered them the business of servicing the SMC bottle-washing and segregation
department in order to avert an impending labor strike. After initial reservations, MAERC
incorporators accepted the offer and before long trial segregation was conducted by SMC
at the PHILPHOS warehouse.
- In legitimate job contracting, the law creates an employer-employee relationship for a
limited purpose, i.e., to ensure that the employees are paid their wages. The principal
employer becomes jointly and severally liable with the job contractor only for the payment
of the employees' wages whenever the contractor fails to pay the same. Other than that,
the principal employer is not responsible for any claim made by the employees.
- On the other hand, in labor-only contracting, the statute creates an employeremployee relationship for a comprehensive purpose: to prevent a circumvention of labor
laws. The contractor is considered merely an agent of the principal employer and the latter
is responsible to the employees of the labor-only contractor as if such employees had
been directly employed by the principal employer. The principal employer therefore
becomes solidarily liable with the labor-only contractor for all the rightful claims of
the employees.
- This distinction between job contractor and labor-only contractor, however, will not
discharge SMC from paying the separation benefits of the workers, inasmuch as MAERC
was shown to be a labor-only contractor; in which case, petitioner's liability is that of a
direct employer and thus solidarily liable with MAERC.
- SMC also failed to comply with the requirement of written notice to both the employees
concerned and the Department of Labor and Employment (DOLE) which must be given at
least one (1) month before the intended date of retrenchment. The fines imposed for
violations of the notice requirement have varied. The measure of this award depends on
the facts of each case and the gravity of the omission committed by the employer. For its
failure, petitioner was justly ordered to indemnify each displaced worker P2,000.00.
Disposition Petition is DENIED.

A2010

- 73 -

Disini

PHIL. FEDERATION OF CREDIT COOPERATIVES INC


(PFCCI) V NLRC (ABRIL)
300 SCRA 72
ROMERO; December 11, 1998.
FACTS
- Victoria Abril was employed by PFCCI in different capacities from 1982 to 1988, when
she went on leave until she gave birth. When she went back in 1989, after 8 months,
another employee had been permanently appointed to her former position of office
secretary. She accepted a position of Regional Field Officer. The contract reads:
"That the employer hires the employee on contractual basis to the position of Regional
Field Officer of Region 4 under PFCCI/WOCCU/Aid Project No. 8175 and to do the
function as stipulated in the job description assigned to him (her): on probationary
status effective February 17/90 for a period not to exceed six (6) months from said
effectivity, subject to renewal of this contract should the employee's performance be
satisfactory."
- Said period having elapsed, respondent was allowed to work until PFCCI presented to
her another employment contract for a period of one year commencing on January 2,
1991 until December 31, 1991, after which period, her employment was terminated.
- LA dismissed her complaint for illegal dismissal against PFCCI.
- NLRC set aside LAs decision and ordered her reinstated to her last position held (RFO)
or to an equivalent position, with full backwages from Jan 1, 1992 until she is reinstated.
ISSUE
WON Abril was a regular employee and thus illegally dismissed
HELD
YES
- It is an elementary rule in the law on labor relations that a probationary employee who is
engaged to work beyond the probationary period of six months, as provided under Art. 281
of the Labor Code, as amended, or for any length of time set forth by the employer, shall
be considered a regular employee.
- Article 281 of the Labor Code, as amended, allows the employer to secure the services
of an employee on a probationary basis which allows him to terminate the latter for just
cause or upon failure to qualify in accordance with reasonable standards set forth by the
employer at the time of his engagement. A probationary employee is one who is on trial by
an employer during which the employer determines whether or not he is qualified for
permanent employment. A probationary employment is made to afford the employer an
opportunity to observe the fitness of a probationer while at work, and to ascertain whether
he will become a proper and efficient employee. Probationary employees, notwithstanding
their limited tenure, are also entitled to security of tenure. Thus, except for just cause as
provided by law, or under the employment contract, a probationary employee cannot be
terminated.
- PFCCI refutes the findings of the NLRC arguing that, after respondent had allegedly
abandoned her secretarial position for eight (8) months, she applied for the position of
Regional Field Officer for Region IV, which appointment, as petitioner would aptly put it,
"had been fixed for a specific project or undertaking the completion or termination of which
had been determined at the time of the engagement of said private respondent and
therefore considered as a casual or contractual employment under Article 280 of the Labor
Code."
- Abril cannot be classified as casual or contractual. (This is why the Court went into a
discussion of the kinds of employment recognized in this jurisdiction)
"Art. 280. Regular and casual employment. The provisions of written agreement to
the contrary notwithstanding and regardless of the oral agreement of the parties, an
employment shall be deemed to be regular where the employee has been engaged to
perform activities which are usually necessary or desirable in the usual business or
trade of the employer, except where the employment has been fixed for a specific
project or undertaking the completion or termination of which has been determined at
the time of the engagement of the employee or where the work or services to be
performed is seasonal in nature and the employment is for the duration of the season.
- An employment shall be deemed to be casual if it is not covered by the preceding
paragraph: Provided, That, any employee who has rendered at least one year of service,
whether such service is continuous or broken, shall be considered a regular employee
with respect to the activity in which he is employed and his employment shall continue
while such activity exists."
- This provision of law comprehends three kinds of employees:
(a) regular employees or those whose work is necessary or desirable to the usual
business of the employer;
(b) project employees or those whose employment has been fixed for a specific
project or undertaking the completion or termination of which has been determined at
the time of the engagement of the employee or where the work or services to be
performed is seasonal in nature and the employment is for the duration of the season;
and

Labor Law 1
(c) casual employees or those who are neither regular nor project employees.
- For contractual employees, stipulations in employment contracts providing for term
employment or fixed period employment are valid when
(1) the period were agreed upon knowingly and voluntarily by the parties without force,
duress or improper pressure being brought to bear upon the employee and absent any
other circumstances vitiating his consent, or
(2) where it satisfactorily appears that the employer and employee dealt with each
other on more or less equal terms with no moral dominance whatever being exercised
by the former over the latter.
- The contract (see facts) contains stipulations so ambiguous as to preclude a precise
application of pertinent labor laws. Since contract of employment is contract of adhesion,
ambiguity is construed strictly against the party who prepared it. Also, Art. 1702 of CC
provides that in case of doubt, all labor contracts shall be construed in favor of the laborer.
The interpretation which the respondent company seeks to wiggle out is wholly
unacceptable, as it would result in a violation of petitioner's right to security of tenure
guaranteed in Section 3 of Article XIII of the Constitution and in Articles 279 and 281 of the
Labor Code.
- Regardless of the designation petitioner may have conferred upon respondent's
employment status, it is, however, uncontroverted that the latter, having completed the
probationary period and allowed to work thereafter, became a regular employee who may
be dismissed only for just or authorized causes under Articles 282, 283 and 284 of the
Labor Code, as amended. Therefore, the dismissal, premised on the alleged expiration of
the contract, is illegal and entitles respondent to the reliefs prayed for.
Disposition The petition is hereby DISMISSED and the decision of the National Labor
Relations Commission dated November 28. 1994 is AFFIRMED.

PANGILINAN V GENERAL MILLING CORPORATION


434 SCRA 159
CALLEJO, SR; July 12, 2004
NATURE
Petition for review on certiorari of a decision of the Court of Appeals
FACTS
- Respondent General Milling Corporation is a domestic corporation engaged in the
production and sale of livestock and poultry. It is, likewise, the distributor of dressed
chicken to various restaurants and establishments nationwide.
- Petitioners were employed by the respondent as emergency workers under separate
"temporary/casual contracts of employment" for a period of five months.
- Upon the expiration of their respective contracts, their services were terminated.
- They later filed separate complaints for illegal dismissal and non-payment of holiday pay,
13th month pay, night-shift differential and service incentive leave pay against the
respondent before the Arbitration Branch of the National Labor Relations Commission,
- Petitioners alleged that their work as chicken dressers was necessary and desirable in
the usual business of the respondent, and added that although they worked from 10:00
p.m. to 6:00 a.m., they were not paid night-shift differential.
- They stressed that based on the nature of their work, they were regular employees of
the respondent; hence, could not be dismissed from their employment unless for just
cause and after due notice.
- Labor Arbiter Voltaire A. Balitaan rendered a decision in favor of the petitioners declaring
that they were regular employees.
- Finding that the termination of their employment was not based on any of the just causes
provided for in the Labor Code, the Labor Arbiter declared that they were allegedly illegally
dismissed.
- On May 25, 1998, the NLRC rendered a decision reversing that of the Labor Arbiter
- The NLRC held that the petitioners, who were temporary or contractual employees of the
respondent, were legally terminated upon the expiration of their respective contracts.
Citing the case of Brent School, Inc. vs. Zamora, the NLRC explained that while the
petitioners' work was necessary and desirable in the usual business of GMC, they cannot
be considered as regular employees since they agreed to a fixed term.
- The petitioners' motion for reconsideration of the decision having been denied by the
NLRC, they filed a petition for certiorari before the Court of Appeals.
- On September 29, 2000, the CA rendered a decision affirming decision of the NLRC
- The CA ruled that where the duties of the employee consist of activities usually
necessary or desirable in the usual business of the employer, it does not necessarily
follow that the parties are forbidden from agreeing on a period of time for the performance
of such activities.
- Petitioners MFR was denied, hence, this petition
ISSUE
WON the petitioners were regular employees of the respondent GMC when their
employment was terminated

A2010

- 74 -

Disini

HELD
NO
- Petitioners were employees with a fixed period, and, as such, were not regular
employees.
- Article 280 of the Labor Code comprehends three kinds of employees: (a) regular
employees or those whose work is necessary or desirable to the usual business of
the employer; (b) project employees or those whose employment has been fixed for
a specific project or undertaking the completion or termination of which has been
determined at the time of the engagement of the employee or where the work or
services to be performed is seasonal in nature and the employment is for the
duration of the season; and, (c) casual employees or those who are neither regular
nor project employees.
- A regular employee is one who is engaged to perform activities which are necessary and
desirable in the usual business or trade of the employer as against those which are
undertaken for a specific project or are seasonal.
- There are two separate instances whereby it can be determined that an employment is
regular: (1) if the particular activity performed by the employee is necessary or desirable in
the usual business or trade of the employer; and, (2) if the employee has been performing
the job for at least a year.
- In the case of St. Theresa's School of Novaliches Foundation vs. NLRC, we held that
Article 280 of the Labor Code does not proscribe or prohibit an employment
contract with a fixed period. We furthered that it does not necessarily follow that
where the duties of the employee consist of activities usually necessary or
desirable in the usual business of the employer, the parties are forbidden from
agreeing on a period of time for the performance of such activities. There is thus
nothing essentially contradictory between a definite period of employment and the
nature of the employee's duties.
- In the case of Brent School Inc. v. Zamora, the SC laid down the guideline before a
contract of employment may be held as valid, to wit: Stipulations in employment contracts
providing for term employment or fixed period employment are valid when the period were
agreed upon knowingly and voluntarily by the parties without force, duress or improper
pressure, being brought to bear upon the employee and absent any other circumstances
vitiating his consent, or where it satisfactorily appears that the employer and employee
dealt with each other on more or less equal terms with no moral dominance whatever
being exercised by the former over the latter.
- An examination of the contracts entered into by the petitioners showed that their
employment was limited to a fixed period, usually five or six months, and did not go
beyond such period.
- The records reveal that the stipulations in the employment contracts were knowingly and
voluntarily agreed to by the petitioners without force, duress or improper pressure, or any
circumstances that vitiated their consent. Similarly, nothing therein shows that these
contracts were used as a subterfuge by the respondent GMC to evade the provisions of
Articles 279 and 280 of the Labor Code.
- The petitioners were hired as "emergency workers" and assigned as chicken dressers,
packers and helpers at the Cainta Processing Plant.
- While the petitioners' employment as chicken dressers is necessary and desirable in the
usual business of the respondent, they were employed on a mere temporary basis, since
their employment was limited to a fixed period. As such, they cannot be said to be regular
employees, but are merely "contractual employees."
- Consequently, there was no illegal dismissal when the petitioners' services were
terminated by reason of the expiration of their contracts.
- Lack of notice of termination is of no consequence, because when the contract specifies
the period of its duration, it terminates on the expiration of such period. A contract for
employment for a definite period terminates by its own term at the end of such period.
Disposition Petition is denied.

DE LEON V NLRC (LA TONDENA)


176 SCRA 615
FERNAN; August 21, 1989
NATURE
Petition for certiorari seeking to annul and set aside: (1) majority decision of the NLRC,
which reversed the Order of Labor Arbiter Hernandez; and, (2) the Resolution denying
petitioner's MFR
FACTS
- DE LEON was employed by LA TONDENA (business of manufacture and distillery of
wines and liquors) on Dec 11, 1981, at the Maintenance Section of its Engineering Dept in
Tondo.
- His work consisted mainly of painting company building and equipment, and other odd
jobs relating to maintenance. He was paid on a daily basis through petty cash vouchers.
- After service of more than 1 year, DE LEON requested that he be included in the payroll
of regular workers. LA TONDENA responded by dismissing him from work.

Labor Law 1
- Weeks after this, he was re-hired indirectly through the Vitas-Magsaysay Village
Livelihood Council, a labor agency of respondent, and was made to perform tasks he used
to do.
- Having been refused reinstatement despite repeated demands, petitioner filed a
complaint before the Office of the Labor Arbiter.
- LA TONDENA claimed he was a casual worker hired only to paint a certain bldg in the
company premises, and such work terminated upon completion of the painting job.
- Labor Arbiter Hernandez ordered reinstatement and payment of backwages to petitioner.
Complainant's being hired on casual basis did not dissuade from the cold fact that such
jobs he performed related to maintenance as a maintenance man is necessary and
desirable to the better operation of the business company.
- On appeal, NLRC reversed such decision because his job cannot be considered
necessary in the usual trade of employer: "Painting the business or factory building is not
a part of the respondent's manufacturing or distilling process of wines and liquors.
ISSUE
WON petitioner is a regular employee

HELD
1. YES
Ratio An employment shall be deemed to be casual if it is not covered by Art.281 13 of
Labor Code: provided, That any employee who has rendered at least one year of service,
whether such service is continuous or broken, shall be considered a regular employee
with respect to the activity in which he is employed and his employment shall continue
while such actually exists.
Reasoning
- During petitioner's period of employment, the records reveal that the tasks assigned to
him included not only painting of company buildings, equipment and tools but also
cleaning and oiling machines, even operating a drilling machine, and other odd jobs
assigned to him when he had no painting job.
- It is not the will and word of the employer that determines whether a certain employment
is regular or casual, to which the desperate worker often accedes, but the nature of the
activities performed in relation to the particular business or trade considering all
circumstances, and in some cases the length of time of its performance and its continued
existence.
Disposition Petition is GRANTED.

SAN MIGUEL CORPORATION V NLRC (GUZMAN)


297 SCRA 277
QUISUMBING; October 7, 1998
NATURE
Petition for certiorari.
FACTS
- In November 1990, Francisco De Guzman, JR. was hired by SMC as helper/bricklayer
for a specific project, the repair and upgrading of furnace C at its Manila Glass Plant. His
contract of employment provided that said temporary employment was for a specific
period of approximately 4 months. On April 30, 1991, De Guzman was able to complete
the repair and upgrading of furnace C. Thus, his services were terminated on that same
day as there was no more work to be done. His employment contract also ended that day.
- On May 10, 1991, De Guzman was again hired for a specific job which involved the
draining/cooling down of fuenace F and the emergency repair of furnace E. This project
was for a specific period of approximately 3 months. After the completion of this task, at
the end of July 1991, DE Guzman's services were terminated.
- On Aug.1, 1991, complainant saw his name in a Memorandum posted at the Company's
Bulletin Board as among those who were considered dismissed.
- On Aug.12, 1994, or after the lapse of more than 3 years from the completion of the last
undertaking for which De Guzman was hired, he filed a complaint for illegal dismissal
against SMC.
- On June 30, 1995, labor Arbiter Felipe Garduque II rendered the decision dismissing said
complaint for lack of merit, sustaining SMC's argument that DE Guzman was a project
employee. The position of a helper does not fall within the classification of regular
13

Art. 281. Regular and casual employment. The provisions of a written agreement to the contrary notwithstanding and
regardless of the oral agreements of the parties, an employment shall be deemed to be regular where the employee has
been engaged to perform activities which are usually necessary or desirable in the usual business or trade of the
employer, except where the employment has been fixed for a specific project or undertaking the completion or
termination of which has been determined at the time of the engagement of the employee or where the work or services
to be performed is seasonal in nature and the employment is for the duration of the season.

A2010

- 75 -

Disini

employees. Hence, complainant never attained regular employment status. Moreover, his
silence for more than three (3) years without any reasonable explanation tended to
weaken his claim.
- Upon appeal, NLRC reversed Labor Arbiter Garduque's decision. In its ruling, NLRC
stated that SMCs scheme of subsequently re-hiring complainant after only 10 days from
the last day of the expiration of his contract of employment for a specific period, and giving
him again another contract of employment for another specific period cannot be
countenanced. This is one way of doing violence to the employee's constitutional right to
security of tenure under which even employees under probationary status are amply
protected.
- SMCs MFR was denied by NLRC. Hence, this petition.
ISSUES
1. WON De Guzman is a regular employee
2. WON De Guzman was illegally dismissed
HELD
1. NO
Art. 280 of the Labor Code defines regular, project and casual employment as follows:
An employment shall be deemed to be regular where the employee has been
engaged to perform activities which are usually necessary or desirable in the usual
business or trade of the employer, except where the employment has been fixed for a
specific project or undertaking the completion or termination of which has been
determined at the time of the engagement of the employee or where the work or
services to be performed is seasonal in nature and the employment is for the duration
of the season.
An employment shall be deemed to be casual if it is not covered by the preceding
paragraph: Provided, That, any employee who has rendered at least one year of
service, whether such service is continuous or broken, shall be considered a regular
employee with respect to the activity in which he is employed and his employment shall
continue while such actually exists.
- The above mentioned provision reinforces the Constitutional mandate to protect the
interest of labor as it sets the legal framework for ascertaining one's nature of
employment, and distinguishing different kinds of employees. Its language manifests the
intent to safeguard the tenurial interest of worker who may be denied the enjoyment of the
rights and benefits due to an employee, regardless of the nature of his employment, by
virtue of lopsided agreements which the economically powerful employer who can
maneuver to keep an employee on a casual or contractual status for as long as it is
convenient to the employer.
- Thus, under Article 280 of the Labor Code, an employment is deemed regular when the
activities performed by the employee are usually necessary or desirable in the usual
business or trade of the employer even if the parties enter into an agreement stating
otherwise. But considered not regular under said Article (1) the so-called "project
employment" the termination of which is more or less determinable at the time of
employment, such as those connected, which by its nature is only for one season of the
year and the employment is limited for the duration of that season, such as the Christmas
holiday season. Nevertheless, an exception to this exception is made: any employee who
has rendered at least 1 year of service, whether continuous or intermitent, with respect to
the activity he performed and while such activity actually exists, must be deemed regular.
- Following Article 280, whether one is employed as a project employee or not would
depend on whether he was hired to carry out a "specific project or undertaking", the
duration and scope of which were specified at the time his services were engaged for that
particular project. Another factor that may be undertaken by the employee in relation to the
usual trade or business of the employer, if without specifying the duration and scope, the
work to be undertaken is usually necessary or desirable in the usual business or trade of
the employer, then it is regular employment and not just "project" must less "casual"
employment.
- Thus, the nature of one's employment does not depend on the will or word of the
employer. Nor on the procedure of hiring and the manner of designating the employee, but
on the nature of the activities to be performed by the employee, considering the
employer's nature of business and the duration and scope of the work to be done.
- Project could refer to 2 distinguishable types of activity. Firstly, a project could refer to a
particular job or undertaking that is within the regular or usual business of the employer
company, but which is distinct at separate, and identifiable as such, from the other
undertakings of the company. Such job or undertaking begins and ends at determined or
determinable times. . . . Secondly, a project could refer to a particular job or undertaking
that is not within the regular business of the corporation. Such a job or undertaking must
also be identifiably separate and distinct from the ordinary or regular business operations
of the employer. The job or undertaking also begins and ends at determined or
determinable times . .
- The plant where De Guzman was employed for only 7 months is engaged in the
manufacturer of glass, an integral component of the packaging and manufacturing
business of petitioner. The process of manufacturing glass requires a furnace, which has a
limited operating life. SMC resorted to hiring project or fixed term employees in having
said furnaces repaired since said activity is not regularly performed. Said furnaces are to

Labor Law 1
be repaired or overhauled only in case of need and after being used continuously for a
varying period of 5-10 years. In 1990, one of the furnaces of petitioner required repair and
upgrading. This was an undertaking distinct and separate from SMC's business of
manufacturing glass. For this purpose, SMC must hire workers to undertake the said
repair and upgrading. De Guzman was, thus, hired by SMC on November 28, 1990 on a
"temporary status for a specific job" for a determined period of approximately four months
- Upon completion of the undertaking, or on April 30, 1991, DE Guzman's services were
terminated. A few days, thereafter, two of SMC's furnaces required "draining/coolong
down" and "emergency repair". De Guzman was again hired on May 10, 1991 to help in
the new undertaking, which would take approximately 3 months to accomplish. Upon
completion of the second undertaking, private respondent's services were likewise
terminated. He was not hired a third time, and his two engagements taken together did
not total one full year in order to qualify him as an exception to the exception falling under
the cited proviso in the second paragraph of Art. 280 of the Labor Code.
2. NO
- De Guzman was hired for a specific project that was not within the regular business of
the corporation. For SMC is not engaged in the business of repairing furnaces. Although
the activity was necessary to enable petitioner to continue manufacturing glass, the
necessity therefor arose only when a particular furnace reached the end of its life or
operating cycle. Or, as on the second undertaking, when a particular furnace required an
emergency repair. In other words, the undertakings where he was hired primarily as
helper/bricklayer have specified goals and purpose which are fulfilled once the designated
work was completed. Moreover, such undertakings were also identifiably separate and
distinct from the usual, ordinary or regular business operations of petitioner, which is glass
manufacturing. These undertakings, the duration and scope of which had been
determined and made known to private respondent at the time of his employment clearly
indicated the nature of his employment as a project employee. Thus, his services were
terminated legally after the completion of the project.
- If NLRCs decision is upheld, it would amount to negating the distinction made in Article
280 of the Labor Code. It would shunt aside the rule that since a project employee's work
defends on the availability of a project, necessarily, the duration of his employment is
coterminous with the project to which he is assigned. It would become a burden for an
employer to retain an employee and pay him his corresponding wages it there was no
project for him to work on.
- While the Constitution is committed to the policy of social justice and the protection of
the working class, it should not be supposed that every dispute will be automatically
decided in favor of labor. Management has also rights, which, as such, are entitled to
respect and enforcement in the interest of fair play. Although the SC has inclined more
often than not toward the worker and has upheld has cause in his conflicts with the
employer, such favoritism has no blinded the Court to the rule that justice is in avery case
for the deserving, to be dispensed in the light of the established facts and the applicable
law and doctrine.
Disposition Petition is hereby GRANTED. The decision of respondent NLRC is hereby
REVERSED, and the judgment of the Labor Arbiter REINSTATED.

TABAS V NLRC (CALIFORNIA MANUFACTURING)


169 SCRA 497
SARMIENTO; January 26, 1989
NATURE
PETITION to review the decision and resolution of the National Labor Relations
Commission.
FACTS
- Petitioners were employees of Livi Manpower Services, Inc. (Livi). Livi subsequently
assigned them to work as promotional merchandisers for California Manufacturing Co.
(California) pursuant to a manpower supply agreement. The agreement provided the
following, among others: (1) that California had no control/supervision over the petitioners
with respect to how they accomplish their work; (2) that Livi is an independent contractor
and that the relationship between Livi and California should not be construed to be of
principal-agent or employer-employee; (3) that California is free and harmless (!?!) from
any liability arising from such laws or from any accident that may befall the workers and
employees of Livi while in the performance of their duties for California; (4) that the
assignment of workers to California shall be on a seasonal and contractual basis; (5) that
most of living allowance and the 10 legal holidays will be charged directly to California at
cost; and (6) that the payroll for the preceding week shall be delivered by LIvi at
Californias premises.
- Petitioners were then made to sign employment contracts with durations of six
months, upon the expiration of which they signed new agreements with the same
period, and so on. Unlike regular California employees, who received not less than
P2,823.00 a month in addition to a host of fringe benefits and bonuses, they received
P38.56 plus P15.00 in allowance daily.

A2010

- 76 -

Disini

- Petitioners filed complaints, demanding to have similar benefits as regular employees;


but pending their claims, California notified them that they would not be rehired. California
then amended their complaint charging California with illegal dismissal. Thereafter, Livi
reabsorbed them into its labor pool on a wait-in or standby status.
- Respondents claim: they are not the petitioners employer (Livi is, therefore, no
employer-employee relationship between them) and that the "retrenchment" had been
forced by business losses as well as expiration of contracts ("unfavorable political
and economic atmosphere coupled by the February Revolution.")
- LA: no employer-employee relationship in the light of the manpower supply contract;
California not liable for the money claims demanded. Livi also absolved from any
obligation because retrenchment was allegedly beyond its control, but were to pay
separation pay and attorneys fees.
- NLRC: affirm labor arbiters deci
ISSUES
1. WON the petitioners are employees of California Manufacturing Company
2. WON the petitioners were illegally dismissed
HELD
1. YES
Ratio The existence of an employer-employees relation is a question of law and being
such, it cannot be made the subject of agreement. The determination of whether or not
there is an employer-employee relation depends upon four standards: (1) the manner of
selection and engagement of the putative employee; (2) the mode of payment of wages;
(3) the presence or absence of a power of dismissal; and (4) the presence or absence of a
power to control the putative employee's conduct. Of the four, the right-of-control test has
been held to be the decisive factor.
Reasoning
- IN RELATION TO THE MANPOWER SUPPLY AGREEMENT: The fact that the manpower supply
agreement between Livi and California had specifically designated the former as the
petitioners' employer and had absolved the latter from any liability as an employer, will not
erase either party's obligations as an employer, if an employer-employee relation
otherwise exists between the workers and either firm. At any rate, since the agreement
was between Livi and California, they alone are bound by it, and the petitioners cannot be
made to suffer from its adverse consequences.
- Art. 106 of the Labor Code 14 still imposes responsibility on both firms: Notwithstanding
the absence of a direct employer-employee relationship between the employer in whose
favor work had been contracted out by a "labor-only" contractor, and the employees, the
former has the responsibility, together with the "labor-only" contractor, for any valid labor
claims by operation of law. The reason, so we held, is that the "labor-only" contractor is
considered "merely an agent of the employer," and liability must be shouldered by either
one or shared by both.
- Livi, as a placement agency, had simply supplied California with the manpower
necessary to carry out Californias merchandizing activities, using the latters premises
and equipment.
- ON PETITIONERS BEING DIRECT EMPLOYEES OF LIVI: Not conclusive will not absolve
California from liability imposed by law and relations of parties are not determined by their
declarations
- ON TEMPORARY OR SEASONAL BASIS HIRING : temporary or casual employee, under Article
218 of the Labor Code, becomes regular after service of one year, unless he has been
contracted for a specific project. Merchandising is not a specific project, it is an activity
related to the day-to-day operations of California.
*The Court need not therefore consider whether it is Livi or California which exercises
control over the petitioner vis-a-vis the four barometers reffered to earlier, since by fiction
of law, either or both shoulder responsibility.
-REASONING FOR DISMISSING THE TERMS AND CONDITIONS OF THE MANPOWER SUPPLY
AGREEMENT: not illegal, under the Labor Code, genuine job contracts are permissible,
provided they are genuine job contracts. But when such arrangements are resorted to
"in anticipation of, and for the very purpose of making possible, the secondment"of the
employees from the true employer, the Court will be justified in expressing its concern. For
then that would compromise the rights of the workers, especially their right to security of
tenure.
2. YES
Ratio Retrenchment of workers, unless clearly warranted, has serious consequences not
only on the State's initiatives to maintain a stable employment record for the country, but
more so, on the workingman himself, amid an environment that is desperately scarce in
jobs.
14 ART. 106. Contractor or subcontractor.-Whenever an employee enters into a contract with another person for the performance of the former's work, the employees of the contractor and of the
letter's subcontractor, if any, shall be paid in accordance with the provisions of this Code.
In the event that the contractor or subcontractor fails to pay wages of his employees in accordance with this Code, the employer shall be jointly and severally liable with his contractor or subcontractor
to such employees to the extent of the work performed under the contract, in the same manner and extent that he is liable to employees directly employed by him.
The Secretary of Labor may, by appropriate regulations, restrict or prohibit the contracting out of labor to protect the rights of workers established under this Code. In so prohibiting or restricting, he may
make appropriate distinctions between labor-only contracting and job contracting as well as differentiations within these types of contracting and determine who among the parties involved shall be
considered the employer for purposes of this Code, to prevent any violation or circumvention of any provisions of this Code.
There is "labor-only" contracting where the person supplying workers to an employer does not have substantial capital or investment in the form of tools, equipment, machineries, work premises, among
others, and the workers recruited and placed by such person are performing activities which are directly related to the principal business of such employer. In such cases, the person or intermediary shall be
considered merely as an agent of the employer who shall be responsible to the workers in the same manner and extent as if the latter were directly employed by him.

Labor Law 1
Reasoning The 6-month contracts of the petitioners were renewed, and accordingly,
under Article 281 (Labor Code), they had become regular employees of California and had
acquired a secure tenure. Hence, they cannot be separated without due process of law.
- ON VALIDITY OF RETRENCHMENT: California has not shown enough evidence that it had in
fact suffered serious business reverses as a result alone of the prevailing political and
economic climate; attribution to February Revolution as cause of alleged losses gratuitous
and without basis in fact.
Disposition petition is GRANTED. Judgment is hereby RENDERED: (1) SETTING
ASIDE the decision, dated March 20, 1987, and the resolution, dated August 19, 1987; (2)
ORDERING the respondent, the California Manufacturing Company, to REINSTATE the
petitioners with fall status and rights of regular employees; and (3) ORDERING the
respondent, the California Manufacturing Company, and the respondents, Livi Manpower
Service, Inc. and/or Lily-Victoria A. Azarcon, to PAY, jointly and severally, unto the
petitioners: (a) backwages and differential pays effective as and from the time they had
acquired a regular status under the second paragraph, of Section 281, of the Labor Code,
but not to exceed three (3) years, and (b) all such other and further benefits as may be
provided by existing collective bargaining agreement(s) or other relations, or by law,
beginning such time; and (4) ORDERING the private respondents to PAY unto the
petitioners attorney's fees equivalent to ten (10%) percent of all money claims hereby
awarded, in addition to those money claims.

PHILIPS SEMICONDUCTORS V FADRIQUELA


427 SCRA 408
CALLEJO, SR; April 14, 2004
FACTS
- On May 8, 1992, respondent Eloisa Fadriquela executed a Contract of Employment with
petitioner Philips Semiconductors as a production operator, initially for 3 months. Because
her performance constantly met petitioners ratings requirements, her contract was
renewed several times, extending to 12 months. However, over the last few months,
respondent incurred several absences for which she offered no valid justification despite a
prompting to do so by the line supervisor. As a consequence, her performance rating
dropped, and respondents contract was no longer renewed.
- Respondent filed a complaint with the NLRC for illegal dismissal, claiming she had not
been duly notified; she furthered that having rendered over 6 months of service, she was
already a regular employee and could not be terminated without just cause.
- Petitioner contended that respondent had not been dismissed; rather, her contract
merely expired and was not renewed.
- The Labor Arbiter dismissed the complaint for lack of merit but awarded her severance of
1 months pay. He stated that petitioner and its unions CBA required one to render 17
months of service to be considered regular. He also added that respondent could not
complain of being deprived of notice and hearing as the line supervisor had asked her to
explain her absences. An appeal with the NLRC yielded the same results. It was pointed
out that as a contractual employee respondent was bound by the stipulations of her
contract of employment, which in this case was a satisfactory performance rating.
- Dissatisfied, respondent filed a petition for certiorari before the CA, which reversed the
decisions of the NLRC and the Labor Arbiter. The appellate court argued that the NLRC
and the Labor Arbiter employed inappropriate bases for their decisions, since the CBA did
not apply to contractual employees like Fadriquela. The CA cited Art. 280 of the Labor
Code which states that regardless of any written or oral agreements between employer
and employee, an employment shall be deemed to be regular where the employee has
been engaged to perform activities which are usually necessary or desirable in the usual
business or trade of the employer. Petitioners contention that employment was obtained
as the need arose was illogical, as this would mean the employee would never attain
regular status. The CA further held that a less punitive penalty would suffice for
absenteeism. Finally, it held that the dialogue between the respondent and line supervisor
was insufficient as to amount to notice, and thus the former was deprived of due process.
- Petitioner filed a motion for reconsideration in which petitioner claimed that its hiring
policy was neither new nor prohibited and that it was a valid exercise of its management
prerogative since demand for its semiconductors is cyclical in nature. It added that it had
the prerogative to set reasonable standards of employment qualification as provided by
law. The motion was denied, hence this petition for review.
ISSUES
1. WON the NLRC and Labor Arbiter erred in not finding respondent to be a regular
employee
2. WON the CBA applies to respondent
3. WON respondent was deprived of due process
4. WON dismissal was a just penalty
HELD
1. YES

A2010

- 77 -

Disini

- According to Article 280 of the Labor Code, there are 2 kinds of regular employees: (1)
those engaged to perform activities which are necessary or desirable in the usual
business or trade of the employer; and (2) those casual employees who have rendered at
least one year of service, whether continuous or broken, with respect to the activities in
which they are employed. The respondent obviously falls under the first type of regular
employee. She had been working continuously for the petitioner for over a year,
evidencing the necessity and indispensability of her services to the petitioners business.
By operation of law, respondent had attained regular status and was thus entitled to
security of tenure as provided in Art. 279 of the code. The said article requires a just cause
before termination, and entitles the employee to reinstatement and other privileges in
absence of one.
- Petitioners hiring policy for contract employees is contrary to the spirit of Articles 279
and 280 of the code; it is but an excuse to prevent regularization and circumvent the law
on security of tenure. This is echoed in Sec. 3 Art XVI of the Constitution which deems
security of tenure a State policy to guarantee social justice. The fact is that the operation
of every business depends on supply and demandthe cyclical nature of ones trade
cannot be invoked as a reason to place an employees status on shaky ground.
- This is not to say that term employment is illegal outright. In Romares v NLRC it was said
that term employment does not circumvent the law when the fixed period was knowingly
and voluntarily agreed upon by both parties and that such agreement was made with no
party holding moral dominance over the other. However, none of these requisites are
present in the instant case.
2. NO
- Petitioners reliance on the CBA is misplaced. The CBA constitutes the law between the
employer and regular employees, but cannot be binding on contractual employees who
are not represented by the bargaining union. The CBA provision requiring 17 months for
regularization runs contrary to what is clearly stipulated in law, which provides that
regularization requires only 1 year.
3. YES
- Respondent was dismissed without the requisite notice and formal investigation.
Dismissals must not be arbitrary and capricious; a mere dialogue between the respondent
and line supervisor cannot possibly suffice as a substitute for actual notice and hearing.
4. NO
- Dismissal is too harsh a penalty for mere absences, especially since the repeated
renewal of respondents contract proves her efficiency as a worker. The SC mandates that
where a penalty less punitive would suffice, whatever missteps may be committed by
labor ought not to be visited with a consequence so severe.
Disposition IN LIGHT OF ALL THE FOREGOING, the assailed decision of the appellate
court is AFFIRMED. The petition at bar is DENIED.

MAGSALIN V NATIONAL ORGANIZATION OF


WORKING MEN (NOWM)
403 SCRA 199
VITUG; May 9, 2003
FACTS
- Coca-Cola Bottlers Phils., Inc., herein petitioner, engaged the services of respondent
workers as sales route helpers for a limited period of five months. After five months,
respondent workers were employed by petitioner company on a day-to-day basis to
substitute for regular sales route helpers whenever the latter would be unavailable or
when there would be an unexpected shortage of manpower in any of its work places or an
unusually high volume of work. The practice was for the workers to wait every morning
outside the gates of the sales office of petitioner company. If thus hired, the workers
would then be paid their wages at the end of the day.
- Ultimately, respondent workers asked petitioner company to extend to them regular
appointments. Petitioner company refused.
- November 7, 1997 - twenty-three (23) of the temporary workers (herein respondents)
filed with the National Labor Relations Commission (NLRC) a complaint for the
regularization of their employment with petitioner company. The complaint was
amended a number of times to include other complainants that ultimately totaled fifty-eight
(58) workers. Claiming that petitioner company meanwhile terminated their services,
respondent workers filed a notice of strike and a complaint for illegal dismissal and unfair
labor practice with the NLRC.
- 01 April 1998 - voluntary arbitration
- 18 May 1998 - the voluntary arbitrator rendered a decision dismissing the complaint on
the thesis that respondents (then complainants) were not regular employees of petitioner
company.
- 11 August 2000, the Court of Appeals reversed and set aside the ruling of the voluntary
arbitrator ; (Petitioners were declared regular employees of Coca Cola Bottlers; dismissal
illegal; ordered to reinstate the workers)
ISSUES
1. WON the nature of work of respondents in the company is of such nature as to be

Labor Law 1
deemed necessary and desirable in the usual business or trade of petitioner that could
qualify them to be regular employees
2. WON the quitclaims executed by the 36 individual respondents were valid
HELD
1. YES
Ratio In determining whether an employment should be considered regular or nonregular, the applicable test is the reasonable connection between the particular activity
performed by the employee in relation to the usual business or trade of the employer.
Reasoning
a. Intentionalist approach - Even while the language of law (Art 280) 15 might have been
more definitive, the clarity of its spirit and intent, i.e., to ensure a regular workers security
of tenure, however, can hardly be doubted.
b. Although the work to be performed is only for a specific project or seasonal, where a
person thus engaged has been performing the job for at least one year, even if the
performance is not continuous or is merely intermittent, the law deems the repeated and
continuing need for its performance as being sufficient to indicate the necessity or
desirability of that activity to the business or trade of the employer. The employment of
such person is also then deemed to be regular with respect to such activity and while such
activity exists.
c. The postproduction activities done by sales route helpers are important. The nature of
the work performed must be viewed from a perspective of the business or trade in its
entirety and not on a confined scope.
d. The repeated rehiring of respondent workers and the continuing need for their services
clearly attest to the necessity or desirability of their services in the regular conduct of the
business or trade of petitioner company.
e. A contract of employment is impressed with public interest. The provisions of
applicable statutes are deemed written into the contract, and the parties are not at liberty
to insulate themselves and their relationships from the impact of labor laws and
regulations by simply contracting with each other.
2. YES
Ratio While quitclaims executed by employees are commonly frowned upon as being
contrary to public policy and are ineffective to bar claims for the full measure of their legal
rights, there are, however, legitimate waivers that represent a voluntary and reasonable
settlement of laborers claims which should be so respected by the Court as the law
between the parties. Where the person making the waiver has done so voluntarily, with a
full understanding thereof, and the consideration for the quitclaim is credible and
reasonable, the transaction must be recognized as being a valid and binding undertaking.
Dire necessity is not an acceptable ground for annulling the release, when it is not shown
that the employee has been forced to execute it.
Disposition Questioned decision of the Court of Appeals, is AFFIRMED with
MODIFICATION in that the Release, Waiver and Quitclaim executed by the thirty-six (36)
individual respondents are hereby declared VALID and LEGAL.

HACIENDA FATIMA V NATIONAL FEDERATION OF


SUGARCANE WORKERS-FOOD AND
GENERAL TRADE
396 SCRA 518
PANGANIBAN; January 28, 2003
NATURE
Before the Court is a Petition for Review under Rule 45 of the Rules of Court, seeking to
set aside CA Decision denying petition for certiorari the Decision of NLRC. NLRC set
aside and vacated the Labor Arbiters finding that there was no illegal dismissal.
FACTS

- According to the Labor arbiter, the respondents


refused to work and/or were choosy in the kind of jobs
15

Art. 280.
Regular and Casual Employment. The provisions of written agreement to
the contrary notwithstanding and regardless of the oral agreement of the parties, an employment shall be
deemed to be regular where the employee has been engaged to perform activities which are usually
necessary or desirable in the usual business or trade of the employer, except where the employment has been
fixed for a specific project or undertaking the completion or termination of which has been determined at the
time of the engagement of the employee or where the work or services to be performed is seasonal in nature
and the employment is for the duration of the season.
An employment shall be deemed to be casual if it is not covered by the preceding paragraph: Provided,
That, any employee who has rendered at least one year of service, whether such service is continuous or
broken, shall be considered a regular employee with respect to the activity in which he is employed and his
employment shall continue while such activity exists.

Disini

A2010 - 78 they wanted to perform. NLRC found that the record


is replete with the workers persistence and
determination of going back to work.

- When the union was certified as the collective bargaining representative in the
certification elections, Hacienda Fatima under the pretext that the result was on appeal,
refused to sit down with the union for the purpose of entering into a CBA. Moreover, the
workers were not given work for more than one month. In protest, the Union staged a
strike which was however settled upon the signing of a Memorandum of Agreement.
- When Company again reneged on its commitment, Union filed the complaint. For all their
persistence, the risk they had to undergo in conducting a strike, complainants now find
themselves being accused of refusing to work and being choosy in the kind of work they
have to perform.
- The CA affirmed that while the work of respondents was seasonal in nature, they were
considered to be merely on leave during the off-season and were therefore still employed
by petitioners. Moreover, the workers enjoyed security of tenure. Any infringement upon
this right was deemed by the CA to be tantamount to illegal dismissal. Hence this Petition.

ISSUES
1. WON CA erred in holding that respondents, admittedly seasonal workers, were regular
employees, contrary to the clear provisions of Article 280 16 of the Labor Code, which
categorically state that seasonal employees are not covered by the definition of regular
employees under paragraph 1, nor covered under paragraph 2 which refers exclusively to
casual employees who have served for at least one year
2. WON CA committed grave abuse of discretion in upholding the NLRCs conclusion that
private respondents were illegally dismissed, that petitioner[s were] guilty of unfair labor
practice, and that the union be awarded moral and exemplary damages.
HELD
1. NO, the CA did not err when it held that respondents were regular employees.
- The fact that respondents do not work continuously for one whole year but only for the
duration of the season does not detract from considering them in regular employment
since in a litany of cases this Court has already settled that seasonal workers who are
called to work from time to time and are temporarily laid off during off-season are not
separated from service in said period, but merely considered on leave until re-employed.
- For respondents to be excluded from those classified as regular employees, it is not
enough that they perform work or services that are seasonal in nature. They must have
also been employed only for the duration of one season. The evidence proves the
existence of the first, but not of the second, condition. The fact that respondents -- with
the exception of Luisa Rombo, Ramona Rombo, Bobong Abriga and Boboy Silva -repeatedly worked as sugarcane workers for petitioners for several years is not denied by
the latter. Evidently, petitioners employed respondents for more than one season.
Therefore, the general rule of regular employment is applicable.
- The test of WON an employee is a regular employee has been laid down in De Leon
v. NLRC, in which this Court held:
- The primary standard of determining regular employment is the reasonable connection
between the particular activity performed by the employee in relation to the usual trade or
business of the employer. The test is whether the former is usually necessary or desirable
in the usual trade or business of the employer. The connection can be determined by
considering the nature of the work performed and its relation to the scheme of the
particular business or trade in its entirety. Also if the employee has been performing the
job for at least a year, even if the performance is not continuous and merely intermittent,
the law deems repeated and continuing need for its performance as sufficient evidence of
the necessity if not indispensability of that activity to the business. Hence, the
employment is considered regular, but only with respect to such activity and while such
activity exists. (Abasolo v. National Labor Relations Commission)
- The sudden changes in work assignments reeked of bad faith. These changes were
implemented immediately after respondents had organized themselves into a union and
started demanding collective bargaining. Those who were union members were
16

Art. 280. Regular and Casual Employment. - The provisions of written agreement to the contrary notwithstanding and
regardless of the oral agreement of the parties, an employment shall be deemed to be regular where the employee has
been engaged to perform activities which are usually necessary or desirable in the usual business or trade of the
employer, except where the employment has been fixed for a specific project or undertaking the completion or
termination of which has been determined at the time of the engagement of the employee or where the work or services
to be performed is seasonal in nature and the employment is for the duration of the season.
An employment shall be deemed to be casual if it is not covered by the preceding paragraph: Provided, That, any
employee who has rendered at least one year of service, whether such service is continuous or broken, shall be
considered a regular employee with respect to the activity in which he is employed and his employment shall continue
while such activity exist.

Labor Law 1
effectively deprived of their jobs. Petitioners move actually amounted to unjustified
dismissal of respondents, in violation of the Labor Code.
2. NO
- Factual findings of labor officials, who are deemed to have acquired expertise in matters
within their respective jurisdictions, are generally accorded not only respect but even
finality. Their findings are binding on the Supreme Court. Verily, their conclusions are
accorded great weight upon appeal, especially when supported by substantial evidence.
Consequently, the Court is not duty-bound to delve into the accuracy of their factual
findings, in the absence of a clear showing that these were arbitrary and bereft of any
rational basis.
- The NLRC found herein petitioners guilty of unfair labor practice. It ruled that from
respondents refusal to bargain, to their acts of economic inducements resulting in the
promotion of those who withdrew from the union, the use of armed guards to prevent the
organizers to come in, and the dismissal of union officials and members, one cannot but
conclude that respondents did not want a union in their haciendaa clear interference in
the right of the workers to self-organization.
Disposition Petition is hereby DENIED and the assailed Decision AFFIRMED. Costs
against petitioners.

MILLARES V NLRC (TRANS-GLOBAL MARITIME


AGENCY, ESSO INTERNATIONAL SHIPPING
COMPANY)
485 SCRA 307
KAPUNAN; July 29, 2002
FACTS
- Petitioner Douglas Millares was employed by private respondent ESSO International
Shipping Company LTD. (Esso Int.) through its local manning agency, private respondent
Trans-Global Maritime Agency, Inc. (Trans-Global) as a machinist. In 1975, he was
promoted as Chief Engineer. He was then receiving a monthly salary of US $1,939.00.
On June 13, 1989, Millares applied for a leave of absence for the period July 9 to August
7, 1989. Trans-global approved it. He then informed Esso Int. of his intention to avail of
the optional retirement plan under the Consecutive Enlistment Incentive Plan (CEIP)
considering that he had already rendered more than twenty (20) years of continuous
service. But the denied petitioner Millares request for optional retirement on the following
grounds, to wit: (1) he was employed on a contractual basis; (2) his contract of enlistment
(COE) did not provide for retirement before the age of sixty (60) years; and (3) he did not
comply with the requirement for claiming benefits under the CEIP, i.e., to submit a written
advice to the company of his intention to terminate his employment within thirty (30) days
from his last disembarkation date. He then requested for an extension of his leave of
absence from August 9 to 24, 1989. But the company told him that they have promoted a
First Assistant Engineer to his position as a result of his previous leave of absence which
expired last August 8, 1989. The adjustment in said rank was required in order to meet
manpower schedules as a result of his inability. Esso International told Millares that in
view of his absence without leave, which is equivalent to abandonment of his position, he
had been dropped from the roster of crew members effective September 1, 1989.
- On the other hand, petitioner Lagda was employed by private respondent Esso
International as wiper/oiler in June 1969. He was promoted as Chief Engineer in 1980, a
position he continued to occupy until his last COE expired on April 10, 1989. He also filed
for a leave of absence and informed the company of his intention to avail the early
retirement. His request was denied on the same grounds and he too was dropped from
work.
- On October 5, 1989, petitioners Millares and Lagda filed a complaint-affidavit for illegal
dismissal and non-payment of employee benefits against private respondents Esso
International and Trans-Global, before the POEA. POEA dismissed it for lack of merit.
NLRC affirmed.
ISSUE
WON the petitioners are contractual employees whose employment are terminated
everytime their contracts expire
HELD
YES
- it is clear that seafarers are considered contractual employees. They can not be
considered as regular employees under Article 280 of the Labor Code. Their employment
is governed by the contracts they sign everytime they are rehired and their employment is
terminated when the contract expires. Their employment is contractually fixed for a
certain period of time. They fall under the exception of Article 280 whose employment has
been fixed for a specific project or undertaking the completion or termination of which has
been determined at the time of engagement of the employee or where the work or
services to be performed is seasonal in nature and the employment is for the duration of
the season. We need not depart from the rulings of the Court in the two aforementioned
cases which indeed constitute stare decisis with respect to the employment status of

A2010

- 79 -

Disini

seafarers. Petitioners make much of the fact that they have been continually re-hired or
their contracts renewed before the contracts expired (which has admittedly been going on
for twenty (20) years). By such circumstance they claim to have acquired regular status
with all the rights and benefits appurtenant to it.
- Such contention is untenable. Undeniably, this circumstance of continuous re-hiring was
dictated by practical considerations that experienced crew members are more preferred.
Petitioners were only given priority or preference because of their experience and
qualifications but this does not detract the fact that herein petitioners are contractual
employees. They can not be considered regular employees.
Disposition IN VIEW OF THE FOREGOING, THE COURT Resolved to Partially GRANT
Private Respondents Second Motion for Reconsideration and Intervenor FAMES Motion
for Reconsideration in Intervention. The Decision of the National Labor Relations
Commission dated June 1, 1993 is hereby REINSTATED with MODIFICATION. The
Private Respondents, Trans-Global Maritime Agency, Inc. and Esso International Shipping
Co.,Ltd. are hereby jointly and severally ORDERED to pay petitioners One Hundred
Percent (100%) of their total credited contributions as provided under the Consecutive
Enlistment Incentive Plan(CEIP).

PETROLEUM SHIPPING LIMITED V NLRC (TANCHICO)


491 SCRA 35
CARPIO; June 16, 2006
FACTS
- On 6 March 1978, Esso International Shipping (Bahamas) Co., Ltd., ("Esso") through
Trans-Global Maritime Agency, Inc. ("Trans-Global") hired Florello W. Tanchico
("Tanchico") as First Assistant Engineer. In 1981, Tanchico became Chief Engineer.
- On 13 October 1992, Tanchico returned to the Philippines for a two-month vacation after
completing his eight-month deployment.
- On 8 December 1992, Tanchico underwent the required standard medical examination
prior to boarding the vessel. The medical examination revealed that Tanchico was
suffering from "Ischemic Heart Disease, Hypertensive Cardio-Muscular Disease and
Diabetes Mellitus." Tanchico took medications for two months and a subsequent stress
test showed a negative result. However, Esso no longer deployed Tanchico. Instead, Esso
offered to pay him benefits under the Career Employment Incentive Plan. Tanchico
accepted the offer.
- On 26 April 1993, Tanchico filed a complaint against Esso, Trans-Global and Malayan
Insurance Co., Inc. ("Malayan") before the Philippine Overseas Employment
Administration (POEA) for illegal dismissal with claims for backwages, separation pay,
disability and medical benefits and 13th month pay.
ISSUES
1. WON Tanchico is a regular employee of petitioners
2. WON Tanchico is entitled to 13th month pay, disability benefits and attorneys fees
HELD
1. NO
- The Court squarely passed upon the issue in Millares v. NLRC17 where one of the issues
raised was whether seafarers are regular or contractual employees whose employment
are terminated everytime their contracts of employment expire. The Court explained:
[I]t is clear that seafarers are considered contractual employees. They can not be
considered as regular employees under Article 280 of the Labor Code. Their
employment is governed by the contracts they sign everytime they are rehired and their
employment is terminated when the contract expires. Their employment is contractually
fixed for a certain period of time. They fall under the exception of Article 280 whose
employment has been fixed for a specific project or undertaking the completion or
termination of which has been determined at the time of engagement of the employee
or where the work or services to be performed is seasonal in nature and the
employment is for the duration of the season. We need not depart from the rulings of
the Court in the two aforementioned cases which indeed constitute stare decisis with
respect to the employment status of seafarers.
- The circumstance of continuous re-hiring was dictated by practical considerations that
experienced crew members are more preferred. Petitioners were only given priority or
preference because of their experience and qualifications but this does not detract the fact
that herein petitioners are contractual employees. They can not be considered regular
employees.
2. On 13th Month Pay
- The Court of Appeals premised its grant of 13th month pay on its ruling that Tanchico
was a regular employee. The Court of Appeals also ruled that petitioners are not exempt
from the coverage of PD 851 which requires all employers to pay their employees a 13th
month pay.
- We do not agree with the Court of Appeals. Again, Tanchico was a contractual, not a
regular, employee. Further, PD 851 does not apply to seafarers.
- Tanchicos employment is governed by his Contract of Enlistment. The Contract has
been approved by the POEA in accordance with Title I, Book One of the Labor Code and

Labor Law 1
the POEA Rules Governing Employment. Hence, in the absence of any provision in his
Contract governing the payment of 13th month pay, Tanchico is not entitled to the benefit.
On Disability Benefits
- Since Tanchico received compensation during his vacation, the Contract did not
terminate on the day he returned to Manila. The Contract remained in force during
Tanchicos vacation period.
- However, the Court of Appeals erred when it ruled that Tanchico is entitled to disability
benefits of 18 days for every year of service. The Court of Appeals ruled that Tanchicos
employment was continuous and that his tenure with petitioners was for 14 years. Again,
the Court of Appeals assumed that Tanchico was a regular employee. The Court of
Appeals failed to consider that Tanchicos employment terminated with the end of each
contract.
- Indications that Tanchico was suffering from ischemia were detected on 8 December
1992 during Tanchicos vacation period. Thus, petitioners paid him disability benefits for 18
days in accordance with the Contract. Tanchico cannot claim that he only acquired the
illness during his last deployment since the Medical Report 26 he submitted to the NLRC
showed that he has been hypertensive since 1983 and diabetic since 1987. In the
absence of concrete proof that Tanchico acquired his disability during his last deployment
and not during his vacation, he is only entitled to disability benefits for 18 days.

SKIPPERS UNITED PACIFIC INC V NLRC (CA &


ROSAROSO)
494 SCRA 66
AUSTRIA-MARTINEZ; July 12, 2006
NATURE
Appeal from a decision of the CA
FACTS
- Private respondent Gervasio Rosaroso was employed as a Third Engineer with Nicolakis
Shipping, S.A., a foreign firm through its recruitment and manning agency, petitioner
Skippers. The employment contract was for the period of one year beginning July 10,
1997 with a salary of $800 per month and other benefits. Rosaroso boarded M/V Naval
Gent on July 15, 1997. He was however ordered to disembark in Bulgaria on August 7,
1997 and repatriated to the Philippines.
- Soon after arrival in Manila, respondent filed a complaint for illegal dismissal and
monetary claims. The Labor Arbiter found the respondent was in fact illegally dismissed
and issued an order directing petitioner, Skippers, to pay Rosaroso separation pay of
$2,4000 or the equivalent of P100,000, representing three months pay and unpaid salary
for seven days of $186.69 or the equivalent of P7,840.98. Atorneys fees of P5,000 was
also awarded. The NLRC and the CA affirmed en toto the ruling of the Arbiter.
- Hence this appeal to the SC.
ISSUE
WON private respondent Rosaroso was illegal dismissed
HELD
YES
- The employer of Rosaroso did not provide the quantum of evidence needed to prove that
dismissal was in fact for cause. The evidence presented was just a telefax coming from
the alleged Chief Engineer of the vessel which the Arbiter up to the CA considered as
mere hearsay. While the Master of the vessel was grated under Paragraph D of Section
17 of the Philippine Overseas employment Administration (POEA) Standard Employment
condition governing the employment of Filipino Seafarers on Board Ocean Going Vessels
the power to dismiss for just cause without furnishing the seafarer with a notice of
dismissal if doing so will prejudice the safety of the crew and the vessel, the SC noted that
the complete report on the circumstances of the dismissal was not forwarded to the
manning agency as called for under the same provision.
Minor issues
- The award of backwages and separation pay in lieu of reinstatement as provided for in
Article 279 of the Labor Code is not applicable in this case. The Seafarer is a contractual
employee whose rights and obligations are governed by the POEA Employment Contract
and by RA 8042 (1995). The Employment contract does not provide for the award of
separation or termination pay. However, under Section 10 of RA 8042 the award of money
claims in cases of illegal dismissal is allowed. Under this provision, an illegal dismissed
seafarer is entitled to indemnity equivalent to his salary for the unexpired term of his
employment contract or three months for every year of the unexpired term, whichever is
less.
- The award by the Arbiter of the peso equivalent of the dollar awards cannot be enforced
as the same is contrary to law. The peso equivalent must be computed at the exchange
rate computed at the time of payment as provided for by RA 8183.

A2010

- 80 -

Disini

Disposition The questioned decision is affirmed with the modification that the dollar
award should be payable in its peso equivalent computed at the prevailing rate of
exchange at the time of payment.

PENTAGON INTERNATIONAL SHIPPING INC V


ADELANTAR
435 SCRA 342
YNARES-SANTIAGO; July 27, 2001
NATURE
Petition for review on certiorari of CA decision which modified an NLRC decision
FACTS
- August 16, 1997 > William B. Adelantar was hired by Dubai Ports Authority of Jebel Ali
under an employment contract (first contract) which provided for an unlimited period of
employment with a monthly salary of Dhs 5,500.
- September 3, 1997 > Adelantar and Pentagon International Shipping, Inc, for and in
behalf of Dubai Ports Authority of Jebel Ali, entered into a Philippine Overseas
Employment Administration (POEA) standard employment contract (second contract), this
time providing for a 12-month period with basic monthly salary of US$380.00 and fixed
overtime pay of US$152.00.
- April 5, 1998 > Adelantars basic salary was increased to Dhs 5,890 and overtime pay
was increased to Dhs 2,356
- June 11, 1998 > Dubai Ports barred Adelantar from entering the port due to a previous
dispute with his superior. On the same date, he was given a letter as he was terminated
for assaulting his superior officer, although he was promised employment in another
company.
- Adelantar filed a complaint for illegal dismissal with money claim against Pentagon with
the NLRC
- LABOR ARBITER: found the dismissal of Adelantar was illegal and ordered Pentagon to
pay Adelantar the amount of Dhs 24,738.00 representing the latters three (3) months
basic salary inclusive of overtime pay
- NLRC: affirmed the Labor Arbiters decision and held that in Section 10 of RA8042
(Migrant Workers and Overseas Filipinos Act of 1995) an illegally dismissed contract
worker is entitled to the salaries corresponding to the unexpired portion of his contract, or
for three (3) months for every year of the unexpired term, whichever is less. They
awarded backwages to Adelantar equivalent to three (3) months of his basic salary, but
exclusive of overtime pay
- CA: September 26, 2002, CA modified the amounts awarded by the Labor Arbiter and
the NLRC and instead awarded full backwages computed from the time of the dismissal
up to the finality of the decision because Section 10 of R.A. No 8042 is not applicable
because said provision only contemplates a fixed period of employment and that A279 LC
should apply considering that Adelantars first contract provided for an unlimited period of
employment.
ISSUE
WON A279 LC should apply given the first contract provided for an unlimited period of
employment
HELD
NO. Sec 10 RA 8042 should apply because the second contract (with POEA), which
provided for a fixed period of 1 year as employment, is applicable at bar. Also, landmark
case of Millares v NLRC applies
Ratio It is clear that seafarers are considered contractual employees. They can not be
considered as regular employees under A280 LC. Their employment is governed by the
contracts they sign every time they are rehired and their employment is terminated when
the contract expires. Their employment is contractually fixed for a certain period of time.
They fall under the exception of A280 LC whose employment has been fixed for a specific
project or undertaking the completion or termination of which has been determined at the
time of engagement of the employee or where the work or services to be performed is
seasonal in nature and the employment is for the duration of the season. (Millares v
NLRC)
Reasoning
- Coyoca v NLRC: Filipino seamen are governed by the Rules and Regulations of the
POEA. The Standard Employment Contract governing the Employment of All Filipino
Seamen on Board Ocean-Going Vessels of the POEA, particularly in Part I, Sec. C
specifically provides that the contract of seamen shall be for a fixed period. In no case
should the contract of seamen be longer than 12 months and any extension of the
Contract period shall be subject to the mutual consent of the parties.
- It should be stressed that whatever status of employment or increased benefits that the
complainant may have gained while under the employ of Dubai Ports Authority, the
undisputed fact remains that prior to his deployment, he agreed to be hired under a 12-

Labor Law 1
month POEA contract, the duration of which is the basis for the determination of the extent
of the respondents liability.
- Moreover, it is an accepted maritime industry practice that employment of seafarers is for
a fixed period only. Constrained by the nature of their employment which is quite peculiar
and unique in itself, it is for the mutual interest of both the seafarer and the employer why
the employment status must be contractual only or for a certain period of time. Seafarers
spend most of their time at sea and understandably, they can not stay for a long and an
indefinite period of time at sea. Limited access to shore society during the employment
will have an adverse impact on the seafarer. The national, cultural and lingual diversity
among the crew during the COE is a reality that necessitates the limitation of its period.
Disposition Petition is partly GRANTED and CA decision is REVERSED and SET ASIDE.
Petitioner Pentagon International Shipping, Inc. is ORDERED to pay private respondent
William B. Adelantar the amount equivalent to the unexpired portion of the September 3,
1997 POEA Standard Contract of Employment plus ten percent (10%) of the award as
attorneys fees.

LOPEZ V MWSS
[PAGE 60]
AUDION ELECTRIC CO INC V NLRC (MADOLID)
308 SCRA 340
GONZAGA-REYES; June 17, 1999
NATURE
Petition for certiorari, seeking annulment of resolution of the NLRC (of which the presiding
officer was our very own Dean Carale )
FACTS
- Madolid was employed by Audion Electric Co. on June 30, 1976 as fabricator and
continuously rendered service in different offices and projects as helper technician,
stockman, and timekeeper. He rendered 13 years of service with a clean record. On
August 3, Madolid received a letter informing him that he will be considered terminated
after the turnover of materials, including companys tools and equipments not later than
August 15, 1989.
- Madolid claims that he was dismissed without justifiable cause and due process and that
his dismissal was done in bad faith which renders the dismissal illegal. For this reason, he
claims that he is entitled to reinstatement with full backwages, and moral and exemplary
damages. He also includes payment of his overtime pay, project allowance, minimum
wage increase adjustment, proportionate 13th month pay and attorney's fees.
Audion rebuts his allegations by saying that the employment contract of Madolid was one
that was co-terminus with the project, thus he should not be considered as a regular
employee. Also, the company contends that it had paid all the alleged unpaid wages.
- The Labor arbiter decided the case in favor of Madolid, ordering Audion to pay him
backwages, OT pay, project allowances, min. wage increase adjustment, 13 th month pay,
and awarding him moral and exemplary damages and attorneys fees. Appeal to NLRC
was dismissed.
ISSUES
1. WON Madolid was a regular employee, thus entitling him to backwages, etc.
2. WON Audion was denied due process with the award of all the claims of Madolid
HELD
1. YES
Ratio Where the employment of project employees is extended long after the supposed
project has been finished, the employees are removed from the scope of project
employees and considered regular employees.
Reasoning
- (citing NLRCs decision): Audions assigning Madolid to its various projects did not make
him a project worker. As found by the Labor Arbiter, it appears that complainant was
employed by respondent xxx as fabricator and or projects as helper electrician, stockman
and timekeeper.' Simply put, complainant was a regular non-project worker.
- Madolids employment status was established by the Certification of Employment dated
April 10, 1989 issued by Audion which certified that private respondent is a bonafide
employee from June 30, 1976 up to the time of issuance on April 10, 1989. This showed
that his exposure to their field of operation was as fabricator, helper/electrician,
stockman/timekeeper. This proves that he was regularly and continuously employed by
Audion in various job assignments from 1976 to 1989, for a total of 13 years. The alleged
gap in employment service does not defeat his regular status as he was rehired for many
more projects without interruption and performed functions which are vital, necessary and
indispensable to the usual business of petitioner.
- Audion could have presented substantial evidence to support its claim that Madolid was
a project worker, like the employment contract (which stated the employees nature of

A2010

- 81 -

Disini

employment) or reports of termination (which were required by DOLE upon termination of


the project, and failure to submit this is an indication of regular status of an employee as
held in cases), but it did not.
2. NO
Ratio Due process is not denied when one is afforded the opportunity to be heard and
present his case, but the same decided not to take the opportunity.
Reasoning
- Madolid clearly specified in his affidavit the specific dates in which he was not paid
overtime pay, project allowances, 13 th month pay, and wage adjustments. The claim of
Audion that it paid him such must be proved by evidence, which it did not do (despite of
having the burden to prove the claim).
- In fact, records show that the company did not appear in hearings, which the court took
to be a waiver of its right to be heard.
- However, award to moral and exemplary damages and attorneys fees are deleted for
being devoid of moral basis.
Disposition Petition denied, resolutions affirmed with modifications (deletion of award of
damages and attorneys fees)

BETA ELECTRIC CORP V NLRC (BETA ELECTRIC


EMPLOYEES ASSOCIATION, PETILLA)
182 SCRA 384
SARMIENTO; February 15, 1990
NATURE
Petition to review the decision of the National Labor Relations Commission affirming the
judgment of the labor arbiter reinstating the private respondent with backwages.
FACTS
- The petitioner hired the private respondent as clerk typist for one month, which
appointment was extended five times in five months (one month /contract).Her
appointments were covered by corresponding written contracts. On June 22, 1987, her
services were terminated without notice or investigation. On the same day, she went to the
labor arbiter on a complaint for illegal dismissal. Both the labor arbiter and the respondent
National Labor Relations Commission ruled for her.Petioner claims the private
respondents appointment was temporary and hence she may be terminated at will.
ISSUES
WON the fact that private respondents employment has been a contract-to-contract basis
alters the character of her employment as a regular employee
HELD
NO
Ratio. The fact that her employment has been a contract-to-contract basis can not alter
the character of employment, because contracts can not override the mandate of law..
Reasoning
- private employee was employed from December 15, 1986 until June 22, 1987 when she
was ordered laid off. Her tenure having exceeded six months, she attained regular
employment.
- petitioner can not rightfully say that since the private respondent's employment hinged
from contract to contract, it was ergo, "temporary", depending on the term of each
agreement. Under the Labor Code, an employment may only be said to be "temporary"
"where [it] has been fixed for a specific undertaking the completion of or termination of
which has been determined at the time of the engagement of the employee or where the
work or services to be performed is seasonal in nature and the employment is for the
duration of the season." Quite to the contrary, the private respondent's work, that of "typistclerk" is far from being "specific" or "seasonal", but rather, one, according to the Code,
"where the employee has been engaged to perform activities which are usually necessary
or desirable in the usual business." And under the Code, where one performs such
activities, he is a regular employee, "[t]he provisions of written agreement to the contrary
notwithstanding.
Disposition Petition DISMISSED. Private respondent is ordered REINSTATED with
backwages equivalent to three years with no qualification or deductions.

UNIVERSAL ROBINA CORPORATION V CATAPANG


473 SCRA 189
CALLEJO, SR; October 14, 2005
FACTS
- Petitioner Universal Robina Corporation is a corporation duly organized and existing
under the Philippine laws, while petitioner Randy Gregorio is the manager of the petitioner
companys duck farm in Calauan, Laguna.

Labor Law 1
- The individual respondents were hired by the petitioner company on various dates from
1991 to 1993 to work at its duck farm in Barangay Sto. Tomas, Calauan, Laguna. The
respondents were hired under an employment contract which provided for a five-month
period. After the expiration of the said employment contracts, the petitioner company
would renew them and re-employ the respondents. This practice continued until sometime
in 1996, when the petitioners informed the respondents that they were no longer renewing
their employment contracts.
- In October 1996, the respondents filed separate complaints for illegal dismissal,
reinstatement, backwages, damages and attorneys fees against the petitioners. The
complaints were later consolidated. On March 30, 1999, after due proceedings, the Labor
Arbiter rendered a decision in favor of the respondents, which NLRC and the CA affirmed.
- On appeal, the petitioners submit that the respondents are not regular employees. They
aver that it is of no moment that the respondents have rendered service for more than a
year since they were covered by the five-month individual contracts to which they duly
acquiesced. The petitioners contend that they were free to terminate the services of the
respondents at the expiration of their individual contracts. The petitioners maintain that,
in doing so, they merely implemented the terms of the contracts.
- The petitioners assert that the respondents contracts of employment were not intended
to circumvent security of tenure. They point out that the respondents knowingly and
voluntarily agreed to sign the contracts without the petitioners having exercised any undue
advantage over them. Moreover, there is no evidence showing that the petitioners exerted
moral dominance on the respondents.[\
ISSUE
WON the respondent employees of the corporation are regular employees and therefore
their termination for causes outside of the Labor Code is patently illegal
HELD
YES
Ratio An employee shall be deemed to be of regular status when he has been
performing a job for at least one year even if the performance is not continuous and
merely intermittent.
Reasoning
- In any case, we find that the CA, the NLRC and the Labor Arbiter correctly categorized
the respondents as regular employees of the petitioner company. In Abasolo v. National
Labor Relations Commission, the Court reiterated the test in determining whether one is a
regular employee:
- The primary standard, therefore, of determining regular employment is the
reasonable connection between the particular activity performed by the employee
in relation to the usual trade or business of the employer. The test is whether the
former is usually necessary or desirable in the usual business or trade of the
employer. The connection can be determined by considering the nature of work
performed and its relation to the scheme of the particular business or trade in its
entirety. Also, if the employee has been performing the job for at least a year, even if
the performance is not continuous and merely intermittent, the law deems repeated
and continuing need for its performance as sufficient evidence of the necessity if
not indispensability of that activity to the business. Hence, the employment is
considered regular, but only with respect to such activity and while such activity
exists.
- It is obvious that the said five-month contract of employment was used by petitioners as
a convenient subterfuge to prevent private respondents from becoming regular
employees. Such contractual arrangement should be struck down or disregarded as
contrary to public policy or morals. To uphold the same would, in effect, permit petitioners
to avoid hiring permanent or regular employees by simply hiring them on a temporary or
casual basis, thereby violating the employees security of tenure in their jobs. Petitioners
act of repeatedly and continuously hiring private respondents in a span of 3 to 5 years
to do the same kind of work negates their contention that private respondents were hired
for a specific project or undertaking only.
- Further, factual findings of labor officials who are deemed to have acquired expertise in
matters within their respective jurisdiction are generally accorded not only respect but
even finality, and bind us when supported by substantial evidence.
Disposition petition is DENIED DUE COURSE. The Decision of the Court of Appeals is
AFFIRMED.

MARAGUINOT V NLRC (DEL ROSARIO, VIVA FILMS)


284 SCRA 539
DAVIDE; January 22, 1998
NATURE
Special civil action for certiorari seeking to annul the decision of NLRC and its Resolution
FACTS

A2010

- 82 -

Disini

- Petitioner Alejandro Maraguinot, Jr. maintains that he was employed by private


respondents as part of the filming crew. About 4 months later, he was designated Asst.
Electrician. He was then promoted to the rank of Electrician.
- Petitioner Paulino Enero claims that private respondents employed him as a member of
the shooting crew.
- Petitioners tasks consisted of loading, unloading and arranging movie equipment in the
shooting area as instructed by the cameraman, returning the equipment to Viva Films
warehouse, assisting in the fixing of the lighting system, and performing other tasks that
the cameraman and/or director may assign.
- Petitioners requested that private respondents adjust their salary in accordance with the
minimum wage law. Petitioners were informed that Mr. Vic del Rosario would agree to
increase their salary only if they signed a blank employment contract. As petitioners
refused to sign, private respondents forced Enero to go on leave then refused to take him
back when he reported for work. Meanwhile, Maraguinot was dropped from the company
payroll but was returned and again asked to sign a blank employment contract, and when
he still refused, private respondents terminated his services. Petitioners thus sued for
illegal dismissal before the Labor Arbiter.
- Private respondents claim that Viva Films is primarily engaged in the distribution and
exhibition of movies, but not in the business of making movies; in the same vein, private
respondent Vic del Rosario is merely an executive producer, i.e., the financier who invests
a certain sum of money for the production of movies distributed and exhibited by VIVA;
that they contract persons called producers -- also referred to as associate producers-to produce or make movies for private respondents; and that petitioners are project
employees of the associate producers who, in turn, act as independent contractors. As
such, there is no employer-employee relationship between petitioners and private
respondents; that it was the associate producer of a film who hired Maraguinot.and he was
released upon payment of his last salary, as his services were no longer needed; that
Enero was hired for a movie, went on vacation and by the time he reported back to work
the move had been completed.
- The Labor Arbiter found that:
-- complainants are the employees of the respondents. The producer cannot be
considered as an independent contractor but should be considered only as a labor-only
contractor and as such, acts as a mere agent of the real employer, the herein
respondents. Also, it is an admitted fact that the complainants received their salaries from
the respondents. It is very clear also that complainants are doing activities which are
necessary and essential to the business of the respondents, that of movie-making.
Complainant Maraguinot worked as an electrician while complainant Enero worked as a
crew [member]. Hence, the complainants were illegally dismissed.
- Private respondents appealed to the NLRC. In its decision, it said that:
1. Complainants were hired for specific movie projects and their employment was coterminus with each movie project the completion/termination of which are pre-determined,
such fact being made known to complainants at the time of their engagement.
2. Each shooting unit works on one movie project at a time. And the work of the shooting
units, which work independently from each other, are not continuous in nature but
depends on the availability of movie projects.
3. Further shown by respondents is the irregular work schedule of complainants on a daily
basis. Maraguinot was supposed to report on 05 August 1991 but reported only on 30
August 1991, or a gap of 25 days. Complainant Enero worked on 10 September 1991 and
his next scheduled working day was 28 September 1991, a gap of 18 days.
4. The extremely irregular working days and hours of complainants work explain the lump
sum payment for complainants services for each movie project. Hence, complainants
were paid a standard weekly salary regardless of the number of working days and hours
they logged in. Otherwise, if the principle of no work no pay was strictly applied,
complainants earnings for certain weeks would be very negligible.
5. Respondents also alleged that complainants were not prohibited from working with
other movie companies.
The NLRC, in reversing the Labor Arbiter, then concluded that these circumstances, taken
together, indicated that complainants (herein petitioners) were project employees.
Petitioners Claim To support their claim that they were regular (and not project)
employees of private respondents, petitioners cited their performance of activities that
were necessary or desirable in the usual trade or business of private respondents and
added that their work was continuous, i.e., after one project was completed they were
assigned to another project.
Respondents Private respondents reiterate their version of the facts and stress that their
evidence supports the view that petitioners are project employees; point to petitioners
irregular work load and work schedule; emphasize the NLRCs finding that petitioners
never controverted the allegation that they were not prohibited from working with other
movie companies; and ask that the facts be viewed in the context of the peculiar
characteristics of the movie industry.
The Office of the Solicitor General (OSG) is convinced that this petition is improper
since petitioners raise questions of fact; and submits that petitioners reliance on Article
280 of the Labor Code to support their contention that they should be deemed regular
employees is misplaced, as said section merely distinguishes between two types of
employees, i.e., regular employees and casual employees, for purposes of determining
the right of an employee to certain benefits. The OSG likewise rejects petitioners

Labor Law 1
contention that since they were hired not for one project, but for a series of projects, they
should be deemed regular employees. In closing, the OSG disagrees with petitioners
claim that the NLRCs classification of the movie producers as independent contractors
had no basis in fact and in law, since, on the contrary, the NLRC took pains in explaining
its basis for its decision.
ISSUES
1. WON this is a proper action
2. WON an employer-employee relationship existed between the petitioners and private
respondents or any one of them
3. WON petitioners were illegally dismissed
HELD
1. YES
Ratio We rule that a special civil action for certiorari under Rule 65 of the Rules of Court is
the proper remedy for one who complains that the NLRC acted in total disregard of
evidence material to or decisive of the controversy. In the instant case, petitioners allege
that the NLRCs conclusions have no basis in fact and in law, hence the petition may not
be dismissed on procedural or jurisdictional grounds.
2. YES
Ratio The relationship between VIVA and its producers or associate producers seems to
be that of agency, as the latter make movies on behalf of VIVA, whose business is to
make movies. As such, the employment relationship between petitioners and producers
is actually one between petitioners and VIVA, with the latter being the direct employer.
The employer-employee relationship between petitioners and VIVA can further be
established by the control test. While four elements are usually considered in
determining the existence of an employment relationship, namely: (a) the selection and
engagement of the employee; (b) the payment of wages; (c) the power of dismissal; and
(d) the employers power to control the employees conduct, the most important element is
the employers control of the employees conduct, not only as to the result of the work to
be done but also as to the means and methods to accomplish the same. These four
elements are present here.
Reasoning (On job contracting)
It is settled that the contracting out of labor is allowed only in case of job contracting. 17
- Assuming that the associate producers are job contractors, they must then be engaged
in the business of making motion pictures. As such, and to be a job contractor under the
preceding description, associate producers must have tools, equipment, machinery, work
premises, and other materials necessary to make motion pictures.The associate producer
did not have substantial capital nor investment in the form of tools, equipment and other
materials necessary for making a movie. If private respondents insist that their associate
producers are labor contractors, then these producers can only be labor-only
contractors.18
- As labor-only contracting is prohibited, the law considers the person or entity engaged in
the same a mere agent or intermediary of the direct employer. But even by the preceding
standards, the associate producers of VIVA cannot be considered labor-only contractors
as they did not supply, recruit nor hire the workers.
Reasoning (On control test)
- VIVAs control is evident in its mandate that the end result must be a quality film
acceptable to the company. The means and methods to accomplish the result are
likewise controlled by VIVA, viz., the movie project must be finished within schedule
without exceeding the budget, and additional expenses must be justified; certain scenes
are subject to change to suit the taste of the company; and the Supervising Producer, the
eyes and ears of VIVA and del Rosario, intervenes in the movie-making process by
assisting the associate producer in solving problems encountered in making the film.
- Aside from control, the element of selection and engagement is likewise present in the
instant case and exercised by VIVA. A sample appointment slip was offered by private
respondents to prove that members of the shooting crew except the driver are project
employees of the Independent Producers. Notably, nowhere in the appointment slip does
it appear that it was the producer or associate producer who hired the crew members;
moreover, it is VIVAs corporate name which appears on the heading of the appointment
slip. What likewise tells against VIVA is that it paid petitioners salaries as evidenced by
vouchers, containing VIVAs letterhead, for that purpose.
17

Section 8, Rule VIII, Book III of the Omnibus Rules Implementing the Labor Code describes permissible job
contracting in this wise:
Sec. 8. Job contracting. -- There is job contracting permissible under the Code if the following conditions are met:
(1) The contractor carries on an independent business and undertakes the contract work on his own account under his
own responsibility according to his own manner and method, free from the control and direction of his employer or
principal in all matters connected with the performance of the work except as to the results thereof; and
(2) The contractor has substantial capital or investment in the form of tools, equipment, machineries, work premises, and
other materials which are necessary in the conduct of his business.

18

Art. 106. Contractor or subcontractor.-- x x x There is labor-only contracting where the person supplying
workers to an employer does not have substantial capital or investment in the form of tools, equipment,
machineries, work premises, among others, and the workers recruited and placed by such persons are
performing activities which are directly related to the principal business of such employer. In such cases, the
person or intermediary shall be considered merely as an agent of the employer who shall be responsible to the
workers in the same manner and extent as if the latter were directly employed by him.

A2010

- 83 -

Disini

3. YES
Ratio A project employee or a member of a work pool may acquire the status of a regular
employee when the following concur: 1) There is a continuous rehiring of project
employees even after cessation of a project; and 2) The tasks performed by the alleged
project employee are vital, necessary and indispensable to the usual business or trade of
the employer. However, the length of time during which the employee was continuously rehired is not controlling, but merely serves as a badge of regular employment.
- In the instant case, the evidence on record shows that petitioner Enero was employed for
a total of two (2) years and engaged in at least eighteen (18) projects, while petitioner
Maraguinot was employed for some three (3) years and worked on at least twenty-three
(23) projects. Moreover, as petitioners tasks involved, among other chores, the loading,
unloading and arranging of movie equipment in the shooting area as instructed by the
cameramen, returning the equipment to the Viva Films warehouse, and assisting in the
fixing of the lighting system, it may not be gainsaid that these tasks were vital, necessary
and indispensable to the usual business or trade of the employer. As regards the
underscored phrase, it has been held that this is ascertained by considering the nature of
the work performed and its relation to the scheme of the particular business or trade in its
entirety.
Reasoning
- It may not be ignored, however, that private respondents expressly admitted that
petitioners were part of a work pool; and, while petitioners were initially hired possibly as
project employees, they had attained the status of regular employees in view of VIVAs
conduct.
- At this time, we wish to allay any fears that this decision unduly burdens an employer by
imposing a duty to re-hire a project employee even after completion of the project for
which he was hired. The import of this decision is not to impose a positive and sweeping
obligation upon the employer to re-hire project employees. What this decision merely
accomplishes is a judicial recognition of the employment status of a project or work pool
employee in accordance with what is fait accompli, i.e., the continuous re-hiring by the
employer of project or work pool employees who perform tasks necessary or desirable to
the employers usual business or trade. Let it not be said that this decision coddles
labor, for as Lao19 has ruled, project or work pool employees who have gained the status
of regular employees are subject to the no work-no pay principle.
- The Courts ruling here is meant precisely to give life to the constitutional policy of
strengthening the labor sector, but, we stress, not at the expense of management. Lest it
be misunderstood, this ruling does not mean that simply because an employee is a project
or work pool employee even outside the construction industry, he is deemed, ipso jure, a
regular employee. All that we hold today is that once a project or work pool employee has
been: (1) continuously, as opposed to intermittently, re-hired by the same employer for the
same tasks or nature of tasks; and (2) these tasks are vital, necessary and indispensable
to the usual business or trade of the employer, then the employee must be deemed a
regular employee, pursuant to Article 280 of the Labor Code and jurisprudence.
Disposition instant petition is GRANTED.

ABESCO CONSTRUCTION AND DEVELOPMENT


CORPORATION V RAMIREZ
487 SCRA 9
CORONA; April 10, 2006
NATURE
Appeal by certiorari
FACTS
- Petitioner company was engaged in a construction business where respondents were
hired on different dates from 1976 to 1992 either as laborers, road roller operators,
painters or drivers.
- In 1997, respondents filed 2 separate complaints for illegal dismissal against the
company and its General Manager before the Labor Arbiter (LA). Petitioners allegedly
dismissed them without a valid reason and without due process of law. The complaints
also included claims for non-payment of the 13th month pay, five days service incentive
leave pay, premium pay for holidays and rest days, and moral and exemplary damages.
The LA later ordered the consolidation of the two complaints.
- Petitioners denied liability and countered that respondents were project employees
since their services were necessary only when the company had projects to be completed.
Petitioners argued that, being project employees, respondents employment was
coterminous with the proj. to which they were assigned. They werent regular employees
19

A work pool may exist although the workers in the pool do not receive salaries and are free to seek other
employment during temporary breaks in the business, provided that the worker shall be available when called
to report for a project. Although primarily applicable to regular seasonal workers, this set-up can likewise be
applied to project workers insofar as the effect of temporary cessation of work is concerned. This is beneficial
to both the employer and employee for it prevents the unjust situation of coddling labor at the expense of
capital and at the same time enables the workers to attain the status of regular employees.

Labor Law 1
who enjoyed security of tenure and entitlement to separation pay upon termination from
work.
- After trial, the LA declared respondents as regular employees because they belonged to
a work pool from which the company drew workers for assignment to different projects,
at its discretion. He ruled that respondents were hired and re-hired over a period of 18
years, hence, they were deemed to be regular employees. He likewise found that their
employment was terminated without just cause. Thus, in its judgment, the LA declared
petitioner company guilty of illegal dismissal and ordered it to reinstate the respondents to
their former positions with backwages and other benefits and that if reinstatement was not
feasible, that separation pay be awarded.
- Petitioners appealed to the NLRC which affirmed the LAs decision. They later filed a
petition for review in the CA arguing that they were not liable for illegal dismissal since
respondents services were merely put on hold until the resumption of their business
operations. They also averred that they had paid respondents their full wages and benefits
as provided by law, hence, the latter had no more right to further benefits.
- The CA, taking note of the fact that petitioners previously used the defense that the
respondents were project employees who were not entitled to security of tenure and now
say that the respondents were not dismissed but their employment merely suspended,
dismissed the appeal and dismissed the MFR as well.
ISSUES
1. WON the respondents were regular employees
2. WON respondents were illegally dismissed
HELD
1. YES
Ratio In determining the nature of ones employment, length of service is not a controlling
factor
Reasoning
- Jurisprudence: The SC ruled that respondents were regular employees but not for the
reasons given by the LA (which both the NLRC and the CA affirmed). Citing Palomar, et
al. v. NLRC, the SC held that contrary to the disquisitions of the LA, employees (like
respondents) who work under different project employment contracts for several years do
not automatically become regular employees; they can remain as project employees
regardless of the number of years they work. Length of time is not a controlling factor in
determining the nature of ones employment.
- Moreover, employees who are members of a work pool from which a company (like
petitioner corp.) draws workers for deployment to its different projects do not become
regular employees by reason of that fact alone. The Court has enunciated in the cases of
Raycor Aircontrol Systems, Inc. v. NLRC, and ALU-TUCP v. NLRC, that members of a
work pool can either be project employees or regular employees
- The principal test for determining whether employees are project employees or regular
employees is whether they are assigned to carry out a specific project or undertaking, the
duration and scope of which are specified at the time they are engaged for that project.
Such duration, as well as the particular work/service to be performed, is defined in an
employment agreement and is made clear to the employees at the time of hiring.
- Petitioners did not have that kind of agreement with respondents. Neither did they inform
the respondents of the nature of their work at the time of hiring. Hence, for failure of
petitioners to substantiate their claim that respondents were project employees, we are
constrained to declare them as regular employees
- Furthermore, petitioners cannot belatedly argue that respondents continue to be their
employees (so as to escape liability for illegal dismissal). Before the LA, petitioners
staunchly postured that respondents were only project employees whose employment
tenure was coterminous with the projects they were assigned to. However, before the CA,
they took a different stance by insisting that respondents continued to be their employees.
Petitioners inconsistent and conflicting positions on their true relation with respondents
make it all the more evident that the latter were indeed their regular employees.
2. YES
Ratio The law requires that the employer furnish the employee 2 written notices: (1) a
notice informing them of the particular acts for which they are being dismissed and (2) a
notice advising them of the decision to terminate the employment, before termination can
be validly effected.
Reasoning
- In resolving the issue of illegal dismissal, the SC simply stated that petitioners failed to
adhere to the two-notice rule, and said that respondents were never given such notices.
Disposition Petition denied

ALU-TUCP V NLRC (NATIONAL STEEL CORP)


234 SCRA 678
FELICIANO; August 2, 1994
NATURE
Petition for Certiorari to review the resolutions of the NLRC

A2010

- 84 -

Disini

FACTS
- Petitioners claim that they have been employed by respondent National Steel
Corporation (NSC) in connection with its Five Year Expansion Program (FAYEP I and II)
for varying lengths of time when they were separated from NSCs service.
- Petitioners filed separate complaints for unfair labor practice, regularization and
monetary benefits. The Labor Arbiter declared petitioners regular project employees who
shall continue their employment as such for as long as such (project) activity exists, but
entitled to the salary of a regular employee pursuant to the provisions in the collective
bargaining agreement. It also ordered payment of salary differentials.
- Both parties appealed. Petitioners argued they were regular, not project employees. NSC
claimed petitioners are project employees as they were employed to undertake a specific
project.
- The NLRC modified the Labor Arbiters decision, affirming the holding that they were
project employees since they were hired to perform work in a specific undertaking. It,
however, set aside the award to petitioners of the same benefits enjoyed by regular
employees for lack of legal and factual basis.
- Petitioners appealed to the SC, arguing that they are regular employees of NSC
because: (i) their jobs are necessary, desirable and work-related to private respondents
main business, steel-making; and (ii) they have rendered service for six (6) or more years
to NSC.
ISSUE
WON petitioners are properly characterized as project employees rather than regular
employees of NSC
SCs NOTE: The issue relates to an important consequence: the services of project
employees are co-terminous with the project and may be terminated upon the end or
completion of the project for which they were hired. Regular employees, in contrast, are
legally entitled to remain in the service of their employer until that service is terminated by
one or another of the recognized modes of termination of service under the Labor Code.
HELD
YES
petitioners are project employees.
- The law governing the matter is Article 280 of the Labor Code:
ART. 280. Regular and casual employment. - The provisions of written
agreement to the contrary notwithstanding and regardless of the oral agreement
of the parties, an employment shall be deemed to be regular where the
employee has been engaged to perform activities which are usually necessary or
desirable in the usual business or trade of the employer, except where the
employment has been fixed for a specific project or undertaking the completion
or termination of which has been determined at the time of the engagement of
the employee or where the work or service to be performed is seasonal in nature
and the employment is for the duration of the season.
An employment shall be deemed to be casual if it is not covered by the
preceding paragraph: Provided, That any employee who has rendered at least
one year of service, whether such service is continuous or broken, shall be
considered a regular employee with respect to the activity in which he is
employed and his employment shall continue while such activity exists.
- As evident in Article 280 of the Labor Code, the principal test for determining whether
particular employees are properly characterized as project employees as distinguished
from regular employees is whether or not the project employees were assigned to carry
out a specific project or undertaking, the duration (and scope) of which were specified at
the time the employees were engaged for that project.
- In business and industry, project could refer to one or the other of at least two
distinguishable types of activities. Firstly, a project could refer to a particular job or
undertaking that is within the regular or usual business of the employer company, but
which is distinct and separate, and identifiable as such, from the other undertakings of the
company. Such job or undertaking begins and ends at determined or determinable times.
Secondly, the term project could also refer to a particular job or undertaking that is not
within the regular business of the corporation. Such job or undertaking must also be
identifiably separate and distinct from the ordinary or regular business operations of the
employer. The job or undertaking also begins and ends at determined or determinable
times.
- Whichever type of project employment is found in a particular case, a common basic
requisite is that the designation of named employees as project employees and their
assignment to a specific project, are effected and implemented in good faith, and not
merely as a means of evading otherwise applicable requirements of labor laws.
- The particular component projects embraced in the FAYEP, to which petitioners were
assigned, were distinguishable from the regular or ordinary business of NSC, which is the
production or making and marketing of steel products. During the time petitioners
rendered services to NSC, their work was limited to one or another of the specific
component projects which made up the FAYEP I and II. It is not shown that petitioners
were hired for or assigned to other purposes.
Re Length of Service

Labor Law 1
- SC affirmed the Labor Arbiter and NLRCs basic finding that the length of service of a
p[roject employee is not the controlling test of employment tenure but whether or not the
employment has been fixed for a specific project or undertaking the completion or
termination of which has been determined at the time of the engagement of the employee.
- The simple fact that the employment of petitioners as project employees had gone
beyond one year does not detract from, or legally dissolve, their status as project
employees.
Disposition Petition for Certiorari is dismissed. Resolutions of NLRC affirmed.

KIAMCO V NLRC (PNOC)


309 SCRA 424
BELLOSILLO; June 29, 1999
FACTS
- Private respondent PHILIPPINE NATIONAL OIL COMPANY (PNOC) through its Energy
Research and Development Division, hired petitioner Cisell Kiamco as a project employee
in its Geothermal Agro-Industrial Plant Project in Valencia, Negros Oriental. The Contract
of Employment1 stipulated among others that Kiamco was being hired by the company as
a technician for a period of 5 months from July 1 1992 to Nov 30 1992, or up to the
completion of the project, whichever would come first.
- After the termination of the contract, a 2nd one was entered into by the parties containing
basically the same terms and conditions. The period of employment was from Dec 1 1992
to April 30 1993.
- Kiamco was again re-hired for 6 months (May 1 1993 to Nov 30 1993)
- On Oct 20 1993 Kiamco received a Memorandum from the administration department
demanding an explanation from him on certain infractions he allegedly committed: 1.
Misconduct 2. Absence without official leave (AWOL) 3. Non-compliance of administrative
reporting procedure on accidents 4. Unauthorized use of company vehicles
- Kiamco tried to explain his side but private respondents found his explanation
unsatisfactory. On Oct 28 1993 Kiamco received a Memorandum placing him under
preventive suspension from Nov 1 1993 to Nov 30 1993 pending further investigation. No
investigation however was ever conducted. Private respondents contended that an
investigation was not necessary since Kiamco had ceased to be an employee ipso facto
upon the expiration of his employment contract on Nov 30 1993.
- On Dec 1 1993 Kiamco reported back to work but was prevented by security guards from
entering the company premises. On May 27 1994 private respondent reported to the
Department of Labor and Employment that petitioner Kiamco was terminated on Nov 1
1993 due to the expiration of his employment contract and the abolition of his position.
- On April 25 1994 Kiamco filed before the NLRC Sub-Regional Arbitration Branch No. VII
a Complaint for illegal suspension and dismissal against the PNOC. He prayed that he be
reinstated to his former position and paid back wages. Labor Arbiter dismissed the
complaint for lack of merit. According to the Labor Arbiter, the three (3) employment
contracts were freely and voluntarily signed by Kiamco and the PNOC representatives.
The contracts plainly stated that Kiamco was being hired for a specific project and for a
fixed term. Therefore Kiamco could not question his dismissal since it was in accordance
with his employment contract.
- Kiamco appealed the decision of the Labor Arbiter to public respondent NLRC which on
Sept 27 1996 reversed the Labor Arbiter and declared Kiamco as a regular employee of
the respondents and to have been illegally dismissed by the latter. Ordering respondents
to REINSTATE the complainant to his former position without loss of seniority rights and
privileges with back wages from the date of his dismissal up to actual reinstatement less
any income he may have earned during the pendency of the case.
- Private respondents filed a MFR of the decision of the NLRC contending that it erred in
holding that Kiamco was a regular employee and that the findings of the Labor Arbiter that
Kiamco was a project employee should be affirmed.
- NLRC modified its Sept 27 1996 Decision declaring that the complainant-appellant is
declared a project employee at respondents Geothermal Plant and to continue with said
employment until the full completion of the project but in the absence of proof to that
effect, complainant is hereby awarded back wages for a period of 6 months or in the
amount of P23,100.00. The order declaring the complainant-appellant as a regular
employee of respondent PNOC, and for said company to reinstate the complainant with
full back wages is hereby deleted.
- In his petition for certiorari, Kiamco charges the NLRC with grave abuse of discretion
amounting to lack or excess of jurisdiction in issuing the questioned Resolution and prays
that it be nullified and he reinstated to his former position. He also seeks payment of back
wages, damages and attorneys fees.
ISSUES
1. WON petitioner is a regular employee or a project employee
2. WON petitioner is entitled to reinstatement without loss of seniority rights and privileges
and to the payment of full back wages
3. WON petitioner is entitled to moral and exemplary damages.
HELD

A2010

- 85 -

Disini

1. Kiamco was correctly labeled by the NLRC as a project employee.


-Article 280 of the Labor Code
Regular and casual employment. - The provisions of written agreement to the contrary
notwithstanding and regardless of the oral agreement of the parties, an employment
shall be deemed to be regular where the employee has been engaged to perform
activities which are usually necessary or desirable in the usual business or trade of the
employer, except where the employment has been fixed - for a specific project or
undertaking the completion or termination of which has been determined at the time of
the engagement of the employee or where the work or service to be performed is
seasonal in nature and the employment is for the duration of the season.
- An employee shall be deemed to be casual if it is not covered by the preceding
paragraph: Provided, that any employee who has rendered at least one year of service,
whether such service is continuous or broken, shall be considered a regular employee
with respect to the activity in which he is employed and his employment shall continue
while such activity exists.
- In Violeta v. NLRC [10 October 1997, 280 SCRA 520.] it was held The principal test for determining whether particular employees are properly characterized
as "project employees," as distinguished from "regular employees," is whether or not the
"project employees" were assigned to carry out a "specific project or undertaking," the
duration (and scope) of which were specified at the time the employees were engaged for
that project. As defined, project employees are those workers hired (1) for a specific
project or undertaking, and (2) the completion or termination of such project or undertaking
has been determined at the time of engagement of the employee.
- Under Policy Instruction No. 20 of the Secretary of Labor, project employees are those
employed in connection with a particular project. Non-project or regular employees are
those employed without reference to any particular project.
- The three Contracts of Employment entered into by Kiamco clearly established that he
was a project employee because (a) he was specifically assigned to work for a particular
project, which was the Geothermal Agro-Industrial Demonstration Plant Project of private
respondents, and (b) the termination and the completion of the project or undertaking was
determined and stipulated in the contract at the time of his employment.
2. YES
- In Santos v. NLRC (154 SCRA 166) it was held The normal consequences of a finding that an employee has been illegally dismissed
are, that the employee becomes entitled to reinstatement to his former position without
loss of seniority rights and the payment of back wages.
- Reinstatement restores the employee who was unjustly dismissed to the position from
which he was removed, that is, to his status quo ante dismissal; while the grant of back
wages allows the same employee to recover from the employer that which he had lost by
way of wages as a result of his dismissal.
- The argument of private respondents that reinstatement and payment of back wages
could not be made since Kiamco was not a regular employee is apparently misplaced. As
quoted above, the normal consequences of an illegal dismissal are the reinstatement of
the aggrieved employee and the grant of back wages. These rights of an employee do not
depend on the status of his employment prior to his dismissal but rather to the legality and
validity of his termination. The fact that an employee is not a regular employee does not
mean that he can be dismissed any time, even illegally, by his employer.
3. NO
- Moral damages are recoverable only where the dismissal of the employee was attended
with bad faith or fraud or constituted an act oppressive to labor or was done in a manner
contrary to morals, good custom or public policy. Exemplary damages, on the other hand,
may be awarded only if the dismissal was effected in a wanton, oppressive or malevolent
manner. The evidence on record does not show any fraud, malice or bad faith on the part
of private respondents that would justify payment to petitioner of moral and exemplary
damages.

PHIL. JAI-ALAI & AMUSEMENT CORP V CLAVE


126 SCRA 299
MELENCIO-HERRERA; December 21, 1983
NATURE
Petition for Certiorari with Preliminary Injunction
FACTS
- Petitioner is a corporation operating a jai-alai fronton for sport and amusement.
- It has its own maintenance group for the upkeep of its premises. For the renovation of its
main building, which work is not included in maintenance, it hired private respondents,
Cadatal, Jr., a plumber, and Delgra, a mason, together with 30 other workers on February
2, 1976 for a period of one month, open to extension should the need for the arise in the
course of the renovation.
- Renovation was completed by October 1976. Management then decided to construct an
annex to the building and private respondents worked on the fire escape.

Labor Law 1
- November 27, 1976 Notice of termination given to the respondents effective November
29 but they still continued to work nonetheless. They worked until December 11 and were
fully paid for the work they rendered up to that date.
- December 13, 1976 - Petitioner filed with the former Department of Labor a report of
termination of the services of private respondents and 30 others, listing them as casual
emergency workers. Private workers alleged illegal termination. Assistant Minister
Leogardo ordered the reinstatement of the workers with full backwages before petitioner
could file a reply to the letter-complaint of the respondents.
- Leogardo said that the respondents were already regular employees according to Art.
170 (now Art. 281) of the CC and that termination was unjust.
- An appeal was filed which Clave, in his capacity as Presidential Executive Assistant,
dismissed it.
ISSUE
WON private respondents are regular employees entitled to security of tenure
HELD
NO
Ratio Casual employees are engaged for a specific project or undertaking and fall within
the exception provided for in Article 281 of the Labor Code, supra . Not being regular
employees, it cannot be justifiably said that petitioner had dismissed them without just
cause. They are not entitled to reinstatement with full backwages.
Reasoning
- A281 defines regular and casual employees. In the case at hand, the casual or limited
character of private respondents' employment, therefore, is evident.
- Private respondents were hired for a specific project - to renovate the main budding,
where major repairs such as painting the main building, repair of the roof, cleaning of
clogged water pipes and drains, and other necessary repairs were required.
- It was made known, and so understood at the start of the hiring, that their services would
last until the completion of the renovation. They rendered service from February 2 to
December 11, 1976, almost 11 months, but less than a year.
- There could be no other reason, however, than that the termination of private
respondents was because their services were no longer needed and they had nothing
more to do since the project for which they were hired had been completed.
Disposition Order of public respondent Vicente Leogardo, Jr., dated December 24, 1976,
and the Orders of the other public respondents dated July 13, 1977, January 25, 1979,
March 19, 1979, and June 5, 1980, are hereby reversed and set aside. The Complaint for
illegal dismissal against petitioner in Case No. R04-12-11832-76 LS (Regional Office No.
IV, Department of Labor) is dismissed, and the Temporary Restraining Order heretofore
issued is hereby made permanent.

SANDOVAL V NLRC
136 SCRA 675
AQUINO; May 31, 1985
NATURE
Appeal by certiorari
FACTS
- 5 workers were assigned to the construction of the LCT Catamaran. After three months
of work, the project was completed and the five workers were served a termination notice.
The termination was reported to the Ministry of Labor. The workers filed a complaint for
illegal dismissal.
- The Labor Arbiter ordered the reinstatement of the workers with backwages. The NLRC
affirmed.
- 55 workers were assigned to work in the construction of a tanker. When the tanker was
finished, the personel manager of Sandoval Shipyards terminated the services of the
welders, helpers, and construction workers. The termination was duly reported to the
Ministry of Labor. 17 workers filed a complaint for illegal dismissal.
- The Director of the Ministrys Capital Region ordered the reinstatement of the
complainants. The Deputy Minister of Labor affirmed. Hence this petition.
ISSUE
WON private respondents were project employees whose work was coterminous with the
project for which they were hired
HELD
YES
Ratio The public respondents in the instant two cases acted with grave abuse of
discretion amounting to lack of jurisdiction in disregarding the precedents cited by the
petitioners.
Reasoning
- Project Employees, as distinguished from regular or non-project employees, are
mentioned in Article 281 of the Labor Code, as those where the employment has been

A2010

- 86 -

Disini

fixed for a specific project or undertaking the completion or termination of which has been
determined at the time of the engagement of the employee.
- The petitioner cited three of its own cases wherein the NLRC, Deputy Minister of Labor,
and the Director of the National Capital Region held that the layoff of its project employees
was lawful.
- In the case of In Re: Sandoval Shipyards, Inc. Application for Clearance to Terminate
Employees, it was held that:
It is significant to note that the corporation does not construct vessels for sale or
otherwise which will demand continuous productions of ships and will need permanent
or regular workers
The completion of their work or project automatically terminates their employment
- The other two cases cited affirmed that the workers of the petitioner were project
employees whose employment was terminated upon the completion of the project.
- Respondent Deputy Minister himself affirmed such finding. He ruled that the
complainants are project workers whose employments are coterminous with the
completion of the project, regardless of the number of projects in which they have worked,
as provided under Policy Instructions No. 20 of the Ministry of Labor and Employment
and as their employment is one for a definite period, they are not entitled to separation
pay.
Disposition REVERSED.

IMBUIDO V NLRC (LIBRANDO)


329 SCRA 357
BUENA; March 31, 2000
NATURE
Petition for review on certiorari of the decision of the NLRC
FACTS
- Petitioner was employed as a data encoder by private respondent International
Information Services, Inc., a domestic corporation engaged in the business of data
encoding and keypunching, from August 26, 1988 until October 18, 1991 when her
services were terminated due to "low volume of work".
- Petitioner filed a complaint for illegal dismissal with prayer for service incentive leave pay
and 13th month differential with NLRC alleging that her employment was terminated not
due to the low volume of work but because she "signed a petition for certification election
among the rank and file employees of respondents," thus charging private respondent with
committing unfair labor practices.
- Private respondent maintained that it had valid reasons to terminate petitioner's
employment and disclaimed any knowledge of the existence or formation of a union
among its rank-and-file employees at the time petitioner's services were terminated.
Private respondent stressed that its business ". . . relies heavily on companies availing of
its services. Its retention by client companies with particular emphasis on data encoding is
on a project to project basis," usually lasting for a period of "two (2) to five (5) months."
Private respondent further argued that petitioner's employment was for a "specific project
with a specified period of engagement." According to private respondent, ". . . the certainty
of the expiration of complainant's engagement has been determined at the time of its
engagement (until 27 November 1991) or when the project is earlier completed or when
the client withdraws," as provided in the contract. "The happening of the second event
[completion of the project] has materialized, thus, her contract of employment is deemed
terminated.
ISSUE
WON petitioner is a "project employee" and not a "regular employee" who has security of
tenure
HELD
- We agree with the findings of the NLRC that petitioner is a project employee. The
principal test for determining whether an employee is a project employee or a regular
employee is whether the project employee was assigned to carry out a specific project or
undertaking, the duration and scope of which were specified at the time the employee was
engaged for that project. A project employee is one whose employment has been fixed for
a specific project or undertaking, the completion or termination of which has been
determined at the time of the engagement of the employee or where the work or service to
be performed is seasonal in nature and the employment is for the duration of the season.
In the instant case, petitioner was engaged to perform activities which were usually
necessary or desirable in the usual business or trade of the employer, as admittedly,
petitioner worked as a data encoder for private respondent, a corporation engaged in the
business of data encoding and keypunching, and her employment was fixed for a specific
project or undertaking the completion or termination of which had been determined at the
time of her engagement, as may be observed from the series of employment contracts 32
between petitioner and private respondent, all of which

Labor Law 1
contained a designation of the specific job contract and a specific period of employment.
- However, even as when petitioner is a project employee, according to jurisprudence "[a]
project employee or a member of a work pool may acquire the status of aregular
employee when the following concur:
1) There is a continuous rehiring of project employees even after the cessation of a
project; and
2) The tasks performed by the alleged "project employee" are vital, necessary and
indispensable to the usual business or trade of the employer.
- The evidence on record reveals that petitioner was employed by private respondent as a
data encoder, performing activities which are usually necessary or desirable in the usual
business or trade of her employer, continuously for a period of more than three (3) years,
from August 26, 1988 to October 18, 1991 and contracted for a total of thirteen (13)
successive projects. We have previously ruled that "[h]owever, the length of time during
which the employee was continuouslyre-hired is not controlling, but merely serves as a
badge of regular employment." Based on the foregoing, we conclude that petitioner has
attained the status of a regular employee of private respondent.
- Being a regular employee, petitioner is entitled to security of tenure and could only be
dismissed for a just or authorized cause, as provided in Article 279 of the
Labor Code, as amended:
Art. 279. Security of Tenure In cases of regular employment, the employer shall not
terminate the services of an employee except for a just cause or when
authorized by this Title. An employee who is unjustly dismissed from work shall be
entitled to reinstatement without loss of seniority rights and other privileges
and to his full backwages, inclusive of allowances, and to his other benefits or their
monetary equivalent computed from the time his compensation was withheld
from him up to the time of his actual reinstatement.
- The alleged causes of petitioner's dismissal (low volume of work and belatedly,
completion of project) are not valid causes for dismissal under Articles 282 and 283 of the
Labor Code. Thus, petitioner is entitled to reinstatement without loss of seniority rights and
other privileges, and to her full backwages, inclusive of allowances, and to her other
benefits or their monetary equivalent computed from the time her compensation was
withheld from her up to the time of her actual reinstatement. However, complying with the
principles of "suspension of work" and "no work, no pay" between the end of one project
and the start of a new one, in computing petitioner's backwages, the amounts
corresponding to what could have been earned during the periods from the date petitioner
was dismissed until her reinstatement when private respondent was not undertaking any
project, should be deducted.
- With regard to petitioner's claim for service incentive leave pay, we agree with the labor
arbiter that petitioner is entitled to service incentive leave pay, as provided
in Article 95 of the Labor Code, which reads:
Art. 95: Right to service incentive leave
(a) Every employee who has rendered at least one year of service shall be entitled to a
yearly service incentive leave of five days with pay.
- Having already worked for more than three (3) years at the time of her unwarranted
dismissal, petitioner is undoubtedly entitled to service incentive leave benefits, computed
from 1989 until the date of her actual reinstatement.
Disposition Petition granted.

DE OCAMPO V NLRC
186 SCRA 360
DAVIDE JR; May 7, 2002
NATURE
The petition seeks a reversal of the decision of the respondent NLRC (ordering
respondent to reinstate, without back wages, the individual complainants who were
regular employees except those who were officers of the union among them or paid
separation pay at their option, equivalent to one month's pay or one-half month's pay for
every year of service, whichever is greater. )
FACTS
- On September 30, 1980, the services of 65 employees of private respondent Makati
Development Corporation were terminated on the ground of the expiration of their
contracts. The said employees filed a complaint for illegal dismissal against the MDC on
October 1, 1980; On October 8, 1980, as a result of the aforementioned termination, the
Philippine Transport and General Workers Association, of which the complainants were
members, filed a notice of strike on the grounds of union-busting, subcontracting of
projects which could have been assigned to the dismissed employees, and unfair labor
practice; that on October 14, 1980, the PTGWA declared a strike and established picket
lines in the perimeter of the MDC premises- On November 4, 1980, the MDC filed with the
Bureau of Labor Relations a motion to declare the strike illegal and restrain the workers
from continuing the strike; that on that same day and several days thereafter the MDC
filed applications for clearance to terminate the employment of 90 of the striking workers,

A2010

- 87 -

Disini

whom it had meanwhile preventively suspended; that of the said workers, 74 were project
employees under contract with the MDC with fixed terms of employment; and that on
August 31, 1982, Labor Arbiter Apolinar L. Sevilla rendered a decision 1 denying the
applications for clearance filed by the MDC and directing it to reinstate the individual
complainants with two months back wages each.
- This is the decision modified by the NLRC 2 which is now faulted by the petitioners for
grave abuse of discretion. The contention is that the public respondent acted arbitrarily
and erroneously in ruling that: a) the motion for reconsideration was filed out of time; b)
the strike was illegal; and c) the separation of the project employees was justified
ISSUES
1. WON the strike held by the workers was legal
2. WON the contract workers are considered regular employees
3. WON the project workers are entitled to separation pay
HELD
1. YES
- under the law then in force, to wit, PD No. 823 as amended by PD No. 849, the strike
was indeed illegal. In the first place, it was based not on the ground of unresolved
economic issues, which was the only ground allowed at that time, when the policy was
indeed to limit and discourage strikes. Secondly, the strike was declared only after 6 days
from the notice of strike and before the lapse of the 30-day period prescribed in the said
law for a cooling-off of the differences between the workers and management and a
possible avoidance of the intended strike. That law clearly provided Sec. 1. It is the policy
of the state to encourage free trade unionism and free collective bargaining within the
framework of compulsory and voluntary arbitration. Therefore all forms of strikes, picketing
and lockout are hereby strictly prohibited in vital industries such as in public utilities,
including transportation and communication, companies engaged in the manufacturer
processing as well as in the distribution of fuel gas, gasoline and fuel or lubricating oil, in
companies engaged in the production or processing of essential commodities or products
for export, and in companies engaged in banking of any kind, as well as in hospitals and in
schools and colleges. However, any legitimate labor union may strike and any employer
may lockout in establishments not covered by General Order No. 5 only on grounds of
unresolved economic issues in collective bargaining, in which case the union or the
employer shall file a notice with the Bureau of Labor Relations at least 30 days before the
intended strike or lockout. (Emphasis supplied)
The Court ruled that the leaders of the illegal strike were correctly punished with
dismissal, but their followers (other than the contract workers) were properly ordered
reinstated, considering their lesser degree of responsibility. The penalty imposed upon the
leaders was only proper because it was they who instigated the strike even if they knew,
or should have known, that it was illegal. It was also fair to rule that the reinstated strikers
were not entitled to backpay as they certainly should not be compensated for services not
rendered during the illegal strike. In our view, this is a reasonable compromise between
the demands of the workers and the rights of the employer.
2. The Court stress the rule in Cartagenas v. Romago Electric Co., that contract workers
are not considered regular employees, their services being needed only when there are
projects to be undertaken. 'The rationale of this rule is that if a project has already been
completed, it would be unjust to require the employer to maintain them in the payroll while
they are doing absolutely nothing except waiting until another project is begun, if at all. In
effect, these stand-by workers would be enjoying the status of privileged retainers,
collecting payment for work not done, to be disbursed by the employer from profits not
earned. This is not fair by any standard and can only lead to a coddling of labor at the
expense of management. However, this rule is not applicable in the case at bar. The
record shows that although the contracts of the project workers had indeed expired, the
project itself was still on-going and so continued to require the workers' services for its
completion. There is no showing that such services were unsatisfactory to justify their
termination. It is obvious that the real reason for the termination of their services-which, to
repeat, were still needed-was the complaint the project workers had filed and their
participation in the strike against the private respondent. These were the acts that
rendered them persona non grata to the management. Their services were discontinued
by the MDC not because of the expiration of their contracts, which had not prevented their
retention or rehiring before as long as the project they were working on had not yet been
completed. The real purpose of the MDC was to retaliate against the workers, to punish
them for their defiance by replacing them with more tractable employees.
3. Noteworthy in this connection is Policy Instruction No. 20 of the Department of Labor,
providing that "project employees are not entitled to separation pay if they are terminated
as a result of the completion of the project or any phase thereof in which they are
employed, regardless of the projects in which they had been employed by a particular
construction company." This rule would entitle project employees to separation pay if the
projects they are working on have not yet been completed when their services are
terminated. And this should be true even if their contracts have expired, on the theory that
such contracts would have been renewed anyway because their services were still
needed.
- Applying this rule, The Court held that the project workers who were separated even
before the completion of the project at the New Alabang Village and not really for the

Labor Law 1
reason that their contracts had expired, are entitled to separation pay. Considering the
workers to have been separated without valid cause, the Court shall compute their
separation pay at the rate of one month for every year of service of each dismissed
employee, up to the time of the completion of the project. This is the most equitable way to
treat their claim in light of their cavalier dismissal by the private respondent despite their
long period of satisfactory service with it.
Disposition The appealed decision of the NLRC is affirmed but with the modification that
the contract workers are hereby declared to have been illegally separated before the
expiration of the project they were working on and so are entitled to separation pay
equivalent to one month salary for every year of service.

A.M. ORETA & CO INC V NLRC (GRULLA)


176 SCRA 218
MEDIALDEA; August 10, 1989

NATURE
Petition for certiorari

FACTS
- Private respondent Grulla was engaged by Engineering Construction and Industrial
Development Company (ENDECO) through A.M. Oreta and Co., Inc. as a carpenter in its
project in Jeddah, Saudi Arabia.
- The contract of employment, which was entered into on June 11, 1980 was for a period
of 12 months. Respondent Grulla left the Philippines for Jeddah, Saudi Arabia on August
5, 1980.
- On August 15, 1980, Grulla met an accident which fractured his lumbar vertebrae while
working at the jobsite. He was rushed to the New Jeddah Clinic and was confined there for
12 days.
- On August 27, 1980, Grulla was discharged from the hospital and was told that he could
resume his normal duties after undergoing physical therapy for two weeks.
- On September 18, 1980, respondent Grulla reported back to his Project Manager and
presented to the latter a medical certificate declaring the former already physically fit for
work. Since then, he stated working again until he received a notice of termination of his
employment on October 9, 1980.
- Grulla filed a complaint for illegal dismissal, recovery of medical benefits, unpaid wages
for the unexpired ten (10) months of his contract and the sum of P1,000.00 as
reimbursement of medical expenses against A.M. Oreta and Company, Inc. and ENDECO
with the POEA.
- The petitioner A.M. Oreta and Company, Inc. and ENDECO filed their answer and
alleged that the contract of employment entered into between petitioners and Grulla
provides, as one of the grounds for termination of employment, violation of the rules and
regulations promulgated by the contractor; and that Grulla was dismissed because he has
not performed his duties satisfactorily within the probationary period of three months.
- POEA held that complainant's dismissal was illegal and warrants the award of his wages
for the unexpired portion of the contract.
- Petitioner appealed from the adverse decision to the respondent Commission.
- Respondent Commission dismissed the appeal for lack of merit and affirmed in toto the
decision of the POEA..

ISSUES
1. WON the employment of respondent Grulla was illegally terminated by the petitioner
2. WON Grulla is entitled to salaries corresponding to the unexpired portion of his
employment contract.

HELD
1. YES
- Article 280 (formerly Article 281) of the Labor Code, as amended, provides:
"Article 280. Regular and Casual Employment. - The provisions of written
agreement to the contrary notwithstanding and regardless of the oral agreements of
the parties, an employment shall be deemed to be regular where the employee has
been engaged to perform activities which are usually necessary or desirable in the
usual business or trade of the employer, except where the employment has been

A2010

- 88 -

Disini

fixed for a specific project or undertaking the completion or termination of which has
been determined at the time of the engagement of the employment or where the
work or service to be performed is seasonal in nature and the employment is far the
duration of the season.
"An employment shall be deemed to be casual if it is not covered by the preceding
paragraph: Provided, that any employee who has rendered at least one year of
service, whether such service is continuous or broken, shall be considered a regular
employee with respect to the activity in which he is employed and his employment
shall continue while such actually exists."
- Policy Instructions No. 12 of the then Minister of Labor (now Secretary of Labor and
Employment) which provides:
"PD 850 has defined the concept of regular and casual employment. What
determines regularity or casualness is not the employment contract, written or
otherwise, but the nature of the job. If the job is usually necessary or desirable to
the main business of the employer, then employment is regular. . . ."
- A perusal of the employment contract reveals that although the period of employment of
respondent Grulla is 12 months, the contract period is renewable subject to future
agreement of the parties. It is clear from the employment contract that the respondent
Grulla was hired by the company as a regular employee and not just a mere probationary
employee.
- On the matter of probationary employment, the law in point is Article 281 (formerly Article
252) of the Labor Code which provides in part:
"Art. 281. Probationary Employment. - . . . . The services of an employee who has
been engaged on a probationary basis may be terminated for a just cause or when
he fails to qualify as a regular employee in accordance with reasonable standards
made known by the employer to the employee at the time of his engagement. An
employee who is allowed to work after a probationary period shall be considered
regular employee."
- The law is clear to the effect that in all cases involving employees engaged on
probationary' basis, the employer shall make known to the employee at the time he is
hired, the standards by which he will qualify as a regular employee. Nowhere in the
employment contract executed between petitioner company and respondent Grulla is
there a stipulation that the latter shall undergo a probationary period for three months
before he can quality as a regular employee. There is also no evidence on record showing
that the Grulla had been apprised of his probationary status and the requirements which
he should comply in order to be a regular employee. In the absence of these requisites,
there is justification in concluding that respondent Grulla was a regular employee at the
time he was dismissed by petitioner.
As such, he is entitled to security of tenure during his period of employment and his
services cannot be terminated except for just and authorized causes enumerated under
the Labor Code and under the emloyment contract.
Granting, in gratia argumenti, that respondent is a probationary employee, he cannot,
likewise, be removed except for cause during the period of probation. Although a
probationary or temporary employee has limited tenure, he still enjoys security of tenure.
During his tenure of employment or before his contract expires, he cannot be removed
except for cause as provided for by law.
- The alleged ground of unsatisfactory performance relied upon by petitioner for dismissing
respondent Grulla is not one of the just causes for dismissal provided in the Labor Code.
Neither is it included among the grounds for termination of employment under Article VII of
the contract of employment executed by petitioner company and respondent Grulla.
- Grulla was not, in any manner, notified of the charges against him before he was
outrightly dismissed. Neither was any hearing or investigation conducted by the company
to give the respondent a chance to be heard concerning the alleged unsatisfactory
performance of his work.
2. YES
- The dismissal of Grulla violated the security of tenure under the contract of employment
which specifically provides that the contract term shall be for a period of twelve (12)
calendar months. Consequently, the respondent Grulla should be paid his salary for the
unexpired portion of his contract of employment which is ten (10) months.

Disposition Petition was dismissed


PURE FOODS CORPORATION V NLRC (CORDOVA,
CRUSIS, ET AL)
174 SCRA 415
DAVIDE, JR; December 12, 1997
NATURE
Petition for certiorari
FACTS

Labor Law 1
- Private respondents (numbering 906) were hired by Pure Foods Corporation (PFC) to
work for a fixed period of five months at its tuna cannery plant in Tambler, General Santos
City. After the expiration of their respective contracts of employment in June and July
1991, their services were terminated. They forthwith executed a "Release and Quitclaim"
stating that they had no claim whatsoever against PFC.
- 29 July 1991: private respondents filed before the NLRC Sub-Regional Arbitration
Branch a complaint for illegal dismissal against PFC and its plant manager, Marciano
Aganon. Labor Arbiter Arturo P. Aponesto dismissed the complaint on the ground that the
private respondents were mere contractual workers, and not regular employees; hence,
they could not avail of the law on security of tenure. The termination of their services by
reason of the expiration of their contracts of employment was, therefore, justified.
- On appeal, NLRC affirmed the Labor Arbiter's decision. But on MR, NLRC held that the
private respondent and their co-complainants were regular employees. It declared that the
contract of employment for five months was a "clandestine scheme employed by PFC to
stifle private respondents' right to security of tenure" and should therefore be struck down
and disregarded for being contrary to law, public policy, and morals. Hence, their dismissal
on account of the expiration of their respective contracts was illegal.
- Accordingly, the NLRC ordered PFC to reinstate the private respondents to their former
position without loss of seniority rights and other privileges, with full back wages; and in
case their reinstatement would no longer be feasible, PFC should pay them separation
pay equivalent to one-month pay or one-half-month pay for every year of service,
whichever is higher, with back wages and 10% of the monetary award as attorney's fees.
- PFC's motion for reconsideration was denied. Hence, this petition.
Purefoods Corp's Contention: that the private respondents are now estopped from
questioning their separation from petitioner's employ in view of their express conformity
with the five-month duration of their employment contracts; that the "Release and
Quitclaim" private respondents had executed has unconditionally released PFC from any
and all other claims which might have arisen from their past employment with PFC.
OSG's Comment: the private respondents were regular employees, since they performed
activities necessary and desirable in the business or trade of PFC. The period of
employment stipulated in the contracts of employment was null and void for being contrary
to law and public policy, as its purpose was to circumvent the law on security of tenure.
The expiration of the contract did not, therefore, justify the termination of their
employment. Also, private respondents' quitclaim was ineffective to bar the enforcement
for the full measure of their legal rights.
Private Respondent's Argument: contracts with a specific period of employment may be
given legal effect provided, however, that they are not intended to circumvent the
constitutional guarantee on security of tenure; the practice of PFC in hiring workers to
work for a fixed duration of five months only to replace them with other workers of the
same employment duration was apparently to prevent the regularization of these so-called
"casuals," which is a clear circumvention of the law on security of tenure.
ISSUES
1. WON employees hired for a definite period and whose services are necessary and
desirable in the usual business or trade of the employer are regular employees
2. WON the private respondents' five-month contracts of employment are valid
3. WON the execution by the private respondents of a "Release and Quitclaim" precluded
them from questioning the termination of their services
HELD
1. YES
- Art. 280 of the Labor Code defines regular and casual employment. Under this provision,
there are two kinds of regular employees: (1) those who are engaged to perform
activities which are necessary or desirable in the usual business or trade of the employer;
and (2) those casual employees who have rendered at least one year of service, whether
continuous or broken, with respect to the activity in which they are employed.
- In the instant case, the private respondents' activities consisted in the receiving,
skinning, loining, packing, and casing-up of tuna fish which were then exported by PFC.
Indisputably, they were performing activities which were necessary and desirable in
petitioner's business or trade.
- not hired for a specific project or undertaking. The term "specific project or
undertaking" under Article 280 of the Labor Code contemplates an activity which is not
commonly or habitually performed or such type of work which is not done on a daily basis
but only for a specific duration of time or until completion; the services employed are then
necessary and desirable in the employer's usual business only for the period of time it
takes to complete the project.
2. NO
- None of the criteria, under which term employment cannot be said to be in circumvention
of the law on security of tenure, had been met in the present case.
-Brent School, Inc. v. Zamora ruling on the legality of fixed-term employment has held that
the decisive determinant in term employment should not be the activities that the
employee is called upon to perform but the day certain agreed upon by the parties for the
commencement and termination of their employment relationship. But, where from the
circumstances it is apparent that the periods have been imposed to preclude acquisition of

A2010

- 89 -

Disini

tenurial security by the employee, they should be struck down or disregarded as contrary
to public policy and morals.
-Criteria under which term employment cannot be said to be in circumvention of the
law on security of tenure:
a) The fixed period of employment was knowingly and voluntarily agreed upon by the
parties without any force, duress, or improper pressure being brought to bear upon the
employee and absent any other circumstances vitiating his consent; or
b) It satisfactorily appears that the employer and the employee dealt with each other on
more or less equal terms with no moral dominance exercised by the former over the latter.
- It was shown that it was really the practice of the company to hire workers on a uniformly
fixed contract basis and replace them upon the expiration of their contracts with other
workers on the same employment duration. This scheme of PFC was apparently designed
to prevent the private respondents and the other "casual" employees from attaining the
status of a regular employee. It was a clear circumvention of the employees' right to
security of tenure and to other benefits like minimum wage, cost-of-living allowance, sick
leave, holiday pay, and 13th month pay.
3. NO
- The execution by the private respondents of a "Release and Quitclaim" did not preclude
them from questioning the termination of their services. Generally, quitclaims by laborers
are frowned upon as contrary to public policy and are held to be ineffective to bar recovery
for the full measure of the workers' rights. The reason for the rule is that the employer and
the employee do not stand on the same footing.
Disposition Petition dismissed. NLRC decision affirmed, subject to modification on the
computation of the separation pay and back wages. Since reinstatement is no longer
possible because PFC's tuna cannery plant had, admittedly, been close in November
1994, the proper award is separation pay equivalent to one month pay or one-half month
pay for every year of service, whichever is higher, to be computed from the
commencement of their employment up to the closure of the tuna cannery plant. The
amount of back wages must be computed from the time the private respondents were
dismissed until the time petitioner's cannery plant ceased operation.

LABAYOG V MY SAN BISCUITS INC


494 SCRA 486
CORONA; July 11, 2006
NATURE
Petition for review on certiorari is the resolution of the Court of Appeals.
FACTS
- 1992: petitioners entered into contracts of employment with respondent company as
mixers, packers and machine operators for a fixed term. On the expiration of their
contracts, their services were terminated.
- April 15, 1993: petitioners filed complaints for illegal dismissal, underpayment of wages,
non-payment of overtime, night differential and 13th month pay, damages and attorneys
fees. The labor arbiter ruled their dismissal to be illegal on the ground that they had
become regular employees who performed duties necessary and desirable in respondent
companys business..
- On appeal to the National Labor Relations Commission (NLRC), the decision of the labor
arbiter was set aside.
- CA set aside the NLRC decision and reinstated the decision of the labor arbiter.
However, on respondents motion for reconsideration, the CA reversed itself. The CA
reasoned that, while petitioners performed tasks which were necessary and desirable in
the usual business of respondent company, their employment contracts providing for a
fixed term remained valid. No force, duress, intimidation or moral dominance was exerted
on them. Respondents dealt with petitioners in good faith and within the valid parameters
of management prerogatives.
ISSUE
WON the contract with a fixed period is valid (therefore determining WON the workers
were dismissed illegally).
HELD
YES, contract is valid.
Ratio Contracts of employment for a fixed period are not unlawful. What is objectionable is
the practice of some scrupulous employers who try to circumvent the law protecting
workers from the capricious termination of employment.
Reasoning
- Petitioners were not regular employees. While their employment as mixers, packers and
machine operators was necessary and desirable in the usual business of respondent
company, they were employed temporarily only, during periods when there was
heightened demand for production. There could have been no illegal dismissal when their
services were terminated on expiration of their contracts.

Labor Law 1
ART. 280. (Labor Code) Regular and Casual Employment. The provisions of written
agreement to the contrary notwithstanding and regardless of the oral agreement of the
parties, an employment shall be deemed to be regular where the employee has been
engaged to perform activities which are usually necessary or desirable in the usual
business of the employer, except where the employment has been fixed for a specific
project or undertaking the completion or termination of which has been determined at
the time of the engagement of the employee or where the work or services to be
performed is seasonal in nature and the employment is for the duration of the season.
Where the duties of the employee consist of activities which are necessary or
desirable in the usual business of the employer, the parties are not prohibited from
agreeing on the duration of employment.
- Article 280 does not proscribe or prohibit an employment contract with a fixed period [11]
provided it is not intended to circumvent the security of tenure.
Two criteria validate a contract of employment with a fixed period:
(1) The fixed period of employment was knowingly and voluntarily agreed upon
by the parties without any force, duress or improper pressure being brought to
bear on the employee and without any circumstances vitiating consent;
(2) It satisfactorily appears that the employer and employee dealt with each other
on more or less equal terms with no moral dominance whatever being
exercised by the former on the latter.
Disposition Resolution of CA is affirmed.

CHUA V CA (SOCIAL SECURITY COMMISSION, SSS,


PAGUIO ET AL)
440 SCRA 121
TINGA; October 6, 2004
NATURE
This is a petition for review of the Decision of the Court of Appeals in CA-G.R. CV No.
38269 dated 06 March 1996, and its Resolution dated 30 July 1996 denying petitioners
Motion for Reconsideration, affirming the Order of the Social Security Commission (SSC)
dated 1 February 1995 which held that private respondents were regular employees of the
petitioner and ordered petitioner to pay the Social Security System (SSS) for its unpaid
contributions, as well as penalty for the delayed remittance thereof.
FACTS
- On 20 August 1985, private respondents Andres Paguio, Pablo Canale, Ruel Pangan,
Aurelio Paguio, Rolando Trinidad, Romeo Tapang and Carlos Maliwat (hereinafter referred
to as respondents) filed a Petition with the SSC for SSS coverage and contributions
against petitioner Reynaldo Chua, owner of Prime Mover Construction Development,
claiming that they were all regular employees of the petitioner in his construction business.
Private respondents alleged that petitioner dismissed all of them without justifiable
grounds and without notice to them and to the then Ministry of Labor and Employment.
They further alleged that petitioner did not report them to the SSS for compulsory
coverage in flagrant violation of the Social Security Act.
- On the other hand, the petitioner claimed that private respondents were project
employees, whose periods of employment were terminated upon completion of the
project. Thus, he claimed, no employer-employee relation existed between the parties.
There being no employer-employee relationship, private respondents are not entitled to
coverage under the Social Security Act. Moreover, petitioner invokes the defense of good
faith, or his honest belief that project employees are not regular employees under Article
280 of the Labor Code. The SSC and CA ruled in favor of the respondents.
ISSUE
WON private respondents were regular employees of the petitioner
HELD
YES
Ratio Elements of the control test: (a) selection and engagement of the employee; (b)
payment of wages; (c) the power of dismissal; and (d) the power of control with regard to
the means and methods by which the work is to be accomplished, with the power of
control being the most determinative factor.
- Even though the employer does not admit, the existence of an employer-employee
relationship between the parties can easily be determined by the application of the "control
test, the elements of which are: (a) selection and engagement of the employee; (b)
payment of wages; (c) the power of dismissal; and (d) the power of control with regard to
the means and methods by which the work is to be accomplished, with the power of
control being the most determinative factor.
- There is no dispute that private respondents were employees of petitioner. Petitioner
himself admitted that they worked in his construction projects, although the period of their
employment was allegedly co-terminus with their phase of work. It is clear that private
respondents are employees of petitioner, the latter having control over the results of the
work done, as well as the means and methods by which the same were accomplished.

A2010

- 90 -

Disini

Suffice it to say that regardless of the nature of their employment, whether it is regular or
project, private respondents are subject of the compulsory coverage under the SSS Law,
their employment not falling under the exceptions provided by the law. This rule is in
accord with the Courts ruling in Luzon Stevedoring Corp. v. SSS to the effect that all
employees, regardless of tenure, would qualify for compulsory membership in the SSS,
except those classes of employees contemplated in Section 8(j) of the Social Security Act.
- In Violeta v. NLRC, this Court ruled that to be exempted from the presumption of
regularity of employment, the agreement between a project employee and his employer
must strictly conform to the requirements and conditions under Article 280 of the Labor
Code. It is not enough that an employee is hired for a specific project or phase of work.
There must also be a determination of, or a clear agreement on, the completion or
termination of the project at the time the employee was engaged if the objectives of Article
280 are to be achieved. This second requirement was not met in this case.
- This Court has held that an employment ceases to be co-terminus with specific projects
when the employee is continuously rehired due to the demands of the employers
business and re-engaged for many more projects without interruption. The Court likewise
takes note of the fact that, as cited by the SSC, even the National Labor Relations
Commission in a labor case involving the same parties, found that private respondents
were regular employees of the petitioner.
- Another cogent factor militates against the allegations of the petitioner. In the
proceedings before the SSC and the Court of Appeals, petitioner was unable to show that
private respondents were appraised of the project nature of their employment, the specific
projects themselves or any phase thereof undertaken by petitioner and for which private
respondents were hired. He failed to show any document such as private respondents
employment contracts and employment records that would indicate the dates of hiring and
termination in relation to the particular construction project or phases in which they were
employed. Moreover, it is peculiar that petitioner did not show proof that he submitted
reports of termination after the completion of his construction projects, considering that he
alleges that private respondents were hired and rehired for various projects or phases of
work therein
- To be exempted from the presumption of regularity of employment, the agreement
between a project employee and his employer must strictly conform to the requirements
and conditions under Article 280 of the Labor Code. It is not enough that an employee is
hired for a specific project or phase of work. There must also be a determination of, or a
clear agreement on, the completion or termination of the project at the time the employee
was engaged if the objectives of Article 280 are to be achieved.

C.E. CONSTRUCTION CORP V CIOCO


437 SCRA 648
PUNO; September 8, 2004
NATURE
Two (2) petitions for review of the Decision of the CA which reversed the NLRC, as well as
its Resolution which denied the parties separate motions for reconsideration.
FACTS
- Cioco, et al. (WORKERS) were hired by C.E. Construction Corp. (COMPANY), a
domestic corporation engaged in the construction business. They were hired as
carpenters and laborers in various construction projects from 1990 to 1999, the latest of
which was the GTI Tower in Makati. Prior to the start of every project, the WORKERS
signed individual employment contracts.
- Sometime in May and June 1999, the WORKERS, along with sixty-six (66) others, were
terminated by the COMPANY on the ground of completion of the phases of the GTI Tower
project for which they had been hired. Alleging that they were regular employees, the
WORKERS filed complaints for illegal dismissal with the NLRC. Claims for underpaid
wages and unpaid overtime pay, premium for holiday and rest days, service incentive
leave pay, night shift differential, and 13th month pay were likewise demanded.
ISSUES
1. WON the WORKERS were regular employees of the COMPANY
2. WON the WORKERS were illegally dismissed
HELD
1. NO
Ratio The Labor Arbiter, the NLRC, and the CA, unanimously found that the WORKERS
were project employees of the COMPANY. This finding is binding on this Court. We again
hold that the fact that the WORKERS have been employed with the COMPANY for several
years on various projects, the longest being nine (9) years, did not automatically make
them regular employees considering that the definition of regular employment in Article
280 of the Labor Code, makes specific exception with respect to project employment. The
re-hiring of petitioners on a project-to-project basis did not confer upon them regular
employment status.

Labor Law 1
2. NO
Ratio The labor arbiter categorically found that the appropriate notices to the WORKERS
and the corresponding reports were submitted by the COMPANY to the DOLE. The NLRC
affirmed this finding of fact on appeal. The rule is that factual findings of administrative
agencies, if supported by substantial evidence, are entitled to great weight. More
importantly, no prior notice of termination is required if the termination is brought about by
completion of the contract or phase thereof for which the worker has been engaged.
Disposition The decision of the CA is MODIFIED. The termination of the WORKERS is
declared valid and legal. The award of backwages is set aside.

MARAGUINOT V NLRC
[PAGE 82]
AGUILAR V NLRC (ACEDILLO)
269 SCRA 596
ROMERO; March 13, 1997
NATURE
Petition for certiorari
FACTS
- Romeo Acedillo worked for the petitioner as a helper-electrician. He was dismissed from
allegedly due to lack of available projects and excess in the number of workers needed.
- He filed a case for illegal dismissal before the NLRC upon learning that the petitioner as
hiring new workers and his request to be reinstated was denied. In reply, petitioner
maintained that its need for workers varied, depending on contracts procured in the course
of its business of contracting refrigeration and other related works. According to them,
Acedillo was a contractual employee.
- NLRC ruled in favor of Acedilo. It held that Acedillo was a regular employee, as seen
from the nature of his job and the length of time he has served. The petitioner was also
held liable for the monetary benefits being claimed by Acedillo since employees, whether
regular or not, are entitled to such.
ISSUE
WON Acedillo is a regular employee
HELD
YES
- definition of project employee: a project employee is one whose "employment has been
fixed for a specific project or undertaking, the completion or termination of which has been
determined at the time of the engagement of the employee or where the work or services
to be performed is seasonal in nature and the employment is for the duration of the
season
- petitioner did not specify the duration and scope of the undertaking at the time Acedillo's
services were contracted. Neither is there any proof that the duration of his assignment
was made clear to him other than the self-serving assertion of petitioner that the same can
be inferred from the tasks he was made to perform
- Acedillos work clearly was an activity "necessary or desirable in the usual business or
trade" of petitioner, since refrigeration requires considerable electrical work. This necessity
is further bolstered by the fact that petitioner would hire him anew after the completion of
each project, a practice which persisted throughout the duration of his tenure
- Petitioners assertion that it held 2 sets of workers. This practice renders untenable
petitioner's position that Acedillo is not a regular employee. Citing Philippine National
Construction Corp v NLRC: "Members of a work pool from which a construction company
draws its project employees, if considered employees of the construction company while
in the work pool, are non-project employees or employees for an indefinite period. If they
are employed in a particular project, the completion of the project or any phase thereof will
not mean severance of (the) employer-employee relationship."
Disposition Petition dismissed

ABESCO CONSTRUCTION AND DEVELOPMENT CO V


RAMIREZ
[PAGE 83]
PALOMARES V NLRC (NATIONAL STEEL
CORPORATION)
277 SCRA 439
ROMERO; August 15, 1997
FACTS

A2010

- 91 -

Disini

- Palomares and Mutia was hired by respondent National Steel Corporation (NSC) by
virtue of contracts of employment for its Five Year Expansion Program or FYEP, Phase I
and II-4, for varying lengths of time. Palomares and Mutia asked for regularization, wage
differential, CBA coverage and other benefits. Palomares, Mutia and four other
complainants were adjudged as regular employees of NSC. The NLRC reversed the
findings of the Labor Arbiter. Respondent Commission held that petitioners were project
employees and that their assumption of regular jobs were mainly due to peakloads or the
absence of regular employees during the latter's temporary leave.
- Petitioners argue that as regards functions and duration of work, contracted employees
should, by operation of law, be considered regular employees. Respondent NSC, on the
other hand, maintains that petitioners are mere project employees, engaged to work on
the latter's Five-Year Expansion Projects (FYEP), Phases I and II-A, hence, dismissible
upon the expiration of every particular project.
- Petitioners were employed for a specific project or projects undertaken by respondent
corporation such as the Five Year Expansion Program include the setting up of a Cold
Rolling Mill Expansion Project, establishing a Billet Steel-Making Plant, installation of a
Five Stand TDM and Cold Mill Peripherals Project.
- Petitioners were hired to work on projects for FYEP I and II-A as shown in the records.
On account of the expiration of their contracts of employment and/or project completion,
petitioners were terminated from their employment. They were, however, rehired for other
component projects of the FYEP because they were qualified. Thus, the Court is
convinced that petitioners were engaged only to augment the workforce of NSC for its
aforesaid expansion program.
ISSUE
WON petitioners should be considered regular employees of respondent corporation
HELD
NO
- They should not be. It should be noted that there were intervals in petitioners' respective
employment contracts with NSC, thus bolstering the latter's position that, indeed,
petitioners are project employees. Since its work depends on availability of such contracts
or projects, necessarily the employment of its work force is not permanent but coterminous with the projects to which they are assigned and from whose payrolls
they are paid. It would be extremely burdensome for their employer to retain them as
permanent employees and pay them wages even if there are no projects to work on. The
fact that petitioners worked for NSC under different project employment contracts for
several years cannot be made a basis to consider them as regular employees, for they
remain project employees regardless of the number of projects in which they have worked.
Even if petitioners were repeatedly and successively re-hired on the basis of a contact of
employment for more than one year, they cannot be considered regularized. Length of
service is not the controlling determinant of the employment tenure of a project
employee. As stated earlier, it is based on whether or not the employment has been
fixed for a specific project or undertaking, the completion of which has been
determined at the time of the engagement of the employee. Furthermore, the second
paragraph of Article 280, providing that an employee who has rendered service for at least
one (1) year, shall be considered a regular employee, pertains to casual employees and
not to project employees such as petitioners.
-The principal test for determining whether an employee is a project employee and
not a regular employee is whether he was assigned to carry out a specific project or
undertaking, the duration and scope of which were specified at the time he was
engaged for that project.
Disposition instant petition is DISMISSED. The decision and resolution of the National
Labor Relations Commission dated November 23, 1994 and March 23, 1995, respectively,
are AFFIRMED.

FILIPINAS PRE-FABRICATED BUILDING SYSTEMS INC


V PUENTE
453 SCRA 820
PANGANIBAN; March 18, 2005
NATURE
Petition for Review
FACTS
- Respondent Puentes contention:
> That he began working with Petitioner Filsystems, Inc., a corporation engaged in
construction business, on June 12, 1989; that he was initially hired by [petitioner] company
as an installer; that he was later promoted to mobile crane operator and was stationed at
the company premises in Quezon City; that his work was not dependent on the completion
or termination of any project; that since his work was not dependent on any project, his
employment with the Filsystems was continuous and without interruption for the past 10

Labor Law 1
years; that on Oct. 1, 1999, he was dismissed from his employment allegedly because he
was a project employee. He filed complaint for illegal dismissal against the petitioner.
- Petitioner-companys claims
> That complainant was hired as a project employee in the companys various projects;
that his employment contracts showed that he was a project worker with specific project
assignments; that after completion of each project assignment, his employment was
likewise terminated and the same was correspondingly reported to the DOLE.
Labor Arbiter dismissed complaint. NLRC affirmed. CA reversed LA and NLRC rulings
holding that respondent was a regular employee of petitioners.
ISSUES
1. WON respondent Roger Puente is a project employee
2. WON he is entitled to reinstatement with full back wages
HELD
1. YES
Ratio Provisions in the Labor Code and DOLE Order No. 19(1993) make it clear that a
project employee is one whose employment has been fixed for a specific project or
undertaking the completion or termination of which has been determined at the time of the
engagement of the employee or where the work or services to be performed is seasonal in
nature and the employment is for the duration of the season. It is a settled rule that the
length of service of a project employee is not the controlling test of employment tenure but
whether or not the employment has been fixed for a specific project or undertaking the
completion or termination of which has been determined at the time of the engagement of
the employee.
Reasoning
1) The contracts of employment of Puente show that he was hired for specific projects.
His employment was coterminous with the completion of the projects for which he had
been hired. Those contracts expressly provided that his tenure of employment depended
on the duration of any phase of the project or on the completion of the construction
projects. Also, petitioners regularly submitted to DOLE reports of the termination of
services of project workers. Such compliance with the requirement confirms that
respondent was a project employee
2) Evidently, although the employment contract did not state a particular date, it did specify
that the termination of the parties employment relationship was to be on a day certain -the day when the phase of work termed Lifting & Hauling of Materials for the World
Finance Plaza project would be completed. Thus, respondent cannot be considered to
have been a regular employee. He was a project employee.
3) That he was employed with Filsystems for 10 yrs. in various projects did not ipso facto
make him a regular employee, considering that the definition of regular employment in
Art.280 of the Labor Code makes a specific exception with respect to project employment.
The mere rehiring of respondent on a project-to-project basis did not confer upon him
regular employment status. The practice was dictated by the practical consideration that
experienced construction workers are more preferred. It did not change his status as a
project employee.
2. YES
Ratio In termination cases, the burden of proving that an employee has been lawfully
dismissed lies with the employer. Employers who hire project employees are mandated to
state and, once its veracity is challenged, to prove the actual basis for the latters
dismissal.
Reasoning
- There is no allegation or proof, however, that the World Finance Plaza project -- or the
phase of work therein to which respondent had been assigned -- was already completed
by Oct.1, 99, the date when he was dismissed. The inescapable presumption is that his
services were terminated for no valid cause prior to the expiration of the period of his
employment; hence, the termination was illegal. Reinstatement with full back wages,
inclusive of allowances and other benefits or their monetary equivalents -- computed from
the date of his dismissal until his reinstatement -- is thus in order. If reinstatement no
longer possible due to the completion of the project during the pendency of this case, he
must be entitled to salary and benefits of the unexpired portion of his employment.
Disposition Petition is PARTLY GRANTED.

7.05 CASUAL EMPLOYEES


NATURE OF WORK

A2010

- 92 -

Disini

A.M. ORETA & CO INC V NLRC (GRULLA)


[PAGE 88]
ONE YEAR SERVICE
KIMBERLY INDEPENDENT LABOR UNION V DRILON
185 SCRA 190
REGALADO; May 9, 1990
FACTS
- Kimberly-Clark Philippines, Inc. (KIMBERLY) executed a three-year collective bargaining
agreement (CBA) with United Kimberly-Clark Employees Union-Philippine Transport and
General Workers' Organization (UKCEUPTGWO) which expired on June 30, 1986.
- Within the 60-day freedom period prior to the expiration of and during the negotiations for
the renewal of the aforementioned CBA, some members of the bargaining unit formed
another union called "Kimberly Independent Labor Union for Solidarity, Activism and
Nationalism-Organized Labor Association in Line Industries and Agriculture (KILUSANOLALIA)
- April 21, 1986, KILUSAN-OLALIA filed a petition for certification election. KIMBERLY and
UKCEU-PTGWO did not object to the holding of a certification election but objected to the
inclusion of the so-called contractual workers whose employment with KIMBERLY was
coursed through an independent contractor, Rank Manpower Company (RANK, for short),
as among the qualified voters.
- On June 2, 1986, Med-Arbiter Bonifacio I. Marasigan, who was handling the certification
election case issued an order declaring the following as eligible to vote in the certification
election, thus:
1) regular rank-and-file laborers/employees of the respondent company; 2) casuals who
have worked at least six (6) months; 3) Contractual employees who are allegedly in the
employ of an independent contractor and who have also worked for at least six (6) months
- During the pre-election conference, 64 casual workers were challenged by KIMBERLY
and UKCEU-PTGWO on the ground that they are not employees of KIMBERLY but of
RANK. It was agreed by all the parties that the 64 voters shall be allowed to cast their
votes but that their ballots shall be segregated and subject to challenge proceedings.
- After the elections, UKCEU-PTGWO won over KILUSAN-OLALIA by 20 votes. This count
considered the votes of the 64 employees as separate.
- In a case regarding the status of the 64 employees in relation to the certification election,
it was held by med-arbiter Sanchez that:
2)The other casual employees not performing janitorial and yard maintenance
services were deemed labor-only contractuals and since labor-only contracting is
prohibited, such employees were held to have attained the status of regular
employees, the regularization being effective as of the date of the decision;
3. UKCEU-PTGWO, having garnered more votes than KILUSAN-OLALIA, was certified
as the exclusive bargaining representative of KlMBERLY's employees;
- Since the members were only considered regular at the time of the decision, their votes
were not re-considered as regards the election.
- winning union and company executed a CBA
- KIMBERLY-OLALIA filed for a TRO on the CBA and included the question of the status of
the 64 members in question.
ISSUE
WON the 64 employees were regular employees at the time of the certification election
HELD
YES
- A280LC provides for two kinds of regular employees: (1) those who are engaged to
perform activities which are usually necessary or desirable in the usual business or trade
of the employer; and (2) those who have rendered at least one year of service, whether
continuous or broken, with respect to the activity in which they are employed
- The individual petitioners herein who have been adjudged to be regular employees (by
law) fall under the second category. These are the mechanics, electricians, machinists,
machine shop helpers, warehouse helpers, painters, carpenters, pipefitters and masons. It
is not disputed that these workers have been in the employ of KIMBERLY for more than
one year at the time of the filing of the petition for certification election by KILUSANOLALIA.
- While the actual regularization of these employees entails the mechanical act of issuing
regular appointment papers and compliance with such other operating procedures as may
be adopted by the employer, it is more in keeping with the intent and spirit of the law to
rule that the status of regular employment attaches to the casual worker on the day
immediately after the end of his first year of service

Labor Law 1
- The law is explicit. As long as the employee has rendered at least one year of service, he
becomes a regular employee with respect to the activity in which he is employed. The law
does not provide the qualification that the employee must first be issued a regular
appointment or must first be formally declared as such before he can acquire a regular
status. Obviously, where the law does not distinguish, no distinction should be drawn.
- On the basis of the foregoing circumstances, and as a consequence of their status as
regular employees, those workers not perforce janitorial and yard maintenance service
were performance entitled to the payment of salary differential, cost of living allowance,
13th month pay, and such other benefits extended to regular employees under the CBA,
from the day immediately following their first year of service in the company.
-These regular employees are likewise entitled to vote in the certification election held in
July 1, 1986. Consequently, the votes cast by those employees not performing janitorial
and yard maintenance service, which forms part of the 64 challenged votes, should be
opened, counted and considered for the purpose of determining the certified bargaining
representative.

SAN MIGUEL CORP V ABELLA


[PAGE 59]
INTEGRATED CONTRACTOR V NLRC (SOLON)

464 SCRA 265


QUISUMBING; August 9, 2005
NATURE
Appeal from a decision of the CA affirming the NLRCs findings which declared respondent
Solon a regular employee of the petitioner and awarded him with 13 th month pay, service
incentive leave pay, reinstatement to his former position with full backwages from the time
his salary was withheld until his reinstatement.
FACTS
- Petitioner is a plumbing contractor. Its business depends on the number and frequency
of the projects it is able to contract with its clients.
- Respondent Solon worked for petitioner several months at a time from 1994 to 1998.
- On Feb. 1998, while Solon was about to log out from work, he was informed that it was
his last day of work as he had been terminated. He went back to petitioners office to sign
a clearance so he could claim his 13th month pay and tax refunds. However, he refused to
sign when he read the clearance indicating that he had resigned. He then filed a complaint
for illegal dismissal without due cause and due process.
- The Labor Arbiter ruled that Solon was a regular employee and could only be removed
for cause. NLRC affirmed with only a modification as to the computation of 13 th month pay.
CA also affirmed.
ISSUE
WON respondent is a regular employee

HELD
YES
Ratio The test to determine whether employment is regular or not is the reasonable
connection between the particular activity performed by the employee in relation to the
usual business or trade of the employer. Also, if the employee has been performing the job
for at least one year, even if the performance is not continuous or merely intermittent, the
law deems the repeated and continuing need for its performance as sufficient evidence of
the necessity, if not indispensability of that activity to the business. (De Leon v NLRC)
Reasoning
- While length of time may not be the controlling test for project employment, it is vital in
determining if the employee was hired for a specific undertaking or tasked to perform
functions vital, necessary and indispensable to the usual business or trade of the
employer. Here, private respondent had been a project employee several times over. His
employment ceased to be coterminous with specific projects when he was repeatedly rehired due to the demands of petitioners business.
Disposition assailed Decision dated October 30, 2001 and the Resolution dated February
28, 2002 of CA are AFFIRMED with MODIFICATION. The petitioner id hereby ORDERED
to (1) reinstate the respondent with no loss of seniority rights and other privileges; and (2)
pay respondent his backwages, 13 th month pay for the year 1998 and Service Incentive
Leave Pay computed from the date of his illegal dismissal up to the date of his actual
reinstatement.

A2010

- 93 -

Disini

7.06 CONTRACT FIXED PERIOD


TESTS VALIDITY
BRENT SCHOOL V ZAMORA
181 SCRA 702
NARVASA; February 5, 1990
NATURE
Appeal from judgment
FACTS
- Alegre was athletic director at Brent, at a yearly compensation of P20,000. Her contract
fixed a specific term of 5 years for its existence, from July, 1971, to July, 1976.
Subsequent subsidiary agreements in March 1973, August 1973, and Sept. 1974
reiterated the same terms and conditions, including the expiry date, as those contained in
the original contract of July, 1971.
- 3 months before the expiration of the stipulated period, in April 1976, Alegre was given a
copy of the report filed by Brent with the Dep. Of Labor advising of the termination of his
services, effective July 16, 1976. The stated ground for termination was completion of
contract, expiration of the definite period of employment.
- Alegre protested the announced termination of his employment. He argued that although
his contract did stipulate that the same would terminate on July 17, 1976, since his
services were necessary and desirable in the usual business of his employer, and his
employment had lasted for five years, he had acquired the status of regular employee and
could not be removed except for valid cause.
- The employment contract of 1971 was executed when the Labor Code of the Philippines
had not yet been promulgated, which came into effect some 3 years after the perfection of
the contract.
ISSUE
WON the provisions of the Labor Code (regarding probationary/regular employees) have
anathematized fixed period employment or employment for a term
HELD
NO
Reasoning
- Before the Labor Code, there was no doubt about the validity of term employment. It was
impliedly but clearly recognized by the Termination Pay law, RA 105220
- The Civil Code, which has always recognized, and continues to recognize, the validity
and propriety of contracts and obligations with a fixed or definite period, and imposes no
restraints on the freedom of the parties to fix the duration of a contract, whatever its object,
be it specie, goods or services, except the general admonition against stipulations
contrary to law, morals, good customs, public order or public policy. Under the Civil Code,
therefore, and as a general proposition, fixed-term employment contracts are not limited,
as they are under the present Labor Code, to those by nature seasonal or for specific
projects with pre-determined dates of completion; they also include those to which the
parties by free choice have assigned a specific date of termination.
-(theres a long, long history about the changes of provisions in the labor code, showing
how fixed period employment became less and less acceptable...)
- Where from the circumstances it is apparent that periods have been imposed to preclude
acquisition of tenurial security by the employee, they should be struck down or
disregarded as contrary to public policy, morals, etc.
- But where no such intent to circumvent the law is shown, where the reason for the law
does not exist, e.g., where it is indeed the employee himself who insists upon a period or
where the nature of the engagement is such that, without being seasonal or for a specific
project, a definite date of termination is a sine qua non, would an agreement fixing a
period be essentially evil or illicit, therefore anathema? Would such an agreement come

20

In cases of employment, without a definite period, in a commercial, industrial, or agricultural establishment or


enterprise, the employer or the employee may terminate at any time the employment with just cause; or without just
cause in the case of an employee by serving written notice on the employer at least one month in advance, or in the
case of an employer, by serving such notice to the employee at least one month in advance or one-half month for every
year of service of the employee, whichever is longer, a fraction of at least six months being considered as one whole
year.
The employer, upon whom no such notice was served in case of termination of employment without just cause, may hold
the employee liable for damages.
The employee, upon whom no such notice was served in case of termination of employment without just cause, shall be
entitled to compensation from the date of termination of his employment in an amount equivalent to his salaries or wages
corresponding to the required period of notice.

Labor Law 1
within the scope of Article 280 21 which admittedly was enacted "to prevent the
circumvention of the right of the employee to be secured in (his) employment?"
- A280LC, under a narrow and literal interpretation would appear to restrict, without
reasonable distinctions, the right of an employee to freely stipulate with his employer the
duration of his engagement, it logically follows that such a literal interpretation should be
eschewed or avoided. The law must be given a reasonable interpretation, to preclude
absurdity in its application. Outlawing the whole concept of term employment and
subverting to boot the principle of freedom of contract to remedy the evil of employer's
using it as a means to prevent their employees from obtaining security of tenure is like
cutting off the nose to spite the face or, more relevantly, curing a headache by lopping off
the head.
- Familiar examples of employment contracts which may be neither for seasonal work nor
for specific projects, but to which a fixed term is essential: overseas employment contracts
to which, the concept of regular employment will all that it implies does not appear ever to
have been applied, Article 280 of the Labor Code not withstanding; appointments to the
positions of dean, assistant dean, college secretary, principal, and other administrative
offices in educational institutions, which are by practice or tradition rotated among the
faculty members, and where fixed terms are a necessity, without which no reasonable
rotation would be possible. Despite the provisions of Article 280, Policy, Instructions No. 8
of the Minister of Labor implicitly recognize that certain company officials may be elected
for what would amount to fixed periods, at the expiration of which they would have to
stand down, in providing that these officials," . . . may lose their jobs as president,
executive vice-president or vice-president, etc. because the stockholders or the board of
directors for one reason or another did not re-elect them."
- Since the entire purpose behind the development of legislation culminating in the present
Article 280 of the Labor Code clearly appears to have been to prevent circumvention of
the employee's right to be secure in his tenure, the clause in said article indiscriminately
and completely ruling out all written or oral agreements conflicting with the concept of
regular employment as defined therein should be construed to refer to the substantive evil
that the Code itself has singled out: agreements entered into precisely to circumvent
security of tenure. It should have no application to instances where a fixed period of
employment was agreed upon knowingly and voluntarily by the parties, without any force,
duress or improper pressure being brought to bear upon the employee and absent any
other circumstances vitiating his consent, or where it satisfactorily appears that the
employer and employee dealt with each other on more or less equal terms with no moral
dominance whatever being exercised by the former over the latter. Unless thus limited in
its purview, the law would be made to apply to purposes other than those explicitly stated
by its framers; it thus becomes pointless and arbitrary, unjust in its effects and apt to lead
to absurd and unintended consequences.
Disposition the public respondent's Decision complained of is REVERSED and SET
ASIDE. Respondent Alegre's contract of employment with Brent School having lawfully
terminated with and by reason of the expiration of the agreed term of period thereof, he is
declared not entitled to reinstatement and the other relief awarded and confirmed on
appeal in the proceedings below.

CIELO V NLRC
MILLARES V NLRC (TRANS-GLOBAL MARITIME
AGENCY, ESSO INTERNATIONAL SHIPPING
COMPANY)
[PAGE 79]

VIERNES V NLRC (BENGUET ELECTRIC COOP)


400 SCRA 557
AUSTRIA-MARTINEZ; April 4, 2003
21

Art. 280. Regular and casual employment. The provisions of written agreement to the contrary notwithstanding and
regardless of the oral agreement of the parties, an employment shall be deemed to be regular where the employee has
been engaged to perform activities which are usually necessary or desirable in the usual business or trade of the
employer, except where the employment has been fixed for a specific project or undertaking the completion or
termination of which has been determined at the time of the engagement of the employee or where the work or service
to be performed is seasonal in nature and the employment is for the duration of the season.
An employment shall be deemed to be casual if it is not covered by the preceding paragraph: Provided, That any
employee who has rendered at least one year of service, whether such service is continuous or broken, shall be
considered a regular employee with respect to the activity in which he is employed and his employment shall continue
while such activity exists.

A2010

- 94 -

Disini

FACTS
- The 15 complainants services were contracted as meter readers by Benguet Electric
Cooperative (BENECO) for less than a months duration from October 8 to 31, 1990.
Their employment contracts, couched in identical terms, read:
You are hereby appointed as METER READER (APPRENTICE) under BENECO-NEA
Management with compensation at the rate of SIXTY-SIX PESOS AND SEVENTYFIVE CENTAVOS (P66.75) per day from October 08 to 31, 1990.
- The said term notwithstanding, the complainants were allowed to work beyond October
31, 1990, or until January 2, 1991. On January 3, 1991, they were each served their
identical notices of termination dated December 29, 1990. The same read:
Please be informed that effective at the close of office hours of December 31, 1990, your
services with the BENECO will be terminated. Your termination has nothing to do with
your performance. Rather, it is because we have to retrench on personnel as we are
already overstaffed.
- The complainants filed separate complaints for illegal dismissal. It is the contention of
the complainants that they were not apprentices but regular employees whose services
were illegally and unjustly terminated in a manner that was whimsical and capricious. On
the other hand, the respondent invokes Article 283 of the Labor Code in defense of the
questioned dismissal.
- The Labor Arbiter dismissed the complaints for illegal dismissal but directed BENECO to
extend the contract of each complainant, with the exception of Viernes who was ordered
to be appointed as regular employee, a months salary as indemnity for failure to give the
30-day notice, and backwages.
- The NLRC declared the complainants dismissal illegal, thus ordering their reinstatement
to their former position as meter readers or to any equivalent position with payment of
backwages limited to one year but deleting the award of indemnity and attorneys fees.
The award of underpayment of wages was affirmed.

ISSUES
1. WON the NLRC committed grave abuse of discretion in ordering the reinstatement of
petitioners to their former position as meter readers on probationary status in spite of its
finding that they are regular employees under Article 280 of the Labor Code
2. WON the NLRC committed grave abuse of discretion in limiting the backwages of
petitioners to one year only in spite of its finding that they were illegally dismissed, which is
contrary to the mandate of full backwages until actual reinstatement but not to exceed 3
years
3. WON the NLRC committed grave abuse of discretion in deleting the award of indemnity
pay which had become final because it was not appealed and in deleting the award of
attorneys fees because of the absence of a trial-type hearing
4. WON the mandate of immediately executory on the reinstatement aspect even pending
appeal as provided in the decision of Labor Arbiters equally applies in the decision of the
NLRC even pending appeal, by means of a motion for reconsideration of the order
reinstating a dismissed employee or pending appeal because the case is elevated on
certiorari before the Supreme Court

HELD
1. YES
Ratio There are two separate instances whereby it can be determined that an
employment is regular: (1) The particular activity performed by the employee is necessary
or desirable in the usual business or trade of the employer; or (2) if the employee has
been performing the job for at least a year.
Reasoning
- Petitioners fall under the first category. They were engaged to perform activities that are
necessary to the usual business of BENECO. We agree with the labor arbiters
pronouncement that the job of a meter reader is necessary to the business of BENECO
because unless a meter reader records the electric consumption of the subscribing public,
there could not be a valid basis for billing the customers. The fact that the petitioners were
allowed to continue working after the expiration of their employment contract is evidence
of the necessity and desirability of their service to BENECOs business. In addition, during
the preliminary hearing of the case on February 4, 1991, BENECO even offered to enter
into another temporary employment contract with petitioners. This only proves BENECOs
need for the services of the petitioners. With the continuation of their employment beyond
the original term, petitioners have become full-fledged regular employees. The fact alone
that the petitioners have rendered service for a period of less than 6 months does not
make their employment status as probationary.
- The principle [exception to the rule in Ratio] enunciated in Brent School vs. Zamora
applies only with respect to fixed term employments. While it is true that petitioners were
initially employed on a fixed term basis as their employment contracts were only for
October 8 to 31, 1990, after October 31, 1990, they were allowed to continue working in

Labor Law 1
the same capacity as meter readers without the benefit of a new contract or agreement or
without the term of their employment being fixed anew. After October 31, 1990, the
employment of petitioners is no longer on a fixed term basis. The complexion of the
employment relationship of petitioners and BENECO is thereby totally changed.
Petitioners have attained the status of regular employees.
2. YES
Reasoning
- A279 LC, as amended by RA 6715 [effective March 21, 1989], provides that an illegally
dismissed employee is entitled to full back wages, inclusive of allowances, and to his other
benefits or their monetary equivalent computed from the time his compensation was
withheld from him up to the time of his actual reinstatement. Since petitioners were
employed on October 8, 1990, the amended provision shall apply to the present case.
Hence, it was patently erroneous, tantamount to grave abuse of discretion on the part of
the NLRC in limiting to one year the back wages awarded to petitioners.
3. YES
Ratio An employer becomes liable to pay indemnity to an employee who has been
dismissed if, in effecting such dismissal, the employer fails to comply with the
requirements of due process
Reasoning
- The indemnity is in the form of nominal damages intended not to penalize the employer
but to vindicate or recognize the employees right to procedural due process which was
violated by the employer. Under A2221 CC, nominal damages are adjudicated in order
that a right of the plaintiff, which has been violated or invaded by the defendant, may be
vindicated or recognized, and not for the purpose of indemnifying the plaintiff for any loss
suffered by him.
- Indemnity is not incompatible with the award of back wages. These two awards are
based on different considerations. Back wages are granted on grounds of equity to
workers for earnings lost due to their illegal dismissal from work. On the other hand, the
award of indemnity is meant to vindicate or recognize the right of an employee to due
process which has been violated by the employer. In this case, BENECO failed to comply
with the provisions of Article 283 of the Labor Code which requires an employer to serve a
notice of dismissal upon the employees and to the Department of Labor, at least one
month before the intended date of termination. As to the award of attorneys fees, the
same is justified by the provisions of Article 111 of the Labor Code.
4. YES
Reasoning
- A223 LC is plain and clear that the decision of the NLRC shall be final and executory
after 10 calendar days from receipt by the parties. In addition, Section 2(b), Rule VIII of
the New Rules of Procedure of the NLRC provides that should there be a motion for
reconsideration entertained pursuant to Section 14, Rule VII of these Rules, the decision
shall be executory after 10 calendar days from receipt of the resolution on such motion.
We find nothing inconsistent or contradictory between the two. The provision of the NLRC
Rules of Procedure merely provides for situations where a motion for reconsideration is
filed. Since the Rules allow the filing of a motion for reconsideration of a decision of the
NLRC, it simply follows that the ten-day period provided under Article 223 of the Labor
Code should be reckoned from the date of receipt by the parties of the resolution on such
motion. In the case at bar, petitioners received the resolution of the NLRC denying their
motion for reconsideration on October 22, 1992. Hence, it is on November 2, 1992 that
the questioned decision became executory.
Disposition Petition PARTLY GRANTED. Decision of the NLRC is MODIFIED. BENECO
is ordered to reinstate petitioners to their former or substantially equivalent position as
regular employees, without loss of seniority rights and other privileges, with full back
wages from the time of their dismissal until they are actually reinstated. The indemnity to
petitioners is REINSTATED. BENECO is also ordered to pay attorneys fees in the amount
of 10% of the total monetary award due to the petitioners. In all other respects the
assailed decision and resolution are AFFIRMED.

PHILIPS SEMICONDUCTORS V FADRIQUELA


[PAGE 77]
PANGILINAN V GENERAL MILLING CORPORATION
[PAGE 74]

SEASONAL EMPLOYEES
MAGCALAS V NLRC (KOPPEL INC)

A2010

- 95 -

Disini

269 SCRA 453


PANGANIBAN; March 13, 1997
NATURE
Appeal from decision of the NLRC
FACTS
- Complainants alleged that they were all regular employees of the respondent company,
having rendered continuous services in various capacities, ranging from leadman,
tinsmith, tradeshelper to general clerk.
- The complainants have worked for a number of years, the minimum of which was 1.5
years and the maximum was 8 years under several supervisors.
- August 30, 1988: they were dismissed without prior notice and investigation, and that
their dismissals were effected for no other cause than their persistent demands for
payment of money claims as mandated by law.
- Respondent company averred that the manufacturing aspect of its operation is handled
by its regular employees, while the installation aspect, by reason of its intermittence, is
carried out by its project or contract employees.
- The complainants herein were among the contract employees hired by the respondent to
install the air-conditioning equipment at the ADB and Interbank projects.
- With the completion of their task in their respective projects, the employment of the
complainants expired as they had no more work to do.
- Labor arbiter ordered petitioners to be reinstated and to be paid backwages from the
time of their dismissal/termination to their actual reinstatement.
- NLRC reversed decision and ordered respondent to pay the petitioners their separation
pay.
ISSUES
1. WON petitioners were regular workers under the contemplation of A280 LC
2. WON petitioners' termination and/or cessation of their employments on August 30, 1988
were justified under the contemplation of A279 of the LC as amended.
HELD
1. YES
Ratio The services of project employees are co-terminous with the project and may be
terminated upon the end or completion of the project for which they were hired. Regular
employees, in contrast, are legally entitled to remain in the service of their employer until
that service is terminated by one or another of the recognized modes of termination of
service under the Labor Code.
Reasoning
- A mere provision in the CBA recognizing contract employment does not sufficiently
establish that petitioners were ipso facto contractual or project employees. In the same
vein, the invocation of Policy No. 20 governing the employment of project employees in
the construction industry does not, by itself, automatically classify private respondent as
part of the construction industry and entitle it to dismiss petitioners at the end of each
project. These facts cannot be presumed; they must be supported by substantial evidence
- The record discloses that the complainants worked not only in one special project but
also variably in other projects/jobsites contracted by Koppel Incorporated. Some of them,
after their tour of duty on these different jobsites were reassigned to the respondent's
plant at Koppel Compound, Paraaque, Metro Manila. A close examination of the record
further reveals that the "special projects" at the ADB and Interbank to which the
complainants were last assigned by the respondent were still in operation before their
alleged termination from employment. Under these factual milieu, we believe that they had
been engaged to work and perform activities which were necessary and desirable in the
air-conditioning and refrigeration installation/repair business of the respondent employer,
especially where, as in this case, the very nature of such trade indicates that it can hardly
fall under the exception of Policy Instruction No. 20 which applies only to the construction
industry. For this reason, and considering the facts narrated in the complainants' sworn
statements were neither disputed nor refuted by contrary evidence by the respondent, it
becomes apparent and increasingly clear that indeed they would and ought to be
classified as regular employees.
- Regular employees cannot at the same time be project employees. Article 280 of the
Labor Code states that regular employees are those whose work is necessary or desirable
to the usual business of the employer. The two exceptions following the general
description of regular employees refer to either project or seasonal employees
- In the realm of business and industry, we note that "project" could refer to one or the
other of at least two distinguishable types of activities. Firstly, a project could refer to
particular job or undertaking that is within the regular or usual business of the employer
company, but which is distinct and separate, and identifiable as such, from the other
undertakings of the company. Such job or undertaking begins and ends at determined or
determinable times. Employees who are hired for the carrying out of one of these separate
projects, the scope and duration of which has been determined and made known to the
employees at the time of employment, are properly treated as "project employees," and
their services may be lawfully terminated at completion of the project.

Labor Law 1
- The employment of seasonal employees, on the other hand, legally ends upon
completion of the project or the season.
- The overwhelming fact of petitioners' continuous employment as found by the labor
arbiter ineludibly shows that the petitioners were regular employees. On the other hand,
we find that substantial evidence, applicable laws and jurisprudence do not support the
ruling in the assailed Decision that petitioners were project employees. The Court here
reiterates the rule that all doubts, uncertainties, ambiguities and insufficiencies should be
resolved in favor of labor. It is a well-entrenched doctrine that in illegal dismissal cases,
the employer has the burden of proof. This burden was not discharged in the present
case.
2. NO
Ratio For a dismissal of an employee to be valid, two requisites must be met: (1) the
employee is afforded due process, meaning, he is given notice of the cause of his
dismissal and an adequate opportunity to be heard and to defend himself; and (2) the
dismissal is for a valid cause as indicated in Article 282 of the Labor Code.
Reasoning
- The services of petitioners were purportedly terminated at the end of the ADB and
Interbank projects, but this could not have been a valid cause for, as discussed above,
they were regular and not project employees.
- As a consequence of their illegal termination, petitioners are entitled to reinstatement and
backwages in accordance with the Labor Code. The backwages however are to be
computed only for three years fromthe date of their dismissal, without deduction or
qualification.
- Where the illegal dismissal transpired before the effectivity of RA 6715 (March 21, 1989),
the award of backwages in favor of the dismissed employees is limited to three years
without deduction or qualification.
Disposition The petition is GRANTED. The assailed Decision and Resolution are
REVERSED and SET ASIDE and the decision of the labor arbiter is REINSTATED, with
backwages to be computed as above discussed.

PHILIPPINE TOBACCO V NLRC


300 SCRA 37
PANGANIBAN; December 10, 1998
NATURE
Petition for review on certiorari
FACTS
- This involves 2 groups of seasonal workers: Lubat group and Luris group. They claimed
separation benefits after closure of processing plant and transfer of tobacco operations to
Ilocos. Petitioner refused to grant separation pay to Lubat group because they had not
been given work during preceding year. It also refused to grant same to Luris group
because of closure due to serious business losses.
ISSUES
1. WON the Luris group has been illegally dismissed
2. WON the Lubat group has been illegally dismissed
HELD
1. YES
- Serious business losses were not proven.
- To justify retrenchment: (1) losses expected should be substantial and not merely de
minimis. (2) substantial loss must be reasonably imminent. (3) retrenchment must be
reasonably necessary. Employer should have taken other measures. (4) alleged losses
must be proven by sufficient and convincing evidence.
- Here, we consider the fact that the petitioner did not actually close its operations but
merely transferred its processing and redrying operations.
- It was also engaged in corn and rental operations.
- Notice of termination, though issued, violated the one month prior notice requisite.
2. YES
- SEASONAL WORKERS WHO ARE CALLED FROM TIME TO TIME AND ARE
TEMPORARILY LAID OFF DURING OFF-SEASON ARE NOT SEPARATED FROM
SERVICE IN SAID PERIOD, BUT ARE MERELY CONSIDERED ON LEAVE UNTIL
REEMPLOYED.
- The employer-employee relationship between petitioner and Lubat group was not
terminated at the end of 1993 season.

SAN MIGUEL CORPORATION V NLRC (GUZMAN)


[PAGE 75]
MANILA HOTEL COMPANY V CIR
9 SCRA 184
BAUTISTA ANGELO; September 30, 1963

A2010

- 96 -

Disini

NATURE
Appeal from CIRs judgment
FACTS
- February 24, 1960 - The Pines Hotel Employees Association(PHEA) filed before the CIR
a petition praying, among other things, that its employees who were working at the Pines
Hotel be paid additional compensation for overtime service rendered due to the exigencies
of the business, as well as additional compensation for Sunday, legal holiday and
nighttime work.
- The Manila Hotel filed its answer denying the material averments of the petition and
alleging, among others, that if overtime service was rendered the same was not
authorized but was rendered voluntarily, for the employees were interested in the "tips"
offered by the patrons of the hotel.
- CIR- judged that the employees were entitled to the additional compensation demanded,
including that for overtime work, because an employee who renders overtime service is
entitled to compensation even if he rendered it without prior authority.
- MR was filed, but the same was denied by the industrial court en banc.1awphl.nt
- The Examining Division (CIR) submitted a report- stated that the amount due the
employees as additional compensation for overtime and night services rendered from
January to December 31, 1958 was P32,950.69.
- The management filed its objection to the report on the ground that it included 22 names
of employees who were not employees of the Pines Hotel at the time the petition was filed
so that insofar as said employees are concerned the petition merely involves a money
claim which comes under the jurisdiction of the regular courts.
- The trial judge, however, overruled this objection holding that, while the 22 employees
were actually not in the service at the time of the filing of the petition, they were however
subsequently employed even during the pendency of the incident, and so their claim
comes within the jurisdiction of the Court of Industrial Relations.
ISSUE
WON the 22 seasonal employees were correctly adjudged the additional compensation
including that for OT
HELD
YES
Ratio Seasonal employees called to work from time to time and temporarily laid off from
during off season are REGULARS but are on LOA w/o pay.
Reasoning
- It appears that the questioned employees were never separated from the service.
- Their status is that of regular seasonal employees who are called to work from time to
time, mostly during summer season.
- The nature of their relationship with the hotel is such that during off season they are
temporarily laid off but during summer season they are re-employed, or when their
services may be needed.
- They are not strictly speaking separated from the service but are merely considered as
on leave of absence without pay until they are re-employed. Their employment
relationship is never severed but only suspended. As such, these employees can be
considered as in the regular employment of the hotel.
Disposition The order appealed from is affirmed.

INDUSTRIAL-COMMERCIAL-AGRICULTURAL
WORKERS' ORGANIZATION V CIR
16 SCRA 562
REYES; March 31, 1966
NATURE
Appeal from decision of the Court of Industrial Relations
FACTS
- Petitioner, Industrial-Commercial- Agricultural Workers' Organization (ICAWO), declared
a strike against the respondent Central Azucarera de Pilar. The strike was amicably settled
the following day, and among the provisions of the "Amicable Settlement" reads:
"That the company shall not discriminate against any worker and the same treatment
shall be accorded to workers (ICAWO affiliates) who declared a strike or not. A petition
for Certification Election will be filed by the ICAWO in view of the other labor union,
CAPAWA, with whom the company has an existing collective bargaining contract, a
union which is considered by the ICAWO as a company union."
- The CAPAWA therein referred to is the herein respondent Central Azucarera de Pilar
Allied Workers Association and the collective bargaining contract, likewise therein referred
to, entered into in 1955, provided:
"The EMPLOYER agrees that in hiring unskilled employees and laborers, the members
of the WORKERS ASSOCIATION should be given preference and the management

Labor Law 1
should notify accordingly to the WORKERS ASSOCIATION of any vacancy existing in
all Departments. New employees and laborers hired who are members of the
WORKERS ASSOCIATION will be on TEMPORARY STATUS and the EMPLOYER
agrees that before they will be considered regular employees and laborers they have to
become members of the CENTRAL AZUCARERA DE PILAR ALLIED WORKERS'
ASSOCIATION within thirty (30) days from the date of employment and if they refuse to
affiliate with the said labor organization within this time they will be immediately
dismissed by the EMPLOYER;"
- Among the strikers were 101 seasonal workers, some of whom have worked as such for
the company since pre-war years. On the opening of the milling season for the year 19561957, the respondent company refused to re-admit those 101 seasonal workers of the
ICAWO on the ground that it was precluded by the closed shop clause in its collective
bargaining agreement with the CAPAWA. Thus, , the ICAWO filed an unfair labor practice
charge against the company. The Court of Industrial Relations in a decision ordered the
reinstatement, with back wages, of these laborers; but on a motion for reconsideration, the
said court, en banc, reversed the said decision.
Not satisfied with the reversal, the ICAWO filed the present petition for certiorari to review
the industrial court's resolution.
- The petitioner contends that they are regular and old employees and, as such, they
should have been re-hired at the start, in the month of October, of each milling season,
which usually last 5 months. The respondents, on the other hand, urge that these laborers
are new, their employment terminating at the end of each milling season and, therefore,
could not be re-admitted without the company violating the closed shop agreement with
the CAPAWA.
ISSUE
WON seasonal workers are new workers
HELD
NO
- Petitioners, even if seasonal workers, were not new workers within the scope of the
closed shop contract between the sugar central and the CAPAWA union; hence their
discharge was illegal.
- The cessation of the Central's milling activities at the end of the milling season is not
permanent or definitive; it is a foreseeable suspension of work, and both activities will be
resumed, as they are in fact resumed, when sugar cane ripe for milling is again available.
There is merely a temporary cessation of the manufacturing process due to passing
shortage of raw materials that by itself alone is not sufficient, in the absence of other
justified reasons, to sever the employment or labor relationship between the parties. The
mere fact that the laborers assent to their medical examination at the beginning of each
milling season does not indicate that a new labor contract is being entered into, in the
absence of a stipulation to such effect. Said examination is in the interest of both the
Central and the labor force.
Disposition Resolution set aside

HACIENDA BINO V CUENCA


456 SCRA 300
CALLEJO SR; April 15, 2005
FACTS
- Hacienda Bino (HB) is a 236-hectare sugar plantation in Negros Occ, owned and
operated by Hortencia Starke. HB consists of 220 workers performing various works, such
as cultivation, planting of cane points, fertilization, watering, weeding, harvesting, and
loading of harvested sugarcanes to cargo trucks. On July 18 1996, Starke issued a notice
that those who signed in favor of CARP are expressing their desire to get out of
employment. 76 workers were affected: they regarded the notice as a notice of their
termination. They filed a complaint for illegal dismissal, wage differentials, 13th month pay,
holiday pay and premium pay for holiday, service incentive leave pay, and moral and
exemplary damages.
- Starkes side: HBs board of directors were petitioning for reclassification of the hacienda
(except the portion earmarked for CARP) from agricultural to industrial, residential and
commercial. She was merely giving priority to those who supported the reclassification.
July 1996 was off-season, so not so many workers needed; the work was seasonal in
nature. She relies on the ruling in Mercado Sr. v. NLRC where sugar farm workers were
classified as seasonal employees, and not regular employees. The workers were free to
offer their services to neighboring haciendas.
- Respondents side: They are regular employees. The hacienda was so big that they work
there year-round and that they do not offer their services to neighboring haciendas. The
Mercado case dealt with a 17.5-heactare hacienda, considerable smaller than HB.
ISSUE
WON the respondents are seasonal employees

A2010

- 97 -

Disini

HELD
NO
- The respondents are regular employees.
- Starkes reliance on Mercado is misplaced because in that case, the workers were
classified as seasonal employees because they were employed for a definite period of
time since the hacienda was much smaller, and they offered their services to and worked
at the neighboring haciendas.
- The Court reiterated the same observations in Hacienda Fatima v. National Federation of
Sugarcane Workers-Food and General Trade and added that the petitioners in the
Mercado case were "not hired regularly and repeatedly for the same phase/s of
agricultural work, but on and off for any single phase thereof." Starke did not present any
evidence that the respondents were required to perform certain phases of agricultural
work for a definite period of time. Although she asserted that the respondents made their
services available to the neighboring haciendas, the records do not, however, support
such assertion.
- The primary standard for determining regular employment is the reasonable connection
between the particular activity performed by the employee in relation to the usual trade or
business of the employer. There is no doubt that the respondents were performing work
necessary and desirable in the usual trade or business of an employer. Hence, they can
properly be classified as regular employees.
- For respondents to be excluded from those classified as regular employees, it is not
enough that they perform work or services that are seasonal in nature. They must have
been employed only for the duration of one season. While the records sufficiently show
that the respondents' work in the hacienda was seasonal in nature, there was, however,
no proof that they were hired for the duration of one season only. In fact, the payrolls,
submitted in evidence by the petitioners, show that they availed the services of the
respondents since 1991. Absent any proof to the contrary, the general rule of regular
employment should, therefore, stand. It bears stressing that the employer has the burden
of proving the lawfulness of his employee's dismissal.
Disposition Petition denied. Reinstate workers without loss of seniority, pay backwages
and wage differentials, and pay attys fees.

POSEIDON FISHING V NLRC (ESTOQUIA)


482 SCRA 717
CHICO-NAZARIO; February 20, 2006
FACTS
- Estoquia was first hired by PF as a Chief Mate in 1988. He later on became Boat
Captain; but was demoted still later to Radio Operator. In 2000, he failed to log a 7:25am
call in one logbook, but was able to record it in another one. When he realized his mistake,
he logged the 7:25 call after a 7:30 am call (same day). The manager noticed this mistake
and summoned Estoquia to get his separation pay. Estoquia refused to accept the sep pay
because he believed he had done nothing illegal to warrant his immediate discharge from
work.
- PF argues that Estoquia was a casual/contractual employee whose services could be
terminated at the end of contract. PF contends that Estoquia was hired on a por viaje
basis, based on the Kasunduan which reads:
NA, kami ay sumasang-ayon na MAGLINGKOD at GUMAWA ng mga gawaing
magmula sa pag-alis ng lantsa sa pondohan sa Navotas patungo sa palakayahan;
pabalik sa pondohan ng lantsa sa Navotas hanggang sa paghango ng mga kargang
isda.
- LA and NLRC found for Estoquia.
ISSUE
WON Estoquia was a regular employee when he was discharged from work
HELD
YES
- PF's intent to evade the application of Article 280 of the Labor Code is unmistakable. In a
span of 12 years, Estoquia worked for petitioner company first as a Chief Mate, then Boat
Captain, and later as Radio Operator. His job was directly related to the deep-sea fishing
business of petitioner Poseidon. His work was, therefore, necessary and important to the
business of his employer. Such being the scenario involved, private respondent is
considered a regular employee of petitioner under Article 280 of the Labor Code.
- Moreover, unlike in the Brent case where the period of the contract was fixed and clearly
stated, the terms of employment of private respondent as provided in the Kasunduan was
not only vague, it also failed to provide an actual or specific date or period for the
contract.
- Furthermore, as petitioners themselves admitted, private respondent was repeatedly
hired as part of the boat's crew and he acted in various capacities onboard the vessel. In
Integrated Contractor and Plumbing Works, Inc. v. NLRC, it was held that the test to
determine whether employment is regular or not is the reasonable connection between the
particular activity performed by the employee in relation to the usual business or trade of
the employer. And, if the employee has been performing the job for at least one year, even

Labor Law 1
if the performance is not continuous or merely intermittent, the law deems the repeated
and continuing need for its performance as sufficient evidence of the necessity, if not
indispensability of that activity to the business.
- The act of hiring and re-hiring in various capacities is a mere gambit employed by
petitioner to thwart the tenurial protection of private respondent. Such pattern of re-hiring
and the recurring need for his services are testament to the necessity and indispensability
of such services to petitioners' business or trade.
ESTOQUIA IS NOT PROJECT/SEASONAL EMPLOYEE
- The activity of catching fish is a continuous process and could hardly be considered as
seasonal in nature. In Philex Mining Corp. v. NLRC, project employees were defined as
those workers hired (1) for a specific project or undertaking, and (2) the completion or
termination of such project has been determined at the time of the engagement of the
employee. The principal test for determining whether particular employees are "project
employees" as distinguished from "regular employees," is whether or not the "project
employees" were assigned to carry out a "specific project or undertaking," the duration
and scope of which were specified at the time the employees were engaged for that
project. In this case, petitioners have not shown that private respondent was informed that
he will be assigned to a "specific project or undertaking." Neither has it been established
that he was informed of the duration and scope of such project or undertaking at the time
of their engagement.
- More to the point, in Maraguinot, Jr. v. NLRC, the SC ruled that once a project or work
pool employee has been:
(1) continuously, as opposed to intermittently, re-hired by the same employer for the
same tasks or nature of tasks; and
(2) these tasks are vital, necessary and indispensable to the usual business or trade of
the employer, then the employee must be deemed a regular employee.
- In fine, inasmuch as private respondent's functions as described above are no doubt
"usually necessary or desirable in the usual business or trade" of petitioner fishing
company and he was hired continuously for 12 years for the same nature of tasks, we are
constrained to say that he belongs to the ilk of regular employee. Being one, private
respondent's dismissal without valid cause was illegal. And, where illegal dismissal is
proven, the worker is entitled to back wages and other similar benefits without deductions
or conditions.
Disposition Petition denied.

SECTION 8: PROBATIONARY EMPLOYEES


8.01 PROBATIONARY EMPLOYEES
DEFINITION
INTERNATIONAL CATHOLIC MIGRATION COMMISSION
V NLRC (GALANG)
169 SCRA 606
FERNAN; January 30, 1989
NATURE
Petition to review the decision of the NLRC
FACTS
- Petitioner International Catholic Migration Commission (ICMC), a non-profit organization
dedicated to refugee service at the Philippine Refugee Processing Center in Morong,
Bataan engaged the services of private respondent Bernadette Galang as a probationary
cultural orientation teacher with a monthly salary of P2,000.00.
- Three (3) months thereafter, private respondent was informed, orally and in writing, that
her services were being terminated for her failure to meet the prescribed standards of
petitioner as reflected in the performance evaluation of her supervisors
- Private respondent filed a complaint for illegal dismissal, unfair labor practice and unpaid
wages against petitioner with the then Ministry of Labor and Employment, praying for
reinstatement with backwages, exemplary and moral damages.
- Labor Arbiter Pelagio A. Carpio rendered his decision dismissing the complaint for illegal
dismissal as well as the complaint for moral and exemplary damages but ordering the
petitioner to pay private respondent the sum of P6,000.00 as payment for the last three (3)
months of the agreed employment period pursuant to her verbal contract of employment.
- Both parties appealed the decision to the National Labor Relations Commission.
- The NLRC, by a majority vote, sustained the decision of the Labor Arbiter and thus
dismissed both appeals for lack of merit.
- Dissatisfied, petitioner filed the instant petition.
ISSUE

A2010

- 98 -

Disini

WON an employee who was terminated during the probationary period of her employment
is entitled to her salary for the unexpired portion of her six-month probationary
employment
HELD
NO
- There is justifiable basis for the reversal of public respondent's award of salary for the
unexpired three-month portion of private respondent's six-month probationary employment
in the light of its express finding that there was no illegal dismissal
- There is no dispute that private respondent was terminated during her probationary
period of employment for failure to qualify as a regular member of petitioner's teaching
staff in accordance with its reasonable standards: private respondent was found by
petitioner to be deficient in classroom management, teacher-student relationship and
teaching techniques.
- Failure to qualify as a regular employee in accordance with the reasonable standards of
the employer is a just cause for terminating a probationary employee specifically
recognized under Article 282 (now Article 28122) of the Labor Code.
- It must be noted that notwithstanding the finding of legality of the termination of private
respondent, public respondent justified the award of salary for the unexpired portion of the
probationary employment on the ground that a probationary employment for six (6)
months is an employment for a "definite period" which requires the employer to exhaust
the entire probationary period to give the employee the opportunity to meet the required
standards.
- The legal basis of public respondent is erroneous. A probationary employee, as
understood under Article 282 (now Article 281) of the Labor Code, is one who is on trial by
an employer during which the employer determines whether or not he is qualified for
permanent employment. A probationary appointment is made to afford the employer an
opportunity to observe the fitness of a probationer while at work, and to ascertain whether
he will become a proper and efficient employee.
- The word "probationary", as used to describe the period of employment, implies the
purpose of the term or period, but not its length.
- Being in the nature of a "trial period" the essence of a probationary period of employment
fundamentally lies in the purpose or objective sought to be attained by both the employer
and the employee during said period. The length of time is immaterial in determining the
correlative rights of both in dealing with each other during said period.
- A281 LC gives ample authority to the employer to terminate a probationary employee for
a just cause or when he fails to qualify as a regular employee in accordance with
reasonable standards made known by the employer to the employee at the time of his
engagement.
- There is nothing under Article 281 of the Labor Code that would preclude the employer
from extending a regular or a permanent appointment to an employee once the employer
finds that the employee is qualified for regular employment even before the expiration of
the probationary period. Conversely, Article 281 of the Labor Code does not likewise
preclude the employer from terminating the probationary employment on justifiable causes
as in the instant case.
- There was no showing, as borne out by the records, that there was circumvention of the
rights of private respondent when she was informed of her termination. Private respondent
was duly notified, orally and in writing, that her services as cultural orientation teacher
were terminated for failure to meet the prescribed standards of petitioner.
- The dissatisfaction of petitioner over the performance of private respondent in this regard
is a legitimate exercise of its prerogative to select whom to hire or refuse employment for
the success of its program or undertaking.
- It was a grave abuse of discretion on the part of public respondent to order petitioner to
pay private respondent her salary for the unexpired three-month portion of her six-month
probationary employment when she was validly terminated during her probationary
employment. To sanction such action would not only be unjust, but oppressive on the part
of the employer
Disposition Petition granted.

PHIL. FEDERATION OF CREDIT COOPERATIVES INC


(PFCCI) V NLRC (ABRIL)
[PAGE 73]

PURPOSE
PHILEMPLOY SERVICES V RODRIGUEZ
22

ART. 281. Probationary employment. Probationary employment shall not exceed six months from the date the
employee started working, unless it is covered by an apprenticeship agreement stipulating a longer period. The services
of an employer who has been engaged in a probationary basis may be terminated for a just cause or when he fails to
qualify as a regular employer in accordance with reasonable standard made known by the employer to the employer at
the time of his engagement. An employee who is allowed to work after a probationary period shall be considered a
regular employee.

Labor Law 1
486 SCRA 302
CARPIO; March 31, 2006
NATURE
Petition for review to annul CA decision
FACTS
- Anita RODRIGUEZ applied with respondent PHILEMPLOY Services and Resources, Inc.
for deployment abroad as a factory worker. When she was asked to report for work, Ms.
Brenda Castro, an official of respondent, demanded from her the sum of P60T as
placement fee.
- Since she could not afford such amount, they agreed that she would initially pay P30T as
downpayment and the balance of P30T, plus 7% interest every month thereafter through
salary deductions. She then paid Ms. Castro the P30T but was not issued any receipt.
- Thereafter, she executed a contract of employment as a domestic helper of one Chao
Hung Ching of Taipei, Taiwan with a monthly salary of NT$14,010, plus free food and
accommodation for a period of 1 year.
- On 13 Jan 1995, she was deployed to Taiwan. As such DH, she worked from 5am until
10pm. Among her chores were to carwash the vehicle of her master, cook the meals,
housecleaning and babysitting. On 24 Jan 1995, she had a talk with her master where she
was told that she is being sent home due to certain problems.
- Complainant pleaded that she continue her employment, confronted as she was with the
debts she had to pay. But she was sent home the following day. While at the airport, a
certain Ms. Go forced her to sign an Affidavit where it stated that her leaving as a DH was
voluntary and that she would assume all the obligations for her travel back to the
Philippines.
- She was only paid the sum of NT$1,931 (12 days work)
- Respondent alleged that it was stipulated and agreed upon in the contract, that she
would undergo a 40-day probationary period before she becomes a regular domestic
helper. Also, she was charged of her placement fees as allowed by law and by the POEA
rules and regulations. During the first 10 days of her probationary period, she was
observed to be inattentive and incompetent to perform her duties and responsibilities.
- Labor Arbiter ruled in favor of Rodriguez.
- NLRC deleted award of P155T for unearned wages since there is no illegal dismissal
that took place.
- CA reversed NLRC decision and reinstated Labor Arbiters.
ISSUES
1. WON Rodriguez was illegally dismissed
2. WON she was afforded due process
HELD
1. NO
Ratio Even if it were true that Anitas foreign employer terminated her services after 10
days of her employment, there could be no illegal dismissal as the termination was
effected during the agreed probationary period.
- There is probationary employment23 where the employee, upon his engagement, is made
to undergo a trial period during which the employer determines his fitness to qualify for
regular employment, based on reasonable standards made known to him at the time of
engagement.
2. NO
Ratio Although Anitas employment was terminated because she failed to meet the
standards of her foreign employer, still it is necessary and obligatory to afford Anita her
basic right to notice.
- Section 2, Rule 1, Book VI of the Omnibus Rules Implementing the Labor Code provides:
(d) In all cases of termination of employment, the following standards of due process
shall be substantially observed:
If the termination is brought about by the completion of a contract or phase thereof, or
by failure of an employee to meet the standards of the employer in the case of
probationary employment, it shall be sufficient that a written notice is served the employee
within a reasonable time from the effective date of termination .
Reasoning
- Anita was repatriated to the Philippines on 25 January 1995. On the night before her
departure, her employers wife merely told her that she was sending her home on account
of some problem. This information given to Anita cannot be considered as equivalent to
the written notice required by law to be served on the employee. The notice should inform
the employee of the ground or grounds for his termination and that his dismissal is being
sought.
* Thus, absence of notice makes her termination DEFECTIVE for which petitioner must be
sanctioned for its non-compliance with the requirements of or for failure to observe due
process.

23

Section 6, Rule 1, Book VI of the Omnibus Rules Implementing the Labor Code

A2010

- 99 -

Disini

- As held in the Agabon case: Where the dismissal is for a just cause, as in the instant
case, the lack of statutory due process should not nullify the dismissal, or render it illegal,
or ineffectual. However, the employer should indemnify the employee for the violation of
his statutory rights.
Disposition Petition GRANTED. CA decision set aside. However, petitioner Philemploy
Services and Resources, Inc. shall pay respondent Rodriguez P30T as nominal damages.

DELA CRUZ V NLRC


418 SCRA 226
CORONA; December 11, 2003
NATURE
Petition for review on certiorari
FACTS
- On May 27, 1996, petitioner Florencio de la Cruz, Jr. was hired by private respondent
Shemberg Marketing Corporation as senior sales manager, a newly created position in
line with the companys objective of product positioning in the consumer market. However,
on Sept. 14, 1996, petitioner was informed that his services were terminated. His request
for a meeting with Shembergs VP and to be furnished a 30-day written notice was denied
by management. Hence, petitioner filed a complaint for illegal dismissal., non-payment of
salary, backwages, 13th month pay and damages.
- Private respondent answered that petitioners dismissal was premised, among others,
on his unauthorized reimbursement of the plane tickets of his wife and child, resulting to
loss of trust and confidence of the company.
- Labor arbiter ruled that petitioner was illegally dismissed and granted his claim for
separation pay, backwages and unpaid wages. Upon appeal, NLRC modified the decision,
deleting the award for separation pay and backwages. Hence, this petition.
ISSUE
WON petitioner was legally dismissed, as he was a probationary employee
HELD
YES
- Petitioner was hired by Shemberg on May 27, 1996 and was terminated on Sept. 14,
1996. A281 LC provides:
Probationary employment shall not exceed six (6) months from the date the employee
started working , unless it is covered in apprenticeship[ agreement stipulating a longer
period. The services of an employee who has been engaged on a probationary basis
may be terminated for a just cause or when he fails to qualify as a regular employee in
accordance with reasonable standards, made known by the employer to the employee
at the time of his engagement. An employee who is allowed to work after a
probationary period shall be considered a regular employee.
- The evidence on record clearly shows that petitioner was well informed of the standards
to be met before he could qualify as a regular employee. Attached to his appointment
papers was a job description of sales manager.
- A probationary employee is one who, for a given period of time, is under observation or
evaluation to determine whether or not he is qualified for permanent employment. During
the probationary period, the employer is given the opportunity to observe the skill,
competence and attitude of the employee while the latter seeks to prove to the employer
that he has the qualifications to meet the reasonable standards for permanent
employment. The length of time is immaterial in determining the correlative rights of both
the employer and the employee in dealing with each other during this period.
- There is no dispute that petitioner, as a probationary employee enjoyed only a
temporary employment status. This meant that he was terminable anytime, permanent
employment not having been attained in the mean time. The employer could well decide
he no longer needed the probationary employees service or hi performance fell short of
expectation. As long as the termination was made before the expiration of the 6-month
probationary period, the employer was well within his rights to sever the employeremployee relationship. A contrary interpretation would defect the clear meaning of the
term probationary. In this case, Shemberg had good reason to terminate petitioners
employment. Petitioner was holding a managerial position in which he was tasked to
perform key functions in accordance with an exacting work ethic. His position required the
full trust and confidence of his employer. While petitioner could exercise some discretion,
this obviously did not cover acts for his own personal benefit. He committed a
transgression which betrayed the trust and confidence of his employer reimbursing his
familys personal travel expenses out of company funds.
Disposition Petition is DISMISSED. The decision of the CA is affirmed.

GRAND MOTORS PARTS CORP V MINISTER OF


LABOR
GUERRERO; July 16, 1984

Labor Law 1
NATURE
Petition for certiorari assailing the Order of the Minister of Labor which affirmed the
decision of the Regional Director of Region VI
FACTS
- Respondent Balicena was the Branch Manager of the petitioner companys Iloilo Branch.
He was the Finance Officer of Warner, Barnes, & Co. (no.2 of the company) when
allegedly, Mr. Alfredo Cisneros (the then acting branch manager of the company in Iloilo)
induced him to apply for the position of Branch Manager, as their company (petitioner) was
looking for a CPA. He applied for the job and was accepted. He started working for the
petitioner company on April 1 but resigned from his position in Warner, Barnes, & Co. only
on April 28.
- However, he was terminated only after working for the company for 4 months (April to
August). Petitioner company alleged that (a) he failed to submit promptly the monthly
Income and Loss Statement, Comparative Projections & Actual Sales Report; (b) the
Comparative Performance Report dated July 8, 1980 on the operation of the Iloilo Branch
for the month of June and May, 1980, the Cash Sales of the Iloilo Branch went down to
P91,318.41 for June, 1980, as compared with the sales for the month of May, 1980 in the
sum of P174,697.77; (c) Belicena in violation of company policy and without clearance
from the head office in Cebu, extended personal accounts in favor of 15 persons which as
of November, 1980 produced delinquent accounts amounting to P18,435.80; and (d)
Belicena claimed lack of knowledge of the vehicular accident caused by a subordinate and
failed to provide prompt administrative disciplinary action against the erring employee.
They claimed that Balicena is only a probationary employee, which would be observed by
the company for 4-6 months and that Balicena knew that there is a possibility that he
would not get the job.
- Balicena on the other hand alleged that he is a regular employee, although he was not
able to present any contract establishing his status as a regular employee; that the
mishap involving the company's vehicle which was used without his permission and
knowledge could not be blamed upon him; that the alleged reports which he failed to send
were not reminded to him, verbally or in writing; that his sales for the period April to
August, 1980 is higher compared to that for the same period in 1979; and that the alleged
accounts remaining unpaid as of November 6, 1980 would have been collected in full if he
were still the Manager, among other things.
- Regional Director and Minister of Labor ruled in favor of Balicena, ordering his
reinstatement, payment of his backwages, and other privileges.
ISSUES
1. WON private respondent's employment as Branch Manager was probationary, and not
regular and permanent
2. WON private respondent, if hes a probationary Branch Manager, was terminated for
just cause
HELD
1. YES
Ratio Indeed, the employer has the right or is at liberty to choose as to who will be hired
and who will be declined. It is within the exercise of this right to select his employees that
the employer may set or fix a probationary period within which the latter may test and
observe the conduct of the former before hiring him permanently. "The right of a laborer to
sell his labor to such persons as he may choose is, in its essence, the same as the right of
an employer to purchase labor from any person whom it chooses. The employer and the
employee have thus an equality of right guaranteed by the Constitution. 'If the employer
can compel the employee to work against the latter's will, this is servitude. If the employee
can compel the employer to give him work against the employer's will, this is oppression.'"
Reasoning
- First, Balicena could not present any written proof of his appointment or employment as
regular and permanent Branch Manager of petitioner corporation. Then there was the fact
that he assumed his work as of April 1 but resigned from his previous company only on
April 28, meaning that if he was really appointed as regular and permanent then he would
have resigned immediately from his old company. But since he was not yet sure of his
status in the petitioner corporation, he resigned late.
-Second, Balicena claims that there was no written contract because the contracts were
given only to those who will pass the probationary period and the rank-and-file employees
not to those managerial ones. This practice is not supported and contrary to usual
business practice. Also, being a CPA, Finance Officer, and No.2 man in his former
company, he should have insisted on a written contract for the security of his tenure in his
new position! (in short, he already had a stable position in his former company and in not
insisting on a written contract, he took the risk of being joblessfor he may be terminated
as Branch Manager!)
-Third, Balicena had never been hired as manager, and the petitioner company and
Balicenas former company are engaged in different kinds of business so it was necessary
for Balicena to undergo a period of probation to test his qualifications, skills and
experience since managing is a new experience for him.

A2010

- 100 -

Disini

2. YES
Ratio A probationary employee may be terminated after six months for a just cause or
when he fails to qualify as a regular employee. It is true that mere allegation of loss of
confidence by employer on his employee is not sufficient cause for his dismissal. But loss
of confidence is a valid ground for dismissing an employee, and proof beyond reasonable
doubt of the employee's misconduct is not required to dismiss him of this charge. It is
sufficient if there is some basis for such loss of confidence.
Reasoning
- Balicena was proven and demonstrated to fail in submitting monthly reports. A monthly
report of operations is essential to the business of the petitioner company, and this cannot
be ignored by a Branch Manager for the viability of its business life may well depend on
these reports.
- Second, it was proven that after being on the job as Branch Manager for nearly a month,
Balicena wrote to the company to ask what his duties were. This manifests that Balicena
had not shown any effort or initiative to familiarize himself with his duties and obligations,
although petitioner corporation provided him with a brochure containing what he was
supposed to do.
- Third, he cannot escape responsibility for the acts of his subordinates (vehicular accident
which he claims that he did not know) and for extending personal accounts to some
individuals in violation of their companys policy.
- Fourth, the sales target were not being reached, and because of this, the Vice
President/GM notified him about this (Can we not improve that performance this July?).
Balicena was clearly not discharging his duties to the satisfaction of the management.
- SO THEREFORE: [The Court is] satisfied that petitioner has valid grounds to charge its
Branch Manager with loss of confidence by reason of the overall performance he has
demonstrated within the probationary period which showed that he is not qualified to be
the regular or permanent Branch Manager of petitioner corporation in Iloilo City. His
dismissal does not appear to Us as arbitrary, fanciful or whimsical. In the last and ultimate
analysis, the prerogative and judgment to hire employees under terms and
conditions designed to achieve success in its business activities belongs to
management which may not be unduly impaired, limited or restricted.
Disposition the Order of the Deputy Minister of Labor dated October 22, 1981 is hereby
REVERSED and SET ASIDE.

INTERNATIONAL CATHOLIC MIGRATION COMMISSION


V NLRC (GALANG)
[PAGE 98]
PHIL. FEDERATION OF CREDIT COOPERATIVES INC
(PFCCI) V NLRC (ABRIL)
[PAGE 73]

ESCORPIZO V UNIVERSITY OF BAGUIO


306 SCRA 497
QUISUMBING; April 30, 1999
NATURE
Special civil action for certiorari
FACTS
- Petitioner Esperanza Escorpizo was initially hired by respondent university on June 13,
1989 as a high school classroom teacher. Under the rules of the respondent university,
appointment to teach during the first two years at the university is probationary in nature.
During the probation period, the teacher is observed and evaluated to determine his
competency. Attainment of a permanent status by a faculty member is conditioned upon
compliance with certain requirements, such as passing the professional board
examination for teachers (PBET).
- On March 18, 1991, respondent university informed Escorpizo that her employment was
being terminated at the end of the school semester in view of her failure to pass the PBET.
But before the start of the school year 1991-1992, Escorpizo reapplied and pleaded that
she be given another chance. She told the respondent school that she had just taken the
PBET and hoped to pass it.
- As Escorpizos appeal was favorably considered, she was allowed to teach during the
school year 1991-1992. However, her continued employment was conditioned on her
passing the PBET. Unfortunately, Escorpizo failed again. Undaunted, Escorpizo took the
examination a third time. At the end of the school year, respondent university evaluated
the teachers performance to determine who would be in the list for the next school year.
Escorpizo, not having passed the PBET yet, was not included.

Labor Law 1

A2010

- 101 -

Disini

- Much later, on June 8, 1992, the results of the PBET were released and this time
Escorpizo passed said examination. Nevertheless, on June 15, 1992, respondent
university no longer renewed Escorpizos contract of employment on the ground that she
failed to qualify as a regular teacher. This prompted Escorpizo to file on July 16, 1992 a
complaint for illegal dismissal, payment of backwages and reinstatement against private
respondents.

- NLRC affirmed POEA decision on appeal. It also ruled that the designation of Flores as
floorman instead of crane operator for which he was hired violated his employment
contract. Orient Express and Nadrico filed for MFR but it was denied.

ISSUE
WON the dismissal was illegal since Escorpizo had attained the status of a regular
employee having rendered very satisfactory performance as probationary teacher for two
years, consistent with the collective bargaining agreement between the respondent
university and petitioner union of which Escorpizo is a member

1. WON NLRC committed grave abuse of discretion in concluding that Flores was never
assigned as crane operator (not important)
2. WON NLRC and POEA committed grave abuse of discretion for ruling that poor job
performance and uncooperative work attitude did not justify his dismissal

HELD
NO
- Escorpizo was not illegally dismissed. Her contract merely expired.
Ratio A probationary employee is one who, for a given period of time, is being observed
and evaluated to determine whether or not he is qualified for permanent employment. A
probationary appointment affords the employer an opportunity to observe the skill,
competence and attitude of a probationer. The word probationary, as used to describe
the period of employment, implies the purpose of the term or period. While the employer
observes the fitness, propriety and efficiency of a probationer to ascertain whether he is
qualified for permanent employment, the probationer at the same time, seeks to prove to
the employer that he has the qualifications to meet the reasonable standards for
permanent employment.
Reasoning
- the rules of the university clearly states that the first two years at the University is
probationary in nature and the following conditions must concur in order that a
probationary teacher may be extended a regular appointment; (1) the faculty member
must satisfactorily complete the probationary period of four semesters or two years, within
which his performance shall be observed and evaluated for the purpose of determining his
competency and fitness to be extended permanent status; and (2) the faculty member
must pass the PBET or an equivalent civil service examination. Escorpizo failed to meet
the 2nd requirement to be a regular employee which is to pass the PBET.
- Though the CBA does not mention that passing the PBET is a prerequisite for attaining
permanent status as a teacher. Nevertheless, the aforecited CBA provision must be read
in conjunction with statutory and administrative regulations governing faculty qualifications.
It is settled that an existing law enters into and forms part of a valid contract without the
need for the parties expressly making reference to it.[16] Further, while contracting parties
may establish such stipulations, clauses, terms and conditions as they may see fit, such
right to contract is subject to limitation that the agreement must not be contrary to law or
public policy.
- DECS Order No. 38, series of 1990, a regulation implementing Presidential Decree No.
1006[18] or the Decree Professionalizing Teaching stipulates that no person shall be
allowed to engage in teaching and/or act as a teacher unless he has registered as
professional teacher with the National Board for Teachers.
Disposition DISMISSED, and the assailed RESOLUTION of public respondent is hereby
AFFIRMED.

HELD

EMPLOYER RIGHT SET PERIOD/OBLIGATION


GRAND MOTORS PARTS CORP V MINISTER OF
LABOR
[PAGE 100]
ORIENT EXPRESS PLACEMENT PHILIPPINES V NLRC
(POEA, FLORES)
273 SCRA 256
BELLOSILLO; June 11, 1997
FACTS
- Antonio Flores was hired as crane operator with a monthly salary of US$500 for 1year
subject to a 3month probationary period. After 1month and 5days, he was repatriated to
the Philippines. He filed a complaint to POEA fro having been terminated for no valid
reason. His employers Orient Express and Nadrico (the foreign principal) claimed that he
was terminated for poor job performance as shown in his performance evaluation sheet.
- POEA decided in favor of Flores held that when the ground invoked for dismissal of an
employee was incompetence or poor job performance, it must be shown that the
reasonable standards of work prescribed by the employer were made known to the
employee. The dismissal was unwarranted because the employers failed to point out the
reasonable standards of work required.

ISSUES

1. Factual determination. NLRC and POEA did overlook the fact that the private
respondent admitted that he was ale to work as crane operator (as shown in his affidavit)
2. NO
- Flores was not validly dismissed. Petition is denied.
- A281 LC, the services of an employee hired on probationary basis may be terminated
when he fails to qualify as a regular employee in accordance with reasonable standards
made known by the employer to the employee at the time of the engagement. The Court
cannot sustain dismissal on this ground because petitioner failed to specify the reasonable
standards by which Flores was alleged to have been evaluated to have poor performance.
- Neither of the 2 original petitioners provided ever made mention that he must first take
and pass a Crane Operators License Examination before he would be allowed to touch a
crane. Neither did he know that he was to be assigned as floorman pending the release of
the exam results. He also did not know that if he failed the exam, he would be subject to a
performance evaluation 1 month after his hiring to determine whether the company was
amenable to continuing his employment. Flores could not be faulted for harboring the
impression that he was hired as crane operator for a definite period of 1 year to
commence upon his arrival at the work-site and to terminate at the end of 1 year. No other
condition was laid out except that he was to be on probation for 3 months.
- Due process dictates that an employee be apprised beforehand of the condition of his
employment and of the terms of advancement therein. Even if unsatisfactory performance
was true, it is not 1 of the just causes for dismissal under the LC. There was no standard
by which such probationary period was made known to him.
Disposition the assailed Decision and Resolution of NLRC, declaring that private
respondent Antonio F. Flores was illegally dismissed is AFFIRMED.

MITSUBISHI MOTORS CORP V CHRYSLER


PHIL LABOR UNION
4333 SCRA 206
CALLEJO; June 29, 2004
NATURE
Petition for review on certiorari
FACTS
- Mitsubishi Motors Philippines Corporation (MMPC) is a domestic corporation engaged in
the assembly and distribution of Mitsubishi motor vehicles. Chrysler Philippines Labor
Union (CPLU) is a legitimate labor organization and the duly certified bargaining agent of
the hourly-paid regular rank and file employees of MMPC. Nelson Paras was a member
of CPLU while wife, Cecille Paras, was the President of the Chrysler Philippines Salaried
Employees Union (CPSU).
- Nelson Paras was first employed by MMPC as a shuttle bus driver. He resigned and
went to Saudi Arabia work. When he returned to the Philippines, he was re-hired as a
welder-fabricator at the MMPC. Sometime in May of 1996, Paras was re-hired on a
probationary basis as a manufacturing trainee at the Plant Engineering Maintenance
Department. As part of the MMPCs policy, Paras was evaluated by his immediate
supervisors after 6 months, and received an average rating. They informed him that based
on his performance rating, he would be regularized. However, the Department and
Division Managers reviewed the performance evaluation made on Parasand unanimously
agreed, along with Paras immediate supervisors, that the performance of Paras was
unsatisfactory. As a consequence, Paras was not considered for regularization. On
November 26, 1996, he received a Notice of Termination dated November 25, 1996,
informing him that his services were terminated effective the said date since he failed to
meet the required company standards for regularization.

Labor Law 1
- Utilizing the grievance machinery in the collective bargaining agreement, the CPLU
demanded the settlement of the dispute which arose from Paras termination. They argued
rd
that Paras was dismissed on his 183 day of employment, or 3 days after the expiration
of the probationary period of 6 months. It was contended that Paras was already a
regular employee on the date of the termination of his probationary employment.
According to CPLU and Paras, the latters dismissal was an offshoot of the heated
argument during the CBA negotiations between MMPC Labor Relations Manager, Atty.
Carlos S. Cao, on the one hand, and Cecille Paras, the President of the Chrysler
Philippines Salaried Employees Union (CPSU) and Paras wife, on the other.
- PETITIONER'S CLAIM:
> Paras and CPLU asserted that pursuant to Article 13 of the New Civil Code, the period
of May 27, 1996 to November 26, 1996 consisted of one hundred eighty-three (183) days.
They asserted that the maximum of the probationary period is six (6) months, which is
equivalent to 180 days; as such, Paras, who continued to be employed even after the
th
180 day, had become a regular employee as provided for in Article 282 of the Labor
Code. They averred that as a regular employee, Paras employment could be terminated
only for just or authorized causes as provided for under the Labor Code, and after due
notice. They posited that in the Letter of Termination dated November 25, 1996, the
ground for Paras termination was not among those sanctioned by the Labor Code;
hence, his dismissal was illegal. Paras and CPLU also stressed that he had already been
in the employ of MMPC from October 3, 1994 to March 3, 1995 as a welder-fabricator.
Such period, in addition to the six-month probationary period, amounted to eleven (11)
months of service, which is sufficient for him to be considered as a regular employee.
- RESPONDENT'S CLAIM:
> The MMPC, for its part, averred that under Article 13 of the New Civil Code, Paras
probationary employment which commenced on May 27, 1996 would expire on November
27, 1996. Since he received the notice of termination of his employment on November 25,
1996, the same should be considered to have been served within the six-month
probationary period.
- CA agreed with the petitioner's claim and held the dismissal of Paras illegal.
ISSUE
WON Paras was illegally dismissed, he being dismissed after the maximum 6 month
probation (MAY 27, 1996 - NOVEMBER 23, 1996)
HELD
YES
Ratio Indeed, an employer, in the exercise of its management prerogative, may hire an
employee on a probationary basis in order to determine his fitness to perform work. Under
Article 281 of the Labor Code, the employer must inform the employee of the standards for
which his employment may be considered for regularization. Such probationary period,
unless covered by an apprenticeship agreement, shall not exceed six (6) months from the
date the employee started working. The employees services may be terminated for just
cause or for his failure to qualify as a regular employee based on reasonable standards
made known to him. Respondent Paras was employed as a management trainee on a
probationary basis. During the orientation conducted on May 15, 1996, he was apprised of
the standards upon which his regularization would be based. He reported for work on May
27, 1996. As per the companys policy, the probationary period was from 3 months to a
maximum of 6 months. As clearly provided for in the last paragraph of Article 13, in
computing a period, the first day shall be excluded and the last day included. Thus, the
one hundred eighty (180) days commenced on May 27, 1996, and ended on November
23, 1996. The termination letter dated November 25, 1996 was served on respondent
Paras only at 3:00 a.m. of November 26, 1996. He was, by then, already a regular
employee of the petitioner under A281 LC
Reasoning
- The basis for which respondent Paras services were terminated was his alleged
unsatisfactory rating arising from poor performance. It is a settled doctrine that the
employer has the burden of proving the lawfulness of his employees dismissal. The
validity of the charge must be clearly established in a manner consistent with due
process.Under Article 282 of the Labor Code, an unsatisfactory rating can be a just cause
for dismissal only if it amounts to gross and habitual neglect of duties. A careful perusal of
the records of this case does not show that respondent Paras was grossly negligent in the
performance of his duties. Considering that respondent Paras was not dismissed for a just
or authorized cause, his dismissal from employment was illegal. Furthermore, the
petitioners failure to inform him of any charges against him deprived him of due process.
Clearly, the termination of his employment based on his alleged unsatisfactory
performance rating was effected merely to cover up and deodorize the illegality of his
dismissal.
Disposition Petition is PARTIALLY GRANTED. The September 13, 2000 Decision of
the Court of Appeals in CAGR SP No. 46030 is hereby AFFIRMED WITH
MODIFICATIONS. The petitioner is ORDERED to pay respondent Nelson Paras
separation pay equivalent to one (1) month, or to at least one-half (1/2) month pay for
every year of service, whichever is higher, a fraction of at least six (6) months to be
considered as one year; and to pay full backwages, computed from the time of his

A2010

- 102 -

Disini

dismissal up to March 25, 1998. That portion of the decision of the Court of Appeals
directing the reinstatement of the respondent Paras is DELETED.

DURATION/EXCEPTION
BUISER V LEOGARDO
131 SCRA 151
GUERRERO; July 31, 1984
FACTS
- Petitioners were employed by the private respondent GENERAL TELEPHONE
DIRECTORY COMPANY as sales representatives and charged with the duty of soliciting
advertisements for inclusion in a telephone directory.
- The records show that petitioners Iluminada Ver Buiser and Ma. Mercedes P. Intengan
entered into an "Employment Contract (on Probationary Status)" on May 26, 1980 with
private respondent, a corporation engaged in the business of publication and circulation of
the directory of the Philippine Long Distance Telephone Company. Petitioner Ma. Cecilia
Rillo-Acuna entered into the same employment contract on June 11, 1980 with the private
respondent.
Among others, the "Employment Contract (On Probationary Status)" included the following
common provisions:
The company hereby employs the employee as telephone representative on a
probationary status for a period of eighteen (18) months, i.e. from May 1980 to October
1981, inclusive. It is understood that daring the probationary period of employment, the
Employee may be terminated at the pleasure of the company without the necessity of
giving notice of termination or the payment of termination pay.
The Employee recognizes the fact that the nature of the telephone sales
representative's job is such that the company would be able to determine his true
character, conduct and selling capabilities only after the publication of the directory, and
that it takes about eighteen (18) months before his worth as a telephone saw
representative can be fully evaluated inasmuch as the advertisement solicited by him
for a particular year are published in the directory only the following year.
- Private respondent prescribed sales quotas to be accomplished or met by the petitioners.
Failing to meet their respective sales quotas, the petitioners were dismissed from the
service by the private respondent.

ISSUE
1. WON the probationary employment of petitioners herein is eighteen (18) months
instead of the mandated six (6) months under the Labor Code, and WON petitioners are
entitled to security of tenure while under said probation for 18 months
2. WON petitioners were dismissed for a just and valid cause
HELD
1. Generally, the probationary period of employment is limited to six (6) months. The
exception to this general rule is when the parties to an employment contract may agree
otherwise, such as when the same is established by company policy or when the same is
required by the nature of work to be performed by the employee.
- In the latter case, there is recognition of the exercise of managerial prerogatives in
requiring a longer period of probationary employment, such as in the present case where
the probationary period was set for eighteen (18) months, i.e. from May, 1980 to October,
1981 inclusive, especially where the employee must learn a particular kind of work such
as selling, or when the job requires certain qualifications, skills, experience or training.
- In the case at bar, it is shown that private respondent Company needs at least eighteen
(18) months to determine the character and selling capabilities of the petitioners as sales
representatives. The Company is engaged in advertisement and publication in the Yellow
Pages of the PLDT Telephone Directories. Publication of solicited ads are only made a
year after the sale has been made and only then win the company be able to evaluate the
efficiency, conduct, and selling ability of its sales representatives, the evaluation being
based on the published ads. Moreover, an eighteen month probationary period is
recognized by the Labor Union in the private respondent company, which is Article V of the
Collective Bargaining Agreement.
2. YES
- The practice of a company in laying off workers because they failed to make the work
quota has been recognized in this jurisdiction. (Philippine American Embroideries vs.
Embroidery and Garment Workers, 26 SCRA 634, 639).
- In the case at bar, the petitioners' failure to meet the sales quota assigned to each of
them constitute a just cause of their dismissal, regardless of the permanent or
probationary status of their employment. Failure to observe prescribed standards of work,
or to fulfill reasonable work assignments due to inefficiency may constitute just cause for
dismissal.

Labor Law 1
- Such inefficiency is understood to mean failure to attain work goals or work quotas,
either by failing to complete the same within the alloted reasonable period, or by producing
unsatisfactory results. This management prerogative of requiring standards availed of so
long as they are exercised in good faith for the advancement of the employer's interest.

INTERNATIONAL CATHOLIC MIGRATION COMMISSION


V NLRC (GALANG)
[PAGE 98]
HOLIDAY INN MANILA V NLRC (HONASAN)
226 SCRA 417
AUSTRIA-MARTINEZ; July 12, 2006
NATURE
Appeal from a decision of the CA
FACTS
- Private respondent Gervasio Rosaroso was employed as a Third Engineer with Nicolakis
Shipping, S.A., a foreign firm through its recruitment and manning agency, petitioner
Skippers. The employment contract was for the period of one year beginning July 10,
1997 with a salary of $800 per month and other benefits. Rosaroso boarded M/V Naval
Gent on July 15, 1997. He was however ordered to disembark in Bulgaria on August 7,
1997 and repatriated to the Philippines.
- Soon after arrival in Manila, respondent filed a complaint for illegal dismissal and
monetary claims. The Labor Arbiter found the respondent was in fact illegally dismissed
and issued an order directing petitioner, Skippers, to pay Rosaroso separation pay of
$2,4000 or the equivalent of P100,000, representing three months pay and unpaid salary
for seven days of $186.69 or the equivalent of P7,840.98. Atorneys fees of P5,000 was
also awarded. The NLRC and the CA affirmed en toto the ruling of the Arbiter.
- Hence this appeal to the SC.
ISSUE
WON private respondent Rosaroso was illegal dismissed

HELD
YES
- The employer of Rosaroso did not provide the quantum of evidence needed to prove that
dismissal was in fact for cause. The evidence presented was just a telefax coming from
the alleged Chief Engineer of the vessel which the Arbiter up to the CA considered as
mere hearsay. While the Master of the vessel was grated under Paragraph D of Section
17 of the Philippine Overseas employment Administration (POEA) Standard Employment
condition governing the employment of Filipino Seafarers on Board Ocean Going Vessels
the power to dismiss for just cause without furnishing the seafarer with a notice of
dismissal if doing so will prejudice the safety of the crew and the vessel, the SC noted that
the complete report on the circumstances of the dismissal was not forwarded to the
manning agency as called for under the same provision.
Minor issues
- The award of backwages and separation pay in lieu of reinstatement as provided for in
Article 279 of the Labor Code is not applicable in this case. The Seafarer is a contractual
employee whose rights and obligations are governed by the POEA Employment Contract
and by RA 8042 (1995). The Employment contract does not provide for the award of
separation or termination pay. However, under Section 10 of RA 8042 the award of money
claims in cases of illegal dismissal is allowed. Under this provision, an illegal dismissed
seafarer is entitled to indemnity equivalent to his salary for the unexpired term of his
employment contract or three months for every year of the unexpired term, whichever is
less.
- The award by the Arbiter of the peso equivalent of the dollar awards cannot be enforced
as the same is contrary to law. The peso equivalent must be computed at the exchange
rate computed at the time of payment as provided for by RA 8183.
Disposition The questioned decision is affirmed with the modification that the dollar
award should be payable in its peso equivalent computed at the prevailing rate of
exchange at the time of payment.

BERNARDO V NLRC (FAR EAST BANK AND TRUST


COMPANY)
310 SCRA 186
PANGANIBAN; July 12, 1999
NATURE

A2010

- 103 -

Disini

Petition for Certiorari of June 20, 1995 Decision of NLRC which affirmed the August, 22
1994 ruling of Labor Arbiter Linsangan and August 4, 1995 Resolution of NLRC denying
the MFR
FACTS
- Complainants numbering 43 are deaf-mutes who were hired on various periods from
1988 to 1993 by respondent Far East Bank and Trust Co. as Money Sorters and Counters
through a uniformly worded agreement called "Employment Contract for Handicapped
Workers".
In compliance with Article 80 of the Labor Code of the Philippines as amended, the BANK
and the EMPLOYEE have entered into this Employment Contract as follows:
1. The BANK agrees to employ and train the EMPLOYEE, and the EMPLOYEE agrees to
diligently and faithfully work with the BANK, as Money Sorter and Counter.
2. The EMPLOYEE shall perform among others, the following duties and responsibilities:
i. Sort out bills according to color;
ii. Count each denomination per hundred, either manually or with the aid of a counting
machine;
iii. Wrap and label bills per hundred;
iv. Put the wrapped bills into bundles; and
v. Submit bundled bills to the bank teller for verification.
3. The EMPLOYEE shall undergo a training period of one (1) month, after which the BANK
shall determine whether or not he/she should be allowed to finish the remaining term of
this Contract.
4. The EMPLOYEE shall be entitled to an initial compensation of P118.00 per day, subject
to adjustment in the sole judgment of the BANK, payable every 15th and end of the month
5. The regular work schedule of the EMPLOYEE shall be five (5) days per week, from
Mondays thru Fridays, at eight (8) hours a day. The EMPLOYEE may be required to
perform overtime work as circumstance may warrant, for which overtime work he/she
[shall] be paid an additional compensation of 125% of his daily rate if performed during
ordinary days and 130% if performed during Saturday or [a] rest day.
6. The EMPLOYEE shall likewise be entitled to the following benefits:
i. Proportionate 13th month pay based on his basic daily wage.
ii. Five (5) days incentive leave.
iii. SSS premium payment.
7. The EMPLOYEE binds himself/herself to abide [by] and comply with all the BANK Rules
and Regulations and Policies, and to conduct himself/herself in a manner expected of all
employees of the BANK.
8. The EMPLOYEE acknowledges the fact that he/she had been employed under a
special employment program of the BANK, for which reason the standard hiring
requirements of the BANK were not applied in his/her case.
9. The Employment Contract shall be for a period of six (6) months unless earlier
terminated by the BANK for any just or reasonable cause.
- DEAF-MUTES HIRED
> 1988 - 2
> 1989 2
> 1990 19
> 1991 6
> 1992 6
> 1993 21
TOTAL: 56 (last: Thelma Malindoy who was employed in 1992 and whose contract
expired on July 1993)
- FAR EAST states:
> disclaimed that BERNARDO ET AL were regular employees AND that they are a special
class of workers the hearing impaired employees
> BERNARDO ET AL were hired temporarily under a special employment arrangement
which was a result of overtures made by some civic and political personalities to the Bank
> BERNARDO ET AL were hired due to "pakiusap" which must be considered in the light
of the context career and working environment which is to maintain and strengthen a
corps of professionals trained and qualified officers and regular employees who are
baccalaureate degree holders from excellent schools which is an unbending policy in the
hiring of regular employees
> training continues so that the regular employee grows in the corporate ladder; that the
idea of hiring handicapped workers was acceptable to them only on a special arrangement
basis
> counting and sorting of money are tellering works which were always logically and
naturally part and parcel of the tellers' normal functions
> the tellers themselves already did the sorting and counting chore as a regular feature
and integral part of their duties > through the "pakiusap" of Arturo Borjal, the tellers were
relieved of this task of counting and sorting bills in favor of deaf-mutes without creating
new positions as there is no position either in FAR EAST or any other Bank in the
Philippines which deals with purely counting and sorting of bills in banking operations.
- NLRC: affirmed ruling of the labor arbiter that BERNARDO ET AL could not be deemed
regular employees under A280 LC; gave credence to conclusion that BERNARDO ET AL
were hired as an accommodation to recommendation of civic oriented personalities; the

Labor Law 1
terms of the contract shall be the law between the parties; AND Magna Carta for Disabled
Persons is not applicable, "considering the prevailing circumstances/milieu of the case."
ISSUES
WON NLRC is guilty of grave abuse of discretion in holding that
1. money sorters and counters working in a bank are not regular employees
2. employment contracts signed and renewed by the petitioners, which provide for a
period of 6 months, were valid
3. not applying the provisions of the Magna Carta for the Disabled (RA 7277), on
proscription against discrimination against disabled persons.
HELD
Preliminary Matter
- Propriety of Certiorari: the Court, as a rule, does not review the factual findings of public
respondents in a certiorari proceeding. In resolving whether the petitioners have become
regular employees, SC shall not change the facts found by NLRC. SCs task is merely to
determine whether the NLRC committed grave abuse of discretion in applying the law to
the established facts
1. YES. Only the employees, who worked for more than 6 months and whose contracts
were renewed are deemed regular. Hence, their dismissal from employment was illegal.
Reasoning
- According to FAR EAST, the employment contracts were prepared in accordance with
A80 LC, which provides
Art. 80. Employment agreement. Any employer who employs handicapped workers
shall enter into an employment agreement with them, which agreement shall include:
(a) The names and addresses of the handicapped workers to be employed;
(b) The rate to be paid the handicapped workers which shall be not less than seventy
five (75%) per cent of the applicable legal minimum wage;
(c) The duration of employment period; and
(d) The work to be performed by handicapped workers.
The employment agreement shall be subject to inspection by the Secretary of Labor or
his duly authorized representatives.
- However, succeeding events and the enactment of RA No. 7277 (the Magna Carta for
Disabled Persons) justify the application of A280 LC.
- FAR EAST entered into contract with a total of 56 handicapped workers and renewed the
contracts of 37 of them. In fact, two of them worked from 1988 to 1993. Verily, the renewal
of the contracts of the handicapped workers and the hiring of others lead to the conclusion
that their tasks were beneficial and necessary to the bank. More important, these facts
show that they were qualified to perform the responsibilities of their positions. In other
words, their disability did not render them unqualified or unfit for the tasks assigned to
them.
- Magna Carta for Disabled Persons mandates that a qualified disabled employee should
be given the same terms and conditions of employment as a qualified able-bodied person.
- Section 5 of the Magna Carta provides:
Sec. 5. Equal Opportunity for Employment. No disabled person shall be denied
access to opportunities for suitable employment. A qualified disabled employee shall be
subject to the same terms and conditions of employment and the same compensation,
privileges, benefits, fringe benefits, incentives or allowances as a qualified able bodied
person.
- Since the Magna Carta accords them the rights of qualified able-bodied persons, they
are thus covered by Article 280 of the Labor Code, which provides:
Art. 280. Regular and Casual Employment. The provisions of written agreement to
the contrary notwithstanding and regardless of the oral agreement of the parties, an
employment shall be deemed to be regular where the employee has been engaged to
perform activities which are usually necessary or desirable in the usual business or
trade of the employer, except where the employment has been fixed for a specific
project or undertaking the completion or termination of which has been determined at
the time of the engagement of the employee or where the work or services to be
performed is seasonal in nature and the employment is for the duration of the season.
An employment shall be deemed to be casual if it is not covered by the preceding
paragraph: Provided, That, any employee who has rendered at least one year of
service, whether such service is continuous or broken, shall be considered as regular
employee with respect to the activity in which he is employed and his employment shall
continue while such activity exists.
- The test of whether an employee is regular
> De Leon v. NLRC: The primary standard, therefore, of determining regular employment
is the reasonable connection between the particular activity performed by the employee in
relation to the usual trade or business of the employer. The test is whether the former is
usually necessary or desirable in the usual business or trade of the employer. The
connection can be determined by considering the nature of the work performed and its
relation to the scheme of the particular business or trade in its entirety. Also if the
employee has been performing the job for at least one year, even if the performance is not
continuous and merely intermittent, the law deems repeated and continuing need for its
performance as sufficient evidence of the necessity if not indispensibility of that activity to

A2010

- 104 -

Disini

the business. Hence, the employment is considered regular, but only with respect to such
activity, and while such activity exist.
- The task of counting and sorting bills is necessary and desirable to the business of
respondent bank. With the exception of sixteen of them, BERNARDO ET AL performed
these tasks for more than six months.
- As held by the Court, "Articles 280 and 281 of the Labor Code put an end to the
pernicious practice of making permanent casuals of our lowly employees by the simple
expedient of extending to them probationary appointments, ad infinitum." The contract
signed by petitioners is akin to a probationary employment, during which the bank
determined the employees' fitness for the job. When the bank renewed the contract after
the lapse of the six-month probationary period, the employees thereby became regular
employees. No employer is allowed to determine indefinitely the fitness of its employees.
- As regular employees, the 27 petitioners are entitled to security of tenure; that is, their
services may be terminated only for a just or authorized cause. Therefore, when FAR
EAST failed to show such cause, they are deemed illegally dismissed and entitled to back
wages and reinstatement without loss of seniority rights and other privileges. Considering
that the job of money sorting is no longer available because it has been assigned back to
the tellers to whom it originally belonged, petitioners are hereby awarded separation pay in
lieu of reinstatement.
2. YES
- Brent Ruling, which upheld the validity of an employment contract with a fixed term, is
not applicable. The term limit in the contract was premised on the fact that the petitioners
were disabled, and that the bank had to determine their fitness for the position. Its validity
is based on A80 LC. Petitioners proved themselves to be qualified disabled persons who,
under the Magna Carta for Disabled Persons, are entitled to terms and conditions of
employment enjoyed by qualified able-bodied individuals; hence, A80 does not apply.
- Moreover, it must be emphasized that a contract of employment is impressed with public
interest. Provisions of applicable statutes are deemed written into the contract, and the
"parties are not at liberty to insulate themselves and their relationships from the impact of
labor laws and regulations by simply contracting with each other." 23 Clearly, the
agreement of the parties regarding the period of employment cannot prevail over the
provisions of the Magna Carta for Disabled Persons, which mandate that petitioners must
be treated as qualified able-bodied employees.
- Because sorting is done in the nighttime, FAR EAST contended that this task "could not
be done by deaf mutes because of their physical limitations as it is very risky for them to
travel at night." But the court found no basis in this because traveling at night involves
risks to handicapped and able-bodied persons alike
- Datu v. NLRC: the determination of whether employment is casual or regular does not
depend on the will or word of the employer, and the procedure of hiring . . . but on the
nature of the activities performed by the employee, and to some extent, the length of
performance and its continued existence.
- The well-settled rule is that the character of employment is determined not by stipulations
in the contract, but by the nature of the work performed. Otherwise, no employee can
become regular by the simple expedient of incorporating this condition in the contract of
employment.
> Romares v. NLRC: A280 was emplaced in our statute books to prevent the
circumvention of the employee's right to be secure in his tenure by indiscriminately and
completely ruling out all written and oral agreements inconsistent with the concept of
regular employment defined therein. Where an employee has been engaged to perform
activities which are usually necessary or desirable in the usual business of the employer,
such employee is deemed a regular employee and is entitled to security of tenure
notwithstanding the contrary provisions of his contract of employment.
3. YES
- The noble objectives of Magna Carta for Disabled Persons are not based merely on
charity or accommodation, but on justice and the equal treatment of qualified persons,
disabled or not. The Court believes, that, after showing their fitness for the work assigned
to them, they should be treated and granted the same rights like any other regular
employees.
Disposition Petition is hereby GRANTED. The June 20, 1995 Decision and the August 4,
1995 Resolution of the NLRC are REVERSED and SET ASIDE. Respondent Far East
Bank and Trust Company is hereby ORDERED to pay back wages and separation pay to
each of the following twenty-seven (27) petitioners

MITSUBISHI MOTORS CORP V CHRYSLER


PHIL LABOR UNION
[PAGE 102]
ALCIRA V NLRC
(MIDDLEBY PHILS/THOMAS, PENA, MAMARADLO)
431 SCRA 508
CORONA; June 9, 2004

Labor Law 1
NATURE
Appeal from decision of CA (dismissing complaint for illegal dismissal)
FACTS
- Alcira was hired by Middleby as an engineering support services supervisor on a
probationary basis for 6 months. Apparently unhappy with Alciras performance, the
company terminated Alciras services. The issue arises on whether the termination
occurred before or after the 6-month probationary period.
- Alcira: he was hired on May 20, 1996
- Middleby: Alcira was hired May 27, 1996
- The appointment paper indicated that the status was probationary (6mos) and a remark
that after 5 mos, performance shall be evaluated and any adjustment shall depend on
work performance
- On November 20, 1996, Alcira was not allowed to work (allegedly by withholding of his
time card by a senior officer).
- Alcira filed a complaint with the Labor Arbiter, contending that he was already a regular
employee as of the date he was dismissed. (he computed the 6mos probationary period
as 30days x 6 mos = 180 days, thus coming to the conclusion that the period ended on
the 180th day after the day of employment).
- The company (and its officers, herein private respondents) presented evidence that
showed Alciras poor performance, tardiness, absences, and violations of company rules
on wearing of uniform during the probationary period, and said that since he failed to meet
the standards, Alciras application to become a regular employee was denied.
LA, NLRC, and CA all ruled in favor of the company, upholding the validity of the
dismissal.
- Central to the matter at hand is art. 281 of Labor Code:
ART. 281. PROBATIONARY EMPLOYMENT. Probationary employment shall not
exceed six (6) months from the date the employee started working, unless it is covered
by an apprenticeship agreement stipulating a longer period. The services of an
employee who has been engaged on a probationary basis may be terminated for a just
cause or when he fails to qualify as a regular employee in accordance with reasonable
standards made known by the employer to the employee at the time of his
engagement. An employee who is allowed to work after a probationary period shall be
considered a regular employee.
ISSUES
1. WON petitioner was allowed to work beyond his probationary period, and was therefore
already a regular employee at the time of his dismissal
2. WON Middleby informed petitioner of standards for regularization at the start of his
employment
3. WON Alcira was illegally dismissed when Middleby opted not to renew his contract on
the last day of the probationary period
HELD
1. NO
Ratio Computation of the 6-month probationary period is reckoned from the date of
appointment up to the same calendar date of the 6th month following, thus dismissal on
November 20, 1996 was well within the probationary period.
Reasoning
- the computation of Alcira (using 30 days x 6 months = 180 days) is wrong. As held in
CALS Poultry Supply Corp v Roco: (O)ur computation of the 6-month probationary period
is reckoned from the date of appointment up to the same calendar date of the 6th month
following.
- In short, since the number of days in each particular month was irrelevant, Alcira was still
a probationary employee when Middleby opted not to "regularize" him on November 20,
1996.
2. YES
Ratio An employer is deemed to substantially comply with the rule on notification of
standards if he apprises the employee that he will be subjected to a performance
evaluation on a particular date after his hiring.
Reasoning
- Middleby substantially notified the petitioner of the standards of a regular employee when
it apprised him, at the start of his employment, that it would evaluate his supervisory skills
after 5 months.
- That the appointment paper contained the remark that Alcira would be subjected to a
performance evaluation is enough notice that the probationary basis of his employment
was conditional (conditioned upon his meeting of performance standards)
3. NO
Ratio Although probationary employees are also accorded security of tenure, this
protection ends upon expiration of the probationary period.
Reasoning
- It is settled that even if probationary employees do not enjoy permanent status, they are
accorded the constitutional protection of security of tenure. This means they may only be

A2010

- 105 -

Disini

terminated for just cause or when they otherwise fail to qualify as regular employees in
accordance with reasonable standards made known to them by the employer at the time
of their engagement.
- But we have also ruled in Manlimos, et. al. vs. National Labor Relations Commission that
this constitutional protection ends on the expiration of the probationary period. On that
date, the parties are free to either renew or terminate their contract of employment.
In this case, Middleby exercised its option not to renew the contract when it informed
Alcira on the last day of his probationary employment that it did not intend to grant him a
regular status.
Disposition Petition DENIED.

CRITERIA REGULARIZATION
ALCIRA V NLRC
(MIDDLEBY PHILS/THOMAS, PENA, MAMARADLO)
[PAGE105]

EXTENSION OF CONTRACT
MARIWASA V LEOGARDO
169 SCRA 465
NARVASA; January 26, 1989
NATURE
Certiorari
FACTS
- Dequila was hired on probation by Mariwasa as a general utility worker on Jan.10, 1979.
After the probationary period of six months, MAriwasa informed him that his work had
proved unsatisfactory and had failed to meet the required standards and to give him a
chance to improve his performance and qualify for regular employment, instead of
dispensing with his service then and there, with his written consent Mariwasa extended his
probation period for another three months from July 10 to October 9, 1979. His
performance, however, did not improve and on that account Mariwasa terminated his
employment at the end of the extended period.
ISSUE
WON employer and employee may by agreement extend the probationary period of
employment beyond the six months prescribed in A282 LC
HELD
YES
- An extension of the probationary period of employment may lawfully be covenanted,
notwithstanding the seemingly restrictive language of Article 282.
Reasoning
- Buiser vs. Leogardo, Jr . recognized agreements stipulating longer probationary periods
as constituting lawful exceptions to the statutory prescription limiting such periods to six
months, when it upheld as valid an employment contract between an employer and two of
its employees that provided for an eigthteen-month probation period. This Court there
held:
- Generally, the probationary period of employment is limited to six (6) months. The
exception to this general rule is when the parties to an employment contract may agree
otherwise, such as when the same is established by company policy or when the same is
required by the nature of work to be performed by the employee. In the latter case, there is
recognition of the exercise of managerial prerogatives in requiring a longer period of
probationary employment.xxxxxx especially where the employee must learn a particular
kind of work such as selling, or when the job requires certain qualifications, skills
experience or training.
- In this case the inability of the probationer to make the grade became apparent only at or
about the end of the six-month period, hence an extension could not have been prearranged as was done in Buiser assumes no adverse significance, given the lack, of any
indication that the extension to which Dequila gave his agreement was a mere stratagem
of petitioners to avoid the legal consequences of a probationary period satisfactorily
completed. For the extension of Dequila's probation was ex gratia, an act of liberality on
the part of his employer affording him a second chance to make good after having initially
failed to prove his worth as an employee. Such an act cannot now unjustly be turned

Labor Law 1
against said employer's account to compel it to keep on its payroll one who could not
perform according to its work standards. The law, surely, was never meant to produce
such an inequitable result.
- By voluntarily agreeing to an extension of the probationary period, Dequila in effect
waived any benefit attaching to the completion of said period if he still failed to make the
grade during the period of extension. The Court finds nothing in the law which by any fair
interpretation prohibits such a waiver. And no public policy protecting the employee and
the security of his tenure is served by prescribing voluntary agreements which, by
reasonably extending the period of probation, actually improve and further a probationary
employee's prospects of demonstrating his fitness for regular employment.

ABSORBED EMPLOYEES
CEBU STEVEDORING CO INC V REGIONAL DIRECTOR
168 SCRA 315
REGALADO; December 8, 1988
FACTS
- Complainants Gelig and Quijano (COMPLAINANTS) were former employees of the Cebu
Customs Arrastre Service (CCAS). Pursuant to an Administrative Order by the Bureau of
Customs, CCAS was abolished "for the reason that the objectives for which it was created
had already been attained". Consequently, all the employees of CCAS, including
COMPLAINANTS, were given their termination separation pay by the Bureau.
- After the abolition, all the employees of CCAS, including COMPLAINANTS, were
absorbed by CSCI with the same positions that they held in the CCAS. Almost 6 months
later, however, COMPLAINANTS were dismissed by CSCI without prior clearance.
COMPLAINANTS consequently filed an action for reinstatement with backwages with the
Labor Regional Office. The Regional Director ruled in COMPLAINANTS favor, which
ruling was affirmed on appeal by the then Ministry of Labor and, subsequently, by the
Office of the President.
ISSUES
1. WON the CSCI was denied due process due to the lack of hearing before the Regional
Director and COMPLAINANTS failure to file their respective position papers
2. WON the COMPLAINANTS were merely casuals and could, therefore, be terminated
even without prior clearance from the then Ministry of Labor and without entitlement to
separation pay
3. WON the positions occupied by COMPLAINANTS with CCAS are identical with the
positions already filled up and being discharged in the main office of CSCI,
COMPLAINANTS may be terminated for redundancy
HELD
1. NO
- The right to be heard, as a preliminary step essential to the rendition of an enforceable
judgment, constitutes a basic element of the constitutional requirement of due process of
law. However, while CSCI was not afforded an opportunity to be heard by oral argument
on its position paper due to its absence at the scheduled hearing, it is likewise true that it
was required to, as in fact it actually did, submit a position paper which, together with the
evidence presented during the hearing, became the basis of the questioned order of the
Regional Director. From this order, CSCI appealed to the Labor Minister, and then to the
Office of the President. It is, therefore, apparent that CSCI was not denied adequate
remedies from the alleged procedural infirmities surrounding the Regional Director's order.
The entire record of the case was reviewed and duly considered on appeal, which
appellate proceeding remedied any inadequacy in the procedural due process with which
the trial proceedings are being faulted.
2. NO
- We agree with the Regional Director that COMPLAINANTS could not be considered
probationary employees because they were already well-trained in their respective
functions. While COMPLAINANTS were still with the CCAS they were already clerks with
10 years of service, on the average. They were, therefore, experienced workers. Findings
of quasi-judicial agencies (like the Labor Regional Office) which have acquired expertise
because their jurisdiction is confined to specific matters are generally accorded not only

A2010

- 106 -

Disini

respect but, at times, even finality where such findings are supported by substantial
evidence.
3. NO
- Despite Art. 283s provision on termination of employment due to redundancy and
retrenchment, records fail to establish clearly that the positions occupied by
COMPLAINANTS are identical with those presently existing in CSCI's office. CSCI also
kept COMPLAINANTS in its employ for almost 6 months without raising this issue. CSCI
does not mention which positions are allegedly duplicated by the positions held by
COMPLAINANTS. CSCI does not even explain why COMPLAINANTS should be the ones
to be terminated, without regard to the comparative lengths of service, qualifications and
performance of all employees concerned.
- CSCIs claim of financial losses is untenable since it appears that it absorbed and
employed for almost 6 months, without any intimation of supposed financial distress, the
majority of the former employees of CCAS. It never advised COMPLAINANTS of a
company retrenchment program; the first time this supposed program was mentioned was
when CSCI was trying to justify the dismissal of COMPLAINANTS before the labor arbiter.
CSCI presented a Statement of Operations, which, however, remains an uncorroborated
and self-serving piece of evidence.
Disposition Petition DISMISSED. COMPLAINANTS ordered reinstated with backwages.
In the event reinstatement becomes impossible, COMPLAINANTS to be given 1-monthsalary worth of separation pay.

DOUBLE PROBATION
A PRIME SECURITY SERVICES INC V NLRC (GUANIO,
MORENO)
322 SCRA 283
PURISIMA; January 19, 2000
NATURE
A special civil action for certiorari seeks to annul the decision of the NLRC
FACTS
- The complaint alleged, among others, that complainant (Othello C. Moreno) had been
working as a security guard for a year with the Sugarland Security Services, Inc., a sister
company of petitioner; that he was rehired as a security guard on January 30, 1988 by the
petitioner and assigned to the same post at the U.S. Embassy Building; that he was
among those absorbed by the petitioner when it took over the security contracts of its
sister company with the U.S. Embassy; that he was forced by petitioner to sign new
probationary contracts of employment for 6 months; that on August 1, 1988, his
employment was terminated; that during his employment, the amount of P20.00 per month
was deducted from his salary allegedly for withholding tax, and the salary he was
receiving was only P2,187.00 a month, which was way below the P2,410.17 stipulated in
the PADPAO memorandum of agreement.
- Petitioner alleged that the private respondent was hired on January 30, 1988, on a
probationary basis, and he signed an authority to deduct from his salary any
reimbursement for any loss or damage caused to properties of the client; that he was
given a copy of petitioners rules and regulations which provide that sleeping on post is
punishable by warning, suspension and dismissal and he was caught sleeping on post on
March 17, 1988, for which he was sent a memorandum giving him a last warning; that on
March 25, 1988, he figured in a quarrel with another security guard, which resulted in a
near shootout; that at the end of his probationary employment, he was given a
psychological test and on the basis of the foregoing, petitioner told him that his
probationary employment had come to an end as he did not pass the company standard
and therefore, he could not be hired as a regular employee.
- LA Guanio handed down a decision in favor of complainant. The respondent was ordered
to reinstate the complainant to his former position and accord to him the status of a regular
employee, and to refund to the complainant the deduction it had made from his salary in
the amount of P20.00 per month. The claim of the complainant for underpayment of
wages is dismissed for lack of merit.
- NLRC affirmed the decision with a slight modification: the refund of the deductions made
by respondent from complainants salaries in the amount of P20.00 per month was
vacated and set aside.
ISSUES
1. WON private respondents employment with A Prime Security Services, Inc. was just a
continuation of his employment with Sugarland Security Services, Inc
2. WON private respondent is a regular employee of petitioner
3. WON private respondents dismissal is illegal
HELD
1. YES

Labor Law 1
Ratio The Court cannot sanction the practice of some companies which, shortly after a
worker has become a regular employee, effects the transfer of the same employee to
another entity whose owners are the same, or identical, in order to deprive subject
employee of the benefits and protection he is entitled to under the law.
Reasoning
- The allegations of the private respondent that Sugarland is a sister company of A Prime
and that the latter absorbed the security contracts and security guards of Sugarland with
the U.S. Embassy were neither denied nor controverted by the petitioner before the Labor
Arbiter.
- Petitioners failure to deny that Sugarland is its sister company and that petitioner
absorbed Sugarlands security contract and security personnel assumes overriding
significance over the resignation theorized upon, evincing petitioners design to ignore or
violate labor laws through the use of the veil of corporate personality.
2. YES
Ratio The complainant became a regular employee upon completion of his six-month
period of probation. Private respondent started working on January 30, 1988 and
completed the said period of probation on July 27, 1988. Thus, at the time private
respondent was dismissed on August 1, 1988, he was already a regular employee with a
security of tenure. He could only be dismissed for a just and authorized cause.
- There is no basis for subjecting private respondent to a new probationary or temporary
employment on January 30, 1988, considering that he was already a regular employee
when he was absorbed by A Prime from Sugarland, its sister company.
3. YES
Ratio The dismissal of complainant, a regular employee, was sans any just, legal and
valid basis. What is more, he was not given a chance to contest his dismissal. He was
deprived of an opportunity to be heard.
Reasoning
- The dismissal of private respondent was presumably based on the results of his
behavioral and neuropsychological tests and on his violation of a company rule on
sleeping on post.
- With respect to the behavioral and neuropsychological tests, the Court agrees with
NLRCs assessment: "Complainants result of his behavioral research and
neuropsychological test to our mind, is of no moment, considering that the said test
appeared to have been conveniently contrived to be conducted, and the result produced
on the very day of his dismissal, in question.
- So also, private respondents alleged violations of sleeping on post, and quarrelling with
a co-worker, may not be proper grounds for dismissal, as the same were first infractions.
Circular No. I of A Prime, governing discipline, suspension and separation from the
service of security guards, provides:
"SECTION VIII - SLEEPING ON POST
1st Offense........- Warning
2nd Offense.......- 30 days suspension without pay
3rd Offense........- Dismissal
SECTION IX - CHALLENGING A POSTED SECURITY/LADY GUARD AND
SUPERIORS
1st Offense - One (1) month suspension
2nd Offense Dismissal
- As the infractions of Sections VIII and IX of Circular No. 1 by private respondent were
first offenses, they were not punishable by dismissal. They were not valid grounds for
terminating the employment of private respondent.
Disposition petition is DISMISSED

TERMINATION AND SALARY


INTERNATIONAL CATHOLIC MIGRATION COMMISSION
V NLRC (GALANG)
[PAGE 98]
ORIENT EXPRESS PLACEMENT PHILIPPINES V NLRC
(POEA, FLORES)
[PAGE 102]
DELA CRUZ V NLRC
[PAGE 100]

RULE PRIVATE SCHOOL TEACHERS

A2010

Disini

- 107 -

CHIANG KAI SHEK COLLEGE V CA (NLRC,


CALAYLAY, AQUINO, GACUTAN, BELO)
[PAGE 40]
LA CONSOLACION COLLEGE V NLRC
366 SCRA 226
PARDO; September 28, 2001
NATURE
Petition for certiorari with preliminary injunction or restraining order
FACTS
- Respondent Jose de la Pea was initially employed by La Consolacion College (LCC) as
a CAT Commandant for the school year 1975-1976 & lasted until Sept. 1979, after which
he resigned. Prior to resignation & despite demands by LCC for him to submit a syllabi in
YDT and CAT containing course objectives, subject matter, content, concepts, skills
activities & evaluation, de la Pea failed to comply.
- 11 years later, or on Dec.2, 1991, respondent applied to the LCC again, for the positions
of CAT Commander. He was appointed instead as classroom teacher in P.E. and health.
The written contract of employment between LCC & respondent expressly provided that
the employment was for 1 academic year, from June 19992 - March 1993, & the same
was accepted by respondent.
- July 1992: A reminder was sent to respondent by the academic evaluation team
regarding the schools standard procedures: timely submission of lesson plans, class
records & attendance at regular meetings which respondent defied & continued to ignore
even after having been reminded. In an emergency meeting of faculty members,
respondent berated one Bayoguing, a member of the academic evaluation team &
threatened him with bodily harm, although no untoward incident ensued.
- As his contract was only for one academic school year, de la Pea applied for
reinstatement in Feb. 1993 but the same was denied by the academic team because of
his unsatisfactory performance.
- Respondent then filed a complaint in the Regional Arbitration Bacolod Branch for illegal
dismissal against LCC & members of the academic team. The Labor Arbiter (LA)
dismissed this complaint. On appeal, the NLRC reversed the LAs decision and held that
respondent attained regular status at the time he was dismissed & that LCC failed to prove
just cause for his dismissal. Petitioner LCC then filed a MFR to the NLRC which was
denied, hence this petition.
ISSUE
WON de la Pea was a permanent employee of LCC
HELD
NO. He did not attain permanent status.
Ratio In resolving issues regarding security of tenure of private school teachers, it is the
Manual of Regulations for Private Schools, not the Labor Code, which is applicable.
Reasoning
- For a private school teacher to acquire permanent status in employment the following
must concur: (1) The teacher is a full-time teacher; (2) The teacher must have rendered 3
consecutive yrs. of service; (3) Such service must have been satisfactory
- The written contract of respondent stated that he shall be employed by the LCC for the
school year June 1992 - March 1993, a fixed term of 10 mos. Clearly, the employment was
not permanent but for a specified duration of one school year. Respondent was a new hire
having previously resigned & never denied the fact that he failed to comply with the
requirements of the school.
Disposition Petition granted, NLRC decision reversed and set aside.

PART TWO
LABOR STANDARDS LAW
PURPOSE
MARIVELES SHIPYARD CORP V CA (REGONDOLA)
415 SCRA 573
QUISUMBING; November 11, 2003
NATURE
Petition for review on certiorari of the decision of the CA
FACTS

Labor Law 1
- In Oct 1993, Mariveles Shipyard Corp engaged the services of Longest Force
Investigation and Security Agency, Inc. to render security services at the formers
premises.
- Petitioner religiously complied with the terms and conditions of the security contract with
Longest Force, promptly paying its bills and contract rates. However, it found the services
being rendered by the assigned guards unsatisfactory and inadequate, causing Mariveles
Shipyard Corp to terminate its contract with Longest Force on April 1995. Longest Force,
in turn, terminated the employment of the security guards it had deployed at the shipyard.
- Private respondents filed a case with the Labor Arbiter. Longest Force, in turn, filed a
cross-claim against Mariveles Shipyard.
- Longest force admitted the following:
> that it employed private respondents as security guards and assigned them to work
at petitioners shipyard, rendering 12 hours duty per shift
> it is liable as to the non-payment of the alleged wage differential, but passed liability
to petitioner alleging that the service fee paid by Mariveles was way below the
PNPSOSIA and PADPAO rates
- Petitioner denied liability on account of alleged illegal dismissal, stressing that no
employer-employee relationship exists between it and security guards
- Labor Arbiter declared Mariveles and Longest Force jointly and severally liable to pay the
money claims of private respondents, and ordered their reinstatement.
- NLRC affirmed in toto the decision of the Labor Arbiter. CA denied due course to
petitioners appeal and dismissed the case
ISSUE
WON petitioner Mariveles should be held jointly and severally liable with Longest Force for
the payment of wage differentials and overtime pay owing to private respondents
HELD
YES, petitioners liability is joint and several that of Longest Force pursuant to Articles 106,
107 and 109 of the Labor Code.
Art 106. CONTRACTOR OR SUBCONTRACTOR Whenever an employer
enters into a contract with another person in the performance of the formers
work, the employees of the contractor and of the latters subcontractor, if any,
shall be paid in accordance with the provisions of this Code.
In the event that the contractor or subcontractor fails to pay the wages of
his employees in accordance with this Code, the employer shall be jointly and
severally liable with this contractor or subcontractor to such employees to the
extent of the work performed under the contract, in the same manner and extent
that he is liable to employee directly employed by him.
Art 107. INDIRECT EMPLOYER The provisions of the immediately preceding
Article shall likewise apply to any person, partnership, association or corporation
which, not being an employer, contracts with an independent contractor for the
performance of any work, task, job or project.
Art 109. SOLIDARY LIABILITY The provisions of existing laws to the contrary
notwithstanding, every employer or indirect employer shall be held responsible
with his contractor or subcontractor for any violation of any provision of this
Code. For purposes of determining the extent of their civil liability under this
Chapter, they shall be considered as direct employers.
- When petitioner contracted for security services with Longest Force, petitioner became
an indirect employer of private respondents. When the agency as contractor failed to pay
the guards, the corporation as principal becomes jointly and severally liable to the guards
wages. The security agency is held liable by virtue of its status as direct employer, while
the corporation is deemed the indirect employer of the guards for the purpose of paying
their wages in the event of failure of the agency to pay them. This statutory scheme gives
the workers the ample protection consonant with labor and social justice provisions of the
1987 Constitution.
- Labor standards are enacted by the legislature to alleviate the plight of workers whose
wages barely meet the spiraling costs of basic needs. Labor laws are considered written in
every contract. Stipulations in violation thereof are considered null. Similarly, legislated
wage increases are deemed amendments to the contract. Thus, employers cannot hide
behind their contracts in order to evade their (or their contractors or subcontractors)
liability for noncompliance with the statutory minimum wage.
- However, the solidary liability of petitioner with that of Longest Force does not preclude
the application of the Civil Code provisions on the right of reimbursement from his codebtor by the one who paid.
Disposition Decision of Appellate Court affirmed. Petitioner and Longest Force are held
liable jointly and severally for underpayment of wages and overtime pay of the security
guards, without prejudice to petitioners right of reimbursement from Longest Force
Investigation and Security Agency, Inc.

SECTION 1: EMPLOYMENT POLICY

A2010

Disini

- 108 -

SECTION 2: RECRUITMENT AND PLACEMENT


OF WORKERS
2.01 RECRUITMENT AND PLACEMENT OF
WORKERS DEFINITION
DEFINITION LAW STRUCTURE
PEOPLE V PANIS
142 SCRA 664
CRUZ; July 11, 1986
NATURE
APPEAL by certiorari to review the orders of the CFI Zambales and Olongapo, by Judge
Panis
FACTS
- Serapio Abug was charged w/ illegal recruitment for operating a private fee-charging
employment agency by charging fees and expenses and promising employment in Saudi
Arabia to four separate individuals w/o a license.
- Abug filed a motion to quash on the ground that the informations did not charge an
offense because he was accused of illegally recruiting only one person in each of the four
informations. Under the proviso in Article 13(b), he claimed, there would be illegal
recruitment only "whenever two or more persons are in any manner promised or offered
any employment for a fee." The motion was granted by the trial court.
- Private respondent: to constitute recruitment and placement, all the acts mentioned in
Article 13(b) should involve dealings with two or more persons as an indispensable
requirement.
- Petitioner: the requirement of two or more persons is imposed only where the
recruitment and placement consists of an offer or promise of employment to such persons
and always in consideration of a fee. The other acts mentioned in the body of the article
may involve even only one person and are not necessarily for profit.
ISSUE
Which is the correct interpretation of Article 13(b) of P.D. 442 24
HELD
NEITHER
- The number of persons dealt with is not an essential ingredient of the act of recruitment
and placement of workers. Any of the acts mentioned in the basic rule in Article 13(b) will
constitute recruitment and placement even if only one prospective worker is involved. The
proviso merely lays down a rule of evidence that where a fee is collected in consideration
of a promise or offer of employment to two or more prospective workers, the individual or
entity dealing with them shall be deemed to be engaged in the act of recruitment and
placement.
- The proviso was intended neither to impose a condition on the basic rule nor to provide
an exception thereto but merely to create a presumption. The presumption is that the
individual or entity is engaged in recruitment and placement whenever he or it is dealing
with two or more persons to whom, in consideration of a fee, an offer or promise of
employment is made in the course of the "canvassing, enlisting, contracting, transporting,
utilizing, hiring or procuring (of) workers."

PEOPLE V SAULO
344 SCRA 605
GONZAGA-REYES; November 15, 2000
FACTS
- Romulo Saulo, together with Amelia and Clodualo de la Cruz, were charged with
violation of A38 of the Labor Code for illegal recruitment in large scale.
- From April to May 1990, the three accused falsely represented themelves to have the
capacity to contract, enlist and recruit workers for employment abroad. They promised job
24

'Recruitment and placement' refers to any act of canvassing, enlisting, contracting, transporting, hiring, or procuring
workers, and includes referrals, contract services, promising or advertising for employment, locally or abroad, whether
for profit or not: Provided, That any person or entity which, in any manner, offers or promises for a fee employment to two
or more persons shall be deemed engaged in recruitment and placement."

Labor Law 1
placements to Maullon, Maligaya and Javier without first securing the required license or
authority from DOLE. In addition, the three were also charged with three counts of estafa.
- Maligaya had learned from a relative of Saulo that the latter was recruiting workers for
Taiwan. He, along with Maullon and Javier, went to visit Saulo in Saulos San Francisco
del Monte home. Saulo told Maligaya that she could leave for Taiwan as soon as she paid
the fees for the processing of documents.
- Saulo pleaded not guilty to the charges against him. Amelia and Clodualdo still remain at
large. Saulo was eventually found guilty of three counts of estafa and illegal recruitment.
- Petitioners Claim:
> Maligaya paid P35,000 evidenced by a receipt dated May 21, 1990. Javier was also told
to pay the same amount but she gave an initial amount of P20,000. She did not ask for a
receipt since she trusted Saulo. Maullo was told to pay P30,000 as processing fee for
work in Taiwan. Maullon made an initial payment of P7,900 to Saulos wife who issued
him a receipt in turn. Maullon then made an additional payment of P6,800 in the presence
of Amelia de la Cruz and another payment of P15,700 to Tumalig, a friend of Saulo, who
also issued him another receipt.
> In all three instances, Saulo failed to deliver what he promised. The prosecution also
presented a certification dated July 26, 1994 which stated that Saulo was not authorized
by the POEA to recruit workers for overseas employment.
- Respondents Comments:
> Saulo interposes for his defense a claim that he was also applying for work abrad
through Amelia de la Cruz which led him to meet the three complainants. They were all
there to follow-up their applications and that he was also deceived by Amelia.
> He denied being an overseas recruiter nor an agent for one. He also denies receiving
the abovementioned amounts from complainants.
> He could not have committed the crime because testimony from a POEA employee
showed that licenses for recruitment are issued only to corporations and not to natural
persons.
ISSUE
WON the appeal of accused should be given merit
HELD
NO
Ratio Recruitment under the Labor Code refers to any act of canvassing, enlisting,
contracting, transporting, utilizing, hiring or procuring workers and includes referrals,
contract services, promising or advertising for employment locally or abroad, whether for
profit or not; Provided, that any person or entity in which, in any manner, offers or
promises for a fee employment to two or more persons shall be deemed engaged in
recruitment and placement.
Reasoning
- The essential elements of illegal recruitment in large scale are as follows:
(a) the accused engages in the recruitment and placement of workers as defined under
Art. 13(b) or in any of the enumerated prohibitions in Art. 31 of the Labor Code
(b) the accused has not complied with the guidelines issued by the Secretary of Labor
and Employment, particularly with respect to the securing of a license or an authority to
recruit and deploy workers whether locally or overseas
(c) accused commits the same against three more persons individually or as a group.
- Saulo proferred inadequate evidence to prove his innocence. Even if Saulo did not sign
all the receipts presented by the complainants, it does not weaken the case in any way. A
person charged with illegal recruitment may be convicted on the strength of the
testimonies of the complainants, if found to be credible and convincing.
- On the argument that licenses for recruitment are issued only to corporations, the Labor
Code states that any person or entity which, in any manner, offers or promises for a fee
employment to two or more persons shall be deemed engaged in recruitment and
placement. A nonlicensee or nonholder of authority is a person, corporation or entity
which has not been issued a valid license or authority to engage in recruitment and
placement by the Secretary of Labor or whose license or recovery has been suspended,
revoked or cancelled by the POEA. Agents or representatives appointed by a licensee or
a holder of authority but whose appointments were not previously recognized by the POEA
fall within the definition stated above.
Disposition Judgment affirmed.

2.02 EMPLOYMENT AGENCY


2.03 ALLOWED ENTITIES
A. GENERAL RULE
B. ALLOWED ENTITIES

A2010

- 109 -

Disini

2.04 PROHIBITED ENTITY

2.05 TECHNIQUES IN REGULATION


LICENSE
PEOPLE V BULI-E
404 SCRA 105
AZCUNA; June 17, 2003
NATURE
Appeal by certiorari
FACTS
- Complainants (8 in number) went to the house of appellant Buli-e in Baguio City upon
learning that she was recruiting workers for overseas employment. Buli-e confirmed that
she was recruiting workers for Taiwan and although she did not have a license of her own,
her boss in Manila (Josefina Alolino) was a licensed recruiter.
- Buli-e told complaints to submit the requirements to her which she in turn will submit to
her boss who was in charge of processing the documents. Complainants complied with
the requirements and paid the downpayment.
- Buli-e brought the complainants to the spouses Alolinos house in Las Pinas to follow up
their applications where they were assured by the spouses that they were licensed to
recruits OCWs and that they can deploy workers within two to three months.
- After months of waiting, complainants were not deployed abroad as promised by
appellants. Hence they went to the POEA in Baguio to find out whether or not appellants
are licensed recruiters. When they learned otherwise, they filed a complaint with the
POEA-CAR and eight separate informations for estafa with the Baguio City Prosecutor.
- During trial, Buli-e testified that she was tasked to find job applicants whom she can refer
to RSI through Josefina. She would be paid for each referral. When Buli-e asked Josefina
had a license, Josefina answered in the affirmative. Buli-e presented, Mrs. Nonette
Villanueva, the unit coordinator of POEA-CAR to testify that RSI was a licensed
employment agency and that Josefina was a licensed recruiter at the time that Buli-e had
dealings with her co-appellant.
- Mrs. Villanueva clarified that licenses of permits to recruit workers are territorial in nature
so that an agency licensed in Manila can only engage in recruitment activities within the
place specified in the license although the applicants may be nonresidents of Metro
Manila.
- Josefina however denied that Buli-e was an agent of Mrs. Fe Go, another Marketing
Director for RSI. Josefina said she accepted referrals from Buli-e even though the latter
was not her agent or connected with RSI because their agency accepts referrals from
everyone.
- The trial court found Buli-e and Josefina guilty of illegal recruitment in a large scale and
estafa. Hence this petition.
ISSUE
1. WON appellants are guilty of illegal recruitment in a large scale
2. WON appellants are guilty of estafa
HELD
1. YES
Ratio The essential elements of the crime of illegal recruitment in large scale are (1) the
accused engages in acts of recruitment and placement of workers defined under Article
13(b) or in any prohibited activities under Article 34 of the Labor Code; (2) the accused
has not complied with the guidelines issued by the Secretary of Labor and Employment,
particularly with respect to the securing of a license or an authority to recruit and deploy
workers, either locally or overseas; and (3) the accused commits the unlawful acts against
three or more persons, individually or as a group. When illegal recruitment is committed in
large scale or when it is committed by a syndicate, it is considered as an offense involving
economic sabotage.
Reasoning
- When complainants approached Buli-e, she gave the impression that she had the ability
to send workers abroad by saying that although she did not have a license, her boss did.
- There is no showing that complainants ever set foot in the RSI office. They were always
brought to the house of the spouses Alolino in Las Pinas.

Labor Law 1
- Josefinas acts clearly show that she and Buli-e acted in concert towards the
accomplishment of a common felonious purpose which was to recruit workers for
overseas employment even though they had no license to do so.
- There is nothing on record to corroborate Josefinas claim that as Marketing Director she
was authorized to solicit applicants for overseas placement through advertisements,
referrals, walk-ins, etc.
- Josefina, despite the suspension and expiration of the RSI license, continued to engage
in recruitment activities for overseas employment.
- Licensed agencies are prohibited from conducting any provincial recruitment, job fairs or
recruitment activities of any form outside of the address stated in the license,
acknowledged branch or extension office, without securing prior authority from the POEA.
2. YES
Ratio It is settled that a person convicted of illegal recruitment under the Labor Code can
also be convicted of violation of the Revised Penal Code provisions on estafa provided
that the elements of the crime are present.
Reasoning
- The elements of estafa are: (1) the accused defrauded another by abuse of confidence
or by means of deceit, and (2) that damage or prejudice capable of pecuniary estimation is
caused to the offended party or third person.
- Appellants deceived complainants into believing that they had the authority and capability
to send them to Taiwan for employment. On the strength of such assurance, complainants
parted with their money in payment of the placement fees.
Disposition AFFIRMED

2.06 ILLEGAL RECRUITMENT


ELEMENTS OF CRIME
PEOPLE V ORDONO
335 SCRA 331
MENDOZA; July 10, 2000
NATURE
Appeal from the decision of RTC
FACTS
- Complainant Presenio Lorena is a resident of Libtong, Tagudin, Ilocos Sur. He only
finished grade school. He earns his living as a farmer. He was introduced to accusedappellant by her relative, Zenaida Ordoo, when they went to his residence. Accusedappellant represented herself as one connected with a recruitment agency and able to
deploy workers abroad, particularly in Korea.
- Attracted by the alleged high salaries in Korea, he was convinced to apply for an
overseas job by the accused-appellant. He paid sums of money and in return accusedappellant secured for Lorena a passport, a two-way plane ticket and some pocket money
in U.S. currency from the money she received from him.
- Accused-appellant informed Lorena that Kuala Lumpur is in Korea and that, upon arrival
there, he would be fetched by a certain Joy Mejia from the Metro Hotel, where he was told
to check in. On January 23, 1993, Lorena took a Philippine Airlines flight to Kuala Lumpur,
Malaysia. As instructed, he stayed at the Metro Hotel for several days, but no one came to
fetch him. Lorena went to the Philippine Embassy for assistance, but he was asked to give
U.S. $500.00 for the processing of his work permit. As he did not have enough money, he
came back to the Philippines on February 4, 1993. He went to see accused-appellant to
confront her, but she told him that if he wanted to try his luck again in finding employment
abroad, he must recruit two more persons so that the placement fees they would pay
would be used for his second job application.
- The other complainant, Jerry Lozano, is also a resident of Libtong, Tagudin, Ilocos Sur.
Like Lorena, he is also a farmer. He never reached high school. Sometime in December
1993, he was called to Lorenas house upon accused-appellants request. There, Lorena
introduced accused-appellant to him. Accused-appellant introduced herself to Lozano as a
recruiter of overseas workers for Korea. Lozano was convinced to apply in the hope of
landing a good job in Korea. Lozano paid her the total amount of P41,000.00 in two
installments. His passport, two-way ticket and pocket money in U.S. dollars were obtained
from the money he had paid to accused-appellant. He arrived in Kuala Lumpur, Malaysia,
not in Korea as he was promised. There, he was apprehended by the Malaysian police at
the airport after finding that he had no other travel documents with him except his
passport. He was kept in a flooded jail for one night. The next day, he was deported to the
Philippines. After arriving home, he saw accused-appellant and confronted her. As in
Lorenas case, he was told to recruit two persons so that the money they would pay for
their placement fees would be used by him for another overseas employment application.

A2010

- 110 -

Disini

- Criminal cases were filed (illegal recruitment and estafa). The trial court found accused
guilty beyond reasonable doubt and imposed the death penalty for the illegal recruitment
cases.
ISSUE
WON accused is guilty of illegal recruitment
HELD
YES
- Illegal recruitment is committed when two elements concur, namely: (1) the offender has
no valid license or authority required by law to enable one to lawfully engage in
recruitment and placement of workers; and (2) he undertakes either any activity within the
meaning of recruitment and placement defined under Art. 13(b), or any of the prohibited
practices enumerated under Art. 34 of the Labor Code. Art. 13(b) of the Labor Code
defines recruitment and placement as any act of canvassing, enlisting, contracting,
transporting, utilizing, hiring, or procuring workers, and includes referrals, contract
services, promising, or advertising for employment, locally or abroad, whether for profit or
not; Provided that any person or entity which in, any manner, offers or promises for a fee
employment to two or more persons, is considered engaged in recruitment and
placement.
- In these cases, these elements concur. The certification issued by the DOLE Regional
Office in La Union that at the time material to these cases the accused-appellant had no
authority to engage in recruitment activities is unrebutted. The acts of accused-appellant,
consisting of her promise of employment to the complainants and of transporting them
abroad, fall squarely within the ambit of recruitment and placement as defined by law.
- Another indication that accused-appellant recruited complainants was the fact that
Presenio Lorenas contact in Kuala Lumpur was Joy Mejia, who is a close friend of
accused-appellant. Indeed, accused-appellants claim that she did not represent herself as
a licensed recruiter but that she merely tried to help complainants secure tourist visas
could not make her less guilty of illegal recruitment, it being enough that she gave the
impression of having had the authority to recruit workers for deployment abroad.
- Accused-appellant is clearly guilty of illegal recruitment. But accused-appellant is correct
that she cannot be sentenced to suffer life imprisonment and pay a fine of P100,000.00 for
each count of illegal recruitment. This is the penalty for illegal recruitment committed either
by a syndicate or in large scale, but this has not been shown in this case. The informations
do not allege the offense of illegal recruitment committed in large scale or by a syndicate
but only of illegal recruitment. Nor does the evidence show that the illegal recruitment was
committed in large scale or by a syndicate. Hence, the applicable penalty is that provided
under Art. 39(c) of the Labor Code which states:
Any person who is neither a licensee nor a holder of authority under this Title found
violating any provision thereof or its implementing rules and regulations shall, upon
conviction thereof, suffer the penalty of imprisonment of not less than four years nor
more than eight years or a fine of not less than P20,000 nor more than P100,000 or
both such imprisonment and fine, at the discretion of the court.
Disposition Decision appealed from is affirmed with modification as to sentence

PEOPLE V SAGAYAGA
423 SCRA 468
CALLEJO SR; February 23, 2004
NATURE
This is an appeal from the Decision of the Regional Trial Court of Manila, Branch 35,
convicting the appellant Leticia Sagayaga of large scale illegal recruitment as defined in
Section 6, Republic Act No. 8042 and sentencing her to suffer life imprisonment.
FACTS
- The appellant was charged with large scale illegal recruitment in an Information.
Sometime in the last week of October 1997, Elmer Janer went to the office of Alvis
Placement Service Corporation to apply for overseas employment as factory worker in
Taiwan. Appellant Leticia Sagayaga, after personally receiving Elmers application,
required him to submit the necessary documents. Appellant further asked Elmer to pay
seventy-five thousand pesos (P75,000.00) as placement fee. Elmer paid the said fee to
appellant in three (3) installments, the first, on November 5, 1997, in the amount of twentyfive thousand pesos (P25,000.00); the second, on November 13, 1997, in the amount of
five thousand pesos (P5,000.00); and the third, on November 19, 1997, in the amount of
forty-five thousand pesos (P45,000.00). All the payments were made inside Alvis
Placement Agency.
- As required, Elmer also had his medical examination at the Angeles Medical Clinic, the
result of which confirmed that he was fit to work. Thereafter, he was told to wait for the
arrival of the employer. After seven (7) months, no employer arrived. Tired of waiting,
Elmer demanded that he be refunded of his money (Id.). Despite appellants promises to
pay, Elmer was not refunded of his money.
- Exasperated, Elmer asked appellant for a promissory note, which appellant executed,
promising to pay Elmer seventy-five thousand (P75,000.00) on May 6, 1998 (pp. 10 and
11, TSN, September 7, 1999). In said promissory note, appellant designated herself as

Labor Law 1
the assistant general manager of the placement agency. When appellant failed to
refund the amount to Elmer on the date stated in the promissory note, the latter went to
the Philippine Overseas Employment Administration (POEA) and filed a sworn complaint
against appellant. Elmer Ramos and Eric Farol filed their complaint based on basically
similar facts.
- Appellants Contention:
> The appellant avers that she is not criminally liable for the crime charged because the
prosecution failed to prove that she had a direct or actual control, management or
direction of the business and recruitment activities of the Alvis Placement Services
Corporation (APSC). She asserts that she had no knowledge of the recruitment activities
of APSC and had no participation whatsoever in its operation. In dealing with the private
complainants, she was merely performing routinary office work as a mere employee. Her
participation as an employee of APSC with respect to the employment application of Elmer
Ramos for Taiwan was to receive his placement fee of P20,000.00. Hence, the appellant
avers, she cannot be held criminally liable for illegal recruitment in large scale. If, at all,
she can be held liable only with respect to the employment applications of Janer and
Farol. Thus, according to the appellant, the trial court erred in sentencing her to life
imprisonment.
ISSUE
WON the accused-appellant was a top ranking officer of said corporation, with authority to
participate directly in the control, management or direction of its business affairs
HELD
YES
Ratio Recruitment is any act of canvassing, enlisting, contracting, transporting, utilizing,
hiring or procuring workers, and includes referrals, contract services, promising or
advertising for employment, locally or abroad, whether for profit or not: Provided, That any
person or entity which, in any manner, offers or promises for a fee employment to two or
more persons shall be deemed engaged in recruitment and placement.
Reasoning
- The appellant, as shown by the records of the POEA, was both the APSC VicePresident-Treasurer and the Assistant General Manager. She was a high corporate officer
who had direct participation in the management, administration, direction and control of
the business of the corporation.
- Under Section 6 (m) of Rep. Act No. 8042, illegal recruitment may be committed by any
person, whether a non-licensee, non-holder of authority, licensee or holder of authority,
thus:
(m) Failure to reimburse expenses incurred by the worker in connection with his
documentation and processing for purposes of deployment, in cases where the
deployment does not actually take place without the workers fault....
- Under the last paragraph of the said section, those criminally liable are the principals,
accomplices and accessories. In case of a juridical person, the officers having control,
management or direction of the business shall be criminally liable.
- At any rate, the accused has expressly admitted in the course of her testimony that she
was at the time the Treasurer of their recruitment agency. As such she was in charge of
the management and control of the financial affairs and resources of the corporation. She
was in charge of collecting all its receivables, safely keeping them, and disbursing them.
She testified that it was part of her duties to receive and collect the monies paid by
applicants. Her disbursing authority has been clearly demonstrated by her co-signing the
checks Exhibits D-2 and G.
- The appellant is guilty of illegal recruitment as a principal by direct participation, having
dealt directly with the private complainants.
- The appellants bare denial of her involvement in the management, administration,
control and operation of APSC cannot prevail over her judicial admissions, the positive
testimonies of the private complainants and the documentary evidence adduced by the
prosecution.
- Section 6 of Rep. Act No. 8042 provides that illegal recruitment shall be considered an
offense involving economic sabotage if committed in large scale, viz, committed against
three (3) or more persons individually or as a group, the imposable penalty for which is life
imprisonment and a fine of not less than P500,000.00 nor more than P1,000,000.00. In
this case, there are three private complainants, namely, Elmer Janer, Eric Farol and Elmer
Ramos. The trial court, thus, correctly convicted the appellant of large scale illegal
recruitment and sentenced her to suffer life imprisonment.
Under Section 6 (m) of Rep. Act No. 8042, illegal recruitment may be committed by any
person, whether a non-licensee, non-holder of authority, licensee or holder of authority,
thus:
(m) Failure to reimburse expenses incurred by the worker in connection with his
documentation and processing for purposes of deployment, in cases where the
deployment does not actually take place without the workers fault....
- Under the last paragraph of the said section, those criminally liable are the principals,
accomplices and accessories. In case of a juridical person, the officers having control,
management or direction of the business shall be criminally liable.

A2010

- 111 -

Disini

PEOPLE V BAYTIC
398 SCRA 18
CARPIO-MORALES; February 20, 2003
NATURE
Appeal from the decision of the Regional Trial Court of Quezon City ffinding the accused
guilty of illegal recruitment in large scale
FACTS
- On 24 September 1998 Kennedy Hapones accompanied by accused Alex Baytic went to
the house of his aunt Ofelia Bongbonga at 514 Tabigo Street, Manggahan, Quezon City.
There they found Ofelia, Nolie and Zenaida, all surnamed Bongbonga, together with Millie
Passi, Yolanda Barrios and Elvira Nacario. Accused Alex Baytic told the girls that he was
looking for workers willing to work in Italy as utility personnel. He explained that interested
applicants should give him money for processing of their medical certificate, certificate of
employment and other travel documents. Since the offer appeared to be a good
opportunity to work abroad, Ofelia Bongbonga on the same day gave the accused
P3,500.00, followed by Millie Passi with P4,000.00 the next day, and Nolie Bongbonga
with P4,000.00 on 5 October as their placement fees. All these transactions were
evidenced by receipts issued by accused Alex Baytic.
- According to private complainant Ofelia Bongbonga, accused Baytic promised her and
her two (2) co-applicants an interview by his cousin, a doctor from Italy, on 7 October 1998
at the Corinthian Gardens. However, on the appointed date of their interview, the accused
failed to appear. Ofelia, Millie and Nolie frantically searched for him but he was nowhere to
be found. Ofelia further testified that sometime in January 1999 they heard over the radio
that accused Baytic was arrested in Pasig City for illegal recruitment activities. Upon
inquiry from the radio station, she learned that the accused was already detained at the
Pasig Provincial Jail, so she followed him there. Thereafter, she and her two (2) other
companions, Millie and Nolie, who were likewise victimized by the accused filed the instant
case against him.
- Accused Alex Baytic, testifying in his defense, not only denied the accusations against
him but also insisted that it was actually Kennedy Hapones, a new acquaintance, who was
the illegal recruiter. He recounted that sometime in November 1999, he went to the house
of Hapones who was trying to recruit him for deployment abroad. According to the
accused, Hapones told him to prepare P250,000.00 although the former eventually
accepted an initial advance payment of P4,500.00. He again met Hapones the following
month when the latter told him and a group of other applicants, including Ofelia
Bongbonga, that their requirements were ready. That was the last time he saw Hapones
who, he later learned, had already left for abroad. He was suspected of being in cahoots
with Hapones because whenever the latter and the applicants talked, Hapones would
always point at him, although he never had the opportunity to know what Hapones had
actually said to them.
- But the trial court sustained the complaining witnesses and gave more credence to their
straightforward and consistent testimonies. It opined that all the essential requisites of the
crime of illegal recruitment in large scale as defined in Art. 13, par. (b), of the Labor Code
were present - The accused made representations to each of the complainants that he
could send them to Italy as janitor/utility aides through direct hiring, which constitutes a
promise of employment which amounted to recruitment as defined under Article 13(b) of
the Labor Code. The testimonies of the three prosecution witnesses that they were
actually recruited for overseas employment by the accused and were induced by him to
part with their money retain undiminished probative worth and weight. The receipts
respectively issued to the complainants are sufficient proofs of his guilt as against
accuseds mere denial of the signatures appearing therein. The modus operandi of the
accused was well established by the corroborative testimonies of the witnesses.
- Accused-appellant now prays that the Court to take a second hard look at his conviction
in view of the alleged failure of the prosecution to prove his guilt beyond reasonable doubt.
He takes exception to the finding of the trial court that all the elements of the crime of
illegal recruitment in large scale are present. He argues that the first element, i.e., the
accused engages in the recruitment and placement of workers, defined under Art. 13, or in
any prohibited activities under Art. 34, of the Labor Code, is not present because he did
not solicit any money from the complainants nor did he promise them employment in Italy.
The truth of the matter, according to him, is that he himself was victimized by Kennedy
Hapones, the real illegal recruiter. He explained that when Hapones could not be
contacted, the complainants vented their anger towards him, being Hapones constant
companion.
Such being the case, accused-appellant insists that the second element, which is the
absence of license or authority to recruit, could not have been present because there was
in fact no need for him to apply for the license as he was not in the recruitment business.
ISSUE
WON the accused is guilty of illegal recruitment

Labor Law 1
HELD
YES
- The appealed decision finding accused-appellant guilty of illegal recruitment in large
scale is affirmed
Ratio Illegal recruitment is committed when two (2) elements concur. First, the offender
has no valid license or authority required by law to enable one to engage lawfully in
recruitment and placement of workers. Second, he or she undertakes either any activity
within the meaning of "recruitment and placement" defined under Art. 13, par. (b), or any
prohibited practices enumerated under Art. 34 of the Labor Code.1awphi1.nt In case of
illegal recruitment in large scale, a third element is added: that the accused commits the
acts against three or more persons, individually or as a group.
Reasoning
- The first element is present. POEA representative Flordeliza Cabusao presented in
evidence a certification from one Hermogenes Mateo, Director III, Licensing Branch,
showing that accused-appellant was neither licensed nor authorized to recruit workers for
overseas employment. The second element is likewise present. Accused-appellant is
deemed engaged in recruitment and placement under Art. 13, par. (b), of the Labor Code
when he made representations to each of the complainants that he could send them to
Italy for employment as utility personnel. Prosecution witness Ofelia Bongbonga
categorically stated that accused-appellant promised her employment for a fee, a
testimony corroborated by both complaining witnesses Nolie Bongbonga and Millie Passi.
His promises and misrepresentations gave the complainants the distinct impression that
he had the authority to engage in recruitment, thus enabling him to collect from them
various amounts for recruitment and placement fees without license or authority to do so.
Accused-appellants vain attempt at exculpating himself by pinpointing Hapones as the
culprit cannot mislead this Court from his transparent and obvious machinations. His selfserving statement that he himself was a victim of Hapones wilts in the face of the
complaining-witnesses testimonies that he made promises of employment, solicited
money from them and even signed receipts as proof of payment.

LARGE SCALE
PEOPLE V BULI-E
[PAGE 110]
PEOPLE V DUJUA
422 SCRA 169
TINGA; February 5, 2004
FACTS
- Ramon Dujua, his mother Rose Dujua, his aunt Editha Singh, and his uncle were
charged with Illegal Recruitment in Large Scale.
- The prosecution presented a Certification issued by Hermogenes C. Mateo, Director II,
Licensing Branch of the POEA, stating that Ramon Dujua is not licensed or authorized by
the POEA to recruit workers abroad. Another Certification, of even date shows that neither
is the World Pack Travel and Tours authorized to recruit workers abroad.
- The RTC rendered its decision convicting Ramon Dujua of illegal recruitment in large
scale.
ISSUE
WON Ramon Dujua committed the crime of illegal recruitment in large scale
HELD
YES
- The essential elements of the crime of illegal recruitment in large scale are:
(1) the accused engages in acts of recruitment and placement of workers defined
under Article 13(b) or in any prohibited activities under Art. 34 of the Labor Code;
(2) the accused has not complied with the guidelines issued by the Secretary of
Labor and Employment, particularly with respect to the securing of a license or an
authority to recruit and deploy workers, either locally or overseas; and
(3) the accused commits the unlawful acts against three or more persons,
individually or as a group.
- All three elements have been established beyond reasonable doubt.
- First, the testimonies of the complaining witnesses satisfactorily prove that appellant
promised them employment and assured them placement overseas. Complainants were
firm and categorical. All of them positively identified appellant as the person who recruited
them for employment abroad. There is no adequate showing that any of them was
impelled by any ill motive to testify against appellant. The absence of receipts in a case
for illegal recruitment does not warrant the acquittal of the appellant and is not fatal to the
prosecutions case. As long as the prosecution is able to establish through credible
testimonial evidence that the appellant has engaged in Illegal Recruitment, a conviction for

A2010

- 112 -

Disini

the offense can very well be justified.


- Second, appellant did not have any license or authority to recruit persons for overseas
work, as shown by the Certification issued by the POEA. Neither did his employer, the
World Pack Travel and Tours, possess such license or authority.
- Third, it bears clarifying that although Romulo Portos was named as among those
recruited by appellant the evidence reveals that Romulo withdrew his application in lieu of
which his wife Melodea Villanueva applied for placement with appellant. Villanueva,
however, is not named as one of appellants victims.
- Nevertheless, it has been alleged and proven that appellant undertook the recruitment of
not less than three persons, namely, Cabus, Caluten and Perlas.
Disposition The decision was affirmed with modifications.

PEOPLE V REYES
242 SCRA 264
MENDOZA; March 9, 1995
NATURE
Appeal from decision of RTC Laguna finding accused Thelma Reyes guilty beyond
reasonable doubt of Illegal Recruitment defined and penalized under Article 38, P.D. No.
442 as amended, and sentencing her to suffer a penalty of Reclusion Perpetua and to pay
a fine of P100,000.00 and to indemnify Rosalino Bitang and Fabian Baradas, Mr. de
Castro, Lorenzo Blanza and Ramon Mendoza the sum of P45,000 and to pay the costs.
FACTS
- Appellant Thelma Reyes was charged together with her husband Nick Reyes, but the
latter was at large and so has remained up to now. The trial proceeded only with respect
to Thelma Reyes in view of her plea of not guilty.
- Evidence show that sometime in 1985, the five victims individually went to the house of
appellant in Los Baos, Laguna, to apply for employment abroad. They were required to
submit travel documents (passports, birth certificates, NBI clearances, etc). They paid
various amounts of money to Nick Reyes for recruitment fees, placement fees, etc. Nick
Reyes handed the money to his wife Thelma Reyes, and afterward issued a receipt.
- Nick Reyes allegedly promised to notify them as soon as they were accepted for
employment so that they could leave for abroad, but this promise was not fulfilled. When
they later checked with the Philippine Overseas Employment Administration (POEA), they
found out that the spouses were not licensed recruiters.
- As soon as they obtained the POEA certification that appellant and her husband were not
licensed to recruit, they demanded from the spouses the return of their money and when
the latter did not give back their money, they filed the complaint in this case.
- Thelma Reyes defense is that of denial. But on cross-examination she admitted that
there were fourteen other cases of Illegal Recruitment filed and/or pending against her and
her husband in different courts of Manila and claimed that some of the cases had been
dismissed or settled after she had refunded the money of the complainants.
- The lower court found Thelam Reyes guilty and sentenced her as stated above. Hence
this appeal.
ISSUES
1. WON the evidence for the prosecution is sufficient to warrant conviction
2. WON the verbal testimonies of private complainants should be given more credit than
the documentary evidence for the accused
3. WON two complainants will suffice for conviction of illegal recruitment in large scale
HELD
1. YES
- An accused can be convicted on the strength of the testimony of a single witness, if such
testimony is credible and positive and produces a conviction beyond reasonable doubt.
That the witness is also the complainant in a case makes little difference so long as the
court is convinced beyond doubt that the witness is telling the truth.
-In determining the value and credibility of evidence, witnesses are to be weighed, not
counted (People v. Villalobos, 209 SCRA 304 [1992]).
- When the issue is the credibility of witnesses, appellate courts will in general not disturb
the findings of the trial court unless certain facts or circumstances of weight have been
overlooked, misunderstood or misapplied which, if considered, might affect the result of
the case.
2. YES
- Evidence shows that after receiving the amounts from complainants, Nick Reyes handed
the money paid to the appellant and that Nick Reyes issued the receipts in question only
after appellant Thelma Reyes had counted it. Mere claim is not enough to overcome the
evidence of the prosecution. If there was anyone whose testimony needed corroboration it
was appellant.
- The fact that the purpose for which payment was made is not stated in the receipts nor
why the receipts purport to have been issued in "Manila" and not in Los Baos where they
said they had made all the payments is something to be explained not by the

Labor Law 1
complainants who have an elementary education only, but by appellant and her husband
because it was the latter who made the receipts. It is not far fetched that they made the
receipts this way precisely to create doubt as to their real import.
3. NO
- Labor Code provisions:
Art. 38 (b): Illegal recruitment when committed by a syndicate or in large scale shall be
considered an offense involving economic sabotage and shall be penalized in
accordance with Article 39 hereof.
Art. 39(a): The penalty of life imprisonment and a fine of One Hundred Thousand
Pesos (P100,000) shall be imposed of illegal recruitment constitutes economic
sabotage as defined herein.
- The information against appellant mentioned only the two complainants Fabian Baradas
and Rosalino Bitang as having been illegally recruited by appellant and her husband. The
trial court, however, held appellant guilty of illegal recruitment on a large scale because
aside from Baradas and Bitang, appellant and her husband allegedly recruited others,
namely, Lorenzo Blanza, Edgardo Garcia, Ramon Mendoza, and Dionisio de Castro.
- When the Labor Code speaks of illegal recruitment "committed against three (3) or more
persons individually or as a group," it must be understood as referring to the number of
complainants in each case who are complainants therein, otherwise, prosecutions for
single crimes of illegal recruitment can be cumulated to make out a case of large scale
illegal recruitment. In other words, a conviction for large scale illegal recruitment must
be based on a finding in each case of illegal recruitment of three or more persons
whether individually or as a group.
- Even if Blanza and Garcia had been illegally recruited so as to make the number of
persons illegally recruited four and make the crime that of illegal recruitment of a large
scale, this can not be the basis of conviction since this was not alleged in the information.
- Appellant must be punished under Art. 39(c) of the Labor Code: Any person who is
neither a licensee nor a holder of authority under this Title found violating any provision
thereof or its implementing rules and regulations shall upon conviction thereof, suffer the
penalty of imprisonment of not less than four years nor more than eight years or a fine of
not less than P20,000 nor more than P100,000 or both such imprisonment and fine, at the
discretion of the court.
Disposition RTC decision is SET ASIDE. Appellant Thelma Reyes is declared guilty of
illegal recruitment on two (2) counts and is sentenced for each crime to suffer
imprisonment of 6 years and 1 day to 8 years and pay a fine of P50,000.00; and ordered
to indemnify Rosalino Bitang in the amount of P13,500.00 and Fabian Baradas in the
amount of P18,000.00 and pay the costs.

REFERRALS
PEOPLE V MERIS
329 SCRA 33
KAPUNAN; March 28, 2000
NATURE
This is an appeal from the Joint Decision of the Regional Trial Court of Manila, Branch 1,
convicting accused-appellant Leonida Meris of illegal recruitment in large-scale and six
counts of estafa.
FACTS
- During the period comprised between January 12, 1991 and February 17, 1991, accused
conspiring and confederating with three others defrauded NAPOLEON RAMOS, by means
of false manifestations and fraudulent representation which they made to Napoleon
Ramos to the effect that they had the power and capacity to recruit and employ him as
Factory Worker in Hongkong and could facilitate the processing of the pertinent papers if
given the necessary amount to meet the requirements.
- They succeeded in inducing said Ramos to give and deliver the amount of P30,000.00
on the strength of said manifestations and representations. The accused obtained P30K,
with intent to defraud, and unlawfully misappropriated the money for their personal use.
- The seventh information in Criminal Case No. 91-94198 charged accused-appellant with
illegal recruitment in large-scale. Meris for a fee recruited and promised employment
abroad to Leo D. delos Santos, Merlita L. Bombarda, Margarita R. madae (sic), Purita A.
Conceja, Cristina I. Nava and Napoleon E. Ramos, without first securing the required
license or authority from the Dept. of Labor.
- Testifying in her own defense, accused-appellant denied the charges of engaging in
recruitment activities and of receiving money from complainants. She described herself as
a public school teacher living in Pangasinan with her four children and unemployed
husband. Like the other complainants, she claimed she was a victim of Julie Micua. When
complainants learned that she had applied for overseas employment, they sought her help
in going to the agency where she applied. Hence, on January 12, 1991, accused-appellant
accompanied the complainants to see Julie Micua who assured them that they would be
leaving for Hongkong within two or three months. Accused-appellant claims she never

A2010

- 113 -

Disini

represented herself as having the capacity to deploy workers abroad. She only told them
that she could accompany them to the agency where she also applied.
ISSUE
WON Meris is guilty of the charges of estafa and illegal recruitment
HELD
YES, she is guilty of estafa and illegal-recruitment (wide-scale).
Reasoning
- All the complainants are one in saying that accused-appellant made representations that
she knew someone who could help them secure employment in Hongkong. Relying on
these representations, they applied for placement for employment abroad and paid
various sums of money therefor. Unfortunately, accused-appellant failed to comply with
her promise of employment or restitute the amounts she received from them.
- The prosecution undoubtedly proved that accused-appellant, without license or authority,
engaged in recruitment and placement activities. This was done in collaboration with Julie
Micua, when they promised complainants employment in Hongkong. Art. 13, par. (b) of the
Labor Code defines recruitment and placement as "any act of canvassing enlisting,
contracting, transporting, utilizing, hiring or procuring workers, and includes referrals,
contract services, promising or advertising for employment, locally or abroad, whether for
profit or not; Provided that any person or entity which, in any manner, offers or promises
for a fee employment to two or more persons shall be deemed engaged in recruitment and
placement."
- Although accused-appellant was not an employee of the alleged illegal recruiter Julie
Micua, the evidence show that she was the one who approached complainants and
prodded them to seek employment abroad. It was through her that they met Julia Micua.
This is clearly an act of referral.
- Illegal recruitment is conducted in a large scale if perpetrated against three (3) or more
persons individually or as a group. This crime requires proof that the accused: (1)
engaged in the recruitment and placement of workers defined under Article 13 or in any of
the prohibited activities under Article 34 of the Labor Code; (2) does not have a license or
authority to lawfully engage in the recruitment and placement of workers; and (3)
committed the infraction against three or more persons, individually or as a group. All
these three essential elements are present in the case.
Disposition Decision in question is affirmed.

PEOPLE V FORTUNA
395 SCRA 354
VITUG; January 16, 2003
NATURE
Appeal from decision of RTC finding petitioner guilty of Illegal Recruitment in Large Scale
FACTS
- Private complainants met Dominga Fortuna y Corrales in a seminar on "Tupperware"
products being then promoted for sale in Cabanatuan City. Fortuna offered the
complainants job placements in Taiwan. Convinced that Fortuna could actually provide
them with jobs abroad, private complainants each gave her the amount of P5,400.00 to
take care of the processing fee for medical examination and other expenses for securing
their respective passports. Private complainants took the medical examination in Manila.
Weeks went by but the promised departure had not materialized. Suspecting that
something was not right, they finally demanded that Fortuna return their money. Fortuna,
in the meanwhile, went "into hiding." After having later learned that Fortuna had neither a
license nor an authority to undertake recruiting activities, Angelyn Magpayo filed a
complaint which, in due time, ultimately resulted in the indictment of Fortuna for illegal
recruitment. During the preliminary investigation, as well as later at the trial, Fortuna gave
assurance to have the money she had received from private complainants returned to
them but Fortuna was unable to make good her promise.
ISSUE
WON the lower court erred in finding petitioner guilty of Illegal Recruitment in Large Scale
HELD
NO
Ratio The rule has been said that a person charged with illegal recruitment may be
convicted on the strength of the testimony of the complainants, if found to be credible and
convincing, and that the absence of receipts to evidence payment to the recruiter would
not warrant an acquittal.
Reasoning
- The requisites constituting the offense of Illegal Recruitment in Large Scale have
sufficiently been proven by the prosecution. First, appellant, undeniably, has not been duly
licensed to engage in recruitment activities; second, she has engaged in illegal recruitment
activities, offering private complainants employment abroad for a fee; and third, she has

Labor Law 1
committed the questioned illegal recruitment activities against three or more persons. The
pertinent provisions of RA 8042 state:
SEC. 6. Definition. illegal recruitment shall mean any act of canvassing, enlisting,
contracting, transporting, utilizing, hiring, or procuring workers and includes referring,
contract of services, promising or advertising for employment abroad when
undertaken by a non-license or non-holder of authority contemplated
xxx
xxx
x x x.
Illegal recruitment is deemed committed in large scale if committed against three (3)
or more persons individually or as a group.
Disposition the appealed decision of the RTC is AFFIRMED

EMPLOYEES
PEOPLE V CORPUZ
412 SCRA 479
YNARES-SANTIAGO; October 1, 2003
NATURE
Appeal from the decision of the RTC of Manila
FACTS
- Private complainants Belinda Cabantog, Concepcion San Diego, Erlinda Pascual and
Restian Surio went to Alga-Moher International Placement Services Corporation to apply
as factory workers in Taiwan. There they met Mrs. Reyes, the president of the said
agency. They were asked to fill up application forms and return to the office to pay
P10,000 as processing fee
- They returned after a month. Mrs. Reyes was not at the office at that time but she called
appellant Corpuz to receive the fees.
- 2 months later, nothing happened to their applications. They asked Corpuz to refund the
money. She said it was already remitted to Mrs. Reyes. Complainants approached Mrs.
Reyes but she told them that the money she received from Corpuz was in payment of the
latters debt.
- The complainants filed a complaint and Corpuz was arrested and detained. While the
case was on the RTC, the complainants received a refund of their money from
complainants sister-in-law. They then executed affidavits of desistance.
- Appelant contends she did not have a part with the illegal recruitment as she merely
received the money on behalf of Mrs. Reyes, whome she was working as a secretary for
about 3 months.
- RTC found Corupuz guilty for violation of RA 8042.
ISSUE
WON Corpuz can be held liable under RA 8042
HELD
NO
- Appellant contends that she is not liable for the foregoing illegal recruitment activities
considering that she was merely an employee having no control over the recruitment
business of the Alga-Moher International Placement Services Corporation and that she did
not actually recruit the private complainants. Moreover, she did not appropriate for her
own use the processing fees she received and she had no knowledge that the agencys
license was suspended by the POEA. RTC convicted her because despite the suspension
of the license since she was able to convince the complainants to give her their money.
- From the foregoing testimony, it is clear that all appellant did was receive the processing
fees upon instruction of Mrs. Reyes. She neither convinced the private complainants to
give their money nor promised them employment abroad
- As stated in the last sentence of Section 6 of RA 8042, the persons who may be held
liable for illegal recruitment are the principals, accomplices and accessories. In case of
juridical persons, the officers having control, management or direction of their business
shall be liable.
- An employee of a company or corporation engaged in illegal recruitment may be held
liable as principal, together with his employer, if it is shown that he actively and
consciously participated in illegal recruitment. However, where it is shown that the
employee was merely acting under the direction of his superiors and was unaware that his
acts constituted a crime, he may not be held criminally liable for an act done for and in
behalf of his employer.
- In the case at bar, the prosecution failed to adduce sufficient evidence to prove
appellants active participation in the illegal recruitment activities of the agency. As already
established, appellant received the processing fees of the private complainants for and in
behalf of Mrs. Reyes who ordered her to receive the same. She neither gave an
impression that she had the ability to deploy them abroad nor convinced them to part with
their money. More importantly, she had no knowledge that the license was suspended the
day before she received the money. Their failure to depart for Taiwan was due to the

A2010

- 114 -

Disini

suspension of the license, an event which appellant did not have control of. Her failure to
refund their money immediately upon their demand was because the money had been
remitted to Mrs. Reyes on the same day she received it from them.
Disposition Decision reversed

PEOPLE V SAGAYAGA
[PAGE 111]
PEOPLE V GUTIERREZ
422 SCRA 32
TlNGA; February 5, 2004
NATURE
Appeal from the decision of the trial court
FACTS
- FLOR GUTIERREZ Y TIMOD with CECILIA BAUTISTA, ESTHER GAMILDE, LINDA
RABAINO and MARILYN GARCIA mutually helped one another in recruiting for overseas
job placement and actual contract EVELYN V. RAMOS, ROSEMARIE I. TUGADE,
GENEROSA G. ASUNCION and ROSALYN B. SUMAYO as domestic helpers in Dubai,
United Arab Emirates, for a fee ranging from P10,000.00 to P15,000.00 each, without first
obtaining the required license and/or authority from the Philippine Overseas Employment
Administration (POEA).
- In her defense, the accused claimed that as an employee of a duly licensed agency
who was tasked to recruit and offer job placements abroad, she could not be held liable for
illegal recruitment. She admitted that she had no authority to recruit in her personal
capacity, but that her authority emanated from a Special Power of Attorney (SPA) and a
Certification issued by a licensed agency.
- At the time complainants applied for overseas employment, the accused was employed
as a Marketing Directress of Sarifudin Manpower and General Services, a duly licensed
agency. A Special Power of Attorney (SPA) from Sarifudin, dated May 1, 1994. A
Certification dated February 3, 1995, issued by the same agency, also states that: MRS.
FLOR T. GUTIERREZ was (sic) employed as OVERSEAS MARKETING DIRECTRESS of
SARIFUDIN MANPOWER AND GENERAL SERVICES, effective May 1994, up to the
present
- But Edwin Cristobal, POEA Labor Employment Officer, found that the said agency
revoked the appointment of Ms. Flor Gutierrez as Overseas Mktg. Director/Manager in a
letter dated Dec. 15, 1995, although POEA has not received nor acknowledged the
representation of Ms. Gutierrez. Cristobal explained that the POEA, Never had a letter
from Sarifudin registering or authorizing Flor Gutierrez... rather, [what] we received [was a]
revocation of her appointment. He also revealed that the name of the accused does not
appear in the records of the POEA as being employed by the agency from the assumption
of its license on June 11, 1993, up to its termination on June 11, 1995.
- Defense likewise alleged that complainants Rosemarie Tugade and Evelyn Ramos
executed Affidavits of Desistance dated May 12, 1995, stating that the accused had
returned to them the amounts they paid her and that the complaint was a result of a
misunderstanding.
- Trial court rendered its Decision finding the accused guilty beyond reasonable doubt of
Illegal Recruitment in Large Scale. Accused Flor Gutierrez filed an appeal seeking the
reversal of her conviction.
ISSUE
WON Gutierres as a representative of a duly licensed recruitment agency, she cannot be
held guilty of Illegal Recruitment in Large Scale
HELD
NO
- Appellant cannot escape liability by claiming that she was not aware that before working
for her employer in the recruitment agency, she should first be registered with the POEA.
Illegal recruitment in large scale is malum prohibitum, not malum in se. Good faith is not a
defense.
- That appellant engaged in recruitment and placement is beyond dispute. The
complaining witnesses categorically testified that the accused promised them on several
occasions that they would be leaving for work abroad. Appellant received complainants
money and documents, a fact that the complainants themselves witnessed and which the
accused acknowledged when she returned the same to them after the filing of the case
against her. Appellant even brought complainant Rosalyn Sumayo to the airport three
times, raising her expectations, but leaving her hanging in mid-air. The accused even had
the audacity to demand cancellation fees from the complainants when they asked for a
refund. Moreover, the Affidavits of Desistance executed by two of the complainants
deserve little weight. The Court attaches no persuasive value to affidavits of desistance,
especially when executed as an afterthought.

Labor Law 1
- Section 11, Rule II, Book II of the Rules and Regulations Governing Overseas
Employment requires the prior approval of the POEA of the appointment of
representatives or agents:
Section 11. Appointment of Representatives. Every appointment of representatives or
agents of licensed agency shall be subject to prior approval or authority of the
Administration.
The approval may be issued upon submission of or compliance with the following
requirements:
a.
Proposed appointment or Special Power of Attorney;
b.
Clearances of the proposed representative or agent from NBI;
c.
A sworn or verified statement by the designating or appointing person or
company assuming full responsibility for all the acts of the agent or representative done
in connection with the recruitment and placement of workers.
Approval by the Administration of the appointment or designation does not authorize
the agent or representative to establish a branch or extension office of the licensed
agency represented.
Any revocation or amendment in the appointment should be communicated to the
administration. Otherwise, the designation or appointment shall be deemed as not
revoked or amended.
Section 1, Rule X of the same Book, in turn, provides that recruitment and placement
activities of agents or representatives appointed by a licensee, whose appointments
were not authorized by the Administration shall likewise constitute illegal recruitment.
- The Certification from the POEA that it has not received nor acknowledged the
representation of Ms. Gutierrez establishes that the appointment of appellant by
Serafudin as a representative or agent was not authorized by the POEA. It may be true
that the POEA received from Serafudin a revocation of appellants appointment, but still is
of no consequence since Serafudin in the first place did not submit her appointment to the
POEA, and so the POEA has nothing to approve.
- As found by the trial court the evidence on record, notably appellants own version,
indicates that she was running her own labor recruitment business.
Disposition Decision of the Regional Trial Court, finding appellant Flor Gutierrez y Timod
guilty beyond reasonable doubt of the crime of Illegal Recruitment in Large Scale and
sentencing her to life imprisonment and to pay a fine of P100,000.00 is AFFIRMED.

2.07 ENFORCEMENT AND SANCTIONS


EXECUTIVE SECRETARY V CA
[PAGE 20]

2.08 LIABILITY OF AGENCY


G & M (PHIL) INC V BATOMALAQUE
461 SCRA 111
CARPIO MORALES; June 23, 2005
NATURE
Appeal from a decision of CA
FACTS
- In Feb.92, Abdul Aziz Car Maintenance Association, a Saudi Arabian entity based in
Riyadh, hired respondent, Willie Batomalaque, as a car painter at a monthly salary of
US$370 for a 2-year period through its agent, petitioner G&M (Phil.), Inc. In accordance
with the employment contract, respondent started working for Abdul Aziz in Mar.92 at a
monthly salary of US$370 which according to him was equivalent to 1,200 Saudi riyals.
- In June 94 respondent was repatriated and later filed a complaint against petitioner,
Abdul Aziz, and Country Empire Insurance Company with the POEA for non-payment and
underpayment of salaries and damages.
- Batomalaque claimed that for the first 4 months of employment, he received a monthly
salary of 900 Saudi riyals, and for the fifth month (July 92) up to the end of the 12th month
(Feb 93), he received a monthly salary of 700 Saudi riyals; that after a 1-year stint with
Abdul Aziz, the workshop where he was working was sold but the new owner did not hire
him; that for 11 months he was jobless; that Abdul Aziz hired him again and started
working for it in Feb 94 for which he was paid 1,200 Saudi riyals; and that he resigned in
May 94 since he was not paid his salary for the months of March and April 94, which 2month salary, was, however, used to purchase his airline ticket on his repatriation to the
Philippines. LA, NLRC and CA ruled in favor of respondent.
ISSUES
1. WON CA erred in awarding respondent his underpayment of salaries and wages

A2010

- 115 -

Disini

2. WON petitioner, as the recruiter and agent of Abdul Aziz, is thus solidarily liable with the
latter for the unpaid wages of respondent
HELD
1. NO
Ratio It is settled that as a general rule, a party who alleges payment as a defense has
the burden of proving it. Specifically with respect to labor cases, the burden of proving
payment of monetary claims rests on the employer, the rationale being that the pertinent
personnel files, payrolls, records, remittances and other similar documents which will
show that overtime, differentials, service incentive leave and other claims of workers have
been paid are not in the possession of the worker but in the custody and absolute
control of the employer.
Reasoning
- Aside from its bare allegation that its principal Abdul Aziz had fully paid respondents
salaries, petitioner did not present any evidence, e.g., payroll or payslips, to support its
defense of payment.
2. YES
Ratio Contractual undertakings submitted to the Bureau of Employment Services
constitute the legal basis for holding petitioner, and other private employment or
recruitment agencies, liable jointly and severally with its principal, the foreign-based
employer, for all claims filed by recruited workers which may arise in connection with the
implementation of the service agreements or employment contracts.
Reasoning
[1] Basis for liability: First, in applying for its license to operate a private employment
agency for overseas recruitment and placement, petitioner was required to submit, among
others, a document or verified undertaking whereby it assumed all responsibilities for the
proper use of its license and the implementation of the contracts of employment with the
workers it recruited and deployed for overseas employment. Second, it was also required
to file with Bureau a formal appointment or agency contract executed by the foreign-based
employer in its favor to recruit and hire personnel for the former, which contained a
provision empowering it to sue and be sued jointly and solidarily with the foreign principal
for any of the violations of the recruitment agreement and the contracts of employment.
And third, it was required as well to post such cash and surety bonds as determined by the
Sec. of Labor to guarantee compliance with prescribed recruitment procedures, rules and
regulations, and terms and conditions of employment as appropriate.
[2] The fact of underpayment does not shift the burden of evidence to the respondent
Batomalaque because partial payment does not extinguish the obligation. Only when the
debtor introduces evidence that the obligation has been extinguished does the burden of
evidence shift to the creditor who is then under a duty of producing evidence to show why
payment does not extinguish the obligation.
Disposition CA decision is AFFIRMED with the MODIFICATION. Batomalaque is only
entitled to 5,200 Saudi riyals, instead of 5,500 Saudi riyals.

2.09 MIGRANT WORKERS ACT


PHIL EMPLOY SERVICES AND RESOURCES INC V
PARAMIO
427 SCRA 732
CALLEJO SR; April 15, 2004
FACTS
- petitioners are a local agency engaged in the business of providing employment
overseas.
- Respondents are overseas workers sent by said agency to work for Kuan Yuan Fiber
Co., Ltd. Hsei-Chang. They had separate contracts and each paid P19,000 as placement
fee.
- They experienced and endured harsh conditions including eating just one meal a day,
substandard living quarters, excessive working hours, unexplained deductions in salaries
among others.
- Paramio, in the course of his work and due to not being fed dinner and breakfast, injured
his thumb while carrying a heavy load. Due to this he was sent home with significant
deductions to his pay.
- Navarra allegedly had an altercation with his superior and due to this was repatriated.
- other respondents opted to go home due to said inhumane work conditions they were
subjected to. They were made to write quitclaims and pay for their own airfare.
- Upon arriving in Manila, they instituted separate actions for illegal dismissal, nonpayment of overtime pay, refund of placement fee, tax refund, refund of plane fares,
attorneys fees and litigation expenses against the agency
- Paramio et al argued that under Section 10, Republic Act No. 8042, otherwise known
as the Migrant Workers and Overseas Filipinos Act of 1995, PSRI was solidarily liable
with Kuan Yuan for their claims. Since they were repatriated prior to the expiration of their

Labor Law 1

A2010

- 116 -

Disini

respective contracts for no valid reason, PSRI was liable to pay their salaries for the
unexpired portion of their contracts
- agency on the other hand contended that all of them were validly dismissed
- Labor Arbiter ruled that their dismissal was illegal and awarded damages. NLRC
reversed. CA affirmed decision of Labor Arbiter.

corresponding to the unexpired portion of their contract, as well as the reimbursement of


their placement fees.

ISSUES
WON the respondents were illegally dismissed

492 SCRA 761

HELD
NO
- We rule that respondents dismissal was not based on just, valid and legal grounds.
- The rule lex loci contractus (the law of the place where the contract is made) governs.
Therefore, the Labor Code, its implementing rules and regulations, and other laws
affecting labor, apply in this case
- In order to effect a valid dismissal of an employee, the law requires that there be just and
valid cause as provided in Article 282 and that the employee was afforded an opportunity
to be heard and to defend himself
- regarding Paramio agency insists that his dismissal was valid based on paragraph 8.2,
Nos. 5 and 6, Article VIII of the employment contract which states that if employer finds
employee with a disease which cannot be cured within a month; or being found losing
ability to work, employer may rescind contract and repatriate employee with airfare costs
against employee.
- However Art. 284 of Labor Code and Section 8, Rule 1, Book VI of the Omnibus Rules
Implementing the Labor Code state that the employer shall not terminate his
employment unless there is a certification by competent public authority that the disease is
of such nature or at such a stage that it cannot be cured within a period of six (6) months
with proper medical treatment
- employer has the duty to prove that employee cannot continue work due to the disease.
- this was not proven. In fact, despite the thumb injury sustained by Paramio, he was even
assigned to the most physically taxing jobs. Only after he could no longer bear the pain did
he ask for a little break.
- thus his dismissal based on his injury was illegal. Consequently, respondent Paramio is
entitled to the full reimbursement of his placement fee with interest at twelve percent
(12%) per annum, plus his salaries for the unexpired portion of his employment contract
for three months for every year of the unexpired term, whichever is less under paragraph
5, Section 10 of Rep. Act No. 8042: Section 10. Money Claims In case of termination
of overseas employment without just, valid or authorized cause as defined by law or
contract, the worker shall be entitled to the full reimbursement of his placement fee with
interest at twelve percent (12%) per annum, plus his salaries for the unexpired portion of
his employment contract or three (3) months for every year of the unexpired term,
whichever is less.
- also, Skippers Pacific, Inc. v. Mira, we ruled that an overseas Filipino worker who is
illegally terminated shall be entitled to his salary equivalent to the unexpired portion of his
employment contract if such contract is less than one year. However, if his contract is for
a period of at least one year, he is entitled to receive his salaries equivalent to the
unexpired portion of his contract, or three months salary for every year of the unexpired
term, whichever is lower
- Regarding Paramio, it was also not proven that he had an altercation with his superior.
The agency never presented evidence to prove such claim. They werent even able to
provide a statement from his superior regarding the issue. Respondent Navarra asserted
that he merely enforced his rights under the employment contract when he requested,
time and again, that the provisions of his contract regarding the accommodation be fulfilled
- Navarra was deployed on November 6, 1996 He was repatriated on May 10, 1997,
approximately five months prior to the expiration of his one-year contract. He shall be
entitled to an amount equivalent to three months salary, or NT$46,080. Similarly, having
admitted that he paid a placement fee of P19,000 only, he is entitled to be fully reimbursed
therefore, plus 12% interest per annum.
- regarding the other respondents, their quitclaims were not valid. The records reveal that
the three respondents agreed to execute the foregoing because they could no longer bear
the working conditions in their place of employment. Despite protestations to their
employer and the attempt to seek help from the OWWA in Taiwan.
- they were constructively dismissed from their employment. There is constructive
dismissal if an act of clear discrimination, insensibility, or disdain by an employer becomes
so unbearable on the part of the employee that it would foreclose any choice by him
except to forego his continued employment
- Under Section 10, paragraph 5 of Rep. Act No. 8042, respondents Sarmiento, Bautista,
Curameng and Guillermo are entitled to the full reimbursement of their placement fees.
Since each of the respondents remitted only P19,000 to the petitioner, each of them is
entitled to P19,000, plus 12% interest per annum.
- Section 10, paragraph 2 of Rep. Act No. 8042, the agency which deployed the
employees whose employment contract were adjudged illegally terminated, shall be jointly
and solidarily liable with the principal for the money claims awarded to the aforesaid
employees. Consequently, the petitioner, as the agency of the respondents, is solidarily
liable with its principal Kuan Yuan for the payment of the salaries due to the respondents

PLACEWELL INTERNATIONAL SERVICES V CAMOTE

YNARES-SANTIAGO; June 26, 2006


NATURE
Petition for certiorari of the September 27, 2005 Decision of the CA in CA-G.R. SP No.
77145, which set aside the November 20, 2002 Resolution of the NLRC and reinstated
with modifications the May 31, 2002 Decision of Labor Arbiter Arturo L. Gamolo.
FACTS
- On August 15, 1999, petitioner deployed respondent to work as building carpenter for
SAAD Trading and Contracting Co. (SAAD) at the Kingdom of Saudi Arabia (KSA) for a 2
yr. contract with a salary of US$370/month.
- At the job site, respondent was allegedly found incompetent by his foreign employer;
thus the latter decided to terminate his services. However, respondent pleaded for his
retention and consented to accept a lower salary of SR 800.00 per month. Thus, SAAD
retained respondent until his return to the Philippines two years after
- On November 27, 2001, respondent filed a complaint for money claims against petitioner
alleging that when he arrived at the job site, he and his fellow Filipino workers were
required to sign another employment contract written in Arabic under the constraints of
losing their jobs if they refused; that for the entire duration of the new contract, he
received only SR 590.00 per month; that he was not given his overtime pay despite
working 9hrs/day; that he and his co-workers sought assistance from the Philippine
Embassy but they did not succeed in pursuing their cause of action because of difficulties
in communication.
- On May 31, 2002, the labor arbiter rendered a decision holding that the modification of
respondents employment contract is not allowed under Section 10 of R.A. 8042; thus, he
should have received the original contracted salary of US$370/month instead of the new
rate given by SAAD. It was also noted that respondent did not refute petitioners allegation
regarding the non-payment of placement and other processing fees prior to deployment.
The labor arbiter also found that there is no differential as far as respondents overtime
pay is concerned considering that he was given overtime pay based on the new rate of
SR 800.00. Since respondent rendered one hour of overtime work per day for only 18
months, and not the entire 24 months as claimed, the total overtime pay he received is
more or less equivalent to the amount he ought to have received if the original contracted
rate of US$370.00 was used. Finally, the labor arbiter awarded respondent attorneys fees
equivalent to 10% of the total judgment award for being compelled to hire a counsel to
protect his rights and interests.
- NLRC set aside the decision. CA set aside NLRCs decision and affirmed Labor Arbiter.
ISSUES
WON respondent has a right to the money claims

HELD
YES
Ratio R.A. No. 8042 explicitly prohibits the substitution or alteration to the prejudice of the
worker, of employment contracts already approved and verified by the Department of
Labor and Employment (DOLE) from the time of actual signing thereof by the parties up to
and including the period of the expiration of the same without the approval of the DOLE.
Reasoning
- The unauthorized alteration in the employment contract of respondent, particularly the
diminution in his salary from US$370.00 to SR 800.00 per month, is void for violating the
POEA-approved contract which set the minimum standards, terms, and conditions of his
employment.
- Moreover, we find that there was no proper dismissal of respondent by SAAD; the
"termination" of respondent was clearly a ploy to pressure him to agree to a lower wage
rate for continued employment. Thus, the original POEA-approved employment contract of
respondent subsists despite the so-called new agreement with SAAD. Consequently, the
solidary liability of petitioner with SAAD for respondents money claims continues in
accordance with Section 10 of R.A. 8042.
- As to petitioners claim that respondent was barred by laches, respondent filed his claim
within the three-year prescriptive period for the filing of money claims set forth in Article
291 of the Labor Code from the time the cause of action accrued. Thus, we find that the
doctrine of laches finds no application in this case.
- The labor arbiter and the CA did not err in awarding attorneys fees to respondent. It is
settled that in actions for recovery of wages or where an employee was forced to litigate

Labor Law 1
and incur expenses to protect his rights and interests, he is entitled to an award of
attorneys fees. However, with regard to Unauthorized Deductions amounting to
P171,780.00; we note that the appellate court did not state any basis for its award, thus,
the same is deleted for lack of factual and legal basis.
Disposition Instant petition is PARTLY GRANTED. The Decision of the Court of Appeals
in CA-G.R. SP No. 77145 dated September 27, 2005 is AFFIRMED with MODIFICATION
that the amount of P171,780 representing Unauthorized Deductions is DELETED for lack
of basis.

SECTION 3: ALIEN EMPLOYMENT


3.01 COVERAGE
NON-RESIDENT ALIEN
ALMODIEL V NLRC (RAYTHEON PHILS INC)
223 SCRA 341
NOCON; June 14, 1993
NATURE
Petition for certiorari of an NLRC decision which set aside the Labor Arbiters decision and
ordered instead the payment of separation pay and financial assistance.
FACTS
- Petitioner Almodiel is a CPA, was a Cost Accounting Manager of respondent Raytheon
Philippines. His major duties were: (1) plan, coordinate, carry out year and physical
inventory; (2) formulate and issue out hard copies of Standard Product costing and other
cost/pricing analysis if needed and (3) set up the written Cost Accounting System for the
whole company.
- The standard cost accounting system was installed and used at the Raytheon plants and
subsidiaries worldwide. As a consequence, the services of a Cost Accounting Manager
allegedly entailed only the submission of periodic reports that would use computerized
forms prescribed and designed by the international head office in California
- Later, Almodiel was summoned told of the abolition of his position on the ground of
redundancy.
- Almodiel thus filed the complaint for illegal dismissal before the Arbitration Branch of
NCR, NLRC.
- The court below found that the parties had no arrived at a definite understanding.
Labor Arbiter rendered judgment in favor of Almodiel, finding that the ground of
redundancy was highly irregular and without legal and factual basis, ordering the
respondents to reinstate complainant, with full backwages, and moral and exemplary
damages.
- NLRC reversed the decision, and instead merely ordered Raytheon to pay P100,000
separation pay/financial assistance
- Petitioner thus filed this petition: The public respondent committed grave abuse of
discretion amounting to (lack of) or in excess of jurisdiction in declaring as valid and
justified the termination of petitioner on the ground of redundancy in the face of clearly
established finding that petitioner's termination was tainted with malice, bad faith and
irregularity.
- Termination of an employees services because of redundancy is governed by Art. 283 of
the Labor code:
Art. 283. Closure of establishment and reduction of personnel. The employer may
also terminate the employment of any employee due to installation of labor-saving
devices, redundancy, retrenchment to prevent losses or the closing or cessation of
operation of the establishment or undertaking unless the closing is for the purpose of
circumventing the provisions of this Title, by serving a written notice on the worker and
the Department of Labor and Employment at least one (1) month before the intended
date thereof. In case of termination due to installation of labor-saving devices or
redundancy, the worker affected thereby shall be entitled to a separation pay equivalent
to at least one (1) month pay for every year of service, whichever is higher. In case of
retrenchment to prevent losses and in cases of closure or cessation of operations of
establishment or undertaking not due to serious business losses or financial reverses,
the separation pay shall be equivalent to at least one (1) month pay or at least one-half
(1/2) month pay for every year of service, whichever is higher. A fraction of at least six
(6) months shall be considered as one (1) whole year.
ISSUE
WON bad faith, malice, and irregularity crept in the abolition of petitioners position of Cost
Accounting Manager on the ground of redundancy, thus making the dismissal illegal

A2010

Disini

- 117 -

HELD
NO
Reasoning
- There simply wasnt evidence showing that there was bad faith or malice or irregularity at
all.
- The wisdom or soundness of such characterization or decision was not subject to
discretionary review on the part of the Labor Arbiter nor of the NLRC so long, of course, as
violation of law or merely arbitrary and malicious action is not shown.
- It is a well-settled rule that labor laws do not authorize interference with the employer's
judgment in the conduct of his business. The determination of the qualification and fitness
of workers for hiring and firing, promotion or reassignment are exclusive prerogatives of
management. The Labor Code and its implementing Rules do not vest in the Labor
Arbiters nor in the different Divisions of the NLRC (nor in the courts) managerial authority.
The employer is free to determine, using his own discretion and business judgment, all
elements of employment, "from hiring to firing" except in cases of unlawful discrimination
or those which may be provided by law. There is none in the instant case.
Disposition Finding no grave abuse of discretion on the part of the National Labor
Relations Commission in reversing and annulling the decision of the Labor Arbiter and that
on the contrary, the termination of petitioner's employment was anchored on a valid and
authorized cause under Article 283 of the Labor Code, the instant petition for certiorari
must fail.
*** the only part relevant to this part of the outline:
Likewise destitute of merit is petitioner's imputation of unlawful discrimination when
Raytheon caused corollary functions appertaining to cost accounting to be absorbed by
Danny Ang Tan Chai, a resident alien without a working permit. Article 40 of the Labor
Code which requires employment permit refers to non-resident aliens. The employment
permit is required for entry into the country for employment purposes and is issued after
determination of the non-availability of a person in the Philippines who is competent, able
and willing at the time of application to perform the services for which the alien is desired.
Since Ang Tan Chai is a resident alien, he does not fall within the ambit of the provision

3.02
TECHNIQUE
EMPLOYMENT PERMIT

REGULATION

AUTHORITY EMPLOYMENT PERMIT ISSUANCE


GENERAL MILLING CORP V TORRES

SECTION 4: EMPLOYMENT OF APPRENTICES,


LEARNERS AND HANDICAPPED WORDERS
4.01 POLICY OBJECTIVES
A. APPRENTICE

4.02 APPRENTICE
REQUIREMENT PROGRAM APPROVAL
NITTO ENTERPRISES V NLRC (CAPILI)
248 SCRA 654
KAPUNAN; September 29, 1995
FACTS
- Nitto Enterprises, a company engaged in the sale of glass and aluminum products, hired
Roberto Capili sometime in May 1990 as an apprentice machinist, molder and core maker
as evidenced by an apprenticeship agreement for a period of 6 months from May 28, 1990
to November 28, 1990 with a daily wage rate of P66.75 which was 75% of the applicable
minimum wage.
- At around 1 pm of August 2, 1990, Capili who was handling a piece of glass which he
was working on, accidentally hit and injured the leg of an office secretary who was treated
at a nearby hospital. Later that same day, after office hours, Capili entered a workshop

Labor Law 1
within the office premises which was not his work station. There, he operated one of the
power press machines without authority and in the process injured his left thumb. Nitto
Enterprises spent the amount of P1,023.04 to cover his medication.
- The following day, Capili was asked to resign [letter was in Tagalog and quite long but
basically narrates the events above as grounds for dismissal]. On August 3, 1990 he
executed a Quitclaim and Release in favor of Nitto Enterprises in consideration of the sum
of P1,912.79. Capili filed a complaint for illegal dismissal and payment of other monetary
benefits.
- The Labor Arbiter found the termination valid and dismissed the money claim for lack of
merit. Nitto, however was ordered to pay Capili the amount of P500 as financial
assistance. The NLRC reversed and declared that Capili was a regular employee of Nitto
Enterprises.

ISSUES
1. WON the NLRC committed grave abuse of discretion in holding that Capili was not an
apprentice
2. WON the NLRC committed grave abuse of discretion in holding that Nitto Enterprises
had not adequately proven the existence of a valid cause in terminating the service of
Capili

HELD
1. NO
Ratio Prior approval by the DOLE of a proposed apprenticeship program is a condition
sine qua non before an apprenticeship agreement can be validly entered into.
Reasoning
- The act of filing the proposed apprenticeship program with the Department of Labor and
Employment is a preliminary step towards its final approval and does not instantaneously
give rise to an employer-apprentice relationship. Article 61 of the Labor Code provides:
Contents of apprenticeship agreement. - Apprenticeship agreements, including the main
rates of apprentices, shall conform to the rules issued by the Minister of Labor and
Employment. The period of apprenticeship shall not exceed 6 months. Apprenticeship
agreements providing for wage rates below the legal minimum wage, which in no case
shall start below 75% per cent of the applicable minimum wage, may he entered into only
in accordance with apprenficeship program duly approved by the Minister of Labor and
Employment. The Ministry shall develop standard model programs of apprenticeship.
- The apprenticeship agreement between Nitto Enterprises and Capili was executed on
May 28, 1990 allegedly employing the latter as an apprentice in the trade of "core
maker/molder." On the same date, an apprenticeship program was prepared by petitioner
and submitted to the DOLE. However, the apprenticeship agreement was filed only on
June 7, 1990. Notwithstanding the absence of approval by the DOLE the apprenticeship
agreement was enforced the day it was signed. Nitto Enterprises did not comply with the
requirements of the law.
2. NO
Ratio The twin requirements of due process, substantive and procedural, must be
complied with, before valid dismissal exists. Without which, the dismissal becomes void.
Reasoning
- This simply means that the employer shall afford the worker ample opportunity to be
heard and to defend himself with the assistance of his representative, if he so desires.
Ample opportunity connotes every kind of assistance that management must accord the
employee to enable him to prepare adequately for his defense including legal
representation.
- As held in the case of Pepsi-Cola Bottling Co., Inc. v. NLRC:
The law requires that the employer must furnish the worker sought to be dismissed with
2 written notices before termination can be legally effected: (1) notice which apprises
the employee of the particular acts or omissions for which his dismissal is sought; and
(2) the subsequent notice which informs the employee of the employer's decision to
dismiss him (Sec. 13, BP 130; Sec. 2-6, Rule XIV, Book V, Rules and Regulations
Implementing the Labor Code as amended). Failure to comply with the requirements
taints the dismissal with illegality. This procedure is mandatory; in the absence of
which, any judgment reached by management is void and inexistent.
- Capili filed a case of illegal dismissal with the Labor Arbiter only 3 days after he was
made to sign a Quitclaim, a clear indication that such resignation was not voluntary and
deliberate. He averred that he was actually employed by petitioner as a delivery boy
("kargador" or "pahinante"). He further asserted that petitioner "strong-armed" him into
signing the resignation letter and quitclaim without explaining to him its contents. Petitioner
made it clear to him that anyway, he did not have a choice.
- Nitto Enterprises cannot disguise the summary dismissal of Capili by orchestrating the
latter's alleged resignation and subsequent execution of a Quitclaim and Release. A
judicious examination of both events belies any spontaneity on Capili's part.

A2010

- 118 -

Disini

Disposition Decision of the NLRC is AFFIRMED.

B. LEARNERS
C. HANDICAPPED WORKERS
REGULAR WORKER
BERNARDO V NLRC (FAR EAST BANK AND TRUST
COMPANY)
[PAGE 104]

SECTION 5: CONDITIONS OF EMPLOYMENT


HOURS OF WORK
5.01 HOURS REGULATION
RATIONALE AND ENFORCEMENT
MANILA TERMINAL CO INC V CIR

5.02 COVERAGE
RATIONALE EXEMPTION MANAGERIAL EMPLOYEES
PEARANDA V BAGANGA PLYWOOD CORP
[PAGE 1]
ASIA PACIFIC CHARTERING (PHILS) INC V FAROLAN
393 SCRA 454
CARPIO-MORALES; December 4, 2002
NATURE
CERTIORARI under Rule 45 of the
FACTS
- Asia Pacific Chartering (Phils) Inc. [APC] was, until 1996, the general sales agent (GSA)
of the Scandinavian Airline System (SAS), an off-line international airline company with
license to do business in the Philippines.
- As GSA, APC sold passenger and cargo spaces for airlines operated by SAS.
- Maria Linda R. Farolan was hired as Sales Manager of APC for its passenger and cargo
GSA operations for SAS, following her conformity to a letter-offer of employment from APC
through its Vice President/Comptroller Catalino Bondoc.
- In a report by Farolan, she indicated that there was a drop in SAS sales revenues which
to her was attributable to market forces beyond her control.
- Noting the marked decline in SAS sales revenues, APC directed its high ranking officer
Roberto Zozobrado to conduct an investigation on the matter and identify the problem/s
and implement possible solutions.
- Zozobrado thus informally took over some of FAROLANs marketing and sales
responsibilities, albeit respondent retained her title as Sales Manager and continued to
receive her salary as such.
- APC claimed that:
> Zozobrado found out that FAROLAN did not adopt any sales strategy nor conduct any
sales meeting or develop other sources of revenue for SAS, she having simply let her
sales staff perform their functions all by themselves;
> In 1994, Soren Jespersen, GM of SAS in Hongkong, Southern China, Taipei and the
Philippines, came to the Philippines to assess the statistics on SAS sales revenues and
SAS was convinced that FAROLAN was not fit for the job of Sales Manager; and in view of
the changes introduced by Zozobrado, SAS-GSA sales operations drew positive results.
- May 1994: Jespersen sent a message to Farolan and Zozobrado congratulating them for
the increase in sales results and for exceeding the target set by 50% while June 1994:
Jespersen sent a msg to Farolan saying that the sales report for June 1994 did
unfortunately not reach target.
- On even date, APC sent respondent a letter of termination on the ground of "loss of
confidence."

Labor Law 1
- This spawned the filing by FAROLAN of a complaint for illegal dismissal against APC,
Bondoc, Zozobrado and one Donald Marshall (the record indicates that he had ceased to
be connected with petitioner when the case was pending before the Labor Arbiter), with
prayer for damages and attorneys fees.
- In her complaint petitioner alleged that Bondoc and Zozobrado had asked her to tender
her resignation as she was not the person whom SAS was looking for to handle the
position of Sales Manager but that she refused, hence, she was terminated by the letter of
July 18, 1994 letter.
- LA: found for FAROLAN that APC dismissed her without just cause, effected with malice,
ill will and bad faith
- NLRC: reversed the LAs decision; recognized the right of APC as employer to terminate
or dismiss employees based on loss of trust and confidence, the right being a
management prerogative.
- Farolans MR denied; appealed to CA
- CA: reversed the NLRC decision; LAs decision upheld with modifications
ISSUE
WON FAROLANs dismissal was legal
HELD
YES
Reasoning
- Requisites for a valid dismissal of an EMPLOYEE:
(a) the employee must be afforded due process, i.e., he must be given opportunity to
be heard and to defend himself; and
(b) dismissal must be for a valid cause as provided in Article 282 of the Labor Code or
any of the authorized causes under Article 283 and 284 of the same Code.
- AS REGARDS (A) THE LA FOUND THAT FAROLAN WAS DEPRIVED OF DUE
PROCESS (this was upheld by the CA):
*W/O any semblance of, or written authority whatsoever, Zozobrado took over the
functions of complainant. Complainant claims that she has been told it was upon the
will of respondent Marshall that she be replaced. Bondoc summoned Farolan and told
her to tender her resignation or face termination, which she refused to do. Thereafter,
she was finally terminated, without being afforded the opportunity to be heard and to
present evidence in her defense. She was never given a written notice stating the
particular acts or omission constituting the grounds for her dismissal as required by
law.
- AS REGARDS (B): the rule is settled that in termination cases, the employer bears the
onus of proving that the dismissal is for just cause failing which the dismissal is not
justified and the employee is entitled to reinstatement.
APCS CLAIMS:
- FAROLAN failed to live up to managements expectation in light of her failure to adopt
sales and marketing strategies to increase sales revenues of SAS, which failure is
reflective of her incompetence and inefficiency, thus resulting to loss of revenues in 1993
and 1994.
- Had it not been through Zozobrados efforts, SAS sales revenues could not have
recovered.
- Jespersen was the one who initiated the termination of respondent because of her
"dismal performance" in handling its operations.
- Reiterated the principle that the right to dismiss a managerial employee is a measure of
self-preservation; Cited the cases of Grand Motor Parts Corp. v. Minister of Labor et al.,
and Buiser et al. v. Legardo.
THE NATURE OF FAROLANS JOB AS SALES MANAGER
- It is not disputed that her job description, and the terms and conditions of her
employment, with the exception of her salary and allowances, were never reduced to
writing.
- Recent decisions of this Court distinguish the treatment of managerial employees from
that of rank and file personnel insofar as the application of the doctrine of loss of trust and
confidence is concerned.
"Thus with respect to rank and file personnel, loss of trust and confidence as ground
for valid dismissal requires proof of involvement in the alleged events in question and
that mere uncorroborated assertions and accusations by the employer will not be
sufficient. But as regards a managerial employee, mere existence of a basis for
believing that such employee has breached the trust of his employer would suffice for
his dismissal."
- Samson v. NLRC: (RULE FOR THIS CASE!)
"Before one may be properly considered a managerial employee, all the
following conditions must be met:
(1) Their primary duty consists of the management of the establishment in which
they are employed or of a department or subdivision thereof;
(2) They customarily and regularly direct the work of two or more employees
therein;
(3) They have the authority to hire or fire other employees of lower rank; or their
suggestions and recommendations as to the hiring and firing and as to the
promotion or any other change of status of other employees are given particular

A2010

- 119 -

Disini

weight. (Section 2(b), Rule I, Book III of the Omnibus Rules Implementing the
Labor Code, emphasis supplied).
- FAROLANS JOB: dealt mainly with servicing of existing clientele. Bondoc, however,
described respondents functions and duties as critical.
- Paper Industries Corp. of the Philippines v. Laguesma:
"Managerial employees are ranked as Top Managers, Middle Managers and First Line
Managers. The mere fact that an employee is designated "manager" does not ipso
facto make him one-designation should be reconciled with the actual job description of
the employee for it is the job description that determines the nature of employment."
- Dismissal on the ground of "loss of confidence" should have a basis and determination
cannot be left entirely to the employer.
-Loss of trust and confidence to be a valid ground for an employees dismissal must be
based on a willful breach and founded on clearly established facts.
- A breach is willful if it is done intentionally, knowingly and purposely, without justifiable
excuse, as distinguished from an act done carelessly, thoughtlessly, heedlessly or
inadvertently.
- The two letters sent by SAS to respondent in 1994 in fact negate willful breach of her
duties by respondent.
- While APC attributed the improvement of sales in 1994 to Zozobrado, the fact remains
that FAROLAN was still the Sales Manager up to July 1994, in charge of those "sales
meetings" during which pertinent market strategies were developed and utilized to
increase sales.
- The Grand Motors case cited by APC, however, involved a probationary employeemanager who failed to, among other things, submit required monthly reports and violated
company policy, clearly mirroring his insubordination and disrespect to express
instructions of management.
- In the Buiser case: "[f]ailure to observe prescribed standards of work, or to fulfill
reasonable work assignments due to inefficiency" may be just cause for dismissal,
petitioner has neither shown what standards of work or reasonable work assignments
were prescribed which respondent failed to observe nor that if she did fail to observe any
such, it was due to inefficiency.
-It bears noting that there is no showing that respondent represented herself as
possessed of the highest degree of skill and care known in the trade.
- FAROLAN was the one approached or offered the job; She thus could not just be
unceremoniously discharged for "loss of confidence" arising from alleged incompetency
- "While an employee may be dismissed because of inefficiency, neglect or
carelessness, the law implies a situation or undertaking by an employee in entering
into a contract of employment that he is competent to perform the work undertaken
and is possessed of the requisite skill and knowledge to enable him to do so, and
that he will do the work of the employer in a careful manner. If he is not qualified to
do the work which he undertakes, if he is incompetent, unskillful or inefficient, or if
he executes his work in a negligent manner or is otherwise guilty of neglect of duty,
he may lawfully be discharged before the expiration of his term of employment."
DAMAGES:
- AS FAROLAN was instructed to resign or be terminated, she was deprived of due
process and denied "basic precepts of fairness" when she was terminated. Her resultant
sufferings thus entitle her to an award of moral damages.
- Award of moral and exemplary damages for an illegally dismissed employee is proper
where the employee had been harassed and arbitrarily terminated by the employer.
-In determining the amount of moral damages recoverable, however, the business, social
and financial position of the offended party and the business or financial position of the
offender are taken into account.
Disposition CAS decision is AFFIRMED with the MODIFICATION that the amount of
moral damages and exemplary damages awarded to Farolan is hereby reduced to
P500,000.00 Pesos and P250,000.00, respectively.

NATIONAL WATERWORKS & SEWERAGE AUTHORITY


V NWSA CONSOLIDATED UNIONS
11 SCRA 766
BAUTISTA ANGELO; August 31, 1964
NATURE
Petition for review of a decision of the Court of Industrial Relations
FACTS
- Petitioner National Waterworks & Sewerage Authority is a government-owned and
controlled corporation created under Republic Act No. 1383, while respondent NWSA
Consolidated Unions are various labor organizations composed of laborers and
employees of the NAWASA. The other respondents are intervenors Centeno, et al.,
hereinafter referred to as intervenors.
- Petitioner and respondent unions, conformably to a suggestion of the Court of Industrial
Relations on the controversy between them, submitted a joint stipulation of facts on the
issues concerning the 40-Hour Week Law, distress pay, minimum wage, filling of

Labor Law 1
vacancies, night compensation, and salary adjustments, reserving the right to present
evidence on matters not covered herein. Respondent intervenors filed a petition in
intervention on the issue for additional compensation for nightwork. Later, however, they
amended their petition by including a new demand for overtime pay in favor of Centeno
and other employees receiving P4200 per annum or more.
- Petitioner filed a motion to dismiss the claim for overtime pay alleging that respondent
Court of Industrial Relations was without jurisdiction to pass upon the same because, as
mere intervenors, the latter cannot raise new issues not litigated in the principal case, the
same not being the lis mota therein involved. To this motion, the intervenors filed an
opposition. Thereafter, respondent court issued an order allowing the issue to be litigated.
Petitionerss motion to reconsider having been denied, it filed its answer to the petition for
intervention.
- Finally, respondent rendered its decision stating among others that (1) the NAWASA is
an agency not performing governmental functions and, therefore, is liable to pay additional
compensation for work on Sundays and legal holidays conformably to Commonwealth Act
No. 444, known as the Eight-Hour Labor Law, even if said days should be within the
staggered five work-days authorized by the President and (2) the intervenors do not fall
within the category of managerial employees as contemplated in Republic Act 2377 and
so are not exempt from the coverage of the Eight-Hour Labor Law; even those intervenors
attached to the General Auditing Office and the Bureau of Public Works come within the
purview of Commonwealth Act No. 444.
- Its motion for reconsideration having been denied, NAWASA filed the present petition for
review.
ISSUE
WON the intervenors are managerial employees within the meaning of Republic Act
2377 and, therefore, not entitled to the benefits of Commonwealth Act No. 444, as
amended
HELD
NO
- One of the distinguishing characteristics by which a managerial employee may be known
as expressed in the explanatory note of Republic Act 2377 is that he is not subject to the
rigid observance of regular office hours. The true worth of his service does not depend so
much on the time he spends in office but more on the results he accomplishes. In fact, he
is free to go out of office anytime.
- Section 2 of Republic Act 2377 provides:
Sec. 2. This Act shall apply to all persons employed in any industry or occupation
whether public or private, with the exception of farm laborers, laborers who prefer to be
paid on piece work basis, managerial employees, outside sales personnel, domestic
servants, persons in the personal service of another and members of the family of the
employer working for him.
The term `managerial employee in this Act shall mean either (a) any person whose
primary duty consists of the management of the establishment in which he is employed
or of a customarily recognized department or subdivision thereof, or (b) any officer or
member of the managerial staff.
- The philosophy behind the exemption of managerial employees from the 8-Hour Labor
Law is that such workers are not usually employed for every hour of work but their
compensation is determined considering their special training, experience, or knowledge
which requires the exercise of discretion and independent judgment, or perform work
related to management policies or general work related to management policies or general
business operations along specialized or technical lines. For these workers it is not
feasible to provide a fixed hourly rate of pay or maximum hours of labor.
- The intervenors herein are holding positions of responsibility. One of them is the
Secretary of the Board of Directors. Another is the private secretary of the general
manager. Another is a public relations officer, and many other chiefs of divisions or
sections and others are supervisors and overseers. Respondent court, however, after
examining carefully their respective functions, duties, and responsibilities found that their
primary duties do not bear any direct relation with the management of the NAWASA, nor
do they participate in the formulation of its policies nor in the hiring and firing of its
employees. The chiefs of divisions and sections are given ready policies to execute and
standard policies to observe for their execution. Hence, it concludes, they have little
freedom of action, as their main function is merely to carry out the companys orders,
plans, and policies.
- To the foregoing comment, the Court agrees. As a matter of fact, they are required to
observe working hours and record their time work and are not free to come and go to their
offices, nor move about at their own discretion. They do not, therefore, come within the
category of managerial employees within the meaning of the law.
Disposition Decision affirmed with modification.

SAMSON V NLRC (SCHERING-PLOUGH CORP)


330 SCRA 460
KAPUNAN; April 12, 2000

A2010

- 120 -

Disini

FACTS
- Samson was a District Sales Manager at SPC. During the 1993 Christmas party, he said
obscene, offensive and insulting words towards the management and the president infront
of other employees. On Jan 25 1994, he was given a letter which enumerated conduct
inimical to SPC. He was given 2 days to explain. He replied on the same day, saying that it
was an informal event and that he was tipsy and denying the accusations. He was
preventively suspended that same day. He was terminated on Feb 3 1994, for loss of
confidence. He filed complaint for illegal dismissal. LA decided in his favor. NLRC reversed
because it found that his gross misconduct warranted termination. NLRC also said that in
terminating the employment of managerial employees, the employer is allowed a wider
latitude of discretion than in the case of ordinary rank-and-file.
ISSUES
1. WON termination was warranted
2. WON petitioner is managerial employee
HELD
1. NO
- The termination was illegal. The companys code of conduct punishes such misconduct
by a verbal warning for a first offense. There was no demoralization among the ranks, or
at least the behavior of the company doesnt show urgency --- he was given a letter weeks
after the event occurred. Also, at a meeting on Jan 3 1994, the president himself civilly
and calmly told him that if there were disagreements, they should be handled in a
professional manner.
2. NO
- Before one may be properly considered a managerial employee, all the following
conditions must be met:
(1)
Their primary duty consists of the management of the establishment in which
they are employed or of a department or sub-division thereof;
(2)
They customarily and regularly direct the work of two or more employees
therein;
(3)
They have the authority to hire or fire other employees of lower rank; or their
suggestions and recommendations as to the hiring and firing and as to the promotion
or any other change of status of other employees are given particular weight.
- Although his position is a District Sales Manager, his job description do not meet the
above conditions for him to be considered a managerial employee.
- Granting he was a managerial employee, loss of confidence as a ground for termination
is still without basis. Loss of trust and confidence to be a valid ground for an employee's
dismissal must be clearly established. A breach is willful if it is done intentionally,
knowingly and purposely, without justifiable excuse, as distinguished from an act done
carelessly, thoughtlessly, heedlessly or inadvertently. It must rest on substantial grounds
and not on the employer's arbitrariness, whims, caprices or suspicion, otherwise, the
employee would remain at the mercy of the employer. When petitioner made the offensive
utterances, it can be said that he merely acted carelessly, thoughtlessly or heedlessly and
not intentionally, knowingly, purposely, or without justifiable excuse.
Disposition Petition granted. NLRC reversed and set aside. Petitioner is thus entitled to
reinstatement to his position as District Sales Manager, unless such position no longer
exists, in which case he shall be given a substantially equivalent position without loss of
seniority rights. He is likewise entitled to the payment of his full backwages.

TESTS- FIELD PERSONNEL


UNION OF FILIPRO EMPLOYEES V VIVAR
205 SCRA 200
GUTIERREZ JR; January 20, 1992
FACTS
- Filipro, Inc. (now Nestle Philippines, Inc.) filed with the National Labor Relations
Commission (NLRC) a petition for declaratory relief seeking a ruling on its rights and
obligations respecting claims of its monthly paid employees for holiday pay.
- Filipro filed a motion for clarification seeking (1) the limitation of the award to three years,
(2) the exclusion of salesmen, sales representatives, truck drivers, merchandisers and
medical representatives (hereinafter referred to as sales personnel) from the award of the
holiday pay; and (3) deduction from the holiday pay award of overpayment for overtime,
night differential, vacation and sick leave benefits due to the use of 251 divisor.
- The respondent arbitrator issued an order declaring that the effectivity of the holiday pay
award shall retroact to November 1, 1974, the date of effectivity of the Labor Code. He
adjudged, however, that the company's sales personnel are field personnel and, as such,
are not entitled to holiday pay. He likewise ruled that with the grant of 10 days' holiday pay,
the divisor should be changed from 251 to 261 and ordered the reimbursement of

Labor Law 1
overpayment for overtime, night differential, vacation and sick leave pay due to the use of
251 days as divisor.
ISSUES
1. WON Nestle's sales personnel are entitled to holiday pay
2. WON concomitant with the award of holiday pay, the divisor should be changed from
251 to 261 days and WON the previous use of 251 as divisor resulted in overpayment for
overtime, night differential, vacation and sick leave pay
HELD
1. NO
- Under Article 82, field personnel are not entitled to holiday pay. Said article defines field
personnel as "non-agricultural employees who regularly perform their duties away from the
principal place of business or branch office of the employer and whose actual hours of
work in the field cannot be determined with reasonable certainty."
- The controversy centers on the interpretation of the clause "whose actual hours of work
in the field cannot be determined with reasonable certainty." The SC concurs with the
following disquisition by the respondent arbitrator:.
- "The requirement for the salesmen and other similarly situated employees to report for
work at the office at 8:00 a.m. and return at 4:00 or 4:30 p.m. is not within the realm of
work in the field as defined in the Code but an exercise of purely management prerogative
of providing administrative control over such personnel. This does not in any manner
provide a reasonable level of determination on the actual field work of the employees
which can be reasonably ascertained. The theoretical analysis that salesmen and other
similarly-situated workers regularly report for work at 8:00 a.m. and return to their home
station at 4:00 or 4:30 p.m., creating the assumption that their field work is supervised, is
surface projection. Actual field work begins after 8:00 a.m. when the sales personnel
follow their field itinerary, and ends immediately before 4:00 or 4:30 p.m. when they report
back to their office. The period between 8:00 a.m. and 4:00 or 4:30 p.m. comprises their
hours of work in the field, the extent or scope and result of which are subject to their
individual capacity and industry and which 'cannot be determined with reasonable
certainty.' This is the reason why effective supervision over field work of salesmen and
medical representatives, truck drivers and merchandisers is practically a physical
impossibility. Consequently, they are excluded from the ten holidays with pay award.'
2. NO
- The divisor to be used is 251. The respondent arbitrator's order to change the divisor
from 251 to 261 days would result in a lower daily rate which is violative of the prohibition
on non-diminution of benefits found in Article 100 of the Labor Code. The Court resolves
that the grant of holiday pay be effective, not from the date of promulgation of the
Chartered Bank case nor from the date of effectivity of the Labor Code, but from October
23, 1984, the date of promulgation of the IBAA case.
Disposition Decision of Labor Arbiter is MODIFIED.

SALAZAR V NLRC (H. L. CARLOS CONSTRUCTION,


CO INC)
256 SCRA 273
KAPUNAN; April 17, 1996
NATURE
Petition for certiorari to annul the decision of the National Labor Relations Commission
FACTS
- On 17 April 1990, private respondent, at a monthly salary of P4,500.00, employed
petitioner as construction/project engineer for the construction of the Monte de Piedad
building in Cubao, Quezon City.
- Allegedly, by virtue of an oral contract, petitioner would also receive a share in the profits
after completion of the project and that petitioners services in excess of eight (8) hours on
regular days and services rendered on weekends and legal holidays shall be
compensable overtime at the rate of P27.85 per hour.
- On 16 April 1991, petitioner received a memorandum issued by private respondents
project manager, Engr. Nestor A. Delantar informing him of the termination of his services
effective on 30 April 1991.
- Petitioner filed a complaint against private respondent for illegal dismissal, unfair labor
practice, illegal deduction, non-payment of wages, overtime rendered, service incentive
leave pay, commission, allowances, profit-sharing and separation pay with the NLRC.
- Labor Arbiter Raul T. Aquino dismissed the complaint for lack of merit.
- NLRC affirmed, and also denied MFR
ISSUE
WON petitioner is entitled to overtime pay, premium pay for services rendered on rest
days and holidays and service incentive leave pay, pursuant to Articles 87, 93, 94 and 95
of the Labor Code

A2010

- 121 -

Disini

HELD
NO
- The NLRC concurred with the Labor Arbiters ruling that petitioner was a managerial
employee and, therefore, exempt from payment of overtime pay, premium pay for holidays
and rest days and service incentive leave pay under the law.
- Petitioner claims that since he performs his duties in the project site or away from
the principal place of business of his employer, he falls under the category of field
personnel.
- However, petitioner accentuates that his case constitutes the exception to the exception
because his actual working hours can be determined as evidenced by the disbursement
vouchers containing payments of petitioners salaries and overtime services. Strangely,
petitioner is of the view that field personnel may include managerial employees.
- In his original complaint, petitioner stated that the nature of his work is supervisoryengineering. Similarly, in his own petition and in other pleadings, petitioner confirmed that
his job was to supervise the laborers in the construction project.
- Although petitioner cannot strictly be classified as a managerial employee under
Art. 82 of the Labor Code, and Sec. 2(b), Rule 1, Book III of the Omnibus Rules
Implementing the Labor Code, nonetheless he is still not entitled to payment of the
aforestated benefits because he falls squarely under another exempt category officers or members of a managerial staff as defined under Sec. 2(c)25 of the
abovementioned implementing rules:
- A case in point is National Sugar Refineries Corporation v. NLRC. On the issue of
whether supervisory employees, as defined in Article 212 (m), Book V of the Labor Code,
should be considered as officers or members of the managerial staff under Article 82,
Book III of the same Code and hence not entitled to overtime, rest day and holiday pay,
this Court ruled: A cursory perusal of the Job Value Contribution Statements of the union
members will readily show that these supervisory employees are under the direct
supervision of their respective department superintendents and that generally they
assist the latter in planning, organizing, staffing, directing, controlling,
communicating and in making decisions in attaining the companys set goals and
objectives. These supervisory employees are likewise responsible for the effective
and efficient operation of their respective departments. From the foregoing, it is
apparent that the members of respondent union discharge duties and responsibilities
which ineluctably qualify them as officers or members of the managerial staff, as
defined in Section 2, Rule 1, Book III of the Rules to Implement the Labor Code
- The same applies to petitioner herein considering in the main his supervisory
duties as private-respondents project engineer, duties which, it is significant to
note, petitioner does not dispute.
- Petitioner, likewise, claims that the NLRC failed to give due weight and consideration to
the fact that private respondent compensated him for his overtime services as indicated in
the various disbursement vouchers he submitted as evidence.
- Petitioners contention is unmeritorious. That petitioner was paid overtime benefits does
not automatically and necessarily denote that petitioner is entitled to such benefits. Art. 82
of the Labor Code specifically delineates who are entitled to the overtime premiums and
service incentive leave pay provided under Art. 87, 93, 94 and 95 of the Labor Code and
the exemptions thereto.
- As previously determined, petitioner falls under the exemptions and therefore has no
legal claim to the said benefits. It is well and good that petitioner was compensated for his
overtime services. However, this does not translate into a right on the part of petitioner to
demand additional payment when, under the law, petitioner is clearly exempted therefrom.

MERCIDAR FISHING CORP V NLRC (AGAO JR)


297 SCRA 440
MENDOZA; October 8, 1998
NATURE
Petition for certiorari to set aside NLRC decision dismissing Mercidars appeal from Labor
Arbiter decision
FACTS
- Fermin AGAO Jr. had been employed as a bodegero or ships quartermaster. He
claimed to have been constructively dismissed by petitioner when it refused him
assignments aboard its boats. He then filed a COMPLAINT against MERCIDAR for
ILLEGAL DISMISSAL, violation of P.D. No. 851, and non-payment of five days service
25

Sec. 2. Exemption. - The provisions of this Rule shall not apply to the following persons if they qualify for
exemption under the condition set forth herein: (c) Officers or members of a managerial staff if they perform the
following duties and responsibilities: (1) The primary duty consists of the performance of work directly related
to management policies of their employer; (2) Customarily and regularly exercise discretion and independent
judgment; (3) [i] Regularly and directly assist a proprietor or a managerial employee whose primary duty
consists of the management of the establishment in which he is employed or subdivision thereof; or [ii] execute
under general supervision work along specialized or technical lines requiring special training, experience, or
knowledge; or [iii] execute under general supervision special assignments and tasks; and (4) who do not
devote more than 20 percent of their hours worked in a work-week to activities which are not directly and
closely related to the performance of the work described in paragraphs (1), (2), and (3) above.

Labor Law 1
incentive leave for 1990.
- He had been sick and was allowed to go on leave without pay for one month but when
he reported to work with a health clearance, he was told to come back another time as he
could not be reinstated immediately. For this reason, he asked for a certificate of
employment from petitioner but it refused to issue the certificate unless he submitted his
resignation.
- MERCIDAR claimed that it was Agao who actually abandoned his work. Agao was
absent without leave for three months. Agao should also be considered field personnel
because: Agaos work is performed away from Mercidars principal place of business,
thus, it has no way of verifying his actual hours of work on the vessel.
- Labor Arbiter: ordered Mercidar to reinstate complainant with backwages, pay his 13th
month pay and incentive leave pay.
- NLRC: affirmed decision of LA.
ISSUE
WON fishing crew members can be classified as field personnel under A82 LC
HELD
NO
- Art. 82. Coverage. - The provisions of this Title [Working Conditions and Rest Periods]
shall apply to employees in all establishments and undertakings whether for profit or not,
but not to government employees, field personnel, members of the family of the employer
who are dependent on him for support, domestic helpers, persons in the personal service
of another, and workers who are paid by results as determined by the Secretary of Labor
in appropriate regulations.
Field personnel shall refer to non-agricultural employees who regularly perform their
duties away from the principal place of business or branch office of the employer and
whose actual hours of work in the field cannot be determined with reasonable certainty.
- In the case of Union of Filipro Employees (UFE) v. Vicar, SC explained that the
requirement of whose actual hours of work in the field cannot be determined with
reasonable certainty must be read in conjunction with Rule IV, Book III 26 of the
Implementing Rules.
- The clause whose time and performance is unsupervised by the employer did not
amplify but merely interpreted and expounded the clause whose actual hours of work in
the field cannot be determined with reasonable certainty. There is no contradiction; the
former clause is still within the scope and purview of Art. 82 which defines field personnel.
Hence, in deciding whether or not an employees actual working hours in the field can be
determined with reasonable certainty, query must be made as to whether or not such
employees time and performance is constantly supervised by the employer.
- During the entire course of their fishing voyage, fishermen employed by petitioner have
no choice but to remain on board its vessel. Although they perform non-agricultural work
away from petitioners business offices, the fact remains that throughout the duration of
their work they are under the effective control and supervision of petitioner through the
vessels patron or master as the NLRC correctly held.
Disposition Petition DISMISSED.

AUTO BUS TRANSPORT SYSTEMS INC V BAUTISTA


458 SCRA 578
CHICO-NAZARIO; May 16, 2005
NATURE
Petition for Review on Certiorari
FACTS
- Since 24 May 1995, respondent Antonio Bautista has been employed by petitioner Auto
Bus Transport Systems, Inc. (Autobus), as driver-conductor with travel routes ManilaTuguegarao via Baguio, Baguio- Tuguegarao via Manila and Manila-Tabuk via Baguio.
Respondent was paid on commission basis, 7% of the total gross income per travel, on a
twice a month basis.
- On January 3, 2000, the bus that the respondent was driving accidentally bumped the
rear portion of Autobus No. 124, as the latter vehicle suddenly stopped at a sharp curve
without giving any warning. Respondent averred that the accident happened because he
was compelled by the management to go back to Roxas, Isabela, although he had not
slept for almost 24 hours. Respondent further alleged that he was not allowed to work
until he fully paid the amount of P75,551.50, representing 30% of the cost of repair of the
damaged buses and that despite respondents pleas for reconsideration, the same was
ignored by management. After a month, management sent him a letter of termination.

26

Rule IV Holidays with Pay


Section 1. Coverage - This rule shall apply to all employees except:
(e) Field personnel and other employees whose time and performance is unsupervised by the employer xxx (Italics
supplied)

A2010

- 122 -

Disini

- Respondent instituted a Complaint for Illegal Dismissal with Money Claims for
nonpayment of 13th month pay and service incentive leave pay against Autobus.
Petitioner, on the other hand, maintained that respondents employment was replete with
offenses involving reckless imprudence, gross negligence, and dishonesty. To support its
claim, petitioner presented copies of letters, memos, irregularity reports, and warrants of
arrest pertaining to several incidents wherein respondent was involved.
- Furthermore, petitioner avers that in the exercise of its management prerogative,
respondents employment was terminated only after the latter was provided with an
opportunity to explain his side regarding the accident.
- Labor Arbiter Tabingan dismissed the complaint for illegal dismissal but ordered
petitioner to pay respondent his 13th month pay from the date of his hiring to the date of his
dismissal, and his service incentive leave pay for all the years he had been in service with
the respondent. Upon appeal, the NLRC deleted the award for 13th month pay and
maintained the award for service incentive leave. The CA affirmed the decision of NLRC.
ISSUES
WON respondent is entitled to service incentive leave
HELD
- A95 LC provides:
Art. 95. RIGHT TO SERVICE INCENTIVE LEAVE
(a) Every employee who has rendered at least one year of service shall be entitled to
a yearly service incentive leave of five days with pay.
Book III, Rule V: SERVICE INCENTIVE LEAVE
SECTION 1. Coverage. This rule shall apply to all employees except:

(d) Field personnel and other employees whose performance is unsupervised by the
employer including those who are engaged on task or contract basis, purely
commission basis, or those who are paid in a fixed amount for performing work
irrespective of the time consumed in the performance thereof; . . .
- The grant of service incentive leave has been delimited by the Implementing Rules and
Regulations of the Labor Code to apply only to those employees not explicitly excluded by
Section 1 of Rule V. According to the Implementing Rules, Service Incentive Leave shall
not apply to employees classified as field personnel. The phrase other employees
whose performance is unsupervised by the employer must not be understood as a
separate classification of employees to which service incentive leave shall not be granted.
Rather, it serves as an amplification of the interpretation of the definition of field personnel
under the Labor Code as those whose actual hours of work in the field cannot be
determined with reasonable certainty.
- The same is true with respect to the phrase those who are engaged on task or contract
basis, purely commission basis. Said phrase should be related with field personnel,
applying the rule on ejusdem generis that general and unlimited terms are restrained and
limited by the particular terms that they follow. Hence, employees engaged on task or
contract basis or paid on purely commission basis are not automatically exempted from
the grant of service incentive leave, unless, they fall under the classification of field
personnel.
- Therefore, petitioners contention that respondent is not entitled to the grant of service
incentive leave just because he was paid on purely commission basis is misplaced. What
must be ascertained in order to resolve the issue of propriety of the grant of service
incentive leave to respondent is whether or not he is a field personnel.
- According to A82 LC, field personnel shall refer to non-agricultural employees who
regularly perform their duties away from the principal place of business or branch office of
the employer and whose actual hours of work in the field cannot be determined with
reasonable certainty. This definition is further elaborated in the Bureau of Working
Conditions (BWC), Advisory Opinion to Philippine Technical-Clerical Commercial
Employees Association[10] which states that:
As a general rule, field personnel are those whose performance of their job/service is
not supervised by the employer or his representative, the workplace being away from
the principal office and whose hours and days of work cannot be determined with
reasonable certainty; hence, they are paid specific amount for rendering specific
service or performing specific work. If required to be at specific places at specific
times, employees including drivers cannot be said to be field personnel despite the fact
that they are performing work away from the principal office of the employee.
- the definition of a field personnel is not merely concerned with the location where the
employee regularly performs his duties but also with the fact that the employees
performance is unsupervised by the employer. As discussed above, field personnel are
those who regularly perform their duties away from the principal place of business of the
employer and whose actual hours of work in the field cannot be determined with
reasonable certainty. Thus, in order to conclude whether an employee is a field
employee, it is also necessary to ascertain if actual hours of work in the field can be
determined with reasonable certainty by the employer. In so doing, an inquiry must be
made as to whether or not the employees time and performance are constantly
supervised by the employer.
- It is of judicial notice that along the routes that are plied by these bus companies, there
are its inspectors assigned at strategic places who board the bus and inspect the

Labor Law 1
passengers, the punched tickets, and the conductors reports. There is also the
mandatory once-a-week car barn or shop day, where the bus is regularly checked as to its
mechanical, electrical, and hydraulic aspects, whether or not there are problems thereon
as reported by the driver and/or conductor. They too, must be at specific place and
specified time, as they generally observe prompt departure and arrival from their point of
origin to their point of destination. In each and every depot, there is always the Dispatcher
whose function is precisely to see to it that the bus and its crew leave the premises at
specific times and arrive at the estimated proper time. These, are present in the case at
bar. The driver, the complainant herein, was therefore under constant supervision while in
the performance of this work. He cannot be considered a field personnel.
- Respondent is not a field personnel but a regular employee who performs tasks usually
necessary and desirable to the usual trade of petitioners business. Accordingly,
respondent is entitled to the grant of service incentive leave.

RATIONALE EXEMPTION
RED V COCONUT PRODUCTS LTD V CIR (TANGLAW
NG PAGGAWA LABOR UNION)
17 SCRA 553
BENGZON; June 30, 1966
NATURE
Petition for review of CIR decision and MFR
FACTS
- Red V Coconut Products, Ltd has a desiccated coconut factory wherein 800 of its
workers were members of members of Tanglaw ng Paggawa labor union. The workers
were classified into 2 groups: the 3-shift group and the 2-shift group.
- In 1958 and 1961, the company and labor union entered into a CBA wherein they agreed
on payment for differentials to night shift workers, as follows: 35c for 2 nd shift, 55c for 3rd
shift.

- In 1962, Tanglaw ng Paggawa and member-workers who belong to the 2-shift group filed
a petition with CIR arguing that they should receive what the 2nd and 3rd shifts of the 3-shif
group, combined, receive as differential pay (90c = 55c + 35c) since their nightwork is
equivalent to the nightwork of the 2nd and 3rd shift of 3-shift group combined. They were
therefore asking for an additional 35c for work done by them from 4pm to 4am. On the
other hand, Red V claims that CIR has no jurisdiction over the case since the claim
asserted by Tanglaw is a simple money claim and that an interpretation of a contract
(CBA) is involved (should have gone to CFI).
- CIR: had jurisdiction since it involved unpaid overtime pay of laborers still employed by
the company.
- Tanglaw filed another petition alleging unfair labor practice for refusing to grant 15 days
leave with pay to members of the union in violation of CBA.
- CIR: workers were engaged in pakiao (piece-work basis) and should not be entitled to
overtime pay under the Eight-Hour labor law (Sec. 2, CA444); that their petition for nigh
shift differentials based on CBA is meritorious because the company having paid night
differentials indiscriminately to the night shift workers of 3-shift group and 2-shift group
alike, the payments should be uniform and equal for the night shifts of both groups(90c).
pay deficiency to workers of 2-shift group.
ISSUES
1. WON the CIR has jurisdiction over the case
2. WON the Eight-Hour Labor Law regarding payment of overtime compensation to pieceworkers is applicable in the case
HELD
1. YES
Ratio To determine the issue of the jurisdiction, resort is to be made to the allegations in
the petition or complaint.
Reasoning
- Red V claims that the case involves mere money claim. The petition did not expressly
mention the Eight-Hour Labor Law. Still, (1) respondents are working for the company and
(2) these workers work in 2 shifts, approximately 12 hours each shift. Therefore, the

A2010

- 123 -

Disini

petition is one for overtime pay by workers still employed by the company which falls
within the jurisdiction of the CIR.
Obiter
- petitioner presented evidence which does not appear from the petition or complaint filed
with CIR (proving that workers were engaged on a piece-work basis) so it cannot affect
CIRs jurisdiction over the case which was already acquired. Jurisdiction, once acquired,
continues until final adjudication of the litigation.
2. YES
Ratio Although the Eight-Hour Labor Law provides that it does not cover those workers
who prefer to be paid on piece-work basis (Sec. 2, CA 444), nothing in said law precludes
an agreement for the payment of overtime precludes an agreement for the payment of
overtime compensation to piece-workers. And in agreeing to the provision for payment of
shift differential to the petitioners-workers aforementioned, in the bargaining agreement,
as well as in actually paying to them said differentials, though not in full, the company in
effect freely adhered to an application and implementation of the Eight-Hour Labor Law, or
its objectives, to said workers.
Reasoning
- It should be observed that while the provision in the bargaining agreements speaks of
shift differentials for the "second shift" and the "third shift" and 2-shift group has no third
shift, said 2-shift group has a second shift, which performs work equivalent to that of the
corresponding shifts of 3-shift group. It follows that respondent court did not err in ordering
the company to pay the full and equivalent amount of said differentials (P .90)
corresponding, under the bargaining agreements, to the workers who performed 12 hours
of work, from 4 P.M. to 4 A.M.
- The laborers are not strictly under the full concept of piece-workers because their hours
of work are fixed by the employer. Piece workers were excluded from the Eight-Hour
Labor Law because said workers are paid depending upon the work they do
"irrespective of the amount of time employed" in doing said work. Such freedom as
to hours of work does not apply to the respondent workers since they are assigned by the
employer to work in two shifts for 12 hours each shift. Thus it cannot be said that for all
purposes these workers fall outside the law requiring payment of compensation for work
done in excess of eight hours. At least for the purpose of recovering the full
differential pay stipulated in the bargaining agreement as due to laborers who
perform 12 hours of work under the night shift, said laborers should be deemed pro
tanto
or
to
that
Group Hrs of work
differentials Payment
extent within
st
1 shift (8hrs)
4am-12nn
None
the
scope of the
3-shift
2nd shift (8hrs)
12nn-8pm
35c
afore-stated
group
3rd shift (8hrs)
8pm-4am
55c
law.
2-shift
1st shift (12hrs)
4am-4pm
None
Disposition
group
2nd shift (12hrs)
4pm-4am
55c
decision
and
resolution of the
Court of Industrial Relations under review are affirmed.

5.03 NORMAL HOURS OF WORK


5.04 HOURS WORKED
IDLE TIME
NATIONAL DEVELOPMENT CO V CIR
6 SCRA 763
REGALA; November 30, 1962
FACTS
- At the National Development Co. (NDC), there were four Eight-hour shifts of work, each
one having a 1-hour mealtime period. Petitioner nevertheless credited the workers with
and paid them for the full 8-hour shift. However, since 1953, whenever workers continued
working through the next shift, petitioner only paid them for 6 hours, claiming the 2 hours
for meals should not be compensated. The National Textile Workers Union (NTWU)
disagreed and asked for an order for overtime pay with the CIR which was granted.
- Petitioner appealed, contending that (1) the CIR had no jurisdiction over claims of
overtime compensation and that (2) the CIR did not make a correct appraisal of the facts
in holding that mealtime periods should be included in overtime work.
ISSUES
1. WON the CIR had jurisdiction over claim of overtime compensation
2. WON the mealtime periods should be considered working time

Labor Law 1
HELD
1. YES
- It was recently held in Campos v. Manila Railroad Co. that the ff are the requisites for
jurisdiction: (1) there must exist an employer-employee relationship between the parties or
the claimant must seek reinstatement; (2) the controversy must relate to a case certified
by the President to the CIR as one involving national interest, or must have a bearing on
an unfair labor practice charge, or must arise either under the Eight-Hour Labor Law, or
under the Minimum Wage Law. Absent either requisite renders the claim a mere money
claim coming under the jurisdiction of regular courts. Because there was clearly an
employer-employee relationship existing between the NDC and the union members and
the claim was based on the Eight-Hour Labor Law, CIR had jurisdiction of over the case.
2. YES
- As stated in Sec 1 of Com. Act No. 444, The legal working day for any person employed
by another shall be of not more than 8 hours daily. When the work is not continuous, the
time during which the laborer is not working and can leave his working place and can rest
completely shall not be counted. It is clear from the provision that idle time spent resting
and during which an employee may leave the workplace is not counted as working time
only where the work is broken or not continuous. In this case, the CIR's finding that work
in the NDC was continuous and did not permit employees and laborers to rest completely
is not without basis in evidence and following the Courts earlier rulings, these findings are
not to be disturbed.
- In addition, because petitioner failed to serve a copy of its motion for reconsideration to
NTWU as required by Sec. 15 of the rules of the CIR, there is no decision of the CIR en
banc that petitioner can bring to the Court for review.
Disposition CIR order and resolution are AFFIRMED and the appeal DISMISSED

A2010

- 124 -

Disini

therefore, exempt from paying additional remuneration or compensation for work


performed on Sundays and legal holidays. (Sec4 CA No.444)
- For the reasons above, employees are only entitled to receive overtime pay for work
rendered in excess of 8hrs on ordinary days including Sundays and legal holidays. The
company was able to prove that it has paid its employees for such overtime wok as shown
above. It was only a matter of computation whether the OT services paid cover actual OT
work performed equivalent to 25% (min rate fixed by law in the absence of other proof to
justify the granting of more beyond said min rate.
- Demands nos. 11 & 12 were denied and LSC was only ordered to pay separation pay
and OT after finding that the suspensions and dismissals were for valid legal grounds.
- LMDU filed MFR to declare that those who work from 6am-6pm were entitled to 4hrs OT
pay, that the time allotted for their meals should not be deducted from the said 4hrs, that
the amounts set aside for the meals of should be considered as part of their actual
compensation, that the employees separated without just cause be paid their unearned
wages from the time of decision became final, and for other relief.
- LSC filed MFR in so far as it interpreted that the period during which a seaman aboard a
tugboat should be considered as working time for the purposes of the Eight-Hour-Labor
Law
- The Court made a resolution, in pursuance of CA 103 as amended, modifying the
previous judgment saying that the 4hrs of OT work included in the regular daily sked of
^am-6pm should be paid independently of the so-called coffee-money after finding that
said extra amount were given to some tugboat crew members for work performed beyond
6pm. LSCs MFR was denied.
- LSC filed the present petition of certiorari when CIR ruled that the 20mins rest given the
claimants after mealtime should not be deducted from the 4hrs of OT work performed.
- LMLU filed a MTD which SC considered as an answer, alleging that the CIR decision
(under consideration in the case) does not present any question of law, the issues being
purely factual. CIR judges also asserted that there is no question of law.

ISSUES

LUZON STEVEDORING CO V LUZON MARINE


DEPARTMENT UNION
101 Phil 257
FELIX; April 29 1957
NATURE
Petition for certiorari filed by Luzon Stevedoring Co (LSC) to review a resolution by CIR.
FACTS
- Luzon Marine Development Union (LMDU) filed a petition with the CIR containing several
demands against LSC, among which were the petition for full recognition of the right of
collective bargaining, close shop and check off.
-While the case was still pending, the union conducted a strike which was ruled down as
illegal. Due to the ruling, the union filed a constancia with the CIR praying that the
remaining unresolved demands of the union be presented in their original petition, be
granted. Other demands included the OT pay, payment of the employees who have not
received their pay corresponding to the second half of Dec 941, reinstatement of certain
individuals suspended without justifiable cause.
-The unresolved demands are the ff: performed work in excess of 8 hrs be paid an OT pay
of 50% of the regular rate and that work performed on Sundays and holidays be paid 2x
the regular rate (point no. 2); that all the officers, etc who havent received their pay
corresponding to the 2nd half of Dec be paid accordingly (point no. 7); that Ciriaco, etc who
have been suspended without justifiable cause for union activities be reinstated (point no
11); that all officers, etc who have been discharged because of union activities be
reinstated (point no. 12).
- Due to these demands the case was set for hearing. The original intervenor, moved for
the withdrawal of said Union from the case, which motion was granted by the Court.
- TC found that the company gave employees 3 free meals every day and about 20
minutes rest after each mealtime; that they worked from 6am-6pm every day including
Sundays and holidays, and for work performed in excess of 8 hours, the officers, patrons
and radio operators were given overtime pay in the amount of P4 each and P2 each for
the rest of the crew up to March, 1947, and after said date, these payments were
increased to P5 and P2.50, respectively, until the time of their separation or the strike; that
when the tugboats underwent repairs, their personnel worked only 8 hours a day
excluding Sundays and holidays; that although there was an effort on the part of claimants
to show that some had worked beyond 6pm, the evidence was uncertain and indefinite
and that demand was denied; that Company, by the nature of its business and as defined
by law is considered a public service operator by the Public Service Commission and,

1. WON hours of work as applied to dryland laborers are equally applicable to seamen
2. WON a person should be penalized for following an opinion of the Sec of Justice
absent any pronouncement
3. WON compensation for 2 years agreed upon legally and deemed retroactively is
presumed to constitute full payment of all services rendered (including OT pay) provided
employees with full knowledge of the law, voluntarily agreed to work in consideration of
such wage and continued working without protest
4. WON members of Union having expressly manifested acquiescence over a period of
almost two years with reference to the sufficiency of their wages and having made no
protest whatsoever with reference to said compensation are estopped from claiming OT
compensation
5. Granting, without conceding, that any OT is due, what is the extent and rule of
retroactivity as set forth and established by the precedents and policies of the CIR as
affirmed by the SC?
6. WON the grant of sizeable amount of back OT wages by the CIR is in consonance with
the dictates of public policy on recovery and financial stability
7. WON a CIR resolution, unsupported in fact and in law, can be declared illegal

HELD
1. YES
Ratio There is no need to set a different criterion to be applied to the seamen. A laborer
doesnt need to leave the premises in order that his period of rest shall not be counted,
such that it is enough that he ceases to work, may rest completely and leave his spot
where he actually has stays while working.27
Reasoning
- LSC claims that the 12hr work is not continuous but interrupted or broken. It was found
true however they could not just leave due to the nature of their duty preventing them to
leave the tugboats
2. The court cannot pass upon the issue because there was nothing on the record saying
that would support the petitioners assertion that in dealing with its employees, it was
guided by such opinion.
3 & 4. Ratio NO. The right of the laborers to OT cannot be waived. CA444, Sec 628
applies
Reasoning
27

CA 444 (Eight-Hour-Labor Law) Sec 1 is applied to contemporary regulations issued by administrative authorities.
SEC. 1. The legal working day for any person employed by another shall be of not more than eight hours daily. When
the work is not continuous, the time during which the laborer is not working AND CAN LEAVE HIS WORKING PLACE
and can rest completely, shall not be counted.

28

SEC. 6. Any agreement or contract between the employer and the laborer or employee contrary to the provisions of
this Act shall be null and void ab initio.

Labor Law 1
- Estoppel and laches cannot be invoked against employees or laborers in an action for
the recovery of compensation for past OT work. It would be contrary to the spirit of the
Eight-Hour Labor Law such that the laborers cannot waive their right to extra
compensation. Secondly, the law principally obligates the employer to observe it that it
punishes the employer for its violation and leaves the employee free and blameless.
Thirdly, the employee or laborer is in such a disadvantageous position as to be naturally
reluctant or even apprehensive in asserting a claim which may cause the employer to
devise a way for exercising his right to terminate the employment.
- Moreover, if the principle of estoppel and laches is to be applied, it would bring about a
situation whereby the employee or laborer who can not expressly renounce the right to
extra compensation, may be compelled to accomplish the same thing by mere silence or
lapse of time, frustrating the purpose of the law by indirection. (Manila Terminal Co. v CIR)
- In the case at hand, the complaining laborers have even declared before the filing of the
case that they informed the overseer of the LSC that they have been working OT claiming
for their compensation. The records do not show that some members of LMLU had
received salaries les than the min wage.
5. Ratio The employee in rendering extra services at the request of his employer has the
right to assume that the latter has complied with the requirements of the law.
Reasoning
- Fear of possible unemployment inhibits the employee from asserting his right under the
law. To allow the workingman to be compensated only from the date of filing would be to
penalize him for his silence. Although LSC claims that the computation of the OT in
arrears should be based from the filing of the petition this case is not in point.
6. Ratio Separation of powers
Reasoning
- The courts cannot go outside the field of interpretation so as to inquire into the motives of
Congress in enacting a piece of legislation.
7. YES
- However, the issue under consideration is predicated on a situation not obtaining in the
case at bar. It supposes that the CIR resolution is unsupported, which does not seem to
be the case.
- The CIR is a court with well-defined powers vested by the law creating it and with such
other powers that pertain to a court of justice. The general rule that before judgment
becomes final, the Court that rendered the same may alter or modify it so as to conform
with the law and the evidence, is applicable to the CIR. The law also provides that after a
judge of the CIR, duly designated by the Presiding Judge therein to hear a particular case,
had rendered a decision, any aggrieved party may request for reconsideration thereof and
the judges of said Court shall sit together, the concurrence of the 3 of them being
necessary for the pronouncement of a decision, order or award. In virtue of these rules
and upon MFR presented by both parties, the resolution subject of the present petition
was issued, the Court en banc finding it necessary to modify a part of the decision, which
is clearly within its power to do.
Disposition on the strength of the foregoing consideration, the resolutions of CIR
appealed from are hereby affirmed

CONTINUOUS WORK
STATES MARINE CORP V CEBU SEAMEN'S ASSOC
7 SCRA 294
PAREDES; February 28, 1963
NATURE
Writ for certiorari
FACTS
- States Marine Corporation and Royal Line, Inc. (SMC) were engaged in the business of
marine coastwise transportation, employing steamships of Philippine registry. They had a
collective bargaining contract with Cebu Seamen's Association, Inc. (CSA)
- On September 12, 1952, the SMC filed with the CIR, a petition against SMC. The Union
alleged that the officers and men working on board the petitioners' vessels have not been
paid their sick leave, vacation leave and overtime pay; that CSA threatened or coerced
them to accept a reduction of salaries, observed by other shipowners; that after the
Minimum Wage Law had taken effect, the petitioners required their employees on board
their vessels, to pay the sum of P.40 for every meal, while the masters and officers were
not required to pay their meals.
- CSA answered by saying that they have suffered financial losses in the operation of their
vessels and that there is no law which provides for the payment of sick leave or vacation
leave to employees or workers of private firms; that as regards the claim for overtime pay,
the petitioners have always observed the provisions of Comm. Act No. 444, (Eight-Hour
Labor Law), notwithstanding the fact that it does not apply to those who provide means of
transportation.
- A decision was rendered in favor of the union. In its decision, it also held that Pepito

A2010

- 125 -

Disini

Severino, one of its boatsmen, rendered overtime work and was not paid. The motion for
reconsideration, having been denied, the companies filed the present writ of certiorari, to
resolve legal question involved.
ISSUES
WON the CIR erred in holding that Severino Pepito, a boatsman, had rendered overtime
work notwithstanding the provisions of section 1, of C.A. No. 444
HELD
NO
- The provisions of sec. 1, of Comm. Act No. 444, states that "When the work is not
continuous, the time during which the laborer is not working and can leave his working
place and can rest completely shall not be counted." Severino Pepito categorically stated
that he worked during the late hours of the evening and during the early hours of the day
when the boat docks and unloads. Aside from the above, he did other jobs such as
removing rusts and cleaning the vessel, which overtime work totalled to 6 hours a day, and
of which he has not been paid as yet. Sec. 1, of Comm. Act No. 444 find no application in
his case.
Disposition petition is dismissed, with costs against the petitioners.

WAITING TIME
ARICA V NLRC
170 SCRA 776
PARAS; February 28, 1989
FACTS
- This case stemmed from a complaint filed on April 9, 1984 against private respondent
Stanfilco for assembly time, moral damages and attorney's fees.
- After the submission by the parties of their respective position papers, Labor Arbiter
Pedro C. Ramos rendered a decision dated October 9, 1985 in favor of private respondent
STANFILCO, holding that: the thirty-minute assembly time long practiced cannot be
considered waiting time or work time and, therefore, not compensable, has become the
law of the case which can no longer be disturbed without doing violence to the timehonored principle of res-judicata.
- NLRC upheld the Labor Arbiters' decision stating that the customary functions referred to
in the provision of the agreement includes the long-standing practice and institutionalized
non-compensable assembly time. This, in effect, estopped complainants from pursuing
this case.
- NLRC denied motion for reconsideration.
- Hence this petition for review on certiorari filed on May 7, 1987.
- Petitioners: the preliminary activities as workers of respondents STANFILCO in the
assembly area is compensable as working time (from 5:30 to 6:00 o'clock in the morning)
since these preliminary activities are necessarily and primarily for private respondent's
benefit: roll call, getting individual work assignments from the foreman; accomplishing the
Laborer's Daily Accomplishment Report; getting work materials; travel to the field bringing
with them their tools, equipment and materials.
- Respondent: the instant complaint is not new, the very same claim having been brought
against herein respondent by the same group of rank and file employees in the case of
Associated Labor Union and Standard Fruit Corporation when ALU was the bargaining
agent of respondent's rank and file workers.
ISSUE
WON NLRC committed a grave abuse of discretion in its resolution of Dec 17, 1986
HELD
- The decision of the Minister of Labor, on May 12, 1978 in the aforecited case (Associated
Labor Union vs. Standard (Phil.) Fruit Corporation made significant findings of facts and
conclusions on the matter. The Minister of Labor held: The thirty (30)-minute assembly
time long practiced and institutionalized by mutual consent of the parties under Article IV,
Section 3, of the Collective Bargaining Agreement cannot be considered as waiting time
within the purview of Section 5, Rule I, Book III of the Rules and Regulations Implementing
the Labor Code. ...
- It is clear that herein petitioners are merely reiterating the very same claim which they
filed through the ALU and which records show had already long been considered
terminated and closed by this Court in G.R. No. L-48510. Therefore, the NLRC can not be
faulted for ruling that petitioners' claim is already barred by res-judicata.
- Be that as it may, petitioners' claim that there was a change in the factual scenario which
are "substantial changes in the facts" makes respondent firm now liable for the same claim
they earlier filed against respondent which was dismissed. It is thus axiomatic that the
non-compensability of the claim having been earlier established, constitute the controlling

Labor Law 1
legal rule or decision between the parties and remains to be the law of the case making
this petition without merit.
- As a rule, the findings of facts of quasi-judicial agencies which have acquired expertise
because their jurisdiction is confined to specific matters are accorded not only respect but
at times even finality if such findings are supported by substantial evidence. The records
show that the Labor Arbiters' decision dated October 9, 1985 pointed out in detail the
basis of his findings and conclusions, and no cogent reason can be found to disturb these
findings nor of those of the National Labor Relations Commission which affirmed the
same.

SEPARATE OPINION
SARMIENTO [dissent]
- It is my opinion that res judicata is not a bar.
- The decision penned by then Minister Blas Ople in ALU v. STANFILCO (NLRC Case No.
26-LS-XI-76) relied upon by the respondents as basis for claims of res judicata, is not, to
my mind, a controlling precedent. In that case, it was held that the thirty-minute "waiting
time" complained of was a mere "assembly time" and not a waiting time as the term is
known in law, and hence, a compensable hour of work.
- Precisely, it is the petitioners' contention that the assembly time in question had since
undergone dramatic changes.
- The petitioners have vehemently maintained that in view thereof, the instant case should
be distinguished from the first case. And I do not believe that the respondents have
successfully rebutted these allegations. The Solicitor General relies solely on the decision
of then Minister Ople, the decision the petitioners precisely reject in view of the changes in
the conditions of the parties. The private respondent on the other hand insists that these
practices were the same practices taken into account in ALU v. STANFILCO. If this were
so, the Ople decision was silent thereon.

TRAVEL TIME
RADA V NLRC (PHILNOR)
205 SCRA 69
REGALADO; 1992
NATURE
Special civil Action for certiorari to review NLRC decision
FACTS
- Hilario Rada was hired by private respondent Philnor as driver under a Contract of
Employment for a Definite Period in connection with Philnors contract with the Manila
North Express Extension. The original contract was for a period of about 24 months
effective July 1, 1977. Due to delays in the project, the petitioner was rehired under a
second contract for a period of ten months. Prior to the end of the second contract and as
the project remained uncompleted, petitioner was again offered another contract for
another period of 19 months. This third contract was extended for several times again due
to delays in the project. The last extension was for a period of three months from October
1 1985 to December 31m 1985. As the project was in fact completed at the end of the end
of the final extension , petitioner sought personnel Clearance from the company. At that
time, he also received the cash equivalent of his unsed leaves and financial assistance.
Also at that time, the petitioner signed a Release, Waiver and Quitclaim.
- Rada, on May 20, 1987, filed before the NLRC a complaint against private respondent
for nonpayment of separation pay and overtime. This complaint was subsequently
amended to include allegation of illegal termination.
- The labor arbiter found for the petitioner but this was reversed by the NLRC. Hence the
appeal to the SC
ISSUE
1. WON the petitioner was a permanent employee of the private respondent
2. WON the petitioner is entitled to overtime pay for three hours per work day
HELD
1. NO
- The Court ruled that the petitioner was a project employee and thus his period of
employment was fixed for a specific project or undertaking. Once the specific project or
undertaking has been completed, the services of the employee not being required, the

A2010

- 126 -

Disini

employment is terminated, subject only to the obligation of the employer to render a report
on the termination of the employment..
2. YES
- It was the job of Rada to pick up and drop off employees of the project at certain
specified points along EDSA. Hence the time he spent in doing this work should be
included in determining the number of hours he had worked. Rada is hence entitled to the
overtime pay claimed.(Note: Ordinarily, the travel time of employees from house to
place of work and vice versa is not included as part of time worked. Time of work
starts when the employee reports at the place of workand ends when he leaves the
same place.)
Disposition Subject to the modification wih regard the overtime pay, the decision of the
NLRC is affirmed.

ENTRY TIME CARDS


PRANGAN V NLRC (MASAGANA SECURITY SERVICES
CORP, PADILLA)
289 SCRA 142
ROMERO; April 15, 1998
FACTS
- November 4, 1980 - MASAGANA SECURITY SERVICES CORP hired PRANGAN as
one of its security guards assigned to the Cat House Bar and Restaurant with a monthly
salary of P2,000.00 until its closure on August 31, 1993.
- May 4, 1994 - PRANGAN filed a complaint against MASAGANA for underpayment of
wages, non-payment of salary from August 16-31, 1993, overtime pay, premium pay for
holiday, rest day, night shift differential, uniform allowance, service incentive leave pay and
13th month pay from the year 1990 to 1993.
- MASAGANA rejected PRANGANS claim alleging it merely acted as an agent of the
latter in securing his employment at the Cat House Bar and Restaurant. Thus, the liability
for the claims of the petitioner should be charged to Cat House Bar and its owner, being
his direct employer.
- Labor Arbiter > brushed aside MASAGANAS contention that it was merely an agent of
the petitioner and ordered to pay within 10 days P9,932.16 premium pay for holiday and
rest days, night shift differential, service incentive leave pay, 13th month pay, uniform
allowance, and unpaid salary and dismissed other claims either for the reason of
prescription and/or lack of merit
- PRANGAN was not satisfied with the LAs decision and he appealed to NLRC contending
that LA erred in concluding that he only worked for four hours and not twelve hours a day.
But, NLRC dismissed his appeal for failure to file the same within ten-day reglementary
period.
- PRANGAN filed a MFR which, in the interest of justice, was favorably granted by the
NLRC resulting in the reinstatement of his appeal. But it was short-lived as the appeal
was eventually dismissed for lack of merit
- PRANGAN is now before us imputing GRAVE ABUSE OF DISCRETION on the part of
NLRC (a) declaring that he rendered only four hours and not twelve hours of work, and (b)
affirming the monetary award.
- As proof of petitioners actual hours of work, MASAGANA submitted the DAILY TIME
RECORDS allegedly signed by the PRANGAN himself showing that he only worked four
hours daily.
- PRANGAN argues that these daily time records were falsified for the simple reason that
he was not required to submit one. He further stressed that, assuming such documents
exist, its authenticity and due execution are questionable and of doubtful source.
ISSUE
WON NLRC erred in finding Prangans work hours to be four hours instead of twelve
HELD
Ratio Matters concerning an employees actual hours of work are within the ambit of
management prerogative. However, when an employer alleges that his employee works
less than the normal hours of employment as provided for in the law, he bears the burden
of proving his allegation with clear and satisfactory evidence.
Reasoning
- Findings of fact of quasi-judicial bodies like the NLRC, particularly when they coincide
with those of the Labor Arbiter, are accorded with respect even finality if supported by
substantial evidence. Substantial evidence is defined as such amount of relevant evidence
which a reasonable mind might accept as adequate to justify a conclusion. Absent such
quantum of evidence, the Court is not precluded from making its own independent
evaluation of facts.
- NLRC, in declaring that PRANGANonly worked for four hours, relied solely on the
supposed daily time records of the petitioner submitted by MASAGANA. We, however, are

Labor Law 1
of the opinion that these documents cannot be considered substantial evidence as to
conclude that petitioner only worked for four hours.
- As PRANGANS employer, MASAGANA has unlimited access to all relevant documents
and records on the hours of work of the petitioner. Yet, even as it insists that petitioner
only worked for four hours and not twelve, no employment contract, payroll, notice of
assignment or posting, cash voucher or any other convincing evidence which may attest to
the actual hours of work of the petitioner were even presented. Instead, what the private
respondent offered as evidence were only petitioners daily time record, which the latter
categorically denied ever accomplishing, much less signing.
- daily time record showed that PRANGAN started work at 10PM and would leave his post
at exactly 2AM. Obviously, such unvarying recording of a daily time record is improbable
and contrary to human experience. It is impossible for an employee to arrive at the
workplace and leave at exactly the same time, day in day out. The very uniformity and
regularity of the entries are badges of untruthfulness and as such indices of dubiety.
- Personnel data sheet of the PRANGAN, duly signed by the formers operation manager,
it shows on its face that the latters hours of work are from 7PM to 7AM or twelve hours a
day. Hence, MASAGANA is estopped from assailing the contents of its own documents.
- attendance sheets of Cat House Bar and Restaurant showed that PRANGAN worked
from 7:00 p.m. to 7:00 a.m. daily, documents which were never repudiated by
MASAGANA.
- As is well-settled, if doubts exist between the evidence presented by the employer and
the employee, the scales of justice must be tilted in favor of the employee. Since it is a
time-honored rule that in controversies between a laborer and his master, doubts
reasonably arising from the evidence, or in the interpretation of agreements and writings
should be resolved in the formers favor.
Disposition instant petition is hereby GRANTED. NLRC decision on July 31, 1996 is
hereby VACATED. Whatever money claims due to the petitioner shall be computed on the
basis of a twelve-hour daily work schedule. For this purpose, the case is hereby
REMANDED to the Labor Arbiter for immediate recomputation of said claims in
accordance with the foregoing findings.

NICARIO V NLRC (MANCAO)


193 SCRA 603
ROMERO; September 17, 1998
NATURE
special civil action for certiorari to review decision of NLRC
FACTS
- Nicario was an employee of Mancao Supermarket (which was managed by Antonio
Mancao). She was terminated, and she filed a complaint for illegal dismissal, with prayer
for backwages, overtime pay, and unpaid wages.
- She alleges that she reported to work everyday from 7:30am till 7:30pm (12 hrs a day),
thus rendering overtime work for 4hrs each day.
- The labor arbiter awarded her OT pay, by taking judicial notice of the fact that all Mancao
establishments operate from 8am til 8pm. On appeal by the supermarket to the NLRC, the
commission deleted the award for OT pay, giving credence to the daily time records
(DTRs) presented by the company, which showed that Nicario worked for 8 hrs a day only,
from 9am-12nn and 2pm-7pm, each day. Also, the NLRC absolved the liability of the
manager. Nicario appealed to the SC.
ISSUES
1. WON Nicario must be given overtime pay
2. WON the manager must be held solidarily liable with the corporation
HELD
1. YES
Ratio The Daily Time Record with uniform entries day in and day out, cannot be given
much credence.
Reasoning
- While the supermarket submitted the daily time records of Nicario to show that she
rendered work for only 8hrs a day, it did not refute nor seek to disprove the judicial notice
taken by the Labor Arbiter that Mancao establishments, including the establishment where
NIcario worked, opens 12 hrs a day, opening at 8:00 a.m. and closing at 8:00 p.m.
In evaluating the evidentiary value of daily time records, especially those which show
uniform entries with regard to the hours of work rendered by an employee, the court has
ruled that such unvarying recording of a daily time record is improbable and contrary to
human experience. It is impossible for an employee to arrive at the workplace and leave at
exactly the same time, day in day out. The uniformity and regularity of the entries are
'badges of untruthfulness and as such indices of dubiety. On the other hand, the
supermarket failed to present substantial evidence, other than the disputed DTRs, to
prove that Nicario indeed worked for only 8 hrs a day.
2. NO

A2010

- 127 -

Disini

Ratio There appearing to be no evidence that Antonio Mancao acted maliciously or


deliberately in the non-payment of benefits to Nicarion, he cannot he held jointly and
severally liable with Mancao supermarket.
Reasoning
- The general rule is that officers of a corporation are not personally liable for their official
acts unless it is shown that they have exceeded their authority. However, the legal fiction
that a corporation has a personality separate and distinct from stockholders and members
may be disregarded if it is used as a means to perpetuate fraud or an illegal act or as a
vehicle for the evasion of an existing obligation, the circumvention of statutes, or to
confuse legitimate issues.
- In this case, there is no showing that Antonio Mancao, as manager of respondent
company, deliberately and maliciously evaded the respondent's company financial
obligation to the petitioner.
Disposition Petition PARTLY GRANTED. Nicario awarded overtime pay, but manager
held not to be solidarily liable with corporation

AKLAN ELECTRIC COOP INC V NLRC

5.05 MEAL PERIOD


MEAL TIME - FREE TIME
PAN AMERICAN WORLD AIRWAYS SYSTEM (PHILS) V
PAN AMERICAN EMPLOYEES ASSOCIATION
1 SCRA 527
REYES; February 23, 1961
NATURE
Appeal by certiorari from the decision of the Court of Industrial Relations
FACTS
- After failing to reach a compromise with their company, PAN AM workers filed a case
against PAN AM PHILIPPINES to seek redress for their grievances at the workplace. They
demanded payment of overtime compensation, inclusion in the computation of overtime
pay the one-hour meal period during overtime, adoption of straight eight hour shift
inclusive of the meal period.
- It appears that most of their grievances were addressed by the Court of Industrial
Relations prompting the management to make this appeal.
ISSUES
1. WON CIR had jurisdiction over claims of overtime pay
2. WON the one-hour meal period should be considered as overtime work
HELD
1. YES
Ratio The Industrial Court may properly take cognizance of overtime claims, if at the time
of the petition, the complainants were still in the service of the employer, or, having been
separated from such service, should ask for reinstatement; otherwise, such claims should
be brought before the regular courts.
Reasoning
- overtime compensation were still in the service of the company when the case was filed,
the jurisdiction of the Court of Industrial Relations cannot be assailed. In fact, since it is not
pretended that, thereafter, the complainants were discharged or otherwise terminated their
relationship with the company for any reason, all of said complainants could still be with
the company up to the present.
2. NO
Ratio The one-hour meal period shall be considered as part of overtime work if the
evidence shows that during that period, the company may call resting employees to render
some services.
Reasoning

Labor Law 1
- Petitioner herein claims that the one-hour meal period should not be considered as
overtime work (after deducting 15 minutes), because the evidence showed that
complainants could rest completely and were not in any manner under the control of the
company during that period. The court below found, on the contrary, that during the socalled meal period, the mechanics were required to stand by for emergency work; that if
they happened not to be available when called, they were reprimanded by the leadman;
that as in fact it happened on many occasions, the mechanics had been called from their
meals or told to hurry up eating to perform work during this period. Far from being
unsupported by substantial evidence, the record clearly confirms the above factual
findings of the Industrial Court.
- The Industrial Court's order for permanent adoption of a straight 8-hour shift including the
meal period was but a consequence of its finding that the meal hour was not one of
complete rest, but was actually a work hour, since, for its duration, the laborers had to be
on ready call. Of course, if the Company practices in this regard should be modified to
afford the mechanics a real rest during that hour (f. ex., by installing an entirely different
emergency crew, or any similar arrangement), then the modification of this part of the
decision may be sought from the Court below. As things now stand, we see no warrant for
altering the decision.
Disposition The judgment appealed from is affirmed. Costs against appellant.

PHILIPPINE AIRLINES INC V NLRC (PROTACIO,


FABROS)
302 SCRA 582
PUNO; February 2, 1999
NATURE
Appeal from the decision of the NLRC
FACTS
- Private respondent was employed as flight surgeon at petitioner company. He was
assigned at the PAL Medical Clinic at Nichols and was on duty from 4:00 in the afternoon
until 12:00 midnight.
- At around 7:00 pm, one night, private respondent left the clinic to have his dinner at his
residence, which was about 5-minute drive away. A few minutes later, the clinic received
an emergency call from the PAL Cargo Services. One of its employees, Mr. Manuel
Acosta, had suffered a heart attack. The nurse on duty, Mr. Merlino Eusebio, called private
respondent at home to inform him of the emergency. The patient arrived at the clinic at
7:50 pm and Mr. Eusebio immediately rushed him to the hospital. When private
respondent reached the clinic at around 7:51 pm, Mr. Eusebio had already left with the
patient. Mr. Acosta died the following day. PAL Medical Director ordered the Chief Flight
Surgeon to conduct an investigation. The Chief Flight Surgeon, in turn, required private
respondent to explain why no disciplinary sanction should be taken against him. Private
respondent asserted that he was entitled to a 30 min. meal break; that he immediately left
his residence upon being informed by Mr. Eusebio about the emergency and he arrived at
the clinic a few minutes later; that Mr. Eusebio panicked and brought the patient to the
hospital without waiting for him.
- The management charged private respondent with abandonment of post while on duty.
In his answer, private respondent reiterated that he only left the clinic to have his dinner at
home. In fact, he returned to the clinic at 7:51 in the evening upon being informed of the
emergency. PAL decided to suspend private respondent for three months.
- Private respondent filed a complaint for illegal suspension against petitioner.
- Labor Arbiter Romulus A. Protasio rendered a decision declaring the suspension of
private respondent illegal.
- Petitioner appealed to the NLRC. The NLRC, however, dismissed the appeal.
ISSUES
1. WON the 3-month suspension of private respondent should be nullified despite the fact
that the private respondent has committed an offense that warranted the imposition of
disciplinary action
2. WON moral damages should be awarded
HELD
1. YES
Ratio The eight-hour work period does not include the meal break. Nowhere in the law
may it be inferred that employees must take their meals within the company premises.
Employees are not prohibited from going out of the premises as long as they return to their
posts on time.

A2010

- 128 -

Disini

Reasoning
- A83 and 85 LC read:
Art. 83. Normal hours of work. The normal hours of work of any employee shall not
exceed eight hours a day.
Health personnel in cities and municipalities with a population of at least one million or
in hospitals and clinics with a bed capacity of at least one hundred shall hold regular
office hours for eight hours a day, for five days a week, ...
Art. 85. Meal periods. Subject to such regulations as the Secretary of Labor may
prescribe, it shall be the duty of every employer to give his employees not less than
sixty (60) minutes time-off for their regular meals.
Sec. 7, Rule I, Book III of the Omnibus Rules Implementing the Labor Code further
states:
Sec. 7. Meal and Rest Periods. Every employer shall give his employees...not less
than one (1) hour time-off for regular meals...
- Private respondent left the clinic that night only to have his dinner at his house, which
was only a few minutes' drive away from the clinic. His whereabouts were known to the
nurse on duty so that he could be easily reached in case of emergency. Upon being
informed of Mr. Acosta's condition, private respondent immediately left his home and
returned to the clinic. These facts belie petitioner's claim of abandonment.
2. NO
Ratio Not every employee who is illegally dismissed or suspended is entitled to damages.
As a rule, moral damages are recoverable only where the dismissal or suspension of the
employee was attended by bad faith or fraud, or constituted an act oppressive to labor, or
was done in a manner contrary to morals, good customs or public policy.
Reasoning
- Bad faith does not simply mean negligence or bad judgment. It involves a state of mind
dominated by ill will or motive. It implies a conscious and intentional design to do a
wrongful act for a dishonest purpose or some moral obliquity.
- there is no showing that the management of petitioner company was moved by some evil
motive in suspending private respondent. It suspended private respondent on an honest,
albeit erroneous, belief that private respondent's act of leaving the company premises to
take his meal at home constituted abandonment of post which warrants the penalty of
suspension
Disposition petition is PARTIALLY GRANTED. The portion of the assailed decision
awarding moral damages to private respondent is DELETED. All other aspects of the
decision are AFFIRMED.

5.06 OVERTIME WORK AND OFFSETTING


PROHIBITION
DEFINITION AND RATIONALE OVERTIME PAY
PNB V PNB EMPLOYEES ASSOCIATION
115 SCRA 507
BARREDO; July 30, 1982
NATURE
Appeal from decision of the Court of Industrial Relations (CIR)
FACTS
- PNB and PNB Employees Association (PEMA) had a dispute regarding the proper
computation of overtime pay. PEMA wanted the cost of living allowance (granted in 1958)
and longevity pay (granted in 1961) to be included in the computation. PNB disagreed and
the 2 parties later went before the CIR to resolve the dispute.
- CIR decided in favor of PEMA and held that PNB should compute the overtime pay of its
employees on the basis of the sum total of the employees basic salary or wage plus cost
of living allowance and longevity pay. The CIR relied on the ruling in NAWASA v NAWASA
Consolidated Unions, which held that for purposes of computing overtime compensation,
regular wage includes all payments which the parties have agreed shall be received
during the work week, including differentiated payments for working at undesirable times,
such as at night and the board and lodging customarily furnished the employee. This
prompted PNB to appeal, hence this case.
ISSUE
WON the cost of living allowance and longevity pay should be included in the computation
of overtime pay as held by the CIR
HELD
NO

Labor Law 1
Ratio Overtime pay is for extra effort beyond that contemplated in the employment
contract; additional pay given for any other purpose cannot be included in the basis for the
computation of overtime pay.
- Absent a specific provision in the CBA, the bases for the computation of overtime pay are
2 computations, namely:
1. WON the additional pay is for extra work done or service rendered
2. WON the same is intended to be permanent and regular, not contingent nor temporary
as a given only to remedy a situation which can change any time.
Reasoning
- Longevity pay cannot be included in the computation of overtime pay for the very simple
reason that the contrary is expressly stipulated in the CBA, which constitutes the law
between the parties.
- As regards cost of living allowance, there is nothing in Commonwealth Act 444 [or the 8hour Labor Law, now Art. 87 Labor Code] that could justify PEMAs posture that it should
be added to the regular wage in computing overtime pay. C.A. 444 prescribes that
overtime work shall be paid at the same rate as their regular wages or salary, plus at least
25% additional. The law did not define what is a regular wage or salary. What the law
emphasized is that in addition to regular wage, there must be paid an additional 25% of
that regular wage to constitute overtime rate of pay. Parties were thus allowed to agree
on what shall be mutually considered regular pay from or upon which a 25% premium
shall be based and added to makeup overtime compensation.
- No rule of universal application to other cases may be justifiably extracted from the
NAWASA case. CIR relies on the part of the NAWASA decision where the SC cited
American decisions whose legislation on overtime is at variance with the law in this
jurisdiction. The US legislation considers work in excess of forty hours a week as
overtime; whereas, what is generally considered overtime in the Philippines is work in
excess of the regular 8 hours a day. It is understandably material to refer to precedents
in the US for purposes of computing weekly wages under a 40-hour week rule, since the
particular issue involved in NAWASA is the conversion of prior weekly regular earnings
into daily rates without allowing diminution or addition.
- To apply the NAWASA computation would require a different formula for each and every
employee. It would require reference to and continued use of individual earnings in the
past, thus multiplying the administrative difficulties of the Company. It would be
cumbersome and tedious a process to compute overtime pay and this may again cause
delays in payments, which in turn could lead to serious disputes. To apply this mode of
computation would retard and stifle the growth of unions themselves as Companies would
be irresistibly drawn into denying, new and additional fringe benefits, if not those already
existing, for fear of bloating their overhead expenses through overtime which, by reason of
being unfixed, becomes instead a veritable source of irritant in labor relations.
**Overtime Pay Rationale Why is a laborer or employee who works beyond the regular
hours of work entitled to extra compensation called, in this enlightened time, overtime
pay? Verily, there can be no other reason than that he is made to work longer than what is
commensurate with his agreed compensation for the statutorily fixed or voluntarily agreed
hours of labor he is supposed to do. When he thus spends additional time to his work, the
effect upon him is multi-faceted; he puts in more effort, physical and/or mental; he is
delayed in going home to his family to enjoy the comforts thereof; he might have no time
for relaxation, amusement or sports; he might miss important pre-arranged engagements;
etc. It is thus the additional work, labor or service employed and the adverse effects just
mentioned of his longer stay in his place of work that justify and are the real reasons for
the extra compensation that is called overtime pay.
**Overtime Pay Definition The additional pay for service or work rendered or performed
in excess of 8 hours a day by employees or laborers in employment covered by the 8 hour
Labor Law [C.A. 444, now Art. 87 Labor Code] and not exempt from its requirements. It is
computed by multiplying the overtime hourly rate by the number of hours worked in
excess of eight.
Disposition decision appealed from is REVERSED

CALTEX REGULAR EMPLOYEES V CALTEX (PHILS)


INC
247 SCRA 398
FELICIANO; August 15, 1995
NATURE
Petition for certiorari
FACTS
- On 12 December 1985, petitioner Union and Caltex (Philippines), Inc. (Caltex) entered
into a Collective Bargaining Agreement ("1985 CBA") which was to be in effect until
midnight of 31 December 1988. The CBA included, among others, the following provision:
"ARTICLE III
HOURS OF WORK
In conformity with Presidential Decree 442, otherwise known as the Labor Code of the
Philippines, as amended, the regular work week shall consist of eight (8) hours per day,

A2010

- 129 -

Disini

seven (7) days, Monday through Sunday, during which regular rates of pay shall be paid
in accordance with Annex B and work on the employee's one 'Day of Rest,' shall be
considered a special work day, during which 'Day of Rest' rates of pay shall be paid as
provided in Annex B. Daily working schedules shall be established by management in
accordance with the requirements of efficient operations on the basis of eight (8) hours
per day for any five (5) days. Provided, however, employees required to work in excess
of forty (40) hours in any week shall be compensated in accordance with Annex B of
this Agreement.
- Sometime in August 1986, the Union called Caltex's attention to alleged violations by
Caltex of Annex "B" of the 1985 CBA,
e.g. non-payment of night-shift differential,
non-payment of overtime pay and non-payment at "first day-off rates" for work performed
on a Saturday.
- Caltexs Industrial Relations manager informed Union that differential payments would be
timely implemented.
- In the implementation of the re-computed claims, however, no differential payment was
made with respect to work performed on the first 2 1/2 hours on a Saturday.
Union instituted a complaint for unfair labor practice against Caltex alleging violation of the
provisions of the 1985 CBA.

- Petitioners Claims:
> charged Caltex with shortchanging its employees when Caltex compensated work
performed on the first 2 1/2 hours of Saturday, an employees' day of rest, at regular rates,
when it should be paying at "day of rest" or "day off" rates.
> Caltex was violating the statutory prohibition against off-setting undertime for overtime
work on another day. The employees had been required to render "overtime work" on a
Saturday but compensated only at regular rates of pay, because they had not completed
the eight (8)-hour work period daily from Monday thru Friday.
- Respondents Comments
> denied the accusations of the Union. It averred that Saturday was never designated as a
day of rest, much less a "day-off".
> the 1985 CBA provided only 1 day of rest for employees at the Manila Office, as well as
employees similarly situated at the Legazpi and Marinduque Bulk Depots. This day of rest,
according to Caltex was Sunday.
- Ruling of Labor Arbiter
> ruled in favor of petitioner Union, while finding at the same time that Caltex was not
guilty of any unfair labor practice.
- In interpreting Article III and Annex "B" of the 1985 CBA, Labor Arbiter concluded that
Caltex's employees had been given two (2) days (instead of one day) of rest, with the
result that work performed on the employee's first day of rest, viz. Saturday, should be
compensated at "First day-off" rates.
- Ruling of NLRC
> NLRC set aside the decision of Labor Arbiter; conclusions of the Labor Arbiter were not
supported by the evidence on record.
- In interpreting the provisions of the 1985 CBA, NLRC concluded that CBA granted only
one (1) day of rest,
e.g., Sunday.
- The Union's motion for reconsideration was denied.
ISSUE
WON Article III in relation to Annex B of the 1985 CBA should be interpreted as giving the
workers two days of rest (Saturday and Sunday), thus entitling the workers to day off
rates on work performed on a Saturday
HELD
NO, the intention of the parties to the 1985 CBA was to provide the employees with only
one (1) day of rest.
- the use of the word "one" describing the phrase "day of rest [of an employee]"
emphasizes the fact that the parties had agreed that only a single day of rest shall be
scheduled and shall be provided to the employee.
- contract clauses governing hours of work in previous CBAs executed in 1973, 1976,
1979 and 1982 contained provisions parallel if not identical to those set out in Article III of
the 1985 CBA. In all these CBAs (1973, 1976, 1979, 1982), Article III provide that only
"work on an employee's one day of rest" shall be paid on the basis of "day of rest rates".
Petitioner Union had never suggested that more than 1 day of rest had been agreed upon,
and Caltex had never treated Article III or any other portion of the CBAs as providing two
(2) days of rest.
- An annex expresses the idea of joining a smaller or subordinate thing with another, larger
or of higher importance. An annex has a subordinate role, without any independent
significance separate from that to which it is tacked on. Annex "B," in the case at bar, is
one such document. It is not a memorandum of amendments or a codicil containing
additional or new terms or stipulations. Annex "B" cannot be construed as modifying or
altering the terms expressed in the body of the agreement contained in the 1985 CBA. It
did not confer any rights upon employees represented by petitioner Union; neither did it

Labor Law 1
impose any obligations upon private respondent Caltex. In fact, the contents of Annex "B"
have no intelligible significance in and of themselves when considered separately from the
1985 CBA.
Petitioner Union also contended that private respondent Caltex in the instant
petition was violating the statutory prohibition against off-setting undertime for overtime
work on another day. 11 Union counsel attempted to establish this charge by asserting
that the employees had been required to render "overtime work" on a Saturday but
compensated only at regular rates of pay, because they had not completed the eight (8)hour work period daily from Monday thru Friday.
- Overtime work consists of hours worked on a given day in excess of the applicable work
period, which here is eight (8) hours. It is not enough that the hours worked fall on
disagreeable or inconvenient hours. In order that work may be considered as overtime
work, the hours worked must be in excess of and in addition to the eight (8) hours worked
during the prescribed daily work period, or the forty (40) hours worked during the regular
work week Monday thru Friday.
- In the present case, under the 1985 CBA, hours worked on a Saturday do not, by that
fact alone, necessarily constitute overtime work compensable at premium rates of pay,
contrary to petitioner's assertion. These are normal or regular work hours, compensable at
regular rates of pay, as provided in the 1985 CBA; under that CBA, Saturday is not a rest
day or a "day off". It is only when an employee has been required on a Saturday to render
work in excess of the forty (40) hours which constitute the regular work week that such
employee may be considered as performing overtime work on that Saturday.
Disposition Petition dismissed

MANILA TERMINAL CO INC V CIR


[ ]

NO COMPUTATION FORMULA BASIC CONTRACT


MANILA TERMINAL CO INC V CIR
[ ]

BUILT-IN COMPENSATION
ENGINEERING EQUIPMENT INC V MINISTER OF
LABOR (ASPERA)
138 SCRA 616
AQUINO; September 23, 1985
NATURE
Petition to review the resolution of the Minister of Labor
FACTS
- Miguel Aspera worked for Engineering Equipment, Inc. in Saudi Arabia for nearly a year
from April 26, 1977 to April 16, 1978.
- He had a monthly salary of P750 (P860) with a six-day work week consisting of ten
working hours.
- According to his written contract of employment, he may be required to work overtime in
excess of the ten-hour work each day and to work on rest days or on Saudi Arabian
holidays.
- He was also promised overtime pay on top of his P750 salary for work rendered in
excess of what was required from him as a minimum.
- Jonathan de la Cruz, the director of Employment Services and the NLRC sustained his
claim and awarded him that amount as overtime pay. The 10 hour-a-week workload was
declared void for going against the Eight Hour Labor Law and section 87 of the Code
which states that any work beyond 8 hours a day should be considered overtime work.
- Petitioners Claim:
> Aspera is a managerial employee and should thus not be entitled to overtime pay in
accordance with Section 82 of the Labor Code.
> Aspera also signed a written contract with a built-in overtime pay in the ten hour
workday which meant that his basic monthly pay was adjusted to reflect the higher amount
covering the guaranteed two-hour extra time whether worked or unworked.
> The contracts were submitted to respondent Jonathan de la Cruz who approved of the
same.
- Respondents Comments:
> Aspera worked 10 hours daily for 335 days and he claims that his monthly salary should
correspond to eight hours of daily work and that for the additional two hours daily, he was
entitled to overtime pay at $1.2162 per hour or to $814.85 for 670 hours during 335
working days.

A2010

- 130 -

Disini

ISSUE
WON the Minister of Labor erred in awarding the overtime pay to Aspera
HELD
YES
- De la Cruz acted with lack of discretion and in excess of his jurisdiction when he
awarded the overtime pay to Aspera.
- He himself approved the contract even he knew of the Eight Hour Labor Law. Because
of that approval, petitioner Engineering Equipment Inc. acted in good faith in enforcing the
contract.
- Aspera also did not deny that he was a managerial employee.
Disposition The resolution of the MOLE is reversed and set aside. Asperas complaint is
dismissed. No costs.

PROOF OF WORK
LAGATIC V NLRC
285 SCRA 251
ROMERO; January 28, 1998
NATURE
Petition by certiorari
FACTS
- Petitioner Romeo Lagatic was employed by Cityland as a marketing specialist. He was
tasked with soliciting sales for the company, with the corresponding duties of accepting
call-ins, referrals, and making client calls and cold calls. Cold calls refer to the practice of
prospecting for clients through the telephone directory.
- In order to assess cold calls made by the sales staff, as well as to determine the results
thereof, Cityland requires the submission of daily progress reports on the same.
- On October 22, 1991, Cityland issued a written reprimand to petitioner for his failure to
submit cold call reports for certain days of September & October 1991. This
notwithstanding, petitioner again failed to submit cold call reports for September & October
1992. Petitioner was required to explain his inaction, with a warning that further noncompliance would result in his termination from the company. In a reply, petitioner claimed
that the same was an honest omission brought about by his concentration on other
aspects of his job. Cityland found said excuse inadequate and suspended him for three
days, with a similar warning.
- Notwithstanding the aforesaid suspension and warning, petitioner again failed to submit
cold call reports for February 1993. He was verbally reminded to submit the same and
was even given up to February 17, 1993 to do so. Instead of complying with said directive,
petitioner wrote a note, TO HELL WITH COLD CALLS! WHO CARES? and exhibited the
same to his co-employees.
- Petitioner received a memorandum requiring him to explain why Cityland should not
make good its previous warning for his failure to submit cold call reports, as well as for
issuing the written statement aforementioned. Petitioner sent a letter-reply alleging that his
failure to submit cold call reports should not be deemed as gross insubordination. He
denied any knowledge of the damaging statement, TO HELL WITH COLD CALLS!
- Finding petitioner guilty of gross insubordination, Cityland served a notice of dismissal
upon him. Petitioner filed a complaint against Cityland for illegal dismissal, illegal
deduction, underpayment, overtime and rest day pay, damages and attorneys fees.
- The labor arbiter dismissed the petition for lack of merit. On appeal, the same was
affirmed by the NLRC; hence the present recourse.
ISSUE
1. WON respondent NLRC gravely abused its discretion in not finding that petitioner was
illegally dismissed
2. WON respondent NLRC gravely abused its discretion in ruling that petitioner is not
entitled to salary differentials, backwages, separation pay, overtime pay, rest day pay,
unpaid commissions, moral and exemplary damages and attorneys fees
HELD
1. NO
Ratio Except as provided for, or limited by, special laws, an employer is free to regulate,
according to his discretion and judgment, all aspects of employment. Employers may,
thus, make reasonable rules and regulations for the government of their employees, and
when employees, with knowledge of an established rule, enter the service, the rule
becomes a part of the contract of employment. It is also generally recognized that

Labor Law 1

A2010

- 131 -

Disini

company policies and regulations, unless shown to be grossly oppressive or contrary to


law, are generally valid and binding on the parties and must be complied with.
Reasoning
- Said company policy of requiring cold calls and the concomitant reports thereon is clearly
reasonable and lawful, sufficiently known to petitioner, and in connection with the duties
which he had been engaged to discharge. There is, thus, just cause for his dismissal.
- Based on the foregoing, we find petitioner guilty of willful disobedience. Willful
disobedience requires the concurrence of at least two requisites: the employees assailed
conduct must have been willful or intentional, the willfulness being characterized by a
wrongful and perverse attitude; and the order violated must have been reasonable, lawful,
made known to the employee and must pertain to the duties which he had been engaged
to discharge.
- Well settled is the dictum that the twin requirements of notice and hearing constitute the
elements of due process in the dismissal of employees. Thus, the employer must furnish
the employee with two written notices before the termination of employment can be
effected. The first apprises the employee of the particular acts or omissions for which his
dismissal is sought; the second informs him of the employers decision to dismiss him.
- The chronology of events clearly shows that petitioner was served with the required
written notices.
- The requirement of a hearing is complied with as long as there was an opportunity to be
heard, and not necessarily that an actual hearing be conducted. Petitioner had an
opportunity to be heard as he submitted a letter-reply to the charge. He, however,
adduced no other evidence on his behalf. In fact, he admitted his failure to submit cold call
reports, praying that the same be not considered as gross insubordination.
- Denials are weak forms of defenses, particularly when they are not substantiated by
clear and convincing evidence. Given the foregoing, we hold that petitioners constitutional
right to due process has not been violated.
2. NO
- There is no law which requires employers to pay commissions, and when they do so,
there is no law which prescribes a method for computing commissions. The determination
of the amount of commissions is the result of collective bargaining negotiations, individual
employment contracts or established employer practice. Since the formula for the
computation of commissions was presented to and accepted by petitioner, such
prescribed formula is in order.
- Petitioner failed to show his entitlement to overtime and rest day pay due, to the lack of
sufficient evidence as to the number of days and hours when he rendered overtime and
rest day work. Entitlement to overtime pay must first be established by proof that said
overtime work was actually performed, before an employee may avail of said benefit.
- Lastly, with the finding that petitioners dismissal was for a just and valid cause, his
claims for moral and exemplary damages, as well as attorneys fees, must fail.
Disposition AFFIRMED.

as well as in filing the petition for certification election before the Department of Labor.
They further averred that they were paid daily wages ranging from P81.00 to P145.00
which were below the minimum fixed by law and that they were required to work six (6)
days a week from 8 oclock in the morning to 7 oclock in the evening without being paid
for the overtime. Neither were they paid their service incentive leave pay and 13th month
pay.
- The Labor Arbiter ruled for the petitioners. On appeal by HI-TECH, the NLRC in its
Decision of 30 May 1997 vacated and set aside the Labor Arbiters Decision and ordered
petitioners to report back to work, or if no longer feasible, directed HI-TECH to pay
petitioners their separation benefits.

SOCIAL SECURITY SYSTEM V CA

SOCIAL SECURITY SYSTEM V CA


[ ]

VILLAR V NLRC (HI-TECH MANUFACTURING


CORP)
331 SCRA 686
BELLOSILLO; May 11, 2000
NATURE
Petitioners, in this petition for certiorari, assail for having been rendered with grave abuse
of discretion the 30 May 1997 Decision of the National Labor Relations Commission
(NLRC) vacating and setting aside the Decision of the Labor Arbiter, as well as its 31 July
1997 Resolution denying reconsideration.
FACTS
- HI-TECH MANUFACTURING CORPORATION (HI-TECH), a corporation duly organized
and existing under Philippine laws, is engaged in the business of manufacturing cartons
for commercial purposes. On different dates, HI-TECH hired petitioners to perform various
jobs for the company such as slitter machine operator, inkman, silk screen printer, truck
helper, rubber dye setter, forklift operator and stitching machine operator.
- Sometime in March 1994 petitioners, who were members of the Federation of Free
Workers Union, filed before the Department of Labor a petition for certification election
among the rank-and-file employees of HI-TECH. The petition was granted and a
certification election was conducted inside the company premises on 31 July 1994.
However, petitioners lost in the election as the HI-TECH employees voted for "No Union."
- On 1 August 1994 and the succeeding days thereafter, petitioners failed to report for
work. They alleged that they were barred from entering the premises of HI-TECH; hence,
they immediately filed before the Labor Arbiter separate complaints for illegal dismissal
and labor standards claims against HI-TECH, Herman T. Go, owner, and Carmen Belano,
general manager.
- Petitioners claimed that they were summarily dismissed from employment by the
management of HI-TECH in retaliation for organizing a labor union in the work premises

ISSUE
WON petitioners are entitled to back wages and other monetary benefits

HELD
Ratio The burden of proving payment of monetary claims rests on the employer. In
Jimenez v. National Labor Relations Commission we held As a general rule, one who pleads payment has the burden of proving it. Even where
the plaintiff must allege non-payment, the general rule is that the burden rests on the
defendant to prove payment, rather than on the plaintiff to prove non-payment. The
debtor has the burden of showing with legal certainty that the obligation has been
discharged with payment.
- The petitioners claims for underpayment of wages, 13th month pay and service incentive
leave pay should be upheld.
- Petitioners executed a JOINT AFFIDAVIT specifying their daily wages, positions and
periods of employment, which was made the basis of the Labor Arbiters computation of
the monetary awards. All that the NLRC needed to do was to refer to the prevailing
minimum wage to ascertain the correctness of petitioners claims. And most importantly,
the burden of proving payment of monetary claims rests on the employer.
- The reason for the rule is that the pertinent personnel files, payrolls, records, remittances
and other similar documents which will show that overtime, differentials, service
incentive leave and other claims of workers have been paid are not in the possession of
the worker but in the custody and absolute control of the employer. Thus, in choosing not
to present evidence to prove that it had paid all the monetary claims of petitioners, HITECH failed once again to discharge the onus probandi. Consequently, we have no choice
but to award those claims to petitioners.

EMPLOYER OBLIGATION

5.07 NIGHT WORK


RATIONALE PROHIBITION
SHELL OIL CO. OF THE PHILS LTD V NATIONAL
LABOR UNION
81 Phil 315
BRIONES; July 26, 1948
NATURE
Petition for review on certiorari
FACTS
- Acting on a petition of the company "National Work Union," the Court of Industrial
Relations has dictated a decision in which, among others things, the petroleum firm is
obliged to pay its laborers for work at night a compensation additional of 50% on its
regular salaries. It seems that the company has need of the nocturnal service of a
determined number of laborers, since some of the necessary tasks are done at night for
the supply of gasoline and lubricants, and for other needs. The company alleges and
argues that theres not any disposition that authorizes the Court of Industrial Relations to
order the payment of additional compensation to laborers that work at night, but, on the
contrary, the law of the Commonwealth Not. 444 exempts all owners of similar obligation
every time that in said law the cases are provided in which the payment of "overtime" is
compulsory, and among such cases of overtime does not figure the work at night. In turn,
the union maintains that the law Not. 444 of the Commonwealth that is invoked does not

Labor Law 1
have any application al present case, therefore the same one is of reach inevitably limited,
being referred And exclusively to the day maxima of work contidiano permitted in the
industrial establishments the day of 8 hours. Our conclusion is that the union has the
reason on its part.
- Commonwealth No. 444 states:
SECTION 1. The legal working day for any person employed by another shall be of not
more than eight hours daily. When the work is not continuous, the time during which
the laborer is not working and can leave his working place and can rest completely
shall not be counted.
SEC. 3. Work may be performed beyond eight hours a day in case of actual or
impending emergencies caused by serious accidents, fire, flood, typhoon, earthquake,
epidemic, or other disaster or calamity in order to prevent loss to life and property or
imminent danger to public safety; or in case urgent work to be performed on the
machines, equipment, or installations in order to avoid a serious loss which the
employer would otherwise suffer, or some other just cause of a similar nature; but in all
such cases the laborers and employees shall be entitled to receive compensation for
the overtime work performed at the same rate as their regular wages or salary, plus at
least twenty-five per centum additional.
In case of national emergency the government is empowered to establish rules and
regulations for the operation of the plants and factories and to determine the wages to
be paid the laborers.
SEC. 4. No person, firm, or corporation, business establishment or place or center of
labor shall compel an employee or laborer to work during Sundays and legal holidays,
unless he is paid an additional sum of at least twenty-five per centum of his regular
remuneration: Provided however, That this prohibition shall not apply to public utilities
performing some public service such as supplying gas, electricity, power, water, or
providing means of transportation or communication.
- The lawyers of the company argues that in these articules the cases are specified in
which payment of extra of additional compensation is authorized: (A) in case of "overtime"
or work over and above the hours regulated by reasons dangers of urgency because of
some disaster or accident, or to avoid lost or to repair them; (b) in case of work by
Sundays and festivals; (c) in case of emergency, and nothing it is necessary refer at work
at night; then the order that treats is illegal, therefore not this authorized by the law. "In
the absence of legislation authorizing the payment of extra compensation for work at night,
the Court of Industrial Relations has not the power or authority to order the petitioner
company to pay extra compensation for by its laborers at night. Expressio unius est
exclusio alterius. Where, in the statute expressly specifies the cases where payment of
extra compensation may be demanded, extra compensation may be allowed in those
cases only, and in not others. The provisions of the Commonwealth Act Not. 444 cannot
be enlarged by implication or otherwise. Expressum facit cessare tacitum. The
argumentation is erronea. The Law Not. 444 is not applicable al present case, being
evident that the same one has an object I specify, to know: (A) to set in 8 hours the day
maxima of work; (b) senalar certain exceptional cases in which the work out of said day
can be authorized; (c) to provide a bonus, that should not be less than 25% of the salary
to regulate, for the "overtime" or I work over and above the 8 hours.
ISSUE
WON the petroleum company is obligated to give additional pay for its night workers
HELD
YES
Ratio The night work that the company Shell requires of its laborers is not an "overtime",
in the sense in which this word is employed in Commonwealth Act 444, but is a complete
day of work, of 8 hours. In other words, the work at night is not only unexceso,
prolongacion or "overtime" of the regular daywork , but is another type of work, absolutely
independent of the day. Therefore there are two shifts: the shift of laborers that work
during the day; and the shift of the ones that work at night. It is not strange that the
legislator have not included this type of work among the cases of "overtime" in the abovementioned law Not. 444.
Reasoning
- And with respect with the apreciation that the work at night is but heavy and burdensome
than that of day and, therefore, he deserves greater remuneration, there is no reason to
revoke it or to alter it. There is not possible argument against the universal fact that the
ordinary, normal, and regular work is that of day and that the work at night is very
exceptional and justified alone certainly imperatively inevitable motives.
- Reasons of hygiene, of medicine, of morale, of culture, of sociologia, they establish with
one accord that the work at night has many objections, and when there is not but remedy
that to cause is alone just that I am paid better than usually to compensate hasa certain
point al working of such objections. It must be remembered that it is distinctly
unphysiological to turn the night into day and deprive the body of the beneficial effects of
sunshine. The human organism revolts against this procedure. Added to artificial lighting
are reversed and unnatural times of eating, resting, and sleeping. Much of the inferiority of
nightwork can doubtless be traced to the failure of the workers to secure proper rest and
sleep, by day. Because of inability or the lack of opportunity to sleep, nightworkers often
spend their days in performing domestic duties, joining the family in the midday meal,

A2010

- 132 -

Disini

'tinkering about the place', watching the baseball game, attending the theater or taking a
ride in the car. It is not strange that nightworkers tend to be less efficient than dayworkers
and lose more time. . . (The Management of Labor Relations, by Watkins & Dodd, page
524.).
> Nightwork. Nightwork has gained a measure of prominence in the modern
industrial system in connection with continuous industries, that is, industries in which
the nature of the processes makes it necessary to keep machinery and equipment in
constant operation. Even in continuous industries the tendency is definitely in the
direction of FOUR shifts of 6 hours each, with provision for an automatic change of shift
for all workers at stated intervals. Some discussion has taken place with regard to the
lengths of the period any workers should be allowed to remain on the night shift. A
weekly change of shifts is common, specially where three or four shifts are in
operation; in other cases the change is made fortnightly or monthly; in still other
instances, no alternation is provided for, the workers remaining on day or nightwork
permanently, except where temporary changes are made for individual convenience.
There is sharp difference of opinion concerning the relative merits of these systems.
Advocates of the weekly change of shifts contend that the strain of nightwork and the
difficulty of getting adequate sleep during the day make it unwise for workers to remain
on the"graveyard" shift for more than a week at a time. Opponents urge that repeated
changes make it more difficult to settle down to either kind of shift and that after the first
week nightwork becomes less trying while the ability to sleep by day increases.
Workers themselves react in various ways to the different systems. This much,
however, is certain: Few persons react favorably to nightwork, whether the shift be
continuous or alternating. Outside of continuous industries, nightwork can scarcely be
justified, and, even in these, it presents serious disadvantages which must be
recognized in planing for industrial efficiency, stabilization of the working force, the
promotion of industrial good-will, and the conservation of the health and vitality of the
workers.
> Nightwork cannot be regarded as desirable, either from the point of view of the
employer or of the wage earner. It is uneconomical unless overhead costs are
unusually heavy. Frequently the scale of wages is higher as an inducement to
employees to accept employment on the night shift, and the rate of production is
generally lower.
> The lack of sunlight tends to produce anemia and tuberculosis and to predispose to
other ills. Nightwork brings increased liability to eyestrain and accident. Serious moral
dangers also are likely to result from the necessity of traveling the streets alone at
night, and from the interference with normal home life. From an economic point of view,
moreover, the investigations showed that nightwork was unprofitable, being inferior to
day work both in quality and in quantity. Wherever it had been abolished, in the long
run the efficiency both of the management and of the workers was raised. Furthermore,
it was found that nightwork laws are a valuable aid in enforcing acts fixing the
maximum period of employment. (Principles of Labor Legislation, by Commons and
Andrews, 4th Revised Edition, p. 142.)
Special regulation of nightwork for adult men is a comparatively recent development.
Some European countries have adopted laws placing special limitations on hours of
nightwork for men, and others prohibit such work except in continuous processes.
(Principles of Labor legislation, 4th Revised Edition by Common & Andrews, p. 147.)
> Nightwork has almost invariably been looked upon with disfavor by students of the
problem because of the excessive strain involved, especially for women and young
persons, the large amount of lost time consequent upon exhaustion of the workers, the
additional strain and responsibility upon the executive staff, the tendency of excessively
fatigued workers to "keep going" on artificial stimulants, the general curtailment of time
for rest, leisure, and cultural improvement, and the fact that night workers, although
precluded to an extent from the activities of day life, do attempt to enter into these
activities, with resultant impairment of physical well-being. It is not contended, of
course, that nightwork could be abolished in the continuous-process industries, but it is
possible to put such industries upon a three- or four-shifts basis, and to prohibit
nightwork for women and children. (Labor's Progress and Problems, Vol. I, p. 464, by
Professors Millis and Montgomery.)
> Nightwork. Civilized peoples are beginning to recognize the fact that except in
cases of necessity or in periods of great emergency, nightwork is socially undesirable.
Under our modern industrial system, however, nightwork has greatly aided the
production of commodities, and has offered a significant method of cutting down the
ever-increasing overhead costs of industry. This result has led employers to believe
that such work is necessary and profitable. Here again one meets a conflict of
economic and social interests. Under these circumstances it is necessary to discover
whether nightwork has deleterious effects upon the health of laborers and tends to
reduce the ultimate supply of efficient labor. If it can proved that nightwork affects
adversely both the quality and quantity of productive labor, its discontinuance will
undoubtedly be sanctioned by employers. From a social point of view, even a relatively
high degree of efficiency in night operations must be forfeited if it is purchased with
rapid exhaustion of the health and energy of the workers. From an economic point of
view, nightwork may be necessary if the employer is to meet the demand for his
product, or if he is to maintain his market in the face of increasing competition or
mounting variable production costs.

Labor Law 1
> Industrial experience has shown that the possession of extra-ordinary physical
strength and self-control facilitates the reversal of the ordinary routine of day work and
night rest, with the little or no unfavorable effect on health and efficiency. Unusual
vitality and self-control, however, are not common possessions. It has been found that
the most serious obstacle to a reversal of the routine is the lack of self-discipline. Many
night workers enter into the numerous activities of day life that preclude sleep, and
continue to attempt to do their work at night. Evidence gathered by the British Health of
Munition Workers' Committee places permanent night workers, whether judged on the
basis of output or loss of time, in a very unfavorable positions as compared with day
workers.
> Systems of nightwork differ. There is the continuous system, in which employees
labor by night and do not attend the establishment at all by day, and the discontinuous
system, in which the workers change to the day turn at regular intervals, usually every
other week. There are, of course, minor variations in these systems, depending upon
the nature of the industry and the wishes of management. Such bodies as the British
Health Munition Workers' Committee have given us valuable conclusions concerning
the effect of nightwork. Continuous nightwork is definitely less productive than the
discontinuous system. The output of the continuous day shift does not make up for this
loss in production.
> There is, moreover, a marked difference between the rates of output of night and day
shifts on the discontinuous plan. In each case investigated the inferiority of night labor
was definitely established. This inferiority is evidently the result of the night worker's
failure to secure proper amounts of sleep and rest during the day. The system of
continuous shifts, especially for women, is regarded by all investigators as undesirable.
Women on continuous nightwork are likely to perform domestic duties, and this added
strain undoubtedly accounts for the poorer results of their industrial activities. The
tendency to devote to amusement and other things the time that should be spent in rest
and sleep is certainly as common among men as among women workers and accounts
largely for the loss of efficiency and time on the part of both sexes in nightwork.
- The case against nightwork, then, may be said to rest upon several grounds. In the first
place, there are the remotely injurious effects of permanent nightwork manifested in the
later years of the worker's life. Of more immediate importance to the average worker is the
disarrangement of his social life, including the recreational activities of his leisure hours
and the ordinary associations of normal family relations. From an economic point of view,
nightwork is to be discouraged because of its adverse effect upon efficiency and output. A
moral argument against nightwork in the case of women is that the night shift forces the
workers to go to and from the factory in darkness. Recent experiences of industrial nations
have added much to the evidence against the continuation of nightwork, except in
extraordinary circumstances and unavoidable emergencies. The immediate prohibition of
nightwork for all laborers is hardly practicable; its discontinuance in the case of women
employees is unquestionably desirable. 'The night was made for rest and sleep and not for
work' is a common saying among wage-earning people, and many of them dream of an
industrial order in which there will be no night shift.
Disposition Petition denied

SECTION 6: CONDITIONS OF EMPLOYMENT


WEEKLY REST PERIODS
6.01 RATIONALE
RATIONALE
MANILA ELECTRIC COMPANY V THE PUBLIC
UTILITIES EMPLOYEES' ASSOCIATION
79 PHIL. 409
FERIA; October 30, 1947
FACTS
- This is an appeal interposed by the petitioner Manila Electric Company against the
decision of the Court of Industrial Relations, which reads as follows:
Although the practice of the company, according to the manifestations of counsel for
said company, has been to grant one day vacation with pay to every workingman
who had worked for seven consecutive days including Sundays, the Court considers
justified the opposition presented by the workingmen to the effect that they need
Sundays and holidays for the observance of their religion and for rest. The Court,
therefore, orders the respondent company to pay 50 per cent increase for overtime
work done on ordinary days and 50 per cent increase for work done during Sundays
and legal holidays irrespective of the number of days they work during the week.

A2010

Disini

- 133 -

- The appellant contends that the said decision is against Section 4, Commonwealth
Act No. 444, which reads as follows:
No person, firm, or corporation, business establishment or place or center of labor
shall compel an employee or laborer to work during Sundays and legal holidays,
unless he is paid an additional sum of at least 25% of his regular remuneration:
Provided, however, That this prohibition shall not apply to public utilities performing
some public service such as supplying gas, electricity, power, water, or providing
means of transportation or communication.
ISSUE
WON the decision of the Court of Industrial Relations is erroneous and contrary to Section
4, Commonwealth Act No. 444
HELD
YES
- Commonwealth Act No. 444 provides that public utilities supplying electricity, gas, power,
water, or providing means of transportation or communication may compel their
employees or laborers to work during Sundays and legal holidays without paying them an
additional compensation of not less than 25 per cent of their regular remuneration on said
days.
-The provisions of the above quoted section 4, are plain and unambiguous and convey a
clear and definite meaning.
- Said section 1 consists of two parts: the first, which is the enactment clause, prohibits a
person, firm or corporation, business establishment, or place or center of labor from
compelling an employee or laborer to work during Sundays and legal holidays, unless the
former pays the latter an additional sum of at least twenty five per centum of his regular
remuneration; and the second part, which is an exception, exempts public utilities
performing some public service, such as supplying gas, electricity, power, water or
providing means of transportation or communication, from the prohibition established in
the enactment clause.
- As the appellant is a public utility that supplies the electricity and provides means of
transportation to the public, it is evident that the appellant is exempt from the qualified
prohibition established in the enactment clause, and may compel its employees or
laborers to work during Sundays and legal holidays without paying them said extra
compensation.
- To construe section 4, Commonwealth Act No. 444, as exempting public utilities, like the
appellant, from the obligation to pay the additional remuneration required by said section 4
should they compel their employees or laborers to work on Sundays and legal holidays,
would not make such exception a class legislation, violative of the constitutional guaranty
of equal protection of the laws.
- And it is evident that the division made by section 4, of Commonwealth Act No. 444, of
persons, firms, and corporations into two classes: one composed of public utilities
performing some public service such as supplying gas, electricity, power, water or
providing means of transportation; and another composed of persons, firms, and
corporations which are not public utilities and do not perform said public service, is not
arbitrary and is based indifferences which are apparent and reasonable.
- The division is not arbitrary, and the basis thereof is reasonable. Public utilities exempted
from the prohibition set forth in the enactment clause of section 4, Commonwealth Act No.
444, are required to perform a continuous service including Sundays and legal holidays to
the public, since the public good so demands, and are not allowed to collect an extra
charge for services performed on those days; while the others are not required to do so
and are free to operate or not their shops, business, or industries on Sundays and legal
holidays.
- If they operate and compel their laborers to work on those days it is but just and natural
that they should pay an extra compensation to them, because it is to be presumed that
they can make money or business by operating on those days even if they have to pay
such extra remuneration.
It would be unfair for the law to compel public utilities like the appellant to pay an
additional or extra compensation to laborers whom they have to compel to work during
Sundays and legal holidays, in order to perform a continuous service to the public. To
require public utilities performing service to do so, would be tantamount to penalize them
for performing public service during said days in compliance with the requirement of the
law and public interest.
Disposition The ruling of the Court of Industrial Relations was set aside.

SEPARATE OPINION
PERFECTO [dissent]
- It will be seen that section 4 of Commonwealth Act No.444 divides the employers into
two classes: 1. Those not engaged in public utilities, and 2. those engaged in public
utilities.
- The first class cannot compel their employees or laborers to work on Sundays and
holidays without giving them an additional salary or renumeration equivalent to not less
than twenty-five per centum of the basic remuneration. There is no limit as to the time or

Labor Law 1

A2010

circumstances under which the additional compensation is to be paid. Employers engaged


in public utilities are excluded from the prohibition.
- This means simply that employers engaged in public utilities may or may not pay the
additional compensation or any additional compensation for compelling their laborers to
work on Sundays and holidays. The exception should not be interpreted as providing that
employers engaged in public utilities cannot be compelled to pay additional compensation
to workers required to work on Sundays and holidays. As to them, section 4of
Commonwealth Act No. 444 may be considered as not having been enacted at all.
Exception or exemption from a negative or prohibitory legal provision is not a positive or
affirmative provision commanding the excepted or exempted person to do what is enjoined
in the general provision.
- The right to collect an additional sum of at least twenty-five per centum of the basic
remuneration is guaranteed to all workers and employees not engaged in public utilities
and that right is enforceable not only in the Court of Industrial Relations but in any other
competent court of justice.
- It is a fact that Sundays and legal holidays are set aside by law as days of rest. The life,
existence, and happiness of a person do not depend only on the satisfaction of his
physical needs. There are moral, intellectual and spiritual needs as imperative as the
physical ones.
- Ordinarily, Sundays and legal holidays are dedicated to reading and instruction so as to
fill the mind with culture or some sort of advancement. On those days the laborer enjoys
longer hours in the company of his family. That gives him an opportunity to satisfy his
moral needs. During Sundays and holidays more time is dedicated to worship and other
religious services. That gives a laborer an opportunity to satisfy his spiritual needs. The
deprivation of that opportunity to satisfy mental, moral, and spiritual needs should not be
ignored, and should be properly compensated.
Petition should be dismissed.

6.02 COVERAGE
6.03 SCHEDULING OF REST DAY
6.04
COMPULSORY
COMPENSATION

WORK

AND

SECTION 7: CONDITIONS OF EMPLOYMENT


HOLIDAYS
7.01 COVERAGE
COVERAGE AND PURPOSE
MANTRADE/FMC DIVISION EMPLOYEES AND
WORKERS UNION V BACUNGAN
SAN MIGUEL V CA (ESPAOL)
375 SCRA 311 (02)
KAPUNAN; January 30, 2002

- 134 -

ISSUES
1. WON CA erred in granting non-Muslim employees Muslim holiday pay
2. WON Macaraya and Espaol have jurisdiction in issuing compliance orders over said
labor standard case
HELD
1. NO
Ratio Wages and other emoluments granted by law to the working man are determined on
the basis of the criteria laid down by laws and certainly not on the basis of the workers
faith or religion.
Reasoning
- Muslim holidays are provided under Articles 169 and 170, Title I, Book V, of Presidential
Decree No. 1083, otherwise known as the Code of Muslim Personal Laws. The
aforementioned provisions should be read in conjunction with Art. 94 of the Labor Code:
Art. 94. Right to holiday pay:
(a) Every worker shall be paid his regular daily wage during regular holidays, except in
retail and service establishments regularly employing less than ten (10) workers;
(b) The employer may require an employee to work on any holiday but such employee
shall be paid a compensation equivalent to twice his regular rate; x x x.
- There should be no distinction between Muslims and non-Muslims as regards payment
of benefits for Muslim holidays. Petitioner asserts that Article 3(3) of Presidential Decree
No. 1083 provides that (t)he provisions of this Code shall be applicable only to Muslims x
x x. However, said article also declares that x x x nothing herein shall be construed to
operate to the prejudice of a non-Muslim.
1999 Handbook on Workers Statutory Benefits:
Considering that all private corporations, offices, agencies, and entities or establishments
operating within the designated Muslim provinces and cities are required to observe
Muslim holidays, both Muslim and Christians working within the Muslim areas may not
report for work on the days designated by law as Muslim holidays.
- As regards the allegation that the issue on Muslim holiday pay was already resolved in
Napoleon E. Fernan vs. San Miguel Corporation Beer Division and Leopoldo Zaldarriaga,
the Court notes that the case was primarily for illegal dismissal and the claim for benefits
was only incidental to the main case.
2. YES
- Regional Director Macaraya acted as the duly authorized representative of the Secretary
of Labor and Employment and it was within his power to issue the compliance order to
SMC.
Reasoning
- Article 128. Visitorial and enforcement power.
(b) Notwithstanding the provisions of Article 129 and 217 of this Code to the contrary,
and in cases where the relationship of employer-employee still exists, the Secretary of
Labor and Employment or his duly authorized representatives shall have the power to
issue compliance orders to give effect to the labor standards provisions of this Code
and other labor legislation based on the findings of labor employment and enforcement
officers or industrial safety engineers made in the course of the inspection.
- Petitioner merely contends that its non-Muslim employees are not entitled to Muslim
holiday pay. The issue could be resolved even without documentary proofs. In any case,
there was no indication that Regional Director Macaraya failed to consider any
documentary proof presented by SMC in the course of the inspection.
Disposition The petition is dismissed.

ASIAN TRANSMISSION CORP V CA (BISIG NG ASIAN


TRANSMISSION LABOR UNION)
[PAGE 45]

7.02 HOLIDAYS

NATURE
Petition for a review of the decision of the Court of Appeals
FACTS
- 17 October 1992: the Department of Labor and Employment (DOLE), Iligan District
Office, conducted a routine inspection in the premises of San Miguel Corporation (SMC) in
Sta. Filomena, Iligan City. In the course of the inspection, it was discovered that there was
underpayment of regular Muslim holiday pay to its employees.
- SMC failed to submit proof that it was paying regular Muslim holiday pay to its
employees. Alan M. Macaraya, Director IV of DOLE Iligan District Office issued a
compliance order directing SMC to consider Muslim holidays as regular holidays and to

Disini

pay both its Muslim and non-Muslim employees holiday pay within thirty (30) days from
the receipt of the order.

7.03 HOLDAY PAY

Labor Law 1
FACULTY PRIVATE SCHOOL
JOSE RIZAL COLLEGE V NLRC
156 SCRA 27
PARAS; 1987
NATURE
- Petition for certiorari with preliminary injunction to review the decision of the National
Labor Relations Commission
FACTS
- Petitioner is a non-stock, non-profit educational institution duly organized and existing
under the laws of the Philippines. It has three groups of employees categorized as follows:
(a) personnel on monthly basis, who receive their monthly salary uniformly throughout the
year, irrespective of the actual number of working days in a month without deduction for
holidays; (b) personnel on daily basis who are paid on actual days worked and they
receive unworked holiday pay and (c) collegiate faculty who are paid on the basis of
student contract hour. Before the start of the semester they sign contracts with the college
undertaking to meet their classes as per schedule.
- Unable to receive their corresponding holiday pay, as claimed, from 1975 to 1977,
private respondent National Alliance of Teachers and Office Workers (NATOW) in behalf of
the faculty and personnel of Jose Rizal College filed with the Ministry of Labor a complaint
against the college for said alleged non-payment of holiday pay
- After the parties had submitted their respective position papers, the Labor Arbiter
rendered a decision on February 5, 1979:
1. The faculty and personnel of the respondent Jose Rizal College who are paid their
salary by the month uniformly in a school year, irrespective of the number of working
days in a month, without deduction for holidays, are presumed to be already paid the
10 paid legal holidays and are no longer entitled to separate payment for the said
regular holidays;
2. The personnel of the respondent Jose Rizal College who are paid their wages daily
are entitled to be paid the 10 unworked regular holidays according to the pertinent
provisions of the Rules and Regulations Implementing the Labor Code;
3. Collegiate faculty of the respondent Jose Rizal College who by contract are paid
compensation per student contract hour are not entitled to unworked regular holiday
pay considering that these regular holidays have been excluded in the programming of
the student contact hours.
- On appeal, respondent National Labor Relations Commission in a decision promulgated
on June 2, 1982, modified the decision appealed from, in the sense that teaching
personnel paid by the hour are declared to be entitled to holiday pay
ISSUE
WON the school faculty who according to their contracts are paid per lecture hour are
entitled to unworked holiday pay
HELD
NO (for regular holidays)/YES(for special holidays)
- Labor Arbiter sustains the view that said instructors and professors are not entitled to
holiday pay, his decision was modified by the National Labor Relations Commission
holding the contrary. Petitioner maintains the position among others, that it is not covered
by Book V of the Labor Code on Labor Relations considering that it is a non-profit
institution and that its hourly paid faculty members are paid on a "contract" basis because
they are required to hold classes for a particular number of hours. In the programming of
these student contract hours, legal holidays are excluded and labelled in the schedule as
"no class day." On the other hand, if a regular week day is declared a holiday, the school
calendar is extended to compensate for that day. Thus petitioner argues that the advent of
any of the legal holidays within the semester will not affect the faculty's salary because this
day is not included in their schedule while the calendar is extended to compensate for
special holidays. Thus the programmed number of lecture hours is not diminished.
- The Solicitor General on the other hand, argues that under Article 94 of the Labor Code,
holiday pay applies to all employees except those in retail and service establishments. To
deprive therefore employees paid at an hourly rate of unworked holiday pay is contrary to
the policy considerations underlying such presidential enactment, apart from the
constitutional mandate to grant greater rights to labor. And under Article 94 of the Labor
Code, the petitioner, although a nonprofit institution is under obligation to give pay even on
unworked regular holidays to hourly paid faculty members subject to the terms and
conditions provided for therein.
- The Court held that the aforementioned implementing rule is not justified by the
provisions of the law which after all is silent with respect to faculty members paid by the
hour who because of their teaching contracts are obliged to work and consent to be paid
only for work actually done.

A2010

- 135 -

Disini

- On the other hand, both the law and the Implementing Rules governing holiday pay are
silent as to payment on Special Public Holidays.
- It is readily apparent that the declared purpose of the holiday pay which is the prevention
of diminution of the monthly income of the employees on account of work interruptions is
defeated when a regular class day is cancelled on account of a special public holiday and
class hours are held on another working day to make up for time lost in the school
calendar. Otherwise stated, the faculty member, although forced to take a rest, does not
earn what he should earn on that day. Be it noted that when a special public holiday is
declared, the faculty member paid by the hour is deprived of expected income, and it does
not matter that the school calendar is extended in view of the days or hours lost, for their
income that could be earned from other sources is lost during the extended days.
Similarly, when classes are called off or shortened on account of typhoons, floods, rallies,
and the like, these faculty members must likewise be paid, whether or not extensions are
ordered.
Disposition Decisions set aside. New decision rendered:
(a) exempting petitioner from paying hourly paid faculty members their pay for regular
holidays, whether the same be during the regular semesters of the school year or during
semestral, Christmas, or Holy Week vacations;
(b) but ordering petitioner to pay said faculty members their regular hourly rate on days
declared as special holidays or for some reason classes are called off or shortened for the
hours they are supposed to have taught, whether extensions of class days be ordered or
not; in case of extensions said faculty members shall likewise be paid their hourly rates
should they teach during said extensions.

DIVISOR AS FACTOR
UNION OF FILIPRO EMPLOYEES V VIVAR
[PAGE 122]
TRANSASIA PHILS EMPLOYER ASSN V NLRC (TRANS
ASIA, DE CASTRO)
320 SCRA 547
KAPUNAN; December 13, 1999
NATURE
Petition for certiorari
FACTS
- On 7 July 1988, Trans-Asia Philippines Employees Association (TAPEA), entered into a
Collective Bargaining Agreement with their employer. The CBA, provided for, the payment
of holiday pay with a stipulation that if an employee is permitted to work on a legal holiday,
the said employee will receive a salary equivalent to 200% of the regular daily wage plus a
60% premium pay.
- Despite the conclusion of the CBA, an issue was still left unresolved with regard to the
claim of TAPEA for payment of holiday pay covering the period from January of 1985 up to
December of 1987.
- The parties underwent preventive mediation meetings with a representative from the
National Mediation and Conciliation Board
- The parties were not able to arrive at an amicable settlement.
- TAPEA filed a complaint before the labor arbiter for the payment of their holiday pay in
arrears.
- Petitioners amended their complaint to include the payment of holiday pay for the
duration of the recently concluded CBA (from 1988 to 1991), unfair labor practice,
damages and attorney's fees.
- Petitioners contend:
> their claim for the holiday pay in arrears is based on the non-inclusion of the same in
their monthly pay. First, petitioners presented Trans-Asia's Employees' Manual which
requires, that the employee should have worked or was on authorized leave with pay on
the day immediately preceding the legal holiday. They argued that if the intention of
Trans-Asia was not to pay holiday pay in addition to the employee's monthly pay, then
there would be no need to impose or specify the pre-condition for the payment. Second,
petitioners proffered as evidence their appointment papers which do not contain any
stipulation on the inclusion of holiday pay in their monthly salary. Third, petitioners noted
the inclusion of a provision in the CBA for the payment of an amount equivalent to 200% of
the regular daily wage plus 60% premium pay to employees who are permitted to work on
a regular holiday. Finally, petitioners cited the current CBA provision which obligates
Trans-Asia to give holiday pay.
> Trans-Asia is guilty of bad faith in negotiating and executing the current CBA since,
after it recognized the right of the employees to receive holiday pay, Trans-Asia allegedly
refused to honor the CBA provision on the same.
- Respondents claim:

Labor Law 1
> Tans-Asia refuted the allegations.
> Trans-Asia asserted that the above circumstances are not indicative of its non-payment
of holiday pay since it has always honored the labor law provisions on holiday pay by
incorporating the same in the payment of the monthly salaries of its employees. In support
of this claim, Trans-Asia pointed out that it has long been the standing practice of the
company to use the divisor of "286" days in computing for its employees' overtime pay and
daily rate deductions for absences.
> Trans-Asia further clarified that the "286" days divisor already takes into account the ten
(10) regular holidays in a year since it only subtracts from the 365 calendar days the
unworked and unpaid 52 Sundays and 26 Saturdays
> Trans-Asia claimed that if the ten (10) regular holidays were not included in the
computation of their employee's monthly salary, the divisor which they would have used
would only be 277.
> Furthermore, Trans-Asia explained that the "286" days divisor is based on Republic Act
No. 6640 wherein the divisor of 262 days is used in computing for the monthly rate of
workers who do not work and are not considered paid on Saturdays and Sundays or rest
days.
- On the second contention of the petitioners, Trans-Asia explained that this holiday pay
rate was included in the CBA in order to comply with Section 4, Rule IV, Book III of the
Omnibus Rules Implementing the Labor Code:
Sec. 4. Compensation for holiday work. - Any employee who is permitted or suffered to
work on any regular holiday, not exceeding eight (8) hours, shall be paid at least two
hundred percent (200%) of his regular daily wage. If the holiday falls on the scheduled
rest day of the employee, he shall be entitled to an additional premium pay of at least
30% of his regular holiday rate of 200% based on his regular wage rate.
- On the contention that Trans-Asia's acquiescence to the inclusion of a holiday pay
provision in the CBA is an admission of non-payment of the same in the past, Trans-Asia
reiterated that it is simply a recognition of the mandate of the Labor Code that employees
are entitled to holiday pay.
- With regard to the accusation of unfair labor practice, Trans-Asia explained that what
petitioners would like the company to do is to give double holiday pay since, as previously
stated, the company has already included its employees monthly salary and, yet,
petitioners want it to pay a second set of holiday pay.
- On 13 February 1989, the labor arbiter rendered a decision dismissing the complaint
- Petitioners appealed to the National Labor Relations Commission. In its Resolution,
dated 23 November 1993, the NLRC dismissed the appeal and affirmed the decision of
the labor arbiter
- Petitioners' motion for reconsideration was, likewise, denied by the NLRC in its
Resolution, dated 13 September 1994.
ISSUES
1. WON NLRC erred in upholding the labor arbiters decision
2. WON NLRC violated the constitutional and legal mandate to resolve all doubts in social
legislation in favor of labor
HELD
1. NO
- Trans-Asia's inclusion of holiday pay in petitioner's monthly salary is clearly established
by its consistent use of the divisor of "286" days in the computation of its employees'
benefits and deductions. The use by Trans-Asia of the "286" days divisor was never
disputed by petitioners.
- Nevertheless, petitioners' cause is not entirely lost. The Court notes that there is a need
to adjust the divisor used by Trans-Asia to 287 days, instead of only 286 days, in order to
properly account for the entirety of regular holidays and special days in a year as
prescribed by Executive Order No. 203 in relation to Section 6 of the Rules Implementing
Republic Act 6727. Section 1 of Executive Order No. 203 provides:
SECTION 1. Unless otherwise modified by law, order or proclamation, the following
regular holidays and special days shall be observed in the country:
A.Regular Holidays
New Year's Day Maundy Thursday Good Friday Araw ng Kagitingan (Bataan &
Corregidor Day) Labor Day Independence Day National Heroes Day Bonifacio Day
Christmas Day Rizal Day
-January 1 -Movable Date -Movable Date -April 9 -May 1
-June 12 -Last Sunday of August -November 30 -December 25 -December 30
B.Nationwide Special Days
All Saints Day Last Day of the Year -November 1 -December 31
- On the other hand, Section 6 of the Implementing Rules and Regulations of Republic Act
No. 6727 provides that the total number of working days is 262 days (see original for the
formula and computation)
- In the present case, since the employees of Trans-Asia are required to work half-day on
Saturdays, 26 days should be added to the divisor of 262 days, thus, resulting to 288
days. However, due to the fact that the rest days of petitioners fall on a Sunday, the
number of unworked but paid legal holidays should be reduced to nine (9), instead of ten
(10), since one legal holiday under E.O. No. 203 always falls on the last Sunday of August,
National Heroes Day. Thus, the divisor that should be used in the present case should be
287 days.

A2010

- 136 -

Disini

- However, the Court notes that if the divisor is increased to 287 days, the resulting daily
rate for the purposes of overtime pay, holiday pay and conversions of accumulated leaves
would be diminished.
- Clearly, this muddled situation would be violative of the proscription on the nondiminution of benefits under Section 100 of the Labor Code. On the other hand, the use of
the divisor of 287 days would be to the advantage of petitioners if it is used for purposes of
computing for deductions due to the employee's absences. In view of this situation, the
Court rules that the adjusted divisor of 287 days should only be used by Trans-Asia for
computations which would be advantageous to petitioners and not for computations which
would diminish the existing benefits of the employees.
2. NO
- As previously stated, the decision of the labor arbiter and the resolutions of the NLRC
were based on substantial evidence and, as such, no ambiguity or doubt exists which
could be resolved in petitioner's favor.
Disposition Affirmed with modification (on the divisor that Trans- Asia should use in
computing the salaries of its employees including holiday pay)

COMPUTATION
AGGA V NLRC (SUPPLY OILFIELD SERVICES INC,
UNDERSEAS DRILLING INC)
298 SCRA 285
PUNO; November 16, 1998
NATURE
Petition for Certiorari
FACTS
- Private respondent Supply Oilfield Services, Inc. hired petitioners to work on board a
drillship owned and operated by private respondent Underseas Drilling, Inc.
- The employment contracts ran for one year with petitioners enjoying two months off with
pay for every two months' duty. The contracts also provided that for service of 12 hours a
day, 7 days a week in a two-shift 24-hour operation, petitioners would receive a fixed
monthly compensation covering "basic rate, allowances, privileges, travel allowances and
benefits granted by law during and after employment with the company."
- Petitioners claim:
> private respondents failed to pay them overtime pay, holiday pay, rest day pay, 13th
month pay and night shift differential. They likewise alleged that private respondents did
not comply with the mandatory insurance requirement. They further averred that while
private respondents made them use passports for overseas contract workers, they were
also instructed to use seaman's books upon reaching port for transfer to, and while
aboard, the oilrig. Petitioners opined that this practice entitled them to the benefits granted
by law to both land-based workers and seamen.
- Respondents contend:
> denied liability. In addition, they alleged that petitioners were insured with Blue Cross
(Asia-Pacific) Insurance, Ltd. against death and permanent disability. Lastly, private
respondents contended that petitioners, as offshore oilriggers, had nothing to do with
manning a vessel or sea navigation. Hence, petitioners were merely land-based workers,
not seamen.
- On July 2, 1992, the POEA dismissed petitioners' complaint for lack of merit.
- Petitioners appealed to the National Labor Relations Commission.
- Upon the other hand, private respondents informed the NLRC that the POEA had already
dismissed the claims for underpayment of labor benefits and lack of insurance coverage in
the consolidated cases, and that the dismissal was affirmed on appeal. The decision has
become final.
- On November 27, 1995, the respondent NLRC promulgated the Decision assailed
herein, dismissing petitioners' appeal.
- In an appeal dated February 26, 1993, the complainants questioned the aforesaid
decision.
- On January 17, 1996, petitioners filed a motion for reconsideration. In an Order 4 dated
January 30, 1996, the respondent NLRC denied petitioners' motion.
ISSUES
1. WON respondent NLRC acted without or in excess of jurisdiction or with grave abuse
of discretion amounting to lack or excess jurisdiction in not declaring that the lumpsum
mode of payment of petitioners monthly salaries by private respondents is illegal
2. WON respondent NLRC erred in not ordering private respondents, jointly and severally,
to pay the admitted underpayment as shown by private respondents computation
3. WON respondent NLRC erred in not ordering the POEA to comply with its mandated
duty to set up standard employment contract and guiding rates for oilrig workers
4. WON respondent NLRC erred in not declaring that private respondents failed to comply
with the legal requirement of mandatory personal insurance
5. WON respondent NLRC erred in not penalizing private respondent

Labor Law 1
6. WON respondent NLRC erred in not awarding damages and attorneys fees to
petitioners
HELD
1. NO
- Petitioners contend that the lumpsum mode of payment of salaries is illegal, citing
Articles 5 and 6 of the New Civil Code, Articles 86, 87, 90, 93 and 94 of PD 442 and Book
V, Rule II, Section 2(a) of the 1991 POEA Rules.
- As correctly observed by the respondents, none of the aforementioned laws and rules
prohibit the subject payment scheme. The cited articles of the New Civil Code merely
provide that agreements in violation of law or public policy cannot be entered into and
have legal effect. The cited provisions of PD 442 simply declare that night shift differential
and additional remuneration for overtime, rest day, Sunday and holiday work shall be
computed on the basis of the employee's regular wage. In like fashion, the 1991 POEA
Rules merely require employers to guarantee payment of wages and overtime pay. Thus,
petitioners' stance is bereft of any legal support.
2. NO
- Petitioners allege that their fixed monthly salaries represented only their basic salaries
and did not include overtime pay, holiday pay, 13th month pay and night shift differential.
- Decisions and/or awards of the Administration shall be final and executory unless
appealed to the National Labor Relations Commission (NLRC) by any or both parties, it
then follows that , to the extent that the POEA has concluded that there is 'no case of
underpayment at bar,' the same has to be bindingly observed by us vis-a-vis complainants'
submitted issue in their draft decision of whether or not there had been underpayments as
claimed by appellants under the provisions of P.D. 442.
3. NO
- The matter of ordering the NLRC to compel the POEA to set up standard employment
contract and guiding rates for oilrig workers is beyond the jurisdiction of this Court.
4. NO
- The POEA and the NLRC have found that private respondents insured petitioners with
Blue Cross (Asia-Pacific) Insurance, Ltd. under two policies which even provide for
coverage superior to that mandated by the rules. Before this Court, however, petitioners
assail these insurance policies as they were allegedly issued by a foreign insurance
company not licensed to do business in the Philippines. The contention is raised for the
first time and cannot be considered.
5. NO
- Evidence shows that petitioners are land-based workers and hence, not entitled to
benefits appertaining to sea-based workers. Petitioners have nothing to do with manning
vessels or with sea navigation. Their use of a seaman's book does not detract from the
fact that they are truly land-based employees. Petitioners' plea that we suspend SOS'
license for making them use two (2) passports is off-line. Again, they never prayed for this
relief before the POEA and the NLRC. This Court is the improper venue for the belated
plea.
6. NO
- The claims for attorney's fees and damages of the petitioners have no basis as private
respondents did not act in bad faith or with malice.
Disposition assailed decision of NLRC affirmed.

SUNDAY

A2010

FACTS
- A labor Enforcement Officer conducted a routine inspection of the Wellington Flour Mills,
owned and operated by Wellington Investment and Manufacturing Corporation, and

Disini

ISSUE
WON a monthly-paid employee, receiving a fixed monthly compensation, is entitled to an
additional pay aside from his usual holiday pay, whenever a regular holiday falls on a
Sunday
HELD
NO
- Wellington simply deducts 51 Sundays from the 365 days normally comprising a year
and used the difference, 314, as basis for determining the monthly salary. The monthly
salary thus fixed actually covers payment for 314 days of the year, including regular and
special holidays, as well as days when no work is done by reason of fortuitous cause, as
above specified, or causes not attributable to the employees.
- The monthly salary in Wellington for all 365 days of a year. The respondents' theory
would make each of the years in question, a year of 368 days. Pursuant to this theory, no
employer opting to pay his employees by the month would have any definite basis to
determine the number of days in a year for which compensation should be given to his
work force.
- There is no provision of law requiring any employer to make such adjustments in the
monthly salary rate set by him to take account of legal holidays falling on Sundays in a
given year, or, contrary to the legal provisions bearing on the point, otherwise to reckon a
year at more than 365 days. What the law requires of employers opting to pay by the
month is to assure that "the monthly minimum wage shall not be less than the statutory
minimum wage multiplied by 365 days divided by twelve," and to pay that salary "for all
days in the month whether worked or not," and "irrespective of the number of working
days therein." That salary is due and payable regardless of the declaration of any special
holiday in the entire country or a particular place therein, or any fortuitous cause
precluding work on any particular day or days (such as transportation strikes, riots, or
typhoons or other natural calamities), or cause not imputable to the worker. The legal
provisions governing monthly compensation are evidently intended precisely to avoid recomputations and alterations in salary on account of the contingencies just mentioned,
which, by the way, are routinely made between employer and employees when the wages
are paid on daily basis.
Disposition The orders complained of, namely: that of the respondent Undersecretary
dated September 22, 1993, and that of the Regional Director dated July 30, 1992, are
NULLIFIED AND SET ASIDE, and the proceeding against petitioner DISMISSED.

PROOF OF PAYMENT

WELLINGTON INVESTMENT V TRAJANO


245 SCRA 561
NARVASA; July 3, 1995
NATURE
Special Civil Action for Certiorari

- 137 -

reported the non-payment of regular holidays falling on a Sunday for monthly-paid


employees. A copy of the report was explained to and received by Wellingtons personnel
manager.
- Wellington sought reconsideration and argued that "the monthly salary of the company's
monthly-salaried employees already includes holiday pay for all regular holidays and
hence there is no legal basis for the finding of alleged non-payment of regular holidays
falling on a Sunday." In a position paper subsequently submitted to the Regional Director,
it asserted that it pays its monthly-paid employees a fixed monthly compensation "using
the 314 factor which undeniably covers and already includes payment for all the working
days in a month as well as all the 10 unworked regular holidays within a year."
- July 28, 1992: the Regional Director ruled that "when a regular holiday falls on a Sunday,
an extra or additional working day is created and the employer has the obligation to pay
the employees for the extra day except the last Sunday of August since the payment for
the said holiday is already included in the 314 factor," and accordingly directed Wellington
to pay its employees compensation corresponding 4 extra working days.
- September 22: the Undersecretary affirmed the challenged order, holding that "the
divisor being used by Wellington does not reliably reflect the actual working days in a
year," and consequently commanded Wellington to pay its employees the "six additional
working days resulting from regular holidays falling on Sundays in 1988, 1989 and 1990."
He said that whenever a regular holiday coincides with a Sunday, an additional working
day is created and left unpaid. In other words, while the said divisor may be utilized as
proof evidencing payment of 302 working days, 2 special days and the ten regular
holidays in a calendar year, the same does not cover or include payment of additional
working days created as a result of some regular holidays falling on Sundays.

BUILDING CARE CORP V NLRC (RODIL)


268 SCRA 666
PANGANIBAN; February 26, 1997
NATURE
Petition for certiorari under rule 56 of the ROC
FACTS

Labor Law 1
- Private respondent, Rodil, a janitor under Building Care Corp. (BCC) and working at Far
East Bank and Trust Co. (FEBTC) alleged that his wages, 13 th month pay and service
incentive leave pay were unpaid; that he was not paid for work rendered during legal
holidays; that on Feb. 11, 1988 he was suspended for a week without just cause or the
requirements of due process.
- BCC on the other hand, alleged that Rodil had been given his wages and holiday pay
and that it was willing to pay his salary differentials from Dec. to Feb. FEBTC once
complained that Rodil had improperly cleaned his area and twice failed to report to the
night shift supervisor when instructed to do so. Also, the suspension on Feb. 11 was
erroneously noted in the logbook but private respondent was allowed to work the next day
as shown by the time records. He came to work only a month later and, when prompted to
explain his long absence, said he had been suspended by his supervisor for no reason
and was advised to report on April 4 to await the investigation on the alleged suspension.
FEBTC later indicated it would no longer accept Rodil and he was advised that he was to
be temporarily assigned as reliever at BCC while no other posts were available. When
advised to report to BCC, he never appeared but instead filed the instant case.
- Rodil maintained that he had been absent for that long period because he had been sick,
then later had to care for his wife who also got sick, and was again absent for the last
week because he was illegally suspended.
- Rodil filed with the NLRC a complaint for illegal dismissal, underpayment and nonpayment of legal holiday pay against petitioner. At the initial hearing, Rodil was offered
reinstatement but he insisted on payment of backwages; petitioner did not agree, so both
parties submitted their respective position papers. The Labor Arbiter issued a decision in
favor of private respondent, hence this petition for certiorari.
ISSUE
WON NLRC erred in awarding Rodil holiday pay and salary differentials
HELD
YES
- If BCC had really paid Rodil his holiday pay, it could easily have presented its payrolls,
which constitute the best proof of payment. To prove payment of salary differentials, it
could have presented proofs of such monetary benefitsbut it did not. It failed to comply
with the mandate of the law; as NLRC ruled, the burden of proof in this regard lies with the
employer, not the employee.
Disposition Petition is DISMISSED and the assailed Decision is AFFIRMED

SECTION 8: CONDITIONS OF EMPLOYMENT


LEAVES
A. SERVICE INCENTIVE LEAVE

8.01 COVERAGE
MAKATI HABERDASHERY INC V NLRC (SANDIGANTUCP AND MEMBERS GRACIANO, ET AL)
179 SCRA 449
FERNAN; November, 15 1989
FACTS
- Private respondents have been working for petitioner Makati Haberdashery, Inc.
(Haberdashery) as tailors, seamstress, sewers, basters and "plantsadoras". They were
paid on a piece-rate basis (except two who are paid on a monthly basis). In addition to
their piece-rate, they were given a daily allowance of P3.00 pesos provided they report for
work before 9:30 a.m. everyday. They were required to work from or before 9:30 a.m. to
6:00 or 7:00 p.m. Mon to Sat, and during peak periods even on Sundays and holidays.
- The Sandigan ng Manggagawang Pilipino filed a complaint for underpayment of the
basic wage; underpayment of living allowance; non-payment of overtime work; non-

A2010

- 138 -

Disini

payment of holiday pay; non-payment of service incentive pay; 13th month pay; and
benefits provided for under Wage Orders Nos. 1, 2, 3, 4 and 5.
- During the pendency of case, private respondent Pelobello left a package w/ a salesman
of the Haberdashery, w/c contained a barong tagalog. When confronted, Pelobello said
that it was respondent Zapatas. Zapata allegedly admitted that he copied the design of
Haberdashery. A memorandum was issued to each of them to explain why no action
should be taken against them for accepting a job order which is prejudicial and in direct
competition with the business of the company. Both respondents did not submit their
explanation and did not report for work. They were dismissed by petitioner. They
countered by filing a complaint for illegal dismissal.
- Labor Arbiter found Haberdashery guilty of illegal dismissal and ordered them to reinstate
Pelobello and Zapata. The charge of unfair labor practice and claims for underpayment re
violation of the minimum wage law were dismissed for lack of merit. And found
Haberdashery to have violated the decrees on the cost of living allowance, service
incentive leave pay and the 13th Month Pay. NLRC affirmed decision.
ISSUES
1. WON an employee-employer relationship existed between Haberdashery and
respondent workers
2. WON the respondent workers were entitled to their monetary claims (COLA, service
incentive, 13th month pay, etc)
3. WON Pelobello and Zapata were illegally dismissed
HELD
1. YES
Ratio The facts at bar indubitably reveal that the most important requisite of control is
present. When a customer enters into a contract with the haberdashery or its proprietor,
the latter directs an employee who may be a tailor, pattern maker, sewer or "plantsadora"
to take the customer's measurements, and to saw the pants, coat or shirt as specified by
the customer. Supervision is actively manifested in all these aspects -- the manner and
quality of cutting, sewing and ironing.
- The test of employee-employer relationship is four-fold: (1) the selection and
engagement of the employee; (2) the payment of wages; (3) the power of dismissal; and
(4) the power to control the employee's conduct. It is the so-called "control test" that is the
most important element. This means the determination of whether the employer controls
or has reserved the right to control the employee not only as to the result of the work but
also as to the means and method by which the same is to be accomplished. - It is evident
that petitioner has reserved the right to control its employees not only as to the result but
also the means and methods by which the same are to be accomplished. That private
respondents are regular employees is further proven by the fact that they have to report
for work regularly from 9:30 a.m. to 6:00 or 7:00 p.m. and are paid an additional allowance
of P3.00 daily if they report for work before 9:30 a.m.
- Private respondents did not exercise independence in their own methods, but on the
contrary were subject to the control of petitioners from the beginning of their tasks to their
completion. Unlike independent contractors who generally rely on their own resources, the
equipment, tools, accessories, and paraphernalia used by private respondents are
supplied and owned by petitioners. Private respondents are totally dependent on
petitioners in all these aspects.
2. Minimum Wage- YES; COLA YES
- As a consequence of their status as regular employees of the petitioners, they can claim
cost of living allowance. This is apparent from the provision defining the employees
entitled to said allowance, thus: ". . . All workers in the private sector, regardless of their
position, designation or status, and irrespective of the method by which their wages are
paid."
13th Month Pay YES
- Section 3(e) of the Rules and Regulations Implementing P.D. No. 851 provides: " The
Decree shall apply to all employers except to:
xxx
xxx
xxx
(e) Employers of those who are paid on purely commission, boundary, or task basis,
and those who are paid a fixed amount for performing a specific work, irrespective of
the time consumed in the performance thereof, except where the workers are paid on
piece-rate basis in which case the employer shall be covered by this issuance insofar
as such workers are concerned."
SERVICE INCENTIVE LEAVE PAY NO
- While private respondents are entitled to Minimum Wage, COLA and 13th Month Pay,
they are not entitled to service incentive leave pay because as piece-rate workers
being paid at a fixed amount for performing work irrespective of time consumed in
the performance thereof, they fall under one of the exceptions stated in Section
1(d), Rule V, Implementing Regulations, Book III, Labor Code. (pls see provision)
Holiday Pay- NO.
- Private respondents cannot also claim holiday pay under Section 1(e), Rule IV,
Implementing Regulations, Book III, Labor Code.
3. NO
- Haberdashery had valid grounds to terminate the services of private respondents.

Labor Law 1
- It does show that a violation of the employer's rules has been committed and the
evidence of such transgression, the copied barong tagalog, was in the possession of
Pelobello who pointed to Zapata as the owner. When required by their employer to explain
in a memorandum issued to each of them, they not only failed to do so but instead went
on AWOL, waited for the period to explain to expire and for petitioner to dismiss them.
Assuming that such acts do not constitute abandonment of their jobs as insisted by private
respondents, their blatant disregard of their employer's memorandum is undoubtedly an
open defiance to the lawful orders of the latter, a justifiable ground for termination of
employment by the employer expressly provided for in Article 283(a) of the Labor Code as
well as a clear indication of guilt for the commission of acts inimical to the interests of the
employer, another justifiable ground for dismissal under the same Article of the Labor
Code, paragraph (c). Well established in our jurisprudence is the right of an employer to
dismiss an employee whose continuance in the service is inimical to the employer's
interest.
- We have ruled that:
"No employer may rationally be expected to continue in employment a person whose
lack of morals, respect and loyalty to his employer, regard for his employer's rules, and
appreciation of the dignity and responsibility of his office, has so plainly and completely
been bared.
"That there should be concern, sympathy, and solicitude for the rights and welfare of
the working class, is meet and proper. That in controversies between a laborer and his
master, doubts reasonably arising from the evidence, or in the interpretation of
agreements and writings should be resolved in the former's favor, is not an
unreasonable or unfair rule. But that disregard of the employer's own rights and
interests can be justified by that concern and solicitude is unjust and unacceptable."
(Stanford Microsystems, Inc. v. NLRC, 157 SCRA 414-415 [1988]).
- The law is protecting the rights of the laborer authorizes neither oppression nor selfdestruction of the employer. More importantly, while the Constitution is committed to the
policy of social justice and the protection of the working class, it should not be supposed
that every labor dispute will automatically be decided in favor of labor.
- The right to dismiss or otherwise impose disciplinary sanctions upon an employee for just
and valid cause, pertains in the first place to the employer, as well as the authority to
determine the existence of said cause in accordance with the norms of due process.

8.02 ENTITLEMENT AND ARBITRATION


8.03 COMPUATION AND LIABILITY
SENTINEL SECURITY AGENCY INC V NLRC (CABANO)
295 SCRA 123
PANGANIBAN; September 3, 1998
FACTS
- The complainants were employees of Sentinel Security Agency, Inc. They were
assigned to render guard duty at the premises of [Philippine American Life Insurance
Company] at Jones Avenue, Cebu City. On December 16, 1993 Philippine American Life
Insurance Company [the Client, ], sent notice to all concerned that the [Agency] was
again awarded the contract of security services together with a request to replace all the
security guards in the companys offices at the cities of Cebu, Bacolod, Cagayan de Oro,
Dipolog and Ilagan. In compliance therewith, [the Agency] issued a Relief and Transfer
Order replacing the complainants as guards [of the Client] and for then to be re-assigned
to other clients. As ordered, the complainants reported but were never given new
assignments but instead they were told that they were replaced because they are already
old. Precisely, the complainants lost no time but filed the subject illegal dismissal cases
and prayed for payment of separation pay and other labor standard benefits.
- Defendants Comment
"[The Client and the Agency] maintained there was no dismissal on the part of the
complainants, constructive or otherwise, as they were protected by the contract of security
services which allows the recall of security guards from their assigned posts at the will of
either party. It also advanced that the complainants prematurely filed the subject cases
without giving the [Agency] a chance to give them some assignments.
- On the part of [the Client], it averred further that there [was] no employer-employee
relationship between it and the complainants as the latter were merely assigned to its
Cebu Branch under a job contract; that [the Agency] ha[d] its own separate corporate
personality apart from that of [the Client]. Besides, it pointed out that the functions of the
complainants in providing security services to [the Clients] property [were] not necessary
and desirable to the usual business or trade of [the Client], as it could still operate and
engage in its life insurance business without the security guards. In fine, [the Client]
maintains that the complainants have no cause of action against it."

A2010

- 139 -

Disini

ISSUES
1. WON the complainants were illegally dismissed
2. WON the Client is jointly and severally liable for their thirteenth-month and service
incentive leave pays
HELD
1. YES
Ratio The transfer of an employee involves a lateral movement within the business or
operation of the employer, without demotion in rank, diminution of benefits or, worse,
suspension of employment even if temporary. The recall and transfer of security guards
require reassignment to another post and are not equivalent to their placement on "floating
status." Off-detailing security guards for a reasonable period of six months is justified only
in bona fide cases of suspension of operation, business or undertaking.
Reasoning
a. The legally recognized concept of transfer was not implemented. The agency hired new
security guards to replace the complainants, resulting in a lack of posts to which the
complainants could have been reassigned. Thus, it refused to reassign Complainant
Andoy when he reported for duty on February 2, 4 and 7, 1994; and merely told the other
complainants on various dates from January 25 to 27, 1994 that they were already too old
to be posted anywhere.
b. A floating status requires the dire exigency of the employers bona fide suspension of
operation, business or undertaking. In security services, this happens when the clients that
do not renew their contracts with a security agency are more than those that do and the
new ones that the agency gets. However, in the case at bar, the Agency was awarded a
new contract by the Client. There was no surplus of security guards over available
assignments. If there were, it was because the Agency hired new security guards. Thus,
there was no suspension of operation, business or undertaking, bona fide or not, that
would have justified placing the complainants off-detail and making them wait for a period
of six months. If indeed they were merely transferred, there would have been no need to
make them wait for six months.
(2) The Client did not, as it could not, illegally dismiss the complainants. Thus, it should
not be held liable for separation pay and back wages. But even if the Client is not
responsible for the illegal dismissal of the complainants, it is jointly and severally liable
with the Agency for the complainants service incentive leave pay.
Disposition The petition is DISMISSED and the assailed Decision and Resolution are
hereby AFFIRMED, but the award of the thirteenth-month pay is DELETED.

AUTO BUS TRANSPORT SYSTEMS INC V BAUTISTA


[PAGE 123]

Labor Law 1
NATIONAL MINES AND ALLIED WORKERS UNION V
SAN ILDEFEONSO COLLEGE, ETC.
299 SCRA 24
DAVIDE JR; November 20, 1998
NATURE
Petition for certiorari seeking to set aside an NLRC decision and resolution denying a
motion for reconsideration
FACTS
- National Mines and Allied Workers Union is the certified bargaining agent of the rank
and file employees of respondent College. Petitioner Juliet Arroyo was the president of the
San Ildefonso College Association of Faculty and Personnel, an affiliate of NAMAWU.
Private respondent Lloren is the directress of the College.
- In February, 1991, ARROYO, a tenured teacher who later became a part-time teacher,
asked that she be allowed to teach on a full-time basis. The COLLEGE denied her request
for her failure to make use of the privilege of her study leave in the two years she was
allowed to do so. The next month, the other individual petitioners, who were issued yearly
appointment, were informed of the non-renewal of their respective contracts.
- In April, 1991, the SICAFP was formalized into a labor union affiliated with NAMAWU.
- The petitioners and NAMAWU filed a complaint for illegal dismissal, unfair labor practice,
forced resignation, harassment, underpayment of wages, non-payment of service
incentive leave pay, and violation of Waeg Order No. IV-1. They demanded reinstatement
and payment of back wages.
- The Labor Arbiter held private respondents guilty of illegal dismissal, unfair labor practice
interfering with the organization of the labor union. The contracts of employment were not
bilateral agreements, but letters of appointment. When the College opted not to renew the
appointments it merely invoked the expiration of the period fixed in the appointments
without giving any other reason or granting the teachers concerned an opportunity to
explaint heir side. The probationary employees were not even informed of their
performance rating when they were denied renewal of their appointment. The non-renewal
was timely made while individual petitioners were in the process of organizing themselves
into a union. These acts of the College amounted to union busting.
- The Office of the Solicitor General moves for the dismissal of the petition except as to
ARROYO; that all petitioners except ARROYO were legally dismissed. The reason why
she failed to complete her masters degree could not be solely attributed to her. She
initially requested a leave of absence, but the COLLEGE suggested that she teach on a
part-time basis because it was in need of teachers at that time. Also, her dismissal was
without due process.
ISSUE
1. WON ARROYO was legally dismissed
2. WON the other petitioners were permanent employees
HELD
1. NO
Reasoning
- it is undisputed that Arroyo had been teaching in the COLLEGE since 1965 and had
obtained a permanent status; she became a part-time teacher, however, from June 1988
to March 1991.
- She did not lose her permanent status when she requested to teach on a part-time basis.
The reason for the request was that she wanted to pursue a master's degree. The
COLLEGE approved the request, and the study leave was extended for another year. It
would have been unjust and unreasonable to allow ARROYO to pursue her master's
degree, from which the COLLEGE would have also benefited in terms of her higher
learning and experience, and at the same time penalize her with the loss of permanent
status. It would as well be absurd and illogical to maintain that by teaching on a part-time
basis after obtaining the permission to take up a master's degree, ARROYO relinquished
her permanent status.
- When ARROYO subsequently requested that she continue teaching on a full-time basis,
private respondents in its letter of 27 March 1991 refused, citing as reason her failure "to
make use of the privilege granted [her] by the administration regarding [her] study leave in
the past four semesters." This letter served as notice of ARROYO's termination from
employment. No further notice was served. It must be emphasized that the letter did not
indicate that a master's degree was necessary for ARROYO to continue her service, as
now claimed by the COLLEGE. In fact, apart from its mere allegation, the COLLEGE failed
to prove that a master's degree was a pre-requisite for ARROYO's teaching position.
ARROYO, a permanent teacher, could only be dismissed for just cause and only after
being afforded due process, in light of paragraph (b), Article 277 of the Labor Code.
- Arroyos dismissal was substantively and procedurally flawed. It was effected without just
cause and due process. Thus, her termination was void. She is therefore entitled to
reinstatement to her former position without loss of seniority rights and other privileges, full
backwages inclusive of allowances, and other benefits computed from the date of her
actual dismissal to the date of reinstatement

A2010

Disini

- 140 -

2. NO
Reasoning
- On the issue of whether the individual petitioners were permanent employees, it is the
Manual of Regulations for Private Schools, and not the Labor Code, which is applicable.
This was settled in University of Sto. Tomas v. NLRC, where we explicitly ruled that for a
private school teacher to acquire permanent status in employment and, therefore, be
entitled to security of tenure, the following requisites must concur: (1) the teacher is a fulltime teacher; (2) the teacher must have rendered three (3) consecutive years of service;
and (3) such service must have been satisfactory.
- Eleven of the individual petitioners were full-time teachers during the school year 19901991, but only two, namely, Odiste and Buan had rendered three consecutive years of
service. There is no showing, however, that the two were on a full-time basis during those
three years and that their services were satisfactory. Evidently, not one of the said
teachers can be considered to have acquired a permanent status.
Disposition the decision of the National Labor Relations Commission in NLRC Case No.
RAB-IV-4-3710-91-RI is AFFIRMED, subject to the modification that private respondent
San Ildefonso College is DIRECTED to (1) reinstate petitioner JULIETA ARROYO to her
former position at the time of her dismissal, or to any equivalent position if reinstatement to
such position is no longer feasible, without of loss of seniority rights and benefits that may
be due her; and (2) pay her back wages from the date of her actual dismissal to the date
of her actual reinstatement.

CIELO V NLRC
193 SCRA 410
CRUZ; January 28, 1991
NATURE
Petition for certiorari to review decision of NLRC setting aside decision of Labor Arbiter for
the reinstatement with backwages of Zosimo Cielo.
FACTS
Henry Lei Trucking hired Zosimo Cielo as a truck driver under 6-month Agreement with
stipulations that the term is can be earlier terminated at the option of either party. The
Agreement also stipulated that there was no employer-employee relationship between the
parties and that the nature of the relationship is merely contractual. Lei asked Cielo to sign
an affidavit of having received full payment of wages, which Cielo refused to sign. A week
before the Agreement was supposed to end, Lei notified Cielo of the termination of his
services. Apparently in the Agreements with the drivers, Lei merely fills in the blanks with
the corresponding data such as the drivers name and address, etc.
ISSUE
WON the Agreement was valid
HELD
NO
Ratio Where from the circumstances it is apparent that the periods were imposed in order
to preclude the acquisition of tenurial security by the employee, they should be struck
down or disregarded for being contrary to public policy, morals,etc.
Reasoning
- The Agreement is void ab initio for having a purpose contrary to public policy. The
agreement was a clear attempt to exploit the employee and deprive him of the protection
of the Labor Code by making it appear that the stipulations are governed by the Civil Code
as in ordinary private transactions. In reality the agreement was a contract of employment
into which were read the provisions of the Labor Code and the social justice policy of the
Constitution. That Cielo refused to sign the affidavit was not a just cause for his
termination as he was only protecting his interest against unguarded waiver of the benefits
due him under the Labor Code. Said affidavit which stipulated payment of wages even
suggested that there was indeed an employer-employee relationship.
Disposition NLRC decision set aside. LA decision reinstated.

GENERAL MILLING CORPORATION V TORRES


196 SCRA 215
FELICIANO; April 22, 1991
NATURE
Petition for certiorari review.
FACTS
- DOLE NCR issued Alien Employment Permit in favor of petitioner Earl Timothy Cone, a
United States citizen, as sports consultant and assistant coach for GMC. GMC and Cone
entered into a contract of employment whereby the latter undertook to coach GMC's
basketball team. Board of Special Inquiry of the Commission on Immigration and

Labor Law 1
Deportation approved petitioner Cone's application for a change of admission status from
temporary visitor to prearranged employee.
- On 9 February 1990, petitioner GMC requested renewal of petitioner Cone's alien
employment permit. GMC also requested that it be allowed to employ Cone as full-fledged
coach. The DOLE Regional Director, Luna Piezas, granted the request. Alien Employment
Permit was issued.
- Private respondent Basketball Coaches Association of the Philippines ("BCAP")
appealed the issuance of said alien employment permit to the respondent Secretary of
Labor who issued a decision ordering cancellation of petitioner Cone's employment permit
on the ground that there was no showing that there is no person in the Philippines who is
competent, able and willing to perform the services required nor that the hiring of petitioner
Cone would redound to the national interest.
ISSUES
1. WON Secretary of Labor gravely abused his discretion when he revoked petitioner
Cone's alien employment permit
2. WON Section 6 (c), Rule XIV, Book I of the Omnibus Rules Implementing the Labor
Code is null and void as it is in violation of the enabling law as the Labor Code does not
empower respondent Secretary to determine if the employment of an alien would redound
to national interest
HELD
1. NO
- Petitioners have failed to show any grave abuse of discretion or any act without or in
excess of jurisdiction on the part of respondent Secretary of Labor in rendering his
decision, revoking petitioner Cone's Alien Employment Permit.
- The alleged failure to notify petitioners of the appeal filed by private respondent BCAP
was cured when petitioners were allowed to file their Motion for Reconsideration before
respondent Secretary of Labor.
2. NO
- The Labor Code itself specifically empowers respondent Secretary to make a
determination as to the availability of the services of a "person in the Philippines who is
competent, able and willing at the time of application to perform the services for which an
alien is desired." In short, the Department of Labor is the agency vested with jurisdiction to
determine the question of availability of local workers.
- Under Article 40 of the Labor Code, an employer seeking employment of an alien must
first obtain an employment permit from the Department of Labor. Petitioner GMC's right to
choose whom to employ is, of course, limited by the statutory requirement of an alien
employment permit.
- Petitioners will not find solace in the equal protection clause of the Constitution. As
pointed out by the Solicitor-General, no comparison can be made between petitioner Cone
and Mr. Norman Black as the latter is "a long time resident of the country," and thus, not
subject to the provisions of Article 40 of the Labor Code which apply only to "non-resident
aliens." In any case, the term "non-resident alien" and its obverse "resident alien," here
must be given their technical connotation under our law on immigration.
- Neither can petitioners validly claim that implementation of respondent Secretary's
decision would amount to an impairment of the obligations of contracts. The provisions of
the Labor Code and its Implementing Rules and Regulations requiring alien employment
permits were in existence long before petitioners entered into their contract of
employment. It is firmly settled that provisions of applicable laws, especially provisions
relating to matters affected with public policy, are deemed written into contracts. Private
parties cannot constitutionally contract away the otherwise applicable provisions of law.
- In short, the Department of Labor is the agency vested with jurisdiction to determine the
question of availability of local workers. The constitutional validity of legal provisions
granting such jurisdiction and authority and requiring proof of non-availability of local
nationals able to carry out the duties of the position involved, cannot be seriously
questioned.
- Petitioners apparently suggest that the Secretary of Labor is not authorized to take into
account the question of whether or not employment of an alien applicant would "redound
to the national interest" because Article 40 does not explicitly refer to such assessment.
This argument (which seems impliedly to concede that the relationship of basketball
coaching and the national interest is tenuous and unreal) is not persuasive. In the first
place, the second paragraph of Article 40 says: "[t]he employment permit may be issued to
a non-resident alien or to the applicant employer after a determination of the nonavailability of a person in the Philippines who is competent, able and willing at the time of
application to perform the services for which the alien is desired."
- The permissive language employed in the Labor Code indicates that the authority
granted involves the exercise of discretion on the part of the issuing authority. In
the second place, Article 12 of the Labor Code sets forth a statement of objectives that the
Secretary of Labor should, and indeed must, take into account in exercising his authority
and jurisdiction granted by the Labor Code.
Disposition Court Resolved to DISMISS the Petition for Certiorari for lack of merit.

A2010

- 141 -

Disini

MANILA TERMINAL COMPANY INC V CIR(MANILA


TERMINAL RELIEF AND MUTUAL AID ASSN)
91 PHIL 625
PARAS; July 16, 1952
FACTS
- Manila Terminal Co undertook arrastre service in Port Area, under control of US Army. It
hired watchmen on 12 hr shifts.
- Manila Terminal began post-war operation of arrastre service under control of Bureau of
Customs. The watchmen continued in the service, with salary raise.
A member of the Manila Terminal Relief and Mutual Aid Association wrote to Dept of Labor
requesting that the matter of overtime pay be investigated, but nothing happened.
- Members of the Association filed demand with Department of Labor, including overtime
pay, but nothing happened.
- Manila Terminal Company instituted system of strict 8 hr shifts.
- The Association was organized for the first time, and an amended petition was filed with
CIR praying that the petitioner be ordered to pay its watchmen or police force overtime
pay.
- The petitioners police force was consolidated with the Manila Harbor Police of the
Customs Patrol Service, a govt agency under Commissioner of Customs and Secretary of
Finance.
- CIR, while dismissing other demands, ordered the petitioner to pay its police force
regular or base pay and overtime compensation. With reference to overtime pay after the
watchmen had been integrated into the Manila Harbor Police, the judge ruled that court
has no jurisdiction because it affects the Bureau of Customs.
- In a separate opinion, Judge Lanting ruled:
> decision should be affirmed in so far as it grants compensation for overtime on regular
days
> as to compensation for work on Sundays and legal holidays, petitioner should pay
compensation that corresponds to the overtime at the regular rate only
> watchmen are not entitled to night differential
ISSUE
WON overtime pay should be granted to the workers
HELD
YES
- Petitioner stressed that the contract between it and the Association stipulates 12 hrs a
day at certain rates including overtime, but the record does not bear out these allegations.
- In times of acute employment, people go from office to office to search for work, and the
workers here found themselves required to render 12 hrs a day. True, there was an
agreement, but did the workers have freedom to bargain much less insist in the
observance of the Eight Hour Labor Law?
- We note that after petitioner instituted 8 hr shifts, no reduction was made in salaries
which its watchmen received under the 12 hr agreement.
- Petitioners allegation that the Association had acquiesced in the 12 hr shifts for more
than 18 mos is not accurate. Only one of the members entered in September 1945. The
rest followed during the next few months.
- The Association cant be said to have impliedly waived the right to overtime pay, for the
obvious reason that it could not have expressly waived it.
- Estoppel and laches cant also be invoked against Association. First, it is contrary to
spirit of the Eight Hour Labor Law. Second, law obligates employer to observe it. Third,
employee is at a disadvantage as to be reluctant in asserting any claim.
- The argument that the nullity of the employment contract precludes recovery by the
Association of overtime pay is untenable. The employer may not be heard to plead its
own neglect as exemption or defense.
- Also, Commonwealth Act 444 expressly provides for payment of extra compensation in
cases where overtime services are required.
- The point that payment of overtime pay may lead to ruin of the petitioner cant be
accepted. It is significant that not all watchmen should receive back overtime pay for the
whole period, since the members entered the firm in different times.
- The Eight-Hour Labor Law was designed not only to safeguard the health and welfare of
the laborer or employee, but in a way to minimize unemployment by forcing employers, in
cases where more than 8-hour operation is necessary, to utilize different shifts of laborers
or employees working only for eight hours each.

AKLAN ELECTRIC COOPERATIVE INC V NLRC


(RETISO)

Labor Law 1
323 SCRA 258
GONZAGA-REYES; January 25, 2000
NATURE
Petition for certiorari and prohibition with prayer for writ of preliminary injunction and/or
temporary restraining order
FACTS
- January 22, 1991 by way of a resolution of the Board of Directors of AKELCO it allowed
the temporary holding of office at Amon Theater, Kalibo, Aklan upon the recommendation
of Atty. Leovigildo Mationg, then project supervisor, on the ground that the office at Lezo
was dangerous and unsafe.
- Majority of the employees including the herein complainants, continued to report for work
at Lezo, Aklan and were paid of their salaries. The complainants claimed that transfer of
office from Lezo, Aklan to Kalibo, Aklan was illegal because it failed to comply with the
legal requirements under P.D. 269, thus the they remained and continued to work at the
Lezo Office until they were illegally locked out therefrom by the respondents. Despite the
illegal lock out however, complainants continued to report daily to the location of the Lezo
Office, prepared to continue in the performance of their regular duties.Complainants who
continuously reported for work at Lezo, Aklan were not paid their salaries from June 1992
up to March 18, 1993.
- LA dismissed the complaints
- NLRC reversed and set aside the LAs decision and held that private respondents are
entitled to unpaid wages from June 16, 1992 to March 18, 1993
- Petitioner claims:
> compensable service is best shown by timecards, payslips and other similar documents
and it was an error for public respondent to consider the computation of the claims for
wages and benefits submitted merely by private respondents as substantial evidence.
ISSUE
WON private respondents are entitled to payment of wages for the period of June 1992
up to March 18,1993 (what is their proof)
HELD
NO
- NLRC based its conclusion on the following: (a) the letter of Leyson, Office Manager of
AKELCO addressed to AKELCO's General Manager, Atty. Mationg, requesting for the
payment of private respondents' unpaid wages from June 16, 1992 to March 18, 1993; (b)
the memorandum of said Atty. Mationg in answer to the letter request of Leyson where he
made an assurance that he will recommend such request; (c) the private respondents'
own computation of their unpaid wages.
- We find that the foregoing does not constitute substantial evidence to support the
conclusion that private respondents are entitled to the payment of wages from June 16,
1992 to March 18, 1993.
- Substantial evidence is that amount of relevant evidence which a reasonable mind
might accept as adequate to justify a conclusion. These evidences relied upon by
public respondent did not establish the fact that private respondents actually rendered
services in the Kalibo office during the stated period.
a. Letter of Pedrito Leyson to Atty. Mationg
> Pedrito Leyson is one of the herein private respondents who are claiming for unpaid
wages and we find his actuation of requesting in behalf of the other private respondents
for the payment of their backwages to be biased and self-serving, thus not credible.
> On the other hand, petitioner was able to show that private respondents did not render
services during the stated period. Petitioner's evidences show that on January 22, 1992,
petitioner's Board of Directors passed a resolution temporarily transferring the Office from
Lezo, Aklan to Amon Theater, Kalibo, Aklan .With the transfer of petitioner's business
office from its former office, Lezo, to Kalibo, Aklan, its equipments, records and facilities
were also removed from Lezo and brought to the Kalibo office where petitioner's official
business was being conducted; thus private respondents' allegations that they continued
to report for work at Lezo to support their claim for wages has no basis.
b. Response of Atty. Mationg to the letter-request of office manager Leyson
> Mationg's offer to recommend the payment of private respondents' wages is hardly
approval of their claim for wages. It is just an undertaking to recommend payment.
Moreover, the offer is conditional. It is subject to the condition that petitioner's Board of
Directors will give its approval and that funds were available. Mationg's reply to Leyson's
letter for payment of wages did not constitute approval or assurance of payment. The fact
is that, the Board of Directors of petitioner rejected private respondents demand for
payment (Board Resolution No. 496, s. 1993).
c. the private respondents' own computation of their unpaid wages
> We hold that public respondent erred in merely relying on the computations of
compensable services submitted by private respondents. There must be competent proof
such as time cards or office records to show that they actually rendered compensable
service during the stated period to entitle them to wages. It has been established that the
petitioner's business office was transferred to Kalibo and all its equipments, records and

A2010

- 142 -

Disini

facilities were transferred thereat and that it conducted its official business in Kalibo during
the period in question. It was incumbent upon private respondents to prove that they
indeed rendered services for petitioner, which they failed to do.

SSS V CA (AYALDE)
348 SCRA 1
YNARES-SANTIAGO; December 14, 2000
NATURE
Petition for review on certiorari
FACTS
- In a petition before the Social Security Commission, Margarita Tana, widow of the late
Ignacio Tana, Sr., alleged that her husband was, before his demise, an employee of
Conchita Ayalde as a farmhand in the two (2) sugarcane plantations she owned in
Pontevedra, La Carlota City (Hda. B-70) and leased from the University of the Philippines
(Hda. B-15-M). She further alleged that Tana worked continuously six (6) days a week,
four (4) weeks a month, and for twelve (12) months every year between January 1961 to
April 1979. For his labor, Tana allegedly received a regular salary according to the
minimum wage prevailing at the time.
- She further alleged that throughout the given period, social security contributions, as well
as medicare and employees compensation premiums were deducted from Tana's wages.
It was only after his death that Margarita discovered that Tana was never reported for
coverage, nor were his contributions/premiums remitted to the SSS. Consequently, she
was deprived of the burial grant and pension benefits accruing to the heirs of Tana had he
been reported for coverage.
- She prayed for the Commission to issue an order directing respondents Conchita Ayalde
and Antero Maghari as her administrator to pay the premium contributions of the deceased
Ignacio Tana, Sr. and report his name for SSS coverage; and for the SSS to grant
petitioner Margarita Tana the funeral and pension benefits due her.
- The SSS revealed that neither Hda. B-70 nor respondents Ayalde and Maghari were
registered members-employers of the SSS, and consequently, Ignacio Tana, Sr. was never
registered as a member-employee. Likewise, SSS records reflected that there was no
way of verifying whether the alleged premium contributions were remitted since the
respondents were not registered members-employers.
- Respondent Antero Maghari raised the defense that he was a mere employee who was
hired as an overseer of Hda. B-70 sometime during crop years 1964-65 to 1971-72, and
as such, his job was limited to those defined for him by the employer which never involved
matters relating to the SSS.
- For her part, respondent Ayalde belied the allegation that Ignacio Tana, Sr. was her
employee, admitting only that he was hired intermittently as an independent contractor to
plow, harrow, or burrow Hda. No. Audit B-15-M. Tana used his own carabao and other
implements, and he followed his own schedule of work hours. Ayalde further alleged that
she never exercised control over the manner by which Tana performed his work as an
independent contractor. Moreover, Ayalde averred that way back in 1971, the University
of the Philippines had already terminated the lease over Hda. B-15-M and she had since
surrendered possession thereof to the University of the Philippines. Consequently,
Ignacio Tana, Sr. was no longer hired to work thereon starting in crop year 1971-72, while
he was never contracted to work in Hda. B-70.
- SSS ruled in favor of Tana. CA ruled in favor of Ayalde.
ISSUE
WON an agricultural laborer who was hired on "pakyaw" basis can be considered an
employee entitled to compulsory coverage and corresponding benefits under the Social
Security Law
HELD
- The mandatory coverage under the SSS Law (Republic Act No. 1161, as amended by
PD 1202 and PD 1636) is premised on the existence of an employer-employee
relationship, and Section 8(d) defines an "employee" as "any person who performs
services for an employer in which either or both mental and physical efforts are used and
who receives compensation for such services where there is an employer-employee
relationship." The essential elements of an employer-employee relationship are: (a) the
selection and engagement of the employee; (b) the payment of wages; (c) the power of
dismissal; and (d) the power of control with regard to the means and methods by which
the work is to be accomplished, with the power of control being the most determinative
factor.
- There is no question that Tana was selected and his services engaged by either Ayalde
herself, or by Antero Maghari, her overseer. Corollarily, they also held the prerogative of
dismissing or terminating Tana's employment. The dispute is in the question of payment
of wages. Claimant Margarita Tana and her corroborating witnesses testified that her
husband was paid daily wages "per quincena" as well as on "pakyaw" basis. Ayalde, on
the other hand, insists that Tana was paid solely on "pakyaw" basis. To support her claim,

Labor Law 1
she presented payrolls covering the period January of 1974 to January of 1976 and
November of 1978 to May of 1979.
- A careful perusal of the records readily show that the exhibits offered are not complete,
and are but a mere sampling of payrolls. While the names of the supposed laborers
appear therein, their signatures are nowhere to be found. And while they cover the years
1975, 1976 and portions of 1978 and 1979, they do not cover the 18-year period during
which Tana was supposed to have worked in Ayalde's plantations. Also an admitted fact is
that these exhibits only cover Hda. B70, Ayalde having averred that all her records and
payrolls for the other plantation (Hda. B-15-M) were either destroyed or lost.
- To our mind, these documents are not only sadly lacking, they are also unworthy of
credence. The fact that Tana's name does not appear in the payrolls for the years 1975,
1976 and part of 1978 and 1979, is no proof that he did not work in Hda. B70 in the years
1961 to 1974, and the rest of 1978 and 1979. The veracity of the alleged documents as
payrolls are doubtful considering that the laborers named therein never affixed their
signatures to show that they actually received the amounts indicated corresponding to
their names. Moreover, no record was shown pertaining to Hda. B-15-M, where Tana was
supposed to have worked. Even Ayalde admitted that she hired Tana as "arador" and
sometimes as laborer during milling in Hda. B-15-M.[16] In light of her incomplete
documentary evidence, Ayalde's denial that Tana was her employee in Hda. B-70 or Hda.
B-15-M must fail. In contrast to Ayalde's evidence, or lack thereof, is Margarita Tana's
positive testimony, corroborated by two (2) other witnesses.
- The witnesses did not waver in their assertion that while Tana was hired by Ayalde as an
"arador" on "pakyaw" basis, he was also paid a daily wage which Ayalde's overseer
disbursed every fifteen (15) days. It is also undisputed that they were made to
acknowledge receipt of their wages by signing on sheets of ruled paper, which are
different from those presented by Ayalde as documentary evidence. In fine, we find that
the testimonies of Margarita Tana and the two other witnesses prevail over the incomplete
and inconsistent documentary evidence of Ayalde.
- No particular form of evidence is required to prove the existence of an employeremployee relationship. Any competent and relevant evidence to prove the relationship
may be admitted. For, if only documentary evidence would be required to show that
relationship, no scheming employer would ever be brought before the bar of justice, as no
employer would wish to come out with any trace of the illegality he has authored
considering that it should take much weightier proof to invalidate a written instrument.
- The testimonial evidence of the claimant and her witnesses constitute positive and
credible evidence of the existence of an employer-employee relationship between Tana
and Ayalde. As the employer, the latter is duty-bound to keep faithful and complete
records of her business affairs, not the least of which would be the salaries of the workers.
- The assertion that Tana is an independent contractor is specious because (1) while Tana
was sometimes hired as an "arador" or plower for intermittent periods, he was hired to do
other tasks in Ayalde's plantations. It is indubitable, as testified by the witnesses, that Tana
worked continuously for Ayalde, not only as "arador" on "pakyaw" basis, but as a regular
farmhand, doing backbreaking jobs for Ayalde's business. There is no shred of evidence
to show that Tana was only a seasonal worker, much less a migrant worker. All witnesses,
including Ayalde herself, testified that Tana and his family resided in the plantation. If he
was a mere "pakyaw" worker or independent contractor, then there would be no reason for
Ayalde to allow them to live inside her property for free. The only logical explanation is
that he was working for most part of the year exclusively for Ayalde, in return for which the
latter gratuitously allowed Tana and his family to reside in her property; and, (2) Ayalde
made much ado of her claim that Tana could not be her employee because she exercised
no control over his work hours and method of performing his task as "arador." A closer
scrutiny of the records, however, reveals that while Ayalde herself may not have directly
imposed on Tana the manner and methods to follow in performing his tasks, she did
exercise control through her overseer.
- Under the circumstances, the relationship between Ayalde and Tana has more of the
attributes of employer-employee than that of an independent contractor hired to perform a
specific project.
- Lastly, as a farm laborer who has worked exclusively for Ayalde for eighteen (18) years,
Tana should be entitled to compulsory coverage under the Social Security Law, whether
his service was continuous or broken.
Disposition Decision of CA reversed. Decision of SSS reinstated.

A2010

Disini

- 143 FERIA; September 30, 1986

NATURE
Petition for Certiorari and Mandamus
FACTS
- Petitioner employees question the validity of the pertinent section of the Rules and
Regulations Implementing the Labor Code as amended on which respondent arbitrator
Froilan M. Bacungan based his decision ruling that Mantrade Devt Corp is not under legal
obligation to pay holiday pay (as provided for in Article 94 of the Labor Code) to its
monthly paid employees who are uniformly paid by the month, irrespective of the number
of working days therein, with a salary of not less than the statutory or established
minimum wage, and that this rule is applicable not only as of March 2, 1976 but as of
November 1, 1974.
- Respondent corporation contends, among others that petitioner is barred from pursuing
the present action in view of (1) Article 263 of the Labor Code; (2) the pertinent provision
of the CBA between petitioner and respondent corporation; and (3) Article 2044 of the Civil
Code; that the special civil action of certiorari does not lie because respondent arbitrator is
not an "officer exercising judicial functions" within the contemplation of Rule 65, Section 1,
of the Rules of Court; that the instant petition raises an error of judgment on the part of
respondent arbitrator and not an error of jurisdiction; that it prays for the annulment of
certain rules and regulations issued by the DOLE, not for the annulment of the voluntary
arbitration proceedings; and that appeal by certiorari under Section 29 of the Arbitration
Law, Republic Act No. 876, is not applicable to the case at bar because arbitration in labor
disputes is expressly excluded by Section 3 of said law.
ISSUES
1. WON decisions of arbitrators are subject to judicial review
2. WON Mantrade employees are entitled to holiday pay
3. WON mandamus lies in the case at bar
HELD
1. YES
- Oceanic Bic Division (FFW) vs. Romero (July 16, 1984): The decisions of voluntary
arbitrators must be given the highest respect and as a general rule must be accorded a
certain measure of finality. It is not correct, however, that this respect precludes the
exercise of judicial review over their decisions. Article 262 of the Labor Code making
voluntary arbitration awards final, inappealable and executory, except where the money
claims exceed P100,000.00 or 40% of the paid-up capital of the employer or where there
is abuse of discretion or gross incompetence refers to appeals to the National Labor
Relations Commission and not to judicial review. Judicial review still lies where want of
jurisdiction, grave abuse of discretion, violation of due process, denial of substantial
justice, or erroneous interpretation of the Law are brought to SCs attention.
2. YES
- Under Art. 94 of the Labor Code, monthly salaried employees are not among those
excluded from receiving holiday pay. But they appear to be excluded under Sec. 2, Rule
IV, Book III of the Rules and Regulations implementing said provision.
- Insular Bank of Asia and America Employees' Union (IBAAEU) vs. Inciong (October 24,
1984): Section 2, Rule IV, Book III of the implementing rules and Policy Instruction No. 9,
issued by the then Secretary of Labor are null and void since in the guise of clarifying the
Labor Code's provisions on holiday pay, they in effect amended them by enlarging the
scope of their exclusion.
- Chartered Bank Employees Association vs. Ople (August 28, 1985): An administrative
interpretation which diminishes the benefits of labor more than what the statute delimits or
withholds is obviously ultra vires.
3. YES
- While it is true that mandamus is not proper to enforce a contractual obligation, the
remedy being an action for specific performance, in view of the above cited subsequent
decisions of this Court clearly defining the legal duty to grant holiday pay to monthly
salaried employees, mandamus is an appropriate equitable remedy.
Disposition Questioned decision of respondent arbitrator is SET ASIDE and respondent
corporation is ordered to GRANT holiday pay to its monthly salaried employees. No costs.

STATES MARINE CORP V CEBU SEAMENS ASSN


[PAGE 126]
MILLARES V NLRC
[PAGE 79]
MANTRADE/FMMC DIVISION EMPLOYEES AND
WORKERS UNION V BACUNGAN
144 SCRA 510

TIPS
ACE NAVIGATION CO INC V CA (NLRC, ALONSAGAY)

Labor Law 1
338 SCRA 380
PUNO; August 15, 2000

NATURE
Petitioner for review of the resolutions
that
dismissed
the
petition
for
certiorari (Ang kulit no? na-dismiss na
nga yung certiorari eh pume-petition
pa!)
FACTS
- In June 1994, Ace Navigation Co., Inc. recruited private respondent Orlando Alonsagay
to work as a bartender on board the vessel M/V "Orient Express" owned by Conning
Shipping Ltd. Under their POEA approved contract of employment, Orlando shall receive a
monthly basic salary of four hundred fifty U.S. dollars (U.S. $450.00), flat rate, including
overtime pay for 12 hours of work daily plus tips of two U.S. dollars (U.S. $2.00) per
passenger per day. He, was also entitled to 2.5 days of vacation leave with pay each
month. The contract was to last for one (1) year.
- Petitioners alleged that on June 13, 1994, Orlando was deployed and boarded M/V
"Orient Express" at the seaport of Hong Kong.
- After the expiration of the contract, Orlando returned to the Philippines and demanded
from Ace Nav his vacation leave pay.
- Ace Nav did not pay him immediately. It told him that he should have been paid prior to
his disembarkation and repatriation to the Philippines.
- Conning did not remit any amount for his vacation leave pay. Ace Nav promised to verify
the matter and asked Orlando to return after a few days. Orlando never returned.
- On November 25, 1995, Orlando filed a complaint before the labor arbiter for vacation
leave pay of four hundred fifty U.S. dollars and unpaid tips amounting to thirty six,
thousand U.S. dollars
- On November 15, 1996, Labor Arbiter Felipe P. Pati ordered Ace Nav and Conning to
pay jointly and severally Orlando his vacation leave pay of US$450.00. The claim for tips
of Orlando was dismissed for lack of merit.
- Orlando appealed to the NLRC on February 3, 1997. In a decision penned by
Commissioner Vicente S.E. Veloso and concurred in by Commissioner Alberto R. Quimpo
the NLRC ordered Ace Nav and Conning to pay the unpaid tips of Orlando which
amounted to US$36,000.00 in addition to his vacation leave pay.
- Ace Nav and Conning filed a motion for reconsideration on February 2, 1998 which was
denied on May 20, 1999.
- On July 2, 1999, Ace Nav and Conning filed a petition for certiorari before the Court of
Appeals
- On July 28, 1999, the Court of Appeals promulgated a three-page resolution and
concurred in by Associate Justices Eubulo G. Verzola and Elvi John S. Asuncion
dismissing the petition.
- Their motion for reconsideration filed was denied.
Petitioners:
> Petitioners argued that the Court of Appeals erred in rigidly and technically applying
Section 13, Rule 1310- Proof of personal service shall consist of a written admission of
the party served, or the official return of the server, or the affidavit of the party serving,
containing a full statement of the date, place and manner of service. If the service is by
ordinary mail, proof thereof shall consist of an affidavit of the person mailing or facts
showing compliance with section 7 of this Rule. If service is made by registered mail,
proof shall be made by such affidavit and the registry receipt issued by the mailing office.
The registry return card shall be filed immediately upon its receipt, or in lieu thereof of the
unclaimed letter together with the certified or sworn copy of the notice given by the
postmaster to the addressee.

> Section 1, Rule 6511 Section 1.-- When any


tribunal, board or officer exercising judicial or quasi
judicial functions has acted without or in excess of its
or his jurisdiction, or with grave abuse of discretion
amounting to lack or excess of jurisdiction, and there
is no appeal, or any plain, speedy, and adequate
remedy in the ordinary course of law, a person
aggrieved thereby may file a verified petition in the
proper court, alleging the facts with certainty and

Disini

A2010 - 144 praying that judgment be rendered annulling or


modifying the proceedings of such tribunal, board,
officer, and granting such incidental reliefs as law and
justice may require.
> They also contend that the respondent court erred in ruling that they are the ones liable
to pay tips to Orlando. They point out that if tips will be considered as part of the salary of
Orlando, it will make him the highest paid employee on M/V "Orient Express." It will create
an unfavorable precedent detrimental to the future recruitment, hiring and deployment of
Filipino overseas workers specially in service oriented businesses. It will also be a case of
double compensation that will unjustly enrich Orlando at the expense of petitioners.
> They also stress that Orlando never complained that they should pay him the said tips.
- Respondent filed a two-page comment to the petition adopting the resolution of the Court
of Appeals dated July 28, 1999.
ISSUES
1. WON the CA erred in rigidly applying Sec 1310
2. WON the CA erred in ruling that they are the ones liable to pay tips to petitioner
(Orlando)
HELD
1. YES
Ratio Rules of procedure are used to help secure and not override substantial justice.
[Heirs of Francisco Guballa Sr. vs. Court of Appeals] Even the Rules of Court mandates a
liberal construction in order to promote their objective of securing a just, speedy and
inexpensive disposition of every action and proceeding. Since rules of procedure are mere
tools designed to facilitate the attainment of justice, their strict and rigid application which
would result in technicalities that tend to frustrate rather than promote substantial justice
must always be avoided. Thus, the dismissal of an appeal on purely technical ground is
frowned upon especially if it will result to unfairness.
Reasoning
- We apply these sound rules in the case at bar. Petitioners' petition for certiorari before
the Court of Appeals contained the certified true copy of the NLRC's decision dated
November 26, 1997. Its order dated May 2, 199917 and the sworn certification of nonforum shopping. Petitioners also explained that their counsel executed an affidavit of proof
of service and explanation in the afternoon of July 1, 1999. However, he forgot to attach it
when he filed their petition the following day because of the volume and pressure of work
and lack of office personnel. However, the Registry which is the proof of mailing to
Orlando's counsel, issued by the Central Post Office was attached on the original petition
they filed with the respondent court. It was also stamped by the NLRC which is proof of
receipt of the petition by the latter. The affidavit of service, which was originally omitted,
was attached on their motion for reconsideration. Significantly, it was dated July 1, 1999.
- the subsequent filing of the affidavit of service may be considered as substantial
compliance with the rules.
2. NO
Reasoning

- The word tip has several meanings.


It is more frequently used to indicate
additional compensation, and in this
sense "tip" is defined as meaning a
gratuity; a gift; a present; a fee; money
given, as to a servant to secure better
or more prompt service.
- Tipping is done to get the attention and secure the immediate services of a waiter, porter
or others for their services. Since a tip is considered a pure gift out of benevolence or
friendship, it can not be demanded from the customer. Whether or not tips will be given is
dependent on the will and generosity of the giver. Although a customer may give a tip as a
consideration for services rendered, its value still depends on the giver. They are given in
addition to the compensation by the employer. A gratuity given by an employer in order to
inspire the employee to exert more effort in his work is more appropriately called a bonus.
- The contract of employment between petitioners and Orlando is categorical that the
monthly salary of Orlando is US$450.00 flat rate. This already included his overtime pay
which is integrated in his 12 hours of work. The words "plus tips of US$2.00 per passenger
per day" were written at the line for overtime. Since payment for overtime was included in
the monthly salary of Orlando, the supposed tips mentioned in the contract should be
deemed included thereat.
- The actuations of Orlando during his employment also show that he was aware his
monthly salary is only US$450.00, no more no less. He did not raise any complaint about
the non-payment of his tips during the entire duration of his employment. After the

Labor Law 1
expiration of his contract, he demanded payment only of his vacation leave pay. He did not
immediately seek the payment of tips. He only asked for the payment of tips when he filed
this case before the labor arbiter. This shows that the alleged non-payment of tips was a
mere afterthought to bloat up his claim. The records of the case do not show that Orlando
was deprived of any monthly salary. It will now be unjust to impose a burden on the
employer who performed the contract in good faith.
- Furthermore, it is presumed that the parties were aware of the plain, ordinary and
common meaning of the word "tip." As a bartender, Orlando can not feign ignorance on
the practice of tipping and that tips are normally paid by customers and not by the
employer.
- However, Orlando should be paid his vacation leave pay. Petitioners denied this liability
by raising the defense that the usual practice is that vacation leave pay is given before
repatriation. But as the labor arbiter correctly observed, petitioners did not present any
evidence to prove that they already paid the amount. The burden of proving payment was
not discharged by the petitioners.
Disposition Reversed and set aside

CASH WAGE/COMMISSIONS
SONGCO V NLRC (AGUAS, F.E. ZUELLIG INC)
183 SCRA 610
MEDIALDEA; March 23, 1990
FACTS
- Private respondent F.E. Zuellig (M), Inc., filed with the Department of Labor an
application seeking clearance to terminate the services of petitioners Jose Songco,
Romeo Cipres, and Amancio Manuel allegedly on the ground of retrenchment due to
financial losses.
- This application was seasonably opposed by petitioners alleging that the company is not
suffering from any losses. They alleged further that they are being dismissed because of
their membership in the union.
- At the last hearing of the case, however, petitioners manifested that they are no longer
contesting their dismissal. The parties then agreed that the sole issue to be resolved is the
basis of the separation pay due to petitioners.
- Petitioners, who were in the sales force of Zuellig received monthly salaries of at least
P40,000. In addition, they received commissions for every sale they made.
- The CBA entered into between Zuellig and F.E. Zuellig Employees Association, of which
petitioners are members, contains the following provision:
ARTICLE XIV Retirement Gratuity
Section l(a)-Any employee, who is separated from employment due to old age,
sickness, death or permanent lay-off not due to the fault of said employee shall
receive from the company a retirement gratuity in an amount equivalent to one (1)
month's salary per year of service. One month of salary as used in this paragraph
shall be deemed equivalent to the salary at date of retirement; years of service shall
be deemed equivalent to total service credits, a fraction of at least six months being
considered one year, including probationary employment.
- On the other hand, Article 284 of the Labor Code then prevailing provides:
Art. 284. Reduction of personnel. The termination of employment of any
employee due to the installation of labor saving-devices, redundancy, retrenchment
to prevent losses, and other similar causes, shall entitle the employee affected
thereby to separation pay. In case of termination due to the installation of laborsaving devices or redundancy, the separation pay shall be equivalent to one (1)
month pay or to at least one (1) month pay for every year of service, whichever is
higher. In case of retrenchment to prevent losses and other similar causes, the
separation pay shall be equivalent to one (1) month pay or at least one-half (1/2)
month pay for every year of service, whichever is higher. A fraction of at least six (6)
months shall be considered one (1) whole year.
- In addition, Sections 9(b) and 10, Rule 1, Book VI of the Rules Implementing the Labor
Code provide:
Sec. 9(b). Where the termination of employment is due to retrechment initiated by
the employer to prevent losses or other similar causes, or where the employee
suffers from a disease and his continued employment is prohibited by law or is
prejudicial to his health or to the health of his co-employees, the employee shall be
entitled to termination pay equivalent at least to his one month salary, or to one-half
month pay for every year of service, whichever is higher, a fraction of at least six (6)
months being considered as one whole year.
Sec. 10. Basis of termination pay. The computation of the termination pay of an
employee as provided herein shall be based on his latest salary rate, unless the
same was reduced by the employer to defeat the intention of the Code, in which
case the basis of computation shall be the rate before its deduction. (Emphasis
supplied)

A2010

- 145 -

Disini

- The Labor Arbiter rendered a decision ordering the respondent to pay the complainants
separation pay equivalent to their one-month salary (exclusive of commissions,
allowances, etc.) for every year of service that they have worked with the company.
- The appeal by petitioners to the National Labor Relations Commission was dismissed for
lack of merit.
- Petitioners' Arguments
> In arriving at the correct and legal amount of separation pay due them, whether under
the Labor Code or the CBA, their basic salary, earned sales commissions and allowances
should be added together. They cited Article 97(f) of the Labor Code which includes
commission as part on one's salary, to wit;
(f) 'Wage' paid to any employee shall mean the remuneration or earnings, however
designated, capable of being expressed in terms of money, whether fixed or
ascertained on a time, task, piece, or commission basis, or other method of
calculating the same, which is payable by an employer to an employee under a
written or unwritten contract of employment for work done or to be done, or for
services rendered or to be rendered, and includes the fair and reasonable value, as
determined by the Secretary of Labor, of board, lodging, or other facilities
customarily furnished by the employer to the employee. 'Fair reasonable value' shall
not include any profit to the employer or to any person affiliated with the employer.
- Respondents Comments
> If it were really the intention of the Labor Code as well as its implementing rules to
include commission in the computation of separation pay, it could have explicitly said so in
clear and unequivocal terms. Furthermore, in the definition of the term "wage",
"commission" is used only as one of the features or designations attached to the word
remuneration or earnings.
ISSUE
WON earned sales commissions and allowances should be included in the monthly salary
of petitioners for the purpose of computation of their separation pa
HELD
YES
- Article 97(f) by itself is explicit that commission is included in the definition of the term
"wage". It has been repeatedly declared by the courts that where the law speaks in clear
and categorical language, there is no room for interpretation or construction; there is only
room for application.
- The ambiguity between Article 97(f), which defines the term 'wage' and Article XIV of the
Collective Bargaining Agreement, Article 284 of the Labor Code and Sections 9(b) and 10
of the Implementing Rules, which mention the terms "pay" and "salary", is more apparent
than real.
- Broadly, the word "salary" means a recompense or consideration made to a person for
his pains or industry in another man's business. Whether it be derived from "salarium," or
more fancifully from "sal," the pay of the Roman soldier, it carries with it the fundamental
idea of compensation for services rendered. - Indeed, there is eminent authority for holding that the words "wages" and "salary" are in
essence synonymous. "Salary," the etymology of which is the Latin word "salarium," is
often used interchangeably with "wage", the etymology of which is the Middle English
word "wagen". Both words generally refer to one and the same meaning, that is, a reward
or recompense for services performed.
- Likewise, "pay" is the synonym of "wages" and "salary". Inasmuch as the words "wages",
"pay" and "salary" have the same meaning, and commission is included in the definition of
"wage", the logical conclusion, therefore, is, in the computation of the separation pay of
petitioners, their salary base should include also their earned sales commissions.
- Granting, in gratia argumenti, that the commissions were in the form of incentives or
encouragement, so that the petitioners would be inspired to put a little more industry on
the jobs particularly assigned to them, still these commissions are direct remuneration
services rendered which contributed to the increase of income of Zuellig.
- Commission is the recompense, compensation or reward of an agent, salesman,
executor, trustees, receiver, factor, broker or bailee, when the same is calculated as a
percentage on the amount of his transactions or on the profit to the principal. The nature of
the work of a salesman and the reason for such type of remuneration for services
rendered demonstrate clearly that commission are part of petitioners' wage or salary.
- The Court took judicial notice of the fact that some salesmen do not receive any basic
salary but depend on commissions and allowances or commissions alone, are part of
petitioners' wage or salary. Also, that some salesman do not received any basic salary but
depend on commissions and allowances or commissions alone, although an employeremployee relationship exists.
- Bearing in mind the preceding discussions, if the opposite view is adopted that
commissions, do not form part of wage or salary, then, in effect, this kind of salesmen do
not receive any salary and therefore, not entitled to separation pay in the event of
discharge from employment. This narrow interpretation is not in accord with the liberal
spirit of our labor laws and considering the purpose of separation pay which is, to alleviate
the difficulties which confront a dismissed employee thrown the the streets to face the
harsh necessities of life.

Labor Law 1
- Additionally, in Soriano v. NLRC, et al., supra, in resolving the issue of the salary base
that should be used in computing the separation pay, the Court held that:
The commissions also claimed by petitioner ('override commission' plus 'net deposit
incentive') are not properly includible in such base figure since such commissions
must be earned by actual market transactions attributable to petitioner.
- Applying this by analogy, since the commissions in the present case were earned by
actual market transactions attributable to petitioners, these should be included in their
separation pay. In the computation thereof, what should be taken into account is the
average commissions earned during their last year of employment.
- In carrying out and interpreting the Labor Code's provisions and its implementing
regulations, the workingman's welfare should be the primordial and paramount
consideration. This kind of interpretation gives meaning and substance to the liberal and
compassionate spirit of the law as provided for in Article 4 of the Labor Code which states
that "all doubts in the implementation and interpretation of the provisions of the Labor
Code including its implementing rules and regulations shall be resolved in favor of labor",
and Article 1702 of the Civil Code which provides that "in case of doubt, all labor
legislation and all labor contracts shall be construed in favor of the safety and decent living
for the laborer.
Disposition The petition was granted.

IRAN V NLRC (RETRALBA)


106 SCRA 444
ROMERO; April 22, 1998
FACTS
- Antonio Iran is engaged in softdrinks merchandising and distribution in Mandaue City,
Cebu, employing truck drivers who double as salesmen, truck helpers, and non-field
personnel in pursuit thereof. He hired private respondents as drivers/salesmen and truck
helpers. Drivers/salesmen drove petitioners delivery trucks and promoted, sold and
delivered softdrinks to various outlets in Mandaue City. The truck helpers assisted in the
delivery of softdrinks to the different outlets covered by the driver/salesmen.
- As part of their compensation, the driver/salesmen and truck helpers of petitioner
received commissions per case of softdrinks sold.
- Sometime in June 1991, Iran discovered cash shortages and irregularities allegedly
committed by private respondents. Pending the investigation of irregularities and
settlement of the cash shortages, Iran required private respondents to report for work
everyday. They were not allowed, however, to go on their respective routes. A few days
thereafter, despite aforesaid order, private respondents stopped reporting for work,
prompting Iran to conclude that the former had abandoned their employment.
Consequently, Iran terminated their services. He also filed a complaint for estafa against
them.
- Private respondents filed complaints against Iran for illegal dismissal, illegal deduction,
underpayment of wages, premium pay for holiday and rest day, holiday pay, service
incentive leave pay, 13th month pay, allowances, separation pay, recovery of cash bond,
damages and attorneys fees.
- Said complaints were consolidated, and assigned to Labor Arbiter Ernesto F. Carreon.
He found that Iran had validly terminated private respondents, there being just cause for
the latters dismissal. Nevertheless, he also ruled that Iran had not complied with
minimum wage requirements in compensating private respondents, and had failed to pay
private respondents their 13th month pay.
- On appeal, NLRC affirmed the validity of private respondents dismissal, but found that
said dismissal did not comply with the procedural requirements for dismissing employees.
MR denied.
ISSUES
1. WON commissions are included in determining compliance with the minimum wage
requirement
2. WON Iran is guilty of procedural lapses in terminating private respondents
If yes, WON P1,000.00 indemnity fee to each of the private respondents is proper
3. WON the advance amount received by private respondents should be credited as part
of their 13th month pay
HELD
1. YES
- The nature of the work of a salesman and the reason for such type of remuneration for
services rendered demonstrate clearly that commissions are part of a salesmans wage or
salary.
- Article 97(f), LC explicitly includes commissions as part of wages. While commissions
are, indeed, incentives or forms of encouragement to inspire employees to put a little more
industry on the jobs particularly assigned to them, still these commissions are direct
remunerations for services rendered.
- Commissions have been defined as the recompense, compensation or reward of an
agent, salesman, executor, trustee, receiver, factor, broker or bailee, when the same is

A2010

- 146 -

Disini

calculated as a percentage on the amount of his transactions or on the profit to the


principal.
- SC has taken judicial notice of the fact that some salesmen do not receive any basic
salary but depend entirely on commissions and allowances or commissions alone,
although an employer-employee relationship exists. Undoubtedly, this salary structure is
intended for the benefit of the corporation establishing such, on the apparent assumption
that thereby its salesmen would be moved to greater enterprise and diligence and close
more sales in the expectation of increasing their sales commissions. This, however, does
not detract from the character of such commissions as part of the salary or wage paid to
each of its salesmen for rendering services to the corporation.
- There is no law mandating that commissions be paid only after the minimum wage has
been paid to the employee. Verily, the establishment of a minimum wage only sets a floor
below which an employees remuneration cannot fall, not that commissions are excluded
from wages in determining compliance with the minimum wage law.
- Philippine Agricultural Commercial and Industrial Workers Union vs. NLRC: drivers and
conductors who are compensated purely on a commission basis are automatically entitled
to the basic minimum pay mandated by law should said commissions be less than their
basic minimum for eight hours work. Were said commissions equal to or even exceed the
minimum wage, the employer need not pay, in addition, the basic minimum pay prescribed
by law.
2. YES
- In terminating employees, the employer must furnish the worker with two written notices
before the latter can be legally terminated: (a) a notice which apprises the employee of the
particular acts or omissions for which his dismissal is sought, and (b) the subsequent
notice which informs the employee of the employers decision to dismiss him.
- First notice informing the employee that his dismissal is being sought is absent in the
present case. This makes the termination of private respondents defective, for which Iran
must be sanctioned for his non-compliance with the requirements of or for failure to
observe due process.
- Section 2 of Book V, Rule XIV of the Omnibus Rules Implementing the Labor Code
requires that in cases of abandonment of work, notice should be sent to the workers last
known address. If indeed private respondents had abandoned their jobs, it was incumbent
upon Iran to comply with this requirement. This, Iran failed to do, entitling respondents to
nominal damages in the amount of P5,000.00 each, in accord with recent jurisprudence, to
vindicate or recognize their right to procedural due process which was violated by Iran.
3. YES
- Iran is entitled to credit only the amounts paid for the particular year covered by said
vouchers.
- While it is true that the vouchers evidencing payments of 13 th month pay were submitted
only on appeal, it would have been more in keeping with the directive of Article 221 of the
Labor Code for the NLRC to have taken the same into account.
- In labor cases, technical rules of evidence are not binding. Labor officials should use
every and all reasonable means to ascertain the facts in each case speedily and
objectively, without regard to technicalities of law or procedure.
- The intent of P.D. No. 851 is the granting of additional income in the form of 13 th month
pay to employees not as yet receiving the same and not that a double burden should be
imposed on the employer who is already paying his employees a 13 th month pay or its
equivalent. An employer who pays less than 1/12 th of the employees basic salary as their
13th month pay is only required to pay the difference.
Disposition NLRC decision modified. Case remanded to the Labor Arbiter for a
recomputation of the alleged deficiencies. No costs.

WAGES AND SALARY


GAA V CA (EUROPHIL INDUSTRIES CORP.)
140 SCRA 304 (85)
PATAJO; December 31, 1985
NATURE
A petition for review on certiorari of the decision of the Court of Appeals affirming the
decision of the Court of First Instance of Manila.
FACTS
- Respondent Europhil Industries Corporation was formerly one of the tenants in Trinity
Building at T.M. Kalaw Street, Manila, while petitioner Rosario A. Gaa was then the
building administrator.
- December 12, 1973: Europhil Industries commenced an action in CFI for damages
against petitioner "for having perpetrated certain acts that Europhil Industries considered a
trespass upon its rights, namely, cutting of its electricity, and removing its name from the
building directory and gate passes of its officials and employees." Court ruled in favor of
Europhil.
- A writ of garnishment was issued pursuant to which Deputy Sheriff Cesar A. Roxas
served a Notice of Garnishment upon El Grande Hotel, where petitioner was then

Labor Law 1
employed, garnishing her "salary, commission and/or remuneration." Gaa filed with the
CFI a motion to lift said garnishment on the ground that her "salaries, commission and or
remuneration" are exempted from execution under Article 1708 of the New Civil Code.
- CA dismissed the petition, saying that Gaa is not a mere laborer as contemplated under
Article 1708. The term laborer does not apply to one who holds a managerial or
supervisory position like that of petitioner, but only to those "laborers occupying the lower
strata."
ISSUE
WON Gaa is a laborer falling under the exception of Art. 1708 of the Civil Code
HELD
NO, Gaa is not a laborer as contemplated by the Civil Code.
Ratio The term "wages" as distinguished from "salary", applies to the compensation for
manual labor, skilled or unskilled, paid at stated times, and measured by the day, week,
month, or season, while "salary" denotes a higher degree of employment, or a superior
grade of services, and implies a position of office.
Reasoning
- The legislature intended the exemption in Article 1708 of the New Civil Code to operate
in favor of laboring men or women in the sense that their work is manual. Persons
belonging to this class usually look to the reward of a day's labor for immediate or present
support, and such persons are more in need of the exemption than any others.
LABORER: everyone who performs any kind of mental or physical labor, but as commonly
and customarily used and understood, it only applies to one engaged in some form of
manual or physical labor.
WAGE: the pay given "as hire or reward to artisans, mechanics, domestics or menial
servants, and laborers employed in manufactories, agriculture, mines, and other manual
occupation and usually employed to distinguish the sums paid to persons hired to perform
manual labor, skilled or unskilled, paid at stated times, and measured by the day, week,
month, or season."
- Petitioner is not an ordinary or rank and file laborer but "a responsibly-placed employee,"
of El Grande Hotel, "responsible for planning, directing, controlling, and coordinating the
activities of all housekeeping personnel" to ensure the cleanliness, maintenance and
orderliness of all guest rooms, function rooms, public areas, and the surroundings of the
hotel. Petitioner is occupying a position equivalent to that of a managerial or supervisory
position.
Disposition Decision of the CA affirmed, with costs against the petitioner.

EQUITABLE BANKING CORP V SADAC


490 SCRA 380
CHICO-NAZARIO; June 8, 2006
NATURE
Petition for Review on Certiorari, with Motion to Refer the Petition to the Court En Banc,
seeking to reverse the Decision and Resolution of the CA which reversed and set aside
the Resolutions of the NLRC.
FACTS
- Respondent Sadac was appointed Vice President of the Legal Department of petitioner
Bank and subsequently General Counsel thereof. Nine lawyers of petitioner Banks Legal
Department accused Sadac of abusive conduct and petitioned for a change in leadership
of the department. On the ground of lack of confidence in Sadac, under the rules of client
and lawyer relationship, petitioner Bank instructed him to deliver all materials in his
custody in all cases in which the latter was appearing as its counsel of record.
- Sadac requested for a full hearing and formal investigation but the same remained
unheeded. He filed a complaint for illegal dismissal with damages against petitioner Bank
and individual members of the Board of Directors thereof. After learning of the filing of the
complaint, petitioner Bank terminated his services. Finally, Sadac was removed from his
office and ordered disentitled to any compensation and other benefits. Labor Arbiter
Jovencio Ll. Mayor, Jr., dismissed the complaint for lack of merit. On appeal, the NLRC
reversed the Labor Arbiter and declared Sadacs dismissal as illegal.
ISSUE
1. WON Sadac is entitled to full backwages including salary increases
2. WON Sadac is entitled to receive certain benefits
3. WON the CA erred in awarding attorneys fees to Sadac
4. WON Sadac is entitled to legal interest
5. WON the petition should be heard by the court en banc
HELD
1. NO
Ratio The outstanding feature of backwages is the degree of assuredness to an employee
that he would have had them as earnings had he not been illegally terminated from his

A2010

- 147 -

Disini

employment. Salary increases, however, are a mere expectancy. There is no vested right
to salary increases.
Reasoning
- That respondent Sadac may have received salary increases in the past only proves fact
of receipt but does not establish a degree of assuredness that is inherent in backwages.
The mere fact that petitioner had been previously granted salary increases by reason of
his excellent performance does not necessarily guarantee that he would have performed
in the same manner and, therefore, qualify for the said increase later. When there is an
award of backwages this actually refers to backwages without qualifications and
deductions. An unqualified award of backwages means that the employee is paid at the
wage rate at the time of his dismissal. The base figure to be used in the computation of
backwages due to the employee should include not just the basic salary, but also the
regular allowances that he had been receiving.
Obiter
Broadly, the word "salary" means a recompense or consideration made to a person for his
pains or industry in another mans business. It carries with it the fundamental idea of
compensation for services rendered. In labor law, the distinction between salary and wage
appears to be merely semantics. That wage and salary are synonymous has been settled.
Both words generally refer to one and the same meaning, that is, a reward or recompense
for services performed. Likewise, "pay" is the synonym of "wages" and "salary".
2. NO
Ratio Sadac did not present any evidence to prove entitlement to these claims.
Reasoning
- Petitioner Banks computation contains no acknowledgment of herein claimed benefits,
namely, check-up benefit, clothing allowance, and cash conversion of vacation leaves. We
cannot sustain the rationalization that the acknowledgment by petitioner Bank in its
computation of certain benefits granted to Sadac means that the latter is also entitled to
the other benefits as claimed by him but not acknowledged by the Bank.
3. NO
Ratio The decision of the CA AFFIRMED with MODIFICATION the NLRC decision, which
modification did not touch upon the award of attorneys fees as granted, hence, the award
stands.
Reasoning
- When a final judgment becomes executory, it thereby becomes immutable and
unalterable. The CAs decision became final and executory. This renders moot whatever
argument petitioner Bank raised against the grant of attorneys fees to Sadac.
4. YES
Ratio The legal interest of 12% per annum shall be imposed from the time judgment
becomes final and executory, until full satisfaction thereof.
Reasoning
- The CA was not in error in imposing the same notwithstanding that the parties were at
variance in the computation of Sadacs backwages. What is significant is that the decision
which awarded backwages to Sadac became final and executory. Therefore, petitioner
Bank is liable to pay interest from the date of finality of the decision.
5. NO
Ratio The instant case is not one that should be heard by the Court en banc.
Reasoning
- We are not herein modifying or reversing a doctrine or principle laid down by the Court
en banc or in a division.
Disposition The petition is PARTIALLY GRANTED and PARTIALLY DENIED. The
decision of the CA is hereby MODIFIED.

GRATUITY AND WAGES


PLASTIC TOWN CENTER CORPORATION V NLRC
(NAGKAKAISANG LAKAS NG MANGGAGAWA (NLM)KATIPUNAN)
172 SCRA 380
GUTIERREZ; April 19,1989
NATURE
Petition for review of the decision of the NLRC
FACTS
- There are 2 provisions of the CBA in question in this case.
1) P1 increase in salary is granted every July 1. Also, section 3 provides: It is agreed and
understood by the parties herein that the aforementioned increase in pay shall be credited
against future allowances or wage orders hereinafter implemented or enforced by virtue of
Letters of Instructions, Decrees and other labor legislation.
- Wage order number 4, effective on May 1 1984, provided for the integration of the
emergency cost of living allowances (ECOLA). It also provided that the minimum daily
wage rate be P32. Petitioner Plastic Town incurred a deficiency of P1 after integrating the

Labor Law 1
ECOLA. They then advanced the implementation of the wage increase as provided for by
the CBA. The petitioner argues that it did not credit the Pl.00 per day across the board
increase under the CBA as compliance with Wage Order No. 5 implemented on June
16,1984 since it gave an additional P3.00 per day to the basic salary pursuant to said
order. It, however, credited the Pl.00 a day increase to the requirement under Wage Order
No. 4 to which the private respondents allegedly did not object.
2) gratuity pay to resigning employees
- Gratuity pay is based on the monthly salary. Petitioner argues that the computation of the
monthly salary should be the equivalent of 26 days of salary, not 30 days as the
respondents aver.
ISSUES
1. WON the petitioners can credit the P1 increase in the CBA as compliance with wage
order number 4
2. WON the monthly salary is equivalent to 26 days
HELD
1. NO.
- In the case at bar, the petitioner alleges that on May 1, 1984, it granted a Pl.00 increase
pursuant to Wage Order No. 4 which in consonance with Section 3 of the CBA was to be
credited to the July 1, 1984 increase under the CBA. It was, therefore, a July increase.
Section 3 of the CBA, however, clearly states that CBA granted increases shall be credited
against future allowances or wage orders. Thus, the CBA increase to be effected on July
1, 1984 can not be retroactively applied to mean compliance with Wage Order No. 4 which
took effect on May 1, 1984.
2. NO. It should be 30 days
- To say that awarding the daily wage earner salary for more than 26 days is paying him
for days he does not work misses the point entirely. The issue here is not payment for
days worked but payment of gratuity pay equivalent to one month or 30 days salary
- From the foregoing, gratuity pay is therefore, not intended to pay a worker for actual
services rendered. It is a money benefit given to the workers whose purpose is "to reward
employees or laborers, who have rendered satisfactory and efficient service to the
company." (Sec. 2, CBA) While it may be enforced once it forms part of a contractual
undertaking, the grant of such benefit is not mandatory so as to be considered a part of
labor standard law unlike the salary, cost of living allowances, holiday pay, leave benefits,
etc., which are covered by the Labor Code. Nowhere has it ever been stated that gratuity
pay should be based on the actual number of days worked over the period of years
forming its basis. We see no point in counting the number of days worked over a ten-year
period to determine the meaning of "two and one- half months' gratuity." Moreover any
doubts or ambiguity in the contract between management and the union members should
be resolved in the light of Article 1702 of the Civil Code that:
- In case of doubt, all labor legislation and all labor contracts shall be construed in favor of
the safety and decent living for the laborer
Disposition Decision affirmed

13TH MONTH PAY


AGABON V NLRC
[PAGE 35]

B. PAYMENT OF WAGES

9.03 FORM
FULL PAYMENT
LOPEZ SUGAR CORPORATION V FRANCO
458 SCRA 515
CALLEJO; May 16, 2005
NATURE
Petition for review on certiorari of the Decision of the Court of Appeals (CA)
FACTS
- Franco, Pabalan, Perrin and Candelario were supervisory employees of the Lopez Sugar
Corporation (the Corporation, for brevity). Franco was barely 20 years old when he was
employed in 1974 as Fuel-in-Charge. His co-employee, Pabalan, was about 28 years old
when he was hired by the Corporation as Shift Supervisor in the Sugar Storage
Department in 1975. Perrin and Candelario were employed in 1975 and 1976,
respectively, as Planter Service Representatives (PSRs), who rose from the ranks and, by
1994, occupied supervisory positions in the Corporations Cane Marketing Section.

A2010

- 148 -

Disini

- The supervisory employees of the Corporation, spearheaded by Franco, Pabalan, Perrin


and Candelario, decided to form a labor union called Lopez Sugar Corporation
Supervisors Association. Franco was elected president and Pabalan as treasurer. Perrin
and Candelario, on the other hand, were among its active members.The officers of the
union and the management held a meeting, which led to the submission of the unions
proposals for a CBA on July 24, 1995.
- The Corporations president issued a Memorandum to the vice-president and department
heads for the adoption of a special retirement program for supervisory and middle level
managers. He emphasized that the management shall have the final say on who would
be covered, and that the program would be irrevocable once approved.
Perrin and Candelario were on leave when they were invited by Juan Masa, Jr., head of
the Cane Marketing Section, to the Northeast Beach Resort in Escalante, Negros
Occidental. The latter informed them that they were all included in the special retirement
program and would receive their respective notices of dismissal shortly.
- Masa, Pabalan, Franco, Perrin and Candelario received copies of the Memorandum
dated August 25, 1995 from the Corporations Vice-President for Administration and
Finance, informing them that they were included in the special retirement program for
supervisors and middle level managers; hence, their employment with the Corporation
was to be terminated effective September 29, 1995, and they would be paid their salaries
until September 27, 1995. The private respondents received their respective separation
pays and executed their respective Release Waiver and Quitclaim after receiving their
clearances from the Corporation.
- The private respondents filed separate complaints against the corporation with the NLRC
for illegal dismissal, unfair labor practice, reinstatement and damages.
- The Corporation maintained that the termination of the employment of the complainants
was in response to the challenges brought about by the General Agreement on Tariff and
Trade (GATT), the AFTA and other international trade agreements, which greatly affected
the local sugar industry and a study done by Sycip, Gorres, Velayo and Company (SGV)
regarding the Corporation and its operations to identify changes that could be
implemented to achieve cost effectiveness and global competitiveness
- Labor Arbiter rendered judgment in favor of the Corporation and ordered the dismissal of
the complainants. The complainants appealed to the NLRC that granted their appeal and
reversed the decision of the Labor Arbiter. The CA rendered judgment dismissing the
petition, on the ground that the NLRC did not commit grave abuse of discretion in
rendering judgment against the Corporation.
ISSUES
1. WON there was proof of redundancy in the fulfillment of the jobs of the employees and
whether there is a criteria, guideline, or standard for selection. If not whether it was meant
to intimidate the union
2. WON the waiver and quitclaim was valid
HELD
1. NO
- The SC agreed with the ruling of the CA that the petitioner illegally dismissed the private
respondents from their employment by including them in its special retirement program,
thus, debilitating the union, rendering it pliant by decapacitating its leadership. As such,
the so-called downsizing of the Cane Marketing Department and SMSD based on the
SGV Study Report was a farce capricious and arbitrary.
- Complainants are not in a position to anticipate how respondent will present its case for
redundancy particular[ly] because no standard, criteria or guidelines for the selection of
dismissed employees was made known to them, and all that they were told was that you
were selected as among those who will be separated from the service; nonetheless, this
early, it is possible to point out certain facts which throw light on the plausibility or want of
it, of the ground relied upon.
1. No contingency has occurred, of the kind mentioned by the Supreme Court in the
Wiltshire case, (over-hiring of workers, decreased volume of business or dropping of a
particular service line) which would explain the dismissal on the ground of redundancy;
over-hiring of workers cannot conceivably occur in the level of the supervisors; on the
other hand, it would have required an event of cataclysmic proportion to justify the
dismissal for redundancy of a full one-third of the supervisors in an establishment, and if
such an event were to occur it would have resulted in tremendous losses which is not true
here because the dismissal is not on account of or to prevent losses;
2. In no other category of employees did positions suddenly become redundant except
among the supervisors who have just organized themselves into a labor union and were
working for their first-ever CBA in the establishment;
3. The dismissal came at the precise time when the Lopez Sugar Central Supervisors
Association (LSCA) had presented its CBA proposals and was expecting the companys
reply as mandated by law; in fact, the reply was overdue, being required to be submitted
by management within ten (10) days from receipt of the union proposal; there is no better
proof that the dismissals have served their hidden purpose than that the CBA negotiation
has ended to all intents and purpose, before management could even present its
counterproposal. Certainly, it would be farfetched to say that the remaining union officers
and members have abandoned its objective of having a CBA for reasons other than the
fear of suffering the fate of those who had been dismissed.

Labor Law 1
- The absence of criteria, guidelines, or standard for selection of dismissed employees
renders the dismissals whimsical, capricious and vindictive; in the case of the
complainants Franco and Pabalan, who are the Union President and Treasurer,
respectively, the reason for their inclusion is obvious. Additionally, it must be mentioned
that in the case of Pabalan, there were three shift supervisors, one for each 8-hour shift
before the program was implemented, namely, Pabalan, Bitera and Lopez; Pabalan and
Bitera (a union director) were terminated, leaving Lopez alone, who worked on 12-hour
shift duty with Henry Villa, department head who was forced to perform the work of shift
supervisor; Pabalan was offered to be rehired as an employee of BUGLAS, a labor-only
contractor but he refused; an employee, Eugenio Bolanos was assigned from another
department to do the work of shift supervisor and three of them (Lopez, Villa and Bolanos)
now divide shift duties among themselves. There is no explanation why among the shift
supervisors it was Pabalan and Bitera who were included in the program.
- In the case of complainants [P]errin and Candelario, both Planter Service
Representatives, the manipulation is even more apparent; one year before the program
was instituted, two new PSRs were hired (Labrador and Cambate) bringing to six the total
number of PSRs; after the termination of [P]errin and Candelario, who have served for
nearly 20 years, two new PSRs were hired (Oropel and Jeres) on contractual basis and
whose compensation is based on pakiao; additionally, Candelario was hired after his
dismissal under the same arrangement as Oropel and Jeres, which lasted only up to
January 1996 when management learned of the filing of the first of these cases; [P]errin,
on his part, was offered the same arrangement but he refused.
- The rehiring of dismissed employees through a labor-only contractor exposes the
program as a circumvention of the law. This is true in the case of the following
supervisors who were terminated with complainant but were subsequently employed to do
exactly the same work, but as employees of BUGLAS, a labor-only contractor which
supplies laborers to respondent LSC. The above re-hiring in addition to other
circumstances earlier mentioned, such as the hiring of 2 men PSRs after Candelario and
[P]errin were terminated; the short-lived rehiring of the former and the offer to hire the
latter which he refused, all indicate that there was no redundancy.
- None of the work has been phased out or rendered obsolete by any event that took
place. As to duplication of functions, it must be mentioned that the positions of
complainants have existed for a long time judging from their years of service with
respondent; the observation of the Supreme Court in the Wiltshire case to the effect that in
a well-organized establishment, duplication of functions is hardly to be expected is
pertinent.
- Foremost, the petitioner failed to formulate fair and reasonable criteria in ascertaining
what positions were declared redundant and accordingly obsolete, such as preferred
status, efficiency or seniority. It, likewise, failed to formulate fair and reasonable
parameters to determine who among the supervisors and middle-level managers should
be retired for redundancy. Using the SGV report as anchor, the petitioner came out with
a special retirement program for its 108 supervisors and middle-level managers, making it
clear that its decision to eliminate them was final and irrevocable. Moreover, the private
respondents were not properly apprised of the existence of the special retirement
program, as well as the criteria for the selection of the supervisors to be retired, and
those to be retained or transferred or demoted.
- Contrary to its submissions, the petitioner downsized the Cane Marketing Department by
eliminating private respondents Perrin and Candelario; and Franco and Candelario from
the Sugar and Molasses Storage Department. The downsizing of personnel was not
among the foregoing recommendations, and yet this was what the petitioner did, through
its special retirement program, by including private respondents Franco and Pabalan,
thereby terminating their employment. It is too much of a coincidence that the two private
respondents were active members of the union.
Recommendations were made relating to the Cane Marketing Department, the report
recommended the beefing up of the petitioners planter service representative force, while
eliminating those who were ineffective. There is no showing in the record that
respondents Perrin and Candelario were eliminated solely because they were inefficient.
Neither is there any substantial evidence on record that the private respondents
performance had been deteriorating; on the contrary, they had been so far so efficient that
they had been given promotions from time to time during their employment. Yet, the
petitioner eliminated private respondents Perrin and Candelario and retained three PSRs,
namely, Danilo Villanueva, Roberto Combate and Danilo Labrador, who were employed
with the petitioner from one to three years and transferred Raymundo de la Rosa, who had
been working there for only six years. Again, it is too much of a coincidence that Franco
and Pabalan, the President and Treasurer, respectively, of the union, were included in the
special retirement program.
- As may be expected, the dismissals generated a general perception that management
was sending a strong message that all employees hold their position at its pleasure, and
that it was within its power to dismiss anyone anytime. With the dismissal of the union
officers and with the membership now effectively threatened, the union virtually collapsed
as an organization. Out of fear, no one would even assume the position of union
President. An indication of this sad state of affairs into which the union has fallen is that
nothing came out of its CBA proposal. It has been a year and three months as of this
writing since the respondent informed the union that its proposal had been referred to the

A2010

Disini

- 149 -

companys external counsel, but no counter-proposal has been submitted and no single
conference has been held since then.
- Redundancy exists when the service capability of the work force is in excess of what is
reasonably needed to meet the demands on the enterprise. A redundant position is one
rendered superfluous by any number of factors, such as over-hiring of workers, decreased
volume of business, dropping of a particular product line previously manufactured by the
company or phasing out of a service activity priorly undertaken by the business. Under
these conditions, the employer has no legal obligation to keep in its payroll more
employees than are necessary for the operation of its business.
- As seen in the case it was seen that contrary to the petitioners claim, the employer must
comply with the following requisites to ensure the validity of the implementation of a
redundancy program: (1) a written notice served on both the employees and the
Department of Labor and Employment at least one month prior to the intended date of
retrenchment; (2) payment of separation pay equivalent to at least one month pay or at
least one month pay for every year of service, whichever is higher; (3) good faith in
abolishing the redundant positions; and (4) fair and reasonable criteria in ascertaining
what positions are to be declared redundant and accordingly abolished.
- And as emphasized in the case of Panlilio v. National Labor Relations Commission that it
is imperative for the employer to have fair and reasonable criteria in implementing its
redundancy program, such as but not limited to (a) preferred status; (b) efficiency; and (c)
seniority.
- The general rule is that the characterization by an employer of an employees services
as no longer necessary or sustainable is an exercise of business judgment on the part of
the employer. The wisdom or soundness of such characterization or decision is not, as a
general rule, subject to discretionary review on the part of the Labor Arbiter, the NLRC and
the CA. Such characterization may, however, be rejected if the same is found to be in
violation of the law or is arbitrary or malicious.
2. NO
- While it may be true that the private respondents signed separate Deeds of Release
Waiver and Quitclaim and received separation pay, nonetheless, we find and so hold that
the NLRC did not err in nullifying the decision of the Labor Arbiter.
- The Release Waiver and Quitclaim were not verified by the complainants. Under
prevailing jurisprudence, the fact that an employee has signed a satisfaction receipt of his
claims does not necessarily result in the waiver thereof. The law does not consider as
valid any agreement whereby a worker agrees to receive less compensation than what he
is entitled to recover. A deed of release or quitclaim cannot bar an employee from
demanding benefits to which he is legally entitled. We have herefore (sic) explained that
the reason why quitclaims are commonly frowned upon as contrary to public policy and
why they are held to be ineffective to bar claims for the full measures of the workers legal
rights is the fact the employer and the employee obviously do not stand on the same
footing. The employer drove the employees to the wall. The latter must have to get hold
of the money. Because out of job, they had to face the harsh necessities of life. x x x
(Marcos vs. NLRC, G.R. No. 111744, September 8, 1995). The private respondents had
no other recourse but to execute the said Release Waiver and Quitclaim because the
petitioner made it clear in its Memorandum dated August 8, 1995 that it had the final say
on who would be included in its special retirement program. Their dismissal from the
petitioner corporation was a fait accompli, solely because they organized a union that
would bargain for reasonable terms and conditions of employment sought to be included
in a CBA. In fine, the private respondents were left to fend for themselves, with no source
of income from then on; prospects for new jobs were dim. Their backs against the wall,
the private respondents were forced to sign the said documents and receive their
separation pay.
Disposition petition is DENIED for lack of merit.

G&M INC V BATOMALAGUE


[PAGE 116]

PAYROLL PAYMENT
CHAVEZ V NLRC
[PAGE 59]
PHIL GLOBAL COMMUNICATIONS INC V DE VERA
[PAGE 52]

CASH WAGE
CONGSON V NLRC
243 SCRA 260
PADILLA; April 5, 1995

Labor Law 1
NATURE
Appeal from a decision of NLRC
FACTS
- Petitioner Dominico C. Congson is the registered owner of Southern Fishing Industry.
Private respondents Bargo, Himeno, Badagos, et al were hired on various dates by
Congson as regular piece-rate workers. They were uniformly paid at a rate of P1 per tuna
weighing 30-80 kilos per movement, that is from the fishing boats down to petitioner's
storage plant at a load/unload cycle of work until the tuna catch reached its final
shipment/destination. They did the work of unloading tuna from fishing boats to truck
haulers; unloading them again at petitioner's cold storage plant for filing, storing, cleaning,
and maintenance; and finally loading the processed tuna for shipment. They worked 7
days/week.
- In June 1990, Congson notified his workers of his proposal to reduce the rate-per-tuna
movement due to the scarcity of tuna. Private respondents resisted Congson's proposed
rate reduction. When they reported for work the next day, they were informed that they
had been replaced by a new set of workers. When they requested for a dialogue with the
management, they were instructed to wait for further notice. They waited for the notice of
dialogue for a full week but in vain.
- So private complainant workers filed complaint against Congson for underpayment of
wages, non-payment of overtime pay, 13th month pay, holiday pay, rest day pay, and 5day service incentive leave pay; and for constructive dismissal. Labor Arbiter and NLRC
ruled in favor of private respondent workers. NLRC found Congson guilty of illegal
dismissal. It held that private respondents did not abandon their work, but that Congson
replaced private respondents with a new set of workers without just cause and the
required notice and hearing. It also affirmed Labor Arbiters findings and monetary awards.
Hence, this appeal.
ISSUES
1. WON there was grave abuse of discretion on the part of respondent NLRC in upholding
Labor Arbiters award of salary differentials
2. WON NLRC was correct in affirming LAs award of separation pay
HELD
1. NO
- Petitioner Congson argues that despite the fact that private respondents' actual cash
wage fell below the minimum wage fixed by law, respondent NLRC should have
considered as forming a substantial part of private respondents' total wages the cash
value of the tuna liver and intestines private respondents were entitled to retrieve.
Petitioner therefore argues that the combined value of private respondents' cash wage
and the monetary value of the tuna liver and intestines clearly exceeded the minimum
wage fixed by law.
- Rule: Congsons practice of paying the private respondents the minimum wage by
means of legal tender combined with tuna liver and intestines runs counter to the above
cited provision of the Labor Code. The fact that said method of paying the minimum wage
was not only agreed upon by both parties in the employment agreement but even
expressly requested by private respondents, does not shield petitioner. Article 102 29 of the
Labor Code is clear. Wages shall be paid only by means of legal tender. The only instance
when an employer is permitted to pay wages informs other than legal tender, that is, by
checks. or money order, is when the circumstances prescribed in the second paragraph of
Article 102 are present.
2. YES
- Congson contends that: assuming arguendo that Labor Arbiter's findings were proper as
to private respondents' illegal dismissal, it did not state the reason why instead of
reinstatement, separation pay has to be awarded. Petitioner submits that under existing
laws and jurisprudence, whenever there is a finding of illegal dismissal, the available and
logical remedy is reinstatement. As a permissible exception to the general rule, separation
pay may be awarded to the employee in lieu of reinstatement, by reason of strained
relationship between the employer and employee. Since there was no finding or even
allegation of strained relationship between .petitioner and private respondents, NLRC
should have deleted the award of separation pay.
- A careful scrutiny of the records of the case discloses the existence of strained
relationship between the petitioner Congson and private respondents: [a] petitioner
consistently refused to re-admit private respondents in his establishment. Petitioner even
replaced private respondents with a new set of workers to perform the tasks of private
respondents [b] private respondents themselves, from the very start, had already
indicated their aversion to their continued employment in petitioner's establishment.
Disposition Petition DISMISSED. Challenged decision of NLRC AFFIRMED.
29

Article 102. Forms of Payment. No. employer shall pay the wages of an employee by means of, promissory notes,
vouchers, coupons, tokens tickets, chits, or any object other than legal tender, even when expressly requested by the
employee.
Payment of wages by check or money order shall be allowed when such manner of payment is customary on the date of
effectivity of this Code, or is necessary as specified in appropriate regulations to be issued by the Secretary of Labor or
as stipulated in a collective bargaining agreement.

A2010

- 150 -

Disini

PAYROLL ENTRIES
KAR ASIA V CORONA
437 SCRA 184
YNARES-SANTIAGO; August 24, 2004
FACTS
- Respondents, regular employees of petitioner KAR ASIA, Inc., an automotive dealer in
Davao City, filed on September 24, 1997 claimubg that they were not paid their cost of
living allowance (COLA) for the months of December 1993 and December 1994.
- Petitioner company and its president Celestino Barretto countered by saying that
respondents had already been paid their COLA for the said periods. Petitioners presented
in evidence the payrolls for December 1993 and December 1994 showing that the
respondents acknowledged in writing the receipt of their COLA, and the affidavits of
Ermina Daray and Cristina Arana, cashiers of KAR ASIA, refuting respondents claim that
they were made to sign blank pieces of paper.
- Labor Arbiter rendered a decision in favor of petitioners. NLRC affirmed the decision of
the Labor Arbiter. Court of Appeals reversed the decision of the NLRC and ordered
petitioner company to pay the respondents the P25.00 per day COLA for the period
December 1 to 31, 1994, plus interest thereon at the rate of 1% per month computed from
the time the same was withheld from respondents up to the time they were actually paid
the respective sums due them
ISSUE
WON not the petitioner company paid the respondents the COLA for December 1993 and
December 1994 as mandated by RTWPB XI Wage Order No. 3
HELD
YES
Ratio A close scrutiny of the payroll for the December 1993 COLA readily disclose the
signatures of the respondents opposite their printed names and the numeric value of
P654.00. Respondents averment that the petitioner company harassed them into signing
the said payroll without giving them its cash equivalent cannot be given credence. He who
asserts not he who denies must prove; unfortunately, the respondents miserably failed to
discharge this burden
Reasoning
- The payrolls for December 1 to 15, 1994 and December 16 to 31, 1994 indicate an
allowance of P327.00 for each period, or a total of P654.00 for the entire month. However,
a casual observation of the payroll for the December 1993 COLA will also show that the
respondents signed for the amount of P654.00. Also, the allowances appearing in the two
separate payslips for December 1 to 15, 1994 and December 16 to 31, 1994 sum up to a
total of P654.00. Although the numeric figures in the December 1994 payroll and the
payslips for the same period were denominated merely as allowances while those in the
December 1993 payroll were specifically identified as COLA, the fact that they add up to
the same figure, i.e., P654.00, is not a coincidence
- While ordinarily a payslip is only a statement of the gross monthly income of the
employee, his signature therein coupled by an acknowledgement of full compensation
alter the legal complexion of the document. The payslip becomes a substantial proof of
actual payment. Moreover, there is no hard-and-fast rule requiring that the employees
signature in the payroll is the only acceptable proof of payment. By implication, the
respondents, in signing the payslips with their acknowledgement of full compensation,
unqualifiedly admitted the receipt thereof, including the COLA for December 1994

9.04 TIME PAYMENT


9.05 PLACE PAYMENT
LABOR ADVISORY ON PAYMENT OF SALARIES THRU
AUTOMATED TELLER MACHINE (ATM)

- Article 104 of the Labor Code, as amended, requires


that payment of wages shall be made at or near the
place of undertaking, except as otherwise provided by

Labor Law 1
such regulations as the Secretary of Labor and
Employment may prescribe under conditions that
would ensure prompt payment and protection of
wages.
- Based on Article 104, as well as the provisions of Sec. 4, Rule VIII, Book III of the Codes
Implementing Rules and considering present-day circumstances, practices and
technology, employers may adopt a system of payment other than in the workplace, such
as through automated teller machine (ATM) of banks, provided that the following
conditions are met:
1. The ATM systems of payment is with the written consent of the employees concerned.
2. The employees are given reasonable time to withdraw their wages from the bank
facility which time, if done during working hours, shall be considered compensable
hours worked.
3. The system shall allow workers to receive their wages within the period or frequency
and in the amount prescribed under the Labor Code, as amended.
4. There is a bank or ATM facility within a radius of one kilometer to the place of work.
5. Upon request of the concerned employee/s, the employer shall issue a record of
payment of wages, benefits and deductions for particular period.
6. There shall be no additional expenses and no diminution of benefits and privileges as a
result of the ATM system of payment.
7. The employer shall assume responsibility in case the wage protection provisions of law
and regulations are not complied with under the arrangement.
- Done in the City of Manila, this 25th day of November 1996.
Sgd. Leonardo A. Quisumbing
Secretary.

9.06 DIRECT PAYMENT


9.07 CONTRACTOR SUB-CONTRACTOR
C. PROHIBITION REGARDING WAGES

9.08 NON-INTERFERRENCE DISPOSAL OF


WAGES
9.09 WAGE DEDUCTION
WAGE DEDUCTION
RADIO COMMUNICATIONS OF THE PHILS INC V SEC
OF LABOR

169 SCRA 38
REGALADO; January 9, 1989
FACTS
- On May 4, 1981, petitioner, a domestic corporation engaged in the telecommunications
business, filed with the National Wages Council an application for exemption from the
coverage of Wage Order No. 1. The application was opposed by respondent United RCPI
Communications Labor Association (URCPICLA-FUR), a labor organization affiliated with
the Federation of Unions of Rizal (FUR).
- On May 22, 1981, the National Wages Council disapproved said application and ordered
petitioner to pay its covered employees the mandatory living allowance of P2.00 daily
effective March 22, 1981.
- As early as March 13, 1985, before the aforesaid case was elevated to this Court,
respondent union filed a motion for the issuance of a writ of execution, asserting therein

A2010

- 151 -

Disini

its claim to 15% of the total backpay due to all its members as "union service fee" for
having successfully prosecuted the latter's claim for payment of wages and for
reimbursement of expenses incurred by FUR and prayed for the segregation and
remittance of said amount to FUR thru its National President.
- On October 24, 1985, without the knowledge and consent of respondent union, petitioner
entered into a compromise agreement with Buklod ng Manggagawa sa RCPI-NFL
(BMRCPI-NFL) as the new bargaining agent of oppositors RCPI employees.
- Thereupon, the parties filed a joint motion praying for the dismissal of the decision of the
National Wages Council for it had already been novated by the Compromise Agreement
re-defining the rights and obligations of the parties. Respondent Union on November 7,
1985 countered by opposing the motion and alleging that one of the signatories thereofBMRCPI-NFL is not a party in interest in the case but that it was respondent Union which
represented oppositors RCPI employees all the way from the level of the National Wages
Council up the Supreme Court. Respondent Union therefore claimed that the Compromise
Agreement is irregular and invalid, apart from the fact that there was nothing to
compromise in the face of a final and executory decision.
- Director Severo M. Pucan issued an Order dated November 25, 1985 awarding to
URCPICLA-FUR and FUR 15% of the total backpay of RCPI employees as their union
service fees, and directing RCPI to deposit said amount with the cashier of the Regional
Office for proper disposition to said awardees.
- Despite said order, petitioner paid in full the covered employees on November 29, 1985,
without deducting the union service fee of 15%.
- In an order dated May 7, 1986, NCR officer-in-charge found petitioner RCPI and its
employees jointly and severally liable for the payment of the 15% union service fee
amounting to P427,845.60 to private respondent URCPICLA-FUR and consequently
ordered the garnishment of petitioner's bank account to enforce said claim.
- Secretary of Labor and Employment issued an order on August 18, 1986 modifying the
order appealed from by holding petitioner solely liable to respondent union for 10% of the
awarded amounts as attorney's fees
ISSUE
WON public respondents acted with grave abuse of discretion amounting to lack of
jurisdiction in holding the petitioner solely liable for "union service fee' to respondent
URCPICLA-FUR

HELD
NO. Attorney's fee due the oppositor is chargeable
against RCPI.
Ratio The defaulting employer or government agency remains liable for attorney's fees
because it compelled the complainant to employ the services of counsel by unjustly
refusing to recognize the validity of the claim. (Cristobal vs. ECC)
Reasoning
- It is undisputed that oppositor (private respondent herein) was the counsel on record of
the RCPI employees in their claim for EC0LA under Wage Order No. 1 since the inception
of the proceedings at the National Wages Council up to the Supreme Court. It had
therefore a valid claim for attorney's fee which it called union service fee.

- As is evident in the compromise agreement,


petitioner was bound to pay only 30% of the amount
due each employee on November 30, 1985, while the
balance of 70% would still be the subject of
renegotiation by the parties. Yet, despite such
conditions beneficial to it, petitioner paid in full the

Labor Law 1

Disini

A2010 - 152 backpay of its employees on November 29, 1985, deductions required of the petitioner and the
ignoring the service fee due the private respondent.

employees do not run counter to the express mandate

- Worse, petitioner supposedly paid to one Atty.

of the law since the same are not unwarranted or

Rodolfo M. Capocyan the 10% fee that properly

without their knowledge and consent. Also, the

pertained to herein private respondent, an unjustified

deductions for the union service fee in question are

and baffling diversion of funds.

authorized by law and do not require individual

- Finally, petitioner cannot invoke the lack of an


individual written authorization from the employees
as a shield for its fraudulent refusal to pay the service
fee of private respondent. Be that as it may, the lack
thereof was remedied and supplied by the execution of
the compromise agreement whereby the employees,
expressly approved the 10% deduction and held
petitioner RCPI free from any claim, suit or complaint
arising from the deduction thereof. When petitioner
was thereafter again ordered to pay the 10% fees to
respondent union, it no longer had any legal basis or
subterfuge for refusing to pay the latter.
- We agree that Article 222 of the Labor Code
requiring an individual written authorization as a
prerequisite to wage deductions seeks to protect the
employee against unwarranted practices that would
diminish his compensation without his knowledge and
consent. However, for all intents and purposes, the

check-off authorizations.
Disposition the order of the Secretary of Labor of August 16, 1986 is hereby AFFIRMED
and the petition at bar is DISMISSED, with double costs against petitioner. The temporary
restraining order issued pursuant to the Resolution of the Court of June 22, 1987 is
LIFTED and declared of no further force and effect.

APODACA V NLRC (MIRASOL, INTRANS PHILS)


172 SCRA 442
GANCAYCO; April 18, 1989
NATURE
Special civil action for certiorari
FACTS
- Petitioner was employed in respondent corporation
- 1975: petitioner was appointed President and General Manager of respondent
corporation
- 1985: Respondent Mirasol persuaded petitioner to subscribe to 1,500 shares of
respondent corporation at P100 per share (total of P150,000). He made an initial payment
of P37,500.
- January, 1986: petitioner resigned
- December, 1986: petitioner instituted with NLRC a complaint for the payment of his
unpaid wages, his cost of living allowance, the balance of his gas and representation
expenses, and his bonus compensation for 1986.
- Private respondents admitted there is due to the petitioner P17,060, but this was applied
to the unpaid balance of his subscription in the amount of P95,439.93
- Petitioner questioned the set off since there was no call or notice for the payment of the
unpaid subscription, and that the alleged obligation is not enforceable.
- The NLRC held that a stockholder who fails to pay his unpaid subscription on call
becomes a debtor of the corporation and that the set-off of said obligation against the
wages and other due to petitioner is not contrary to law, morals, public policy
ISSUE
1. WON the NLRC has jurisdiction to resolve a claim for non-payment of stock
subscriptions to a corporation
2. WON an obligation arising from the non-payment of stock subscription can be offset
against a money claim of any employee against an employer
HELD
1. NO
Reasoning
- The NLRC has no jurisdiction to determine such intra-corporate dispute between the
stockholder and the corporation as in the matter of unpaid subscriptions. This is within the
exclusive jurisdiction of the Securities and Exchange Commission.
2. NO
Reasoning
- Assuming arguendo that the NLRC may exercise jurisdiction in this case, the unpaid
subscriptions are not due and payable until a call is made by the corporation for payment.
It does not appear that a notice of such call has been sent to petitioner. The records only
show that the respondent corporation deducted the amount due to petitioner from the
amount receivable from him from for the unpaid subscriptions. This set-off was without
lawful basis. As there was no notice or call for payment, the same is not yet due or
payable.

Labor Law 1
- Assuming that there had been a call for payment, the NLRC still cannot validly set it off
against the wages and other benefits due petitioner.
- Art. 113 of the Labor code allows such a deduction from the wages of the employees by
employer in only 3 instances: (a) In cases where the worker is insured with his consent by
the employer, and the deduction is to recompense the employer for the amount paid by
him as premium on the insurance; (b) For union dues, in cases where the right of the
worker or his union to checkoff has been recognized by the employer or authorized in
writing by the individual worker concerned; and (c) In cases where the employer is
authorized by law or regulations issued by the Secretary of Labor.
Disposition The petition is GRANTED and the questioned decision of the NLRC dated
September 18, 1987 is hereby set aside and another judgment is hereby rendered
ordering private respondents to pay petitioner the amount of P17,060.07 plus legal interest
computed from the time of the filing of the complaint on December 19, 1986, with costs
against private respondents.

CHECK OFF
MANILA TRADING & SUPPLY CO V MANILA TRADING
LABOR ASSN
93 PHIL 288
REYES; April 29, 1953
NATURE
Petition for certiorari to set aside decision of CIR.
FACTS
- On October 10, 1950, the Manila Trading Labor Association, composed of workers of
Manila Trading and Supply Co., made a demand upon said company for increase of
wages, increase of personnel, Christmas bonus, and other gratuities and privileges. As the
demand was refused and the Department of Labor - whose intervention had been sought
by the association - failed to effect an amicable settlement, the Head of the Department
certified the dispute to the Court of Industrial Relations on October 25, and there it was
docketed as case No. 521-V. The company, on its part, on that same day applied to the
Court of Industrial Relations for authority to lay off 50 laborers due to "poor business," the
application being docketed as Case No. 415-V (4).
- To resolve the disputes involved in the two cases the Court of Industrial Relations
conducted various hearings between October 26, 1950, and January 18, 1951. Of their
own volition the president and vice-president of the association attended some if not all of
the hearings, and though they absented themselves from work for that reason they
afterwards claimed that they were entitled to their wages. The Court of Industrial Relations
found merit in the claim, and at their instance, ordered the company to pay them their
wages corresponding to the days they were absent from work while in attendance at the
hearings.
- Contending that the industrial court had no authority to issue such an order, the company
asks this Court to have it annulled. Opposing the petition, the association, on its part,
contends that the order comes within the broad powers of the industrial court in the
settlement of disputes between capital and labor.
ISSUE
WON Court of Industrial Relations may require an employer to pay the wages of officers of
its employees' labor union while attending the hearing of cases between the employer and
the union

HELD
Ratio When in case of strikes, and according to the CIR even if the strike is legal, strikers
may not collect their wages during the days they did not go to work, for the same reasons
if not more, laborers who voluntarily absent themselves from work to attend the hearing of
a case in which they seek to prove and establish their demands against the company, the
legality and propriety of which demands is not yet known, should lose their pay during the
period of such absence from work.
Reasoning
- The age-old rule governing the relation between labor and capital or management and
employee is that of a "fair day's wage for a fair day's labor.' If there is no work performed
by the employee there can be no wage or pay, unless of course, the laborer was able,
willing and ready to work but was illegally locked out, dismissed or suspended. It is hardly
fair or just for an employee or laborer to fight or litigate against his employer on the
employer's time.
- The respondent association, however, claims that it was not the one that brought the
cases to the Court of Industrial Relations, and the point is made that "if the laborer who is
dragged to court is deprived of his wages while attending court hearings, he would in

A2010

- 153 -

Disini

effect be denied the opportunity to defend himself and protect his interests and those of
his fellow workers." But while it is true that it was the Secretary of Labor who certified the
dispute involved in case No. 521-V to the Court of Industrial Relations, the fact remains
that the dispute was initiated by a demand from the labor association. The truth, therefore,
is that while one of the cases was filed by the employer, the offer was initiated by the
employees. It may be conceded that the employer is in most cases in a better position to
bear the burdens of a litigation than the employees. But as was said in the case of J. P.
Heilbronn Co. vs. National Labor Union, supra, "It is hardly fair for an employee or laborer
to fight or litigate against his employer on the employer's time." The most that could be
conceded in favor of the claimants herein is to have the absences occasioned by their
attendance at the hearings charged against their vacation leave if they have any, or as
suggested by three of the Justices who signed the decision in the case just cited, to have
the wages they failed to earn charged as damages in the event the cases whose hearings
they attended are decided in favor of the association. But the majority of the Justices
make no commitment on this latter point.
Disposition Petition for certiorari is granted and the order complained of set aside.

9.10 DEPOSIT
DENTECH MANUFACTURING V NLRC (MARBELLA)
172 SCRA 588
GANCAYCO; April 19, 1989
FACTS
- Dentech Manufacturing Corporation is a domestic corporation organized under Philippine
laws owned and managed by the petitioner Jacinto Ledesma. The firm is engaged in the
manufacture and sale of dental equipment and supplies.
- Private respondents Benjamin Marbella, Armando Torno, Juanito Tajan, Jr. and Joel
Torno are members of the Confederation of Citizens Labor Union (CCLU), a labor
organization registered with the DOLE. They used to be the employees of Dentech,
working as welders, upholsterers and painters. They were already employed with the
company when it was still a sole proprietorship. They were dismissed from the firm
beginning February 14, 1985.
- They filed a Complaint with the NLRC against Dentech and Ledesma for, among others,
illegal dismissal and violation of PD 851. They were originally joined by another employee,
one Raymundo Labarda, who later withdrew his Complaint. At first, they only sought the
payment of their 13th month pay under PD 851 as well as their separation pay, and the
refund of the cash bond they filed with the company at the start of their employment. Later
on, they sought their reinstatement as well as the payment of their 13th month pay and
service incentive leave pay, and separation pay in the event that they are not reinstated. It
is alleged that they were dismissed from the firm for pursuing union activities.
- Dentech argued that they are not entitled to a 13th month pay. They maintained that
each of the private respondents receive a total monthly compensation of more that P1,000
and that under Section 1 of PD 851, such employees are not entitled to receive a 13th
month pay. Also, the company is in bad financial shape and that pursuant to Section 3, the
firm is exempted from complying with the provisions of the Decree. Dentech also
contended that the refund of the cash bond filed by the Marbella, et al., is improper
inasmuch as the proceeds of the same had already been given to a certain carinderia to
pay for their outstanding accounts.

ISSUES
1. WON the private respondents are entitled as a matter of right to a 13th month pay
2. WON the refund of the cash bond is proper

HELD
1. YES
Reasoning
- PD 851 was signed into law in 1975 by then President Ferdinand Marcos. Under the
original provisions of Section 1, all employers are required to pay all their employees
receiving a basic salary of not more than P1,000 a month, regardless of the nature of their
employment, a 13th month pay not later than December 24 of every year. Under Section 3
of the rules and regulations implementing PD 851, financially distressed employers, i.,e.,
those currently incurring substantial losses, are not covered by the Decree. Section 7
requires, however, that such distressed employers must obtain the prior authorization of
the Secretary of Labor before they may qualify for such exemption.

Labor Law 1
- On May 1, 1978, PD 1364 was signed into law. The Decree enjoined the DOLE to stop
accepting applications for exemption under PD 851. On August 13, 1986, President
Corazon Aquino issued Memorandum Order No. 28 which modified Section 1 of PD 851.
The said issuance eliminated the P1,000 salary ceiling.
- It clearly appears that Dentech has no basis to claim that it is exempted from complying
with the provisions of the law relating to the 13th month pay. The P1,000 salary ceiling
provided in PD 851 pertains to basic salary, not total monthly compensation. Dentech
admits that Marbella, at al., work only five days a week and that they each receive a basic
daily wage of P40 only. A simple computation of the basic daily wage multiplied by the
number of working days in a month results in an amount of less than P1,000. Thus, there
is no basis for the contention that the company is exempted from the provision of PD 851
which mandated the payment of 13th month compensation to employees receiving less
than P1,000 a month. [NOTE: Corys Memo (1986) is not yet applicable as of the time
Marbella, et al., were dismissed (1985).]
- Even assuming, arguendo, that Marbella, et al., are each paid a monthly salary of over
P1,000, Dentech is still not in a position to claim exemption. The rules and regulations
implementing PD 851 provide that a distressed employer shall qualify for exemption from
the requirements of the Decree only upon prior authorization from the Secretary of Labor.
No such prior authorization had been obtained by Dentech.
2. YES
Reasoning
- The refund of the cash bond is in order. Article 114 of the Labor Code prohibits an
employer from requiring his employees to file a cash bond or to make deposits, subject to
certain exceptions.
- Art. 114. Deposits for loss or damage. - No employer shall require his worker to
make deposits from which deductions shall be made for the reimbursement of loss
of or damage to tools, materials, or equipment supplied by the employer, except
when the employer is engaged in such trades, occupations or business where the
practice of making deductions or requiring deposits is a recognized one, or is
necessary or desirable as determined by the Secretary of Labor in appropriate rules
and regulations.
- Dentech has not satisfactorily disputed the applicability of this provision to the case at
bar. Considering further that it failed to show that it is authorized by law to require
Marbella, et al., to file the cash bond in question, the refund is in order.
- The allegation that the proceeds of the cash bond had already been given to a certain
carinderia to pay for the accounts of the private respondents does not merit serious
consideration. No evidence or receipt has been shown to prove such payment.
Disposition Petition is hereby DISMISSED for lack of merit.

FIVE J TAXI V NLRC


235 SCRA 556
REGALADO; August 22, 1994
NATURE
Special civil action for certiorari to annul NLRC decision
FACTS
- Maldigan and Sabsalon were hired by the petitioners as taxi drivers. They paid daily
"boundary" of P700 for air-conditioned or P450 for non-air-conditioned taxi, P20 for car
washing, and a P15 deposit to answer for any deficiency in their "boundary," for every
actual working day.
- In less than 4 months, Maldigan already failed to report for work. Later, petitioners
learned that he was working for "Mine of Gold" Taxi Company. - Sabsalon was held up by
his armed passenger who took all his money and stabbed him. After his hospital
discharge, he went to his home province to recuperate. He was re-admitted by petitioners
after 4 years under the same terms and conditions, but he was only allowed to drive only
every other day. However, on several occasions, he failed to report for work during his
schedule.
-Sept. 1991: Sabsalon failed to remit his "boundary" for the previous day. Also, he
abandoned his taxicab in Makati without fuel refill worth P300. He adamantly refused to
report to work despite demands. Afterwards it was revealed that he was driving a taxi for
"Bulaklak Company."
- 1989: Maldigan requested petitioners for the reimbursement of his daily cash deposits for
2 years, but petitioners told him that nothing was left of his deposits as these were not
even enough to cover the amount spent for the repairs of the taxi he was driving
- When Maldigan insisted on the refund, petitioners terminated his services. Sabsalon
claimed that his termination from employment was effected when he refused to pay for the
washing of his taxi seat covers. They filed a complaint for illegal dismissal and illegal
deductions
- Labor arbiter dismissed case holding that the unreasonable delay in filing the case (two
years) was not consistent with the natural reaction of a person who claimed to be unjustly
treated.

A2010

- 154 -

Disini

- NLRC: Private respondents dismissal was legal since they voluntarily left to work for
another company. The deductions were held illegal and it ordered petitioners to reimburse
the accumulated deposits and car wash payments, plus interest thereon at the legal rate
from the date of promulgation of judgment to the date of actual payment, and 10% of the
total amount as and for attorney's fees
ISSUE
WON private respondents are entitled to the refund of deposits
HELD
YES
- NLRC held that the daily deposits made by respondents to defray any shortage in their
"boundary" is covered by the general prohibition in Article 114 of the Labor Code and that
there is no showing that the Secretary of Labor has recognized the same as a "practice" in
the taxi industry.
Art. 114. Deposits for loss or damage. No employer shall require his worker to make
deposits from which deductions shall be made for the reimbursement of loss of or
damage to tools, materials, or equipment supplied by the employer, except when the
employer is engaged in such trades, occupations or business where the practice of
making deposits is a recognized one, or is necessary or desirable as determined by the
Secretary of Labor in appropriate rules and regulations.
- Article 114 does not apply to or permit deposits to defray any deficiency which the
taxi driver may incur in the remittance of his "boundary ." Also, when private
respondents stopped working for petitioners, the alleged purpose for which petitioners
required such unauthorized deposits no longer existed. In other case, any balance due to
private respondents after proper accounting must be returned to them with legal interest.
-The evidence shows that Sabsalon was able to withdraw his deposits through vales or he
incurred shortages, such that he is even indebted to petitioners in the amount of
P3,448.00. With respect to Maldigan's deposits, nothing was mentioned questioning the
same. Since the evidence shows that he had not withdrawn the same, he should be
reimbursed the amount of his accumulated cash deposits.
- On car wash payment: No refund. There was nothing to prevent private respondents
from cleaning the taxi units themselves, if they wanted to save their P20. Also, car
washing after a tour of duty is a practice in the taxi industry, and is, in fact, dictated by fair
play.
- On attorneys fees: Article 222 of the Labor Code, as amended by Section 3 of PD. 1691,
states that non-lawyers may appear before the NLRC or any labor arbiter only (1) if they
represent themselves, or (2) if they represent their organization or the members thereof.
While it may be true that Guillermo H. Pulia was the authorized representative of private
respondents, he was a non-lawyer who did not fall in either of the foregoing categories.
Hence, by clear mandate of the law, he is not entitled to attorney's fees.
Disposition NLRC decision MODIFIED by deleting the awards for reimbursement of car
wash expenses and attorney's fees and directing NLRC to order and effect the
computation and payment by petitioners of the refund for Maldigan's deposits, plus legal
interest thereon from the date of finality of this resolution up to the date of actual payment
thereof.

9.11 WITHHOLDING OF WAGES; RECORD


KEEPING
SPECIAL STEEL CORP V VILLAREAL
[PAGE 32]

PACIFIC CUSTOMS BROKERAGE V INTER-ISLAND


DOCKMEN AND LABOR UNION AND CIR
89 PHIL 722
BAUTISTA ANGELO; August 24, 1951
NATURE
Petition for review on certiorari
FACTS

Labor Law 1
- Inter-island Dockmen and Labor
Union filed petition in CIR against
Pacific Customs Brokerage praying
that said company be ordered to desist
from dismissing members of said
union, to turn over to the treasurer of
union all dues and fees withheld by the
company and to reinstate with backpay
the workers.
- A motion was filed by labor union
praying
that
Pacific
Customs
Brokerage be ordered to pay union
members their wages which was
allegedly withheld by it for certain
alleged damages caused by said
members for staging a strike. To this
motion, Pacific objected.
ISSUE
WON Pacific Customs Brokerage can be compelled by CIR to pay wages of the union
members in spite of the writ of garnishment issued by CFI in civil case, directing the sheriff
to levy upon moneys of Pacific Customs Brokerage Workers Union which are in the
possession of Pacific Customs Brokerage
HELD
YES
- Pacific Customs Brokerage contends otherwise because the moneys having been
garnished, are in custodia legis, and cant be controlled by CIR. The Court noticed that
this is the same argument advanced by petitioner before the respondent court in its effort
to frustrate the purpose of the motion of the labor union, and the respondent court found
said argument untenable. Art 1708 of new Civil Code provides, Laborers wages shall not
be subject to execution or attachment, except for debts incurred for food, shelter, clothing,
medical attendance.
- Pacific Customs Brokerage doesnt dispute that money garnished is intended to pay
wages of members of labor union. There is nothing to show that such money was
garnished or attached for debts incurred for food, shelter, clothing and medical
attendance. The writ of garnishment issued by the court, while it purports to include all
moneys and properties belonging to the employing company, cannot, in any manner,
touch or affect what said company has in its possession to pay the wages of its laborers.
- When CFI issued writ of garnishment, its scope could not have been extended to include
money intended to pay the wages of members of labor union.
- But before the order of the respondent court can be enforced there is need of lifting the
garnishment by presentation of a motion to that effect by the labor union.
- Petitioner's contention that the motion should be denied because it is predicated on a
labor contract entered into between the petitioner and the Pacific Customs Brokerage
Workers Union has no foundation in fact, it appearing that the members of the two labor
unions are one and the same. The members of the Pacific Customs Brokerage Workers'
Union are the same laborers now members of the petitioner union.

GAA V CA
[PAGE 148]

RECORD-KEEPING
SOUTH MOTORISTS ENTERPRISES V TOSOC [SEC OF
DOLE]
181 SCRA 386
MELENCIO-HERRERA; January 23, 1990
NATURE
Certiorari

A2010

- 155 -

Disini

FACTS
- January 1983, complaints for non-payment of emergency cost of living allowances were
filed by 46 workers, Tosoc, et als., against SOUTH MOTORISTS(SM) before the Naga
City District Office of Regional Office No. 5 of the then Ministry of Labor
- 10 January 1983 a Special Order was issued by the District Labor Officer directing its
Labor Regulation Officers to conduct an inspection and verification of SOUTH
MOTORISTS' employment records.
- On the date of the inspection and verification, SOUTH MOTORISTS was unable to
present its employment records on the allegation that they had been sent to the main
office in Manila.
- The case was then set for conference on 25 January 1983 but was reset twice.
- SM kept on requesting for postponements on the ground that the documents were still
being prepared and collated and that a formal manifestation or motion would follow.
Nothing did.
- After the submission of an Inspection Report on the basis of which an Order dated 14
April 1983 was issued by Labor Officer Domingo Reyes directing SMto pay Tosoc, et als.,
the total amount of P184,689.12 representing the latter's corresponding emergency cost of
living allowances.
- SM FILED a M for R BUT was denied.
- 11 July 1988, the Secretary of Labor and Employment affirmed the appealed Order.
- 28 July 1988, SM FILED another MR BUT WAS DENIED; FILED ANOTHER MR BUT
WAS STILL DENIED.
CLAIMS:
- SOUTH MOTORISTS: this falls under the original and exclusive jurisdiction of Labor
Arbiters (LA- a trier of facts, may determine after hearing such questions as WON an EREE relp exists; WON the workers were project workers; WON the employees worked
continuously or WON they should receive emergency cost of living allowances and if
entitled, how much each should receive..)
- TOSOC et al: maintain otherwise.
ISSUE
WON Regional Directors of DOLE have jurisdiction to validly act on/ award money claims
HELD
YES
Ratio Regional Directors are empowered to hear and decide, in a summary proceeding,
claims for recovery of wages and other monetary claims and benefits, including legal
interest, subject to the concurrence of the following requisites:
1) the claim is presented by an employee or person employed in domestic or household
service, or househelper under the Code;
2) the claim arises from employer-employee relations;
3) the claimant no longer being employed, does not seek reinstatement; and
4) the aggregate money claim of each employee or househelper does not exceed
P5,000.00 (Art. 129, Labor Code, as amended by R.A. 6715).
But where these requisites do not concur, the Labor Arbiters shall have exclusive original
jurisdiction over claims arising from employer-employee relationship except claims for
employees' compensation, social security, medicare and maternity benefits (parag. 6, Art.
217, Labor Code as amended by R.A. 6715).
Reasoning
- Two provisions of law are crucial to the issueA129 and A217 of the LC, as recently
amended by Republic Act No. 6715, approved on 2 March 1989. Said amendments, being
curative in nature, have retroactive effect and, thus, should apply in this case (BRIAD
AGRO vs. DE LA CERNA, G.R. No. 82805, and CAMUS ENGINEERING vs. DE LA
CERNA, G.R. No. 83225, 9 November 1989).
- The aforesaid Articles, as amended, respectively read as follows:
Art. 129. Recovery of wages, simple money claims and other benefits. Upon
complaint of any interested party, the Regional Director of the Department of Labor
and Employment or any of the duly authorized hearing officers of the Department is
empowered, through summary proceeding and after due notice, to hear and decide
cases involving the recovery of wages and other monetary claims and benefits,
including legal interest, owing to an employee or person employed in domestic or
household service and househelper under this Code, arising from employer-employee
relations: Provided, That such complaint does not include a claim for reinstatement:
Provided, further, That the aggregate claim of each employee or househelper does
not exceed five thousand pesos (P5,000.00). . . .
and
Art. 217. Jurisdiction of Labor Arbiters and the Commission. (a) Except as otherwise
provided under this Code, the Labor Arbiters shall have original and exclusive
jurisdiction to hear and decide, within thirty (30) calendar days after the submission of
the case by the parties for decision without extension, even in the absence of
stenographic notes, the following cases involving all workers, whether agricultural or
non-agricultural:
xxx xxx xxx

Labor Law 1
(6) Except claims for employees compensation, social security, medicare and
maternity benefits, all other claims arising from employer-employee relations, including
those of persons in domestic or household service, involving an amount exceeding
five thousand pesos (P5,000), whether or not accompanied with a claim for
reinstatement.
xxx xxx xxx
- In accordance said articles, those awards in excess of P5,000.00, particularly those
given to Gavino, Euste, Brequillo, Cis, Agreda, Galona, Tosoc, Guinoo, Cea, Guinoo, and
Osoc, each of which exceeds P5,000.00, should be ventilated in a proceeding before the
LAs.
- SM also caused the resetting of all subsequent hearings on the ground that the
documents were still being prepared and collated. - Having been given the opportunity to
put forth its case, SM has only itself to blame for having failed to avail of the same
- What is more, its repeated failure to attend the hearings, and to submit any motion as
manifested may be construed as a waiver of its right to adduce evidence to controvert the
worker's claims.
Disposition The award P l84,689.12 was MODIFIED. The individual claims of Gavino,
Euste ,Brequillo, Cis, Agreda, Galona, Tosoc, Guinoo, Cea, Guinoo, and Osoc, each of
which exceeds P5,000.00, were remanded to the LA for proper disposition. All other
individual awards not in excess of P5,000.00 were AFFIRMED.

D. OTHER FORMS OF RENUMERATIO N

A2010

- 156 -

Disini

WON respondent NLRC committed grave abuse of discretion in not reversing that portion
of the decision of the labor arbiter ordering petitioner to pay private respondent his share
in the service charge which was collected during the time he was not working in the hotel
HELD
NO
- Damalerio is entitled not only to full backwages but also to other benefits, including a just
share in the service charges, to be computed from the start of his preventive suspension
until his reinstatement.
- However, mindful of the animosity and strained relations between the parties, emanating
from this litigation, we uphold the ruling a quo that in lieu of reinstatement, separation pay
may be given to the private respondent, at the rate of one month pay for every year of
service. Should petitioner opt in favor of separation pay, the private respondent shall no
longer be entitled to share in the service charges collected during his preventive
suspension.
Disposition Petition dismissed

TIPS
ACE NAVIGATION CO INC V CA
[PAGE 146]

9.12 SERVICE CHARGES

9.13 THIRTEENTH MONTH PAY [PD 851]

SERVICE CHARGES

COVERAGE

MARANAW HOTELS AND RESORT CORPORATION V


NLRC (DAMALERIO)
303 SCRA 540
PURISIMA; February 23, 1999
NATURE
Special Civil Action in the Supreme Court
FACTS
- Damalerio, a room attendant of the Century Park Sheraton Hotel, operated by Maranaw
Hotel and Resort Corporation, was seen by hotel guest Glaser with left hand inside the
latter's suitcase. Confronted with what he was doing, Damalerio explained that he was
trying to tidy up the room. Not satisfied with the explanation of Damalerio, Glaser lodged a
written complaint before Despuig, shift-in-charge of security of the hotel. Glaser also
reported that Damalerio had previously asked from him souvenirs, cassettes, and other
giveaways. The complaint was later brought by Despuig to the attention of Major Buluran,
Chief of Security of the hotel.
- Damalerio was given a Disciplinary Action Notice (DAN ). The next day, an administrative
hearing was conducted on the matter. Among those present at the hearing were the room
attendant, floor supervisor, chief of security, personnel representative, and senior floor
supervisor, and union representative.
- Damalerio received a memorandum issued by San Gabriel, Sr. Floor Supervisor, bearing
the approval of Kirit, Executive Housekeeper, stating that he (Damalerio) was found to
have committed qualified theft in violation of House Rule No. 1, Section 3 of Hotel Rules
and Regulations. The same memorandum served as a notice of termination of his
employment.
- Damalerio filed with the Labor Arbiter a Complaint for illegal dismissal against the
petitioner. After the parties had sent in their position papers, Labor Arbiter rendered
judgment finding the dismissal of complainant to be illegal and ordering the respondents to
reinstate him to his former or equivalent position without loss of seniority rights and with
backwages from April 15, 1992 when he was preventively suspended up to actual
reinstatement and other benefits, including but not limited to his share in the charges
and/or tips which he failed to receive, and all other CBA benefits that have accrued since
his dismissal.
- From the aforesaid Labor Arbiter's disposition, the petitioner appealed to the NLRC,
which modified the appealed decision by giving petitioner the option of paying Damalerio a
separation pay equivalent to one month pay for every year of service, instead of
reinstating him.
- Petitioner interposed a motion for reconsideration but to no avail. NLRC denied the
same. Undaunted, petitioner brought this matter to the Court.
ISSUES

ULTRA VILLA FOOD HAUS V GENISTON


309 SCRA 17
KAPUNAN; June 23, 1999.
FACTS
- Geniston claims he is employed as a do it all guy in the UVFH owned by Rosie Tio. He
claims he was employed as a waiter, driver and maintenance guy in the restaurant. Tio
claims Geniston is her personal driver. Tio works as a manager of the CFC Corporation.
- On May 12, 1992, Tio called Genistons house to ask him to report for work even though
it was a holiday because she needed to do something important in the office. (The election
that year was on May 11. May 11 & 12 were holidays.) His wife took the call and informed
Tio that he wasnt there as he was working as a poll watcher.
- Tio says that Geniston abandoned his work. Geniston was dismissed. He filed case for
illegal dismissal and asked for benefits, including 13th month pay. LA found that he wasnt
an employee of UVFH, but instead was a personal driver of Tio. It found that his claim of
being a waiter isnt true because the positions of waiter and driver are incongruent --- as a
waiter he would have to be at the resto all day; as a driver, he would have to be away from
the resto. LA also told Tio to indemnify Geniston for P1k for failing to comply with the due
process requirement. NLRC reversed LAs decision.
ISSUES
1. WON Geniston was a personal driver and not an employee of UVHF
2. WON personal drivers are entitled to 13th month pay, according to the law
3. WON Geniston abandoned his work
HELD
1. YES
- The facts support Tios claim that Geniston was her personal driver. He was not in the
payroll of UVFH; UVFH employees attested that he was not one of them; warehousemen
of CFC Corp described Genistons relationship to Tio, ie, he brings her to work,
waits/sleeps in her car until she goes out for lunch, brings her back after lunch, then
waits/sleeps in her car until she goes home.
2. NO
- Art 141 of the LC defines Domestic or household service as to include services of
family drivers.
- The Revised Guidelines on the Implementation of the 13th Month Pay Law excludes
employers of household helpers from the coverage of PD 851, thus:
2.. Exempted Employers
The following employers are still not covered by P.D. No. 851:
a. . . .;
b. Employers of household helpers . . .;
c. . . .;
d. . . .

Labor Law 1
- BUT Geniston was awarded 13th month pay in view of Tios practice of according private
respondent such benefit. Indeed, petitioner admitted that she gave private respondent
13th month pay every December.
3. NO
- To constitute abandonment, two requisites must concur: (1) the failure to report to work
or absence without valid or justifiable reason, and (2) a clear intention to sever the
employer-employee relationship as manifested by some overt acts, with the second
requisite as the more determinative factor. The burden of proving abandonment as a just
cause for dismissal is on the employer. Petitioner failed to discharge this burden.
Note
- The court also found that Geniston is not entitled to the other benefits he was asking for
because Art 82 (LC) excludes domestic helpers from the mandatory grant of overtime pay,
holiday pay, premium pay and service incentive leave.
Disposition NLRC decision is reversed.

PETROLEUM SHIPPING V NLRC


[PAGE 79]

MANNER OF WAGE PAYMENT


JACKSON BLDG V NLRC (GUMOGDA)
246 SCRA 329
QUIASON; July 14, 1995
FACTS
- Ferdinand Gumogda underwent an appendectomy. Because his doctor advised him to
rest for at least 30 days, Gumogda filed for a 45-day LOA. He came back 50 days after the
operation but to his surprise, her wasnt allowed to return to work because according to
petitioners, Gumogda had abandoned his work. The Labor Arbiter and NLRC ruled in favor
of Gumogda. Petitioners appealed.
ISSUES
1. WON private respondent abandoned his work;
2. WON petitioners are liable for the payment of private respondent's back wages,
differential pay, thirteenth-month pay and service-incentive leave pay for 1991
HELD
1. NO
- For abandonment to be a valid ground for dismissal, two requisites must be copresent:
the intention by an employee to abandon coupled with an overt act from which it may be
inferred that the employee had no more intention to resume his work (People's Security,
Inc. vs. National Labor Relations Commission, 226 SCRA 146 [1993]).
- In the instant case, the said requisites are not present.
2. YES
- Gumogda heeded doctors advise and even exceeded the number of days
recommended by his doctor for his recuperation. In fact, he reported back for work 50
days after his operation. This would clearly show that private respondent was ready to
assume his responsibilities considering that he had fully recovered from the operation.
Furthermore, the filing of a complaint from illegal dismissal by private respondent is
inconsistent with the allegation of petitioners that he had abandoned his job. Surely, an
employee's posture will be illogical if he abandons his work and then immediately files an
action for his reinstatement. Article 279 of the Labor Code of the Philippines provides that
"an employee who is unjustly dismissed from work shall be entitled to reinstatement
without loss of seniority rights and other privileges and to his full back wages, inclusive of
allowances, and to his other benefits or their monetary equivalent computed from the time
his compensation was withheld from him up to the time of his actual reinstatement." Also,
Gumogda is likewise entitled to the thirteenth-month pay. Presidential Degree No. 851,
as amended by Memorandum Order No. 28, provides that employees are entitled to
the thirteenth-month pay benefit regardless of their designation and irrespective of
the method by which their wages are paid.
Disposition Petition is DISMISSED.

WAGE DIFFERENCE
JPL MARKETING PROMOTIONS V CA (GONZALES,
ABESA & ANNIPOT)
463 SCRA 136
TINGA; July 8, 2005
NATURE

A2010

- 157 -

Disini

Petition for review of the decision of the CA


FACTS
- JPL Marketing and Promotions is a domestic corporation engaged in the business of
recruitment and placement of workers.
- Gonzales, Abesa III and Aninipot were employed by JPL as merchandisers on separate
dates and assigned at different establishments in Naga City and Daet, Camarines Norte
as attendants to the display of California Marketing Corporation (CMC), one of petitioners
clients.
- On 13 August 1996, JPL notified private respondents that CMC would stop its direct
merchandising activity effective 15 August 1996.
- They were advised to wait for further notice as they would be transferred to other clients.
- On 17 October 1996, Abesa and Gonzales filed before the NLRC complaints for illegal
dismissal, praying for separation pay, 13 th month pay, service incentive leave pay and
payment for moral damages. Aninipot filed a similar case thereafter.
- Labor Arbiter Gelacio L. Rivera, Jr. dismissed the complaints for lack of merit.
- The Labor Arbiter held that:
1. The private respondents may be deemed to have severed their relation with JPL, and
cannot charge JPL with illegal dismissal, as they applied for different jobs even before the
lapse of the six (6)-month period given by law to JPL to provide them with new
assignments.
2. The claims for 13th month pay and service incentive leave pay should be denied
since private respondents were paid way above the applicable minimum wage
during their employment.
- NLRC affirmed the finding that there was no illegal dismissal, but ordered the payment of
separation pay, service incentive leave pay, and13th month pay.
- JPL filed a petition with the CA, claiming that private respondents were not entitled to the
separation pay, service incentive leave pay and 13th month pay.
- CA affirmed in toto the NLRC resolution. While conceding that there was no illegal
dismissal, it justified the award of separation pay on the grounds of equity and social
justice. The Court of Appeals rejected JPLs argument that the difference in the
amounts of private respondents salaries and the minimum wage in the region
should be considered as payment for their service incentive leave and 13 th month
pay.
- MFR denied, hence, this petition
ISSUES
1. WON private respondents were illegally dismissed, and thus entitled to separation pay
2. WON private respondents are entitled to 13th month pay and service incentive leave pay
HELD
1. NO
- Under Arts. 283 and 284 of the Labor Code, separation pay is authorized only in cases of
dismissals due to any of these reasons: (a) installation of labor saving devices; (b)
redundancy; (c) retrenchment; (d) cessation of the employer's business; and (e) when the
employee is suffering from a disease and his continued employment is prohibited by law
or is prejudicial to his health and to the health of his co-employees. However, separation
pay shall be allowed as a measure of social justice in those cases where the employee is
validly dismissed for causes other than serious misconduct or those reflecting on his moral
character, but only when he was illegally dismissed.
- The common denominator of the instances where payment of separation pay is
warranted is that the employee was dismissed by the employer
- In the instant case, there was no dismissal to speak of. What they received from JPL was
not a notice of termination of employment, but a memo informing them of the termination
of CMCs contract with JPL. More importantly, they were advised that they were to be
reassigned. At that time, there was no severance of employment to speak of.
- Art. 286 of the Labor Code allows the bona fide suspension of the operation of a
business or undertaking for a period not exceeding six (6) months, wherein an
employee/employees are placed on the so-called floating status.
- As clearly borne out by the records of this case, private respondents sought employment
from other establishments even before the expiration of the six (6)-month period provided
by law. JPL did not terminate their employment; they themselves severed their relations
with JPL. Thus, they are not entitled to separation pay.
2. YES
- JPL cannot escape the payment of 13th month pay and service incentive leave pay to
private respondents. Said benefits are mandated by law and should be given to
employees as a matter of right.
- Admittedly, private respondents were not given their 13th month pay and service incentive
leave pay while they were under the employ of JPL. Instead, JPL provided salaries which
were over and above the minimum wage.
- The Court rules that the difference between the minimum wage and the actual salary
received by private respondents cannot be deemed as their 13 th month pay and service
incentive leave pay as such difference is not equivalent to or of the same import as the
said benefits contemplated by law. Thus, as properly held by the Court of Appeals and by

Labor Law 1
the NLRC, private respondents are entitled to the 13 th month pay and service incentive
leave pay.
Disposition Petition granted in part. Award of separation pay deleted.

HOUSEHELPERS
ULTRAVILLA FOOD HOUSE V GENISTON
[PAGE 157]

GOVERNMENT EMPLOYEES
ALLIANCE OF GOVERNMENT WORKERS V MINISTER
OF LABOR (PNB)
124 SCRA 1
GUTIERREZ JR; August 3, 1983
NATURE
Petition to review decision of the Minister of Labor and Employment
FACTS
- Petitioner Alliance of Government Workers (AGW) is a registered labor federation while
the other petitioners are its affiliate unions with members who are employees of PNB,
MWSS, GSIS, SSS, PVTA, PNC, PUP, and PGEA.
- PD 851 was enacted:
WHEREAS, it is necessary to further protect the level of real wages from the ravage of
world-wide inflation;
WHEREAS, there has been no increase case in the legal minimum wage rates since
1970;
WHEREAS, the Christmas season is an opportune time for society to show its concern
for the plight of the working masses so they may properly celebrate Christmas and
New Year.
NOW, THEREFORE, I, FERDINAND E. MARCOS, by virtue of the powers vested in
me by the Constitution do hereby decree as follows:
SECTION 1. All employers are hereby required to pay all their employees receiving a
basic salary of not more than Pl,000 a month, regardless of the nature of their
employment, a 13th-month pay not later than December 24 of every year.
SECTION 2. Employers already paying their employees a 13th-month pay or its
equivalent are not covered by this Decree.
SECTION 3. This Decree shall take effect immediately. Done in the City of Manila, this
16th day of December 1975.
- According to the petitioners, P.D. No. 851 requires all employers to pay the 13th-month
pay to their employees with one sole exception found in Section 2. BUT Sec.3 of the Rules
and Regulations Implementing PD 851 included other types of employers not exempted by
the decree. (1) Distressed employees, (2) Government employees30, (3) Those already
paying 13th month pay, (4) Household helpers, (5) Those paid on purely commission,
boundary or task basis.
- Sec 3 is then challenged as a substantial modification by rule of a Presidential Decree
and an unlawful exercise of legislative power.
- Sol Gen: What the P.D. No. 851 intended to cover are only those in the private sector
whose real wages require protection from world-wide inflation. This is emphasized by the
"whereas" clause which states that 'there has been no increase in the legal minimum
wage rates since 1970'. This could only refer to the private sector, and not to those in the
government service because at the time of the enactment of PD 851 in 1975, only the
employees in the private sector had not been given any increase in their minimum wage.
The employees in the government service had already been granted in 1974 a ten percent
across-the-board increase.
ISSUE
WON the branches, agencies, subdivisions, and instrumentalities of the Government,
including GOCCs are required to pay all their employees receiving a basic salary of not
more than P1,000.00 a month, a 13th month pay (not later than December 24 of every
year)
HELD
NO
Ratio Since the terms and conditions of government employment are fixed by law,
government workers cannot use the same weapons employed by workers in the private
sector to secure concessions from their employers. Subject to the minimum requirements
30

b) The Government and any of its political subdivisions, including government-owned and controlled corporations,
except)t those corporation, operating essentially as private, ,subsidiaries of the government

A2010

Disini

- 158 -

of wage laws and other labor and welfare legislation, the terms and conditions of
employment in the unionized private sector are settled through the process of collective
bargaining. In government employment, however, it is the legislature and, where properly
given delegated power, the administrative heads of government which fix the terms and
conditions of employment.
Reasoning
- An analysis of the "whereases" of P.D. No. 851 shows that the President had in mind
only workers in private employment when he issued the decree. There was no intention to
cover persons working in the government service.
- Under the present Constitution, govemment-owned or controlled corporations are
specifically mentioned as embraced by the civil service. The amendment was intended to
correct the situation where more favored employees of the government could enjoy the
benefits of two worlds. They were protected by the laws governing government
employment.
- Why are the GOCCs part of the Civil Service?
(1) Nature of the public employer and peculiar character of public service: the Govt
protects the interests of ALL people in the public service, hence there would never be
conflicting interests.
(2) Govt agencies have a right to demand undivided allegiance.
(3) Governmental machinery must be impartial and non-political.
(4) To meet increasing social challenges of the times- the tendency towards a greater
socialization of economic forces.
- Section 631, Article XII-B of the Constitution gives added reasons why the government
employees represented by the petitioners cannot expect treatment in matters of salaries
different from that extended to all others government personnel.
- The Solicitor-General correctly points out that to interpret P.D. No. 851 as including
government employees would upset the compensation levels of government employees in
violation of those fixed.
Disposition Petition is DISMISSED for lack of merit.

SEPARATE OPINION
FERNANDO [concur pro hac vice]
- The approach taken by opinion of the Court is distinguished by its conformity to the
prevailing doctrine of statutory construction that unless so specified, the government does
not fall within the terms of any legislation or decree.
- "Since the terms and conditions of government employment are fixed by law,
government workers cannot use the same weapons employed by workers in the private
sector to secure concessions from their employers.
- In government employment, however, it is the legislature and, where properly given
delegated power, the administrative heads of government which fix the terms and
conditions of employment. This is effected through statutes or administrative circulars,
rules, and regulations, not through collective bargaining agreements.

MAKASIAR [dissent]
- It will be noted that the PD 851 provides only one exception in its Section 2: "Employers
already paying their employees a 13th-month pay or its equivalent..." Hence, all other
employers, whether of the private sectors or of GOCCs and government agencies, are
thereunder obligated to pay their employees.
- If the President intended to favor only employees of the private sector, he could have
easily inserted the phrase "in the private sector between the words "wages" and "from" in
the first WHEREAS, and between the words masses" and "so" in the third WHEREAS; or
the President could have included the other four classes of employers in the questioned
Section 3.
- The position taken by public respondents is repugnant to the social justice guarantee
under the new Constitution. The laboring masses of the government- owned and
-controlled agencies are entitled to such dignity, welfare and security as well as an
equitable share in the profits of respondents which will inevitably contribute to enhancing
their dignity, welfare and security, as much as those of the workers and employees of the
private sector.
- Basic rule is that all doubts should be interpreted in favor of labor.
- To deny them this right would render the State culpable of failing to "afford protection to
labor, promote... equality in employment as well as "just and humane conditions of
work."

TERMINATED EMPLOYEES

31

SEC. 6. The National Assembly shall provide for the standardization of compensation of government officials and
employees, including those in government-owned or controlled corporations, taking into account the nature of the
responsibilities pertaining to, and the qualifications required for the positions concerned.

Labor Law 1
ARCHILLES MANUFACTURING CORP V NLRC
(MANUEL, ET AL)
244 SCRA 750
BELLOSILLO; June 2, 1995
NATURE
Appeal on certiorari
FACTS
- Private respondents Geronimo Manuel, Arnulfo Diaz, Jaime Carunungan and Benjamin
Rindon were employed by Archilles Manufacturing Corporation, (Alberto Yu - Chairman)
and (Adrian Yu-VP) as laborers in its steel factory located in Bulacan, each receiving a
daily wage of P96.00.
- ARCHILLES was maintaining a bunkhouse in the work area which served as resting
place for its workers. In 1988 a mauling incident nearly took place involving a relative of an
employee. As a result, ARCHILLES prohibited its workers from bringing any member of
their family to the bunkhouse. But despite this prohibition, private respondents continued
to bring their respective families to the bunkhouse, causing annoyance and discomfort to
the other workers. This was brought to the attention of ARCHILLES.
- The management ordered private respondent to remove their families from the
bunkhouse and to explain their violation of the company rule. Private respondents
removed their families from the premises but failed to report to the management as
required; instead, they absented themselves from 14 to 18 May 1990. Consequently,
ARCHILLES terminated their employment for abandonment and for violation of the
company rule regarding the use of the bunkhouse. 3
- Private respondents filed a complaint for illegal dismissal. The Labor Arbiter found the
dismissal of private respondents illegal and ordered their reinstatement as well as the
payment to them the backwages, proportionate 13th month pay for the year 1990 and
attorney'sfees. ARCHILLES appealed.
- NLRC set aside the decision of the LA and ruled that the dismissal of private
respondents was valid. However, it ordered ARCHILLES to pay private respondents their
"withheld" salaries from 19 September 1991and to pay their proportionate 13th month pay
for 1990.
ISSUE
WON dismissal for cause results in the forfeiture of the employee's right to a 13th month
pay
HELD
NO
- Paragraph 6 of the Revised Guidelines on the Implementation of the 13th Month Pay
Law (P. D. 851) provides that "an employee who has resigned or whose services were
terminated at any time before the payment of the 13th month pay is entitled to this
monetary benefit in proportion to the length of time he worked during the year, reckoned
from the time he started working during the calendar year up to the time of his resignation
or termination from the service . . . The payment of the 13th month pay may be demanded
by the employee upon the cessation of employer-employee relationship. This is consistent
with the principle of equity that as the employer can require the employee to clear himself
of all liabilities and property accountability, so can the employee demand the payment of
all benefits due him upon the termination of the relationship."
- Furthermore, Sec. 4 of the original Implementing Rules of P.D. 851 mandates employers
to pay their employees a 13th month pay not later than the 24th of December every year
provided that they have worked for at least one (1) month during a calendar year. In effect,
this statutory benefit is automatically vested in the employee who has at least
worked for one month during the calendar year. As correctly stated by the Solicitor
General, such benefit may not be lost or forfeited even in the event of the
employee's subsequent dismissal for cause without violating his property rights.

RATIONALE PD 851 WHEREAS CLAUSE AND


LIMITATIONS
BASIC WAGE/COMMISSIONS
BOIE TAKEDA V DELA SERNA
228 SCRA 329
NARVASA; December 10, 1993
NATURE
Petition for review via certiorari and for issuance of writ of prohibition (consolidated)

A2010

- 159 -

Disini

FACTS
(HISTORY OF 13TH MONTH PAY PD 851)
- Initially, PD 851 ordered the payment of 13th month pay to workers receiving basic salary
of not more than P1,000.00 a month, regardless of the nature of the employment.
- DECEMBER 22, 1975: Rules and Regulations Implementing P.D. 851 promulgated by
Labor Minister Ople, defined 13th month pay, and basic salary as including all
remunerations or earnings paid by an employer to an employee for services rendered but
may not include cost of living allowances granted pursuant to Presidential Decree No.
525 or Letter of Instructions No. 174, profit sharing payments, and all allowances and
monetary benefits which are not considered or integrated as part of the regular or basic
salary of the employee at the time of the promulgation of the Decree on December 16,
1975; and exempted employers of those who are paid on purely commission,
boundary, or task basis, and those who are paid a fixed amount for performing a
specific work from payment of 13th month pay
- Supplementary Rules and Regulations implementing P.D. 851 were subsequently issued
by Minister Ople, which enumerated items not included in the computation of the 13 th
month pay: overtime pay, earnings and other remunerations which are not part of the
basic salary
- AUGUST 13, 1986: President Corazon C. Aquino promulgated Memorandum Order No.
28 which modified PD 851 by removing the salary ceiling of P1,000.00 a month set by
the latter,
- NOVEMBER 16, 1987: Revised Guidelines on the Implementation of the 13th Month Pay
Law were promulgated by Sec. Drilon which, among other things, enumerated
remunerative items not embraced in the concept of 13th month pay (allowances and
monetary benefits which are not considered or integrated as part of the regular or basic
salary, such as the cash equivalent of unused vacation and sick leave credits, overtime,
premium, night differential and holiday pay, and cost-of-living allowances), and
specifically dealt with employees who are paid a fixed or guaranteed wage plus
commission (Employees who are paid a fixed or guaranteed wage plus commission
are also entitled to the mandated 13th month pay based on their total earnings during
the calendar year, i.e., on both their fixed or guaranteed wage and commission)
- DOLE conducted a routine inspection in the premises of both Boie Takeda and Philippine
Fuji Xerox Corp. It was found that both companies failed to pay the 13 th month pay of their
employees for the years 1986, 1987, and 1988. Both companies were ordered to restitute
the said underpayment within 5-10 days. Both companies appealed but were denied.
- BASIC CONTENTION OF THE PETITIONERS : commissions should not be included in the
computation of the basic salary as basis for the 13th month pay
- BASIC CONTENTION OF THE RESPONDENTS: Commissions are now included in the
computation for the 13th month pay, as clarified by the Revised Guidelines issued by Sec.
Drilon
ISSUE
WON the Revised Guidelines on the Implementation of the 13th Month Pay Law issued by
Labor Sec. Drilon should be declared null and void as being violative of the law said
Guidelines were issued to implement, hence issued with grave abuse of discretion
correctible by the writ of prohibition and certiorari (Thus, commissions should not be
included in the computation for basic salary as basis for 13th month pay)
HELD
YES
- In including commissions in the computation of the 13th month pay, the second
paragraph of Section 5(a) of the Revised Guidelines on the Implementation of the 13th
Month Pay Law unduly expanded the concept of "basic salary" as defined in P.D. 851. It is
a fundamental rule that implementing rules cannot add to or detract from the
provisions of the law it is designed to implement. Administrative regulations
adopted under legislative authority by a particular department must be in harmony
with the provisions of the law they are intended to carry into effect. They cannot
widen its scope. An administrative agency cannot amend an act of Congress.
Ratio. In remunerative schemes consisting of a fixed or guaranteed wage plus
commission, the fixed or guaranteed wage is patently the "basic salary" for this is what the
employee receives for a standard work period. Commissions are given for extra efforts
exerted in consummating sales or other related transactions. They are, as such, additional
pay, which this Court has made clear do not form part of the "basic salary."
Reasoning
- San Miguel Corp. vs. Inciong discussion on history of 13 th Month Pay Law. The exclusion
of all allowances and monetary benefits such as profit-sharing payments, COLA, overtime
pay, premiums for special holiday, and the like indicate the intention to strip basic salary of
other payments, and any and all additions which may be in the form of allowances or
fringe benefits. If they were not excluded, it is hard to find any earnings and other
remunerations (exclusionary phrase) expressly excluded in the computation of the 13th
month pay. Then the exclusionary provision would prove to be idle and with no purpose.
Disposition the consolidated petitions are hereby GRANTED. The second paragraph of
Section 5 (a) of the Revised Guidelines on the Implementation of the 13th Month Pay Law
issued on November 126, 1987 by then Labor Secretary Franklin M. Drilon is declared null
and void as being violative of the law said Guidelines were issued to implement, hence

Labor Law 1
issued with grave abuse of discretion correctible by the writ of prohibition and certiorari.
The assailed Orders of January 17, 1990 and October 10, 1991 based thereon are SET
ASIDE. SO ORDERED

PHIL DUPLICATORS V NLRC (PHIL DUP. EMPLOYEES


UNION)
241 SCRA 380
FELICIANO; February 15, 1995
FACTS
- The Court rendered a decision dismissing a petition for certiorari by Phil. Duplicators, Inc
(PDI). The Court upheld the decision of public respondent NLRC ordering PDI to pay 13 th
month pay to private respondent employees computed on the basis of their fixed wages
plus sales commissions.
- PDI filed an MFR, invoking the decisions in the 2 consolidated cases of Boie-Takeda
Chem. vs Hon. Dionisio de la Serna and Phil. Fuji Xerox Corp. vs Hon. Cresenciano
Trajano. PDI alleged that the decision in the Duplicators case should be reversed since
the Boie-Takeda decision went directly opposite and contrary to the conclusion reached
in the former further seeking to dismiss the money claims of private respondent union. In
view of the nature of the issues raised, the Court considered the MFR and accepted it as a
banc case.
ISSUES
1. WON the Duplicators decision goes against the Boie-Takeda decision
2. WON the sales commission earned by the salesmen of PDI constitute a part of their
wage and should be included in the computation of 13th month pay
HELD
1. NO
- The doctrines enunciated in the 2 cases present different factual situations. The so-called
commissions received by the Boie-Takeda medical reps or by the rank and file employees
of Fuji were characterized as productivity bonuses. These are additional monetary
benefits generally tied to the capacity for revenue production of a corporation. As such,
they more closely resemble profit-sharing payments and are not directly related to the
amount of work actually done by an employee.
-The commissions paid to the medical reps were not sales commissions in the same
sense as in the Duplicators case. Medical representatives are not salesmen; they merely
promote products and leave samples with physicians. As such, no actual sales are made
placing the commissions in the nature of a profit-sharing bonus.
2. YES
- The commissions received by every duplicating machine sold constitute part of the basic
compensation of PDIs salesmen, apart from a small fixed wage. It is important to note that
the fixed portion of their salaries represent only 15-30% of an employees total earnings in
a year. Considering this, the sales commissions were an integral part of PDIs basic salary
structure and not mere profit-sharing payments or fringe benefits.
-The Supplementary Rules and Regulations Implementing P.D. 851(The 13 th Month Pay
Law) clarifies the scope of items excluded in the computation of 13 th month pay. Section 4
of the Law states that Overtime pay, earnings and other remunerations which are not part
of the basic salary shall not be included in the computation of the 13 th month pay. What
constitutes other remunerations not part of basic salary is a question to be resolved on a
case-to-case basis. In the instant case, it is important to distinguish the productivity
bonuses granted in Boie-Takeda from the sales commissions of the Duplicators case.
- A productivity bonus is something extra given to an employee for which no specific
additional services are rendered. Since a bonus is a gratuity of the employer, the recipient
cannot demand its payment as a matter of right. If an employer cannot be compelled to
pay a productivity bonus to his employees, then it follows that the bonus should not fall
under basic salary when computing 13th month pay.
- Sales commissions, on the other hand, are directly proportional to the extent or energy of
an employees work. Such commissions are paid upon the specific results achieved by a
salesman and form an integral part of his basic pay and should thus be included in the
computation of 13th month pay.
Disposition MFR is denied for lack of merit

IRAN V NLRC
[PAGE 148]

A2010

- 160 -

Disini

HONDA PHILS INC V SAMAHAN NG MALAYANG


MANGGAGAWA SA HONDA
460 SCRA 186
YNARES-SANTIAGO; June 15, 2005
FACTS
- A Collective Bargaining Agreement (CBA) was forged between petitioner Honda and
respondent union Samahan ng Malayang Manggagawa sa Honda (respondent union).
Among others, the CBA provides that the Company will maintain the present practice in
the implementation of the 13th month pay, shall grant a 14 th Month Pay, computed on the
same basis as computation of 13th Month Pay and shall continue the practice of granting,
in its discretion, financial assistance to covered employees in December of each year, of
not less than 100% of basic pay. This CBA is effective until year 2000.
- In 1998, the two parties started re-negotiations for the 4 th and 5th years of their CBA
(meaning for yr 1999 to 2000). However, the talks bogged down. The union filed a Notice
of Strike on the ground of bargaining deadlock. Thereafter, Honda filed a Notice of
Lockout. DOLE intervened and ordered the parties to cease and desist from committing
acts that would aggravate the situation. Both parties complied accordingly.
- On May 11, 1999, however, respondent union filed a second Notice of Strike on the
ground of unfair labor practice alleging that Honda illegally contracted out work to the
detriment of the workers. The DOLE again intervened and the striking employees were
ordered to return to work and the management accepted them back under the same terms
prior to the strike staged.
- On November 22, 1999, the management of Honda issued a memorandum[4]
announcing its new computation of the 13th and 14th month pay to be granted to all its
employees whereby the thirty-one (31)-day long strike shall be considered unworked days
for purposes of computing said benefits. As per the companys new formula, the amount
equivalent to 1/12 of the employees basic salary shall be deducted from these bonuses,
with a commitment however that in the event that the strike is declared legal, Honda shall
pay the amount deducted (In effect, this enabled them to devise a formula using 11/12 of
the total annual salary as base amount for computation instead of the entire amount for a
12-month period.
- The union opposed the pro-rated computation of the bonuses.
ISSUE
WON the pro-rated computation of the 13th month pay and the other bonuses in question
is valid and lawful
HELD
NO
Reasoning
- The said pro-rated computation is violative of the provisions of the CBA. A collective
bargaining agreement refers to the negotiated contract between a legitimate labor
organization and the employer concerning wages, hours of work and all other terms and
conditions of employment in a bargaining unit. As in all contracts, the parties in a CBA
may establish such stipulations, clauses, terms and conditions as they may deem
convenient provided these are not contrary to law, morals, good customs, public order or
public policy. Thus, where the CBA is clear and unambiguous, it becomes the law
between the parties and compliance therewith is mandated by the express policy of the
law.
- It is violative of the provision of P.D. No. 851 which, provided that the minimum 13th
month pay required by law shall not be less than one-twelfth (1/12) of the total basic salary
earned by an employee within a calendar year.
- The act has ripened into a practice and therefore can no longer be withdrawn, reduced,
diminished, discontinued or eliminated. Honda did not adduce evidence to show that the
13th month, 14th month and financial assistance benefits were previously subject to
deductions or pro-rating or that these were dependent upon the companys financial
standing.
- It is more in keeping with the underlying principle for the grant of this benefit. It is
primarily given to alleviate the plight of workers and to help them cope with the exorbitant
increases in the cost of living. To allow the pro-ration of the 13th month pay in this case is
to undermine the wisdom behind the law and the mandate that the workingmans welfare
should be the primordial and paramount consideration.
- To rule otherwise inevitably results to dissuasion, if not a deterrent, for workers from the
free exercise of their constitutional rights to self-organization and to strike in accordance
with law.
Disposition Denied.

SUBSTITUTE PAYMENTS
FRAMANLIS FARMS INC V MOLE
171 SCRA 87

Labor Law 1
GRINO-AQUINO; March 8, 1989
NATURE
Petition for certiorari to reverse the denied MFR denied by MOLE order.
FACTS

(The facts are difficult to digest because they involved

A2010

- 161 -

Disini

- Under Section 3 of PD No. 851 32, such benefits in the form of food or free electricity,
assuming they were given, were not a proper substitute for the 13th month pay required
by law.
- Neither may year-end rewards for loyalty and service be considered in lieu of 13th month
pay according to Section 10 of the Rules and Regulations Implementing Presidential
Decree No.
- The failure of the Minister's decision to identify the pakyaw and non-pakyaw workers
does not render said decision invalid. The workers may be identified or determined in the
proceedings for execution of the judgment.
Disposition petition for certiorari is dismissed with costs against the petitioners.

numbers. For recitation purposes, the reasoning is


enough.)
- Employees of the petitioners filed against their employer and the other petitioners 2 labor
standard cases in the RTC alleging that they were not paid emergency cost of living
allowance (ECOLA), minimum wage 13 th month pay, holiday pay, and service incentive
leave pay.
- Petitioners, in an answer to the amended complaint, alleged that (1) the private
respondents were not regular workers, but were migratory (sacadas) or pakyaw workers
who were hired seasonally, or only during the milling season, to so piece-of work on the
farms, hence they were not entitled to benefits being claimed, (2) they applied for an
exception to pay for the living allowance although the MOLE has no ruling yet.
- The claims for holiday pay, service incentive pay, social amelioration bonus and
underpayment fo minimum wage were not controverted. On the other claims, the
petitioners submitted only random payrolls which showed that the women workers were,
although the male workers received P10 more or less, per day.
- In an Order, the Minister of Labor (MOLE), through Assistant Regional Director Dante
Ardivilla, adopting the recommendations of the Chief of the Labor Regulation Section,
Bacolod District Office, directed the respondents (now petitioners) to pay: (1) deficiency
payments under PD 925,PD 1614 , under Ministry Order No. 5, under PD 1678, service
incentive leave pay, holiday pay and social amelioration bonus and 13 th month pay and
emergency living allowance under PD 1123
- Upon the petitioners' appeal of that Order, the Deputy MOLE modified it ordering the
employer to all non-pakyaw workers their claim for holiday and incentive leave pay, their
13th month pay, pay differentials and ECOLA excluding the pakyaw workers from holiday
and service incentive leave pay
- Framanlis filed for MFR, which was denied hence, this petition for certiorari

ISSUE
1. WON Minister erred in requiring the petitioners to pay wage differentials to their pakyaw
workers who worked for at least eight hours daily
2. WON benefits in form of food and electricity are equivalent to the 13th month pay
HELD
1. NO
- In 1976, PD No. 928 fixed a minimum wage for agricultural workers in any plantation or
agricultural enterprise irrespective of WON the worker was paid on a piece-rate basis.
However, effective July 1, 1978, the minimum wage was increased (Sec. 1, PD 1389).
Subsequently, PD 1614 provided for another increase in the daily wage of all workers
effective April 1, 1979. The petitioners admit that those were the minimum rates prevailing
then. Therefore, the respondent Minister did not err in requiring the petitioners to pay
wage differentials to their pakyaw workers who worked for at least eight hours daily and
earned less than P8.00 per day in 1978 to 1979.
2. NO
- With regard to the 13th month pay, petitioners admitted that they failed to pay their
workers 13th month pay. However, they argued that they substantially complied with the
law by giving their workers a yearly bonus and other non-monetary benefits amounting to
not less than 1/12th of their basic salary in weekly subsidy of choice pork meat, free
choice pork meat and free light or electricity which were allegedly "the equivalent" of the
13th month pay.

14th MONTH PAY


KAMAYA PORT HOTEL V NLRC (FEDERATION OF
FREE WORKERS)
177 SCRA 87
FERNAN; August 31, 1989
NATURE
Petition for review on certiorari
FACTS
- Respondent Memia Quiambao with thirty others who are members of the Federation of
Free Workers (FFW) were employed by Kamaya as hotel crew. On the basis of the
profitability of the company's business operations, management granted a 14th
month pay to its employees starting in 1979. In January 1982, the hotel converted into
a training center for Libyan scholars. Hoever, the Libyans preterminated their program
leaving Kamayan without any business, aside from the fact that it was not paid for the use
of the hotel premises. All in all Kamayan allegedly suffered losses amounting to P2
million.
- Although Kamayan reopened the hotel premises to the public, it was not able to pick-up
its lost patronage. In a couple of months it effected a retrenchment program until finally, it
totally closed its business.
- FFW then filed with the Ministry of Labor and Employment a complaint against petitioner
for illegal suspension, violation of the CBA and non-payment of the 14th month pay.
Records however show that the case was submitted for decision on the sole issue of
alleged non-payment of the 14th month pay for the year 1982.
- The LA rendered a decision ordering Kamaya to pay the 14th month pay. On appeal, the
NLRCaffirmed the grant of the 14th month pay on the ground that the granting of this 14th
month pay has already ripened into a company practice which respondent company
cannot withdraw unilaterally. This 14th month pay is now an existing benefit which cannot
be withdrawn without violating article 100 of the Labor Code. To allow its withdrawal now
would certainly amount to a diminution of existing benefits which complainants are
presently enjoying.
ISSUE
WON the latter tribunal committed grave abuse of discretion when it adopted the Labor
Arbiter's decision saying that the 14th month pay cannot be withdrawn without violating
Article 100 of the Labor Code
HELD
YES
- Art. 100 of the LC states: Prohibition against elimination or diminution of benefits.Nothing in this Book shall be construed to eliminate or in any way diminish supplements,
or other employee benefits being enjoyed at the time of promulgation of this Code.
32

Section 3. Employees covered The Decree shall apply to all employees except to:
xxx
xxx
xxx
"The term 'its equivalent' as used in paragraph (c) hereof shall include Christmas bonus, mid-year bonus, profit-sharing
payments and other cash bonuses amounting to not less than 1/12 of the basic salary but shall not include cash and
stock dividends, cost of living allowances and all other allowances regularly enjoyed by the employee, as well as nonmonetary benefits.
"Where an employer pays less than 1/12 of the employee's basic salary the employer shall pay the difference."

Labor Law 1
- It is patently obvious that Article 100 is clearly without applicability. The date of effectivity
of the Labor Code is May 1, 1974. In the case at bar, petitioner extended its 14th month
pay beginning 1979 until 1981. What is demanded is payment of the 14th month pay for
1982. Indubitably from these facts alone, Article 100 of the Labor Code cannot apply.
- Moreover, there is no law that mandates the payment of the 14th month pay. This is
emphasized in the grant of exemption under Presidential Decree 851 (13th Month Pay
Law) which states: "Employers already paying their employees a 13th month pay or its
equivalent are not covered by this Decree." Necessarily then, only the 13th month pay is
mandated. Having enjoyed the additional income in the form of the 13th month pay,
private respondents' insistence on the 14th month pay for 1982 is already an unwarranted
expansion of the liberality of the law.
- Verily, a 14th month pay is a misnomer because it is basically a bonus and, therefore,
gratuitous in nature. The granting of the 14th month pay is a management prerogative
which cannot be forced upon the employer. It is something given in addition to what is
ordinarily received by or strictly due the recipient. It is a gratuity to which the recipient has
no right to make a demand.
- This Court is not prepared to compel petitioner to grant the 14th month pay solely
because it has allegedly ripened into a company practice" as the labor arbiter has put it.
Having lost its catering business derived from Libyan students, Kamaya Hotel should not
be penalized for its previous liberality.
An employer may not be obliged to assume a "double burden" of paying the 13th month
pay in addition to bonuses or other benefits aside from the employee's basic salaries or
wages. Restated differently, we rule that an employer may not be obliged to assume the
onerous burden of granting bonuses or other benefits aside from the employee's basic
salaries or wages in addition to the required 13th month pay.
Disposition petition is hereby GRANTED. The portion of the decision of the National
Labor Relations Commission dated June 25, 1986 ordering the payment of 14th month
pay to private respondents is set aside.

DIMINUTION
DAVAO FRUITS CORP V ASSOCIATED LABOR UNIONS
[PAGE 3]

MANAGEMENT FUNCTION
BUSINESSDAY INFORMATION SYSTEMS AND
SERVICES INC V NLRC (MOYA)
221 SCRA 9
GRIO-AQUINO; April 5, 1993
NATURE
PETITION for certiorari of the decision of the National Labor Relations Commission.
FACTS
- BSSI was engaged in the manufacture and sale of computer forms. Due to financial
reverses, its creditors, the Development Bank of the Philippines (DBP) and the Asset
Privatization Trust (APT), took possession of its assets, including a manufacturing plant in
Marilao, Bulacan.
- As a retrenchment measure, some plant employees, including the private respondents,
were laid off on May 16, 1988, after
prior notice, and were paid separation pay equivalent to one-half (1/2) month pay for
every year of service. Upon receipt on her separation may, the private respondents signed
individual releases and quitclaims in favor of BSSI.
- BSSI retained some employees in an attempt to rehabilitate its business as a trading
company.
- However, barely two and a half months later, these remaining employees were likewise
discharged because the company decided to cease business operations altogether. Unlike
the private respondents, that batch of employees received separation pay equivalent to a
full month's salary for every year of service plus mid-year bonus.
- Protesting against the discrimination in the payment of their separation benefits, the
twenty-seven (27) private respondents filed complaints against the BSSI and Raul Locsin.
ISSUES
1. WON there was unlawful discrimination in the payment of separation benefits to the
employees.
2. WON the company is obliged to pay mid-year bonus.
3. WON Locsin should be held liable.
HELD

A2010

- 162 -

Disini

1. YES
- Petitioners' right to terminate employees on account of retrenchment to prevent losses or
closure of business operations, is recognized by law, but it may not pay separation
benefits unequally for such discrimination breeds resentment and ill-will among those who
have been treated less generously than others.
- The respondents cited financial business difficulties to justify their termination of the
complainants' employment. They were given one-half (1/2) month of their salary for every
year of service. Due to continuing looms, they closed operations where they dismissed the
second batch of employees who were given one (1) month pay for every year they served.
The third batch of employees were terminated and were likewise given one (1) monthly
pay for every year of service. The business climate when the complainants were
terminated did not at all defer improvement-wise. The interval between the dates of
termination was so close to each other, so that, no improvement in business maybe likely
expected.
- The law requires the granting of the same amount of separation benefits to the affected
employees in any of the cases. The respondent argued that the giving of more separation
benefit to the second and third batches of employees separated was their expression of
gratitude and benevolence to the remaining employees who have tried to save and make
the company viable in the remaining lays of operations. This justification is not plausible.
There are workers in the first batch who have rendered more years of service and more
efficient than those separated subsequently, yet, they did not receive the same
recognition.
- There was impermissible discrimination against the private respondents in the payment
of their separation benefits. The law requires an employer to extend equal treatment to its
employees. It may not, in the guise of exercising management prerogatives, grant greater
benefits to some and less to others. Management prerogatives are not absolute
prerogatives but are subject to legal limits, collective bargaining agreements, or general
principles of fair play and justice
2. NO
- The grant of a bonus is a prerogative, not an obligation, of the employer. The matter of
giving a bonus over and above the worker's lawful salaries and allowances is entirely
dependent on the financial capability of the employer to give it. The fact that the
company's business was no longer profitable (it was in fact moribund) plus the fact that
the private respondents did not work up to the middle of the year (they were discharged in
May 1993) were valid reasons for not granting them a mid-year bonus.
3. NO
- A corporate officer is not personally liable for the money claims of discharged corporate
employees unless he acted with evident malice and bad faith in terminating their
employment. There is no evidence in this case that Locsin acted in bad faith or with malice
in carrying out the retrenchment and eventual closure of the company, hence, he may not
be held personally and solidarily liable with the company for the satisfaction of the
judgment in favor of the retrenched employees.
Disposition The resolution of the NLRC ordering the petitioner company to pay
separation pay differentials to the private respondents is AFFIRMED. However, the award
of mid-year bonus to them is hereby deleted and set aside. Petitioner Raul Locsin is
absolved from any personal liability to the respondent employees. No costs.

ASIAN TRANSUNION CORP V CA


[PAGE 45]

NATURE BONUS WHEN DEMANDABLE


AMERICAN WIRE AND CABLE DAILY RATED
EMPLOYEES UNION V AMERICAN WIRE AND CABLE
CO INC
[PAGE 4]
LUZON STEVEDORING CORP V CIR
15 SCRA 660
BENGZON; December 31, 1965
NATURE
Appeal from judgment and order of the Court of Industrial Relations
FACTS
- The appeal is a consolidation of three actions filed for or against Luzon
Stevedoring Corporation and Luzteveco Employees Association in connection with a strike
called by the Union on January 2, 1959. The atrike came after another strike in 1958 and
which was just decided on by the said Court.

Labor Law 1
- In any case, the strike was declared illegal due to four factors cited as follows:
a. the strike was declared without prior notice
b. the reduction from fifteen to ten days Christmas bonus could not be unfair labor
practice considering the nature of the collective bargaining contracts then existing
between the parties
c. the strike, being not only illegal, was conducted in a manner not sanctioned by law,
specially the commission of
illegal acts at he picket line
d. the workers at the Pandacan Bulk Oil Terminal are bound by the provisions of the
no strike clause of the CBA.
ISSUE
WON the reduction of the bonus constituted unfair labor practice (the issue is being
limited to this as this is the only issue called to be discussed under the outline)
HELD
NO
- As a rule a bonus is an amount granted and paid to an employee for his industry and
loyalty which contributed to the success of the employers business and made possible
the realization of profits. It is an act of generosity for which the employee ought to be
thankful and grateful. From a legal point of view, a bonus is not a demandable and
enforceable obligation. It would be different if this bonus was made part of the wage,
salary, or compensation.
Reasoning
- There was no showing that the Christmas bonus was pat of the CBA as part of the salary
or compensation. Thus, the grant of this is contingent upon the profits being realized. The
reduced bonus in 1958 was a necessary consequence of a reduced profit in that year. and
there being no clear showing that the reduction was aimed to discriminate against the
Union, the finding of the CIR stands.
Disposition Ruling of the CIR is affirmed

LIBERATION STEAMSHIP CO INC V CIR


23 SCRA 1105
REYES JBL; June 27, 1968
NATURE
PETITIONS for review by certiorari of a resolution of the Court of Industrial Relations.
FACTS
- Petitions filed separately by the Liberation Steamship Co., Inc. (LISTCO) and the
National Development Company (NDC) for the review of the CIR resolution en banc of
September 2, 1965 modifying the decision of the trial Judge of May 13, 1964
- NDC, a government-owned and controlled corporation, in 1961 was the owner and
operator of the vessels M/S "Doa Alicia", "Doa Nati" and "Doa Aurora". It can be
gathered from the records that prior to April 15, 1961, said corporation decided to dispose
of these three vessels; and in the bidding that ensued, LISTCO won. The crew members
of the three vessels, through the Philippine Maritime Industrial Union (PMIU), made
representations with both the seller and the purchaser to retain them in the service of' the
vessels. And when in the final deed of sale no provision on the hiring of the complement of
the vessels was included, the crew-members declared a strike on April 15, 1961. On April
25, 1961, the dispute was certified by the President to the CIR
- April 29, 1961 - the Industrial Court ordered that the three Doa vessels mentioned in the
presidential certification had already been sold by the government to the Liberation
Steamship Company. Hence, the said company is indispensable party in this litigation,
without whom no final determination of this case can be had.
- May 3, 1961- acting upon NDC's petition, alleging that the strike was causing the
corporation an actual loss of about P15,000.00 daily, the court issued a return-to-work
order, the pertinent part of which reads:
"During the pendency of this case, the management shall refrain from dismissing any
employee or laborer, unless with the express authority of this Court.
- June 17, 1961 (Over a month arid a half after this order) - representatives of the LISTCO
posted notices around the M/S "Doha Alicia" to the effect that the officers and members of
the crew not otherwise appointed by the said new owner will be ejected. On the same day,
30 security guards and about 50 men with luggages came aboard the said vessels and
never departed therefrom until the vessel left port on June 21, 1961, only after the
remaining members of the original crew had been sent down.
- The unlicensed crew members of the three "Doa" vessels thus petitioned the Industrial
Court for an order to restrain LISTCO from carrying out its ejection threat of the officers
and/or crew members of the M/S "Doha Alicia" and of the two other "Doa" vessels upon
their delivery to the new owner.
- June 30, 1961 - restraining order was issued against NDC and LISTCO, directing the
maintenance of status quo during the pendency of the dispute.
- Petitioners' demands included
To NDC

A2010

- 163 -

Disini

> payments by NDC of a gratuity equivalent to one month salary for every year of service
from their employment up to the termination of their services on account of the sale of the
vessels to LISTCO
> payment of strike-duration pay
> commutation of accumulated vacation and sick leaves
> unpaid overtime services rendered from the dates of their employment
> gratuity and accumulated vacation and sick leaves to the officers and/or crew members
who were on leave and were required by the NDC to man the Pew cargo liners from
Japan to the Philippines.
To LISTCO
> retention as officers and/or crew members of the "Doa" vessels
> observance or continuation of the collective bargaining contract between NDC and the
union until its expiration in June, 1962,
> separation pay for any officer and/or crew members retained but separated by LISTCO
from the service within one year from the turnover of the vessels.
- May 13, 1964 - the TRIAL COURT rendered judgment:
> demand gratuity pay was denied
> entitled to accumulation of sick and vacation leaves with pay not exceeding 5 months
> claim for unpaid overtime was ruled out on the ground of prescription
> denied the demand for gratuity because gratuity is essentially voluntary and the
management cannot be compelled to give the same.
> NDC responsible for the ejection of the crew of the M/S "Doa Alicia", in view of its
failure to incorporate in the deed of sale in favor of LISTCO a provision on the retention of
the services of the complement of the vessels, in spite of the latter's requests therefor
prior to the consummation of the sale.
> NDC ordered to pay the back wages of the ejected crew up to the date of their actual
reinstatement.
> LISTCO was completely exonerated from any liability, the trial court reasoning that the
lay off of the crew of the M/S "Doa Alicia" was committed on June 21, 1961, or before
said respondent became subject to the restraining order of June 30, 1961.
- September 2, 1965 CIR, upon the MFR of NDC, modified the decision of the trial Judge
> NDC and LISTCO solidarity liable for payment of the backwages
> LISTCO equally responsible, the court en banc took into account the fact that as of April
29, 1961, it was already an indispensable party to the case. Thus, with knowledge of the
restraining order of May 3, 1961 to the "management" against unauthorized dismissal of
employees and laborers, the court held that LISTCO could not claim to have acted in good
faith when it ejected the crew of the M/S "Doa Alicia" on June 21, 1961.
> increased the allowable accumulated vacation and sick leaves with pay of the
petitioners, from 5 to 10 months because of RA1081.
> new sale of the "Doa" vessels had taken place during the pendency of the motion for
reconsideration, the case was ordered reopened, but only for the purpose of determining
the merits of the demand for gratuity pay.
- LISTCO assails
> ruling on paying, jointly and severally with the NDC, back wages to the affected officers
and crew members of the M/S "Doa Alicia", claiming
(1) that the Industrial Court was without jurisdiction over its persons, LISTCO not being a
party to the labor dispute certified to it by the President;
(2) that the restraining order of May 3, 1961 did not include this petitioner; and
(3) that it cannot legally be compelled to retain the services of the original crew of the M/S
"Doa Alicia"
- NDC raises
(1) legality of the strike staged by the crews of the three vessels and of their right to
strike-duration pay
(2) liability for such strike-duration pay and for reinstatement of the officers and crewmembers who were not reemployed after the conclusion of the Agreement of November
28, 1961
(3) jurisdiction of' the Court of Industrial Relations over the officers of the vessels
(4) legality of the ruling that the crew-members are entitled to accumulated sick and
vacation leaves with pay
(5) correctness of the order of the court en banc to reopen the case, insofar as the union's
demands for gratuity are concerned
ISSUES
1. WON CIR has jurisdiction over the case since, at that time, LISTCO is not an employer
of the petitioners
2. WON LISTCO is bound by the TRO
3. WON NDC is liable for backwages
4. WON crew-members are entitled to accumulated sick and vacation leaves with pay
5. WON crew-members are entitled to gratuity
HELD
1. YES
- It cannot be denied that when the certification was made by the President on April 25,
1961, and the Court of Industrial Relations assumed jurisdiction over the case, the three
"Doa" vessels were still owned and operated by the NDC. Understandably, the

Labor Law 1
presidential certification mentioned only the crew of the vessels and the NDC as parties to
the dispute. Although not originally named as respondent. the court, informed of the
consummation of the sale, ordered the inclusion of LISTCO as an indispensable party.
- LISTCO cannot contest the authority of the trial judge in ordering it to be impleaded in
the proceeding.
(1) this being a certified case, the CIR, in the exercise of its arbitration power, can direct
the inclusion or exclusion of parties therefrom; it is clothed with authority to issue such
order or orders as may be necessary to make effective the exercise of its jurisdiction,
which may include the bringing in of parties into the case.
(2) what confers jurisdiction on the Industrial Court is not the form or manner of
certification by the President, but the referral to said court of the industrial dispute between
the employer and the employee. Thus, the court is not deprived of jurisdiction over a case
simply because the certification of the President is erroneous. That LISTCO was not so
named in the certification would not make it any less the employer of the petitioning
employees within the contemplation of law, since by the transfer of ownership of the
vessels it actually became such employer.
2. YES
- April 29, 1961 LISTCO, as the new owner of the vessels, was included as an
indispensable party in the litigation, "without which no final determination of this case can
be had." It was, therefore, made of record that LISTCO was then already the owner and
operator of the ships, there having been no showing that the management thereof was
lodged in another; it was a party against which any appropriate order shall be binding and
enforceable. The order of the trial judge to "the management", to reinstate the strikers
under the last terms existing before the dispute arose and to refrain from dismissing any
employee or laborer, could not have been directed solely against the NDC but also to
LISTCO which had the power to admit or discharge employees.
3. YES
- there is no reason for exempting the NDC from liability for payment of the employees'
back wages. CIRs back to work order simultaneously ordering management to refrain
from dismissing laborers without the labor court's authority was already in full force, having
been issued since May 3. Yet, in its letter dated June 17, 1961 and sent to the Master of
the M/S "Doa Alicia", the General Manager of the NDC "enjoined" the officers and crew
members thereof, who were not selected by the new owner to debark. This letter, in effect,
was a defiance of the Industrial Court's injunction, just as the LISTCO's replacement
of the "Doa Alicia" crew was in disregard of the same order. This cooperation and
concordant action of both appellants, plainly contrary to the express CIR order of May 3,
justifies their being held solidarily liable for the back wages of the officers and crew of said
motor vessel.
4. YES
- RIGHT TO ACCUMULATION OF SICK AND VACATION LEAVES WITH PAY - The lower
court's recognition of the right of the employees of the NDC, admittedly a governmentowned and controlled corporation to accumulation of sick and vacation leaves with pay is
based on the provisions of Government Enterprises Counsel Circular No. 4 of March 1948
and of Sections 294-286 of the Administrative Code as amended by Republic Act No.
1081, which increased the allowable accumulated vacation and sick of government
employees to 10 months. The fact that the officers and unlicensed members of the crew of
the vessel had a collective bargaining contract that did not contain any provision on the
payment of accumulated leaves does not by itself bar the employees' resort to the Leave
Law. The rule is that the law forms part of, and into, every contract, unless clearly
excluded therefrom in those cases where such exclusion is allowed.
5. There should be a reopening of the case to determine whether such conditions
operated in the instant case
- GRANT OF GRATUITY; NORMALLY DISCRETIONARY BUT MAY BECOME PART OF
COMPENSATION.- While normally discretionary, the grant of a gratuity or bonus by
reason of its long and regular concession may become regarded as part of regular
compensation. (Phil. Education Co., Inc., vs. C.I.R., 92 Phil., 382, 385). For this reason,
where there is a resale of the vessels to another party during the pendency of the motion
for reconsideration, the court may order the reopening of the case insofar as the demands
for gratuity are concerned, in order to determine whether aforecited conditions operated in
the instant case.
Disposition resolution appealed from is hereby affirmed

MARCOS V NLRC (INSULAR LIFE ASSURANCE CO


LTD)
248 SCRA 146
REGALADO; September 8, 1995
NATURE
Petition for certiorari
FACTS
- Petitioners were regular employees of private respondent Insular Life Assurance Co.,
Ltd., but they were dismissed when their positions were declared redundant. A special

A2010

- 164 -

Disini

redundancy benefit was paid to them. However, not included in this redundancy benefit
were their respective service awards and other prorated bonuses which they had earned
at the time they were dismissed. Because of this, petitioners questioned the redundancy
package. Nevertheless, they signed a Release and Quitclaim but with a written protest
reiterating their previous demand that they were nonetheless entitled to receive their
service awards.
- Petitioners inquired from the Legal Service of the Department of Labor and Employment
whether respondent corporation could legally refuse the payment of their service awards
as mandated in their Employee's Manual.
- DOLE ruled in their favor. However, this decision was overturned by the NLRC affirming
the validity of the Release and Quitclaim which consequently bar the petitioners to
demand for service awards and other bonuses. Thus, this petition.
ISSUE
WON respondent NLRC committed reversible error or grave abuse of discretion in
affirming the validity of the "Release and Quitclaim" and, consequently, that petitioners are
not entitled to payment of service awards and other bonuses

HELD
YES
Ratio On Release and Quitclaim - The fact that an employee has signed a satisfaction
receipt for his claims does not necessarily result in the waiver thereof. The law does not
consider as valid any agreement whereby a worker agrees to receive less compensation
than what he is entitled to recover. A deed of release or quitclaim cannot bar an employee
from demanding benefits to which he is legally entitled. Renuntiatio non praesumitur.
While there may be possible exceptions to this holding, we do not perceive any in the case
at bar.
Reasoning
a. The element of total voluntariness in executing that instrument is negated by the fact
that they expressly stated therein their claim for the service awards, a manifestation
equivalent to a protest and a disavowal of any waiver thereof.
b. Petitioners even sought the opinion of the Department of Labor and Employment to
determine where and how they stood in the controversy. This act only shows their
adamant desire to obtain their service awards and to underscore their disagreement with
the "Release and Quitclaim" they were virtually forced to sign in order to receive their
separation pay.
c. While rights may be waived, the same must not be contrary to law, public order, public
policy, morals or good customs or prejudicial to a third person with a right recognized by
law.
- Article 6 of the Civil Code renders a quitclaim agreement void ab initio where the
quitclaim obligates the workers concerned to forego their benefits while at the same time
exempting the employer from any liability that it may choose to reject. This runs counter to
Art. 22 of the Civil Code which provides that no one shall be unjustly enriched at the
expense of another.
Ratio On Service Awards and other Bonuses - The petitioners are entitled to receive
service awards and other bonuses. The contention of the respondent that service award is
a bonus and therefore is an act of gratuity which the complainants have no right to
demand and service awards are governed by respondent's employee's manual and (are)
therefore contractual in nature is not impressive.
Reasoning
a. Anniversary and performance bonuses have ripened into a company practice
therefore become demandable. It is not disputed that it is respondent's practice to give an
anniversary bonus every five years from its incorporation. The prerogative of the
employer to determine who among its employees shall be entitled to receive bonuses
which are, as a matter of practice, given periodically cannot be exercised arbitrarily.
b. Pursuant to their policies on the matter, the service award differential is given at the
end of the year to an employee who has completed years of service divisible by 5.
c. A bonus is not a gift or gratuity, but is paid for some services or consideration and is in
addition to what would ordinarily be given. The term "bonus" as used in employment
contracts, also conveys an idea of something which is gratuitous, or which may be claimed
to be gratuitous, over and above the prescribed wage which the employer agrees to pay.
- If one enters into a contract of employment under an agreement that he shall be paid
a certain salary by the week or some other stated period and, in addition, a bonus, in case
he serves for a specified length of time, there is no reason for refusing to enforce the
promise to pay the bonus, if the employee has served during the stipulated time, on the
ground that it was a promise of a mere gratuity.
Disposition The assailed decision and resolution of respondent National Labor Relations
Commissions are hereby SET ASIDE and the decision of Labor Arbiter Alex Arcadio Lopez
is REINSTATED.

PHILIPPINE NATIONAL CONSTRUCTION CORP V


NLRC (ANGELES, PABLO, JR)

Labor Law 1

A2010

307 SCRA 218


18 May 1999
NATURE
Petition for certiorari of a decision of NLRC.
FACTS
- ANGELES and PABLO, JR. [COMPLAINANTS, for brevity] were employed by PNCC as
tollway guards. Acting on a private complaint regarding mulcting activities of some of its
tollway personnel, PNCC created an investigating team. During its investigation, said team
saw COMPLAINANTS accept cash and a dog from a motorist.
- After due investigation, COMPLAINANTS were dismissed by PNCC for serious
misconduct. When the COMPLAINANTS complaint for illegal dismissal reached NLRC,
the latter held that COMPLAINANTS act of receiving a sum of money and a dog from
motorists constituted bribery which was a sufficient ground for their dismissal. NLRC,
nonetheless, ordered PNCC to pay COMPLAINANTS their mid-year bonus for 1994,
among others. Hence, the present petition.
ISSUE
WON COMPLAINANTS entitled to the disputed mid-year bonus

HELD
NO
Ratio A bonus is a gift from the employer and the grant thereof is a management
prerogative. A bonus becomes a demandable or enforceable obligation only when it is
made part of the compensation of the employee. Whether a bonus forms part of wages
depends upon the circumstances for its payment. If it is additional compensation which
the employer promised and agreed to give without any conditions imposed for its payment,
such as success of business or greater production or output, then it is part of the wage.
But if it is paid only if profits are realized or if a certain level of productivity is achieved, it
cannot be considered part of the wage. Where it is payable only to some employees
and only when their labor becomes more efficient or more productive, it is only an
inducement for efficiency, a prize therefor, not a part of the wage [citing Metro Transit vs
NLRC, 245 SCRA 767 (1995)].
YEAR

MID- YEAR BONUS

CHRISTMAS BONUS

13TH MO. PAY

previous
years

one mo. basic

one mo. basic

one mo. Basic

1984

[one mo. basic]

-none-

one-half mo. Basic

1985

one-half mo. basic

-none-

one-half mo. Basic

1986

one-half mo. basic

one-half mo. basic

one mo. Basic

1987

one-half mo. basic

one-half mo. basic

one mo. basic

- COMPLAINANTS neither alleged nor adduced evidence to show that the bonus they are
claiming is a regular benefit which has become part of their compensation. Thus, the
presumption is that it is not a demandable obligation from the employer and the latter may
not be compelled to grant the same to undeserving employees.
Disposition NLRC decision set aside.

PRODUCERS BANK OF THE PHILIPPINES V NLRC


(PRODUCERS BANK EMPLOYEES ASSN)
355 SCRA 489
GONZAGA-REYES; March 28, 2001
NATURE
A special civil action for certiorari with prayer for preliminary injunction and/or restraining
order seeking the nullification of the decision of NLRC
FACTS
- The present petition originated from a complaint filed by private respondent with the
Arbitration Branch, National Capital Region, National Labor Relations Commission

- 165 -

Disini

(NLRC), charging petitioner with diminution of benefits, non-compliance with Wage Order
No. 6 and non-payment of holiday pay. In addition, private respondent prayed for
damages.
- Labor Arbiter Nieves found private respondent's claims to be unmeritorious and
dismissed its complaint.
- NLRC4 granted all of private respondent's claims, except for damages ordering
respondent- appellee to pay complainant-appellant:
1. The unpaid bonus (mid-year and Christmas bonus) and 13th month pay;
2. Wage differentials under Wage Order No. 6 for November 1, 1984 and the
corresponding adjustment thereof; and
3. Holiday pay under Article 94 of the Labor Code, but not to exceed three (3) years.
- Petitioner now contends that the NLRC gravely abused its discretion in ruling as it did for
the succeeding reasons stated in its Petition
ISSUES
1. WON petitioner should pay the unpaid bonus
2. WON petitioner should pay the 13th month pay
3. WON petitioner complied with Wage Order No.6
4. WON petitioner complied with Art.94 of the Labor Code on holiday pay

HELD
1. NO
Ratio A bonus is an amount granted and paid to an employee for his industry and loyalty
which contributed to the success of the employer's business and made possible the
realization of profits. It is an act of generosity granted by an enlightened employer to spur
the employee to greater efforts for the success of the business and realization of bigger
profits. The granting of a bonus is a management prerogative, something given in addition
to what is ordinarily received by or strictly due the recipient.13 Thus, a bonus is not a
demandable and enforceable obligation, except when it is made part of the wage, salary
or compensation of the employee.
- However, an employer cannot be forced to distribute bonuses which it can no longer
afford to pay. To hold otherwise would be to penalize the employer for his past generosity.
Reasoning
- private respondent declared in its position papers filed with the NLRC that Producers
Bank of the Philippines has been providing several benefits to its employees since 1971
when it started its operation. Among the benefits it had been regularly giving is a mid-year
bonus equivalent to an employee's one-month basic pay and a Christmas bonus
equivalent to an employee's one whole month salary (basic pay plus allowance). However,
it has changed this practice. In a tabular form, here are the bank's violations:
- Private respondent argues that the mid-year and Christmas bonuses, by reason of their
having been given for thirteen consecutive years, have ripened into a vested right and, as
such, can no longer be unilaterally withdrawn by petitioner without violating Art.100 of PD
No. 4429 which prohibits the diminution or elimination of benefits already being enjoyed by
the employees.
- Petitioner was not only experiencing a decline in its profits, but was reeling from
tremendous losses triggered by a bank-run which began in 1983. In such a depressed
financial condition, petitioner cannot be legally compelled to continue paying the same
amount of bonuses to its employees. Thus, the conservator was justified in reducing the
mid-year and Christmas bonuses of petitioner's employees. To hold otherwise would be to
defeat the reason for the conservatorship which is to preserve the assets and restore the
viability of the financially precarious bank.
2. NO
Ratio The intention of the law was to grant some relief - not to all workers - but only to
those not actually paid a 13th month salary or what amounts to it, by whatever name
called. It was not envisioned that a double burden would be imposed on the employer
already paying his employees a 13th month pay or its equivalent whether out of pure
generosity or on the basis of a binding agreement. To impose upon an employer already
giving his employees the equivalent of a 13th month pay would be to penalize him for his
liberality and in all probability, the employer would react by withdrawing the bonuses or
resist further voluntary grants for fear that if and when a law is passed giving the same
benefits, his prior concessions might not be given due credit.
Reasoning
- Petitioner argues that it is not covered by PD 851 since the mid-year and Christmas
bonuses it has been giving its employees from 1984 to 1988 exceeds the basic salary for
one month (except for 1985 where a total of one month basic salary was given). Hence,
this amount should be applied towards the satisfaction of the 13th month pay, pursuant to
Section 2 of PD 851. PD 851, which was issued by President Marcos on 16 December
1975, requires all employers to pay their employees receiving a basic salary of not more
than P 1,000 a month, regardless of the nature of the employment, a 13th month pay, not
later than December 24 of every year.30 However, employers already paying their
employees a 13th month pay or its equivalent are not covered by the law.

Labor Law 1
- It is noted that, for each and every year involved, the total amount given by petitioner
would still exceed, or at least be equal to, one month basic salary and thus, may be
considered as an "equivalent" of the 13th month pay mandated by PD 851.
Thus, petitioner is justified in crediting the mid-year bonus and Christmas bonus as part of
the 13th month pay.
3. YES
Ratio The creditability provision in Wage Order No. 6 is based on important public policy,
that is, the encouragement of employers to grant wage and allowance increases to their
employees higher than the minimum rates of increases prescribed by statute or
administrative regulation. To obliterate the creditability provisions in the Wage Orders
through interpretation or otherwise, and to compel employers simply to add on legislated
increases in salaries or allowances without regard to what is already being paid, would be
to penalize employers who grant their workers more than the statutorily prescribed
minimum rates of increases. Clearly, this would be counter-productive so far as securing
the interest of labor is concerned. The creditability provisions in the Wage Orders prevent
the penalizing of employers who are industry leaders and who do not wait for statutorily
prescribed increases in salary or allowances and pay their workers more than what the
law or regulations require.
Reasoning
- Wage Order No.6, which came into effect on 1 November 1984, increased the statutory
minimum wage of workers, with different increases being specified for agricultural
plantation and non-agricultural workers. The bone of contention, however, involves
Section 4 thereof33
- On 16 November 1984, the parties entered into a CBA providing for the following salary
adjustments34
- Petitioner argues that it complied with Wage Order No. 6 because the first year salary
and allowance increase provided for under the collective bargaining agreement can be
credited against the wage and allowance increase mandated by such wage order.
- It would be inconsistent with the above stated rationale underlying the creditability
provision of Wage Order No. 6 if, after applying the first year increase to Wage Order No.
5, the balance was not made chargeable to the increases under Wage Order No. 6 for the
fact remains that petitioner actually granted wage and allowance increases sufficient to
cover the increases mandated by Wage Order No. 5 and part of the increases mandated
by Wage Order No. 6.
4. YES
Ratio We agree with the labor arbiter that the reduction of the divisor to 303 was done for
the sole purpose of increasing the employees' overtime pay, and was not meant to
exclude holiday pay from the monthly salary of petitioner's employees. In fact, it was
expressly stated in the inter-office memorandum that the divisor of 314 will still be used in
the computation for cash conversion and in the determination of the daily rate. Thus,
based on the records of this case and the parties' own admissions, the Court holds that
petitioner has complied with the requirements of Article 94 of the Labor Code
Reasoning
- Article 94 of the Labor Code provides that every worker shall be paid his regular daily
wage during regular holidays and that the employer may require an employee to work on
any holiday but such employee shall be paid a compensation equivalent to twice his
regular rate.
- In this case, the Labor Arbiter found that the divisor used by petitioner in arriving at the
employees' daily rate for the purpose of computing salary-related benefits is 314.
However, the divisor was reduced to 303 by virtue of an inter-office memo.
- the acting Conservator approved the use of 303 days as divisor in the computation of
Overtime pay.
- Corollarily, the Acting Conservator also approved the increase of meal allowance from
P25.00 to P30.00 for a minimum of four (4) hours of work for Saturdays.
- the Labor Arbiter observed that the reduction of the divisor to 303 was for the sole
purpose of increasing the employees' overtime pay and was not meant to replace the use
of 314 as the divisor in the computation of the daily rate for salary-related benefits.
As to private respondent's claim for damages, the NLRC was correct in ruling that there is
no basis to support the same.
Disposition Decision of public respondent is SET ASIDE, with the exception of public
respondent's ruling on damages.

33

All wage increase in wage and/or allowance granted by employers between June 17, 1984 and the effectivity of this
Order shall be credited as compliance with the minimum wage and allowance adjustments prescribed herein, provided
that where the increases are less than the applicable amount provided in this Order, the employer shall pay the
difference. Such increases shall not include anniversary wage increases provided in collective bargaining agreements
unless the agreement expressly provide otherwise.

34

Article VIII. Section 1. Salary Adjustments. ...(i) Effective March 1, 1984 - P225.00 per month as salary increase plus
P100.00 per month as increase in allowance to employees within the bargaining unit on March 1, 1984.
(ii) Effective March 1,1985 -P125.00 per month as salary increase plus P100.00 per month as increase in allowance to
employees within the bargaining unit on March 1,1985.
(iii) Effective March 1,1986 -P125.00 per month as salary increase plus P100.00 per month as increase in allowance to
employees within the bargaining unit on March 1, 1986.
- In addition, the collective bargaining agreement of the parties also included a provision on the chargeability of such
salary or allowance increases against government-ordered or legislated income adjustments

A2010

- 166 -

Disini

PHIL DUPLICATORS INC V NLRC


[PAGE 161]
MANILA ELECTRIC CO V QUISUMBING
[PAGE 19]
PHILIPPINE APPLIANCE CORPORATION (PHILACOR)
V CA (PHILACOR WORKERS UNION)
430 SCRA 525
YNARES-SANTIAGO; June 3, 2004
NATURE
Appeal by Certiorari to set aside CA decision denying petitioners partial appeal as well as
CA resolution denying the MFR.
FACTS
- Petitioner is a domestic corp. engaged in manufacturing refrigerators, freezers, and
washing machines. Respondent United Philacor Workers Union NAFLU is the duly
elected collective bargaining representative of the rank and file employees of petitioner.
- During one collective bargaining negotiation, petitioner offered P4000 to each employee
as an early conclusion bonus, or a unilateral incentive for the speeding up of negotiations
between the parties and to encourage respondent union to exert their best efforts to
conclude a CBA. Upon conclusion of the CBA negotiations, petitioner accordingly gave
this early signing bonus.
- After this CBA expired in Aug.1999, the 2 parties began negotiations for a new CBA but
after 11 meetings, respondent union declared a deadlock and a few days later filed a
notice of strike. A conciliation and mediation conference was held but it still left the ff.
issues unresolved: wages, rice subsidy, signing and retroactive bonus. Failure to come to
an agreement led respondent union to go on an 11-day strike which resulted in stoppage
of manufacturing operations as well as losses for petitioner. This constrained petitioner to
file a petition before the DOLE and the Labor Secretary Laguesma resolved the dispute by
issuing an order which, among others, granted a signing bonus of P3,000 to the union.
- Petitioner filed a MFR, stating that it accepted the decision but took exception to the
award of the signing bonus, claiming that it is not demandable or enforceable since it is in
the nature of an incentive. Labor Sec. denied this motion. Petitioner then filed for Certiorari
with the CA which was dealt with similarly. The Labor Secs award of signing bonus was
affirmed since petitioner itself offered the same incentive to expedite the CBA negotiations,
which they did not withdraw and was still outstanding when the dispute reached the
DOLE. Petitioner filed a MFR which was again denied, leading to this petition.
ISSUE
WON the signing bonus awarded by the Labor Secretary (and affirmed by respondent CA)
was proper
HELD
NO
Ratio A signing bonus may not be demanded as a matter of right if it is not agreed upon
by the parties or unilaterally offered as an additional incentive. It is not a demandable and
enforceable obligation. The condition for awarding it must be duly satisfied.
Reasoning
- 2 things militate against the grant of the signing bonus: first, the non-fulfillment of the
condition for which it was offered, i.e., the speedy and amicable conclusion of the CBA
negotiations; and second, the failure of respondent union to prove that the grant of the
said bonus is a long established tradition or a regular practice on the part of petitioner.
Petitioner admits, and respondent union does not dispute, that it offered an early
conclusion bonus or an incentive for a swift finish to the CBA negotiations.
- A signing bonus is justified by and is the consideration paid for the goodwill that existed
in the negotiations that culminated in the signing of a CBA. In the case at bar, the CBA
negotiation between petitioner and respondent union failed. Respondent union went on
strike for eleven days and blocked the ingress to and egress from petitioners work plants.
The labor dispute had to be referred to the Secretary of Labor and Employment because
neither of the parties was willing to compromise their respective positions regarding the
four remaining items which stood unresolved. While we do not fault any one party for the
failure of the negotiations, it is apparent that there was no more goodwill between the
parties and that the CBA was clearly not signed through their mutual efforts alone. Hence,
the payment of the signing bonus is no longer justified and to order such payment would
be unfair and unreasonable for petitioner.
- We have consistently ruled that although a bonus is not a demandable and enforceable
obligation, it may nevertheless be granted on equitable considerations as when the giving
of such bonus has been the companys long and regular practice. To be considered a
regular practice, however, the giving of the bonus should have been done over a long
period of time, and must be shown to have been consistent and deliberate. The test or

Labor Law 1
rationale of this rule on long practice requires an indubitable showing that the employer
agreed to continue giving the benefits knowing fully well that said employees are not
covered by the law requiring payment thereof. Respondent does not contest the fact that
petitioner initially offered a signing bonus only during the previous CBA negotiation.
Previous to that, there is no evidence on record that petitioner ever offered the same or
that the parties included a signing bonus among the items to be resolved in the CBA
negotiation. Hence, the giving of such bonus cannot be deemed as an established
practice considering that the same was given only once.
Disposition petition is GRANTED. CA decision affirming the Order of the Secretary of
Labor and Employment is REVERSED and SET ASIDE.

9.15 PRODUCTIVITY INCENTIVES ACT OF 1990


RA 6971
E. WAGE RECOVERY, LIABILITIES, AND
WORKER PREFERENCE
EMPLOYER, INDEPENDENT CONTRACTOR AND SUBCONTRACTOR AND LABORONLY CONTRACTING

A2010

- 167 -

MANILA ELECTRIC CO V BENAMIRA


[PAGE 62]

9.17 WORKER PREFERENCE BANKRUPTCY


CIVIL CODE LABOR CODE
REPUBLIC V PERALTA
150 SCRA 37
FELICIANO; May 20, 1987

SENTINEL SECURITY AGENCY INC V NLRC


[PAGE 140]
LAPANDAY AGRICULTURA DEVT CORP V CA
[PAGE 7]
OSM SHIPPING V NLRC (GUERRERO)
398 SCRA 606
PANGANIBAN; March 5, 2003
FACTS
- Fermin Guerrero was hired by OSM for and in behalf of its principal, Phil Carrier Shipping
Agency Services Co. (PCSASCO) as a Master Mariner of M/V Princess Hoa for a contract
period of 10 months. Guerrero alleged that for almost 7 months (from start of his work in
July 1994 until Jan 1995), despite the services he rendered, no compensation or
remuneration was ever paid to him. He was forced to disembark because he cannot even
buy his basic personal necessities (wawa naman!). He filed for illegal dismissal and nonpayment of wages, etc.
- OSM: Philippine Carrier Shipping Lines Co. (PCSLC) is the disponent owner/employer,
and PCSLC is now responsible for the payment of complainant's wages (Because
Concorde Pacific, an American company w/c owns M/V Princess Hoa, decided to use ship
in the coastwise trade. Since the M/V Princess Hoa was a foreign registered vessel and
could not be used in the coastwise trade, the shipowner converted the vessel to Philippine
registry on Sept 28, 1994 by way of bareboat chartering it out to another entity named
Philippine Carrier Shipping Lines Co. [PCSLC]. To do this, the shipowner had to terminate
its management agreement with PCSASCO on Sept 28, 1994 by a letter of termination.
Consequently, PCSASCO terminated its crew agreement with OSM in a letter dated Dec
5, 1994. Because of the bareboat charter of the vessel to PCSLC and its subsequent
conversion to Philippine registry and use in coastwise trade as well as to the termination of
the management agreement and crew agency agreement, a termination of contract
ensued whereby PCSLC, the bareboat charterer, became the disponent owner/employer
of the crew.)
- NLRC: OSM Shipping Phils. Inc. and its principal, PCSASCO are jointly and severally
ordered to pay complainant
ISSUE
WON OSM is liable for the payment of unpaid salary of Guerrero
HELD
YES
- Petitioner, as manning agent, is jointly and severally liable with its principal, PCSASCO,
for private respondent's claim. This conclusion is in accordance with Section 1 of Rule II
of the POEA Rules and Regulations35.
35

SEC. 1. Requirements for Issuance of License. Every applicant for license to operate a private employment agency
or manning agency shall submit a written application together with the following requirements:
xxx
xxx
f. A verified undertaking stating that the applicant:

Disini

- Joint and solidary liability is meant to assure aggrieved workers of immediate and
sufficient payment of what is due them. The fact that petitioner and its principal have
already terminated their agency agreement does not relieve the former of its liability. The
reason for this ruling was given by this Court in Catan National Labor Relations
Commission, which we reproduce in part as follows:
"This must be so, because the obligations covenanted in the [manning] agreement
between the local agent and its foreign principal are not coterminus with the term of
such agreement so that if either or both of the parties decide to end the agreement, the
responsibilities of such parties towards the contracted employees under the agreement
do not at all end, but the same extends up to and until the expiration of the,
employment contracts of the employees recruited and employed pursuant to the said
recruitment agreement. Otherwise, this will render nugatory the very purpose for which
the law governing the employment of workers for foreign jobs abroad was enacted."
Disposition NLRC Decision REINSTATED and AFFIRMED

NATURE:
Review on certiorari
FACTS:
- The Republic of the Philippines seeks the review on certiorari of the Order of the CFI of
Manila in its Civil Case No. 108395 entitled "In the Matter of Voluntary Insolvency of
Quality Tobacco Corporation, Quality Tobacco.
- In its questioned Order, the trial court held that the above-enumerated claims of USTC
and FOITAF (hereafter collectively referred to as the "Unions") for separation pay of their
respective members embodied in final awards of the NLRC were to be preferred over the
claims of the Bureau of Customs and the BIR. The trial court, in so ruling, relied primarily
upon Article 110 of the Labor Code.
- The Solicitor General, in seeking the reversal of the questioned Orders, argues that
Article 110 of the Labor Code is not applicable as it speaks of "wages," a term which he
asserts does not include the separation pay claimed by the Unions. "Separation pay," the
Solicitor General contends: is given to a laborer for a separation from employment
computed on the basis of the number of years the laborer was employed by the employer;
it is a form of penalty or damage against the employer in favor of the employee for the
latter's dismissal or separation from service
ISSUE
WON separation pay of their respective members embodied in final awards of the NLRC
were to be preferred over the claims of the Bureau of Customs and the BIR (WON
separation pay is included in the term wages36)
HELD
1. YES

xxx
xxx
xxx
(3)
Shall assume joint and solidary liability with the employer for all claims and liabilities which may arise in
connection with the implementation of the contract; including but not limited to payment of wages, health and disability
compensation and reparation.

36

Article 97 (f) of the Labor Code defines "wages" in the following terms:
Wage' paid to any employee shall mean the remuneration or earnings, however designated, capable of
being expressed in terms of money, whether fixed or ascertained on a time, task, piece, or commission
basis, or other method of calculating the same, which is payable by an employer to an employee under a
written or unwritten contract of employment for work done or to be done, or for services rendered or to be
rendered, and includes the fair and reasonable value, as determined by the Secretary of Labor, of board,
lodging, or other facilities customarily furnished by the employer to the employee. 'Fair and reasonable
value' shall not include any profit to the employer or to any person affiliated with the employer.(emphasis
supplied)

Labor Law 1
Ratio For the specific purposes of Article 110 37 and in the context of insolvency
termination or separation pay is reasonably regarded as forming part of the remuneration
or other money benefits accruing to employees or workers by reason of their having
previously rendered services to their employer; as such, they fall within the scope of
"remuneration or earnings for services rendered or to be rendered ." Liability for
separation pay might indeed have the effect of a penalty, so far as the employer is
concerned. So far as concerns the employees, however, separation pay is additional
remuneration to which they become entitled because, having previously rendered
services, they are separated from the employer's service.
Reasoning
- We note, in this connection, that in Philippine Commercial and Industrial Bank (PCIB) us.
National Mines and Allied Workers Union, the Solicitor General took a different view and
there urged that the term "wages" under Article 110 of the Labor Code may be regarded
as embracing within its scope severance pay or termination or separation pay. In PCIB,
this Court agreed with the position advanced by the Solicitor General. We see no reason
for overturning this particular position.
- The resolution of the issue of priority among the several claims filed in the insolvency
proceedings instituted by the Insolvent cannot, however, rest on a reading of Article 110 of
the labor Code alone.
- Article 110 of the Labor Code, in determining the reach of its terms, cannot be viewed in
isolation. Rather, Article 110 must be read in relation to the provisions of the Civil Code
concerning the classification, concurrence and preference of credits, which provisions find
particular application in insolvency proceedings where the claims of all creditors, preferred
or non-preferred, may be adjudicated in a binding manner.
Disposition MODIFIED and REMANDED to the trial court for further proceedings in
insolvency.

PHILIPPINE EXPORT V CA (DIEHL)


251 SCRA 354
VITUG; December 12, 1995
NATURE
Petition for review on certiorari
FACTS
- On 13 May 1988, private respondent Raimund Diehl, a resident alien, lodged a complaint
for illegal dismissal against the Philippine German Wire Mesh Reinforcing Corporation
("FILFORCE") with the National Labor Relations Commission ("NLRC"). Parenthetically,
five (5) years earlier, or on 28 July 1983, FILFORCE had mortgaged its plant and other
property located at EPZA, Mariveles, Bataan, in favor of herein petitioner Philippine Export
and Foreign Loan Guarantee Corporation ("PHILGUARANTEE"), a government owned
and controlled corporation, to secure a guarantee which the latter executed in favor of
Kuwait Asia Bank, E.C., over fifty one percent (51%) of the US$1,357,600.00 loan which
had been extended to FILFORCE by the bank.
- The Labor Arbiter rendered a judgment favorable to Diehl. Since no appeal was filed, the
decision became final and the Labor Arbiter issued a writ of execution directing NLRC
Sheriff to execute the judgment against FILFORCE and Basilio Sison. Failing to collect the
sum due, the Sheriff was directed to cause the satisfaction of the award by levying on the
property of FILFORCE. The Deputy Sheriff effected the levy and scheduled a public
auction sale.
- Since the assets had previously been mortgaged to it, PHILGUARANTEE filed a thirdparty claim which resulted in the suspension of the scheduled auction sale. Upon the
submission by Diehl of an indemnity bond issued by Plaridel Surety and Insurance
Company, with a face value of P1,320,772.11, the Deputy Sheriff issued a notice resetting
the auction sale. PHILGUARANTEE promptly filed a petition/manifestation before the
Labor Arbiter questioning, among other things, the integrity of the indemnity bond posted
by Diehl and, at the same time, asserting its superior right and prior lien over the levied
property. Deputy Sheriff proceeded, nonetheless, with the auction sale at which Diehl was
declared the sole and winning bidder.
- PHILGUARANTEE went to the Regional Trial Court of Makati and there filed a complaint
for "Annulment of Sale, Recovery of Possession and Injunction with Urgent Prayer for the
Issuance of a Writ of Preliminary Injunction and/or Temporary Restraining Order and/or
Status Quo Order".
ISSUE
WON CA can issue a preliminary injunction
HELD
37

Article 110. Worker preference in case of bankruptcy In the event of bankruptcy or liquidation of an employer's
business, his workers shall enjoy first preference as regards wages due them for services rendered during the period
prior to the bankruptcy or liquidation, any provision of law to the contrary notwithstanding. Union paid wages shall be
paid in full before other creditors may establish any claim to a share in the assets of the employer. (emphasis supplied).

A2010

- 168 -

Disini

- The appellate court did not commit error. The question of whether or not the trial court
below was in any good position to take cognizance over the complaint filed by
PHILGUARANTEE and to issue an injunctive relief depended, in turn, on whether or not
the acts complained of arose out of, or were connected or interwoven with, cases falling
under the exclusive jurisdiction of the Labor Arbiter or the NLRC. While, ostensibly, the
complaint filed with the trial court was for the annulment of sale, recovery of possession
and injunction, in essence, however, the action challenged the legal propriety of the
execution sale, as well as the acts performed by the Labor Arbiter and the Deputy Sheriff
in the conduct thereof, and the subsequent issuance of an alias writ of execution. In
reality, petitioner's action to annul the execution sale was a motion to quash the writ of
execution on a case aptly within the jurisdiction of the Labor Arbiter. The case brought
before the trial court, being a matter growing out of the labor dispute decided by the Labor
Arbiter, clearly fell outside the competence of the trial court.
- Another reason that militates against the trial court's assumption of jurisdiction over the
case is Article 254 of the Labor Code which states:
Art. 254. Injunction prohibited. No temporary or permanent injunction or restraining
order in any case involving or growing out of labor disputes shall be issued by any
court or other entity,
The Court, however, cannot end its ponencia on this simple case without calling attention
to serious lapses in the proceedings before the Labor Arbiter concerning the third party
claim of PHILGUARANTEE. Section 2, Rule VI, of the Manual of Instructions for Sheriffs
of the NLRC prescribes in detail the procedure that must be followed in the event that the
property levied upon to satisfy a final judgment is claimed by any person other than the
losing party, viz.:
Sec. 2. Proceedings. If property levied upon be claimed by any person other than
the losing party or his agent, such person shall make an affidavit of his title thereto or
right to the possession thereof, stating the grounds of such right or title and shall file
the same with the sheriff and copies thereof served upon the Labor Arbiter or proper
officer issuing the writ and upon the prevailing party. Upon receipt of the third party
claim, all proceedings with respect to the execution of the property subject of the third
party claim shall automatically be suspended and the Labor Arbiter or proper officer
issuing the writ shall conduct a hearing with due notice to all parties concerned and
resolve the validity of the claim within ten (10) working days from receipt thereof and
his decision is appealable to the Commission within ten (10) working days from notice,
and the Commission shall likewise resolve the appeal within the same period.
However, should the prevailing party put up an indemnity bond in a sum not less than
the value of the property levied, the execution shall proceed. In case of disagreement
as to such value, the same shall be determined by the Labor Arbiter, National Labor
Relations Commission or the Philippine Overseas Employment Administration issuing
the writ, as the case may be.
Evidently, the Court's exhortation in Guimoc v. Rosales, i.e., that "(i)n executing an order,
resolution, or decision of the NLRC, the sheriff of the Commission, or other officer acting
as such, must be guided strictly by the Sheriff's Manual . . .," was not properly heeded.
- We could consider the following:
1. The Manual requires that the indemnity bond that must be posted up by the
prevailing party should be in a sum not less than the value of the property levied. Here,
Diehl has put up a bond of only P1,320,772.11; the appraised value, however, totals
P4,934,000.00.
2. The Manual provides that in case of disagreement on the value of the property
levied, the matter shall be determined by the Labor Arbiter. Not only did
PHILGUARANTEE promptly challenge the integrity of the bond submitted by Diehl but
it also did question the amount of the bond. Since the difference is substantial, it should
have behooved the Labor Arbiter to take more than just a passing glance on the claim
of PHILGUARANTEE.
- A final observation. On 21 March 1989, Article 110 of the Labor Code was amended by
Republic Act No. 6715 so as to read:
Art. 110. Worker preference in case of bankruptcy. In the event of bankruptcy or
liquidation of an employer's business, his workers shall enjoy first preference as
regards their wages and other monetary claims, any provisions of law to the contrary
notwithstanding. Such unpaid wages and monetary claims shall be paid in full before
claims of the Government and other creditors may be paid.
- In Development Bank of the Philippines vs. National Labor Relations Commission (183
SCRA 328, 336-339), the Court has said:
The amendment expands worker preference to cover not only unpaid wages but also
other monetary claims to which even claims of the Government must be deemed
subordinate.
xxx xxx xxx
Notably, the terms "declaration" of bankruptcy or "judicial" liquidation have been
eliminated. Does this mean then that liquidation proceedings have been done away
with?
We opine in the negative, upon the following considerations:
1. Because of its impact on the entire system of credit, Article 110 of the Labor Code
cannot be viewed in isolation but must be read in relation to the Civil Code scheme on
classification and preference of credits.
xxx xxx xxx

Labor Law 1
2. In the same way that the Civil Code provisions on classification of credits and the
Insolvency Law have been brought into harmony, so also must the kindred provisions
of the Labor Law be made to harmonize with those laws.
3. In the event of insolvency, a principal objective should be to effect an equitable
distribution of the insolvent's property among his creditors. To accomplish this there
must first be some proceeding where notice to all of the insolvent's creditors may be
given and where the claims of preferred creditors may be bindingly adjudicated (De
Barretto vs. Villanueva, No. L-14938, December 29, 1962, 6 SCRA 928). The rationale
therefore has been expressed in the recent case of DBP vs. Secretary of Labor (G.R.
No. 79351, 28 November 1989), which we quote:
xxx xxx xxx
4. A distinction should be made between a preference of credit and a lien. A preference
applies only to claims which do not attach to specific properties. A lien creates a charge
on a particular property. The right of first preference as regards unpaid wages
recognized by Article 110 does not constitute a lien on the property of the insolvent
debtor in favor of workers. It is but a preference of credit in their favor, a preference in
application. It is a method adopted to determine and specify the order in which credits
should be paid in the final distribution of the proceeds of the insolvent's assets. It is a
right to a first preference in the discharge of the funds of the judgment debtor.
xxx xxx xxx
6. Even if Article 110 and its implementing Rule, as amended, should be interpreted to
mean "absolute preference," the same should be given only prospective effect in line
with the cardinal rule that laws shall have no retroactive effect, unless the contrary is
provided (Article 4, Civil Code). Thereby, any infringement on the constitutional
guarantee on non-impairment of the obligation of contracts (Section 10, Article III, 1987
Constitution) is also avoided. In point of fact, DBP's mortgage credit antedated by
several years the amendatory law, RA No. 6715. To give Article 110 retroactive effect
would be to wipe out the mortgage in DBP's favor and expose it to a risk which it
sought to protect itself against by requiring a collateral in the form of real property.
In fine, the right to preference given to workers under Article 110 of the Labor Code
cannot exist in any effective way prior to the time of its presentation in distribution
proceedings. It will find application when, in proceedings such as insolvency, such
unpaid wages shall be paid in full before the "claims of the Government and other
creditors" may be paid. But, for an orderly settlement of a debtor's assets, all creditors
must be convened, their claims ascertained and inventoried, and thereafter the
preferences determined in the course of judicial proceedings which have for their object
the subjection of the property of the debtor to the payment of his debts or other lawful
obligations. Thereby, an orderly determination of preference of creditors' claims is
assured (Philippine Savings Bank vs. Lantin, No. L-33929, September 2, 1983, 124
SCRA 476); the adjudication made will be binding on all parties-in-interest, since those
proceedings are proceedings in rem; and the legal scheme of classification,
concurrence and preference of credits in the Civil Code, the Insolvency Law, and the
Labor Code is preserved in harmony.
Disposition Petition DENIED. Assailed decision of the CA AFFIRMED.

BARAYOGA V ASSET PRIVATIZATION TRUST


473 SCRA 690
PANGANIBAN; October 24, 2005

NATURE
Petition for review of the decision of the CA reversing
the findings of the NLRC
FACTS
- Bisudeco-Philsucor

Corfarm Workers Union is composed of workers of Bicolandia


Sugar Development Corporation (BISUDECO), a sugar plantation mill located in Himaao,
Pili, Camarines Sur.
- On December 8, 1986, Respondent Asset Privatization Trust (APT), a public trust was
created under Proclamation No. 50, as amended, mandated to take title to and possession
of, conserve, provisionally manage and dispose of non-performing assets of the Philippine
government identified for privatization or disposition.
- Pursuant to Section 23 of Proclamation No. 50, former President Corazon Aquino issued
Administrative Order No. 14 identifying certain assets of government institutions that were
to be transferred to the National Government and among the assets transferred was the
financial claim of the Philippine National Bank against BISUDECO in the form of a secured
loan.
- Consequently, by virtue of a Trust Agreement executed between the National
Government and APT on February 27, 1987, APT was constituted as trustee over
BISUDECOs account with the PNB. Sometime later, on August 28, 1988, BISUDECO
contracted the services of Philippine Sugar Corporation (Philsucor) to take over the

A2010

- 169 -

Disini

management of the sugar plantation and milling operations until August 31, 1992.
- Meanwhile, because of the continued failure of BISUDECO to pay its outstanding loan
with PNB, its mortgaged properties were foreclosed and subsequently sold in a public
auction to APT, as the sole bidder. On April 2, 1991, APT was issued a Sheriffs Certificate
of
Sale.
- On July 23, 1991, the union filed a complaint for unfair labor practice, illegal dismissal,
illegal deduction and underpayment of wages and other labor standard benefits plus
damages.
- On March 2, 1993, the union filed an amended complaint, impleading as additional party
respondents APT and Pensumil, the company that took over its sugar milling operations.
- In their Position Paper, the union alleged that when Philsucor initially took over the
operations of the company, it retained BISUDECOs existing personnel under the same
terms and conditions of employment. Nonetheless, at the start of the season sometime in
May 1991, Philsucor started recalling workers back to work, to the exception of the union
members. Management told them that they will be re-hired only if they resign from the
union. Just the same, thereafter, the company started to employ the services of outsiders
under
the
pakyaw
system.
- BISUDECO, Pensumil and APT all interposed the defense of lack of employer-employee
relationship.
ISSUE
WON APT is liable for the claims of petitioners against their former employer
HELD
NO
- Responsibility for the liabilities of a mortgagor towards its employees cannot be
transferred via an auction sale to a purchaser who is also the mortgagee-creditor of the
foreclosed assets and chattels. Clearly, the mortgagee-creditor has no employeremployee relations with the mortgagors workers. The mortgage constitutes a lien on the
determinate properties of the employer-debtor, because it is a specially preferred credit to
which the workers monetary claims is deemed subordinate.
- The duties and liabilities of BISUDECO, including its monetary liabilities to its employees,
were not all automatically assumed by APT as purchaser of the foreclosed properties at
the auction sale. Any assumption of liability must be specifically and categorically agreed
upon. In Sundowner Development Corp. v. Drilon the Court ruled that, unless expressly
assumed, labor contracts like collective bargaining agreements are not enforceable
against the transferee of an enterprise. Labor contracts are in personam and thus binding
only between the parties.
- No succession of employment rights and obligations can be said to have taken place
between the two. Between the employees of BISUDECO and APT, there is no privity of
contract that would make the latter a substitute employer that should be burdened with the
obligations of the corporation.
- The rule has been laid down that the sale or disposition must be motivated by good faith
as an element of exemption from liability. Indeed, an innocent transferee of a business
establishment has no liability to the employees of the transferor to continue employing
them. Nor is the transferee liable for past unfair labor practices of the previous owner,
except, when the liability therefor is assumed by the new employer under the contract of
sale, or when liability arises because of the new owners participation in thwarting or
defeating the rights of the employees.
- The liabilities of the previous owner to its employees are not enforceable against the
buyer or transferee, unless (1) the latter unequivocally assumes them; or (2) the sale or
transfer was made in bad faith. Thus, APT cannot be held responsible for the monetary
claims of petitioners who had been dismissed even before it actually took over
BISUDECOs assets.
- it should be remembered that APT merely became a transferee of BISUDECOs assets
for purposes of conservation because of its lien on those assets -- a lien it assumed as
assignee of the loan secured by the corporation from PNB. Subsequently, APT, as the
highest bidder in the auction sale, acquired ownership of the foreclosed properties.
- Relevant to this transfer of assets is Article 110 of the Labor Code, as amended by
Republic Act No. 6715, which reads:
Article 110. Workers preference in case of bankruptcy. In the event of bankruptcy or
liquidation of the employers business, his workers shall enjoy first preference as
regards their unpaid wages and other monetary claims shall be paid in full before the
claims of the Government and other creditors may be paid.
- This Court has ruled in a long line of cases that under Articles 2241 and 2242 of the
Civil Code, a mortgage credit is a special preferred credit that enjoys preference
with respect to a specific/determinate property of the debtor. On the other hand, the
workers preference under Article 110 of the Labor Code is an ordinary preferred
credit. While this provision raises the workers money claim to first priority in the
order of preference established under Article 2244 of the Civil Code, the claim has
no
preference
over
special
preferred
credits.
- Thus, the right of employees to be paid benefits due them from the properties of their
employer cannot have any preference over the latters mortgage credit. In other words,

Labor Law 1
being a mortgage credit, APTs lien on BISUDECOs mortgaged assets is a special
preferred lien that must be satisfied first before the claims of the workers.
- In Development Bank of the Philippines v. NLRC the rationale of this ruling was
explained as follows:
A preference applies only to claims which do not attach to specific properties. A lien
creates a charge on a particular property. The right of first preference as regards
unpaid wages recognized by Article 110 does not constitute a lien on the property of
the insolvent debtor in favor of workers. It is but a preference of credit in their favor, a
preference in application. It is a method adopted to determine and specify the order in
which credits should be paid in the final distribution of the proceeds of the insolvents
assets. It is a right to a first preference in the discharge of the funds of the judgment
debtor. Furthermore, workers claims for unpaid wages and monetary benefits cannot
be paid outside of a bankruptcy or judicial liquidation proceedings against the
employer. It is settled that the application of Article 110 of the Labor Code is contingent
upon the institution of those proceedings, during which all creditors are convened, their
claims ascertained and inventoried, and their preferences determined. Assured thereby
is an orderly determination of the preference given to creditors claims; and preserved
in harmony is the legal scheme of classification, concurrence and preference of credits
in the Civil Code, the Insolvency Law, and the Labor Code. The Court hastens to add
that the present Petition was brought against APT alone. In holding that the latter,
which has never really been an employer of petitioners, is not liable for their claims, this
Court is not reversing or ruling upon their entitlement to back wages and other unpaid
benefits from their previous employer.
Disposition Petition denied

RECEIVERSHIP
RUBBERWORLD (PHILS) INC V NLRC
336 S 433
PARDO; July 26, 2000
NATURE
Petition to annul the resolution of the National Labor Relations Commission
FACTS
- Aquilino Magsalin, Pedro Manibo, Ricardo Borja, Benjamin Camitan, Alicia M. San
Pedro, and Felomena Tolin were employed as dispatcher, warehouseman, issue monitor,
foreman, jacks cementer and outer sole attacher, respectively. On August 26, 1994,
Rubberworld filed with the Department of Labor and Employment a notice of temporary
shutdown of operations to take effect on September 26, 1994. Before the effectivity date,
however, Rubberworld was forced to prematurely shutdown its operations. On November
11, 1994, private respondents filed with the National Labor Relations Commission a
complaint against petitioner for illegal dismissal and non-payment of separation pay.On
November 22, 1994, Rubberworld filed with the Securities and Exchange Commission
(SEC) a petition for declaration of suspension of payments with a proposed rehabilitation
plan
- On December 28, 1994, SEC issued the following order:
"Accordingly, with the creation of the Management Committee, all actions for claims
against Rubberworld Philippines, Inc. pending before any court, tribunal, office, board,
body, Commission or sheriff are hereby deemed SUSPENDED."Consequently, all pending
incidents for preliminary injunctions, writ or attachments, foreclosures and the like are
hereby rendered moot and academic.
- On January 24, 1995, petitioners submitted to the labor arbiter a motion to suspend the
proceedings invoking the SEC order dated December 28, 1994. The labor arbiter did not
act on the motion and ordered the parties to submit their respective position papers.
- On December 10, 1995, the labor arbiter rendered a decision, which provides: "In the
light of the foregoing, respondents are hereby declared guilty of Illegal Shurtdown. On
February 5, 1996, petitioners appealed to the National Labor Relations Commission
(NLRC) alleging abuse of discretion and serious errors in the findings of facts of the labor
arbiter. On August 30, 1996, NLRC issued a resolution affirming the decision with
modification in that the award of moral and exemplary damages were deleted
ISSUE
WON the Department of Labor and Employment, the Labor Arbiter and the National Labor
Relations Commission may legally act on the claims of respondents despite the order of
the Securities and Exchange Commission suspending all actions against a company
under rehabilitation by a management committee created by the Securities and Exchange
Commission.
HELD
NO
- The petition is hereby granted. The decision of the labor arbiter dated December 10, 1995
and the NLRC resolution dated August 30, 1996, are SET ASIDE.

A2010

- 170 -

Disini

Ratio Presidential Decree No. 902-A is clear that "all actions for claims against
corporations, partnerships or associations under management or receivership pending
before any court, tribunal, board or body shall be suspended accordingly." The law did not
make any exception in favor of labor claims. The justification for the automatic stay of all
pending actions for claims is to enable the management committee or the rehabilitation
receiver to effectively exercise its/his powers free from any judicial or extra judicial
interference that might unduly hinder or prevent the 'rescue' of the debtor company. To
allow such other actions to continue would only add to the burden of the management
committee or rehabilitation receiver, whose time, effort and resources would be wasted in
defending claims against the corporation instead of being directed toward its restructuring
and rehabilitation. Thus, the labor case would defeat the purpose of an automatic stay. To
rule otherwise would open the floodgates to numerous claims and would defeat the rescue
efforts of the management committee.
- This finds ratiocination in that the power to hear and decide labor disputes is deemed
suspended when the Securities and Exchange Commission puts the corporation under
rehabilitation. Thus, when NLRC proceeded to decide the case despite the SEC
suspension order, the NLRC acted without or in excess of its jurisdiction to hear and decide
cases. As a consequence, any resolution, decision or order that it rendered or issued
without jurisdiction is a nullity.

9.18 WAGE RECOVERY AND ATTORNEYS FEES


PLACEWELL INTERNATIONAL V CAMOTE
[PAGE 117]
PHILIPPINE FISHERIES DEVELOPMENT AUTHORITY
V NLRC
213 SCRA 621
GUTIERREZ JR; September 4, 1992
NATURE
PETITION to review the resolutions of the National Labor Relations Commission
FACTS
- Petitioner, PFDA is a GOCC created by PD977.
- November 1, 1984, Wage Order No. 6 was already in effect. After bidding, Odin Security
Agency (Odin) and PFDA entered into a Security Services Contract for the Iloilo Fishing
Port Complex on November 11, 1985. The compensation to be paid for the security
guards would supposedly include, among others, the minimum wage based on Wage
Order No. 5 (NOTE: not based on the latest Wage Order, which was already in effect 1 yr
before the contract was entered into).
- On October 24, 1987, during the effectivity of the said Security Agreement, Odin
requested PFDA to adjust the contract rate in accordance with Wage Order No. 6. Among
the important provisions in the said Order was that the increases in the minimum wage
and allowance rates shall be borne by the principal/client and the contracts shall be
deemed amended accordingly (Sec9). The Security Services Contract provided for an
automatic escalation of the rate per guard in case of wage increase. The request for
adjustment was made further on two more occasions but were ignored by PFDA. So, Odin
filed a complaint for unpaid amound of re-adjustment rate under Wage Order No. 6
together with wage salary differentials from the integration of the COLA under Wage Order
No. 1, 2, 3, and 5 pursuant to EO 178 plus P25k as attys fees and cost of litigation. PFDA
filed a Motion to Dismiss (no jurisdiction, security guards no legal personality, if ever they
have, action involves interpretation of contract Labor Arbiter has no authority)
- LA: dismissed complaint: PFDA under scope and jurisdiction of the Civil Service
Commission Odin appealed to the NLRC
- NLRC: set aside order, entered decision in favor of Odin MFR filed by PFDA (due
process violation, resolution granting relief without legal basis; if with legal basis for the
award, stipulation allowing an increase of wage rate void ab initio) MFR denied
Petitioners contention
(1) The National Labor Relations Commission failed to ob serve due process
(2) Granting the award of the National Labor Relations Commission is valid, reliefs granted
are not legal
(3) Assuming the award complies with the requirements of due process, the National
Labor Relations Commission erred when it failed to declare the contract for security
services void.
ISSUES
1. WON NLRC had jurisdiction over the case
2. WON due process was observed
3. WON the contract for security services void
4. WON PFDA should carry the burden of the wage increase

Labor Law 1
HELD
1. YES
Ratio Notwithstanding that the petitioner is a government agency, its liabilities, which are
joint and solidary with that of the contractor, are provided in Articles 106, 107 and 109 of
the Labor Code. This places the petitioner's liabilities under the scope of the NLRC.
Moreover, Book Three, Title 11 on Wages specifically provides that the term "employer"
includes any person acting directly or indirectly in the interest of an employer in relation to
an employee and shall include the Government and all its branches, subdivisions and
instrumentalities, all government-owned or controlled corporations and institutions as well
as non-profit private institutions, or organizations (Art. 97 [b], Labor Code..)
Reasoning
- Generally, since petitioner is a GOCC with a special charter, it is under the scope of the
civil service. However, the guards are not employees of PFDA. Since no EER, cant invoke
jurisdiction of Civil Service Commission. The contract entered was merely job contracting,
PFDA being an indirect employer. Therefore, under NLRC.
2. YES
Ratio A decision on the merits is proper where the issues raised by the parties did not
involve intricate questions of law.
Reasoning
- The Wage Orders are statutory and mandatory and can not be waived. The petitioner
can not escape liability since the law provides the joint and solidary liability of the principal
and the contractor for the protection of the laborers. There can be no question that the
security guards are entitled to wage adjustments. The computation of the amount due to
each individual guard can be made during the execution of the decision where hearings
can be held. (See Section 3, Rule VIII of the New Rules of Procedure of the NLRC)
3. ESTOPPED FROM QUESTIONING VALIDITY
Reasoning
- Cannot question validity on the ground that it did not comply with the bidding
requirements set by law. Undeniably, services were rendered already and the petitioner
benefitted from said contract for two (2) years now.
4. YES. But Odin would pay first, subject to reimbursement of from PFDA.
Ratio In job contracting, the petitioner as principal is jointly and severally liable with the
contractor for the payment of unpaid wages. The Statutory basis for the joint and several
liability is set forth in Articles 107, and 109 in relation to Article 106 of the Labor Code.
- Eagle Security vs. NLRC (explained interpretation of wage order): If explicitly say
that payment of the increase are to be borne by the principal or client, doesnt mean that
principal would DIRECTLY pay the security guards the wage and allowance because NO
PRIVITY of CONTRACT between them. Security guards contractual relationship is with
their immediate employer (Security Agency). Security agency tasked as an employer with
the payment of the wages. In order for the security agency to comply with the new wage
order and allowance rates, amendment of the contract between security agency and the
principal as to the consideration to cover the service contractor the payment of the
increases mandated should be done. In the end, therefore, ultimate liability for the
payment of the increases rests with the principal."
Reasoning
- As was held in the stated case, the security guards direct recourse is from their direct
employer Odin. Also Odin is equally guilty when it entered into the contract with PFDA
without considering Wage Order No. 6, which was already in effect on the date when they
entered into the bidding for the security contract.
- on the fixing of contract rate of security agencies by the Philippine Association of
Detective and Protective Agency Operators: no moment. Their memo was not
necessary to make Wage Order No. 6 effective upon Odin; it was merely an internal
agreement among the operators to set the ceiling of the contract rates. This was aimed to
curb the practice of security agencies which were in cutthroat competition to request for
wage adjustments after proposals were accepted in good faith to the prejudice of the
parties.
-The private respondent is the employer of the security guards and as the employer, it is
charged with knowledge of labor laws and the adequacy of the compensation that it
demands for contractual services is its principal concern and not any other's. By filing the
complaint in its own behalf and in behalf of the security guards, the private respondent
wishes to exculpate itself from liability on the strength of the ruling in the Eagle case that
the ultimate liability rests with the principal.
Disposition the questioned resolutions of the National Labor Relations Commission are
hereby AFFIRMED with the modification that both the petitioner and the private
respondent are ORDERED to pay jointly and severally the unpaid wage differentials under
Wage O rder No. 6 without prejudice to the right of reimbursement for one-half of the
amount which either the petitioner or the private respondent may have to pay to the
security guards

CHAVEZ V NLRC
[PAGE 59]

A2010

- 171 -

Disini

MAYON HOTEL AND RESTAURANTS V ADANA


458 SCRA 609
PUNO; May 16, 2005
NATURE
This is a petition for certiorari to reverse and set aside the Decision issued by the Court of
Appeals and the Resolution denying petitioners motion for reconsideration.
FACTS
- Petitioner Mayon Hotel & Restaurant is a single proprietor business registered in the
name of petitioner Pacita O. Po, whose mother, petitioner Josefa Po Lam, manages the
establishment. The hotel and restaurant employed about sixteen employees.
- Due to the expiration and non-renewal of the lease contract for the rented space
occupied by the said hotel and restaurant at Rizal Street, the hotel operations of the
business were suspended. The operation of the restaurant was continued in its new
location at Elizondo Street, Legazpi City, while waiting for the construction of a new Mayon
Hotel & Restaurant. Only nine of the sixteen employees continued working at its new site.
- On various dates, the 16 employees filed complaints for underpayment of wages
and other money claims against petitioners.
- Executive Labor Arbiter rendered a Joint Decision in favor of the employees.
- On appeal to the NLRC, the decision of the Labor Arbiter was reversed, and all the
complaints were dismissed.
- Respondents filed a motion for reconsideration with the NLRC and when this was
denied, they filed a petition for certiorari with the CA which rendered the now assailed
decision.
ISSUE
WON the cost of the food and snacks provided to respondents as facilities should have
been included in reckoning the payment of respondents wages
HELD
NO
- While petitioners submitted Facility Evaluation Orders issued by the DOLE Regional
Office whereby the cost of meals given by [petitioners] to [respondents] were specified for
purposes of considering the same as part of their wages, We cannot consider the cost of
meals in the Orders as applicable to [respondents]. [Respondents] were not interviewed
by the DOLE as to the quality and quantity of food appearing in the applications of
[petitioners] for facility evaluation prior to its approval to determine whether or not
[respondents] were indeed given such kind and quantity of food. Also, there was no
evidence that the quality and quantity of food in the Orders were voluntarily accepted by
[respondents]. On the contrary; while some [of the respondents] admitted that they were
given meals and merienda, the quality of food serve[d] to them were not what were
provided for in the Orders and that it was only when they filed these cases that they came
to know about said Facility Evaluation Orders. [Petitioner] Josefa herself, who applied for
evaluation of the facility (food) given to [respondents], testified that she did not inform
[respondents] concerning said Facility Evaluation Orders.
- Even granting that meals and snacks were provided and indeed constituted facilities,
such facilities could not be deducted without compliance with certain legal requirements.
The employer simply cannot deduct the value from the employee's wages without
satisfying the following: (a) proof that such facilities are customarily furnished by the trade;
(b) the provision of deductible facilities is voluntarily accepted in writing by the employee;
and (c) the facilities are charged at fair and reasonable value. The records are clear that
petitioners failed to comply with these requirements. There was no proof of respondents
written authorization. Indeed, the Labor Arbiter found that while the respondents admitted
that they were given meals and merienda, the quality of food served to them was not what
was provided for in the Facility Evaluation Orders and it was only when they filed the
cases that they came to know of this supposed Facility Evaluation Orders. Petitioner
Josefa Po Lam herself admitted that she did not inform the respondents of the facilities
she had applied for.
- More important, we note the uncontroverted testimony of respondents on record that
they were required to eat in the hotel and restaurant so that they will not go home and
there is no interruption in the services of Mayon Hotel. Food or snacks or other
convenience provided by the employers are deemed as supplements if they are granted
for the convenience of the employer. The criterion in making a distinction between a
supplement and a facility does not so much lie in the kind (food, lodging) but the purpose.
Considering, therefore, that hotel workers are required to work different shifts and are
expected to be available at various odd hours, their ready availability is a necessary
matter in the operations of a small hotel, such as petitioners business.
Disposition petition is hereby DENIED. The Decision of January 17, 2003 of the Court
of Appeals in CA-G.R. SP No. 68642 upholding the Joint Decision of July 14, 2000 of the
Labor Arbiter in RAB V Case Nos. 04-00079-97 and 04-00080-97 is AFFIRMED, with the
following MODIFICATIONS:
(1) Granting separation pay of one-half (1/2) month for every year of service
to respondents Loveres, Macandog and Llarena;

Labor Law 1
(2) Granting retirement pay for respondents Guades, Nicerio, and Alamares;
(3) Removing the deductions for food facility from the amounts due to all
respondents;
(4) Awarding moral damages of P20,000.00 each for respondents Loveres,
Macandog, Llarena, Guades, Nicerio, Atractivo, and Broola;
(5) Deleting the award of exemplary damages of P10,000.00 from all
respondents except Loveres, Macandog, Llarena, Guades, Nicerio, Atractivo,
and Broola; and
(6) Granting attorneys fees of P10,000.00 each to all respondents.
The case is REMANDED to the Labor Arbiter for the RECOMPUTATION of the total
monetary benefits awarded and due to the employees concerned in accordance with
the decision. The Labor Arbiter is ORDERED to submit his compliance thereon within
thirty (30) days from notice of this decision, with copies furnished to the parties.

AKLAN ELECTRIC V NLRC


[PAGE 144]
INTERNATIONAL SCHOOL ALLIANCE V QUISUMBING
[PAGE 32]

PHILEX GOLD VS PHILEX BULAWAN SUPERVISORS


468 SCRA 111
AZCUNA; August 25, 2005
NATURE
Petition for certiorari to review CA decision
FACTS
- Philex Gold operates a gold mining concession in Sipalay, Negros Occidental where the
sole and exclusive bargaining representative of all supervisors is the Philex Bulawan
Supervisors Union. On July 1, 1997, Philex Gold made the employees of its Benguet
operations supervisory employees in its Sipalay operations.
- As it turned out, the salaries of the Padcal employees were higher than those locally
hired in Bulawan despite the fact that they were of similar rank and classification doing
parallel duties and functions.
- The union filed a complaint against Philex for discriminatory wage policy which Article
248 (e)38 of the Labor Code prohibits and defines as Unfair Labor practice.
- The Voluntary Arbitrator initially found for the Union but on motion for reconsideration
reversed its original order and instead ordered an P800 across the board increase. In its
order, the Voluntary Arbitrator also decreed that the principal officers of Philex be solidarily
liable for the payment of the increase.
- On appeal to the CA, the original ruling of the arbitrator was reinstated. Hence this
appeal to the SC
ISSUES
1. WON the notice sent through petitioners Liaison office can be considered as notice to
counsel
2. WON the petitioners-corporate officer are solidarily liable
3. WON the doctrine of equal pay for equal work should not remove management
prerogative to institute difference in salary on the basis of seniority, skill, experience, and
dislocation factor in the same class of supervisory workers doing the same kind of work
HELD
1. NO
- Section 4, Rule III of the NCMB Procedural Guidelines in the Conduct of Voluntary
Arbitration Proceedings states that where a party is represented by counsel or authorized
representative, service shall be made on the latter. As the notice was sent to a place
other than that of the counsel, the receipt of the notice by the counsel shall be the
reckoning date for the 10-day reglementary period to file a motion for reconsideration.
2. NO
- The corporation is a juridical entity with legal personality separate and distinct from those
acting for and in its behalf, and in general, from the people comprising it. The rule is that
obligations incurred by the corporation, acting through its directors, officers and
employees, are its sole liabilities. There are circumstances where the director, trustee or
officer of the corporation may be held solidarily liable with the corporation. These include
acts by these individuals which are in bad faith or with gross negligence.
38

Art 248 (e) to discriminate in regard to wages, hours of work and other terms and conditions of employment in order
to encourage or discourage membership in any labor organization

A2010

- 172 -

Disini

3. NO
- In this case, the petitioner failed to differentiate the basic salary from any kind of salary
increase or additional benefit which may have been given to the Padcal supervisors due to
their seniority, experience and other factors. The records only show that an ex-Padcal
supervisor is paid a higher salary than a locally hired supervisor of he same rank. The
companys prerogative must be exercised in good faith and with due regard to the rights of
labor. A priori, they are not absolute prerogatives but are subject to legal limits, collective
bargaining agreements, and the general principles of fair play and justice.
Disposition Petition is denied

BANKARD EMPLOYEES UNION V NLRC (BANKARD


INC)

423 SCRA 148


CARPIO-MORALES; February 17, 2004
FACTS
- Bankard, Inc. (Bankard) classifies its employees by levels. On May 28, 1993, its Board of
Directors approved a "New Salary Scale" for the purpose of making its hiring rate
competitive in the industrys labor market. The "New Salary Scale" increased the hiring
rates of new employees. Accordingly, the salaries of employees who fell below the new
minimum rates were also adjusted to reach such rates under their levels.
- Bankards move drew the Bankard Employees Union-WATU (petitioner), the duly
certified exclusive bargaining agent of the regular rank and file employees of Bankard, to
press for the increase in the salary of its old, regular employees. Bankard took the
position, however, that there was no obligation on the part of the management to grant to
all its employees the same increase in an across-the-board manner.
- Petitioners filed Notices of Strike on the ground of discrimination and other acts of
Unfair Labor Practice. The strike was averted, however, when the dispute was certified by
the Secretary of Labor and Employment for compulsory arbitration.
- NLRC, finding no wage distortion, dismissed the case for lack of merit. MFR was denied.
Hence, this petition for certiorari.
ISSUE
WON the unilateral adoption by an employer of an upgraded salary scale that increased
the hiring rates of new employees without increasing the salary rates of old employees
amounts to discrimination

HELD
NO
Ratio Absent any indication that the voluntary increase of salary rates by an employer was
done arbitrarily and illegally for the purpose of circumventing the laws or was devoid of
any legitimate purpose other than to discriminate against the regular employees, this
Court will not step in to interfere with this management prerogative.
Reasoning
- Petitioner cannot make a contrary classification of private respondents employees
without encroaching upon recognized management prerogative of formulating a wage
structure, in this case, one based on level.
- While seniority may be a factor in determining the wages of employees, it cannot be
made the sole basis in cases where the nature of their work differs. Moreover, for
purposes of determining the existence of wage distortion, employees cannot create their
own independent classification and use it as a basis to demand an across-the-board
increase in salary.
- Apart from the findings of fact of the NLRC and the Court of Appeals that some of the
elements of wage distortion are absent, petitioner cannot legally obligate Bankard to
correct the alleged "wage distortion" as the increase in the wages and salaries of the
newly-hired was not due to a prescribed law or wage order. The wordings of Article 124
are clear. If it was the intention of the legislators to cover all kinds of wage adjustments,
then the language of the law should have been broad, not restrictive as it is currently
phrased.

- Moreover, Bankards right to increase its hiring rate,


to establish minimum salaries for specific jobs, and to
adjust the rates of employees affected thereby is

Labor Law 1
embodied under Section 2, Article V (Salary and Cost
of Living Allowance) of the parties Collective
Bargaining Agreement (CBA).
Disposition present petition is hereby DENIED.

F. MINIMUM WAGES

9.19 WAGES AND THE CONSTITUTION


MINIMUM WAGES
ATOK BIG WEDGE MINING CO INC V ATOK BIG
WEDGE MUTUAL BENEFIT ASSOCIATION
92 PHIL 755
LABRADOR; March 3, 1953
NATURE
PETITION for review by certiorari of a decision of the Court of Industrial Relations
FACTS
- On September 4, 1950, a demand was submitted to petitioner by respondent union
through its officers for various concessions, among which were (a) an increase of P0.50 in
wages, (b) commutation of sick and vacation leave if not enjoyed during the year, (c)
various privileges, such as free medical care, medicine, and hospitalization, (d) right to a
closed shop, check off, etc., (e) no dismissal without prior just cause and with a prior
investigation, etc.
- Some of the demands, were granted by the petitioner, and the others were rejected.
Hearings were held in the respondent court (CIR)
- After the hearings the respondent court rendered a decision fixing the minimum wage for
the laborers at P3.20, declaring that additional compensation representing efficiency
bonus should not be included as part of the wage, and making the award effective from
September 4, 1950 (the date of the presentation of the original demand, instead of from
April 5, 1951, the date of the amended demand).
ISSUES
1. WON the Court of Industrial Relations erred in fixing the minimum wage at P3.20
2. WON the Court of Industrial Relations erred in declaring that the additional
compensation representing the efficiency bonus should not be included as part of the
wage
3. WON the Court of Industrial Relations erred in making the award effective from
September 4, 1950
HELD
1. NO
- Petitioner contends that the laborer and his family need only the amount of P2.58 for
food so this should be the basis for the determination of his wage, not what he actually
spends. Furthermore, that it is not justifiable to fix a wage higher than that provided by
Republic Act No. 602 and that respondent union made the demand in accordance with a
pernicious practice of claiming more after an original demand is granted.
- The respondent court found that P2.58 is the minimum amount actually needed by the
laborer and his family. But this does not mean that it is his actual expense. A person's
needs increase as his means increase. This is true not only as to food but as to everything
else-education, clothing, entertainment, etc. The law guarantees the laborer a fair and
just wage. The minimum must be fair and just. The "minimum wage" can by no
means imply only the actual minimum. Some margin or leeway must be provided,
over and above the minimum, to take care of contingencies, such as increase of
prices of commodities and increase in wants, and to provide means for a desirable
improvement in his mode of living. That the P3 minimum wage fixed in the law is still far
below what is considered a fair and just minimum is shown by the fact that this amount is
only for the year after the law takes effect, as thereafter the law fixes it at P4. Neither may
it be correctly contended that the demand for increase is due to an alleged pernicious
practice. Frequent demands for increase are indicative of a healthy spirit of wakefulness to
the demands of a progressing and an increasingly more expensive world.

A2010

- 173 -

Disini

2. NO
- Petitioner contends that the efficiency bonus paid the laborer should have been included
in his minimum wage, in the same manner as the value of living quarters. Whether or not
bonus forms part of wages depends upon the circumstances or conditions for its
payment. If it is an additional compensation which the employer promised and
agreed to give without any conditions imposed for its payment, such as success of
business or greater production or output, then it is part of the wage. But if it is paid
only if profits are realized or a certain amount of productivity achieved, it can not be
considered part of the wages. In the case at bar, it is not payable to all but to laborers
only. It is also paid on the basis of actual production or actual work accomplished. If the
desired goal of production is not obtained, or the amount of actual work accomplished, the
bonus does not accrue. It is evident that under the circumstances it is paid only when the
labor becomes more efficient or more productive. It is only an inducement for efficiency, a
prize therefore, not a part of the wage.
3. NO
- Both parties agreed that any award should be retroactive to the date of the presentation
of the demands, which is September 4, 1950. The terms of the stipulation are clearly
against petitioner's contention. There being no question as to its agreement, the same
must be given force and effect.
Disposition Dismissed with costs

BENEFICIARIES
PEOPLE V GATCHALIAN
104 PHIL 664
BAUTISTA ANGELO; September 30, 1958

FACTS
- Alfonso Gatchalian owner or manager of the New Life Drug Store was charged with a
violation of Section 3 of Republic Act No. 602 for paying Expedito Fernandez, a monthly
salary of P60 to P90 on or about August 4, 1951, up to and including December 31, 1953
which is less than that provided for by law, thereby leaving a difference of an unpaid salary
to the latter in the total amount of P1,016.64 for the period above-mentioned.
- He filed a written motion to dismiss based on two grounds which in substance merely
consist in that the violation charged does not constitute a criminal offense but carries only
a civil liability, and even if it does, the section of the law alleged to have been violated
does not carry any penalty penalizing it.
- The city Attorney of Zamboanga filed his answer to the motion to dismiss contending that
the law which was violated by the accused carries with it both civil and criminal liability, the
latter being covered by Section 15 which provides for the penalty for all willful violations of
any of the provisions of the Minimum Wage Law.
- The Court issued an order dismissing the informations with costs de oficio and cancelling
the bail bond filed by the accused. The court in the same order directed the Regional
Representative of the Department of Labor to immediately institute a civil action against
the erring employer for the collection of the alleged underpayment of wages due the
employees.
- A motion for reconsideration having been denied, the Government took the present
appeal.

ISSUE
WON law which violated by the accused carries with it both civil and criminal liability

HELD
YES
- It is clear from the provisions of Republic Act 602 that while Section 3 explicitly requires
every owner of an establishment located outside of Manila or its environs to pay each of
its employees P3.00 a day on the effective date of the Act, and one year thereafter P4.00
a day, Section 15 imposes both a criminal penalty for a wilful violation of any of the above
provisions and a civil liability for any underpayment of wages due an employee.
- The intention of the law is clear: to slap not only a criminal liability upon an erring
employer for any willful violation of the acts sought to be enjoined but to attach
concurrently a civil liability for any underpayment he may commit as a result thereof.
- The law speaks of a willful violation of 11 any of the provisions of this Act", which is allembracing, and the same must include what is enjoined in Section 3 thereof which
embodies the very fundamental purpose for which the law has been adopted.

Labor Law 1
- The Minimum Wage Law (Republic Act 602) was patterned after the U. S. Fair Labor
Standards Act of 1938.
- While in substance they are similar, they however contain some differences in their
phraseology and in the apportionment of their provisions. Thus, while Section 15 (a),
paragraph 2, of the Fair Labor Standards Act makes it unlawful for an employer not to pay
the minimum wage prescribed therein, our Minimum Wage Law does not contain a similar
provision. Again, the Fair Labor Standards Act enumerates in one single section all those
acts which are declared unlawful and are not spread out in different sections as done in
our law.
- It should also be noted that while Section 16 of the Fair Labor Standards Act which
provides for the penalties to be imposed for any willful violation of the provisions of the Act
specifically states that those penalties refer to acts declared unlawful under Section 15 of
the same Act, our law does not contain such specification.
- This distinction is very revealing. It clearly indicates that while the Fair Labor Standards
Act intends to subject to criminal action only acts that are declared unlawful, our law by
legislative fiat intends to punish not only those expressly declared unlawful but even those
not so declared but are clearly enjoined to be observed to carry out the fundamental
purpose of the law. One such provision is undoubtedly that which refers to the payment of
the minimum wage embodied in Section 3.
- Indeed, the main objective of the law is to provide for a rock-bottom wage to be observed
and followed by all employers of all agricultural and industrial establishment. This objective
would be defeated were we to adopt a restrictive interpretation of the above penal clause,
for all employer who knows that he cannot be amenable to a criminal action would be
prone to subvert the law because if he is detected it would be easy for him to pay the
underpayment and the corresponding interest as would be the case were he to assume
merely a civil liability.
- This would be a mockery and a decision of the law not contemplated by our lawmaker
which would certainly render it nugatory and abortive. If the law is to survive, it must be
real, militant and effective.
- "The establishment of the maximum wage benefits directly the low-paid employees, who
now receive inadequate wages on which to support themselves and their families. It
benefits all wage earners indirectly by setting a floor below which their remuneration
cannot fail. It raises the standards of competition among employers, since it would protect
the fair-minded employer who voluntarily pays a wage that supports the wage earner from
the competition of the employer who operates at lower cost by reasons of paying his
workers a wage below subsistence. If, in fact, the employer cannot pay a subsistence
wage then he should riot continue his operation unless he improves his methods and
equipment so as to make the payment of the minimum wage feasible for him otherwise the
ernployer is wasting the toil of the worker and the material resources used in the
employment. Second methods of operation, progressive and fair-minded Management,
and an adequate minimum wage go hand in hand."
- Counsel for appellee however entertains a different interpretation. He contends that if
Section 15 (a) should be interpreted in a manner that would embrace a wilful violation of
any of the provisions of the law we would have a situation where even the officials
entrusted with its enforcement may be held criminally liable which is not contemplated in
the law.

- These provisions are substantive in


nature and had been adopted for
common observance by the persons
affected. They cannot be eluded nor
subverted lest the erring employer
runs into the sanction of the law. On
the other hand, the provisions adverted
to by counsel are merely administrative
in character which had been adopted
to set the machinery by which the law
is to be enforced. They are provisions
established for observance by the
officials
entrusted
with
its
enforcement. Failure to comply with
them would therefore subject them
merely to administrative sanction. They

A2010

- 174 -

Disini

do not come under the penal clause


embodied ed in Section 15(a).
- It is true that Section 3 under which appellee was charged does not state that it shall be
unlawfull for an employer to pay his employees wages below the minimum wage but
merely requires that the employer shall pay wages not below the minimum wage. But
failure of such declaration does not make the non-observance of the provisions less
unlawful than otherwise, for such provision embodies precisely the raison d'etre of the law
itself. Indeed, Section is the very provision on which all the other provisions of the law are
built.
Disposition The order appealed from was set aside. It was ordered that the cases be
remanded to the court a quo for further proceedings, with costs against appellee.

SEPARATE OPINION
REYES JBL [concur]
- The act complained of, therefore, is an offense penalized tinder the Act. Our duty being
to interpret the Act in consonance with its primary purpose to benefit the laborer, we
should consider that the only sanction for not paying the minimum wage were to be the
payment of interest on the unpaid salary, the situation of the wage earner would have
been in no way advanced and the Minimum Wage Act would be practically nullified, for a
laborer is in no position to, engage in protracted litigation with his employer. As pointed in
the opinion of Justice Bautista Angelo the criminal liability is the only effective sanction
under the circumstances.

BENGZON [dissent]
- Sec.. 15, subsections (a) and (e) should be read together substantially as follows:
Any person who violates any provision of this Act shall be punished with fines, etc. *
* * cxcept that where the violation consist in paying, the employee less than the
minimum wage the employer shall be punished by requiring him to satisfy the
difference, plus attorney's fees.
- Subsection (e) punishes the particular "violation" of paying less than the minimum wage.
It is a special provision, which under well-known rules of construction, should prevail over
the general provision in subsection (a). In other words, although the Act does not
expressly say so, subsection (e) is an exception1 to subsection (a).
- It is a mistake to suppose that this interpretation fails to punish the employer who
disregards the Act; because the liability imposed by subsection (e) is unquestionably a
sanction, penal in nature, which except for the the law, would not be demandable.
Additional liability, is their prevailing idea. And yet, why should the law impose on the
employer (who pays in accordance with a contract freely entered into), additional burdens
not imposed on the other employers willfully violating other fundamental provisions of the
Act.
- It may be argued that subsection (e) is not really a sanction independent of subsection
(a), or additional thereto, because it merely repeats the well-known principle that "every
person criminally liable is also civily liable."
- The answer is twofold: first, attorney's fees are not usually included in such principle-a
robber is not required to pay attorney's fees; second, the employee is not really the injured
party because he accepted the employment under a contract "Scienti et volenti nulla fit
injuria." There is no violation of any right of the employee for which reparation is due.
There is only a breach of the statute, for which the Government can choose, and did
choose the proper sanction, namely, payment of the salary differential under subsection
(e). The Congress did not choose more than that; contrary to what it did in analogous
situations.

MONTEMAYOR [dissent]
- Under the interpretation given by the majority, any employer who underpays his
employees in violation of the Minimum Wage Law (Section 3) would be subject to criminal
prosecution. The net result would be the unjust punishment of innocent government
officials and the discouragement and destruction of infant and small industries.
- From the explanatory note of Senate Bill No. 202, we may have an idea of the attitude of
the Legislature on this particular point:
"One thing to be remembered is that the country has not yet attained that degree of
industrialization where wages can be set at fully satisfactory levels from the viewpoints of
human values. Compromises must still be made until this full industrial status is attained.
Another thing is that hasty and unjudicioas action in passing minimum wage laws may be
deterrent to private capital which, on the contrary needs to be encouraged to invest in
local industries if the industrialization of our country must someday be a fact." -- As the
majority opinion correctly observes, our Minimum Wage Law is patterned after that of the
United States Fair Labor Standards Act (F. L. S. A.) of 1938, as amended. The
establishment of this minimum wage in this country being a sort of experiment, it being the
first time that it was being tried out, and not knowing whether or not it would be a success,

Labor Law 1
it is to be presumed that the Legislature acted cautiously and warily, and even while
adopting as a pattern the United States F.L.S.A., it did not wish or Intend to make our law
more stern and strict in its enforcement and application, particularly as regards its
penalties. But the majority opinion would make our law more rigorous and severe, more
comprehensive and more devastating in the application of its penal provisions.

- What the Legislature intended to


penalize with fine and prison sentence
were only those acts which it
enumerated and declared unlawful, not
the mere failure to follow and comply
with the obligations imposed upon an
employer, such as, the nonpayment of
the minimum wage.
- It will be noticed that our law expressly provides that any employer underpaying an
employee in violation of the Act shall be liable to said employee in the amount of the
underpayment, with legal interest, plus a reasonable amount for attorney's fees. This
amount may be recovered not only by the employee himself, but by the Secretary of Labor
on his behalf. In my opinion, this civil responsibility to be enforced with the aid of the
Department of Labor, was regarded as sufficient punishment and deterrent on the
employer. Being a civil action, the employee only needs preponderance of evidence to win
his suit. The Legislature may have been of the belief that application of the penal sanction
in the form of fine and prison sentence would be too radical a measure, would scare and
discourage new and infant industries, besides inducing violators to resort to underhanded
but effective measures to hide and conceal infringement of the law, to say nothing of the
added difficulty in securing conviction, which requires not only preponderance of evidence,
but proof of guilt beyond reasonable doubt.
- To show that the Legislature did not intend to be too severe and stern in the application
of this new law which was merely being tried out, while House Bill No, 1732 was being
discussed in the Senate, Senator Taada believing that the bill was too lenient for those
who violated its provisions by providing for a fine of only not more than P1,000.00 or
imprisonment of not more than six months, he proposed that the fine be increased to not
more than P10,000.00-half the amount of the fine provided in Section 16(a) of the U.S.
Fair Labor Standards Act; but the committee sponsoring the bill, through Senator Torres,
objected to the proposed amendment as being too excessive and severe, specially at the.
beginning of the operation of the law, upon which Senator Taada, withdrew his
amendment, with the understanding that in a year or so after the promulgation of the law, it
will be amended.
- The majority of this Tribunal on its own initiative would make the Minimum Wage Law
militant and effective by a blanket and indiscriminate application of Section 15(a) to all
violators of its provisions, whether or not such violation is expressly or specifically
declared unlawful by the law itself. Stated otherwise, this Tribunal steps in, nay, rushes in
to put teeth in a legislation which it considers toothless and would make effective and
militant what it regards would otherwise be ineffectual and inadequate. This Tribunal
would, like one unlicensed to practice medicine, prescribe a cure for a supposed
legislational malady. That is not and has never been the province, much less,
the prerogative of the Judiciary. Otherwise, the courts would be indulging in judicial
legislation.
- If the Minimum Wage Law is found to be inadequate and ineffective, let the Legislature
make the necessary changes and amendments. In fact, that was the legislative plan from
the beginning-observe the operation and working of the law and then make changes, if
deemed necessary. But evidently, the Legislature is satisfied with the operation and mode
of application of the law, because although approved on April 6, 1951, and made effective
120 days thereafter, in other words, after a seven-year operation, it (Legislature) has not
seen fit to introduce any major changes4 specially in the application of the penalty.
- Anyway, after all is said and done, the least that could be said about the applicability of
Section 15(a) of the Minimum Wage law to violations of Section 3 is doubtful. Even
brushing aside and not considering the grave doubts entertained by the undersigned as to
the applicability of said Section 15(a) to violations of Section 3, we have in evidence the
opinion of the Court of First Instance of Zamboanga and according to it, the opinion of the
major sector of the Zamboanga bar, sustaining the view that Section 15(a) is not
applicable to violations of Sections 3 of the Minimum Wage Law. We have the well settled
principle in the interpretation of penal laws that in case of doubt, the interpretation
favorable to the accused should be adopted.

REYES A [dissent]
- It is doubtful if the penal sanction prescribed in section 15(a) of that Act-presumably for
the acts and practices therein declared unlawful-could be rightfully applied to the act of
underpaying an employee, since paragraph (e) of that same section already provides a
specific remedy therefor. Considering that penal statutes are strictly construed against the

A2010

- 175 -

Disini

state and in case of doubt courts must adopt the construction favorable to the accused,
the courts should not presume to legislate by putting into the law more teeth than the

EFFECT INABILITY TO PAY


PHIL. APPAREL WORKERS UNION V NLRC (PHIL
APPAREL INC)
106 SCRA 444
MAKASIAR; July 31, 1981
NATURE
Petition for review on certiorari of the NLRC decision
FACTS
- Phil. Apparel, Inc. (a corporation engaged in the garments industry) had a 1973-1976
CBA with the Phil. Apparel Workers Union (representing its approximately 2000
employees). Prior to said CBAs expiry (31 July 1976), the union submitted to the company
proposals for its renewal. Negotiations were held thereafter.
- 2 April 1977: the company and the union agreed, partially, that the employees would be
given an across-the-board increase in regular wages of P0.80 per day retroactive to April
1st. A memorandum of undertaking was signed to this effect. Because there were other
unresolved issues, the new CBA was not signed until 3 September 1977.
- In the meantime, PD 1123 was issued by the President on 21 April 1977 to take effect 1
May 1977. Section 4 of said decree provided that employees then receiving P50/month as
ECOLA should be given an increase in the sum of P60/month, but distressed employers
may be exempted while in such condition.
- The Secretary of Labor subsequently promulgated the IRR, Section 1(k) of which also
exempts employers that have granted, in addition to the allowance under PD 525, at least
P60 monthly wage increase on or after 1 January 1977, provided that those who paid less
than this amount shall pay the difference.
- 1 May 1977: the company gave its employees the P60 ECOLA provided in PD1123.
- 13 May 1977: the company wrote to DOLE asking if the wage increase of P0.80/day
agreed upon between the company and the union could be credited against the ECOLA
provided in PD1123. The reply of 15 July 1977 was in the affirmative if management and
labor had agreed to grant said amount.
- After the new CBA was signed in 3 Sept 1977, the union raised the question of
creditability of the April 1 increase of P0.80/day to the May 1 ECOLA. The matter was
taken up in grievance procedure but no agreement was reached. The union thereafter filed
a complaint with the DOLE against the company for ULP.
- 30 May 1978: Labor Arbiter dismissed the unions complaint saying that the issue should
be resolved through further proceedings under the grievance machinery established in the
CBA. NLRC set aside LAs decision, and dismissed the complaint upon its merits. Hence,
this petition for review.
ISSUE
WON the companys application of the negotiated wage increase as partial compliance
with PD 1123 is proper
HELD
NO
- The company cannot fall within the exceptions provided for in par. (k) of the Rules
implementing PD 1123 on the grant of ECOLA because when said decree took effect on
May 1, 1977 there was neither a perfected CBA (signed 3 Sept 1977) nor an actual
payment of the negotiated wage increase made retroactive to April 1, 1977.
- The letter of Undersecretary Inciong dated 13 May 1977 that there was an agreement
between the parties on the wage increase was based on a wrong premise because there
were still other unresolved bargaining items and it was only on 3 September 1977 that the
parties reached an agreement thereon.
- There being no actual agreement yet and actual payment of the agreed P0.80 wage
increase, there could have been no grant of wage increases by the employer w/in the
contemplation of par. k, Sec. 1 of the implementing rules of PD 1123 to exempt the
employer.
- In this case, the company is guilty of bad faith when it signed the CBA on 3 September
1977 without letting the union know that it was going to apply part of the allowances being
paid under PD 1123 to the wage increases provided for in the CBA.
- The stability of the economy does not depend on the employer alone, but on government
economic policies concerning productivity in all areas. It is the living wage of the workers
which is the basis of a stable economy. If a company cannot pay a living wage, it has

Labor Law 1

A2010

no business operating at the expense of the lives of its workers from the very start.
The preservation of the lives of citizens is a basic duty of the State, more vital than the
preservation of corporate profits.
- Par. (k), Sec. 1 of the Rules Implementing Pd 1123 is null and void as the Secretary of
Labor exceeded his authority in enacting it.
Disposition Certiorari granted. NLRC decision set aside. Company directed to pay, in
addition to the increased allowance provided for in PD 1123, the negotiated wage increase
of P0.80 daily, as well as other wage increases embodied in the CBA. Costs against
company.

SEPARATE OPINION
TEEHANKEE [concur]
- The memorandum of agreement signed by the parties clearly shows that the negotiated
staggered wage increases were intended to be exclusive of any statutory increase in the
minimum wage and the ECOLA is in effect another increase in the minimum wage.
- There is basis for the majority holding that the questioned portion of the rules
implementing PD1123 is void, but I do not deem it necessary to concur in such a ruling
given that the issue of validity is not the lis mota of the case.

MELENCIO-HERRERA [dissent]
- It is inequitable that an employer, who has granted increases in pay on a given day, is
further ordered to give additional increases a few days thereafter.
- Section 1(k) of the Implementing rules of PD1123 is part of the law and binding on the
courts. Great weight should be given to the interpretation of a given statute by the
government agency called upon to implement it.

9.20 AGENCIES
MACHINERY
A.
NATIONAL
COMMISSION

FOR
WAGES

WAGE
AND

FIXING

PRODUCTIVITY

B. REGIONAL TRIPARTITE WAGES AND PRODUCTIVITY


BOARD
FUNCTIONS
NASIPIT LUMBER CO V NLRC (NWPC)
289 SCRA 339 (03)
SANDOVAL-GUTIERREZ; September 8, 2003
NATURE
A petition for certiorari with prayer for issuance of a temporary restraining order and/or writ
of preliminary injunction.
FACTS
- November 19, 1993: the Regional Tripartite Wages and Productivity Board (RTWPB) of
Region X, Northern Mindanao, Cagayan de Oro City, issued a Wage Order mandating a
P7.00 increase in the minimum daily wage of all workers and employees in the private
sector in Region X receiving a daily wage of not more than P130.00 per day and, an
additional P10.00 allowance per day.
- Nasipit Lumber Company et al. filed their separate application for exemption from
compliance, claiming they are distressed establishments whose paid-up capital has been
impaired by at least 25%.
- After finding that the petitioners indeed sustained financial losses which impaired their
respective paid-up capital, the RTWPB granted petitioners a full exemption from
compliance with the said Wage Order for a period of one (1) year or from December 8,
1993 to December 7, 1994.
- Petitioners, citing the continuous business decline in the wood processing industry, filed
a consolidated petition for extension of their full exemption from compliance for another
year. RTWPB denied the application, relying on Section 7 of the NWPC Revised
Guidelines No. 1: Establishments shall be granted full exemption of one (1) year from
effectivity of the Order for all categories of exemption.
- National Wages and Productivity Commission denied the appeal interposed by the
petitioners, stating that the rationale behind the policy is to afford protection to workers

- 176 -

Disini

who may be unfairly affected by the deleterious effect of a prolonged exemption which is
not in accord with the very purpose of the issuance of a Wage Order.
ISSUE
WON NWPC exceeded its jurisdiction

HELD
NO, NWPC did not exceed its jurisdiction.
Reasoning
- ART. 121. Powers and Functions of the Commission. The Commission shall have
the following powers and functions:
x x x
(c) To prescribe rules and guidelines for the determination of appropriate minimum
wage and productivity measures at the regional, provincial or industry levels;
(d) To review regional wage levels set by the Regional Tripartite Wages and
Productivity Boards to determine if these are in accordance with prescribed guidelines
and national development plans;
- JUSTICE PANGANIBAN: The foregoing clearly grants the NWPC, x x x, the power to
prescribe the rules and guidelines for the determination of minimum wage and
productivity measures. x x x, the NWPC has the power not only to prescribe guidelines to
govern wage orders, but also to issue exemptions therefrom, x x x. In short, the NWPC
lays down the guidelines which the RTWPB implements.
- In affirming the RTWPBs Resolution denying petitioners application for extension for
another year of their full exemption from compliance with Wage Order No. RX-03, the
NWPC did not act with grave abuse of discretion. On the contrary, it merely applied its
own Guideline No. 01, Series of 1992 limiting the duration of exemption to only one (1)
year.
Disposition Petition is dismissed. Decision of the NWPC is affirmed.

9.21 AREA MINIMUM WAGES AND CRITERIA


9.22 WAGE ORDER
METHODS OF FIXING
EMPLOYERS CONFEDERATION OF THE PHIL V
NATIONAL WAGES AND PRODUCTIVITY
COMMISSION
[PAGE]

REQUIREMENTS VAILIDITY
CAGAYAN SUGAR MILLING CO V SECRETARY
284 SCRA 150
PUNO; January 15, 1998
NATURE
Petition for certiorari impugning the Decision of the Secretary of Labor upholding the Order
of the Regional Director finding petitioner guilty of violating Regional Wage Order No.
RO2-02
FACTS
- Regional Wage Order No. RO2-02 was issued by the Regional Tripartite Wage and
Productivity Board of the DOLE. It provided that the statutory minimum wage rates
applicable to workers and employees in the private sector in Region II shall be increased
by P14.00 per day in Cagayan. Labor inspectors from DOLE found that petitioner violated
the wage order as it did not implement an across the board increase in the salary of its
employees. At the hearing at the DOLE Regional Office for the alleged violation, petitioner
maintained that it complied with the wage order as it paid the mandated increase in the
minimum wage. Regional Director Martinez ruled that petitioner violated RO2-02. He
ordered petitioner to pay the deficiency in the salary of its employees in the total amount of
P555,133.41.
- Petitioner appealed to public respondent Labor Secretary Quisumbing. On the same
date, the Regional Wage Board issued Wage Order No. RO2-02-A amending the earlier
wage order in providing that the workers and employees in the private sector shall receive
an across the board wage increase of P14.00 in Cagayan, retroacting to the date of

Labor Law 1
effectivity of RO2-02. Private respondent CARSUMCO EMPLOYEES UNION moved for
execution of the order. Regional Director Martinez granted the motion and issued the writ
of execution. Petitioner moved for reconsideration. The DOLE regional sheriff served on
petitioner a notice of garnishment of its account with the Far East Bank and Trust
Company. The sheriff also seized petitioner's dump truck and scheduled its public sale.
Hence, this petition, with a prayer for the issuance of a TRO.
ISSUE
WON RO2-02-A violates petitioners right to due process
HELD
YES
Ratio RO2-02-A is invalid for lack of public consultations and hearings and non-publication
in a newspaper of general circulation, in violation of the Labor Code.
Reasoning
- Article 123 of the Labor Code provides that in the performance of their wage-determining
functions, the Regional Board shall conduct public hearings and consultations, giving
notices to interested parties. Moreover, it mandates that the Wage Order shall take effect
only after publication in a newspaper of general circulation in the region. In passing RO202-A without going through the process of public consultation and hearings, the Regional
Board deprived petitioner and other employers of due process as they were not given the
opportunity to ventilate their positions regarding the proposed wage increase. The
contention that, despite the wording of RO2-02 providing for a statutory increase in
minimum wage, the real intention of the Regional Board was to provide for an across the
board increase is absurd. There was no ambiguity in the provision of RO2-02 as it
provided in clear and categorical terms for an increase in statutory minimum wage of
workers in the region. The subsequent passage of RO2-02-A changed the essence of the
original Order.
Disposition petition is GRANTED.

PIECE WORKER
LAMBO V NLRC
317 SCRA 420
MENDOZA; October 26, 1999
NATURE
Petition for certiorari
FACTS
- Petitioners Avelino Lambo and Vicente Belocura were employed as tailors by private
respondents J.C. Tailor Shop and/or Johnny Co on September 10, 1985 and March 3,
1985, respectively. Petitioners were paid on a piece-work basis, according to the style of
suits they made. Regardless of the number of pieces they finished in a day, they were
each given a daily pay of at least P64.00
- On January 17, 1989, petitioners filed a complaint against private respondents for illegal
dismissal and sought recovery of overtime pay, holiday pay, premium pay on holiday and
rest day, service incentive leave pay, separation pay, 13th month pay, and attorneys fees.
The Labor Arbiter ruled in their favor.
- The NLRC reversed the decision of the labor arbiter. It found that the petitioners were not
dismissed but merely threatened with a closure of a business if they insisted on their
demand for a straight payment of their minimum wage, after petitioners, on January 17,
1989, walked out of a meeting with private respondents and other employees. In that
meeting, the employees voted to maintain the company policy of paying them according to
the volume of their work. Only the petitioners insisted that they be paid the minimum wage
and other benefits. The NLRC held petitioners guilty of abandonment of work.
- Petitioners deny that they abandoned their work. They allege that they were dismissed
as they were about to file a petitione with DOLE for payment of benefits.
ISSUES
1. WON the petitioners are regular employees
2. WON the petitioners abandoned their work
HELD
1. There are two categories of employees paid by results: (1) those whose time and
performance are supervised by the employer. (Here, there is an element of control and
supervision over the manner as to how the work is to be performed. A piece-rate worker
belongs to this category especially if he performs his work in the company premises.); and
(2) those whose time and performance are unsupervised. (Here, the employers control is
over the result of the work. Workers on pakyao and takay basis belong to this group.).
Petitioners belong to the 1st group.
- The relationship at bar passes the four-fold test. The mere fact that they were employed
on a per piece basis does not negate their status as regular employees. Payment by the

A2010

- 177 -

Disini

piece is just a method of compensation and does not define the essence of the relations.
Nor does the fact that petitioners are not covered by the SSS affect the employeremployee relationship
2. To justify a finding of abandonment of work, there must be proof of a deliberate and
unjustified refusal on the part of an employee to resume his employment. Private
respondents failed to discharge this burden. Other than the self-serving declarations in
the affidavits of their two employees, private respondents did not adduce proof of overt
acts of petitioners showing their intention to abandon their work. On the contrary, the
evidence shows that petitioners lost no time in filing the case for illegal dismissal against
private respondent. This fact negates any intention on their part to sever their
employment relationship
Disposition Petition granted

PULP AND PAPER INC V NLRC (ANTONIO)


279 SCRA 408
PANGANIBAN; September 24, 1997
NATURE
Instant petition for certiorari
FACTS
- A case of illegal dismissal and underpayment of wages was filed by MS. EPIFANIA
ANTONIO [private respondent herein] against PULP AND PAPER DISTRIBUTORS INC.,
[petitioner herein] She alleges that she was a regular employee of the corporation having
served thereat as Wrapper sometime in September 1975. On November 29, 1991, for
unknown reasons, she was advised verbally of her termination and was given a prepared
form of Quitclaim and Release which she refused to sign. Instead she brought the present
complaint for illegal dismissal. In charging the company of underpayment of wages,
complainant in the same position paper alleges that, rarely during her employment with
the respondent she received her salary, a salary which was in accordance with the
minimum wage law. She was not paid overtime pay, holiday pay and five-day service
incentive leave pay, hence she is claiming for payments thereof by instituting the present
case.
- Company denied having terminated the services of the complainant and alleges inter alia
that starting 1989 the orders from customers became fewer and dwindled to the point that
it is no longer practical to maintain the present number of packer/wrappers. Maintaining
the same number of packers/wrappers would mean less pay because the work allocation
is no longer the same as it was. Such being the case, the respondent has to reduce
temporarily the number of packers/wrappers. Complainant was among those who were
temporarily laid-off from work. Complainant last worked with the company on June 29,
1991.
- As regards complainant's allegation that on November 29, 1991, she was forced to sign
a quitclaim and release by the respondent, the latter clarified that considering that five
months from the time the complainant last worked with the company, the management
decided to release the complainant and give her a chance to look for another job in the
meantime that no job is available for her with the company. In other words, complainant
was given the option and considering that she did not sign the documents referred to as
the Quitclaim and Release, the respondent did not insist, and did not terminate the
services of the complainant. It was just surprise [sic] to receive the present complaint. In
fact, respondent added that the reason why the complainant was called on November 29,
1991 was not to work but to receive her 13th month pay of P636.70 as shown by the
voucher she signed.
- As regards the claim of the complainant for underpayment, respondent did not actually
denied the same but give the reservation that should the same be determined by this
Office it is willing to settle the same considering the fact that complainant herein being
paid by results, it is not in a proper position to determine whether the complainant was
underpaid or not.
ISSUES
1. WON the computation should apply the minimum wage and not the wage rate of
workers
2. WON the private respondent's separation pay should not have been computed at one
month's pay for every year of service because private respondent should be considered
retrenched, the separation pay should be "one month's pay or at least one/half (1/2) month
pay for every year of service, whichever is higher, and not one (1) month's pay for every
year of service as public respondent had ruled."
3. WON petitioner's challenge to the computation of salary differential must be dismissed
as "the work of the private respondent is seasonal, being dependent upon the availability
of job-orders" and not "twenty-six (26) days a month because as a piece worker whose
work was seasonal."
HELD
1. Minimum wage is applicable.

Labor Law 1
- In the absence of wage rates based on time and motion studies determined by the labor
secretary or submitted by the employer to the labor secretary for his approval, wage rates
of piece-rate workers must be based on the applicable daily minimum wage
determined by the Regional Tripartite Wages and Productivity Commission. To
ensure the payment of fair and reasonable wage rates, Article 101 of the Labor Code
provides that "the Secretary of Labor shall regulate the payment of wages by results,
including pakyao, piecework and other nontime work." The same statutory provision
also states that the wage rates should be based, preferably, on time and motion studies,
or those arrived at in consultation with representatives of workers' and employers'
organizations. In the absence of such prescribed wage rates for piece-rate workers,
the ordinary minimum wage rates prescribed by the Regional Tripartite Wages and
Productivity Boards should apply. This is in compliance with Section 8 of the Rules
Implementing Wage Order Nos. NCR-02 and NCR-02-A the prevailing wage order at the
time of dismissal of private respondent, viz:
Sec. 8. Workers Paid by Results. a) All workers paid by results including those who are
paid on piece work, takay, pakyaw, or task basis, shall receive not less than the
applicable minimum wage rates prescribed under the Order for the normal working
hours which shall not exceed eight (8) hours work a day, or a proportion thereof for
work of less than the normal working hours.
- The adjusted minimum wage rates for workers paid by results shall be computed in
accordance with the following steps:
1) Amount of increase in AMW x 100 = % increase Previous AMW
2) Existing rate/piece x % increase = increase in rate/piece;
3) Existing rate/piece + increase in rate/piece = adjusted rate/piece.
b) The wage rates of workers who are paid by results shall continue to be established in
accordance with Art. 101 of the Labor Code, as amended and its implementing
regulations.
- On November 29, 1991, private respondent was orally informed of the termination of her
employment. Wage Order No. NCR-02, in effect at the time, set the minimum daily wage
for non-agricultural workers like private respondent at P118.00. This was the rate used by
the labor arbiter in computing the separation pay of private respondent. We cannot find
any abuse of discretion, let alone grave abuse, in the order of the labor arbiter which was
later affirmed by the NLRC.
- Moreover, since petitioner employed piece-rate workers, it should have inquired from the
secretary of labor about their prescribed specific wage rates. In any event, there being no
such prescribed rates, petitioner, after consultation with its workers, should have submitted
for the labor secretary's approval time and motion studies as basis for the wage rates of its
employees. This responsibility of the employer is clear under Section 8, Rule VII, Book III
of the Omnibus Rules Implementing the Labor Code:
Sec. 8. Payment by result.
(a) On petition of any interested party, or upon its initiative, the Department of Labor
shall use all available devices, including the use of time and motion studies and
consultations with representatives of employers' and workers' organizations, to
determine whether the employees in any industry or enterprise are being compensated
in accordance with the minimum wage requirements of this Rule.
(b) The basis for the establishment of rates for piece, output or contract work shall be
the performance of an ordinary worker of minimum skill or ability.
(c) An ordinary worker of minimum skill or ability is the average worker of the lowest
producing group representing 50% of the total number of employees engaged in similar
employment in a particular establishment, excluding learners, apprentices and
handicapped workers employed therein.
(d) Where the output rates established by the employer do not conform with the
standards prescribed herein, or with the rates prescribed by the Department of Labor in
an appropriate order, the employees shall be entitled to the difference between the
amount to which they are entitled to receive under such prescribed standards or rates
and that actually paid them by employer.
- In the present case, petitioner as the employer unquestionably failed to discharge
the foregoing responsibility. Petitioner did not submit to the secretary of labor a
proposed wage rate based on time and motion studies and reached after
consultation with the representatives from both workers' and employers'
organization which would have applied to its piece-rate workers. Without those
submissions, the labor arbiter had the duty to use the daily minimum wage rate for
non-agricultural workers prevailing at the time of private respondent's dismissal, as
prescribed by the Regional Tripartite Wages and Productivity Boards. Put differently,
petitioner did not take the initiative of proposing an appropriate wage rate for its piece-rate
workers. In the absence of such wage rate, the labor arbiter cannot be faulted for applying
the prescribed minimum wage rate in the computation of private respondent's separation
pay. In fact, it acted and ruled correctly and legally in the premises.
- It is clear, therefore, that the applicable minimum wage for an eight-hour working day is
the basis for the computation of the separation pay of piece-rate workers like private
respondent. The computed daily wage should not be reduced on the basis of
unsubstantiated claims that her daily working hours were less than eight. Aside from its
bare assertion, petitioner presented no clear proof that private respondent's regular
working day was less than eight hours. Thus, the labor arbiter correctly used the full

A2010

- 178 -

Disini

amount of P118.00 per day in computing private respondent's separation pay. We agree
with the following computation:
- Considering therefore that complainant had been laid-off for more than six (6) months
now, we strongly feel that it is already reasonable for the respondent to pay the
complainant her separation pay of one month for every year of service, a fraction of six (6)
months to be considered as one whole year. Separation pay should be computed based
on her minimum salary as will be determined hereunder.
Separation pay 1 month = 16 yearsP118.00 x 26 x 16 years = P49,088.00
The amount "P118.00" represents the applicable daily minimum wage per Wage Order
Nos. NCR-02 and NCR-02-A; "26", the number of working days in a month after excluding
the four Sundays which are deemed rest days; "16", the total number of years spent by
private respondent in the employ of petitioner.
2. NO
- Petitioner misapprehended the ground relied upon by public respondent for awarding
separation pay. In this case, public respondent held that private respondent was
constructively dismissed, pursuant to Article 286 of the Labor Code which reads:
- Art. 286. When employment not deemed terminated. The bonafide suspension of the
operation of a business or undertaking for a period not exceeding six (6) months, or the
fulfillment by the employee of a military or civic duty shall not terminate employment. In all
such cases, the employer shall reinstate the employee to his former position without loss
of seniority rights if he indicates his desire to resume his work not later that one (1) month
from his resumption of operations of his employer or from his relief from the military or
civic duty.
- Petitioner failed to discern that public respondent, in finding that the services of private
respondent were terminated, merely adopted by analogy the rule on constructive
dismissal. Since private respondent was not reemployed within six (6) months from the
"suspension" of her employment, she is deemed to have been constructively dismissed.
Otherwise, private respondent will remain in a perpetual "floating status." Because
petitioner had not shown by competent evidence any just cause for the dismissal of private
respondent, she is entitled to reinstatement or, if this is not feasible, to separation pay
equivalent to one (1) month salary for every year of service. Private respondent, however,
neither asked for reinstatement nor appealed from the labor arbiter's finding that she was
not illegally dismissed; she merely prayed for the grant of her monetary claims. Thus, it is
sustained that the award of separation pay made by public respondent, for employees
constructively dismissed are entitled to separation pay. Because she did not ask for more,
we cannot give her more. We repeat: she appealed neither the decision of the labor arbiter
nor that of the NLRC. Hence, she is not entitled to any affirmative relief.
- Furthermore, we cannot sustain petitioner's claim that private respondent was
retrenched. For retrenchment to be considered a ground for termination, the employer
must serve a written notice on the workers and the Department of Labor and Employment
at least one month before the intended date thereof. Petitioner did not comply with this
requirement.
3. YES
- it should be dismissed. As earlier observed, private respondent is entitled to the minimum
wage prevailing at the time of the termination of her employment. The same rate of
minimum wage, P118.00, should be used in computing her salary differential resulting
from petitioner's underpayment of her wages. Thus, the labor arbiter correctly deducted
private respondent's actually received wage of P60 a day from the prescribed daily
minimum wage of P118.00, and multiplied the difference by 26 working days, and
subsequently by 16 years, equivalent to her length of service with petitioner. Thus, the
amount of P31,149.56 as salary differential.
- Contrary to the assertion of petitioner, neither the assailed Decision nor the pleadings of
private respondent show that private respondent's work was seasonal. More important,
petitioner utterly failed to substantiate its allegation that private respondent's work was
seasonal. We observe that the labor arbiter based the computation of the salary
differential on a 26-day month on the presumption that private respondent's work was
continuous. In view of the failure of petitioner to support its claim, we must sustain the
correctness of this computation.
Disposition petition is DISMISSED and the assailed Decision is AFFIRMED.

WAGE DISTORTION
NATIONAL FEDERATION OF LABOR V NLRC
(FRANKLIN BAKER COMPANY OF THE PHILIPPINES)
234 SCRA 311
FELICIANO; July 21, 1994
NATURE
Petition for Review on Certiorari
FACTS

Labor Law 1
- Wage Orders Nos. 3, 4, 5 and 6 were promulgated by the then Pres. Marcos. Before the
effectivity of Wage Order No. 3, the wage rates of regular employees and of casual (or
non-regular employees of private respondent Franklin Baker Company of the Philippines
(Davao Plant) ("Company") were such that there was a positive differential of P4.56
between the 2. The effect of the implementation of the successive Wage Orders upon the
daily wage rates of these two (2) groups of employees was summarized by petitioner in
the following table:
- Upon the effectivity of Wage Order No. 5, grievance meetings were held by petitioner
National Federation of Labor ("NFL") and private respondent Company sometime in June
1984, addressing the impact which implementation of the various Wage Orders had on the
wage structure of the Company. The wage rates of bith casual and regular employees
were the same at P34. Then all the casual or non-regular employees of private respondent
Company (at least in its Davao Plant) were "regularized," or converted into regular
employees, pursuant to the request of petitioner NFL.
- On 1 July 1984, the effectivity date of the 1984 CBA between NFL and the Company, all
regular employees of the Company received an increase of P1.84 in their daily wage; the
regular daily wage of the regular employees thus became P35.84 as against P34.00 per
day for non-regular employees.
- Through Wage Order No. 6, casual employees received an increase of their daily wage
from P34 to 36. At the same time, Company unilaterally granted an across-the-board
increase of P2 in the daily rate of all regular employees, increasing their daily wage from
P35.84 to P37.84. Further, on 1 July 1985, all regular employees who were members of
the collective bargaining unit got a raise of P1.76 in their basic daily wage, which pushed
that daily wage from P37.84 to P39.60, as against the non-regular's basic wage of
P36/day. And, by Nov 87, the lowest paid regular employee had a basic daily rate of
P64.64, P10.64 more than the statutory minimum wage paid to a non-regular employee.
- Petitioners principal contention: A wage distortion in the wage structure of private
respondent Company continued to exist although a gap of P1.84 between the daily wage
rate of regular employees and that of casual employees had been re-established upon the
effectivity of the CBA increase on 1 July 1984.
ISSUE
WON NLRC committed a grave abuse of discretion amounting to lack or excess of
jurisdiction, when it concluded that the wage distortion had ceased to exist, after 1 July
1984
HELD
NO
- The re-establishment of a significant gap or differential between regular employees and
casual employees by operation of the CBA was more than substantial compliance with the
requirements of the several Wage Orders (and of Article 124 of the Labor Code 39)
In this case the Court summarized the principles regarding wage distortion:
[a] The concept of wage distortion assumes an existing grouping or classification of
employees which establishes distinctions among such employees on some relevant or
legitimate basis. This classification is reflected in a differing wage rate for each of the
existing classes of employees.
[b] Wage distortions have often been the result of government-decreed increases in
minimum wages. There are however, other causes of wage distortions, like merger of 2
companies (with differing classifications of employees and different wage rates) where the
surviving company absorbs all the employees of the dissolved corporation.
[c] Should a wage distortion exist, there is no legal requirement that, in the rectification of
that distortion by readjustment of the age rates of the differing classes of employees, the
gap which had previously or historically existed or restored in precisely the same amount.
In other words correction of a wage distortion may be done by reestablishing a substantial
or significant gap (as distinguished from the historical gap) between the wage rates of the
differing classes of employees.
[d] The reestablishment of a significant difference in wage rates may be the result of
resort to grievance procedures or collecting bargaining negotiations.
The Court also emphasized that: Whether or not a new additional scheme of classification
of employees for compensation purposes should be established by the Company (and the
legitimacy or viability of the bases of distinction there embodied) is properly a matter for
management judgment and discretion, and ultimately, perhaps, a subject matter for
bargaining negotiations between employer and employees
Disposition Petition DISMISSED for lack of merit,

PRUBANKERS ASSOCIATION V PRUDENTIAL BANK &


TRUST COMPANY
39

Article 124. Standards/Criteria for Minimum Wage Fixing. - . . .


xxx
xxx
xxx
As used herein, a wage distortion shall mean a situation where an increase in prescribed wage rates results in the
elimination or severe contraction of intentional quantitative differences in wage or salary rates between and among
employee groups in an establishment as to effectively obliterate the distinctions embodied in such wage structure based
on skills, length of service, or other logical bases of differentiation.

A2010

- 179 -

Disini

302 SCRA 74
PANGANIBAN; January 25, 1999
FACTS
- On November 18, 1993, the Regional Tripartite Wages and Productivity Board of Region
V issued Wage Order No. RB 05-03 which provided for a Cost of Living Allowance (COLA)
to workers in the private sector who had rendered service for at least three months before
its effectivity, and for the same period thereafter. The amount of COLA depended on the
region where it was implemented
- On November 23, 1993, the Regional Tripartite Wages and Productivity Board of Region,
VII issued Wage Order No. RB VII-03, which directed the integration of the COLA
mandated pursuant to Wage Order No. RO VII-02-A into the basic pay of all workers. It
also established an increase in the minimum wage rates for all workers and employees in
the private sector. Again amount differed depending on the region
- The petitioner then granted a COLA of P17.50 to its employees at its Naga Branch, the
only branch covered by Wage Order No. RB 5-03, and integrated the P150.00 per month
COLA into the basic pay of its rank-and-file employees at its Cebu, Mabolo and P. del
Rosario branches, the branches covered by Wage Order No. RB VII-03
- Prubankers association told Prudential Bank to extend the application of the wage orders
to its employees outside Regions V and VII, claiming that the regional implementation of
the said orders created a wage distortion in the wage rates of the banks employees
nationwide
- they decided to submit the dispute for voluntary arbitration. Issue being won the
enforcement of the wage orders created a wage distortion. (there was no mention of the
decision so I think the ruled that it created a wage distortion)
- CA ruled that there was no distortion saying that the RA justified it. It noted, I that the
underlying considerations in issuing the wage orders are diverse, based on the distinctive
situations and needs existing in each region
ISSUE
WON a wage distortion resulted from respondent's implementation of the aforecited Wage
Orders
HELD
NO
- The statutory definition of' wage distortion I'S found Article 124 of the Labor Code, as
amended by Republic Act No. 6727:
As used herein, a wage distortion shall mean a situation where an increase in
prescribed wage results in the elimination or severe contraction of intentional
quantitative differences in wage or salary rates between and among employee groups
in an establishment as to effectively obliterate the distinctions embodied in such wage
structure based on skills, length of service, or other logical bases of differentiation
- Wage distortion involves four elements:
1. An existing hierarchy of positions with corresponding salary rates 2. A significant
change in the salary rate of a lower pay class without a concomitant increase in the salary
rate of a higher one 3. The elimination of the distinction between the two levels
4. The existence of the distortion in the same region of the country
- no wage distortion resulted when respondent implemented the subject Wage Orders ill
the covered branches-. In the said branches, there was an increase in the salary rates of
all pay classes. Furthermore, the hierarchy of positions based on skills, length of service
and other logical bases of differentiation was preserved
- petitioner maintains that, as a result of the two Wage Orders, the employees in the
affected regions have higher compensation than their counterparts of the same level in
other regions
- The Court is not persuaded. A wage parity between employees in different rungs is not at
issue here, but a zeage disparity between employees in the same rung but located in
different regions of the country
- a wage distortion arises when a wage order engenders wage parity, between employees
in different rungs of the organizational ladder of the same establishment. It bears
emphasis that wage distortion involves a parity in the salary rates of different pay classes
which, as a result, eliminates the distinction between the different ranks in the same region
- Petitioner's claim of wage distortion must also be denied for one other reason. The
difference in wages between employees in the same pay scale in different regions is not
the mischief sought to be banished by the law. In fact it recognizes the disparity.
- It must be understood that varying in each region of the country are controlling factors
such as the cost of living; supply and demand of basic goods, services and necessities;
and the purchasing power of the peso.

ASSOCIATED LABOR UNIONS-TUCP V NLRC


(MAGBANUA. DEL MONTE PHILIPPINES INC)
235 SCRA 395
MENDOZA; August 16, 1994

Labor Law 1
NATURE
Special civil action of certiorari to set aside the decision and resolution dated June 22,
1992 and September 14, 1992 respectively of the National Labor Relations Commission
FACTS
- On July 1, 1989, Republic Act No. 6727, otherwise known as the Wage Rationalization
Act, took effect, granting a P25.00/day increase in the statutory minimum wage of all
workers and employees in the private sector, subject to certain conditions.
- In implementation of the law, private respondent Del Monte Philippines, Inc. gave a
P25.00/day increase to the P54.00/day wages of its temporary employees or "broilers."
Because the regular employees, members of petitioner union, who were then receiving
P100.80 a day were not granted a similar increase, they complained to the management
of private respondent.
- On February 14, 1990, the parties executed a Memorandum Agreement wherein private
respondent, "in positive response to the union's representations and notwithstanding that it
has no legal or contractual obligation," granted the members of petitioner union a
P10.00/day wage increase effective January 1, 1990, subject to the latter's right to claim
P15.00/day as balance, through compulsory arbitration.
- On June 5, 1990, petitioners filed a complaint against private respondent in the National
Labor Relations Commission (NLRC). They alleged that a wage distortion had been
created. On appeal the NLRC affirmed the Labor Arbiter's findings and denied petitioner's
motion for reconsideration.
ISSUE
WON the wage increases mandated by the parties Collective Bargaining Agreement
should be considered (should take effect)
HELD
YES
- Art. 124 of the Labor Code, as amended by Republic Act No. 6727,
expressly provides that where the application of any prescribed wage increase by
virtue of a law or wage order issued by any Regional Board results in distortions of the
wage structure within an establishment, the employer and the union shall negotiate to
correct the distortions. The law recognizes, therefore, the validity of negotiated
wage increases to correct wage distortions. The legislative intent is to
encourage the parties to seek solution to the problem of wage distortions
through voluntary negotiation or arbitration, rather than strikes, lockouts, or
other concerted activities of the employees or management. Recognition and
validation of wage increases given by employers either unilaterally or as a result of
collective bargaining negotiations for the purpose of correcting wage distortions are in
keeping with the public policy of encouraging employers to grant wage and allowance
increases to their employees which are higher than the minimum rates of increases
prescribed by statute or administrative regulation.
- Apex Mining, Inc. v. NLRC
To compel employers simply to add on legislated increases in salary or allowances
without regard to what is already paid, would be to penalize employers who grant their
workers more than the statutorily prescribed minimum rates of increases. Clearly, this
would be counterproductive so far as securing the interest of labor is concerned.
- Cardona v. NLRC
There was no wage distortion where the employer made salary adjustments in terms of
restructuring of benefits and allowances and re was an increase pursuant to the CBA.
- Finally, whether or not a wage distortion exists by reason of the grant of a wage increase
to certain employees is essentially a question of fact. In this case, the findings of the Labor
Arbiter, affirmed by the NLRC, that no wage distortion exists being based on substantial
evidence, are entitled to respect and finality.
Disposition Petition dismissed

BANKARD EMPLOYEES UNION V NLRC (BANKARD


INC)

423 SCRA 148


CARPIO-MORALES; February 17, 2004
FACTS
- Bankard, Inc. (Bankard) classifies its employees by levels. On May 28, 1993, its Board of
Directors approved a "New Salary Scale" for the purpose of making its hiring rate
competitive in the industrys labor market. The "New Salary Scale" increased the hiring
rates of new employees. Accordingly, the salaries of employees who fell below the new
minimum rates were also adjusted to reach such rates under their levels.
- Bankards move drew the Bankard Employees Union-WATU (petitioner), the duly
certified exclusive bargaining agent of the regular rank and file employees of Bankard, to

A2010

- 180 -

Disini

press for the increase in the salary of its old, regular employees. Bankard took the
position, however, that there was no obligation on the part of the management to grant to
all its employees the same increase in an across-the-board manner.
- Petitioners filed Notices of Strike on the ground of discrimination and other acts of Unfair
Labor Practice. The strike was averted, however, when the dispute was certified by the
Secretary of Labor and Employment for compulsory arbitration.
- NLRC, finding no wage distortion, dismissed the case for lack of merit. MFR was denied.
Hence, this petition for certiorari.
ISSUE
WON the unilateral adoption by an employer of an upgraded salary scale that increased
the hiring rates of new employees without increasing the salary rates of old employees
resulted in wage distortion within the contemplation of Article 124 of the Labor Code

HELD
NO
Ratio Petitioner cannot make a contrary classification of private respondents employees
without encroaching upon recognized management prerogative of formulating a wage
structure, in this case, one based on level.
Reasoning
- While seniority may be a factor in determining the wages of employees, it cannot be
made the sole basis in cases where the nature of their work differs. Moreover, for
purposes of determining the existence of wage distortion, employees cannot create their
own independent classification and use it as a basis to demand an across-the-board
increase in salary.
- Apart from the findings of fact of the NLRC and the Court of Appeals that some of the
elements of wage distortion are absent, petitioner cannot legally obligate Bankard to
correct the alleged "wage distortion" as the increase in the wages and salaries of the
newly-hired was not due to a prescribed law or wage order. The wordings of Article 124
are clear. If it was the intention of the legislators to cover all kinds of wage adjustments,
then the language of the law should have been broad, not restrictive as it is currently
phrased.

- Moreover, Bankards right to increase its hiring rate,


to establish minimum salaries for specific jobs, and to
adjust the rates of employees affected thereby is
embodied under Section 2, Article V (Salary and Cost
of Living Allowance) of the parties Collective
Bargaining Agreement (CBA).
Disposition the present petition is hereby DENIED.

9.23 EFFECT OF BENEFITS


PRUBANKERS ASSN V PRUDENTIAL BANK AND CO
[PAGE 181]

SECTION 10: WOMEN WORKERS


10.01 WOMEN AND THE CONSTITUTION

Labor Law 1
WOMEN WORKERS
PHIL TELEGRAPH AND TELEPHONE CO V NLRC
[PAGE 26]

10.02 COVERAGE
10.03 PROHIBITED ACTS
1) NIGHT WORK AND EXCEPTION
2) DISCRIMINATION
3) MARRIAGE
4) GENERAL

10.04 FACILITIES
10.05 SPECIAL CLASSIFICATION SPECIAL
WOMEN WORKERS
10.06 MATERNITY LEAVE
10.07 SEXUAL HARRASSMENT
WORK RELATED ENVIRONMENT
EDUCATION OR TRAINING0RELATED ENVIRONMENT
DUTY EMPLOYER
LIABILITY EMPLOYER
REMEDIES
LIBRES V NLRC (NATIONAL STEEL CORPORATION,
ET AL)
307 SCRA 675
BELLOSILLO; May 28, 1999
NATURE
Petition for certiorari seeking to annul the decision of public respondent NLRC sustaining
the Labor Arbiters finding that petitioner was validly suspended by private respondents, as
well as the NLRC resolution denying petitioners motion to reconsider its decision
FACTS
- Petitioner Libres was an Assistant Manager at National Steel Corp.
- In August, 1993, he received a Notice of investigation requesting him to submit a written
explanation regarding the charge of sexual harassment made by Capiral, Hynsons (his
boss) secretary, and warned him that failure to file this would be construed as a waiver of
his right to be heard.
- He submitted his explanation, denying the accusation, offering to submit himself for
clarificatory interrogation
- Hynson conducted an internal investigation, letting Libres and Capiral provide their sides
of the issue
- Hynson Jr. submitted his report to the Management Evaluation Committee (MEC). The
MEC concluded that the charges against petitioner constituted a violation of the Plant's
Rules and Regulations. It opined that "touching a female subordinate's hand and
shoulder, caressing her nape and telling other people that Capiral was the one who
hugged and kissed or that she responded to the sexual advances are unauthorized acts
that damaged her honor". Referring to the Manual of the Philippine Daily Inquirer in
defining sexual harassment, the MEC finally concluded that petitioner's acts clearly
constituted sexual harassment as charged and recommended petitioner's suspension for
thirty (30) days without pay.
- Libres filed a complaint for illegal suspension and unjust discrimination against

A2010

- 181 -

Disini

respondent NSC and its officers, private respondents herein, before the Labor Arbiter.
Citing the failure of the MEC to grant him audience despite his offer to answer clarificatory
questions, petitioner claimed denial of due process.
- Petitioner primarily disputes the failure of the NLRC to apply RA No. 7877, "An Act
Declaring Sexual Harassment Unlawful in the Employment, Education or Training
Environment and for Other Purposes," in determining whether he actually committed
sexual harassment. He asserts that his acts did not fall within the definition and criteria of
sexual harassment as laid down in Sec. 3 of the law. 5 Specifically, he cites public
respondent's failure to show that his acts of fondling the hand and massaging the
shoulders of Capiral "discriminated against her continued employment," "impaired her
rights and privileges under the Labor Code," or "created a hostile, intimidating or offensive
environment."
- Petitioner questioned the set off since there was no call or notice for the payment of the
unpaid subscription, and that the alleged obligation is not enforceable.
- The NLRC held that a stockholder who fails to pay his unpaid subscription on call
becomes a debtor of the corporation and that the set-off of said obligation against the
wages and other due to petitioner is not contrary to law, morals, public policy
ISSUE
1. WON RA 7877 is applicable in this case (to determine whether he actually committed
sexual harassment.)
2. WON the delay in instituting the complaint shows that it was only an afterthought
3. WON the requirements of due process were sufficiently complied with
HELD
1. NO
Reasoning
- Republic Act No. 7877 was not yet in effect at the time of the occurrence of the act
complained of. It was still being deliberated upon in Congress when petitioner's case was
decided by the Labor Arbiter. As a rule, laws shall have no retroactive effect unless
otherwise provided, or except in a criminal case when their application will favor the
accused. Hence, the Labor Arbiter have to rely on the MEC report and the common
connotation of sexual harassment as it is generally as understood by the public. Faced
with the same predicament, the NLRC had to agree with the Labor Arbiter. In so doing, the
NLRC did not commit any abuse of discretion in affirming the decision of the Labor Arbiter.
2. NO
Reasoning
- it could be expected since Libres was Capiral's immediate superior. Fear of retaliation
and backlash, not to forget the social humiliation and embarrassment that victims of this
human frailty usually suffer, are all realities that Capiral had to contend with. Moreover, the
delay did not detract from the truth derived from the facts. Petitioner Libres never
questioned the veracity of Capiral's allegations. In fact his narration even corroborated the
latter's assertion in several material points. He only raised issue on the complaint's
protracted filing.
3. YES
Reasoning
- Due process as a constitutional precept does not always require a trial type proceeding.
It is satisfied when a person is notified of the charge against him and given an opportunity
to explain or defend himself. The essence is simply to be heard, an opportunity to seek a
reconsideration of the action complained of. Petitioner was given a Notice of Investigation,
he was asked to submit a a written explanation to his superior, he was allowed to air his
grievance in a private session, and upon the release of the suspension order made by the
MEC he was requested its reconsideration (but he was denied)
- The personal confrontation with the MEC officers, which he requested, was not
necessary, for they had already exhaustively presented their claims and defenses in
different fora. Litigants may be heard through pleadings, written explanations, position
papers, memoranda or oral arguments (Howevers Savings and Loan Association v
NLRC).
Disposition petition is DISMISSED, no grave abuse of discretion having been committed
by public respondent National Labor Relations Commission in upholding the suspension of
petitioner Carlos G. Libres as justified and in accordance with due process. Consequently,
its decision of 28 August 1995 as well as its resolution of 31 October 1995 is AFFIRMED.

PHIL AEOLUS AUTOMOTIVE UNITED CO V NLRC


(CORTEZ)
331 SCRA 237 / 382 PHIL 250
BELLOSILLO; April 28, 2000
NATURE
This petition seeks to set aside the Decision and the Resolution of NLRC which modified
the decision of the Labor Arbiter finding petitioners not guilty of illegal dismissal.
FACTS

Labor Law 1
- Petitioner Philippine Aeolus Automotive United Corporation (PAAUC) is a corporation
duly organized and existing under Philippine laws, petitioner Francis Chua is its President
while private respondent Rosalinda C. Cortez was a company nurseof petitioner
corporation until her termination on 7 November 1994.
- On 5 October 1994 a memorandum was issued by Ms. Myrna Palomares, Personnel
Manager of petitioner corporation, addressed to private respondent Rosalinda C. Cortez
requiring her to explain within forty-eight (48) hours why no disciplinary action should be
taken against her (a) for throwing a stapler at Plant Manager William Chua, her superior,
and uttering invectives against him on 2 August 1994; (b) for losing the amount of
P1,488.00 entrusted to her by Plant Manager Chua to be given to Mr. Fang of the CLMC
Department on 23 August 1994; and, (c) for asking a co-employee to punch-in her time
card thus making it appear that she was in the office in the morning of 6 September 1994
when in fact she was not. The memorandum however was refused by private respondent
although it was read to her and discussed with her by a co-employee. She did not also
submit the required explanation, so that while her case was pending investigation the
company placed her under preventive suspension for thirty (30) days effective 9 October
1994 to 7 November 1994.
- On 20 October 1994, while Cortez was still under preventive suspension, another
memorandum was issued by petitioner corporation giving her seventy-two (72) hours to
explain why no disciplinary action should be taken against her for allegedly failing to
process the ATM applications of her nine (9) co-employees with the Allied Banking
Corporation. On 21 October 1994 private respondent also refused to receive the second
memorandum although it was read to her by a co-employee. A copy of the memorandum
was also sent by the Personnel Manager to private respondent at her last known address
by registered mail.
- Meanwhile, private respondent submitted a written explanation with respect to the loss of
the P1,488.00 and the punching-in of her time card by a co-employee.
- On 3 November 1994 a third memorandum was issued to private respondent, this time
informing her of her termination from the service effective 7 November 1994 on grounds of
gross and habitual neglect of duties, serious misconduct and fraud or willful breach of
trust.
- On 6 December 1994 private respondent filed with the Labor Arbiter a complaint for
illegal dismissal, non-payment of annual service incentive leave pay, 13th month pay and
damages against PAAUC and its president Francis Chua.
- LA rendered a decision holding the termination of Cortez as valid and legal, at the same
time dismissing her claim for damages for lack of merit.
- NLRC reversed the decision of the LA and found petitioner corporation guilty of illegal
dismissal of private respondent Cortez.
ISSUES
1. WON the NLRC gravely abused its discretion in holding as illegal the dismissal of
private respondent
2. WON she is entitled to damages in the event that the illegality of her dismissal is
sustained
HELD
1. NO
- Cortez claims that as early as her first year of employment her Plant Manager, William
Chua, already manifested a special liking for her, so much so that she was receiving
special treatment from him who would oftentimes invite her "for a date," which she would
as often refuse. On many occasions, he would make sexual advances - touching her
hands, putting his arms around her shoulders, running his fingers on her arms and telling
her she looked beautiful. The special treatment and sexual advances continued during her
employment for four (4) years but she never reciprocated his flirtations, until finally, she
noticed that his attitude towards her changed. He made her understand that if she would
not give in to his sexual advances he would cause her termination from the service; and
he made good his threat when he started harassing her. She just found out one day that
her table which was equipped with telephone and intercom units and containing her
personal belongings was transferred without her knowledge to a place with neither
telephone nor intercom, for which reason, an argument ensued when she confronted
William Chua resulting in her being charged with gross disrespect.
- The Supreme Court, in a litany of decisions on serious misconduct warranting dismissal
of an employee, has ruled that for misconduct or improper behavior to be a just cause for
dismissal (a) it must be serious; (b) must relate to the performance of the employees
duties; and, (c) must show that the employee has become unfit to continue working for the
employer. The act of private respondent in throwing a stapler and uttering abusive
language upon the person of the plant manager may be considered, from a lay man's
perspective, as a serious misconduct. However, in order to consider it a serious
misconduct that would justify dismissal under the law, it must have been done in relation to
the performance of her duties as would show her to be unfit to continue working for her
employer. The acts complained of, under the circumstances they were done, did not in any
way pertain to her duties as a nurse. Her employment identification card discloses the
nature of her employment as a nurse and no other. Also, the memorandum informing her
that she was being preventively suspended pending investigation of her case was
addressed to her as a nurse.

A2010

- 182 -

Disini

2. YES
- The gravamen of the offense in sexual harassment is not the violation of the employee's
sexuality but the abuse of power by the employer. Any employee, male or female, may
rightfully cry "foul" provided the claim is well substantiated. Strictly speaking, there is no
time period within which he or she is expected to complain through the proper channels.
The time to do so may vary depending upon the needs, circumstances, and more
importantly, the emotional threshold of the employee.
- Private respondent admittedly allowed four (4) years to pass before finally coming out
with her employer's sexual impositions. Not many women, especially in this country, are
made of the stuff that can endure the agony and trauma of a public, even corporate,
scandal. If petitioner corporation had not issued the third memorandum that terminated the
services of private respondent, we could only speculate how much longer she would keep
her silence. Moreover, few persons are privileged indeed to transfer from one employer to
another. The dearth of quality employment has become a daily "monster" roaming the
streets that one may not be expected to give up one's employment easily but to hang on
to it, so to speak, by all tolerable means. Perhaps, to private respondent's mind, for as
long as she could outwit her employer's ploys she would continue on her job and consider
them as mere occupational hazards. This uneasiness in her place of work thrived in an
atmosphere of tolerance for four (4) years, and one could only imagine the prevailing
anxiety and resentment, if not bitterness, that beset her all that time. But William Chua
faced reality soon enough. Since he had no place in private respondent's heart, so must
she have no place in his office. So, he provoked her, harassed her, and finally dislodged
her; and for finally venting her pent-up anger for years, he "found" the perfect reason to
terminate her.
- In determining entitlement to moral and exemplary damages, we restate the bases
therefor. In moral damages, it suffices to prove that the claimant has suffered anxiety,
sleepless nights, besmirched reputation and social humiliation by reason of the act
complained of.22 [Art. 2217, New Civil Code of the Philippines.] Exemplary damages, on
the other hand, are granted in addition to, inter alia, moral damages "by way of example or
correction for the public good"23 [Art. 2229, id.] if the employer "acted in a wanton,
fraudulent, reckless, oppressive or malevolent manner."24 [Art. 2232, id.]
- Anxiety was gradual in private respondent's five (5)-year employment. It began when her
plant manager showed an obvious partiality for her which went out of hand when he
started to make it clear that he would terminate her services if she would not give in to his
sexual advances. Sexual harassment is an imposition of misplaced "superiority" which is
enough to dampen an employee's spirit in her capacity for advancement. It affects her
sense of judgment; it changes her life. If for this alone private respondent should be
adequately compensated. Thus, for the anxiety, the seen and unseen hurt that she
suffered, petitioners should also be made to pay her moral damages, plus exemplary
damages, for the oppressive manner with which petitioners effected her dismissal from the
service, and to serve as a forewarning to lecherous officers and employers who take
undue advantage of their ascendancy over their employees.
Disposition Decision of NLRC finding the dismissal of private respondent without just
cause and ordering petitioners to pay her back wages computed from the time of her
dismissal, which should be full back wages, is AFFIRMED. However, in view of the
strained relations between the adverse parties, instead of reinstatement ordered by public
respondent, petitioners should pay private respondent separation pay equivalent to one
(1) month salary for every year of service until finality of this judgment. In addition,
petitioners are ordered to pay private respondent P25,000.00 for moral damages and
P10,000.00 for exemplary damages.

SECTION 11: MINORS


11.01 MINORS AND THE CONSTITUTION
11.02 LAW
11.03 DISCRIMINATION
SECTION 12: HOUSEKEEPERS
12.01 COVERAGE
12.02 HOUSEHELPERS

Labor Law 1
12.03 NON-HOUSEHOLD WORK ASSIGNMENT
BARCENAS V NLRC (REV SIM DEE)
187 SCRA 498
GANCAYCO; April 19, 1989
FACTS
- In 1978, Chua Se Su (Su, for short) in his capacity as the Head Monk of the Buddhist
Temple of Manila and Baguio City and as President and Chairman of the Board of
Directors of the Poh Toh Buddhist Association of the Phils. Inc. hired the petitioner,
Filomena Barcenas, who speaks the Chinese language as secretary and interpreter.
- Her position required her to receive and assist Chinese visitors to the temple, act as
tourist guide for foreign Chinese visitors, attend to the callers of the Head Monk as well as
to the food for the temple visitors, run errands for the Head Monk such as paying the
Meralco, PLDT, MWSS bills and act as liaison in some government offices. Aside from her
pay and allowances under the law, she received an amount of P500 per month plus free
board and lodging in the temple.
- In December, 1979, Su assumed the responsibility of paying for the education of
Barcenas nephew. In 1981, Su and Barcenas had amorous relations. In May, 1982, or five
months before giving birth to the alleged son of Su on October 12, 1982, she was sent
home to Bicol. Upon the death of Su in July, 1983, she remained and continued in her job.
- . In 1985, Manuel Chua (Chua, for short) was elected President and Chairman of the
Board of the Poh Toh Buddhist Association of the Philippines, Inc. and Rev. Sim Dee (Dee,
for short) was elected Head Buddhist Priest. Thereafter, Chua and Dee discontinued
payment of her monthly allowance and the additional P500 effective 1983. In addition,
Barcenas and her son were evicted forcibly from their quarters in the temple by six police
officers. She was brought first to the Police precinct in Tondo and then brought to Aloha
Hotel where she was compelled to sign a written undertaking not to return to the Buddhist
temple in consideration of the sum of P10,000. She refused and Chua shouted threats
against her and her son. Her personal belongings including assorted jewelries were never
returned.
- The Labor Arbiter ruled for Barcenas but the NLRC reversed.

ISSUES
1. WON Barcenas was a regular employee of the Manila Buddhist Temple
2. WON Barcenas was illegally dismissed

HELD
1. YES
Reasoning
- We agree with the petitioner's claim that she was a regular employee of the Manila
Buddhist Temple as secretary and interpreter of its Head Monk, Su. As Head Monk,
President and Chairman of the Board of Directors of the Poh Toh Buddhist Association of
the Philippines, Su was empowered to hire the petitioner under Article V of the By-laws of
the Association which states:
"The President or in his absence, the Vice President shall represent the Association in
all its dealings with the public, subject to the Board, shall have the power to enter into
any contract or agreement in the name of the Association, shall manage the active
business operation of the Association, shall deal with the bank or banks."
- Chua and Dee, on the other hand, claimed that Barcenas was never an employee of the
Poh Toh Temple but a servant who confined herself to the temple and to the personal
needs of the late Chua Se Su and thus, her position is co-terminus with that of her master.
However, the work that she performed in the temple could not be categorized as mere
domestic work. Barcenas, being proficient in the Chinese language, attended to the
visitors, mostly Chinese, who came to pray or seek advice before Buddha for personal or
business problems; arranged meetings between these visitors and Su and supervised the
preparation of the food for the temple visitors; acted as tourist guide of foreign visitors;
acted as liaison with some government offices; and made the payment for the temple,
Meralco, MWSS and PLDT bills. Indeed, these tasks may not be deemed activities of a
household helper. They were essential and important to the operation and religious
functions of the temple.
2. NO
Reasoning
- Her status as a regular employee ended upon her return to Bicol in May, 1982 to await
the birth of her lovechild allegedly by Su. The records do not show that she filed any leave

A2010

- 183 -

Disini

from work or that a leave was granted her. Neither did she return to work after the birth of
her child on October 12,1982, whom she named Robert Chua alias Chua Sim Tiong
[Whoa, wait a minute! If youre alert youll realize that Sim is the NEW Head Monks name!
Hmmm dont you think something elses going on here? ]. The NLRC found that it was
only in July, 1983 after Su died that she went back to the Manila Buddhist Temple.
- She herself supplied the reason for her return. She stated:
"It was the death-bed instruction to her by Chua Se So to stay at the temple and to take
care of the two boys and to see to it that they finish their studies to become monks and
when they are monks to eventually take over the two temples as their inheritance from
their father."
- Thus, her return to the temple was no longer as an employee but rather as Su's mistress
who is bent on protecting the proprietary and hereditary rights of her son and nephew. In
her pleadings, the petitioner claims that they were forcefully evicted from the temple,
harassed and threatened by respondents and that the Poh Toh Buddhist Association is a
trustee corporation with the children as cestui que trust. These claims are not proper in
this labor case. They should be appropriately threshed out in the complaints already filed
by the petitioner before the civil courts. Due to these claims, we view the respondents'
offer of P10,000 as indicative more of their desire to evict the petitioner and her son from
the temple rather than an admission of an employer-employee relation.
- The petitioner's claim for unpaid wages since May, 1982 which she filed only in 1986,
has already prescribed. Under Article 292 of the Labor Code, all money claims arising
from employer-employee relations must be filed within three years from the time the cause
of action accrued, otherwise they shall forever be barred.
- Finally, while petitioner contends that she continued to work in the temple after Su died,
there is, however, no proof that she was re-hired by the new Head Monk. In fact, she
herself manifested that respondents made it clear to her in no uncertain terms that her
services as well as her presence and that of her son were no longer needed. However,
she persisted and continued to work in the temple without receiving her salary because
she expected Chua and Dee to relent and permit the studies of the two boys.
Consequently, under these circumstances, no employer-employee relationship could have
arisen.
Disposition Decision of the NLRC is AFFIRMED.

APEX MINING CO V NLRC


196 SCRA 251
GANCAYCO; April 22, 1991
NATURE
Special civil action for certiorari to annul NLRC decision
FACTS
- Sinclita Candida was employed by Apex Mining Company, Inc. to perform laundry
services at its staff house. At first, she was paid on a piece rate basis. Later, she was paid
on a monthly basis.
- While she was hanging her laundry, she accidentally slipped and hit her back on a stone.
She reported the accident to her immediate supervisor and to the personnel officer. As a
result of the accident she was not able to continue with her work.
- She was permitted to go on leave for medication and was offered P2k which was
eventually increased to P5k to persuade her to quit her job, but she refused the offer and
preferred to return to work. Petitioner did not allow her to return to work and dismissed her.
- Labor arbiter ordered Apex Mining Company to pay the complainant Salary Differential,
Emergency Living Allowance, 13th Month Pay Differential and separation pay of one
month for every year of service NLRC affirmed.
ISSUE
WON the househelper in the staff houses of an industrial company is a domestic helper
HELD
NO
- Petitioner is a regular employee
- Rule XIII, Section l(b), Book 3 of the Labor Code:
The term "househelper" as used herein is synonymous to the term "domestic servant"
and shall refer to any person, whether male or female, who renders services in and
about the employer's home and which services are usually necessary or desirable for
the maintenance and enjoyment thereof, and ministers exclusively to the personal
comfort and enjoyment of the employer's family.
- The foregoing definition clearly contemplates such househelper or domestic servant who
is employed in the employer's home to minister exclusively to the personal comfort and
enjoyment of the employer's family. The definition cannot be interpreted to include
househelp or laundrywomen working in staffhouses of a company
- The criteria is the personal comfort and enjoyment of the family of the employer in the
home of said employer.

Labor Law 1
- While it may be true that the nature of the work of a househelper, domestic servant or
laundrywoman in a home or in a company staffhouse may be similar in nature, the
difference in their circumstances is that in the former instance they are actually serving the
family while in the latter case, whether it is a corporation or a single proprietorship
engaged in business or industry or any other agricultural or similar pursuit, service is being
rendered in the staffhouses or within the premises of the business of the employer. In such
instance, they are employees of the company or employer in the business concerned
entitled to the privileges of a regular employee.
Disposition Petition dismissed

12.04 CONDITIONS OF EMPLOYMENT


ULTRA VILLA FOOD HOUSE V GENISTON
[PAGE 158]

SECTION 13: HOMEWORKERS


13.01 COVERAGE AND REGULATION
13.02 EMPLOYER

SECTION 14: TERMINATION OF EMPLOYMENT


A. GENERAL CONCEPT

14.01 SECURITY OF TENURE


A. NATURE OF SECURITY OF TENURE
SONZA V ABS-CBN BROADCASTING CORP
[PAGE 42]
ALHAMBRA INDUSTRIES V. NLRC (RUPISAN)
238 SCRA 232
BELLOSILLO; November 18, 1994
NATURE
Special civil action in the Supreme Court. Certiorari
FACTS
- Alhambra employed Rupisan as salesman on 6-mo probationary basis. Alhambra made
surprise audit, alleged violations were purportedly committed by him. He was placed
under 1-mo preventive suspension. He protested. He alleges that charges against him
had become academic when he was given clearance of all accountabilities.
- A day before end of suspension, he was terminated. He sued Alhambra.
- Labor Arbiter found that the termination was for just cause, but there was a violation of
due process (failure to furnish copy of audit report).
- Both parties appealed to NLRC which affirmed Arbiters findings.
ISSUE
WON NLRC committed grave abuse of discretion in sustaining finding of Labor Arbiter that
Rupisan was illegally dismissed but directing his reinstatement so he could have
explained
HELD
YES
- Employment is no longer just an ordinary human acctivity. For most families the main
source of their livelihood, employment has now leveled off with property rights which no
one may be deprived of without due process of law.

A2010

- 184 -

Disini

- Termination of employment is not anymore a mere cessation or severance of contractual


relationship but an economic phenomenon affecting members of the family. This explains
why under the broad principles of social justice the dismissal of employees is adequately
protected by the laws of the state.
- A termination without just cause entitles a worker to reinstatement regardless of whether
he was accorded due process. On the other hand, termination of a worker for cause, even
without procedural due process, does not warrant reinstatement, but the employer incurs
liability for damages.
- Since the Labor Arbiter found a valid ground for dismissal, it erred when it directed
reinstatement.
- To order reinstatement and compel the parties to start the procedure from step one
would be circuitous because almost invariably that same issue of validity of the ground of
dismissal would be brought back to the Labor Arbiter for adjudication. We laid down in
Wenphil Corporation v. NLRC3 that an otherwise justly grounded termination without
procedural due process would only sanction payment of damages
- Standards of due process in judicial as well as administrative proceedings have long
been established. In its bare minimum due process of law simply means giving notice and
opportunity to be heard before judgment is rendered.
- When the private respondent filed a complaint against petitioner, he was afforded the
right to an investigation by the labor arbiter.
- Although belatedly, private respondent was afforded due process before the labor arbiter
wherein the just cause of his dismissal had been established. With such finding, it would
be arbitrary and unfair to order his reinstatement with backwages.
- It will be highly prejudicial to the interests of the employer to impose on him the services
of an employee who has been shown to be guilty of the charges that warranted his
dismissal from employment. Indeed, it will demoralize the rank and file.
- However, the petitioner must nevertheless be held to account for failure to extend to
private respondent his right to an investigation before causing his dismissal.

MANILA ELECTRIC COMPANY V NLRC (LOMABAO,


MASAYA)
186 SCRA 763
NARVASA; July 2, 1991
NATURE
CERTIORARI
FACTS
- Jose Masaya made an unauthorized electric service connection which supplied electricity
to the house of Antonio Sanchez (who paid the former Php 200 for making the said
connection.)
- Sanchez neither applied with Meralco for electric service nor made the requisite deposit
for it.
- This clandestine and illicit connection was eventually discovered by Meralco who then
charged him (through a letter) with a violation of the Company Code on Employee
Discipline, and thereafter conducted a formal investigation of the matter.
- Those who gave testimony at that investigation were Jose Masaya himself, and Renato
Repuyan, Meralco field investigator.
- Prior to being interrogated about the illegal connection and in response to preliminary
questions by the investigator, Masaya stated for the record that he had received the letter
accusing him of misconduct, that he had a copy of the code of discipline and understood
the nature of the precise charge against him, and that he did not need to be assisted by a
lawyer or a representative of his Union because he said that what he was about to say
was pawing katotohanan lamang.
- Repuyan testified on the fact of the undenied and indisputable installation of the illegal
electrical connection at the residence of Antonio Sanchez (his description of the manner of
its accomplishment being substantially the same as Masaya's own), and also, the
disclosures made to him by Sanchez's house helpers and the owner of the house
- After the investigation, and on the basis of the results thereof, Meralco filed with the
Ministry of Labor and Employment an application for clearance to terminate Masaya's
services, serving copy on the latter.
- Meralco also placed Masaya under preventive suspension.
- A week later, Masaya filed a complaint for illegal dismissal against Meralco.
- After the trial, LA Andres M. Lomabao rendered a decision in Masaya's favor; saying that
the record of the investigation conducted by Meralco should not be accorded credence;
that Meralco's contention that Masaya had "surreptitiously effected the direct connection of
. . . electric service" was not credible, because Masaya "was employed as a bill collector,
not as a lineman collector, hence, he does not know how to install electrical connection;"
and that the money received by Masaya from Sanchez (P200 or P250) was not in
consideration of any clandestine connection but was accepted as "representation
expenses in following up Mr. Sanchez' application for installation of electric facilities . . .
with the Engineer's Office at the City Hall of Manila.

Labor Law 1
- NLRC affirmed the Arbiter's decision; said that since Meralco was charging Masaya of a
criminal offense, it should prove beyond reasonable doubt (pbrd) said crime which it was
not able to do as it was not shown that Masaya was given the opportunity to be heard by
counsel or at least, a representative to confront his accuser; that based on the doctrine of
PBRD, there is no causal connection between Masaya' s duties to the crime imputed to
him, mere substantial evidence is insufficient to hold Masaya guilty of installing electrical
connection let alone deprive him of his right to labor."
ISSUE
WON the LA & the NLRC committed GABD in failing to take into consideration or
excluding Masayas admissions in their prononcement that Masaya was illegally dismissed
HELD
YES
Reasoning
NLRCS ERROR:
- Masaya was in truth asked if he wished to be assisted by a lawyer or a representative of
his Union, and his response was in the negative because, in his own words, "ang
sasabihin ko naman dito ay pawang katotohanan lamang"
- In administrative or quasi-judicial proceedings, PBRD is not required as basis for a
judgment of the legality of an employer's dismissal of an employee, nor even
preponderance of evidence, substantial evidence being sufficient.
- LC: the rules of evidence prevailing in courts of law or equity shall not be controlling and
it is the spirit and intention of this Code that the Commission and its members and the
Labor Arbiters shall use every and all reasonable means to ascertain the facts in each
case speedily and objectively and without regard to the technicalities of law or procedure,
all in the interest of due process. . . .
- SC: the ground for an employer's dismissal of an employee need be established only by
substantial evidence.
- It is absolutely of no consequence that the misconduct with which an employee may be
charged also constitutes a criminal offense
-The proceedings being administrative, the quantum of proof is governed by the
substantial evidence rule and not, as the respondent Commission seems to imagine, by
the rule governing judgments in criminal actions.
-The Court cannot close its eyes to the following facts of record, to wit:
1) the reality of the illegal electrical connection;
2) the letter to Masaya accusing him of misconduct
3) Masaya's acknowledgment that, having a copy of the company's code of discipline,
he understood the nature of the accusation against him, and his declining to be
assisted by a lawyer or a representative of his Union because, according to him, "ang
sasabihin ko naman dito ay pawang katotohanan lamang;"
4) his voluntary admission that it was he who had made the illegal electrical
connection, describing the manner by which he had made it, and that he had received
P250.00 from the occupant of the house, Antonio Sanchez; and
5) his plea to the company for forgiveness for having made the illegal connection.
- on record: testimony regarding identification of Masaya by Antonio Sanchez' servants
and by Castaeda, the owner of the house occupied by Sanchez.
- nothing in the record to demonstrate that Masaya's admissions were made otherwise
than voluntarily.
- Such an offense is obviously of so serious a character as to merit the penalty of
dismissal from employment, as stated in the Meralco Code on Employee Discipline:
SECTION 7. Dishonesty.
xxx xxx xxx
3) Directly or indirectly tampering with electric meters or metering installation of the
Company or the installation of any device, with the purpose of defrauding the
Company.
-The Labor Code pronounces "fraud or willful breach by the employee of the trust
reposed in him by his employer or duly authorized representative," or "serious
misconduct" on the part of the employee to be lawful ground to terminate
employment.
Ratio And this Court has held that the "dismissal of a dishonest employee is as much
in the interests of labor as it is of management. The labor force in any company is
protected and the workers' security of tenure strengthened when pilferage of
equipment, goods and products which endangers the viability of an employer and,
therefore, the workers' continued employment is minimized or eliminated and
consequently labor-management relations based on mutual trust and confidence
are promoted."
(*IN short: Tenurial Security is not an absolute right for the law provides that an employee
may be dismissed for just cause. )
Disposition Petition for certiorari is GRANTED, the decisions of the NLRC and LA are
ANNULLED AND SET ASIDE, and the petitioner's termination of the employment of
private respondent is AUTHORIZED and APPROVED

CITYTRUST BANKING CORPORATION V NLRC (RUIZ)

A2010

- 185 -

Disini

258 SCRA 621


MENDOZA; July 11, 1996
NATURE
Special civil action in the Supreme Court. Certiorari
FACTS
- Private respondent Ruiz was the internal auditor of petitioner Citytrust Banking
Corporation. She was designated manager of the Quiapo branch of the bank, but she
refused the appointment on the ground that it was a demotion. As a consequence, she
was suspended and, upon clearance given by the Department of Labor, she was
terminated on November 8, 1974.
- Private respondent filed a complaint for illegal dismissal. She was ordered reinstated as
branch manager, the NLRC urging her to accept the position, otherwise her refusal would
be considered a ground for her loss of employment. Private respondent appealed to the
Minister of Labor (now Secretary of Labor and Employment) but again she lost. Both
parties then appealed to the Office of the President, which ordered petitioner to reinstate
private respondent to her former position as internal auditor and to pay her backwages
from the time her compensation was withheld up to the time of her reinstatement.
- Petitioner moved for a reconsideration on the ground that the position of internal auditor
had been abolished (although the position of resident inspector was created in its stead),
and therefore in lieu of reinstatement, it should only be made to pay private respondent's
separation pay. The Office of the President modified its decision and ordered petitioner to
reinstate private respondent to a substantially equivalent position without loss of seniority
rights and to grant her the benefits and privileges to which she would be entitled had she
not been dismissed.
- Subsequently, petitioner reinstated private respondent as manager of the Auditing
Department. Private respondent accepted the appointment but questioned her
reinstatement to that position on the ground that it was not substantially equivalent to the
position of resident inspector (the position created in place of internal auditor). She also
questioned the award of backwages as the report of the socio-economic analyst allegedly
did not include backwages from April 1974 to June 1974 when she was on leave with pay
and vacation and sick leave in 1974 and other fringe benefits to which she was entitled
before her termination.
- Labor Arbiter Apolinario N. Lumabao issued an order holding that the position of manager
of the Auditing Department was not substantially equivalent to that of resident inspector.
possible as it appears (that) the position is already filled up (,) to relocate complainant to a
substantially equivalent position with all the emoluments and privileges of a Resident
Inspector. Respondent is hereby further ordered to pay.
- The NLRC affirmed the Labor Arbiter's order with modification by ordering the following
to be added to the award: (a) Her vacation and sick leave privilege during the period of her
separation in accordance with the disposition hereinbefore stated in the body of this
Resolution, and (b)the normal increases which complainant would have received during
the period of her separation.
- In connection with the computation of the award in her favor, private respondent sought
the production of the bank's payrolls for 1974-1981. Her motion was opposed by
petitioner which offered instead P74,344.00, the total amount of backwages as computed
by the socio-economic analyst of the Department of Labor, plus P9,040.00 in
transportation allowance and P1,050.00 mid-year bonus for 1974.
- Private respondent refused the offer, hence the NLRC directed the analyst to compute
the award on the basis of the payrolls from 1974 to 1981. Petitioner appealed to the
NLRC en banc, but its petition was dismissed, on the ground that the order appealed from
was interlocutory.
- Petitioner filed a petition for Certiorari and Prohibition with this Court, assailing the
dismissal of its appeal. The petition was at first dismissed for lack of merit. Petitioner's
motion for reconsideration was also dismissed. On July 21, 1986 this Court modified its
decision and petitioner was ordered to pay private respondent "backwages limited to three
years without qualification or deduction at the salary rate of private respondent at the time
of dismissal."
- The Labor Arbiter issued an alias writ of execution after finding that the amount
corresponded to the amount found due private respondent in the decision of the NLRC
and the resolution of this Court, consisting of salary differentials and other fringe benefits
which were not paid to her from the time that she was reinstated on August 14, 1978 as
manager of the Auditing Department.
- Petitioner moved to quash the alias writ of execution. As its motion was denied, it filed a
petition for Injunction in the NLRC en banc to stop the implementation of the alias writ of
execution and prayed for a recomputation of the monetary award pursuant to this Court's
resolution of July 21, 1986. Its petition was, however, denied, as was its motion for
reconsideration, in the resolutions of the NLRC. Hence, this petition.
ISSUE
WON private respondent is entitled to only three years of backwages and no more
HELD

Labor Law 1
NO
- Private respondent is, in addition, entitled to reinstatement without loss of seniority rights.
Art. 280 of the Labor Code provides:
ART. 280. Security of Tenure. In cases of regular employment, an employer shall not
terminate the services of an employee except for a just cause or when authorized by
this title. An employee who is unjustly dismissed from work shall be entitled to
reinstatement without loss of seniority rights and to his backwages computed
from the time his compensation was withheld from him up to the time of his
reinstatement. (emphasis supplied)
- Backwages are for earnings which a worker has lost due to his illegal dismissal. Private
respondent was illegally dismissed from November 8, 1974 to August 13, 1978. In its May
28, 1985 Report, the socio-economic analyst computed private respondent's backwages
for this period but he erroneously considered as backwages private respondent's salary
differential from August 14, 1978 to October 31, 1984. On August 14, 1978, private
respondent had already been reinstated, albeit to a lower paying position as manager of
the Auditing Department. Hence the award of backwages should be up to August 13,
1978 only. What she was entitled to receive after that date was the difference between
the salary of internal auditor (resident inspector) and that of manager of the Auditing
Department to which she was actually appointed. This position, as already noted, was
found to be not a substantially equivalent position to that of internal auditor or resident
inspector.
- The resolution of July 21, 1986 of this Court, which limited the award of backwages,
referred to the backwages for the period November 8, 1974 to August 13, 1978 as
component of the relief granted by law to those who are illegally dismissed. The Court at
that time limited the award of backwages to three years without qualification and deduction
to avoid delays incident to the determination of the earnings of the laid-off employees
during the pendency of the case and of deducting them from the backwages later
awarded.
- The second component of the relief granted under then Art. 280 of the Labor Code was
reinstatement either to their former position or if, this was not possible, to a substantially
equivalent position. Reinstatement contemplates a restoration to a position from which
one has been removed or separated so that the employee concerned may resume the
functions of the position he already held. Private respondent was the internal auditor of
petitioner at the time of her dismissal. Since this position had been replaced by the
position of resident inspector, private respondent should have been appointed resident
inspector. The position of manager of the Auditing Department to which she was
appointed was not a substantially equivalent position, as found by the Labor Arbiter in his
order of February 26, 1979 and later by the NLRC.
- The order to reinstate an employee to a former position or to a substantially equivalent
position is a positive mandate of the law with which strict compliance is required. This is
an affirmation that those deprived of a recognized and protected interest should be made
whole so that the employer will not profit from his misdeeds.
- Since private respondent retired from the bank on March 1, 1991, reinstatement is now
academic. She should therefore be paid the difference in pay of a resident inspector and
a manager of the Auditing Department from August 14, 1978 up to March 1, 1991.
Disposition Petition dismissed.

PHILIPS SEMICONDUCTORS V FADRIQUELA


[PAGE 77]
QUIJANO V BARTOLABAC
480 SCRA 204
TINGA; January 27, 1999
FACTS
- Quijano was employed by Mercury Drug Corporation as a warehouseman --- a
clerical/rank and file position. He was dismissed, so he filed a complaint with the NLRC for
illegal dismissal. The case reached the SC. In 1998, the SC ruled for his reinstatement to
his old position or to a substantially similar position. The SC denied the companys mfr,
and came out with a resolution in 1999 for Quijanos reinstatement.
- Whats this case all about, then? The respondents in this case are the LA and the NLRC
commissioner, respectively. Quijano filed a case against then for violation of Canon 1 and
Rule 1.01 of the Code of Professional Responsibility. WHY? They gave out orders contrary
to the resolution of the SC. The LA said to make him self-service attendant because accdg
to mercury there were only 4 positions open. All 4 positions required college graduates,
but LA said he thinks Quijano could handle the self-service attendant job. The NLRC
commissioner said since there are no available positions, he should just be given
separation pay.
ISSUE
WON Bartolabac & Quimpo erred
HELD

A2010

- 186 -

Disini

YES
- The decision of the SC was already final and executory. They had no place to use
discretion in executing a final and executory order of the Supreme Court. SUPREME. If
the final & executory orders of the SC would be second-guessed by other bodies, then
cases would never reach finality. The implementation of the final and executory decision is
mandatory. (The court was disappointed in the IBP recommendation to dismiss the
complaint against Bartolabac & Quimpo.)
- The SC wont compel to instantly restore the position of warehouseman if it had already
been abolished. It ruled that Quijano should be reinstated to original or substantially
similar position. They took notice of Mercury Drugs nationwide operation. SC couldnt
believe that they wouldnt have a position for Quijano.
- Our Constitution mandates that no person shall be deprived of life, liberty, and property
without due process of law. It should be borne in mind that employment is considered a
property right and cannot be taken away from the employee without going through legal
proceedings. In the instant case, respondents wittingly or unwittingly dispossessed
complainant of his source of living by not implementing his reinstatement. In the process,
respondents also run afoul of the public policy enshrined in the Constitution ensuring the
protection of the rights of workers and the promotion of their welfare.
Disposition Bartolabac & Quimpo suspended from the practice of law for 3 months for
violation of Canon 1 and Rule 1.01 of CPR.

B. IMPORTANCE OF EMPLOYMENT
EMPLOYMENT
GONZALES V NLRC (ATENEO DE DAVAO
UNIVERSITY)
313 SCRA 169
BELLOSILLO; August 26, 1999
FACTS
- Lorlene Gonzales was a Grade 6 teacher in Ateneo de Davao University from 1974 to
1993, when she was terminated. In 1991, the Grade School Headmaster sent her a letter
informing her of 2 complaints from parents of her students for alleged use of corporal
punishment. She demanded to know who the parents were because Ateneo wouldnt tell
her. When she found out that Ateneo was soliciting complaints from parents of her
students, she demanded an investigation.
- Ateneo sent her a notice of investigation, schedule, Committee composition, affidavits
of the parents, and the rules of procedure. She refused to take part in the investigation
unless the rules of procedure were revised. The committee, under advise of counsel, did
not revise the rules, since it had been used for a different teacher in the past. The
investigation went on, without her participation. In 1993, she was asked to tender her
resignation, otherwise she would be considered resigned.
- Lorlene filed for illegal dismissal with the LA. The LA found that she was indeed illegally
dismissed because although she was afforded due process, Ateneo failed to establish
substantial evidence as to Lorlenes guilt. It was established that she is a very good
teacher, equipped with the appropriate educational qualifications, trainings, seminars and
work experiences. Such fact was affirmed by her present and former students, their
parents, colleagues and the former headmaster of the grade school. As a matter of fact, 6
out of the nine 9 students and their parents/guardians retracted and withdrew their
statements.
- NLRC reversed LAs decision, saying the dismissal was valid and legal.
ISSUE
WON dismissal was valid and legal
HELD
NO
- In view of the foregoing, the conclusion of the NLRC is unwarranted.
No due process The committee refused to revise the rules of procedure. As a result,
Lorlene wasnt afforded a chance defend herself and to examine / cross-examine the
accusers.
Failure to prove by substantial evidence The evidence of Ateneo didnt measure up to
the standard laid down in Ang Tibay v CIR: "substantial evidence is more than mere

Labor Law 1
scintilla. It means such relevant evidence as a reasonable mind might accept as adequate
to support a conclusion."
Lorlenes evidence She was able to prove that shes a competent and dedicated teacher
of Ateneo for 17 years.
- Employment is not merely a contractual relationship; it has assumed the nature of
property right. It may spell the difference whether or not a family will have food on their
table, roof over their heads and education for their children. It is for this reason that the
State has taken up measures to protect employees from unjustified dismissals. It is also
because of this that the right to security of tenure is not only a statutory right but, more so,
a constitutional right.
Disposition NLRC decision reversed and set aside. LA decision reinstated, affirmed and
adopted.

C. STATE REGULATION - RATIONALE


RATIONALE
LLOSA-TAN V SILAHIS INTERNATIONAL HOTEL
181 SCRA 738
PARAS; February 5, 1990
NATURE
Petition for certiorari seeking to set aside the decision and resolutions of the NLRC
FACTS
- The complainant was a front office cashier of Silahis International Hotel since November
2, 1976 until her questioned dismissal on October 30, 1982.
- Since 1977, the Silahis International Hotel, had a standing corporate policy (Corporate
Policy No. 014), which orders all cashiers of SMC and its affiliates to refuse the cashing of
personal checks of employees and officials, endorsement by any executive of the Sulo
Management Company, or Philippine Village Hotel or Silahis International Hotel or Sulo
Hotel notwithstanding, because based on experience, a number of these checks
unfortunately bounce to the detriment of SMC and its affiliates.
- On August 22, 1982, while petitioner was on duty, she was approached by Mr.
Gayondato, the general cashier of Puerto Azul Beach Resorta sister company of Silahis
International Hotel and nephew of the Executive Vice President, to encash two (2) US
dollar checks with a combined value of US$1,200.00 or P10,389.60.
- Although petitioner politely explained the existence of Policy No. 014 prohibiting such
transactions, Gayondato persisted and assured that the presentation of aforesaid checks
to the front office cashier was upon instructions of the Executive Vice President.
- Petitioner, eventually encashed the aforesaid checks, notwithstanding Corporate Policy
No. 014.
- Thereafter, the said checks bounced.
- On October 1, 1982, respondent Vanessa Suatengco issued a memorandum to the
petitioner requiring her to explain in writing why she should not be terminated for
encashing the two (2) personal checks without proper authorization.
- Despite petitioner's explanation, her services were terminated effective October 30,
1982.
- Petitioner filed a complaint against respondents for illegal dismissal.
- Labor Arbiter Virginia G. Son rendered a decision in favor of petitioner.
- Hotel appealed the decision of the LA to the NLRC, and the NLRC rendered a decision
setting aside the decision of the Labor Arbiter and dismissing the complaint for illegal
dismissal for lack of merit
- Petitioners 2 MFRs having been denied, recourse was made to the SC
ISSUE

A2010

- 187 -

Disini

WON the acts of petitioner constitute gross negligence resulting in a valid ground for the
termination of her employment
HELD
NO
- Gross negligence has been defined as the want of any or slight care or the utter
disregard of consequences.
- Admittedly, the encashment of the checks in question is a violation of Policy No. 014 of
said hotel. But as found by the Labor Arbiter, it was established that: (a) complainant was
not motivated by bad faith; (b) Policy No. 014 is not strictly or consistently enforced but
has been relaxed repeatedly to meet business exigencies; and (c) complainant's
encashment of the checks in question was not only with the knowledge but with clearance
from her superiors who are more knowledgeable as to the circumstances under which the
enforcement of the same may be relaxed.
- Moreover, it cannot be said that complainant was precipitate or that she has acted in
utter disregard of consequences. On the contrary, she refused to encash subject checks
despite the request of Mr. Gayondato, the general cashier of Puerto Azul, but was
persuaded only upon the assurances of the latter that such was the wish of the Executive
Vice President and that said encashment was necessary to meet certain disbursements in
Puerto Azul. In addition, she informed personally Mr. Samuel Grulla, Assistant Manager of
the Silahis International Hotel, of said encashment, who also told her that such is "alright".
- Finally, against the background of her previous experience when she refused to encash
a similar check for Mr. Katte, the Food and Beverage Manager of Silahis International
Hotel, and that she was reprimanded by the management of the Silahis International Hotel
for her refusal, as well as threatened with suspension or dismissal from her job, coupled
with the advice of Mr. Nestor Famatigan, Jr., Silahis International Hotel Comptroller, to use
her discretion in handling similar requests in the future, it is not at all surprising that she
opted to take subject course of action.
- It is well settled that dismissal based on loss of trust and confidence arising from alleged
misconduct of employee, is not to be used as a shield to dismiss an employee arbitrarily.
Although the power to dismiss is a normal prerogative of the employer, the same is not
without limitations. The right of the employer must not be exercised arbitrarily and without
just cause. Otherwise, the constitutional guarantee of security of tenure of the workers
would be rendered nugatory. While dismissing or laying off of an employee is a
management's prerogative, it must nevertheless be done without abuse of discretion.
Furthermore, the right of employer to freely select or discharge his employees is regulated
by the State, because the preservation of the lives of the citizens is a basic duty of the
State, more vital than the preservation of the corporate profit. In addition, security of
tenure is a right of paramount value guaranteed by the Constitution and should not be
denied on mere speculation. Protection for labor and social justice provisions of the
Constitution and the labor laws and rules and regulations are interpreted in favor of the
exercise of labor rights.
Disposition The assailed decision of the NLRC is DISMISSED, and SET ASIDE and
private respondent Silahis International Hotel is ordered to reinstate petitioner Anita LlosaTan to her former position or similar position without loss of seniority rights with full
backwages beginning October 30, 1982 for a period of three (3) years therefrom.

D. COVERAGE
CONTRACT EMPLOYEE
LABAJO V ALEJANDRO
165 SCRA 747
FELICIANO; September 26, 1988
NATURE
Petition for certiorari with preliminary injunction to review NLRC resolution
FACTS
- The 6 private respondents had all been contracted by the petitioners to work as
classroom teachers at the San Andres HS, a private learning institution situated in
Maramag, Bukidnon. They then filed a complaint before the Ministry of Labor and
Employment, alleging that they had each received a letter from petitioner Fr. Labajo,
Director of the San Andres High School which contained: Please be informed that your
service at the San Andres High School will be terminated effective March 31, 1985.Thank
you for all services you have rendered to the school. Thus, their dismissal was without
justifiable cause and violated their rights to due process and security of tenure.
Petitioners Claims
> It was admitted that they had not paid in full the employment benefits claimed by the
teachers. It was alleged, however, that private respondents, prior to their acceptance of
teaching jobs at the San Andres High School, "were already made aware that the school

Labor Law 1
could not give them everything due them under existing laws" and, hence, were estopped
from claiming such benefits.
> At time of their dismissal, they were merely probationary employees of the San Andres
HS whose services were terminated for just cause (upon expiration on 31 March 1985 of
their respective contracts and before any of them had achieved regular or permanent
status in their jobs.)
* Labor Arbiter ruled in favor of the teachers. It held that they were not probationary
employees, and that they could only be dismissed for cause and only after having been
accorded due process.
* NLRC affirmed Labor Arbiters decision.
ISSUE
WON the respondents were illegally dismissed
HELD
NO
Ratio As probationary and contractual employees, private respondents enjoyed security
of tenure, but only to a limited extent i.e., they remained secure in their employment
during the period of time their respective contracts of employment remained in effect. As
petitioners were not under obligation to renew those contracts of employment, the
separation of private respondents in this case cannot be said to have been without
justifiable cause, much less illegal.
Reasoning
- Par 75 of the Manual of Regulations for Private Schools is applicable in this case: Fulltime teachers who have rendered three years of satisfactory service shall be considered
permanent. This 3-year period is the maximum period or upper limit of probationary
employment allowed. Whether or not one has indeed attained permanent status in one's
employment, before the passage of 3 years, is a matter of proof.
- NONE of them had been able to accumulate at least 3 years of service with the San
Andres HS at the time of their separation.
- Private respondent AMAR argued that the 12 years of teaching experience he had
accumulated prior to his acceptance of employment at San Andres qualified him as a
regular employee thereof. This is not persuasive since it is the length of time Mr. Amar has
been teaching at San Andres that is material in determining whether or not he in fact
qualified as a regular employee.
- Respondent ALEJANDRO asserted that her appointment as "Night Principal" after
having served a year thereat as a non-regular full-time teacher amounted to a
promotion which raised her status to that of a regular employee. This is also not
persuasive because mere appointment as "Night Principal" is not, by itself and absent any
additional evidence, sufficient proof that her employment status had in fact been upgraded
from probationary to regular.
- The contracts of employment entered into by the San Andres HS separately with each of
the respondents stipulated, among others: (a) that employment of the individual concerned
took effect at the beginning of the school year, or sometime in the month of June; and (b)
that payment of that individual's salary would be made "every month for 10 months." We
read these stipulations together to mean that such contracts each had an effective term of
ten (10) months, i.e., from June until either March or April of the following year. New
contracts for another period of ten months were negotiated between them at the beginning
of each school year. It does not appear from the record or from the stipulations in those
contracts, however, that renewal was obligatory upon either party.
- Private respondents claimed that Fr. Labajos allegedly "unusual antedated letter of
termination" did not sufficiently inform them of the reasons for their dismissal, nor did it
satisfy the due process requirements in termination cases. These contentions ignore the
fact that their employment was on a contractual basis and for a stipulated period of time.
- The use of the word "terminated" was inept and unfortunate but need not preclude
recognition of the real nature of that letter. Such letter was either a formal reminder that
their contracts were due to expire OR advance notice that such contracts would no longer
be renewed for the next school year OR both. Assuming that prior notice of expiration of
the contractual term was necessary in this case, we consider that Fr. Labajo's letter
substantially complied with that requirement.
* Since the six (6) private respondents were not illegally dismissed, the twin remedies of
reinstatement and backwages are not available to them. Dispositive NLRC Resolution is
SET ASIDE, except for the portion directing petitioners to pay P52,173.67 in favor of
private respondents.

PROBATIONARY EMPLOYEE
SKILLWORD MANAGEMENT AND MARKETING
CORPORATION V NLRC (MANUEL)
186 SCRA 465
MEDIALDEA; June 13, 1990

A2010

- 188 -

Disini

NATURE
Petition for certiorari
FACTS
- On June 24, 1983, Francisco Manuel was deployed to Saudi Arabia to work as driver by
petitioner Skillworld Management and Marketing, a duly licensed recruitment agency
operated by petitioners-spouses Serafin and Alicia Ramos. Upon his arrival in Jeddah,
Manuel signed a 2-year employment contract with his foreign employer, petitioner Shary
Limousine for a monthly basic salary of $300. 2 months later, Manuel was repatriated to
the Philippines. Upon his arrival in the Philippines, Manuel confronted the Ramoses who
promised to deploy him to other projects.
- After the lapse of more than one year without being deployed to other projects of
petitioners, Manuel filed a complaint with the POEA against petitioners for illegal
dismissal. He alleged that while he was employed as driver of Shary Limousine in its
branch at Jeddah he was stopped, and his driver's license sought for inspection, by Saudi
Arabian police. He showed the police two documents given to him by his employer, Shary
Limousine who made him believe that these pertained to a driver's temporary license.
However, Manuel was informed that the documents were not valid for a drivers license.
Together with eleven other drivers, they brought the matter before their superiors. Three
days after bringing the matter to his superior, respondent was ordered to pack his things.
He was taken to Riyadh and from there, repatriated to the Philippines. Upon respondent's
arrival in the Philippines, he requested the Ministry of Foreign Affairs for a translation of
what purported to be his driver's license. When translated it was only a certification of
employment with Shary Limousine in its branch at Jeddah.
- Petitioners alleged that Manuels dismissal was for a valid and just cause. Petitioners
alleged that Manuel was dismissed because of disobedience, absenteeism, refusal to
work and banding together to engage in concerted activities against the employer.
- POEA rendered judgment in favor of Manuel, directing petitioners to pay him $6,900.00
or its peso equivalent. Upon appeal, the NLRC affirmed said decision.
- According to petitioners, because of the probationary status of the employment of
Manuel, he may be dismissed at any time. Furthermore, this agreement was contained in
paragraph four (4) of the employment contract signed by Manuel.
ISSUE
WON Manuel was illegally dismissed
HELD
YES
- There is no dispute that as a probationary employee, Manuel had but a limited tenure.
Although on probationary basis, however, he still enjoys the constitutional protection on
security of tenure. During his tenure of employment therefore, or before his contract
expires, he cannot be removed except for cause as provided for by law.
- The alleged causes for which private respondent was dismissed (disobedience,
absenteeism, refusal to work, etc.) were not established. Respondent NLRC found that the
purported temporary licenses to drive issued to Manuel and his co-drivers by their
employer-the Shary Rent a Car/Limousine, turned out to be mere certifications to the
effect that they are Filipino citizens who are holders of given passport numbers and that
they were sent to work with the Shary Limousine Branch in Jeddah. It is for this reason
that after being accosted twice at checkpoints by Saudi police, who informed complainant
and his co-drivers that the alleged temporary licenses were not valid, they brought the
matter first to their Lebanese superior and then to the Philippine Embassy. - - Further,
records show that Manuel reported for work regularly and even rendered regular overtime
services; that he did not even attempt to join a strike or any other form of mass action
while working in Jeddah, because he knew that the laws in Jeddah are very strict and
being a foreigner he did not have the courage to join much less lead a strike which is
prohibited there; that he and his co-workers merely inquired from the Philippine Embassy
why they were allowed to drive without licenses; and that their action prompted the
Philippine Embassy to write their employer, which is perfectly in order as it was designed
to protect them in foreign soil.

MANAGERIAL EMPLOYEE
INTERORIENT MARITIME ENTERPRISES INC V NLRC
(TAYONG)
235 SCRA 268
FELICIANO; August 11, 1994
NATURE
PETITION for reviewof a decision of the National Labor Relations Commission
FACTS

Labor Law 1
- Captain Rizalino Tayong, a licensed Master Mariner with experience in commanding
ocean-going vessels, was employed on 1989 by petitioners for 1 yr as stated in his
employment contract. He assumed command of petitioners vessel at the port of
Hongkong. His instructions were to replenish bunker and diesel fuel, to sail to South Africa
and there to load 120,000 metric tons of coal. However, while in HK and unwarding cargo,
he received a weather report that a storm would hit HK, so precautionary measures were
taken to secure the safety of the vessel and its crew, considering that the vessels turbocharger was leaking and the vessel was 14 yrs old. He also followed-up the
requisition by the former captain for supplies of oxygen and acetylene necessary for the
welding-repair of the turbo-charger and the economizer.
-The vessel then sailed from HK for Singapore. Captain Tayong reported a water leak
from M.E. Turbo Chapter No. 2 Exhaust gas casing so he was instructed to black off
the cooling water and maintain reduced RPM unless authorized by the owners.
However, the vessel stopped in mid-ocean for 6 hrs and 45 minutes due to a leaking
economizer. He was instructed to shut down the economizer and use the auxiliary boiler
instead.
- The Chief Engineer reminded Captain Tayong that the oxygen and acetylene supplies
had not been delivered. He then informed the shipowner that the departure of the vessel
for South Africa may be affected because of the delay in the delivery of the supplies. The
shipowner advised Captain Tayong to contact its technical director who would provide a
solution for the supply of said oxygen and acetylene. The technical director recommended
to Captain Tayong that by shutting off the water to the turbo charger and using the
auxiliary boiler, there should be no further problem. Captain Tayong agreed to the
recommendation of the technical director, but communicated his reservations regarding
proceeding to South Africa without the requested supplies. So the shipowner advised him
to wait for the supplies.
- Finally, the vessel arrived at South Africa. However, Captain Tayong was instructed to
turn-over his post to the new captain, and was repatriated to the Philippines after serving
petitioners for around 2 wks. He was not informed of the charges against him, and was
just sent a letter after arriving in the Philippines. He therefore instituted a complaint for
illegal dismissal before the POEA, claiming his unpaid salary for the unexpired portion of
the written employment contract, plus attorneys fees.
- POEA: dismissed complaint, there was valid cause for his untimely repatriation (the
company alleged that due to Captain Tayongs refusal to sail immediately to South Africa,
the vessel was placed off-hire by the charterers, and the charterers refused to pay the
charter hire or compensation corresponding to 12 hours, amounting to US
$15,500.00.They fired Captain Tayong for lost of confidence; POEA believed that the
Captains concern for the oxygen and acetylene was not legitimate as these supplies were
not necessary or indispensable for running the vessel.)
- NLRC: reversed and set aside POEA decision because Captain Tayong had not been
afforded an opportunity to be heard and that no substantial evidenced was adduced to
establish the basis for petitioners loss of trust or confidence. Captain had acted in
accordance with his duties to maintain the seaworthiness of the vessel and to insure the
safety of the ship and crew.
ISSUE
WON Captain Tayong was arbitrarily dismissed and without cause as reasonably
established in an appropriate investigation (whether or not Captain Tayong had
reasonable grounds to believe that the safety of the vessel and the crew under his
command or the possibility of substantial delay at sea required him to wait for the
delivery of the supplies needed for the repair of the turbo-charger and the
economizer before embarking on the long voyage from Singapore to South Africa)
HELD
YES
Ratio It is well settled in this jurisdiction that confidential and managerial employees
cannot be arbitrarily dismissed at any time, and without cause as reasonably
established in an appropriate investigation. Such employees, too, are entitled to
security of tenure, fair standards of employment and the protection of labor laws.
Reasoning
- Captain Tayong was denied any opportunity to defend himself. Petitioners curtly
dismissed him from his command and summarily ordered his repatriation to the
Philippines without informing him of the charge or charges against him, and much less
giving him a chance to refute any such charge. In fact, it was only 2 months after his
repatriation that Captain Tayong received a telegram dated 24 October 1989 from InterOrient requiring him to explain why he delayed sailing to South Africa.
- NLRCs conclusion was supported by substantial evidence: The official report of the
technical director, which stated that a disruption in the normal functioning of the vessels
turbo charger and economizer had prevented the full or regular operation of the vessel
and that he was the one who recommended the reduction of RPM during the voyage to
South Africa instead of waiting in Singapore for the supplies that would permit shipboard
repair of the malfunctioning machinery and equipment, supported NLRCs conclusion that
Captain Tayong did not arbitrarily and maliciously delay the voyage to South Africa.
- Captain Tayong's decision (arrived at after consultation with the vessel's Chief Engineer)
to wait seven (7) hours in Singapore for the delivery on board the Oceanic Mindoro of the

A2010

- 189 -

Disini

requisitioned supplies needed for the welding-repair, on board the ship, of the turbocharger and the economizer equipment of the vessel, did not constitute merely arbitrary,
capricious or grossly insubordinate behavior on his part. In the view of the NLRC, that
decision of Captain Tayong did not constitute a legal basis for the summary dismissal of
Captain Tayong and for termination of his contract with petitioners prior to the expiration of
the term thereof.
Obiter
- The captain of a vessel is a confidential and managerial employee within the
meaning of the above doctrine. A master or captain, for purposes of maritime
commerce, is one who has command of a vessel. A captain commonly performs three (3)
distinct roles: (1) he is a general agent of the shipowner; (2) he is also commander and
technical director of the vessel; and (3) he is a representative of the country under whose
flag he navigates. Of these roles, by far the most important is the role performed by the
captain as commander of the vessel; for such role (which, to our mind, is analogous to
that of "Chief Executive Officer" [CEO] of a present-day corporate enterprise) has to do
with the operation and preservation of the vessel during its voyage and the protection of
the passengers (if any) and crew and cargo. In his role as general agent of the shipowner,
the captain has authority to sign bills of lading, carry goods aboard and deal with the
freight earned, agree upon rates and decide whether to take cargo. The ship captain, as
agent of the shipowner, has legal authority to enter into contracts with respect to
the vessel and the trading of the vessel, subject to applicable limitations
established by statute, contract or instructions and regulations of the shipowner. To
the captain is committed the governance, care and management of the vessel.
Clearly, the captain is vested with both management and fiduciary functions.
- Indeed, if the ship captain is convinced, as a reasonably prudent and competent
mariner acting in good faith that the shipowner's or ship agent's instructions
(insisted upon by radio or telefax from their officers thousand of miles away) will
result, in the very specific circumstances facing him, in imposing unacceptable
risks of loss or serious danger to ship or crew, he cannot casually seek absolution
from his responsibility, if a marine casualty occurs, in such instructions. 23
- Compagnie de Commerce v. Hamburg: xxx where by the force of circumstances, a man
has the duty cast upon him of taking some action for another, and under that obligation
adopts a course which, to the judgment of a wise and prudent man, is apparently the best
for the interest of the persons for whom he acts in a given emergency, it may properly be
said of the course so taken that it was in a mercantile sense necessary to take it."
- ON management prerogative: that prerogative is nevertheless not to be exercised, in the
case at bar, at the cost of loss of Captain Tayong's rights under his contract with
petitioner's and under Philippine law.
Disposition petitioners having failed to show grave abuse of discretion amounting to loss
or excess of jurisdiction on the part of the NLRC in rendering its assailed decision, the
Petition for Certiorari is hereby DISMISSED, for lack of merit. Costs against petitioners

E. MANAGEMENT RIGHTS AND SECURITY OF TENURE


MANAGEMENT RIGHTS AND SECURITY OF TENURE
COLEGIO DE SAN JUAN DE LETRAN V ASSN OF
EMPLOYEES AND FACULTY OF LETRAN
340 SCRA 587
KAPUNAN; September 18, 2000
NATURE
Petition for review on certiorari
FACTS
- Private respondent Ambas, the newly elected president of the Association of Employees
and Faculty of Letran (Union) wanted to continue the renegotiation of its CBA with
petitioner Colegio de San Juan de Letran (Letran) for the last 2 years of the CBAs 5 year
lifetime. However, petitioner claimed the CBA was already prepared for signing by the
parties. The CBA was submitted to a referendum by the union members, who rejected it.
- Petitioner accused the union officers of bargaining in bad faith before the NLRC which
decided in favor of petitioner but was later reversed on appeal with the NLRC.
- The Union notified the National Conciliation and Mediation Board (NCMB) of its intention
to strike on the grounds of petitioners refusal to bargain. Later, the parties agreed to
disregard the unsigned CBA and start negotiating a new 5 year CBA for which the Union
submitted its proposals. Ambas protested a recent changing of her schedule and petitioner
sent the Union a letter dismissing Ambas for alleged insubordination after which the Union
amended its notice of strike to include the said dismissal.
- Both parties again discussed the ground rules for the CBA renegotiation but petitioner
stopped the negotiations after purportedly receiving information that a new group of

Labor Law 1
employees (ACEC) filed a petition for certification election, giving rise to the issue of
majority representation of the employees.
- The Union finally went on strike and the Sec. of Labor and Employment assumed
jurisdiction, ordering those on strike to return to work and for petitioner to accept them
under the same terms before the strike. All were readmitted except Ambas. The Sec.
issued an order declaring petitioner guilty of unfair labor practice and directing the
reinstatement of Ambas with backwages. Letrans MFR was denied and the CA affirmed
the Sec.s decision, hence this petition.
ISSUES
1. WON petitioner is guilty of unfair labor practice by refusing to bargain with the union
2. WON the termination of the Ambas amounts to an interference of the employees right
to self-organization
HELD
1. YES
- Petitioner is guilty of unfair labor practice by its stern refusal to bargain in good faith with
respondent union.
- Article 252 defines collective bargaining as the performance of a mutual obligation to
meet and convene promptly and expeditiously in good faith for the purpose of negotiating
an agreement. The Union, in sending its proposals during the 2 nd CBA negotiations, kept
up its end of the bargain while Letran devised ways and means to prevent the negotiation.
- Letran also failed to make a timely reply to the Unions proposals (no counter-proposal a
month later), violating Article 250 which requires such a reply within 10 days upon receipt
of a written notice of said proposals. Letrans refusal to reply is an indication of bad faith,
showing a lack of sincere desire to negotiate.
- In a last ditch effort, Letran suspended the bargaining process on the ground that it
allegedly received information that ACEC had filed a petition for certification election. The
mere filing of a petition for certification election does not ipso facto justify the suspension
of negotiations when there is no legitimate representation issue raised; also, such an
action for intervention had already prescribed.
2. YES
- While we recognize the right of the employer to terminate the services of an employee
for just cause, the dismissal of employees must be made within the parameters of law and
pursuant to the tenets of equity and fair play and must be exercised in good faith. It must
not amount to interfering with, restraining or coercing employees in the exercise of their
right to self-organization as it would amount to unlawful labor practice under Article 248.
-It would appear that Letran terminated Ambas in order to strip the union of a leader who
would fight for her co-workers rights at the bargaining table and frustrate their desire to
form a new CBA. The charge of insubordination was a mere ploy to give a color of legality
to the action to dismiss her. Management may have the prerogative to discipline its
employees for insubordination but when it interferes with employees right to selforganization, it amounts to union-busting which is a prohibited act.
Disposition petition is DENIED for lack of merit

SAN MIGUEL BREWERY SALES FORCE UNION V


OPLE
170 SCRA 25
GRIO-AQUINO; February 8, 1989
FACTS
- A collective bargaining agreement was entered into by petitioner San Miguel Corporation
Sales Force Union and the private respondent, San Miguel Corporation. One provision of
the CBA was employees within the appropriate bargaining unit shall be entitled to a basic
monthly compensation plus commission based on their respective sales."
- Few months after the said CBA, the company introduced a marketing scheme known as
the "Complementary Distribution System" (CDS) whereby its beer products were offered
for sale directly to wholesalers through San Miguel's sales offices.
- The labor union filed a complaint for unfair labor practice in the Ministry of Labor, with a
notice of strike on the ground that the CDS was contrary to the existing marketing scheme
whereby the Route Salesmen were assigned specific territories within which to sell their
stocks of beer, and wholesalers had to buy beer products from them, not from the
company. It was alleged that the new marketing scheme violates Section 1, Article IV of
the collective bargaining agreement because the introduction of the CDS would reduce the
take-home pay of the salesmen and their truck helpers for the company would be unfairly
competing with them.
ISSUES
1. WON the CDS violates the collective bargaining agreement
2. WON it is an indirect way of busting the union
HELD
1. NO

A2010

Disini

- 190 -

- CDS is a valid exercise of management prerogatives:


Ratio Except as limited by special laws, an employer is free to regulate, according to his
own discretion and judgment, all aspects of employment, including hiring, work
assignments, working methods, time, place and manner of work, tools to be used,
processes to be followed, supervision of workers, working regulations, transfer of
employees, work supervision, lay-off of workers and the discipline, dismissal and recall of
work.
- So long as a company's management prerogatives are exercised in good faith for the
advancement of the employer's interest and not for the purpose of defeating or
circumventing the rights of the employees under special laws or under valid agreements,
this Court will uphold them
2. NO
Ratio Nothing in the record as to suggest that the unilateral action of the employer in
inaugurating the new sales scheme was designed to discourage union organization or
diminish its influence, but rather it is undisputable that the establishment of such scheme
was part of its overall plan to improve efficiency and economy and at the same time gain
profit to the highest. While it may be admitted that the introduction of new sales plan
somewhat disturbed the present set-up, the change however was too insignificant as to
convince this Office to interpret that the innovation interferred with the worker's right to
self-organization.
Reasoning
- Petitioner failed to consider is the fact that corollary to the adoption of the assailed
marketing technique is the effort of the company to compensate whatever loss the workers
may suffer because of the new plan over and above than what has been provided in the
collective bargaining agreement. To us, this is one indication that the action of the
management is devoid of any anti-union hues."
Disposition Dismissed

F. GUIDELINE ON IMPOSITION OF PENALTIES


VALIAO V CA
[PAGE 11]
FARROL V CA (RCPI)
325 SCRA 331
YNARES-SANTIAGA; February 10, 2000
FACTS
- Wenifrado Farrol was the station cashier of RCPI Cotabato City Station.
- There was a P50K cash shortage in the branchs Peragram Petty Cash Funds. Farrol
was required to explain the cash shortage. He paid to P25K to RCPI
- He was then required to explain why he should not be dismissed. Petitioner wrote to the
Field Auditor stating that the missing funds were used for the payment of the retirement
benefits earlier referred by the Branch Manager and that he already paid P25k. After he
made 2 more payments of the cash shortage, he was placed under preventive
suspensions. He still made 2 payments of the balance.
- RCPI then sent Farrol a letter informing him of the termination of his services for alleging
that part of the cash shortage was used for payment of salaries and retirement benefits,
disregard of policies involving statistical reports, malversation/misappropriation (which is a
ground for dismissal), and loss of trust and confidence.
- Unaware of the termination letter, he requested his reinstatement since his preventive
suspension had expired. Ferrol even manifested his willingness to settle the case. RCPI
informed him that his employment had already been terminated. The conflict was sent to
the grievance committee. Two years later, it was submitted for voluntary arbitration.
- VA ruled in favor of Farrol. RCPI filed a petition for certiorari before the CA which
reversed VA decision. CA also dismissed MFR.
- Farrol now filed a petition for review on certiorari on the ground that his dismissal was
illegal because he was not afforded due process and that he cannot be held liable for the
loss of trust and confidence reposed in him.

ISSUE
WON he was illegally terminated
HELD
YES
- BOP resides on the employer to prove that there was valid cause for dismissal, and that
he was afforded the opportunity to be heard and defend himself.

Labor Law 1
- For the 1st notice, RCPI required petitioner to explain why he failed to account for the
shortage. The 2nd notice was that informing Farrol of his termination. it does not clearly cite
the reasons for dismissal, nor were there facts and circumstances in support thereof.
- Even assuming there was a breach of trust and confidence, there was no evidence that
Farrol was a managerial employee. The term trust and confidence is restricted to
managerial employees.
- RCPI alleges that under its rules, petitioners infarction is punishable by dismissal.
However, employers rules cannot preclude the state from inquiring whether strict and rigid
application or interpretation would be too harsh to the employee. This is Farrols 1st
offense, to which the Court holds that dismissal is too harsh and grossly disproportionate.
Disposition CA is REVERSED and SET ASIDE and new one entered REINSTATING the
decision of the Voluntary Arbitrator subject to the MODIFICATION that petitioners
separation pay be recomputed to include the period within which backwages are due. For
this purpose, this case is REMANDED to the Voluntary Arbitrator for proper computation of
backwages, separation pay, 13th month pay, sick leave conversion and vacation leave
conversion.

VH MANUFACTURING INC V NLRC (GAMIDO)


322 SCRA 417
DE LEON; January 19, 2000
NATURE
Before us is a petition for certiorari
FACTS
- Since November 5, 1985 Gamido was employed in VH Manufacturings business of
manufacturing liquefied petroleum gas (LPG) cylinders. He served as a quality control
inspector with the principal duty of inspecting LPG cylinders for any possible defects. His
service with the company was abruptly interrupted on February 14, 1995, when he was
served a notice of termination of his employment.
- His dismissal stemmed from an incident on February 10, 1995 wherein VHs company
President, Alejandro Dy Juanco, allegedly caught private Gamido sleeping on the job. On
that same day, private respondent was asked through a written notice from the petitioners
Personnel Department to explain within twenty-four (24) hours why no disciplinary action
should be taken against him for his violation of Company Rule 15-b which provides for a
penalty of separation for sleeping during working hours. Without delay, private respondent
replied in a letter which reads: "Sir, ipagpaumanhin po ninyo kung nakapikit ako sa aking
puwesto dahil hinihintay ko po ang niliha hi Abreu para i quality pasensiya na po kung
hindi ko po namalayan ang pagdaan ninyo dahil maingay po ang painting booth."
Notwithstanding his foregoing reply, he was terminated.
- Feeling aggrieved, he filed a complaint for illegal dismissal, praying for reinstatement to
his position as quality control inspector. Labor Arbiter declared that Gamidos dismissal is
anchored on a valid and just cause. NLRC reversed the decision.
ISSUE
WON Gamidos dismissal was too harsh a penaltly for his violation of company rule 15-b
HELD
YES
- Basically, the reason cited for the dismissal of private respondent is sleeping on the job
in violation of Company Rule 15-b. But according to Gamido, he was not sleeping on the
job but was merely idle, waiting for the next cylinder to be checked.
- In view of the gravity of the penalty of separation, as provided by the Company Rules
and Regulation., in termination disputes, the burden of proof is always on the employer to
prove that the dismissal was for a just and valid cause. What is at stake here is not only
the job itself of the employee but also his regular income therefrom which is the means of
livelihood of his family.
- A thorough review of the record discloses that, contrary to the findings of the Labor
Arbiter, petitioners claim that private respondent slept on the job was not substantiated by
any convincing evidence other than the bare allegation of the officer.
- Next, VHs reliance on the authorities it cited that sleeping on the job is always a valid
ground for dismissal, is misplaced. The authorities cited involved security guards whose
duty necessitates that they be awake and watchful at all times inasmuch as their function,
to use the words in Luzon Stevedoring Corp. v. Court of Industrial Relations, is "to protect
the company from pilferage or loss." Accordingly, the doctrine laid down in those cases is
not applicable to the case at bar.

A2010

- 191 -

Disini

- Finally, while an employer enjoys a wide latitude of discretion in the promulgation of


policies, rules and regulations on work-related activities of the employees, those
directives, however, must always be fair and reasonable, and the corresponding penalties,
when prescribed, must be commensurate to the offense involved and to the degree of the
infraction. In the case at bar, the dismissal meted out on private respondent for allegedly
sleeping on the job, under the attendant circumstances, appears to be too harsh a
penalty, considering that he was being held liable for first time, after nine 9 of unblemished
service, for an alleged offense which caused no prejudice to the employer, aside from
absence of substantiation of the alleged offense. Neither was it shown that private
respondents alleged negligence or neglect of duty, if any, was gross and habitual. Thus,
reinstatement is just and proper.
Disposition petition is hereby DISMISSED, and the challenged Decision and Order of
public respondent NLRC are AFFIRMED.

REYNO V MANILA ELECTRIC COMPANY


434 SCRA 660
SANDOVAL-GUTIERREZ; July 22, 2004
NATURE
Petition for review on certiorari under Rule 45 of the 1997 Rules of Civil Procedure
FACTS
- Reyno was employed by MERALCO where he eventually occupied the position of
Assistant Squad Leader of Squad 12 at the Inspection Department. Petitioner and his
team of inspectors were in charge of monitoring and inspecting electric meters installed at
the premises of respondents customers; ensuring the accuracy of the electric
consumption recorded in these meters; and reporting and apprehending violators who use
insidious schemes or devices to reduce their electric consumption deliberately.
- Later, MERALCO implemented an incentive scheme aimed at encouraging its inspectors
to perform their duties zealously. Under this incentive scheme, the inspector concerned
shall be paid an additional 30-minute overtime pay for every submitted report of major
violation/s committed by customers against respondent.
- Roger Sacdalan, Senior Investigator of respondents Special Presidential Committee
(SPC), received several complaints against Gilbert Villapa, Leader of Squad 12, about an
illegal connection.
- SPC conducted an investigation wherein members of Squad 12 were summoned to
explain. However, they failed to establish Villapas involvement in such illegal connection.
Instead, their declarations pointed to Reynos irregular performance of his duties.
- This prompted SPC to conduct clarificatory hearing. But the hearing was cancelled for
failure of Reynos counsel to appear despite notice. When the case was called for hearing
as scheduled, his counsel again failed to appear. He then opted to proceed with the
clarificatory hearing without the assistance of his counsel.
- After evaluating the records on hand, the SPC found petitioner guilty of dishonesty,
serious misconduct and willful breach of trust. Respondent then sent petitioner a notice
terminating his services.
- Reyno filed with the Labor Arbiter a complaint for illegal dismissal and payment of
overtime pay, premium pay for holidays and rest days, damages and attorneys fees.
ISSUES
1. WON Reyno was deprived of his right to cross examine witnesses before the Labor
Arbiter
2. WON Reyno was illegally dismissed
HELD
1. NO
- His right to cross-examine the three witnesses, did not err as it was not required to apply
strictly the Rules of Evidence. At any rate, MERALCO had valid reasons why it did not
present those three witnesses during the proceedings before the Labor Arbiter
2. NO
- The standard of substantial evidence is satisfied where the employer, as in this case, has
reasonable ground to believe that the employee is responsible for the misconduct and his
participation therein renders him unworthy of trust and confidence demanded by his
position. Reyno violated MERALCOs Code of Employee Discipline and committed serious

Labor Law 1
misconduct in the performance of his duties have been proved by the affidavits of
petitioners own subordinates in Squad 12 of which he was the Assistant Squad Leader.
Moreover, MERALCO had lost his trust and confidence in petitioner. Under Article 282 of
the Labor Code, as amended, these are just causes for his dismissal from the service.
- The longer an employee stays in the service of the company, the greater is his
responsibility for knowledge and compliance with the norms of conduct and the code of
discipline in the company.
- An employees length of service with the company even aggravates his offense. He
should have been more loyal to company from which he has derived his family bread and
butter for seventeen (17) years.
Disposition Petition is DENIED. The assailed Decision dated January 17, 2001 and
Resolution dated May 3, 2001 of the Court of Appeals in CA-G.R. SP No. 53987 are
hereby AFFIRMED.

A2010

- 192 -

Disini

- Gabriel is not entirely faultless. As a supervisor, he is required to act judiciously and to


exercise his authority in harmony with PLDTs policies. When he jeopardized the status of
the rank and file employees whom he ordered to by-pass the standard operating
procedures of the company, to the detriment of his employer, he was not entirely
blameless. The irregularity attributable to him could not be disregarded. He must not be
rewarded, in fairness to the employers own legitimate concerns such as company morale
and discipline.
Disposition the resolution f the NLRC is affirmed subject to the deletion of the other
awards of unspecified benefits and proportionate privileges.

FACTORS
ASSOCIATED LABOR UNION V NLRC
[PAGE 181]

PHILIPPINE LONG DISTANCE TELEPHONE V NLRC


(GABRIEL)
303 SCRA 9
QUISUMBING; February 11, 1999
NATURE
Appeal from the order of the NLRC
FACTS
- Private respondent, Enrique Gabriel, was foreman of petitioner PLDT and was a
supervisor with territorial responsibility for Camp Crames First to 20 th Avenue and portions
of Project 4, all located in Quezon City. On two occasions (September 5, 1989 and
October 16, 1989) he ordered Medel Mercado and Juancho Jocson to install two
telephone lines each at Unit R, Facilities Center Building, located at Shaw Boulevard,
Mandaluyong.
- The ordered installations were investigated because (a) the Facilities Center Building
had no entrance cable facilities or conduit wires for telephone connection, (b)
Mandaluyong was not within Gabriels area of jurisdiction, and (c) installers Mercado and
Jocson were not under his direct supervision.
- During the investigation, Gabriel. while acknowledging responsibility for his action,
claimed that his actuation was motivated by the desire to provide customer satisfaction.
He also claimed that the telephones were installed after the documents of approval were
issued by PLDT. He dismissed from service on September 3, 1990 on the ground that he
committed grave misconduct, breach of trust, and violations of company rules and
regulations.
- Gabriel filed an illegal dismissal complaint with the Labor Arbiter on September 6, 1990.
Said Arbiter affirmed the dismissal but the same was reversed by the NLRC and ordered
PLDT to reinstate Gabriel to the position he held as at the time of the complained
dismissal, with full backwages, benefits, and proportionate privileges. Hence the appeal.
ISSUE
WON Gabriel is guilty of serious misconduct and/or breach of trust anent the irregular
installation of the telephones
HELD
NO
- The facts of the case do not point to any misconduct or breach of trust on the part of
Gabriel. There was also no provision in the written rule of PLDT which penalizes
unwarranted installation of telephone lines with dismissal. In any case, the installations
were approved by the company. There was also no evidence that Gabriel profited
personally with the transaction. The dismissal of Gabriel is illegal.
Reasoning
- Dismissal is the ultimate penalty and should not be imposed if the employee has been in
service for a considerable length of time and has not been the recipient of any disciplinary
actions. Where a penalty less punitive would suffice, whatever missteps may have been
committed by the worker ought not to be visited with a consequence so severe such as
dismissal. This interpretation gives meaning and substance to the liberal and
compassionate spirit of the law as provided for in Article 4 of the Labor Code which states
that all doubts in the implementation and interpretation of the provisions of the Labor
Code including its implementing rules and regulations shall be resolved in favor of labor.

DISMISSAL AS PENALTY
CEBU FILVENEER CORPORATION V NLRC
(VILLAFLOR)
286 SCRA 556
PUNO; February 24, 1998
FACTS
- Villaflor was the chief accountant of CFC. The top execs were Italians: Cordaro
(president), Kun (GM), Marinoni (Production manager). Guillermo was the accounting
clerk of Villaflor.
- Kun resigned from the company and asked for the liquidation of his investment: P125k.
Two weeks later, he asked Guillermo for a blank check and a blank check voucher.
Guillermo gave him. Three days later, Villlaflor noticed that a check voucher was missing.
She asked Guillermo, who said that Mr. Kun has it.
- Villaflor immediately informed Mr. Cordaro of what happened. She also wrote to the
bank demanding the return of the encashed check.
- Marinoni charged Villaflor of complicity in Kuns irregular disbursement of company
funds. Two days later, she was prevented entry to the office by the security guards. Her
office drawer and safe were also forcibly opened upon order of Marinoni. Villaflor reported
the incident to the PNP.
- Marinoni suspended her for 30 days without pay for failure to come to work for half a
day (the day she was prevented entry). The next day she was preventively suspended for
30 days pending investigation of her involvement in Kuns booboo. The company also
printed a newspaper ad for an accountant.
- Villaflor filed for illegal dismissal with the LA. LA decided in her favor. NLRC affirmed.
ISSUE
WON Villaflor was illegally dismissed
HELD
YES
- Due to its far reaching implications, our Labor Code decrees that an employee cannot be
dismissed, except for the most serious causes. Article 282 enumerates the causes for
which the employer may terminate an employee.
- Company says its loss of trust. The SC said that Villaflors omission cannot be
described as willful to justify dismissal. A breach is willful if it is done intentionally,
knowingly and purposely. Petitioners merely proved the omission of the private respondent
but there is no evidence whatsoever that it was done intentionally.
- Company says shes grossly or habitually negligent in the performance of her duties.
The SC said that since she has not been remiss in the performance of her duties in the
past, she cant be charged with habitual negligence. Neither is her negligence gross in
character. Gross negligence implies a want or absence of or failure to exercise slight
care or diligence or the entire absence of care. It evinces a thoughtless disregard of
consequences without exerting any effort to avoid them. She had not the slightest
reason to distrust Kun because he was the GM and appears to have conducted himself
well in the performance of his duties in the past. At most, its error of judgment, not gross
negligence.
Disposition NLRC decision affirmed.

Labor Law 1
GOLDEN THREAD KNITTING INDUSTIRES V NLRC
(MACASPAC)
304 SCRA 720
BELLOSILLO; March 11, 1999
NATURE
Petition to review decision of NLRC
FACTS
- several employees of Golden Thread Knitting Industries (GTK) were dismissed for
different reasons. 2 employees were allegedly for slashing the companys products
(towels), 2 for redundancy, 1 for threatening the personnel manager and violating the
company rules, and 1 for abandonment of work.
- The laborers filed complaints for illegal dismissal. They allege that the company
dismissed them in retaliation for establishing and being members of the Labor Union.
GTK, on the other hand, contend that there were valid causes for the terminations. The
dismissals were allegedly a result of the slashing of their products, rotation of work, which
in turn was caused by the low demand for their products, and abandonment of work. WRT
to the cases involving the slashing of their products and threats to the personnel manager,
the dismissals were in effect a form of punishment.
- The labor arbiter ruled partially in favor of GTK. He said that there was no showing that
the dismissals were in retaliation for establishing a union. He, however, awarded
separation pay to some employees.
- NLRC, however, appreciated the evidence differently. It held that there was illegal
dismissal and ordered reinstatement.
ISSUE
WON there was illegal dismissal
HELD
YES
Ratio Dismissal is the ultimate penalty that can be meted to an employee. It must
therefore be based on a clear and not on an ambiguous or ambivalent ground.
Reasoning
- WRT to the case involving slashing of towels, the employees were not given procedural
due process. There was no notice and hearing, only outright denial of their entry to the
work premises by the security guards. The charges of serious misconduct were not
sufficiently proved.
- WRT to the employees dismissed for redundancy, there was also denial of procedural
due process. Hearing and notice were not observed. Thus, although the characterization
of an employees services is a management function, it must first be proved with evidence,
which was not done in this case. the company cannot merely declare that it was
overmanned.
- WRT to the employee dismissed for disrespect, the SC believed the story version of the
company (which essentially said that the personnel manager was threatened upon mere
service of a suspension order to the employee), but ruled that the dismissal could not be
upheld.
the dismissal will not be upheld where it appears that the employees act of disrespect
was provoked by the employer. xxx the employee hurled incentives at the personnel
manager because she was provoked by the baseless suspension imposed on her. The
penalty of dismissal must be commensurate with the act, conduct, or omission to the
employee.
- The dismissal was too harsh a penalty; a suspension of 1 week would have sufficed.
GTK exercised their authority to dismiss without due regard to the provisions of the
Labor Code. The right to terminate should be utilized with extreme caution because its
immediate effect is to put an end to an employee's present means of livelihood while its
distant effect, upon a subsequent finding of illegal dismissal, is just as pernicious to the
employer who will most likely be required to reinstate the subject employee and grant
him full back wages and other benefits.
Disposition Decision AFFIRMED

CENTRAL PANGASINAN ELECTRIC COOP INC V


MACARAEG
395 SCRA 720
PUNO; January 22, 2003
NATURE
Petition for review on certiorari
FACTS

A2010

- 193 -

Disini

- De Vera was employed as teller and Geronima Macaraeg as cashier by Central


Pangasinan Electric cooperative inc. They accommodated and encashed two hundred
eleven crossed checks of Evelyn Joy Estrada (de Veras sister) amounting to
P6,945,128.95 payable to the cooperative despite the absence of any transaction or any
outstanding obligation with it. They credited the checks as part of their collection and
deposited the same together with their cash collection to the coops account at the Rural
Bank of Central Pangasinan.
- The finance department noticed these checks which bounced (insufficient funds).De Vera
and Macaraeg were confronted with the discovery. De Vera admitted that the checks were
issued by her sister and that she encashed them from the money collected from
petitioners customers.
- De Vera testified and admitted that she encashed the checks of Evelyn Joy Estrada
because the latter is her older sister. Macaraeg admitted that she knew of the
accommodations given by respondent de Vera to her sister; that she allowed her
subordinate to do it because respondent de Vera is her kumare, and that she knew that
Mrs. Estradas checks were sufficiently funded.
- On March 19, 1999, on the basis of the findings and recommendation of Atty. Fernandez
(presided over the hearing), the General Manager issued to respondents separate notices
of termination for serious misconduct, and breach of trust and confidence reposed on
them by management.
- Respondents questioned their dismissal before the National Conciliation and Mediation
Board (NCMB),claiming that their dismissal was without just cause and in violation of the
Collective Bargaining Agreement (CBA), which requires that the case should first be
brought before a grievance committee. Eventually, the parties agreed to submit the case
to a voluntary arbitrator for arbitration.
- LA-ruled in favor of defendants and ordered their reinstatement
CA-affirmed
ISSUES
1. WON the procedure leading to the termination of respondents Maribeth de Vera and
Geronima Macaraeg was in violation of the provisions of the CBA
2. WON the respondents were validly dismissed
HELD
1. Issue is moot and academic
- The parties active participation in the voluntary arbitration proceedings, and their failure
to insist that the case be remanded to the grievance machinery, shows a clear intention on
their part to have the issue of respondents illegal dismissal directly resolved by the
voluntary arbitrator.
2. YES
- The respondents were validly dismissed. Article 282(c) of the Labor Code allows an
employer to dismiss employees for willful breach of trust or loss of confidence. Proof
beyond reasonable doubt of their misconduct is not required, it being sufficient that there
is some basis for the same or that the employer has reasonable ground to believe that
they are responsible for the misconduct and their participation therein rendered them
unworthy of the trust and confidence demanded of their position.
Reasoning
- the acts of the respondents were clearly inimical to the financial interest of the petitioner.
During the investigation, they admitted accommodating Evelyn Joy Estrada by encashing
her checks from its funds for more than a year. They did so without petitioners
knowledge, much less its permission.
- there was willful breach of trust on the respondents part, as they took advantage of their
highly sensitive positions to violate their duties.
- the acts of the respondents caused damage to the petitioner. During those times the
checks were illegally encashed, petitioner was not able to fully utilize the collections,
primarily in servicing its debts.
- it is not material that they did not misappropriate any amount of money, nor incur any
shortage relative to the funds in their possession. The basic premise for dismissal on the
ground of loss of confidence is that the employees concerned hold positions of trust. The
betrayal of this trust is the essence of the offence for which an employee is penalized.
- the respondents held positions of utmost trust and confidence. As teller and cashier,
respectively, they are expected to possess a high degree of fidelity. They are entrusted
with a considerable amount of cash. Respondent de Vera accepted payments from
petitioners consumers while respondent Macaraeg received remittances for deposit at
petitioners bank. They did not live up to their duties and obligations.

PHILIPS SEMICONDUCTORS V FADRIQUELA


[PAGE 77]

G. RULES MANAGERIALS AND RANK AND RANK FILE


EMPLOYEES

Labor Law 1
SALVADOR V PHILIPPINE MINING SERVICE CORP
395 SCRA 729
PUNO; January 22, 2003
FACTS
- JOSE V. SALVADOR was first employed by respondent in 1981. He rose from the ranks
and assumed the position of Plant Inspection Foreman in 1991. He was tasked to: (1)
supervise plant equipment and facility inspection; (2) confirm actual defects; (3) establish
inspection standards and frequency; (4) analyze troubles and recommend counter
measures; and (5) prepare weekly/monthly inspection schedule.[3]
- As early as March 1, 1985, respondent instituted the shift boss scheme whereby the
foreman from the Plant Section and the foreman from the Mining Section rotate as shift
boss throughout their night shift to oversee and supervise both the mining and plant
operations. The shift boss was entrusted with the care, supervision and protection of the
entire plant.
- Aside from his employment with respondent, petitioner co-owned and managed LHOTAB Enterprises, with his partner Ondo Alcantara. They were engaged in the manufacture
and sale of hollow blocks. On September 29, 1997, petitioners employment relation with
respondent was tainted with charges of pilferage and violation of company rules and
policy, resulting to loss of confidence. Respondents evidence disclose that on September
29, 1997, at about 9:30 a.m., Koji Sawa, respondents Assistant Resident Manager for
Administration, was on his way back to his office in the plant. He and his driver, Roberto
Gresones, saw petitioner operating respondents payloader, scooping fine ore from the
stockpile and loading it on his private cargo truck. As the truck was blocking the access
road leading to the stockyards gate, Sawas car stopped near the stockpile and the driver
blew the horn thrice. Petitioner did not hear him because of the noise emanating from his
operation of the payloader. Sawas driver found a chance to pass through when the
payloader maneuvered to get another scoop from the fine ore stockpile.
- As it was contrary to respondents standard operating procedure for the plant foreman to
operate the payloader, Sawa went to the administration office to check the delivery receipt
covering the loading operation of petitioner that morning. However, sales-in-charge
Eduardo Guangco was in the wharf, overseeing the loading of respondents product.
Hence, it was only in the afternoon that Sawa was able to verify the delivery receipt
covering petitioners loading transaction. The delivery receipt showed that it was dolomite
spillage that was purchased by buyer Ondo Alcantara, not the fine ore that he saw
petitioner loading on his truck. The receipt also showed it was not the respondent but
Alcantara, the buyer, who was responsible for loading the spillage he purchased from the
plant.
- On the basis of the foregoing facts PMSC terminated Salvador for pilferage of company
property. Labor Arbiter and NLRC ruled in favor of Salvador but CA reversed. Hence, this
recourse.
ISSUES
1. WON the charge of pilferage against petitioner was supported by substantial evidence
to warrant his dismissal from the service
2. WON the employer was well within its rights in imposing a harsh penalty considering the
length of the employees service
HELD
1. YES
Ratio The settled rule in administrative and quasi-judicial proceedings is that proof
beyond reasonable doubt is not required in determining the legality of an employers
dismissal of an employee, and not even a preponderance of evidence is necessary as
substantial evidence is considered sufficient. Substantial evidence is more than a mere
scintilla of evidence or relevant evidence as a reasonable mind might accept as adequate
to support a conclusion, even if other minds, equally reasonable, might conceivably opine
otherwise. Thus, substantial evidence is the least demanding in the hierarchy of evidence.
Reasoning
- The Labor Code provides that an employer may terminate the services of an employee
for just cause and this must be supported by substantial evidence. In the case at bar, our
evaluation of the evidence of both parties indubitably shows that petitioners dismissal for
loss of trust and confidence was duly supported by substantial evidence.
2. NO
Ratio As a general rule, employers are allowed wider latitude of discretion in terminating
the employment of managerial employees as they perform functions which require the
employers full trust and confidence.
Reasoning
- To be sure, length of service is taken into consideration in imposing the penalty to be
meted an erring employee. However, the case at bar involves dishonesty and pilferage by
petitioner which resulted in respondents loss of confidence in him. Unlike other just
causes for dismissal, trust in an employee, once lost is difficult, if not impossible, to regain.
Moreover, petitioner was not an ordinary rank-and-file employee. He occupied a high
position of responsibility. As foreman and shift boss, he had over-all control of the care,
supervision and operations of respondents entire plant. It cannot be over-emphasized that

A2010

- 194 -

Disini

there is no substitute for honesty for sensitive positions which call for utmost trust.
Fairness dictates that respondent should not be allowed to continue with the employment
of petitioner who has breached the confidence reposed on him.
- In the case at bar, respondent has every right to dismiss petitioner, a managerial
employee, for breach of trust and loss of confidence as a measure of self-preservation
against acts patently inimical to its interests. Indeed, in cases of this nature, the fact that
petitioner has been employed with the respondent for a long time, if to be considered at
all, should be taken against him, as his act of pilferage reflects a regrettable lack of loyalty
which he should have strengthened, instead of betrayed.
Disposition The petition is DENIED.

CAOILE V NLRC (COCA-COLA BOTTLERS,


PHILIPPINES INC)
299 SCRA 76
QUISUMBING; November 24, 1998
NATURE
Special action for certiorari
FACTS
- Private respondent CCBPI, through the local plant management, contracted the services
of Mr. Redempto de Guzman for the installation of a Private Automatic Branch Exchange
(PABX) housewiring in the plant premises for the sum of P65,000.00. Since the project fell
under the direct supervision of petitioner, all cash advances by the contractor were
coursed through him.
- Mr. De Guzman, the contractor, requested for an initial cash advance of P10,000.00.
Petitioner caused the preparation of the Payment Request Memo in the amount of
P15,000.00 and the issuance of a check in the same amount. After securing the
endorsement of the contractor, petitioner encashed the check with the plant teller Mr.
Dominador S. Pila and handed over P10,000.00 to Mr. De Guzman while retaining the
amount of P5,000.00 for himself.
- The contractor requested for second and third cash advances in the amounts of
P5,000.00 and P10,000.00 respectively. As in the first cash advance, petitioner caused the
preparation of 2 checks in the amounts of P10,000.00 and P15,000.00 respectively. After
securing the endorsements of the contractor the requested cash advances while retaining
for himself the difference of P10,000.00.
- After the project was completed, the contractor requested payment of the balance of the
contract price in the amount of P25,000.00. Petitioner caused the issuance of a check in
the amount of P24,350.00 (after deducting 1% of the total contact price by way of
witholding tax). Petitioner secured the endorsement of the contractor, encashed the check
with the teller, then handed over to the contractor only P19,350.00 while retaining fore
himself the amount of P5,000.00.
- Upon completion of an additional project requested of the contractor, petitioner caused
the issuance a check, and after securing the endorsement of the contractor, petitioner
encashed the check and delivered P8,000.00 to the contractor and retained P500.00 for
himself.
- Mr. de Guzman executed an affidavit exposing the fraudulent acts perpetrated by
petitioner, which prompted the company to conduct an investigation.
- Petitioner was served a Notice of investigation. During the investigation, petitioner
admitted that the initials in the check vouchers were his but denied having encashed the
checks and delivering the cash payments to the contractor.
- It was established through the testimony of Mrs. Macasinag and Mr. Pila that petitioner
personally withdrew the checks from the GM Secretary and had them encashed with the
teller after Mr.de Guzman has endorsed the same.
- Mr. Mariano A. Limjap, Senior VP and Administration Director issued a memo sustaining
the findings and recommendation of the local plant management for the termination of
complainant from his employ on the grounds of grave misconduct and dishonesty
considering that his position as EDP Supervisor is bestowed with the highest trust and
confidence by the respondent as may be seen from the description of his duties and
responsibilities.
- As a consequence of his dismissal, petitioner filed a compliant for illegal dismissal with
damages
- Labor Arbiter rendered a decision finding that petitioner was illegally dismissed

Labor Law 1
- Private respondents appealed to NLRC which reversed the Labor Arbiter's
decision. NLRC held that petitioner committed acts constituting a breach of trust and
confidence reposed on him by his employer, thereby justifying his dismissal.
ISSUE
WON the NLRC committed grave abuse of discretion amounting to lack or excess of
jurisdiction in reversing and setting aside the Labor Arbiter's decision finding private
respondents guilty of illegal dismissal
HELD
NO
Ratio Law and jurisprudence have long recognized the right of employers to dismiss
employees by reason of loss of trust and confidence. As provided for in the Labor Code,
"Art. 282. An employer may terminate an employment for any of the following causes: x x
x (c) Fraud or willful breach of the trust reposed in him by his employer or his duly
authorized representative. x x x." In the case of supervisors or personnel occupying
positions of responsibility, this Court has repeatedly held that loss of trust and confidence
justifies termination. Obviously, as a just cause provided by law, this ground for terminating
employment, springs from the voluntary or willful act of the employee, or "by reason of
some blameworthy act or omission on the part of the employee".
Reasoning
- Loss of confidence as a just cause for termination of employment is premised from the
fact that an employee concerned holds a position of trust and confidence. But, in order to
constitute a just cause for dismissal, the act complained of must be "work-related" such as
would show the employee concerned to be unfit to continue working for the employer.
- it must be noted the recent decisions of this Court has distinguished the treatment of
managerial employees from that of rank-and-file personnel, insofar as the application of
the doctrine of loss of trust and confidence is concerned. Thus with respect to rank-andfile personnel, loss of trust and confidence as ground for valid dismissal requires
proof of involvement in the alleged events in question, and that mere
uncorroborated assertion and accusations by the employer will not be sufficient.
But, as regards as a managerial employee, mere existence of a basis for believing
that such employee has breached the trust of his employer would suffice for his
dismissal. Hence, in the case of managerial employees, proof beyond reasonable doubt
is not required, it being sufficient that there is some basis for such loss of confidence, such
as when the employer has reasonable ground to believe that the employee concerned is
responsible for the purported misconduct, and the nature of his participation therein
renders him unworthy of the trust and confidence demanded by his position.
- In the present case, petitioner is not an ordinary rank-and-file employee. He is the EDP
Supervisor tasked to directly supervise the installation of the PABX housewiring project in
respondent company's premises. He should have realized that such sensitive position
requires the full trust and confidence of his employer. Corollary, he ought to know that his
job requires that he keep the trust and confidence bestowed on him by his employer
unsullied.
Disposition Petition is DISMISSED for lack of merit.

G. TERMINATION OF EMPLOYMENT BY
EMPLOYEE

14.02 CAUSES
A. JUST CAUSES
B. WITHOUT JUST CAUSE
C. RESIGNATION
DEFINITION
HABANA V NLRC (HOTEL NIKKO)
298 SCRA 537
KAPUNAN; November 16, 1998
NATURE
Petition for certiorari seeking reversal of NLRC decision which affirmed LA

A2010

- 195 -

Disini

FACTS
- On March 16, 1989, petitioner Antonio Habana was employed by Hotel Nikko Manila
Garden (Nikko) as Rooms Division Director (RDD). One of his tasks as RDD was to
conduct regular and surprise inspection of all work areas to ensure quality of performance.
In the course of his employment, petitioner encountered several problems: his frequent
clashes with Dolores Samson (his Senior Rooms Mgr); frequent absence and tardiness;
rampant violations of hotel rules due to his failure to effectively manage his own division;
and complaints regarding the overall quality (or lack thereof) of service of Nikko. As a
result, private respondent Mr. Okawa, who replaced private respondent Mr. Yokoo as the
executive asst. for Sales, issued a memorandum instructing petitioner, along with 2 others,
to conduct and report daily inspection of the guestrooms and public areas. Petitioner sent
a memorandum of protest claiming that Mr. Okawas orders was a form of harassment to
ease him out of his position and illustrated in detail the other forms of alleged
harassment supposedly perpetrated by Mr. Okawa.
He, however, manifested that he had no intention to resign.
- But on May 2, 1990, petitioner went to the Hotels Comptroller asking for his severance
pay of P120,000 plus accrued benefits of P11, 865.28. The check was not given to him
until he submitted his resignation letter (part of standard procedure). He also executed an
Affidavit of Quitclaim, along with his resignation. The very next day, however, respondents
received a letter from petitioner (addressed to Mr. Okawa) who insisted that he was forced
to resign because he could no longer endure Mr. Okawas acts of harassment against him.
2 weeks later, petitioner filed a complaint for illegal dismissal and damages against Hotel
Nikko and its officers, including his direct superiors, Yokoo and Okawa. The LA dismissed
the complaint finding that petitioner voluntarily resigned and that the alleged acts of
harassment were non-existent. On appeal, the NLRC affirmed the LAs decision likewise
finding that petitioner voluntarily resigned as manifested by his act of negotiating for a
huge amount of separation pay. When his MFR was dismissed, he came to the SC.
ISSUE
WON the resignation was forced upon Habana or he did so voluntarily
HELD
The resignation was voluntary.
Ratio Voluntary resignation is the voluntary act of an employee who finds himself in a
situation where he believes that personal reasons cannot be sacrificed in favor of the
exigency of the service and he has no other choice but to disassociate himself from his
employment.
Reasoning
- In this case, petitioner was clearly having trouble performing his job, which undeniably
carries immense responsibilities. Notable too was petitioners failure to see eye to eye with
his immediate bosses, Mr. Yokoo and Mr. Okawa. Because of these difficulties, it was
quite reasonable for petitioner to think of, and eventually, relinquishing his position
voluntarily (and get a fat sum as severance pay in the bargain) instead of waiting to be
fired.
- Petitioner laments that he was completely stripped of his powers and functions as
Director when Mr. Okawa tasked him with inspecting the hotels guest and public areas.
Conducting these daily inspections, in effect, demoted him to a mere room inspector one
notch higher than a bellboy. He claims that the humiliation he endured in going room to
room, inspecting toilets and garbage areas, was all part of a malicious scheme to harass
him out of his position. These orders were not borne out of mere whim and caprice. They
were made in response to the complaints they were getting. Moreover, these measures
executed by the hotels top management were legitimate exercise of management
prerogatives.
- Petitioner asserts that private respondents coerced and intimidated him to resigning
through their collective acts of harassment. Contrariwise, private respondents contend that
it was petitioner who approached them indicating his desire to resign due to his difficulty in
coping with his responsibilities and his differences with his immediate boss, Mr. Okawa.
- Petitioner could not have been intimidated by private respondents to quit. In his
memorandum, petitioner emphatically vowed not to resign despite private respondents
alleged acts of harassment. Surprisingly, however, after only a few days he did quit
alleging that he was forced and harassed to do so. If petitioner was adamant in his
intention not to be coerced into leaving, how could he suddenly be forced to resign?
Petitioner glaringly contradicted himself. His excuse is thus, unbelievable and unjustifiable.
- Moreover, the issue in this case is factual in nature and firm is the principle that factual
findings of the NLRC, particularly when they coincide with those of the LA, are accorded
respect, even finality, and will not be disturbed for as long as such findings are supported
by substantial evidence. We have painstaking reviewed the records of this case and we
find no justifiable reason to overturn the findings of both the LA and the NLRC.
Disposition Petition is DISMISSED

REQUISITES

Labor Law 1
AZCOR MANUFACTURING V NLRC (CAPULSO)
303 SCRA 26
BELLOSILLO; February 11, 1999
NATURE
Petition for certiorari
FACTS
- Candido Capulso filed with the Labor Arbiter a complaint for constructive illegal dismissal
and illegal deduction of P50.00 per day for the period April to September 1989.
The evidence presented by Capulso showed that he worked for AZCOR as ceramics
worker for more than two (2) years starting from 3 April 1989 to 1 June 1991. From April to
September 1989 the amount of P50.00 was deducted from his salary without informing
him of the reason therefor.
- In the second week of February 1991, upon his doctor's recommendation, Capulso
verbally requested to go on sick leave due to bronchial asthma. It appeared that his illness
was directly caused by his job as ceramics worker where, for lack of the prescribed
occupational safety gadgets, he inhaled and absorbed harmful ceramic dusts. His
supervisor, Ms. Emily Apolinaria, approved his request. Later, on 1 June 1991, Capulso
went back to petitioner AZCOR to resume his work after recuperating from his illness. He
was not allowed to do so by his supervisors who informed him that only the owner, Arturo
Zuluaga, could allow him to continue in his job. He returned five (5) times to AZCOR but
when it became apparent that he would not be reinstated, he immediately filed the instant
complaint for illegal dismissal.
- Capulso presented the following documentary evidence in support of his claim: (a) His
affidavit and testimony to prove that he was terminated without just cause and without due
process; (b) Identification card issued by AZCOR which he continued to use even after his
supposed employment by Filipinas Paso; (c) Certification of SSS premium payments; (d)
SSS Member Assistance Form wherein he stated that he worked with AZCOR from March
1989 to April 1991; (e) Certification of Employee Contribution with SSS; and, (f) Payslips
issued by AZCOR.
- AZCOR alleged that Capulso was a former employee of AZCOR who resigned on 28
February 1990 as evidenced by a letter of resignation and joined Filipinas Paso on 1
March 1990 as shown by a contract of employment; in February 1991 Capulso allegedly
informed his supervisor, Ms. Emilia Apolinaria, that he intended to go on terminal leave
because he was not feeling well; on 1 March 1991 he submitted a letter of resignation
addressed to the President of Filipinas Paso, Manuel Montilla; and, in the early part of
June 1991 Capulso tried to apply for work again with Filipinas Paso but there was no
vacancy.
- Petitioners submitted the following documentary evidence: (a) Sworn Statement of Ms.
Emilia Apolinaria and her actual testimony to prove that respondent indeed resigned
voluntarily from AZCOR to transfer to Filipinas Paso, and thereafter, from Filipinas Paso
due to failing health; (b) Contract of Employment between Filipinas Paso and respondent
which took effect 1 March 1991; (c) Letter of resignation of respondent from AZCOR dated
28 February 1990, to take effect on the same date; (d) Undated letter of resignation of
respondent addressed to Filipinas Paso to take effect 1 March 1991; (e) BIR Form No. W4 filed 6 June 1990; (f) Individual Income Tax Return of respondent for 1990; and, (g) BIR
Form 1701-B which was an alphabetical list of employees of Filipinas Paso for the year
ending 31 December 1990.
- Labor Arbiter rendered a decision dismissing the complaint for illegal dismissal for lack of
merit, but ordered AZCOR and/or Arturo Zuluaga to refund to Capulso P200.00
representing the amount illegally deducted from his salary.
- NLRC modified the Labor Arbiter's decision by: (a) declaring the dismissal of Capulso as
illegal for lack of just and valid cause; (b) ordering petitioners to reinstate Capulso to his
former or equivalent position without loss of seniority rights and without diminution of
benefits; and, (c) ordering petitioners to jointly and solidarily pay Capulso his back wages
computed from the time of his dismissal up to the date of his actual reinstatement.
- Petitioners' motion for reconsideration was denied by the NLRC. Meanwhile, during the
pendency of the case before this Court, Capulso succumbed to asthma and heart disease,
and died.
- Petitioners insist that Capulso voluntarily resigned. They also contend that they could
not be held jointly and severally liable for back wages since AZCOR and Filipinas Paso
are separate and distinct corporations with different corporate personalities; and, the mere
fact that the businesses of these corporations are interrelated and both owned and
controlled by a single stockholder are not sufficient grounds to disregard their separate
corporate entities.
ISSUE
WON NLRC erred in finding that Capulso was illegally dismissed and in holding petitioners
jointly and solidarily liable to Capulso for back wages
HELD
NO
- On resignation, requisites

A2010

- 196 -

Disini

Ratio To constitute a resignation, it must be unconditional and with the intent to operate
as such. There must be an intention to relinquish a portion of the term of office
accompanied by an act of relinquishment.
- The fact that Capulso signified his desire to resume his work when he went back to
petitioner AZCOR after recuperating from his illness, and actively pursued his case for
illegal dismissal before the labor courts when he was refused admission by his employer,
negated any intention on his part to relinquish his job at AZCOR.
- a closer look at the subject resignation letters readily reveals the following: (a) the
resignation letter allegedly tendered by Capulso to Filipinas Paso was identically worded
with that supposedly addressed by him to AZCOR; (b) both were pre-drafted with blank
spaces filled up with the purported dates of effectivity of his resignation; and, (c) it was
written in English, a language which Capulso was not conversant with considering his low
level of education. No other plausible explanation can be drawn from these circumstances
than that the subject letters of resignation were prepared by a person or persons other
than Capulso. And the fact that he categorically disowned the signatures therein and
denied having executed them clearly indicates that the resignation letters were drafted
without his consent and participation.
- Even assuming for the sake of argument that the signatures were genuine, the
resignation letters still cannot be given credence in the absence of any showing that
Capulso was aware that what he was signing then were in fact resignation letters or
that he fully understood the contents thereof.
- On illegal dismissal
> In illegal dismissal cases, the onus of proving that the dismissal of the employee was for
a valid and authorized cause rests on the employer, and failure to discharge the same
would mean that the dismissal is not justified and therefore illegal.
- On joint and several liability
> The doctrine that a corporation is a legal entity or a person in law distinct from the
persons composing it is merely a legal fiction for purposes of convenience and to
subserve the ends of justice. This fiction cannot be extended to a point beyond its reason
and policy. Where, as in this case, the corporate fiction was used as a means to
perpetrate a social injustice or as a vehicle to evade obligations or confuse the legitimate
issues, it would be discarded and the two (2) corporations would be merged as one, the
first being merely considered as the instrumentality, agency, conduit or adjunct of the
other.
Disposition petition is DISMISSED. NLRC Decision is MODIFIED. Petitioners AZCOR
MANUFACTURING, INC., FILIPINAS PASO and ARTURO ZULUAGA are ORDERED to
pay, jointly and solidarily, the heirs of private respondent Candido Capulso the amounts
representing his back wages, inclusive of allowances and other benefits, and separation
pay to be computed in accordance with law.

METRO TRANSIT ORG V NLRC (GARCIA)


284 SCRA 308
BELLOSILLO; January 16 1998
FACTS
- Garcia had been working for Metro Transit (METRO) for almost 8 years as station teller.
On April 22 1992, he called up his immediate supervisor if he could go on LOA to go to
Cebu to look for his wife and children who suddenly left home without his knowledge. After
a few weeks of fruitless search he returned to Manila.
- When he reported to the office on May 15 1992 Garcia was not allowed to resume work
but was directed to proceed to the legal department of METRO where he would undergo
investigation. He was asked by Pili about his absence from work. After he explained to Pili
his predicament, Pili cut short the inquiry and informed him right away that it would be
better for him to resign rather than be terminated for his absences. Still in a state of
extreme agitation and weighed down by a serious family problem, Garcia at once
prepared a resignation letter. Then he left again for the province to look for his family. But
like his first attempt his effort came to naught. Soon after the Personnel Committee of
METRO approved his resignation.
- Garcia sought advice from his labor union and asked that the union intervene in his case
by bringing the matter of his forced resignation before their grievance machinery for
arbitration. METRO rejected Garcia's plea that he be not considered resigned from his
employment. Garcia filed a complaint for illegal dismissal. Labor Arbiter and NLRC ruled in
favor of Garcia.
- Petitioner: private respondent absented himself on 22 April 1992 without official leave
and then later on freely and willingly relinquished his employment because he was
establishing his own business.
ISSUE
WON Garcia resigned from his employment
HELD
NO

Labor Law 1
- An examination of the circumstances surrounding the submission of the letter
indicates that the resignation was made without proper discernment so that it could
not have been intelligently and voluntarily done.
- What Pili did as petitioner's representative was to advise Garcia, who at that time was
thoroughly confused and bothered no end by a serious family problem, that he had better
resign or face the prospect of an unceremonious termination from service for
abandonment of work. At that precise moment, the employee could not be said to have
fully understood what he was doing, i.e., writing his resignation letter, nor could have
foreseen the consequences thereof, for it is established that as soon as he came out of
the investigation office he prepared his resignation letter right then and there at a table
nearby with no time for reflection. It is noteworthy that shortly thereafter he consulted his
union president for help regarding his forced resignation. This does not indicate by any
means a resignation that was knowingly and voluntarily done. On the contrary, it shows
that his writing and handing in the resignation letter to petitioner were a knee-jerk
reaction triggered by that singular moment when he was left with no alternative but
to accede, having been literally forced into it by being presented with the more
unpleasant fate of being terminated.
- the voluntariness of complaint's resignation can hardly be believed if he was not forced
by circumstances due to the following:
- First he was already in the employ of respondent for almost eight years with a high
paying job and benefits; Second, no offense or violation has been attributed to the
complainant during his period of employment; Third, the filing of this instant complaint by
the complainant for illegal dismissal negates or is inconsistent with abandonment and
voluntary resignation. Lastly, there is no iota of evidence that complainant is indeed
engaged in business, and belies the contents of his resignation.
- Evidently the complainant was asked to make a choice whether to tender his resignation
or be terminated for his absences which to our mind is anchored on justifiable grounds.
Such compulsion to make an unnecessary choice placed undue and unjustifiable pressure
on the employee who otherwise would not have thought of leaving his position as Station
Teller if he had not been induced to do so. This being the case, the resignation filed by the
complainant did not become effective.

VOLUNTARY RESIGNATION
PHIL WIRELESS INC V NLRC (LUCILA)
310 SCRA 653
PARDO; July 20, 1999
NATURE
Petition for certiorari to set aside a decision of the NLRC
FACTS
- January 8, 1976 Phil. Wireless Inc. (Pocketbell) hired respondent Doldwin Lucila as an
operator/encoder. Three years later, Lucila was promoted as Head Technical and
Maintenance Department of the Engineering Department. On September 11, 1987, he
was promoted as Technical Services Supervisor and later on October 1, 1990, he became
Project Management Superintendent.
- December 8, 1990 Lucila tendered his resignation.
- December 3, 1991 Lucila filed with the NLRC a complaint for illegal/constructive
dismissal.
- Lucila alleges that his promotion from Technical Services Supervisor to Project
Management Superintendent was actually a demotion because it was demeaning, illusory
and humiliating. He based it on the fact that he was not given a secretary/assistant and
subordinates.
- June 29, 1992 Labor Arbiter Villarente declared that Lucila actually resigned and
dismissed the complaint for lack merit.
- June 15, 1993 NLRC reversed the findings of the Labor Arbiter and ordered for Lucilas
reinstatement with payment of backwages or separation pay.
ISSUE
WON Lucila was constructively dismissed
HELD
NO
Ratio Constructive dismissal is an involuntary resignation resorted to when continued
employment is rendered impossible, unreasonable or unlikely; when there is a demotion in
rank and/or diminution in pay; or when a clear discrimination, insensibility or disdain by an
employer becomes unbearable to the employee.
Reasoning
- In this case, the Court ruled that Lucila voluntarily resigned and was not pressured into
doing so.
- Voluntary resignation is defined as the act of en employee who finds himself in a
situation where he believes that personal reasons cannot be sacrificed in favor of the

A2010

- 197 -

Disini

exigency of the service and he has no other choice but to disassociate himself from his
employment.
- Lucilas basis for his demotion is inadequate as the Court ruled that there is no
demotion where there is no reduction in position rank or salary as a result of such transfer.
Disposition The petition is hereby granted. The questioned decision of the NLRC is set
aside and the decision of the Labor Arbiter is reinstated and affirmed. No costs.

PASCUA V NLRC
287 SCRA 554
PANGANIBAN; March 13, 1998
NATURE
Review on certiorari
FACTS
- The complainants are among the employees of Henry Lao at the Tiongsan Super
Bazaar. On August 7, 1991, Henry Lao received a telephone call who informed him that
one of his sales ladies had just stolen a Karaoke, the previous night. There, said
saleslady made a confession, that, there were others who were involved in the stealing of
goods. She was required by Henry Lao to write down their names. Violeta Soriano and
Susan Castillo were included in her list. The eighteen (18) sales ladies who admitted their
guilt resigned. The remaining workers were placed under the watchful eyes of
respondent.
- On August 21, 1991, Lilia Pascua was caught repairing three (3) pairs of pants that
belonged to Mrs. Manaois and allegedly were not bought at the Tiongsan Super Bazaar.
Respondent scolded Lilia Pascua for this offense, because it is against the respondents
policy that repair jobs of items not bought at the bazaar should not be accepted. She was
given a warning, that this prohibition should be strictly followed. Lilia Pascua did not report
for work the next day. She went to see the respondents bookkeeper for the computation
of her separation pay. Respondent paid her separation pay.
- On August 24, 1991, Victoria Santos was caught charging a meter of a cloth for the
price of a yard. For this offense, she was suspended for a period of thirty (30) days. She
never returned to work since then.
- Mimi Macanlalay was employed on June 10, 1989. Previously, she worked for Mrs. Tan.
On September 19, 1991, Mrs. Tan went to the Tiongsan Super Bazaar, and she saw Mimi
Macanlalay working as a cashier. Mrs. Tan informed Mr. Lao, that Mimi Macanlalay was
previously dismissed by her for dishonesty. Mimi Macanlalay later on resigned.
- Violeta Soriano was employed on May 16, 1984. After the August 7, 1991 incident, she
was assigned as a cashier. She was reverted back as a sales lady after a few weeks
when Mr. Lao learned, that, she had some knowledge of the schemes of the resigned
employees. On November 9, 1991, Mr. Lao required her to explain in writing, why she
should not be the subject matter of a disciplinary action, for her failure to fill up her daily
time record. Respondent reviewed her past records and found out that, she was the
subject matter of a disciplinary action in the past. She was terminated [sic] on December
8, 1991.
- Susan De Castro refused to receive her salary on November 18, 1991, because she
insisted on receiving more than what is indicated in the payrolls. Respondent told her that
if she is not satisfied with her salary, she can find employment elsewhere. She failed to
report for work on the following day. In any case, respondent states, that, she can be
dismissed for lack of trust and confidence, for her involvement in the pilferage of goods.
Petitioners filed at the Regional Arbitration Branch of the NLRC separate complaints
against Henry Lao for illegal dismissal and claims for violation of labor standards
pertaining to payment of wages. Subsequently, the labor arbiter ruled that the dismissals
were illegal and awarded back wages and separation pay to petitioners.
- The NLRC, which modified the appealed decision and found the termination of
petitioners employment to be due either to voluntary resignation or dismissals with just
cause.
ISSUES
1. WON petitioners employment terminated because of resignation, abandonment or
dismissal
2. WON petitioners employment terminated in accordance with law
HELD
1. ILLEGAL DISMISSAL (except for Santos).
- Petitioner Pascua was aware of the close relationship between Henry Lao and Mrs.
Manaois. Thus, Pascua feared that, if she turned down Mrs. Manaois request, she would
be subjected to public scolding by Lao. Thus, accommodation of the said request may
have been an act of disobedience of her employers order, but hardly an instance of the
wrongful and perverse attitude that would warrant a penalty as grave as dismissal. That
after the incident, Henry Lao kept pushing me by my shoulders as he repeatedly told me in
a loud manner, pakuwenta mo na ang separation pay mo at hindi ka na rin makakabalik.
Puntahan mo ang accountant. which made me nervous and afraid especially that he kept

Labor Law 1
on pushing me even when I was already on top of the stairs; It is evident from the above
that Petitioner Pascua was forced to resign -- an act which was tantamount to a dismissal,
an illegal one at that.
- The NLRC could not explain the contradictions in Petitioner De Castros case. If she had
not been dismissed but was still an employee of private respondent, then why did she file
this case for illegal dismissal? And even more perplexing: Why would the NLRC conclude
that reinstatement was no longer possible because of the parties respective imputations
of charges against each other? Furthermore, the labor arbiters finding that there was no
evidence on record to establish her dismissal is refuted by the uncontested allegations of
Petitioner De Castro.
- Prior to her employment at Tiongsan Super Bazaar Petitioner Macanlalay had been a
saleslady at Rommels which was owned by a certain Mrs. Tan. On September 20, 1991,
while she was working as a cashier at Tiongsan, Mrs. Tan saw her; thereupon, Mrs. Tan
reported to Henry Lao that Petitioner Macanlalay had previously been dismissed for
alleged dishonesty. Petitioner was then called by Lao and unceremoniously told: Kunin
mo na ang separation pay mo. Pa total mo na sa accountant. At huwag ka ng
magtrabajo dito. Clearly, she did not resign; she was orally dismissed by Lao. It is this
lack of clear, valid and legal cause, not to mention due process, that made her dismissal
illegal, warranting reinstatement and the award of back wages.
- The NLRC justified Petitioner Sorianos dismissal by alleging that it was due to her failure
to make regular entries in her daily time records. We believe, however, that this alleged
just cause was convincingly disputed by Petitioner Soriano in her letter dated November
9, 1991.
- We agree that Petitioner Santos voluntarily resigned. The labor arbiter did not find
Petitioner Santos to have been illegally dismissed. Rather, after her suspension for
charging for a meter of cloth bought [at] the price of a yard, she offered to resign. The
solicitor general supports this by stating that even the Labor Arbiter discovered this when
he ruled that there [was] no evidence on record to support Santos dismissal.
2. NO
Reasoning
- Basic is the doctrine that resignation must be voluntary and made with the intention of
relinquishing the office, accompanied with an act of relinquishment. Based on the
evidence on record, we are more than convinced that Petitioners Lilia Pascua, Mimi
Macanlalay, Susan C. De Castro and Violeta Soriano did not voluntarily quit their jobs.
Rather, they were forced to resign or were summarily dismissed without just cause.
Petitioners -- except Victoria L. Santos -- forthwith took steps to protest their layoff and
thus cannot, by any logic, be said to have abandoned their work.
- In labor cases, the employer has the burden of proving that the dismissal was for a just
cause; failure to show this, as in the instant case, would necessarily mean that the
dismissal was unjustified and, therefore, illegal. To allow an employer to dismiss an
employee based on mere allegations and generalities would place the employee at the
mercy of his employer; and the right to security of tenure, which this Court is bound to
protect, would be unduly emasculated. Considering the antecedents in the summary
dismissals effected against Petitioners Pascua, Macanlalay, De Castro and Soriano, the
causes asserted by private respondent are, at best, tenuous or conjectural; at worst, they
are mere afterthoughts.
- Under the Labor Code, as amended, the dismissal of an employee which the employer
must validate has a twofold requirement: one is substantive, the other procedural. Not
only must the dismissal be for a just or an authorized cause as provided by law (Articles
282, 283 and 284 of the Labor Code, as amended); the rudimentary requirements of due
process -- the opportunity to be heard and to defend oneself -- must be observed as well.
- Petitioners Pascua and Macanlalays acceptance of separation pay did not necessarily
amount to estoppel; nor did it connote a waiver of their right to press for reinstatement,
considering that such acceptance -- particularly by Petitioner Pascua who had to feed her
four children -- was due to dire financial necessity.
Disposition REVERSED.

AZCOR MANUFACTURING INC V NLRC


[PAGE 197]
VALDEZ V NLRC (NELBUSCO INC)
286 SCRA 87
REGALADO; February 9, 1998
NATURE
Special civil action for certiorari
FACTS
- Sometime in December, 1986, petitioner was hired by private respondent as a bus driver
on commission basis, with an average earning of P6,000.00 a month. On February 28,
1993, the airconditioning unit of the bus which petitioner was driving suffered a mechanical
breakdown. Respondent company told him to wait until the airconditioning unit was

A2010

- 198 -

Disini

repaired. Meanwhile, no other bus was assigned to petitioner to keep him gainfully
employed.
- Thereafter, petitioner continued reporting to his employer's office for work, only to find out
each time that the airconditioning unit had not been repaired. Several months elapsed but
he was never called by respondent company to report for work. Later, petitioner found out
that the bus formerly driven by him was plying an assigned route as an ordinary bus, with
a newly-hired driver.
- On June 15, 1993, petitioner filed a complaint against private respondent for illegal
dismissal, with money claims for labor standard benefits, and for reimbursement of his
bond and tire deposit. He claimed that the reason why respondent company did not allow
him to drive again was due to his refusal to sign an undated company-prepared
resignation letter and a blank affidavit of quitclaim and release.
- Private respondent, on the other hand, admitted that it told petitioner to wait until the
airconditioning unit of the bus was repaired. However, private respondent alleged that after
the bus driven by the petitioner broke down due to his fault and negligence, the latter did
not report for work. He supposedly informed the management later that he was voluntarily
resigning from his employment in order to supervise the construction of his house.
Consequent to his resignation, petitioner demanded the return of his cash bond and tire
deposit. Respondent company required him to secure the necessary management
clearance and other pertinent papers relative to his resignation. Instead of complying with
those requirements, petitioner filed the instant complaint.
ISSUE
WON petitioner was illegally dismissed because he did not voluntarily resigned as claimed
by respondents
HELD
- The reason for the stoppage of operation of the bus assigned to petitioner was the
breakdown of the airconditioning unit, which is a valid reason for the suspension of its
operation. However, such suspension regarding that particular bus should likewise last
only for a reasonable period of time. The period of six months was more than enough for it
to cause the repair thereof. Beyond that period, the stoppage of its operation was already
legally unreasonable and economically prejudicial to herein petitioner who was not given a
substitute vehicle to drive.
- The so-called "floating status" of an employee should last only for a legally prescribed
period of time. When that "floating status" of an employee lasts for more than six months,
he may be considered to have been illegally dismissed from the service. Thus, he is
entitled to the corresponding benefits for his separation, and this would apply to the two
types of work suspension heretofore noted, that is, either of the entire business or of a
specific component thereof.
- It was not denied by private respondent that it tried to force private respondent to sign an
undated company-prepared resignation letter and a blank undated affidavit of quitclaim
and release which the latter validly refused to sign. Furthermore, the bus which petitioner
used to drive was already plying a transportation route as an ordinary bus and was being
driven by another person, without petitioner having been priorly offered the same
alternative arrangement.
- The other allegation of private respondent that petitioner voluntarily resigned from work
obviously does not deserve any consideration. It would have been illogical for herein
petitioner to resign and then file a complaint for illegal dismissal. Resignation is
inconsistent with the filing of the said complaint.
- Resignation is defined as the voluntary act of an employee who finds himself in a
situation where he believes that personal reasons cannot be sacrificed in favor of the
exigency of the service, and, that he has no other choice but to disassociate himself from
his employment. Resignation is a formal pronouncement of relinquishment of an office. It
must be made with the intention of relinquishing the office accompanied by an act of
relinquishment.
- The cardinal rule in termination cases is that the employer bears the burden of proof to
show that the dismissal is for just cause, failing in which it would mean that the dismissal
is not justified. This rule applies adversely against herein respondent company since it has
utterly failed to discharge that onus by the requisite quantum of evidence.
- Under Article 279 of the Labor Code, as amended, an employee who is unjustly
dismissed from work shall be entitled to reinstatement without loss of seniority rights and
other privileges and to his full back wages, inclusive of allowances, and to other benefits
or their monetary equivalent computed from the time his compensation was withheld from
him up to the time of his actual reinstatement.
Disposition Decision of respondent National Labor Relations Commission is SET ASIDE
and the decision of the Labor Arbiter REINSTATED

VALIDITY OF POLICY
MANILA BROADCASTING COMPANY V NLRC
(OLAIREZ, BANGLOY)

Labor Law 1
294 SCRA 486
MENDOZA; 1998
NATURE
Petition for certiorari to set aside the decision of the National Labor Relations Commission,
affirming the decision of the Labor Arbiter which found private respondent to have been
illegally dismissed and which ordered him reinstated with damages.
FACTS
- Private respondent Samuel L. Bangloy was production supervisor and radio
commentator of the DZJC-AM radio station in Laoag City. The radio station is owned by
petitioner Manila Broadcasting Company.
- On February 28, 1992, private respondent applied for leave of absence for 50 days,
from March 24 to May 13, 1992, in order to run for Board Member in Ilocos Norte under
the Kilusang Bagong Lipunan (KBL). He made his application pursuant to 11(b) of R.A.
No. 6646 which provides:
Sec. 11(b) . . . Any mass media columnist, commentator, announcer, or personality who
is a candidate for any elective public office shall take a leave of absence from his work
as such during the campaign period.
- After a week, private respondents application was returned to him, together with a copy
of an office memorandum of Eugene Jusi, Assistant Vice-President for Personnel and
Administration, to Atty. Edgardo Montilla, Executive Vice-President and General Manager
of the FJE Group of Companies, in which it was stated that as a matter of company
policy, any employee who files a certificate of candidacy for any elective national or local
office would be considered resigned from the company.
- It would appear that private respondent nonetheless ran in the election but lost. On May
25, 1992, he tried to return to work, but was not allowed to do so by petitioner on the
ground that his employment had been terminated.
- Private respondent filed a complaint for illegal dismissal against petitioner before the
Department of Labor and Employment.
ISSUES
1. WON the company policy that any employee who files a certificate of candidacy for any
elective national or local office would be considered resigned from the company valid
2. WON the company policy was made known to employees before it was sought to be
applied to private respondent
HELD
1. YES
- the policy is valid and justified.
2. NO
- There are a number of circumstances which raise some doubts whether the company
policy was strictly enforced.
Ratio Although 11(b) of R.A. No. 6646 does not require mass media commentators and
announcers such as private respondent to resign from their radio or TV stations but only to
go on leave for the duration of the campaign period, we think that the company may
nevertheless validly require them to resign as a matter of policy.
- The policy is justified on the following grounds:
1) Working for the government and the company at the same time is clearly
disadvantageous and prejudicial to the rights and interest not only of the company but the
public as well. In the event an employee wins in an election, he cannot fully serve, as he
is expected to do, the interest of his employer. The employee has to serve two (2)
employers, obviously detrimental to the interest of both the government and the private
employer.
2) In the event the employee loses in the election, the impartiality and cold neutrality of an
employee as broadcast personality is suspect, thus readily eroding and adversely affecting
the confidence and trust of the listening public to employers station.
These are valid reasons for petitioner. No law has been cited by private respondent
prohibiting a rule such as that in question.
Disposition Decision AFFIRMED

14.03 NO TERMINATION PERFORMANCE OF


MILITARY OR CIVIC DUTY
C. TERMINATION OF EMPLOYMENT BY
EMPLOYER
1. PRELIMINARY MATTERS

A2010

- 199 -

Disini

14.04 BASIS OF RIGHT AND REQUIREMENTS


BASIS
GUTIERREZ V SINGER SEWING MACHINE
411 SCRA 512
QUISUMBING; September 3, 2003
NATURE
review is the decision of the Court of Appeals
FACTS
- Petitioner Mario Gutierrez was initially hired by Singer Sewing Machine Company as
Audit Assistant on contractual basis in 1993. He became an Accounts Checker on
probationary status on February 8, 1994. Thereafter, he acquired regular status as Asset
Auditor on March 1, 1995, receiving a monthly salary of P4,455, until September 9, 1996,
when he was dismissed from employment. Singer premised the petitioners termination on
the following incidents:
- On August 1, 1996, at around 3:15 p.m., Ms. Emelita Garcia, Personnel Supervisor of
Singer, caught Gutierrez and three other Asset Auditors, watching a video tape inside the
Asset/Legal Department Office. Despite Ms. Garcias reminder that it was no longer break
time and that the other occupants of the room might be disturbed, Gutierrez and company
ignored Ms. Garcia and continued to watch the video. The following day, August 2, 1996,
Ms. Evangeline Que-Ilagan, Administration Manager of Singer, noticed a sign posted at
the door of the Asset/Legal Department Office, which read MAIPARIT TI UMISBO DITOY.
When she asked who placed the sign at the door, Gutierrez admitted responsibility. When
Ms. Que-Ilagan asked what it meant, Gutierrez answered, BAWAL ANG UMIHI DITO (No
Urinating Here). Ms. Que-Ilagan then asked if Gutierrez had seen anyone urinate at the
door where the sign was posted and the latter replied in the negative. Ms. Que-Ilagan then
asked why he placed such a sign, to which Gutierrez replied, Gusto ko, eh (It is my
pleasure). She admonished him not to do the same thing again and requested him to
remove the sign, but Gutierrez refused to do so.
- Later that same day, August 2, 1996, Gutierrez personally explained his side to the Asset
Manager, Mr. Leonardo Consunji, at the latters office. Gutierrez claimed that he only
admitted to the posting of the sign in order to take the cudgels for a co-employee. He also
explained that their use of the video equipment was upon the orders of their supervisor,
Mr. Romeo C. Ninada. The latter wanted to test the quality of their video players. Mr.
Consunji brought the matter to the attention of Mr. Ninada. The latter promptly issued a
Memo dated August 6, 1996, requiring Gutierrez to explain his side. Gutierrez then
informed Mr. Ninada that he had already discussed the matter with Mr. Consunji. In his
letter to Mr. Consunji dated August 21, 1996, Mr. Ninada opined, [T]he case does not
deserve to be devoted with too much time and effort as he considered it a minor
offense.
- Nevertheless, Mr. Consunji issued a Memo dated August 28, 1996, informing Gutierrez
of the latters violation of company rules and regulations, specifically citing the following:
> Part V-B.9 Use of Companys time, materials, equipment and other assets for personal
use or business; and
> Part V-B.18 Acts of vandalism such as defacing or destroying Company documents and
records; posting, altering or removing any printed matter, announcements or signs in the
Bulletin Boards unless specifically authorized.
- Under the Company Code of Discipline, these infractions were classified as 4 th Degree
Offenses with the corresponding sanction of dismissal. In the same Memo, Gutierrez was
directed to explain in writing why the aforesaid penalty should not be imposed on him. He
was given until August 30, 1996, to comply with the directive. As Gutierrez insisted that
he had previously verbally explained his side to Mr. Consunji, no written explanation was
submitted by him.
- On September 9, 1996, another Memo was issued by Mr. Consunji, worded as follows:
> After a thorough investigation of the incident and after having found your explanations to
be unsatisfactory and due to your refusal to comply with my memo to you dated August
28, 1996 which constitutes willful defiance or disregard of Company authority, the
management deems it fitting and proper to impose upon you the penalty of dismissal
effective immediately upon receipt hereof.
- On September 19, 1996, petitioner filed a motion/request for reconsideration with Singer,
but the latter stood pat on its decision to dismiss him.
- Thus, petitioner filed the complaint for illegal dismissal with claims for damages before
the Labor Arbiter, docketed as NLRC NCR Case No. 00-10-06201-96. In a decision dated
August 13, 1997, Labor Arbiter Renato A. Bugarin dismissed the complaint for lack of
merit.
- Aggrieved, Singer filed a petition for certiorari with this Court, which in turn was referred,
by resolution dated December 2, 1998, to the Court of Appeals.The Court of Appeals
reversed the NLRC, thereby upholding and reinstating the decision of the Labor Arbiter.

Labor Law 1
Gutierrez now comes to the Court via a petition for review on certiorari seeking to reverse
and set aside the decision of the Court of Appeals, with a prayer for moral damages and
attorneys fees.
ISSUE
WON the appellate court erred in reversing the NLRC which declared respondents guilty
of illegal dismissal of the petitioner from his employment
HELD
YES
Ratio We agree with the NLRC that petitioners dismissal from employment was
unjustified and illegal. Petitioners dismissal was based on his alleged violation of two
company rules and regulations, namely: (1) acts of vandalism; and (2) use of companys
time, materials, equipment and other assets for personal use/business. These acts were
found by the Labor Arbiter to constitute serious misconduct or willful disobedience under
paragraph (a) of Article 282 of the Labor Code. The Labor Arbiter characterized Gutierrez
undesirable or unreasonable behavior and unpleasant deportment with his fellow
employees, all the more his supervisors, as within the scope of the analogous just causes
for termination under paragraph (e) of the same article.
- Singer averred that petitioners defiance of the reasonable rules and regulations being
implemented by Singer was enough reason for his dismissal. Singer emphasized that the
two violations of company rules and regulations on the two consecutive days, were
manifestations that petitioner was challenging the authorities of Singer.
In its impugned decision, however, the NLRC stated:
- We agree with the complainant that the questioned poster contained an innocuous and
harmless statement, which when translated in tagalog means Bawal Umihi Dito and that
such posting cannot be interpreted as an act of vandalism. The affidavit of Ms. Ilagan, in
relation with such poster, is not sufficient to establish complainants guilt of vandalism.
The complainant likewise justified his action in relation to his act of watching video films
during office hours by arguing that he, together with four (4) other co-employees, were
asked by their immediate supervisor, Mr. Romy Ninada to test the video tape player. Such
claim was not denied by Mr. Ninada, who could have been easily required by the
respondents to do so. Mr. Ninada was the logical officer to negate the claim of the
complainant that he was authorized to test the quality of the VHS and CTV 143 to
guarantee the excellency (sic) of respondent firms products.
- Though no admission was made that the use of the video player was upon the orders of
the immediate supervisor of Gutierrez, Mr. Ninada himself considered the same to be a
minor infraction, not worth the time and effort of the company spent on the matter.
- We might add that, as contended by petitioner, the act of posting the sign does not fall
squarely within the scope of the cited company rules and regulations, Part V-B.18, on
vandalism. The rule prohibits unauthorized posting in the Bulletin Board, while the
present case involved posting of a sign at one of the office doors, a different matter. We
must also stress that, even on the assumption that Gutierrez in fact committed the cited
infractions, in our view they are not major violations but only minor ones which do not
merit the supreme penalty of dismissal from employment. Time and again, this Court has
underscored the need for restraint in the dismissal of workers:
- Extreme caution should be exercised in terminating the services of a worker for his job
may be the only lifeline on which he and his family depend for survival in these difficult
times. That lifeline should not be cut off except for a serious, just and lawful cause, for, to
a worker, the loss of his job may well mean the loss of hope for a decent life for him and
his loved ones.
- In the present case, the penalty of dismissal appears in our view unjustified, much too
harsh and quite disproportionate to the alleged infractions. Not only were the alleged
violations minor in nature, in this case the evidence adduced to prove them did not fairly
show they fall exactly within the rules and regulations allegedly violated. Otherwise stated,
the evidence did not square fully with the charges. That is why the Labor Arbiter found
only analogous causes which, in our view do not sufficiently justify the extreme penalty of
termination.
- The penalty imposed on the erring employee ought to be proportionate to the offense,
taking into account its nature and surrounding circumstances. In the application of labor
laws, the courts and other agencies of the government are guided by the social justice
mandate in our fundamental law.
- To be lawful, the cause for termination must be a serious and grave malfeasance to justify
the deprivation of a means of livelihood. This is merely in keeping with the spirit of our
Constitution and laws which lean over backwards in favor of the working class, and
mandate that every doubt must be resolved in their favor.
- To conclude, the Court of Appeals erred in reversing the decision of the NLRC which
declared respondents guilty of illegal dismissal.

MANILA TRADING AND SUPPLY CO INC V ZULUETA


69 PHIL 485
LAUREL; January 30, 1940

A2010

Disini

- 200 -

NATURE
Petition for Certiorari
FACTS
- On July 7, 1938, the Secretary of Labor apprised the Court of Industrial Relations of a
labor dispute existing between the petitioner company and its employees who were
members of the Philippine Labor Union
- A preliminary hearing was held after which, on August 6, 1938 the respondent court
entered an order requiring the company, inter alia not to dismiss any of its employees and
laborers except for good cause and with its permission.
- Subsequently, on June 30, 1939, one of the gatekeepers of the petitioners, Filomeno
Ramollo, was suspended for a breach of duty. The breach consisted in that as gatekeeper
of the petitioner he permitted, contrary to instructions, one of the customers to pass thru
the exit gate without paying for the work done on the car. Before this, it is also alleged that
he refused to work in the setting up department of the company when ordered by his
superior.
- The Philippine Labor Union submitted a petition requesting the reinstatement of the
suspended laborer, to which an answer was filed by the company.
- In its order of July 28, 1939, the respondent court found that the laborer was guilty of the
breach imputated to him, but, deciding that his suspension from June 30 to July 28, 1939
was a sufficient punishment, ordered his immediate reinstatement.
- The petitioner moved for reconsiderations, but the respondent Court of Industrial
Relations, sitting in banc, denied the motion.
ISSUE
WON the Court of Industrial Relations can order the readmission of a laborer who has
been found derelict in the performance of his duties
HELD
NO
- The right of an employer to freely select or discharge his employees, is subject to
regulation by the State. An employer cannot legally be compelled to continue with the
employment of a person who admittedly was guilty of misfeasance or malfeasance
towards his employer, and whose continuance in the service of the latter is patently
inimical to his interest. The law, in protecting the rights of the laborer, authorizes neither
oppression nor self-destruction of the employer. There may, of course, be cases where the
suspension or dismissal of an employee is whimsical or unjustified or otherwise illegal
scrutinized carefully and the proper authorities will go to the core of the controversy and
not close their eyes to the real situation.
Disposition Writ of Certiorari granted

AGABON V NLRC
[PAGE 35]
PLDT V TOLENTINO
438 SCRA 555
CORONA; September 21, 2004
FACTS
- Arturo R. Tolentino Tolentino was employed in petitioner PLDT for 23 years.
- He started in 1972 as an installer/helper and, at the time of his termination in 1995, was
the division manager of the Project Support Division, Provincial Expansion Center, Meet
Demand Group.
- His division was in charge of the evaluation, recommendation and review of documents
relating to provincial lot acquisitions. Sometime in 1995, Jonathan de Rivera, a supervisor
directly under respondent Tolentino, was found to have entered into an internal
arrangement with the sellers of a parcel of land which he recommended for acquisition
under PLDTs expansion program. Quirino Donato, the attorney-in-fact of the landowner,
executed an affidavit disclosing his internal arrangement with de Rivera.
- Donatos affidavit revealed that all follow-up calls regarding the transaction were to be
directed to the office of respondent and de Rivera. Upon being apprised of this internal
arrangement, PLDT dismissed de Rivera. After he was dismissed, de Rivera submitted a
sworn statement to PLDT implicating respondent as the person behind the anomalous
internal arrangement. Respondent, in an affidavit, denied this and pointed out that his
authority to approve real estate acquisitions was limited to land valued below P200,000.
- Petitioner PLDT sent a notice of dismissal, effective October 27, 1995, to respondent
Tolentino. Attached to this notice was a handwritten note from Nicanor E. Sacdalan, VicePresident of the Provincial Expansion Center, Meet Demand Group, giving respondent
Tolentino the option to resign. Petitioner did not grant respondents request for a formal
hearing but delayed the implementation of his dismissal. On December 4, 1995, petitioner
informed respondent that his dismissal was already final and effective on December 5,
1995.

Labor Law 1
- Respondent then filed a complaint for illegal dismissal, moral and exemplary damages
and other monetary claims against petitioner PLDT in January, 1996. The labor arbiter
found that petitioner PLDT failed to prove and substantiate the charges against respondent
- On appeal, the NLRC reversed the labor arbiters decision on the ground that respondent
was a managerial employee and that loss of trust and confidence was enough reason to
dismiss him.
- Respondents petition for certiorari was referred by this Court to the Court of Appeals
which rendered the assailed decision reinstating the decision of the labor arbiter, that is,
ordering respondents reinstatement.

A2010

- 201 -

Disini

deferential act by management makes us doubt PLDTs claim that its relations with
respondent were strained. The option to resign would not have been given had animosity
existed between them.
- Furthermore, respondent was dismissed in December, 1995 when petitioner PLDT was
still under the Cojuangco group. PLDT has since then passed to the ownership and control
of its new owners, the First Pacific group which has absolutely nothing to do so with this
controversy. Since there are no strained relations between the new management and
respondent, reinstatement is feasible.
Disposition The petition was denied.

ISSUE
WON the Court of Appeals erred in ruling that the dismissal was not founded on clearly
established facts sufficient to warrant separation from employment
HELD
NO
- The petition is without merit. PLDTs basis for respondents dismissal was not enough to
defeat respondents security of tenure.
- There is no dispute over the fact that respondent was a managerial employee and
therefore loss of trust and confidence was a ground for his valid dismissal. The mere
existence of a basis for the loss of trust and confidence justifies the dismissal of the
employee because:
[w]hen an employee accepts a promotion to a managerial position or to an office
requiring full trust and confidence, she gives up some of the rigid guaranties available to
ordinary workers. Infractions which if committed by others would be overlooked or
condoned or penalties mitigated may be visited with more severe disciplinary action. A
companys resort to acts of self-defense would be more easily justified.
- Proof beyond reasonable doubt is not required provided there is a valid reason for the
loss of trust and confidence, such as when the employer has a reasonable ground to
believe that the managerial employee concerned is responsible for the purported
misconduct and the nature of his participation renders him unworthy of the trust and
confidence demanded by his position.
- However, the right of the management to dismiss must be balanced against the
managerial employees right to security of tenure which is not one of the guaranties he
gives up. This Court has consistently ruled that managerial employees enjoy security of
tenure and, although the standards for their dismissal are less stringent, the loss of trust
and confidence must be substantial and founded on clearly established facts sufficient to
warrant the managerial employees separation from the company. Substantial evidence is
of critical importance and the burden rests on the employer to prove it. Due to its subjective
nature, it can easily be concocted by an abusive employer and used as a subterfuge for
causes which are improper, illegal or unjustified.
- In the case at bar, this Court agrees with the Court of Appeals that the petitioners
dismissal was not founded on clearly established facts sufficient to warrant separation from
employment. The factual findings of the court a quo on the issue of whether there was
sufficient basis for petitioner PLDT to dismiss respondent Tolentino are binding on this
Court. In the exercise of the power of review, the factual determinations of the Court of
Appeals are generally conclusive and binding on the Supreme Court.
- The evidence relied upon by petitioner PLDT de Riveras sworn statement and
Donatos affidavit does not, in our view, establish respondent Tolentinos complicity in
the internal arrangement engineered by his subordinate de Rivera.
- To be sure, respondent Tolentino was remiss in his duties as division manager for failing
to discover the internal arrangement contrived by his subordinate. However, dismissal
was not the proper sanction for such negligence. It was not commensurate to the lapse
committed, especially in the light of respondents unblemished record of long and
dedicated service to the company. In Hongkong Shanghai Bank Corporation vs. NLRC, we
had occasion to rule that:
The penalty imposed must be commensurate to the depravity of the malfeasance,
violation or crime being punished. A grave injustice is committed in the name of justice
when the penalty imposed is grossly disproportionate to the wrong committed.
[D]ismissal is the most severe penalty an employer can impose on an employee. It
goes without saying that care must be taken, and due regard given to an employees
circumstances, in the application of such punishment.
- Certainly, a great injustice will result if this Court upholds Tolentinos dismissal.
An employee illegally dismissed is entitled to full backwages and reinstatement
pursuant to Article 279 of the Labor Code, as amended by RA 6715.
- Although a managerial employee, respondent should be reinstated to his former position
or its equivalent without loss of seniority rights inasmuch as the alleged strained relations
between the parties were not adequately proven by petitioner PLDT which had the burden
of doing so. In Quijano vs. Mercury Drug Corporation, the Court ruled that strained
relations are a factual issue which must be raised before the labor arbiter for the proper
reception of evidence. In this case, petitioner PLDT only raised the issue of strained
relations in its appeal from the labor arbiters decision. Thus, no competent evidence exists
in the records to support PLDTs assertion that a peaceful working relationship with
respondent Tolentino was no longer possible. In fact, the records of the case show that
PLDT, through VP Sacdalan, gave respondent Tolentino the option to resign. [18] Such a

PEREZ V MEDICAL CITY GENERAL HOSPITAL


484 SCRA 138
AZCUNA; March 6, 2006

NATURE
Petition for certiorari
FACTS
- September 9, 1999:Prompted by
reports of missing medicines and
supplies
in
the
Emergency
Room/Trauma Room (ER/TR) and upon
the suggestion of one of the Hospitals
staff nurses, Medical City General
Hospital, opened
22
lockers
of
employees assigned to the ER/TR. The
Hospital found four lockers with items
belonging to it. The employees
corresponding
to
the
lockers
(Dominador Perez, Celine Campos,
Lailanie Espiritu and Mateo Butardo)
were directed to submit written
explanations as to why these items
were inside their lockers.
- Perez, Campos and Butardo submitted
their
written
explanations,
while
Espiritu
opted
to
resign.
An
administrative hearing was held where
the three employees who responded
were represented by a union counsel.
At the end of the proceedings, the
charge against Butardo was dismissed
while Perez and Campos, herein
petitioners, were found to have
violated category seven of the company

Labor Law 1
rules, a serious infraction meriting
dismissal. The Hospital offered them
the opportunity to voluntarily resign
with separation pay, under a clause
provided in the Collective Bargaining
Agreement. They refused and the
Hospital dismissed them from the
service.
- January 19, 2000: petitioners filed a
complaint for illegal dismissal with the
NLRC.
- Labor Arbiter found respondents
guilty of illegal dismissal and ordered
the reinstatement of petitioners with
backwages and without loss of
seniority rights. NLRC reversed the
Labor Arbiters decision and dismissed
the complaint. CA affirmed. Hence, this
petitiom.
- Petitioners maintain that they have
sufficiently accounted for the presence
of these items inside their lockers and
that the evidence presented against
them is insufficient to show that they
are guilty of misappropriating company
property. Moreover, assuming ex gratia
argumenti that there was violation of
company rules, the penalty of dismissal
would be too harsh considering their
long years of dedicated service to the
Hospital.
ISSUES
1. WON there was sufficient basis to
hold that petitioners misappropriated
hospital property
2. WON dismissal was the appropriate
penalty
HELD
1. YES
- The Supreme Court is not a trier of
facts, and this rule applies with greater
force in labor cases. Hence, the factual
findings of the NLRC are generally

A2010

- 202 -

Disini

accorded not only respect but even


finality if supported by substantial
evidence and especially when affirmed
by the CA. However, a disharmony
between the factual findings of the
Labor Arbiter and the NLRC opens the
door to a review by this Court.
- Contrary to the position taken by the
Labor Arbiter, the Hospitals dismissal
of petitioners did not rest on
speculative
inferences.
Petitioners
themselves
have
admitted
that
properties belonging to the Hospital
were found inside their lockers. As to
how these items got inside the lockers,
petitioners
acknowledged
having
placed them there against company
rules. In view of these admissions,
there is ample evidence to support a
charge for pilferage unless petitioners
can
satisfactorily
explain
their
possession.
- It was made clear to all hospital staff
that hospital equipment should only be
kept in the supplies locker.
2. NO
- The power to dismiss an employee is
a recognized prerogative that is
inherent in the employers right to
freely manage and regulate his
business. An employer cannot be
expected to retain an employee whose
lack of morals, respect and loyalty to
his employer or regard for his
employers rules and appreciation of
the dignity and responsibility of his
office has so plainly and completely
been bared. An employer may not be
compelled to continue to employ a
person whose continuance in service
will patently be inimical to his interest.
The dismissal of an employee, in a way,
is a measure of self-protection.
Nevertheless,
whatever
acknowledged right the employer has

Labor Law 1
to discipline his employee, it is still
subject to reasonable regulation by the
State in the exercise of its police
power. Thus, it is within the power of
this Court not only to scrutinize the
basis for dismissal but also to
determine
if
the
penalty
is
commensurate
to
the
offense,
notwithstanding the company rules.
- In this case, the Court agrees with the
Labor Arbiter that dismissal would not
be proportionate to the gravity of the
offense considering the circumstances
present in this case. During Perez and
Campos' long tenure (19 and 7 years,
respectively) with the Hospital, it does
not appear that they have been the
subject of disciplinary sanctions and
they
have
kept
their
records
unblemished. Moreover, the Court also
takes into account the fact that
petitioners are not managerial or
confidential
employees
in
whom
greater trust is placed by management
and from whom greater fidelity to duty
is correspondingly expected.
- The reinstatement of petitioners is in
line with the social justice mandate of
the Constitution. Nevertheless, the
Court does not countenance the
wrongful act of pilferage but simply
maintains that the extreme penalty of
dismissal is not justified and a lesser
penalty would suffice. Under the facts
of this case, suspension would be
adequate.
Without
making
any
doctrinal pronouncement on the length
of the suspension in cases similar to
this, the Court holds that considering
petitioners
non-employment
since
January 2000, they may be deemed to
have already served their period of
suspension. Consequently, the Labor
Arbiters order of reinstatement is
upheld, with the deletion of the award

A2010

- 203 -

Disini

of backwages, so as not to put a


premium on acts of dishonesty.
Disposition Petition partially granted.
REQUIREMENTS
SUBSTANTIVE AND PROCEDURAL DUE PROCESS
FUJITSU COMPUTER PRODUCTS OF THE PHILS V CA
(DE GUZMAN, ALVAREZ)
454 SCRA 737
CALLEJO SR; April 8, 2000
NATURE
A petition for review assailing the Decision of the Court of Appeals in reversing the
decision of the National Labor Relations Commission (NLRC).
FACTS
- Petitioner Fujitsu Computer Products Corporation of the Philippines (FCPP) is a
corporation organized and existing under Philippine laws engaged in the manufacture of
hard disc drives, MR heads and other computer storage devices for export.
- Respondent Victor de Guzman began working for FCPP on September 21, 1997 as
Facilities Section Manager. As of 1999, he was also holding in a concurrent capacity the
position of Coordinator ISO 14000 Secretariat. Allan Alvarez, on the other hand, was
employed as a Senior Engineer on April 21, 1998. He was assigned at the Facilities
Department under the supervision of respondent De Guzman.
- The garbage and scrap materials of FCPP were collected and bought by the Saros
Trucking Services and Enterprises (Saros). On January 15, 1999, respondent De Guzman
as Facilities Section Manager, for and in behalf of FCPP, signed a Garbage Collection
Agreement with Saros, and the latters signatory therein was its owner and general
manager, Larry Manaig.
- De Guzman served as middleman between Sta. Rosa Bible Baptist Church and Saro.
The Church was looking for scrap metal, and was willing to buy the purlins at P3. The
scrap metal was then delivered from FCPP to Sta. Rosa Bible Baptist Church.
- Ernesto Espinosa, HRD and General Affairs Director of FCPP, received a disturbing
report from Manaig. Manaig reported that respondent De Guzman had caused the
anomalous disposal of steel [purlins] owned by FCPP. Two of Manaigs employees,
Roberto Pumarez and Ma. Theresa S. Felipe, executed written statements detailing how
respondent De Guzman had ordered the steel purlins to be brought out. Thereafter,
petitioner Espinosa sent a two-page Inter-Office Memorandum dated July 24, 1999 to
respondent De Guzman, effectively placing him under preventive suspension.
- On July 28, 1999, respondent Alvarez sent an e-mail message to his co-employees,
expressing sympathy for the plight of respondent De Guzman. Respondent Alvarez used a
different computer, but the event viewer system installed in the premises of petitioner
FCPP was able to trace the e-mail message to him. Respondent Alvarez submitted a
written Explanation dated September 29, 1999 where he apologized, readily admitted that
he was the sender of the e-mail message in question, and claimed that he acted alone
with his own conviction. He alleged, however, that he was only expressing his sentiments,
and that he was led by his desire to help a friend in distress.
- Respondent Alvarez was informed that his services were terminated on the ground of
serious misconduct effective August 13, 1999. Respondent De Guzmans employment
was, thereafter, terminated effective August 23, 1999 through an Inter-Office
Memorandum.
- The respondents then filed a complaint for illegal dismissal against the petitioners with
prayer for reinstatement, full backwages, damages and attorneys fees before the NLRC.
Labor Arbiter Antonio R. Macam ruled in favor of FCPP, stating that it was justified in
terminating the employment of the respondents. According to the Labor Arbiter,
respondent De Guzman, a managerial employee, was validly dismissed for loss of trust
and confidence. Citing a number of cases,[24] the Labor Arbiter stressed that where an
employee holds position of trust and confidence, the employer is given wider latitude of
discretion in terminating his services for just cause.
- The NLRC sustained the ruling of the Labor Arbiter and dismissed the respondents
appeal for lack of merit. The NLRC also affirmed the Labor Arbiters finding that
respondent De Guzman, a managerial employee who was routinely charged with the
custody and care of the petitioners property, was validly dismissed on the ground of willful
breach of trust and confidence. In so far as the dismissal of respondent Alvarez was
concerned, the Commission held that the circumstances surrounding the sending of the
clearly malicious and premeditated e-mail message constituted no less than serious

Labor Law 1

A2010

- 204 -

Disini

misconduct. Hence, respondent Alvarezs dismissal was also justified under the
circumstances.
- The CA reversed the ruling of the NLRC and held that the
respondents were illegally
dismissed. According to the appellate court, the non-payment of the scrap steel purlins by
the Sta. Rosa Bible Baptist Church (Sta. Rosa) to Saros was not a valid cause for the
dismissal of respondent De Guzman. Contrary to the findings of the Labor Arbiter,
respondent De Guzman did not betray the trust reposed on him by his employer, as the
transaction involving the sale of scrap steel purlins was between Sta. Rosa and Saros.
Anent the dismissal of respondent Alvarez, the CA ruled that his act of sympathizing and
believing in the innocence of respondent De Guzman and expressing his views was not
of such grave character as to be considered serious misconduct which warranted the
penalty of dismissal.

- There is no showing that the sending of such e-mail message had any bearing or relation
on respondent Alvarezs competence and proficiency in his job. To reiterate, in order to
consider it a serious misconduct that would justify dismissal under the law, the act must
have been done in relation to the performance of his duties as would show him to be unfit
to continue working for his employer.
Disposition Petition is denied. Decision of the CA is affirmed, with costs against the
petitioners.

ISSUES
1. WON De Guzman is guilty of breach of confidence, thus warranting dismissal
2. WON Alvarez committed serious misconduct in sending the e-mail

NATURE
Petition for review of the decision of the CA finding the retrenchment of the petitioners to
be valid

HELD
1. NO
- De Guzman is not guilty of breach of confidence.
Ratio To be a valid ground for dismissal, loss of trust and confidence must be based on a
willful breach of trust and founded on clearly established facts. A breach is willful if it is
done intentionally, knowingly and purposely, without justifiable excuse, as distinguished
from an act done carelessly, thoughtlessly, heedlessly or inadvertently. It must rest on
substantial grounds and not on the employers arbitrariness, whims, caprices or suspicion;
otherwise, the employee would eternally remain at the mercy of the employer. In order to
constitute a just cause for dismissal, the act complained of must be work-related and
shows that the employee concerned is unfit to continue working for the employer.
Reasoning
- The term trust and confidence is restricted to managerial employees. In this case, it is
undisputed that respondent De Guzman, as the Facilities Section Manager, occupied a
position of responsibility, a position imbued with trust and confidence.
- The Court had the occasion to reiterate in Nokom v. National Labor Relations
Commission the guidelines for the application of the doctrine of loss of confidence:
Loss of confidence should not be simulated;
> It should not be used as a subterfuge for causes which are improper, illegal or
unjustified;
> It may not be arbitrarily asserted in the face of overwhelming evidence to the contrary;
and
> It must be genuine, not a mere afterthought to justify earlier action taken in bad faith.
- The scrap metals, including the steel purlins, were already classified as scrap materials
and ready for disposal. No less than the written statements of the witnesses for the
petitioners confirm this.
- No fraud or bad faith could be attributed to respondent De Guzman, as evinced by his
readiness to disclose his participation in the transaction between Saros and Sta. Rosa.
- Loss of trust and confidence as a just cause for termination of employment is premised
on the fact that the employee concerned is invested with delicate matters, such as the
handling or care and protection of the property and assets of the employer. After such
scrap materials are weighed, loaded onto a truck and carried out of the company
premises, the petitioner FCPP can no longer be considered the owner thereof, and ceases
to exercise control over such property. In this case however, Saros, as the new owner of
the scrap materials in question, including the steel purlins, was free to contract with
anyone as it wished.
- A condemnation of dishonesty and disloyalty cannot arise from suspicions spawned by
speculative inferences. Because of its subjective nature, this Court has been very
scrutinizing in cases of dismissal based on loss of trust and confidence because the same
can easily be concocted by an abusive employer. Thus, when the breach of trust or loss
of confidence theorized upon is not borne by clearly established facts, as in this case,
such dismissal on the ground of loss of confidence cannot be allowed.
2. NO
- Alvarez did not commit serious misconduct in sending the e-mail.
Ratio Misconduct has been defined as improper or wrong conduct. It is the transgression
of some established and definite rule of action, a forbidden act, a dereliction of duty, willful
in character, and implies wrongful intent and not mere error of judgment. The misconduct
to be serious must be of such grave and aggravated character and not merely trivial and
unimportant.

FACTS
- Petitioners are former supervisors of respondent Philex Mining Corp. Philex sustained
financial losses in its operations and adopted several measures including reducing
personnel through early voluntary retirement and retrenchment programs to save costs.
The labor union representing the rank-and-file employees and the union representing the
supervisory employees signed a MOA with Philex prescribing the criteria for retrenchment.
- Petitioners, with 6 other supervisors and 49 rank-and-file employees, received from
Philex termination notices informing them of their retrenchment. Philex paid them
separation pay, and all of them signed Deeds of Release and Quitclaim in Philexs favor.
Claiming that Philex dismissed them illegally, these supervisors and rank-and-file
employees separately submitted for voluntary arbitration the legality of their separation
from service.
The rank-and-file employees case
- The rank-and-file employees case was referred to Arbitrator Valdez. Valdez ruled in the
employees favor, declared their dismissal illegal, and ordered their reinstatement. He held
that Philex failed to prove its claim of financial losses and that the criteria for retrenchment
in the rank-and-files MOA were arbitrary and inconsistent with the CBA then in force. The
CA reversed Valdezs finding on Philexs financial condition and held that Philex had a
valid reason to undertake retrenchment. Nevertheless, the appellate court affirmed
Valdezs ruling that Philex is liable for illegal dismissal because the criteria for
retrenchment in the rank-and-files MOA were inequitable. Philex further appealed to this
Court, which denied Philexs petition.
The supervisory employees case
- The supervisors case was referred to Arbitrator Advincula, who issued an order to
reinstate petitioners and their co-complainants, after Philex failed to timely file its Position
Paper. On Philexs motion, Advincula admitted Philexs Position Paper and
Supplementary Position Paper. He rendered judgment finding sufficient basis or just
cause for Philex to undertake a retrenchment.
Advincula also held that petitioners were barred from questioning their separation from
service because they availed of the early retirement program and executed the Deeds of
Release and Quitclaim releasing Philex from further liability. Petitioners appealed to the
CA, which denied the petition for lack of merit. The appellate court no longer ruled on the
validity of Philexs retrenchment program because it treated its decision in the rank-andfile employees case as the law of the case on that issue.

Reasoning
- For misconduct or improper behavior to be a just cause for dismissal, (a) it must be
serious; (b) must relate to the performance of the employees duties; and (c) must show
that the employee has become unfit to continue working for the employer.
- The Court finds that respondent Alvarezs act of sending an e-mail message as an
expression of sympathy for the plight of a superior can hardly be characterized as serious
misconduct as to merit the penalty of dismissal.

ARIOLA V PHILEX MINING CORP


446 SCRA 514
CARPIO; August 9, 2005

ISSUES
1. WON petitioners retired or whether Philex dismissed them from service
2. WON petitioners dismissal was illegal
HELD
1. NO
Ratio If the intent to retire is not clearly established or if the retirement is involuntary, it is
to be treated as a discharge.
Reasoning
- Although there is no dispute that petitioners received varied amounts denominated as
retirement gratuity, the records show that Philex paid these amounts because of
petitioners retrenchment. Under Philexs Retirement Gratuity Plan, retirement gratuity is
paid not only to retiring employees but also to those who, like petitioners, are dismissed
for cause beyond their control such as retrenchment. Philex treated the retirement
gratuity as petitioners basic separation pay as indicated in Deeds of Release and
Quitclaims petitioners signed. Significantly, Philex paid petitioners such separation pay
after notifying them of their retrenchment.
Obiter
- In the letter addressed to petitioner Biete, Roxas of Philex Retirement Trust informed
Biete that he was entitled to receive retirement gratuity because his separation, as a
result of the retrenchment program, is for cause beyond his control. Biete submitted
Roxas letter to the CA after that court had rendered its decision. However, at that time,

Labor Law 1
petitioners did not yet file their MFR. Considering the import of the letter, it was error for
the CA not to have considered the letter in resolving petitioners MFR. There can be no
denial of due process where the party claiming to be aggrieved is the one who is guilty of
not disclosing to the court the vital document that contains the most conclusive evidence
regarding the matter in dispute. Philex cannot feign ignorance of this letter.
2. YES
Ratio A substantive defect invalidates a dismissal because the ground for dismissal is
negated by such defect, rendering the dismissal without basis.
Reasoning
- Philexs financial condition justified petitioners retrenchment. What Philex failed to do
was implement its retrenchment program in a just and proper manner. Its failure to use a
reasonable and fair standard in the computation of the supervisors demerits points is not
merely a procedural but a substantive defect which invalidates petitioners dismissal.
When the defect is procedural, the dismissal remains valid because the basis of the
dismissal is not in any way affected by such defect.
Disposition The petition is GRANTED. The decision of the CA is SET ASIDE. We
ENTER another judgment finding petitioners to have been illegally dismissed and ordering
Philex to reinstate petitioners with full backwages, provided that the amounts petitioners
received shall be deducted therefrom. If reinstatement is no longer possible, Philex shall
pay backwages as computed above plus separation pay.

PHILIPPINE NATIONAL BANK V CABANSAG


460 SCRA 514
PANGANIBAN; June 21, 2005
NATURE
Petition for review on certiorari
FACTS
- Florence Cabansag arrived in Singapore as a tourist. She applied for the Singaore
branch of PNB. At that time, PNB had 2 types of employees: 1) employees hired in Manila
and assigned in Singapore 2) locally hired.
- Ruben Tobias, the general manager of the bank, found her qualified and recommended
her to the President of the bank in Manila. The latter approved
- Cabansag then applied for an Employment pass with the Ministry of Manpower of the
Government of Singapore. She was issued said pass.
- On December 7, 1998, she was offered a temporary appointment, as Credit Officer,
wherein she was to be on probation for 3 months. Cabansag accepted the position and
assumed office. In the meantime, the Philippine Embassy in Singapore processed the
employment contract of Florence O. Cabansag and, on March 8, 1999, she was issued by
the Philippine Overseas Employment Administration, an Overseas Employment
Certificate, certifying that she was a bona fide contract worker for Singapore
- On April 15, 1999, she was asked to resign. Tobias said that it was a cost cutting
measure. He likewise said that the PNB branch would be transformed into a remittance
office. Cabansag then asked Tobias that she be furnished with a Formal Advice from the
PNB Head Office in Manila. However, Tobias flatly refused. Cabansag did not submit any
letter of resignation.
- On April 16,1999, Tobias again demanded that she submit a resignation letter. She was
warned that he will be dismissed if she does not. Cabansag asked for more time in order
for her to look for another job. Cabansag said that she should be out by May15, 1999.
- However, on April 19, 1999, Tobias again asked that Cabansag submit her letter of
resignation. Cabansag refused. The next day she was terminated.
- NLRC ruled in favor of Cabansag. CA affirmed.
ISSUES
1. WON the NLRC has jurisdiction over the case at bar
2. WON the arbitration of the NLRC in the National Capital Region is the most convenient
venue or forum to hear and decide the instant controversy
3. WON Cabansag was illegally dismissed
HELD
1. YES
- As enunciated in A217 of the Labor Code, labor arbiters clearly have original and
exclusive jurisdiction over claims arising from employer-employee relations, including
termination disputes involving all workers, among whom are overseas Filipino workers
- When Cabansag obtained an employment pass from the Singapore Ministry of
Manpower, it did not imply a waiver of ones national labor laws. The permit only grants
one a status as a worker in the issuing country. She also applied for an Overseas
Employment Certificate from the POEA through the Philippine Embassy in Singapore. This
entitles her to all benefits and processes under our statutes
- Moreover, petitioner admits that it is a Philippine corporation doing business through a
branch office in Singapore. Significantly, respondents employment by the Singapore
branch office had to be approved by Benjamin P. Palma Gil,[19] the president of the bank

A2010

- 205 -

Disini

whose principal offices were in Manila. This circumstance militates against petitioners
contention that respondent was locally hired; and totally governed by and subject to the
laws, common practices and customs of Singapore, not of the Philippines. Instead, with
more reason does this fact reinforce the presumption that respondent falls under the legal
definition of migrant worker.
2. YES
- The law gives her two choices:
(1) at the Regional Arbitration Branch (RAB) where she resides or
(2) at the RAB where the principal office of her employer is situated
3. YES
- Cabansag was already a regular employee at the time she was terminated, since her 3
months probationary period has already ended.
- The twin requirements of notice and hearing constitute the essential elements of
procedural due process, and neither of these elements can be eliminated without running
afoul of the constitutional guarantee
- In dismissing employees, the employer must furnish them two written notices:
1) one to apprise them of the particular acts or omissions for which their dismissal is
sought; and
2) the other to inform them of the decision to dismiss them. As to the requirement of a
hearing, its essence lies simply in the opportunity to be heard.
- Respondent was not notified of the specific act or omission for which her dismissal was
being sought. Neither was she given any chance to be heard, as required by law. At any
rate, even if she were given the opportunity to be heard, she could not have defended
herself effectively, for she knew no cause to answer to
- All that petitioner tendered to respondent was a notice of her employment termination
effective the very same day, together with the equivalent of a one-month pay. This Court
has already held that nothing in the law gives an employer the option to substitute the
required prior notice and opportunity to be heard with the mere payment of 30 days salary.
- Moreover, Articles 282,[26] 283[27] and 284[28] of the Labor Code provide the valid
grounds or causes for an employees dismissal. The petitioner has not asserted any
grounds as a valid reason for terminating the employment of respondent
Disposition Petition denied

GENUINO ICE CO INC V MAGPANTAY


493 SCRA 195
AUSTRIA-MARTINEZ; June 27, 2006
NATURE
Review on certiorari
FACTS
- Alfonso Magpantay (respondent) was employed as a machine operator with Genuino Ice
Company, Inc. (petitioner). On November 18, 1996, respondent filed against petitioner a
complaint for illegal dismissal with prayer for moral and exemplary damages. In his
Position Paper, respondent alleged that he was dismissed from service effective
immediately by virtue of a memorandum, after which he was not allowed anymore to enter
the company premises. Respondent bewailed that his termination from employment was
done without due process.Petitioner countered that he was not illegally dismissed, since
the dismissal was based on a valid ground, i.e., he led an illegal strike at petitioners sister
company, Genuino Agro Industrial Development Corporation, which lasted from November
18 to 22, 1995, resulting in big operation losses on the latters part. Petitioner also
maintained that respondents dismissal was made after he was accorded due process.
- Petitioner initially claimed that respondents acts were tantamount to serious misconduct
or willful disobedience, gross and habitual neglect of duties, and breach of trust.
Subsequently, petitioner amended its position paper to include insubordination among the
grounds for his dismissal, since it came out during respondents cross-examination, and
the matter was reported only after the new personnel manager assumed his position in
August 1996.
- Labor Arbiter of the National Labor Relations Commission (NLRC) dismissed the case for
lack of merit finding that petitioner had valid cause to dismiss respondent. Labor Arbiters
Decision affirmed. Motion for reconsideration of the NLRC Decision was denied. Special
civil action for certiorari with the CA was filed. Petitioner filed its Comment, contending that
the petition was filed out of time, considering that contrary to respondents claim that the
NLRC Resolution dated August 31, 1999 was received on December 20, 1999, it was
actually received on September 15, 1999, as shown in the registry return card. Petitioner
also reiterated its arguments that respondent was dismissed for cause and with due
process.
- CA rendered the assailed Decision granting the petition and declaring respondents
dismissal as illegal. Petitioner filed a motion for reconsideration which the CA denied.
ISSUES
1. WON the petition was filed by petitioner out of time
2. WON he was illegally dismissed (and on what ground)

Labor Law 1
3. WON there was due process under Section 2 (d), Rule 1, Book VI of the Omnibus
Rules Implementing the Labor Code provides for the standards of due process
HELD
1. NO
- The New Rules of Procedure of the NLRC provides the rule for the service of notices and
resolutions in NLRC cases, to wit:
Sec. 4. Service of notices and resolutions. a) Notices or summons and copies of
orders, resolutions or decisions shall be served on the parties to the case personally by
the bailiff or the duly authorized public officer within three (3) days from receipt thereof
by registered mail; Provided, that where a party is represented by counsel or
authorized representative, service shall be made on such counsel or authorized
representative;
- The presumption is that the decision was delivered to a person in his office, who was
duly authorized to receive papers for him, in the absence of proof to the contrary. It is
likewise a fundamental rule that unless the contrary is proven, official duty is presumed to
have been performed regularly and judicial proceedings regularly conducted, which
includes the presumption of regularity of service of summons and other notices. The
registry return of the registered mail as having been received is prima facie proof of the
facts indicated therein. Thus, it was necessary for respondent to rebut that legal
presumption with competent and proper evidence. Records show that Ducut is not an
employee of the FEU Legal Aid Bureau, but is connected with the Computer Services
Department. The FEU Legal Aid Bureau has its own personnel which include Ms. dela
Paz who is the one authorized to receive communications in behalf of the office. It has
been ruled that a service of a copy of a decision on a person who is neither a clerk nor
one in charge of the attorneys office is invalid. The CA was correct in ruling that the
reckoning period should be the date when respondents counsel actually received the
NLRC Resolution dated August 31, 1999, which was on December 20, 1999. Petitioner,
however, pointed out that a certain Ruby D.G. Sayat received a copy of their Motion for
Reconsideration filed by registered mail on August 16, 2000. Respondent contended that
at the time Sayat received the motion, she was then detailed at the office and was
authorized to receive said pleading, and that it was an isolated and exceptional instance.
On this matter, the FEU Acting Postmaster certified that Sayat is a permanent employee of
the FEU Legal Aid Bureau. As such, she is authorized to receive communications in behalf
of the office and need not possess an express authority to do so. More importantly, the
Court has consistently frowned upon the dismissal of an appeal on purely technical
grounds. While the right to appeal is a statutory, not a natural right, it is, nonetheless, an
essential part of our judicial system. Courts should proceed with caution so as not to
deprive a party of the right to appeal, but rather, ensure amplest opportunity for the proper
and just disposition of a cause, free from the constraints of technicalities.
2. NO, on the ground of habitual neglect of duties but YES on the ground of
insubordination. The Court sustained the CAs finding that respondents four-day absence
does not amount to a habitual neglect of duty; however, the Court found that respondent
was validly dismissed on ground of willful disobedience or insubordination.
- FOR HABITUAL NEGLECT OF DUTY: Neglect of duty, to be a ground for dismissal,
must be both gross and habitual. Gross negligence connotes want of care in the
performance of ones duties. Habitual neglect implies repeated failure to perform ones
duties for a period of time, depending upon the circumstances. On the other hand, fraud
and willful neglect of duties imply bad faith on the part of the employee in failing to perform
his job to the detriment of the employer and the latters business. Thus, the single or
isolated act of negligence does not constitute a just cause for the dismissal of the
employee. Thus, the Court agrees with the CA that respondents four-day absence is not
tantamount to a gross and habitual neglect of duty. As aptly stated by the CA, (W)hile he
may be found by the labor courts to be grossly negligent of his duties, he has never been
proven to be habitually absent in a span of seven (7) years as GICIs employee. The
factual circumstances and evidence do not clearly demonstrate that petitioners
[respondent] absences contributed to the detriment of GICIs operations and caused
irreparable damage to the company.
- FOR INSUBORDINATION OR WILLFUL DISOBEDIENCE: On this point, the CA opined
that petitioner included insubordination as a mere after-thought. It noted that petitioner
seemed to be irresolute in stating the cause of respondents dismissal, as in its Position
Paper, it originally relied on respondents four-day absence or participation in the illegal
strike as a cause for dismissal but later on amended its Position Paper to include
insubordination. Thus, the CA did not make any factual finding or conclusion in its
Decision vis--vis petitioners allegation of respondents insubordination.
While its perception may be true, it should not have deterred the CA from making any
resolution on the matter. For one, respondent was able to argue against petitioners
allegation of insubordination before the Labor Arbiter and the NLRC. For another, it was
respondent himself who raised the subject before the CA, wherein he stated in his
Petition. Further, the proceedings before the Labor Arbiter and the NLRC are non-litigious
in nature. As such, the proceedings before it are not bound by the technical niceties of the
law and procedure and the rules obtaining in courts of law, as dictated by Article 221 of the
Labor Code:
ART. 221. Technical rules not binding and prior resort to amicable settlement. In any
proceeding before the Commission or any of the Labor Arbiters, the rules of evidence

A2010

- 206 -

Disini

prevailing in courts of law or equity shall not be controlling and it is the spirit and
intention of this Code that the Commission and its members and the Labor Arbiters
shall use every and all reasonable means to ascertain the facts in each case speedily
and objectively and without regard to technicalities of law or procedure, all in the
interest of due process. This rule applies equally to both the employee and the
employer. In the interest of due process, the Labor Code directs labor officials to use
all reasonable means to ascertain the facts speedily and objectively, with little regard to
technicalities or formalities. What is essential is that every litigant is given reasonable
opportunity to appear and defend his right, introduce witnesses and relevant evidence
in his favor, which undoubtedly, was done in this case. Willful disobedience, or
insubordination as otherwise branded in this case, as a just cause for dismissal of an
employee, necessitates the concurrence of at least two requisites: (1) the employee's
assailed conduct must have been willful, that is, characterized by a wrongful and
perverse attitude; and (2) the order violated must have been reasonable, lawful, made
known to the employee and must pertain to the duties which he had been engaged to
discharge. Company policies and regulations are generally valid and binding on the
parties and must be complied with until finally revised or amended, unilaterally or
preferably through negotiation, by competent authority. For misconduct or improper
behavior to be a just cause for dismissal, the same must be related to the performance
of the employees duties and must show that he has become unfit to continue working
for the employer. In the case at bench, petitioner informed respondent, through a
Memorandum dated November 14, 1995, that he was being transferred to its GMA,
Cavite operations effective November 20, 1995.
- Due to his refusal to report to the Cavite plant, petitioner reiterated its order transferring
respondent in its Memorandum dated November 24, 1995, where respondent was also
warned that his failure to report to the Cavite plant will be considered as an absence
without leave (AWOL) and insubordination. Respondent was required to comply with the
order within 24 hours from receipt, otherwise, disciplinary action will be imposed on
respondent. Respondent replied with a request that he remain in the Otis plant since a
transfer to the Cavite plant will entail additional expenditure and travel time on his part.
Petitioner again wrote respondent inviting him to appear before the Plant Level
Investigation on December 11, 1995 for the latter to be able to clarify his reasons for
refusing the transfer. Finally, petitioner issued its Memorandum dated December 12, 1995
informing respondent of its decision to terminate his services. The rule is that the transfer
of an employee ordinarily lies within the ambit of the employers prerogatives. The
employer exercises the prerogative to transfer an employee for valid reasons and
according to the requirement of its business, provided the transfer does not result in
demotion in rank or diminution of the employees salary, benefits and other privileges. In
this case, petitioners order for respondent to transfer to the GMA, Cavite Plant is a
reasonable and lawful order was made known to him and pertains to his duties as a
machine operator. There was no demotion involved or diminution of salary, benefits and
other privileges, and in fact, petitioner was even willing to provide respondent with
monetary allowance to defray whatever additional expenses he may incur with the
transfer. Such being the case, respondent cannot adamantly refuse to abide by the order
of transfer without exposing himself to the risk of being dismissed. Hence, his dismissal
was for just cause in accordance with Article 282 (a) of the Labor Code. Consequently,
respondent is not entitled to reinstatement or separation pay and backwages.
3. YES
- Simply stated, the employer must furnish the employee a written notice containing a
statement of the cause for termination and to afford said employee ample opportunity to
be heard and defend himself with the assistance of his representative, if he so desires,
and the employee must be notified in writing of the decision dismissing him, stating clearly
the reasons therefor.
- The CA found that petitioner failed to observe the twin requirements of notice and
hearing, stating that its Memorandum dated December 13, 1995 does not squarely meet
the standards of due process. The circumstances surrounding respondents dismissal,
however, prove the contrary. The CA failed to take into account that prior to the
Memorandum dated December 13, 1995, petitioner sent respondent several memoranda
apprising him of the possible implications of his refusal to comply with the order of
transfer. Thus, in its Memorandum dated November 24, 1995, petitioner notified
respondent that his continued non-compliance with the order of transfer might bring about
disciplinary action. Respondent replied to this memorandum, stating the reasons for his
refusal, i.e., additional expenses, longer travel time, and union concerns. Petitioner sent
another Memorandum on December 9, 1995, asking respondent to appear on December
11, 1995, for further clarification of his reasons for refusing the transfer. Despite the
meeting, and since respondent, apparently, stubbornly refused to heed petitioners order, it
was then that the Memorandum dated December 13, 1995 was issued to respondent
informing him of the managements decision to terminate his services. Clearly,
respondents right to due process was not violated.
Disposition petition is GRANTED. The CA Decision dated August 3, 2000 and
Resolution dated March 16, 2001 are SET ASIDE, and the NLRC Decision dated June 30,
1999 is REINSTATED.

Labor Law 1
14.05 JUST CAUSES SUBSTANTIVE DUE
PROCESS GROUNDS FOR TERMINATION
A. SERIOUS MISCONDUCT
DEFINITION AND ACTS
VALIAO V CA
[PAGE 11]
VILLAMOR GOLF CLUB V PEHID
472 SCRA 36
CALLEJO; October 4, 2005
NATURE
Petition for review on certiorari of CA decision
FACTS
- Rodolfo Pehid was employed by the Villamor Golf Club (VGC) as an attendant in the
mens locker room, and, thereafter, he became the Supervisor-in-Charge. His
subordinates included Superal, Parilla, Mendoza, Velasquez, Casabon, Buenaventura and
Modelo. Pehid and these employees agreed to establish a common fund from the tips they
received from the customers, guests and members of the club for their mutual needs and
benefits. Each member was to contribute the amount of P100 daily. The contributions of
the employees had reached the aggregate amount of P17,990 based on the logbook
maintained in the locker room. This agreement was not known to the VGC management.
- An audit of the Locker Room Section of the golf club was conducted stating, among
others, that based on the information relayed, there was an undeclared and unrecorded
aggregate amount of P17,990 for the fund from May 98 to October 98. Further, not one
in the said section admitted custody of such amount and there was no record that the
money had been distributed among those employed in the locker room. In said report,
Capuyan recommended that an investigation be conducted to determine the whereabouts
of said amount and who was accountable therefor.
- After the requisite formal investigation by the Administrative Board of Inquiry, Pehid
received order that his employment was terminated. Based on its findings, Pehid
committed gross misconduct in the performance of his duties in violation of Paragraph
IV-E(d) of the VGC Rules and Regulations. He was also informed that he committed acts
of dishonesty which caused and tend to cause prejudice to the club for misappropriating
the common fund of P17,990.00 for his personal benefit.
- Pehid filed a complaint for illegal dismissal, unfair labor practice, separation
pay/retirement benefits, damages and attorneys fees against petitioners VGC. LA ruled in
favor of Pehid saying that his dismissal was illegal. NLRC set aside and reversed the
decision of LA.
- CA set aside and reversed NLRC decision. The CA declared that Paragraph IV-E(a) and
(d) of the VGC Rules40 expressly provide that the funds referred to therein are funds of the
club and that the P17,990 did not form part of such fund but belonged to the locker room
personnel. The CA also declared that the management of the VGC had no personal
knowledge about the funds and, in fact, had not sanctioned its existence. Moreover, VGC
was not prejudiced by the loss of the fund. Hence, this petition by VGC.
Petitioners contentions:
> That when confronted with the letter-complaint against him, Pehid admitted that his
accountability arose from the proceeds of the sale of the golf club and golf shares
entrusted to him, which he used for his personal needs without the knowledge of the
persons concerned;
> That there is substantial evidence that Pehid was the custodian of fund belonging to the
members of the locker room and that his misappropriation of the same constituted gross
misconduct;
> That it is an act of manifest dishonesty within the context of Paragraph IV-E(d) of the
Rules of Conduct of the club, in relation to A282(e) of the Labor Code, tending to prejudice
the VGC
> That, based on the substantial evidence Pehid misappropriated the fund as his coemployees in the locker room even positively identified him as the custodian thereof; and

40
1.
a)
d)

E. Dishonesty
The following shall constitute violation of this section.
Misappropriation or malversation of Club funds.
All other acts of dishonesty which cause or tend to cause prejudice to VGC

A2010

- 207 -

Disini

> that Pehids failure to account for and distribute the common fund which the locker
personnel had established for their mutual aid and benefit is a manifest dishonesty falling
within the scope of the proviso
Respondents arguments:
> That he was dismissed without just cause and due process of law;
> that there was no basis or evidence to show that he had custody of the common fund
which was used for his own benefit;
> that he incurred the ire of his superiors for testifying in support of Tansiongco, a former
Director of Personnel who was dismissed by VGC; and
> that one of Tansiongcos accusers was the brother of Velasquez, one of the locker boys
who complained against him.
ISSUES
1. WON CA decision is contrary to law and jurisprudence and therefore reversible
2. WON the incident of the case shall fall within the provision of Article 282 paragraph (e)
of the Labor Code
HELD
1. NO
- Company policies and regulations are, unless shown to be grossly oppressive or
contrary to law, generally valid and binding and must be complied with by the parties
unless finally revised or amended, unilaterally or preferably through negotiation. However,
while an employee may be validly dismissed for violation of a reasonable rule or regulation
adopted for the conduct of the companys business, an act allegedly in breach thereof
must clearly and convincingly fall within the express intendment of such order.
- The CA was correct in ruling that the NLRC had overlooked and misapplied certain facts
and circumstances of substance, which, if properly appreciated, would affect the
disposition of the case.
- Theres no doubt that funds alleged to have been embezzled by the petitioner, belonged
to the personnel of respondent VGC and not to respondent VGC. Under the afore-quoted
VGC rule (see footnote), the dishonesty of an employee to be a valid cause for dismissal
must relate to or involve the misappropriation or malversation of the club funds, or cause
or tend to cause prejudice to VGC. The substantial evidence on record indicates that the
P17,990, which was accumulated from a portion of the tips given by the golfers from May
1998 to October 1998 and was allegedly misappropriated by the respondent as the
purported custodian thereof, did not belong to VGC but to the forced savings of its locker
room personnel. Hence, VGC was not prejudiced. So it is within law and jurisprudence
that CA reversed NLRC ruling.
2. NO
Ratio The principle in statutory construction of ejusdem generis: Where general words
follow an enumeration of persons or things, by words of a particular and specific meaning,
such general words are not to be construed in their widest extent, but are to be held as
applying only to persons or things of the same kind or class as those specifically
mentioned.
Reasoning
- Based on the grounds of termination provided under A282 of the Labor Code and the
VGC Rules and Regulations, the common denominator thereof to constitute gross
misconduct as a ground for a valid termination of the employee, is that it is committed in
connection with the latters work or employment. In the instant case, as previously pointed
out, the alleged petitioners misappropriation or malversation was committed, assuming it
to be true, against the common funds of the Locker Room personnel, which did not belong
nor sanctioned by respondent VGC. A fortiori, respondent VGC was not prejudiced or
damaged by the loss or misappropriation thereof.
Obiter
- Important for our purposes in the outline: Serious misconduct as a valid cause for
the dismissal of an employee is defined as improper or wrong conduct; the transgression
of some established and definite rule of action, a forbidden act, a dereliction of duty, willful
in character, and implies wrongful intent and not mere error in judgment. To be serious
within the meaning and intendment of the law, the misconduct must be of such grave and
aggravated character and not merely trivial or unimportant. However serious such
misconduct, it must be in connection with the employees work to constitute just cause for
his separation. The act complained of must be related to the performance of the
employees duties such as would show him to be unfit to continue working for the
employer.
Disposition Petition is DENIED for lack of merit. CA decision AFFIRMED.

LAKPUE V BELGA
473 SCRA 617
YNARES-SANTIAGO; October 20, 2005
FACTS

Labor Law 1
- Petitioner Tropical Biological Phils., Inc. (Tropical), a subsidiary of Lakpue Group of
Companies, hired on March 1, 1995 respondent Ma. Lourdes Belga (Belga) as
bookkeeper and subsequently promoted as assistant cashier. On March 19, 2001, Belga
brought her daughter to the Philippine General Hospital (PGH) for treatment of bronchopneumonia. On her way to the hospital, Belga dropped by the house of Marylinda O.
Vegafria, Technical Manager of Tropical, to hand over the documents she worked on over
the weekend and to give notice of her emergency leave.
- While at the PGH, Belga who was pregnant experienced labor pains and gave birth on
the same day. On March 22, 2001, or two days after giving birth, Tropical summoned
Belga to report for work but the latter replied that she could not comply because of her
situation. On March 30, 2001, Tropical sent Belga another memorandum ordering her to
report for work and also informing her of the clarificatory conference scheduled on April 2,
2001. Belga requested that the conference be moved to April 4, 2001 as her newborn was
scheduled for check-up on April 2, 2001. When Belga attended the clarificatory
conference on April 4, 2001, she was informed of her dismissal effective that day.
ISSUE
WON Belga was illegally dismissed
HELD
YES
- Tropical terminated Belga on the following grounds: (1) Absence without official leave for
16 days; (2) Dishonesty, for deliberately concealing her pregnancy; (3) Insubordination, for
her deliberate refusal to heed and comply with the memoranda sent by the Personnel
Department on March 21 and 30, 2001 respectively
- Tropical cites the following paragraphs of Article 282 of the Labor Code as legal basis for
terminating Belga:
Article 282. Termination by employer. An employer may terminate an employment
for any of the following causes:
(a) Serious misconduct or willful disobedience by the employee of the lawful orders of
his employer or representative in connection with his work;
(c)
Fraud or willful breach by the employee of the trust reposed in him by his
employer or duly authorized representative
- We have defined misconduct as a transgression of some established and definite rule of
action, a forbidden act, a dereliction of duty, willful in character, and implies wrongful intent
and not mere error in judgment. Such misconduct, however serious, must, nevertheless,
be in connection with the employees work to constitute just cause for his separation
- Her absence for 16 days was justified considering that she had just delivered a child,
which can hardly be considered a dereliction of duty or wrongful intent on the part of
Belga.
-Tropical harps on the alleged concealment by Belga of her pregnancy. This argument,
however, begs the question as to how one can conceal a full-term pregnancy. We agree
with respondents position that it can hardly escape notice how she grows bigger each
day. While there may be instances where the pregnancy may be inconspicuous, it has not
been sufficiently proven by Tropical that Belgas case is such
- The charge of disobedience for Belgas failure to comply with the memoranda must
likewise fail. Disobedience, as a just cause for termination, must be willful or intentional.
In the instant case, the memoranda were given to Belga two days after she had given
birth. It was thus physically impossible for Belga to report for work and explain her
absence, as ordered
- Tropical avers that Belgas job as Treasury Assistant is a position of responsibility since
she handles vital transactions for the company. It adds that the nature of Belgas work
and the character of her duties involved utmost trust and confidence.
- In order to constitute a just cause for dismissal, the act complained of must be workrelated such as would show the employee concerned to be unfit to continue working for
the employer. More importantly, the loss of trust and confidence must be based on the
willful breach of the trust reposed in the employee by his employer. A breach of trust is
willful if it is done intentionally, knowingly and purposely, without justifiable excuse, as
distinguished from an act done carelessly, thoughtlessly, heedlessly or inadvertently
- Belga was an assistant cashier whose primary function was to assist the cashier in such
duties as preparation of deposit slips, provisional receipts, post-dated checks, etc. As
correctly observed by the Court of Appeals, these functions are essentially clerical.

COCA-COLA BOTTLERS PHIL INC V KAPISANAN NG


MALAYANG MANGGAGAWA SA COCA-COLA
452 SCRA 480
CALLEJO; February 28, 2005

A2010

- 208 -

Disini

NATURE
This is a petition for review of the Resolution 1 the Court of Appeals reversing the
Resolution of the National Labor Relations Commission
FACTS
- Petitioner Coca-Cola Bottlers Phil., Inc. is a domestic corporation engaged in the
manufacture, sale and distribution of softdrinks.
- On July 1, 1982, the petitioner hired Florentino Ramirez as "driver-helper" with the
following duties: (a) as driver, he checks the trucks oil, water, wheels, etc.; (b) as helper,
he is charged of loading and unloading trucks load; putting bottles in the coolers and
displays company products to each outlet or customers store.2
- Ramirez became a member of the respondent Kapisanan ng Malayang Manggagawa
Sales Force Union, the bargaining representative of the rank- and-file employees of the
petitioner company. In 1996, he was the "shop steward" of the union at the companys
Batangas Sales Office.
- Sometime in October 1996, it happened that the route salesman for Route M11 was
unavailable to make his usual routes. Since Ramirez had been driving for the route
salesman for so long, the petitioner company decided to assign him as temporary
replacement of the regular route salesman for routes M11, AMC and LPR.
- Thereafter, in a Letter dated December 5, 1996, the Officer-in-Charge of the Batangas
Sales Office, Victor C. dela Cruz, informed the Officer-in-Charge of DSS-District 44,
Rolando Manzanares, that a review of the copies of the invoices relating to the
transactions of Ramirez in Rt. M11 revealed the following discrepancies: (a) the number of
cases delivered to customers; (b) empty bottles retrieved from them, and (c) the amounts
in Sales Invoices Nos. 3212215, 3288587, 3288763, 3288765 and 3288764
- Ramirez received a Memorandum from District Office Nos. 44 and 45 requiring him to
report to the said office starting December 5, 1996 until such time that he would be notified
of the formal investigation of the charges against him.
- During the formal investigation conducted by a panel of investigators on December 20,
1996, Ramirez was not represented by counsel. He also manifested that he was waiving
his right to be represented by counsel when the members of the panel asked him about it.
- Ramirez was then asked to explain the discrepancies subject of the charges
- On February 11, 1997, Ramirez received a notice from the company informing him that
his services were being terminated; his employment was terminated effective February 12,
1997.
- On March 17, 1997, Ramirez and the union filed a Complaint for unfair labor practice
and illegal dismissal against the company with the Arbitration Branch of the NLRC.
- Ramirez likewise claimed that he was denied of his right to due process, based on the
following grounds: Firstly, individual complainant was dismissed without having been first
issued a "notice of dismissal" which supposedly should contain the charges against him,
which would be made as basis for his termination. Secondly, individual complainant was
dismissed without affording him an ample opportunity to defend himself, as he was not
notified in advance of the subject of the administrative investigation. Thirdly, individual
complainant was terminated without just and valid cause, and in gross violation of his right
to due process. Lastly, individual complainant was terminated by respondents in utter bad
faith, as the decision on the said termination was arrived at, without any just and valid
cause. Simply put, respondents simply acted oppressively, malevolently, and with grave
abuse of prerogatives.7
- Petitioner company alleged that the dismissal of Ramirez was based on the facts
unearthed during the formal investigation, and that he was guilty of serious misconduct, a
valid ground for termination of employment. Even if he was occupying the position of route
driver/helper, he was nevertheless performing the functions and duties of a route
salesman, and, as such, he not only committed fraud, but also willfully breached the trust
and confidence reposed on him by the petitioner company.
- According to the petitioner company, considering the sanctions imposed on Ramirez for
prior breaches of company rules, his dismissal from employment was with basis. The
petitioner company also insisted that Ramirez was accorded his right to due process: he
was notified of the charges against him, was subjected to a formal investigation during
which he was allowed to explain the discrepancies, and was notified of the outcome
thereof, as well as the bases of the termination of his employment.
- On July 31, 1998, the Labor Arbiter rendered judgment dismissing the complaint for lack
of merit. The LA found that based on the evidence, there was a justifiable basis for the
dismissal of Ramirez. According to the LA, it was of no moment that the official
designation of Ramirez was "driver-helper," since he committed the infractions while he
was performing the functions of an "acting salesman." The LA further found that due
process had been complied with.9
- Ramirez appealed the decision to the NLRC
- On September 20, 1999, the NLRC rendered a Resolution affirming the decision of the
LA.
- Upon the denial of his motion for reconsideration, Ramirez filed a petition for certiorari
under Rule 65 of the Rules of Court with the Court of Appeals
- In a Decision dated October 25, 2000, the CA dismissed the petition. It ruled that the
petitioners designation at the time of the infraction was of no moment; when he agreed to
be an "acting salesman" for Route M11, AMC and LPR, he actually performed the duties

Labor Law 1
of a salesman, and in so doing, assumed the responsibilities of the position. The CA
further ratiocinated that notwithstanding Ramirezs lack of training, he had assumed and
performed the duties of a salesman; hence, he was obligated to do so with due care,
dedication, and with due regard to the exercise of the degree of diligence to prevent the
commission of any serious error, mistake or blunder on his part.
- The petitioner filed a motion for the reconsideration of the decision
- This time, the CA found merit in petitioners cause
ISSUE
WON respondent Florentino Ramirez was dismissed by the petitioner without just or valid
cause
HELD
- with just cause, but too severe penalty
- The respondent, by his acts and omissions, committed irregularities in the performance
of his duties. However the penalty imposed on respondent by the petitioner company was
too severe. In order to effect a valid dismissal of an employee, the law requires that there
be just and valid cause as provided in Article 282 and that the employee was afforded an
opportunity to be heard and to defend himself. Pursuant to Article 282 of the Labor Code,
an employees services can be terminated for the following just causes:
(a) Serious misconduct or willful disobedience by the employee of the lawful orders of
his employer or representative in connection with his work;
(b) Gross and habitual neglect by the employee of his duties;
(c) Fraud or willful breach by the employee of the trust reposed in him by his employer
or duly-authorized representative.
(d) Commission of a crime or offense by the employee against the person of his
employer or any immediate member of his family or his duty-authorized representative;
and
(e) Other causes analogous to the foregoing.
- In termination disputes, the burden of proof is always on the employer to prove that the
dismissal was for a just and valid cause. Considering the nature of the charges and the
penalties therefore, the petitioner is bound to adduce clear and convincing evidence to
prove the same.
- It is recognized that company policies and regulations, unless shown to be grossly
oppressive or contrary to law, are generally valid and binding on the parties and must be
complied with until finally revised or amended, unilaterally or preferably through
negotiation, by competent authority. The Court has upheld a companys management
prerogatives so long as they are exercised in good faith for the advancement of the
employers interest and not for the purpose of defeating or circumventing the rights of the
employees under special laws or under valid agreements. For misconduct or improper
behavior to be a just cause for dismissal, the same must be related to the performance of
the employees duties and must show that he has become unfit to continue working for the
employer.
- In cases when an employer may dismiss an employee on the ground of willful
disobedience, there must be concurrence of at least two requisites: (1) the employees
assailed conduct must have been willful or intentional, the willfulness being characterized
by a wrongful and perverse attitude; and (2) the order violated must have been
reasonable, lawful, made known to the employee and must pertain to the duties which he
had been engaged to discharge.
- That the individual petitioner has not been specifically trained as salesman is
undisputed. In acting as a salesman, he was tasked with a duty involving trust and
specialized skills for which he was never trained. His alleged failure to comply strictly with
all the procedures, of which he was unfamiliar, was to be expected. Yet Ramirez was
penalized as a full-fledge salesman, not as a driver-helper who was forced to perform the
functions of acting salesman or perhaps risk being charged with insubordination. Then it
was not just any penalty meted out to him, as if there is only one punishment possible for
him: the supreme sanction of dismissal.
- Perhaps, individual petitioner should first have been given a mere warning, then a
reprimand or even a suspension, but certainly not outright dismissal from employment.
One must keep in mind that a workers employment is property in the constitutional sense,
and he cannot be deprived thereof without due process and unless it was commensurate
to his acts and degree of moral depravity.
- In order to validly dismiss an employee on the ground of loss of trust and confidence
under Article 282 of the Labor Code of the Philippines, the following guidelines must be
followed:
1. The loss of confidence must not be simulated;
2. It should not be used as a subterfuge for causes which are illegal, improper or
unjustified;
3. It may not be arbitrarily asserted in the face of overwhelming evidence to the
contrary;
4. It must be genuine, not a mere afterthought, to justify earlier action taken in bad
faith; and
5. The employee involved holds a position of trust and confidence.
- Considering the factual backdrop in this case, we find and so rule that for his infractions,
the respondent should be meted a suspension of two (2) months.

A2010

- 209 -

Disini

Disposition PARTIALLY GRANTED

GENUINO ICE CO INC V MAGPANTAY


[PAGE 206]
PREMIERE DEVT BANK V MANTAL

485 SCRA 234


YNARES-SANTIAGO; March 23, 2006
NATURE

Petition for review on certiorari seeking to annul and


set aside the Decision of the Court of Appeals in CAG.R. SP No. 80975 dated January 17, 2005 and its
Resolution dated April 7, 2005 holding the petitioner
Premiere Development Bank liable for illegal
suspension and illegal dismissal, ordering it to
reinstate respondent Elsie Escudero Mantal to her
former position and to pay her full backwages from
date of suspension and dismissal until actual
reinstatement, half month salary and half month 13th
month pay, as well as attorneys fees.
FACTS
- Respondent is a regular employee of petitioners Cubao branch, serving as accounting
clerk since July 17, 1996. On November 24, 2000, the branch manager, Rosario Detalla,
instructed respondent: "Elsie, baka may mag-confirm sa Bank Guarantee ng GIA Fuel,
sabihin mo OKAY NA, may kulang pa lang dokumento."
- Later that day, Emmie Crisostomo of Filpride Energy Corporation inquired whether GIA
Fuel and Lubricant Dealer has a credit line or maintains an account with petitioner Bank
which respondent confirmed after checking the files on the computer. Crisostomo also
inquired if the bank guarantee signed by Detalla is in order, and likewise respondent
replied in the affirmative. However, upon verification from petitioners head office,
Crisostomo was informed that the bank guarantee was spurious.
- On the same day, respondent was summoned to the head office and was required to
write down what she knew about the subject bank guarantee. Respondent also received a
memorandum placing her under preventive suspension effective immediately for a period
of 30 days. During the investigation, Detalla admitted issuing the falsified bank guarantee.
- On December 21, 2000, Detalla tendered her irrevocable letter of resignation.
Respondent was asked to execute a resignation letter on December 22, 2000, but she
declined. The following day, respondent received a Notice of Termination dated December
22, 2000.
- Respondent filed a complaint for illegal suspension, illegal dismissal, unpaid salary and
13th month pay, moral and exemplary damages. The Labor Arbiter rendered a decision
holding petitioner liable for illegal suspension and illegal dismissal and ordering the
reinstatement of respondent to her former position, with full backwages, half month salary
and half month 13th month pay, and attorneys fees. NLRC reversed the labor arbiters
decision, and dismissed the complaint for lack of merit. The motion for reconsideration

Labor Law 1
having been denied, respondent appealed to the Court of Appeals which affirmed the
Labor Arbiter.
ISSUE
WON respondent was validly suspended and dismissed from her position as accounting
clerk

HELD
NO
Ratio Misconduct is improper or wrongful conduct. It is the transgression of some
established and definite rule of action, a forbidden act, a dereliction of duty, willful in
character, and implies wrongful intent and not mere error in judgment. Under Article 282 of
the Labor Code, the misconduct, to be a just cause for termination, must be of such grave
and aggravated character, not merely of a trivial or unimportant nature. For serious
misconduct to warrant the dismissal of an employee, it (1) must be serious; (2) must relate
to the performance of the employees duty; and (3) must show that the employee has
become unfit to continue working for the employer.
Reasoning
- Respondent did what was expected of her as an employee of the bank. Before
answering the telephone inquiry, respondent verified the existence of the GIA Fuel and
Lubricant Dealer account through the bank computer. If ever she was negligent, it would
only constitute a single or isolated act which is not a just cause for the dismissal of the
respondent from her employment.
In addition, although respondents position as accounting clerk involves a high degree of
responsibility requiring trust and confidence, carrying with it the duty to observe proper
company procedures in the fulfillment of her job as it relates closely to the financial
interests of the company, the charge against her is not reasonably connected to her job of
opening of savings, current and/or time deposits and the payment of withdrawals. The
duty and ultimately, the responsibility of approving transactions relating to bank
guarantees lie with the branch manager and the management personnel of the petitioners
head office. Thus, in Metropolitan Bank and Trust Company v. Barrientos, the Court held
that respondent therein was not liable of misconduct for allowing the opening of fictitious
accounts, because he was merely a cashier and had no authority to approve new
accounts and had no way of knowing the anomalous transactions.
Disposition petition is DENIED. The Decision of the Court of Appeals in CA-G.R. SP No.
80975 dated January 17, 2005 finding petitioner guilty of illegal dismissal and ordering the
reinstatement of respondent to her former position, with full backwages, inclusive of
allowances and to the other benefits or their monetary equivalent from the time her
compensation was withheld up to her actual reinstatement, plus attorneys fees, and the
Resolution dated April 7, 2005 denying the motion for reconsideration, are AFFIRMED.

MOLINA V PACIFIC PLANS INC


484 SCRA 498
CALLEJO; March 10, 2006
NATURE
Petitions for Review on Certiorari assailing the decision and resolution of the CA reversing
the decision of the NLRC.
FACTS
- The accident occurred on July 9, 1912.
- Because of injuries, plaintiff spent 10 days in the hospital. The first 4-5 days he couldnt
leave his bed. After being discharged, he received medical attention from a private
practitioner for several days.
- Plaintiff testified that he had down no work since the accident, that his earning capacity
was P50/month
- He described himself as being well at the end of July; the trial took place September 19
- Plaintiff sold distillery products and had about 20 regular customers who purchased in
small quantities, necessitating regular, frequent deliveries
- It took him about 4 years to build up the business he had at the time of the accident, and
since the accident, he only kept 4 of his regular customers.
- The lower court refused to allow him any compensation for injury to his business due to
his enforced absence therefrom.
ISSUE
How to determine the amount of damages to award plaintiff
HELD

A2010

- 210 -

Disini

- The judgment of the lower court is set aside, and the plaintiff is awarded the following
damages; ten pesos for medical expenses; one hundred pesos for the two months of his
enforced absence from his business; and two hundred and fifty pesos for the damage
done to his business in the way of loss of profits, or a total of three hundred and sixty
pesos. No costs will be allowed in this instance.
Reasoning
- Actions for damages such as the case at bar are based upon article 1902 of the Civil
Code: "A person who, by act or omission, causes damage to another where there is fault
or negligence shall be obliged to repair the damage so done." Of this article, the supreme
court of Spain, in considering the indemnity imposed by it, said: "It is undisputed that said
reparation, to be efficacious and substantial, must rationally include the generic idea of
complete indemnity, such as is defined and explained in article 1106 of the said (Civil)
Code."
- Art 1106. Indemnity for losses and damages includes not only the amount of the loss
which may have been suffered, but also that of the profit which the creditor may have
failed to realize, reserving the provisions contained in the following articles.
- Art 1107. The losses and damages for which a debtor in good faith is liable, are those
foreseen or which may have been case is will gradually increase. The injury to plaintiff's
business begins where these profits leave off, and, as a corollary, there is where
defendant's liability begins. Upon this basis, we fix the damages to plaintiff's business at
P250.
- Before us is a Petition for Review on Certiorari assailing the Decision and Resolution of
the Court of Appeals (CA) in CA-G.R. SP No. 81298 reversing the Decision of the National
Labor Relations Commission (NLRC) in NLRC-NCR (South) Case No. 30-07-03393-01.
Pacific Plans, Inc. (PPI) is a domestic corporation engaged in the business of selling preneed plans, such as educational, pension, and memorial plans. It maintains regional
offices throughout the Philippines. At the time material to this case, Metro Manila regional
offices were divided into two sales divisions - the South Sales Division and the North
Sales Division. Metro Manila VI was part of the North Sales Division. Among the corporate
officers of PPI were Geoffrey Martinez, Executive Vice-President for Finance; Luciano
Abia, Senior Assistant Vice-President, Metro Manila Marketing Division; and Atty. Manuel
Reyes, the Head of the Legal Department. Roy Padiernos then occupied the position of
Regional Manager of Metro Manila VI.
- PPI solicited subscribers and buyers of its pre-need plans through clusters of sales
associates. One of them was Ruth Padiernos, wife of Roy Padiernos.
Sometime in October 1994, PPI hired Agripino Molina as Regional Manager of Metro
Manila VI, replacing Roy Padiernos who was promoted as First Vice-President for
Marketing Operations. As Regional Manager, Molina performed both administrative and
marketing functions, whose duties and responsibilities included the following:
a. formulating and recommending short and long range marketing plans for the Region
and executing approved plans;
b. generating new and conserving existing pre-need plan businesses;
c. motivating, training, and developing a dedicated and effective counselor force;
d. conducting researches to determine sales potentials and share of the market,
pricing, and profitability of Company's products, competition and the directing of
product development for the Region;
e. hiring and terminating counselors, unit managers or group managers in accordance
with policies previously laid out;
f. recommending the creation of additional positions or termination of services of any
employee within the Region;
g. recommending promotions or changes in salaries of personnel within the Region and
lateral shifts of supervisor, their assistants, understudies of positions of equal rank;
h. training and developing understudies for each position within the Region to provide
immediate replacement whenever vacated;
i. changing methods and procedures not affecting the other Regions, provided,
however, that radical changes should first be cleared with [the] superior;
j. controlling the operations of the Region and establishing a system of periodic work
reporting;
k. coordinating the Regions activities with those of the other Regions;
l. keeping [the] superior informed of [the] Region's activities and specially of [the]
decision on matters for which he may be held responsible;
m. realizing the Companys objective for service, growth, and profit;
n. establishing and maintaining harmonious and dignified relationship with plan holders,
counselors, employees, the public, government instrumentalities, other pre-need plan
companies; [and]
o. further enhancing the prestige of the Company and maintaining its position of
leadership in its field.
- Since Metro Manila VI was consistently on top in terms of nationwide sales and
productivity, Molina was promoted Assistant Vice-President with the same functions as
those of a regional manager of the same sales region.
- Caritas Health Shield, Inc. (Caritas for brevity), a health maintenance organization
(HMO) engaged in selling health and hospitalization plans, was established on December
16, 1998. Geoffrey Martinez resigned as Executive Vice-President of PPI and became the
President and Chief Executive Officer of Caritas. Among the incorporators and members
of the Board of Directors were Luciano Abia and Atty. Manuel Reyes. Molina was hired as

Labor Law 1
Assistant Vice-President and Marketing Head of Area 10. His wife, Fe Molina, was the
head of a sales agency of Caritas.
- In the meantime, from February 2000, there was a considerable decrease in the sales
output production of PPIs Metro Manila Region VI.
- On March 21, 2000, Molina received a Memorandum from PPI, through its Senior
Assistant Vice-President for Human Relations, Patricio A. Picazo, informing him that,
based on written reports, he committed the following: 1) recruiting and pirating activities in
favor of Caritas, in particular, initiating talks and enticing associates to join Caritas, and a
number of associates have already signed up; 2) he called for a meeting with his
associates sometime in November 1999, and solicited contributions from them for the bill
but later asked for reimbursement from the company; and 3) acts of misdemeanor on
several occasions, such as coming to the office under the influence of liquor, initiating a
smear campaign against PPI, and other acts inimical to the companys interest. Molina
was also required to submit, on March 23, 2000, a written explanation why he should not
be held administratively liable for said acts which, it opined, might constitute conduct
unbecoming of an officer, conflict of interest, and breach of trust and confidence. Molina
was also informed that he was preventively suspended pending formal investigation
effective immediately until April 24, 2000.
- In a letter addressed to Picazo dated March 22, 2000, Molina categorically denied the
acts attributed to him. He, however, requested that he be furnished with copies of the
alleged written reports to enable him to prepare the required written explanation. However,
instead of acceding to the request of copies of the written reports, Picazo wrote a letter
dated April 3, 2000, citing the particulars of the charges against Molina, thus:
I. Conflict of Interest
1. Recruiting and pirating activities in favor of Caritas Health Shield, Inc.
* You have acted as conduit for Caritas in recruiting/pirating Mr. Restie Acosta on
March 04, 2000 and Ms. Eppie Acosta on March 06, 2000.
*Your failure to stop and/or tolerating your wife's activities in recruiting for Caritas
Ms. Lennie Gatmaitan who belongs to Ms. Celeste Villena, a PPI GA.
II. Misappropriation of Funds
1. Solicitation of associates' personal funds in the amount of P200.00 per person, to
which 12 persons contributed for a total P2,400.00, for payment of official function
during the meeting held at Barrio Fiesta last November 27, 1999. Amount solicited
was subsequently reimbursed from the company but not returned to the associates
concerned.
III. Dereliction of Duties
1. You failed to prevent associates from leaving the company in favor of
competitors, thus causing demoralization among your sales associates.
2. You even encouraged associates to transfer to Caritas.
IV. Conduct unbecoming of a Company Officer
1. Often reporting to office under the influence of liquor.
2. Sowing intrigue in the case of Vilma del Rosario which almost caused her early
retirement from the company and transfer to Caritas.
3. Sowing intrigues between Mr. Roy Padiernos and Mr. Abia.
4. Showing disrespect to immediate superior, Mr. Roy Padiernos, by shouting at him
and walking out in one of the meetings called by him after the retirement of Atty.
Haceta.
- During the investigation the following day, April 4, 2000, Molina reiterated his request to
be provided with a copy of the written reports. Picazo denied the request in a
Memorandum dated April 6, 2000, and reiterated his order for Molina to submit his written
explanation on April 11, 2000, and to address his concerns during the investigation
scheduled on April 14, 2000. Molina failed to submit any written explanation. On April 24,
2000, PPI issued a Memorandum advising Molina that he would be reinstated in the
payroll effective April 25, 2000 without requiring him to report for work during the pendency
of his investigation.
- Molina filed a "Motion to Dismiss Complaints and Motion for Full Reinstatement" on May
2, 2000. He asserted that the charges should be dismissed since he was compelled to
prepare a written explanation on the basis of "summarized specific acts," denying him the
right to be informed of the exact charges and to confront those who made written reports
against him. As to the issue of reinstatement, he alleged that he should be allowed to
report for work, conformably with Rule XIV, Section 4 of the Implementing Rules of the
Labor Code.
- On May 11, 2000, Picazo wrote Molina that his motion to dismiss the charges would be
resolved after the investigation. He was warned that his non-appearance at the
investigation would be considered a waiver of his right to be heard.
- On the same day, May 11, 2000, Abia issued an inter-office Memorandum announcing
the appointment of Sercy F. Picache as the Officer-In-Charge (OIC) for Metro VI and XVI
effective May 6, 2000.
- Molina and his counsel attended the May 19, 2000 investigation and filed a Motion to
Suspend Proceedings, praying that the administrative investigation be deferred until the
resolution of the "prejudicial" issues raised in his previous motion.
- When Picazo failed to respond, Molina filed, on June 1, 2000, a complaint for damages
with a prayer for a temporary restraining order and preliminary injunction based on Article
19 of the New Civil Code. PPI filed a Motion to Dismiss, maintaining that the courts have

A2010

- 211 -

Disini

no jurisdiction over matters arising from employee-employer relationship. The trial court
denied the motion as well as PPIs motion for reconsideration.
- Meanwhile, in letter dated June 13, 2000, Molina was notified of the termination of
administrative investigation. PPI considered his failure to submit a written explanation as a
waiver of his right to be heard, and as such, the investigating committee had evaluated the
evidence at hand and submitted its recommendations to the "higher management" for
decision. Also, it confirmed the denial of his Motion to Suspend Proceedings
- On June 23, 2000, the trial court issued an Order granting Molina's prayer for temporary
restraining order, which was later made permanent per its Order dated July 12, 2000. The
motion for reconsideration filed by PPI on July 26, 2000 was likewise denied. Thereafter, it
filed a petition for certiorari before the CA, assailing the writ of preliminary injunction
issued by the RTC and its order denying the motion to dismiss the complaint. On July 16,
2001, the CA rendered judgment in favor of PPI and nullified the writ of preliminary
injunction issued by the RTC as well as the order denying the motion of PPI for the
dismissal of the complaint.
- On July 30, 2001, PPI resolved to dismiss Molina from employment on its finding that the
latter violated its standard operating procedure.
- Molina forthwith filed a complaint with the NLRC against PPI and Alfredo C. Antonio,
Patricio A. Picazo, and Certerio B. Uy, in their capacity as President, Senior Assistant
Vice-President of Human Resources Development, and Division Head, respectively, for
illegal dismissal and illegal suspension with claim for monetary benefits.
- In his Position Paper, Molina principally argued that he was denied the right to due
process due to the failure of PPI to furnish him a copy of the written reports of the sales
associates and co-employees, the basis of the accusations against him. Since an OIC for
his position was already appointed even before all his pending motions were resolved, he
surmised that there were really no such reports, and that the alleged accusations were
merely concocted in order to replace him with someone close to Picazo. Molina
maintained that since he was denied the opportunity to dispute the authenticity and
substantive contents of the reports, his alleged violations of company rules and policies
were hearsay and, therefore, lacked probative value. Besides, the termination of his
employment was made without the 30-day prior notice; his dismissal from employment
took effect immediately, only six days after PPI received the CA decision decreeing that
the NLRC has the rightful jurisdiction over the case. Thus, he prayed for the following
relief:
1. Total Money Claims
a) Salary with (overriding) commission from March 21 to April 24, 2000 - suspended
w/o pay - P45,000.00 (P25,000[.00] mo. salary & P20,000[.00] [overriding])
b) Unpaid (overriding) commission from April 25, 2000 to present - P400,000[.00]
c) Unpaid salary from August 1, 2001 to present - P125,000[.00]
d) One mo. salary for every yr. of service in lieu of reinstatement - 7 years =
P175,000.00
2. Leave Credits - P100,000.00 for 7 years
3. Profit Bonus for Year 2000 & 2001 - P400,000.00
4. Moral Damages - P300,000.00
5. Exemplary Damages - P500,000.00
6. Actual Damages - for lifetime medical attendance and medicines at 16 more years
life expectancy - P1,249,384.00
7. Attorney's Fees - P300,000.00
8. Amount debited from complainant's ATM [as partial payment for hospitalization
expenses incurred by him which PPI had advanced] - P12,000.00
9. Retention of complainant's car, as additional penalty for illegal dismissal.
- For its part, PPI stressed that Caritas was its competitor in the pre-need plans business,
and that Molina and his wife recruited and enticed some of the sales associates of PPI to
work for Caritas, in violation of its policy against conflict of interest. Some of these sales
associates were the spouses Eppie and Restie Acosta, Lenita Gatmaitan, Lolita Casaje,
Lydia Magalso, Lydia San Miguel, and Alice Halili, and including Vilma del Rosario, the
secretary of Roy Padiernos. PPI, likewise, averred that Molina had the habit of coming to
the office under the influence of liquor; he constantly shouted to lady employees and
solicited money from his sales associates in connection with an official company function
without returning the same after PPI reimbursed him for the expenses incurred;
disseminated intrigues and created divisiveness among the employees and PPIs senior
officers; and disrespected Padiernos, his superior, by shouting at him during one of the
meetings with other senior officers, and walked out of the meeting afterwards. Supporting
its claims that Molina committed breach of trust, serious misconduct, fraud, and gross
neglect of duty by reason thereof, PPI appended to its position paper the
statements/affidavits of Marivic Uy, Ruth and Roy Padiernos, Eppie and Restie Acosta,
Celeste Villena, and Vilma del Rosario.
- On the claim of Molina that he was denied due process, PPI averred that he was given
sufficient opportunity to present his personal submissions before finally issuing the notice
of dismissal but Molina persistently refused to submit his explanation. PPI further argued
that he was not entitled to the payment of 13th and 14th month salaries, overriding
commission, profit bonus, actual, moral or exemplary damages, and attorneys fees. PPI
maintained that, under Article 217(a) of the Labor Code, as amended, and the ruling of this
Court in Baez v. Valdevilla, Molina should be held liable for P1,000,000 as moral

Labor Law 1
damages and an amount not less than P428,400.00 for the salary he received during the
time when the restraining order/ writ of injunction was erroneously enforced.
- In his Reply, Molina averred that the affidavits submitted by PPI were antedated since he
was never furnished copies of said reports/affidavits despite demands. PPI even failed to
present the reports/affidavits before the RTC where his complaint for damages against PPI
and its officers was pending. He and Roy Padiernos had been at odds because the latter
appointed his brother and wife as agency manager and group manager of PPI to which he
objected. Molina averred that the P200.00 collected from each of the employees of PPI
during their luncheon meeting was a voluntary contribution, and that they spent P4,000.00,
more than the amount collected from the employees. He contended that he had no motive
to recruit sales associates or employees of PPI to be employed by Caritas because the
depletion of sales associates would diminish his effectiveness as an area manager,
including his overriding commission, profit bonus and fringe benefits. He admitted that he
may have raised his voice in the heat of arguing a point during meetings, but averred that
it should not be considered as disrespect or misdemeanor.
- Molina further emphasized that Caritas was not a competitor of PPI, as the former was
engaged in selling health care and is supervised by the Department of Health (DOH),
while the latter is into the business of selling pre-need plans and supervised by the
Securities and Exchange Commission (SEC). Finally, he averred that the so-called
"associates" of PPI were not actually employees but "independent journeymen" who
derived income on commission basis, free to engage in any kind of selling activities not in
direct competition with PPI.
- Molina admitted having had drinking sessions with Certerio Uy, Ilustre Acosta and
Reynaldo Villena, who provided the hard liquor and pulutan, but only after office hours. He
claimed that his officemates mistook him for being drunk when he went to his office even
after office hours because of his "mestizo complexion."
- In its response, PPI averred that, based on the sales data, the acts of Molina caused
demoralization of the sales associates, resulting in a sudden decrease of the region's
output from P343,009,643.00 in 1998 to P263,099,773.00 in 1999, and P228,752,090.00
in 2000. PPI insisted that he should be held liable for not less than P507,348.00,
P2,000,000, and P1,000,000 as actual, moral and exemplary damages, and attorney's
fees, respectively, and P273,600.00 which was the balance on his car plan agreement
with PPI.
- In his Rejoinder and Sur-Rejoinder Molina submitted the affidavit of Geoffrey Martinez,
who belied the reports of Uy, Villena, Del Rosario, and the spouses Padiernos and Acosta.
He also appended the affidavits of Natividad Gatchalian, San Miguel, Gatmaitan, and
Magalso, who all disputed, in one way or another, Molina's alleged violations. To counter
the imputations of conflict of interest, Molina also alleged that Abia and Atty. Reyes were
incorporators of Caritas, and that Villena had in her possession a license to sell Caritas
products. With regard to the declining sales output of his region, Molina attributed the
same to the Asian regional crisis that hit the Philippines sometime in 1997. He noted,
however, that the same records revealed that despite the financial bane, Metro VI still
managed to be on top from 1998 up to 2000 in terms of its sales relative to the other
regions.
- Molina denied any liability for the car plan, claiming that he already settled the obligation
when PPI demanded full payment as, in fact, all the papers related thereto, including the
Release of Mortgage, were already in his possession.
- In its Sur-Rejoinder, PPI stressed its claim that Caritas was a business competitor, as
may be inferred from the benefits available under its health care agreement and the preneed contract of PPI. Particularly with regard to the pension plan contract, it noted the
following similarities: (a) Caritas also provides Term Life Insurance, Accidental Death
Insurance, Credit Life Insurance, and Waiver of Installment Due to Disability; (b) there are
similarities in the provisions on contract price, grace period, cancellation, reinstatement,
and transfer and termination; and (c) unlike other health care programs that provide a oneyear coverage, renewable every year thereafter, Caritas offers a continuous five year
coverage and sells the same in units payable in five-year installment basis, with maturity
period and guaranteed return of investment in the form of Full-Term Medical Expense
Fund computed at P10,000.00 for every unit purchased with increment of 10% yearly after
the maturity period, which may be withdrawn in cash by its member. It stressed that this
was similar to the pension program offered by PPI which was also sold in per unit basis,
payable by installment in certain number of years or lump sum payment, and upon
maturity also gives P10,000.00 pension benefit per unit purchased by the plan holder. With
respect to the alleged interest of Atty. Reyes with Caritas, PPI adduced in evidence a
Deed of Sale to prove that as early as February 1999 he had already divested his
stockholdings in Caritas.
- On November 18, 2002, Labor Arbiter Roma C. Asinas rendered a Decision dismissing
the complaint and the counterclaims for lack of merit. The labor arbiter ruled that Molina
was lawfully dismissed from his employment for serious misconduct in office and fraud or
willful breach of trust and confidence. It declared that Molinas mere denial of the charges
against him did not overthrow the overwhelming evidence against him tending to show
that he committed the allegations against him. Moreover, his wife was then an agency
manager of Caritas, and some PPI sales associates were with Caritas because they were
recruited by Molina. The labor arbiter also ruled that other employees of respondent
attested to the fact that they were being recruited and enticed by the complainant to join
Caritas. This act of pirating constituted serious misconduct in office, fraud or willful breach

A2010

- 212 -

Disini

of trust and confidence, which are just causes for termination of employment under Article
282 of the Labor Code, as amended. As such, PPI could not legally be compelled to
continue Molinas employment due to breach of trust.
- The labor arbiter likewise held that Molina was afforded his right to due process, but that
he refused to give an answer to the charges leveled against him, and instead insisted that
he be furnished a copy of the alleged reports against him. Since he was given ample
opportunity to answer the charges and explain his side during the investigation, and a
formal or trial-type hearing is not at all times essential, Molinas right to due process was
not violated. The labor arbiter stressed that the requirements of due process are satisfied
where the parties are afforded fair and reasonable opportunity to explain their side of the
controversy at hand.
- Molina appealed the decision to the NLRC, which rendered judgment in his favor. The
NLRC reversed the decision of the Labor Arbiter and ordered Molinas immediate
reinstatement to his former position as Assistant Vice President without demotion in rank
and salary; and the payment of his backwages from August 1, 2001 up to his actual
reinstatement, and other accrued monetary benefits. However, the NLRC denied all other
claims for damages.
- According to the NLRC, the charges of coming to the office under the influence of liquor
and making PPI reimburse the expenses already paid by Molina's co-employees were not
supported by the records. The "loss of trust and confidence" had no factual basis since the
alleged acts of Molina did not result to any loss in favor of PPI.
- Anent Molinas recruitment activities, the NLRC ratiocinated that PPI failed to show that
Caritas was a competitor of PPI. Caritas caters to the health care needs of its clients while
PPI to the pre-need (pension, educational, and memorial) requirements of its plan holders.
Any similarity between PPI and Caritas extra features like term life insurance, accidental
death insurance, credit life insurance, and waiver of installment due to disability, did not
ipso facto make Caritas a competitor of PPI. Thus, there was no conflict of interest in
Molinas act of trying to recruit counselors for Caritas to help his wife. Moreover, PPI failed
to establish that recruiting for Caritas affected Molinas decisions in the performance of his
duties with PPI. According to the NLRC, the drop in the sales and productivity of
complainants area of responsibility may be due to market forces and depressed economic
condition at that time; absent any clear and convincing proof, it cannot be attributed to the
alleged acts of Molina which constituted willful breach of trust or confidence.
- PPI filed a motion for reconsideration, and appended a Letter dated June 13, 2002 from
the SEC to Caritas, indicating that its HMO Plan was similar to the previous plans offered
by pre-need companies, hence, under the regulatory suspension of the SEC; another
letter of SEC ordering Caritas to immediately desist from selling its HMO plan with the full
term medial expense fund; and the letter of Caritas, through counsel, endorsing the
objectionable features of the HMO plan.
- The NLRC, however, was not persuaded, and resolved to deny PPIs motion in its Order
dated September 30, 2003. On November 19, 2003, the NLRC declared its Decision final
and executory as of November 14, 2003.
- PPI filed a Petition for Certiorari with the CA for the nullification of the decision and
resolution of the NLRC and the reinstatement of the decision of the Labor Arbiter.
- On August 13, 2004, the CA rendered a decision reversing the Decision and Resolution
of the NLRC, and reinstating the November 18, 2002 Decision of the Labor Arbiter. Later,
the CA denied Molinas Motion for Reconsideration in its Resolution dated September 27,
2004.
- The issues for resolution are the following: whether the decision of the NLRC was
already final and executory when PPI filed its petition for certiorari in the CA; and whether
the NLRC committed grave abuse of discretion amounting to excess or lack of jurisdiction
in issuing the assailed decision and resolution.
- On the first issue, we find and so hold that the decision of the NLRC had become final
and executory when PPI filed its Petition for Certiorari in the CA. PPI received a copy of
the NLRC Decision on July 11, 2003 and filed the Motion for Reconsideration thereof on
July 18, 2003, which motion was denied on September 30, 2003. Under Rule VII, Section
2 of the NLRC Omnibus Rules of Procedure, the decision of the NLRC becomes final and
executory after ten (10) calendar days from receipt of the same. PPI received a copy of
the NLRC decision on November 30, 2003; hence, such decision became final and
executory on December 3, 2003. Nonetheless, the Court ruled in St. Martin Funeral Home
v. NLRC that, although the 10-day period for finality of the NLRC decision may have
elapsed as contemplated in the last paragraph of Section 223 of the Labor Code, the CA
may still take cognizance of and resolve a petition for certiorari for the nullification of the
decision of the NLRC on jurisdictional and due process considerations. Indeed, the
remedy of the aggrieved party from an adverse decision of the NLRC is to timely file a
motion for reconsideration as a precondition for any further or subsequent remedy, and if
the motion is denied, such party may file a special civil action in accordance with law and
jurisprudence considering that these matters are inseparable in resolving the main issue of
whether the NLRC committed grave abuse of discretion.
- The Labor Arbiter and the NLRC act in quasi-judicial capacity in resolving cases after
hearing and on appeal, respectively. On the presumption that they have already acquired
expertise in their jurisdiction, which is confined on specific matters, their findings of facts
are oftentimes accorded not only with respect but even finality if supported by substantial
evidence. However, in spite of the statutory provision making "final" the decision of the
NLRC, the Court has taken cognizance of petitions challenging such decision where there

Labor Law 1
is a clear showing that there is want of jurisdiction, grave abuse of discretion, violation of
due process, denial of substantial justice, or erroneous interpretation of law.
- In this case, the Labor Arbiter declared that there is substantial evidence on record
warranting the dismissal of petitioner as Assistant Vice President for serious misconduct in
office, fraud or willful breach of trust and confidence. The NLRC disagreed with the Labor
Arbiter and reversed the latters findings. The CA, for its part, concurred with the findings
of the Labor Arbiter. In view of the discordance between the findings of the Labor Arbiter
and the CA on one hand, and the NLRC on the other, there is a need for the Court to
review the factual findings and the conclusions based on the said findings. As this Court
held in Diamond Motors Corporation v. Court of Appeals:
- A disharmony between the factual findings of the Labor Arbiter and the National Labor
Relations Commission opens the door to a review thereof by this Court. Factual findings of
administrative agencies are not infallible and will be set aside when they fail the test of
arbitrariness. Moreover, when the findings of the National Labor Relations Commission
contradict those of the labor arbiter, this Court, in the exercise of its equity jurisdiction, may
look into the records of the case and reexamine the questioned findings
- Article 282 of the Labor Code of the Philippines provides:
Art. 282. Termination by employer. An employer may terminate an employment for
any of the following causes:
a. Serious misconduct or willful disobedience by the employee of the lawful orders of
his employer or representative in connection with his work;
b. Gross and habitual neglect by the employee of his duties;
c. Fraud or willful breach by the employee of his duties of the trust reposed in him by
his employer or duly authorized representative;
d. Commission of a crime or offense by the employee against the person of his
employer or any immediate member of his family or his duly authorized representative;
and
e. Other causes analogous to the foregoing.
- Misconduct has been defined as improper or wrong conduct; the transgression of some
established and definite rule of action; a forbidden act, a dereliction of duty, unlawful in
character and implies wrongful intent and not mere error of judgment. The misconduct to
be serious must be of such grave and aggravated character and not merely trivial and
unimportant. Such misconduct, however, serious, must nevertheless, be in connection
with the employees work to constitute just cause for his separation.
- The loss of trust and confidence, in turn, must be based on the willful breach of the trust
reposed in the employee by his employer. Ordinary breach will not suffice. A breach of
trust is willful if it is done intentionally, knowingly and purposely without justifiable excuse,
as distinguished from an act done carelessly, thoughtlessly, heedlessly or inadvertently.
The Court has laid down the guidelines for the application of the doctrine for loss of
confidence, thus:
1. the loss of confidence must not be simulated;
2. it should not be used as a subterfuge for causes which are illegal, improper or
unjustified;
3. it may not be arbitrarily asserted in the face of overwhelming evidence to the
contrary;
4. it must be genuine, not a mere afterthought, to justify earlier action taken in bad faith;
and
5. the employee involved holds a position of trust and confidence.
In Samson v. Court of Appeals, the Court enumerated the conditions for one to be
considered a managerial employee:
(1) Their primary duty consists of the management of the establishment in which they
are employed or of a department or subdivision thereof;
(2) They customarily and regularly direct the work of two or more employees therein;
(3) They have the authority to hire or fire other employees of lower rank; or their
suggestions and recommendations as to the hiring and firing and as to the promotion
or any other change of status of other employees are given particular weight.
- As a general rule, employers are allowed wide latitude of discretion in terminating the
employment of managerial personnel. The mere existence of a basis for believing that
such employee has breached the trust and confidence of his employer would suffice for
his dismissal.
- In this case, petitioner was not a mere employee of respondent. He was the Assistant
Vice-President with the same functions of a regional manager of the same sales region,
Metro Manila VI. Taking into account his job description, he was one of the top managers
of the respondent, tasked to perform key and sensitive functions in the interest of his
employer and, thus, bound by the more exacting work ethic.
- We find, however, that the charge of misappropriation of funds was not proven with
substantial evidence. As gleaned from the handwritten statement of Ilustre Acosta, the
General Manager of the Springs and Blessings General Agency under Metro Manila VI, it
appears that, aside from him and petitioner, there were 10 other attendees during the
luncheon conference on November 27, 1999 at the Barrio Fiesta, Cubao, Quezon City.
Petitioner received the amount of only P2,386.00 from respondent to pay for the cost of
the luncheon for the conference, based on Petty Cash Voucher signed by petitioner, 74 but
the conferees spent more than P4,000.00. Upon petitioners suggestion, the conferees
agreed to contribute P200.00 each, or the total amount of P2000.00 to answer for the
difference. Petitioner had no obligation to return the contributions of the conferees, nor

A2010

- 213 -

Disini

was he liable for said amount. Significantly, except for Ilustre Acosta, the other attendees
in the conference never complained against petitioner or requested him to return their
respective contributions of P200.00.
- Regarding the charge that the petitioner peddled false and malicious informations
against Abia and Padiernos, Abia has not executed any affidavit to confirm paragraph 9 of
the affidavit of Roy Padiernos. As admitted by del Rosario, the informations allegedly
relayed to her by the petitioner pertaining to Roy Padiernos were confirmed by Zita
Domingo.
- The petitioner does not deny having had a heated exchange of words with Roy
Padiernos in the course of a meeting. However, such incident does not constitute proof
that the petitioner thereby showed disrespect to Roy Padiernos, nor a valid cause for
petitioners dismissal. It does happen that in the course of exchange of views during a
meeting, participants may become so assertive to the point of being overbearing or
unyielding and in the process lose their temper, on their sincere belief of being right. There
is no evidence on record that petitioner committed the same or similar acts thereafter.
- To prove its charge of conduct unbecoming of a company officer, more specifically of
drinking alcoholic beverages within the premises of the company during office hours or
going to work drunk, respondent relied on the statement/affidavit of Celeste Villena, the
Agency Manager of the Wondrous and Miraculous General Agency under Metro Manila VI;
and Marivic Uy, the General Manager of the DMBP General Agency under Metro Manila
VI. Both claimed that they always saw petitioner drunk during office hours, most especially
during cut-offs when many sales counselors were present. Petitioner admitted having had
drinking sessions with Certerio Uy, the husband of Marivic Uy, Ilustre Acosta and Reynaldo
Villena, the husband of Celeste Villena, and who, according to petitioner, provided the
hard liquor and the pulutan. He, however, denied reporting to office drunk and insisted that
he reported for work sober.
- We are inclined to give credence to petitioners claim, noting that in her handwritten
letter-report to Norman Gonzales dated March 10, 2000, Villena made no mention of the
petitioner going to office drunk. It was only in her affidavit dated January 16, 2002 that
Villena made such declaration. Villena did not explain her failure to report the matter to
Gonzales on March 10, 2000, and why she made the charge for the first time in her
Affidavit dated January 16, 2002. Uy is the wife of no less than Certerio Uy, the Senior
Vice-President of the Manila North Sales Division of respondent. If petitioners "drinking
problem" had any ring of truth, she should have immediately reported the matter to her
husband or to other officials concerned. Uys unexplained silence until March 10, 2000
thus renders her report implausible.
- Respondent avers that petitioner served directly as agent of Caritas, a business
competitor of the respondent, when he connived with his wife in recruiting Sales
Associates of the Metro Sales Division VI to transfer to Caritas as sales associates.
Respondent claims that, by his acts, petitioner failed to dedicate his full time on the job
with respondent and prevented said sales associates from doing the same. Aside from
violating its policy against conflict of interest, petitioners acts adversely affected his
decisions in the performance of his duties and obligations to respondent.
- Loyalty of an employee to his employer consists of certain very basic and common sense
obligations. An employee must not, while employed, act contrary to the employers
interest. The scope of the duty of loyalty that an employee owes to his employer may vary
with the nature of their relationship. Employees occupying a position of trust and
confidence owe a higher duty than those performing low-level tasks. Assisting an
employees competitor can even constitute a breach of the employees duty of loyalty. An
employees self-dealing may breach that duty. However, it has been ruled that
- A reality of contemporary life is that many families will consist of two wage earners, one
wage earner with two jobs, or both. For some employees, particularly those earning low or
modest incomes, second sources of income are an economic necessity. For them, a
second job or "moonlighting" is the only way to make ends meet. Conversely, employers
need the assurance that employees will not disserve them by furthering their own interests
or those of competitors at the employers expense.
- A slight assistance to a direct competitor could constitute a breach of the employees
duty of loyalty. However, when competition is indirect or minimal, the employer may be
required to show that the employee received substantial assistance from the competitor. If
an employee usurped a corporate opportunity or secretly profited from a competitive
activity, the employer may receive the value of the lost opportunity or the secret profit.
- An employees skill, aptitude, and other subjective knowledge obtained in the course of
employment are not the property of his employer. However, an employee occupying a
managerial position or office is obliged to protect the trade secret of his employer
consisting of formula, process, device or compilation which it uses in its business and
gives it an opportunity to obtain an advantage over competitors who do not know of such
trade secret. However, the rule does not apply to a matter of public knowledge or of
general knowledge within the industry. Moreover, an employer has a protectible interest in
the customer relationships of its former employee established and/or nurtured while
employed by the employer, and is entitled to protect itself from the risk that a former
employee might appropriate customers by taking unfair advantage of the contract
developed while working for the employer. While acting as an agent of his employer, an
employee owes the duty of fidelity and loyalty. Being a fiduciary, he cannot act
inconsistently with his agency or trust. He cannot solicit his employers customers or coemployees for himself or for a business competitor of his employer. If such employee or

Labor Law 1
officer connives with and induces another to betray his employer in favor of a business
competitor of his employer, he is held accountable for his mischief.
- In this case, we are not persuaded that Caritas is the business competitor of respondent.
The evidence on record shows that while Abia, the Senior Vice-President of respondents
Metro Manila Marketing, is one of the incorporators of Caritas and is even a member of
the Board of Directors, respondent did not dismiss him from employment. The Head of the
Legal Division of the respondent, Atty. Reyes, was also an incorporator of Caritas and a
member of its Board of Directors, and although he appears to have sold his shares to
Herminigildo C. Belen for P127,312.34, he only did so on March 7, 1999. There is no
evidence on record whether the transfer of such shares of stocks has already been
reflected in the books of Caritas. Celeste Villena, one of the Sales Associates of
respondent, is herself licensed by Caritas to sell plans for the latter. Villena has likewise
not been prohibited from selling pre-need plans for Caritas. Fe Molina, who is the head of
a sales agency of Caritas, is also a sales agency head of respondent. Petitioner, his wife,
and Villena were not charged nor meted any sanction by the respondent for conflict of
interest. Petitioner was the Assistant Vice-President, Marketing Head, Area 10, of Caritas,
and for a while, without any protest from respondent. If Caritas is a business competitor of
the respondent, it should have meted sanctions not only on petitioner but also on Abia,
Reyes, Fe Molina and Villena as well.
- The truth of the matter is that, as averred by Caritas President Geoffrey Martinez, Caritas
is engaged in health care and hospitalization package, whereas respondent sells
educational, pension, and pre-need plans. Caritas is an HMO and is directly supervised by
the DOH, while respondent is under the supervision of the SEC. The so-called sales
associates of the respondent are non-salaried employees and are paid on commission
basis only. Their commissions are based on their individual initiative and industry. That the
contracts executed by the beneficiaries of both corporations have similar provisions
regarding contract price, grace period, cancellation, reinstatement, transfer and
termination, do not constitute proof that Caritas and respondent are business competitors.
There is also no proof that the two corporations compete with each other in the same or
similar business; in fact, the business of Caritas and that of the respondent complement
each other.
- Respondent relied on the declarations of Ruth Padiernos, Spouses Eppie and Ilustre
Acosta, Celeste Villena, and Marivic Uy to prove its charge that Fe Molina pirated sales
associates working for respondent and that petitioner tolerated the actuations of his wife
and even connived with her.
- The Court finds, however, that the evidence adduced by respondent insufficient to
warrant the petitioners dismissal from employment.
- Ruth Padiernos, wife of Roy Padiernos, averred in her written statement dated March 8,
2000, that as far back as July 1999, she had a conference with her husband and Abia
where she reported that petitioner connived with his wife in pirating sales associates. She
was assured that something would be done to arrest the problem. 90 However, Ruth
Padiernos failed to name any such sales associate who was recruited by Fe Molina. There
is likewise no evidence that Abia ever confronted petitioner relative to the charge. Roy
Padiernos confronted petitioner, but the latter denied the charge. Since then, no further
action was taken against the petitioner by respondent, until the letter of Picazo dated
March 21, 2000 was sent to him. Roy Padiernos did not explain why he executed his
affidavit regarding the matter almost three years later, only on January 18, 2002. In an
Affidavit dated January 18, 2002, it was made to appear that Ruth Padiernos claimed that
petitioners wife, the Unit Manager of the Ark Group under Metro Manila Sales Group VI
and also an Agency Manager of Caritas, recruited sales associates under respondent to
work for Caritas, and that petitioner did the same; and that she (Padiernos) learned that
almost all the productive Sales Associates in Metro Manila VI were already connected with
Caritas, using "different names." Although notarized, the affidavit has no probative weight
because it was unsigned.
- Celeste Villena, for her part, declared in her handwritten statement dated March 10, 2000
that Fe Molina recruited Lenie Gatmaitan to join Caritas and that she confronted
petitioner.92 In her Affidavit dated January 16, 2002, she alleged that petitioner and his
wife, Fe Molina, recruited Gatmaitan to join Caritas. However, the signature of the notary
public does not appear in said affidavit. For his part, Ilustre Acosta, averred in his
handwritten statement dated March 11, 2000, that on March 4, 2000, petitioner informed
him that Geoffrey Martinez called petitioner to inquire if petitioner would have no objection
for him (Ilustre) to be with Caritas and that petitioner replied that he had no objection if that
was Ilustres decision. Ilustre maintained this claim in his Affidavit dated January 16, 2002.
Eppie Acosta, the wife of Ilustre Acosta, averred in her handwritten statement of March 12,
2000, that on March 6, 2000, petitioner commented about their low sales production, and
she retorted that he was the cause, hence, may have grudges against him. Petitioner
replied that he and his wife did not interfere with each others business dealings, and that
petitioner even declared "Mare, for all you know, ikaw na lang ang hindi nag-ca-Caritas."
She reiterated her claim in her affidavit dated January 16, 2000. Marivic Uy averred that
the wife of petitioner had been pirating sales associates of respondent since 1999 to join
Caritas and that she tried to recruit Morena Siasoco, one of the Group Managers.
Petitioner failed to stop his wife, but rather tolerated her actuations. She reiterated her
claim in her Affidavit dated January 16, 2002

A2010

- 214 -

Disini

- However, there is no evidence on record to prove that respondent expressly prohibited


its Sales Associates from selling for Caritas. Neither is there evidence on record to prove
that Caritas prohibited its sales associates from selling pre-need plans of respondent.
- Respondent likewise failed to present the affidavits of Siasoco, Casaje, Magalso, San
Miguel and Halili. In contrast to the evidence of respondent, Gatchalian, San Miguel,
Siasoco, and Gatmaitan executed their respective affidavits declaring that neither
petitioner nor his wife ever recruited them. 99 They admitted that they sold plans for Caritas,
but without any prodding from petitioner and his wife. Geoffrey Martinez declared, in his
affidavit, that Siasoco, San Miguel, Casaje, Magalso, and Halili joined Caritas voluntarily
and individually, through him, and he was not aware that petitioner and his wife
recommended them to Caritas. Lenita Gatmaitan called him and inquired if she could join
Caritas, and he replied in the affirmative. He never called petitioner concerning Ilustre
Acosta; on the contrary, it was the latter who called to inquire if he was entitled to a
discount if he purchased a Caritas health plan. He talked to Vilma Del Rosario and
convinced her to apply as Branch Manager of Caritas, which she did, but backed out later
on.
Disposition IN LIGHT OF ALL THE FOREGOING, the instant petition is hereby
GRANTED. The August 13, 2004 Decision and September 27, 2004 Resolution of the
Court of Appeals are REVERSED AND SET ASIDE. The decision and resolution of the
NLRC are reinstated.

WILLFUL DISOBEDIENCE
MICRO SALES OPERATION NETWORK V NLRC
472 SCRA 328
QUISUMBING; October 11, 2005
NATURE
For review on certiorari of the Resolutions the CA dismissing petitioners special civil
action for certiorari against the NLRC Resolution, which affirmed the Labor Arbiters
Decision finding petitioners herein liable for illegal dismissal.
FACTS
- Micro Sales Operation Network is a domestic corporation engaged in local transportation
of goods by land. Petitioner Willy Bendol was the companys operations manager at the
time of the controversy.
- Private respondents Larry Hermosa, Leonardo de Castro, and Ramil Basinillo were
employed by the company as driver, warehouseman, and helper, respectively. Hermosa
was hired on November 17, 1997, de Castro on February 1, 1996, and Basinillo on
February 4, 1998.
- Hermosa failed to promptly surrender the ignition key of the companys vehicle after
discharging his duties. Such failure was allegedly contrary to the companys standard
operating procedure. Thus, he was asked to explain within 24 hours why disciplinary
action should not be meted on him. He explained that he kept the ignition key because the
vehicle was stalled when its battery broke down. Unsatisfied with Hermosas explanation,
the company dismissed him on January 9, 1999.
- LA found that private respondents were illegally dismissed. NLRC affirmed the Labor
Arbiters decision. It also denied petitioners motion for reconsideration.
CA dismissed the petition for being defective in form.
ISSUES
1. WON the private respondents were unjustly dismissed
2. WON there was willful disobedience on the part of the private respondents, justifying
their dismissal
HELD
1. YES
- Hermosa was unjustly dismissed
2. NO
- For willful disobedience to be a valid cause for dismissal, the following twin elements
must concur: (1) the employee's assailed conduct must have been willful, that is,
characterized by a wrongful and perverse attitude; and (2) the order violated must have
been reasonable, lawful, made known to the employee and must pertain to the duties
which he had been engaged to discharge.
- Both elements are lacking. We find no hint of perverse attitude in Hermosas written
explanation. On the contrary, it appears that the alleged company procedure for leaving
the ignition key of the companys vehicles within office premises was not even made
known to him. Petitioners failed to prove Hermosa willfully disobeyed the said company
procedure. At any rate, dismissal was too harsh a penalty for the omission imputed to him.
Disposition NLRC Resolution affirming the Labor Arbiters Decision, finding petitioners
liable for illegal dismissal, is AFFIRMED.

Labor Law 1
BASCON V CA (METRO CEBU COMMUNITY
HOSPITAL)
422 SCRA 122
QUISUMBING; February 5, 2004
FACTS
- ELIZABETH BASCON and NOEMI COLE, petitioners, were employees of Metro Cebu
Community Hospital, Inc. (MCCH) and members of the Nagkahiusang Mamumuo sa
Metro Cebu Community Hospital (NAMA-MCCH), a labor union of MCCH employees.
Bascon had been employed as a nurse by MCCH since May 1984. At the time of her
termination from employment in April 1996, she already held the position of Head Nurse.
Cole had been working as a nursing aide with MCCH since August 1974. Both were
dismissed for allegedly participating in an illegal strike.
- The controversy arose from an intra-union conflict between the NAMA-MCCH and the
National Labor Federation (NFL), the mother federation of NAMA-MCCH. In November
1995, NAMA-MCCH asked MCCH to renew their CBA, which was set to expire on
December 31, 1995. NFL, however, opposed this move. Mindful of the apparent intraunion dispute, MCCH decided to defer the CBA negotiations until there was a
determination as to which union had the right to negotiate a new CBA. Believing that their
union was the certified CBA agent, NAMA-MCCH staged a series of mass actions inside
MCCHs premises starting February 27, 1996. The DOLE in Region 7 issued two
certifications stating that NAMA-MCCH was not a registered labor organization. This
finding, however, did not deter NAMA-MCCH from filing a notice of strike with the Region 7
Office of the National Conciliation and Mediation Board (NCMB). Said notice was,
however, disregarded by the NCMB for want of legal personality of the union.
- MCCH notified the petitioners that they were to be investigated for their activities in the
mass actions. Petitioners, however, denied receiving said notices. In a notice dated April
8, 1996, MCCH ordered petitioners to desist from participating in the mass actions
conducted in the hospital premises with a warning that non-compliance would result in the
imposition of disciplinary measures. Petitioners again claimed they did not receive said
order. Bascon and Cole were then served notices terminating their employment effective
April 12, 1996 and April 19, 1996, respectively.
- The Labor Arbiter found the termination to be valid and legal. The Labor Arbiter held that
petitioners were justly dismissed because they actually participated in the illegal mass
action. It also concluded that petitioners received the notices of hearing, but deliberately
refused to attend the scheduled investigation.
- The NLRC reversed the ruling and ordered the reinstatement of petitioners with full back
wages. First, it found that petitioners merely wore armbands for union identity, per
instruction of their union officials. Said wearing of armbands while nursing patients, is a
constitutional right, which cannot be curtailed if peacefully carried out. Second, it ruled that
the placards complained of by MCCH did not contain scurrilous, indecent or libelous
remarks. Finally, it concluded that, in a belated but crude attempt to camouflage the illegal
dismissal of petitioners, MCCH merely fabricated the notices allegedly sent to petitioners.
On the charge of gross insubordination, it ruled that petitioners were not guilty, because
the elements had not been sufficiently proven, to wit: (1) reasonableness and lawfulness
of the order or directive, (2) sufficiency of knowledge on the part of the employee of such
order, and (3) the connection of the order with the duties which the employee had been
engaged to discharge.
- MCCH filed a special civil action for certiorari before the CA. The CA granted the petition
but ordered payment of separation pay.

A2010

HELD
1. NO
Ratio While a union officer can be terminated for mere participation in an illegal strike, an
ordinary striking employee must have participated in the commission of illegal acts during
the strike. There must be proof that they committed illegal acts during the strike. But proof
beyond reasonable doubt is not required. Substantial evidence, which may justify the
imposition of the penalty of dismissal, may suffice.
Reasoning
- Article 264 (a) of the Labor Code provides in part that:
Any union officer who knowingly participates in illegal strike and any worker or union
officer who knowingly participates in the commission of illegal acts during a strike may
be declared to have lost his employment status

Disini

Reasoning
- Article 282 of the Labor Code provides in part:
An employer may terminate an employment for any of the following causes: (a) Serious
misconduct or willful disobedience by the employee of the lawful orders of his employer
or representative in connection with his work.
- We find lacking the element of willfulness characterized by a perverse mental attitude on
the part of petitioners in disobeying their employers order as to warrant the ultimate
penalty of dismissal. Wearing armbands and putting up placards to express ones views
without violating the rights of third parties, are legal per se and even constitutionally
protected. Thus, MCCH could have done well to respect petitioners right to freedom of
speech instead of threatening them with disciplinary action and eventually terminating
them.
- Neither are we convinced that petitioners exercise of the right to freedom of speech
should be taken in conjunction with the illegal acts committed by other union members in
the course of the series of mass actions. It bears stressing that said illegal acts were
committed by other union members after petitioners were already terminated, not during
the time that the latter wore armbands and put up placards.
- Finally, even if willful disobedience may be properly appreciated, still, the penalty of
dismissal is too harsh. Not every case of willful disobedience by an employee of a lawful
work-connected order of the employer may be penalized with dismissal. There must be
reasonable proportionality between, on the one hand, the willful disobedience by the
employee and, on the other hand, the penalty imposed. In this case, evidence is wanting
on the depravity of conduct and willfulness of the disobedience on the part of petitioners,
as contemplated by law. Wearing armbands to signify union membership and putting up
placards to express their views cannot be of such great dimension as to warrant the
extreme penalty of dismissal, especially considering their long years of service and the
fact that they have not been subject of any disciplinary action in the course of their
employment with MCCH.
Disposition Petition is GRANTED. The Decision of the CA is REVERSED. MCCH is
hereby ordered to reinstate petitioners without loss of seniority rights and other privileges
and to pay them full back wages, inclusive of allowances, and other benefits computed
from the time they were dismissed up to the time of their actual reinstatement.

ISSUES
1. WON petitioners were validly terminated for allegedly participating in an illegal strike
2. WON petitioners were validly terminated for gross insubordination to the order to stop
wearing armbands and putting up [of] placards

- 215 -

- The CA found that petitioners actual participation in the illegal strike was limited to
wearing armbands and putting up placards. There was no finding that the armbands or the
placards contained offensive words or symbols. Thus, neither such wearing of armbands
nor said putting up of placards can be construed as an illegal act. In fact, per se, they are
within the mantle of constitutional protection under freedom of speech. Evidence shows
that various illegal acts were committed by unidentified union members in the course of
the protracted mass action. And we commiserate with MCCH, patients, and third parties
for the damage they suffered. But we cannot hold petitioners responsible for acts they did
not commit. The law, obviously solicitous of the welfare of the common worker, requires,
before termination may be considered, that an ordinary union member must have
knowingly participated in the commission of illegal acts during a strike.
2. NO
Ratio Willful disobedience of the employers lawful orders, as a just cause for dismissal of
an employee, envisages the concurrence of at least two requisites: (1) the employee's
assailed conduct must have been willful, that is, characterized by a wrongful and perverse
attitude; and (2) the order violated must have been reasonable, lawful, made known to the
employee and must pertain to the duties which he had been engaged to discharge.

R TRANSPORT CORP V EJANELRA


[PAGE 55]

B. GROSS AND HABITUAL NEGLECT OF


DUTIES
REQUISITES
JUDY PHILIPIINES V NLRC
289 SCRA 755
MARTINEZ; April 29, 1998
NATURE
Special civil action for certiorari to annul NLRC decision
FACTS

Labor Law 1
- Virginia Antiola was employed as an assorter of baby infant dresses by Judy Philippines,
Inc. in its export business. She was directed by her supervisor, to sort out baby infant
dresses pursuant to an instruction sheet.
- Petitioner required Antiola to explain in writing why she should not be meted disciplinary
sanctions for her erroneous assortment and packaging of 2,680 dozens of infant wear.
She admitted her error and asked for forgiveness. Antiolas supervisor and the packer also
received a memo requiring them to explain why they should not be penalized. Both
submitted their explanations.
- Petitioner found Antiola guilty of negligence and she was dismissed from employment.
The supervisor was suspended for one month on the ground of negligence through
command responsibility. The packer was found innocent on the ground that when she
undertook the packing of the infant wear, the same were already sealed in black plastic
bags and could no longer be checked.
- The National Federation of Labor Union (NAFLU), in behalf of Antiola, filed a complaint
for unfair labor practice and illegal dismissal against Judy Philippines, Inc. They alleged
that the dismissal was unjustified because the infant wear erroneously assorted by Antiola
should not have been shipped to the buyer had the companys supervisor and the buyers
quality comptroller exercised due diligence in the performance of their duties in ensuring
that the goods were properly assorted.
- Labor arbiter held that the dismissal was lawful on the ground of fault and negligence
causing an irreparable damage to the goodwill of the petitioners business, especially
considering that the latter is an export oriented entity
- NLRC held that to qualify as a valid cause for dismissal under Art. 282(b) of the Labor
Code, neglect must not only be gross, it should be Gross and habitual neglect in
character. NLRC ordered petitioner to reinstate Antiola, with one year backwages
ISSUES
1. WON the appeal before the NLRC had been seasonably made
2. WON the offense committed by Antiola constitute a just cause for dismissal under
article 282 of the labor code.
HELD
1. YES
- Under Article 223 of the Labor Code, as amended, the period to appeal to the
Commission is ten calendar days, to wit:
Article 223. Appeal. - Decisions, awards, or orders of the Labor Arbiter are final
and executory unless appealed to the Commission by any or both parties within
ten (10) calendar days from receipt of such decisions, awards or orders.
- It is admitted that Antiola received the labor arbiters decision on May 2, 1990. She filed
her appeal on May 14, 1990, a Monday.
- In subsequent cases, We ruled that if the tenth day to perfect an appeal from the
decision of the Labor Arbiter to the NLRC falls on a Saturday, the appeal shall be made on
the next working day as embodied in Section 1, Rule VI of the NLRC Rules of Procedure
promulgated on January 14, 1992. This conclusion recognizes the fact that on Saturdays
the offices of NLRC and certain post offices are closed.
- Even assuming arguendo that the appeal was filed beyond the period allowed by law,
technical rules of procedure in labor cases are not to be strictly applied if the result would
be detrimental to the working man.
2. NO
- Gross negligence implies a want or absence of or failure to exercise slight care or
diligence, or the entire absence of care. It evinces a thoughtless disregard of
consequences without exerting any effort to avoid them.
- Article 282 (b) of the Labor Code requires that xxx such neglect must not only be
gross, it should be Gross and habitual neglect in character.
- The employers obligation to give his workers just compensation and treatment carries
with it the corollary right to expect from the workers adequate work, diligence and good
conduct.
- Considering however that private respondent worked with the company for 4 years with
no known previous bad record, the ends of social and compassionate justice would be
better served if she was merely suspended from work rather than terminated.
- Petitioner should be reinstated but not awarded backwages. RA 6715, which provides
that an illegally dismissed employee is entitled to full backwages, inclusive of allowances,
and to his other benefits or their monetary equivalent computed from the time his
compensation was withheld from him up to the time of his actual reinstatement, has no
retroactive effect.
Disposition NLRC decision AFFIRMED but MODIFIED in that petitioner. is ordered to pay
private respondent Virginia Antiola backwages for a period of three years, without
qualification or deduction.

CHAVEZ V NLRC
[PAGE 59]

A2010

- 216 -

Disini

CHALLENGE SOCKS CORP V CA (NLRC, ANTONIO ET


AL)
474 SCRA 356
YNARES-SANTIAGO; November 8, 2005
NATURE
CERTIORARI under RULE 45
FACTS
- CHALLENGE SOCKS CORP (CSC) hired Elvie Buguat as knitting operator.
- In the course of her employment, she incurred absences and tardiness without prior
approval and had been neglectful of her duties.
- May 25, 1998: she failed to check the socks she was working on causing excess use of
yarn and damage to the socks design.
- She was suspended for 5 days and warned that a repetition of the same act would mean
dismissal from the service.
- February 2, 1999: she committed the same infraction and was given a warning.
- Despite the previous warnings, Buguat continued to be habitually absent and inattentive
to her task.
- March 1, 1999: she again failed to properly count the bundle of socks assigned to her.
- March 2, 1999: CSC terminated her services on grounds of habitual absenteeism without
prior leave, tardiness and neglect of work.
[8]
- Thereafter, Buguat filed a complaint for illegal dismissal.
- LA: Buguat was illegally dismissed; ordered CSC to reinstate her without loss of seniority
rights and benefits, but w/o backwages; ruled that mistake in counting bundles of socks is
tolerable and should be punished by suspension only.
- NLRC: adopted the findings of LA. Denied CSC's Appeal and MR.
- CA: reversed and set aside LAs and NLRCs decisions; CSC was ordered to pay
BUGUAT full backwages; remanded to the Regional LA for the computation of the
backwages.
- CA also noted that petitioner failed to comply with the twin-notice requirement in
terminating an employee hence, the dismissal was considered ineffectual.
ISSUE
WON Buguats termination is valid
HELD
YES
Reasoning
- One of the just causes for terminating an employment under Article 282 of the Labor
Code is gross and habitual neglect by the employee of her duties. This cause includes
gross inefficiency, negligence and carelessness. Such just causes is derived from the
right of the employer to select and engage his employees.
- As a knitting operator, Elvie was required to check the socks she was working on and to
count the bundles of socks she had to pack to be forwarded to the Looping Section.
- Her repeated commission of the same offense could be considered willful disobedience.
Elvie, despite the suspension and warning, continued to disregard the company rules and
regulations.
- Habitual neglect implies repeated failure to perform ones duties for a period of time.
Buguats repeated acts of absences without leave and her frequent tardiness reflect her
indifferent attitude to and lack of motivation in her work. Her repeated and habitual
infractions, committed despite several warnings, constitute gross misconduct. Habitual
absenteeism without leave constitute gross negligence and is sufficient to justify
termination of an employee.
- Her repeated negligence is not tolerable; neither should it merit the penalty of
suspension only.
- The record of an employee is a relevant consideration in determining the penalty that
should be meted out.
- An employees past misconduct and present behavior must be taken together in
determining the proper imposable penalty. The totality of infractions or the number of
violations committed during the period of employment shall be considered in determining
the penalty to be imposed upon an erring employee. The offenses committed by him
should not be taken singly and separately but in their totality. Fitness for continued
employment cannot be compartmentalized into tight little cubicles of aspects of character,
conduct, and ability separate and independent of each other.
- It is the totality, not the compartmentalization, of such company infractions that Buguat
had consistently committed which justified her dismissal.
- Terminating an employment is one of petitioners prerogatives.
- Management has the prerogative to discipline its employees and to impose appropriate
penalties on erring workers pursuant to company rules and regulations.
- The Court has upheld a companys management prerogatives so long as they are
exercised in good faith for the advancement of the employers interest and not for the

Labor Law 1
purpose of defeating or circumventing the rights of the employees under special laws or
under valid agreements.
- In the case at bar, petitioner exercised in good faith its management prerogative as there
is no dispute that Buguat had been habitually absent, tardy and neglectful of her work, to
the damage and prejudice of the company. Her dismissal was therefore proper.
- The law imposes many obligations on the employer such as providing just compensation
to workers, observance of the procedural requirements of notice and hearing in the
termination of employment. On the other hand, the law also recognizes the right of the
employer to expect from its workers not only good performance, adequate work and
diligence, but also good conduct and loyalty. The employer may not be compelled to
continue to employ such persons whose continuance in the service will patently be
inimical to his interests.
- The employer has the burden of proving that the dismissed worker has been served two
notices: (1) one to apprise him of the particular acts or omissions for which his dismissal is
sought, and (2) the other to inform him of his employers decision to dismiss him.
- A review of the records shows that private respondent was served a written termination
notice on the very day she was actually dismissed from the service. It was not shown that
CSC notified Elvie in advance of the charge or charges against her nor was she given an
opportunity to refute the charges made against her.
- Agabon v. National Labor Relations Commission: Upheld as valid the dismissal for just
cause even if there was no compliance with the requirements of procedural due process.
While the procedural infirmity cannot be cured, it should not invalidate the dismissal.
However, the employer should be held liable for non-compliance with the procedural
requirements of due process.
Disposition CAS DECISION IS AFFIRMED; backwages is DELETED; Nominal damages
(for violation of Buguats statutory due process) in the amount of P30,000.00.

GROSS AND HABITUAL NEGLIGENCE DEFINED


VALIAO V CA
[PAGE 11]
REYES V MAXIMS TEA HOUSE
398 SCRA 288
QUISUMBING; February 27, 2003
NATURE
Peition for review on certiorari of a decision of the Court of Appeals
FACTS
- Respondent Maxim's Tea House (hereinafter Maxim's for brevity) had employed Reyes
as a driver since October 1995. He was assigned to its M.H. del Pilar Street, Ermita,
Manila branch. His working hours were from 5:00 P.M. to 3:00 A.M., and among his duties
was to fetch and bring to their respective homes the employees of Maxim's after the
restaurant closed for the day.
- In the wee hours of the morning of September 27, 1997, petitioner was driving a
Mitsubishi L300 van and was sent to fetch some employees of Savannah Moon, a
ballroom dancing establishment in Libis, Quezon City. Petitioner complied and took his
usual route along Julia Vargas Street in Pasig City. He was headed towards Meralco
Avenue at a cruising speed of 50 to 60 kilometers per hour, when he noticed a ten-wheeler
truck coming his way at full speed despite the fact that the latter's lane had a red signal
light on. Petitioner maneuvered to avoid a collision, but nonetheless the van he was
driving struck the truck. As a result, petitioner and seven of his passengers sustained
physical injuries and both vehicles were damaged
- The management of Maxim's required petitioner to submit, within forty-eight hours, a
written explanation as to what happened that early morning of September 27, 1997. He
complied but his employer found his explanation unsatisfactory and as a result he was
preventively suspended for thirty (30) days. Subsequently, Maxim's terminated petitioner
for cause.
- Feeling that the vehicular accident was neither a just nor a valid cause for the severance
of his employment, petitioner filed a complaint for illegal dismissal docketed as NLRC
NCR Case No. 00-12-08773-97. In his decision, the Labor Arbiter found that petitioner was
grossly negligent in failing to avoid the collision. Instead of filing the requisite pleading for
appeal, petitioner filed a "Motion for Partial Reconsideration" with the NLRC. The NLRC
opted to treat petitioner's motion as an appeal. The NLRC reversed the decision of the
Labor Arbiter on the ground that there was no negligence on petitioner's part.
Respondents moved for reconsideration of the foregoing decision, but said motion was
denied by the Commission in its resolution
- Respondents then filed a special civil action for certiorari with the Court of Appeals, The
appellate court decided in favor of the employer and its manager. Hence, the instant case.
ISSUE

A2010

- 217 -

Disini

WON petitioners dismissal from employment is valid and legal


HELD
NO
- The issue of whether a party is negligent is a question of fact. As a rule, the Supreme
Court is not a trier of facts and this applies with greater force in labor cases. But where the
findings of the NLRC and the Labor Arbiter are contradictory, as in this case, the reviewing
court may delve into the records and examine for itself the questioned findings.
- Under the Labor Code, gross negligence is a valid ground for an employer to terminate
an employee. Gross negligence is negligence characterized by want of even slight care,
acting or omitting to act in a situation where there is a duty to act, not inadvertently but
willfully and intentionally with a conscious indifference to consequences insofar as other
persons may be affected. In this case, however, there is no substantial basis to support a
finding that petitioner committed gross negligence.
- In sustaining the Labor Arbiter's finding that petitioner was grossly negligent, the
appellate court stressed that the cited episode was the second vehicular accident
involving petitioner, and as such it "may clearly reflect against [his] attitudinal character as
a driver." The Court notes, however, that the Commission found that in the first vehicular
accident involving petitioner "he was the victim of the reckless and negligent act of a fellow
driver." An imputation of habitual negligence cannot be drawn against petitioner, since the
earlier accident was not of his own making.
The test to determine the existence of negligence is as follows: Did petitioner in doing the
alleged negligent act use that reasonable care and caution which an ordinarily prudent
person would use in the same situation? It is not disputed that petitioner tried to turn left to
avoid a collision. To put it otherwise, petitioner did not insist on his right of way,
notwithstanding the green light in his lane. Still, the collision took place as the ten-wheeler
careened on the wrong lane. Clearly, petitioner exerted reasonable effort under the
circumstances to avoid injury not only to himself but also to his passengers and the van he
was driving. To hold that petitioner was grossly negligent under the circumstances goes
against the factual circumstances shown. It appears to us he was more a victim of a
vehicular accident rather than its cause.
- There being no clear showing that petitioner was culpable for gross negligence,
petitioner's dismissal is illegal.
Disposition Petition granted.

CEBU FILVENEER CORPORATION V NLRC


[PAGE 194]
CITIBANK NA V GATCHALIAN
240 SCRA 212
PUNO; January 18, 1995
FACTS
- Petitioner bank received thirty-one (31) applications from alleged APBCI employees for
the issuance of Citibank credit cards, popularly known as Mastercard.
- A Citibank employee verified by phone the data which appeared on the application forms.
It was Florence Verendia, as secretary of the APBCI General Manager, who answered the
check calls. The applications were then approved and the corresponding new and
unsigned credit cards were issued. Petitioner bank's policy is for new and unsigned credit
cards to be released only to the cardholders concerned or their duly authorized
representatives. However, a Citibank employee may himself take delivery of new and
unsigned credit cards after accomplishing a Card Pull-Out Request Form wherein the
employee assumes the responsibility of delivering the same to the cardholder concerned.
- Supnad (an employee of bank) and Verendia, conspired together to get the fictitious
cards. They got seven cards from bank employee Llonillo. As a result, the two (Supnad
and Verendia) used the cards in commercial establishments causing injury to the bank in
the amount of 200k.
- the Bank found out about this and conducted an investigation
- Investigation resulted in the decision to terminate Llonilla and to file charges against
Verendia and Supnad
-the labor arbiter ruled that Llonilla be reinstated based on evidence that what Llonilla did
was not gross negligence
ISSUE
WON Llonillas negligence was gross
HELD
YES
- Gross negligence implies a want or absence of or failure to exercise slight care or
diligence, or the entire absence of care. It evinces a thoughtless disregard of
consequences without exerting any effort to avoid them. The evidence on record
succinctly established the gross negligence of respondent Llonillo. She admitted that the

Labor Law 1
first time she was asked by Verendia to pick up one of the newly approved and unused
credit cards, she immediately acceded. Yet at that time, she had not personally met nor
previously seen Verendia. When asked how she came to know to whom she would give
the card, respondent Llonillo responded that Verendia described herself over the phone
and that was how she was able to identify Verendia when she first met her. Thus, on the
basis of a mere description over the telephone, respondent Llonillo delivered the credit
cards to Verendia.
- Furthermore, not only is her negligence gross, it was also habitual it being found out that
she picked up the newly approved credit cards on five (5) separate occasions and
delivered the same to Verendia and the latter's messenger. Certainly, these repetitive acts
and omissions bespeak of habituality.
- Company says shes grossly or habitually negligent in the performance of her duties.
The SC said that since she has not been remiss in the performance of her duties in the
past, she cant be charged with habitual negligence. Neither is her negligence gross in
character. Gross negligence implies a want or absence of or failure to exercise slight care
or diligence or the entire absence of care. It evinces a thoughtless disregard of
consequences without exerting any effort to avoid them. She had not the slightest reason
to distrust Kun because he was the GM and appears to have conducted himself well in the
performance of his duties in the past. At most, its error of judgment, not gross negligence.
Disposition NLRC decision affirmed.

CHUA V NLRC (SCHERING-PLOUGH CORP ET AL)


453 SCRA 244
MELO; March 11, 2005
NATURE
Petition for review on certiorari of a decision and resolution of the CA
FACTS
- On June 1, 1995, Dennis Chua was hired as a Professional Medical Representative by
Schering-Plough Corporation (SPC), and thereafter became a regular employee on
December 1, 1995.
- As a Professional Medical Representative, he was tasked to promote SPC and its
products to physicians, hospitals, paramedics, including trade and government outlets in
his assigned territory.
- One of the petitioners duties was to submit a Daily Coverage Report (DCR) every
Monday, or at least to mail the same to the Field Operations Manager. Furthermore, he
was required to have call cards signed by any of the eighty (80) doctors under his
coverage to show that he indeed visited them and handed out promotional items. This
system enabled the SPC to know how many doctors the petitioner had visited in a week
and the number of call cards he was required to submit.
- Respondent Roberto Z. Tada, Field Operations Manager of the corporation for the Bicol
Region, noticed that the petitioner filed his DCRs late, and in batches at that. Specifically,
a batch of DCRs up to January 10, 1997 was filed only on March 13, 1997, while another
batch was filed only on March 18, 1997. The petitioner also failed to submit the DCRs for
the period covering February 10, 1997 to April 7, 1997. Respondent Tada also found
some discrepancies in the DCRs submitted by the petitioner.
- On April 6, 1997, respondent Tada confronted the petitioner regarding the said
discrepancies, to which Tada merely replied, Pagbigyan mo na lang ako, boss. Tulungan
mo na lang ako, boss.
- On April 8, 1997, Tada went to the petitioners residence and confiscated all the
paraphernalia used by the latter for his fieldwork, including the call cards and medicine
samples. The car assigned to the respondent was likewise confiscated.
- On April 9, 1997, the petitioner filed an application for a three-day sick leave, but
indicated therein that he was going on leave only for two (2) days, from April 10 to 11,
1997. However, after the lapse of his applied leave of absence, the petitioner failed to
report for work.
- On April 15, 1997, the petitioner had already filed a complaint for illegal dismissal with
the National Labor Relations Commission (NLRC) against the SPC, Epitacio Titong, Jr. (as
President and General Manager), Danny T. Yu (as Division Manager) and Roberto Z. Tada
(as Field Operations Manager
- On April 16, 1997, the petitioner received a telegram from the SPC instructing him to
report to the office on April 18, 1997 and to see respondent Danny T. Yu who was the
Division Manager. The petitioner, however, failed to comply.
- On April 18, 1997, respondent Tada sent a Memorandum to the petitioner requiring the
latter to explain the late submission of DCRs, insufficiency of the information on the call
cards, etc.
- The same letter informed the petitioner that he was under preventive suspension
effective April 11, 1997 while the case was under investigation.
- On May 8, 1997, while the case for illegal dismissal was pending resolution before the
arbitration branch of the NLRC, the SPC sent another letter to the petitioner, informing him
that his employment was terminated.

A2010

- 218 -

Disini

- On September 30, 1998, Labor Arbiter Ramon Valentin C. Reyes rendered a Decision
declaring the petitioners dismissal from employment as illegal. The Labor Arbiter held
that the SPC failed to establish any ground for the petitioners dismissal and ordered the
SPC to reinstate him.
- SPC appealed the decision of the Labor Arbiter to the NLRC.
- On October 19, 1999, the NLRC issued a Resolution, finding respondent to have validly
dismissed complainant.
- The petitioner filed a motion for reconsideration of the said resolution, but the same was
dismissed.
- The petitioner sought relief from the CA, which affirmed, in toto, the resolution of the
NLRC, and consequently denied the petitioners MFR
ISSUE
WON petitioners dismissal form employment was illegal
HELD
NO
- The petitioners termination from employment was anchored on the following: (a) gross
and habitual neglect; (b) serious misconduct; and (c) willful disobedience to the lawful
orders of the employer. Thus, it all boils down to the filing of the requisite DCRs due every
Monday. As found by both the NLRC and the CA, the petitioner failed to file the DCRs on
time on several occasions, and instead filed them in batches. Furthermore, the petitioner
failed to submit the DCRs for February 10, 1997 to April 7, 1997. Considering that about
ninety percent (90%) of the petitioners work as a medical representative entails fieldwork,
such DCRs were vital to his job; the DCRs were the primary basis upon which the
petitioners employer could track his accomplishments and work progress. Without the
said DCRs, the employer would have no basis to determine if the petitioner was actually
performing his assigned tasks or not.
- In the same light, the petitioner also failed to submit several doctors call cards, and
submitted others which were incomplete; that is, undated although signed by the doctors.
It must be stressed that the said call cards were also vital to the petitioners fieldwork. The
requirement of asking the doctors to affix their signatures in the call cards, the date of the
visit, as well as the samples and promotional items, if any, given to the doctors, enabled
the SPC to verify whether such doctors were indeed visited by the petitioner.
- Gross negligence under Article 282 of the Labor Code, as amended, connotes want of
care in the performance of ones duties, while habitual neglect implies repeated failure to
perform ones duties for a period of time, depending upon the circumstances. Clearly, the
petitioners repeated failure to submit the DCRs on time, as well as the failure to submit
the doctors call cards constitute habitual neglect of duties. Needless to state, the
foregoing clearly indicates that the employer had a just cause in terminating the
petitioners employment.
***But because there was a violation of the petitioners statutory right to two notices prior
to the termination of his employment for a just cause, he is entitled to nominal damages of
P30,000.00, absent sufficient evidence to support an award for actual or moral damages.
(In line with the ruling in Agabon)
Disposition The decision of the Court of Appeals is affirmed with modification that
petitioner is entitled to above stated award for nominal damages..

GENUINO ICE CO INC V MAGPANTAY


[PAGE 206]
PREMIER DEVT BANK V MANTAL
[PAGE 210]

SIMPLE NEGLIGENCE
PAGUIO TRANSPORT CORP V NLRC (MELCHOR)
294 SCRA 657
PANGANIBAN; August 28, 1998
NATURE
Petition for review of NLRC decision
FACTS
- Complainant Wilfredo Melchor was hired by respondent company as a taxi driver under
the "boundary system. He was to drive the taxi unit assigned to him on a 24-hour
schedule per trip every two 2 days, for which he used to earn an average income from
P500 to P700 per trip, exclusive of the P650 boundary and other deductions.
- He was involved in a vehicular accident along Quirino Ave when he accidentally bumped
a car. He was allegedly advised to stop working and have a rest. When reported for work,
he was told that his service was no longer needed.

Labor Law 1
- He then filed complaint for illegal dismissal.
- Paguio maintained that Melchor was not illegally dismissed since there was no employeremployee relationship. (no control, no payment of compensation) Even if EER existed,
complainant's termination arose out of a valid cause since he was already involved in 3
accidents.
- NLRC ruling: there was illegal dismissal
ISSUES
1. WON an employer-employee relationship exists
2. WON dismissal was for a just cause
3. WON Melchor was afforded due process
4. WON doctrine of strained relations applies
HELD
1. YES
Ratio The relationship of taxi owners and taxi drivers is the same as that between jeepney
owners and jeepney drivers under the "boundary system." This relationship is that of
employer-employee and not of lessor-lessee. The fact that the drivers do not receive fixed
wages but get only the excess of that so-called boundary they pay to the owner/operator is
not sufficient to withdraw the relationship between them from that of employer and
employee.
Reasoning
- He was considered an employee because he was engaged to perform activities which
were usually necessary or desirable in the usual trade of the employer.
- This is different from lease of chattels, wherein the lessor loses complete control over the
chattel leased. In the case of jeepney owners/operators and jeepney drivers, the former
exercise supervision and control over the latter.
2. NO
Ratio Employer has the burden of proving that the dismissal of an employee is for a just
cause. The failure of the employer to discharge this burden means that the dismissal is not
justified and that the employee is entitled to reinstatement and backwages.
- Mere involvement in an accident, absent any showing of fault or recklessness on the part
of an employee, is not a valid ground for dismissal.
3. NO
Ratio The twin requirements of notice and hearing are essential elements of due process.
The employer must furnish the worker two written notices: (1) one to apprise him of the
particular acts or omissions for which his dismissal is sought and (2) the other to inform
him of his employer's decision to dismiss him. The essence of due process lies simply in
an opportunity to be heard, and not always and indispensably in an actual hearing.
4. NO
Ratio Strained relations must be demonstrated as a fact.
- The doctrine on "strained relations" cannot be applied indiscriminately since every labor
dispute almost invariably results in "strained relations"; otherwise, reinstatement can never
be possible simply because some hostility is engendered between the parties as a result
of their disagreement.
Reasoning
- Paguios allegation that private respondent was incompetent and reckless in his manner
of driving, which led to his involvement in three vehicular accidents, is not supported by
the records. No evidence was properly submitted by petitioner to prove or give credence
to his assertions.

C. FRAUD WILLFUL BREACH OF TRUST


SANTOS V SAN MIGUEL CORPORATION
399 SCRA 172
SANDOVAL-GUTIERREZ; March 14, 2003
NATURE
Petition for review on certiorari
FACTS
- Petitioner Carmelita Santos was appointed Finance Director of respondent SMCs Beer
Division for Luzon Operations.
On September 15, 1987, SMC issued a Memorandum prohibiting the encashment of
personal checks at respondent's Plants and Sales Offices. Thereafter, SMC noticed that
petitioner encashed her 3 personal checks in various Metro Manila Sales Offices.
SMC commenced an audit investigation. Petitioner received from respondent an interoffice memorandum requiring her to explain in writing why no disciplinary action should be
taken against her in view of her unauthorized encashment of her 3 personal checks at
respondent's sales offices.
- Petitioner admitted that she encashed three personal checks at respondent's sales
offices but claimed that such act was not irregular since all personnel in respondent's Beer

A2010

- 219 -

Disini

Division were allowed to encash their personal checks at any sales office upon clearance
from the region management concerned. She stated that her encashment of personal
checks had prior clearance. She further clarified that only two of the three checks she
encashed were dishonored for insufficiency of funds, but she promptly funded the checks
upon receipt of notice of such dishonor, thereby causing no damage to respondent.
- Meanwhile, the audit results revealed that, aside from petitioner's reported encashment
of 3 personal checks, she had previously encashed 50 personal checks in varying
amounts, which were not endorsed by the Sales Operations Manager or the Region
Finance Officer. Additionally, petitioner encashed 2 other personal checks. After receiving
such report, SMC formed an Investigating Panel to conduct a full-blown investigation.
- The Investigating Panel found the encashment by petitioner of her personal checks with
the region/sales offices as highly irregular transactions to the detriment of the Company. It
recommended that Santos be terminated from employment.
- In a memorandum, SMC adopted the findings of the Investigating Panel and informed
petitioner of her termination from employment for abuse of position as Finance Director,
engaging in highly irregular transactions to the detriment of the company and employer's
loss of trust and confidence.
- The complaint filed by petitioner against SMC for illegal dismissal was dismissed by the
Labor Arbiter for lack of merit. The NLRC reversed the Labor Arbiters decision. Upon an
MR filed by SMC, the NLRC dismissed the complaint filed by Santos. Hence, this
recourse.
ISSUE
WON SMC dismissed the petitioner from employment without just cause
HELD
NO
- Under the Labor Code, a valid dismissal from employment requires that: (1) the
dismissal must be for any of the causes expressed in Article 282 of the Labor Code and
(2) the employee must be given an opportunity to be heard and to defend himself.Article
282(c) of the same Code provides that "willful breach by the employee of the trust reposed
in him by his employer" is a cause for the termination of employment by an employer. This
ground should be duly established. Substantial evidence is sufficient as long as such loss
of confidence is well-founded or if the employer has reasonable ground to believe that the
employee concerned is responsible for the misconduct and her act rendered her unworthy
of the trust and confidence demanded of her position. It must be shown, though, that the
employee concerned holds a position of trust. The betrayal of this trust is the essence of
the offense for which an employee is penalized.
- Petitioner argues that her position as Finance Director of respondent's Beer Division is
not one of trust but one that is merely functional and advisory in nature. She possesses no
administrative control over the plants and region finance officers, including cashiers. She
reports to two superiors. Petitioner's argument is misplaced. As Finance Director, she is in
charge of the custody, handling, care and protection of respondent's funds. The
encashment of her personal checks and her private use of such funds, albeit for short
periods of time, are contrary to the fiduciary nature of her duties.
- Moreover, petitioner has functional control over all the plant and region finance officers,
including cashiers, within the Luzon Operations Area. In fact, she is the highest ranking
managerial employee for the finance section of the Luzon Beer Division Operations.
Obviously, her position is a factor in abetting the encashment of her personal checks.
- Indeed, there is substantial ground for respondent's loss of confidence in petitioner. She
does not deny encashing her personal checks at respondent's sales offices and diverting
for her own private use the latter's resources. The audit investigation accounted for all the
checks she encashed, some of which were dishonored for insufficiency of funds. The
Investigating Panel concluded that petitioner not only encashed her personal checks at
respondent's sales offices, but also used company funds to temporarily satisfy her
insufficient accounts. This Court has held that misappropriation of company funds,
although the shortages had been fully restituted, is a valid ground to terminate the
services of an employee of the company for loss of trust and confidence.
- Petitioner contends that there is a prolonged practice of other payroll personnel,
including persons in managerial levels, who encashed personal checks but remained
unpunished by respondent. She asserts that her administrative superiors even
encouraged her to encash her checks at the nearest sales office since her appearance at
the bank for encashment would entail undue digression from her daily work routine.
- Prolonged practice of encashing personal checks among respondent's payroll personnel
does not excuse or justify petitioner's misdeeds. Her willful and deliberate acts were in
gross violation of respondent's policy against encashment of personal checks of its
personnel, embodied in its Memorandum. She cannot feign ignorance of such
memorandum as she is duty-bound to keep abreast of company policies related to
financial matters within the corporation. Equally unmeritorious are her claims that the acts
complained of are regular, being with the knowledge and consent of her superiors,
Francisco Gomez de Liano and Ben Jarmalala, and that she is being charged because
she resisted the sexual advances of her superior. Suffice it to state that she could have
proved these matters during the investigation had she attended the proceedings.

Labor Law 1
LAKPUE DRUG INC V BELGA
[PAGE 208]

LOSS OF CONFIDENCE REQUISITES


JARDINE DAVIES INC V NLRC (REYES)
311 SCRA 289
QUISUMBING; July 28, 1999
FACTS
- Petitioner, the exclusive distributor of Union 76 lubricating oil, engaged the services of
a private investigation agency due to reports that petitioners products, particularly Union
76, were being illegally manufactured, blended, packed and distributed. Upon confirmation
of the investigator through a surveillance report and having secured a search warrant,
petitioner seized some of the fake items found in the apartment of private respondent, a
former sales representative of petitioner.
- a criminal complaint for unfair competition violating Article 189 of the RPC (repealed by
Section 239 of the Intellectual Property Code) was filed against Reyes along with
administrative charges for serious misconduct inimical to the interest of petitioner. He was
advised to go on an indefinite leave which later led to his termination.
- the materials seized were released in view of a petition filed by Reyes younger brother
Donato, who convinced the court that the materials belonged to him and that he was
legally engaged in the business of general merchandising (Lubrix Conglomerate) reselling
oil and lubricant products to the public.
- with that, Reyes sued petitioner for illegal dismissal but the complaint was dismissed by
the Labor Arbiter as he thought otherwise. Upon appeal with the NLRC, the decision was
reversed on the ground that there was no cogent reason for petitioner to lose trust and
confidence in private respondent, there being no shadow of an act amounting to serious
misconduct, fraud or breach of trust.
- petitioners MFR was denied, hence this petition.
ISSUE
WON there was reason for petitioner to lose trust and confidence in private respondent
and justify his dismissal
HELD
NO
Ratio The right of an employer to dismiss employees on account of loss of trust of
confidence must not be exercised arbitrarily and without showing just cause, so as not to
render the employees constitutional right to security of tenure nugatory.
Reasoning
- Article 282 provides that an employer may terminate an employment for fraud or willful
breach by the employee of the trust reposed in him by his employer. It is settled that loss
of confidence as a just cause for termination must be premised on the fact that an
employee concerned holds a position of trust and confidence, as in this case. And in order
to constitute just cause, the act complained of must be work-related. Proof beyond
reasonable doubt is not required, so long as there is some basis for the loss of confidence,
but basis thereof must still be clearly and convincingly established, arising from particular
proven facts which the employer bears to prove.
- in the instant case, the surveillance report of the private investigator was unreliable as
the conclusions therein were mere deductions not supported by substantial corroborating
evidence. Petitioner also failed to controvert proof presented by private respondent that
the reselling of the oil was in support of petitioners marketing policy. It was also odd that
petitioners agents did not submit the alleged fake merchandise to be tested in their labs,
virtually affirming the articles were genuine, having been purchased from petitioners
dealers.
- another confirmation that petitioner lacked basis for its distrust of private respondent was
the release of the seized articles, with Donato even presenting receipts to prove they were
purchased from authorized dealers.
- Considering this, private respondent was illegally dismissed. As such, he is entitled to
backwages. Since he was terminated before the effectivity of RA 6715, he is entitled to
only 3 years of backwages, and not full backwages as would be granted now. Because the
antagonism and imputations of the criminal act strained the parties relationship,
reinstatement would not be feasible. Instead, a more equitable disposition would be an
award of separation pay.
Disposition instant petition is DENIED for lack of merit

PLDT V TOLENTINO
[PAGE 202]
DELA CRUZ V NLRC

A2010

Disini

- 220 [PAGE 100]

PHILIPPINE NATIONAL CONSTRUCTION


CORPORATION V MATIAS
458 SCRA 148
PANGANIBAN; May 6, 2005
FACTS
- Rolando Matias was employed by Construction and Development Corporation of the
Philippines (CDCP) as Chief Accountant and Administrative Officer. During his
employment with the company, various parcels of land situated at Don Carlos Bukidnon
were placed in the names of certain employees as trustees for the purpose of owning vast
tracts of land more than the limit a corporation can own which were primarily intended for
CDCP agricultural businesses. By internal arrangement documents transferring back the
properties to the corporation were executed. A piece of land was registered in the name of
Matias.
- CDCP was later converted a government owned or controlled corporation, and the name
of CDCP was changed to Philippine National Construction Corporation (PNCC). Under a
new set up, PNCC offered a retrenchment program and on December 31, 1984 Matias
availed of the said program.
- Sometime in 1985, the Conjuangco Farms owned by Mr. Danding Conjuangco acquired
CDCP Farms Corporation wh[ich] took over the operations of said farms. Not long after,
or in 1989, CDCP Farms Corporation ceased to operate.
- In July 1992, two former CDCP employees, namely Reynaldo Tac-an and Luciano
Tadena went to the house of Matias and brought with them duly accomplished documents
and Special Power of Attorney for his signature and informed him that the lands in
Bukidnon under his name with all the others were invaded by squatters, and that the said
land were covered by the Comprehensive Agrarian Reform Program (CARP) where
Matias name was included in the list of landowners. Matias reluctantly signed the
document and after six months, he signed an acknowledgment receipt of P100,000.00.
- The original title registered in the name of Matias was cancelled and a new title was
issued. The transfer of said parcel of land was made possible because Rolando Matias
and Elena Esmeralda Matias received managers checks from the Land Bank of the
Philippines in the amount of P102,355.96 and P219.22 and bond worth P203,478.48 as
payment of Land Transfer Acquisition.
- On August 12, 1996, Matias was rehired by PNCC as Project Controller in Zambales
PMMA Project.
- Not long after, Mr.Alday, Head of the Realty Management Group of PNCC invited Matias
to his office and showed him a listing of parcels of land in the name of different persons
with the corresponding status including the latters name. On the basis of the listing, Mr.
Alday told Matias that the transfer of the property registered in the latters name was not
yet consummated by the LBP and then requested Matias to execute a Deed of
Assignment in favor of PNCC pertaining to the said property, which Matias did and
guaranteed in writing that the parcel of land is free from any lien or encumbrance.
- On April 20, 1998, a memorandum was issued to Matias by PNCC directing the former
to explain in writing why none of the following actions, falsification, estafa, dishonesty, and
breach of trust and confidence, should be taken against him in connection with the Deed
of Assignment. PNCC alleges that respondent fraudulently breached its trust and
confidence when, without its knowledge and consent, he disposed of the Bukidnon
property; though actually belonging to petitioner, that property had purportedly been
merely placed in trust under his name. Thereafter, he assigned the same property to
petitioner, allegedly despite his full knowledge that the title had already been transferred -with his active planning and participation -- to the Republic of the Philippines .
- In due time, Matias submitted his written explanation. However, he was later advised
that he was terminated from the service on the ground of loss of trust and confidence.
Hence, Matias filed a complaint for illegal dismissal and money claims against PNCC
alleging that the dismissal on the ground of loss of trust and confidence was without
basis.
ISSUE
WON the dismissal of Matias on the ground of loss of trust and confidence was without
basis
HELD
YES
Ratio: To constitute a valid cause to terminate employment, loss of trust and confidence
must be proven clearly and convincingly by substantial evidence. To be a just cause for
terminating employment, loss of confidence must be directly to the duties of the
employee to show that he or she is woefully unfit to continue working for the employer.
Reasoning
- Undeniably, the position of project controller -- the position of respondent at the time of
his dismissal -- required trust and confidence, for it related to the handling of business

Labor Law 1

A2010

Disini

- 221 -

expenditures or finances. However, his act allegedly constituting breach of trust and
confidence was not in any way related to his official functions and responsibilities as
controller. In fact, the questioned act pertained to an unlawful scheme deliberately
engaged in by petitioner in order to evade a constitutional and legal mandate.
- It has oft been held that loss of confidence should not be used as a subterfuge for
causes which are illegal, improper and unjustified. It must be genuine, not a mere
afterthought to justify an earlier action taken in bad faith. Be it remembered that at stake
here are the sole means of livelihood, the name and the reputation of the employee.
Thus, petitioner must prove an actual breach of duty founded on clearly established facts
sufficient to warrant his loss of employment.
- We stress once more that the right of an employer to dismiss an employee on account of
loss of trust and confidence must not be exercised whimsically. To countenance an
arbitrary exercise of that prerogative is to negate the employees constitutional right to
security of tenure. In other words, the employer must clearly and convincingly prove by
substantial evidence the facts and incidents upon which loss of confidence in the
employee may be fairly made to rest; otherwise, the latters dismissal will be rendered
illegal.

employer has reasonable ground to believe that the employee concerned id responsible
for the purported misconduct, and the nature of his participation renders him unworthy of
the trust and confidence demanded by his position.

CRUZ V CA (NLRC, CITYTRUST BANK)


494 SCRA 226
AUSTRIA-MARTINEZ; July 12, 2006

against a claim that the dismissal of an employee was

- Art 282 ( c) LC states that the loss of trust and


confidence must be based on willful breach. It should
be done intentionally, knowingly and purposely
without

justifiable

excuse.

It

must

not

be

indiscriminately used as a shield by the employer

arbitrary. And, in order to constitute a just cause for

NATURE
Special civil action for certiorari under Rule 65 PROC seeking to annul CA decision
affirming NLRC decision and resolution .

dismissal, the act complained of must be work-related

FACTS

and shows that the employee concerned is unfit to

- Felix Cruz was an employee of Ciytrust Banking Corporation. He held a


confidential position of Micro Technical Support Officer, whose
duties include: evaluating and recommending requests for Micro
Computers received by the bidding committee, further evaluating
and accepting of bids done by the Technical Commitee. He was
recognized with awards and citations due to his good
performance.
- There were feedbacks and informations that there were irregularities in the bidding
process and purchase of the computers. A special investigation was conducted which
found that there were unauthorized and unreported commissions and rebated given out by
one of its computer suppliers (MECO) for purchases made by Citytrust.
- Citytrust sent a show cause memorandm to Cruz placing him under preventive
suspension and directing him to appear in an administrative hearing by the Ad Hoc
Committee. The committee found him guilty of fraud, serious misconduct, gross
dishonesty and serious violation of the bank policies. For the resultant loss of confidence,
Citytrust terminated Cruz from employment.
- Cruz filed before the Labor Arbiter an action for illegal dismissal and damages for being
denied due process and hastily dismissed. LA decision favored Cruz.
- Citytrust appealed to the NLRC, setting aside LA decision and dismissing the case fro
lack of merit. Cruz filed MFR, but was denied for lack of merit.
- Cruz filed petition for Certiorari with SC, which was referred to the CA for appropriate
action and disposition.
- CA dismissed the petition. It held that although the signature of the petitioner does not
appear in the check vouchers, other pieces of evidence prove that he benefited from the
proceeds of the checks issued and that there is substantial evidence to hold the petitioner
liable for soliciting; that his acts constituted a willful breach of the suppliers trust and
confidence; that the dismissal was the result of a thorough investigation and hearing.

ISSUES
1. WON CA committed grave abuse of discretion
2. WON he denied due process
HELD
1. NO
- Petitioner failed to prove such.
- Petitioner was dismissed on the ground, among others, of loss of trust and confidence.
Loss of trust and confidence, as a valid ground for dismissal, must be substantiated by
evidence.
- WRT to rank-and-file personnel, loss of trust and confidence requires proof of
involvement in the alleged events in question. But as regards a managerial employee, the
mere existence of a basis fro believing that such employee has breached the trust of his
employer would suffice for his dismissal. Proof beyond reasonable doubt is not required, it
being sufficient that there is some basis for such loss of confidence such when the

continue working for the employer. In addition, loss of


confidence is premised on the fact that the employee
concerned holds a position of responsibility, trust and
confidence or that the employee concerned is
entrusted with confidence with respect to delicate
matters. The betrayal of this trust is the essence of the
offense for which an employee is penalized
- Cruzs job entails the observance of proper company procedures. His functions are also
extended to all branches nationwide, involving high degree of responsibility requiring a
substantial amount of trust and confidence.
- Petitioners acceptance of commissions and rebates from MECO, without knowledge and
consent from Citytrust, and without said rebates being reported and turned over to the
latter, are acts which can be considered willful breach of the trust and confidence reposed
by Citytrust on him.
- An employer cannot be compelled to retain an employee who s guilty of acts inimical to
the interests of the employer.
2. NO, he was not denied due process.
- The basic requirement of notice and hearing in termination cases is for the employer to
inform the employee of the specific charges against him and to hear his side and
defenses. This does not mean a full adversarial proceeding. The parties may be heard
through pleadings, written explanations, position papers, memorandum or oral argument.
In all of these instances, the employer plays an active role by providing the employee with
the opportunity to present his side and answer the charges in substantial compliance with
due process.
- The fact alone that he was not able to confront the witnesses against him during the
investigation conducted by Citytrust does not mean that he was denied his right to due
process. What is frowned upon is the absolute lack of notice and hearing.
- Citytrust complied with the first requirement of notice when it informed petitioner through
a letter of the charges against him, directing him to explain in writing why his employment
should not be terminated and to appear in a hearing to be conducted by the company to

Labor Law 1
give him further opportunity to explain his side. Citytrust also complied with the second
requirement of notice when it sent a memorandum informing him of his dismissal from
employment and the reasons therefore.
Dispositon instant petition is DISMISSED for lack of merit.

BREACH OF TRUST LOSS OF CONFIDENCE


CENTRAL PANGASINAN ELEC CORP V MACARAEG
[PAGE 195]

POSITION, TRUST AND CONFIDENCE


SANTOS V SAN MIGUEL CORP
[PAGE 219]
PANDAY V NLRC (LUZON MAHOGANY TIMBER
INDUSTRIES INC)
209 SCRA 122
GUTIERREZ; May 20, 1992
NATURE
Petition seeking the review of the order rendered by the NLRC authorizing the separation
from the service of Panday to the payment by the private respondent of separation pay
equivalent to one-half month salary for every year of service. It likewise ordered the
payment of the complainant's 13th month pay for 1977 but dismissed his claim for living
allowance for lack of merit.
FACTS
- Panday was hired by Luzon Mahogany Timber Industries since Aug. 23, 1973.
Sometime in Dec. 1977, Panday was called by Martin Gaw, the owner-manager who
instructed him to cut off the living allowance of the employees. Panday requested that a
memorandum to this effect be made so that he would not be blamed by the workers. Gaw
got angry and shouted "what for is the memorandum? I am telling you to do so." He then
butted, "Ano ba talaga Naning ang ibig mong sabihin? Sa tuwing magsasalita ka, panay
ka "policy" ng companya at panay ka records".
- From the time of that incident, Panday was deprived of free light. He was no longer
given any accounting work. His per diem was abruptly cut off. All that was left for him to do
was the simple clerical job of registering or paying SSS premiums. Still complainant
continued to bear it out. In 1979, however he was totally divested of all his duties and he
was compelled to approach Manager Martin Gaw to clear up matters. Gaw referred him to
Mr. Gerry Lumban who was supposed to give him some work to do. It turned out, however,
that no such instructions were given to said Mr. Lumban.
- In 1979, Panday filed a request for vacation leave with pay for 15 days from April 14-30,
1979. On that same day he brought his son to Manila for medical treatment and stayed
there up to the end of the month. Upon his return to Isabela, he asked for his salaries only
to learn that his application for leave was disapproved.
- Hence, he filed this case for illegal dismissal, non-payment of 13th month pay for 1977,
emergency allowance under P.D. 525 since 1975 up to 1977 and unpaid wages for April
16-30, 1979.
- The Office of the Minister found and ruled that Panday was constructively dismissed from
the service. Luzon Timber was ordered to comment on the petition but failed to so. Thus,
the actual findings are affirmed. The only complaint of Panday which remains is his
claim that Deputy Minister Vicente Leogardo, Jr. should have ordered his
reinstatement with backwages.
ISSUE
WON Pandays prayer for reinstatement should have been granted
HELD
NO
- Panday, as branch accountant occupied a position involving trust and confidence and in
the light of the estranged relation between the complainant and the respondent that may
not permit the full restoration of an employment relationship based on trust and
confidence, we have to allow termination of the employer-employee relationship but upon
the payment of separation pay equivalent to one-half (1/2) month for every year of service
rendered.
- The case of Lepanto Consolidated Mining Co. v. Court of Appeals provides a definition of
a "position of trust and confidence". It is one where a person is "entrusted with

A2010

- 222 -

Disini

confidence on delicate matters, or with the custody, handling, or care and


protection of the employer's property"
- A few examples were given by the Court in the case of Globe-Mackay Cable and Radio
Corporation v. National Labor Relations Commission and Imelda Salazar, G.R. No. 82511,
March 3, 1992, to illustrate the principle:
- where the employee is a Vice-President for Marketing and as such, enjoys the full
trust and confidence of top management
- or is the Officer-In-Charge of the extension office of the bank where he works
- or is an organizer of a union who was in a position to sabotage the union's efforts to
organize the workers in commercial and industrial establishments
- or is warehouseman of a non-profit organization whose primary purpose is to facilitate
and maximize voluntary gifts by foreign individuals and organizations to the Philippines
- or is a manager of its Energy Equipment Sales
- Credit and Collection Supervisor (Tabacalera Insurance Co. v. National Labor
Relations Commission)
- If the respondent had been a laborer, clerk or other rank-and-file employee, there would
be no problem in ordering her reinstatement with facility. An officer in such a key position
as Vice President for Marketing(or as Chief Accountant as in the present case) can work
effectively only if she enjoys the full trust and confidence of top management.
- The case of Metro Drug Corp. v. National, Labor Relations Commission, aptly describes
the difference in treatment between the positions of trust on one hand and mere clerical
positions on the other. It states:
Managerial personnel and other employees occupying positions of trust and confidence
are entitled to security of tenure, fair standards of employment, and the protection of
labor laws. However, the rules on termination of employment, penalties for infractions,
and resort to concerted action are not necessarily the same as those for ordinary
employees.
A special and unique employment relationship exists between a corporation and its
cashiers. More than most key positions, that of cashier calls f or the utmost trust and
confidence. . . .
When an employee accepts a promotion to a managerial position or to an office
requiring full trust and confidence she gives up some of the rigid guaranties available to
ordinary workers. Infractions which if committed by others would be overlooked or
condoned or penalties mitigated may be visited with more severe disciplinary action. A
company's resort to acts of self-defense would be more easily justified. It would be
most unfair to require an employer to continue employing as its cashier a person whom
it reasonably believes is no longer capable of giving full and wholehearted
trustworthiness in the stewardship of company funds.
- Reinstatement in the present case is no longer possible not only because of the strained
relationship between the employee and the employer but also because of the length of
time that has passed from the date the incident occurred to its resolution. Instead of
reinstating the employee, this Court has in several cases awarded separation pay
although the employee was found to be illegally dismissed.
The following reasons have been advanced by the Court for denying reinstatement
- reinstatement can no longer be effected in view of the long passage of time
- because of the realities of the situation
- that it would be inimical to the employer's interest
- that reinstatement may no longer be feasible
- that it will not serve the best interests of the parties involved
- that the company would be prejudiced and by the workers' continued employment
- that it will not serve any prudent purpose as when supervening facts have transpired
which make execution on that score unjust or inequitable
Disposition the prayer for reinstatement is DENIED but the order rendered by Deputy
Minister Vicente Leogardo, Jr. dated May 29, 1984 is modified to cover five (5) years
backwages. The order is AFFIRMED in other respects.

CRUZ V COCA-COLA BOTTLERS PHILS INC


460 SCRA 340
YNARES-SANTIAGO; June 15, 2005
FACTS
- Cruz has been working for respondent companys plant in Calamba, Laguna, as a
driver/helper since June 1983. At times, however, Cruz gets designated as Acting
Salesman for companys soft drinks and other beverages. On July 25, 1998, petitioner
was assigned as acting salesman of Route DA1, covering the small barangays. Together
with his helper, Mr. Pablito Aguila, Cruz loaded their truck with CCBPI products. After the
required verification and confirmation of the products loaded on the truck by the Checker
and the guard at the gate, Cruz proceeded to leave the plant vicinity.
- After gate inspection, however, Cruz drove back inside the plant on the pretext of
refueling. While waiting in line to refuel, Cruz allegedly asked Aguila to load an additional

Labor Law 1
thirty cases of assorted canned soft drinks as plus load. Aguila reminded him about the
required documents but he merely stated Ayos na and continued with the refueling of the
truck.
- On his second exit from the plant premises, Cruz did not slow down for the mandatory
inspection even as the security guards at the gate flagged him down.
- One of the guards pursued the truck and when he caught up with petitioner at the Walter
Mart Shopping Mall in Barangay Real, Calamba, the latter could not produce the proper
documents for the extra thirty cases loaded on his truck. He was then directed to return to
the plant and unload the products. At this point, it was confirmed that Cruz did not actually
secure any paper for the added products nor did he follow the established procedure
before taking out the extra cases.
- Cruz admitted the incident but alleged that he forgot to secure the requisite documents
for the products. On August 5, 1998, an investigation was conducted on the alleged
violations committed by petitioner. On August 19, 1998, respondent company terminated
the services of petitioner effective upon receipt of the memorandum.
ISSUE
WON Cruz was validly dismissed
HELD
YES
- The Labor Arbiter, the NLRC and the Court of Appeals were unanimous in their findings
that petitioner was guilty of dishonest acts but differed only on the propriety of the penalty
imposed upon petitioner.
- After a careful evaluation of the evidence on record of this case, we found no compelling
reason to disturb the unanimous findings of the Court of Appeals, the NLRC and the Labor
Arbiter.
- Several factors militate against petitioners claim of good faith. Petitioners length of
service, which spans almost fifteen years, works against his favor in this case. We have
held that the longer an employee stays in the service of the company, the greater is his
responsibility for knowledge and compliance with the norms of conduct and the code of
discipline in the company. Considering that petitioner has worked at respondent company
for a long period of time, one expects that securing the LOGP or TGP would be automatic
for him.
- Faced with the overwhelming evidence presented by respondents on one hand and the
mere general denial of petitioner on the other, the invocation of the protective mantle of
the law in favor of labor cannot be upheld in this case. This principle cannot be adopted
where there is clear and convincing evidence of the truth. While this court endeavors to
live up to its mandate that the workingmans welfare should be the primordial and
paramount consideration, it cannot do so if it will be at the expense of justice and will
result in the oppression or self-destruction of the employer. The interests of both the
employers and employees are intended to be protected and not one of them is given
undue preference.
- Termination of employment by reason of loss of confidence is governed by Article 282(c)
of the Labor Code, which provides that an employer can terminate the employment of the
employee concerned for fraud or willful breach by an employee of the trust reposed in him
by his employer or duly authorized representative. Loss of confidence, as a just cause for
termination of employment, is premised on the fact that the employee concerned holds a
position of responsibility, trust and confidence. He must be invested with confidence on
delicate matters such as the custody, handling, care and protection of the employers
property and/or funds.
- Admittedly, the company rules violated by Cruz are punishable, for the first offense, with
the penalty of suspension. However, company has presented evidence showing that Cruz
has a record of other violations from as far back as 1986. To be sure, the nature of
petitioners offenses is downright inimical to the interests of respondent company. By
virtue of his job, Cruz is entrusted with the property and funds, which belong to respondent
company. His actions on that fateful day highlight, not only his consistent and deliberate
defiance of company rules and regulation, but also his duplicity in handling respondent
companys properties. It would appear that company had tolerated his work ethic far too
long. We therefore find that it was justified in terminating petitioner after the flagrant
dishonesty he committed.
Disposition Instant petition is DENIED. Dismissal of petitioner is declared valid but
respondent company is ORDERED to pay petitioner the amount of P20,000.00 as nominal
damages for non-compliance with statutory due process.

GUIDELINES
VITARICH CORP V NLRC (RECODO)
307 SCRA 509
BELLOSILLO; May 20, 1999
NATURE

A2010

- 223 -

Disini

Special Civil action in the SC. Certiorari


FACTS
- Private respondent, Isagani Recodo, started working at Vitarich as an Accounting clerk.
He gradually moved up the organization ladder until he was made Sales Manager for
Western Visayas in 1988. He was dismissed in October 15, 1992 for alleged violation of a
memorandum dated August 4, 1992 and also for violation of company policies relating to
credit extensions and cash advances. He was also terminated for loss of trust and
confidence.
- Apparently, his new boss, Onofre Sebastian, was under pressure from senior
management to address and correct all the problems he had inherited from his
predecessor. The problems included high account receivable level in the sales territory of
Recodo. The two had a meeting sometime middle of July to address the problems,
including the A/R level of one Rex Cordova.
- The August 4 Memorandum referred to contains instructions to Recodo to ground
salesmen with thirty say overdue A/R so that the levels of said A/R can be regularized.
Apparently, Recodo received the said memo garbled and had to verify its contents on
September 5, 1992. In the meantime, he postponed the grounding of Cordova until August
20 to bring about the desired reduction. The reduction hoped for in fact happened when
Cordovas A/R went down from Pesos 800,000 to just Pesos 250,000. Huffing and puffing,
Sebastian was asked to explain why he should not be terminated for failure to ground
Cordova in compliance with the August 4 memo of Sebastian.
- Recodo complied with the order to explain and an investigation was conducted by the
Head of Personnel, a certain Enriquez. In his report, Enriquez found that there was no
defensible ground for terminating Recodos services in the absence of documented
warnings given to Recodo to justify any loss of trust and confidence in him. Nonetheless,
Recodo was terminated on October 15, 1992.
- Private respondent filed a complaint for illegal termination, non-payment of managerial
bonus, and for moral and exemplary damages. The Labor Arbiter ruled illegal dismissal.
The NLRC initially overturned the ruling but on appeal by Recodo, the finding of the Labor
Arbiter was upheld. Hence this action.
ISSUE
WON the NLRC committed a grave abuse of discretion in finding in favor of Recode
HELD
NO
. In rectifying its previous appreciation and assessment of Recodos dismissal, the NLRC
did not commit any abuse of discretion. A careful scrutiny of the records reveal that the
decision of the Labor Arbiter is suffused with established facts and a correct understanding
of them.
Reasoning
- While it may be true that there was a delay on the part of Recodo in implementing his
superiors order with regard Cordovas grounding, the question is whether the delay
constitutes disobedience and whether this disobedience was willful to merit loss of
confidence. The SC, in AHS Philippines, Inc. vs. CA, explained that willful disobedience
of the employers lawful orders, as a just cause for dismissal of an employee, envisages
the concurrence of at least two requisites:
a. the employees assailed conduct must be willful or
intentional, the willfulness
being characterized by a wrongful and perverse attitude;
b. the order violated must have been reasonable, lawful, made known to the
employee and must pertain to the duties
which he had been engaged to
discharge.
- In the case at bar, the non-compliance by Recodo was not an open defiance but as one
of the discretions which he had to take under the circumstances in his capacity as sales
manager. As it turned out, the result both Recodo and Sebastian hoped for was achieved
by not immediately grounding Cordova.
- While an employer is allowed wide latitude to dismiss employees on loss of trust and
confidence, still the loss thereof must have some basis and must be proved by the
employer otherwise the social justice policy of the labor lawsand the constitution will be for
naught. The guidelines for the application of the doctrine of loss of confidence are:
a. loss of confidence should not be simulated
b. it dhould not be used as subterfuge for causes which are improper, illegal, or
unjustified
c. it should not be arbitrarily asserted in the face of
overwhelming evidence to
the contrary
d. it must be genuine, not a mere afterthought to justify
earlier action taken in bad
faith
Disposition the resolution f the NLRC is affirmed with the modification that corresponding
back wages of respondent be forthwith updated and released to him.

COCA-COLA BOTTLERS PHIL INC V KAPISANAN NG


MALAYANG MANGGAGAWA SA COCA-COLA
[PAGE 209]

Labor Law 1
WILLFUL BREACH
ATLAS CONSOLIDATED MINING & DEVELOPMENT
CORP V NLRC (VILLACENCIO)
290 SCRA 479
PUNO; May 21, 1998
NATURE
petition for certiorari under Rule 65 of the Revised Rules of Court of Decision dated
December 27, 1994 of NLRC which ordered the payment of separation pay and
backwages to private respondent Isabelo O. Villacencio, and its Resolution dated August
18, 1995 denying petitioner's Motion for Reconsideration.
FACTS
- private respondent Isabelo O. Villacencio worked with petitioner ACMDC from January
23, 1970 to February 2, 1990. He started as an ordinary laborer/helper in the Mill
Department. In 1973, he became supervisor of the Tailings Disposal Department. In 1982,
he was elevated as a junior staff of the department. Finally, he was promoted general
foreman of the Tailings Disposal and Water Supply Department with a monthly salary of
P7,440.00. He held this position until his services were terminated on February 2, 1990.
- As general foreman, Villacencio was the second-to-the-highest man in the department
which has a field office located in Magdugo, Toledo City. Under Villacencio were some fifty
nine (59) workers whom he supervised through regular field inspections. Villacencio was
assigned a service jeep and a service motorcycle which he used alternately. He was given
the privilege to withdraw the necessary fuel/gasoline for the vehicles at the Transport
Department located inside the main compound of ACMDC.
- September 8, 1989 - Engineer Sanchez of the Services Division wrote a memorandum
requesting that Villacencio be investigated for alleged anomalies at the Magdugo Tailings
Field Office. Villacencio was charged before the Special Investigation Board with acts of
malfeasance consisting of:
1. withdrawal of company-owned gasoline for the refueling of his personal jeep;
2. use of company personnel on company time as well as company-owned materials
for the assembly of a jeep not belonging to the company; and
3. granting of authority to non-company personnel to withdraw company-owned
stocks.
- January 1990 - He was summoned and investigations were conducted. the Special
Investigation Board found Villacencio guilty of the charge of withdrawing on various dates
a total of 192 liters of company-owned gasoline which he used to refuel his private jeep
and of the charge of using company personnel on company time in the assembly of his
jeep. The third charge was dismissed for insufficiency of evidence. Villacencio was
dismissed from work on February 2, 1990.
- February 19, 1990 - Villavicencio lodged a complaint against ACMDC before the
Regional Arbitration Cebu City, for illegal dismissal with prayer for reinstatement and
backwages plus damages. The case was assigned to Labor Arbiter Reynoso A.
Belarmino.
- Meanwhile, ACMDC initiated a criminal complaint against Villacencio for the
misappropriation of 192 liters of gasoline amounting to P1,086.72. An Information for
Estafa was filed against Villacencio before the Municipal Trial Court of Toledo City. After
trial, he was found guilty and sentenced to prision correccional as maximum, and to pay
ACMDC the amount of P1,086.72 for the misappropriated gasoline.
- Villacencio appealed his conviction to RTC Toledo City. For failure of the prosecution to
establish the guilt of Villacencio beyond reasonable doubt, the appellate court acquitted
him
- August 9, 1993 - Labor Arbiter Belarmino rendered a Decision dismissing Villacencio's
complaint of illegal dismissal for lack of merit.
- December 27, 1994 - NLRC reversed the Labor Arbiter's decision.
- Both parties filed their respective Motion for Reconsideration. ACMDC's motion assailed
the public respondent's decision for allegedly misapprehending the Labor Arbiter's
decision. On the other hand, Villacencio's motion prayed for reinstatement and award of
backwages in addition to separation pay.
- August 18, 1995 NLRC rendered a Resolution granting Villacencio's prayer for
backwages and denying ACMDC's motion.
ISSUES
1. WON NLRC acted with grave abuse of discretion amounting to lack of jurisdiction in
reversing the Decision of the Labor Arbiter and holding Villavicencios dismissal illegal
2. WON there is willful breach of trust
HELD
1. NO
- In illegal dismissal cases, the employer bears the burden of proof to show that the
dismissal is for a just or authorized cause. The charges against private respondent are: (1)

A2010

- 224 -

Disini

withdrawal of 192 liters of gasoline from company stocks for his private use; and (2)
knowingly allowing company personnel to work on company time in the assembly of a
privately-owned jeep. To prove the first charge, petitioner presented the Tenders Logbook
showing the unsigned entries of gasoline withdrawals allegedly made by Villavicencio .
Wilfredo Caba and Bienvenido Villacencio also testified that Villavicencio refused to sign
the entries when requested to do so.
(1) The evidence for the Villavicencio shows that during his more than twenty (20)-year
stint with petitioner, he received several awards and commendations for his contribution in
the areas of production, services and smooth operation of his department. The
management recognized his ability in handling his subordinates and in protecting
company assets in relation to his assigned duties. As a stickler for company rules, he
never held back on issuing warnings, admonitions and even suspensions against erring
subordinates. Consequently, he earned the ire of some of his subordinates. Among them
were Wilfredo Caba., June Climaco, Felix Gonzales and Bienvenido Villacencio. In sum,
Villavicencios position is that the logbook entries do not prove that he received the 192
liters of gasoline since his signature does not appear therein and that the witnesses
presented by the petitioner to explain the absence of his signature in the logbook entries
were motivated by vengeance since he offended their feelings when he disciplined them
and denied their requests for promotion.
- The Standard Guidelines of ACMDC require that all withdrawals of consumable items
and the borrowing of company materials and equipments should be recorded in the
Tender's Logbook by the tender on duty and should be signed by the withdrawing party.
The tender on duty is also required to immediately report to his supervisor any
discrepancy, error or irregularity. Needless to stress, the best evidence of any withdrawal
is the Tender's Logbook. In the case at bar, the gasoline withdrawal entries were made by
tenders Caba and Villacencio. Villavicencios signature does not appear in the logbook,
thus, there is no proof that he actually withdrew and received the gasoline.
(2) The Authorization to Work Overtime dated May 14, 1989 indicates that A. Saavedra, A.
Sepada and V. Rago were among those authorized to work overtime 'to assist in
emergency repair of busted 280 CIC Tailings Line' on that date. The same does not show
or affirm petitioner's contention that said workers were not actually authorized or did not
actually perform the required work but were at the Magdugo Field Office working on
private respondent's personal jeep. On the contrary, the Authorization to Work Overtime
appears regular on its face, as in fact, the same bears the imprimatur indicated by the
signature not only of private respondent alone but of three (3) other officers: the
Supervisor, J.V. Climaco, Jr., the Department Head, J. N. Tecson, and the Division
Manager, C. N. Sanchez. If ever there was an irregularity, these officers would likewise
have to be answerable to the company, instead of letting private respondent bear the
burden alone.
2. NO
- We reject the ruling of the Labor Arbiter that since Villavicencio neglected to inspect the
logbook and thus failed to discover the irregularity, he committed breach of trust.
Ratio Settled is the rule that under Article 283(c) of the Labor Code, the breach of trust
must be willful. A breach is willful if it is done intentionally, knowingly and purposely,
without justifiable excuse, as distinguished from an act done carelessly, thoughtlessly,
heedlessly or inadvertently. It must rest on substantial grounds and not on the employer's
arbitrariness, whims, caprices or suspicion; otherwise, the employee would eternally
remain at the mercy of the employer. It should be genuine and not simulated; nor should it
appear as a mere afterthought to justify earlier action taken in bad faith or a subterfuge for
causes which are improper, illegal or unjustified. It has never been intended to afford an
occasion for abuse because of its subjective nature. Private respondent explained that he
failed to inspect the logbook for about two (2) months before its disappearance because
he was preoccupied with some emergency works brought about by a storm. With the
foregoing explanation, it cannot be said that Villavicencios failure was willful.
Disposition the assailed Decision and Resolution of public respondent NLRC are
AFFIRMED.

COVERAGE
FUJITSU COMPUTER PRODUCTS CORP V CA
[PAGE 204]

PROOF
RAMATEK PHILS V DE LOS REYES
474 SCRA 129
CARPIO; October 25, 2005
NATURE
petition for review resolutions of CA (denying appeals for being filed out of time)

Labor Law 1
FACTS
- Anelia de los Reyes was employed by Ramatek as a comptroller. Subsequently,
Ramatek entered into a sub-contracting agreement with Sicar Micro-Electronics Corp, of
which Anelias husband Nestor was a major stockholder, treasurer, and COO.
- Some time after, Sicar filed a civil action for damages against the Ramatek officials for
the unilateral termination of their contract. Later, the chairman of the board of directors of
Ramatek informed Anelia that she should file a leave of absence while the case was
ongoing. Afterwards, the chairman emailed Anelia, requesting her to tender her voluntary
resignation from the company. The email said in part: IT IS WITH GREAT REGRET THAT
I MUST INFORM YOU OF MY REACTION TO THE SICAR AFFAIR. YOUR
CONNECTION IN THIS MATTER HAS CAUSED ME TO LOSE MY FAITH AND TRUST IN
YOU. IT IS A MAJOR CONFLICT OF INTEREST SITUATION.
- In a letter dated a month later, the company required Anelia to explain within 72 hours
some of her allegedly questionable transactions. Such included awards of work bids to
bidders who did not give the lowest bids, purchase of equipment not at the lowest prices,
and failure to submit company documents despite demand.
- Anelia did not answer (she failed to claim the letter sent by Ramatek through registered
mail) nor did she appear in the administrative investigation. Ramatek, soon after,
terminated Anelias employment for committing anomalies amounting to breach of trust
and confidence. Anelia filed for illegal suspension and illegal dismissal.
LA ruled in favor of Anelia. NLRC affirmed. Appeal to CA (by Ramatek) was denied for
being filed out of time.
ISSUES
1. WON the appeal was filed out of time
2. WON dismissal based on loss of trust and confidence was valid
HELD
1. NO
Ratio The latest amendment to Rule 65, ROC allows filing of an appeal within 60 days
after the notice of denial of a motion for reconsideration.
Reasoning
- The amended rule now reads:
- Sec. 4. When and where petition filed. The petition shall be filed not later than sixty
(60) days from notice of the judgment, order or resolution. In case a motion for
reconsideration or new trial is timely filed, whether such motion is required or not, the sixty
(60) day period shall be counted from notice of the denial of the said motion.
- In the present case, the petition filed in the Court of Appeals was indeed filed beyond the
60-day period if computed from the time the notice of judgment was received and
interrupted only by the filing of the motion for reconsideration. However, if the 60-day
period is reckoned from the receipt of the notice denying the motion for reconsideration, as
provided under Circular No. 56-2000, then the petition for certiorari was filed on time.
2. NO
Ratio That the dismissal was based on loss of trust and confidence was not sufficiently
proven by evidence. Ramateks evidence are insubstantial and inadequate to support a
conclusion that Anelia engaged in anomalous transactions. Since the company had the
burden of proving the same, said dismissal cannot be held valid.
Reasoning
- The SC upholds the findings of the Labor Arbiter that Anelia was able to prove that the
charges against her were false and baseless.
Despite the gravity of the charges, there is nothing competent in the records to
substantiate the same. Xxx Ramatek has the burden to prove just cause, but it failed to
undertake the burden. On the other hand, complainant explained to the satisfaction of
this Office that the charges against her are utterly false and baseless.
- Ramatek having failed to substantiate their charges against Anelia with competent and
credible evidence, this Office perceives that the primordial inspiration for her dismissal
was the filing by her husband of a civil suit against the company officials, a matter which
respondents cannot legally use against complainant to deprive her of her tenurial rights.
This is because the suit was not filed by Anelia against Ramatek or its officials but by her
husband. There is no showing that the filing of the suit was a joint decision by the couple
or was instigated by complainant as to charge complainant with disloyalty or a conflict of
interests. Moreover, it appears that Anelias husband was merely asserting and exercising
his right to seek redress in the courts, a matter which Ramatek should not begrudge
Anelia about. Finally, the case was amicably settled by the parties such that there can be
no rational justification for respondents to dismiss Anelia just because a plaintiff in the civil
suit happened to be her husband.
- Loss of confidence as a ground for dismissal does not require proof beyond reasonable
doubt. The law requires only that there be at least some basis to justify it. Thus, there
must be some evidence to substantiate the claim and form a legal basis for loss of
confidence. The employer cannot exercise arbitrarily and without just cause the right to
dismiss an employee for loss of trust and confidence.
Disposition Resolutions of CA set aside. Decision of NLRC AFFIRMED.

A2010 - 225 LACK OF DAMAGE

Disini

CADIZ V CA (PHILIPPINE COMMERCIAL BANK


[EQUITABLE PCIBANK])
474 SCRA 232
TINGA; October 25, 2005
NATURE
Certiorari
FACTS
- Cadiz, Bongkingki and Gloria were employed as signature verifier, bookkeeper, and
foreign currency denomination clerk/bookkeeper-reliever, respectively, in the main office
branch (MOB) of Philippine Commercial International Bank (respondent bank).
- Cadiz reserved S/A No. 1083-4 in July 1987 as reflected on respondent banks new
account register.
- Foreign denominated checks payable to other payees were diverted into the said
account.
- The various deposit slips, covering the said checks, did not bear the machine validation
of any of the tellers-in-charge.
- Petitioner Cadiz agreed to pay Alqueza the equivalent amount of $600.00 but it was
made to appear that Alfiscar paid the said amount.
- In view of these findings, petitioners were served with show-cause memoranda asking
them to explain the lapses.
- Finding their explanations unsatisfactory, petitioners were terminated from employment.
LA-adjudged that petitioners were illegally dismissed and ordered their reinstatement and
payment of backwages.
- NLRC-reversed
- CA-affirmed reversal by NLRC
ISSUES
1. WON petitioners were validly dismissed (with just cause and were afforded due
process)
2. WON petitioners should be relieved of any liability considering that respondent bank
did not suffer a pecuniary loss
HELD
1. YES
- Petitioners had surreptitiously diverted funds deposited by depositors to S/A No. 1083-4
which was under their control and disposition.
- Their behavior in the course of the discharge of their duties is clearly malfeasant, and
constitutes ground for their termination on account of just cause.
- respondent bank complied with the two-notice rule prescribed in Article 277(b) of the
Labor Code. Petitioners were given all avenues to present their side and disprove the
allegations of respondent bank. An informal meeting was held between the branch
manager of MOB, the three petitioners and Mr. Gener, the Vice-President of the PCIB
Employees Union.
2. NO
- In University of the East v. NLRC the court held that lack of material or pecuniary
damages would not in any way mitigate a persons liability nor obliterate the loss of trust
and confidence.
- In the case of Etcuban v. Sulpicio Lines, this Court definitively ruled that:
. . . Whether or not the respondent bank was financially prejudiced is immaterial. Also,
what matters is not the amount involved, be it paltry or gargantuan; rather the
fraudulent scheme in which the petitioner was involved, which constitutes a clear
betrayal of trust and confidence. . . .

D. COMMISSION OF A CRIME
E. ANALOGOUS CAUSES
QUARELSOME BOSSY
CATHEDRAL SCHOOL OF TECHNOLOGY V NLRC
(VALLEJERA)
214 SCRA 551
October 13, 1992

Labor Law 1
NATURE
Petition for certiorari of a decision of NLRC.
FACTS
- Starting as an aspirant to the Congregation of the Religious of Virgin Mary (RVM),
VALLEJERA worked on a volunteer basis as a library aide of CST, an educational
institution run by the RVM sisters. Eventually she became a regular employee of CST,
again as library aide.
- It was around such regular employment, however, that trouble developed. The sisters
began receiving complaints from students and employees about VALLEJERA's difficult
personality and sour disposition at work. On one occasion, VALLEJERA was summoned
to the Office of the Directress by SISTER APOLINARIA, shortly after the resignation of the
school's Chief Librarian on account of irreconcilable differences with VALLEJERA, for the
purpose of clarifying the matter. SR APOLINARIA also informed VALLEJERA of the
negative reports received by her office regarding the latter's frictional working relationship
with co-workers and students and reminded VALLEJERA about the proper behavior in the
interest of peace and harmony in the school library. VALLEJERA resented the
observations about her actuations and was completely unreceptive to the advice given by
her superior. She reacted violently to SR APOLINARIA and angrily offered to resign,
repeatedly saying, "OK, I will resign. I will resign." Thereafter, without waiting to be
dismissed from the meeting, she stormed out of the office.
- On separate occasions thereafter, CST and SR APOLINARIA (PETITIONERS, for
brevity) sent people to convince VALLEJERA to settle her differences with the former.
VALLEJERA remained adamant in her refusal to submit to authority. Eventually, SR
APOLINARIA, by letter, informed VALLEJERA to look for another job as the school had
decided to accept her resignation. VALLEJERA filed a complaint for illegal dismissal. An
issue arose as to whether there was lawful cause for her dismissal.
ISSUE
WON there was there lawful cause for VALLEJERAs dismissal
HELD
YES
Ratio The reason for which VALLEJERAs services were terminated, namely, her
unreasonable behavior and unpleasant deportment in dealing with the people she closely
works with, is analogous to the other "just causes" enumerated under ART.282, Labor
Code.
- PETITIONERS' averments on VALLEJERAs disagreeable character as "quarrelsome,
bossy, unreasonable and very difficult to deal with," are supported by testimonies of
several co-employees and students of CST. In fact, her overbearing personality caused
the chief librarian to resign, Furthermore, the complaints about her objectionable behavior
were confirmed by her reproachable actuations during her meeting with SR APOLINARIA,
when VALLEJERA, upon being advised of the need to improve her working relations with
others, obstreperously reacted and unceremoniously walked out on her superior, and
arrogantly refused to subsequently clear up matters or to apologize therefor. To make
matters worse, she ignored the persons sent by PETITIONERS to intervene in an effort to
bring the matter to a peaceful resolution. The conduct she exhibited on that occasion
smacks of sheer disrespect and defiance of authority and assumes the proportion of
serious misconduct or insubordination, any of which constitutes just cause for dismissal
from employment.
- As CST is run by a religious order, it is but expected that good behavior and proper
department, especially among the ranks of its own employees, are major considerations in
the fulfillment of its mission. Under the circumstances, the sisters cannot be faulted for
deciding to terminate VALLEJERA whose presence "has become more a burden rather
than a joy" and had proved to be disruptive of the harmonious atmosphere of the school.
Disposition NLRC decision that VALLEJERA was illegally dismissed, SET ASIDE.

HEAVYLIFT MANILA INC V CA (GALAY, NLRC)


473 SCRA 541
QUISUMBING; October 20, 2005
NATURE
A petition for certiorari
FACTS
- Petitioner Heavylift, a maritime agency, thru a letter signed by Josephine Evangelio,
Admin. and Finance Manager of Heavylift, informed respondent Ma. Dottie Galay,
Heavylift Insurance and Provisions Assistant, of her low performance rating and the
negative feedback from her team members regarding her work attitude. The letter also
notified her that she was being relieved of her other functions except the development of
the new Access program.
- Subsequently, Galay was terminated for alleged loss of confidence.

A2010

- 226 -

Disini

- Thereafter, she filed with the Labor Arbiter a complaint for illegal dismissal and
nonpayment of service incentive leave and 13th month pay against petitioners.
- Petitioners alleged that Galay had an attitude problem and did not get along with her coemployees for which she was constantly warned to improve. Petitioners aver that Galays
attitude resulted to the decline in the companys efficiency and productivity.
- The Labor Arbiter found that Galay was illegally terminated for petitioners failure to prove
that she violated any company regulation, and for failure to give the proper notice as
required by law.
- Petitioner appealed to the NLRC. The latter, however, denied the appeal for lack of merit
and affirmed the decision of the Labor Arbiter.
- CA denied the motion for lack of justifying circumstances, and because the attached
board resolution was issued after the petition was filed (petitioners failed to: state the full
names and actual addresses of all the petitioners; attach the copies of all pleadings and
supporting documents; properly verify the petition; and certify against forum-shopping)
ISSUES
1. WON petitioners were denied due process with the CAs dismissal of the petition on
technical grounds
2. WON attitude problem is a valid ground for the termination of an employee.
3. If issue 2 is in the affirmative, WON this was sufficiently proved
4. WON the procedural requirements for an effectual dismissal were present
5. WON the awards of service incentive pay and 13th month pay were proper
HELD
1. YES
Ratio The Rules of Court are designed for the proper and prompt disposition of cases. In
not a few instances, we relaxed the rigid application of the rules to afford the parties
opportunity to fully ventilate their cases on the merits. In that way, the ends of justice
would be better served. Additionally, verification of a pleading is a formal, not a
jurisdictional requisite. It is intended to secure an assurance that what are alleged in the
pleading are true and correct and not the product of the imagination or a matter of
speculation, and that the pleading is filed in good faith.
2. YES
Ratio An employee who cannot get along with his co-employees is detrimental to the
company for he can upset and strain the working environment. Without the necessary
teamwork and synergy, the organization cannot function well. Thus, management has the
prerogative to take the necessary action to correct the situation and protect its
organization. When personal differences between employees and management affect the
work environment, the peace of the company is affected. Thus, an employees attitude
problem is a valid ground for his termination. It is a situation analogous to loss of trust and
confidence that must be duly proved by the employer. Similarly, compliance with the twin
requirement of notice and hearing must also be proven by the employer.
3. NO
Ratio We are not convinced that in the present case, petitioners have shown sufficiently
clear and convincing evidence to justify Galays termination. Though they are correct in
saying that in this case, proof beyond reasonable doubt is not required, still there must be
substantial evidence to support the termination on the ground of attitude. The mere
mention of negative feedback from her team members, and the letter, are not proof of her
attitude problem. Likewise, her failure to refute petitioners allegations of her negative
attitude does not amount to admission. Technical rules of procedure are not binding in
labor cases. Besides, the burden of proof is not on the employee but on the employer who
must affirmatively show adequate evidence that the dismissal was for justifiable cause.
4. NO
Ratio The letter did not constitute the required notice. It did not inform her of the specific
acts complained of and their corresponding penalty. Additionally, the letter never gave
respondent Galay an opportunity to explain herself, hence denying her due process.
5. YES
Ratio Apropos the award of service incentive pay and 13th month pay, we find that they
were properly prayed for by Galay. These were subsumed in the complaint and under the
position papers general prayer of such other relief as are just and equitable under the
law.
Disposition Decision of the Labor Arbiter and the Resolution of the NLRC are hereby
affirmed.

PROBABLE CAUSE
STANDARD ELECTRIC MANUFACTURING CORP V
STANDARD ELECTRIC EMPLOYEES UNION
CALLEJO; August 25, 2005
NATURE

Labor Law 1
Petition for review on certiorari to review the CA decision annulling the NLRC Resolution
which affirmed the LA decision
FACTS
- Rogelio Javier, a radio machine operator, employee of Standard Electric Manufacturing
Corp. (SEMC) and member of the Standard Electric Employees Union (Union), failed to
report for work and failed to report the reason for his absence. This failure to report for
work and failure to report the reason therefor happened several times until he was later
found to have been arrested and detained for the charge of rape.
- Javier informed SEMC (through a letter and through his counsel) that he was detained
for the charge of rape which is why he failed to report for work. He requested that SEMC
defer the implementation of its intention to dismiss him. The SEMC denied his request and
issued a Memorandum terminating his employment for having been absent without leave
(AWOL) for more than 15 days and for committing rape.
- Javier, after the RTC granted his demurrer to evidence and ordered his release from jail,
reported for work but the SEMC refused to accept him back. A grievance meeting between
the Union, Javier and the SEMC was later held, but the SEMC refused to re-admit Javier.
The Union and Javier then filed a complaint for illegal dismissal against SEMC before the
NLRC, alleging that since his detention for rape was non-existent, the termination of his
employment was illegal. SEMC averred that Javiers prolonged absences caused
irreparable damage to its orderly operation and that it could not afford to wait for Javiers
indefinite return from detention, if at all.
- The LA dismissed the complaint but ordered SEMC to pay Javier P71, 760 as separation
pay. On appeal, the NLRC affirmed the LAs ruling (held that Javier was given a chance to
explain his side), and later denied a subsequent MFR. Javier and the Union then filed a
petition for certiorari with the CA, which reversed the findings of both the LA and the NLRC
and ordered the reinstatement of Javier to his former position. The appellate court cited
Magtoto v NLRC and City Govt of Makati v Civil Service as precedents and declared that it
was not Javiers intention to abandon his job; his incarceration reasonably justified his
failure to report for work and negated the theory that he was on AWOL. The CA also held
that Javier could not be terminated on the ground of commission of a crime, as he was
acquitted of the rape charges. Hence, despite the fact that Javier was allegedly afforded
the opportunity to explain his side (the basis of the LA and NLRC decisions), the same
was unnecessary since there was no just or authorized cause for the dismissal. The MFR
by SEMC was denied by the CA, hence, this recourse.
ISSUE
WON the CA erred in holding that the termination was illegal
HELD
NO
- The CA was correct in holding that the termination was illegal and correctly applied the
Magtoto case.
Ratio Separation from employment founded on a false or non-existent cause is illegal
Reasoning
- In the Magtoto case, Alejandro JONAS Magtoto was arrested. He was charged with
violation of Arts 136 and 138 of the RPC. Although Magtoto informed his employer and
pleaded that he be considered on leave until released, his employer denied the request.
About seven months after his arrest, Magtoto was released after the City Fiscal dismissed
the criminal charges for lack of evidence. On the same date, he informed his employer of
his intent to start working again but the employer rejected the offer. In ruling that his
termination was illegal, the SC ruled:
The employer tries to distance itself from the detention by stressing that the petitioner
was dismissed due to prolonged absence. However, Mr. Magtoto could not report for
work because he was in a prison cell. The detention cannot be divorced from
prolonged absence. One caused the other. Since the causes for the detention, which in
turn gave the employer a ground to dismiss the petitioner, proved to be non-existent,
we rule that the termination was illegal and reinstatement is warranted.
- Respondent Javier was dismissed by the petitioner for: (a) being AWOL from July 31,
1995 up to January 30, 1996; and (b) committing rape. However, on demurrer to
evidence, Javier was acquitted of the charge. With Javiers acquittal, the cause of his
dismissal from his employment turned out to be non-existent.
- A non-existent cause for dismissal was explained in Pepito v. Secretary of Labor (96
SCRA 454):
... A distinction, however, should be made between a dismissal without cause and a
dismissal for a false or non-existent cause. In the former, it is the intention of the
employer to dismiss his employee for no cause whatsoever, in which case the
Termination Pay Law would apply. In the latter case, the employer does not intend to
dismiss the employee but for a specific cause which turns out to be false or nonexistent. Hence, absent the reason which gave rise to his separation from
employment, there is no intention on the part of the employer to dismiss the employee
concerned. Consequently, reinstatement is in order. And this is the situation here.
Petitioner was separated because of his alleged involvement in the pilferage in
question. However, he was absolved from any responsibility therefor by the court. The
cause for his dismissal having been proved non-existent or false, his reinstatement is

A2010

- 227 -

Disini

warranted. It would be unjust and unreasonable for the Company to dismiss petitioner
after the latter had proven himself innocent of the cause for which he was dismissed.
- The petitioner acted with precipitate haste in terminating respondent Javiers
employment on the ground that he had raped the complainant therein. Respondent Javier
had yet to be tried for the said charge. In fine, the petitioner prejudged him, and
preempted the ruling of the RTC. Petitioner had, in effect, adjudged Javier guilty without
due process of law. While it may be true that after the preliminary investigation of the
complaint, probable cause for rape was found and respondent Javier had to be detained,
these cannot be made as legal bases for the immediate termination of his employment.
Disposition petition DISMISSED for lack of merit. CA decision is AFFIRMED with
MODIFICATION. Petitioner is ordered to reinstate Rogelio Javier to his former position or,
if no longer possible, a substantially equivalent position without loss of seniority rights and
other privileges appurtenant thereto, with full backwages from the time it refused to allow
his reinstatement on May 24, 1996 until actually reinstated; or, if reinstatement is no longer
feasible, to pay him separation pay equivalent to one (1) month salary for every year of
service.

CONVICTION MORAL TURPITUDE


IRRI V NLRC (MICOSA)
221 SCRA 760
NOCON; May 12, 1993
NATURE
Petition for certiorari
FACTS
- International Rice Research Institute (IRRI) is an international organization recognized by
the Philippine government and accorded privileges, rights and immunities normally
granted to organizations of universal character. In 1977, it hired Nestor Micosa, who
thereby became bound by IRRI Employment Policy and Regulations, the Miscellaneous
Provisions of which states:
"C. Conviction and Previous Separation.
XXX
'2. An employer who has been convicted of a (sic) criminal offense involving moral
turpitude may be dismissed from the service.'"
- On February 6, 1987, Micosa stabbed to death Reynaldo Ortega inside a beer house in
Laguna. He was accused of homicide. During the pendency of the criminal case, Micosa
voluntarily applied for inclusion in IRRI's Special Separation Program. However, IRRI's
Director General expressed deep regret that he had to disapprove Micosa's application for
separation because of IRRI's desire to retain the skills and talents that persons like him
possess.
- Trial court found Micosa guilty of homicide, but appreciated in his favor the mitigating
circumstances of incomplete self-defense and voluntary surrender, and no aggravating
circumstance. Subsequently, Micosa applied for suspension of his sentence under the
Probation Law.
- On February 8, 1990, IRRI's Director General personally wrote Micosa that his
appointment as laborer was confirmed, making him a regular core employee whose
appointment was for an indefinite period and who "may not be terminated except for
justifiable causes as defined by the pertinent provisions of the Philippine Labor Code."
- On March 30, 1990, IRRIs HR head wrote Micosa urging him to resign from employment
in view of his conviction in the case for homicide.
- Laguna Parole and Probation Office No. II wrote IRRI informing the latter that said office
found Micosa's application for probation meritorious as he was evaluated "to possess
desirable social antecedents in his life."
- Micosa informed IRRI that he had no intention of resigning from his job.
- IRRIs HR head replied to Micosa's letter insisting that the crime for which he was
convicted involves moral turpitude and informing him that he is thereby charged of
violating Section I-AA, Par VII, C-2 of the Institute's Personnel Manual (quoted above).
- Micosa explained to IRRI that the slaying of Ortega arose out of his act of defending
himself from unlawful aggression; that his conviction did not involve moral turpitude and
that he opted not to appeal his conviction so that he could avail of the benefits of
probation, which the trial court granted to him.
- Micosa sought the assistance of IRRI's Grievance Committee who recommended to the
Director General, his continued employment. However, IRRI issued a notice to Micosa that
the latter's employment was to terminate effective May 25, 1990.
- Micosa then filed a case for illegal dismissal. Labor Arbiter found the termination was
illegal and ordered his reinstatement with full backwages from the date of his dismissal up
to actual reinstatement. NLRC affirmed decision.
Petitioners claims:
> Micosa's conviction of homicide, which is a crime involving moral turpitude, is a valid
ground for his dismissal under the Miscellaneous Provisions of IRRI's Employment Policy
Regulations. IRRI has the prerogative to issue rules and regulations including those

Labor Law 1
concerning employee discipline and that its employees are bound by the aforesaid
personnel manual
- While IRRI admits that Micosa's interests in his employment and means of livelihood
are adversely affected; that a convict should not be discriminated against in society and
that he should be given the same opportunities as those granted to other fellow citizens, it
claims that one's right is deemed superior than that of another. It believes that it has a
superior right to maintain a very high degree or standard not only to forestall any internal
problem hampering operations but also to prevent even the smallest possibility that said
problems could occur considering that it is an international organization with concomitant
obligation to the host country to avoid creating disturbance or give occasion for such
disturbance.
ISSUE
WON a conviction of a crime involving moral turpitude is a ground for dismissal from
employment
HELD
NO, it is not one of the causes enumerated in the Labor Code.
- Article 282 of the Labor Code enumerates the just causes wherein an employer may
terminate an employment. Conviction of a crime involving moral turpitude is not one of
these justifiable causes. Article 282 (c) or (d) may not be applied by analogy. Analogous
causes must have an element similar to those found in the specific just cause enumerated
under Article 282.
- Under Article 282 (c) fraud or willful breach by the employees of the trust reposed in him
by his employer or duly authorized representative refers to any fault or culpability on the
part of the employee in the discharge of his duty rendering him absolutely unworthy of the
trust and confidence demanded by his position. The breach of trust must be related to the
performance of the employee's function.
- Commission of a crime by the employee under Article 282 (d) refers to an offense
against the person of his employer or any immediate member of his family or his duly
authorized representative.
- The commission of the crime of homicide was outside the perimeter of the IRRI complex,
thus, the conviction of Micosa for homicide was not work-related, his misdeed having no
relation to his position as laborer and was not directed or committed against IRRI or its
authorized agent.
- IRRI failed to show how the dismissal of Micosa would be in consideration of the safety
and welfare of its employees, its reputation and standing in the community and its special
obligations to its host country. Micosa's service record is unblemished. IRRI's Director
General even expressed his confidence in him when he disapproved his application for
special separation and decided to promote him to the status of a regular core employee,
with the commensurate increases in benefits. In addition, the employees at IRRI's
Grievance Committee interceded favorably in behalf of Micosa when they recommended
his retention despite his conviction showing that the very employees which IRRI sought to
protect did not believe that they were placing their very own lives in danger with Micosa's
retention.
- Likewise, Micosa, although found guilty as charged, was also found worthy of probation.
This means that there existed no undue risk that Micosa will commit another crime during
his period of probation and that his being placed on probation would be to the benefit of
society as a whole.
- Even under IRRI's Employment Policy and Regulations, the dismissal of Micosa on the
ground of his conviction for homicide cannot be sustained. The miscellaneous provisions
of said personnel manual mentions of conviction of a crime involving moral turpitude as a
ground for dismissal. IRRI simply assumed that conviction of the crime of homicide is
conviction of a crime involving moral turpitude.
- Moral turpitude has been defined in Can v. Galing citing In Re Basa and Tak Ng v.
Republic as everything which is done contrary to justice, modesty, or good morals; an act
of baseness, vileness or depravity in the private and social duties which a man owes his
fellowmen, or to society in general, contrary to justice, honesty, modesty or good morals.
As to what crime involves moral turpitude, is for the Supreme Court to determine. The
conclusion of IRRI that conviction of the crime of homicide involves moral turpitude is
unwarranted considering that the said crime which resulted from an act of incomplete selfdefense from an unlawful aggression by the victim has not been so classified as involving
moral turpitude.
- The facts of the incident show that Micosa's intention was not to slay the victim but only
to defend his person. The appreciation in his favor of the mitigating circumstances of selfdefense and voluntary surrender, plus the total absence of any aggravating circumstance
demonstrate that Micosa's character and intentions were not inherently vile, immoral or
unjust.
- Corollary issue: WON conviction of homicide involves moral turpitude
Homicide may or may not involve moral turpitude depending on the degree of the crime.
Moral turpitude is not involved in every criminal act and is not shown by every known and
intentional violation of statute, but whether any particular conviction involves moral
turpitude may be a question of fact and frequently depends on all the surrounding
circumstances. Moral turpitude is somewhat a vague and indefinite term, the meaning of
which must be left to the process of judicial inclusion or exclusion as the cases are

A2010

- 228 -

Disini

reached.
Disposition petition is DISMISSED for lack of merit.

OANIA V NLRC (PHILEX MINING)


244 SCRA 668
ROMERO; June 1, 1995
FACTS
- Alfredo Oania, a welder, and Aurelio Caluza and Santiago Biay, miners, were employed
by Philex Mining Corporation (Philex) in Benguet. They were accused of mauling their coworker, Felipe Malong, at the gasoline area within the company compound.
- Philex conducted investigation regarding the incident. After a formal hearing wherein
petitioners were duly notified and accorded the opportunity to be heard, the company
arrived at the decision to terminate their employment on the ground that petitioners
violated Art I, par 1 of the company rules and regulations41
- Malong instituted a criminal complaint (frustrated murder) vs. petitioners. But later,
Malong desisted from pursuing the criminal case because he said his conscience
bothered him.
- With Malong's affidavit of desistance, petitioners sought reconsideration of their dismissal
from employment. Philex refused. Petitioners filed complaints for illegal dismissal before
the labor arbiter.
- Labor Arbiter: The termination of employment of petitioners was not justified was based
on findings that there was no proof that the mauling of Malong was "caused by a dispute
involving their employment" with private respondent (which, the Labor Arbiter believed,
was the only dispute clearly prohibited by the company rule).
- Petitioners had been illegally dismissed from employment. Philex to reinstate them to
their former positions or substantially equivalent positions and to pay each of them one
year's backwages.
- NLRC: Reversed. there is prima facie evidence that the complainants injured physically
a co- employee under circumstance(s) which constitute an infraction of specific company
rules; and that the respondent had valid cause to terminate their employment."
ISSUES
1. WON the mauling comes under Art 1 of the company rules and regulations
2. WON there was illegal dismissal
HELD
1. YES
- The provision in question obviously covers situations where any company employee
inflicts or attempts to inflict physical harm or injury upon any person. There are two
separate instances contemplated here. The first part of the sentence conceives of a
situation wherein such injury was done "on the job site on company time or property,"
regardless of the reason. What is material is the venue. The second half of the sentence
deals with a situation where an employee attempts to inflict or actually inflicts bodily injury
upon another "anywhere at anytime," regardless of the venue, as long as it arose in
connection with a dispute "involving one's employment." The site matters not; what is
crucial in the subject matter, i.e. it should have something to do with the employee's job.
Clearly, the commas in the sentence may be dispensed with without sacrificing the intent
behind the provision.
2. YES
- Violation of a company rule prohibiting the infliction of harm or physical injury against any
person under the particular circumstances provided for in the same rule may be deemed
analogous to "serious misconduct" stated in Art. 282 (a). (H)owever, there is no substantial
evidence definitely pointing to petitioners as the perpetrators of the mauling of Malong.
What is an established fact is that, after investigation, private respondent dismissed them
and, thereafter, a criminal complaint was filed against petitioners. It is of record that
Malong desisted from suing the perpetrators before the regular courts. In criminal cases,
an affidavit of desistance may create serious doubts as to be the liability of the accused
- On the issue of the legality of the dismissal, two requisites must concur to constitute a
valid dismissal: (a) the dismissal must be for any of the causes expressed in Art. 282 of
the Labor Code, and (2) the employee must be accorded due process, basic of which are
the opportunity to be heard and to defend himself.

LIM V NLRC (PEPSI-COLA FAR EAST TRADE DEVT)


259 SCRA 485
DAVIDE JR; July 26, 1996
NATURE
Petition for certiorari
41

"Inflicting or attempting to inflict bodily injury on the job-site on company time or property for any reason, or
attempting to inflict or inflicting bodily injury anywhere at anytime, in any dispute involving one's employment

Labor Law 1
FACTS
- Pepsi is a manufacturer of concentrates sold to Pepsi-Cola Bottlers Co. Inc. Petitioner
Sixta Lim had been employed with the Pepsi Group since January 1, 1981, working as a
secretary for Pepsi Bottling Co. Pepsi employed Lim as a secretary on June 15, 1983.
- At the time of her dismissal she was a staff accountant.
> She assisted and worked closely with the Plant Accountant to carry out the
accounting department's tasks necessary to ensure an accurate, timely, and
coordinated compilation of data for each accounting transaction.
> Her work involved cost accounting production, cost accounting financial reporting,
payroll reporting, statutory reporting and preparation of daily trade accounts receivable
reports, petty cash fund custodianship, and check preparation.
- Pepsi regularly evaluated its employees' performance using following ratings: Marginal
(obviously well below the acceptable level for the position), Fair Below (shows noticeable
need for improvement), Commendable (fully meeting the performance requirements of
the position), Superior (noticeably better than required performance) and Distinguished
Outstanding (obviously far above an acceptable job).
- Lims overall performance appraisals rated as follows: (a) "S" (Superior) as of May 1,
1984; (b) "C" (Commendable) for the period for December 1, 1987 to August 31,1988;
and (c) "U' (C minus), quantified as 81.10% for the period from September 1, 1988 to May
31, 1989.
- In 1989, Pepsi changed its rating scale to: Significantly Above Target (SA, exceeds
position requirements by a wide margin; exceptional), Above Target (AT, usually exceeds
position requirements), On Target (OT, meets and sometimes exceeds position
requirements), Below Target (BT, meets some or many but not all position requirements)
and Significantly Below Target (SB, below position requirements by a wide margin;
unacceptable).
- July 1, 1989 to December 31, 1989 Lim received an overall rating of BT.
> This was heavily influenced by her ratings in production reporting which made up
40% of her final rating. Her supervisor noted several discrepancies which could have
been avoided had Sixta been more diligent in her work.
> In cost accounting and financial reporting (20% of the rating), Lim also was given a
BT. Her supervisor noted that she did not seem to be aware of the importance of the
reports she issued and her work always needed to be reviewed. She also needed a
systematic workplan.
> For the remaining 60%, she was given an OT. Overall, she was given a BT.
- Lim questioned the change in the rating style as well as the ratings and appraisals given
to her by her supervisors. She asserted her previous positive ratings and expressed
disbelief over the sudden decline of her ratings. Pepsi conducted another evaluation and
Lims overall rating was a BT. Following that evaluation, she was given a report which
outlined the areas where she could improve.
- Lim then wrote Mr. Mihara of Pepsi Co. in Japan and Mihara replied, saying that he
would discuss the matters with her upon arrival in the Philippines. Pepsi, however, did not
wait for Mihara and offered to pay Lims termination benefits if she resigned.
- Lim refused to do so and on May 6, 1991, she was informed that she was terminated as
an employee of Pepsi. On May 14, 1991, she filed a complaint for illegal dismissal with
the Labor Arbiter. The Labor Arbiter decided matters in her favor, ordering Pepsi to
reinstate Lim to her former position or to pay her separation pay, 13 th month and
backwages.
- The NLRC reversed the Labor Arbiters ruling.
Petitioners Claim
> Lims BT performance appraisal was sufficient ground to dismiss her under Article 282
(b) of the Labor Code.
Respondents Comments
> Lim argues her alleged inefficiency was not among the just causes prescribed by law for
the dismissal of an employee and even assuming that such dismissal was justified, she
was still entitled to separation benefits of P268,000.00 in accordance with company policy
plus damages and attorney's fees.
ISSUE
WON Lims alleged gross inefficiency was an adequate ground for her dismissal
HELD
NO
Ratio "Gross inefficiency" is closely related to "gross neglect," for both involve specific
acts of omission on the part of the employee resulting in damage to the employer or to his
business. The Court has ruled that failure to observe prescribed standards of work, or to
fulfill reasonable work assignments due to inefficiency may constitute just cause for
dismissal.
Reasoning
- Pepsi had not characterized as "gross inefficiency" whatever failures, shortcomings, or
deficiencies may have been attributable to the petitioner.
- Lim obtained an unfavorable rating, but not to the extent, under the company's
standards, to warrant even a probationary measure which is given to the lowest rating of
Significantly Below Target (SB).

A2010

- 229 -

Disini

- In Pepsi's brochure entitled Managing Performance for the 90's, a BT rating does not
merit dismissal from the service; as a matter of fact, the lower rating - Significantly Below
Target (SB) - is not even a ground for termination of employment, but may only justify
putting the employee "on probation, telling the said employee that improvement is
necessary.
- If the company truly found the petitioner's "inefficiency" to be of such a gross character,
then it should have rated her even lower than SB, since the latter only requires that the
employee be put on probation.
- Pepsi also violated the petitioner's right to due process. Prior to the issuance of her
termination letter, Pepsi never called Lims attention to any alleged "gross inefficiency" on
her part. Likewise, she was never warned of possible disciplinary action due to any
alleged "gross inefficiency." The evaluation report merely indicated her areas for
improvement.
Disposition the instant petition is GRANTED. Private Respondent Pepsi-Cola Far East
Trade Development Co., Inc. is ordered to reinstate petitioner Sixta C. Lim to her position
as Staff Accountant without loss of seniority rights, and to pay her (a) backwages from the
time she was illegally dismissed until she was effectively reinstated, less whatever she
may have received through payroll reinstatement and whatever amount she may have
earned from employment elsewhere during the period of her illegal dismissal, and (b)
other monetary benefits that may be due her from the date of her illegal dismissal until
such effective reinstatement.

F. OTHER JUST CAUSES CLAIMED BY EMPLOYER


1. ABANDONMENT
DEFINED
NUEVA ECIJA ELECTRIC COOP (NEECO) II V NLRC
461 SCRA 169
CHICO-NAZARIO; June 23, 2005
NATURE
Petition for review
FACTS
- Petitioner NEECO II staunchly asserts that since its new GM assumed office on 01
March 1995, the GM never saw private respondent Eduardo Cairlan report for work
prompting the former to issue a memorandum dated 22 November 1995, which required
private respondent to explain in writing why he was not reporting for duty. Private
respondent was likewise directed in the said memo to report to its main office at
Calipahan, Talavera, Nueva Ecija. For failure of the private respondent to comply with the
said memorandum, Mr. dela Cruz directed a certain Mr. Marcelo to conduct an
investigation on the whereabouts of the petitioner. It was then that NEECO II uncovered
that private respondent was at that time already working with the Provincial Government of
Nueva Ecija as driver allegedly under an assumed name of Eduardo Caimay. For these
reasons, petitioner contended that it was left with no other alternative but to terminate
private respondents services.
- Petitioners GM terminated private respondents services on ground of abandonment.
Immediately thereafter, private respondent talked with the GM regarding this matter and
the latter promised him that the issue would be brought to the attention of NEECOs Board
of Directors for appropriate action. But nothing came out of the GMs promise prompting
private respondent to institute a Complaint for illegal dismissal with prayer for
reinstatement and payment of backwages since the NEECOs Board of Directors did not
act upon his termination.
- The Labor Arbiter rendered a Decision declaring that private respondent was illegally
dismissed on the following grounds: First, petitioners assertion that it required private
respondent to explain in writing why he was not reporting for duty as driver assigned at
Quezon Service Center merited scant consideration since a copy of the alleged
memorandum dated 22 November 1995, purportedly as its Annex A, was nowhere to be
found in the record of the case. Second, petitioners contention that private respondent
Cairlan was later discovered to be working with the Provincial Government of Nueva Ecija
under an assumed name of Eduardo Caimay remained unsubstantiated as petitioner
failed to adduce independent evidence that said Eduardo Caimay and private
respondent Eduardo Cairlan are one and the same person. Third, the Labor Arbiter held
that the private respondent was denied his right to due process since the letter of
termination dated 15 January 1996 stated that said termination is retroactively effected on
1 January 1996. Finally, according to the Labor Arbiter, petitioner failed to corroborate its
claim that private respondent was guilty of dereliction of duty.
Public respondent NLRC dismissed for lack of merit. The NLRC affirmed in toto the
decision of Labor Arbiter. Hence this petition.

Labor Law 1
ISSUES
1. WON petitioner was accorded due process
2. WON petitioner is guilty of illegally dismissing private respondent
HELD
1. YES
Ratio The rules of evidence prevailing in courts of law or equity shall not be controlling
and it is the spirit and intention of this Code that the Commission and its members and the
Labor Arbiters shall use every and all reasonable means to ascertain the facts in each
case speedily and objectively and without regard to technicalities of law or procedure, all
in the interest of due process.
Reasoning
- The Labor Arbiter shall motu proprio determine whether there is need for a formal trial or
hearing.
- Under Section 4, Rule V of the New Rules of Procedure of the NLRC, the Labor Arbiter is
given the latitude to determine the necessity for a formal hearing or investigation, once the
position papers and other documentary evidence of the parties have been submitted
before him. The parties may ask for a hearing but such hearing is not a matter of right of
the parties. The Labor Arbiter, in the exercise of his discretion, may deny such request
and proceed to decide the case on the basis of the position papers and other documents
brought before him without resorting to technical rules of evidence as observed in regular
courts of justice.
- In the present case, a scrupulous study of the records reveals that the Labor Arbiter did
not abuse his discretion conferred upon him by the Rules in not conducting a formal
hearing. On this, the findings of the Court of Appeals, consistent with that of the NLRC
and the Labor Arbiter, ought to be sustained.
2. YES
Ratio Abandonment42 is the deliberate and unjustified refusal of an employee to resume
his employment; it is a form of neglect of duty; hence, a just cause for termination of
employment by the employer under Article 282 of the Labor Code, which enumerates the
just causes for termination by the employer: i.e., (a) serious misconduct or willful
disobedience by the employee of the lawful orders of his employer or the latters
representative in connection with the employees work; (b) gross and habitual neglect by
the employee of his duties; (c) fraud or willful breach by the employee of the trust reposed
in him by his employer or his duly authorized representative; (d) commission of a crime or
offense by the employee against the person of his employer or any immediate member of
his family or his duly authorized representative; and (e) other analogous causes.
Reasoning
- Private respondents alleged abandonment of work through his employment with the
Provincial Government of Nueva Ecija was not clearly established and proven. The
evidence submitted by petitioner to buttress its allegation that private respondent
abandoned his work consists merely of indexes of payments to employees under the
name Eduardo Caimay without any further evidence showing that Eduardo Caimay and
private respondent Eduardo Cairlan is one and the same person. The best evidence that
could have established the allegation that Eduardo Caimay and private respondent
Eduardo Cairlan is one and the same person is Eduardo Caimays Personal Data Sheet
which definitely would have the pertinent personal information about him and a picture
that would identify him and not a testimony of a representative from the Provincial
Government of Nueva Ecija, as adverted to by petitioner to justify its motion for a trial type
hearing.
- Worse, private respondent received his notice of termination only on 15 January 1996
which termination is effective as early as 01 January 1996, all in gross violation of the
requirements provided for by law.
- Further negating petitioners contention of abandonment, as noted by the Labor Arbiter,
is private respondents letter dated 04 March 1996 addressed to Mr. Danilo dela Cruz
reiterating the formers plea for reconsideration of his dismissal. This letter depicts private
respondents fervor and yearning to continue working with petitioner the very antithesis
of abandonment
Disposition AFFIRMED.

GABUAY V OVERSEA PAPER SUPPLY INC


436 SCRA 514
CALLEJO; August 13, 2004
NATURE
Petition for review of the decision of the Court of Appeals
FACTS

42

The elements of abandonment are: (a) failure to report for work or absence without valid or justifiable reason; and (b)
a clear intention to sever the employer-employee relationship, with the second element as the more determinative factor
manifested by some overt acts (Tomas Lao Construction v. NLRC, 278 SCRA 716 [1997]).

A2010

- 230 -

Disini

- The respondent Oversea Paper Supply, Inc. is a domestic corporation engaged in the
business of selling paper products. On different dates, the respondent corporation hired
the petitioners for the positions machine operators, driver and helpers.
- On April 7, 1999, the respondent corporations sales and operations manager, James C.
Tan, required all employees to fill up and submit their bio-data not later than April 17, 1999
so that their 201 files could be updated. All the employees complied except for the
petitioners. Petitioners William Lacambra and Rodolfo Gabuay even failed to report for
work starting April 19 and 21, 1999, respectively.
- Thereafter, the respondent corporation required petitioners to explain why they refused to
submit their updated bio-data and requiring each of them to (a) return to work, and (b)
explain why they were absent. Despite the receipt of such notices, the petitioners, except
for Reynante Lacambra, did not reoport back to work.
- On April 21, 1999, petitioner Rodolfo Gabuay filed a complaint for illegal dismissal,
payment of separation pay, accumulated vacation and sick leave, and reinstatement with
full backwages before the arbitration branch of the National Labor Relations Commission
(NLRC). On April 26, 1999, petitioners William Lacambra, Reynante Lacambra, Rolando
Vicente and Tomacito Tabuli filed a similar complaint.
- The petitioners alleged that they were barred from reporting for work after they refused
to fill up their bio-data for the respondent corporation. They also claimed that they were
not paid vacation and sick leave benefits; that their 13th month pay for 1996 to 1998 was
underpaid; and, that the respondents violated their right to security of tenure and payment
of separation pay.
ISSUE
WON the petitioners were legally dismissed by reason of abandonment of work
HELD
- As correctly ruled by the Labor Arbiter, the NLRC and the CA, the petitioners were not
illegally dismissed. Even after the petitioners received notices from the respondent
corporation requiring them to report for work and to explain their unauthorized absences
and failure to submit their updated bio-data, they still failed to report for work. It can then
be inferred that the petitioners had abandoned their work. Indeed, the factors considered
for finding a valid abandonment are present in the case at bar: the petitioners failure to
report for work or absence was without valid or justifiable cause, and their refusal to report
for work notwithstanding their receipt of letters requiring them to return to work, show their
clear intention to sever the employer-employee relationship.
- Consistent with the finding that the petitioners abandoned their work, the award of
financial assistance in the form of separation pay should be deleted. Separation pay is
defined as the amount that an employee receives at the time of his severance and is
designed to provide the employee with the wherewithal during the period that he is looking
for another employment. Under the Labor Code, the award of separation pay is
sanctioned when termination was due to an authorized cause, i.e., (a) installation of labor
saving device, redundancy, retrenchment to prevent losses, closure or cessation of
business operations not due to serious business losses or financial reverses; and, (b)
disease prejudicial to the health of the employee and his fellow employees.
- Separation pay is, likewise, awarded in lieu of reinstatement if it can be shown that the
reinstatement of the employee is no longer feasible, as when the relationship between
employer and employee has become strained. In some cases, it is awarded as a measure
of social justice. In the present case, the petitioners were not dismissed, either legally or
illegally; the petitioners abandoned their jobs. They failed to return to work despite the
respondents directive requiring them to do so. There is, thus, no room for the award of
financial assistance in the form of separation pay. To sustain the claim for separation pay
under the circumstances herein established would be to reward the petitioners for
abandoning their work.
Disposition Petition denied

REQUISITES
LEONARDO V NLRC (REYNALDO'S MKTG CORP)
333 SCRA 589
DE LEON JR; June 16, 2000

NATURE
Petitions for certiorari seeking the
annulment of a Decision of the public
respondent, NLRC.
FACTS

Labor Law 1
- Petitioner AURELIO FUERTE was
originally
employed
by
private
respondent REYNALDO'S MARKETING
CORPORATION on August 11, 1981 as
a muffler specialist, receiving P45.00
per day. He was appointed as
supervisor
in
1988and
his
compensation was increased.
- DANILO LEONARDO was hired by
private respondent on March 4, 1988
as an auto-aircon mechanic.
- FUERTE alleges that on January 3,
1992, he was instructed to report at
private respondent's main office where
he was informed by the company's
personnel manager that he would be
transferred to its Sucat plant due to his
failure to meet his sales quota, and for
that reason, his supervisor's allowance
would be withdrawn.
- For a short time, FUERTE reported
for work at the Sucat plant; however,
he protested his transfer, subsequently
filing
a
complaint
for
illegal
termination.
- LEONARDO abandoned his post
following an investigation wherein he
was asked to explain an incident of
alleged "sideline" work which occurred
on April 22, 1991. It would appear that
late in the evening of the day in
question, the driver of a red Corolla
arrived at the shop looking for
LEONARDO. The driver said that, as
prearranged, he was to pick up
LEONARDO who would perform a
private service on the vehicle. When
reports of the "sideline" work reached
management,
it
confronted
LEONARDO
and
asked
for
an
explanation. According to private
respondent,
LEONARDO
gave
contradictory
excuses,
eventually
claiming that the unauthorized service
was for an aunt.

A2010

- 231 -

Disini

- When pressed to present his aunt, it


was then that LEONARDO stopped
reporting for work. He filed
a
complaint for illegal dismissal some ten
months after his termination.
ISSUES
1. WON the demotion of Fuerte by the
private respondent is proper
2. WON Fuerte's action constitutes
abandonment
3. WON the dismissal of Leornado is
justified
HELD
1. YES
- Private respondent's justification is
well-illustrated
in
the
record.
Complainant Fuerte's failure to meet
his sales quota which caused his
demotion
and
the
subsequent
withdrawal of his allowance is fully
supported
by
Exhibit
"4"
of
respondents' position paper showing
that his performance for the months of
July 1991 to November 1991 is below
par.
Reasoning
- FUERTE nonetheless decries his
transfer as being violative of his
security of tenure, the clear implication
being that he was constructively
dismissed. We have held that an
employer acts well within its rights in
transferring an employee as it sees fit
provided that there is no demotion in
rank or diminution in pay. 11 The two
circumstances are deemed badges of
bad faith, and thus constitutive of
constructive dismissal. In this regard,
constructive dismissal is defined in the
following manner:
an involuntary resignation resorted to
when continued employment becomes
impossible, unreasonable, or unlikely;

Labor Law 1
when there is a demotion in rank or
diminution in pay; or when a clear
discrimination, insensibility or disdain
by an employer becomes unbearable
to the employee. 12
- However, this arrangement appears
to us to be an allowable exercise of
company rights. An employer is
entitled
to
impose
productivity
standards for its workers, and in fact,
non-compliance may be visited with a
penalty even more severe than
demotion. Thus, the practice of a
company in laying off workers because
they failed to make the work quota has
been recognized in this jurisdiction.
- In the case at bar, the petitioners'
failure to meet the sales quota
assigned to each of them constitute a
just cause of their dismissal, regardless
of the permanent or probationary
status of their employment. Failure to
observe prescribed standards of work,
or
to
fulfill
reasonable
work
assignments due to inefficiency may
constitute just cause for dismissal.
Such inefficiency is understood to
mean failure to attain work goals or
work quotas, either by failing to
complete the same within the allotted
reasonable period, or by producing
unsatisfactory
results.
This
management prerogative of requiring
standards may be availed of so long as
they are exercised in good faith for the
advancement
of
the
employer's
interest.
2. NO
- his actions do not constitute
abandonment. The filing of a complaint
for illegal dismissal, as in this case, is
inconsistent
with
a
charge
of
abandonment.
Ratio
To constitute abandonment
there must be (1) failure to report

A2010

- 232 -

Disini

for work or absence without valid


or justifiable reason; and (2) a clear
intention, as manifested by some
overt acts, to sever the employeremployee relationship.
3. YES
- He was not terminated by the
company but Leonardo abandoned his
position in light of the pending
investigation
against
him.
Abandonment is a valid ground for
dismissal.
- He protests that he was never
accorded due process. This begs the
question, for he was never terminated;
he only became the subject of an
investigation
in
which
he
was
apparently loath to participate. As
testified to by Merlin P. Orallo, the
personnel manager, he was given a
memorandum asking him to explain the
incident in question, but he refused to
receive it. In an analogous instance, we
held that an employee's refusal to sign
the minutes of an investigation cannot
negate the fact that he was accorded
due process.
Disposition Petition dismissed.
R.P. DINGLASAN CONSTRUCTION INC V ATIENZA
433 SCRA 263
PUNO; June 29 2004
NATURE
Special Civil Action in the Supreme Court. Certiorari
FACTS
- This is an appeal from the decision and resolution of the Court of Appeals, dated January
17, 2001 and October 30, 2002, respectively, upholding the finding of constructive
dismissal against petitioner.
- Petitioner R.P. Dinglasan Construction, Inc. provided janitorial services to Pilipinas Shell
Refinery Corporation (Shell Corporation) in Batangas City. Private respondents Mariano
Atienza and Santiago Asi served as petitioners janitors assigned with Shell Corporation
since 1962 and 1973, respectively. Private respondents claim that on July 7, 1994,
petitioner called for a meeting and informed private respondents and three (3) other
employees that their employment with Shell Corporation would be terminated effective
July 15, 1994. They were told that petitioner lost the bidding for janitorial services with
Shell. Petitioner notified respondents that they may reapply as helpers and redeployed in
other companies where petitioner had subsisting contracts but they would receive only a
minimum wage. Private respondents refused as the offer would be a form of demotion --they would lose their seniority status and would not be guaranteed to work at regular
hours.
- In December 1994, private respondents filed a complaint against petitioner for nonpayment of salary with the district office of the Department of Labor and Employment
(DOLE) in Batangas City. In February 1995, during the conciliation proceedings with the

Labor Law 1
DOLE, petitioner sent notices to respondents informing them that they would be reinstated
with Shell Corporation as soon as they submit their barangay clearance, medical
certificate, picture and information sheet as per the new identification badge requirements
of Shell Corporation. Thereafter, petitioner again met with private respondents, who were
then accompanied by the barangay captain and a councilor, and the latter confirmed to the
former their willingness to be reinstated. Private respondents duly submitted the
documents required for their reinstatement.
- In May 1995, respondents demanded the payment of their backwages starting from July
15, 1994. On June 1, 1995, petitioner notified private respondents that they have been
declared absent without leave (AWOL) as they allegedly failed to signify their intention to
return to work and submit the badge requirements for their reinstatement. On June 13,
1995, private respondents wrote petitioner and insisted that they had complied with the
badge requirements. Accompanied by the barangay officials, private respondents
attempted to meet with the officers of petitioner but the latter refused to dialogue with
them. As proof of their compliance with the Shell requirements, private respondents
submitted to the DOLE their x-ray results, dated May 17 and 19, 1995 and their barangay
certification, dated May 13, 1995. The case was eventually referred to the National Labor
Relations Commission (NLRC) for compulsory arbitration. Private respondents amended
their complaint charging petitioner with illegal dismissal and non-payment of 13 th month
pay, with a claim for payment of attorneys fees and litigation expenses, and a prayer for
reinstatement with payment of full backwages from July 15, 1994.
- Petitioner gave a different version of the incident. It allegedly informed respondents and
the other affected employees that they would be deployed to petitioners other principal
companies but that their work would be different. Except for private respondents, all the
affected employees accepted its offer of redeployment and reported back to work.
Respondents failed to submit a resignation letter to signify their intention not to return to
work. Thereafter, during the pendency of the labor case, petitioner in two (2) separate
notices, informed private respondents that they could be reinstated at Shell Corporation
with no diminution in their salary provided that they submit the documents for the new
identification badge requirement of Shell Corporation. Private respondents, however,
refused to return to work until they were paid their backwages. Consequently, petitioner
was constrained to consider them as having abandoned their work and to terminate their
employment on September 19, 1995. Petitioner, thus, justified the dismissal of private
respondents on the grounds of gross and habitual neglect of duties and abandonment of
work. On September 3, 1998, labor arbiter Andres Zavalla rendered a decision finding that
private respondents were illegally dismissed from service and ordering their reinstatement.
- On appeal, the decision of the labor arbiter was affirmed by the NLRC. Without moving
for reconsideration, petitioner immediately filed a petition for certiorari before the Court of
Appeals but petitioner suffered the same fate. On the procedural aspect, the Court of
Appeals ruled that the petition could not prosper as petitioner failed to move for a
reconsideration of the NLRC decision. On the substantive issues, the appellate court
upheld the findings of the labor arbiter and the NLRC that: (1) private respondents were
constructively dismissed as petitioners offer of reassignment involved a diminution in pay
and demotion in rank that made their continued employment unacceptable; and, (2)
private respondents could not be considered to have abandoned their work.
- As petitioners motion for reconsideration was denied, petitioner filed this appeal
ISSUES
1. WON the respondents dismissal is justified
2. WON the Court of Appeals, contrary to existing law, erred in dismissing the petition for
certiorari and affirming the decision of the NLRC insofar as the monetary award is
concerned
HELD
1. Ratio In an illegal dismissal case, the onus probandi rests on the employer to prove
that its dismissal of an employee is for a valid cause. In the case at bar, petitioner failed to
discharge its burden. It failed to establish that private respondents deliberately and
unjustifiably refused to resume their employment without any intention of returning to work.
- To constitute abandonment of work, two (2) requisites must concur: first, the employee
must have failed to report for work or must have been absent without justifiable reason;
and second, there must have been a clear intention on the part of the employee to sever
the employer-employee relationship as manifested by overt acts. Abandonment as a just
ground for dismissal requires deliberate, unjustified refusal of the employee to resume his
employment. Mere absence or failure to report for work, after notice to return, is not
enough to amount to abandonment.
Reasoning
- In the case at bar, the evidence of private respondents negates petitioners theory that
they abandoned their work. Firstly, private respondents reported back to petitioners office
a number of times expressing their desire to continue working for petitioner without
demotion in rank or diminution of salary. This fact was established by the corroborating
testimony of barangay councilman Valentin Clerigo who, together with the barangay
captain, accompanied private respondents to petitioners office at least ten (10) times to
negotiate their redeployment on more acceptable terms. Secondly, in seeking
reinstatement, private respondents also sought the intervention of the DOLE to arbitrate
the labor issue between the parties. Thirdly, private respondents submitted the barangay

A2010

Disini

- 233 -

clearances and x-ray results required from them by petitioner for their reinstatement as
witnessed by the barangay officials. Lastly, the records would bear that private
respondents lost no time and sought their reinstatement by filing an illegal dismissal case
against petitioner, which act is clearly inconsistent with a desire to sever employeremployee relations and abandon their work. All these overt acts on the part of private
respondents negate petitioners claim of abandonment of work and prove beyond doubt
their steadfast desire to continue their employment with petitioner and be reinstated to
their former position. Moreover, petitioner failed to explain why it waited for 14 months
from the time private respondents allegedly did not return to work before it dismissed them
for being AWOL.
- We hold that private respondents were constructively dismissed by petitioner.
Constructive dismissal is defined as quitting when continued employment is rendered
impossible, unreasonable or unlikely as the offer of employment involves a demotion in
rank and diminution of pay. In the case at bar, petitioner committed constructive dismissal
when it offered to reassign private respondents to another company but with no
guaranteed working hours and payment of only the minimum wage. The terms of the
redeployment thus became unacceptable for private respondents and foreclosed any
choice but to reject petitioners offer, involving as it does a demotion in status and
diminution in pay. Thereafter, for six (6) months, private respondents were in a floating
status. Interestingly, it was only after private respondents filed a complaint with the DOLE
that petitioner backtracked in its position and offered to reinstate private respondents to
their former job in Shell Corporation with no diminution in salary. Eventually, however,
petitioner unilaterally withdrew its offer of reinstatement, refused to meet with the private
respondents and instead decided to dismiss them from service.
2. On the second issue, petitioner cannot impugn for the first time the computation of the
monetary award granted by the labor arbiter to private respondents.
Doctrine The settled rule is that issues not raised or ventilated in the court a quo cannot
be raised for the first time on appeal as to do so would be offensive to the basic rules of
fair play and justice. The computation of monetary award granted to private respondents is
a factual issue that should have been posed at the arbitration level when the award was
first granted by the labor arbiter who received and evaluated the evidence of both parties,
or, at the latest, raised by petitioner in its appeal with the NLRC.
- Petitioner omitted to do any of these. All throughout the proceedings below, from the
labor arbiter to the NLRC, and even in its petition before the Court of Appeals, petitioner
repeatedly pounded only on the sole issue of the validity of its dismissal of private
respondents. Thus, at this late stage of the proceedings, it cannot ask the Court to review
the bases and verify the correctness of the labor arbiters computation of the monetary
award which it never assailed below. A first-hand evaluation of the evidence of the parties
upon which the monetary award is based belongs to the labor arbiter. This Court is not a
trier of facts and factual issues are improper in a petition for review on certiorari. Likewise,
the Court notes that in seeking reinstatement and payment of their monetary claims,
private respondents have traversed a long and difficult path. This case has passed
the DOLE, the labor arbiter, the NLRC, the Court of Appeals and now this Court, with the
finding of illegal dismissal having been consistently affirmed in each stage. Private
respondents had been rendering janitorial services as early as 1962 and, at the time of
their dismissal, were receiving a measly P4,000.00 monthly salary. It is time to put a
period to private respondents travail. If there is anything that frustrates the search for
justice by the poor, it is the endless search for it.

CHAVEZ V NLRC
[PAGE 59]
FLOREN HOTEL V NLRC (CALIMLIM, RICO, ET AL)
458 SCRA 128
QUISUMBING; May 6, 2005
FACTS
- At the time of their termination, private respondents Roderick A. Calimlim, Ronald T. Rico
and Jun A. Abalos were working in the hotel as room boys, private respondent Lito F.
Bautista as front desk man, and private respondent Gloria B. Lopez as waitress. They all
started working for the hotel in 1993, except for Jun A. Abalos who started only in 1995.
- In the afternoon of June 6, 1998, petitioner Dely Lim randomly inspected the hotel rooms
to check if they had been properly cleaned. When she entered Room 301, she found
private respondent Lito F. Bautista sleeping half-naked with the air-conditioning on. Lim
immediately called the attention of the hotels acting supervisor, Diosdado Aquino, who
had supervision over Bautista. Lim admonished Aquino for not supervising Bautista more
closely, considering that it was Bautistas third offense of the same nature.
When she entered Room 303, she saw private respondents Calimlim and Rico drinking
beer, with four bottles in front of them. They had taken these bottles of beer from the
hotels coffee shop. Like Bautista, they had switched on the air conditioning in Room 303.
- That same afternoon, Dely Lim prepared a memorandum for Bautista, citing the latter for
the following incidents: (1) sleeping in the hotel rooms; (2) entertaining a brother-in-law for
extended hours during duty hours; (3) use of hotel funds for payment of SSS loan without

Labor Law 1
management consent; (4) unauthorized use of hotels air-con; and (5) failure to pay cash
advance in the amount of P4,000.
- Dely Lim tried to give Bautista a copy of the memorandum but Bautista refused to
receive it. Bautista then went on absence without leave. Calimlim and Rico, embarrassed
by the incident, went home. When they returned to work the next day, they were served
with a notice of suspension for one week.
- Like Bautista, they refused to receive the notice of suspension, but opted to serve the
penalty. Upon their return on June 15, 1998, they saw a memorandum dated June 13,
1998 on the bulletin board announcing (a) the suspension as room boys of Calimlim and
Rico, or alternately, (b) returning to work on probation as janitors for the following
reasons: unsatisfactory work, having a drinking spree inside the hotels rooms, cheating
on the Daily Time Record, being absent without valid reason, leaving work during duty
time, tardiness, and sleeping on the job. The memorandum also included Calimlim and
Ricos new work schedule.
- Calimlim and Rico submitted handwritten apologies and pleaded for another chance,
before they went AWOL
- On June 25, 1998, Calimlim, Rico and Bautista filed separate complaints, for illegal
dismissal and money claims, before the Labor Arbiter. Abalos and Lopez later also filed
separate complaints for underpayment of wages, non-payment of their 13th month pay,
and service incentive leave pay. On July 7, 1998, after they stopped working, Abalos and
Lopez amended their complaints. They claimed that petitioners orally dismissed them
when they refused to withdraw their complaints.
- Petitioners alleged that they did not dismiss private respondents but that private
respondents had abandoned their jobs.
- Private respondents filed a manifestation and motion dated November 24, 1998, praying
that petitioners be ordered to reinstate them to their former positions since after all,
according to petitioners, they were not dismissed.
- Petitioners opposed the motion and argued that private respondents cannot be
reinstated since they were not illegally dismissed but they had abandoned their jobs and
management simply considered them dismissed for abandonment.
- On March 19, 1999, the Labor Arbiter dismissed the complaints but ordered petitioners to
pay private respondents their proportionate 13th month pay, and service incentive leave
pay. He likewise ordered petitioners to pay Calimlim and Rico indemnity.
- The Labor Arbiter found that Calimlim, Rico, and Bautista did not report for work and they
did not show any order of dismissal, thus constructively, they abandoned their work and
were not illegally dismissed. The Labor Arbiter also ruled that Calimlim and Ricos
demotion and reassignment were valid exercises of management prerogatives. The
reassignment was intended to enable management to supervise them more closely and, in
any event, did not involve a diminution of wages. The Labor Arbiter, however, held
petitioners liable for indemnity to Calimlim and Rico for not observing the twin notices rule.
- Private respondents appealed to the National Labor Relations Commission
- On March 22, 2000, the NLRC rendered its decision. It reversed the decision of the
Labor Arbiter and ordered the hotel management to immediately reinstate complainantsappellants to their former positions without loss of seniority rights, with full backwages and
other benefits until they are actually reinstated. In the event that reinstatement was no
longer possible, the respondent-appellees should pay herein private respondents their
separation pay in addition to the payment of their full backwages; their incentive leave pay
and their 13th month pay, together with P1,000 to each of them as indemnity.
- Later, the NLRC also denied petitioners motion for reconsideration. The petitioners
appealed to the Court of Appeals.
- On September 10, 2002, the Court of Appeals decided the petition as follows: (1) The
Court declares that the private respondents Roderick A. Calimlim and Jose Abalos [should
be Ronald T. Rico] were illegally dismissed by petitioner Floren Hotel/Ligaya Chu who is
ORDERED to reinstate them to their former positions without loss of [seniority] rights, with
full backwages and other benefits until they are actually reinstated; but if reinstatement is
no longer possible, Floren Hotel/Ligaya Chu shall pay their separation pay in addition to
their backwages. (2) Declaring private respondents Lito Bautista, Jun Abalos and Gloria
Lopez to have abandoned their employment, and, therefore, not entitled to either
backwages nor separation pay; and (3) ORDERING Floren Hotel/Ligaya Chu to pay all the
private respondents their 13th month pay and incentive leave pay as computed in the
Decision of the Labor Arbiter
ISSUES
1. WON the Court of Appeals erred in giving due course to the petition for certiorari filed
before the appellate court
2. WON the private respondents were illegally dismissed
3 WON the Court of Appeals erred in ordering petitioners to pay Calimlim and Rico
indemnity of P1,500
4 WON the appellate court erred in ordering petitioners to pay all of private respondents
their proportionate 13th month pay and incentive leave pay
HELD
1. NO
- Acceptance of a petition for certiorari as well as the grant of due course thereto is
addressed to the sound discretion of the court. Section 1, Rule 65 of the Rules of Court in

A2010

- 234 -

Disini

relation to Section 3, Rule 46 of the same rules does not specify the precise documents,
pleadings or parts of the records that should be appended to the petition other than the
judgment, final order, or resolution being assailed. The Rules only state that such
documents, pleadings or records should be relevant or pertinent to the assailed resolution,
judgment or orders.
2. YES
- Petitioners claimed that all five private respondents were guilty of abandoning their jobs.
Thus, it was incumbent upon petitioners to show that the two requirements for a valid
dismissal on the ground of abandonment existed in this case. Specifically, petitioners
needed to present, for each private respondent, evidence not only of the failure to report
for work or that absence was without valid or justifiable reasons, but also of some overt act
showing the private respondents loss of interest to continue working in his or her job.
- It was true that private respondents abandoned their jobs, then petitioners should have
served them with a notice of termination on the ground of abandonment as required under
Sec. 2, Rule XIV, Book V, Rules and Regulation Implementing the Labor Code, in effect at
that time. Said Section 2 provided that:
Notice of Dismissal. Any employer who seeks to dismiss a worker shall furnish him a
written notice stating the particular acts or omission constituting the grounds for his
dismissal. In cases of abandonment of work, the notice shall be served at the workers
last known address.
- But petitioners failed to comply with the foregoing requirement, thereby bolstering further
private respondents claim that they did not abandon their work but were illegally
dismissed.
- None of the private respondents in this case had any intention to sever their working
relationship. Just days after they were dismissed, private respondents Calimlim, Rico,
Bautista, Abalos and Lopez filed complaints to protest their dismissals. The wellestablished rule is that an employee who takes steps to protest his layoff cannot be said to
have abandoned his work. That private respondents all desired to work in the hotel is
further shown by the fact that during the proceedings before the Labor Arbiter, shortly after
private respondents received petitioners position paper where the latter averred that
private respondents were never terminated, private respondents filed a manifestation and
motion asking that petitioners be ordered to allow them back to work. This is nothing if not
an unequivocal expression of eagerness to resume working.
3. YES (should have reinstated)
- Article 279 of the Labor Code gives to Calimlim and Rico the right to reinstatement
without loss of seniority rights and other privileges or separation pay in case reinstatement
is no longer possible, and to his full backwages, inclusive of allowances and other
benefits. It was thus error for the Court of Appeals to affirm the NLRC decision to award
Calimlim and Rico indemnity in addition to the measure of damages provided in Article
279. The award of indemnity is a penalty awarded only when the dismissal was for just or
authorized cause but where the twin-notice requirement was not observed.
4. NO
- Petitioners did not question the propriety of the award of proportionate 13th month pay
and service incentive leave in the Court of Appeals. They assailed the NLRC decision on
only one ground: Respondent NLRC committed grave abuse of discretion in reversing
the Labor Arbiters decision insofar as it relates to the issues of illegal dismissal. Hence,
the correctness of the cited award in the NLRC ruling was never brought before the
appellate court and is deemed to have been admitted by petitioners. It cannot therefore
be raised anymore in this petition. The decision of the NLRC as regards the award of 13 th
month pay and service incentive leave pay became binding on petitioners because the
failure to question it before the Court of Appeals amounts to an acceptance of the ruling.
In any event, the award appears to us amply supported by evidence and in accord with
law.
Disposition Assailed decision MODIFIED

INFERENCE
HDA. DAPDAP V NLRC (BARRIENTOS JR)
285 SCRA 9
BELLOSILLO; January 26, 1998
FACTS
- Nine workers of Hda. Dapdap I, a sugar farm in Victorias, Negros Occidental, filed a
complaint for illegal dismissal against its owner Magdalena Fermin alleging that they had
been working in the farm since 1977 but were unjustly terminated, without notice and
without any valid ground, on 27 January 1992.
- The only reason for their dismissal was their refusal to return the 6-hectare lot given to
them for cultivation under an "Amicable Settlement in connection with an illegal dismissal
case previously filed against the management of Hda. Dapdap I by its workers.
- In addition, complainants charged Magdalena Fermin with unfair labor practice for trying
to bust the National Federation of Sugar Workers Food and General Trades (NFSW-FGT)
Union which forged the 1986 "Amicable Settlement."

Labor Law 1
- Eight of the original complainants withdrew from the complaint and returned to work on
the ground that their misunderstanding with management was already settled.
- Pedro Barrientos Jr. was left as the sole complainant who amended the complaint by
impleading Lumbia Agricultural and Development Corporation (LADCOR), the real owner
of Hda. Dapdap I, as co-respondent with its President Magdalena Fermin.
- LADCOR denied that complainant was terminated and alleged that complainant
voluntarily abandoned his work to transfer to the adjacent farm of a certain Mr. Ramos.
- In addition, LADCOR alleged that it had a personality separate and distinct from its
president, Magdalena Fermin, hence the latter could not be held personally liable for the
alleged illegal dismissal.
- The Labor Arbiter ruled in favor of complainant. ] While LADCOR was absolved from the
charge of unfair labor practice it was held liable for illegal dismissal on the ground that its
claim of voluntary abandonment by complainant of his work was not credible in view of the
immediate institution of the case for illegal dismissal.
- LADCOR appealed to the NLRC.
- The NLRC affirmed the Labor Arbiter's decision in toto. The defense that complainant
voluntarily abandoned his work was similarly rejected on the additional grounds that no
notice of dismissal was sent by LADCOR to complainant as required by Sec. 2, Rule 14,
Book V, of the Rules Implementing the Labor Code and no concurrence of the intention to
abandon on the part of complainant and overt acts from which it could be inferred that he
was no longer interested in working for LADCOR.
ISSUE
WON petitioner was illegally dismissed
HELD
YES
- The Court is not a trier of facts. Whether respondent voluntarily abandoned his work
issue of credibility best left to the determination of the Labor Arbiter. Great respect and
even finality is accorded the conclusions of the Labor Arbiter and the NLRC in accordance
with the well-settled rule that findings of fact of labor arbiters affirmed by the NLRC are
binding on the Supreme Court.
- Judicial review in such cases is limited only to issues of jurisdiction or grave abuse of
discretion amounting to lack of jurisdiction.
- No such grave abuse of discretion was committed by the NLRC as it correctly applied the
consistent ruling in labor cases that a charge of abandonment is totally inconsistent with
the immediate filing of a complaint for illegal dismissal.
- It is indeed inconceivable that an employee like herein respondent who has been
working at Hda. Dapdap I since 1977 and cultivating a substantial portion of a 6-hectare
lot therein for himself would just abandon his work in 1992 for no apparent reason.
- Nor could intent to abandon be presumed from private respondent's subsequent
employment with another employer as petitioner alleges. The fact that the start of such
employment, i.e., after 1 March 1992 as petitioner alleges, coincides with the date of the
original complaint strongly indicates that such employment was only meant to help
respondent and his family survive during the pendency of his case.
- It has been said that abandonment of position cannot be lightly inferred, much less
legally presumed from certain equivocal acts such as an interim employment.
Disposition Petition was dismissed.

SPECIFIC ACTS
PREMIERE DEVT BANK V NLRC (LABANDA)
293 SCRA 49
MARTINEZ; July 23, 1988
NATURE
Petition for certiorari
FACTS
- August 8, 1985: Ramon T. Ocampo, a depositor of Premiere Devt Bank (PDB), issued a
check in the amount of P6,792.66 in favor of and for deposit to the account of Country
Banker's Insurance Corporation (CBISCO), also a depositor of PDB. On the same day,
after the check and the deposit slip were presented to respondent Teodora Labanda, who
was employed as teller at PDB Taytay Branch, they were turned over to the Branch
cashier for verification of the fund balance and signature of the drawer. There was a
confirmation of the check and the same was accepted by Labanda for deposit to the
current account of CBISCO.
- The check was posted by Manuel S. Torio, the Taytay Branch bookkeeper. But instead of
posting it to CBISCO's account, the same was posted to the account of Ocampo treating it
as "On-Us Check," that is, drawn against the Taytay Branch where the check was
deposited.
- January 13, 1986: the wife of Ocampo, together with the auditor from CBISCO, went to

A2010

- 235 -

Disini

PDB and complained to PDB Chairman Dr. Procopio C. Reyes that her husband was
being held accountable for the amount. It was only then that PDB discovered the
misposting of the check issued by Ocampo, resulting in the overstatement of his
outstanding daily balance by P6,792.66. The overstatement remained undetected until
Ocampo withdrew the money from PDB.
- Due to this incident, PDB Asst VPres Pacita M. Araos sent a demand letter to Labanda
requesting her to explain in writing the misposting and erroneous crediting of the subject
check in issue as well as the circumstances surrounding the incident within three (3) days
from receipt thereof, and in case she fails to do so, necessary action shall be taken
against her.
- PDB Exec VPres Renato G. Dionisio, upon instructions of Reyes, sent the internal
auditors of the bank to investigate and make a detailed report about the incident.
- January 22, 1986: the auditors came out with a report finding Labanda and bookkeeper
Torio primarily liable for the incident. These findings prompted Dionisio to send a letter to
Labanda requiring her to shoulder 20% of the amount lost via salary deduction. Labanda
replied, objecting to such move, reasoning out that she is the breadwinner in the family.
She further asked the bank to furnish her a copy of the audit report and requested for a
full-dress investigation. For this reason, petitioners held in abeyance the salary
deductions.
- March 13, 1986: Labanda was placed under preventive suspension pending investigation
of the incident. She was requested to report on April 4, 1986 so that she can present her
side of the story. Labanda then wrote a letter to Reyes requesting information on the
duration of her suspension and at the same time asking for an expeditious investigation. In
response thereto, she was informed that the period of her suspension shall last until the
investigation is completed and a decision is made thereon.
- On the date of said inquiry, Labanda executed a statement. However, she manifested
before Atty. Revelo during the inquiry that she will not sign any of the preliminary
statements she made unless the same is with the consent and advice of her husband. She
also told the inquiring officer that she could not inform petitioners of the dates when she
would be available for investigation.
- April 8, 1986: another letter was sent to Labanda by Reyes informing the former that her
refusal to sign or authenticate preliminary statements given on April 4, 1986 was a clear
indication of her unwillingness to cooperate or an effort to hide something or suppress the
truth.
- The dates of the hearing were rescheduled by petitioners several times. The first
rescheduled hearing was on April 14, 1986 where Labanda sent her lawyer bringing with
him a letter asking that she be given time to confer with her counsel for which she was
given until April 23. Notices were sent to inform her of the rescheduled dates with warning
that failure to attend the same shall be taken as a tacit admission of her liability and the
case shall be resolved based on the evidence available. In the meantime, Bookkeeper
Torio admitted liability and was allowed to resign.
- April 7, 1986: the bank officials received a letter from Labanda through her counsel
demanding payment of actual damages in the amount of P50,000.00 for their alleged
arbitrary, illegal and oppressive acts. Petitioners did not heed the demand.
- May 23, 1986: Labanda filed a complaint for damages before the court. Petitioners
motion to dismiss, and subsequent motion for reconsideration were both denied. The
petition for certiorari was also dismissed by CA, without prejudice to the refiling of the
complaint with the labor arbiter. The decision became final and executory on July 30,
1987.
- Eight months from the finality of the CA decision and two years from the alleged
termination of her employment, Labanda filed an illegal dismissal case before the Labor
Arbiter on the ground that her dismissal was without lawful cause and without due
process. After trial, the Labor Arbiter dismissed the labor case, ruling that Labanda was
not illegally dismissed, and that she abandoned her job when she filed a complaint for
compensatory damages with the regular court.
- NLRC reversed the decision of the Labor Arbiter ruling that Labandas indefinite
preventive suspension amounted to constructive dismissal. It ordered PDB to immediately
reinstate Labanda to her former position with backwages and other benefits for a period
not exceeding three (3) years without qualifications and deductions computed on the
amount of P87,750.00. It denied the subsequent MFR.
ISSUES
1. WON there was legal cause in placing Labanda under preventive suspension
2. WON the filing of an action for damages against one's employer is tantamount to
abandonment of job
3. WON PDB violated due process requirements in dismissing Labanda
4. WON Labandas action is barred by laches
HELD
1. NO
- Labanda's preventive suspension is without valid cause since she was outrightly
suspended by petitioner. As of the date of her preventive suspension on March 13, 1986
until the date when the last investigation was rescheduled on April 23, 1986, more than 30
days had expired. The preventive suspension beyond the maximum period amounted to
constructive dismissal.

Labor Law 1
- The question of whether or not an employee has abandoned his/her work is a factual
issue, not reviewable by this Court.
2. NO
- Labanda did not abandon her job. To constitute abandonment, two elements must
concur: (1) the failure to report for work or absence without valid or justifiable reason, and
(2) a clear intention to sever the employer-employee relationship, with the second element
as the more determinative factor and being manifested by some overt acts.
Abandoning one's job means the deliberate, unjustified refusal of the employee to resume
his employment and the burden of proof is on the employer to show a clear and deliberate
intent on the part of the employee to discontinue employment.
- The law, however, does not enumerate what specific overt acts can be considered as
strong evidence of the intention to sever the employee-employer relationship. An
employee who merely took steps to protest her indefinite suspension and to subsequently
file an action for damages, cannot be said to have abandoned her work nor is it indicative
of an intention to sever the employer-employee relationship. Her failure to report for work
was due to her indefinite suspension. Petitioner's allegation of abandonment is further
belied by the fact that Labanda filed a complaint for illegal dismissal. Abandonment of
work is inconsistent with the filing of said complaint.
3. YES
- The twin requirements of notice and hearing constitute the essential elements of due
process which are set out in Rule XIV, Book V of the Omnibus Rules Implementing the
Labor Code.
- Granting arguendo that there was abandonment in this case, it nonetheless cannot be
denied that notice still has to be served upon the employee sought to be dismissed, as the
second sentence of Section 2 of the pertinent implementing rules explicitly requires
service thereof at the employee's last known address. While it is conceded that it is the
employer's prerogative to terminate the services of an employee, especially when there is
a just cause therefor, the requirements of due process cannot be taken lightly. The law
does not countenance the arbitrary exercise of such a power or prerogative when it has
the effect of undermining the fundamental guarantee of security of tenure in favor of the
employee.
4. NO
- Laches is the failure for an unreasonable and unexplained length of time to do that which
in exercising due diligence, could or should have been done earlier. It is negligence or
omission to assert a right within a reasonable time, warranting the presumption that the
party entitled to assert it either has abandoned or has declined to assert it. A party cannot
be held guilty of laches when he has not incurred undue delay in the assertion of his
rights.
- Under the law, an illegal dismissal case is an action predicated on the injury to the rights
of the dismissed employee which prescribes in four (4) years. On April 4, 1988 or eight
months from the finality of the Court of Appeals' decision and two years from the alleged
termination of employment by respondent Labanda, she filed her complaint with the Labor
Arbiter which is within the four-year reglementary period. She did not sleep on her rights
for an unreasonable length of time.
- SolGen: Labanda never intended to abandon her job. First, after her indefinite
suspension, she requested that the "full-dressed" investigation be done at the quickest
time possible, and appealed to petitioner Reyes to consider that she was the breadwinner
in the family. Second, she actively fought for her right to security of tenure by filing first
with the RTC an action for damages, and later with the Labor Arbiter a complaint for illegal
dismissal. Moreover, Labanda's inability to report for work was not voluntary but was
rather the result of her indefinite suspension, which in reality was a constructive dismissal.
Petitioners never took the initiative to notify Labanda to report back to work or charge the
latter with abandonment of work. These show that Labanda did not abandon her job but
was illegally dismissed from employment without due process of law.
Disposition Petition is DISMISSED. The challenged NLRC Resolution is AFFIRMED.

1. LOANS
BORROWING MONEY
MEDICAL DOCTORS INC V NLRC (MAGLAYA, ELOA)
136 SCRA 1
MAKASIAR; April 24, 1985
NATURE
An appeal of the decision of the NLRC.
FACTS
- Evelyn Eloa (complainant) was given a probationary appointment as Clerk by the
Makati Medical Center from July 16, 1975 to January 15, 1976, and assigned at the OutPatient Charity Department of said Medical Center.
- Two of the conditions embodied in the appointment:

A2010

Disini

- 236 -

'Comply with all existing policies, rules and regulations and those that may be adopted
or promulgated in the future deemed necessary in the internal affairs of the employer;
'If at anytime during the probationary employment of the employee her services are
judged to be unsatisfactory, the employer may terminate such employment.'
- The termination or dismissal was and is predicated mainly on the fact that Evelyn Elona
borrowed P50 from one of the patients, Mrs. Leticia Lavapiez, allegedly in violation of
respondent's policies, rules and regulations against solicitation of any consideration from
indigent patients. The borrowing took place at Mrs. Lavapiezs house and after she was
discharged from the OPCD. The amount of P50 that was borrowed was also returned,
remitted or paid by complainant to Mrs. Lavapiez
- Eloa worked in this capacity of clerk continuously until February 14, 1976 when she was
dismissed or terminated.
- NLRC: Borrowing money and paying the same is not an act of dishonesty, of immorality,
of illegality, or of omissions punishable by law as to be a ground for dismissal as in this
case. We so hold that the Rules and Regulations & Policies of respondent Medical Center
are whimsical, capricious, arbitrary and oppressive The facts and the law point
unerringly to her side. She has completed her probationary period. Her employment
contract is not covered by an apprenticeship agreement stipulating a longer period.
ISSUE
WON Eloa was justly dismissed on sole reason of borrowing money from the patients
HELD
NO, Eloa was not dismissed justly.
Ratio Borrowing money is neither dishonest, nor immoral nor illegal, much less criminal.
Reasoning
- Private respondent paid the money she borrowed from the hospital patient. She was
even recommended for permanent appointment from her probationary status, from clerk to
secretary, by her immediate superior, Sis. Consolacion Briones.
- It may be added that she must have been compelled to borrow P50.00 from her patient
because of economic necessity, which circumstance should evoke sympathy from this
Court, the very constitutional organ mandated by the fundamental law to implement the
social justice guarantee for the protection of the lowly, efficient and honest employee, who
is economically disadvantaged, like herein petitioner.
Disposition Petition is dismissed, and decision of the labor arbiter is affirmed, with the
modification that backwages should cover three (3) years.

SEPARATE OPINION
AQUINO [dissent]
- Nicolas A. Zarate, the chief of the public information assistance division, apprised the
Makati Medical Center of Evelyn's conduct. Zarate alleged that Evelyn "has the habit of
borrowing money from OPD patients of that hospital." Evelyn allegedly borrowed P100
from Leticia Lavapiez after she delivered a baby. She attempted to borrow money from
Teofila Luzon and tried to ask for lunch from another patient, Mrs. Fabian. A copy of the
denunciation was furnished Mayor Nemesio Yabut.
- To have more time for investigating the charge, Evelyn's probationary appointment was
extended by one month or up to February 15, 1976. After due investigation, Consolacion
Briones, the supervisor of the Outpatient Charity Department, submitted a report
exonerating Evelyn. The Barangay Secretariat of Makati also recommended Evelyn's
exoneration.
- Eloa should not be reinstated or placed under permanent status because, as correctly
observed by Commissioner Villatuya of NLRC, she was dismissed when she was still a
probationary employee. It is true that the probationary status does not exceed six months
but under the peculiar circumstances of this case Evelyn's probationary or temporary
status was extended for one month due to the investigation. This may well be considered
an exceptional case. Evelyn is not the kind of employee who can invoke security of tenure.

PEARL S. BUCK FOUNDATION V NLRC


182 SCRA 446
GUTIERREZ; February 21, 1990
NATURE
Appeal from the decision of the NLRC as well as the resolution denying the motion for
reconsideration
FACTS
- Petitioner Pearl S. Buck Foundation, Inc. extends financial, education and medical
assistance to indigent "Amerasian" youth through funds provided by individuals and
church groups in the US. Private respondent Rubini Gosiaco Querimit was employed by
the petitioner as a case worker in the Olongapo City branch. One of the wards assigned to
Mrs. Querimit as such case worker was Richard Aliarte, Amerasian son of Andrea Aliarte.

Labor Law 1
- It appears that Mrs. Querimit borrowed P300 from Andrea Aliarte. It is not clear from the
records when she paid said debt but Mrs. Querimit once again borrowed P3,000.00 from
Aliarte, who requested assistance from petitioner for the collection of the indebtedness.
Mrs. Querimit paid the amount allegedly only after the petitioner had exerted incessant
pressure on her. Thereafter, she received a letter dated from the petitioner's resident
director informing her that her services would be terminated. Mrs. Querimit filed in the
NLRC a complaint for illegal dismissal, underpayment, overtime pay and maternity
benefits.
- The labor arbiter dismissed the complaint for lack of merit. On appeal, the NLRC opined
that borrowing money is not a ground for termination of employment under the Labor Code
and that the loan is a "personal transaction" between Andrea Aliarte and Mrs. Querimit
"the respondent not being a privy to (the) transaction and hence, had no cause to dismiss
the complainant from her job more so that the loan had earlier been paid and settled." The
petitioner filed an MFR. After it was denied, the petitioner filed the instant petition.
ISSUE
WON private respondent was illegally dismissed
HELD
1. NO
Ratio Borrowing money is neither dishonest, nor immoral, nor illegal, much less criminal.
However, said act becomes a serious misconduct that may justly be asserted as a ground
for dismissal when reprehensible behavior such as the use of a trust relationship as a
leverage for borrowing money is involved.
Reasoning
- The fact that Aliarte has retracted her complaint is of no moment. She loaned money to
the respondent, not once but twice and there can be no other assumption where the
money came from except from the trust funds intended for the ward. The NLRC should
have considered that a higher degree of prudence is required of the foundation's
employees especially when it comes to financial matters affecting the petitioner's wards.
The petitioner solicits or "begs" for money from abroad to support its wards. It cannot be a
third person where that money is involved.
Disposition The petition is GRANTED. The decision of the NLRC is REVERSED and
SET ASIDE. The decision of the Labor Arbiter is REINSTATED.

2. COURTESY RESIGNATION
BATONGBACAL V ASSOCIATED BANK
168 SCRA 600
FERNAN; December 21, 1988
NATURE
Review of the decision of the NLRC
FACTS
- Bienvenido Batongbacal, a lawyer, worked for Citizens Bank and Trust Company from
1961. On 1975, Citizens Bank and Trust Company merged with the Associated Banking
Corporation. The merged corporate entity later became known as Associated Bank. In the
new bank, petitioner resumed his position as assistant vice-president.
- On March 1982, he learned that his salary was very much below compared to the other
Asst. VPs of the bank. He wrote to the Board of Directors asking that he be paid the
proper amount. Apparently, said letter fell on deaf ears.
- On March 15, 1982, the board approved the following resolution:
BE IT RESOLVED that the new management be given the necessary flexibility in
streamlining the operations of the Bank and for the purpose it is hereby resolved that
the Bank officers at the Head Office and the Branches with corporate rank of Manager
and higher be required, as they hereby are required to submit IMMEDIATELY to the
President their courtesy resignations.
- Petitioner did not submit his courtesy resignation. On May 3, 1983, he received a letter
from the Board saying that his resignation has been accepted. Petitioner wrote to the
executive VP asking for reconsideration. He stated therein that he thought the call for the
submission of courtesy resignations was only for erring "loathsome" officers and not those
like him who had served the bank honestly and sincerely for sixteen years.
- Starting May 4, 1983, he was not paid. He filed for illegal dismissal and damages with
the NLRC. The NLRC ruled in favor of the petitioner. On MFR, the NLRC reversed.
ISSUE
WON the bank may legally dismiss for refusal to tender the courtesy resignation which the
bank required in line with its reorganization plan
HELD
NO

A2010

- 237 -

Disini

- While it may be said that the private respondent's call for courtesy resignations was
prompted by its determination to survive, we cannot lend legality to the manner by which it
pursued its goalBy directing its employees to submit letters of courtesy resignation, the
bank in effect forced upon its employees an act which they themselves should voluntarily
do. It should be emphasized that resignation per se means voluntary relinquishment of a
position or office. 11 Adding the word "courtesy" did not change the essence of resignation.
That courtesy resignations were utilized in government reorganization did not give private
respondent the right to use it as well in its own reorganization and rehabilitation plan.
There is no guarantee that all employers will not use it to rid themselves arbitrarily of
employees they do not like, in the guise of "streamlining" its organization. On the other
hand, employees would be unduly exposed to outright termination of employment which is
anathema to the constitutional mandate of security of tenure
- The record fails to show any valid reasons for terminating the employment of petitioner.
There are no proofs of malfeasance or misfeasance committed by petitioner which
jeopardized private respondent's interest.
- However, we agree with the Solicitor General and the NLRC that petitioner is not entitled
to an award of the difference between his actual salary and that received by the assistant
vice-president who had been given the salary next higher to his. There is a semblance of
discrimination in this aspect of the bank's organizational set-up but we are not prepared to
preempt the employer's prerogative to grant salary increases to its employees. In this
connection, we may point out that private respondent's claim that it needed to trim down
its employees as a self-preservation measure is belied by the amount of salaries it was
giving its other assistant vice-presidents
Disposition Remanded to the NLRC to determine WON the petitioner is a managerial
employee

3. WORK ATTITUDE
ABSENCES
MANILA ELECTRIC CO V NLRC
[PAGE 186]
GSP MANUFACTURING CORP V CABANBAN
495 SCRA 123
CORONA; July 14, 2006
NATURE
Petition for review on certiorari from a decision and a resolution of the Court of Appeals.
FACTS
- Cabanban worked with GSP Manufacturing Corporation (GSP) as a sewer from February
7, 1985 until her alleged termination on March 1, 1992.
- On June 16, 1992, respondent filed with the National Labor Relations Commission
(NLRC), National Capital Region Arbitration Branch, a complaint against petitioners for
illegal dismissal, non-payment of holiday pay, service incentive leave pay and 13th month
pay. She claimed she was terminated by petitioners because she failed to dissuade her
daughter from continuing her employment at the Sylvia Santos Company, a business
competitor of petitioners.In their defense, petitioners argued that respondent abandoned
her work on March 14, 1992 and that they reported this to the Department of Labor and
Employment on May 15, 1992.
- Labor arbiter found petitioners guilty of illegal dismissal. Petitioners appealed to the
NLRC, it was dismissed. Petition to CA was also dismissed. They claim that these
findings, based solely on statements made by respondent in the affidavit attached to her
position paper, were arrived at arbitrarily.
ISSUE
WON respondent is guilty of abandonment
HELD
NO
- Abandonment as a just ground for dismissal requires the deliberate, unjustified refusal of
the employee to perform his employment responsibilities. Mere absence or failure to
work, even after notice to return, is not tantamount to abandonment. The records are
bereft of proof that petitioners even furnished respondent such notice.
- Furthermore, it is a settled doctrine that the filing of a complaint for illegal dismissal is
inconsistent with abandonment of employment. An employee who takes steps to protest
his dismissal cannot logically be said to have abandoned his work. The filing of such
complaint is proof enough of his desire to return to work, thus negating any suggestion of
abandonment.
- Clearly, petitioners claim that respondents complaint was an afterthought, having been
filed a long time after the date of the supposed abandonment, was utterly without merit.

Labor Law 1
As the Court of Appeals correctly pointed out, citing the case of Pare v. NLRC, respondent
had four years within which to institute her action for illegal dismissal. Compared to the six
months it took the aggrieved employee in that case to file his complaint for illegal
dismissal, respondents 84 days was not unreasonably long at all.
Disposition petition is hereby DENIED. The assailed decision and resolution of the Court
of Appeals in CA-G.R. SP No. 51161 are hereby AFFIRMED.

4. TERM EMPLOYMENT
BRENT SCHOOL V ZAMORA
[PAGE 94]

ROMARES V NLRC
294 SCRA 411
MARTINEZ; August 19, 1998
NATURE
Appeal from a decision of NLRC
FACTS
- Complainant-petitioner Romares has been hired and employed by respondent PILMICO
since Sept 1, 89 to Jan 15, 93, in a broken tenure but all in all totalled to over a year's
service. Complainant's period of employment started on Sept 1, 89 up to Jan 31, 90 or for
a period of 5 months. Then on Jan 16 91, he was hired again up to June 15, 91, or for a
period covering another 5 months. Then on Aug 16, 92, he was hired again up to Jan 15,
93 or for a period of another 5 months. Thus, from Sept 1, 1989 up to January 15, 1993,
complainant has worked for 15 months more or less and has been hired and terminated 3
times. In all his engagements by respondent, he was assigned at respondent's
Maintenance/Projects/Engineering Dept performing maintenance work, particularly the
painting of company buildings, maintenance chores, like cleaning and sometimes
operating company equipment and sometimes assisting the regulars in the Maintenance/
Engineering Dept.
- Petitioners arguments: That having rendered a total service of more than 1 year and
by operation of law, complainant has become a regular employee of respondent; That
complainant has performed tasks and functions which were necessary and desirable in
the operation of respondent's business which include painting, maintenance, repair and
other related jobs; That complainant was never reprimanded nor subjected to any
disciplinary action during his engagement with the respondent; That without any legal
cause or justification and in the absence of any time to know of the charge or notice nor
any opportunity to be heard, respondent terminated him; That his termination is violative of
the security of tenure clause provided by law; That complainant be awarded damages and
be reinstated to his former position, be awarded backwages, moral and exemplary
damages and atty's fees.
- Respondents arguments: That complainant was a former contractual employee of
respondent and as such his employment was covered by contracts; That complainant was
hired as mason in the Maintenance/Project Department and that he was engaged only for
a specific project under such department; That when his last contract expired on Jan 15,
1993, it was no longer renewed and thereafter, complainant filed this instant complaint;
and; That since petitioner's employment contracts were for fixed or temporary periods, as
an exception to the general rule, he was validly terminated due to expiration of the
contract of employment.
- LA ruled in favor of petitioner finding him to be a regular employee and hence should be
reinstated. NLRC reversed LA decision ruling that petitioner was engaged in a fixed term
employment and as such, his termination was valid due to expiration of employment
contract. Hence, this appeal.
ISSUE
WON dismissal of complainant (under the just cause that such employment was of term
employment) was justified
HELD
NO
[a] Petitioner was deemed a regular employee. Petitioners work with PILMICO as a
mason was definitely necessary and desirable to its business. PILMICO cannot claim that
petitioner's work as a mason was entirely irrelevant to its line of business in the production
of flour yeast feeds and other flour products. During each rehiring, the summation of which
exceeded
1
year,
petitioner
was
assigned
to
PILMICO's
Maintenance/Projects/Engineering Dept performing the same kind of maintenance work
such as painting of company buildings cleaning and operating company equipment, and

A2010

- 238 -

Disini

assisting the other regular employees in their maintenance works. Such a continuing need
for the services of petitioner is sufficient evidence of the necessity and indispensability of
his services to PILMICO's business or trade.
[b] Even assuming arguendo that petitioner was temporary EE, he was converted to
regular employee ff this rule: If the employee has been performing the job for at least one
year, even if the performance is not continuous or merely intermittent, the law deems the
repeated and continuing need for its performance as sufficient evidence of the necessity is
not indispensability of that activity to the business. Hence, the employment is also
considered regular but only with respect to such activity and while such activity exists.
[c] In rehiring petitioner, employment contracts ranging from 2 to 3 months with an express
statement that his temporary job/service as mason shall be terminated at the end of the
said period or upon completion of the project was obtrusively a convenient subterfuge
utilized to prevent his regularization. It was a clear circumvention of the employee's right to
security of tenure and to other benefits. It likewise evidenced bad faith on the part of
PILMICO.
[d] NLRC erred in finding that the contract of employment of petitioner was for a fixed or
specified period. From Brent v Zamora: The decisive determinant in "term employment"
should not be the activities that the employee is called upon to perform but the day certain
agreed upon by the parties for the commencement and termination of their employment
relationship. But, if from the circumstances it is apparent that the periods have been
imposed to preclude acquisition of tenurial security by the employee, they should be struck
down or disregarded as contrary to public policy and morals.
Note however that, "term employment" cannot be said to be in circumvention of the law on
security of tenure if: (1) The fixed period or employment was knowingly and voluntarily
agreed upon by the parties without any force, duress, or improper pressure being brought
to bear upon the employee and absent any other circumstances vitiating his consent; or
(2) It satisfactorily appears that the employer and the employee dealt with each other on
more or less equal terms with no moral dominance exercised by the former or the latter
None of these requisites were complied with.
Disposition Petition GRANTED. NLRC decision SET ASIDE. LA decision REINSTATED

MEDENILLA V PHIL VETERANS BANK


PURISIMA; March 13, 2000
FACTS
- Petitioners were employees of the Philippine Veterans Bank (PVB). On June 15, 1985,
their services were terminated as a result of the liquidation of PVB. On the same day of
their termination, petitioners were rehired through PVB's Bank Liquidator.
- All of them were required to sign employment contracts which provided that:
(1) The employment shall be strictly on a temporary basis and only for the duration of the
particular undertaking for which a particular employee is hired; (3) The Liquidator reserves
the right to terminate the services of the employee at any time during the period of such
employment if the employee is found not qualified, competent or, efficient in the
performance of his job, or have violated any rules and regulations, or such circumstances
and conditions recognized by law.
- January 18, 1991 their employment was terminated. The reasons for which were "(a) To
reduce costs and expenses in the liquidation of closed banks in order to protect the
interests of the depositors, creditors and stockholders of Bank. (b) The employment were
on strictly temporary basis."
- Petitioners filed for illegal dismissal. Labor Arbiter found for employees. NLRC however
reversed decision
ISSUES
1. WON NLRC gravely abused its discretion in holding that the employment contract
entered into by the complainants and the Liquidator of PVB was for a fixed-period
2. WON NLRC act with grave abuse of discretion in finding that there was no illegal
dismissal
HELD
1. NO
- Employment contract between parties states that:
(1) The employment shall be on a strictly temporary basis and only for the duration of the
particular undertaking for which you are hired and only for the particular days during which
actual work is available as determined by the Liquidator or his representatives since the
work requirements of the liquidation process merely demand intermittent and temporary
rendition of services."
- The Court has repeatedly upheld the validity of fixed-term employment. Philippine
National Oil Company-Energy Development Corporation vs. NLRC gave two guidelines by
which fixed contracts of employment can be said NOT to circumvent security of tenure:
1. The fixed period of employment was knowingly and voluntarily agreed upon by the
parties, without any force, duress or improper pressure being brought to bear upon the
employee and absent any other circumstances vitiating his consent;
or:

Labor Law 1
2. It satisfactorily appears that the employer and employee dealt with each other on
more or less equal terms with no moral dominance whatever being exercised by the
former on the latter."
- The employment contract entered into by the parties herein appears to have observed
the said guidelines. Furthermore, it is evident from the records that the subsequent rehiring of petitioners which was to continue during the period of liquidation and the process
of liquidation ended prior to the enactment of RA 7169 entitled, "An Act to Rehabilitate
Philippine Veterans Bank
2. YES
- The reason given by the Liquidator for the termination of petitioners' employment was "in
line with the need of the objective of the Supervision and Examination Sector, Department
V, Central Bank of the Philippines, to reduce costs and expenses in the liquidation of
closed banks in order to protect the interest of the depositors, creditors and stockholders
- In cases of illegal dismissal, the burden is on the employer to prove that there was a
valid ground for dismissal. Mere allegation of reduction of costs without any proof to
substantiate the same cannot be given credence by the Court. As the respondents failed
to rebut petitioners' evidence, the irresistible conclusion is that the dismissal in question
was illegal.
- the failure of respondent bank to dispute complainants' evidence pertinent to the various
unnecessary and highly questionable expenses incurred renders the termination process
as a mere subterfuge, as the same was not on the basis as it purports to see, for reason
that immediately after the termination from their respective positions, the same were given
to other employees who appear not qualified. What respondent's counsel did was merely
to dispute by pleadings the jurisdiction of this Office and the claims for damages, which
evidentiary matters respondent is required to prove to sustain the validity of such
dismissals."
- As held by this Court, if the contract is for a fixed term and the employee is dismissed
without just cause, he is entitled to the payment of his salaries corresponding to the
unexpired portion of the employment contract

MAGSALIN V NATIONAL ORGANIZATION OF


WORKING MEN
[PAGE 77]
LABAYOG V MY SAN BISCUITS INC
[PAGE 89]

5. PAST INFRACTIONS
PAST OFFENSES
STELLAR INDUSTRIAL SERVICE INC V NLRC
(PEPITO)
252 SCRA 323
REGALADO; January 24, 1996
NATURE
Special Civil Action for Certiorari
FACTS
- Stellar Industrial Services, Inc., an independent contractor engaged in the business of
providing manpower services, employed private respondent Roberto H. Pepito as a janitor
on January 27, 1975 and assigned the latter to work as such at the Maintenance Base
Complex of the Philippine Airlines in Pasay City.
- Pepito worked for 15 years.
- According to petitioner, private respondent committed infractions of company rules
ranging from tardiness to gambling, but he was nevertheless retained as a janitor out of
humanitarian consideration and to afford him an opportunity to reform.
- Stellar finally terminated private respondent's services on January 22, 1991 because of
Absent Without Official Leave/Virtual Abandonment of Work Absent from November 2 December 10, 1990.
- Private respondent had insisted that during the period in question he was unable to
report for work due to severe stomach pain and that, as he could hardly walk by reason
thereof, he failed to file the corresponding official leave of absence. Attached was a
medical certificate.
- Petitioner filed a complaint for illegal dismissal, illegal deduction and underpayment of
wages with prayer for moral and exemplary damages and attorney's fees.
- LA was of the view that Pepito was not entitled to differential pay, or to moral and
exemplary damages for lack of bad faith on the part of the company, he opined that private
respondent had duly proved that his 39-day absence was justified on account of illness

A2010

- 239 -

Disini

and that he was illegally dismissed without just cause. He ordered the respondent to
immediately reinstate complainant to his former position as Utilityman, without loss of
seniority rights and with full backwages and other rights and privileges appurtenant to his
position until he is actually reinstated.
- The respondent is further ordered to pay the complainant reasonable attorney's fees
equivalent to 10% of the amount recoverable by the complainant.
- LAs decision was affirmed by NLRC
ISSUES
1. WON serious misconduct for nonobservance of company rules and regulations may be
attributed to Pepito
2. WON the extreme penalty of dismissal meted to him by Stellar may be justified under
the circumstances

HELD
1. NO
- Stellar's company rules and regulations on the matter could not be any clearer, to wit:
"Absence Without Leave"
Any employee who fails to report for work without any prior approval from his
superior(s) shall be considered absent without leave.
In the case of an illness or emergency for an absence of not more than one
(1) day, a telephone call or written note to the head office, during working
hours, on the day of his absence, shall be sufficient to avoid being penalized.
In the case of an Illness or an emergency for an absence of two (2) days or
more, a telephone call to the head office, during regular working hours, on the
first day of his absence, or a written note to the head office, (ex. telegram)
within the first three (3) days of his absence, and the submission of the proper
documents (ex. medical certificate) On the first day he reports after his
absence shall be sufficient to avoid being penalized.
1st offense- three (3) days suspension
2nd offense- seven (7) days suspension
3rdoffense- fifteen (15) days suspension
4th offense- dismissal with a period of one (1) year
- There was substantial compliance with said company rule by private respondent. He
immediately informed his supervisor of the fact that he could not report for work by reason
of illness. At the hearing, it was also established without contradiction that Pepito was able
to talk by telephone to one Tirso Pamplona, foreman, and he informed the latter that he
would be out for two weeks as he was not feeling well. Added to this is his letter to the
chief of personnel which states that, on November 2, 1990, he relayed to his supervisor
his reason for not reporting for work and that, thereafter, he made follow-up calls to their
office when he still could not render services. As earlier noted, these facts were never
questioned nor rebutted by petitioner.
- While there is no record to show that approval was obtained by Pepito with regard to his
absences, the fact remains that he complied with the company rule that in case of illness
necessitating absence of two days or more, the office should be informed beforehand
about the same that is, on the first day of absence. Since the cause of his absence could
not have been anticipated, to require prior approval would be unreasonable. On this score,
then, no serious misconduct may be imputed to Pepito. Necessarily, his dismissal from
work, tainted as it is by lack of just cause, was clearly illegal.
2. NO
- Petitioner's reliance on Pepito's past infractions as sufficient grounds for his eventual
dismissal, in addition to his prolonged absences, is unavailing. The correct rule is that
previous infractions may be used as justification for an employee's dismissal from work in
connection with a subsequent similar offense.
- In the present case, private respondent's absences, as already discussed, were incurred
with due notice and compliance with company rules and fie had not thereby committed a
"similar offense" as those lie had committed in the past. Furthermore, as correctly
observed by the labor arbiter, those past infractions had either been "satisfactorily
explained, not proven, sufficiently penalized or condoned by the respondent." In fact, the
termination notice furnished Pepito only indicated that he was being dismissed due to his
absences from November 2. 1990 to December 10, 1990 supposedly without any
acceptable excuse therefor. There was no allusion therein that his dismissal was due to
his supposed unexplained absences on top of his past infractions of company rules. To
refer to those earlier violations as added grounds for dismissing him is doubly unfair to
private respondent. Significantly enough, no document or any other piece of evidence was
adduced by petitioner showing previous absences of Pepito, whether with or without
official leave.
Disposition Petition dismissed

LA CARLOTA PLANTERS ASSN V NLRC (COMPACION)

Labor Law 1

A2010

298 SCRA 252


VITUG; October 27, 1998
NATURE

Petition for certiorari which seeks to set aside and


nullify the decision of the NLRC promulgated on 25
September 1995 setting aside the LAs decision and
directing

the

respondent

to

pay

complainant

backwages and separation pay in lieu of reinstatement,


computed at one (1) month per year of service.
FACTS
- Compacion alleges that he was a regular employee of petitioner since 1988 hired as
truck driver; that on December 14, 1992, at the instance of the petitioner, he drove the
truck overloaded of sugarcane bound for La Carlota Sugar Central; that while driving
through Sitio Bacus, Ma-ao, Bago City, the road was very slippery causing the truck to be
outbalance (sic) resulting to the truck turning right side down; that he was not drunk when
he drove the truck on December 14, 1992; that the Security Guards of Central La Carlota
issued a clearance to the effect that he is cleared from whatever issues against him; that
Rene Baylon reported the incident only on March 1993 when the incident happened on
December 14, 1992 as shown by the Police Blotter; that because of his illegal dismissal,
he sought the help of a legal counsel who helped him in filing this case for which he claims
for payment of attorney's fees.
- On the other hand, petitioner alleges that Compacion is a truck driver of Nature's Beauty
Trucking Services; as such, he was assigned to Ma-ao Transloading Station, a loading
station of sugarcanes bound for Central La Carlota located at Brgy. Ma-ao, Bago City,
Negros Occidental; that on December 14, 1992, Compacion who was very drunk and with
a knife entered the Ma-ao Transloading Station and harassed the office personnel to the
extent of stabbing the person of Gerry Flores who fortunately was able to escape the said
assault; that despite the repeated warning made by the Shifting In-charge Rene Baylon
not to drive the truck, he drove the ten wheeler truck loaded with 18 tons of sugarcane
bound for La Carlota Central in a reckless manner causing the truck to turn right side
down resulting in a damage to property paid by the owner to Mr. Eulalio Pagunsan, owner
of the bananas and pig pen hit and destroyed by the truck; that the said Mr. Eulalio
Pagunsan observed that the driver Felix Compacion, at the time of the accident, was very
drunk; that because of this accident which happened because of reckless driving, the
truck underwent major repair; that after the accident, driver Felix Compacion was nowhere
to be found, never reporting the accident to the police authorities or to the owner; that
despite repeated calls, he refused to meet the owner nor did he report to the office thus
prompting the latter to write him a letter dated January 4, 1993 suspending him for 30
days; further requiring him to report to the office and explain why he should not be
terminated.
- Petitioner further averred that during his employment, Compacion was paid wages and
other benefits in accordance with law; that at the time of the accident, there was no rain
and the road was not slippery; that at the time he stopped reporting, he has an
outstanding account with respondent in the amount of P3,650.00; that prior to this accident
on December 14, 1992, specifically on November 27, 1992, Felix Compacion was caught
stealing diesel fuel from the drums owned by La Carlota Planters Association for which he
was admonished and warned not to repeat the same.
ISSUE
WON there was valid, legal and just cause for the dismissal of private respondent by
petitioners

HELD
NO
Ratio The correct rule has always been that such previous offenses may be so used as
valid justification for dismissal from work only if the infractions are related to the

- 240 -

Disini

subsequent offense upon which basis the termination of employment is decreed. The
previous infraction, in other words, may be used if it has a bearing to the proximate
offense warranting dismissal.
Reasoning
- Petitioners contend that sufficient factual and legal bases exist to justify the dismissal of
private respondent for misconduct. It cites various infractions allegedly committed in the
past by private respondent; to wit:
a. Private respondent was caught twice stealing diesel fuel from the drum of the
petitioner's association;
b. He entered the transloading office on December 14, 1992 drunk, armed with bayonet
knife, and harassed the personnel therein, even unsuccessfully stabbing one Gerry Flores
for two (2) times; and
c. Private respondent failed to report for work since December 14, 1992 which is an
obvious sign of guilt.
- The reliance by petitioners on the past offenses of private respondent supposedly
dictating his eventual dismissal is unavailing. The complainant may have been at fault
when he figured in a vehicular accident causing damage to the company truck; that fault,
nevertheless, cannot be considered a just cause for dismissal. Indeed, it has once been
held that the penalty of dismissal would be grossly disproportionate to the offense of
driving through reckless imprudence resulting in damage to property. The claim of
drunkenness on the part of private respondent has not been substantiated; the allegation
is based solely on the uncorroborated statement made by one Rene Baylon in his affidavit
executed on 24 April 1993, months after the accident had occurred in December of 1992.
Disposition the Court is constrained to dismiss, as it hereby so DISMISSES, the instant
petition for certiorari.

6. PROFESSIONAL TRAINING
RESIDENCY TRAINING
FELIX V BUENASEDA
[PAGE 55]

7. LOVE AND MORALS


IMMORALITY
SANTOS V NLRC (HAGONOY INSTITUTE ET AL)
287 SCRA 117
ROMERO; March 6, 1998
NATURE
Petition for certiorari
FACTS
- Mrs. Martin and Petitioner Santos were both teachers at the Hagonoy Institute. Both
were married to different people. During the course of their employment, they fell in love,
and rumors about their relationship spread.
- Private respondent advised Mrs. Martin to take a leave of absence, which she ignored. A
week later, she was barred from reporting for work and was not allowed to enter
Hagonoys premises, effectively dismissing her from employment
- Mrs. Martins case for illegal dismissal was successful because the private respondent
failed to accord her the necessary due process in her dismissal.
- Meanwhile, HI set up a committee to investigate the veracity of the rumors. After 2
weeks, the committee confirmed the illicit relationship
- in view of this finding, petitioner was charged administratively for immorality and was
required to present his side of the controversy. 5 months later, he was informed of his
dismissal. He thus filed a complaint for illegal dismissal.
- After a full blown trial was conducted, the Labor Arbiter dismissed his complaint, but
awarded him money as financial assistance.
- petitioner filed an appeal with the NLRC which was dismissed for lack of merit
ISSUE

Labor Law 1
WON the illicit relationship between the petitioner and Mrs. Martin could be considered
immoral as to constitute just cause to terminate an employee under Article 282 of the
Labor Code
HELD
YES
Reasoning
- Section 94 10 of the Manual of Regulations for Private Schools: Causes of Terminating
Employment. In addition to the just cases enumerated in the Labor Code, the employment
of school personnels, including faculty, may be terminated for any of the following
causes:xxx xxx xxx E. Disgraceful or immoral conduct.
- To constitute immorality, the circumstances of each particular case must be holistically
considered and evaluated in light of the prevailing norms of conduct and applicable laws.
America jurisprudence has defined immorality as a course of conduct which offends the
morals of the community and is a bad example to the youth whose ideals a teacher is
supposed to foster and to elevate, the same including sexual misconduct. Thus, in
petitioner's case, the gravity and seriousness of the charges against him stem from his
being a married man and at the same time a teacher.
- Having an extra-marital affair is an affront to the sanctity of marriage, which is a basic
institution of society. Even our Family Code provides that husband and wife must live
together, observe mutual love, respect and fidelity. This is rooted in the fact that both our
Constitution and our laws cherish the validity of marriage and unity of the family. Our laws,
in implementing this constitutional edict on marriage and the family underscore their
permanence, inviolability and solidarity.
- As a teacher, petitioner serves as an example to his pupils, especially during their
formative years and stands in loco parentis to them. To stress their importance in our
society, teachers are given substitute and special parental authority under our laws.
- Teachers must adhere to the exacting standards of morality and decency. He must freely
and willingly accept restrictions on his conduct that might be viewed irksome by ordinary
citizens. The personal behavior of teachers, in and outside the classroom, must be beyond
reproach.
- Accordingly, teachers must abide by a standard of personal conduct which not only
proscribes the commission of immoral acts, but also prohibits behavior creating a
suspicion of immorality because of the harmful impression it might have on the students. Likewise, they must observe a high standard of integrity and honesty.
- From the foregoing, it seems obvious that when a teacher engages in extra-marital
relationship, especially when the parties are both married, such behavior amounts to
immorality, justifying his termination from employment.
Disposition Petition DISMISSED

LOVE
CHUA-QUA V CLAVE
189 SCRA 117
REGALADO; August 30, 1990
NATURE
Petition for certiorari.
FACTS
- This would have been just another illegal dismissal case were it not for the controversial
and unique situation that the marriage of herein petitioner, then a classroom teacher, to
her student who was fourteen (14) years her junior, was considered by the school
authorities as sufficient basis for terminating her services.
- Private respondent Tay Tung High School, Inc. is an educational institution in Bacolod
City. Petitioner had been employed therein as a teacher since 1963 and, in 1976 when this
dispute arose, was the class adviser in the sixth grade where one Bobby Qua was
enrolled. Since it was the policy of the school to extend remedial instructions to its
students, Bobby Qua was imparted such instructions in school by petitioner. In the course
thereof, the couple fell in love and on December 24, 1975, they got married in a civil
ceremony solemnized in lloilo City by Hon. Cornelio G. Lazaro, City Judge of Iloilo.
- Petitioner was then thirty (30) years of age but Bobby Qua, being sixteen (16) years old,
consent and advice to the marriage was given by his mother, Mrs. Concepcion Ong.
Their marriage was ratified in accordance with the rites of their religion in a church
wedding solemnized by Fr. Nick Melicor at Bacolod City on January 10, 1976.
- On February 4, 1976, private respondent filed with the subregional office of the
Department of Labor at Bacolod City an application for clearance to terminate the
employment of petitioner on the following ground: "For abusive and unethical conduct
unbecoming of a dignified school teacher and that her continued employment is inimical to
the best interest, and would downgrade the high moral values, of the school."
- Petitioner was placed under suspension without pay on March 12, 1976.

A2010

- 241 -

Disini

- Executive Labor Arbiter rendered an "Award" in favor of private respondent granting


the clearance to terminate the employment of petitioner.
- NLRC unanimously reversed the Labor Arbiter's decision and ordered petitioner's
reinstatement with backwages.
- Minister of Labor reversed the decision of theNLRC.
- Petitioner appealed the said decision to the Office of the President of the Philippines.
Presidential Executive Assistant Jacobo C. Clave, rendered its decision reversing the
appealed decision.
- However, in a resolution dated December 6, 1978, public respondent, acting on a motion
for reconsideration of herein private respondent and despite opposition thereto,
reconsidered and modified the aforesaid decision, this time giving due course to the
application of Tay Tung High School, Inc. to terminate the services of petitioner.
ISSUE
WON there is substantial evidence to prove that the antecedent facts which culminated in
the marriage between petitioner and her student constitute immorality and or grave
misconduct
HELD
NO
Ratio To constitute immorality, the circumstances of each particular case must be
holistically considered and evaluated in the light of prevailing norms of conduct and the
applicable law.
Reasoning
- Contrary to what petitioner had insisted on from the very start, what is before us is a
factual question, the resolution of which is better left to the trier of facts.
- Considering that there was no formal hearing conducted, we are constrained to review
the factual conclusions arrived at by public respondent, and to nullify his decision through
the extraordinary writ of certiorari if the same is tainted by absence or excess of
jurisdiction or grave abuse of discretion. The findings of fact must be supported by
substantial evidence; otherwise, this Court is not bound thereby.
- We rule that public respondent acted with grave abuse of discretion.
- As earlier stated, from the outset even the labor arbiter conceded that there was no direct
evidence to show that immoral acts were committed.
- Nonetheless, indulging in a patently unfair conjecture, he concluded that "it is however
enough for a sane and credible mind to imagine and conclude what transpired during
those times." In reversing his decision, the National Labor Relations Commission
observed that the assertions of immoral acts or conducts are gratuitous and that there is
no direct evidence to support such claim, a finding which herein public respondent himself
shared.
- What is revealing, however, is that the reversal of his original decision is inexplicably
based on unsubstantiated surmises and non sequiturs which he incorporated in his
assailed resolution in this wise:
". . . While admittedly, no one directly saw Evelyn Chua and Bobby Qua doing immoral
acts inside the classroom, it seems obvious and this Office is convinced that such a
happening indeed transpired within the solitude of the classroom after regular class
hours. The marriage between Evelyn Chua and Bobby Qua is the best proof which
confirms the suspicion that the two indulged in amorous relations in that place during
those times of the day..."
- With the finding that there is no substantial evidence of the imputed immoral acts, it
follows that the alleged violation of the Code of Ethics governing school teachers would
have no basis. Private respondent utterly failed to show that petitioner took advantage of
her position to court her student. If the two eventually fell in love, despite the disparity in
their ages and academic levels, this only lends substance to the truism that the heart has
reasons of its own which reason does not know. But, definitely, yielding to this gentle and
universal emotion is not to be so casually equated with immorality. The deviation of the
circumstances of their marriage from the usual societal pattern cannot be considered as a
defiance of contemporary social mores.
- It would seem quite obvious that the avowed policy of the school in rearing and
educating children is being unnecessarily bannered to justify the dismissal of petitioner.
This policy, however, is not at odds with and should not be capitalized on to defeat the
security of tenure granted by the Constitution to labor. In termination cases, the burden of
proving just and valid cause for dismissing an employee rests on the employer and his
failure to do so would result in a finding that the dismissal is unjustified.
Disposition Petition for certiorari granted. Decision of respondent annulled and set aside.

DUNCAN ASSOCIATION V GLAXO-WELLCOME


[PAGE 43]

8. VIOLATION COMPANY RULES

Labor Law 1
APARENTE SR V NLRC (COCA-COLA BOTTLERS
PHIL)
331 SCRA 82
DE LEON JR; April 27, 2000
FACTS
- Rolando Aparante, Sr. was first employed by private respondent Coca-Cola Bottlers
Phils., Inc. (CCBPI), General Santos City Plant as assistant mechanic in April 1970. He
rose through the ranks to eventually hold the position of advertising foreman until his
termination on May 12, 1988 for alleged violation of company rules and regulations. His
monthly salary at the time of his termination was P5,600.
- On November 9, 1987, Aparante drove CCBPI's advertising truck to install a panel sign.
He sideswiped Marilyn Tejero, a ten-year old girl. He brought Tejero to Heramil Clinic for
first aid treatment. As the girl suffered a 2 cm fracture on her skull which was attributed to
the protruding bolt on the truck's door, she was subsequently transferred to the General
Santos City Doctor's Hospital where she underwent surgical operation. She stayed in the
hospital for about a month.
- Five days after the accident, he reported the incident to CCBPI. At about the same time,
he submitted himself to the police authorities at Polomolok, South Cotabato for
investigation where it was discovered that he had no driver's license at the time of the
accident. In view thereof, FGU Insurance Corporation, an insurer of CCBPI's vehicles, did
not reimburse the latter for the expenses it incurred in connection with Tejero's
hospitalization a total amount of P19,534.45.
- CCBPI conducted an investigation of the incident where Aparente was given the
opportunity to explain his side and to defend himself.
On May 12, 1988, Aparente was dismissed for having violated the company rules and
regulations particularly Sec. 12 of Rule 005-858 for blatant disregard of established control
procedures resulting in company damages.
- The Labor Arbiter ordered his reinstatement without back wages. The NLRC affirmed but
reversed its ruling upon motion of CCBPI. It declared the dismissal as one for just cause
and effected after observance of due process.

A2010

- 242 -

Disini

3. YES
Ratio The law warrants the dismissal of an employee without making any distinction
between a first offender and a habitual delinquent where the totality of the evidence was
sufficient to warrant his dismissal. In protecting the rights of the laborer, the law authorizes
neither oppression nor self-destruction of the employer.
Reasoning
- Company policies and regulations, unless shown to be grossly oppressive or contrary to
law, are generally valid and binding on the parties and must be complied with until finally
revised or amended, unilaterally or preferably through negotiation, by competent authority.
The Court has upheld a company's management prerogatives so long as they are
exercised in good faith for the advancement of the employer's interest and not for the
purpose of defeating or circumventing the rights of the employees under special laws or
under valid agreements.
- First, Aparente's dismissal is justified by Company rules and regulations. It is true that his
violation of company rules is his first offense. Nonetheless, the damage caused to private
respondent amounted to more than P5,000, thus, the penalty of discharge is properly
imposable as provided by CCBPI's Code of Disciplinary Rules and Regulations.
- Second, Article 282, in order that an employer may dismiss an employee on the ground
of willful disobedience, there must be concurrence of at least two requisites: The
employee's assailed conduct must have been willful or intentional, the willfulness being
characterized by a wrongful and perverse attitude; and the order violated must have been
reasonable, lawful, made known to the employee and must pertain to the duties which he
had been engaged to discharge. We have found these requisites to be present in the case
at bar. The evidence clearly reveals the willful act of Aparente in driving without a valid
driver's license, a fact that he even tried to conceal during the investigation conducted by
CCBPI. Such misconduct should not be rewarded with re-employment and back wages,
for to do so would wreak havoc on the disciplinary rules that employees are required to
observe.
- In the instant case, we find the award to petitioner of separation pay by way of financial
assistance equivalent to 1/2 month's pay for every year of service equitable. Although
meriting termination of employment, petitioner's infraction is not as reprehensible or
unscrupulous as to warrant complete disregard for the fact that this is his first offense in an
employment that has spanned 18 long years.
Disposition Decision of the NLRC is AFFIRMED.

ISSUES
1. WON the NLRC erred in holding that CCBPI afforded petitioner due process
2. WON the NLRC erred in upholding the dismissal despite its initial finding that the
CCBPI had implicitly tolerated petitioners driving without a license
3. WON the infraction committed by petitioner warrants the penalty of dismissal despite
the fact that it was his first offense during his 18 long years of satisfactory and
unblemished service

9. CRIMINAL CASE
EFFECT OF ACQUITTAL

HELD
1. NO
Ratio The essence of due process does not necessarily mean or require a hearing but
simply a reasonable opportunity or a right to be heard or as applied to administrative
proceedings, an opportunity to explain one's side. In labor cases, the filing of position
papers and supporting documents fulfill the requirements of due process.
Reasoning
- Aparente was fully aware that he was being investigated for his involvement in the
vehicular accident that took place on November 9, 1987. It was also known to him that as
a result of the accident, the victim suffered a 2 cm fracture on her skull which led to the
latter's surgical operation and confinement in the hospital for which CCBPI incurred
expenses amounting to P19,534.45 which FGU Insurance Corporation refused to
reimburse upon finding that he was driving without a valid driver's license. Thus, being
aware of all these circumstances and the imposable sanctions under CCBPI's Code of
Disciplinary Rules and Regulations, he should have taken it upon himself to present
evidence to lessen his culpability.
2. NO
Reasoning
- According to Aparente, he informed the company that he had lost his license five months
before the accident. Notwithstanding such fact, the company allowed him to continue
driving the vehicle assigned to him. Thus, he shifts the blame to the company, claiming
that it should have simply ordered him to desist from driving the vehicle once it was
informed of the loss of his license. His contention is belied by his very own admission in
his position papers filed before the Labor Arbiter and the NLRC that the company had in
fact prohibited him from driving immediately after he lost his license, and had requested
him to secure a new license. However, through misrepresentations, he led CCBPI to
believe that he had procured another driver's license. Thus, he was permitted to drive
again.

RAMOS V NLRC
298 SCRA 225
PUNO; October 21, 1998
NATURE
Petition for certiorari to annul NLRC decision
FACTS
- In 1978, Elizabeth Ramos was employed by United States Embassy Filipino Employees
Credit Cooperative (USECO)
- In 1993, the USECO Board created an Audit and Inventory Committee to determine
whether USECO has a sound financial management and control mechanism.
- The committee found anomalies in USECOs lending transactions. Petitioner and her coemployees, Luz Coronel and Nanette Legaspi, were called to shed light on some items in
the Audit Committee Report, such as unrecorded loans, fabricated ledger, falsification of
documents, accommodations of payroll checks, encashment of check/CPAs, resigned
members, unrecorded loan of resigned members and withdrawal of more than the
deposits.
- During the meeting, Beth admitted her serious offense in regard to falsification of
documents. When asked by the Board to explain how recently resigned members and
other resigned employees in the past were able to secure loans, Beth replied that she just
wanted to help members without regard to existing policies.
- In her written explanation, Beth said that the loans are approved based on prerogatives
of individuals in authority. She said that, it is unfortunate that the USECU Staff had to
resort to creating dummy records. But since the loans are duly acknowledged by the
borrowers in other legitimate documents, it is readily apparent that the records were made
simply to accommodate those borrowers beyond the authorized limits, but never, never to
defraud USECU.

Labor Law 1
- Ramos was preventively suspended for 30 days. Later, petitioner was placed on forced
leave with pay, pending the completion of the investigation.
- USECO commissioned an external auditing firm to examine the irregularities discovered
in its lending practices. The auditor confirmed the irregularities and also discovered
shortages in bank deposits.
- USECO dismissed the petitioner for loss of trust and confidence. Petitioner countered
with a complaint for illegal dismissal, illegal suspension, underpayment of salary, moral
damages and attorneys fees.
- Labor Arbiter sustained the suspension and dismissal of petitioner but ordered the
payment of her unpaid salary.
ISSUES
1. WON there is just cause for petitioners suspension and dismissal
2. WON the NLRC committed grave abuse of discretion in granting private respondents
second motion for reconsideration
HELD
1. YES
- Position of petitioner as Management Assistant requires a high degree of trust and
confidence.
- Loss of confidence is a valid ground for dismissal of an employee. In the case at bar,
USECO proved that its loss of confidence on petitioner has a rational basis. The findings
of the labor arbiter on this factual issue are supported by the evidence.
- Petitioner's explanation that the "loan practices" were made for the benefit of the
borrowing members and not to defraud USECO cannot exonerate her. Her unsound
practices endangered the financial condition of USECO because of the possibility that the
loans could not be collected at all.
- Petitioner was not denied due process before she was suspended and later dismissed.
The records show that petitioner was called by the USECO Board of Directors and
confronted with the findings of the Audit, and Inventory Committee showing the
irregularities she committed. She was asked to explain in writing these irregularities.
Petitioner submitted her written explanation. Thus, petitioner cannot complain that she did
not understand the charges against her. She is educated and she immediately explained
her side. Due process simply demands an opportunity to be heard and this opportunity
was not denied her.
2. NO
- Section 14 of the Rules of the NLRC provides:
Section. 14. Motions for Reconsideration.--Motions for reconsideration of any order,
resolution or decision of the Commission shall not be entertained except when based
on palpable or patent errors, provided that the motion is under oath and filed within ten
(10) calendar days from receipt of the order, resolution or decision, with proof of service
that a copy of the same has been furnished, within the reglementary period, the
adverse party, and provided further that only one such motion from the same party shall
be entertained.
- The NLRC initially reversed the ruling of the labor arbiter on the grounds that: (1)
petitioner was denied procedural due process and (2) the criminal case for estafa filed
against her has been dismissed by the Manila Prosecutor's Office for insufficiency of
evidence, particularly, for lack of proof that the USECO was damaged by the acts
attributed to petitioner.
- As discussed above, petitioner was not denied due process.
- Similarly, it is a well established rule that the dismissal of the criminal case against an
employee shall not necessarily be a bar to his dismissal from employment on the
ground of loss of trust and confidence. The NLRC corrected these patent errors when
it granted private respondent's second motion for reconsideration.
Disposition Petition dismissed for lack of merit.

CONVICTION
SAMPAGUITA GARMENTS CORP V NLRC (SANTOS)
233 SCRA 260
CRUZ; June 17, 1994
NATURE
Petition for review of a resolution of the NLRC
FACTS
- Theft was claimed to have been done by Santos, employee of Sampaguita. It was
alleged she attempted to bring out a piece of cloth w/o permission.
- She was dismissed on this ground. She filed complaint for illegal dismissal. Labor
Arbiter ruled in favor of Sampaguita. NLRC reversed and ordered reinstatement.
- Sampaguita also filed criminal action against Santos. She was found guilty.
- Santos moved for execution of NLRC decision. Sampaguita opposed and invoked her
conviction in the criminal case.

A2010

- 243 -

Disini

ISSUE
WON subsequent conviction in criminal prosecution for an offense will affect a previous
administrative decision which absolved the employee of the same offense
HELD
YES
- Once judgment has become final and executory, it can no longer be disturbed except
only for correction of clerical errors or where supervening events render its execution
impossible or unjust.
- Here, the decision of NLRC ordering reinstatement had become final and executory.
Even so, we find that NLRC wasnt correct in sustaining implementation
- The affirmance by RTC and CA of private respondent's conviction is justification enough
for NLRC to exercise this authority and suspend execution of its decision. Such
conviction, also upheld by this Court is a supervening cause that rendered unjust and
inequitable the decision mandating the private respondent's reinstatement.
- Separation pay shall be allowed as a measure of social justice only in those instances
where the employee is validly dismissed for causes other than serious misconduct or
those reflecting on his moral character. A contrary rule would, as the petitioner correctly
argues, have the effect of rewarding rather than punishing the erring employee for his
offense.
- The only award to which private respondent may be entitled is for the amount as a
penalty for effecting her dismissal without complying with the procedural requirements.

DISMISSAL-CRIMINAL CASE
LACORTE V INCIONG (ESTRELLA, ASEAN
FABRICATORS INC)
166 SCRA 1
FERNAN; September 27, 1988
NATURE
Certiorari and Mandamus
FACTS
- Salvador Lacorte was hired as a warehouseman whose duties were among others, to
receive and store the raw and junk materials used by respondent in its business.
- January 19, 1977: Lacorte offered to purchase some obsolete, defective and non-usable
junk materials from AFI, who agreed and issued a cash invoice for the purchase of the
scrap items.
- When Lacorte tried to bring out these items he was accosted by AFI' s security guard
and in the course of the investigation, it was discovered that the items sought to be
brought out by complainant weighed more than what he actually purchased.
- Furthermore, it was found out that the items were not junk since some parts were brand
new and usable.
- As a consequence the respondent filed a case for qualified theft against complainant
before the Provincial Fiscal of Bulacan.
- The criminal complaint was however, dismissed for insufficiency of evidence.
- The application of AFI to terminate LACORTE was granted as the latter was found by
Labor Regional Director Estrella, to have committed certain acts in breach of the trust and
confidence of his employer.
- On appeal, Deputy Minister of Labor Amado Gat Inciong affirmed the aforementioned
order. Hence, this present recourse.
ISSUE
WON public respondents acted arbitrarily and/or with grave abuse of discretion
(considering that the criminal complaint was dismissed) connection with the grant of the
application for clearance to terminate the employment of petitioner filed by AFI
HELD
YES
- The purpose of the proceedings before the fiscal is to determine if there is sufficient
evidence to warrant the prosecution and conviction of the accused. In assessing the
evidence before him, the fiscal considers the basic rule that to successfully convict the
accused the evidence must be beyond reasonable doubt and not merely substantial.
- On the other hand, to support findings and conclusion of administrative bodies only
substantial evidence is required.
- The evidence presented before the two bodies may not be necessarily Identical.
- The appreciation of the facts and evidence presented is an exercise of discretion on the
part of administrative officials over which one cannot impose his conclusion on the other.
- Sea-Land Service, Inc. v. NLRC: The conviction of an employee in a criminal case is not
indispensable to warrant his dismissal, and the fact that a criminal complaint against the

Labor Law 1
employee has been dropped by the fiscal is not binding and conclusive upon a labor
tribunal.
- Also, the Court did not believe Lacortes claim that the real reason behind his termination
was his union activities.
- As regards Lacortes claim that there was no actual weighing and examination of the
boxes containing the scrap materials he allegedly stole, the Court ruled that it was too late
in the day for Lacorte to raise these matters of facts in this petition and that his evidence
does not substantiate his claim.
- The Court considered the records of this case as a whole, and was convinced that there
is substantial basis for the Orders issued by respondent labor officials.
Disposition Petition is dismissed for lack of merit.

GUILT OR INNOCENCE
CHUA V NLRC
218 SCRA 545
FELICIANO; February 8, 1993
NATURE
Petition for certiorari
FACTS
- The Union of Filipro Employees, of which petitioner Chua was a member, declared a
strike against the private respondent company, Nestle Philippines, Inc. During the strike,
several of the striking employees threw stones at the trucks entering and leaving the
company premises. One truck. whose driver was rendered unconscious by a stone hitting
him on the head, rammed a private vehicle and crashed into a beauty parlor resulting in
the death of three persons and extensive damage to private property. Consequently, a
criminal complaint for multiple murder and frustrated murder was filed against petitioner
and several other employees who were believed to be responsible for the stoning incident
which resulted in the deaths and property damage. The criminal complaint was dismissed
for insufficiency of evidence. The strike itself was, however, declared illegal in two
decisions of the National Labor Relations Commission (NLRC) which were affirmed by the
Supreme Court.
- Subsequently, the union and its striking members offered to return to work and were
readmitted by the company except 69 union officers and 33 union members, including
petitioner. The union's counsel wrote to the private respondent requesting the
reinstatement of five employees, including petitioner. The request, however, was denied.
Petitioner received a notice of dismissal from private respondent for having participated in
the illegal strike.
- Two days later, petitioner initiated a complaint for illegal dismissal against private
respondent company. The Labor Arbiter rendered a decision finding that petitioner had
been validly dismissed. It was held that the evidence introduced by private respondent, in
the form of the testimony of Maniego, Personnel Supervisor of its Cabuyao Plant, that he
positively saw and identified petitioner as one of the union members who actively
participated and manned the barricades during the strike is "a concrete manifestation of
an illegal act that is frowned upon by law." Wishing to be reinstated also, petitioner
appealed the Labor Arbiter's decision to the NLRC which, however, affirmed in toto the
decision of the Labor Arbiter. Hence, this petition.
ISSUE
WON the NLRC committed grave abuse of discretion in affirming the decision of the Labor
Arbiter
HELD
NO
- We find this contention to be without merit, Petitioner's participation in the illegal strike
and his commission of illegal acts while the strike was in progress, i.e., he participated in
the barricade which barred people from entering and/or leaving the employer's premises,
had been sufficiently established by substantial evidence, including the testimony of Mr.
Maniego, Personnel Supervisor at the Cabuyao Plant. Mr. Maniego testified, among other
things, that he was not able to report to work because of the presence of the barricade.
The law prohibits any person engaged in picketing from obstructing free ingress to or
egress
from
the
employer's
premises
for
lawful
purposes.
- While the criminal complaint where petitioner was included as one of the accused was
dismissed for insufficiency of evidence, the Court considers that the dismissal of the
criminal complaint did not preclude a finding by the competent administrative authorities,
that petitioner had indeed committed acts inimical to the interest of his employer.
- Private respondent's guilt or innocence in the criminal case is not determinative of the
existence of a just or authorized cause for his dismissal. This doctrine follows from the
principle that the quantum and weight of evidence necessary to sustain conviction in
criminal cases are quite different from the quantum of evidence necessary for affirmance
of a decision of the Labor Arbiter and of the NLRC.

A2010

Disini

- 244 -

- Since petitioner's participation in the unlawful and violent strike was amply shown by
substantial evidence, the NLRC was correct in holding that the dismissal of petitioner was
valid being based on lawful or authorized cause.
Disposition Petition dismissed.

10. MOONLIGHTING
AGABON V NLRC
[PAGE 35]

11. SUSPICION
EASTERN TELECOMMUNICATIONS PHILS INC V
DIAMSE
491 SCRA 239
YNAREZ-SANTIAGO; June 16, 2006
FACTS
- Maria Charina Damse is the Head of Building Services of ETPI. She requested a cash
advance of P150k for the renewal of ETPIs business permits. The companys policy is
cash advances should be liquidated 15 days from the completion of the project or activity,
or else it will be deducted from the employees salary, benefits or receivables.
- She was able to use a total of P97,151. The last payment was made on Feb 26, 2001.
She wasnt able to liquidate the cash advance within 15 days.
- On July 13, 2001, ETPIs Finance Dept advised her to liquidate the amount. She
submitted a liquidation report on August 13, 2001. This report was refused by the Fin Dept
for being late. She was told that the entire amount would just be deducted from her
monthly salary starting Sept 2001. By Dec 2001, a total of P23k had been deducted from
her salary. She then requested for reimbursement for P97,151. This was reviewed by her
supervisor and approved by HR and Fin Dept, and the amt was credited to her ATM
payroll acct.
- The Internal Audit Dept (IAD) apparently didnt know what was going on. In Jan 2002,
IAD found that her payroll acct had P86k. They required her to withdraw P52,533 for the
unliquidated amt minus the deductions. She complied. The next day, they asked her again
for P74,462.82, which is the difference bet the P97k++ credited to her acct minus the P23k
deductions. She complied again. (I dont understand how the computations were made.)
- The next day, ETPI required Diamse to explain why she should not be disciplined for
unauthorized diversion or application of company funds, and for acts of dishonesty, fraud,
deceit and willful breach of trust. She explained what that the liquidation report wasnt
accepted by the Fin Dept and she was instead advised to do as she did. A month later,
she was dismissed.
- LA ruled in her favor. NLRC reversed. CA reversed NLRC and ordered separation pay
etc instead of reinstatement because of the strained relations bet the parties.
ISSUE
WON Diamse was illegally terminated
HELD
NO
- Employer wasnt able to prove that the employee was terminated for valid and just
cause.
LOSS OF TRUST AND CONFIDENCE v. SUSPICION
- To be a valid cause for dismissal, the loss of trust and confidence must be based on a
willful breach and founded on clearly established facts. A breach is willful if it is done
intentionally, knowingly and purposely, without justifiable excuse, as distinguished from an
act done carelessly, thoughtlessly, heedlessly or inadvertently. Loss of trust and
confidence must rest on substantial grounds and not on the employer's arbitrariness,
whims, caprices or suspicion, otherwise, the employee would eternally remain at the
mercy of the employer.
- The SC held that the mere delay in the liquidation of the cash advance cannot sustain a
finding of loss of trust and confidence. It was based on mere suspicion, without evidence
to show that Diamse misappropriated funds. In fact, all documents submitted were found
to be authentic. The evidence on record shows that Diamse was able to liquidate the cash
advance and that the ensuing delay in its liquidation was attributable to ETPI.
- It cannot be presumed that Diamse misappropriated the funds because to do so would
do violence to her right to security of tenure and the well-settled rule that the burden of
proof is on the employer to establish the ground for dismissal. Suspicion has never been
a valid ground for dismissal and the employee's fate cannot, in justice, be hinged upon
conjectures and surmises.

Labor Law 1

A2010

- More suspicion with regard to the P86k in her ATM acct: The company suspected that it
came from the P97k erroneously credited to her acct. They didnt bother to prove it. They
werent able to show any bank statements to that effect.
Disposition Petition denied. CA decision affirmed and modified in that this case be
REMANDED to the Labor Arbiter for the sole purpose of computing Diamse's full
backwages, etc.

14.06 TRANSFERS
SUSPENSION

DISCHARGE

AND

LANZADERAS V AMETHYST SECURITY AND


GENERAL SERVICES INC
404 SCRA 505
QUISUMBING; June 20, 2003
FACTS
- Amethyst has been supplying guard for Resin Industrial Chemical Corp (RICC) and its
sister company Phil. Iron Construction and Marine Works (PICMW) since 1968. One
condition was that Amethyst must supply guards between 25 45 years old.
- In 1998, RICC/PICMW reminded Amethyst of this condition. Amethyst in turn required of
the guards assigned to RICC/PICMW to submit copies of their birth certificates. Those
beyond the limit were told to report to the office for reassignment.
- Amethyst was able to renegotiate with RICC/PICMW to the effect that those beyond the
age limit could be assigned as firewatch guards in the same company. (SO they had a
choice of being assigned as firewatch guards in the same company or be transferred to
Cagayan de Oro.)
- The petitioners chose neither option and didnt report for work. They filed illegal dismissal
with the LA.
- LA held Amethyst and RICC/PICMW solidarily liable for P1.25M to the petitioners. On
appeal, NLRC reversed and set aside the LAs decision on the ground that the relief of
petitioners from their posts was a legitimate exercise of prerogative on RICC/PICMWs
part.
- CA denied petitioners appeal on procedural grounds.
ISSUE
WON petitioners were illegally dismissed

HELD
NO
- In the first place, the petitioners knew of the age limit and acted in bad faith when they
werent honest about their ages.
- The condition imposed by respondent RICC/PICMW, as a principal or client of the
contractor Amethyst, regarding the age requirement of the security guards to be
designated in its compound, is a valid contractual stipulation. It is an inherent right of
RICC/PICMW, as the principal or client, to specify the qualifications of the guards who
shall render service pursuant to a service contract. It stands to reason that in a service
contract, the client may require from the service contractor that the personnel assigned to
the client should meet certain standards and possess certain qualifications, conformably to
the client's needs.
- Security of tenure, although provided in the Constitution, does not give an employee an
absolute vested right in a position as would deprive the company of its prerogative to
change their assignment or transfer them where they will be most useful. When a transfer
is not unreasonable, nor inconvenient, nor prejudicial to an employee; and it does not
involve a demotion in rank or diminution of his pay, benefits, and other privileges, the
employee may not complain that it amounts to a constructive dismissal.
- Case law recognizes the employer's right to transfer or assign employees from one area
of operation to another, or one office to another or in pursuit of its legitimate business
interest, provided there is no demotion in rank or diminution of salary, benefits and other
privileges and not motivated by discrimination or made in bad faith, or effected as a form
of punishment or demotion without sufficient cause. This matter is a prerogative inherent
in the employer's right to effectively control and manage the enterprise.
- The petitioners were given an option to stay at RICC/PICMW as firewatch guards or to
be transferred to CDO as security guards. The petitioners didnt report to the office to
receive new deployment instructions. They have no excuse not to heed managements
exercise of management prerogative.
Disposition Petition denied. CA affirmed.
Note The SC also denied on procedural grounds but went into the issues to settle the
matter completely.

- 245 -

Disini

WESTIN PHIL PLAZA HOTEL V NLRC (RODRIGUEZ)


306 SCRA 631
QUISUMBING; May 3, 1999
NATURE
Petition to review a decision of the NLRC
FACTS
- Private respondent Len Rodriguez was continuously employed by petitioner in various
capacities (pest controller, room attendant, bellman, and doorman) from July 1, 1977 until
his dismissal on February 16, 1993.
- On December 28, 1992, private respondent received a memorandum from the
management transferring him from doorman to linen room attendant. The position of
doorman is categorized as guest-contact position while linen room attendant is a nonguest contact position.
- The transfer was allegedly taken because of the negative feedback on the manner of
providing service to hotel guests by private respondent.
- Instead of accepting his new assignment, private respondent went on vacation leave
- The President of the National Union of Workers in Hotels, Restaurants and Allied
Industries (NUWHRAIN) appealed to management concerning private respondent's
transfer, but the director for human resources development, clarified that private
respondent's transfer is merely a lateral movement. She explained that management
believed that private respondent was no longer suited to be in a guest-contact position, but
there was no demotion in rank or pay.
- When private respondent reported back to work, he still did not assume his post at the
linen room.
- On February 11, 1993, private respondent was served with a memorandum asking him to
explain in writing why no disciplinary action should be taken against him for
insubordination. The memorandum noted that while private respondent regularly came to
the hotel everyday, he just stayed at the union office.
- In his reply private respondent merely questioned the validity of his transfer without
giving the required explanation.
- On February 16, 1993, petitioner terminated private respondent's employment on the
ground of insubordination.
- Private respondent filed with the Department of Labor and Employment a complaint for
illegal dismissal against petitioner.
- The labor arbiter declared that the dismissal was legal. Accordingly, the complaint was
dismissed for lack of merit.
- On appeal, public respondent reversed the judgment of the labor arbiter. It held that there
was no just cause in dismissing private respondent.
- Its motion for reconsideration having been denied, petitioner filed this instant petition.
ISSUES
1. WON private respondent was guilty of insubordination, thus giving petitioner just and
valid cause for dismissal
2. WON the order of transfer was legal
HELD
1. YES
- Under Article 282 (a) of the Labor Code, as amended, an employer may terminate an
employment for serious misconduct or willful disobedience by the employee of the lawful
orders of his employer or representative in connection with his work. But disobedience to
be a just cause for dismissal envisages the concurrence of at least two (2) requisites: (a)
the employee's assailed conduct must have been willful or intentional, the willfulness
being characterized by a wrongful and perverse attitude; and, (b) the order violated must
have been reasonable, lawful, made known to the employee and must pertain to the
duties which he has been engaged to discharge.
- In the present case, the willfulness of private respondent's insubordination was shown by
his continued refusal to report to his new work assignment: 1. Upon receipt of the order of
transfer, private respondent simply took an extended vacation leave; 2. When he reported
back to work, he did not discharge his duties as linen room attendantwhile he came to
the hotel everyday, he just went to the union office; 3. when he was asked to explain why
no disciplinary action should be taken against him, private respondent merely questioned
the transfer order without submitting the required explanation.
2. YES
- It must be emphasized that this Court has recognized and upheld the prerogative of
management to transfer an employee from one office to another within the business
establishment, provided that there is no demotion in rank or a diminution of his salary,
benefits and other privileges.
- This is a privilege inherent in the employer's right to control and manage its enterprise
effectively.
- Besides, it is the employer's prerogative, based on its assessment and perception of its
employee's qualifications, aptitudes and competence, to move him around in the various
areas of its business operations in order to ascertain where the employee will function with
utmost efficiency and maximum productivity or benefit to the company.

Labor Law 1
- An employee's right to security of tenure does not give him such a vested right in his
position as would deprive the company of its prerogative to change his assignment or
transfer him where he will be most useful.
- Petitioner is justified in reassigning private respondent to the linen room. Petitioner's right
to transfer is expressly recognized in the collective bargaining agreement between the
hotel management and the employees union as well as in the hotel employees handbook.
The transfer order was issued in the exercise of petitioner's management prerogative in
view of the several negative reports vis-a-vis the performance of private respondent as
doorman. It was a lateral movement as the positions of doorman and linen room attendant
are equivalent in rank and compensation. It was a reasonable relocation from a guest
contact area to a non-guest contact area.
Disposition Petition granted. NLRC decision reversed.

CASTILLO V NLRC (PCIB)


308 SCRA 326
GONZAGA-REYES; June 1999
NATURE
Petition for certiorari seeking to annul the NLRC Decision
FACTS
- Castillo was an employee of Philippine Commercial & International Bank (PCIB) as
Foreign Remittance Clerk. A Jordanian national, went to PCIBs Ermita branch to claim a
foreign remittance in the amount of US$2T. He paid P450 as commission charges as
computed by petitioner. Upon re-computation, the correct amount of the charges
amounted to only P248.75.
- Because of this incident, Castillo received a Memorandum regarding her
REASSIGNMENT. In line with the Banks policy on flexibility employee development and
internal control, effective immediately, you are hereby reassigned temporarily as
Remittance Clerk-Inquiry.
- She then filed with the NCR Arbitration Branch a complaint-affidavit for illegal dismissal
asking for her reinstatement as Foreign Remittance Clerk plus moral and exemplary
damages.
- She received another memorandum: Relative to your reassignment as Remittance
Clerk-Inquiry, for internal control purposes, you are hereby instructed that your specific
duties and responsibilities will be confined to handling of inquiring by phone, by walk-in
clients over the counter and to assist the FX Supervisor-Inquiry & Investigation in verifying
inquiries of correspondent banks, agencies, other banks and branches.
- Castillo claimed that there was no legal basis for her transfer and demotion order. Also,
PCIB immediately appointed another employee in her place and refused to allow petitioner
to perform her usual functions as she became a mere fixture in the office premises to her
gross humiliation. She was allegedly barred from the office premises and was thereby
constructively dismissed without any legal ground and without due process.
- Labor Arbiter ruled that Castillo was constructively dismissed, thus she was entitled to
reinstatement with full backwages without loss of seniority rights, privileges and other
rights granted by law.
- NLRC reversed LA: there was no demotion because the position to which she was
being reassigned belongs to the same job level as her former position and both positions
have the same rate of compensation.
ISSUE
WON Castillo was constructively and illegally dismissed
HELD
NO
Ratio The Court, as a rule, will not interfere with an employers prerogative to regulate all
aspects of employment which includes among others, work assignment, working methods,
and place and manner of work. It is the prerogative of the employer to transfer and
reassign employees for valid reasons and according to the requirement of its business,
provided that the transfer is not unreasonable, inconvenient, or prejudicial to the
employee, and that there is no demotion in rank or a diminution of his salary, benefits and
other privileges. An employees right to security of tenure does not give him such a vested
right in his position as would deprive the company of its prerogative to change his
assignment or transfer him where he will be most useful.
- Constructive dismissal: The employer has the burden of proving that the transfer and
demotion of an employee are for valid and legitimate grounds. Where the employer fails to
overcome this burden of proof, the employees demotion shall no doubt be tantamount to
unlawful constructive dismissal.
Reasoning
- PCIB was acting within its management prerogative to protect its interest and that of its
clients. NLRC upheld PCIBs contention that the remittance clerk payment order/collection
item is given the same weight in terms of duties and responsibilities as that of a remittance
clerk inquiry. These positions are of co-equal footing, co-important and of the same level

A2010

- 246 -

Disini

of authority and that the transfer did not entail any reduction of wages and other benefits.
This is because both positions are in fact Remittance Clerks, which, in PCIBs
classification system, are both slotted at level S-III.
- It is not true that Castillo has become a mere fixture in the office premises without any
function and was given no responsibilities. As a matter of act, had she accepted her new
position, she would have assumed a bigger responsibility, a big departure from her former
position where she merely did routine processing work.
Disposition Petition dismissed.

OSS SECURITY & ALLIED SERVICES INC V NLRC


(LEGASPI)
325 SCRA 157
DE LEON JR; February 9, 2000
NATURE
Petition for certiorarii
FACTS
- Private respondent Eden Legaspi worked as a security guard of OSS Security Agency
from June 16, 1986. On January 17, 1986 petitioner Miguel and Victoria Vasquez
acquired the assets and properties of OSS and absorbed some of its personnel, including
Legaspi, who was assigned to render security services to the different clients of petitioner.
She was last assigned at the Vicente Madrigal Condominium II located in Ayala Avenue,
Makati. In a memorandum, to petitioner, the Building Administrator of VM Condominium II,
complained of the laxity of the guards in enforcing security measures and requested that
petitioner reorganize the men and women assigned to the building to instill more discipline
and proper decorum by changing, if need be, some of the personnel, replacing, if possible,
on a temporary basis, the women complement, to find out if it would improve the service.
- In compliance therewith, petitioner issued Duty Detail Order, relieving Legaspi and
another lady security guard of their assignment at VM, for reassignment to other units or
detachments where vacancy exists. Thereafter, petitioner detailed Legaspi to the Minami
International Corporation in Taytay, Rizal for 1 month to replace lady security guard who
was on leave. However, Legaspi did not report for duty at her new assignment.
- Legaspi filed her complaint for under payment and constructive dismissal. The Labor
Arbiter upheld Legaspis position and ordered OSS to reinstate complainant to her former
position and pay the latter backwages for 18 months. Upon appeal, the NLRC affirmed
said decision. Hence, this petition.
ISSUE
WON the transfer of Legaspi was illegal and tantamount to unjust dismissal
HELD
NO
- Service-oriented enterprises, such as petitioner's business of providing security
services, generally adhere to the business adage that "the customer or client is always
right". To satisfy the interests, conform to the needs, and cater to the whims and wishes of
its clients, along with its zeal to gain substantial returns on its investments, employers
adopt means designed towards these ends. These are called management prerogatives in
which the free will of management to conduct its own affairs to achieve its purpose, takes
from. Accordingly, an employer can regulate, generally without restraint, according to its
own discretion and judgment, every aspect of business.
- In the employment of personnel, the employer can prescribe the hiring, work
assignments, working methods, time, place and manner of work, tools to be used,
processes to be followed, supervision of workers, working regulations, transfer of
employees, work supervision, lay-off of workers and the discipline, dismissal and recall of
work, subject only to limitations imposed by laws.
- Thus, the transfer of an employee ordinarily lies within the ambit of management
prerogatives. However, a transfer amounts to constructive dismissal when the transfer is
unreasonable, inconvenient, or prejudicial to the employee, and it involves a demotion in
rank or diminution of salaries, benefits and other privileges. In the case at bench, nowhere
in the record does it show that that the transfer of Legaspi was anything but done in good
faith, without grave abuse of discretion, and in the best interest of the business enterprise.
- No malice should be imputed from the fact that Legaspi was relieved of her assignment
and, a day later, assigned a new post. We must bear in mind that, unlike other contracts of
service, the availability of assignment for security guards is primarily at heart subservient
to the contracts entered into by the security agency with its client-third parties. As such,
being sidelined temporarily is a standard stipulation in employment contracts. When a
security guard is placed "off detail" or on "floating" status, in security agency parlance, it
means "waiting to be posted." Legaspi has not even been "off detail" for a week when she
filed her complaint.
- Evidence is wanting to support the Labor Arbiter's conclusion that petitioner
discriminated against private respondent when it ordered her relief and transfer of

Labor Law 1
assignment. Petitioner proved that such transfer was effected in good faith to comply with
the reasonable request of its client, Madrigal Condominium Corporation Incorporated
(MCCI), for a more disciplined service of the security guards on detail. The renewal of the
contract of petitioner with MCCI hinged on the action taken by the former on the latter's
request. Most contracts for security services stipulate that the client may request the
replacement of the guards assigned to it. Besides, a relief and transfer order in itself does
not sever employment relationship between a security guard and her agency. 29 Neither
was the transfer for any ulterior design, such as to rid itself of an undesirable worker or to
penalize an employee for his union activities and thereby defeat his right to selforganization.
- It appears that Legaspi declined the post assigned to her inasmuch as she considered it
"a booby trap of crippling and dislocating her from her employment". She lived in V. Mapa,
Sta. Mesa, Manila, and her new assigned post is in Taytay, Rizal, as against her previous
post at VM Condominium II in Makati. Her new assigned post would entail changes in her
routine, something that she was not agreeable with. But the mere fact that it would be
inconvenient for her, as she has been assigned to VM Condominium II for a number of
years, does not by itself make her transfer illegal. Even Legaspi admitted that she was
assigned to render security service to the different clients of petitioner. An employee has a
right to security of tenure, but this does not give her such a vested right in her position as
would deprive petitioner of its prerogative to change her assignment or transfer her where
her service, as security guard, will be most beneficial to the client. Thus, there was no
basis to order reinstatement and back wages inasmuch as she was not constructively
dismissed. Neither is private respondent entitled to the award of money claims for
underpayment, absent evidence to substantiate the same.

MENDOZA V RURAL BANK OF LUCBAN


433 SCRA 756
PANGANIBAN; July 7, 2004
NATURE
Petition for Review under Rule 45, ROC
FACTS
- April 1999, the Board of Directors of respondent bank (BANK) issued a Board Resolution
announcing the reshuffling of assignments, without changes in compensation and other
benefits, of several officers and employees of the bank (in line with the banks policy to
familiarize bank employees with the various phases of bank operations and to further
strengthen the existing internal control system). One of the employees assigned to a new
position is petitioner Elmer Mendoza, who was transferred from his post as an appraiser to
a Clerk for Meralco collection.
- In May, Mendoza expressed his resentment on the reshuffling, saying that it was a
demotion and that he heard intrigues that his demotion to a Clerk-Meralco collection was
due to the malicious machination of a certain public official who is the friend of the Board
chairman and with whom the relatives of Mendoza had filed a falsification case against.
He also said that he had been working for 6 years in good standing in the bank and that
the reshuffling is a blatant harassment on the part of the Board, an act which implicitly
forces him to resign, and which constitutes an unfair labor practice. He requested to have
his position as an appraiser retained. However, Mr. Daya (Board Chairman) explained in a
reply that the reshuffling is not a demotion since his compensation as an appraiser is
retained and no reductions were made. Also, Mr. Daya explained the objectives of the
reshuffling, particularly the maintenance of an effective internal control system
recommended by Bangko Sentral ng Pilipinas, and that it was the banks management
prerogative to do so. Also, Mendoza could retain position upon a formal request to the
board.
- In June, petitioner submitted 2 applications for LOA, and during his 2 nd LOA he filed a
complaint before the Arbitration Branch of NLRC for illegal dismissal, underpayment,
separation pay and damages.
- LA: in favor of Mendoza (entitled to reinstatement + full backwages/ separation pay if
reinstatement not possiblemoral + exemplary + attys fees)
- NLRC: in favor of Bank: no bad faith or malice on banks part; petitioner only feel
inconvenienced; petitioner not only employee reshuffled; no clear, competent, convincing
evidence that he holds a vested right to the position of Appraiser.
- CA: affirm (Mendozas claims self-serving, no diminution, could retain title upon formal
request, no bad faith/malice; no constructive dismissal he was the one who separated
himself from the banks employ)
ISSUES
1. WON Mendoza was constructively dismissed
2. WON the transfer of employees/ reshuffling was a valid exercise of the banks
management prerogatives
(Secondary Issues)
3. WON Serrano v. NLRC is applicable
4. WON NLRC and CA proceedings null

A2010

- 247 -

Disini

HELD
1. NO
- Findings of NLRC and CA were supported by substantial evidence
Ratio Constructive dismissal is defined as an involuntary resignation resorted to when
continued employment is rendered impossible, unreasonable or unlikely; when there is a
demotion in rank or a diminution of pay; or when a clear discrimination, insensibility or
disdain by an employer becomes unbearable to the employee.
Reasoning
- Petitioner presented no evidence to support his claims. (More on the second issue)
2. YES
Ratio In the pursuit of its legitimate business interest, management has the prerogative to
transfer or assign employees from one office or area of operation to another -- provided
there is no demotion in rank or diminution of salary, benefits, and other privileges; and the
action is not motivated by discrimination, made in bad faith, or effected as a form of
punishment or demotion without sufficient cause. This privilege is inherent in the right of
employers to control and manage their enterprise effectively. The right of employees to
security of tenure does not give them vested rights to their positions to the extent of
depriving management of its prerogative to change their assignments or to transfer them.
-Managerial prerogatives, however, are subject to limitations provided by law, collective
bargaining agreements, and general principles of fair play and justice.
-TEST of validity of transfer of employees (Blue Dairy Corporation v. NLRC): "The
managerial prerogative to transfer personnel must be exercised without grave abuse of
discretion, bearing in mind the basic elements of justice and fair play. Having the right
should not be confused with the manner in which that right is exercised. Thus, it cannot
be used as a subterfuge by the employer to rid himself of an undesirable worker. In
particular, the employer must be able to show that the transfer is not unreasonable,
inconvenient or prejudicial to the employee; nor does it involve a demotion in rank or a
diminution of his salaries, privileges and other benefits. Should the employer fail to
overcome this burden of proof, the employees transfer shall be tantamount to constructive
dismissal, which has been defined as a quitting because continued employment is
rendered impossible, unreasonable or unlikely; as an offer involving a demotion in rank
and diminution in pay. Likewise, constructive dismissal exists when an act of clear
discrimination, insensibility or disdain by an employer has become so unbearable to the
employee leaving him with no option but to forego with his continued employment."
- Employees may be transferred based on their qualifications, aptitudes and
competencies to positions in which they can function with maximum benefit to the
company.
Reasoning
- Mendozas transfer complied with the test. Transfer made in pursuit of valid objectives
(see above, 1st paragraph inside parenthesis); Mendoza was not singled out; no
diminution of salary, privileges, and other benefits.
3. NO
- No constructive dismissal, not entitled to monetary benefits as awarded in the Serrano
case.
4. NO
- Petitioners arguments regarding the Banks appeal before the NLRC filed beyond the
reglementary period was not raised in CA, thus cannot be entertained if raised for the 1 st
time.
Disposition Petition is DENIED, and the June 14, 2002 Decision and the September 25,
2002 Resolution of the Court of Appeals are AFFIRMED.

12. RESIGNATION AND EFFECTIVITY


EMCO PLYWOOD CORP V ABELGAS
[PAGE 14]
SHIE JIE CORP/SEASTER EX-IM CORP V NATIONAL
FEDERATION OF LABOR
463 SCRA 569
SANDOVAL-GUTIERREZ; July 15, 2005
FACTS
- Respondents were employed by petitioner as fish processors. Respondents staged a
walk-out and abandoned their work, bringing operations to a standstill. They were
suspended for a week. Petitioner claims that instead of coming to work, some of the
respondents submitted resignation letters and quitclaims. Petitioner then sent the rest a
notice terminating their services for abandonment of work.
- the Labor Arbiter found petitioners guilty of unfair labor practice for illegally dismissing
respondents and awarding the latter claims. On appeal, the NLRC reversed the decision.
The CA later reversed the decision again based on Article 277 which requires that the
employer prove that the termination was for a valid or just cause. Hence this petition.

Labor Law 1
ISSUE
WON respondents made valid resignations and were thus not illegally dismissed
HELD
NO
- Voluntary resignation is defined as the act of an employee, who finds himself in a
situation in which he believes that personal reasons cannot be sacrificed in favor of the
exigency of the service; thus, he has no other choice but to disassociate himself from his
employment. Acceptance of a resignation tendered by an employee is necessary to make
the resignation effective, which was not shown in the instant case.
- To constitute a resignation, it must be unconditional and with the intent to operate as
such. There must be an intention to relinquish a portion of the term of office accompanied
by an act of relinquishment.
- It is illogical that respondents would file complaints of illegal dismissal 17 days after filing
their resignation letters. Such acts negate any intention on their part to relinquish their
jobs. It was held in Molave Tours Corp. vs NLRC, By vigorously pursuing the litigation of
his action against petitioner, private respondent clearly manifested that he has no intention
of relinquishing his employment, which act is wholly incompatible to petitioners assertion
that he voluntarily resigned.
Disposition WHEREFORE the petition is DENIED

13. ABOLITION OF POSITION


BENGUET ELECTRIC COOPERATIVE V FIANZA
425 SCRA 41
YNARES-SANTIAGO; March 9, 2004
NATURE
Review on certiorari
FACTS
- Josephine Fianza had been employed with petitioner Benguet Electric Cooperative
(BENECO) as Property Custodian under the Office of the General Manager.
- BENECOs General Manager, Versoza, issued Office Order No. 42 addressed to Fianza
communicating that she is temporarily detailed to the Finance Department to assume the
duties of a Bill Distributor without any change in salary rate. This is line with their efforts to
reduce the cost of operation.
- Fianza acknowledged receipt of the letter under protest. She avers that it amounts to a
demotion because there are significant differences in the educational qualifications, work
experience, skills and job description and the working conditions of a Bill Distributor are
totally different and more strenuous and expose her to unfavourable and dangerous
circumstances, and therefore not similarly situated as that of Property Custodian.
- In response, Versoza issued a Memorandum informing her that the position of Property
Custodian may eventually phased out upon approval of the already proposed Table of
Organization as part of business decision.
- Still, Fianza refused to heed the order of the General Manager and continued to work as
Property Custodian despite successive issuance of Memorandum until the management
no longer authorized her to perform the duties and functions of a Property Custodian.
ISSUES
1. WON Fianzas transfer from Property Custodian to Bill Distributor is valid
2. WON the position of Property Custodian is abolished and WON the abolition is valid
HELD
1. YES
Ratio The management has a wide latitude to regulate, according to his own discretion
and judgment, all aspects of employment, including the freedom to transfer and reassign
employees according to the requirements of its business. However, the transfer of an
employee may constitute constructive dismissal when it amounts to an involuntary
resignation resorted to when continued employment is rendered impossible, unreasonable
or unlikely; when there is a demotion in rank and/or a diminution in pay, or when a clear
discrimination, insensibility or disdain by an employer becomes unbearable to the
employee.
2. YES
Ratio The abolition of a position deemed no longer necessary is a management
prerogative, and this Court, absent any findings of malice and arbitrariness on the part of
management, will not efface such privilege if only to protect the person holding that office.
Reasoning
- There was no showing that the position of Property Custodian was abolished in order to
single out Fianza, or that malice and ill-will attended the phasing out of the position.

14. DISHONESTY

A2010

- 248 -

Disini

NAGUIT V NLRC (MANILA ELECTRIC)


408 SCRA 617
CARPIO-MORALES; August 12, 2003
NATURE
Petition for certiorari seeking to annul and set aside the decision and resolution of the
NLRC.
FACTS
- Petitioner Aniceto W. Naguit, Jr., an employee of respondent Manila Electric Company
(MERALCO) was dismissed after 32 years of service. At the time of his dismissal, he was
Administrative Officer of MERALCO.
- On June 5, 1987, petitioner informed his Supervisor-Branch head Sofronio Ortega, Jr.
that he would render overtime work on June 6, 1987, a Saturday, and that after concluding
his field work on that day, he would proceed to Pagbilao, Quezon to accompany his wife
who was a principal sponsor to a kins wedding.
- On June 6 Naguit proceeded to his field assignment to conduct supervisory survey on
re-sequence of customers account numbers, and to supervise MERALCOs Operation
FC (apprehension of customers with illegally connected service). At 12:00 noon, he, along
with his co-employee Accounts Representative Fidel Cabuhat who drove his (petitioners)
jeep, proceeded to Pagbilao, Quezon.
- On June 8, the timekeeper prepared an Overtime Notice and the corresponding
Timesheet[9] wherein it was reflected that petitioner worked from 8:00 a.m. to 5:00 p.m. on
June 6 and 7. Petitioner corrected the documents by erasing the entries made for June 7.
The documents were approved by petitioners supervisor Ortega. Petitioner was
thereafter paid for overtime work on June 6.
- Documents including petty cash voucher covering Cabuhats alleged overtime work on
June 6 were also prepared on account of which petitioner, as custodian of petty cash,
released to Cabuhat the amount of P192.00 representing meal allowance and rental for a
jeep.
- More than two years later, petitioner received from the Legal and Investigation Staffs
Head of MERALCO a letter stating that the Special Presidential Committee (SPC) is in
receipt of information that he caused reimbursement of transportation expenses for the
work of Cabuhat not actually rendered. It requested that he report to the Ortigas office Feb
27, 1990 for the administrative proceedings.

- During the administrative proceedings, Naguit


wiaved his right to counsel and gave sworn statements
denying the charges.
- Evidence against petitioner consisted primarily of the sworn statements of Cabuhat who
was charged along with petitioner with falsification of time card; Olivia Borda, billings clerk;
and five customers of MERALCO. The statements tried to establish that, petitioner
induced Cabuhat to prepare a petty cash voucher covering expenses for meal and rental
of a jeep for the June 6 alleged conduct by the latter of field verification of Bill Omissions;
that on petitioners invitation, Cabuhat also repaired to Pagbilao, Quezon on June 6; and
that petitioner gave the petty cash payable to Cabuhat making it appear that some
collections for bill omissions were received from customers on June 6 when in fact no
such collections were ever received from the customers in whose name official receipts
were issued.
- SPC found Naguit and Cabuhat guilty of falsification of time cards under Sec. 7, par. 7 of
the Company Code on Employee Discipline. Additionally, petitioner was found guilty
under Sec. 6, par. 24 of the Code for encouraging Cabuhat to commit an act constituting a
violation of the Code.
- MERALCO thus informed petitioner that he was, for falsification of time card and
encouraging and inducing another employee to perform an act constituting a violation of
the Company Code on Employee Discipline, dismissed from the service with forfeiture of
all rights and privileges.
- Naguit filed a complaint with the NLRC against MERALCO for illegal dismissal, he
praying for reinstatement, backwages, damages, attorneys fees and other awards he is
entitled to. Labor Arbiter found for Naguit.
- Meralco appealed which reversed the Labor Arbiters decision.
Petitioners claims
> the factual findings of the Labor Arbiter clearly show that he, as an Administrative Officer,
is covered by respondent MERALCOs policy pertaining to field personnel, particularly
when he is designated to perform field assignments. As such, he did not bother to correct
the Overtime Notice which indicated that he worked from 8 a.m. to 5 p.m., albeit he
actually worked until 12 noon, the company policy being that even if an employee who had

Labor Law 1
a field assignment did not actually render 8 hours of work, he is deemed to have worked
for such duration provided he had completed the assigned task as he claims he did.
ISSUES
1. WON Naguit is guilty of falsification
2. WON NLRC committed grave abuse of discretion when they gave full credence to
Cabuhats affidavits that he was induced to claim overtime pat despite Cabuhats failure to
affirm such in the arbitral proceedings
3. WON there was valid ground for dismissal
HELD
1. NO
- The petitioner was in good faith when he did not correct the entry in the Notice of
Overtime and Timesheet reflecting that he worked up to 5pm on June 6.
- Petitioner advised his superior Ortega about his rendering overtime work the following
day, June 6, 1987, after which he would head for Pagbilao after concluding his work. If
petitioner had intended to do overtime work up to 5:00 p.m., there would have been no
need for him to advise Ortega that he would thereafter go to Pagbilao. Since Ortega never
refuted petitioners claim about his advising him of his proceeding to Pagbilao and in fact
the grant and release of petitioners overtime pay was approved by Ortega, who had the
discretion to judge the number of hours that can be foregone in light of his (Ortegas)
explanation that office personnel on field assignment forego the convenience of the office,
they [being] exposed to the heat of the sun and the like, this Court would not, as the
Labor Arbiter did not, attribute malice to petitioner. Thus, the Labor Arbiter held that
Ortega opined that half day would not be allowed. But, the fact remains that such
discretion is exercised, the limit of which was not shown to have been disseminated to the
employees, the qualifying factor being whether the job was satisfactory or not. If on the
contrary, there was indeed no such practice or, that complainant, being an office
personnel, is removed from coverage thereof and governed strictly by the time-rule such
that he would have been off at the actual completion of the assigned task, he would not
have bothered to inform his branch head - in effect a request for permission of his planned
trip to Pagbilao, Quezon thereafter. That would have been meaningless gesture on the
part of the complainant.
- With the incentive scheme or tolerance of Naguit, there is no resulting prejudice to
Meralco so to speak of nor intention on the part of complainant to cause it. What was done
was consistent with management policy on covering the overtime work in the branch.
2. YES
- In labor cases, where the adverse party is deprived of the opportunity to cross-examine
affiants, affidavits are considered hearsay unless the affiants are placed on the witness
stand to testify thereon. Cabuhats affidavits are inadmissible as evidence.
3. YES
- Naguit, despite his knowledge that Cabuhat did not hire any jeep nor conduct field
verification on June 6, released the petty cash representing Cabuhats meal allowance
and rental fee for a jeep. As custodian of the petty cash fund, he had the duty to ascertain
that the circumstances which brought about any claim therefrom were in order. He cannot
now shirk from this responsibility by indirectly pinning the blame on the approving officer
and asserting that the transgression was the result of mere inadvertence, given his
admission that he very well knew that Cabuhat did not conduct any field work on June 6,
1987, he (Cabuhat) having merely driven for him to Pagbilao.
- Petitioner thus committed dishonesty and breached MERALCOs trust, which dishonesty
calls for reprimand to dismissal under MERALCOs rules.
- Dismissal is, however, too severe as a penalty in petitioners case, given his 32 years of
service during which he had no derogatory record.
At the time petitioner was dismissed, he was still below the retirement age of employees of
MERALCO at 60. However, he is now about 65. Imposing a penalty less harsh than
dismissal and ordering his reinstatement are thus functus oficio, the Labor Arbiters order
for his reinstatement not having been executed.
Disposition Decision and Resolution of the NLRC are hereby SET ASIDE. Respondent
MERALCO is, in light of the foregoing discussions, hereby ORDERED to pay petitioner
Aniceto W. Naguit, Jr. his retirement benefits to be computed from the inception of his
service up to the time he reached 60 years of age, in accordance with its retirement plan.

CONSTRUCTIVE DISCHARGE
DEFINED
PHIL JAPAN ACTIVE CARBON CORP V NLRC
(QUINANOLA)
171 SCRA 164
GRINO-AQUINO; March 8, 1989

A2010

- 249 -

Disini

NATURE
A petition for review
FACTS
- Quinanola had been employed in Phil. Japan since January 19, 1982, as Assistant
Secretary/Export Coordinator. He was promoted to the position of Executive Sec. to the
Executive Vice President and General Manager. On May 31, 1986, for no apparent
reason at all and without prior notice to her, she was transferred to the Production
Department as Production Secretary, swapping positions with Ester Tamayo. Although the
transfer did not amount to a demotion because her salary and workload remained the
same, she believed otherwise so she rejected the assignment and filed a complaint for
illegal dismissal. LA found that the transfer would amount to constructive dismissal and
her refusal to obey the order was justified. Upon appeal to the NLRC, the Commission
approved the Labor Arbiter's decision but reduced to P10,000 the award of moral
damages and the attorney's fees to 10% of the judgment.
ISSUE
WON Quinanola was constructively and illegally dismissed as a result of her transfer or
assignment to the Office of the Production Manager even if she would have received the
same salary rank, rights and privileges
HELD
NO
- A constructive discharge is defined as: "A quitting because continued employment is
rendered impossible, unreasonable or unlikely; as, an offer involving a demotion in rank
and a diminution in pay." In this case, Quinanolas assignment as Production Secretary of
the Production Department was not unreasonable as it did not involve a demotion in rank
(her rank was still that of a department secretary) nor a change in her place of work (the
office is in the same building), nor a diminution in pay, benefits, and privileges. It did not
constitute a constructive dismissal.
- It is the employer's prerogative, based on its assessment and perception of its
employees' qualifications, aptitudes, and competence, to move them around in the various
areas of its business operations in order to "ascertain where they will function with
maximum benefit to the company. An employee's right to security of tenure does not give
him such a vested right in his position as would deprive the company of its prerogative to
change his assignment or transfer him where he will be most useful. When his transfer is
not unreasonable, nor inconvenient, nor prejudicial to him, and it does not involve a
demotion in rank or a diminution of his salaries, benefits, and other privileges, the
employee may not complain that it amounts to a constructive dismissal.
- On the other hand, we reject the petitioner's contention that the private respondent's
absence from work on June 2 to June 3, 1986 constituted an abandonment of her job in
the company resulting in the forfeiture of the benefits due her. While she was guilty of
insubordination for having refused to move out of her position as Executive Secretary to
the Executive Vice-President and General Manager of the company, dismissal from the
service would be a draconian punishment for it, as her complaint for illegal dismissal was
filed in good faith.
Disposition the decision of the NLRC insofar as it orders the petitioner to reinstate the
private respondent is affirmed, but she shall be reinstated to her position as Production
Secretary of the Production Department of petitioner's corporation without loss of seniority
rights and other privileges. The awards of backwages, moral damages and attorney's fees
to the private respondent are hereby set aside. No pronouncement as to costs.

DUSIT HOTEL NIKKO V NUWHRAIN


466 SCRA 374
CALLEJO; August 9, 2005
NATURE
Petition for review on certiorari of the Decision of the Court of Appeals
FACTS
The Case for Rowena Agoncillo
- Agoncillo was employed by the Hotel. After some time, she was promoted as Supervisor
of Outlet Cashiers and later promoted as Senior Front Office Cashier.
- The Hotel decided to trim down the number of its employees from the original count of
820 to 750.
- The Hotel offered a Special Early Retirement Program (SERP) to all its employees. It
was stated therein that the program was intended to provide employees financial benefits
prior to prolonged renovation period and, at the same time, to enable management to
streamline the organization by eliminating redundant positions and having a more efficient
and productive manpower complement.

Labor Law 1
- Union president Rasing, sought a commitment from the management that the
employees terminated due to redundancy will not be replaced by new employees; nor will
their positions be given to subcontractors, agencies or casual employees.
- A total of 243 employees, including Agoncillo, 161 of whom were Union officers and
members, were separated from the Hotels employment. As a result, the membership of
the Union was substantially reduced.
- The Hotel wrote DOLE saying that the Hotel terminated the employment of 243
employees due to redundancy. On the same day, Agoncillo was summoned by Hotel
Comptroller Reynaldo Casacop, who gave her a letter of even date informing the latter of
her separation from service due to redundancy effective close of office hours of April 30,
1996.
- Casacop advised Agoncillo to just avail of the Hotel's SERP, as embodied in the interoffice memorandum of Masuda. He informed her that she had the option to avail of the
program and that, in the meantime, he will defer the processing of her termination papers
to give her time to decide. On April 3, 1996, Agoncillo finally told Casacop that she would
not avail of the SERP benefits. By then, she had decided to file a complaint for illegal
dismissal against the Hotel.
- Meanwhile, the Hotel temporarily closed operations because of the renovation thereof.
- When news spread among the hotel employees that Agoncillo would contest her
termination before the NLRC, she was summoned by Personnel Manager Leticia
Delarmente to a conference. Delarmente and Dizon repeatedly asked Agoncillo to give
back the original copy of the April 1, 1996 termination letter. Agoncillo told them that the
letter was already in the possession of her counsel. Agoncillo was relieved when she was
given another letter of even date stating that, by reason of her non-availment of the SERP,
she was still considered an employee but on temporary lay-off due to the ongoing
renovation of the Hotel and that she will just be advised accordingly of her work schedule
when the Hotel reopens.
- Delarmente and Dizon offered to reinstate Agoncillo but not to her former position as
Senior Front Office Cashier. Agoncillo objected but informed them that she could accept
the position of Reservation Clerk. However, no response was received.
- She was told by Dizon that the Hotel was willing to reinstate her but as an Outlet
Cashier. Dizon explained that the Hotel had already hired new employees for the
positions of Reservation Clerks. Agoncillo, however, pointed out that she was already an
Outlet Cashier Supervisor before her promotion as Senior Front Office Cashier and that if
she accepted the position, it would be an unjustified demotion on her part. After
Agoncillos meeting with Dizon, the latter kept on promising to find a suitable position for
her. In those meetings, Dizon always offered reinstatement to positions that do not require
guest exposure like Linen Dispatcher at the hotel basement or Secretary of
Roomskeeping. When Agoncillo refused, Dizon just instructed her to return. Agoncillo had
no specific position or assigned task to perform.
- When the Hotel resumed operations, the Union filed a Notice of Strike for unfair labor
practice with the DOLE.
The Case for the Hotel
- Pursuant to the reorganization program, a reclassification of positions ensued upon
resumption of the Hotels operation. Consequently, the position of Agoncillo as Senior
Front Office Cashier was abolished and a new position of Guest Services Agent absorbing
its functions was created. Considering that the new position requires skills in both
reception and cashiering operations, respondent Hotel deemed it necessary to transfer
Agoncillo to another position as Outlet Cashier, which does not require other skills aside
from cashiering.
- The transfer of Agoncillo from Senior Front Office Cashier to Outlet Cashier does not
entail any diminution of salary or rank. Despite which, she vehemently refused the
transfer and insisted that she be reinstated to her former position. Since Agoncillo was not
amenable to the said transfer, she did not assume her new position and since then had
stopped reporting for work despite the Hotels patient reminder to act on the contrary.
Instead, she filed a complaint to question the prerogative of the management to validly
transfer her to another position as she considers the transfer an act of constructive
dismissal amounting to illegal termination and unfair labor practice in the form of union
busting.
ISSUE
WON Agoncillo was illegally dismissed
HELD
YES
- We agree with the contention of the petitioners that it is the prerogative of management to
transfer an employee from one office to another within the business establishment based
on its assessment and perception of the employees qualification, aptitude and
competence, and in order to ascertain where he can function with the maximum benefit to
the company. But, like other rights, there are limits thereto. The managerial prerogative to
transfer personnel must be exercised without grave abuse of discretion, bearing in mind
the basic elements of justice and fair play. Having the right should not be confused with the
manner in which that right is exercised.

A2010

- 250 -

Disini

- There is constructive dismissal when there is a demotion in rank and/or diminution in


pay; or when a clear discrimination, insensibility or disdain by an employer becomes
unbearable to the employee.
- In the present case, the Hotel recalled the termination of respondent Agoncillo when they
learned that she was going to file a complaint against them with the NLRC for illegal
dismissal. However, instead of reinstating her to her former position, she was offered the
position of Linen Dispatcher in the hotel basement or Secretary of the Roomskeeping
Section, positions much lower than that of a Supervisor of Outlet Cashiers which the
respondent held before she was promoted as Senior Front Office Cashier. With the said
positions, the respondent would not certainly be receiving the same salary and other
benefits as Senior Front Office Cashier.
- The offers by the petitioners to transfer Agoncillo to other positions were made in bad
faith, a ploy to stave off a suit for illegal dismissal. In fact, Agoncillo had not been
transferred to another position at all.
- Even assuming, for the sake of argument, that the hotel had a valid ground for dismissing
[the] complainant and that it had merely spared her such fate, the hotel is still guilty of
illegal dismissal. Had the hotel made the transfer of complainant in good faith and in the
normal course of its operation, it would have been justified. In this case, however, the
supposed transfer was made only after complainant had been earlier terminated.
Complainants statement in her affidavit that she was summoned by the hotel after news of
her plan to contest her dismissal circulated remains unrefuted. Furthermore, the hotel has
not explained why there was no official memorandum issued to complainant formally
informing her of her transfer. All these lead to only one conclusion that the alleged
transfer was not made in good faith as a valid exercise of management prerogative but
was intended as a settlement offer to complainant to prevent her from filing a case.
Disposition Petition is DENIED for lack of merit. Costs against the petitioners.

MOBILE PROTECTIVE AND DETECTIVE AGENCY V


OMPAD
458 SCRA 308
PUNO; May 9, 2005
NATURE
Petition for review on certiorari of the decision of the CA
FACTS
- Private respondent, Alberto Ompad, was employed by the petitioner as a security guard
in 1990. He was assigned to the various clients of Mobile. In June 1997, respondent was
assigned as a security guard at Manila Southwoods when he inquired from the project
manager of Southwoods if they have already paid their backwages to the security agency.
Ompad claims that when the Agency found out about his query, he was relieved from his
post and never given another assignment.
- The petitioner on the other hand claims that Ompad was assigned to another client, Valle
Verde Country Club from August 29 to October 31, 1997 after he was relieved from his
post at the Manila Southwoods. Petitioner further claims that one of the guards at Valle
Verde attested that Ompad had told her that he would earn better if he just drives his
tricycle full time. On October 15, 1997, Ompad reported for work but he was limping due
to an accident he suffered while driving his tricycle. Petitioner claims that he stopped
reporting for work after that date. On September 23, 1998, Domingo Alonzo, operations
manager of Mobile saw respondent and inquired as to whether he was still interested in
reporting for work. The petitioner allegedly answered in the negative and it was at that
time that Alonzo advised him to resign. Ompad, he claims, submitted his hand written
resignation which also was a quit claim.
- Petitioner contended that the letter of resignation was forced on him in return for monies
owed him. As he needed the money, he had no choice but to comply. He however was
only being given Pesos 5,000 which he rejected. He filed this case the following day.
- Ompad alleged that he was illegal terminated and claimed underpayment or nonpayment of wages, overtime pay, premium pay for holiday and rest day, separation pay,
etc.
- Labor Arbiter dismissed the complaint for lack of merit. The NLRC reversed the decision.
The CA also dismissed the action for reconsideration, noting that there was no
voluntariness in the acts of Ompad in submitting the resignation letters. Hence this action.
ISSUE
WON Ompad was illegally dismissed
HELD
YES
- The resignation letters of Ompad are dubious as they were written in a language
obviously not his and lopsidedly worded to free the Agency from liabilities. The affidavits
issued by the witnesses of Mobile are suspect considering that these witnesses were/are
in fact employed by the petitioner.
Reasoning

Labor Law 1
- All the documentary evidence proves that respondent was assigned to Valle Verde from
September 29 to October 31, 1997 and that he stopped reporting for work on October 16,
1997. After this period, respondent did not seem to have be given any further assignment.
- The SC ruled that while it is true that security guards may be put on floating status the
same should last for only six months. In the case at bar, there was no showing that Mobile
lacked engagements to which they can post their guards. Absent any dire exigency
justifying their failure to give respondent further assignment, the only logical conclusion is
that respondent was constructively dismissed.
- Even assuming that Mobile was justified in not immediately giving Ompad any
assignment after October, the length of time that he was put on floating status is
tantamount to constructive dismissal.
- In an illegal dismissal case, the onus probandi is on the employer to prove that the
dismissal was in fact for valid cause. It was in this case also the burden of Mobile to
submit evidence that the resignation was voluntary on the part of Ompad.
Disposition Petition dismissed.

DUNCAN ASSOCIATION V GLAXO WELLCOME


[PAGE 43]
R.P. DINGLASAN CONSTRUCTION INC V ATIENZA
433 SCRA 263
PUNO; June 29, 2004
NATURE
This is an appeal from the decision and resolution of the Court of Appeals, dated January
17, 2001 and October 30, 2002, respectively, upholding the finding of constructive
dismissal against petitioner.
FACTS
- Petitioner R.P. Dinglasan Construction, Inc. provided janitorial services to Pilipinas Shell
Refinery Corporation (Shell Corporation) in Batangas City. Private respondents Mariano
Atienza and Santiago Asi served as petitioners janitors assigned with Shell Corporation
since 1962 and 1973, respectively.
- July 7, 1994 - Dinglasan called for a meeting and informed them and 3 other employees
that their employment with Shell Corporation would be terminated effective July 15, 1994.
They were told that Dinglasan lost the bidding for janitorial services with Shell. Dinglasan
notified them that they may reapply as helpers and redeployed in other companies where
DInglasan had subsisting contracts but they would receive only a minimum wage. Atienza
refused as the offer would be a form of demotion --- they would lose their seniority status
and would not be guaranteed to work at regular hours.
- December 1994 Atienza et al filed a complaint against Dinglasan for non-payment of
salary with the DOLE district office in Batangas City.
- February 1995 - during the conciliation proceedings with the DOLE, Dinglasan sent
notices to Atienza et al informing them that they would be reinstated with Shell Corporation
as soon as they submit their barangay clearance, medical certificate, picture and
information sheet as per the new identification badge requirements of Shell Corporation.
Barangay officials met with Dinglasan to signify Atienza et als willingness to to be
reinstated bringing with them said requirements
- May 1995 Atienza et al demanded the payment of their backwages starting from July
15, 1994.
- June 1, 1995 Dinglasan notified Atienza et al they have been declared absent without
leave (AWOL) as they allegedly failed to signify their intention to return to work and submit
the badge requirements for their reinstatement.
- June 13, 1995 Atienza et al wrote Dinglasan and insisted that they had complied with
the badge requirements. Accompanied by the barangay officials, Atienza et al attempted to
meet with the officers of Dinglasan but the latter refused to dialogue with them. As proof
of their compliance with the Shell requirements, Atienza et al submitted to the DOLE their
x-ray results, dated May 17 and 19, 1995 and their barangay certification, dated May 13,
1995.
- NLRC > Atienza et al charged DInglasan with illegal dismissal and non-payment of 13th
month pay, with a claim for payment of attorneys fees and litigation expenses, and a
prayer for reinstatement with payment of full backwages from July 15, 1994.
> Dinglasan gave a different version of the incident. It allegedly informed Atienza et al and
the other affected employees that they would be deployed to petitioners other principal
companies but that their work would be different. Except for Atienza et al, all the affected
employees accepted its offer of redeployment and reported back to work. Atienza et al
failed to submit a resignation letter to signify their intention not to return to work.
- during the pendency of the labor case Dinglasan in 2 separate notices informed
Atienza et al that they could be reinstated at Shell Corporation with no diminution in their
salary provided that they submit the documents for the new identification badge
requirement of Shell Corporation. Atienza et al, however, refused to return to work until
they were paid their backwages. Consequently, Dinglasan was constrained to consider
them as having abandoned their work and to terminate their employment on September

A2010

- 251 -

Disini

19, 1995. Dinglasan, thus, justified the dismissal of Atienza et al on the grounds of gross
and habitual neglect of duties and abandonment of work.
- LABOR ARBITER > September 3, 1998, LA Andres Zavalla rendered a decision finding
that private respondents were illegally dismissed from service and ordering their
reinstatement.
- NLRC > the decision of the labor arbiter was affirmed
- CA > PROCEDURAL: petition could not prosper as petitioner failed to move for a
reconsideration of the NLRC decision; SUBSTANTIVE: upheld the findings of the labor
arbiter and the NLRC that: (1) private respondents were constructively dismissed as
petitioners offer of reassignment involved a diminution in pay and demotion in rank that
made their continued employment unacceptable; and, (2) private respondents could not
be considered to have abandoned their work; DInglasans motion for reconsideration was
denied
ISSUES
1. WON there is valid dismissal on the ground that they failed to report back to the office
and this abandoned their work
2. WON there was constructive dismissal
HELD
1. Ratio In an illegal dismissal case, the onus probandi rests on the employer to prove
that its dismissal of an employee is for a valid cause. In the case at bar, Dinglasan failed
to discharge its burden. It failed to establish that Atienza et al deliberately and unjustifiably
refused to resume their employment without any intention of returning to work.
- To constitute abandonment of work, two (2) requisites must concur: first, the employee
must have failed to report for work or must have been absent without justifiable reason;
and second, there must have been a clear intention on the part of the employee to sever
the employer-employee relationship as manifested by overt acts. Abandonment as a just
ground for dismissal requires deliberate, unjustified refusal of the employee to resume his
employment. Mere absence or failure to report for work, after notice to return, is not
enough to amount to abandonment.
Reasoning
- the evidence negates the theory that they abandoned their work.
(1) Atienza et al reported back to Dinglasans office a number of times expressing their
desire to continue working without demotion in rank or diminution of salary. This fact was
established by the corroborating testimony of barangay officials, accompanied Atienza et
al to Dinglasans office at least ten (10) times to negotiate their redeployment on more
acceptable terms.
(2) in seeking reinstatement, Atienza et al also sought the intervention of the DOLE to
arbitrate the labor issue between the parties.
(3) Atienza et al submitted the barangay clearances and x-ray results required from them
by petitioner for their reinstatement as witnessed by the barangay officials.
(4) the records would bear that Atienza et al lost no time and sought their reinstatement by
filing an illegal dismissal case against Dinglasan, which act is clearly inconsistent with a
desire to sever employer-employee relations and abandon their work.
- All these overt acts on the part of Atienza et al negate Dinglasans claim of abandonment
of work and prove beyond doubt their steadfast desire to continue their employment with
petitioner and be reinstated to their former position.
2. YES
Ratio Constructive dismissal is defined as quitting when continued employment is
rendered impossible, unreasonable or unlikely as the offer of employment involves a
demotion in rank and diminution of pay.
Reasoning
- Dinglasan committed constructive dismissal when it offered to reassign Atienza et al to
another company but with no guaranteed working hours and payment of only the minimum
wage. The terms of the redeployment thus became unacceptable for private respondents
and foreclosed any choice but to reject petitioners offer, involving as it does a demotion in
status and diminution in pay. Thereafter, for six (6) months, Atienza et al were in a floating
status. Interestingly, it was only after Atienza et al filed a complaint with the DOLE that
Dinglasan backtracked in its position and offered to reinstate Atienza et al to their former
job in Shell Corporation with no diminution in salary. Eventually, however, Dinglasan
unilaterally withdrew its offer of reinstatement, refused to meet with the Atienza et al and
instead decided to dismiss them from service.
Disposition petition is DISMISSED and the impugned decision and resolution of the
Court of Appeals, dated January 17, 2001 and October 30, 2002, respectively, are
AFFIRMED in toto.

GO V CA (MOLDEX PRODUCTS INC)


430 SCRA 358
YNARES-SANTIAGO; May 28, 2004
NATURE
Petition for review decision of CA (which set aside resolutions of NLRC)

Labor Law 1

A2010

FACTS
- Fernando Go was hired by Moldex Products Inc. in 1986 as a salesman, then, over the
years, was promoted to a Senior Sales Manager. As such officer, he was responsible for
overseeing and managing the sales force of the company such as dealing with clients,
getting orders, entering into an agreement with clients (subject to approval of higher
management).
- Sometime in 1998, the EVP o Moldex called the attention of Go regarding the discovery
of alleged anomalies purportedly committed by the sales people under Gos control. Such
anomalies stemmed from the disbursement of funds by Moldex to govt officials to secure
big supplpy contracts from the govt.
- Because of the issue, a number of employees were dismissed, including those under
Gos supervision. Go himself was terminated, allegedly on account of command
responsibility. Moldex claimed that Go, obviously feeling guilty for not exercising effective
supervision over his subordinates, submitted a letter of resignation dated October 12,
1998 but effective on November 16, 1998. Moldex added that Go went on leave from Oct
12, 1998 to Nov 16, 1998. While on leave, he worked for the release of his clearance and
the payment of 13th month pay and leave pay benefits. On the other hand, Go contends
that he was not investigated. The investigation only involved other sales people. He filed a
complaint for constructive dismissal. LA ruled for Go (there was illegal dismissal), NLRC
affirmed, but CA set aside the decisions, relying on evidence that Go was actively
performing his normal duties and functions during the months immediately prior to his
resignation, contrary to the finding of constructive dismissal.
ISSUE
WON there was constructive dismissal
HELD
NO
Ratio Constructive dismissal exists where there is a cessation of work because continued
employment is rendered impossible, unreasonable or unlikely. It is present when an
employees functions, which were originally supervisory in nature, were reduced, and such
reduction is not grounded on valid grounds such as genuine business necessity.
Reasoning
- Apparently, Go still fully exercised the prerogatives and the responsibilities of his office
as the Senior Sales Manager during the time that the said functions were supposedly
removed from him. Therefore, there can be no constructive dismissal to speak of.
- Go claims that his separation from employment with Moldex was a case of constructive
dismissal, an allegation which the company refutes with its own set of evidence pointing to
the Gos voluntary resignation.
- It should be remembered that Go has submitted a letter of resignation. It is thus
incumbent upon him to substantiate his claim that his resignation was not voluntary but in
truth was actually a constructive dismissal. This the petitioner failed to do. His bare
allegations, when uncorroborated by evidence, cannot be given credence.
- on the other hand, Moldex presented confidential sales evaluation forms that prove that
Go was still performing his duties and responsibilities one month prior to his resignation.
- While on leave, he worked for the release of his clearance and the payment of his 13 th
month pay and leave pay benefits. In doing so, he in fact performed all that an employee
normally does after he resigns. Resignation is the formal pronouncement or
relinquishment of an office. The voluntary nature of Gos acts has manifested itself clearly
belie his claim of constructive dismissal.
The totality of the evidence indubitably shows that Go resigned from employment without
any coercion or compulsion from respondent. His resignation was voluntary.
Disposition: Petition denied, and decision of CA AFFIRMED.

ACUNA V CA
[PAGE 12]

DISCHARGE

Disini

ISSUE
WON the dismissal of private respondents was a constructive dismissal or an illegal
dismissal
HELD
- Constructive dismissal or a constructive discharge has been defined as a quitting
because continued employment is rendered impossible, unreasonable or unlikely, as an
offer involving a demotion in rank and a diminution in pay. In the instant case, private
respondents were not demoted in rank nor their pay diminished considerably. They were
simply told without prior warning or notice that there was no more work for them. After
receiving the notice of hearing of the petition for certification election on 27 October 1992,
petitioners immediately told private respondents that they were no longer employed.
Evidently it was the filing of the petition for certification election and organization of a union
within the company which led petitioners to dismiss private respondents and not
petitioners' allegations of absence or abandonment by private respondents. The formation
of a labor union has never been a ground for valid termination, and where there is an
absence of clear, valid and legal cause, the law considers the termination illegal.

GLOBE TELECOM INC V FLORENDO


390 SCRA 201
September 27, 2002
NATURE
Petition for review on certiorari of a decision of CA.
FACTS
- FLORENDO, a Senior Account Manager of Globe, filed a complaint for constructive
dismissal against Globe with some key officials [GLOBE et al., for brevity] and
FLORENDOs immediate superior Cacholo Santos [SANTOS, for brevity]. FLORENDO
complained that SANTOS never submitted her performance evaluation report thereby
depriving her of salary increases and incentives which other employees of the same rank
had been receiving; reduced her to a house-to-house selling agent (i.e. a direct sales
agent) of company products ("handyphone") despite her rank as supervisor of company
dealers and agents; never supported her in the sales programs she presented; and,
withheld all her other benefits.
- GLOBE et al., on the other hand, claimed that FLORENDO abandoned her work; that her
complaint rested on a purely private disagreement with her immediate superior, and that
she filed the complaint without consulting the companys grievance process.
ISSUE
WON FLORENDO can be constructively dismissed from service

POSEIDON FISHING V NLRC


[PAGE 98]

CONSTRUCTIVE
DISMISSAL

- 252 -

(MRI), Eduardo Dayot and Susan Dayot. Private respondents sought the assistance of a
labor organization which helped them organize the Mark Roche Workers Union (MRWU).
Apparently irked by the idea of a union within the company, petitioners ordered private
respondents to withdraw the petition and further threatened them that should they insist in
the organization of a union they would be dismissed. Unfazed, private respondents
refused. As expected, private respondents were discharged from work. Petitioners
disclaimed knowledge of any deficiency owing to private respondents since all the benefits
due them as required by law were fully paid, except overtime pay which they were not
entitled to on account of their being piece-rate workers. The Labor Arbiter rendered his
decision declaring as illegal the constructive dismissal of private respondents and ordered
their reinstatement, payment of backwages, salary differentials and proportionate 13th
month pay and service incentive leave pay. On appeal, the National Labor Relations
Commission (NLRC) affirmed the decision of the Labor Arbiter, but set aside the award of
service incentive leave on the ground that private respondents were not entitled thereto as
they were piece-rate workers. Petitioners moved for reconsideration, but it was denied.
Hence, the present petition.

AND

ILLEGAL

MARK ROCHE V NLRC


313 SCRA 356
BELLOSILLO; August 31, 1999
FACTS
- On different dates, private respondents filed separate complaints for underpayment of
wages and non-payment of overtime pay against petitioners Mark Roche International

HELD
YES
Ratio Constructive dismissal exists where there is cessation of work because "continued
employment is rendered impossible, unreasonable or unlikely, as an offer involving a
demotion in rank and a diminution in pay." All these are discernible in FLORENDOs
situation. She was singularly edged out of employment by the undesirable treatment she
received from her superior, who discriminated against her without reason . (See above for
SANTOS acts against FLORENDO.) And although FLORENDO continued to have the
rank of a supervisor, her functions were reduced to a mere direct sales agent. This was
tantamount to a demotion. She might not have suffered any diminution in her basic salary
but GLOBE et al. did not dispute her allegation that she was deprived of all benefits due to
another of her rank and position, benefits which she apparently used to receive.
- Far from blaming SANTOS alone, FLORENDO also attributes her degraded state to
GLOBE et al. She cited GLOBE et al.'s indifference to her plight as she was twice left out
in a salary increase, without GLOBE et al. giving her any reason. It eludes belief that

Labor Law 1
GLOBE et al. were entirely in the dark as the salary increases were granted across-theboard to all employees except FLORENDO. It is highly improbable that the exclusion of
FLORENDO had escaped GLOBE et al.'s notice. The absence of an evaluation report
from SANTOS should have been looked into by GLOBE et al. for proper action. If a salary
increase was unwarranted, then it should have been sufficiently explained by GLOBE et
al. to FLORENDO. And despite GLOBE et al.s claim that FLORENDO did not brought her
problem against SANTOS to the company's grievance machinery, it remains
uncontroverted that FLORENDO had inquired from GLOBE et al. why her other benefits
had been withheld and sought clarification for her undeserved treatment but GLOBE and
SANTOS remained mum.
- Thus, the dispute was not a mere private spat between FLORENDO and her superior;
the case overflowed into the realm of FLORENDO's employment. And at the very least,
GLOBE et al. were negligent in supervising all of their employees.
- In constructive dismissal, the employer has the burden of proving that the transfer and
demotion of an employee are for just and valid grounds such as genuine business
necessity. The transfer must not involve a demotion in rank or a diminution of salary and
other benefits. If the employer cannot overcome this burden of proof, the employee's
demotion shall be tantamount to unlawful constructive dismissal. The award of back
wages in the instant case is justified upon the finding of illegal dismissal.
Disposition CA decision that FLORENDO abandoned her work, SET ASIDE. GLOBE et
al. to pay FLORENDO full back wages from the time she was constructively dismissed
until her reinstatement, and to cause immediate reinstatement of FLORENDO to her
former position, without loss of seniority rights and other benefits.

PREVENTIVE SUSPENSION
GLOBE-MACKAY CABLE AND RADIO CORP V NLRC
(SALAZAR)
206 SCRA 702
ROMERO; March 3, 1992
NATURE
Appeal from a decision of NLRC
FACTS
- Imelda L. Salazar was employed by Globe-Mackay Cable and Radio Corporation
(GMCR) as general systems analyst. Also employed by petitioner as manager for
technical operations' support was Delfin Saldivar with whom private respondent was
allegedly very close.
- GMCR, prompted by reports that company equipment and spare parts worth thousands
of dollars under the custody of Saldivar were missing, caused the investigation of the
latter's activities.
- The report prepared by the company's internal auditor indicated that Saldivar had
entered into a partnership styled Concave Commercial and Industrial Company with
Richard A. Yambao, owner and manager of Elecon Engineering Services (Elecon), a
supplier of petitioner often recommended by Saldivar; that Saldivar had taken petitioner's
missing Fedders airconditioning unit for his own personal use without authorization and
also connived with Yambao to defraud petitioner of its property; that Imelda Salazar
violated company regulations by involving herself in transactions conflicting with the
company's interests. Evidence showed that she signed as a witness to the articles of
partnership between Yambao and Saldivar. It also appeared that she had full knowledge of
the loss and whereabouts of the Fedders airconditioner but failed to inform her employer.
- GMCR placed Salazar under preventive suspension for 1 month, thus giving her 30 days
within which to, explain her side. But instead of submitting an explanation, private
respondent filed a complaint against petitioner for illegal suspension, which she
subsequently amended to include illegal dismissal, vacation and sick leave benefits, 13th
month pay and damages, after petitioner notified her in writing that she was considered
dismissed in view of her inability to refute and disprove the findings.
- Labor Arbiter ordered petitioner company to reinstate private respondent to her former or
equivalent position and to pay her full backwages and other benefits
- NLRC affirmed the aforesaid decision

A2010

- 253 -

Disini

ISSUES
1. WON the suspension was illegal
2. WON Art.27943 of the Labor Code should apply
HELD
1. NO
Ratio By itself, preventive suspension does, not signify that the company has adjudged
the employee guilty of the charges she was asked to answer and explain. Such
disciplinary measure is resorted to for the protection of the company's property pending
investigation any alleged malfeasance or misfeasance committed by the employee.
Reasoning
- The investigative findings of GMCR which pointed to Saldivar's acts in conflict with his
position as technical operations manager, necessitated immediate and decisive action on
any employee closely, associated with Saldivar. The suspension of Salazar was further
impelled by the discovery of the missing Fedders airconditioning unit inside the apartment
private respondent shared with Saldivar. Under such circumstances, preventive
suspension was the proper remedial recourse available to the company pending Salazar's
investigation.
- If at all, the fault, lay with Salazar when she ignored petitioner's memo giving her ample
opportunity to present her side. Instead, she filed her complaint for illegal suspension
without giving her employer a chance to evaluate her side of the controversy.
2. YES
Ratio Where a case of unlawful or unauthorized dismissal has been proved by the
aggrieved employee, or on the other hand, the employer whose duty it is to prove the
lawfulness or justness of his act of dismissal has failed to do so, then the remedies
provided in Article 279 should find, application.
Reasoning
- It must be recalled that the present Constitution has gone further than the 1973 Charter
in guaranteeing vital social and economic rights to marginalized groups of society,
including labor. To be sure, both Charters recognize "security of tenure" as one of the
rights of labor which the State is mandated to protect. But there is no gainsaying the fact
that the intent of the framers of the present Constitution was to give primacy to the rights
of labor and afford the sector "full protection," at least greater protection than heretofore
accorded them, regardless of the geographical location of the workers and whether they
are organized or not.
- that the right of an employee not to be dismissed from his job except for a just or
authorized cause provided by law has assumed greater importance under the 1987
Constitution with the singular prominence labor enjoys under the article on Social Justice.
And this transcendent policy has been translated into law in the Labor Code
- The intendment of the law in prescribing the twin remedies of reinstatement and payment
of backwages is, in the former, to restore the dismissed employee to her status before she
lost her job, for the dictionary meaning of the word "reinstate" is "to restore to a state from
which one had been removed" and in the latter, to give her back the income lost during the
period of unemployment.
- Over time, the following reasons have been advanced by the Court for denying
reinstatement under the facts of the case and the law applicable thereto; that
reinstatement can no longer be effected in view of the long passage of time or because of
the realities of the situation; or that it would be "inimical to the employer's interest; " or that
reinstatement may no longer be feasible; or that it will not serve the best interests of the
parties involved; or that the company would be prejudiced by the workers' continued
employment; or that it will not serve any prudent purpose as when supervening facts have
transpired which make execution on that score unjust or inequitable or, to an increasing
extent, due to the resultant atmosphere of "antipathy and antagonism" or "strained
relations" or "irretrievable estrangement" between the employer and the employee. In lieu
of reinstatement, the Court has variously ordered the payment of backwages and
separation pay or solely separation pay.
- If in the wisdom of the Court, there may be a ground or grounds for non-application of the
Art.279, this should be by way of exception, such as when the reinstatement may be
inadmissible due to ensuing strained relations between the employer and the employee.
- Here, it has not been proved that the position of private respondent as systems analyst is
one that may be characterized as a position of trust and confidence such that if reinstated,
it may well lead to strained relations between employer and employee. Hence, this does
not constitute an exception to the general rule mandating reinstatement for an employee
who has been unlawfully dismissed.
- To rely on the Maramara report as a basis for Salazar's dismissal would be most
inequitous because the bulk of the findings centered principally oh her friend's alleged
thievery and anomalous transactions as technical operations' support manager. Said
43

The following provision on security of tenure is embodied in Article 279 reproduced herein but with the amendments
inserted by RA 6715:
In cases of regular employment, the employer shall not terminate the services of-an employee except for a just cause or
when authorized by this Title. An employee who is unjustly dismissed from work shall be entitled to reinstatement without
loss of seniority rights AND OTHER PRIVILEGES and to his FULL backwages, inclusive of allowances, and to his other
benefits or their monetary equivalent computed from the time his compensation was withheld from him up to the time of
his ACTUAL reinstatement.

Labor Law 1
report merely insinuated that in view of Salazar's special relationship with Saldivar,
Salazar might have had direct knowledge of Saldivar's questionable activities. Direct
evidence implicating private respondent is wanting from the records.
Disposition The assailed resolution of NLRC is AFFIRMED. Petitioner GMCR is ordered
to REINSTATE Salazar and to pay her backwages equivalent to her salary for a period of
2 years only. The decision is immediately executory.

SEPARATE OPINIION
MELENCIO-HERRERA [dissent]
- I believe there is just cause for dismissal per investigative findings.

PHIL AIRLINES INC V NLRC (CASTRO)


292 SCRA 40
ROMERO; July 8, 1998
NATURE
Appeal from a decision of the NLRC affirming the decision of the LA
FACTS
- Private Respondent Edilberto Castro, an employee (manifesting clerk) of PAL was
apprehended by govt. authorities while about to board a flight to H.K. Castro and coemployee Arnaldo Olfindo were found to be in possession of P39,850 and P6,000
respectively, in violation of Central Bank (CB) Circular 265, as amended by CB Circular
383, 1 in relation to Section 34 of R.A. 265, as amended.
- Upon knowledge of this incident, PAL required respondent to explain within 24 hrs why
he should not be charged administratively. He failed to comply and was placed on
preventive suspension effective March 27, 1984 for grave misconduct. An investigation
was later conducted wherein respondent admitted ownership of the confiscated money but
denied any knowledge of CB Circular 265. Respondent, through the PAL Employees
Association (PALEA) then sought not only the dismissal of his case but also prayed for his
reinstatement.
- 3 years and 6 months after his suspension, PAL issued a resolution finding respondent
guilty of the offense charged but nonetheless reinstated the latter explaining that the
period within which he was out of work shall serve as penalty for suspension. Upon
reinstatement, respondent filed a claim against PAL for backwages and salary increases
granted under the CBA covering the period of his suspension which the latter, however,
denied on account that under the existing CBA, an employee under suspension is not
entitled to CBA salary increases granted during the period covered by his penalty.
- Labor Arbiter De Vera rendered a decision in favor of Castro; limiting his suspension to 1
month; ordering PAL to pay his salaries, benefits, and other privileges from April 26, 1984
up to Sept. 18, 1987 and to pay his salary increases accruing during the period aforesaid.
Moral damages and exemplary damages were likewise awarded. On appeal, the NLRC
affirmed the LA decision but deleted the award of moral and exemplary damages, hence,
this petition.
ISSUE
WON an employee who has been preventively suspended beyond the maximum 30-day
period is entitled to backwages and salary increases granted under the CBA during his
period of suspension
HELD
YES
- The rules are rather clear under Secs. 3 and 4, Rule XIV of the Omnibus Rules
Implementing the Labor Code:
Sec.3. Preventive suspension. The employer can place the worker concerned under
preventive suspension if his continued employment poses a serious and imminent
threat to the life or property of the employer or of his co-workers
Sec.4. Period of suspension. No preventive suspension shall last longer than 30 days.
The employer shall thereafter reinstate the worker in his former or in a substantially
equivalent position or the employer may extend the period of suspension provided that
during the period of extension, he pays the wages and other benefits due to the
workers. In such case, the worker, shall not be bound to reimburse the amount paid to
him during the extension if the employer decides, after completion of the hearing, to
dismiss the worker.
Reasoning
- It is undisputed that the period of suspension of respondent lasted for 3 years and 6
months. PAL, therefore, committed a serious transgression when it manifestly delayed the
determination of respondents culpability in the offense charged. The provisions of the
rules are explicit and direct; hence, there is no reason to further elaborate on the same.
- PAL faults the LA and the NLRC for allegedly equating preventive suspension as
remedial measure with suspension as penalty for administrative offenses. This argument
is inaccurate. As held in Beja Sr. v CA: Imposed during the pendency of an administrative

A2010

Disini

- 254 -

investigation, preventive suspension is not a penalty in itself. It is merely a measure of


precaution so that the employee who is charged may be separated, for obvious reasons,
from the scene of his alleged misfeasance while the same is being investigated. While the
former may be imposed on a respondent during the investigation of the charges against
him, the latter is the penalty which may only be meted upon him at the termination of the
investigation or the final disposition of the case. A cursory reading of the records reveals
no reason to ascribe grave abuse of discretion against the NLRC; its decision was
grounded upon petitioners manifest indifference to the plight of its suspended employee
and its consequent violation of the Implementing Rules of the Labor Code.
- As the NLRC correctly ruled: The long period of preventive suspension could even be
considered constructive dismissal because were it not for his letters demanding his
reinstatement, PAL by its inaction appeared to have no plan to employ respondent back to
work. The manifest inaction of PAL over the pendency of the administrative charge is
indeed violative of Castros security of tenure because without any justifiable cause and
due process, his employment would have gone into oblivion.
- PAL contends that when respondent consented to the resolution that the entire period of
suspension shall constitute his penalty for the offense charged, the latter is thereby
estopped to question the validity of said suspension. We concur with the labor arbiter
when he ruled that the ensuing conformity by respondent does not cure petitioner's blatant
violation of the law, and the same is therefore null and void- We do not question the right
of the petitioner to discipline its erring employees and to impose reasonable penalties
pursuant to law and company rules and regulations. Having this right, however, should
not be confused with the manner in which that right must be exercised. Thus, the exercise
by an employer of its rights to regulate all aspects of employment must be in keeping with
good faith and not be used as a pretext for defeating the rights of employees under the
laws and applicable contracts. Petitioner utterly failed in this respect.
Disposition Petition is DISMISSED for lack of merit. Assailed decision is AFFIRMED.

VALIAO V CA
[PAGE 11]
CADIZ V CA
[PAGE 224]
MARICALUM MINING CORP V DECORION
487 SCRA 182
TINGA; April 12, 2006
NATURE
Appeal from decision of CA
FACTS
- Decorion was a regular employee of Maricalum Mining who started out as a Mill
Mechanic and was later promoted to Foreman I.
- On April 11, 1996, the Concentrator Maintenance Supervisor called a meeting which
Decorion failed to attend as he was then supervising the workers under him. Because of
his alleged insubordination for failure to attend the meeting, he was placed under
preventive suspension on the same day. He was also not allowed to report for work the
following day.
- May 12, 1996, Decorion was served a Notice of Infraction and Proposed Dismissal to
enable him to present his side. On May 15, he submitted to the Personnel Department his
written reply. A grievance meeting was held upon Decorion's request on June 5, during
which he manifested that he failed to attend the meeting on April 11 because he was then
still assigning work to his men. He maintained that he has not committed any offense and
that his service record would show his efficiency.
- July 23, 1996, Decorion filed before the Labor Arbiter a complaint for illegal dismissal and
payment of moral and exemplary damages and attorney's fees.
- In the meantime, the matter of Decorion's suspension and proposed dismissal was
referred to Atty. Roman G. Pacia, Jr., Maricalum Mining's Chief and Head of Legal and
Industrial Relations, and recommended that Decorion's indefinite suspension be made
definite with a warning that a repetition of the same conduct would be punished with
dismissal. Maricalum Mining's Resident Manager issued a memorandum on August 2
placing Decorion under definite disciplinary suspension of 6 months which would include
the period of his preventive suspension which was made to take effect retroactively.
- On September 4, 1996, Decorion was served a memorandum informing him of his
temporary lay-off due to Maricalum Mining's temporary suspension of operations and shut
down of its mining operations for 6 months, with the assurance that in the event of
resumption of operations, he would be reinstated to his former position without loss of
seniority rights.

Labor Law 1
- Decorion, through counsel, wrote to Maricalum Mining on October 8, 1996, requesting
that he be reinstated to his former position. The request was denied with the explanation
that priority for retention and inclusion in the skeleton force was given to employees who
are efficient and whose services are necessary during the shutdown.
- Labor Arbiter found Decorion's dismissal illegal and ordered his reinstatement with
payment of backwages and attorney's fees. According to the labor arbiter, Decorion's
failure to attend the meeting called by his supervisor did not justify his preventive
suspension. Further, no preventive suspension should last longer than 30 days.
- The NLRC, however, reversed the labor arbiter's decision and dismissed Decorion's
complaint. The reversal was premised on the finding that the case was litigated solely on
Decorion's allegation that he was dismissed on April 11, 1996. However, during the
grievance meeting held on June 5, 1996, Decorion left it up to management to decide his
fate, indicating that as of that time, there was no decision to terminate his services yet.
According to the NLRC, to consider the events that transpired after April 11, 1996 and
make the same the basis for the finding of illegal dismissal would violate Maricalum
Mining's right to due process.
- CA reinstated decision of labor arbiter. It held that Decorion was placed under preventive
suspension immediately after he failed to attend the meeting called by his supervisor on
April 11, 1996. At the time he filed the complaint for illegal dismissal on July 23, 1996, he
had already been under preventive suspension for more than 100 days in violation of Sec.
9, Rule XXIII, Book V of the Omnibus Rules Implementing the Labor Code (Implementing
Rules) which provides that no preventive suspension shall last longer than 30 days.
-

Maricalum Minings MFR was denied.

ISSUE
WON Decorion was dismissed or merely under preventive suspension
HELD
- Decorions preventive suspension has already ripened into a constructive dismissal.
- Sections 8 and 9 of Rule XXIII, Book V of the Implementing Rules provide:
Section 8. Preventive suspension. The employer may place the worker
concerned under preventive suspension if his continued employment poses a
serious and imminent threat to the life or property of the employer or his coworkers.
Section 9. Period of Suspension No preventive suspension shall last longer
than thirty (30) days. The employer shall thereafter reinstate the worker in his
former or in a substantially equivalent position or the employer may extend the
period of suspension provided that during the period of extension, he pays the
wages and other benefits due to the worker. In such case, the worker shall not
be bound to reimburse the amount paid to him during the extension if the
employer decides, after completion of the hearing, to dismiss the worker.
- Preventive suspension is justified where the employee's continued employment
poses a serious and imminent threat to the life or property of the employer or of the
employee's co-workers. Without this kind of threat, preventive suspension is not
proper.
- Decorion was suspended only because he failed to attend a meeting called by his
supervisor. There is no evidence to indicate that his failure to attend the meeting
prejudiced his employer or that his presence in the company's premises posed a
serious threat to his employer and co-workers. The preventive suspension was
unjustified.
- Decorion's suspension persisted beyond the 30-day period allowed by the
Implementing Rules. In Premiere Development Bank v. NLRC the Court ruled that
preventive suspension which lasts beyond the maximum period allowed by the
Implementing Rules amounts to constructive dismissal.
- At the time Decorion filed a complaint for illegal dismissal, he had already been
suspended for 103 days. Decorion's preventive suspension had already ripened
into constructive dismissal at. While actual dismissal and constructive dismissal do
take place in different fashion, the legal consequences they generate are identical.
His employment may not have been actually terminated in the sense that he was
not served walking papers but there is no doubt that he was constructively
dismissed as he was forced to quit because continued employment was rendered
impossible, unreasonable or unlikely by Maricalum Mining's act of preventing him
from reporting for work.
- Article 286 of the Labor Code, which provides that the bona fide suspension of
the operation of a business or undertaking for a period not exceeding six (6)
months shall not terminate employment, may not be applied in this case. The
instant case involves the preventive suspension of an employee not by reason of
the suspension of the business operations of the employer but because of the
employee's failure to attend a meeting. The allowable period of suspension in such
a case is only 30 days as provided by the Implementing Rules.
Disposition Petition denied. CAs decision affirmed.

RATIONALE

A2010

- 255 -

Disini

KWIKWAY ENGG WORKS V NLRC (VARGAS)


195 SCRA 526
MEDIALDEA; March 22, 1991
FACTS
- Respondent Vargas was formerly employed by Kwikway as bookkeeper and secretary.
As bookkeeper, it was her duty to fill up the check vouchers and indicate therein the name
of the customer agent and the amount payable to each before they are presented to the
agents for signing.
- The new branch manager (BM) discovered that several blank vouchers already
contained the signatures of the mechanic agents. BM confronted the branch cashier in
charge of the vouchers, Marina Corpus, concerning the irregularity. Corpus explained that
Vargas was aware of this practice. When asked for an explanation, Vargas stated that the
procedure has been the practice in that office since the time of the former branch manager
who had knowledge thereof.
- BM informed the head office with his discovery. Kwikways VP conducted an
investigation. On the following day, Vargas and Corpus were placed under preventive
suspension for an indefinite period of time on the ground of loss of trust and confidence.
- Vargas was informed of the result of the investigation. Kwikway offered her a chance to
resign with separation pay, which she accepted.
- The Labor Arbiter rendered a decision directing the reinstatement of respondent Vargas
to her former position with backwages. NLRC affirmed the decision of the labor arbiter.
- Petitioner: the nature of the position of Vargas involves trust and confidence. That
private respondent's acts of dishonesty as well as her active participation in violating and
infringing company accounting procedure which allowed the cashier to personally
misappropriate sums of money provide sufficient basis for dismissing respondent. That
Vargas was aware that her cashier Corpus was committing acts of dishonesty and
misappropriation of company funds but she did not report the matter to her superiors in the
company. That the actuations of Vargas were in violation of the company's code of
conduct, which is punishable by dismissal.
ISSUES
1. WON the dismissal of respondent Vargas was for a just and valid cause
2. WON respondent Vargas was deprived of her constitutional right to due process
HELD
1. YES
- The rule is settled that if there is sufficient evidence to show that the employee has been
guilty of breach of trust or that his employer has ample reason to distrust him, the labor
tribunal cannot justly deny to the employer the authority to dismiss such employee.
- Jurisprudence abounds with cases recognizing the right of the employer to dismiss the
employee on loss of confidence. More so in the case of supervisors or personnel
occupying positions of responsibility, loss of trust justifies termination
- The mere existence of basis for believing that the employee has breached the trust of
employer is sufficient and does not require proof beyond reasonable doubt.
- Clearly, respondent Vargas's position involves a high degree of responsibility requiring
trust and confidence. Her position carries with it the duty to observe proper company
procedures in the fulfillment of her job as it relates closely to the financial interests of the
company.
2. YES
Notice and Hearing
- The twin requirements of notice and hearing constitute essential elements of due
process in cases of employee dismissal: the requirement of notice is intended to inform
the employee concerned of the employer's intent to dismiss and the reason for the
proposed dismissal; upon the other hand, the requirement of hearing affords the employee
an opportunity to answer his employer's charges against him accordingly to defend
himself therefrom before dismissal is effected. Neither of these two requirements can be
dispensed with without running afoul of the due process requirement of the 1987
Constitution.
- In the instant case, the records are bereft of any indication that a formal notice of the
charge was given to the respondent prior to the suspension or that the said investigation
gave adequate opportunity to the respondent to defend herself. It is important to stress
that an employee whose services are sought to be terminated, has the right to be informed
beforehand of his proposed dismissal or suspension as well as of the reasons therefor and
to be afforded an adequate opportunity to defend himself from the charges leveled against
him. We give respect to the following conclusions of the labor arbiter and respondent
Commission:
It is patent from the respondent's submission that written notice specifying the causes
for termination was never furnished to complainant. Neither does it appear that she
was given enough opportunity to explain her side and defend herself with the
assistance of a representative of her choice is she so desires.
Preventive Suspension

Labor Law 1
- Further, the preventive suspension of respondent Vargas for an indefinite period
amounted to a dismissal and is violative of Section 4, Rule XIV of the Implementing Rules
of the Labor Code which limits the preventive suspension to thirty (30) days. The said rule
also provides that "the employer shall thereafter reinstate the worker in his former or in a
substantially equivalent position or the employer may extend the period of suspension
provided that during the period of extension, he pays the wages and other benefits due to
the worker." (Pacific Cement Company Inc. v. NLRC
Disposition the petition GRANTED. The questioned decision of the respondent NLRC
insofar as it ordered the reinstatement of respondent Rosalinda Vargas with payment of
three (3) years backwages is REVERSED and SET ASIDE.
- Petitioner company is ordered to pay an indemnity of P1,000.00 to respondent Vargas
(For failure of the employer to comply with the requirements of due process in terminating
the employees service, it shall be liable to indemnify the employee in the sum of
P1,000.00 as damages)

GATBONTON V NLRC (MIT, CALDERON)


479 SCRA 416
AUSTRIA-MARTINEZ; January 23, 2006
NATURE
Petition for review on certiorari
FACTS
- Gatbonton is a professor at the Mapua Institute of Technology, a member of the Faculty
of Civil Engineering. In November 1998, a student filed a complaint of sexual harassment
against Gatbonton. He was then placed under preventive suspension for 30 days pending
investigation.
- Gatbonton filed a complaint against MIT for illegal suspension.
- Petitioner questioned the validity of the administrative proceedings with the Manila RTC
but the case was terminated on May 21, 1999 when the parties entered into a compromise
agreement wherein MIT agreed to publish in the school organ the rules and regulations
implementing the Anti-Sexual Harassment Act. The Labor Arbiter later declared that the
preventive suspension imposed was illegal.
- Both respondents and petitioner filed their appeal from the Labor Arbiters Decision, with
Gatbonton questioning the dismissal of his claim for damages. The NLRC favorable
granted the appeal of respondent MIT. Gatbonton.
ISSUE
WON the NLRC erred in dismissing Gatbontons claim for damages stemming from an
alleged illegal suspension
HELD
YES
Ratio Preventive suspension is a disciplinary measure for the protection of the companys
property pending investigation of any alleged malfeasance or misfeasance committed by
the employee. The employer may place the worker concerned under preventive
suspension if his continued employment poses a serious and imminent threat to the life or
property of the employer or of his co-workers. However, when it is determined that there is
no sufficient basis to justify an employees preventive suspension, the latter is entitled to
the payment of salaries during the time of preventive suspension.
Reasoning
- R.A. No. 7877 imposed the duty on educational or training institutions to promulgate
rules and regulations in consultation with and jointly approved by the employees or
students or trainees, through their duly designated representatives, prescribing the
procedures for the investigation of sexual harassment cases and the administrative
sanctions therefor.
- Petitioners preventive suspension was based on respondent MITs Rules and
Regulations for the Implemention of the Anti-Sexual Harassment Act which provides that
any member of the educational community may be placed immediately under preventive
suspension during the pendency of the hearing of the charges of grave sexual harassment
against him if the evidence of his guilt is strong and the school head is morally convinced
that the continued stay of the accused during the period of investigation constitutes a
distraction to the normal operations of the institution or poses a risk or danger to the life or
property of the other members of the educational community.
- But the said rules and regulations were published only on February 23, 1999.
- The Mapua Rules is one of those issuances that should be published for its effectivity,
since its purpose is to enforce and implement R.A. No. 7877, which is a law of general
application. Thus, at the time of the imposition of petitioners preventive suspension on
January 11, 1999, the Mapua Rules were not yet legally effective, and therefore the
suspension had no legal basis.
- The Court also finds that there is insufficient legal basis to justify the preventive
suspension because of the absence of the required requisites. Under the Mapua Rules,

A2010

Disini

- 256 -

an accused may be placed under preventive suspension during pendency of the hearing
under any of the following circumstances:
(a) if the evidence of his guilt is strong and the school head is morally convinced that the
continued stay of the accused during the period of investigation constitutes a distraction to
the normal operations of the institution; or
(b) the accused poses a risk or danger to the life or property of the other members of the
educational community.
Disposition
petition is PARTIALLY GRANTED. The decision of Labor Arbiter is
reinstated while the decisions of the CA and the NLRC are set aside.

NUMBER OFFENSES
APARENTE V NLRC
[PAGE 240]

OTHER
CAUSES

CAUSES,

BUSINESS

RELATED

RECOGNITION OF RIGHT BUSINESS-RELATED


CAUSES/[PROTECTION
AGABON V NLRC
[PAGE 35]
UICHICO V NLRC
273 SCRA 35
HERMOSISIMA JR; June 2, 1997
NATURE
Special civil action for certiorari
FACTS
- Private respondents were employed by Crispa, Inc. for many years in the latter's
garments factory. Their services were terminated on the ground of retrenchment due to
alleged serious business losses suffered by Crispa, Inc. in the years immediately
preceding 1990.
- Three (3) separate complaints for illegal dismissal and diminution of compensation
against Crispa, Inc., Valeriano Floro (a major stockholder and director), and the petitioners
(high-ranking officers and directors).
- Labor Arbiter rendered a decision dismissing the complaints for illegal dismissal but at
the same time ordering Crispa, Inc., Floro and the petitioners to pay respondent
employees separation pays equivalent to seventeen (17) days for every year of service
- The NLRC found Crispa, Inc., Valeriano Floro, together with the petitioners liable for
illegal dismissal and modified the award of separation pay in the amount of one (1) month
for every year of service instead of seventeen (17) days.
- Private respondents sought a clarification of public respondent NLRC's Resolution dated
September 30, 1993 insofar as the computation of separation pay by the Examination and
computation division was concerned as well as the failure of the Resolution to award them
full backwages despite the finding of illegal dismissal.
- On April 21, 1995, the NLRC, treating the Motion to Clarity Judgment as an Appeal,
granted the same in this wise: hereby directed to include in the computation, six months
backwages.
- Petitioners filed a Motion for Reconsideration of the April 21, 1995 Resolution, which was
denied. Hence, this petition
ISSUES
1. WON Respondents were illegally dismissed (i.e. there was no valid retrenchment)
2. WON petitioners may be held solidarily liable with the company
HELD
1. YES
Reasoning
- Article 28344 of the Labor Code covers retrenchment.
44

Art. 283. Closure of establishment and reduction of personnel. - The employer may also terminate the employment of
any employee due to the installation of labor saving devices, redundancy, retrenchment to prevent losses or the closing
or cessation of operation of the establishment or undertaking unless the closing is for the purpose of circumventing the
provisions of this Title, by serving a written notice on the worker and the Ministry of Labor and Employment at least one

Labor Law 1
Retrenchment, or "lay-off" in layman's parlance, is the termination of employment
initiated by the employer through no fault of the employee's and without prejudice to the
latter, resorted to by the management during periods of business recession, industrial
depression, or seasonal fluctuations, or during lulls occasioned by lack of orders,
shortage of materials, conversion of a plant for a new production program or the
introduction of new methods or more efficient machinery, or of automation.
Simply put, it is an act of employer of dismissing employees because of losses in the
operation of a business, lack of work, and considerable reduction on the volume of his
business, a right consistently recognized and affirmed by this court.
- Any claim of actual or potential business losses must satisfy certain established
standards before any reduction of personnel becomes legal, viz:
1. The losses expected and sought to be avoided must be substantial and not merely
de minimis in extent;
2. The substantial losses apprehended must be reasonably imminent, as such
imminence can be perceived objectively and in good faith by the employer;
3. The retrenchment must be reasonably necessary and likely to effectively prevent the
expected losses.
4. The alleged losses. If already realized, and the expected imminent losses sought to
be forestalled, must be proved by sufficient and convincing evidence
- The Statement of Profit and Losses submitted by Crispa, Inc. to prove its alleged losses,
without the accompanying signature of a certified public accountant or audited by an
independent auditor, are nothing but self-serving documents which ought to be treated as
a mere scrap of paper devoid of any probative value.
2. YES
Reasoning
- The general rule is that obligations incurred by the corporation, acting through its
directors, officers and employees, are its sole liabilities. There are times, however, when
solidary liabilities may be incurred but only when exceptional circumstances warrant such
as in the following cases:
1. When directors and trustees or, in appropriate cases, the officers of a corporation:
(a) vote for or assent to patently unlawful acts of the corporation; (b) act in bad faith or
with gross negligence in directing the corporate affairs; (c) are guilty of conflict of
interest to the prejudice of the corporation, its stockholders or members, and other
persons
- In labor cases, particularly, corporate directors and officers are solidarily liable with the
corporation for the termination of employment of corporate employees done with malice or
in bad faith. In this case, it is undisputed that petitioners have a direct hand in the illegal
dismissal of respondent employees.
Disposition Petition is DISMISSED.

FILIPINAS V GATLABAYAN
487 SCRA 673
CALLEJO; April 19, 2006
NATURE
Petition for Review on Certiorari of the decision of the CA
FACTS
- Filipinas Systems FILSYSTEMS, Inc. (Filsystems) is a domestic corporation engaged in
the construction business. Filsystems employed complainants as grillers, carpenters and
laborers on various dates from 1992 to July 24, 1999. Their average length of service with
Filsystems ranged from two to nine years. Their latest and last assignment was in
Meralco, in Antipolo, after which they reported back to the main plant. Complainants
narrated that they were verbally informed that their services were no longer needed on
July 24, 1999 (considering that they were only project employees and the project to which
they were last assigned had already been completed). The said employees filed their
respective complaints for illegal dismissal against Filsystems.
- Filsystems, on the other hand, averred that the complainants were employed merely as
project employees.
ISSUE
WON the employees were legally dismissed by reason of retrenchment
HELD
NO
- Petitioners claim that respondents were retrenched must be rejected. Indeed, petitioners
even failed to adduce in evidence any notes addressed to respondents notifying them that

(1) month before the intended date thereof. In case of termination due to the installation of labor saving devices or
redundancy, the worker affected thereby shall be entitled to a separation pay equivalent to at least his one (1) month pay
or to at least one (1) month pay for every year of service, whichever is higher. In case of retrenchment to prevent losses
and in cases of closures or cessation of operations of establishment or undertaking not due to serious business losses or
financial reverses, the separation pay shall be equivalent to one (1) month pay or at least one-half (1/2) month pay for
every year of service, whichever is higher. A fraction of at least six (6) months shall be considered as one (1) whole
year."

A2010

- 257 -

Disini

their employment was terminated on the ground of retrenchment. In Sebuguero v.


National Labor Relations Commission, this Court said:
The requirement of notice to both the employees concerned and the Department of
Labor and Employment (DOLE) is mandatory and must be written and given at least
one month before the intended date of retrenchment. In this case, it is undisputed that
the petitioners were given notice of the temporary lay-off. There is, however, no
evidence that any written notice to permanently retrench them was given at least one
month prior to the date of the intended retrenchment. ..But what the law requires is a
written notice to the employees concerned and that requirement is mandatory. The
notice must also be given at least one month in advance of the intended date of
retrenchment to enable the employees to look for other means of employment and
therefore to ease the impact of the loss of their jobs and the corresponding income.
That they were already on temporary lay-off at the time notice should have been given
to them is not an excuse to forego the one-month written notice because by this time,
their lay-off is to become permanent and they were definitely losing their employment.
- Moreover, the minutes of the meeting on August 22, 2000 (supposedly between the
management of petitioner corporation, and rank and file and supervisory employees),
show that only 13 rank and file employees attended. There is no evidence on record that
respondents attended the meeting. There is likewise no evidence on record that
petitioners complied with the requirements of Article 283 of the Labor Code of the
Philippines, which reads:
Article 283. Closure of establishment and reduction of personnel The employer may
also terminate the employment of any employee due to the installment of labor saving
devices, redundancy, retrenchment to prevent losses or the closing or cessation of
operation of the establishment or undertaking unless the closing is for the purpose of
circumventing the provisions of this Title, by serving a written notice on the workers and
the Ministry of Labor and Employment at least one (1) month before the intended date
thereof. In case of termination due to the installation of labor saving devices or
redundancy, the worker affected thereby shall be entitled to a separation pay equivalent
to at least one (1) month pay or to at least one (1) month pay for every year of service,
whichever is higher. In case of retrenchment to prevent losses and in cases of
closures or cessation of operations of establishment or undertaking not due to serious
business losses or reverses, the separation pay shall be equivalent to one (1) month
pay or at least one half (1/2) month pay for every year of service, whichever is higher.
A fraction of at least six (6) months shall be considered one (1) whole year.
- The general standards in retrenchment are: firstly, the losses expected should be
substantial and not merely de minimis in extent. If the loss purportedly sought to be
forestalled by retrenchment is clearly shown to be insubstantial and inconsequential in
character, the bona fide nature of the retrenchment would appear to be seriously in
question. Secondly, the substantial loss apprehended must be reasonably imminent, as
such imminence can be perceived objectively and in good faith by the employer. There
should, in other words, be a certain degree of urgency for the retrenchment, which is after
all a drastic recourse with serious consequences for the livelihood of the employees
retired or otherwise laid-off. Because of the consequential nature of retrenchment, it must,
thirdly, be reasonably necessary and likely to effectively prevent the expected losses. The
employer should have taken other measures prior or parallel to retrenchment to forestall
losses, i.e., cut other costs than labor costs. An employer who, for instance, lays off
substantial numbers of workers while continuing to dispense fat executive bonuses and
perquisites or so-called golden parachutes, can scarcely claim to be retrenching in good
faith to avoid losses. The employers prerogative to bring down labor costs by retrenching
after less drastic means e.g., reduction of both management and rank-and-file bonuses
and salaries, going on reduced time, improving manufacturing efficiencies, trimming of
marketing and advertising costs, etc. have been tried and found wanting.
- Alleged losses if already realized, and the expected imminent losses sought to be
forestalled, must be proved by sufficient and convincing evidence. The reason for
requiring this quantum of proof is readily apparent: any less exacting standard of proof
would render too easy the abuse of this ground for termination of services of employees.
Thus, petitioner failed to adduce a morsel of evidence to prove the factual basis for a valid
retrenchment.
Disposition Petition denied

BUSINESS SERVICES OF THE FUTURE TODAY INC V


CA
480 SCRA 571
QUISUMBING; January 30, 2006
NATURE
Petition for review on certiorari the decision of the Court of Appeals declaring the
dismissal of the private respondents illegal and awarding to them separation pay,
backwages, and 13th month pay, and attorneys fees.
FACTS
- Mailboxes, Etc. (Davao) is the local franchisee of Mailboxes, Etc. (MBE), a US-based

Labor Law 1
corporation operating business support and communication service centers worldwide. It
is operated locally by petitioner Business Services of the Future Today, Inc. (BSFTI),
whose stockholders are petitioner Ramon Allado and his nominees.
- On January 8, 1996, Allado hired private respondents, spouses Gilbert and Ma.
Celestina Veruasa, as manager and assistant manager, respectively, of Mailboxes, Etc.
(Davao). During the course of their employment the spouses advances exceeded their
unpaid salaries by P43,402.54.
- On January 8, 1998, Allado personally gave notices of termination effective immediately
to the spouses. They gave as reason, the negative cashflow and BSFTIs failure to infuse
additional capital to the business. No written notice of closure of business was given to the
Department of Labor and Employment (DOLE). Allado then padlocked the offices of
Mailboxes, Etc. (Davao), confiscated all its business records, and appropriated for himself
all the transferable rights and equipment of the office.
- Despite repeated demands, the petitioners did not pay the alleged balance of
P129,488.93 due to the spouses. Thereafter, the Veruasa spouses instituted a complaint
for illegal dismissal.
ISSUE
WON the dismissal was illegal
HELD
NO
- The BSFTIs closure was bona fide and under ART. 283. of the Labor Code closure of
establishment is a valid ground for termination of employment. The records before us
revealed that it suffered losses from 1996 to 1998. The failure of the petitioners to notify
the DOLE through a written notice of the termination of its corporate life should not nullify
the dismissal, or render it illegal, or ineffectual. However, the employer should indemnify
the employee, in the form of nominal damages, for the violation of his right to statutory due
process.
Ratio Article 283 of the Labor Code is the applicable law. It states,
ART. 283. Closure of establishment and reduction of personnel. The employer may
also terminate the employment of any employee due to the installation of labor saving
devices, redundancy, retrenchment to prevent losses or the closing or cessation of
operation of the establishment or undertaking unless the closing is for the purpose of
circumventing the provisions of this Title, by serving a written notice on the worker and
the Ministry of Labor and Employment at least one (1) month before the intended date
thereof. In case of termination due to the installation of labor saving devices or
redundancy, the worker affected thereby shall be entitled to a separation pay equivalent
to at least his one (1) month pay or to at least one (1) month pay for every year of
service, whichever is higher. In case of retrenchment to prevent losses and in cases of
closures or cessation of operations of establishment or undertaking not due to serious
business losses or financial reverses, the separation pay shall be equivalent to one (1)
month pay or at least one-half (1/2) month pay for every year of service, whichever is
higher. A fraction of at least six (6) months shall be considered as one (1) whole year.
- For the cessation of business operations due to serious business losses or financial
reverses to be valid, the employer must give the employee and the DOLE written notices
30 days prior to the effectivity of his separation.
- However. in Agabon v. National Labor Relations Commission,[15] we ruled that where
the dismissal is for an authorized cause, the lack of statutory due process should not
nullify the dismissal, or render it illegal, or ineffectual. However, the employer should
indemnify the employee, in the form of nominal damages, for the violation of his right to
statutory due process. The amount of such damages is addressed to the sound discretion
of the Court, taking into account the relevant circumstances.
Disposition Petition partially granted.

A. INSTALLATION OF LABOR SAVING DEVICES


B. REDUNDANCY

BUSINESS JUDGMENT
WILTSHIRE FILE CO INC V NLRC
193 SCRA 665
FELICIANO; February 7, 1991
NATURE
Petition for review on certiorari
FACTS
- Private respondent Vicente T. Ong was the Sales Manager of petitioner Wiltshire File
Co., Inc. ("Wiltshire") from 16 March 1981 up to 18 June 1985. On 13 June 1985, upon
private respondent's return from a business and pleasure trip abroad, he was informed by

A2010

- 258 -

Disini

the President of petitioner Wiltshire that his services were being terminated. Private
respondent maintains that he tried to get an explanation from management of his
dismissal but to no avail. On 18 June 1985, when private respondent again tried to speak
with the President of Wiltshire, the company's security guard handed him a letter which
formally informed him that his services were being terminated upon the ground of
redundancy.
- Private respondent filed, on 21 October 1985, a complaint before the Labor Arbiter for
illegal dismissal alleging that his position could not possibly be redundant because nobody
(save himself) in the company was then performing the same duties. Private respondent
further contended that retrenching him could not prevent further losses because it was in
fact through his remarkable performance as Sales Manager that the Company had an
unprecedented increase in domestic market share the preceding year. For that
accomplishment, he continued, he was promoted to Marketing Manager and was
authorized by the President to hire four (4) Sales Executives five (5) months prior to his
termination.
- In its answer, petitioner company alleged that the termination of respondent's services
was a cost-cutting measure: that in December 1984, the company had experienced an
unusually low volume of orders: and that it was in fact forced to rotate its employees in
order to save the company. Despite the rotation of employees, petitioner alleged; it
continued to experience financial losses and private respondent's position, Sales Manager
of the company, became redundant.
- On 2 December 1986, during the proceedings before the Labor Arbiter, petitioner, in a
letter 1 addressed to the Regional Director of the then Ministry of Labor and Employment,
notified that official that effective 2 January 1987, petitioner would close its doors
permanently due to substantial business losses.
- In a decision dated 11 March 1987, the Labor Arbiter declared the termination of private
respondent's services illegal and ordered petitioner to pay private respondent backwages,
unpaid salaries in the amount of, accumulated sick and vacation leaves in the amount of,
hospitalization benefit package in the amount, unpaid commission in the amount of, moral
damages in the amount of and attorney's fees in the amount of. On appeal by petitioner
Wiltshire, the National Labor Relations Commission ("NLRC") affirmed in toto on 9
February 1988 the decision of the Labor Arbiter.
- In this Petition for Certiorari, it is submitted that private respondent's dismissal was
justified and not illegal. Petitioner maintains that it had been incurring business losses
beginning 1984 and that it was compelled to reduce the size of its personnel force.
Petitioner also contends that redundancy as a cause for termination does not necessarily
mean duplication of work but a "situation where the services of an employee are in excess
of what is demanded by the needs of an undertaking
ISSUE
WON private respondents dismissal was justified on the ground of retrenchment
HELD
YES
- The Court resolved to grant due course to the Petition for Certiorari. The Resolutions of
the National Labor Relations Commission dated 9 February 1988 and 7 March 1988 are
hereby SET ASIDE and NULLIFIED. The Temporary Restraining Order issued by this
Court on 21 March 1988 is hereby made PERMANENT.
Ratio. Having reviewed the record of this case, the Court has satisfied itself that indeed
petitioner had serious financial difficulties before, during and after the termination of the
services of private respondent. For one thing, the audited financial statements of the
petitioner for its fiscal year ending on 31 July 1985 prepared by a firm of independent
auditors, showed a net loss in the amount of P4,431,321.00 and a total deficit or capital
impairment at the end of year of P6,776,493.00. 2 In the preceding fiscal year (1983-1984),
while the company showed a net after tax income of P843,506.00, it actually suffered a
deficit or capital impairment of P2,345,172.00. Most importantly, petitioner Wiltshire finally
closed its doors and terminated all operations in the Philippines on January 1987, barely
two (2) years after the termination of private respondent's employment. We consider that
finally shutting down business operations constitutes strong confirmatory evidence of
petitioner's previous financial distress. The Court finds it very difficult to suppose that
petitioner Wiltshire would take the final and irrevocable step of closing down its operations
in the Philippines simply for the sole purpose of easing out a particular officer or
employee, such as the private respondent.
- Turning to the legality of the termination of private respondent's employment, we find
merit in petitioner's basic argument. The Court was unable to sustain public respondent
NLRC's holding that private respondent's dismissal was not justified by redundancy and
hence illegal. In the first place, while the letter informing private respondent of the
termination of his services used the word "redundant", that letter also referred to the
company having "incur[red] financial losses which [in] fact has compelled [it] to resort to
retrenchment to prevent further losses". 3 Thus, what the letter was in effect saying was
that because of financial losses, retrenchment was necessary, which retrenchment in turn
resulted in the redundancy of private respondent's position.
- In the second place, the Court does not believe that redundancy in an employer's
personnel force necessarily or even ordinarily refers to duplication of work. That no other
person was holding the same position that private respondent held prior to the termination

Labor Law 1
of his services, does not show that his position had not become redundant. Indeed, in any
well-organized business enterprise, it would be surprising to find duplication of work and
two (2) or more people doing the work of one person. Redundancy, for purposes of our
Labor Code, exists where the services of an employee are in excess of what is reasonably
demanded by the actual requirements of the enterprise. Succinctly put, a position is
redundant where it is superfluous, and superfluity of a position or positions may be the
outcome of a number of factors, such as overhiring of workers, decreased volume of
business, or dropping of a particular product line or service activity previously
manufactured or undertaken by the enterprise. 4 The employer has no legal obligation to
keep in its payroll more employees than are necessarily for the operation of its business.
- In the third place, in the case at bar, petitioner Wiltshire, in view of the contraction of its
volume of sales and in order to cut down its operating expenses, effected some changes
in its organization by abolishing some positions and thereby effecting a reduction of its
personnel. Thus, the position of Sales Manager was abolished and the duties previously
discharged by the Sales Manager simply added to the duties of the General Manager, to
whom the Sales Manager used to report.
- It is of no legal moment that the financial troubles of the company were not of private
respondent's making. Private respondent cannot insist on the retention of his position
upon the ground that he had not contributed to the financial problems of Wiltshire. The
characterization of private respondent's services as no longer necessary or sustainable,
and therefore properly terminable, was an exercise of business judgment on the part of
petitioner company. The wisdom or soundness of such characterization or decision was
not subject to discretionary review on the part of the Labor Arbiter nor of the NLRC so
long, of course, as violation of law or merely arbitrary and malicious action is not shown. It
should also be noted that the position held by private respondent, Sales Manager, was
clearly managerial in character.

DOLE PHILIPPINES INC V NLRC (BARRANCO)


365 SCRA 124
KAPUNAN; September 13, 2001
NATURE
Certiorari
FACTS
- Dole Philippines, Inc., is a corporation organized and existing under Philippine laws. It is
engaged in the business of growing, canning, processing and manufacturing pineapples
and other allied products. The other petitioners were Dole corporate officers at the time
the cases were instituted
- Private respondents were Doles employees of different ranks and positions.
- In 1990 and 1991, DOLE carried out a massive manpower reduction and restructuring
program aimed at reducing the total workforce and the number of positions in the
companys table of organization.
- Dole intimates that the 1990-1991 reduction was a continuation of previous efforts to
restructure its organization.
- Previously, in 1982, Dole reduced its manpower by 509 workers but prolonged collective
bargaining negotiations, which ended in 1990, prevented the company from proceeding
with its restructuring
- Among the factors considered by the company in undertaking the reduction program was
the high absenteeism rate, which in 1989 accounted for 16% of total man hours. The high
absenteeism rate translated to higher paid sick leaves, higher operating costs for medical
facilities, and higher transportation costs due to under-filled and late hauls.
- Dole also cites the exacerbation of operating cost problems due to factors beyond [its]
control, i.e., the Gulf War, oil price increases, mandated wage increases, the 9% import
levy, power rate hikes, [and] increased land rentals, existing at that time.
- Furthermore, the bloody December 1989 coup detat shook investor confidence and put
in doubt the continued economic progress of the country.
- Pursuant to its restructuring efforts, Dole abolished the positions of foremen, bargaining
capataces and foreladies. Employees occupying these positions were either promoted or
were dismissed on grounds of redundancy.
- To address the surplus of manpower relative to its operations, Dole also decided to
reduce the number of employees company-wide. In 1990, the company offered a Special
Voluntary Resignation (SVR) program of which many employees, including a number of
private respondents, availed.
- Upon approval of the applications, notices of termination were sent to the employees
who availed of the SVR.
- After receiving the benefits, said employee-applicants executed a Release stating that
the employee had no claims against Dole in connection with his or her employment.
Subsequently, the dismissed employees executed another Release of Claim in favor of
Dole.
- A total of 2,357 hourly and monthly salaried employees were separated from Dole during
this period.
- After assessing the outcome of the SVR, Dole found that it could still do with lesser
employees, and proceeded to dismiss more of them in March 1991.

A2010

- 259 -

Disini

- Overall, 2,792 employees were separated under the SVR Program.


- In Oct. 1991, complaints for illegal dismissal were filed against Dole by De Lara et al.
- LA Solano dismissed the complaints for lack of merit; their dismissal was valid.
- NLRC reversed the LA and ordered reinstatement. Denied DOLEs MR.
ISSUE
WON DOLES redundancy program is valid and if so, won the dismissal of the respondent
employees are valid as well
HELD
YES
- Redundancy is one of the authorized causes for the dismissal of an employee.
- The lack of notice to the DOLE does not render the redundancy program void.
Reasoning
- Wiltshire File Co. Inc., vs. NLRC: x x x redundancy in an employers personnel force
necessarily or even ordinarily refers to duplication of work. That no other person was
holding the same position that private respondent held prior to the termination of his
services, does not show that his position had not become redundant. Indeed, in any wellorganized business enterprise, it would be surprising to find duplication of work and two
(2) or more people doing the work of one person. We believe that redundancy, for
purposes of the Labor Code, exists where the services of an employee are in excess of
what is reasonably demanded by the actual requirements of the enterprise. Succinctly
put, a position is redundant where it is superfluous, and superfluity of a position or
positions may be the outcome of a number of factors, such as overhiring of workers,
decreased volume of business, or dropping of a particular product line or service activity
previously manufactured or undertaken by the enterprise.
- The characterization of an employees services as no longer necessary or sustainable,
and, therefore, properly terminable, is an exercise of business judgment on the part of the
employer. The wisdom or soundness of such characterization or decision is not subject to
discretionary review provided, that violation of law or arbitrary or malicious action is not
shown.
- Doles redundancy program does not appear to be tainted by bad faith even if it hired
casual employees to replace those dismissed (as it has always hired casual employees to
augment its manpower in accordance with the demands of the industry.)
- The petition alleges that the redundancy program is part of a wide-scale restructuring of
the company.
- This purported restructuring is supported by the companys undisputed history towards
these ends, which culminated in the abolition of certain positions and the Special
Voluntary Resignation program in 1990-1991.
- Among the avowed goals of such restructuring is the reduction of absenteeism in the
company. The harsh economic and political climate then prevailing in the country also
emphasized the need for cost-saving measures.
- Reorganization as a cost-saving device is acknowledged by jurisprudence. An employer
is not precluded from adopting a new policy conducive to a more economical and effective
management, and the law does not require that the employer should be suffering financial
losses before he can terminate the services of the employee on the ground of redundancy.
- The law, however, does not prevent employers from saving on labor costs.
- International Macleod, Inc. vs. Intermediate Appellate Court: held that the determination
of the need for the phasing out of a department as a labor and cost saving device because
it was no longer economical to retain said department is a management prerogative, with
which the courts will not interfere.
- De Ocampo vs. NLRC: [t]he reduction of the number of workers in a company made
necessary by the introduction of the services of Gemac Machineries in the maintenance
and repair of its industrial machinery is justified. There can be no question as to the right
of the company to contract the services of Gemac Machineries to replace the services
rendered by the terminated mechanics with a view to effecting more economic and
efficient methods of production. So long as the undertaking to save on labor costs is not
attended by malice, arbitrariness, or intent on the part of the employer to circumvent the
law, as in this case, the Court will not interfere with such endeavor.
Ratio If an employee consented to his retrenchment or voluntarily applied for
retrenchment with the employer due to the installation of labor-saving devices,
redundancy, closure or cessation of operation or to prevent financial losses to the
business of the employer, the required previous notice to the DOLE is not necessary as
the employee thereby acknowledged the existence of a valid cause for termination of his
employment (International Harvester, Inc. vs. NLRC).
- Here, most of the private respondents even filled up application forms to be considered
for the redundancy program and thus acknowledged the existence that their services were
redundant.
- Private respondents executed two releases in favor of petitioner company.
- Not all quitclaims are per se invalid or against public policy. But those
(1) where there is clear proof that the waiver was wangled from an unsuspecting or
gullible person or
(2) where the terms of settlement are unconscionable on their face are invalid. In these
cases, the law will step in to annul the questionable transaction. There is no showing
here that private respondents are unsuspecting or gullible persons. Neither are the

Labor Law 1
terms of the settlement unconscionable. Indeed, private respondents received a
generous separation package, as set out in the narration of facts above.
Disposition Petition is GRANTED and the decision of the NLRC ANNULLED and SET
ASIDE. TRO LIFTED

ASUFRIN JR V SAN MIGUEL CORP


425 SCRA 270
YNARES-SANTIAGO; March 10, 2004
FACTS
- Coca Cola Plant, then a department of respondent San Miguel Beer Corporation (SMC),
hired petitioner as a utility/miscellaneous worker.
- He became a regular employee paid on daily basis as a Forklift Operator. Then became
a monthly paid employee promoted as Stock Clerk.
- The sales office and operations at the Sum-ag, Bacolod City Sales Office were
reorganized. Several positions were abolished including petitioners position as Stock
Clerk. After reviewing petitioners qualifications, he was designated warehouse checker at
the Sum-ag Sales Office.
- SMC implemented a new marketing system known as the pre-selling scheme at the
Sum-ag Beer Sales Office. As a consequence, all positions of route sales and warehouse
personnel were declared redundant. Respondent notified the DOLE Director of Region VI
that 22 personnel of the Sales Department of the Negros Operations Center would be
retired.
- SMC thereafter wrote a letter to petitioner informing him that, owing to the
implementation of the pre-selling operations scheme, all positions of route and
warehouse personnel will be declared redundant and the Sum-ag Sales Office will be
closed effective April 30, 1996. Thus, from April 1, 1996 to May 15, 1996, petitioner
reported to respondents Personnel Department at the Sta. Fe Brewery, pursuant to a
previous directive.
- Thereafter, the employees of Sum-ag sales force were informed that they can avail of
respondents early retirement package pursuant to the retrenchment program, while those
who will not avail of early retirement would be redeployed or absorbed at the Brewery or
other sales offices.
- Petitioner opted to remain and manifested to Acting Personnel Manager Salvador
Abadesco his willingness to be assigned to any job, considering that he had three children
in college.
- Petitioner was surprised when he was informed by the Acting Personnel Manager that
his name was included in the list of employees who availed of the early retirement
package. Petitioners request that he be given an assignment in the company was
ignored by the Acting Personnel Manager.
- Petitioner thus filed a complaint for illegal dismissal with the NLRC.
- The Labor Arbiter dismissed the complaint for lack of merit.
- Petitioner appealed to the NLRC which set aside the Labor Arbiters decision and
ordered respondent SMC to reinstate petitioner to his former or equivalent position with full
backwages.
- Respondent filed a petition with the Court of Appeals which reversed the decision of the
NLRC and reinstated the judgment of the Labor Arbiter dismissing the complaint for illegal
dismissal.
- Petitioners motion for reconsideration] was denied.
ISSUE
WON the dismissal of petitioner is based on a just and authorized cause
HELD
NO
- Dole Philippines, Inc. v. NLRC, citing the leading case of Wiltshire File Co., Inc. v.
NLRC:
redundancy in an employers personnel force necessarily or even ordinarily
refers to duplication of work. That no other person was holding the same position
that private respondent held prior to the termination of his services, does not show
that his position had not become redundant. Indeed, in any well-organized business
enterprise, it would be surprising to find duplication of work and two (2) or more
people doing the work of one person. We believe that redundancy, for purposes of
the Labor Code, exists where the services of an employee are in excess of what is
reasonably demanded by the actual requirements of the enterprise. Succinctly put,
a position is redundant where it is superfluous, and superfluity of a position or
positions may be the outcome of a number of factors, such as overhiring of workers,
decreased volume of business, or dropping of a particular product line or service
activity previously manufactured or undertaken by the enterprise.
- The determination that employees services are no longer necessary or sustainable and,
therefore, properly terminable is an exercise of business judgment of the employer.
- The wisdom or soundness of this judgment is not subject to discretionary review of the
Labor Arbiter and the NLRC, provided there is no violation of law and no showing that it
was prompted by an arbitrary or malicious act.

A2010

- 260 -

Disini

- It is not enough for a company to merely declare that it has become overmanned. It
must produce adequate proof that such is the actual situation to justify the dismissal of the
affected employees for redundancy.
- Persuasive as the explanation proffered by respondent may be to justify the dismissal of
petitioner, a number of disturbing circumstances, however, leave the Court unconvinced.
1. Of the 23 SMC employees assigned at the Sum-ag Sales Office/Warehouse, 9
accepted the offer of SMC to avail of the early retirement whose separation benefits
was computed at 250% of their regular pay. The rest, including petitioner, did not
accept the offer. Out of the remaining fourteen 14, only petitioner clearly manifested,
through several letters, his desire to be redeployed to the Sta. Fe Brewery or any sales
office and for any position not necessarily limited to that of a warehouse checker. In
short, he was even willing to accept a demotion just to continue his employment.
Meanwhile, other employees who did not even write a letter to SMC were redeployed
to the Sta. Fe Brewery or absorbed by other offices/outlets outside Bacolod City.
2. Petitioner was in the payroll of the Sta. Fe Brewery and assigned to the Materials
Section, Logistics Department, although he was actually posted at the Sum-ag
Warehouse. Thus, even assuming that his position in the Sum-ag Warehouse became
redundant, he should have been returned to the Sta. Fe Brewery where he was
actually assigned and where there were vacant positions to accommodate him.
3. It appears that despite respondents allegation that it ceased and closed down its
warehousing operations at the Sum-ag Sales Office, actually it is still used for
warehousing activities and as a transit point where buyers and dealers get their stocks.
4. In selecting employees to be dismissed, a fair and reasonable criteria must be used,
such as but not limited to (a) less preferred status, e.g. temporary employee; (b)
efficiency; and (c) seniority. In the case at bar, no criterion whatsoever was adopted by
respondent in dismissing petitioner. - Furthermore, as correctly observed by the NLRC,
respondent has not shown how the cessation of operations of the Sum-ag Sales Office
contributed to the ways and means of improving effectiveness of the organization with
the end in view of efficiency and cutting distribution overhead and other related costs.
- Respondent, thus, clearly resorted to sweeping generalization[s] in dismissing
complainant. Indeed, petitioners predicament may have something to do with an incident
where he incurred the ire of an immediate superior in the Sales Logistics Unit for exposing
certain irregularities committed by the latter.
- Whether it be by redundancy or retrenchment or any of the other authorized causes, no
employee may be dismissed without observance of the fundamentals of good faith.
- It is not difficult for employers to abolish positions in the guise of a cost-cutting measure
and we should not be easily swayed by such schemes which all too often reduce to near
nothing what is left of the rubble of rights of our exploited workers.
- Given the nature of petitioners job as a Warehouse Checker, it is inconceivable that
respondent could not accommodate his services considering that the warehousing
operations at Sum-ag Sales Office has not shut down.
- To sustain the position taken by the appellate court would be to dilute the workingmans
most important right: his constitutional right to security of tenure.
- While respondent may have offered a generous compensation package to those whose
services were terminated upon the implementation of the pre-selling scheme, we find
such an offer, in the face of the prevailing facts, anathema to the underlying principles
which give life to our labor statutes because it would be tantamount to likening an
employer-employee relationship to a salesman and a purchaser of a commodity.
- To quote what has been aptly stated by former Governor General Leonard Wood in his
inaugural message before the 6th Philippine Legislature on October 27, 1922 labor is
neither a chattel nor a commodity, but human and must be dealt with from the standpoint
of human interest.
- As has been said: We do not treat our workers as merchandise and their right to security
of tenure cannot be valued in precise peso-and-centavo terms. It is a right which cannot
be allowed to be devalued by the purchasing power of employers who are only too willing
to bankroll the separation pay of their illegally dismissed employees to get rid of them.
- This right will never be respected by the employer if we merely honor it with a price tag.
The policy of dismiss now and pay later favors moneyed employers and is a mockery of
the right of employees to social justice.
Disposition. The petition was granted.

FINANCIAL LOSS
ESCAREAL V NLRC (PHILIPPINE REFINING CO INC)
213 SCRA 472
DAVIDE JR; September 2, 1992
NATURE
Petition for certiorari
FACTS
- Escareal was hired by the PRC for the position of Pollution Control Manager effective on
16 September 1977 with a starting monthly pay of P4,230 00; the employment was made

Labor Law 1
permanent effective on 16 March 1978. The contract of employment provides, inter alia,
that his "retirement date will be the day you reach your 60th birthday, but there is provision
(sic) for voluntary retirement when you reach your 50th birthday. Bases for the hiring of
Escareal are LOI No. 588 implementing the National Pollution Control Decree, P.D No.
984, and Memorandum Circular No. 02, implementing LOI No. 588, which amended
Memorandum Circular No. 007, Series of 1977, issued by the National Pollution Control
Commission (NPCC).
- 1 April 1979: Escareal was also designated as Safety Manager pursuant to Article 162 of
the Labor Code (P.D. 442, as amended) and the pertinent implementing rule thereon. At
the time of such designation, Escareal was duly accredited as a Safety Practitioner by the
Bureau of Labor Standards, Department of Labor and Employment (DOLE) and the Safety
Organization of the Philippines.
- In addition, the pertinent rules on Occupational Health and Safety implementing the
Labor Code provide for the designation of full-time safety men to ensure compliance with
the safety requirements prescribed by the Bureau of Labor Standards. Consequently,
Escareal's designation was changed to Pollution Control and Safety Manager.
- In the course of his employment, Escareal's salary was regularly upgraded; the last pay
hike was granted on 28 March 1988 when he was officially informed that his salary was
being increased to P23,100.00 per month effective 1 April 1988. This last increase is
indisputably a far cry from his starting monthly salary of P4,230.00.
- Sometime in the first week of November 1987, PRC's Personnel Administration Manager
George B. Ditching informed Escareal about the company's plan to declare the position of
Pollution Control and Safety Manager redundant. Ditching attempted to convince Escareal
to accept the redundancy offer or avail of the company's early retirement plan. Escareal
refused and instead insisted on completing his contract as he still had about three and a
half (3 1/2) years left before reaching the mandatory retirement age of sixty (60).
- 15 June 1988: Escareal's immediate superior, PRC's Engg Dept Manager Jesus P.
Javelona, formally informed Escareal that the position of "Safety and Pollution Control
Manager will be declared redundant effective at the close of work hours on 15th July
1988." Escareal was also notified that the functions and duties of the position to be
declared redundant will be absorbed and integrated with the duties of the Industrial
Engineering Manager; as a result thereof, Escareal "will receive full separation benefits
provided under the PRC Retirement Plan and additional redundancy payment under the
scheme applying to employees who are 50 years old and above and whose jobs have
been declared redundant by Management."
- Escareal protested his dismissal via his 22 June 1988 letter to Javelona. This
notwithstanding, the PRC unilaterally circulated a clearance dated 12 July 1988, to take
effect on 15 July 1988, indicating therein that its purpose is for Escareal's "early
retirement" and not redundancy. Escareal confronted Javelona; the latter, in his letter
dated 13 July 1988, advised the former that the employment would be extended for
another month, or up to 15 August 1988. Escareal responded with a letter dated 25 July
1988 threatening legal action.
- 14 July 1988: PRC's Industrial Relations Manager Bernardo N. Jambalos III sent a
Notice of Termination to the DOLE informing the latter that Escareal was being terminated
on the ground of redundancy effective 16 August 1988.
- 5 August 1988:Escareal had a meeting with Cesar Bautista and Dr. Reynaldo Alejandro,
PRC's President and Corporate Affairs Director, respectively. To his plea that he be
allowed to finish his contract of employment as he only had three (3) years left before
reaching the mandatory retirement age, Bautista retorted that the termination was final.
- 8 August 1988: Escareal presented to Javelona a computation showing the amount of
P2,436,534.50 due him (Escareal) by way of employee compensation and benefits.
- On the date of the effectivity of his termination, Escareal was only fifty-seven (57) years
of age. He had until 21 July 1991, his sixtieth (60th) birth anniversary, before he would
have been compulsorily retired. Also, on the date of effectivity of Escareal's termination, 16
August 1988, (UP Chemical Engg graduate) Miguelito S. Navarro, PRC's Industrial
Engineering Manager, was designated as the Pollution Control and Safety Officer.
- In view of all this, Escareal filed a complaint for illegal dismissal with damages against
the private respondent PRC before the NLRC. Labor Arbiter Manuel P. Asuncion rendered
a decision ordering PRC to pay Escareal his redundancy pay in accordance with existing
company policy on the matter, without prejudice to the grant of additional benefits offered
by PRC during the negotiation stage of the case, though it never materialized for failure of
the parties to reach an agreement.
- On appeal, NLRC affirmed the Labor Arbiter's decision, with modification ordering PRC
to pay Escareal his retirement pay in accordance with the company policy and other
benefits granted to him thereunder, less outstanding obligations of the complainant with
the company at the time of his dismissal. Separate MFRs of PRC and Escareal were both
dismissed. Hence, this petition.
ISSUES
1. WON PRC had valid and acceptable basis to declare the position of Pollution Control
and Safety Manager redundant
2. WON Escareal's right to security of tenure was violated by PRC
3. WON Escareal's employment was for a fixed definite period to end at his 60th birthday
because of the stipulation as to the retirement age of sixty (60) years
4. WON Escareal is entitled to backwages and retirement benefits

A2010

- 261 -

Disini

5. WON Escareal is entitled to damages and attorney's fees


HELD
1. NO
- Wiltshire File Co., Inc. vs. NLRC: Redundancy, for purposes of the Labor Code, exists
where the services of an employee are in excess of what is reasonably demanded by the
actual requirements of the enterprise; a position is redundant when it is superfluous, and
superfluity of a position or positions may be the outcome of a number of factors, such as
257the overhiring of workers, a decreased volume of business or the dropping of a
particular product line or service activity previously manufactured or undertaken by the
enterprise.
- Redundancy in an employer's personnel force, however, does not necessarily or even
ordinarily refer to duplication of work. That no other person was holding the same position
which the dismissed employee held prior to the termination of his services does not show
that his position had not become redundant.
- PRC had no valid and acceptable basis to declare the position of Pollution Control and
Safety Manager redundant as the same may not be considered as superfluous; said
positions are required by law. Thus, it cannot be gainsaid that the services of Escareal are
in excess of what is reasonably required by the enterprise. Otherwise, PRC would not
have allowed ten (10) long years to pass before opening its eyes to that fact; neither would
it have increased the Escareal's salary to P23,100.00 a month effective 1 April 1988. That
Escareal's positions were not duplicitous is best evidenced by PRC's recognition of their
imperative need thereof, this is underscored by the fact that Miguelito S. Navarro, the
company's Industrial Engineering Manager, was designated as Pollution Control and
Safety Manager on the very same day of Escareal's termination.
- Indeed, the proposition that a department manned by a number of engineers presumably
because of the heavy workload, could still take on the additional responsibilities which
were originally reposed in an altogether separate section headed by Escareal, is difficult to
accept.
- If PRC felt that either Escareal was incompetent or that the task could be performed by
someone more qualified, then why is it that the person designated to the position hardly
had any experience in the field concerned? And why reward Escareal, barely five (5)
months before the dismissal, with an increase in salary?
- If based on the ground of redundancy, such a move would be invalid as the creation of
said position is mandated by the law; the same cannot therefore be declared redundant.
- If the aim was to generate savings in terms of the salaries that PRC would not be paying
Escareal any more as a result of the streamlining of operations for improved efficiency,
such a move could hardly be justified in the face of PRC's hiring of ten (10) fresh
graduates for the position of Management Trainee and advertising for vacant positions in
the Engineering/Technical Division at around the time of the termination.
- There would seem to be no compelling reason to save money by removing such an
important position. As shown by their recent financial statements, PRC's year-end net
profits had steadily increased from 1987 to 1990.
- While concededly, Article 283 of the Labor Code does not require that the employer
should be suffering financial losses before he can terminate the services of the employee
on the ground of redundancy, it does not mean either that a company which is doing well
can effect such a dismissal whimsically or capriciously. The fact that a company is
suffering from business losses merely provides stronger justification for the termination.
2. YES
- It is evident that Escareal's right to security of tenure was violated by the private
respondent PRC. Both the Constitution (Section 3, Article XIII) and the Labor Code (Article
279, P.D. 442, as amended) enunciate this right as available to an employee.
- Security of tenure is a right which may not be denied on mere speculation of any unclear
and nebulous basis.
- In this regard, it could be concluded that the respondent PRC was merely in a hurry to
terminate the services of Escareal as soon as possible in view of the latter's impending
retirement; it appears that said company was merely trying to avoid paying the retirement
benefits Escareal stood to receive upon reaching the age of sixty (60). PRC acted in bad
faith.
3. NO
- There is no indication that PRC intended to offer uninterrupted employment until
Escareal reached the mandatory retirement age, the contract of employement merely
informs Escareal of the compulsory retirement age and the terms pertaining to the
retirement.
- The letter to Escareal confirming his appointment does not categorically state when the
period of employment would end. It stands to reason then that Escareal's employment
was not one with a specific period.
4. YES
- Article 279, LC: an "employee who is unjustly dismissed from work shall be entitled to
reinstatement without loss of seniority rights and other privileges and to his full
backwages, inclusive of allowances, and to his other benefits or their monetary equivalent
computed from the time his compensation was withheld from him up to the time of his
actual reinstatement."
- Torillo vs. Leagardo, Jr. / Santos vs. NLRC: "The normal consequences of a finding that
an employee has been illegally dismissed are, firstly, that the employee becomes entitled

Labor Law 1
to reinstatement to his former position without loss of seniority rights and, secondly, the
payment of backwages corresponding to the period from his illegal dismissal up to actual
reinstatement. xxx Though the grant of reinstatement commonly carries with it an award of
backwages, the inappropriateness or non-availability of one does not carry with it the
inappropriateness or non-availability of the other. xxx Put a little differently, payment of
backwages is a form of relief that restores the income that was lost by reason of unlawful
dismissal, separation pay, in contrast, is oriented towards the immediate future, the
transitional period the dismissed employee must undergo before locating a replacement
job."
- Reinstatement of Escareal would have been proper. However, since he reached the
mandatory retirement age on 21 July 1991, reinstatement is no longer feasible. He should
thus be awarded his backwages from 16 August 1988 to 21 July 1991, inclusive of
allowances and the monetary equivalent of the other benefits due him for that period, plus
retirement benefits under the PRC's compulsory retirement scheme which he would have
been entitled to had he not been illegally dismissed.
5. NO
- In his complaint and the attached Affidavit-Complaint, Escareal does not mention any
claim for damages and attorney s fees; furthermore, no evidence was offered to prove
them. An award therefor would not be justified.
Disposition Petition granted.

LAW REQUIRED POSITION


ESCAREAL V NLRC
[PAGE 260]

WHEN REDUNDANCY
LOPEZ SUGAR CORP V FRANCO
[PAGE 150]
WILTSHIRE FILE CO INC V NLRC
[PAGE 257]
TIERRA INTERNATIONAL CONSTRUCTION CORP V
NLRC (OLIVAR)
211 SCRA 73
PADILLA; July 3, 1992

A2010

HELD
1. YES, the termination was for a valid cause.
Ratio In redundancy, what is looked into is the position itself, the nature of the services
performed by the employee and the necessity of such position.
Reasoning
- Termination of an employee's services because of a reduction of work force due to a
decrease in the scope or volume of work of the employer is synonymous to, or a shade of
termination because of redundancy under Article 283 of the Labor Code.
- Redundancy exists where the services of an employee are in excess of what is
reasonably demanded by the actual requirements of the enterprise. A position is
redundant where it is superfluous, and superfluity of a position or positions may be the
outcome of a number of factors, such as over-hiring of workers, decreased volume of
business, or dropping of a particular product line or service activity previously
manufactured or undertaken by the enterprise.
- Olivar received his notice of termination advising him that his position will be deleted
because of a reduction of force due to a decrease in scope of work assigned. 28 other
positions were also abolished. Olivar was not singled out and that his termination was not
arbitrary or malicious on the part of the employer.
- The law does not make any distinction between a technical and a non-technical position
for purposes of determining the validity of termination due to redundancy. Neither does the
law nor the stipulations of the employment contract here involved require that junior
employees should first be terminated (in answer to NLRCs reasoning that junior
employees should be terminated first before the technical and senior positions).
2. YES, Olivar is entitled to separation pay.
Ratio Not only are existing laws read into contracts in order to fix the obligations as
between the parties, but the reservation of essential attributes of sovereign power is also
read into contracts as a postulate of the legal order.
Reasoning
- There is no mention of an award of separation pay in the contract between the parties.
HOWEVER, Tierra admits that Article 283 of the Labor Code governs its employeremployee relationship with the private respondent as the same is deemed written in the
employment contract signed by the parties. Thus, although a contract is the law between
the parties, thereto, this provisions of law which regulate such contracts are deemed
included and shall limit and govern the relations between the parties.
Disposition. Decision of the NLRC is reversed and set aside, and the decision of the
POEA is revived. No pronouncements as to costs.

ESCAREAL V NLRC
[PAGE 260]

NATURE
Petition for the annulment of the decision of NLRC.
FACTS
- March 7 1984: private respondent Isidro P. Olivar was hired by FEBROE, a foreign
shipping company, through its local agent Tierra International Construction Corporation, to
work as shift supervisor in its Base Operating Support (BOS) project for the U.S. Navy in
the British Indian Ocean Territory of Diego Garcia, for a period of one (1) year with a basic
monthly salary of US $680.00.
- Olivars employment contract was renewed in 1985; the last renewal was on 8 May 1986.
But on 1 October 1986, he was dismissed from employment, and subsequently repatriated
to the Philippines.
- Olivar alleged that he was a victim of improper termination of employment thru gradual
and systematic removal of high salaried employees.
- FEBROE averred that in July and August 1986, its management undertook a
comprehensive audit and evaluation of its entire work force to promote economy,
efficiency and profitability in its operations, and to reduce personnel whose positions were
considered redundant or surplusage and/or to re-assign personnel to other available
useful positions. One of the positions listed for abolition was the position of the olivar as
"13401 Supervisor, Technical."
- POEA held that the termination was for authorized cause. POEA then ordered Tierra and
FEBROE to pay Olivar his separation pay.
- Tierra contended that the employment contract does not provide for separation pay in
case of termination based on redundancy or reduction of force due to a decrease in
volume or scope of work.
- NLRC reversed the decision of POEA and ordered the company to pay Olivar
corresponding to the unexpired portion of his contract.
ISSUES
1. WON termination of Olivar is illegal
2. WON Olivar is entitled to separation pay

Disini

- 262 -

EDGE APPAREL INC V NLRC


349 PHIL 972
VITUG; February 12, 1998
NATURE
Petition for certiorari and prohibition assailing the decision of the NLRC
FACTS
- Pursuing its retrenchment program, petitioner Edge Apparel, Inc., dismissed private
respondents from employment. The subsequent receipt of their separation pay benefits
did not deter them from going through with their complaint for illegal dismissal against
petitioner. The charge averred that the retrenchment program was a mere subterfuge used
by Edge Apparel to give a semblance of regularity and validity to the dismissal.
- Satisfied with the legality of the retrenchment program, the Labor Arbiter dismissed the
complaint of private respondents against petitioner. The decision was appealed to the
NLRC. In their appeal, private respondents claimed that the documents submitted to
demonstrate the alleged losses had been "bloated" so as to reflect financial losses. The
NLRC held that there was no basis for the retrenchment of the workers .
- The NLRC found, however, that the workers were terminated due to redundancy and
ordered petitioner to pay private respondents additional separation pay in accordance with
Art. 283. The NLRC noted that the workers were assigned to the sewing line for simple
garments which was phased out due in fact to the dropping of this particular line of
business. It was held that the dismissal of private respondents could be considered to
have been due to redundancy since it was only private respondents' line of work which
was phased out by petitioner. Edge Apparels MFR was denied, hence this petition.
ISSUE
WON private respondents were dismissed due to redundancy
HELD

Labor Law 1
NO
Ratio The fact that a mere portion of the business of an employer is shut down does not
necessarily remove that measure from the ambit of the term "retrenchment" within the
meaning of Art. 283.
Reasoning
- The Labor Arbiter, affirmed by the NLRC, found petitioner to have complied with the
requirements of the law in effecting a valid retrenchment. The law acknowledges the right
of every business entity to reduce its work force if it is compelled by economic factors that
would otherwise endanger its stability or existence. In exercising its right to retrench
employees, the firm may choose to close all, or a part of, its business to avoid further
losses or mitigate expenses.
Obiter
- Redundancy exists where the services of an employee are in excess of what would
reasonably be demanded by the actual requirements of the enterprise. A position is
redundant when it is superfluous, and superfluity of a position or positions could be the
result of a number of factors, such as the overhiring of workers, a decrease in the volume
of business or the dropping of a particular line or service previously manufactured or
undertaken by the enterprise. An employer has no legal obligation to keep on the payroll
employees more than the number needed for the operation of the business.
Retrenchment, in contrast to redundancy, is an economic ground to reduce the number of
employees. A dismissal due to redundancy entitles the worker to separation pay
equivalent to 1 month pay for every year of service. When the termination of employment
is due to retrenchment to prevent losses, the separation pay is only an equivalent of 1/2
month pay for every year of service. In the above instances, a fraction of at least 6 months
is considered as 1 whole year.
Disposition The appealed decision is MODIFIED by deleting the additional award of
separation pay to private respondents.

CRITERIA-SELECTION OF EMPLOYEE
PANLILIO V NLRC (FINDSTAFF PLACEMENT
SERVICES INC, OMAN SHERATON HOTEL INC)
281 SCRA 53
ROMERO; October 17, 1997
NATURE
Review of the decision of the NLRC
FACTS
- Petitioner Moises B. Panlilio was recruited by private respondent Findstaff Placement
Services (FPS) for employment in the Sheraton Hotel in Oman as Recreational Manager
in October 1991. The contract was for a period of two years. However, in March 1992, his
services were terminated.
- He filed a complaint for illegal dismissal before the POEA. POEA rendered a decision in
favor of the petitioner on the ground that the alleged redundancy of his position was not
adequately proven.
- FPS filed an appeal before the NLRC, submitting affidavits from the officers of the
Director of Personnel and Training Division of Sheraton Hotel by FPS substantiating the
redundancy of petitioners position. However, NLRC affirmed the decision of the POEA.
On MFR, NLRC reversed its decision.
ISSUE
WON Panlilio was illegally dismissed
HELD
YES
- The redundancy of his position wasnt fully substantiated. The affidavits and documents it
submitted are entitled to little weight, for it does not prove the superfluity of petitioners
position. In fact, these documents do not even present the necessary factors which would
confirm that a position is indeed redundant, such as overhiring of workers, decreased
volume of business or dropping of a product line or service activity.
- Undoubtedly, said documents still do not sufficiently explain the reason why petitioners
position had become redundant, but only elucidated the fact that he was not a victim of
any discrimination in effecting the termination.
- We have held that it is important for a company to have fair and reasonable criteria in
implementing its redundancy program, such as but not limited to, (a) preferred status, (b)
efficiency and (c) seniority. Unfortunately for FPS, such appraisal was not done in the
instant case.
Disposition Decision reversed

GOLDEN THREAD KNITTING INDUSTRIES INC V


NLRC

A2010

Disini

- 263 [PAGE 194]

TANJUAN V PHIL POSTAL SAVINGS BANK


411 SCRA 168
PANGANIBAN; September 16, 2003
NATURE
Petition for Review of a CA ruling
FACTS
- Tanjuan (petitioner) was employed by Philippine Postal Savings Bank, Inc. (respondent),
a government financing institution and a subsidiary of the Philippine Postal Corporation
(Philpost), as Property Appraisal Specialist and Officer-in-Charge of its Credit Supervision
and Control Department. At the time material to this case, he was on his fourth year of
service. Torres (respondent Torres), PPSBIs President and Chief Executive Officer, issued
Memorandum 145-98 addressed to petitioner and five (5) other employees belonging to its
Accounts Management Department and Credit Supervision and Control Department
charging them with negligence in the performance of duties and misrepresentation.
Petitioner submitted his written explanation alleging that he merely reviewed and validated
the findings of the Property Appraiser.
- OP Order No. 003-99 was issued by respondent Torres to petitioner informing him of his
preventive suspension for a period of ninety (90) days in view of the pending
administrative investigation against him, later reduced to 30 days. Petitioners suspension
would only be up to February 11, 1999, after which he could already report back to work.
- Board of Directors of respondent PPSBI issued Board Resolution No. 99-14 approving
the banks reorganization via retrenchment of employees and re-alignment of functions
and positions for the purpose of preventing further serious losses. In furtherance of the
Boards decision, a letter was released by respondent Torres addressed to all employees
of respondent PPSBI, informing them of the impending reorganization and enjoining them
to apply for their desired plantilla positions under the new organizational set-up not later
than July 20, 1999, otherwise they shall not be included in the selection process and shall
be deemed to have opted to be separated instead. Petitioner did not apply for any
position in the new organizational set-up.
- Petitioner received a Notice of Termination dated October 4, 1999 informing him that
pursuant to respondent PPSBIs adoption of a new organizational structure under Board
Resolution No. 99-14, his employment therewith shall cease [at] the close of office hours
on November 4, 1999 or thirty (30) calendar days from date of receipt of the notice on the
ground of abolition of position. The Department of Labor and Employment was likewise
seasonably notified prior to the effectivity date of petitioners termination as required by
law. However, the release of his separation pay of one and a half (1 1/2) months salary for
every year of service was withheld in view of the pendency of a criminal case against him
with the Office of the Ombudsman for alleged irregularities in the granting of loans for
which he could likewise be held pecuniarily liable.
- Petitioner filed a complaint for illegal dismissal with money claims.
- Labor Arbiter declared Philippine Postal Savings Bank, Inc. guilty of illegal dismissal.
Respondents appealed to the NLRC asseverating that they were denied due process of
law when Labor Arbiter allegedly hastily decided that they did not adduce evidence to
support their claim of business losses to justify retrenchment. In support of their appeal,
respondents submitted in evidence different documents.
- Petitioner duly opposed the presentation of the aforesaid documents contending that
[these] cannot be presented for the first time on appeal. Moreover, even if the same can
be admitted on appeal, the aforesaid documents are insufficient to prove the existence of
business losses. Finally, petitioner posits that if serious losses were in fact incurred by
respondent PPSBI, the same was due to the mismanagement of its officers which should
not be borne by its rank and file employees.
- NLRC issued a Resolution admitting the evidence presented by respondents on appeal
and finding the same adequate to prove the existence of business losses on the part of
respondent PPSBI. Petitioner elevated the case to the CA which affirmed the NLRC.
ISSUES
1. WON proof of business losses maybe admitted on appeal
2. WON the CA has a Power to review findings of fact
3. WON the Retrenchment was valid
HELD
1. YES
- It is well-settled that the NLRC is not precluded from receiving evidence, even for the first
time on appeal, because technical rules of procedure are not binding in labor cases. This
rule applies equally to both the employee and the employer. In the interest of due

Labor Law 1
process, the Labor Code directs labor officials to use all reasonable means to ascertain
the facts speedily and objectively, with little regard to technicalities or formalities. However,
delay in the submission of evidence should be clearly explained and should adequately
prove the employers allegation of the cause for termination.
- In the instant case, it is undisputed that the evidence of business losses for the years
1996 up to 1999 was introduced before the NLRC only. The CA correctly noted, however,
that respondents reserved the right to introduce the evidence to the labor arbiter, if and
when required to do so. Reasons of confidentiality and the volatile nature of PPSBIs
business as a banking institution prompted respondents to limit the presentation of this
evidence at the outset. Indeed, it would have been foolhardy for the NLRC and the CA to
reject the evidence, just because it had not been presented before the labor arbiter. Such
evidence was absolutely necessary to resolve the issue of whether petitioners
employment was validly terminated. For strictly adhering to technical rules of procedure at
the expense of equity, the Commission has in fact been chided by the Court in Philippine
Telegraph and Telephone Corporation v. NLRC, from which we quote:
Thus, even if the evidence was not submitted to the labor arbiter, the fact that it was
duly introduced on appeal to respondent commission is enough basis for the latter to
have been more judicious in admitting the same, instead of falling back on the mere
technicality that said evidence can no longer be considered on appeal. Certainly, the
first course of action would be more consistent with equity and the basic notions of
fairness.
- As to petitioners claim that he was denied due process because of the belated
admission of the evidence, suffice it to say that he was given every opportunity to refute it
and to submit counter-evidence. The essence of due process consists simply in according
parties reasonable opportunity to be heard and to submit any evidence they may have in
support of their defense.
2. YES
- Section 9 of Batas Pambansa Blg. 129, as amended by Republic Act No. 7902,
expanded the jurisdiction of the CA as follows:
SEC. 9. Jurisdiction. - The Court of Appeals shall exercise:
(1) Original jurisdiction to issue writs of mandamus, prohibition, certiorari, habeas
corpus, and quo warranto, and auxiliary writs or processes, whether or not in aid of its
appellate jurisdiction;
- The Court of Appeals shall have the power to try cases and conduct hearings, receive
evidence and perform any and all acts necessary to resolve factual issues raised in cases
falling within its original and appellate jurisdiction, including the power to grant and
conduct new trials or further proceedings. Verily, the appellate court, pursuant to the
exercise of its original jurisdiction over petitions for certiorari, has the power to review
NLRC cases. Such review extends to the factual findings of the labor arbiter when, as in
this case, these are at variance with those of the NLRC.
- The CA acted within its power when it disregarded the labor arbiters findings and upheld
the contrary ruling of the NLRC. In turn, the factual findings of the former affirming those of
the latter are generally binding on this Court and will not as a rule be reviewed on appeal.
Petitioner has not shown any reason for us to depart from this rule.
3. YES
- The Court has consistently recognized and affirmed the employers management right
and prerogative to terminate the services of its employees in order to obviate or minimize
business losses. Retrenchment, one of the authorized causes for termination under the
Labor Code, has been defined as the termination of employment initiated by the employer
through no fault of the employees and without prejudice to the latter, resorted by
management during periods of business recession, industrial depression, or seasonal
fluctuations[;] or during lulls occasioned by lack of orders, shortage of materials,
conversion of the plant for a new production program or the introduction of new methods
or more efficient machinery, or of automation.
- For the exercise of such prerogative, Article 283 of the Labor Code provides these
conditions:
Art. 283.
Closure of establishment and reduction of personnel. -- The employer
may also terminate the employment of any employee due to the installation of labor
saving devices, redundancy, retrenchment to prevent losses or the closing or cessation
of operation of the establishment or undertaking unless the closing is for the purpose of
circumventing the provisions of this Title, by serving a written notice on the worker and
the Ministry of Labor and Employment at least one (1) month before the intended date
thereof. In case of termination due to the installation of labor saving devices or
redundancy, the worker affected thereby shall be entitled to a separation pay equivalent
to at least his one (1) month pay or to at least one (1) month pay for every year of
service, whichever is higher. In case of retrenchment to prevent losses and in cases of
closures or cessation of operations of establishment or undertaking not due to serious
business losses or financial reverses, the separation pay shall be equivalent to one (1)
month pay or at least one-half () month pay for every year of service, whichever is
higher. A fraction of at least six (6) months shall be considered as one (1) whole year.
-Thus, the requisites for valid retrenchment are the following: (1) necessity of the
retrenchment to prevent losses, and proof of such losses; (2) written notice to the
employees and to the Department of Labor and Employment (DOLE) at least one month
prior to the intended date of retrenchment; and (3) payment of separation pay equivalent
to one-month pay or at least one-half month pay for every year of service, whichever is

A2010

- 264 -

Disini

higher. There is no dispute that respondents have seasonably complied with the
procedural requirement of serving written notice to petitioner and the DOLE. On the other
hand, the withholding of separation pay, which has not been raised as an issue in this
Petition, was satisfactorily resolved by the CA.
- The only remaining question is whether respondents have sufficiently and convincingly
established business reverses of the kind or the amount that would justify the
retrenchment. As this Court has held, before any reduction of personnel becomes legal,
any claim of actual or potential business losses must satisfy established standards as
follows: (1) the losses incurred are substantial and not de minimis; (2) the losses are
actual or reasonably imminent; (3) the retrenchment is reasonably necessary and is likely
to be effective in preventing the expected losses; and (4) the alleged losses, if already
incurred, or the expected imminent losses sought to be forestalled are proven by sufficient
and convincing evidence. The employer has the burden of proving that the losses are
serious, actual and real.
- The Court had previously ruled that financial statements audited by independent external
auditors constituted the normal method of proof of the profit-and-loss performance of a
company. Audit reports conducted by no less than the Commission on Audit (COA)
showec the losses alleged by respondents to have been the primary reason for the Board
of Directors decision to effect retrenchment and reorganization were clearly not at all
fictitious, imaginary or mere conjectures as claimed by petitioner. Furthermore, the fact
that respondent PPSBI was being regularly monitored by the Bangko Sentral ng Pilipinas
(BSP) and the Philippine Deposit Insurance Corporation (PDIC) due to its precarious
financial position for several years positively affirms respondents asseveration of
continuous serious business losses. The findings of the CA affirming those of the NLRC
showed real and grave financial reverses, which made downsizing the only recourse for
the bank to follow. Indeed, the retrenchment of petitioner was the consequence of the
banks reorganization and a cost-saving device recognized by jurisprudence.
Disposition Petition is DENIED, and the assailed Decision and Resolution AFFIRMED.

LOPEZ SUGAR CORP V FRANCO


[PAGE 150]

EMPLOYMENT OF INDEPENDENT CONTRACTOR


EFFECT
ASIAN ALCOHOL CORPORATION V NLRC
305 SCRA 416
PUNO; March 25, 1999
NATURE
Petition for Review on certiorari of NLRC decision
FACTS
- In Sept 91, the Parsons family, who originally owned the controlling stocks in Asian
Alcohol Corp (AAC), were driven by mounting business losses to sell their majority rights
to Prior Holdings, Inc. which took over AACs management and operation.
- To prevent further losses, Prior Holding implemented an organizational plan and other
cost-saving measures. Some 117 employees out of a total workforce of 360 were
separated. 72 of them occupied redundant positions that were abolished. Of these
positions, 21 held by union members and 51 by non-union members.
- The 6 private respondent workers are among those union members whose positions
were abolished due to redundancy. In Oct, 92, they received individual notices of
termination effective Nov 30, 92. They were paid the equivalent of 1 month salary for
every year of service as separation pay, the money value of their unused sick, vacation,
emergency and seniority leave credits, 13th month pay for the year 1992, medicine
allowance, tax refunds, and goodwill cash bonuses for those with at least 10 years of
service. All of them executed sworn releases, waivers and quitclaims. Except for 2, they all
signed sworn statements of conformity to the company retrenchment program. And except
for one, they all tendered letters of resignation.
- The 6 private respondents filed with the NLRC complaints for illegal dismissal with a
prayer for reinstatement with backwages, moral damages and atty's fees. They alleged
that AAC used the retrenchment program as a subterfuge for union busting. They claimed
that they were singled out for separation by reason of their active participation in the
union. They also asseverated that Asian Alcohol was not bankrupt as it has engaged in an
aggressive scheme of contractual hiring.
- LA ruled in favor of AAC saying that retrenchment was valid. NLRC reversed LA ruling
that there was illegal dismissal. It rejected the evidence by AAC to prove its business
reversals. It ruled that the positions of private respondents were not redundant for the
simple reason that they were replaced by casuals. Hence, this petition.
ISSUE

Labor Law 1

A2010

- 265 -

Disini

WON the positions of the herein private respondent workers are unnecessary redundant
and superfluous, thereby justifying the termination of their employment
HELD
YES
Ratio Retrenchment and redundancy under A283 of Labor Code 45 are just causes for the
employer to terminate the services of workers to preserve the viability of the business. In
exercising its right, however, management must faithfully comply with the substantive and
procedural requirements laid down by law and jurisprudence
Reasoning
- On Retrenchment: AAC was able to prove the ff. requirements for retrenchment to be
valid: (1) the retrenchment was reasonably necessary and likely to prevent business
losses, which, if already incurred, are not merely de minimis, but substantial, serious,
actual and real, or if only expected, are reasonably imminent as perceived objectively and
in good faith by the employer; (2) ER served written notice both to the EEs and to DOLE at
least 1 mo. prior to the intend date of retrenchment;(3) ER paid the retrenched employees
separation pay equivalent to 1 mo. pay or at least 1/2 month pay for every year of service,
whichever is higher; (4) ER exercised its prerogative to retrench employees in good faith
for the advancement of its interest of its interest and not to defeat or circumvent the
employees' right to security of tenure; and (5) ER used fair and reasonable criteria in
ascertaining who would be dismissed and who would be retained among the EEs, such as
status (i.e., whether they are temporary, casual, regular or managerial employees),
efficiency, seniority, physical fitness, age, and financial hardship for certain workers.
- On Redundancy: Redundancy exists when the service capability of the work force is in
excess of what is reasonably needed to meet the demands on the enterprise. A redundant
position is one rendered superfluous by any of these factors: overhiring of workers,
decreased volume of business, dropping of a particular product line previously
manufactured by the company or phasing out of a service activity priorly undertaken by
the business. Under these conditions, ER has no legal obligation to keep in its payroll
more EEs than are necessary for the operation of its business.
- Procedure Requisites for valid implementation of valid redundancy program: (1) written
notice served on both the employees and the Department of Labor and Employment at
least one month prior to the intended date of retrenchment; (2) payment of separation pay
equivalent to at least one month pay or at least one month pay for every year of service,
whichever is higher; (3) good faith in abolishing the redundant positions; and (4) fair and
reasonable criteria in ascertaining what positions are to be declared redundant and
accordingly abolished.
- On Effect of Employment of Independent Contractor : An employer's good faith in
implementing a redundancy program is not necessarily destroyed by availment of the
services of an independent contractor to replace the services of the terminated
employees. We have previously ruled that the reduction of the number of workers in a
company made necessary by the introduction of an independent contractor is justified
when the latter is undertaken in order to effectuate more economic and efficient methods
of production. In the case at bar, private respondents failed to proffer any proof that the
management acted in a malicious or arbitrary manner in engaging the services of an
independent contractor to operate the Laura wells. Absent such proof, the Court has no
basis to interfere with the bona fide decision of management to effect more economic and
efficient methods of production.
Disposition Petition GRANTED. NLRC decision set aside. LA ruling reinstated.

PROCEDURE REQUIREMENT
ASIAN ALCOHOL CORP V NLRC
[PAGE 264]

HEARING
WILTSHIRE FILE CO INC V NLRC
[PAGE 257]

45

Art. 283. Closure of establishment and reduction of personnel. The employer may also terminate the employment
of any employee due to the installation of labor saving devices, redundancy, retrenchment, to prevent losses or the
closing or cessation of operation of the establishment or undertaking unless the closing is for the purpose of
circumventing the provisions of this Title, by serving a written notice on the workers and the Ministry of Labor and
Employment at least one (1) month before the intended date thereof. In case of termination due to the installation of
labor saving devices or redundancy, the worker affected thereby shall be entitled to a separation pay equivalent to at
least one (1) month pay or to at least one (1) month pay for every year of service, whichever is higher. In case of
retrenchment to prevent losses and in case of closures or cessation of operations of establishment or undertaking not
due to serious business losses or financial reverses, the separation pay shall be equivalent to one (1) month pay at least
one-half (1/2), month pay for every year of service, whichever is higher. A fraction of at least six (6) month shall be
considered one (1) whole year.

VENUE OF COMPLAINT
WILTSHIRE FILE CO INC V NLRC
[PAGE 257]

C. RETRENCHMENT TO PREVENT LOSSES


DEFINED
FF MARINE CORP V NLRC
455 SCRA 154
TINGA; April 8, 2005
FACTS
- On 26 October 1998, petitioners filed with the Department of Labor and Employment
(DOLE) a notice that petitioner corporation was undertaking a retrenchment program to
curb the serious business reverses brought about by the Asian economic crisis. Notices
and separation were given to the affected employees.
- 21 employees were retrenched including petitioner Magno who despite receiving
separation pay, filed a case for illegal dismissal. Magno claimed that he was beguiled into
accepting the separation pay since petitioners terminated his services on the pretext that
the dredging machine where he was assigned was temporarily stalled in Zambales.
Magno eventually learned that the company had been adducing to others a different
reason for retrenchment, primarily the Asian financial crisis
- Labor Arbiter Salimathar V. Nambi promulgated a Decision upholding the validity of
retrenchment. NLRC reversed decision. CA affirmed NLRC decision.
- Material to the resolution of the case was the issue of admissibility and competency as
evidence of the 31 December 1997 and 1996 Financial Statements of petitioners.
However, the appellate court ruled that a perusal of the certification issued by Banaria,
Banaria and Company (independent auditor) regarding the Financial Statements reveals
that the same were executed on 30 March 1998 or nine (9) months prior to the filing of the
complaint for illegal dismissal on 12 January 1999. Thus, the financial statements could
have been offered as evidence before the Labor Arbiter and the NLRC. Thus, the Court of
Appeals reproached petitioners for having suppressed material evidence
- Accordingly, the appellate court found that petitioners failed to substantiate the
substantive requirements of a valid retrenchment
ISSUE
WON retrenchment was valid
HELD
NO
- Retrenchment is the termination of employment initiated by the employer through no fault
of the employees and without prejudice to the latter, resorted to by management during
periods of business recession, industrial depression, or seasonal fluctuations or during
lulls occasioned by lack of orders, shortage of materials, conversion of the plant for a new
production program or the introduction of new methods or more efficient machinery, or of
automation
- There are three (3) basic requisites for a valid retrenchment to exist, to wit: (a) the
retrenchment is necessary to prevent losses and such losses are proven; (b) written notice
to the employees and to the DOLE at least one (1) month prior to the intended date of
retrenchment; and (c) payment of separation pay equivalent to one (1) month pay or at
least one-half (1/2) month pay for every year of service, whichever is higher
- Also according to jurisprudence: the employers prerogative to bring down labor costs by
retrenching must be exercised essentially as a measure of last resort, after less drastic
means alleged losses if already realized, and the expected imminent losses sought to
be forestalled, must be proved by sufficient and convincing evidence
- The law recognizes this under Article 283 of the Labor Code. However, the employer
bears the burden to prove his allegation of economic or business reverses.
- petitioners adduced before the Labor Arbiter the 1994 and 1995 Financial Statements.
Said Financial Statements, however, were prepared only by petitioners accountant,
Rosalie Bengzon, and approved by the manager Bernadette Rosales. They were not
audited by an independent external auditor. The financial statements show that in 1994
and 1995, petitioner corporation earned an income of only P77,609.79 and P155,339.96,
respectively. In contrast, the 1996 and 1997 Financial Statements, however, showed
losses of P18,005,918.08, and P21,316,072.89, respectively. It was only before the Court
of Appeals that the financial statements for the years 1996 and 1997 as audited by an

Labor Law 1
independent external auditor were introduced. They were not presented before the Labor
Arbiter and the NLRC although they were executed on 30 March 1998, several months
prior to the filing of the complaint for illegal dismissal by Magno on 12 January 1999
- The appellate courts affirmance of the decision of the NLRC is principally anchored on
the ground that petitioners failed to adduce the 1996 and 1997 Financial Statements
audited by an independent external auditor before the Labor Arbiter and the NLRC. By
merely upholding the evidentiary weight accorded to financial statements duly audited by
independent external auditors, grave abuse of discretion on the part of the NLRC is hardly
imaginable as it is unfounded
- It is essentially required that the alleged losses in business operations must be proven.
The employer bears the burden of proving the existence or the imminence of substantial
losses with clear and satisfactory evidence that there are legitimate business reasons
justifying a retrenchment. Should the employer fail to do so, the dismissal shall be deemed
unjustified
- Moreover, petitioners failed to act in consonance with the rule that retrenchment shall be
a remedy of last resort. retrenchment is not perfectly justified as there was no showing that
the retrenchment was the last recourse resorted to by petitioners. Although petitioners
allege in their petition before this Court that they had undertaken cost-cutting measures
before they resorted to retrenchment, their contention does not inspire belief for the
evidence shows that the petition for certiorari filed by petitioners with the Court of Appeals
is bereft of any allegation of prior resort to cost-cutting measures other than retrenchment

DISTINCTION REDUNDANCY AND RETRENCHMENT


AG&P UNITED RANK AND FILE ASSN V NLRC
(ATLANTIC GULF AND PACIFIC COMPANY OF
MANILA INC)
265 SCRA 159
MENDOZA; November 29, 1996
NATURE
Special Civil Action for Certiorari
FACTS
- Petitioner union is the duly certified bargaining agent of the rank and file employees of
the respondent corporation.
- As a result of a deadlock in the negotiations for a collective bargaining agreement, the
union declared a strike on September 22, 1987.
- Less than a month later, the Department of Labor and Employment assumed jurisdiction
over the dispute. Then Secretary Franklin Drilon rendered a decision on February 10,
1988 from which both parties appealed.
- On January 11, 1988, prior to the rendition of the decision of the Secretary of Labor and
Employment, the president of respondent company announced the adoption by the
company of several cost-cutting measures to forestall impending financial losses. Among
these was a so-called "redundancy program," which, as implemented on March 1, 1988,
resulted in the layoff of around 177 employees, some of whom were officers and members
of the petitioner union. The affected employees were given separation pay equivalent to
one month pay for every year of service, for which they signed documents of waiver.
- On March 14, 1988, however, petitioners filed a complaint for unfair labor practice and
illegal dismissal.
- After trial, Labor Arbiter Mendoza found the complaint to be without merit and accordingly
dismissed it. He found the "redundancy program" necessary for the company's existence
and considered private respondent's practice of rehiring or reemploying dismissed
employees under the said program as a managerial prerogative, made not only in
pursuance of the company's policy of giving preference to its dismissed workers, but
actually beneficial to the workers as well. Moreover, he held that petitioners' acceptance of
termination pay and other benefits constituted a waiver of their right to question their
dismissal.
- On appeal, the NLRC reversed the labor arbiter's ruling.
- The company moved for a reconsideration. On May 29, 1992, the NLRC under R.A. No.
6715, reconsidered the decision and reinstated the decision of the labor arbiter.
- Petitioners filed a motion for reconsideration but their motion was denied
ISSUE
WON the NLRC gravely abused its discretion by declaring the legality of the redundancy
program of the company
HELD
NO
- It is now settled that the NLRC has the power to admit on appeal additional evidence to
show lawful cause for dismissal, provided that the delay in the submission of said

A2010

- 266 -

Disini

evidence is explained and the same clearly proves the employer's allegation of a valid
cause for dismissing his employees.
- In the case at bar, evidence of losses for the years 1987 up to 1990 was, belatedly
introduced in the NLRC. But the delay was satisfactorily explained by respondent
company, as the audit conducted on its financial report by Sycip Gorres Velayo and Co.
was completed only in 1991. The additional evidence presented confirmed private
respondent's allegation that the losses expected by the company were substantial and
reasonably imminent to justify the layoff of the individual petitioners.
- It is necessary to distinguish "redundancy" from "retrenchment." Both are mentioned in
Art. 283 of the Labor Code as just causes for the closing of establishments or reduction of
personnel. "Redundancy" exists when the services of an employee are in excess of what
is required by an enterprise. "Retrenchment," on the other hand, is one of the economic
grounds for dismissing employees and is resorted to primarily to avoid or minimize
business losses. Private respondent's "redundancy program, " while denominated as
such, is more precisely termed "retrenchment" because it is primarily intended to prevent
serious business losses.
- The Labor Code recognizes retrenchment as one of the authorized causes for
terminating the employer-employee relationship and the decision to retrench or not to
retrench is a management prerogative. In the case at bar, the company losses were duly
established by the financial statements presented by both parties.
- Petitioners contend that the "redundancy program" was actually a union-busting scheme
of management, aimed at removing union officers who had declared a strike. This
contention cannot stand in the face of evidence of substantial losses suffered by the
company. Moreover, while it is true that the company rehired or reemployed some of the
dismissed workers, it has been shown that such action was made only as company
projects became available and that this was done in pursuance of the company's policy of
giving preference to its former workers in the hiring of project employees. The rehiring or
reemployment does not negate the imminence of losses, which prompted private
respondent to retrench.
- Lastly, it is not disputed that petitioners signed documents of waiver. Petitioners insist
that the documents are without any effect because quitclaims and releases are contrary to
public policy and therefore, null and void. Not all quitclaims and releases are, however,
contrary to public policy. If the agreement was voluntarily entered into and represents a
reasonable settlement, it is binding on the parties and may not later be disowned simply
because of a change of mind. It is only where there is clear proof that the waiver was
wangled from an unsuspecting or gullible person, or the terms of settlement are
unconscionable on its face, that the law will step in to annul the questionable transaction.
But where it is shown that the person making the waiver did so voluntarily, with full
understanding of what he was doing, and the consideration for the quitclaim is credible
and reasonable, the transaction must be recognized as a valid and binding undertaking.
Disposition Dismissed.

DISTINCTION CLOSURE AND RETRENCHMENT


JAT GEN SERVICES V NLRC (MASCARINAS)

421 SCRA 78
QUISUMBING; January 26, 2004
NATURE
Certiorari on decision of CA affirming NLRC affirming LA finding petitioner liable for illegal
dismissal and ordering petitioners to pay private respondent Mascarinas separation pay,
backwages, legal holiday pay, service incentive leave pay and 13 th month pay in the
aggregate sum of P85,871.00.
FACTS
- JAT is engaged in the business of selling second-hand heavy equipment. It hired private
respondent as helper tasked to coordinate with the cleaning and delivery of the heavy
equipment sold to customers. Initially, private respondent was hired as a probationary
employee and was paid P165 per day that was increased to P180 in July 1997 and P185
in January 1998.
- In October 1997, the sales of heavy equipment declined because of the Asian currency
crisis. Consequently, JAT temporarily suspended operations. It advised its employees,
including private respondent, not to report for work starting on the first week of March
1998. JAT indefinitely closed shop effective May 1998.
- A few days after, Mascarinas filed a case for illegal dismissal and underpayment of
wages.
- On December 14, 1998, JAT filed an Establishment Termination Report with the DOLE,
notifying its decision to close its business operations due to business losses and financial
reverses.
ISSUE

Labor Law 1
WON private respondent was illegally dismissed

HELD
NO
Ratio Article 283 of the Labor Code is clear that an employer may close or cease his
business operations or undertaking even if he is not suffering from serious business
losses or financial reverses, as long as he pays his employees their termination pay in the
amount corresponding to their length of service, It would, indeed, be stretching the intent
and spirit of the law if we were to unjustly interfere in managements prerogative to close
or cease it s business operations just because said undertaking is not suffering from any
loss. (Industrial Timber Corp. v NLRC)
Reasoning
- While the CA defined the issue to be the validity of dismissal due to alleged closure of
business, it cited jurisprudence relating to retrenchment to support its resolution and
conclusion. While they are often used interchangeably and are interrelated, they are
actually two separate and independent causes for termination of employment.
Termination of an employment may be predicated on one without the need of resorting to
the other.
- Closure of business, on one hand, is the reversal of fortune of the employer whereby
there is a complete cessation of business operations and/or an actual locking-up of the
doors of establishment, usually due to financial losses. Closure of business as an
authorized cause for termination of employment aims to prevent further financial drain
upon an employer who cannot pay anymore his employees since business has already
stopped.
- On the other hand, retrenchment is reduction of personnel usually due to poor financial
returns so as to cut down on costs of operations in terms of salaries and wages to prevent
bankruptcy of the company. It issometimes also referred to as downsizing. Retrenchment
is an authorized cause for termination of employment which the law accords an employer
who is not making good in its operations in order to cut back on expenses for salaries and
wages by laying off some employees. The purpose of retrenchment is to save a financially
ailing business establishment from eventually collapsing.
- However, the burden of proving that such closure is bona fide falls upon the employer.
JATs submitted financial statements were bereft of details on the extent of alleged losses
incurred.
- So we proceed to determining if there was valid dismissal on the ground of closure or
cessation of operations for reasons other than substantial business losses. JATs income
statement showed declining sales enough to justify the closure of business.
The dismissal being valid, backwages need not be awarded.
Dispositive petition is given due course. The assailed Resolutions of the Court of Appeals
in CA-G.R. SP No. 60337 are AFFIRMED with the MODIFICATION that the award of
P66,924.00 as backwages is deleted. The award of separation pay amounting to
P10,296.00 and the other monetary awards, namely salary differentials in the amount of
P1,066.00, legal holiday pay in the amount of P1,850.00, service incentive leave pay in
the amount of P925.00 and the 13 th month pay in the amount of P4,910, or a total of
P29,047.00 are maintained. No pronouncements as to costs.

ALABANG COUNTRY CLUB V NLRC (ALABANG


COUNTRY CLUB INDEPENDENT EMPLOYEES UNION)
466 SCRA 329
CARPIO-MORALES; August 9, 2005
NATURE
Petition for review on certiorari. Alabang Country Club Inc. (ACCI) seeks to set aside the
appellate courts decision which denied its motion for reconsideration, and ordered the
reinstatement of herein 63 respondents-members of the labor organization Alabang
Country Club Independent Employees Union, without loss of seniority rights and other
privileges, and payment of full backwages.
FACTS
- ACCIs internal auditor conducted a study on the profitability of ACCIs Food and
Beverage Department
- Her report showed that from 1989-1993, the F&B Dept had been incurring substantial
losses of around P8,727,135
- Realizing that it was no longer profitable for ACCI to maintain its own F&B Dept, the
management decided to cease from operating it, and to open it to a contractor, which
would be willing to operate its own food and beverage business within the club.
- ACCI entered into an agreement with La Tasca Restaurant for it to operate the F&B Dept.
- ACCI sent its F&B Dept employees letters informing them their services would be
terminated effective 1 month later, that they would get separation pay of 125% of their
monthly salary for every year of service, that La Tasca agreed to absorb all affected
employees immediately with the status of regular employees without need of undergoing

A2010

- 267 -

Disini

probationary period, and that they would receive the same salary they were receiving at
the time of their termination
- The Union filed a complaint for illegal dismissal, unfair labor practice, regularization and
damages with prayer for the issuance of a writ of preliminary injunction against ACCI
- The ACCI ceased operating its F&B Dept as scheduled, and La Tasca began operating
its own F&B business
- Meanwhile, in the proceedings before the Labor Arbiter, respondent union informed that
the F&B division had been reporting gaining profits as shown by the Statement of Income
and Deficit prepared by SGV. They thus argued that compliance with standards for losses
to justify their retrenchment were not met by ACCI.
- ACCI averred that it may exercise management prerogatives to adopt a cost-saving and
cost-consciousness program t improve efficiency in its operations, etc.
- By decision of April 30, 1999, the Labor Arbiter dismissed the complaint for illegal
dismissal on the ground that a business entity has the right to reduce its work force if
necessitated by compelling economic factors which endanger its existence or stability.
- On appeal, the NLRC acknowledged the right of ACCI to regulate, according to its own
discretion and judgment, all aspects of employment including the lay-off of workers
because of losses in the operation of its business, lack of work and considerable reduction
in the volume of business. It thus dismissed the appeal.
- Private respondents thereupon brought their case, via petition for certiorari before the
Court of Appeals, alleging that the Labor Arbiter and the NLRC committed grave abuse of
discretion and utter ignorance of the law in completely disregarding the audited financial
statements prepared by SGV&Co. showing that ACCIs F & B Department had been
consistently earning profits.
- By decision of August 14, 2002, the Court of Appeals reversed those of the NLRC and
the Labor Arbiter. It held that due to ACCIs failure to prove by sufficient and competent
evidence that its alleged losses were substantial, continuing and without any immediate
prospect of abating them, the bona fide nature of the retrenchment appeared doubtful
- ACCIs motion for reconsideration was denied by the appellate court by Resolution of
March 6, 2003
- During the pendency of the whole case, petitioner the separation package of sixty-two
(62) of the sixty-three (63) individual respondents on account of which they executed
Releases, Waivers and Quitclaims in its favor.
ISSUE
WON respondents were terminated for an authorized cause
HELD
YES
Reasoning
- The court first distinguished between retrenchment and closure of a business
undertaking, because the respondents were relying on a case (Lopez Sugar Corp. v
Federation of Free Workers) involving retrenchment on the ground of serious business
losses being allowed subject to certain conditions.
- The court, however, viewed this case as one involving closure of business undertaking.
- **Retrenchment is the reduction of personnel for the purpose of cutting down on costs
of operations in terms of salaries and wages resorted to by an employer because of losses
in operation of a business occasioned by lack of work and considerable reduction in the
volume of business.
- **Closure of a business or undertaking due to business losses is the reversal of
fortune of the employer whereby there is a complete cessation of business operations to
prevent further financial drain upon an employer who cannot pay anymore his employees
since business has already stopped.
- While the Labor Code provides for the payment of separation package in case of
retrenchment to prevent losses, it does not obligate the employer for the payment thereof
if there is closure of business due to serious losses.
- In the present case, when petitioner decided to cease operating its F & B Department
and open the same to a concessionaire, it did not reduce the number of personnel
assigned thereat. It terminated the employment of all personnel assigned at the
department.
- As in the case of retrenchment, however, for the closure of a business or a department
due to serious business losses to be regarded as an authorized cause for terminating
employees, it must be proven that the losses incurred are substantial and actual or
reasonably imminent; that the same increased through a period of time; and that the
condition of the company is not likely to improve in the near future .
- As did the appellate court, this Court finds that the study report submitted by the internal
auditor of petitioner, the only evidence submitted to prove its alleged losses, is self-serving
and falls short of the stringent requirement of the law that the employer prove sufficiently
and convincingly its allegation of substantial losses.
- Petitioners failure to prove that the closure of its F & B Department was due to
substantial losses notwithstanding, this Court finds that individual respondents were
dismissed on the ground of closure or cessation of an undertaking not due to serious
business losses or financial reverses, which is allowed under Article 283 of the Labor
Code:

Labor Law 1
Art. 283. Closure of establishment and reduction of personnel. The employer
may also terminate the employment of any employee due to the installation of laborsaving devices, redundancy, retrenchment to prevent losses or the closing or cessation
of the establishment or undertaking unless the closing is for the purpose of
circumventing the provisions of this Title, by serving a written notice on the worker and
the Ministry of Labor and Employment at least one (1) month before its intended date
thereof. In case of termination due to the installation of labor-saving devices or
redundancy, the worker affected thereby shall be entitled to a separation pay equivalent
to at least one (1) month pay for every year of service, whichever is higher. In case of
retrenchment to prevent losses and in cases of closures or cessation of operations of
the establishment or undertaking not due to serious business losses or financial
reverses, the separation pay shall be equivalent to one (1) month pay or at least onehalf () month pay for every year of service, whichever is higher. A fraction of at least
six (6) months shall be considered as one (1) whole year.
- While petitioner did not sufficiently establish substantial losses to justify closure of its F &
B Department on this ground, there is basis for its claim that the continued maintenance of
said department had become more expensive through the years. An evaluation of the
financial figures appearing in the audited financial statements prepared by the SGV&Co.
shows that ninety one to ninety six (91% - 96%) percent of the actual revenues earned by
the F & B Department comprised the costs and expenses in maintaining the department.
Petitioners decision to place its F & B operations under a concessionaire must then be
respected, absent a showing of bad faith on its part.
- While the closure of F & B Department is found to be justified, petitioner is, under the
above-quoted provision of Art. 283 of the Labor Code, mandated to pay separation pay
computed from the time individual respondents commenced their employment until the
time the department ceased operations, in an amount equivalent to one (1) month pay or
at least one-half () month pay for every year of service, whichever is higher. In
petitioners case, it in fact voluntarily doled out to some of individual respondents
separation pay equivalent to one month and a quarter (1) for every year of service, a
fraction of a year being considered as one year.
- Respondents not having been illegally dismissed, they are not entitled to backwages.
By petitioners information, it had paid, during the pendency of the case, the separation
package of sixty-two (62) of the sixty-three (63) individual respondents on account of
which they executed Releases, Waivers and Quitclaims in its favor.
Disposition Petition GRANTED

COVERAGE
PHILIPPINE TUBERCULOSIS SOCIETY INC V NLRC
294 SCRA 567
MENDOZA; August 25, 1998
NATURE
This is a petition for certiorari to set aside the decision of the NLRC declaring the
retrenchment of 116 employees of petitioner invalid and ordering the reinstatement and
backwages to them.
FACTS
- The Philippine Tuberculosis Society, Inc. is a non-stock and non-profit domestic
corporation with the primary objective of fighting tuberculosis in the Philippines. It has
employees who are represented by private respondent National Labor Union.
- In the proceedings before the NLRC, it was shown that, in 1989, the Society began to
experience serious financial difficulties when it incurred a deficit of P2 million. The
shortfall increased to P9,100,000.00 in 1990 and was certain to become worse were it not
for quick measures taken by petitioner.
- First, the Society leased a property in Tayuman to a fastfood outlet, cancelled its service
agreement with a janitorial company, and sold its equity in the Philippine Long Distance
Telephone Company (PLDT). Second, it withdrew from the Pag-Ibig Fund Program,
negotiated with the Government Service Insurance System for the restructuring of its
obligations, and applied for exemption from minimum wage increases. Finally, it
disapproved the overtime pay of supervisory and managerial employees, obtained the
waiver of personnel of their entitlement to wage differentials, and implemented the
retrenchment of one hundred sixteen (116) employees.The retrenchment is the subject of
the present suit.
- On September 27, 1991, respondent NLU filed a notice of strike against the Society with
the National Conciliation and Mediation Board (NCMB), charging the Society with unfair
labor practice in terminating the services of the aforementioned employees.
- Conferences were scheduled by the NCMB, which however failed to resolve the case.
On November 6, 1991, then Secretary of Labor and Employment Ruben Torres certified
the case to the NLRC on the ground that the labor dispute seriously affected the national
interest.

A2010

- 268 -

Disini

- On August 31, 1993, the NLRC rendered a decision declaring as invalid the
retrenchment of the employees concerned on the ground that the Society did not take
seniority into account in their selection.
ISSUE
WON NLRC committed grave abuse of discretion in rendering its decision
HELD
NO
- Clearly (under the Labor Code), retrenchment or reduction of the workforce in cases of
financial difficulties is recognized as a ground for the termination of employment.
- Although petitioner is a non-stock and non-profit organization, retrenchment as a
measure adopted to stave off threats to its existence is available to it.
- However, the employers prerogative to layoff employees is subject to certain limitations
set forth in Lopez Sugar Corporation v. Federation of Free Workers as follows:
- Firstly, the losses expected should be substantial and not merely de minimis in extent.
If the loss purportedly sought to be forestalled by retrenchment is clearly shown to be
insubstantial and inconsequential in character, the bonafide nature of the retrenchment
would appear to be seriously in question. Secondly, the substantial loss apprehended
must be reasonably imminent, as such imminence can be perceived objectively and in
good faith by the employer. There should, in other words, be a certain degree of urgency
for the retrenchment, which is after all a drastic recourse with serious consequences for
the livelihood of the employees retired or otherwise laid-off. Because of the consequential
nature of retrenchment, it must, thirdly, be reasonably necessary and likely to effectively
prevent the expected losses. The employer should have taken other measures prior or
parallel to retrenchment to forestall losses, i.e., cut other costs than labor costs. An
employer who, for instance, lays off substantial numbers of workers while continuing to
dispense fat executive bonuses and perquisites or so-called golden parachutes, can
scarcely claim to be retrenching in good faith to avoid losses. To impart operational
meaning to the constitutional policy of providing full protection to labor, the employers
prerogative to bring down labor costs by retrenching must be exercised essentially as a
measure of last resort, after less drastic means - e.g., reduction of both management and
rank-and-file bonuses and salaries, going on reduced time, improving manufacturing
efficiencies, trimming of marketing and advertising costs, etc. - have been tried and found
wanting.
- Lastly, but certainly not the least important, alleged losses if already realized, and the
expected imminent losses sought to be forestalled, must be proved by sufficient
and convincing evidence. The reason for requiring this quantum of proof is readily
apparent: any less exacting standard of proof would render too easy the abuse of this
ground for termination of services of employees.
- In addition to the above, the retrenchment must be implemented in a just and proper
manner. As held in Asiaworld Publishing House, Inc. v. Ople:
there must be fair and reasonable criteria to be used in selecting employees to be
dismissed, such as: (a) less preferred status (e.g. temporary employee); (b) efficiency
rating; and (c) seniority.
- There is substantial evidence in the record to support the NLRCs finding that the Society
suffered financial distress as a result of growing deficits which were not likely to abate.
Petitioner presented to the NLRC the balance sheets, financial statements, and the
reports of its external auditors for the years 1989 and 1990. We cannot, therefore, say
that the finding of the NLRC is unsupported by substantial evidence. Accordingly, the
NLRC could rightly conclude:
Given the claim of the Society that its present financial troubles were occasioned by a
dearth of funding from its traditional sources of revenue, . . . it is Our considered view
that the Societys claim to retrench employees . . . is valid.
- Nor do we think the NLRC erred in holding that though the Society was justified in
ordering a retrenchment, its implementation of the scheme rendered the retrenchment
invalid. That is because in selecting the employees, the Society disregarded altogether
the factor of seniority. As the NLRC noted:
- We noted with concern that the criteria used by the Society failed to consider the
seniority factor in choosing those to be retrenched, a failure which, to our mind, should
invalidate the retrenchment, as the omission immediately makes the selection process
unfair and unreasonable. Things being equal, retaining a newly hired employee and
dismissing one who had occupied the position for years, even if the scheme should result
in savings for the employer, since he would be paying the newcomer a relatively smaller
wage, is simply unconscionable and violative of the senior employees tenurial rights. In
Villena vs. NLRC, 193 SCRA 686, February 7, 1991, the Supreme Court considered the
seniority factor an important ingredient for the validity of a retrenchment program.
According to the Court, the following legal procedure should be observed for a
retrenchment to be valid; (a) one-month prior notice to the employee as prescribed
by Article 282 of the Labor Code; and b) use of a fair and reasonable criteria in
carrying out the retrenchment program, such as 1) less preferred status (as in the
case of temporary employees) 2) efficiency rating, 3) seniority, and 4) proof of
claimed financial losses.
- Petitioner has not explained why the said employees had to be laid off without
considering their many years of service to the Society. The fact that these employees had

Labor Law 1
accumulated seniority credits indicates that they had been retained in the employ of the
Society because of loyal and efficient service. The burden of proving the contrary is on
petitioner.
Disposition Petition DISMISSED for lack of showing that in rendering its decision, NLRC
committed grave abuse of discretion.

BALBALEC V NLRC (RURAL BANK OF BANGUED)


251 SCRA 399
KAPUNAN; December 19, 1995
FACTS
- The Rural Bank of Bangued dismissed Paulino Balbalec, Juan Bolante and Rolando
Beleno, petitioners, alleging that its workforce was being retrenched for losses suffered by
the bank during the years 1984-1988. It stressed that management's decision to dismiss
them was made to "protect the bank's stability and profitability" and to "enable the bank to
comply with provisions of the new minimum wage law."
- Petitioners alleged that they were singled out for dismissal after they refused to sign an
agreement for deferment of wage increases under RA 6727, an agreement which all other
employees signed. The bank, on the other hand, averred that retrenchment of its
workforce was necessary to prevent business losses. It further alleged that said
termination would not hamper its operations and denied that petitioners were singled out,
claiming that the latter ranked last in seniority among its employees.
- The Labor Arbiter decided that there was no valid retrenchment, hence complainants
were illegally dismissed. The NLRC sustained the Labor Arbiter's findings. On motion for
reconsideration, however, the NLRC partially reversed itself with respect to the issue of
retrenchment, upholding the bank's dismissal of petitioners on the basis of a valid
retrenchment.

ISSUE
WON the bank's dismissal of petitioners was justified by a valid retrenchment

HELD
YES
Ratio The law recognizes the right of every business entity to reduce its workforce if the
same is made necessary by compelling economic factors which would endanger its
existence or stability. In spite of overwhelming support granted by the social justice
provisions of our Constitution in favor of labor, the fundamental law itself guarantees, even
during the process of tilting the scales of social justice towards workers and employees,
"the right of enterprises to reasonable returns of investment and to expansion and growth."
[quoting Art XIII, Sec 3, last paragraph of the Consti]
Reasoning To hold otherwise would not only be oppressive and inhuman, but also
counterproductive and ultimately subversive of the nation's thrust towards a resurgence in
our economy which would ultimately benefit the majority of our people. Where appropriate
and where conditions are in accord with law and jurisprudence, the Court has authorized
valid reductions in the workforce to forestall business losses, the hemorrhaging of capital,
or even to recognize an obvious reduction in the volume of business which has rendered
certain employees redundant.
- Article 283 of the Labor Code provides:
Art. 283. Closure of establishment and reduction of personnel - The employer may also
terminate the employment of any employee due to the installation of labor saving
devices, redundancy, retrenchment to prevent losses or the closing or cessation of
operation of the establishment or undertaking unless the closing is for the purpose of
circumventing the provisions of this Title, by serving a written notice on the worker and
the Ministry of Labor and Employment at least one (1) month before the intended date
thereof. In case of termination due to the installation of labor saving devices or
redundancy, the worker affected thereby shall be entitled to a separation pay equivalent
to at least his one (1) month pay or to at least one (1) month pay for every year of
service, whichever is higher. In case of retrenchment to prevent losses and in cases of
closures or cessation of operations of establishment or undertaking not due to serious
business losses or financial reverses, the separation pay shall be equivalent to one (1)
month pay or at least onehalf (1/2) month pay for every year of service, whichever is
higher. A fraction of at least six (6) months shall be considered as one (1) whole year.
- The article not only contemplates the termination of employment of workers or
employees to minimize established business losses but also to prevent impending losses,
for the law's phraseology explicitly uses the phrase "retrenchment to prevent losses."
However, retrenchment strikes at the very core of an individual's employment and the
burden clearly falls upon the employer to prove economic or business losses with
appropriate supporting evidence. After all, not every asserted potential loss is sufficient

A2010

- 269 -

Disini

legal warrant for a reduction of personnel and the evidence adduced in support of a claim
of actual or potential business losses should satisfy certain established standards, to wit:
1. The losses expected and sought to be avoided must be substantial and not merely
de minimis;
2. The apprehended substantial losses must be reasonably imminent, as such
imminence can be perceived objectively and in good faith by the employer;
3. The retrenchment should reasonably be necessary and likely to prevent effectively
the expected losses;
4. The losses, both the past and forthcoming, must be proven by sufficient and
convincing evidence.
- The bank posted losses in the amount of P12,920.22, P3,888.34 and P17,865.12 for the
years 1984, 1985 and 1986, respectively. Against these losses, the bank registered a net
income of P12,702.59 and P68,085.56 in 1988 and 1989, respectively. Its net profit of
P60,085.56 would instead have been a net loss had not the bank deferred the
implementation of the wage increase under RA 6640. In addition, the bank was faced with
a serious financial problem resulting from a considerable reduction of its total resources by
27.23% and total loan investments by 35.79% from 1984 to 1988. Past due loan ratio
ranged from 29.13% to 32.13% in 1986 to 1988 such that it was required by the Central
Bank to set aside a reserve for bad debts in the amount of P359,464.50 as of December
31, 1988. It should be noted that unlike huge commercial banks with large capitalization,
the bank involved is a small rural bank barely afloat and surviving on a measly
capitalization of P500,000. Were we to deny its urgent request to streamline its work force
to enable it to maintain stability and modest profitability, we would be sending a small
financial institution teetering on the verge of financial ruin tumbling down on the road to
bankruptcy.
- The State recognizes the pivotal role of small rural banks in the development of the
countryside through its loan portfolios and other services to the rural folk. While courts
must be constantly vigilant in validating claims of business losses to prevent unscrupulous
employers from feigning such losses in order to dismiss their personnel, we are satisfied
that the Rural Bank of Bangued undertook the drastic act of cutting down its workforce in
order to prevent imminent substantial loss to its business. As to petitioners' claim that they
were singled out in the process because they happened to be the only employees who did
not sign an agreement for deferment of wages under RA 6727, suffice it to say that such
assertion is merely speculative because the bank clearly followed the standard for
dismissing its personnel by selecting the last in seniority among its 10 employees.
Dispositive Decision of the NLRC AFFIRMED with MODIFICATION.

PROCEDURE
MAYON HOTEL & RESTAURANT V ADANA
458 SCRA 609
PUNO; May 16, 2005
NATURE
Petition for certiorari to annul CA decision
FACTS
- On various dates starting in 1981, petitioner hotel and restaurant hired the 16
respondents.
- Due to the expiration and non-renewal of the lease contract for the rented space
occupied by the said hotel and restaurant, operations of the business were suspended.
The operation of the restaurant was continued in its new location, while waiting for the
construction of a new Mayon Hotel & Restaurant. Only nine of the sixteen employees
continued working in the Mayon Restaurant at its new site.
- On various dates, the 16 employees filed complaints for underpayment of wages and
other money claims against petitioners
- The Labor Arbiter awarded substantially all of respondents money claims and held that
based on the evidence presented, Josefa Po Lam is the owner/proprietor of Mayon Hotel
& Restaurant, and is the proper respondent in these cases.
- NLRC reversed labor arbiters decision and dismissed all complaints.
- CA reinstated labor arbiters decision
ISSUES
1. WON Josefa Po Lam should to be held liable as the owner of petitioner Mayon Hotel &
Restaurant, and the proper respondent in this case
2. WON respondents Loveres, Guades, Macandog, Atractivo, Llarena and Nicerio were
illegally dismissed
3. WON respondents are entitled to their money claims
HELD
1. YES
- The records of the case belie petitioner Josefa Po Lams claim that she is merely an
overseer. While Josefa Po Lam claims that it is her daughter, Pacita Po, who owns the

Labor Law 1
hotel and restaurant when the latter purchased the same, Josefa failed to submit the
document of sale as allegedly the sale was only verbal although the license to operate
said hotel and restaurant is in the name of Pacita which, despite our Order to Josefa to
present the same, she failed to comply. While several documentary evidences were
submitted by Josefa wherein Pacita was named therein as owner of the hotel and
restaurant, there were documentary evidences also that were submitted by Josefa
showing her ownership of said enterprise Respondents testified that it was Josefa who
exercises all the acts and manifestation of ownership of the hotel and restaurant like
transferring employees from the Greatwall Palace Restaurant which she and her husband
Roy Po Lam previously owned; it is Josefa to whom the employees submit reports, draws
money for payment of payables and for marketing, and attends to Labor Inspectors during
ocular inspections.
2. YES
- The records are unequivocal that since petitioner Mayon Hotel & Restaurant suspended
its hotel operations and transferred its restaurant operations in Elizondo Street,
respondents Loveres, Macandog, Llarena, Guades and Nicerio have not been permitted to
work for petitioners. Respondent Alamares, on the other hand, was also laid-off when the
Elizondo Street operations closed, as were all the other respondents. Since then,
respondents have not been permitted to work nor recalled, even after the construction of
the new premises at Pearanda Street and the reopening of the hotel operations with the
restaurant in this new site.
- Article 286 of the Labor Code is clear there is termination of employment when an
otherwise bona fide suspension of work exceeds six months. The cessation of
employment for more than six months was patent and the employer has the burden of
proving that the termination was for a just or authorized cause.
- Even after six months of what should have been just a temporary lay-off, the same
respondents were still not recalled to work. More than three years after the supposed
temporary lay-off, the employment of all of the respondents with petitioners had ceased,
notwithstanding that the new premises had been completed and the same operated as a
hotel with bar and restaurant. This is clearly dismissal or the permanent severance or
complete separation of the worker from the service on the initiative of the employer
regardless of the reasons therefor.
- We find substantial evidence that petitioners intended the termination to be permanent.
First, respondents filed the complaint for illegal dismissal immediately after the closure of
the hotel operations in Rizal Street, notwithstanding the alleged temporary nature of the
closure of the hotel operations, and petitioners allegations that the employees assigned to
the hotel operations knew about this beforehand. Second, in their position paper
submitted to the Labor Arbiter, petitioners made no mention of any intent to recall these
respondents to work upon completion of the new premises. Third, the various pleadings
on record show that petitioners held respondents, particularly Loveres, as responsible for
mismanagement of the establishment and for abuse of trust and confidence. The
vehemence of petitioners accusation of mismanagement against respondents, especially
against Loveres, is inconsistent with the desire to recall them to work. Fourth, petitioners
memorandum on appeal also averred that the case was filed not because of the business
being operated by them or that they were supposedly not receiving benefits from the
Labor Code which is true, but because of the fact that the source of their livelihood,
whether legal or immoral, was stopped on March 31, 1997, when the owner of the building
terminated the Lease Contract. Fifth, petitioners had inconsistencies in their pleadings in
referring to the closure. Sixth, that petitioners terminated all the other respondents, by not
employing them when the Hotel and Restaurant transferred to its new site on Pearanda
Street. Indeed, in this same memorandum, petitioners referred to all respondents as
former employees of Mayon Hotel & Restaurant.
- Even assuming that the cessation of employment was merely temporary, it became
dismissal by operation of law when petitioners failed to reinstate respondents after the
lapse of six months, pursuant to Article 286
- Serious business losses do not excuse the employer from complying with the clearance
or report required under Article 283 of the Labor Code and its implementing rules before
terminating the employment of its workers. In the absence of justifying circumstances, the
failure of petitioners to observe the procedural requirements set out under Article 284,
taints their actuations with bad faith, especially since they claimed that they have been
experiencing losses in the three years before 1997.
-Even assuming that the closure was due to a reason beyond the control of the employer,
it still has to accord its employees some relief in the form of severance pay.
- While we recognize the right of the employer to terminate the services of an employee
for a just or authorized cause, the dismissal of employees must be made within the
parameters of law and pursuant to the tenets of fair play. And in termination disputes, the
burden of proof is always on the employer to prove that the dismissal was for a just or
authorized cause. Where there is no showing of a clear, valid and legal cause for
termination of employment, the law considers the case a matter of illegal dismissal.
- Under these circumstances, the award of damages was proper. As a rule, moral
damages are recoverable where the dismissal of the employee was attended by bad faith
or fraud or constituted an act oppressive to labor, or was done in a manner contrary to
morals, good customs or public policy.
3. YES

A2010

- 270 -

Disini

- Petitioners failed to submit the pertinent employee files, payrolls, records, remittances
and other similar documents which would show that respondents rendered work entitling
them to payment for overtime work, night shift differential, premium pay for work on
holidays and rest day, and payment of these as well as the COLA and the SILP. By
choosing not to fully and completely disclose information and present the necessary
documents to prove payment of labor standard benefits due to respondents, petitioners
failed to discharge the burden of proof. Indeed, petitioners failure to submit the necessary
documents which as employers are in their possession, inspite of orders to do so, gives
rise to the presumption that their presentation is prejudicial to its cause.
- The cost of meals and snacks purportedly provided to respondents cannot be deducted
as part of respondents minimum wage. Respondents were not interviewed by the DOLE
as to the quality and quantity of food appearing in the applications of petitioners for facility
evaluation prior to its approval to determine whether or not respondents were indeed given
such kind and quantity of food. Also, there was no evidence that the quality and quantity
of food in the Orders were voluntarily accepted by respondents.
- Even granting that meals and snacks were provided and indeed constituted facilities,
such facilities could not be deducted without compliance with certain legal requirements.
The employer simply cannot deduct the value from the employee's wages without
satisfying the following: (a) proof that such facilities are customarily furnished by the trade;
(b) the provision of deductible facilities is voluntarily accepted in writing by the employee;
and (c) the facilities are charged at fair and reasonable value.
- More important, food or snacks or other convenience provided by the employers are
deemed as supplements if they are granted for the convenience of the employer. The
criterion in making a distinction between a supplement and a facility does not so much lie
in the kind (food, lodging) but the purpose. Considering, therefore, that hotel workers are
required to work different shifts and are expected to be available at various odd hours,
their ready availability is a necessary matter in the operations of a small hotel, such as
petitioners business. The deduction of the cost of meals from respondents wages,
therefore, should be removed.
- The 5% of the gross income of the establishment cannot be considered as part of the
respondents wages. Although called profit share, such is in the nature of share from
service charges charged by the hotel.
- Petitioners themselves have admitted that the establishment employs more or less
sixteen employees, therefore they are estopped from claiming that the applicable
minimum wage should be for service establishments employing 15 employees or less.
- The employer cannot exempt himself from liability to pay minimum wages because of
poor financial condition of the company. The payment of minimum wages is not
dependent on the employers ability to pay.
Disposition Petition is denied. The case is REMANDED to the Labor Arbiter for the
RECOMPUTATION of the total monetary benefits awarded and due to the employees
concerned in accordance with the decision.

TEMPORARY RETRENCHMENT
SEBUGERO V NLRC (GTI SPORTSWEAR)
248 SCRA 532
DAVIDE JR; September 27, 1995
NATURE
Special civil action for certiorari
FACTS
- Petitioners were among the thirty-eight (38) regular employees of private respondent
GTI Sportswear who were given "temporary lay-off"notices due to alleged lack of work and
heavy losses caused by the cancellation of orders from abroad and by the garments
embargo. Believing that their "temporary lay-off" was a ploy to dismiss them, resorted to
because of their union activities and was in violation of their right to security of tenure, laidoff employees filed complaint with Labor Arbiter.
- GTI denied the claim of illegal dismissal and asserted that it was its prerogative to lay-off
its employees temporarily for a period not exceeding six months to prevent losses, and
that the lay-off affected both union and non-union members. It justified its failure to recall
the 38 laid-off employees after the lapse of six months because of the subsequent
cancellations of job orders made by its foreign principals.
- 22 of the 38 complainants accepted the separation pay. The petitioners herein did not.
- LA said there was justification to lay-off temporarily some employees. That their
principals transferred their orders were proven by correspondence. Although, as a general
rule, Respondent company has the prerogative and right to resort to temporary lay-off,
such right is likewise limited to a period of six (6) months applying Art. 286. However,
Respondent company should have recalled them after the end of the six month period or
at the least reasonably informed them (complainants) that the Respondent company is still
not 'in a position to recall them, and if the same cannot be met, then the company should
implement retrenchment and pay its employees separation pay. Hence, there is in this
complaint a clear case of constructive dismissal.
Reinstatement is not prudent,

Labor Law 1
separation pay is in order.
- GTI appealed to NLRC. NLRC concurred with the findings of the Labor Arbiter that there
was a valid lay-off of the petitioners due to lack of work, but disagreed with the latter's
ruling granting back wages. NLRC said that having established lack of work, it follows that
retrenchment took place and not constructive dismissal.
- Petitioners filed this action.
ISSUES
1. WON the ground for termination of employment was redundancy
2. WON termination was legal
HELD
1. NO, the ground in this case is retrenchment.
- Redundancy exists where the services of an employee are in excess of what is
reasonably demanded by the actual requirements of the enterprise.
- Retrenchment on the other hand, is used interchangeably with the term "lay-off." It is the
termination of employment initiated by the employer through no fault of the employee's
and without prejudice to the latter, resorted to by management during periods of business
recession, industrial depression, or seasonal fluctuations, or during lulls
2. YES, it is legal, but it is defective.
- Six months is the period set by law that the operation of a business or undertaking may
be suspended thereby suspending the employment of the employees concerned. The
temporary lay-off wherein the employees likewise cease to work should also not last
longer than six months. After six months, the employees should either be recalled to work
or permanently retrenched.
- Failing to comply with this would be tantamount to dismissing the employees and the
employer would thus be liable for such dismissal.
- We must determine whether there was compliance with the law regarding a valid
retrenchment at anytime within the six month-period that they were temporarily laid-off.
Three basic requisites for valid retrenchment:
- it is necessary to prevent losses and losses are proven
- written notice to employees and DOLE at least 1 mo prior to intended date of
retrenchment
- separation pay
- Here, both the Labor Arbiter and the NLRC found that the private respondent was
suffering and would continue to suffer serious losses.
- In this case, it is undisputed that the petitioners were given notice of the temporary layoff. There is, however, no evidence that any written notice to permanently retrench them
was given at least one month prior. There is also nothing in the records to prove that a
written notice was ever given to the DOLE.
- With respect to the payment of separation pay, the NLRC found that GTI offered to give
the petitioners their separation pay but that the latter rejected such offer which was
accepted only by 22 out of the 38 original complainants.
- CONCLUSION: RETRENCHMENT IS DEFECTIVE IN THE FACE OF FINDING THAT
REQUIRED NOTICES TO PETITIONERS AND DOLE ARE NOT GIVEN. But this doesnt
make the retrenchment illegal.
- Where the dismissal of employee is for just cause and is proven to be but he is not
accorded right to due process, dismissal is upheld but employer must be sanctioned for
non-compliance.

REQUIREMENTS STANDARDS
LOPEZ SUGAR CORP V FED OF FREE WORKERS
PHILIPPINE LABOR UNION ASSOCIATION (PLUANACUSIP)
189 SCRA 179
FELICIANO; August 30, 1990
NATURE
Certiorari
FACTS
- Lopez Sugar Corporation (LPC), allegedly, to prevent losses due to major economic
problems, and exercising its privilege under the 1975-1977 CBA entered into with PLUANACUSIP, caused the retrenchment and retirement of a number of its employees.
- LPC filed with the MOLE a combined report on retirement and application for clearance
to retrench affecting eighty six (86) of its employees. Of these 86 employees, 59 were
retired and 27 were to be retrenched in order to prevent losses.
- Federation of Free Workers (FFW), as the certified bargaining agent of the rank-and-file
employees of LPC, also filed with the MOLE a complaint for unfair labor practices and
recovery of union dues.

A2010

- 271 -

Disini

- In said complainant, FFW claimed that the terminations undertaken by LPC were
violative of the security of tenure of its members and were intended to "bust" the union and
hence constituted an unfair labor practice.
- FFW claimed that after the termination of the services of its members, LPC advised 110
casuals to report to its personnel office.
- FFW further argued that to justify retrenchment, serious business reverses must be
"actual, real and amply supported by sufficient and convincing evidence." FFW prayed for
reinstatement of its members who had been retired or retrenched.
- LPC denied having hired casuals to replace those it had retired or retrenched. It
explained that the announcement calling for 110 workers to report to its personnel office
was only for the purpose of organizing a pool of extra workers which could be tapped
whenever there were temporary vacancies by reason of leaves of absence of regular
workers.
- LA: denied LPC s application for clearance to retrench its employees on the ground that
for retrenchment to be valid, the employer's losses must be serious, actual and real and
must be amply supported by sufficient and convincing evidence. The application to retire
was also denied on the ground that LPC's prerogative to so retire its employees was
granted by the 1975-77 CBA had long ago expired. LPC was, therefore, ordered to
reinstate 27 retired or retrenched employees represented by PLUAand FFW and to pay
them full backwages from the time of termination until actual reinstatement.
- On appeal, the NLRC, finding no justifiable reason for disturbing the decision of the
Labor Arbiter, affirmed that decision
ISSUE
WON NLRC acted with GAD in denying LPC s combined report on retirement and
application for clearance to retrench
HELD
NO
- A283 of the LC provides:
Article 283. Closure of establishment and reduction of personnel. The employer may
also terminate the employment of any employee due to the installation of labor saving
devices, redundancy, retrenchment to prevent losses or the closing or cessation of
operation of the establishment or undertaking unless the closing is for the purpose of
circumventing the provisions of this Title, by serving a written notice on the workers and
the Ministry of Labor and Employer at least one (1) month before the intended date
thereof. In case of termination due to the installation of labor saving devices or
redundancy, the worker affected thereby shall be entitled to a se pay equivalent to at
least his one (1) month pay or to at least one (1) month pay for every year of service,
whichever is higher. In case of retrenchment to prevent losses and in cases, of
closures or cessation of operations of establishment or undertaking not due to serious
business losses or financial reverses, the separation pay shall be equivalent to one (1)
month pay or at least one half (1/2) month pay for every year of service, whichever is
higher. A fraction of at least six (6) months shall be considered one (1) whole year.
- In ordinary connotation, the phrase "to prevent losses" means that retrenchment or
termination of the services of some employees is authorized to be undertaken by the
employer sometime before the losses anticipated are actually sustained or realized. It is
not, in other words, the intention of the lawmaker to compel the employer to stay his hand
and keep all his employees until sometime after losses shall have in fact materialized ; if
such an intent were expressly written into the law, that law may well be vulnerable to
constitutional attack as taking property from one man to give to another
- When, or under what circumstances does the employer becomes legally privileged to
retrench and reduce the number of his employees?
- The general standards in terms of which the acts of petitioner employer must be
appraised:
1) the losses expected should be substantial and not merely de minimis in
extent. If the loss purportedly sought to be forestalled by retrenchment is clearly shown
to be insubstantial and inconsequential in character, the bona fide nature of the
retrenchment would appear to be seriously in question.
2) The substantial loss apprehended must be reasonably imminent, as such imminence
can be perceived objectively and in good faith by the employer. There should, in other
words, be a certain degree of urgency for the retrenchment, which is after all a
drastic recourse with serious consequences for the livelihood of the employees retired
or otherwise laid-off.
3) Because of the consequential nature of retrenchment, it must be reasonably
necessary and likely to effectively prevent the expected losses. The employer should
have taken other measures prior or parallel to retrenchment to forestall losses, i.e., cut
other costs than labor costs. To impart operational meaning to the constitutional policy
of providing "full protection" to labor, the employer's prerogative to bring down labor
costs by retrenching must be exercised essentially as a measure of last resort, after
less drastic means e.g., reduction of both management and rank-and-file bonuses
and salaries, going on reduced time, improving manufacturing efficiencies, trimming of
marketing and advertising costs, etc. have been tried and found wanting.
4) If already realized, and the expected imminent losses sought to be forestalled, must
be proved by sufficient and convincing evidence. The reason for requiring this

Labor Law 1
quantum of proof is readily apparent: any less exacting standard of proof would render
too easy the abuse of this ground for termination of services of employees.
-Garcia v. National Labor Relations Commissions:
. . . But it is essentially required that the alleged losses in business operations must be
prove[n] (NAFLU vs. Ople, [1986]). Otherwise, said ground for termination would be
susceptible to abuse by scheming employers who might be merely feigning business
losses or reverses in their business ventures in order to ease out employees .
- WON an employer would imminently suffer serious or substantial losses for economic
reasons is essentially a question of fact for the Labor Arbiter and the NLRC to determine.
- In the instant case, the LA found no sufficient and convincing evidence to sustain
petitioner's essential contention that it was acting in order to prevent substantial and
serious losses.
- The principal difficulty with LPC' s case as above presented was that no proof of actual
declining gross and net revenues was submitted. No audited financial statements showing
the financial condition of petitioner corporation during the above mentioned crop years
were submitted.
- LPC conspicuous failed to specify the cost-reduction measures actually undertaken in
good faith before resorting to retrenchment. Upon the other hand, it appears from the
record that petitioner, after reducing its work force, advised 110 casual workers to register
with the company personnel officer as extra workers.
- LPC argued that it did not actually hire casual workers but that it merely organized a pool
of "extra workers" from which workers could be drawn whenever vacancies occurred by
reason of regular workers going on leave of absence but the LA and the NLRC did not
accord much credit to LPC's explanation.
*AS REGARDS The RETIREMENTS effected by LPC
- On this point, SC finds for LPC saying that although the CBA expired on 31 December
1977, it continued to have legal effects as between the parties until a new CBA had been
negotiated and entered into. This proposition finds legal support in Article 253 of the
Labor Code, which provides:
Article 253 Duty to bargain collectively when there exists a collective bargaining
agreement. When there is a collective bargaining agreement, the duty to bargain
collectively shall also mean that neither party shall terminate nor modify such
agreement during its lifetime. However, either party can serve a written notice to
terminate or modify the agreement at least sixty (60) days prior to its expiration date. It
shall be the duty of both parties to keep the status quo and to continue in full force and
effect the terms and conditions of the existing agreement during the 60-day period
and/or until a new agreement is reached by the parties. (Emphasis supplied)
- Accordingly, in the instant case, despite the lapse of the formal effectivity of the CBA by
virtue of its own provisions, the law considered the same as continuing in force and effect
until a new CBA shall have been validly executed.
- Hence, LPC acted within legal bounds when it decided to retire several employees in
accordance with the CBA. That the employees themselves similarly acted in accordance
with the CBA is plain from the record.
- Even after the expiration of the CBA, LPC's employees continued to receive the benefits
and enjoy the privileges granted therein. If the workers chose to avail of the CBA despite
its expiration, equity if not the lawdictates that the employer should likewise be able
to invoke the CBA.
- The fact that several workers signed quitclaims will not by itself bar them from joining in
the complaint. Quitclaims executed by laborers are commonly frowned upon as contrary to
public policy and ineffective to bar claims for the full measure of the worker's legal rights.
- AFP Mutual Benefit Association, Inc. v. AFP-MBAI-EU:
In labor jurisprudence, it is well establish that quitclaims and/or complete releases
executed by the employees do not estop them from pursuing their claims arising from
the unfair labor practice of the employer. The basic reason for this is that such
quitclaimants and/or complete releases are against public policy and, therefore, null
and void. The acceptance of termination pay does not divest a laborer of the right to
prosecute his employer for unfair labor practice acts.
- Cario vs. ACCFA, (1966) ~ Justice Sanchez, said:
Acceptance of those benefits would not amount to estoppel. The reason is plain.
Employer and employee, obviously, do not stand on the same footing The employer
drove the employee to the wall. The latter must have to get hold of money. Because,
out of job, he had to face the harsh necessities of life. He thus found himself in no
position to resist money proffered. His, then, is a case of adherence, not of choice. One
thing sure, however, is that petitioners did not relent their claim. They pressed it. They
are deemed not to have waived any of their rights. Renuntiatio non praesumitur
Disposition Petition for Certiorari is partially GRANTED and NLRC s decision affirming
that portion of the Decision of the Labor Arbiter ordering the reinstatement judgment of
employees who had been retired by LPC under the applicable provisions of the CBA is
AFFIRMED.
(*all illegally retrenched were ordered to be reinstated and given backwages; those who
executed quitclaims-said amount shall be deducted from their backwages and where
reinstatement is no longer possible, backwages + separation pay na lang. BUT those who
were retired by LPC were found to be valid as per the CBA.

A2010

- 272 -

Disini

EMCO PLYWOOD CORP V ABELGAS


[PAGE 14]

BLUCOR MINERALS CORP V AMARILLA


458 SCRA 37
PANGANIBAN; May 4, 2005
NATURE
Petition for review on certiorari
FACTS
- Respondent Amarilla is a regular employee of herein Petitioner Blucor Minerals
Corporation ('Blucor,' for brevity), having worked at Blucor's mining business since
January 4, 1987 and assigned in the latter's internal security force. Co-respondent Aldiano
is also a regular employee of Blucor, assigned at the latter's services and engineering
department since August 9, 1995 and continued to work thereat in the course of Blucor's
usual business operations. Co-respondent Parcon is likewise a regular employee of Blucor
since September 10, 1998, assigned at the latter's field operation. On July 31, 2000,
Blucor notified Amarilla, Aldiano and Parcon to the effect that Blucor is terminating their
employment
due
to
retrenchment,
effective
August
31,
2000.
- Respondents questioned their termination from employment by filing a complaint before
the arbitration branch of NLRC, for illegal dismissal against Blucor Minerals Corporation
and its President, alleging that herein petitioner did not incur substantial losses in its
operation so as to justify their retrenchment.
- Blucor Minerals Corporation opposed the complaint, alleging that they maintained two
operations called Blucor-1 and Blucor-2 and that due to the economic crisis affecting the
country and the fact that the Blucor-1 operation yielded gold with low grade or quality,
Blucor incurred serious business losses, so that it resorted to cost-reduction measures by
closing the Blucor-2 operation and retrenching some workers, including herein
respondents. In support thereof, Blucor presented its annual income tax return and
financial statements showing the net loss in the amount of P2,038,846.10 for the year
2000.
- After the parties filed their respective pleadings, Executive Labor Arbiter Villanueva
rendered his decision, declaring Blucor Minerals Corporation and its President guilty of
illegal dismissal
- Labor Arbiter Villanueva also ruled that since reinstatement is no longer feasible in this
case, payment of separation pay is more appropriate.
- Blucor Minerals Corporation appealed to NLRC, contending that the Labor Arbiter
committed grave abuse of discretion in ignoring the financial statement for the year 2000
operations as adequate proof of their serious business losses warranting the retrenchment
of complainants. Blucor argued that the jurisprudence, i.e., Lopez Sugar Corporation v.
Federation of Free Workers and Asian Alcohol Corporation v. NLRC, cited by the Labor
Arbiter are obsolete and no longer controlling.
- Subsequently, NLRC promulgated its assailed resolution, vacating and setting aside the
decision of Labor Arbiter Villanueva, and dismissing in the main the complaint for illegal
dismissal. It held that Blucor was justified in terminating the services of [respondents] due
to retrenchment under Article 283 of the Labor Code. The NLRC ordered Blucor to pay
respondents their separation pay, service incentive leave pay and proportionate 13th
month pay.
- Respondents moved for the reconsideration of the September 14, 2001 resolution of the
NLRC. Alleging that the said resolutions of NLRC were issued with grave abuse of
discretion amounting to lack or excess of jurisdiction, respondents filed a Petition for
Certiorari with the CA.
- The CA ruled that the dismissal was unjustified, as Blucor had failed to prove with clear
and satisfactory evidence that legitimate business reasons existed to justify retrenchment.
By themselves, Blucor's Income Tax Return and Audited Financial Statements for the year
2000 did not sufficiently prove the existence or the imminence of substantial losses.
ISSUE
WON the CA was correct in disregarding the Income Tax Return and the Audited Financial
Statements of the herein petitioners as basis for retrenchment due to serious business
losses
HELD
YES
- Before any reduction of personnel becomes legal, any claim of actual or potential
business losses must satisfy the following established standards: (1) the losses incurred
are substantial, not de minimis; (2) the losses are actual or reasonably imminent; (3) the
retrenchment can be fairly regarded as necessary and likely to be effective in preventing
the expected losses; and (4) sufficient and convincing evidence prove the alleged losses,
if already incurred, or the expected imminent losses sought to be forestalled are proven.

Labor Law 1
- It is a well-settled rule that the employer bears the burden of proving the existence or the
imminence of substantial losses, a burden that is by nature an affirmative defense. It is the
duty of the employer to prove with clear and satisfactory evidence that legitimate business
reasons exist to justify retrenchment; failure to do so necessarily results in a finding that
the dismissal was unjustified. Absent any convincing evidence that the alleged losses are
substantial and actual, the dismissal of employees would be unjustified.
- To prove that it incurred losses, petitioners presented its Income Tax Return and Audited
Financial Statements for the year 2000 alone. Previously, however, the company had
admittedly enjoyed profitable initial years of operation. This situation falls short of the
stringent requirement of the law that the employer must sufficiently and convincingly prove
its allegation of substantial losses. It is interesting to note that while petitioners admit that
the company began to incur losses only in 2000, Blucor effectively terminated
respondents' services on August 31 of that same year. When respondents requested
copies of the company's financial statement to see for themselves the state of Blucor's
finances, petitioners informed them that their request was premature. The latter would now
have this Court decide that the corporation indeed suffered substantial losses
necessitating retrenchment, based solely on the tax returns and financial statements for
the year 2000.
- Petitioners likewise failed to show any reasonable necessity for the retrenchment.
Although the law recognizes it as a valid means to avert substantial losses, retrenchment
must be exercised as a "measure of last resort when other less drastic means have been
tried and found to be inadequate.
Disposition Petition denied

A2010

- 273 -

Disini

(4) that the employer exercises its prerogative to retrench employees in good faith for
the advancement of its interest and not to defeat or circumvent the employees' right to
security of tenure; and
(5) that the employer used fair and reasonable criteria in ascertaining who would be
dismissed and who would be retained among the employees, such as status (i.e.,
whether they are temporary, casual, regular or managerial employees), efficiency,
seniority, physical fitness, age, and financial hardship for certain workers.
- What the law speaks of is serious business losses or financial reverses. Sliding
incomes or decreasing gross revenues are not necessarily losses, much less serious
business losses within the meaning of the law. The bare fact that an employer may have
sustained a net loss, such loss, per se, absent any other evidence on its impact on the
business, nor on expected losses that would have been incurred had operations been
continued, may not amount to serious business losses mentioned in the law. 50 The
employer must also show that its losses increased through a period of time and that the
condition of the company will not likely improve in the near future.
Disposition Petition granted.

NATURE OF LOSS
LOPEZ SUGAR CORP V FEDERATION OF FREE
WORKERS
[PAGE 150]

SAN MIGUEL CORP V ABELLA


[PAGE 59]

EDGE APPAREL INC V NLRC


[PAGE 262]

PHIL CARPET EMPLOYEES ASSN V STO TOMAS


483 SCRA 128
CALLEJO SR; February 22, 2006

BOGO-MEDELLIN SUGARCANE PLANTERS ASSN V


NLRC (ALU, MONTILLA)
296 SCRA 108
PANGANIBAN; September 25, 1998

FACTS
- Phil. Carpet Mfg. Corp. (PCMC) retrenched 88 workers in 2004. Of the 88, 77 were
Union members. The Union alleged illegal dismissal. The company said they were
suffering losses due to the Asian financial crisis, the 9/11 and the Iraq war. They showed
figures to the effect that the net income and net profit of the company decreased in
increasing percentages from 2000 to 2001 to 2002 to 2003.
- [The SC later found out that the decrease in net income ensued because PCMC
declared cash dividends for its shareholders amounting to P28,000,000.00.]
ISSUE
WON the dismissals were illegal
HELD
YES
- Retrenchment is an authorized cause for the termination of employment under Article
283 of the Labor Code. However, the retrenchment effected by PCMC is invalid due to a
substantive defect, non-compliance with the substantial requirements to effect a valid
retrenchment; it necessarily follows that the termination of the employment of petitioner
Union's members on such ground is, likewise, illegal.
- Retrenchment is defined as the termination of employment initiated by the employer
through no fault of the employee and without prejudice to the latter, resorted by
management during periods of business recession, industrial depression or seasonal
fluctuations or during lulls over shortage of materials. It is a reduction in manpower, a
measure utilized by an employer to minimize business losses incurred in the operation of
its business.
- The prerogative of an employer to retrench its employees must be exercised only as a
last resort, considering that it will lead to the loss of the employees' livelihood. It is justified
only when all other less drastic means have been tried and found insufficient or
inadequate. Moreover, the employer must prove the requirements for a valid retrenchment
by clear and convincing evidence; otherwise, said ground for termination would be
susceptible to abuse by scheming employers who might be merely feigning losses or
reverses in their business ventures in order to ease out employees.
- The requirements are:
(1) that the retrenchment is reasonably necessary and likely to prevent business losses
which, if already incurred, are not merely de minimis, but substantial, serious, actual
and real, or if only expected, are reasonably imminent as perceived objectively and in
good faith by the employer;
(2) that the employer served written notice both to the employees and to the
Department of Labor and Employment at least one month prior to the intended date of
retrenchment;
(3) that the employer pays the retrenched employees separation pay equivalent to one
month pay or at least 1/2 month pay for every year of service, whichever is higher;

FACTS
- The workers performed the functions of computer, sampler and scalers. They joined and
became members of Associated Labor Unions, with Bonifacio Montilla as its. The
company told him to withdraw his membership from the Associated Labor Unions or else
they will not be hired at the start of the milling season and will be dismissed. The workers
did not heed the warning and stuck to their membership with the private respondent union.
As a consequence and as earlier warned of being dismissed if they persisted in their union
activities, notices of termination were sent to the workers informing them that their
services will be terminated due to financial difficulties. While the said notices stated that
their services will be terminated 30 days from date, they were not allowed to work within
that 30 day period and Montilla was immediately replaced by Gavino Negapatan. The
workers alleged that their dismissal was sought due to their membership [in] the private
respondent union as they have not violated any company rules and regulations. There is
also no allegation to this effect by the respondents and the latter strongly advocated
retrenchment to prevent losses as their basis in terminating the [private respondents].
Aggrieved of the respondents' actuations they filed the present complaint or before the
expiration of the 30 days notice. Just on the 30th day of the notice of termination, four of
the workers were paid their corresponding separation/gratuity pay and accordingly signed
their Quitclaim and Release.
ISSUE
WON the retrenchment was justified
HELD
NO
- Retrenchment is the termination of employment effected by management during periods
of business recession, industrial depression, seasonal fluctuations, lack of work or
considerable reduction in the volume of the employer's business. Resorted to by an
employer to avoid or minimize business losses, it is a management prerogative
consistently recognized by this Court and allowed under Article 283 of the Labor Code as
follows:
"ART. 283.
Closure of
establishment and reduction of personnel. The employer may also terminate the
employment of any employee due to the installation of labor saving devices,
redundancy, retrenchment to prevent losses or the closing or cessation of operation of
the establishment or undue taking unless the closing is for the purpose of
circumventing the provisions of this Title by serving a written notice on the workers and
the Ministry of Labor and Employment at least one (1) month before the intended date
thereof . . . In case of retrenchment to prevent losses . . ., the separation pay shall be

Labor Law 1
equivalent to one (1) month pay for every year of service, which ever is higher. A
fraction of at least six (6) months shall be considered one (1) whole year."
- In a number of cases, the Court has laid down the following requisites of a valid
retrenchment: (1) the losses incurred are substantial and not de minimis; (2) the losses
are actual or reasonably imminent; (3) the retrenchment is reasonably necessary and is
likely to be effective in preventing the expected losses; and (d) the alleged losses, if
already incurred, or the expected imminent losses sought to be forestalled, are proven by
sufficient and convincing evidence. In the present case, petitioners miserably failed to
prove (1) substantial losses and (2) the reasonable necessity of the retrenchment.
No Sufficient and Substantial
Evidence of Business Loss
- To justify retrenchment, the employer must prove serious business losses. Indeed, not
all business losses suffered by the employer would justify retrenchment under this article.
The Court has held that the "'loss' referred to in Article 283 cannot be just any kind or
amount of loss; otherwise, a company could easily feign excuses to suit its whims and
prejudices or to rid itself of unwanted employees."
- In the case at bar, Petitioner Corporation claimed that the retrenchment of private
respondents was justified, because it suffered business losses, as evidenced by its
Comparative Statement of Revenue and Expenses for crop years for two years. In their
rebuttal, petitioners allege the following: (1) the comparative financial statement of the
corporation duly reflects its income and expenses in a given taxable year and, despite its
different nomenclature, is substantially the same as a profit and loss statement or any
other financial statement; and (2) the National Internal Revenue Code (NIRC) requires the
certification of an independent certified public accountant only if the taxpayer's gross
receipts exceed P25,000 in any quarter of any taxable year.
- The contentions of petitioners are untenable. A comparative statement of revenue and
expenses for two years, by itself, is not conclusive proof of serious business losses. The
Court has previously ruled that financial statements audited by independent external
auditors constitute the normal method of proof of the profit and loss performance of a
company. While Petitioner Corporation avers that it was not required to file audited
financial statements under Section 232 of the Tax Code, it failed to establish its exemption
through any evidence showing that its quarterly gross revenues did not exceed P25,000.
Thus, its claim that it did not need to have its financial statements certified by a certified
public accountant is without basis in fact and in law and does not excuse it from complying
with the usual requirement. Besides, the requirement of the Tax Code is one thing, and the
requirement of the Labor Code is quite another. Moreover, the financial statement of
Petitioner Corporation for two crop years is insufficient proof of serious business losses
that would justify the retrenchment of private respondents.

CAMA V JONIS FOOD SERVICES


425 SCRA 259
QUISUMBING; March 10, 2004
NATURE
Petition for review on certiorari of a decision and resolution of the CA
FACTS
- Respondent Jonis Food Services, Inc., (JFSI) is a corporation engaged in the coffee
shop and restaurant business, with several branches or outlets. Feliciano is its president
and general manager.
- Petitioners were employees of JFSI having been hired on various dates during the 1970s
to the 1990s.
- In the 1990s, JFSI had eight (8) outlets for its coffee shop and restaurant business.
- In 1997, faced with dropping sales, however, it shut down three of these shops to avert
serious business losses.
- 1998 saw JFSI operations in the red. JFSI incurred a total net loss of P2,541,537.70 as
of December 31, 1998. As a result, JFSI shut down more outlets, leaving it with just three
operating outlets at the end of 1998.
- Bleak business conditions continued to plague the company and by the end of the first
quarter of 1999, the remaining branches were also closed.
- On April 5, 1999, the petitioners, filed a complaint for illegal dismissal, separation pay,
service incentive leave pay, 13 th month pay, attorneys fees, remittance of SSS and PagIbig contributions, and refund of excess withholding taxes against JFSI.
- Following failed attempts to reach an amicable settlement between complainants and
respondents, formal hearings ensued before the Labor Arbiter.
- On October 25, 1999, the Labor Arbiter rendered a decision finding respondent not liable
for illegal dismissal, but directing it to pay the complainants separation pay, service
incentive leave pay and attorneys fees
- In holding that the petitioners were entitled to separation pay, the Labor Arbiter
recognized that JFSI did suffer business losses, but that he did not consider these serious
enough so as to warrant denial of the petitioners separation pay.
- Respondents appealed to the NLRC

A2010

- 274 -

Disini

- NLRC affirmed the decision appealed from with the modification of deleting the award for
attorneys fees.
- JFSI moved for reconsideration, but this was denied
- It filed a special civil action for certiorari with the Court of Appeals
- The Court of Appeals granted the writ of certiorari prayed for by JFSI, and declared the
decision of the NLRC null and void for having been issued with grave abuse of discretion.
- Petitioners moved for reconsideration, but the appellate court denied the motion in its
resolution
ISSUE
WON the termination of petitioners employment due to serious business losses suffered
by JFSI precluded payment of separation pay
HELD
YES
- In the instant case, it was duly established that respondent company incurred losses in
1998, which led it to close several outlets, resulting in the lay-off of petitioners. Both the
Labor Arbiter and the NLRC ruled that the losses were not so severe as to prevent the
respondent from paying separation pay to the petitioners. The Court of Appeals ruled to
the contrary.
- To find out whether the appellate court erred or not, a review of the evidence based on
financial statements using ratio analyses is called for.
- To test the liquidity of JFSI for the periods under consideration, working capital ratio is
used. The current ratios derived clearly show that in 1997, JFSI did not have sufficient
current assets to pay its current liabilities. It was even less liquid in 1998, indicating that
the firm faced difficulty in paying current obligations as they fell due.
- Using the gross profit ratio, which indicates the mark-up on the companys products, we
find that in 1997, for every peso of sales generated by respondent, gross profit or mark-up
amounted to almost 52 centavos, but this went down in 1998 to just a fraction over 49
centavos
- If we were to employ the net profit (loss) ratio, which is widely used as a measure of the
overall profitability of business operations, it can be seen that for every peso of sales
earned by the respondent in 1997, less than half a centavo (0.39%) represented profit.
This is an inadequate and unsatisfactory profit ratio. The situation was worse in 1998,
since for every peso of sales made by respondent, no profit was made, but instead a loss
was incurred. A loss of 50.1% recorded in 1998 appears to us quite serious.
- The Constitution, while affording full protection to labor, nonetheless, recognizes the
right of enterprises to reasonable returns on investments, and to expansion and growth.
- In line with this protection afforded to business by the fundamental law, Article 283 of the
Labor Code clearly makes a policy distinction. It is only in instances of retrenchment to
prevent losses and in cases of closures or cessation of operations of establishment or
undertaking not due to serious business losses or financial reverses that employees
whose employment has been terminated as a result are entitled to separation pay.
- In other words, Article 283 of the Labor Code does not obligate an employer to pay
separation benefits when the closure is due to serious losses. To require an employer to
be generous when it is no longer in a position to do so, in our view, would be unduly
oppressive, unjust, and unfair to the employer.
Disposition The petition is denied for lack of merit.

PHILIPPINE CARPET V STO. TOMAS


[PAGE 272]

SLIDING INCOME
SAN MIGUEL JEEPENEY SERVICE V NLRC
265 SCRA 35
PANGANIBAN; November 28, 1996
NATURE
Civil action of Certiorari alleging grave abuse of discretion of NLRC
FACTS
- The 23 complainants were formerly working (as drivers, dispatchers and mechanic) with
San Miguel Jeepney Service (SMJS), with services ranging from two to eight years.
SMJS had a contract with the U.S. Naval Base Facility located in San Miguel, San
Antonio, Zambales, to provide transportation services to personnel and dependents inside
said facility.
- When this contract expired, owner and general manager of SMJS, opted not to renew
the existing contract nor bid on the new contract, due to financial difficulties, he having
suffered a net loss the prior year. As a consequence, the services of the complainants
were terminated.
- By that time, however, the 23 had already filed a complaint for non-compliance with the

Labor Law 1
minimum wage law from 1980 onwards, plus non-payment of the 13 th month pay, legal
holiday pay, overtime pay, service incentive leave pay and separation pay.
- Labor Arbiter: Complainants were not entitled to such benefits (13 th month, holiday,
service incentive pay), being workers on a purely commission basis.
- NLRC: Modified the LAs ruling, holding that all the complainants are regular employees
in the contemplation of Art 280; thus, they are entitled to separation pay.
Petitioners Claims
> They cannot be held liable for separation pay because it had been experiencing
financial reverses since 1986. They cited figures showing sliding incomes.
> They argue that in order to award separation pay, there must be some numerical and
factual basis for the computation thereof, which they claim is absent in this case.
ISSUES
1. WON there is factual bases for SMJS contention that it has suffered serious business
losses
2. WON complainant-workers should be granted separation pay
HELD
1. NO
Ratio What the law speaks of is serious business losses or financial reverses. Clearly,
sliding incomes (decreasing gross revenues) are not necessarily losses, much less
serious business losses within the meaning of the law.
- Requisites of a valid retrenchment: (a) the losses expected should be substantial and
not merely de minimis in extent; (b) the substantial losses apprehended must be
reasonably imminent; (c) the retrenchment must be reasonably necessary and likely to
effectively prevent the expected losses; and (d) the alleged losses, if already incurred, and
the expected imminent losses sought to be forestalled, must be proved by sufficient and
convincing evidence.
Reasoning
- In the nature of things, the possibility of incurring losses is constantly present, in greater
or lesser degree, in the carrying on of business operations, since many of the factors
which impact upon the profitability or viability of such operations may be substantially
outside the control of the employer. Employer bears the burden of proving his allegation of
economic or business reverses with clear and satisfactory evidence, it being in the nature
of an affirmative defense.
- In this case, the loss per se, absent any other evidence, and viewed in the light of the
amounts of gross receipts the business generated historically, may not be deemed the
serious business loss contemplated by law. Neither did petitioners present any evidence
whatsoever regarding the impact of the said net loss on the business nor on expected
losses that would have been incurred had operations been continued.
2. YES
Ratio Being regular employees, they are entitled to security of tenure and their services
may be terminated only for causes provided by law. They are also to be accorded the
benefits provided under the Labor Code, including separation pay for loss of employment
resulting from retrenchment to prevent losses or closure/cessation of operation not due to
serious business losses.
Reasoning
- Complainants were unarguably performing work necessary and desirable in the business
of SMJS. Without the services rendered by private respondents, petitioners could not
have conducted their business of providing transportation services within the naval base.
This plus the fact that private respondents had each rendered from two to eight years of
service cause them to come squarely within Art. 280 of the LC and are considered regular
employees. The mere fact that they were paid on commission basis does not affect or
change their status as regular employees.
- Being regular employees, they are entitled to the protection of minimum wage status.
Hence, the separation pay due them may be computed on the basis of the minimum wage
prevailing at the time their services were terminated.
Disposition Assailed resolution is AFFIRMED. The separation pay of the private
respondents equivalent to one-half month pay for every year of service shall be computed
at the then prevailing minimum daily wage of P53.

PROOF OF LOSS
LOPEZ SUGAR CORP V FEDERATION OF FREE
WORKERS
[PAGE 270]
BOGO-MEDELLIN SUGAR CANE PLANTERS ASSN INC
V NLRC
[PAGE 273]

A2010

- 275 -

Disini

MITSUBISHI MOTORS V CHRYSLER


[PAGE 102]

DANZAS INTERCONTINENTAL INC V DAGUMAN


456 SCRA 383
TINGA; April 15, 2005
NATURE
Petition for review of the CA decision
FACTS
- Danzas Intercontinental, Inc. is engaged in the business of forwarding and brokerage
Respondents:
- Henry Daguman, Amador Castro, Richard Salvador and Jonas Culala, in a letter dated
August 14, 1999, were notified by the respondents that due to losses the brokerage
department of Danzas to which the complainants belonged shall be closed on September
14, 1999 for which they will be given a separation pay equal to one month or one half
month for every year of service which ever is higher. However, respondents contend that
the brokerage department was not closed and is in fact under operation by Elizabeth
Gotiam, Sevilla Enzon, Shelbie Ocampo, Jenie Prado, Roden Reyes and Rodel Trias who
are newly hired personnel of the respondents precisely to take over the tasks of
complainants and that the names of these people appear as such newly hired employees
in the companys payroll and in the lists of employees that were submitted to the SSS and
the HMDF. the brokerage department was not really closed but its responsibilities just
transferred to Ms. Elizabeth Gotiam and her staff, hence, their dismissal should be
considered illegal and they should be reinstated to their former positions and should be
awarded backwages plus moral and exemplary damages.
- Petitoners:
> Danzas and Claude Schaer claim that they had to close the brokerage department
because its accumulated losses had already amounted to P5.449M and that as a
consequence of such closure they had to resort to the retrenchment of the personnel
belonging to the brokerage department which include the complainants. They further aver
that new employees were hired not to replace private respondents but to monitor the
activities of outside brokers who were engaged to finish the work for the companys
remaining clients. Thus, the engagement of new employees was coterminous with the
completion of the work for these clients and only for the purpose of winding up the
companys brokerage business on account of E.O. 11. Belatedly, they claimed that,
whether they like it or not, the brokerage department can no longer be opened because
respondent company is no longer allowed to engage in the business of brokerage under
Executive Order No. 11 which limits the exercise of the brokers profession to Filipino
nationals only.
- The Labor Arbiter dismissed the complaint . The NLRC affirmed said decision
- Respondents claim that the NLRC committed grave abuse of discretion in affirming the
findings of the Labor Arbiter that there was sufficient evidence to prove that the Brokerage
Department of Danzas Intercontinental has been suffering losses to justify its closure. The
CA denied petitioners appeal.
ISSUES
1. WON petitioners were able to prove that the brokerage department was incurring
substantial business losses which would justify retrenchment.
2. WON the brokerage department was indeed closed down
HELD
1. NO
- The labor arbiter and the NLRC both found that petitioners validly exercised their
management prerogatives recognized under Article 283 of the Labor Code, viz:
- ART. 283. Closure of establishment and reduction of personnel. -- The employer may
also terminate the employment of any employee due to the installation of labor saving
devices, redundancy, retrenchment to prevent losses or the closing or cessation of
operation of the establishment or undertaking unless the closing is for the purpose of
circumventing the provisions of this Title, by serving a written notice on the worker and the
Ministry of Labor and Employment at least one (1) month before the intended date
thereof. In case of termination due to the installation of labor saving devices or
redundancy, the worker affected thereby shall be entitled to a separation pay equivalent to
at least his one (1) month pay or to at least one (1) month pay for every year of service,
whichever is higher. In case of retrenchment to prevent losses and in cases of closures or
cessation of operations of establishment or undertaking not due to serious business
losses or financial reverses, the separation pay shall be equivalent to one (1) month pay
or at least one-half () month pay for every year of service, whichever is higher. A
fraction of at least six (6) months shall be considered one (1) whole year.

Labor Law 1
- It is neither the function of the law nor its intent to supplant the prerogative of
management in running its business, such as, to compel the latter to operate at a
continuing loss simply because it has to maintain its workers in employment. Such an act
would be tantamount to a taking of property without due process of law.HOWEVER, the
burden of proving that the termination was for a valid or authorized cause rests on the
employer who must comply with certain substantive and procedural requirements. For
instance, the requirements for a valid retrenchment which must be proved by clear and
convincing evidence are: (1) that retrenchment is reasonably necessary and likely to
prevent business losses which, if already incurred, are not merely de minimis, but
substantial, serious, actual and real, or if only expected, are reasonably imminent as
perceived objectively and in good faith by the employer; (2) that the employer served
written notice both to the employees and to the Department of Labor and Employment at
least one month prior to the intended date of retrenchment; (3) that the employer pays the
retrenched employees separation pay equivalent to one (1) month pay or at least one-half
() month pay for every year of service, whichever is higher; (4) that the employer
exercises its prerogative to retrench employees in good faith for the advancement of its
interest and not to defeat or circumvent the employees right to security of tenure; and (5)
that the employer used fair and reasonable criteria in ascertaining who would be
dismissed and who would be retained among the employees, such as status, efficiency,
seniority, physical fitness, age, and financial hardship for certain workers.
- The condition of business losses justifying retrenchment is normally shown by audited
financial documents like yearly balance sheets and profit and loss statements as well as
annual income tax returns. Financial statements must be prepared and signed by
independent auditors. Otherwise, they may be assailed as self-serving. Since the losses
incurred must be substantial and actual or reasonably imminent, it is necessary that the
employer show that the losses increased through a period of time and that the condition of
the company is not likely to improve in the near future.
- The same evidence is generally required when the termination of employees is by
reason of closure of the establishment or a division thereof for economic reasons,
although the more overriding consideration is, of course, good faith. The employer must
prove that the cessation of or withdrawal from business operations was bona fide in
character and not impelled by a motive to defeat or circumvent the tenurial rights of
employees. Parenthetically, if the business losses that justify the closure of the
establishment are duly proved, the right of affected employees to separation pay is lost for
obvious reasons. Otherwise, the employer closing his business is obligated to pay his
employees their separation pay.
- It is worth noting in this regard that the employers prerogative to close or abolish a
department or section of his establishment for economic reasons such as to minimize
expenses and reduce capitalization is as much recognized as managements prerogative
to close the entire establishment and cease operations due to adverse economic
conditions. In the instant case, petitioners presented in evidence an affidavit of the
companys financial comptroller, financial statements for the year 1999 and a quarterly
report in support of the companys claim of losses. Petitioners maintain that they were not
given an opportunity to present the companys audited financial statements before the
NLRC because private respondents appeal thereto was dismissed without notice to
petitioners. Since the case was brought to the CA on certiorari, the audited financial
statements attached to petitioners Comment on Petition were no longer considered.
Consequently, the CA ruled that the financial documents presented by petitioners are
insufficient to prove their claim of business losses.
- As they have the burden of proving the existence of an authorized cause, petitioners
should have presented the companys audited financial statements before the labor arbiter
who is in the position to evaluate evidence. That they failed to do so and only presented
these documents to the CA on certiorari is lamentable considering that the admission of
evidence is outside the sphere of the CAs certiorari jurisdiction. Neither can this Court in
the present petition admit in evidence the companys audited financial statements much
more make a ruling on the question of whether the company incurred substantial losses
justifying retrenchment on the basis thereof as this Court is not a trier of facts. Besides, it
cannot even be ascertained from the audited financial statements attached to the instant
petition whether the losses incurred by the company were indeed attributable to the
brokerage department. We therefore agree with the appellate court that petitioners failed
to substantiate their claim of valid retrenchment.
2. NO
- The petitioners evidence in support of their claim that the companys brokerage
department was closed down is wanting. For instance, the records contain a letter from
petitioners addressed to their clients insinuating that private respondents voluntarily left
the company and that the brokerage department was merely reorganized and not closed
down.
- Petitioners assertion that the brokerage department was closed down because it has
been found to be unprofitable and because the company did not want to run afoul of E.O.
11 which prohibits corporations with foreign equity to engage in the brokerage business is
controverted by the foregoing letter which clearly expresses that the brokerage
department was merely restructured to improve brokerage services. Moreover, petitioners
allegations are unsubstantiated and self-serving. Apart from a letter from the SEC
requiring the company to appear and show cause why its articles of incorporation should
not be amended to delete the business of customs brokerage from its primary purpose,

A2010

- 276 -

Disini

which does not even appear to have been complied with, petitioners have not presented
any evidence in support of its bullish claim that the brokerage department has been closed
down.
- On the contrary, respondents contention to the effect that the company continued in
operation is supported by documentary evidence such as affidavits, transshipment permit
and certificate of importation to the effect that the brokerage department continued in
operation at least until June 2000.
- The hiring of new employees purportedly to oversee the performance of outside brokers
negates the need for the termination of private respondents due to closure of the
brokerage department. The termination of respondents was therefore unjustified either as
retrenchment to prevent losses because petitioners evidence to prove business losses
was insufficient, or closure of the establishment because the brokerage department did
not actually cease operations.

BURDEN OF PROOF
SY V CA (SAHOT)
398 SCRA 301
QUISUMBING; February 27, 2003
NATURE
Petition for review on certiorari of a decision of the Court of Appeals
FACTS
- Respondent Jaime Sahot started working as a truck helper for the petitioner when he
was 23. Later on the company were renamed several times until it became SBT Trucking
Corporation. For 36 years before his dismissal, respondent continuously served the
trucking business of the petitioners.
- in 1994, Sahot was 59 years old, he had been incurring absences as he was suffering
from various ailments, particularly the pain in his left thigh. He filed a week-long leave
when he was treated for his various ailments. He filed a formal request for extension of his
leave, and during this time he was threatened that if he refused to go back to work he
would be terminated. He could not retire on pension because petitioners never paid his
correct SSS premiums. He could no longer work as his left thigh hurt abominably.
Eventually petitioners dismissed him from work on June 30.
- Sahot filed a complaint for illegal dismissal. Petitioners claim that Sahot was their
industrial partner; that respondent only became their employee in 1994; that Sahot went
on leave and never reported back to work nor did he file an extension of his leave
(therefore, should be deemed to have voluntarily resigned)
- LA: pro-petitioners (no illegal dismissal, they were industrial partners, but still pay
financial assistance)
- NLRC: pro-respondent (employee, no abandonment of job, entitled to separation pay for
29 years)
- CA: affirmed with modification (employee, with separation pay for 36 years)
ISSUES
1. WON there was an employer-employee relationship between petitioners and
respondent Sahot (or WON Sahot is an industrial partner of the petitioners)
2. WON there was a valid dismissal
3. WON Sahot is entitled to separation pay
HELD
1. YES. No partnership, Sahot was employee.
Ratio The elements to determine the existence of an employment relationship are: (a) the
selection and engagement of the employee; (b) the payment of wages; (c) the power of
dismissal; and (d) the employers power to control the employees conduct. The most
important element is the employers control of the employees conduct, not only as to the
result of the work to be done, but also as to the means and methods to accomplish it.
Reasoning
- Private respondent actually engaged in work as an employee: he did not have the
freedom to determine where he would go, what he would do, and how he would do it;
merely followed instructions of petitioners as long as he was paid his wages.
- ON PARTNERSHIP: A1767, NCC- contract of partnership is where 2 or more persons
bind themselves to contribute money, property or industry to a common fund, with the
intention of dividing the profits among themselves.
as applied in this case: no written agreement exist to prove partnership; no proof
respondent was receiving a share in the profits, no proof that he actively participated in
the management, administration and adoption of policies of the business.
- if doubt exists between the evidence presented by the employer and the employee, the
scales of justice must be tilted in favor of the latter
2. NO
Ratio In termination cases, the burden is upon the employer to show by substantial
evidence that the termination was for lawful cause and validly made. A277, LC puts the

Labor Law 1
burden of proving that the dismissal of an employee was for a valid or authorized cause on
the employer, without distinction whether the employer admits or does not admit the
dismissal. For an employees dismissal to be valid, (a) the dismissal must be for a valid
cause, and (b) the employee must be afforded due process.
Reasoning
- ON VALID CAUSE: if disease as a ground for termination, refer to A284, LC and Sec8,
Book VI, Rule I of the Omnibus Implementing Rules of the Labor Code where a
certification by competent public health authority that the disease is of such nature or at
such a stage that it cannot be cured within a period of 6 months even with proper medical
treatment. If curable, then employee would be required to take a leave, then reinstate to
formal position upon restoration of his normal health. The requirement for a medical
certificate cannot be dispensed with; otherwise, it would sanction the unilateral and
arbitrary determination by the employer of the gravity or extent of the employees illness
and thus defeat the public policy in the protection of labor.
as applied in the case: petitioners did not comply with the medical certificate
requirement before Sahots dismissal was effected
- ON DUE PROCESS: The employer is required to furnish an employee with 2 written notices
before the latter is dismissed: (1) the notice to apprise the employee of the particular acts
or omissions for which his dismissal is sought, which is the equivalent of a charge; and (2)
the notice informing the employee of his dismissal, to be issued after the employee has
been given reasonable opportunity to answer and to be heard on his defense.
as applied in the case: No notice given, but instead what they did to threaten the
employee with dismissal, then actually implement the threat when the occasion
presented itself because of private respondents painful left thigh
3. YES
Ratio. An employee who is terminated because of disease is entitled to separation pay
equivalent to at least one month salary or to one-half month salary for every year of
service, whichever is greater.
as applied in the case: entitled to separation pay computed at P2,080 times 36 years
or P74, 880.
Disposition petition is DENIED and the decision of the Court of Appeals dated February
29, 2000 is AFFIRMED. Petitioners must pay private respondent Jaime Sahot his
separation pay for 36 years of service at the rate of one-half monthly pay for every year of
service, amounting to P74, 880.00, with interest of six per centum (6%) per annum from
finality f this decision until fully paid.

NATIONAL BOOKSTORE INC V CA (YMASA, GABRIEL)


378 SCRA 194
BELLOSILLO; February 27, 2002
FACTS
- Petitioner National Bookstore employed private respondents Ymasa and Gabriel as Cash
Custodian and Head Cashier. They were routinely tasked with counting the previous days
sales and placing them in separate plastic bags to be deposited in INTERBANK and PCIB.
The bags were held for safekeeping in the Branch vault but upon retrieval to deposit the
money with roving tellers, the money was counted again but the amount for PCIB was
short of P42,758.
- Private respondents were asked by Management to explain in writing why they should
not be dismissed for the loss of company funds and were placed under preventive
suspension. Private respondents in turn denied responsibility, emphasizing they had no
access to the vault and that they were thoroughly searched by the guard before leaving.
They also asserted their loyalty and sincerity in their work as they had been employed
there over 13 years.
- Petitioner found their explanation unsatisfactory and terminated them for gross neglect of
duty and loss of confidence. Private respondents filed a complaint for illegal dismissal. The
Labor Arbiter found in their favor, stating that the dismissal was not founded on valid and
justifiable grounds. Petitioners appeal with the NLRC was denied, as was their petition for
certiorari with the CA for lack of merit.
ISSUES
WON private respondents were illegally dismissed
HELD
YES
- The onus of proving that the dismissal of the employee was for a valid and authorized
cause rests on the employer. Failure to discharge the same would mean the dismissal was
not justified and therefore illegal.
- The requisites for a valid dismissal are (a) the employee must be afforded due process
(b) the dismissal must be for a valid cause. Petitioner complied with the first requisite by
furnishing the employees with written notices stating cause for termination, and having
decided to do so, the reasons therefor.

A2010

- 277 -

Disini

- Petitioner accused private respondents of gross neglect of duty and loss of confidence.
Gross negligence is defined as the failure to exercise slight care or diligence. A perusal of
the records show they werent even remotely negligent of their duties. They were able to
illustrate with candor and sincerity the procedure they took prior to the losspetitioners
allegations on the other hand, were not supported by any substantive evidence. Assuming
arguendo they were negligent, a single act cannot be categorized as habitual and thus
cannot be a just cause for dismissal.
- Loss of confidence on the other hand must be based on the willful breach of trust and
founded on clearly established facts. Petitioner failed to establish with certainty the facts
upon which such a breach of confidence could be based. Private respondents were thus
illegally dismissed.
Disposition petition is DENIED for lack of merit

WHEN EFFECTED
LOPEZ SUGAR CORP V FEDERATION OF FREE
WORKERS
[PAGE 270]
CAJUCOM V TPI
451 SCRA 70
SANDOVAL-GUTIERREZ; February 11, 2005
NATURE
Petition for review on certiorari
FACTS
- TPI Philippines Cement Corporation (TP Cement) and TPI Philippines Vinyl Corporation
(TP Vinyl) employed Atty. Benedicto A. Cajucom VII as Vice-President for Legal Affairs
with a monthly salary of P70,000.00.
- As a result of the economic slowdown then experienced in this country, respondent TP
Cement, having no viable projects, shortened its corporate term from 50 years to 2 years
and 7 months. Thus, respondents implemented cost-cutting measures resulting in the
retrenchment or termination from the service of their employees, including petitioner.
- Petitioners Claim
> Petitioner contested respondents action, claiming that his retrenchment was based
erroneously on respondents probable losses, instead of their actual, substantial and
imminent losses, as shown by the following:
(1) an increase or raise in his
monthly salary from P70,000.00 in 1995 to P80,000.00 in 1996;
(2) hiring by
respondents of more marketing and accounting employees for the period from July 1997
to December 1998; (3) acquisition, in 1998, of a warehouse; and (4) expansion in 1998 of
their operations by including sales and marketing of oil products. Petitioner further
claimed that respondents were motivated by revenge in terminating his services. This
stemmed from his October 7, 1996 memorandum to respondents Executive VicePresident Thun Tritasavit, also a respondent herein, questioning his financial transactions
detrimental to respondents interests.
ISSUE
WON the dismissal of the complainant is illegal
HELD
NO
Ratio To be valid, three requisites must concur, as provided in Article 283 of the Labor
Code, as amended, namely: (1) The retrenchment is necessary to prevent losses and the
same is proven; (2) Written notice to the employees and to the DOLE at least one month
prior to the intended date thereof; and (3) Payment of separation pay equivalent to one
month pay or at least month pay for every year of service, whichever is higher.
Reasoning
1. The Court is persuaded that retrenchment due to substantial losses has been
sufficiently established and that the dismissal of complainant pursuant to Art. 283 of the
Labor Code, was justified. TPI acted in good faith in dismissing Cajucom as proven by the
ff acts of the company:
TPI began downsizing their operations as early as April 1996
They moved to a smaller and cheaper three-storey building to reduce rental costs.
Voluntary termination from the service of some employees was encouraged upon legal
consultation to the complainant.
They increased the salary of complainant from P70,000.00 to P80,000.00. In order to
accommodate such increase, respondent Tritasavit agreed to deduct the same from his
own salary, thereby, reducing his (respondent Tritasavits) total monthly salary and
making it lower than that of complainant.

Labor Law 1
They sold some company vehicles and used the proceeds to meet their operational
expenses and pay their obligations.
The audited financial reports prepared by Sycip Gorres Velayo and Co. show that as of
31 December 1997, TPI Philippines Cement Corporation incurred losses at
P12,375,166.00.
Respondent was in fact very honest to complainant by forewarning him, a year in
advance, of the possibility of his separation from the service, should there be no
changes in the economic condition, and by helping complainant in seeking another job
by referring him to other companies.
2. The fact that respondents did not comply with the one-month notice requirement would
not invalidate the dismissal but the petitioner is entitled to an award of nominal damages
which the Court fixed at P20,000.00.
3. It bears reiterating that under Article 283, petitioner is entitled to an award of separation
pay equivalent to one-half (1/2) months pay for every year of service (with a fraction of at
least six (6) months considered one (1) whole year). Since he had been employed for
four (4) years, or from June 1, 1995 to December 30, 1998, with a monthly salary of
P80,000.00, he should be paid P160,000.00 as separation pay.
Disposition petition is party GRANTED. The challenged Decision and the Resolution of
the Court of Appeals are AFFIRMED with MODIFICATION in the sense that respondents
are hereby ordered to pay petitioner (1) P160,000.00 as separation pay; and (2)
P20,000.00 as nominal damages.

PROCEDURE (FOR BOTH RETRENCHMENT AND


REDUNDANCY)
SEBUGERO V NLRC
[PAGE 270]
EMCO PLYWOOD CORP V ABELGAS
[PAGE 14]
INDUSTRIAL TIMBER CORP V ABABON
480 SCRA
YNARES-SANTIAGO; January 5, 2006
NATURE
Petitions for review under Rule 45 ROC assailing the decision of CA, which set aside the
decision of NLRC and the resolution denying MFR
FACTS
- Industrial Plywood Group Corporation (IPGC) is the owner of a plywood plant located at
Agusan, Pequeo, Butuan City, leased to Industrial Timber Corporation (ITC) August 30,
1985 for a period of five years. Thereafter, ITC commenced operation of the plywood
plant and hired 387 workers.
- March 16, 1990, ITC notified the DOLE and its workers that effective March 19, 1990 it
will undergo a "no plant operation" due to lack of raw materials and will resume only after it
can secure logs for milling.
- Meanwhile, IPGC notified ITC of the expiration of the lease contract in August 1990 and
its intention not to renew the same.
- June 26, 1990, ITC notified the DOLE and its workers of shutdown due to the nonrenewal of anti-pollution permit that expired in April 1990. This and the alleged lack of logs
for milling constrained ITC to lay off all its workers until further notice. This was followed by
a final notice of closure or cessation of business operations on August 17, 1990 with an
advice for all the workers to collect the benefits due them under the law and CBA.
- October 15, 1990, IPGC took over the plywood plant after it was issued a Wood
Processing Plant Permit , which included the anti-pollution permit, by the DENR
coincidentally on the same day the ITC ceased operation of the plant.
- Virgilio Ababon, et al. to file a complaint against ITC and IPGC for illegal dismissal, unfair
labor practice and damages. They alleged, among others, that the cessation of ITC's
operation was intended to bust the union and that both corporations are one and the same
entity being controlled by one owner.

- January 20, 1992, after requiring both


parties to submit their respective
position papers, LA held that there was
lack of evidence to prove that it was
used to perpetuate fraud or illegal act;
upheld the validity of the closure; and

A2010

- 278 -

Disini

ordered ITC to pay separation pay of


1/2 month for every year of service.
- Ababon, et al. appealed to the NLRC. NLRC set aside the decision of the LA and ordered
the reinstatement of the employees to their former positions, and the payment of full back
wages, damages and attorney's fees.
- ITC and IPGC filed MFR. However, it was dismissed for being filed out of time having
been filed only on the date of actual receipt by the NLRC on June 29, 1993, three days
after the last day of the reglementary period. Thus, they filed a Petition for Relief from
Resolution, which was treated as a 2nd MFR by the NLRC and dismissed for lack of merit.
Petitioners filed a Notice of Appeal with the SC. Subsequently, they filed MFR/ 2 nd petition
for relief with the NLRC.
- SC dismissed the Notice of Appeal for being a wrong mode of appeal from the NLRC
decision. On the other hand, the NLRC granted the Second Petition for Relief and set
aside all its prior decision and resolutions.
- October 2, 1995, Virgilio Ababon, et al. filed a Petition for Certiorari with the SC.
Pursuant to St. Martin's Funeral Home v. NLRC, it was referred to the CA for appropriate
action and disposition.
- October 21, 2002, the CA set aside the decision on the 2nd petition for relief of the NLRC
and reinstated its initial decision and resolution.
- Both filed their respective MFRs which were denied, hence, the present consolidated
petitions for review
ISSUES
1. WON NLRC should have taken cognizance of the most recent MFR
2. WON Ababon, et al. were illegally dismissed due to the closure of ITC's business; and
whether they are entitled to separation pay, backwages, and other monetary awards
HELD
1. YES
- Substantial justice is best served by allowing the petition for relief despite procedural
defect of filing MFR
- Under Art 218 (c) LC, NLRC may, in the exercise of its appellate powers, correct, amend,
or waive any error, defect or irregularity whether in substance or in form. Further, Art 221
LC provides that in any proceeding before the Commission or any of the LA, the rules of
evidence prevailing in courts of law or equity shall not be controlling and it is the spirit and
intention of this Code that the Commission and its members and the Labor Arbiters shall
use every and all reasonable means to ascertain the facts in each case speedily and
objectively and without regard to technicalities of law or procedure, all in the interest of
due process.
- Sec14 Rules of Procedure of the NLRC that MFR shall not be entertained except when
based on palpable or patent errors, provided that the motion is under oath and filed within
10 calendar days from receipt of the order, resolution or decision should not be interpreted
as to sacrifice substantial justice to technicality. The limitation of the period is to forestall or
avoid an unreasonable delay in the administration of justice, from which the NLRC
absolved ITC and IPGC because the filing of their MFR was due to excusable negligence.
2. NO
- The records reveal that respondent ITC actually underwent 'no plant operation' since 19
March 1990 due to lack of log supply. This s admitted by complainants. Since then several
subsequent incidents prevented respondent ITC to resume its business operations.
Without the raw materials respondent ITC has nothing to produce. Without the permits it
cannot lawfully operate the plant. And without the contract of lease respondent ITC has no
option but to cease operation and turn over the plant to the lessor.
- The right to close the operation of an establishment or undertaking is one of the
authorized causes in terminating employment of workers, the only limitation being that the
closure must not be for the purpose of circumventing the provisions on termination of
employment in the LC.
- According to Art 283 LC, a partial or total closure or cessation of operations of
establishment or undertaking may either be due to serious business losses or financial
reverses or otherwise. Under the first kind, the employer must sufficiently and convincingly
prove its allegation of substantial losses, while under the second kind, the employer can
lawfully close shop anytime as long as cessation of or withdrawal from business
operations was bona fide in character and not impelled by a motive to defeat or
circumvent the tenurial rights of employees, and as long as he pays his employees their
termination pay in the amount corresponding to their length of service. Just as no law
forces anyone to go into business, no law can compel anybody to continue the same. It
would be stretching the intent and spirit of the law if a court interferes with management's
prerogative to close or cease its business operations just because the business is not
suffering from any loss or because of the desire to provide the workers continued
employment.
- In sum, under Art 283 LC, three requirements are necessary for a valid cessation of
business operations: (a) service of a written notice to the employees and to the DOLE at
least one month before the intended date thereof; (b) the cessation of business must be
bona fide in character; and (c) payment to the employees of termination pay amounting to

Labor Law 1
one month pay or at least one-half month pay for every year of service, whichever is
higher.
- In these consolidated cases, we find that ITC's closure or cessation of business was
done in good faith and for valid reasons.

- Having established that ITC's closure of the plywood


plant was done in good faith and that it was due to
causes beyond its control, the conclusion is inevitable
that said closure is valid. Consequently, Ababon, et al.
could not have been illegally dismissed to be entitled
to full backwages. However, they are entitled to
separation pay equivalent to one month pay or at least
one-half month pay for every year of service,
whichever is higher.
- Although the closure was done in good faith and for valid reasons, we find that ITC did
not comply with the notice requirement. While an employer is under no obligation to
conduct hearings before effecting termination of employment due to authorized cause,
however, the law requires that it must notify the DOLE and its employees at least one
month before the intended date of closure.
Disposition the Decision of CA, which set aside the decision and resolution of NLRC, are
hereby REVERSED. The May 24, 1995 Decision of the NLRC reinstating the decision of
the Laf finding that the closure or cessation of ITC's business valid, is AFFIRMED with the
MODIFICATIONS that ITC is ordered to pay separation pay equivalent to one month pay
or to at least one-half month pay for every year of service, whichever is higher, and
P50,000.00 as nominal damages to each employee.

RE-HIRING EFFECT
ATLANTIC GULF AND PACIFIC CO V NLRC (GAMBOA,
TUASON)
307 SCRA 714
PUNO; May 28, 1999
NATURE
Special civil action for certiorari
FACTS
- Atlantic Gulf (AG&G) is a corporation engaged in general construction work. It reportedly
incurred huge operating losses of P134.8 million and net losses of P35.42 million as the
construction industry experienced a major slump. To save itself, Atlantic implemented a
redundancy program wherein 177 employees were separated from employment. Among
them were private respondents Gamboa, Tuason, and Din, all members of the AG&P
United Rank and File Association (AG&P URFA). They all received the benefits due them
under the LC. More than a year later, they charged the company with unfair labor practice
and illegal dismissal.
- The LA rendered a decision in their favor as it was similar to the cases filed by the 36
employees affected by AG&Ps retrenchment program in 1988. There they found that the
companys redundancy program has no basis for implementation on account of the
substantial profits obtained by the company from 1983 to 1988. Thus, the redundancy
program was but a mere scheme to get rid of the complainants, all active union members.
The receipt by the complainants of their respective separation pay, even if without
protestation or reservation, does not stop them from asserting their right to security of
tenure and self organization. As mere salaried employees, complainants were faced with
no other choice but to accept the money to enable them to meet the demands of everyday

A2010

- 279 -

Disini

living. However, the NLRC set aside the decision of the LA with regard to Gamboa,
Tuason and Din.
- Since the cases a quo stemmed from the same circumstances as the ones previously
filed by the thirty six (36) AG & P employees, they share the same core issue: Was the
redundancy/retrenchment program undertaken by AG & P in 1988 and the resulting
termination of the employment of 177 of its employees, including private respondents
herein, valid?
ISSUE
WON the redundancy/retrenchment program undertaken by AG & P in 1988 and the
resulting termination of the employment of 177 of its employees, including private
respondents herein, was valid
HELD
- As already stated, the Labor Code recognizes retrenchment as one of the authorized
causes for terminating the employer-employee relationship and the decision to retrench or
not to retrench is a management prerogative. In the case at bar, the company losses were
duly established by the financial statements presented by both parties.
In the case at bar, there is no question that respondents income had been
continuously decreasing P205 million in 1984; P175 million in 1985 and P101 million
in 1986. In 1987, however, it declared a loss of P34 million. The declining trend in
respondents income and losses in 1987 confirms its allegation that respondent is
predicting a bleak future considering the slump not only in foreign contracts but with
respect to domestic contracts as well. True enough, respondent incurred further
tremendous losses in 1990 in the amount of P176,181,505.00. In other words, the
losses or abrupt down fall in income which respondent wanted to abate by resorting to
the reduction in the number of employees was imminent and real.
Indeed, the records show that aside from its redundancy program, respondent
company had to resort to other cost-cutting measures in order to stave off impending
losses.
- Petitioners contend that the redundancy program was actually a union-busting scheme
of management, aimed at removing union officers who had declared a strike. This
contention cannot stand in the face of evidence of substantial losses suffered by the
company. Moreover, while it is true that the company rehired or reemployed some of the
dismissed workers, it has been shown that such action was made only as company
projects became available and that this was done in pursuance of the companys policy of
giving preference to its former workers in the hiring of project employees. The rehiring or
reemployment does not negate the imminence to (sic) losses, which prompted private
respondent to retrench.
- As to the waivers and quitclaims signed, not all waivers and quitclaims are invalid as
against public policy. If the agreement was voluntarily entered into and represents a
reasonable settlement, it is binding on the parties and may not later be disowned simply
because of a change of mind. In the case at bar, the documents of waiver were executed
by the affected employees without any force or duress used against them by private
respondent or its representatives. The documents embodied reasonable settlements of
the parties claims. As a matter of fact, the employees received separation pay equivalent
to one month pay for every year of service, which was more than what they were entitled
to receive.
- It is necessary to distinguish redundancy from retrenchment. Both are mentioned in
Article 283 of the Labor Code as just causes for the closing of establishments or reduction
of personnel. Redundancy exists when the services of an employee are in excess of what
is required by an enterprise. Retrenchment, on the other hand, is one of the economic
grounds for dismissing employees and is resorted to primarily to avoid or minimize
business losses. Private respondents redundancy program, while denominated as such,
is more precisely termed retrenchment because it is primarily intended to prevent serious
business losses.
As already stated, the Labor Code recognizes retrenchment as one of the authorized
causes for terminating the employer-employee relationship and the decision to retrench
or not to retrench is a management prerogative. In the case at bar, the company losses
were duly established by the financial statements presented by both parties.
- The petitioners contention that the redundancy program was actually a union-busting
scheme of management, aimed at removing union officers who had declared a strike. This
contention cannot stand in the face of evidence of substantial losses suffered by the
company.
Disposition petition is GRANTED. The public respondents Decision dated 30 September
1996 and Resolution dated December 6, 1996 are SET ASIDE. The Temporary
Restraining Order issued on 4 February 1998 is made permanent.

LIABILITY
CAPITOL MEDICAL CENTER V MERIS
470 SCRA 236

Labor Law 1
CARPIO MORALES; September 16, 2005
FACTS
- Capitol hired Dr. Meris one of its stockholders, as in charge of its Industrial Service Unit
(ISU) at a monthly salary of P10,270.00.
- Until the closure of the ISU, Dr. Meris performed dual functions of providing medical
services to Capitols more than 500 employees and health workers as well as to
employees and workers of companies having retainer contracts with it.
- Dr. Meris received from Capitols president and chairman of the board, Dr. Clemente, a
notice advising him of the managements decision to close or abolish the ISU and the
consequent termination of his services as Chief thereof
- Dr. Meris, doubting the reason behind the managements decision to close the ISU and
believing that the ISU was not in fact abolished as it continued to operate and offer
services to the client companies with Dr. Clemente as its head and the notice of closure
was a mere ploy for his ouster in view of his refusal to retire despite Dr. Clementes
previous prodding for him to do so, sought his reinstatement but it was unheeded.
- Dr. Meris thus filed a complaint against Capitol and Dr. Clemente for illegal dismissal
and reinstatement with claims for backwages, moral and exemplary damages, plus
attorneys fees.
ISSUE
WON Capitol validly terminated Dr. Meriss service as ISU head
HELD
NO
- The right to close the operation of an establishment or undertaking is explicitly
recognized under the Labor Code as one of the authorized causes in terminating
employment of workers, the only limitation being that the closure must not be for the
purpose of circumventing the provisions on termination of employment embodied in the
Labor Code. The phrase closures or cessation of operations of establishment or
undertaking includes a partial or total closure or cessation. And the phrase closures or
cessation x x x not due to serious business losses or financial reverses recognizes the
right of the employer to close or cease his business operations or undertaking even if he is
not suffering from serious business losses or financial reverses, as long as he pays his
employees their termination pay in the amount corresponding to their length of service.
- As long as the companys exercise of the same is in good faith to advance its interest
and not for the purpose of defeating or circumventing the rights of employees under the
law or a valid agreement, such exercise will be upheld.
- The ultimate test of the validity of closure or cessation of establishment or undertaking is
that it must be bona fide in character.[39] And the burden of proving such falls upon the
employer.
- From the letter of Dr. Clemente to Dr. Meris, it is gathered that the abolition of the ISU
was due to the almost extinct demand for direct medical service by the private and semigovernment corporations in providing health care for their employees; and that such
extinct demand was brought about by the existing trend of industrial companies
allocating their health care requirements to Health Maintenance Organizations (HMOs) or
thru a tripartite arrangement with medical insurance carriers and designated hospitals.
- The records of the case, however, fail to impress that there was indeed extinct demand
for the medical services rendered by the ISU.
- The termination of the services of Dr. Meris not having been premised on a just or
authorized cause, he is entitled to either reinstatement or separation pay if reinstatement
is no longer viable, and to backwages.
Disposition The decision of the Court of Appeals dated February 15, 2002 is hereby
AFFIRMED with MODIFICATION. As modified, judgment is hereby rendered ordering
Capitol Medical Center, Inc. to pay Dr. Cesar Meris separation pay at the rate of One (1)
Month salary for every year of his employment, with a fraction of at least Six (6) Months
being considered as One (1) Year, full backwages from the time of his dismissal from April
30, 1992 until the expiration of his term as Chief of the ISU or his mandatory retirement,
whichever comes first; other benefits due him or their money equivalent; and attorneys
fees.

D. CLOSING OF BUSINESS
RIGHT
MAC ADAMS METAL V MAC ADAM METAL
ENGINEERING
414 SCRA 411
CORONA; October 24, 2003

A2010

- 280 -

Disini

NATURE
Petition for review on certiorari of the decision of the CA
FACTS
- Mac Adams Metal Engineering (MAME) and GBS Engineering Services (GBS) were both
owned by spouses Geronimo and Lydia Sison. From the facts of the case, the spouses
decided to retire from business in 1988 when the wife became sickly. In July 1992, in a
genral meeting of the workers, she announced her plan to close shop effective early 1993.
The announcement was done early so that the employees can look for alternative work.
- After the announcement, some employees decided to form a union, MAMEWU,
ostensibly for the purpose of making representations with the management to reconsider
its decision to close shop or to see to it that all benefits due the employees would be paid.
In the course of the negotiation, the Union demanded among others that the separation
pay to be given should be 45 days for every year of service. This was not accepted by
Mrs. Sison. It was found out later that even before Mrs sison could announce her plans to
shut down, member of the Union were already sabotaging the operations through work
slowdowns, picketing, refusal to report to work, and ultimately strikes.
- It was alleged by the workers that MAME and GBS were being closed to bust the Union
and that these two businesses will continue under the business names, MBS Machine and
Industrial supply (MBS) and MVS Heavy Equipment Rental and Builders (MVS). These
two companies were subsequently impleaded as run-away shops MAME and GBS. MBS
is partly owned by Geronimo Sison while MVS is owned by Dominic Sison, son of the
Spouses Sison. It should be noted that the MBS was engaged in manufacturing carton
boxes and other allied products and MVS was engaged in leasing out heavy equipment.
- the Union filed a complaint for unfair labor practice against the two companies. The
Labor Arbiter found for the respondents which was affirmed by the NLRC. The CA also
affirmed the ruling of the Labor Arbiter and the NLRC that there was a legitimate and bona
fide closure and cessation of the business by MAME and GBS.

ISSUE
WON the closure of MAME and GBS was done in good faith and for legitimate business
reasons
HELD
YES
- The owner of a business can lawfully close his shop. No law can force anyone to go into
business, no law can compel anyone to continue the same. It would be stretching the
intent and spirit of the law if the government were to unjustly interfere with the
managements prerogative to close or cease its business operations, just because said
business is not suffering from losses or simply to provide the workers the continued
employment.
Reasoning
- The governing article under the labor code with regard closure is Article 283. Under this
article, it seems clear that business closures will not be interfered with provided however
that what is considered as due to workers be complied with. In addition to the separation
pay required to be paid, the establishment would also have to serve the workers and the
the DOLE notice one month before the intended date of closure.
- As part of the proof submitted by the respondents, they presented various notices not
only to the employees and DOLE but also to the SSS, BIR, DTI, And the Municipal
licensing Division of Antipolo.
Disposition Petition dismissed.

CATATISTA V NLRC (VICTORIAS MILLING CO INC)


247 SCRA 46
ROMERO; August 3, 1995
NATURE
petition for review of the NLRC decision Cebu City dated March 8, 1991 which reversed
Labor Arbiter decision dated October 18, 1988 and the resolution dated July 23, 1991
which denied petitioners' motion for reconsideration.
FACTS
- Petitioners CATATISTA, JAIME MONSERATE, FRANCISCO ELISAN, FERNANDO DE
LA PENA, DIEGO TUPAS, ROSENDO MONSERATE, ERNESTO SIBUNAL, DIOSCORO
HINO-O, AURELIO DESCATAMIENTO, LODOVICO DELA PENA, and DIONISIO
BALLADOS, were regular plantation workers in Hacienda Binanlutan, one of the six
haciendas operated and managed by private respondent Victorias Milling Company, Inc.
with an area of 14.8 hectares planted to sugarcane.
- June 1984 Victorias decided to permanently stop and close its sugarcane operations in
Hacienda Binanlutan "due to low sugar prices which affected the viability and profitability
of said hacienda" and convert it instead into an ipil-ipil plantation. In view of such decision,

Labor Law 1
management subsequently held a conference with all thirteen field workers to explain to
them the reason for this move, as well as the computation of their termination pay
- July 10, 1984 in a letter, each of the thirteen petitioners was formally informed of
Victorias decision to close and stop sugarcane operations and the reason for such closure
and that "considering that they have been hired specifically for Hacienda Binanlutan's
operations and considering further that there is no plan to revive sugarcane operations at
said company farm," their services would be terminated effective August 1, 1984.
- Petitioners received their termination pay or retirement pay under the pension plan,
whichever was higher.
- March 18, 1987 Catatista et al filed a complaint against Victorias with the arbitration
branch of NLRC for illegal dismissal, damages and attorney's fees.
- October 18, 1988 - Labor Arbiter Rodolfo Lagoc ordered Victorias to reinstate Catatista
et al without loss of seniority and with backwages for three (3) years without any
qualifications
- NRLC > reversed the decision of the Labor Arbiter
- Catatista et als MFR to NLRC decision was denied for lack of merit.
- Petitioners claim that NLRC abused its discretion in rendering its questioned decision
when it misappreciated or failed to consider the following:
1. Victorias net profit from its company-wide operations for 1983 and 1984 at a staggering
amount of P 11,508,406.00 and P48,229,400.00, respectively
2. Victorias own admission of profits in the raw and refined sugar production as stated in
page 4, of its Position Paper dated November 2, 1988;
3. Victorias payment of wages and benefits for company hacienda workers more than
what is prescribed by law. It even granted wage increases at P1.20/day in 1983 to 1985
CBA which private respondent alleged to be the years it incurred losses in the hacienda
operation;
4. There is a stipulation in the CBA wherein the company shall comply with statutory salary
increases in the event it is decreed by law-,
5. Victorias had not submitted a separate financial statement or balance sheet for its
company haciendas except only an income statement. Obviously, such income statement
cannot show such items as retained earnings, fixed assets, and real networth which could
be found only in financial statements or balance sheets that is determinative of losses or
profits of any business;
6. Only Hacienda Binanlutan, where Catatista et al were employed, has closed its
operation exempting other company haciendas such as Florencia, Candelaria, Pacita,
Begonia and Bacayan;
7. Catatista et al have already worked for two months in the ipil-ipil plantation before they
were terminated from the service;
8. Catatista et al worked also in other company owned haciendas after milling season in
Hacienda Binanlutan and in case the need arises and return again to Hacienda Binanlutan
upon ressumption of work;
9. Victorias change only on crops from sugarcane to ipilipil trees which is definitely the
same agricultural undertaking with the same need for workers for its operation;
10. After Catatista et als termination, Victorias hired a laboronly contractor to supply
workers in the ipil-ipil plantation to the exclusion of Catatista et al;
11. The uncontroverted fact that Hacienda Binanlutan was not losing in its operation in
1984 but was converted to ipil-ipil plantation only for reason of 'land use' which is the idea
of Mr. Virgilio Flores who was then Vicgtorias Division Head of Research and
Development
ISSUES
WON Catatusta et al were illegally terminated from work resulting from the closure of
Hacienda Binanlutan, one of the company farms owned by Victorias
(Corollary to the above issue is the validity of the closure of a business based on the
following)
A. WON Victorias is in good faith when it closed its business
B. WON such closure is due to causes beyond its control as enunciated in the case of
Armed Forces of the Philippines Mutual Benefit Association v. Armed Forces Mutual
Benefit Association, Inc. Employees Union
HELD
NO
- Article 283 of the Labor Code provides that:
"The employer may also terminate the employment of any employee due to the
installation of labor-saving devices, redundancy, retrenchment to prevent losses or the
closing or cessation of operation of the establishment or undertaking unless the closing
is for the purpose of circumventing the provisions of this title, by serving a written notice
on the workers and the Department of Labor and Employment at least one (1) month
before the intended date thereof. x x x In case of retrenchment to prevent losses and in
cases of closures or cessation of operations of establishment or undertaking not due to
serious business losses or financial reverses, the separation pay shall be equivalent to
one (1) month pay or at least one-half (1/2) month pay for every year of service,
whichever is higher. A fraction of at least six (6) months shall be considered one (1)
whole year."

A2010

- 281 -

Disini

- The termination of employment of the employees of Hacienda Binanlutan brought about


by the closure is to be considered as retrenchment as Hacienda Binanlutan is only one of
the six haciendas of private respondent. Clearly, private respondent's purpose in
converting said hacienda into an Ipil-ipil plantation and terminating the service of
petitioners is to cut down on losses which it had adequately shown to have suffered
through an income statement for the fiscal year which ended August 31, 1984.
- This Court has held that "the requisites of a valid retrenchment are: (a) the losses
expected should be substantial and not merely de minimis in extent; (b) the substantial
losses apprehended must be reasonably imminent; (c) the retrenchment must be
reasonably necessary and likely to effectively prevent the expected losses; and (d) the
alleged losses, if already incurred, and the expected imminent losses sought to be
forestalled, must be proved by sufficient and convincing evidence."
- We see no grave abuse of discretion on the part of NLRC when it found that "company
haciendas including Hacienda Binanlutan incurred huge losses from years 1982 to 1983 in
the amount of P2,842,778.03."9 Private respondent showed that Hacienda Binanlutan
itself suffered a net loss of P22,624.88.10 It is significant to note that petitioners failed to
dispute these submissions of private respondent which more than satisfy the first and
fourth requirements for a valid retrenchment. The losses incurred are clearly substantial
and sufficiently proven by means of an income statement of Hacienda Binanlutan and the
financial statement of the company haciendas. Said losses are not only imminent but had,
in fact, already been incurred by private respondent since 1982. This was even more
alarming in 1984 considering the worldwide economic situation, as well as the low sugar
prices during that year, events which were obviously beyond the control of private
respondent.
- Having determined that private respondent suffered losses and had to resort to
retrenchment of its employees in Hacienda Binanlutan to prevent further losses, this Court
holds that private respondent was within its rights in closing Hacienda Binanlutan and in
terminating the service of petitioners.
- In any case, Article 283 of the Labor Code is clear that an employer may close or cease
his business operations or undertaking even if he is not suffering from serious business
losses or financial reverses, as long as he pays his employees their termination pay in the
amount corresponding to their length of service. It would, indeed, be stretching the intent
and spirit of the law, if we were to unjustly interfere in management's prerogative to close
or cease its business operations just because said business operation or undertaking is
not suffering from any loss. This Court, in the case of
- Maya Farms Employees Organization, et al. v. NLRC
"The rule is well-settled that labor laws discourage interference with an employer's
judgment in the conduct of his business. Even as the law is solicitous of the welfare of
employees, it must also protect the right of an employer to exercise what are clearly
management prerogatives. As long as the company's exercise of the same is in good
faith to advance its interest and not for the purpose of defeating or circumventing the
rights of employees under the laws or valid agreements, such exercise will be upheld."
- Dangan v. NLRC > management's prerogative to close or abolish a department or
section of the employer's establishment for economic reasons. We reasoned out that since
the greater right to close the entire establishment and cease operations due to adverse
economic conditions is granted an employer, the closure of a part thereof to minimize
expenses and reduce capitalization should similarly be recognized.
- Special Events & Central Shipping Office Workers Union v. San Miguel
Corporation > the determination of the usefulness of a section, being a company
prerogative, the closure may not be questioned, specially in this case where it is impelled
by economic reasons due to the continuous losses sustained in its operation, coupled with
the lack of demand for the services of such section.
A. YES
- We are satisfied with private respondent's good faith when it closed Hacienda
Binanlutan.
(1) the necessary notice to the affected employees and to the then Ministry of Labor and
Employment have been sufficiently complied with.
(2) the affected employees were paid their separation pay which was computed at one
month pay for every year of service or based on private respondent's pension plan,
whichever was higher.
(3) there was then no brewing labor dispute which could have served as a motive for
closing Hacienda Binanlutan, contrary to petitioners' insinuation that their demand for meal
allowances prompted private respondent to close Hacienda Binanlutan, a reason which
we deem to be rather trivial. In fact, petitioners admitted that they received their meal
allowances
(4) it took Catatista et al almost three years to file their complaint which does not seem to
be the natural reaction of persons aggrieved.
B. YES
- A283 LC provides that the employer may terminate the employment of his employees to
prevent losses, For an employer to validly terminate the service of his employees under
this ground, he has to comply with two requirements, namely:
(a) serving a written notice on the workers and the DOLE at least one month before the
taking effect of the closure, and

Labor Law 1
(b) payment of separation pay equivalent to one month pay or at least one-half (1/2)
month pay for every year of service, whichever is higher, with a fraction of at least six
months to be considered one whole year.
Disposition petition is hereby DISMISSED. The decision of public respondent NLRC
(Fourth Division) dated March 8, 1991 is AFFIRMED.

ALABANG COUNTRY CLUB INC V NLRC


[PAGE 266]
CAPITOL MEDICAL CENTER INC V MERIS
[PAGE 279]
INDUSTRIAL TIMBER CORP V ABABON
[PAGE 277]

PARTIAL CLOSURE
JAT GENERAL SERVICE V NLRC
[PAGE 266]

CHENIVER DECO PRINT TECHNICS CORP V NLRC


(CFW-MAGKAKAISANG LAKAS NG MGA
MANGGAGAWA SA CHENIVER)
325 SCRA 758
QUISUMBING; February 17, 2000
NATURE
Special civil action for certiorari
FACTS
- Cheniver is a corporation operating its printing business in Makati. The respondents are
members of the labor union and former employees of Cheniver.
- June 5, 1992 Cheniver informed its employees that it will transfer its operations to
Batangas. Reasons for the transfer are expiration of lease contract on the premises of the
Makati palnt, and local authorities action to force out Chenivers operations from Makati
because of alleged hazards to residents nearby.
- Cheniver gave its employees until the end of June to inform management if they wanted
with Cheniver in its transfer, otherwise it would hire replacements. Aug1 was the
scheduled start of operations in the new plant in Batangas.
- Aug 4, 1992 Cheniver wrote its employees to report to the new location within 7days,
otherwise they will be deemed to have lost interest in the job and would be replaced.
However, no one reported for work in batangas, even after extension of period of time to
report to work.
- Respondents filed a complaint for unfair labor practice and illegal dismissal, and
demanded separation pay (among others).
- LA ruled that the transfer of operations was valid and absolved cheniver of charges for
unfair labor practice and illegal dismissal. It however ordered payment of separation pay.
NLRC affirmed.
cheniver contends that the transfer of its business is neither closure nor retrenchment,
thus separation pay should not be awarded. Also, employees were not terminated but they
resigned because they find the new site to far from their residences
ISSUE
WON employees are entitled to separation pay considering that the transfer of the plant
was valid
HELD
1. YES
Ratio Art. 283 of the Labor Code provides (in part):
ART. 283. Closure of establishment and reduction of personnel. - The employer may
terminate the employment of any employee due to the installation of labor saving
devices, redundancy, retrenchment to prevent losses or the closing or cessation of

A2010

- 282 -

Disini

operation of the establishment or undertaking unless the closing is for the purpose of
circumventing the provisions of this Title
xxx
- In case of retrenchment to prevent losses and in cases of closures or cessation of
operations of establishment or undertaking not due to serious business losses or financial
reverses, the separation pay shall be equivalent to one (1) month pay or at least one-half
(1/2) month pay for every year of service, whichever is higher. xxx
Reasoning
- there appears no complete dissolution of Chenivers business undertaking but the
relocation of its plant to Batangas, in our view, amounts to cessation of petitioner's
business operations in Makati. It must be stressed that the phrase closure or cessation of
operation of an establishment or undertaking not due to serious business losses or
reverses under Art. 283 includes both complete cessation of all business operations and
the cessation of only part of a company's business
- There is no doubt that petitioner has legitimate reason to relocate its plant because of the
expiration of the lease contract on the premises it occupied. That is its prerogative. But
even though the transfer was due to a reason beyond its control, Cheniver has to accord
its employees some relief in the form of severance pay.
- Since the closure of the plant is not on account of serious business losses, Cheniver
shall give respondents separation pay equivalent to at least 1 month or month pay for
every year of service
- that the employees resigned is not convincing. The transfer of Cheniver to another place
hardly accessible to its workers resulted in the latter's untimely separation from the service
not to their own liking, hence, not construable as resignation
Disposition petition denied. NLRC resolutions AFFIRMED.

REQUISITE
ME-SHURN CORP V ME-SHURN WORKERS UNION
448 SCRA 41
PANGANIBAN; January 11, 2005
FACTS
- Regular rank and file employees of Me-Shurn Corporation organized Me-Shurn Workers
Union-FSM, an affiliate of the February Six Movement (FSM), their union had a pending
application for registration with the Bureau of Labor Relations (BLR).
- Ten days after the organization petitioner corporation started placing on forced leave all
the rank and file employees who were members of the unions bargaining unit.
- The union filed a Petition for Certification Election with the Med-Arbitration Unit of the
Department of Labor and Employment (DOLE), Reg Office No. 3.
- Instead of filing an answer to the Petition, the corporation filed a comment stating that it
would temporarily lay off employees and cease operations, on account of its alleged
inability to meet the export quota required by the Board of Investment.
- Respondent union filed a Notice of Strike against petitioner corporation on the ground of
unfair labor practice (illegal lockout and union busting).
- The corporation imposed a precondition for the resumption of operation and the rehiring
of laid off workers which was for remaining union officers to sign an Agreement containing
a guarantee that upon their return to work, no union or labor organization would be
organized and instead, the union officers were to serve as mediators between labor and
management. After the signing of the Agreement, the operations of the corporation
resumed in September 1998.
ISSUE
WON the dismissal of the employees of petitioner Meshurn Corporation is for an
authorized cause (WON there was a valid reason for the closure of the business or
temporary cessation of operation)
HELD
NO
- To justify the closure of a business and the termination of the services of the concerned
employees, the law requires the employer to prove that it suffered substantial actual
losses. The cessation of a companys operations shortly after the organization of a labor
union, as well as the resumption of business barely a month after, gives credence to the
employees claim that the closure was meant to discourage union membership and to
interfere in union activities. These acts constitute unfair labor practices.
- The reason invoked by petitioners to justify the cessation of corporate operations was
alleged business losses. Yet, other than generally referring to the financial crisis in 1998
and to their supposed difficulty in obtaining an export quota, interestingly, they never

Labor Law 1
presented any report on the financial operations of the corporation during the period
before its shutdown. Neither did they submit any credible evidence to substantiate their
allegation of business losses.
- However, as previously stated, in all the proceedings before the two quasi-judicial bodies
and even before the CA, no evidence was submitted to show the corporations alleged
business losses. It is only now that petitioners have belatedly submitted the corporations
income tax returns from 1996 to 1999 as proof of alleged continued losses during those
years.
- The claim of petitioners that they were compelled to close down the company to prevent
further losses is belied by their resumption of operations barely a month after the
corporation supposedly folded up.
- Moreover, petitioners attribute their loss mainly to their failure to obtain an export quota
from the Garments and Textile Export Board (GTEB). Yet, as pointed out by respondents,
the corporation resumed its business without first obtaining an export quota from the
GTEB. Besides, these export quotas pertain only to business with companies in the
United States and do not preclude the corporation from exporting its products to other
countries. In other words, the business that petitioner corporation engaged in did not
depend entirely on exports to the United States.
- If it were true that these export quotas constituted the determining and immediate cause
of the closure of the corporation, then why did it reopen for business barely a month after
the alleged cessation of its operations?
- Proper written notices of the closure were not sent to the DOLE and the employees at
least one month before the effectivity date of the termination, as required under the Labor
Code. Notice to the DOLE is mandatory to enable the proper authorities to ascertain
whether the closure and/or dismissals were being done in good faith and not just as a
pretext for evading compliance with the employers just obligations to the affected
employees. This requirement is intended to protect the workers right to security of tenure.
The absence of such requirement taints the dismissal.
- The determination to cease operations is a management prerogative that the State does
not usually interfere in. Indeed, no business can be required to continue operating at a
loss, simply to maintain the workers in employment. That would be a taking of property
without due process of law. But where it is manifest that the closure is motivated not by a
desire to avoid further losses, but to discourage the workers from organizing themselves
into a union for more effective negotiations with management, the State is bound to
intervene.

TEMPORARY CESSATION OF OPERATION


BASIS
SAN PEDRO HOSPITAL OF DIGOS INC V SEC OF
LABOR
263 SCRA 98
PANGANIBAN; October 11, 1996
NATURE
Petition for certiorari of a decision of NLRC.
FACTS
- The HOSPITALs collective bargaining agreement (CBA) 46 with the UNION, which is the
exclusive bargaining agent 47 of the hospital's rank-and-file workers, ended in Dec 1990.
The following year, both parties negotiated for the renewal of their CBA, but they failed to
reach an agreement. Eventually, the UNION went on strike, until the DOLE Secretary
assumed jurisdiction over the labor dispute. 48 The Secretary issued an order directing the
striking workers to return to work; the HOSPITAL to accept all returning workers under the
same terms prior to the work stoppage; and both parties to desist from committing any act
that may aggravate the situation.
- Before the HOSPITAL could receive such order, however, it already notified the DOLE by
letter, that it would temporarily suspend operations for 6 months effective June 15, 1991,
or up to Dec 15, 1991. The HOSPITAL thus refused the return of its striking workers on
account of such suspension of operations. Several conciliation conferences were held,
where the HOSPITAL stated that it would submit the necessary documents showing its
serious financial condition "should the need be in earnest". No such documents came, and
after the Secretarys ocular inspection, he ruled, among others, that the subject
suspension was not for a valid cause but was actually for defeating the workers' right to

A2010

- 283 -

Disini

self-organization. He also enjoined the HOSPITAL to enter into a new CBA with the
UNION.
ISSUES
1. WON the Secretary is correct in concluding that the subject temporary suspension was
for circumventing the return-to-work order he issued
2. WON the Secretary gravely abused his discretion when he ordered the HOSPITAL to
enter into a new CBA despite his knowledge that it had actually ceased operations. Is it
correct
HELD
1. YES
Ratio Temporary suspension of operations is a valid exercise of management prerogative
provided it is not carried out in order to circumvent the provisions of the Labor Code or to
defeat the rights of the employees under the Code. The determination to suspend
operations is a management prerogative that the State usually does not interfere with, as
no business can be required to continue operating at a loss simply to maintain the workers
in employment. To require such continued operation would be tantamount to a taking of
property without due process, which the employer has a right to resist. But where it is
shown that the closure is motivated not by a desire to prevent further losses, but to
discourage the workers from organizing themselves into a union for more effective
negotiation with management, the State is bound to intervene.
- The burden of proving that such a temporary suspension is bona fide falls upon the
employer. In this instance, the HOSPITAL had to establish the fact of its precarious
financial health; that its cessation of operation was really necessitated by its financial
condition; and that said condition would probably be improved by such suspension. The
HOSPITAL could have partly met the foregoing requirements by submitting its financial
statements or records as proof of its financial crisis. Its unexplained and continued failure
to submit its financial statements raises grave doubts as to the truth of the claimed
financial crisis and the real purpose of the subject suspension. The HOSPITAL was
recklessly pushing its luck when it believed that the Secretary could be convinced without
first obtaining and examining its financial statements and the notes thereto. The fact that
the conciliator never asked for them is no excuse for not presenting them, as such was the
HOSPITALs duty. Neither is it acceptable for petitioner to allege that the latest financial
statement (for 1991) were still being prepared, since the financial statement for the prior
years 1989 and 1990 would have sufficed. Petitioner, having wretchedly failed to justify by
even the most rudimentary proof its temporary suspension, must bear the consequences
thereof. Thus, the Secretary did not act with grave abuse of discretion in finding the
temporary suspension illegal.
2. NO
- Art. 286 of the Code provides that "bona fide suspension for a period not exceeding 6
months . . . shall not terminate employment." Sec.12, Rule 1, Book VI of the Omnibus
Rules provides that the employer-employee relationship shall be deemed suspended in
case of the suspension referred to above, it being implicitly assumed that once operations
are resumed, the employment relationship is revived. If a valid suspension of operation
merely suspends the relationship, with more reason will an illegal suspension, as in
this case, not affect the employment relationship.
- There is, thus, no basis for the HOSPITAL to claim that a new CBA should not be entered
into or that collective bargaining should not be conducted during the temporary
suspension. The HOSPITAL expressly represented that the suspension was to be for 6
months only. Absent any other information, the natural presumption will be that the
HOSPITAL would resume operations after 6 months, and therefore, it follows that a new
CBA will be needed to govern the employment relations of the parties, the old one having
already expired. Clearly then, under the circumstances, the Secretary cannot be to have
gravely abused his discretion for ordering the parties to enter into a new CBA.
Disposition
DOLE Secretarys decision AFFIRMED. But since the HOSPITAL
permanently closed after the 6-month suspension, which closure it has duly justified, SC
sustained the permanent closure and did not anymore compel the HOSPITAL to negotiate
with the UNION for a new CBA. SC, in addition, penalized the HOSPITAL for the illegal
temporary suspension by awarding backwages to the UNION members from the time they
were refused admittance by the HOSPITAL, until the expiration of the temporary
suspension.

JAT GENERAL SERVICES V NLRC


[PAGE 266]
MAYON HOTEL & RESTAURANT V ADONA
[PAGE 269]

46

The CBA is the end result of the collective bargaining process [see ART.250, Labor Code]. Both the employer and the
exclusive bargaining agent [see footnote 2] have the duty to bargain collectively, which duty is explained under
ARTS.252-253.
47
An exclusive bargaining agent is defined under ART.255.
48
See ART.263(g).

ME-SHURN CORP V ME-SHURN WORKERS UNION


[PAGE 282]

Labor Law 1
EFFECT ON EMPLOYER-EMPLOYEE RELATIONSHIP
SAN PEDRO HOSPITAL OF DIGOS V SEC OF LABOR
[PAGE 282]

E. INSTALLATION
ABAPO V CA (SAN MIGUEL CORP)
439 SCRA 594
SANDOVAL-GUTIERREZ; September 30, 2004
NATURE
Petition for review of the decision of CA
FACTS
- The petitioners are former employees of private respondent San Miguel Corporation
(SMC) who were hired on different dates and assigned to SMC's Mandaue Brewery Plant.
- Sometime in 1991, SMC conducted a viability study of its business operations and
adopted a modernization program. SMC then brought into the Mandaue plant high-speed
machines to be used in the manufacture of its beer. These machines were installed in
bottling lines 6 and 7. The main line operation known as lines 1, 2, 3, 4, and 5 ceased to
operate. As a consequence, several functions of the employees were declared redundant.
- On February 13, 1992, SMC sent to Office of Region VII, Department of Labor and
Employment (DOLE), two letters with the information that it was terminating the services of
the employees named in the attached list. On February 7, 1992 and September 28, 1992,
SMC sent notices to the affected employees informing them that they would be given a
separation pay amounting to 175% of their respective monthly salaries for every year of
service, a 3-year free hospitalization coverage, and free training and consultation for
livelihood programs and small-scale business enterprises, as well as assistance in local
and overseas job placements.
- Accordingly, the employees concerned received the said amounts and executed their
respective quitclaims before the Officers of Region VII DOLE.
- Two years later, those employees filed with the Labor Arbiter two separate complaints,
alleging that SMC did not institute a modernization program, hence, the company
committed an illegal mass lay-off. They prayed that their separation from the service be
declared illegal and that they be paid their backwages, separation pay in lieu of
reinstatement, and other benefits. Labor Arbiter rendered a Decision dismissing the
complaints. On appeal, NLRC affirmed the Labor Arbiters decision.
- Petitioners then filed with CA a special civil action for certiorari which was dismissed.
Petitioners' motion for reconsideration was denied. The grounds relied upon by the CA in
dismissing the petition are: petitioners failed to attach to their petition a certified true copy
of the assailed Resolution of the NLRC in NLRC Case No. V-0099-97; and they filed the
petition 7 days late.
ISSUE
WON CA gravely abused its discretion in dismissing the petition
HELD
- Section 3, Rule 46 of the 1997 Rules of Civil Procedure, as amended, is clear that failure
to comply with the requirement that the petition shall be accompanied by a certified true
copy of the resolution being challenged is a sufficient ground for the dismissal of the
petition.
- With respect to the delay in the filing of the petition, petitioners accepted the finding of
the Court of Appeals that there was indeed a delay of 7 days.
- Granting that the petition is in order, it is still dismissible. In a similar case (involving the
same issue the validity of the termination of SMC employees at the Mandaue Brewery),
it was held that the installation of labor-saving devices by SMC at the Mandaue plant was
a proper ground for terminating employment. The quitclaims and releases, signed by the
employees concerned as reasonable settlements, are binding upon the parties.
Disposition Petition denied. CA decision affirmed in toto

F. FLOATING STATUS
DISEASE
SY V CA
[PAGE 276]

A2010

- 284 -

Disini

G. SPECIAL CASE OF BUSINESS TRANSFERS


NATURE OF LABOR CONTRACT
SUNDOWNER DEV CORP V DRILON (NUWHRAIN,
PENANO)
180 SCRA 14
GANCAYCO; December 6 1989
FACTS
- Hotel Mabuhay, Inc. (Mabuhay) leased the premises belonging to Santiago Syjuco, Inc.
(Syjuco) located in Ermita, Manila. Due to non-payment of rentals, Syjuco filed an
ejectment case against Mabuhay. Mabuhay offered to amicably settle the case by
surrendering the premises to Syjuco and to sell its assets and personal property to any
interested party.
- Syjuco offered the said premises for lease to petitioner Sundowner Dev Corp.
- Mabuhay then offered to sell its assets and personal properties in the premises to
petitioner to which petitioner agreed.
- Syjuco turned over the possession of the leased premises to petitioner who took
possession and occupied the same.
- The National Union of Workers in Hotel, Restaurant and Allied Services (NUWHRAIN)
picketed the leased premises, barricaded the entrance to the leased premises and denied
petitioner's officers, employees and guests free access to and egress from said premises.
- Secretary of Labor ordered all striking employees to return to work and for respondent
Mabuhay to accept all returning employees pending final determination of the issue of the
absorption of the former employees of Mabuhay.
- Mabuhay: it had sold all its assets and personal properties to petitioner and that there
was no sale or transfer of its shares whatsoever and that Mabuhay completely ceased
operation and surrendered the premises to petitioner so that there exists a legal and
physical impossibility on its part to comply with the return to work order specifically on
absorption.
- NUWHRAIN: there was connivance between Mabuhay and petitioner in selling the
assets and closing the hotel to escape its obligations to the employees of Mabuhay and so
it prays that petitioner accept the workforce of Mabuhay and pay backwages from April 15,
1986 to April 28, 1987, the day Mabuhay stopped operation.
- Drilon issued an order requiring petitioner to absorb the members of the union and to
pay backwages from the time it started operations up to the date of the order. \
- Petitioner filed motion for reconsideration of the aforesaid order alleging that the theory
of implied acceptance and assumption of statutory wrong does not apply in the instant
case; that the prevailing doctrine that there is no law requiring bona fide purchasers of the
assets of an on-going concern to absorb in its employ the employees of the latter should
be applied in this case; that the order for absorption of the employees of Mabuhay as
well as the payment of their backwages is contrary to law.
ISSUES
1. WON the purchaser of the assets of an employer corporation can be considered a
successor employer of the latter's employees (WON Sundowner can be compelled to
absorb the employees of respondent Mabuhay)
2. WON there was bad faith
HELD
1. NO
Ratio As a general rule, there is no law requiring a bona fide purchaser of assets of an
on-going concern to absorb in its employ the employees of the latter.
- However, although the purchaser of the assets or enterprise is not legally bound to
absorb in its employ the employers of the seller of such assets or enterprise, the parties
are liable to the employees if the transaction between the parties is colored or clothed with
bad faith. (see no. 2)
- Nature of labor contract: The rule is that unless expressly assumed, labor contracts
such as employment contracts and collective bargaining agreements are not enforceable
against a transferee of an enterprise, labor contracts being in personam, thus binding only
between the parties. A labor contract merely creates an action in personam and does not
create any real right which should be respected by third parties. This conclusion draws its
force from the right of an employer to select his employees and to decide when to engage
them as protected under our Constitution, and the same can only be restricted by law
through the exercise of the police power.
2. NO
- contrary to the claim of the public respondent that the transaction between petitioner and
Mabuhay was attended with bad faith, the court finds no cogent basis for such contention.
Thus, the absorption of the employees of Mabuhay may not be imposed on petitioner.

Labor Law 1
- It is undisputed that when Mabuhay surrendered the leased premises to Syjuco and
asked Syjuco to offer same to other lessees, it was Syjuco who found petitioner and
persuaded petitioner to lease said premises. Mabuhay had nothing to do with the
negotiation and consummation of the lease contract between petitioner and Syjuco.

H. PROCEDURAL DUE PROCESS NATURE


AND REQUIREMENTS
1. REQUIREMENTS
IN GENERAL
AGABON V NLRC
[PAGE 35]

A2010

- 285 -

Disini

hand, the requirement of hearing affords the employee the opportunity to answer his
employer's charges against him and accordingly to defend himself before dismissal is
effected.
- The initial act of the bank in convening the disciplinary council would have been a proper
step had Salaw been allowed the assistance of counsel. That would have complied with
the demands of due process had complainant been given the opportunity to present his
own defense and confront the witnesses, if any, and examine the evidence against him.
- But the records indicate that there was a denial of Salaws constitutional right when his
subsequent request to refute the allegations against him was granted and a hearing was
set "without counsel or representative."
- Section 5 (of Rule 14, Book 5 of the IRR of the Labor Code) of the said Rule requires
that "the employer shall afford the worker ample opportunity to be heard and to defend
himself with the assistance of his representative, if he so desires."
- The evidence which Associated Bank anchors on is the sworn statement of Salaw which
was executed without the assistance of counsel.
- Even if administrative and quasi-judicial bodies are not strictly governed by the
procedural rules in adjudicating cases, the right to counsel is a requirement of substantial
due process.
Disposition Judgment set aside and the judgment of the labor arbiter is reinstated.

ESSENCE OF DUE PROCESS


CENTRAL PANGASINAN ELEC COOP V MACARAEG
[PAGE 195]
VALIAO V CA
[PAGE 11]

RIGHT TO COUNSEL
SALAW V NLRC (ASSOCIATED BANK, TENGCO,
TUAZON)
202 SCRA 7
SARMIENTO; September 27, 1991
NATURE
Petition for certiorari to review the decision of the NLRC
FACTS
- Espero Salaw was employed by the Associated Bank in 1967 as a credit-investigatorappraiser. His job involved the following activities: inspecting, investigating, appraising,
and identifying the company's foreclosed assets; giving valuation to its real properties, and
verifying the genuineness and encumbrances of the titles of properties mortgaged to the
bank.
- November 27, 1984 The investigation unit of the Philippine Constabulary were able to
make Salaw execute a sworn statement, stating that he disposed acquired assets of the
bank in the form of 20 sewing machines electronic generators for P60,000 with the help of
another bank employee Reynaldo Madrigal. The said sworn statement was executed
without assistance of counsel.
- December 1984 Salaw was informed by the bank manager Tuazon that the banks
disciplinary would be convened and that he attend the proceedings without counsel.
- April 1, 1985 Salaw was terminated effective March 27, 1985 for alleged serious
misconduct or willful disobedience and fraud or willful breach of the trust reposed upon
him by the bank.
- Salaw then filed a complaint for illegal termination against the Associated Bank, Tengco
(as chairman of the board) and Tuazon (as bank manager).
- The Labor Arbiter ruled that Salaw was illegally dismissed and ordered his reinstatement
or payment of backwages and benefits.
- The respondents appealed and the NLRC reversed the decision of the Labor Arbiter.
ISSUE
WON Salaw was illegally terminated
HELD
YES
Ratio The dismissal must not only be for a valid or authorized cause as provided by law
but the rudimentary requirements of due process-notice and hearing-must also be
observed before an employee may be dismissed.
Reasoning
- Petitioner Salaw was terminated without due process of law.
- The requirement of notice is intended to inform the employee concerned of the
employer's intent to dismiss him and the reason for the proposed dismissal; on the other

NOTICE
COLEGIO DE SAN JUAN DE LETRAN - CALAMBA V.
VILLAS
399 SCRA 550
CORONA; March 26, 2003
NATURE
Petition for review on certiorari
FACTS
- Respondent Belen Villas was employed by the petitioner School as high school teacher.
She applied for a study leave for six months. The principal of the high school department
told Villas that her request for study leave was granted for one (1) school year subject to
certain conditions.
- Respondent alleged that she intended to utilize the first semester of her study leave to
finish her masteral degree at the Philippine Womens University (PWU). Unfortunately, it
did not push through so she took up an Old Testament course in a school of religion and at
the same time utilized her free hours selling insurance and cookware to augment her
familys income. However, during the second semester of her study leave, she studied and
passed 12 units of education subjects at the Golden Gate Colleges in Batangas City.
- In response to the letters sent her by petitioner to justify her study leave, she submitted a
certification from Golden Gate Colleges and a letter explaining why she took up an Old
Testament course instead of enrolling in her masteral class during the first semester.
- The President and Rector of the School, Fr. Ramonclaro G. Mendez, O. P., wrote her,
stating that her failure to enroll during the first semester was a violation of the conditions of
the study leave and that the reasons she advanced for failure to enroll during the first
semester were not acceptable.
- Respondent then filed a case for illegal dismissal and the case was assigned to the
Voluntary Arbitrator who found that respondent was illegally dismissed. Hence this petition.
ISSUE
WON respondents alleged violation of the conditions of the study grant constituted
serious misconduct which justified her termination from petitioner School
HELD
1. NO
Reasoning
- Under the Labor Code, there are twin requirements to justify a valid dismissal from
employment: (a) the dismissal must be for any of the causes provided in Article 282 of the
Labor Code (substantive aspect) and (b) the employee must be given an opportunity to be
heard and to defend himself (procedural aspect).
- In the case at bar, the requirements for both substantive and procedural aspects were
not satisfied.
- We affirm the findings of the Court of Appeals that there was no violation of the
conditions of the study leave grant. Thus, respondent could not be charged with serious
misconduct warranting her dismissal as a teacher in petitioner School. Petitioner has failed
to convince us that the alleged violations of the study leave grant constituted serious

Labor Law 1
misconduct which justified the termination of respondents employment. The most
respondent could be charged with was simple misconduct
- Misconduct is improper or wrongful conduct. It is the transgression of some established
and definite rule of action, a forbidden act, a dereliction of duty, willful in character, and
implies wrongful intent and not mere error of judgment. Under Article 282 of the Labor
Code, the misconduct, to be a just cause for termination, must be serious. This implies
that it must be of such grave and aggravated character and not merely trivial or
unimportant.
- We similarly affirm the Voluntary Arbitrators decision that respondent is not entitled to
moral and exemplary damages and attorneys fees because there is no evidence showing
that bad faith or malice attended the dismissal of respondent. Moral damages are
recoverable only where the dismissal is attended by bad faith or fraud, or constitutes an
act oppressive to labor, or is done in a manner contrary to morals, good customs or public
policy. A dismissal may be contrary to law but, by itself alone, it does not necessarily
establish bad faith
Disposition Petition is DENIED.

AGABON V NLRC
[PAGE 35]

STA. CATALINA COLLEGE V NLRC (TERCERO)


416 SCRA 233
CARPIO MORALES; November 19, 2003
NATURE
Petition for review on cetiorari
FACTS
- In June 1955, Hilaria was hired as an elementary school teacher at the Sta. Catalina
College. In 1970, she applied for and was granted a one year leave of absence without
pay on account of the illness of her mother. After the expiration in 1971 of her leave of
absence, she had not been heard from by petitioner school. In the meantime, she was
employed as a teacher at the San Pedro Parochial School during school year 1980-1981
and at the Liceo de San Pedro, Bian, Laguna during school year 1981-1982.
- In 1982, she applied anew at petitioner school which hired her.
- On May 31, 1997, Hilaria reached the compulsory retirement age of 65. Retiring
pursuant to Article 287 of the Labor Code, as amended by Republic Act 7641, petitioner
school pegged her retirement benefits at P59,038.35, computed on the basis of fifteen
years of service from 1982 to 1997. Her service from 1955 to 1970 was excluded in the
computation, petitioner school having asserted that she had, in 1971, abandoned her
employment.
- Hilaria insisted, however, that her retirement benefits should be computed on the basis of
her thirty years of service, inclusive of the period from 1955 to 1970. She thus concluded
that she was entitled to P190,539.90.
ISSUE
WON Hilarias services for petitioner school during the period from 1955 to 1970 should be
factored in the computation of her retirement benefits
HELD
- Hilaria cannot be credited for her services in 1955-1970 in the determination of her
retirement benefits. For, after her one year leave of absence expired in 1971 without her
requesting for extension thereof as in fact she had not been heard from until she
resurfaced in 1982 when she reapplied with petitioner school, she abandoned her
teaching position as in fact she was employed elsewhere in the interim and effectively
relinquished the retirement benefits accumulated during the said period.
- It is not disputed that the approved one year leave of absence without pay of Hilaria
expired in 1971, without her, it bears repeating, requesting for extension thereof or
notifying petitioner school if and when she would resume teaching. Nor is it disputed that
she was rehired only in 1982 after filing anew an application, without her proffering any
explanation for her more than a decade of absence.
Under the circumstances,
abandonment of work at petitioner school in 1971 is indubitably manifest.
- It should be noted that when Hilaria abandoned her teaching position in 1971, the law in
force was Republic Act 1052 or the Termination Pay Law, as amended by Republic Act
1787, Section 1 of which provides:
SEC. 1. In cases of employment, without a definite period, in a commercial, industrial,
or agricultural establishment or enterprise, the employer or the employee may
terminate at any time the employment with just cause; or without just cause in the case
of an employee by serving written notice on the employer at least one month in
advance, or in the case of an employer, by serving such notice to the employee at least

A2010

Disini

- 286 -

one month in advance or one-half month for every year of service of the employee,
whichever is longer, a fraction of at least six months being considered as one whole
year.
The employer, upon whom no such notice was served in case of termination of
employment without just cause may hold the employee liable for damages.
The employee, upon whom no such notice was served in case of termination of
employment without just cause shall be entitled to compensation from the date of
termination of his employment in an amount equivalent to his salaries or wages
corresponding to the required period of notice.
xxx
- Above-stated law should thus apply in the case at bar. Abandonment of work being a just
cause for terminating the services of Hilaria, petitioner school was under no obligation to
serve a written notice to her.
- As Hilaria was considered a new employee when she rejoined petitioner school upon reapplying in 1982, her retirement benefits should thus be computed only on the basis of her
years of service from 1982 to 1997.
- In the absence of a retirement plan or agreement providing for retirement benefits of
employees in the establishment, an employee upon reaching the age of sixty (60) years or
more, but not beyond sixty-five (65) years which is hereby declared the compulsory
retirement age, who has served at least five (5) years in the said establishment, may retire
and shall be entitled to retirement pay equivalent to at least one-half (1/2) month salary for
every year of service, a fraction of at least six (6) months being considered as one whole
year.
Disposition Petition granted in part.

TWO NOTICE RULE


AGABON V NLRC
[PAGE 35]
CAINGAT V NLRC
453 SCRA 142
QUISUMBING; March 10, 2005
NATURE
This petition for review on certiorari of the decision of the CA upholding the dismissal of
petitioner.
FACTS
- In 1983, petitioner Bernardino A. Caingat was hired by respondent Sta. Lucia Realty and
Development, Inc. (SLRDI) as officer-in-charge of maintaining the facilities and checking
the deliveries of construction materials in the different malls and subdivisions developed
by the SLRDI. In 1990, petitioner became the General Manager of SLRDIs sister
companies, R.S. Night Hawk Security and Investigation Agency, Inc., and R.S.
Maintenance and Services, Inc.
- In 1991, the Finance Manager discovered that petitioner deposited company funds in the
latters personal account and used the funds to pay his credit card purchases, utility bills,
trips abroad, and acquisition of a lot in Laguna.
- The company then sent the petitioner a letter which reads:
- Upon verification of financial records of the R.S. Nighthawk Security & Investigation
Agency, Inc. and the R.S. Maintenance & Services Corporation, where you were
designated as Manager, it was found that you have misappropriated company funds in the
sum of about P5,000,000.00 from 1992 up to the present.
- On August 13, 1996, without conducting any investigation, respondent R.S. Maintenance
filed a complaint docketed as Civil Case No. 65841 for sum of money and damages with
prayer for writ of preliminary attachment before the Pasig Regional Trial Court, Branch 67.
- On August 27, 1998, petitioner filed before the Labor Arbiter, a complaint, for illegal
dismissal against the respondents.
- Petitioner avers that he was dismissed from his employment by respondents without just
and legal cause and respondents dismissed him from employment without due process of
law.
ISSUES
1. WON he was dismissed for just cause
2. WON petitioner was denied due process of law
3. WON the violation of the due process requirement renders the dismissal illegal
HELD
1. YES
Ratio The petitioner held a position of utmost trust and confidence. He was entrusted with
cash fees for the maintenance and security services of 80 subdivisions and clubhouses.
He used company funds for payment of his personal bills. Instead of accounting for his
alleged unauthorized disbursements, petitioner suddenly departed from the country and

Labor Law 1
returned only after more than three years and claimed that he was illegally dismissed and
he was entitled to the alleged misappropriated funds by way of commission apart from the
monthly salary he received. Given the circumstances of this case, we can conclude that
managements loss of trust and confidence on petitioner was well justified. Private
respondents had every right to dismiss petitioner. Petitioners long period of
disappearance from the scene and departure for abroad before making a claim of illegal
dismissal does not contribute to its credibility.
2. YES
Ratio The due process prescribed in Article 277 of the Labor Code, as amended, and in
Sections 2 and 7, Rule I, Book VI of the Implementing Rules of the Labor Code, are
mandatory. Two notices should be sent to the employee. The first notice apprises the
employee of the particular acts or omissions for which his dismissal is sought; while the
second informs the employee of the employers decision to dismiss him. The latter must
come after the employee is given a reasonable period from receipt of the first notice within
which to answer the charge, and ample opportunity to be heard and defend himself with
the assistance of his representative, if he so desires. In this case, the respondents only
sent the first notice, gleaned from the June 20, 1996 memorandum. There was no second
notice. Neither the public notice in the Philippine Daily Inquirer, a newspaper of general
circulation, nor the demand letter could constitute substantial compliance. What the public
notice did was to inform the public that petitioner was already separated as of June 20,
1996, the same day he was suspended.
- The management failed to observe fully the procedural requirement of due process for
the termination of petitioners employment in its failure to provide two notices as required
in Article 277 of the Labor Code, as amended, and in Sections 2 and 7, Rule I, Book VI of
the Implementing Rules of the Labor Code, which are mandatory.
3. NO, the dismissal is still valid as previously held in Agabon v. NLRC
Ratio In Agabon v. NLRC, we said that if the dismissal was for cause, the lack of statutory
due process should not nullify the dismissal, or render it illegal or ineffectual. But the
violation of the petitioners right to statutory due process by respondents warrants the
payment of indemnity in the form of nominal damage.
Disposition The assailed Resolutions of the Court of Appeals in are hereby MODIFIED.
Petitioner Bernardino A. Caingats dismissal from employment by private respondents is
upheld on the ground of loss of trust and confidence on a managerial employee, a just
cause for termination. However, for failure to comply fully with the requirement of notice as
part of due process, the private respondents are ORDERED to pay petitioner Bernardino
A. Caingat the amount of P30,000.00 as nominal damages.

HAYLIFT MANILA INC V CA


[PAGE 226]
GENUINO ICE CO INC V MAGPANTAY
[PAGE 206]

HEARING
MAGOS V NLRC
300 SCRA 484
BELLOSILLO; December 28, 1998
NATURE
This special civil action for certiorari
FACTS
- Danilo J. Magos became an employee of PEPSI on 5 April 1987. He rose from the ranks
until he was appointed Route/Area Manager covering different areas in Northern
Mindanao. On 1 March 1991 he was assigned to handle the Butuan Plant in Surigao City.
In July 1991 PEPSI entered into a Sales and Distributorship Agreement with one Edgar
Andanar covering the entire Siargao Island. The Agreement included, among others, the
following terms: (a) that the Distributor shall be the sole agent of PEPSI in the entire
Siargao Island, Surigao City, and (b) that PEPSI would not directly or indirectly sell to or
serve anybody in the covered territory of the Distributor unless extremely necessary.
- Andanar complained formally to the Plant General Manager Val Lugti that petitioner was
still serving Tony Chua and Boy Lim, clients who were both within the area of the
agreement. On 15 April 1992 District Manager Reynaldo Booc issued a memorandum to
petitioner to stop effective immediately "giving deals to Siargao Island dealers, unless and
only, if Andanar cannot supply them due to unavoidable circumstances beyond his
control," and only up to a specified limit. Magos reported to Booc a negative trend in the
sales of PEPSI in Siargao Island as a result of Andanar's shortage of stocks and the
conversion to Coke of several wholesalers, notably Boy Lim. On 16 and 24 June 1992
Ramonito Endozo and Ramon Ganzon, respectively, reported the sales of Pepsi products
by salesman Prudencio Palen to Boy Lim at "7:1" deal allegedly upon instructions of

A2010

- 287 -

Disini

petitioner. Such sales were either made to fictitious dealers or diverted to other dealers
without receipts.
- On the basis of these reports, Magos was notified by Booc of his temporary recall
effective 1 July 1992 on the ground of his "continued refusal to follow orders/instructions of
a superior after 2 or more successive reminders or warnings." 4 He was also required to
submit a written explanation, which he did on 30 June 1992 citing among others: (a) the
lack of proper turnover of jurisdiction to distributor and guidelines thereto; (b) the rapid
conversion to Coke of previous big account dealers like Boy Lim in Siargao; and, (c) the
lack of ability of Andanar to supply such dealers. Finding Magos' explanation insufficient
PEPSI on 27 July 1992 notified Magos of an administrative investigation against him on
grounds of disobedience and breach of trust and confidence as shown by the reports of
Endozo and Ganzon, the audit reports of the Home Office Auditors, the complaint of
Andanar and the memorandum of Booc. 6 On 7 September 1992, Magos was notified of
his termination for disobedience and breach of trust and confidence. 7
- On 25 September 1992 Magos filed a complaint for illegal dismissal and non-payment of
wages, 13th month pay, premium pay for holidays and rest days, night shift pay and
allowances. After petitioner waived his right to a formal hearing, the Labor Arbiter set a
date for the submission of position papers. However, petitioner failed to submit a position
paper even after his two motions for extension to file the same were granted. After the
lapse of the extended period by nine (9) days, the Labor Arbiter issued an order submitting
the case for resolution and considering the petitioner to have waived his right to submit
evidence. 9 Petitioner's subsequent motion for reconsideration was denied on the ground
that such a motion was not allowed by the NLRC rules.
- On 18 March 1993 the Labor Arbiter ruled that the dismissal was valid on grounds of
insubordination and loss of confidence upon proof of Magos' sale of PEPSI products
despite the oppossion of his superiors. Magos' claims for 13th month pay, holiday pay, rest
day pay and night shift differentials were denied as he was a managerial employee. The
Labor Arbiter, however, found that Magos was dismissed without due process as it was
done in an arbitrary and perfunctory manner without any investigation to provide him with
an opportunity to present his side. Accordingly, PEPSI was ordered to give Magos
financial assistance of P2,000.00.
- Petitioner appealed to the NLRC imputing grave abuse of discretion to the Labor Arbiter
for denying him the right to present evidence on his behalf and for sustaining the legality of
his dismissal.
- On 16 May 1995 the NLRC also found that the dismissal of petitioner was done in an
arbitrary manner as there was no record of any investigation conducted. However, the
Commission opined that there was enough breach of confidence to justify Magos'
dismissal considering that he was duty-bound to follow and obey the instructions of his
superiors irrespective of his personal convictions. Thus, inspite of a finding of good faith
on Magos' part and lack of damage on PEPSI, it affirmed the Arbiter's finding of illegal
dismissal and the award of indemnity. Additionally, in consideration of Magos' good faith
and long service, the Commission also awarded him one-half (1/2) month separation pay
for every year of service.
- Both parties sought recosideration Magos faulted the NLRC for its failure to award him
the reliefs prayed for in his complaint, while PEPSI questioned the grant of separation pay
in Magos favor. On 29 August 1995 the NLRC denied both motions for reconsideration.
ISSUE
WON the employee was given due process as regards his termination
HELD
YES
- Both the NLC and the Labor Arbiter found that no formal hearing was conducted
regarding petitioner's dismissal. Although a hearing is essential to due process, we did
hold that no formal hearing was necessary when the petitioner had already admitted his
responsibility for the act he was accused of. Even though petitioner in this case never
admitted the accusations of dishonesty against him, he impliedly acknowledged his
insubordination as shown in his petition.
- During the investigation, Petitioner had however admitted as a sign of good faith, various
"saving measures" that he undertook to prevent the competitors from easing out Private
Respondent from its dominant market hold in Siargao Island
- Petitioner was subsequently terminated despite all the pertinent explanations he had
given to his immediate superiors for alleged violation of the above-mentioned Company
Rules and Regulations and for alleged loss of trust and confidence.
- Evidently, Magos regarded his sales to Siargao Island dealers covered by the Sales and
Distributorship Agreement with Andanar as "saving measures." As a consequence, he
earned the ire of his superiors for persistently ignoring the agreement. He further risked
losing his job by presenting what he deemed as "pertinent explanations" to justify the
questioned sales. Thus, even if no hearing was conducted on Magos' disobedience, the
requirement of due process was sufficiently met where petitioner was accorded the
chance to explain his side. The award of indemnity in the sum of P2,000.00 thereto is no
longer warranted in the light of this finding.

Labor Law 1
CAURDANETAAN PIECE WORKERS UNION V
LAGUESMA
285 SCRA 291
PANGANIBAN; February 24, 1998
NATURE
Petition for Certiorari
FACTS
- Petitioner union has ninety-two (92) members who worked as 'cargador' at the
warehouse and ricemills of private respondent Corfarm at Umingan, Pangasinan since
1982.
- They were paid by private respondent on a piece rate basis. When private respondent
denied some benefits to these cargadores, the latter organized petitioner union. Upon
learning of its formation, private respondent barred its members from working with them
and replaced them with non-members of the union sometime in the middle of 1992.
- On July 9, 1992, petitioner filed a petition for certification election before the Regional
Office No. I of the Department of Labor and Employment
- While this petition for certification election was pending, petitioner also filed on November
16, 1992, a complaint for illegal dismissal, unfair labor practice, refund of illegal
deductions, payment of wage differentials, various pecuniary benefits provided by laws,
damages, legal interest, reinstatement and attorney's fees, against private respondent
before the Regional Arbitration Branch No. 1 of Dagupan City
- On November 24, 1992, Labor Arbiter Ricardo Olairez, directed the parties to submit
position paper and to appear for hearing on the said date. Only the complainant petitioner
submitted its position paper.
- Likewise in the scheduled hearing on December 14, 1992, private respondent did not
appear thus Labor Arbiter Olairez allowed the president of petitioner union Juanito
Costales to testify and present its evidence ex-parte.
- On December 16 1992, another notice was sent to the parties to appear on [the] January
7, 1993 hearing by Labor Arbiter Emiliano de Asis.
- Before the scheduled hearing on January 7, 1993, complainant petitioner filed a motion
to amend complaint and to admit amended complaint.
- On March 18, 1993, Med-Arbiter Sinamar E. Limos issued an Order granting the petition
for certification election earlier filed.
- Meanwhile, Labor Arbiter Rolando D. Gambito in the illegal dismissal case ordered
respondents to submit their position paper, together with their documentary evidence, if
any, within 10 days from receipt of the order
- On September 7, 1993, public respondent Laguesma issued a Resolution denying the
appeal filed by private respondent against the order of Med-Arbiter Limos granting the
petition for certification election.
- Acting on said denial, private respondent filed a motion for reconsideration which was
granted in an Order dated January 4, 1994 by public respondent Laguesma dismissing the
petition for certification election for lack of employer-employee relationship.
- Petitioner in turn filed a motion for reconsideration but it was denied by public respondent
Laguesma when he reaffirmed the dismissal of petition for certification election.
- Thus, the union filed its first petition for certiorari
- On September 14, 1993, Labor Arbiter Rolando D. Gambito issued his decision finding
the dismissal of petitioner's members illegal. On appeal by both parties, NLRC set aside
the appealed decision and remanded the case to the labor arbiter for further proceedings.
Petitioner's motion for reconsideration was later denied.
- The solicitor general, who was supposed to represent both public respondents, joined
petitioner in praying that the petition in the First Case "be granted and that judgment be
rendered annulling" the assailed Orders of Respondent Laguesma. The SolGen likewise
filed another "Manifestation and Motion (In Lieu of Comment)" in the Second Case,
praying that "judgment be rendered annulling the resolution of NLRC ordering public
respondent to proceed with the case instead of remanding the same to the labor arbiter of
origin.
- In a Resolution dated March 29, 1995, 13 this Court ordered the consolidation of the two
cases.
ISSUE
WON private respondent was denied due process
HELD
NO
- Petitioner assails the NLRC for setting aside the labor arbiter's decision and remanding
the case for further proceedings. Petitioner argues that the order of remand "will only
prolong the agony of the 92 union members and their families for living or existing without
jobs and earnings to give them support."
- Further, petitioner contends that the Labor Arbiter had rendered a decision on September
14, 1993 in favor of petitioner based on the available records of the case after giving more
than ample opportunities to private respondents herein to submit their position paper and
other pleadings alleging their evidences against the causes of action of petitioner alleged

A2010

- 288 -

Disini

in the complaint for illegal dismissal, unfair labor practice, non-payment of various benefits
granted by existing laws during their employment, illegal deductions or diminution of their
underpaid daily wages, non-payment of wage increases and other causes of action
pleaded
by
the
complainant
or
herein
petitioner.
- Private respondent had been duly informed of the pendency of the illegal dismissal case,
but it chose not to participate therein without any known justifiable cause. The labor arbiter
sent notices of hearing or arbitration to the parties, requiring them to submit position
papers at 1:30 p.m. on November 14, 1992. Respondent Corfarm did not attend the
hearing. According to Respondent NLRC, there was no proof that Respondent Corfarm
received such notice. In any case, petitioner filed a Motion to Admit Amended Complaint
on December 23, 1992. Again, another notice for hearing or arbitration on January 7, 1993
was sent to the parties. This was received by petitioner's counsel as evidenced by the
registry return receipt duly signed by private respondent's counsel, Atty. Alfonso Bince, Jr.
It was only on January 28, 1993, however, that Atty. Bince entered his appearance as
counsel for Respondent Corfarm. On May 10, 1993, Corfarm was again given a new
period of ten (10) days within which to submit its position paper and documentary
evidence; "otherwise, [the labor arbiter] will be constrained to resolve this case based on
available evidence on record." As evidenced by a registry return receipt, a copy of said
directive was received by respondent's counsel on May 25, 1993. Still and all, Corfarm
failed to file its position paper. Clearly, private respondent was given an opportunity to
present its evidence, but it failed or refused to avail itself of this opportunity without any
legal reason. Due process is not violated where a person is given the opportunity to be
heard, but chooses not to give his side of the case.
- Contrary to the conclusions of the NLRC and the arguments of private respondent, the
findings of the labor arbiter on the question of illegal dismissal were based on credible,
competent and substantial evidence.
- It is to be borne in mind that proceedings before labor agencies merely require the
parties to submit their respective affidavits and position papers. Adversarial trial is
addressed to the sound discretion of the labor arbiter. To establish a cause of action, only
substantial evidence is necessary; i.e., such relevant evidence as a reasonable mind
might accept as adequate to support a conclusion, even if other minds equally reasonable
might conceivably opine otherwise.
- As ruled in Manalo vs. Roldan-Confesor:
"Clear and convincing proof is '. . . more than mere preponderance, but not to extent of
such certainty as is required beyond reasonable doubt as in criminal cases . . .'while
substantial evidence '. . . consists of more than a mere scintilla of evidence but may be
somewhat less than a preponderance . . .' Consequently, in the hierarchy of evidentiary
values, We find proof beyond reasonable doubt at the highest level, followed by clear
and convincing evidence, preponderance of evidence, and substantial evidence, in that
order."
- Evidence to determine the validity of petitioner's claims, which the labor arbiter relied
upon, was available to Respondent NLRC. These pieces of evidence are in the case
records.
- It must be stressed that labor laws mandate the speedy administration of justice, with
least attention to technicalities but without sacrificing the fundamental requisites of due
process. In this light, the NLRC, like the labor arbiter, is authorized to decide cases based
on the position papers and other documents submitted, without resorting to the technical
rules of evidence. Verily, Respondent NLRC noted several documentary evidence
sufficient to arrive at a just decision. Indeed, the evidence on record clearly supports the
conclusion of the labor arbiter that the petitioners were employees of respondent, and that
they were illegally dismissed.
- Why Respondent NLRC refused to rule directly on the appeal cannot be explained. The
remand of a case or an issue to the labor arbiter for further proceedings is unnecessary,
considering that the NLRC was in a position to resolve the dispute based on the records
before it and particularly where the ends of justice would be served thereby. Remanding
the case would needlessly delay the resolution of the case which has been pending since
1992.
Disposition both petitions are GRANTED

NATIONAL SEMICONDUCTOR (HK) DISTRIBUTION,


LTD V NLRC (SANTOS)
291 SCRA 348
BELLOSILLO; June 26, 1998
FACTS
- Petitioner National Semiconductor (HK) Distribution, Ltd., a foreign corporation,
manufactures and assembles electronic parts for export.
- Respondent Edgar Philip C. Santos was employed by NSC as a technician in its Special
Products Group assigned to the graveyard shift starting at ten o clock in the evening until
six o clock in the morning.
- On 8 January 1993 Santos did not report for work on his shift. He resumed his duties as
night shift Technician Support only on 9 January 1993. However, at the end of his shift
the following morning, he made two (2) entries in his daily time record (DTR) to make it

Labor Law 1
appear that he worked on both the 8th and 9th of January 1993.
- His immediate supervisor, Mr. Joel Limsiaco, checked the DTRs and found out that
Santos indeed did not report for work on 8 January. But when he checked Santos DTR
again in the morning of 9 January 1993 he found the entry made by Santos for the day
before.
- Informal investigations were conducted by management. Santos was required in a
memorandum to explain in writing within 48 hours from notice why no disciplinary action
should be taken against him for dishonesty, falsifying daily time record DTR and violation
of company rules and regulations.
- Santos submitted his written explanation alleging that he was ill on the day he was
absent. As regards the entry on 8 January, he alleged that it was merely due to oversight
or carelessness on his part.
- Finding Santos' explanation unsatisfactory, NSC dismissed him on the ground of
falsification of his DTR, which act was inimical to the company and constituted dishonesty
and serious misconduct.
- Santos filed a complaint for illegal dismissal and non-payment of back wages, premium
pay for holidays and rest days, night shift differential pay, allowances, separation pay,
moral damages and attorneys fees.
- The Labor Arbiter found that Santos was dismissed on legal grounds although he was
not afforded due process, hence, NSC was ordered to indemnify him P1,000.00.
- NSC appealed to the NLRC which affirmed the Labor Arbiter.
- NSC imputes grave abuse of discretion to the NLRC in affirming the Labor Arbiters
award of night shift differentials and P1,000.00 indemnity for alleged violation of due
process. It contends that the question of non-payment of night shift differentials was never
raised as an issue nor pursued and proved by Santos in the proceedings before the Labor
Arbiter; that Santos was already paid his night shift differentials, and any further payment
to him would amount to unjust enrichment; and, that the P1,000.00 indemnity is totally
unjustified as he was afforded ample opportunity to be heard.
ISSUES
1. WON the complainant should be awarded a night shift differential
2. WON the requirements of due process substantially were complied with in dismissing
the worker
HELD
1. YES
- The fact that Santos neglected to substantiate his claim for night shift differentials is not
prejudicial to his cause.
- The burden of proving payment rests on petitioner NSC. Santos allegation of nonpayment of this benefit, to which he is by law entitled, is a negative allegation, which need
not be supported by evidence unless it is an essential part of his cause of action.
- His main cause of action is his illegal dismissal, and the claim for night shift differential is
but an incident of the protest against such dismissal. Thus, the burden of proving that
payment of such benefit has been made rests upon the party who will suffer if no evidence
at all is presented by either party.
- Jimenez v. National Labor Relations Commission:
As a general rule, one who pleads payment has the burden of proving it. Even
where the plaintiff must allege non-payment, the general rule is that the burden rests
on the defendant to prove payment, rather than on the plaintiff to prove nonpayment. The debtor has the burden of showing with legal certainty that the
obligation has been discharged by payment.
- By choosing not to fully and completely disclose information to prove that it had paid all
the night shift differentials due to private respondent, petitioner failed to discharge the
burden of proof.
- Complainant should be awarded a night shift differential but limited to three (3) years
considering the prescriptive period of money claims.
2. YES
- Santos was accorded full opportunity to be heard before he was dismissed.
- The essence of due process is simply an opportunity to be heard, or as applied to
administrative proceedings, an opportunity to explain ones side.
- Petitioner furnished private respondent notice as to the particular acts which constituted
the grounds for his dismissal.
- By requiring him to submit a written explanation within 48 hours from receipt of the
notice, the company gave him the opportunity to be heard in his defense. Private
respondent availed of this chance by submitting a written explanation. - Furthermore,
investigations on the incident were actually conducted on 9 January 1993 and 11 January
1993.
- Respondent was notified on 14 January 1993 of the managements decision to terminate
his services.
- It is clear that the minimum requirements of due process have been fulfilled by petitioner.
- That the investigations conducted by petitioner may not be considered formal or
recorded hearings or investigations is immaterial.
- A formal or trial type hearing is not at all times and in all instances essential to due
process, the requirements of which are satisfied where the parties are afforded fair and
reasonable opportunity to explain their side of the controversy. It is deemed sufficient for

A2010

- 289 -

Disini

the employer to follow the natural sequence of notice, hearing and judgment.
Disposition Petition was dismissed.

LA CARLOTA PLANTERS ASSN INC V NLRC


[PAGE 238]
LAVADOR V J MARKETING CORP
497 SCRA
SANDOVAL-GUTIERREZ; June 28, 2005

NATURE
Petition for review on certiorari
FACTS
- January 7, 1991: Elsie T. Lavador was
employed
by
J
Marketing
Corporation as a daily paid worker.
Eventually, she was promoted as
assistant cashier in respondents
branch office at Butuan City, receiving
a monthly salary of P3,834.00.
- Claiming that respondent failed to
remit the P1,000.00 payment of Robert
Braza; to issue official receipt for the
P1,259.00 check payment of Chelito M.
Delliva; and to apply the same to his
account, respondent issued inter-office
memoranda dated June 9, 1999 and
August 23, 1999 charging petitioner
with misappropriation and directing
her to submit a written explanation. In
the meantime, respondent reassigned
her as receptionist.
- September 1, 1999: after evaluating
the
evidence
against
petitioner,
respondent issued a notice terminating
her services for loss of trust and
confidence.
- Lavador filed with the Office of the
Labor Arbiter a complaint for illegal
dismissal against respondent and
Rogelio U. Soyao, its Executive Vice
President and General Manager.
- Labor Arbiter rendered a Decision
finding that petitioner was not illegally
dismissed from the service but
ordering respondents to pay her

Labor Law 1
P12,392.73 as salary differential and
P1,239.27 attorneys fees.
- Upon appeal, the NLRC rendered a
Decision affirming with modification
the Decision of the Labor Arbiter in the
sense that the award of salary
differential and attorneys fee is
deleted. Petitioner's MFR was denied.
- On appeal, CA rendered a Decision
affirming with modification the assailed
NLRC Decision. While the said court
upheld the termination of petitioners
employment, however, it ordered
respondents to pay her P10,000.00 as
damages for violating her right to due
process. Petitioner's MFR was denied.
ISSUE
WON petitioner was deprived of her
right to due process
HELD
YES
- Section 2, Rule XXIII, Book V of the
Implementing Rules of the Labor Code
provides fro the standards of due
process and requirements of notice to
be followed in all cases of termination
of employment.
- Santos vs. San Miguel Corporation:
Procedural due process requires the
employer to give the employee two
notices. First is the notice apprising
him of the particular acts or omissions
for which his dismissal is sought.
Second is the subsequent notice
informing him of the employers
decision to dismiss him.
- Homeowners Savings and Loan Association,
Inc.
vs.
NLRC:
Actual adversarial
proceeding becomes necessary only for
clarification or when there is a need to
propound searching questions to
unclear witnesses. This is a procedural
right which the employee must,

A2010

- 290 -

Disini

however, ask for. It is not an inherent


right.
Petitioner
requested
that
an
investigation
be
conducted
but
respondents
vehemently
refused.
Clearly, petitioner was deprived of her
right to due process.
- Agabon vs. National Labor Relations
Commission: Procedurally, (1) if the
dismissal is based on a just cause
under Article 282, the employer must
give the employee two written notices
and a hearing or opportunity to be
heard if requested by the employee
before terminating the employment: a
notice specifying the grounds for which
dismissal is sought, a hearing or an
opportunity to be heard and after
hearing or opportunity to be heard, a
notice of the decision to dismiss; x x x.
- From the foregoing rules four
possible situations may be derived: (1)
the dismissal is for a just cause under
Article 282 of the Labor Code, for an
authorized cause under Article 283, or
for health reasons under Article 284,
and due process was observed; (2) the
dismissal is without just or authorized
cause but due process was observed;
(3) the dismissal is without just or
authorized cause and there was no due
process; and (4) the dismissal is for
just or authorized cause but due
process was not observed.
xxx In the fourth situation, the
dismissal should be upheld. While the
procedural infirmity cannot be cured,
it should not invalidate the dismissal.
However, employer should be held
liable for non-compliance with the
procedural requirements of due
process.
xxx The violation of the petitioners
right to statutory due process by the
private respondent warrants the

Labor Law 1
payment of indemnity in the form of
nominal damages. The amount of
such damages is addressed to the
sound discretion of the court, taking
into
account
the
relevant
circumstances.
xxx
Considering
the
prevailing
circumstances in the case at bar, we
deem it proper to fix it at P30,000.00.
We believe this form of damages
would serve to deter employers from
future violations of the statutory due
process rights of employees. At the
very least, it provides a vindication or
recognition of this fundamental right
granted to the latter under the Labor
Code and its Implementing Rules.
- In this case, the dismissal of
petitioner from the service is due to
dishonesty or a just cause. But due
process was not observed as no
hearing was conducted despite her
request. Thus, respondents should be
held liable for violation of her right to
due process and should pay her
indemnity in the form of nominal
damages, pursuant to our ruling in
Agabon, which we fix at P20,000.00.
Disposition Petition is GRANTED.

SEPARATE OPINION
PANGANIBAN

- reiterates his dissent in Agabon v.


NLRC & Serrano v. NLRC that a
violation of due process should be
sanctioned by reinstatement and back
wages, not merely nominal damages.
POSITION PAPER
SHOPPES MANILA INC V NLRC (CAYUCA & TORNO)
419 SCRA 354
CALLEJO; January 14, 2004

A2010

- 291 -

Disini

NATURE
Petition for review of decision of CA.
FACTS
- Shoppes Manila is a domestic corporation engaged in garments manufacturing using the
brand name KAMISETA.
- On May 6, 1994, the petitioner employed private respondent Lorie Torno as trimmer In
April 1997 the private respondent and a co-employee, Maricar Buan, were tasked to
handle the inventory of finished products.
- Shoppes Manila received information from the head of its production department that
Buan and the private respondent had been stealing KAMISETA items from the factory.
- On July 30, 1997, the private respondents supervisor, Ms. Myrasol O. Silva, conducted
the inspection and submitted a report to the effect that she found Kamiseta property in
Tornos residence. Shoppes Manila issued a disciplinary action form suspending the
private respondent indefinitely without pay. On August 25, 1997, a notice of dismissal was
addressed to the private respondent specifying the charge against her.
- Torno failed to appear during the scheduled hearing. Consequently, the petitioner
decided to dismiss the private respondent from her employment. Despite mandatory
conferences, the parties did not reach an amicable settlement.
- In due course, the parties submitted their respective position papers and replies. The
petitioner filed a motion for the labor arbiter to conduct a formal investigation on its claim
that a full blown hearing during which the witnesses can be cross-examined by the
opposing counsel was necessary to ascertain the truth.
- LA Tumanong (labor arbiter given jurisdiction over the case) was replaced by LA Cuyuca
who issued an order declaring that the case was submitted for decision. The petitioner
filed a manifestation and motion informing LA Cuyuca that a formal hearing had been set
by LA Tumanong and requested that the case be set for hearing anew. However, no
action was taken by LA Cuyuca on the said motion. In August 1998 LA Cayuca rendered a
decision in favor of Torno, holding that she was illegally dismissed. LA Cuyuca declared
that the private respondent was denied of her right to due process before she was
dismissed from her employment and that the petitioner failed to show that it notified the
private respondent of the charges against her.
- Shoppes Manila appealed to the NLRC, arguing that LA Cuyucas failure to conduct a
hearing deprived the petitioner of its vested right; consequently, her decision was null and
void. NLRC dismissed the appeal.
Petitioners Claim
- When LA Tumanong agreed to conduct a formal hearing of the case and, indeed, set the
case for hearing, the petitioner thenceforth acquired a vested right.
ISSUE
WON Shoppes Manila had a vested right to a formal hearing because the first labor arbiter
granted its motion (This case is under Position Paper as Requirement in the outline)
HELD
NO, Shoppes Manila is not entitled to said right.
Ratio The holding of a formal hearing or trial is discretionary with the labor arbiter and is
something that the parties cannot demand as a matter of right.
- The requirements of due process are satisfied when the parties are given the opportunity
to submit position papers wherein they are supposed to attach all the documents that
would prove their claim in case it be decided that no hearing should be conducted or was
necessary.
Reasoning
- Pursuant to Section 5, Rule V of the New Rules of Procedure of the NLRC, the labor
arbiter has the authority to determine whether or not there is a necessity to conduct formal
hearings in cases brought before him for adjudication.
- It is entirely within the authority of the labor arbiter to decide a labor case before him,
based on the position papers and supporting documents of the parties, without a trial or
formal hearing.
- The order of LA Tumanong granting the petitioners motion for a hearing of the case was
not conclusive and binding on LA Cuyuca who had the discretion either to hear the case
before deciding it, or to forego with the hearing (if in her view, there was no longer a need
therefore as the case could be resolved on its merits based on the records).
- The Court affirms the decision of the CA that Torno was not legally dismissed. In order to
effect a valid dismissal, the law requires that (a) there be just and valid cause as provided
under Article 282 of the Labor Code; and (b) the employee be afforded an opportunity to
be heard and to defend himself. Shoppes Manila had failed to show that it had complied
with the two-notice requirement: (a) a written notice containing a statement of the cause
for the termination to afford the employee ample opportunity to be heard and defend
himself with the assistance of his representative, if he so desires; (b) if the employer
decides to terminate the services of the employee, the employer must notify him in writing
of the decision to dismiss him, stating clearly the reason therefor
Disposition Petition is dismissed, and decision of CA is affirmed.

C.F. SHARP & CO V ZIALCITA

Labor Law 1
495 SCRA 387
QUISUMBING; July 17, 2006
NATURE
Petitioner seeking the reversal of the Decision of the CA dismissing its petition for
certiorari against the NLRC, as well as the Resolution denying its motion for
reconsideration
FACTS
- Petitioner C.F. Sharp & Co., Inc. hired respondent Renato Zialcita as a clerk in its
Crewing Department. It promoted him to Assistant Crewing Manager and tasked him to
process shipping papers of the petitioners seamen. The petitioner alleged that seaman
Marcial Tanoy returned to the country bringing with him US$1,000 from another seaman,
Fernando Guerrero. On May 27, 1993, Tanoy gave the money to respondent Zialcita. On
June 14, 1993, when Guerreros father and sister came to the petitioners office to get the
money, the respondent denied he had it. On June 25, 1993, the Guerreros came back
together with Tanoy. This time, the respondent acknowledged he had the money but could
not give it to them then. On June 29, 1993, he returned US$800 to Guerreros father. The
respondent also issued a promissory note on the balance. The Guerreros reported the
matter to the Senior Crewing Manager, who immediately paid the deficiency. Respondent
was served with a notice giving him 72 hours to respond to the Guerreros complaint, and
placed on preventive suspension immediately.
- Respondent averred that Tanoy had approached him on May 28, 1993, but he refused to
accept the money due to the petitioners policy against unauthorized handling of
remittances from its seamen. When the Guerreros went to his office, he informed them
that he did not have the money, and on June 23, 1993, Tanoy returned and begged him to
accept the money since he was leaving for the Visayas and Guerreros wife would pick it
up the following day. She did not come so he brought the money home. On June 25, 1993,
the Guerreros went to his office but he advised them to come back since the money was
in his house. The respondent then brought back the money to his office and placed it
inside his drawer. When the Guerreros returned on June 29, 1993, he discovered that
US$200 was missing. He gave them the US$800 and promised to return the US$200. The
respondent later learned that the Guerreros submitted to the Senior Crewing Manager a
complaint against him. On July 2, 1993, he reimbursed the petitioner the balance of
US$200.
- Not finding the explanation acceptable, the petitioner dismissed Zialcita. Respondent
filed a complaint for illegal dismissal with money claims and damages before the NLRC.
The Labor Arbiter ordered respondent to reinstate the complainant. The NLRC vacated the
decision on the ground that the case was decided prematurely without affording the
petitioner the opportunity to present rebuttal evidence. Upon remand, the Labor Arbiter
dismissed the complaint for lack of merit. The NLRC reversed the Arbiters decision. The
CA affirmed the NLRC, finding that the date when the respondent received the money
from Tanoy was pivotal. If he received it on May 27, 1993, he would be guilty of gross
misconduct for giving the Guerreros a run-around. But if he received it only on June 23,
1993, the charge of gross misconduct against him would fail. Since petitioner failed to
show what sanction is imposed in its policies for the cited violation, the appellate court
upheld the NLRC ruling that the respondent should be reinstated and punished only with 1
month suspension.
ISSUES
1. WON Tanoys affidavit should be given credence although he was not cross-examined
2. WON there is just cause to dismiss respondent
HELD
1. YES
Ratio Trial-type hearings are not required in labor cases and these may be decided on
verified position papers, with supporting documents and their affidavits. It is not necessary
for the affiants to appear and testify and be cross-examined by the counsel for the adverse
party. Be that as it may, Tanoys affidavit is still insufficient to establish that the respondent
was guilty of gross misconduct as Tanoy failed to allege the specific date when he gave
the money to the respondent. There is no indubitable proof that as of June 14, 1993,
respondent already had the money.
2. NO
Ratio To be a valid ground for dismissal, loss of trust and confidence must be premised on
the fact that the employee concerned is invested with delicate matters, such as the
handling or care and protection of the property and assets of the employer. It was not
shown that respondent handled company property and assets. Respondent was remiss in
his duties when he received the money, failed to turn it over to the proper custodians, and
failed to produce the entire amount when it was finally claimed. However, dismissal is
incommensurate to the offense committed, especially in the absence of any malicious
intent or fraud on the respondents part.
Obiter

A2010

Disini

- 292 -

- In termination cases, the employer bears the onus of proving that the dismissal was for
just cause. When the breach of trust or loss of confidence alleged is not borne by clearly
established facts, as in this case, such dismissal on the cited grounds cannot be allowed.
Disposition petition is DENIED. The assailed Decision of the CA is AFFIRMED.

FAILURE OF DUE PROCESS


EFFECT OF FAILURE SUBSTANTIVE PROCEDURAL
AGABON V NLRC
[PAGE 35]
ALADDIN V CA
460 SCRA 234
AZCUNA; June 21, 2005
NATURE
Petition for certiorari
FACTS
- Petitioner Aladdin Transport Corp hired Rafael Rodriguez in 1990 as an accounting clerk.
In 1997, respondent alleged that her sister had a fight with their personnel manager. He
was then barred from entering the companys premises. He was then asked to take a
leave of absence for a month.
- While he was on leave, he received a letter from their personnel manager asking him to
shed light about the SSS contribution that he allegedly did not remit. Respondent merely
said he tried to report to the office, but petitioner did not allow him. Later he received a
letter informing him of his preventive suspension. He wrote a letter to the President of the
company but did not receive any reply. He then filed a complaint before the labor arbiter.
- Petitioner alleges that respondent used company funds and lent them with interest to his
co-employees. Petitioner also alleges that respondent has illegally been making payroll
salary deductions and has failed to remit SS contribution of his co-employees.
- The Labor Arbiter then ruled in favor of the petitioner. NLRC affirmed. The CA, however,
found that even though there was just cause in the dismissal, respondent was not given
due process. It ordered the payment of backwages and reinstatement.
ISSUE
WON the CA erred in ordering the payment of backwages
HELD
YES
- Recently, this Court has had occasion to revisit the Serrano doctrine and the present rule
is set forth in the Agabon v. NLRC, et al.,[5] namely, that where the dismissal is based on
a just cause, the failure to give the required notice does not invalidate the same, but
merely holds the employer liable for damages for violating said notice of requirement. The
amount of damages was fixed at Thirty Thousand Pesos (P30,000) by way of nominal
damages.
Disposition Petition GRANTED

GLAXO WELLCOME PHILIPPINES INC V


NAGKAKAISANG EMPLEYADO NG WELLCOME-DFA
453 SCRA 256
PANGANIBAN; March 11, 2005
NATURE
Petition for Review of the decision of the CA
FACTS
- Union NAGKAKAISANG EMPLEYADO NG WELLCOME-DFA (NEW-DFA) filed a Petition
for Certification Election with the DOLE-NCR seeking to represent the bargaining unit
comprised of all the regular rank-and-file employees of [petitioner] company GLAXOWELLCOME.

Labor Law 1
- Several days before the election GLAXO-WELLCOME issued a circular relative to the
improvement of the companys retirement policy bringing different employees to different
resorts.
- In the meantime, GLAXO-WELLCOME adopted a new Car Allocation Policy. Under the
provisions of the said car plan, a prioritization schedule in the assignment of company
vehicles is to be fixed based on the sales performance of the employees. Pursuant to the
same, several company cars had to be re-assessed and re-assigned in favor of other
employees more qualified under the priority list. Incidentally, included among the vehicles
that had to be re-allocated in accordance with the priority schedule of the new car plan
were [those] of union officers Norman Cerezo and Jossie Roda de Guzman.
- Accordingly, a memorandum was sent by the company to [Respondent] de Guzman
advising her that she would have to surrender the vehicle assigned to her in light of the
new car policy. De Guzman refused to turn over said car and instead sought
reconsideration from the companys National Sales Manager. The latter did not accede to
de Guzmans request. De Guzman, thru counsel, wrote the company, asking that the
withdrawal of her car be held in abeyance. The company, however, rejected her petition.
On December 7, 1990, de Guzman received another memorandum from the company,
again instructing her to return the vehicle. The following day, de Guzman sent a letter to
the company reiterating her plea for the suspension of the withdrawal of her car. On
December 17, 1990, a final warning was sent to de Guzman instructing her to return her
assigned vehicle or else she would be charged for insubordination and be dismissed.
Finally, because of de Guzmans staunch refusal to comply with the order, through a letter
dated December 20, 1990, she was cited, and at the same time, terminated for gross
insubordination. Norman Cerezo was of the same case.
- The Union alleged undue interference due to a massive electioneering and manipulative
acts of GLAXO-WELLCOME prior to and during the certification election and that the new
Car Allocation Policy adopted by the company was intended to harass, retaliate and
discriminate against union officers and members. Union also challenged the legality of the
suspension and dismissal of two of its officers, namely: Norman Cerezo and Jossie Roda
de Guzman. It argued that the suspension and dismissal were effected without any prior
hearing (Which was the only sticking issue in this case).
- Labor Arbiter dismissed the charges of unfair labor practice, illegal dismissal and illegal
suspension filed against GLAXO-WELLCOME by union. NLRC affirmed the dismissal of
the complaint. NLRC likewise denied the motion for reconsideration.The CA affirmed the
ruling of the National Labor Relations Commission (NLRC) adopted the findings of the
labor arbiter. It held that respondents had failed to proffer convincing evidence to
prove that petitioners assailed acts were ill-motivated and deliberately orchestrated
to interfere with or otherwise influence the conduct of the certification elections.
- Moreover, the CA ruled that there was nothing objectionable per se about the
programs or incentive schemes that the company had provided for the employees.
The appellate court said that the grant of benefits to the employees, as well as the
adoption of the Car Allocation Policy, constituted a proper exercise of the
companys management prerogatives. This plain company practice had been set
up to make petitioners employee benefits competitive with those of other
pharmaceutical corporations. De Guzman and Cerezo were among those adversely
affected by the policy, because they had failed to meet the sales performance
required thereunder, not because they were officers of the union.
- However, the CA held that the dismissal of De Guzman and the suspension of
Cerezo had not been validly effected. Opining that their defiant actuation toward
management constituted willful disobedience, which was a just cause for the termination
of their employment, the appellate court conceded the validity of the dismissal and
suspension. Nonetheless, the CA said that those actions (dismissal and suspension)
effected by petitioner could not be deemed legal, because it had failed to comply
with procedural due process mandated by the Labor Code and with the two-notice
requirement under the Implementing Rules. According to the CA, petitioner did not
accord private respondents the benefit of a proper charge, an opportunity to defend
themselves, and a formal investigation.
- The appellate court opined that the Memoranda were merely demands for respondents
to comply with the order to turn over their assigned cars. Those Memoranda merely
intimated the possibility that De Guzman and Cerezo might be charged and dismissed if
they continued to disobey the order.
ISSUE
WON the Court of Appeals erred in ruling that petitioner did not observe procedural due
process in terminating and suspending the employment of de Guzman and Cerezo,
respectively
HELD
YES, since there was substantial compliance through the memoranda.
- In the present case, petitioner sent respondents a total of three Memoranda stating that
their stubborn refusal to comply with the car policy and to surrender the subject vehicle
constituted gross insubordination, for which they could be dismissed. The December 5,
1990 Memorandum sent to Respondent De Guzman specified her acts that constituted
gross insubordination.

A2010

- 293 -

Disini

- To each Memorandum, respondents were able to reply and expla sincein, with the aid of
their counsel, why they had refused to return the vehicles; and, in effect, why they should
not be dismissed for gross insubordination. Initially, they asked petitioner not to implement
the car policy in the light of the Complaint and the Motion for the Issuance of a Writ of
Preliminary Injunction that they had filed. They explained that they could not work
effectively and efficiently for the company without the cars that had been assigned to
them.
- In their written replies to petitioners succeeding Memoranda -- which reiterated that their
actions constituted gross insubordination and could result in their termination -respondents, still through their counsel, reasoned that they were not claiming ownership of
the car. They said that their refusal to surrender the car to the company could not be
denominated as gross insubordination, because they were merely acting upon the advice
of their counsel. They added that, to enjoin the implementation of the car policy, they had
already lodged with the NLRC a complaint for unfair labor practice.
- Their counsel further alleged that De Guzman was apprehensive that she might not
immediately be given a replacement upon the return of the car. He stressed that the
vehicle was necessary to prevent adverse effects on the sales performance of
respondents. Ultimately, after petitioner had sent them a final warning, to which they also
ably replied, it served them a letter terminating their employment.
- Neither Section 2 of Book V of Rule XXIII nor Section 2(d) of Rule 1 of Book VI of the
Implementing Rules require strict literal compliance with the stated procedure; only
substantial compliance is needed. On this basis, the Memoranda sent to respondents
may be deemed to have sufficiently conformed to the first notice required under the
Implementing Rules. The Memoranda served the purpose of informing them of the
pending matters beclouding their employment and of extending to them an opportunity to
clear the air. In fact, not only were respondents duly informed of the particular acts for
which their dismissal was sought; they were, in truth and in fact, able to defend
themselves and to respond to the charges with the assistance of a counsel of their own
choosing. Respondents were amply informed of the cause of their dismissal. Their
correspondence with petitioner took almost a month, which was sufficient cooling time
within which the parties could have, and in fact had, tried to settle the problem amicably.
Moreover, petitioners Memoranda amply gave them a distinct, different and effective first
level of remedy (which was to surrender the vehicles) to protect their jobs. Furthermore,
they were still able to file a Complaint with the labor arbiter, with better knowledge of the
cause of their dismissal, with longer time to prepare their case, and with greater
opportunity to take care of the financial needs of their family pendente lite.
- Agabon v. NLRC effectively reverted to Wenphil and ruled that a dismissal due to
abandonment -- a just cause -- was not illegal or ineffectual, even if done without due
process; but that the employer should indemnify the employee with nominal damages for
non-compliance with statutory due process.
- To stress, if the dismissal is based on a just cause under Article 282 of the Labor Code,
the employer must give the employee (1) two written notices and (2) a hearing (or at least,
an opportunity to be heard). The first notice is intended to inform the employee of the
employers intent to dismiss and the particular acts or omissions for which the dismissal is
sought. The second notice is intended to inform the employee of the employers decision
to dismiss. This decision, however, must come only after the employee has been given a
reasonable period, from receipt of the first notice, within which to answer the charge; and
ample opportunity to be heard with the assistance of counsel, if the employee so desires.
- The twin requirements of (a) two notices and (b) hearing are necessary to protect the
employees security of tenure, which is enshrined in the Constitution, the Labor Code and
related laws.
Disposition Petition is GRANTED and the challenged Decision REVERSED. The
Decision of the NLRC dated August 28, 1998, affirming that of the labor arbiter dated
August 15, 1995, is REINSTATED.

2. OTHER PROCEDURAL MATTERS

BURDEN AND DEGREE OF PROOF


BURDEN
EQUITABLE PCI BANK V CAGUIOA
466 SCRA 658
PANGANIBAN; August 12, 2005
NATURE
Petition for Review of CA Decision

Labor Law 1
FACTS
- Generosa A. Caguioa was a Senior Manager of Equitable PCI Bank. She had worked for
said bank for 35 years. On Feb 3, 2000, George L. Go, respondents Chairman of the
Board, called her attention to the complaint of client Antonio Jarina regarding an
accounting activities, involving her participation, which cost him considerable damage. It
was about bank checks, which were issued to Jarina in exchange for cash at a discounted
rate -- all by means of his own capital. Jarina initially benefited from the new transactions
but in August 1997, no further interest was credited into his account and his principal had
not been returned to him. The scheme failed, resulting in his loss of investments of
around P4.3M. She denied any knowledge of the discounting activities. - VP and Internal
Auditor Nuguid directed her to explain and show cause why she should not be
administratively/financially dealt with the banks Code of Conduct in connection with the
said incidents. She pointed to an Arlene Pascual as the culprit and claimed that Jarina
picked on her as Ms. Pascual had already resigned and moved to the US. But a certain
Evelyn Magadia testified that Arlene would not have acted on her own had it not been for
the instigation of Caguioa and Ruth Amador. Magadia even declared that she liaised for
Arlene Pascual in the check discounting transactions, and Arlene would sometimes
instruct her to go to the Banks New York branch in order to pick up and deliver envelopes
containing cash to complainant Genie Caguioa. Magadia stated that the envelopes
containing cash were the complainants shares in the proceeds of the check discounting
activity.
- After evaluation and investigation, bank found her guilty of having personally participated
in the check discounting activity of Mr. Antonio Jarina, and therefrom, personally
benefited/profited to the prejudice of the bank. She was dismissed with automatic
forfeiture of benefits. She then filed complaint. LA upheld the dismissal of Caguioa, finding
her to have violated its Code of Conduct. NLRC reversed ruling that dismissal was illegal
and invalid. CA held that Caguioa was illegally dismissed for lack of clear and convincing
evidence which establish her direct participation in the alleged check discounting
activity/transaction remotely detrimental to the business and interest of the petitioner bank.
Revelations made by Evelyn Magadia were insufficient to justify the dismissal.
ISSUES
1. WON the totality of the evidence proves that the bank legally terminated the
employment of respondent
2. WON CA gravely erred in ruling that the information provided by Mrs. Magadia and the
complaint of Mr. Jarina were hearsay
HELD
1. YES
- The Court is not proscribed from determining and resolving factual issues where, as in
the present case, (a) the findings and conclusions of the labor arbiter, on one hand, and
the NLRC and the CA, on the other, are inconsistent on material and substantial points; (b)
the findings of the NLRC and the CA are capricious and arbitrary; and (c) the CAs findings
that are premised on a supposed absence of evidence are in fact contradicted by the
evidence on record.
- Due process requires that in reaching a decision, a tribunal must consider the entire
evidence presented. Thus, it must (1) accord all contending parties the opportunity to be
heard and (2) consider every piece of evidence presented in favor of each party.
- As the employer, petitioner bank has the burden of proving the legality of respondents
dismissal. The petitioners case rises or falls on the strength of the employers evidence,
not on the weakness of the employees defense. The employer, however, needs only to
adduce substantial evidence, which has been defined to be such relevant evidence that a
reasonable mind might accept as adequate to support a conclusion.
- The records show that more than substantial evidence supports LAs finding that Caguioa
had direct participation in the check-discounting scheme, and that her dismissal was valid.
Constituting substantial evidence were the Sworn Statement of Sibayan on the
investigation he conducted; the Complaint of Jarina; the audit trail; and the schedule of
withdrawals and deposits in Jarinas account, a fact sworn to by Vivian Tiu (a VP of the
Bank) and uncontroverted by Caguioa. Such evidence is adequate to justify the conclusion
that respondent was indeed a direct participant in and beneficiary of the unauthorized
check-discounting transactions.
2. YES
- It is not necessary that the affidavits and other documents presented conform with the
technical rules of evidence since in labor cases the rules of evidence prevailing in courts
of law or equity are not controlling. It is sufficient that the documents submitted by the
parties have a bearing on the issue at hand and support the positions taken by them.
Therefore, the information provided by Magadia, as corroborated by the Sworn Statement
of Sibayan, and the Complaint of Jarina, should be accorded probative value.
- See A221 of LC and Sec 3 of Rule V of the New Rules of Procedure of the NLRC
Disposition Petition GRANTED and the assailed Decision and Resolution REVERSED
and SET ASIDE. The Decision of the labor arbiter is REINSTATED.

CF SHARP & CO INC V ZIALCITA

A2010

Disini

- 294 [PAGE 290]

DEGREE
CENTRAL PANGASINAN ELEC COOP INC V
MACARAEG
[PAGE 195]
SALVADOR V PHIL MINING SERVICE CORP
[PAGE 195]

PRESCRIPTION PERIOD
AZCOR MANUFACTURING INC V NLRC
[PAGE 197]

OFFER TO REINSTATE
RANARA V NLRC
212 SCRA 631
CRUZ; August 14, 1992
NATURE
Appeal from NLRC decision
FACTS
- Petitioner Carlos Ranara had been working as a driver with respondent Oro Union
Construction Supply, when he was told by Fe Leonar, secretary of the other private
respondent, Jimmy Ting Chang, not to come back the following day. Thinking that she was
only joking, he reported for work as usual on Nov. 11, 1989, but was surprised to find
some other person handling the vehicle previously assigned to him. It was only then that
Ranara realized that he had really been separated. When he approached Leonar to ask
why his services were being terminated, she replied that he was hard headed and that
Chang sis not like his services.
- Ranara filed a complaint 3 days later with DOLE for illegal dismissal, reinstatement with
full back wages, underpayment of wages, overtime pay, non-payment of 13th month pay,
service incentive leave, separation pay and moral damages.
- The private respondents denied charges, contending that the petitioner had not been
illegally dismissed. Chang said had not authorized Leonar, or even his mother who was
the officer-in-charge during his absence at the time, to terminate Ranara's employment.
The truth was that it was Ranara who abandoned his work when he stopped reporting
from Nov. 11, 1989.
- LA held that Ranara had not been illegally dismissed. The decision stressed that at the
hearing of December 28, 1989, Chang offered to re-employ the petitioner as he was
needed in the store but the latter demurred, saying he was no longer interested. This
attitude, according to the Labor Arbiter, showed that it was the petitioner who chose to
stop working for Chang and not the latter who terminated his employment. The decision
was affirmed on appeal by NLRC. Hence, this appeal.
ISSUE
WON petitioner Ranara did not abandon his work and was illegally dismissed
HELD
YES
- We reject as a rank falsity the private respondents' claim that the petitioner had not been
illegally dismissed and in fact abandoned his work.
- On illegal dismissal: The secretary would not have presumed to dismiss him if she had
not been authorized to do so, considering the seriousness of this act. It is worth noting that
neither Chang's mother, who was the officer-in-charge in his absence, nor Chang himself
upon his return, reversed her act and reinstated the petitioner.
- It is clear that the petitioner was illegally dismissed without even the politeness of a
proper notice. Without cause and without any investigation, formal or otherwise, Ranara
was simply told that he should not report back for work the following day. When he did so
just the same, thinking she had only spoken in jest, he found that somebody else had
been employed in his place. When he protested his replacement, he was even scolded for
being "hard-headed" and not accepting his dismissal.
- On abandonment: The charge of abandonment does not square with the recorded fact
that three days after Ranara's alleged dismissal, he filed a complaint with the labor
authorities. The two acts are plainly inconsistent. Neither can Ranara's rejection of

Labor Law 1
Chang's offer to reinstate him be legally regarded as an abandonment because the
petitioner had been placed in an untenable situation that left him with no other choice.
- On Offer to Reinstate: The fact that his employer later made an offer to re-employ him
did not cure the vice of his earlier arbitrary dismissal. The wrong had been committed and
the harm done. Notably, it was only after the complaint had been filed that it occurred to
Chang, in belated gesture of good will, to invite Ranara back to work in his store. Chang's
sincerity is suspect. We doubt if his offer would have been made if Ranara had not
complained against him. At any rate, sincere or not, the offer of reinstatement could not
correct the earlier illegal dismissal of the petitioner.
Disposition Petition GRANTED. Petitioner is entitled to separation pay and 3 years' back
wages in lieu of reinstatement.

G. SANCTIONS AND REMEDIES

14.07 GENERAL RULE


NATURE OF REMEDIES TWIN REMEDIES
PHESCHEM V MOLDEZ
458 SCRA 339
PUNO; May 9, 2005
FACTS
- PHESCHEM INDUSTRIAL CORPORATION employed respondent PABLITO V. MOLDEZ
as operator of its payloader and bulldozer at its quarrying site in Palompon, Leyte
- He was suspended from work without pay for a period of seven (7) days, without being
informed of its reason. he reported back to work only to be told that the President of the
corporation, Tomas Y. Tan, was ill and his suspension had been extended for another 7
days without pay. Again, no reason was offered for said decision. After the lapse of the
extension, respondent was not allowed to return to work. When he inquired about the
status of his employment, Viacrusis assured him that he will be rehired in due time. Eight
(8) months passed but respondent received no word from petitioner
- filed a complaint against petitioner for illegal suspension and dismissal. He prayed for
reinstatement, or if reinstatement is not possible, an award of separation pay
-corporation averred that dismissal for just cause. They said that the bulldozer operated by
Moldez malfunctioned due to his negligence thus resulting to their loss of more or less
500,000 due to repair and non-utilization.
- Labor Arbiter held petitioner guilty of illegal dismissal and ordered the reinstatement of
respondent and payment of his backwages. This was affirmed by NLRC and CA
ISSUE
WON CA erred in affirming decisions and WON Moldez is entitled to separation pay in
spite of his reinstatement
HELD
- Decision of CA is affirmed. However, the circumstances in the case at bar do not warrant
the award of separation pay to respondent in lieu of reinstatement to his former position
- The legal consequences of an illegal dismissal are reinstatement of the employee
without loss of seniority rights and other privileges, and payment of his full backwages,
inclusive of allowances, and other benefits or their monetary equivalent. The law intended
reinstatement to be the general rule. It is only when reinstatement is no longer feasible
that payment of separation pay is awarded to an illegally dismissed employee
- Payment of separation pay as a substitute for reinstatement is allowed only under
exceptional circumstances, (1) when reasons exist which are not attributable to the fault or
beyond the control of the employer, such as, when the employer, who is in severe financial
strait and has suffered serious business losses, has ceased operations, implemented
retrenchment, or abolished the position due to the installation of labor-saving devices; (2)
when the illegally dismissed employee has contracted a disease and his reinstatement will
endanger the safety of his co-employees; or, (3) where strained relationship exists
between the employer and the dismissed employee

NUEVA ECIJA ELECTRIC CORP V NLRC


[PAGE 229]
LAKPUE DRUG INC V BELGA
[PAGE 208]

A2010

- 295 -

Disini

GREAT SOUTHERN MARITIME SERVICES CORP V


ACUNA
425 SCRA 422
AUSTRIA-MARTINEZ; February 28, 2005
NATURE
Petition for Certiorari
FACTS
- Petitioner Great Southern Maritime Services Corporation is a manning agency organized
and existing under Philippine laws. It is the local agent of petitioner Ferry Casinos Limited.
Petitioner Pioneer Insurance and Surety Corporation is the surety company of petitioner
GSMSC.
- On October 7, 1993, respondents Jennifer Anne B. Acua, Haydee Anne B. Acua,
Marites T. Clarion, Marissa C. Enriquez, Graciela M. Torralba, and Mary Pamela A.
Santiago filed a complaint for illegal dismissal against petitioners before the Philippine
Overseas Employment Administration (POEA)
- Respondents claim that: between the months of March and April 1993, they were
deployed by petitioner GSMSC to work as croupiers (card dealers) for petitioner Ferry
Casinos Limited under a six-month contract with monthly salaries of US$356.45 plus fixed
overtime pay of US$107 a month and vacation leave pay equivalent to two months salary
pro rata, except for respondent Jennifer Anne B. Acua who had a monthly salary of
US$250.56 plus fixed overtime pay of US$87.17 and vacation leave pay equivalent to two
months salary pro rata; sometime in July 1993, Sue Smits, the Casino Manager, informed
them that their services were no longer needed; considering that their plane tickets were
already ready and they were subjected to harassment, they had no alternative but to sign
documents on July 11 and 12, 1993 specifying that they were the ones who terminated
their employment; they were repatriated on July 25, 1993.
- Petitioners denied the allegations of respondents and averred that respondents
voluntarily resigned from employment.
- On October 5, 1995, the POEA decided the case against petitioners
- The POEA ruled that the respondents were illegally dismissed since petitioners failed to
prove that respondents voluntarily resigned from employment. It held that the alleged
resignation letters are only declarations of release and quitclaim.
- Petitioners appealed to the NLRC
- NLRC set aside the decision of the POEA and dismissed the complaint for illegal
dismissal. The NLRC held that the contested letters are not only declarations of release
and quitclaim but resignations as well. It further held that there is no concrete evidence of
undue pressure, force and duress in the execution of the resignation letters. The NLRC
gave credence to petitioners claim that respondents pre-terminated their contracts en
masse because two of the respondents, Haydee Anne B. Acua and Marites T. Clarion,
are now working in Singapore.
- Respondents filed a motion for reconsideration but the NLRC denied
- On July 18, 1997, respondents filed a petition for certiorari before us, docketed as G.R.
No. 129673.
- On October 3, 1997, petitioners prayed for outright dismissal of the petition for: (a) failure
of respondents to submit a verified statement of the material dates to show that the
petition was filed on time, and (b) filing a certification on non-forum shopping signed only
by their counsel. In addition, petitioners argued that the issues raised are factual and there
is no showing that the NLRC committed grave abuse of discretion.
- On January 27, 1998, the Solicitor General, in lieu of Comment, manifested that he is
unable to sustain the position of the NLRC because the allegation that respondents
voluntarily resigned was not substantially established and respondents non-compliance
with the formal requirements of the petition should be waived since the petition is
meritorious.
- The NLRC filed its own Comment praying for the dismissal of the petition and the
affirmance of its decision with finality. It argued that in reversing the POEA, it focused its
attention on the correct evaluation of the evidence on record which substantially showed
that petitioners did not dismiss respondents but that the latter resigned en masse on July
12, 1993.
- The Court referred the petition to the Court of Appeals which set aside the decision of the
NLRC and reinstated the decision of the POEA.
- On July 21, 1999, petitioners filed a motion for reconsideration but the Court of Appeals
denied it in a Resolution dated September 22, 1999.
ISSUE
WON the Petition for Certiorari filed by respondents should have been denied outright
HELD
NO
- Petitioners maintain that the petition for certiorari should have merited outright dismissal
for non-compliance with the mandatory requirements of the rules. There is no statement
indicating the material dates when the decision of the NLRC was received and when the

Labor Law 1
motion for reconsideration was filed. Likewise, the certification on non-forum shopping was
not signed by respondents but by their counsel. In any event, petitioners insist that
respondents voluntarily resigned from their employment.
- Section 3 of Rule 46 of the Rules of Court provides that there are three material dates
that must be stated in a petition for certiorari brought under Rule 65: (a) the date when
notice of the judgment or final order or resolution was received, (b) the date when a
motion for new trial or for reconsideration when one such was filed, and, (c) the date when
notice of the denial thereof was received. This requirement is for the purpose of
determining the timeliness of the petition, since the perfection of an appeal in the manner
and within the period prescribed by law is jurisdictional and failure to perfect an appeal as
required by law renders the judgment final and executory.
- As a general rule, these requirements are mandatory, meaning, non-compliance
therewith is a sufficient ground for the dismissal of the petition.
- In the case before us, the failure to comply with the rule on a statement of material dates
in the petition may be excused since the dates are evident from the records. A thorough
scrutiny of the records reveals that the January 15, 1997 decision of the NLRC was
received by respondents counsel on January 24, 1997. On February 19, 1997,
respondents filed a motion for reconsideration 25 which was denied by the NLRC in a
Resolution dated April 30, 1997. 26 Respondents counsel received the resolution on May
30, 1997 and they filed the petition for certiorari on July 18, 1997.
- In view of the retroactive application of procedural laws, Section 4, Rule 65 of the 1997
Rules of Procedure, is the governing provision. It provides that when a motion for
reconsideration is timely filed, the 60-day period for filing a petition for certiorari shall be
counted from notice of the denial of said motion. While respondents motion for
reconsideration was filed 16 days late, the NLRC nonetheless acted thereon and denied it
on the basis of lack of merit. In resolving the merits of the motion despite being filed out of
time, the NLRC undoubtedly recognized that it is not strictly bound by the technicalities of
law and procedure. Thus, the 60-day period for filing of a petition for certiorari should be
reckoned from the date of the receipt of the resolution denying the motion for
reconsideration, i.e., May 30, 1997, and thus, the filing made on July 18, 1997 was well
within the 60-day reglementary period.
- As regards the verification signed only by respondents counsel, this procedural lapse
could have warranted the outright dismissal of respondents petition for certiorari before
the Court of Appeals. However, it must be remembered that the rules on forum shopping,
which were precisely designed to promote and facilitate the orderly administration of
justice, should not be interpreted with such absolute literalness as to subvert its own
ultimate and legitimate objective which is the goal of all rules of procedure - that is, to
achieve substantial justice as expeditiously as possible.
- Needless to stress, rules of procedure are merely tools designed to facilitate the
attainment of justice. They were conceived and promulgated to effectively aid the court in
the dispensation of justice. Courts are not slaves to or robots of technical rules, shorn of
judicial discretion. In rendering justice, courts have always been, as they ought to be,
conscientiously guided by the norm that on the balance, technicalities take a backseat
against substantive rights, and not the other way around. Thus, if the application of the
Rules would tend to frustrate rather than promote justice, it is always within our power to
suspend the rules or except a particular case from its operation.
- As the Court eloquently stated in the case of Aguam vs. Court of Appeals It is a far better
and more prudent course of action for the court to excuse a technical lapse and afford the
parties a review of the case on appeal to attain the ends of justice rather than dispose of
the case on technicality and cause a grave injustice to the parties, giving a false
impression of speedy disposal of cases while actually resulting in more delay, if not a
miscarriage of justice.
- Indeed, where a decision may be made to rest on informed judgment rather than rigid
rules, the equities of the case must be accorded their due weight because labor
determinations should not only be secundum rationem but also secundum caritatem.
- In this case, the Court of Appeals aptly found compelling reasons to disregard
respondents procedural lapses in order to obviate a patent injustice.
Disposition Petition Denied

A2010

- 296 -

Disini

and purchasing officer.


- On April 8, 1993, respondent spouses and their children went on a world tour and
entrusted the business operations to Cabatulan.
- On May 11, 1993, an altercation ensued between petitioner and a certain Isidro Alaan
(security aide) regarding the purchase of spare parts. At about 1:30pm of the same day,
petitioner was asked to meet Vincent Cosmiano (Julios brother). Before meeting Vincent,
Cabatulan passed by the premises of J.C. Trucking but was refused admission by Alaan
who was armed with an armalite rifle. Vincent Cosmiano advised Cabatulan not to report
for work in the meantime and await the arrival of respondent spouses. Cabatulan agreed.
- On May 25, 1993, respondent spouses arrived. Petitioner was summoned to their
residence and was informed that his services were no longer needed.
- On June 11, 1993 petitioner filed a complaint for illegall dismissal, underpayment of
wages, unpaid wages, proportionate 13 th month pay and attorneys fees against
respondent spouses.
- LA found for petitioner and granted 126K backwages; 35.7K separation pay; 1,750
unpaid wages; 4,985.18 13th month pay, 100K moral damages and 26,843.52 attorneys
fees totaling P295,278.70.
- On May 28, 1997, NLRC affirmed with the modification that monetary awards reduced to
P168,435.18, deleted moral damages and attorneys fees reduced. Petitioner didnt file
MFR, spouses did. NLRC granted.
- On Dec 12, 1997, NLRC deleted backwages and accrued salaries and awarded
separation pay and indemnity in lieu thereof. Petitioner filed MFR to reinstate decision of
May 28, 1997.
- On March 14, 2000, CA granted the petition and reinstated May 28, 1997 decision.
Petitioner filed a motion for partial reconsideration praying that he be awarded all the
monetary awards and/or benefits under Art. 279 of the Labor Code.
- On Jan 15, 2001, CA denied the motion.
ISSUE
WON petitioner can avail of all the remedies in Art. 279

HELD

NATURE
Certiorari on decision and resolution of CA reinstating the May 28, 1997 resolution of
NLRC.

YES
Ratio Where an ironhanded application of the rules will result in an unmistakable failure or
miscarriage of justice, technicalities should be transgressed in order to resolve the case.
(Fulgencio v NLRC)
Reasoning
- We agree with the ruling of the CA that as a rule, a party who did not appeal from a
decision of a court cannot obtain affirmative relief other than that granted in the appealed
decision. But it is absurd for the NLRC, and for the CA for that matter, to rule that the
petitioner was dismissed without a lawful or valid cause and yet declare that he is not
entitled to monetary benefits as provided by law.
Obiter
- Under the existing law, an employee who is unjustly dismissed from work shall be
entitled to reinstatement without loss of seniority rights. It must be emphasized, though,
that the Court has declared that there are specific circumstances obtaining where
reinstatement is not a practicable remedy, as when the relations between the employer
and the employee have been so severely strained that it is no longer fitting to order
reinstatement or when the employee decides not to be reinstated. It must be stressed that
the petitioner was charged by the respondent spouses with qualified theft and was even
coerced into withdrawing the labor case against them. No other conclusion may be
deduced other than the categorical fact that antagonism already caused a severe strain in
the relationship between respondent spouses and petitioner. Separation pay is the amount
that an employee receives at the time of his severance from the service and is designed to
provide the employee with the wherewithal during the period that he is seeking another
employment. The grant of separation pay does not impede an award for backwages as
the latter represents the amount of earning lost by reason of unjustified dismissal . A more
equitable settlement, therefore, would be an award of separation pay equivalent to at least
one month pay for every year of service in addition to his full backwages, allowances and
other benefits.
Dispositive the petition is PARTIALLY GRANTED. The Decision of the Court of Appeals,
affirming the Resolution of the NLRC dated May 28, 1997, is AFFIRMED WITH
MODIFICATIONS. The respondent spouses Julio and Cecilia Cosmiano are ORDERED to
pay to the petitioner separation pay equivalent to one months salary for every year of
service in lieu of reinstatement, plus full backwages, without deduction or qualification,
counted fro the date of his dismissal until the finality of this decision, and indemnity of
P2,000.00. No costs.

FACTS
- Respondent spouses Julio and Cecilia Cosmiano were engaged in the trucking business
under the name J.C. Trucking. They rented out heavy equipment such as dump trucks,
bucket loaders, fork loaders and log haulers. They hired petitioner as operations manager

TRIAD SECURITY & ALLIED SERVICES INC V


ORTEGA
481 SCRA 591

CABATULAN V BUAT

451 SCRA 234


CALLEJO SR; February 14, 2005

Labor Law 1
CHICO-NAZARIO; February 6, 2006
NATURE
Petition seeking to set aside CA decision which denied their petition and lifted a TRO
earlier issued
FACTS
- Petitioner Triad Security and Allied services is a security agency owned by Anthony Que.
- Respondents were formerly employed by Triad Security as security guards
- In March, 1999, respondents filed a complaint against petitioners and a certain Gen.
Carbonell for underpayment/nonpayment of salaries, overtime pay, premium pay for
holiday and rest day, service incentive leave pay, holiday pay, and attorneys fees. The
complaint was amended on 20 April 1999 to include the charges of illegal dismissal, illegal
deductions, underpayment/nonpayment of allowance, separation pay, and claims for 13 th
month pay, moral and exemplary damages as well as night shift differential.
- According to respondents: during the time that they were in the employ of petitioners,
compensation was below the minimum wage; they were also made to render services
everyday for 12 hours but were not paid the requisite overtime pay, nightshift differential,
and holiday pay. Petitioners failed to provide them with weekly rest period, service
incentive leave pay, and 13th month pay. So respondents filed a complaint before the
Labor Standards Enforcement Division of the Department of Labor on 6 January 1999.
- Upon learning of the complaint, respondents services were terminated without the
benefit of notice and hearing.
- Petitioners denied respondents claim of illegal dismissal. Petitioners explained that
management policies dictate that the security guards be rotated to different assignments
to avoid fraternization; that they be required to take refresher courses at their
headquarters. Respondents allegedly refused to comply with these policies and instead
went on leave or refused to report at headquarters. Petitioners claim that respondents
worked for only eight hours a day, six days a week and that they received their premium
pays for services rendered during holidays and rest day. The service incentive leave of
respondents was allegedly made payable as soon as respondents applied for said benefit.
- Labor Arbiter Jose G. de Vera found in favor of respondents, ordered the Triad to
reinstate the guards, pay backwages, separation pay in the event reinstatement is not
feasible
- On 18 September 2000, petitioners filed a motion to recompute money claims as
decreed arguing therein that respondents money claims as contained in the 28 February
2000 decision were baseless and that their former counsel was not furnished a copy of
the computation nor was he allowed to submit comments thereon.
- On 20 May 2003, petitioners filed before the Court of Appeals a petition for certiorari with
prayer for the issuance of a temporary restraining order and/or writ of preliminary
injunction.
- In a resolution promulgated on 12 June 2003, the Court of Appeals issued a temporary
restraining order enjoining the execution or enforcement of the 23 April 2003 order of the
labor arbiter.
- Petitioners victory with the Court of Appeals was, however, short-lived. In the decision
now assailed before us, the Court of Appeals ruled that backwages payable to
respondents should be computed from the date of their termination from their jobs until
actual reinstatement as provided in Article 223 of the Labor Code. As petitioners failed to
observe said pertinent provision of the law, the labor arbiter could not be charged with
having committed a grave abuse of discretion when he issued the assailed 23 April 2003
order.
- Moreover, the Court of Appeals took note of the procedural but fatal flaw committed by
petitioners when they immediately elevated their case via petition for certiorari before the
Court of Appeals without first seeking recourse from the NLRC in violation not only of the
Rules of Procedure of said body but also of the doctrine of exhaustion of administrative
remedies.
- Petitioners motion for reconsideration was denied by the Court of Appeals in a resolution
dated 20 November 2003.
ISSUES
1. WON CA erred when it declared that the remedy adopted by the petitioners is
erroneous
2. WON Petitioners should be held liable to the additional amount as contained in the
April 23, 2003 order considering that the February 28, 2000 decision has already been
fully satisfied
HELD
1. NO
Ratio It is a basic tenet of procedural rules that for a special civil action for a petition for
certiorari to prosper, the following requisites must concur: (1) the writ is directed against a
tribunal, a board or an officer exercising judicial or quasi-judicial functions; (2) such
tribunal, board or officer has acted without or in excess of jurisdiction, or with grave abuse
of discretion amounting to lack or excess of jurisdiction; and (3) there is no appeal or any
plain, speedy and adequate remedy in the ordinary course of law.

A2010

- 297 -

Disini

- NLRC has the authority to rule on matters involving grave abuse of discretion that may be
committed by a labor arbiter (ART. 223. APPEAL Decisions, awards, or orders of the
Labor Arbiter are final and executory unless appealed to the Commission by any or both
parties within ten (10) calendar days from receipt of such decisions, awards, or orders.
Such appeal may be entertained only on any of the following grounds:
(a) If there is prima facie evidence of abuse of discretion on the part of the Labor Arbiter.
Reasoning
- Petitioners CLAIM (based on a limited understanding of ART 223): that based on the
rules of procedure of the NLRC, the order granting the issuance of the 2 nd alias writ of
execution could not have been the proper subject of an appeal before the NLRC neither
could petitioners have sought the remedy of certiorari from the NLRC. Petitioners argue
that the rules of procedure of the NLRC do not provide for any remedy or procedure for
challenging the order granting a writ of execution; hence, the pertinent provision of the
Revised Rules of Court should apply which in this case is Section 1 of Rule 41. It states:
Section 1. Subject of appeal An appeal may be taken from a judgment or final
order that completely disposes of the case, or of a particular matter therein when
declared by these Rules to be appealable.
- No appeal may be taken from:
(f)
An order of execution;
- In all the above instances where the judgment or final order is not appealable, the
aggrieved party may file an appropriate special civil action under Rule 65.
- Moreover, Rule III, Section 4 of the Rules of Procedure of the NLRC expressly proscribes
the filing of a petition for certiorari SECTION 4. PROHIBITED PLEADINGS &
MOTIONS. The following pleadings, motions or petitions shall not be allowed in the cases
covered by these Rules:
c) Petition for Certiorari, Mandamus or Prohibition.Therefore, inasmuch as the NLRC
had no authority to issue the writ of certiorari, recourse to the Court of Appeals was only
proper.
- Petitioners also maintain that the doctrine of exhaustion of administrative remedies is not
absolute as it accepts of certain exceptions such as when an appeal would not be an
adequate remedy there being an order or execution already issued and the implementation
of said writ loomed as a great probability.

2. YES
Reasoning
- As the law now stands, an illegally
dismissed employee is entitled to two
reliefs,
namely:
backwages
and
reinstatement. These are separate and
distinct from each other. However,
separation pay is granted where
reinstatement is no longer feasible
because of strained relations between
the employee and the employer. In
effect, an illegally dismissed employee
is entitled to either reinstatement, if
viable,
or
separation
pay
if
reinstatement is no longer viable and
backwages.
- Backwages and separation pay are,
therefore, distinct reliefs granted to
one who was illegally dismissed from
employment. The award of one does
not preclude that of the other as this
court had, in proper cases, ordered the
payment of both.
expenses and risks."
Disposition Court AFFIRMS the Decision of the Court of Appeals dated 31 July 2003 and
the Order dated 23 April 2003 of the Labor Arbiter declaring petitioners liable for additional
accrued backwages. The amount of money claims due the respondents is, however,

Labor Law 1
MODIFIED.

RATIONALE FOR REMEDIES


GLOBE MACKAY V
[PAGE 252]

14.08 REINSTATEMENT
DEFINED
UNION OF SUPERVISORS V SEC OF LABOR
128 SCRA 442
MAKASIAR; March 29, 1984
FACTS
- On November 12, 1981, the Supreme Court rendered a decision in this case finding that
"Luna's discharge was discriminatory and constituted unfair labor practice . . . He is
therefore entitled to reinstatement with back wages ." and accordingly ordered respondent
Republic Bank (now Republic Planters Bank.) ". . . TO IMMEDIATELY REINSTATE
COMPLAINANT NORBERTO LUNA TO HIS FORMER POSITION WITHOUT LOSS OF
SENIORITY RIGHTS AND OTHER BENEFITS AND INCREASES RECOGNIZED BY LAW
OR GRANTED BY PRIVATE RESPONDENT DURING THE PERIOD OF HIS ILLEGAL
DISMISSAL, WITH BACK WAGES EQUIVALENT TO THREE (3) YEARS WITHOUT
QUALIFICATION . . ."
- SC even ordered that: "THIS DECISION IS HEREBY MADE IMMEDIATELY
EXECUTORY."
- On November 17, 1981, upon receipt of a copy of the said decision, complainant Luna
presented himself to the Republic Planters Bank for reinstatement to his former position as
San Juan Branch Manager. The Republic Planters Bank refused to take him back,
claiming that the decision was still under study by the counsel of the said bank.
- On December 1, 1981, the Republic Planters Bank sought leave to file a motion for
reconsideration and/or clarification.
- On January 15, 1982, the Republic Planters Bank begged leave to forego the filing of a
motion for reconsideration and/or clarification, and in lieu thereof, submit manifestation
and motion.
- On May 25, 1982, the Solicitor General, in behalf of the respondent Secretary of Labor
filed a manifestation and motion wherein it was prayed that public respondent be excused
from filing the corresponding comment referred to in the resolution of this Court dated May
10, 1982 on the ground that "the issues being raised therein only affect the private interest
of said bank and hence it may not be proper for public respondent to make a comment"
- On July 8, 1982, the respondent Republic Planters Bank submitted its sur-rejoinder to
petitioner's rejoinder to private respondent's reply to petitioner's comment. The Republic
Planters Bank in its sur-rejoinder has shown that it has already paid the complainant
Norberto Luna his backwages equivalent to three (3) years without qualification.
ISSUE
WON Luna is entitled to reinstatement in Republic Planters Bank (or WON Republic
Planters Bank should be made to suffer the consequences of the Unfair Labor Practice
committed by Republic Bank)
HELD
YES
- In its generally accepted sense, reinstatement is a restoration to a state from which one
has been removed or separated. It is the return to the position from which he was
removed (San Miguel Brewery, Inc. v. Santos and CIR, 112 Phil. 986) and assuming again
the functions of the office already held (Abeto v. Rodas, 82 Phil. 67).
- Reinstatement pre-supposes that the previous position from which one had been
removed still exists, or that there is an unfilled position more or less of a similar
nature as the one previously occupied by the employee. Here, in the case, there was
no closure of shop notwithstanding that the respondent Republic Bank was almost at the
brink of financial ruin. Despite the widespread restructuring and reorganization following
the substantial change in the corporate structure of the former Republic Bank into the
present Republic Planters Bank, to offset the impending financial collapse, the position
previously held by the complainant Norberto Luna was not abolished, but is now held by

A2010

- 298 -

Disini

the incumbent manager who replaced Luna. Although the position formerly held by the
herein complainant is now held by another, there is every reason for the Republic Planters
Bank to reinstate him because there is an "unfilled position more or less of a similar nature
as the one previously occupied by the employee" (Philippine Engineering Corporation v.
CIR, supra). And, Section 4, Rule 1, Book VI of the implementing Rules and Regulations
of the Labor Code states, to wit:
"An employee who is separated from work without just cause should be reinstated to
his former position, unless such position no longer exists, at the time of his
reinstatement, in which case he shall be given a substantially equivalent position in the
same establishment without loss of seniority rights"
- It must be remembered that the respondent Bank reached the point of almost complete
financial ruin were it not for the rehabilitation progress initiated by the government
necessitating widespread restructuring and reorganization of the Bank. At that time of
economic crisis of the respondent Bank, the herein petitioner was the manager of the San
Juan Branch. It cannot be said that he did not contribute, directly or indirectly, to the
downhill economy of the respondent Bank.
- Clearly, the respondent Bank, after surviving said crisis, and emerging therefrom as
Republic Planters Bank, should be given the best managers. Managers who can
contribute to its struggle to survive. Managers who have not only the capacity and
efficiency but likewise they must also have the "proven" skill that is essential for the
respondent Republic Planters Bank to go on and weather these times of economic
difficulties.
- The Labor Code provision on reinstatement, to wit:
"Art. 280. An employee who is unjustly dismissed from work shall be entitled to
reinstatement without loss of seniority rights and to his backwages computed from the
time his compensation was withheld from him up to the time of his reinstatement."
- Reinstatement is aimed to restore the situation as nearly as possible to status quo ante
the unfair labor practice. This requires that those deprived of a recognized and protected
interest by violations of the law should be made whole so as to prevent the violator from
profiting from his misdeeds Yet the reinstatement remedy must always be adapted to
economic-business conditions.
- There is no question that the Republic Planters Bank is the successor of the Republic
Bank. But, in order to keep the respondent Republic Planters Bank on its feet once again,
it had to undergo innovations to ensure recovery. The respondent Republic Planters Bank
actually changed its methods of operation and this change was motivated by economic
factors.
- Considering these "economic-business conditions" together with the economic crisis WE
are in now, it is inevitable that these be reflected in the desire for efficient and productive
management. This honest intention can only be effectuated if the complainant Norberto
Luna is reinstated to a substantially equivalent position without loss of seniority rights and
the incumbent manager who is now holding the position formerly held by herein
complainant be allowed to continue with his "tested" competence and integrity in the
management of the San Juan Branch of the Republic Planters Bank.
Disposition immediately reinstate the complainant norberto luna to an equivalent position
without loss of seniority rights from the time of his illegal dismissal up to actual
reinstatement; and Not to make any deductions from the three years backwages already
paid to norberto luna. This resolution is hereby made immediately executory.

PHESCHEM INDUSTRIAL CORP V MOLDEZ


[PAGE 293]

EMPLOYEE RIGHT
QUIJANO V MERCURY DRUG
292 SCRA 109
PUNO; July 8, 1998
FACTS
- Petitioner DANDY QUIJANO was a warehouseman at the central warehouse of
MERCURY DRUG CORPORATION in Libis, Quezon City, since 1983. During his 8-year
stay in the company, he received high performance ratings and a corresponding 15%
increase in salary per annum. Through the years, the company has also recognized and
commanded him for his dedication to his work. He has actively articulated the employees'
concerns and, since 1990, has written to the management about the malpractices
committed by some officers of the company. He exposed the existence of a "five-six" loan
system in their workplace operated by some of its officers. He incurred the ire of the
manager, Antonio Altavano, who operated the usurious transactions.
- Then followed the harassment of Quijano. In April 1991, Mercury Drug charged him with
4 violations of company policies, all allegedly committed on March 19, 1991. It started at
about 11 am when he allegedly left his workplace without permission. He was charged
with loafing and abandonment of work. Then, between 11:30 am to 12:30 pm, he allegedly
entered the warehouse employees' locker room and angrily uttered in a loud voice:

Labor Law 1
"Niloloko tayo ng kalbong yan.", referring to the warehouse manager, Altavano. He was
charged with disrespect to his superiors. Thirty minutes later, at about 1:00 pm, he
allegedly grabbed the public address system at the central warehouse without permission
and angrily announced: "Wala kay Mrs. Azcona ang incentive natin, na kay Mr.
Concepcion. Niloloko lang tayo (ng superiors natin)." He was charged with disrupting the
work of his co-employees. Finally, after an hour and a half, at about 2:30 pm, he allegedly
saw Mr. Simon peeping through a rack divider, and shouted: "Anong tinitingin-tingin mo?"
He was charged with using abusive language in company premises.
- Consequently, in April 1991, 4 notices of corrective/disciplinary action were served on
him. These were the very first disciplinary sanctions imposed on petitioner in his 8 years of
service and all were allegedly committed on the same day, March 19, 1991.
- Quijano gave a different version of the incidents. He alleged that on said date, he had
been following-up the payment of incentives due to his co-employees. Altavano, informed
him that the incentives were already in the office of Mrs. Vivian Azcona. However, when he
inquired from Azcona about their incentives, she referred him to the office of Mr.
Concepcion and asked him to inform his co-employees. Petitioner did as he was told. He
used the microphone for a few minutes and informed his co-employees about the status of
their incentives. His co-employees submitted a joint written statement confirming his
allegations. They further declared that his brief use of the microphone did not distract them
in the performance of their work. Quijano also denied uttering rude or insulting language in
referring to or communicating with his superiors. Again his co-employees submitted
statements to corroborate his denial. Finally, he claimed that the charges against him were
merely concocted by the warehouse manager and the supervisor in retaliation to his
exposure of the latter's usurious loan scheme in the warehouse, thereby taking undue
advantage of the plight of his co-employees. Quijano was cleared of the four charges.
- However, his employment woes did not end there. He was served another notice of
corrective action for serious misconduct for allegedly challenging his superior to a fistfight
and uttering death threats to the manager, Altavano, on April 25, 1991, or about 7 months
earlier. The next day, a Special Investigating Committee, found him guilty not only of
challenging his superior to a fistfight and issuing death threats to the manager, but also
guilty of the 4 charges of misbehavior earlier hurled against him. On November 19, 1991,
Mercury Drug sent him a notice of termination of employment. The dismissal was to take
effect the next day, November 20, 1991.
- The labor arbiter ruled that Quijano was illegally dismissed from service for lack of just
cause. The NLRC affirmed. However, it modified the labor arbiter's decision by: (1) limiting
the award of backwages to three years; (2) deleting the award of moral and exemplary
damages; and (3) ordering respondent to pay petitioner separation pay in lieu of
reinstatement. Acting on the MFR, the NLRC modified but only as to the period of
computation of back wages. It refused to reinstate petitioner in view of the brewing
antagonism between him and his supervisor and awarded him separation pay instead.

ISSUE
WON the NLRC committed grave abuse of discretion when it awarded separation pay in
lieu of reinstatement

HELD
YES
Ratio The doctrine of "strained relations" should be strictly applied so as not to deprive an
illegally dismissed employee of his right to reinstatement. Every labor dispute almost
always results in "strained relations", and the phrase cannot be given an overarching
interpretation, otherwise, an unjustly dismissed employee can never be reinstated.
Reasoning
- An illegally dismissed employee is entitled to reinstatement as a matter of right. Where
reinstatement is not feasible, expedient or practical, as where reinstatement would only
exacerbate the tension and strained relations between the parties, or where the
relationship between the employer and employee has been unduly strained by reason of
their irreconcilable differences, particularly where the illegally dismissed employee held a
managerial or key position in the company, it would be more prudent to order payment of
separation pay instead of reinstatement. Unscrupulous employers, however, have taken
advantage of the overgrowth of this doctrine of "strained relations" by using it as a cover to
get rid of its employees and thus defeat their right to job security. - Mercury Drugs
charges of misbehavior against Quijano cannot serve as basis to justify his dismissal, let
alone his non-reinstatement. These charges had been found to be baseless and both the
labor arbiter and the NLRC agreed that there was no just cause for petitioner's dismissal.
It can even be granted in arguendo that a certain antagonism may characterize the
relationship of petitioner and the respondents. However, the antagonism was caused
substantially if not solely by the misdeeds of respondent's superiors. The arbiter found as
a fact that the false charges were filed against Quijano by two of his superiors to punish
him for exposing their usurious loan operations. Hence, to deny his reinstatement due to

A2010

- 299 -

Disini

the "strained relations" with his accusers whose charges were found to be false would
result in rewarding the accusers and penalizing the victim. This would set a bad precedent
for no employer should be allowed to profit from his own misdeed. In addition, it is most
inequitable to rule that the antagonism engendered by Quijanos performance of his legal
right to expose the usurious lending operations of some warehouse officers will cause him
to lose the security of his job. The expose is work related and is intended to protect the
economic welfare of employees, and hence its exercise cannot be visited by any
punishment especially by the supreme penalty of separation from service. Again, it bears
emphasis that the State guarantees a worker security of tenure which can well be his most
precious economic right. Thus, all efforts must be exerted to protect him from unjust
deprivation of his job.
- The alleged antagonism is a mere conclusion bereft of evidentiary support. Mercury Drug
did not raise the defense of strained relationship before the labor arbiter. Consequently,
this issue which is factual in nature was not the subject of evidence on the part of both the
petitioner and the respondent. There is thus no competent evidence upon which to base
the conclusion that the relationship between the petitioner and the respondent has
reached the point where it is now best to sever their employment relationship. The NLRC's
ruling on the alleged brewing antagonism between the petitioner and the respondent is a
mere guesswork and cannot justify the non-reinstatement of petitioner to his job.
Disposition Petition is GRANTED. MERCURY DRUG is ordered: (1) to reinstate
QUIJANO to his former or substantially equivalent position; (2) to pay back wages from
the time of his illegal dismissal until his reinstatement (3) to pay moral and exemplary
damages in the amount of P50,000 and P25,000, respectively, and; (4) to pay 10% of the
total amount due as attorney's fees.

ROSARIO V VICTORY RICEMILL


397 SCRA 760
CALLEJO; February 19, 2003
NATURE
Petition for review on certiorari seeking to reverse CA decision
FACTS
- Emilio Uy owned Victory Ricemill. He employed Rosario as truck driver. Petitioner was
tasked to, among others, haul palay from various points and bring them to respondents
ricemill. In addition, petitioner acted as personal driver to the family of Mr. Uy during their
trips to Manila.
- Uy dismissed Rosario. According to respondent, petitioner was guilty of insubordination
when he refused to serve as driver of Mr. Uys son when the latter needed a driver. Also,
petitioner was instructed to deliver 600 bags of cement to Felix Hardware. Instead,
petitioner delivered the same to one Eduardo Interior, who had not since then paid for
P60k. Because of petitioners tendency to disobey the orders to him, Uy was constrained
to engage the services of another driver. Petitioner resented the new driver and became
uncooperative, disrespectful and quarrelsome. Petitioner, armed with a dagger, fought with
the other driver and inflicted an injury on the latter. Petitioner likewise inflicted injuries on
the head of a co-employee, when he intervened in the fight and tried to pacify petitioner.
- A complaint for illegal dismissal with money claims was filed by petitioner
- Labor arbiter found that there were valid causes for the termination of petitioners
employment.
- The NLRC found that petitioner was denied due process during the proceedings with the
regional labor arbiter as petitioner was not given the opportunity to present his additional
rebuttal evidence. On the other hand, respondent was allowed to submit in evidence
various exhibits to discredit the rebuttal testimony of petitioner. Case was remanded.
- Petitioner submitted the affidavit of Roque, who averred that the 600 bags of cement
delivered to Eduardo Interior had been paid as evidenced by in the sum of P58,950.00
payable to respondent.
- Regional labor arbiter promulgated his decision stating that he found no reason to
deviate from his previous decision.
- NLRC affirmed the ruling of the regional labor arbiter
- CA found that respondent had justifiable cause to dismiss petitioner. CA also observed
that although there was no strict compliance with the two-notice rule, it could be gleaned
from the records that petitioner was given ample opportunity to explain his side. Moreover,
even granting that respondent fell short of the two-notice requirement, such irregularity,
according to the CA, does not militate against the legality of the dismissal.
ISSUES
1. WON petitioners termination was for a just and lawful cause
2. WON petitioners dismissal from his employment was in accordance with the due
process requirement of the law
3. WON petitioner is entitled to backwages
HELD
1. YES

Labor Law 1
- Petitioners actuations clearly constituted willful disobedience and serious misconduct
justifying his dismissal under Article 282(a) of the Labor Code which provides:
Art. 282. Termination by employer. An employer may terminate an employment for
any of the following causes:
(a) Serious misconduct or willful disobedience by the employee of the lawful orders of
his employer or representative in connection with his work;
- Willful disobedience of the employers lawful orders, as a just cause for the dismissal of
an employee, envisages the concurrence of at least two requisites: (1) the employees
assailed conduct must have been willful or intentional, the willfulness being characterized
by a "wrongful and perverse attitude;" and (2) the order violated must have been
reasonable, lawful, made known to the employee and must pertain to the duties which he
had been engaged to discharge.
2. NO
- To effect the dismissal of an employee the law requires not only that there be just and
valid cause as provided under Article 282. It likewise enjoins the employer to afford the
employee the opportunity to be heard and to defend himself. The employer is mandated to
furnish the employee with two written notices: (a) a written notice containing a statement
of the cause for the termination to afford the employee ample opportunity to be heard and
defend himself with the assistance of his representative, if he so desires; (b) if the
employer decides to terminate the services of the employee, the employer must notify him
in writing of the decision to dismiss him, stating clearly the reason therefore
- While respondent furnished petitioner the written notice informing him of his dismissal,
respondent failed to furnish petitioner the written notice apprising him of the charge or
charges against him. Consequently, petitioner was deprived of the opportunity to respond
thereto
- When the dismissal is effected for a just and valid cause, the failure to observe
procedural requirements does not invalidate nor nullify the dismissal of an employee. The
consequence of the failure either of the employer or the employee to live up to this precept
is to make him liable in damages, not to render his act void. The measure of damages is
the amount of wages the employee should have received were it not for the termination of
his employment without prior notice. If warranted, nominal and moral damages may also
be awarded.
3. YES
- Under the Labor Code, only the absence of a just cause for the termination of
employment can make the dismissal of an employee illegal.
Art. 279. Security of Tenure. In cases of regular employment, the employer shall not
terminate the services of an employee except for a just cause or when authorized by
this Title. An employee who is unjustly dismissed from work shall be entitled to
reinstatement without loss of seniority rights and other privileges and to his full
backwages, inclusive of allowances, and to his other benefits or their monetary
equivalent computed from the time his compensation was withheld from him up to the
time of his actual reinstatement.
- Thus, only if the termination of employment is not for any of the causes provided by law
is it illegal and, therefore, the employee should be reinstated and paid backwages.
- On the other hand, if it is shown that the employee was dismissed for any of the just
causes mentioned in said Art. 282, then, in accordance with that article, he should not be
reinstated. However, he must be paid backwages from the time his employment was
terminated until it is determined that the termination of employment is for a just cause
because the failure to hear him before he is dismissed renders the termination of his
employment without legal effect. .
Disposition CA decision AFFIRMED with MODIFICATION. Uy is ordered to pay petitioner
full backwages from the time his employment was terminated up to the time the herein
decision becomes final

EFFECT OF FAILURE TO ASK RELIEF


GENERAL BAPTIST BIBLE COLLEGE V NLRC (BASA)
219 SCRA 549
CAMPOS JR; March 5, 1993
NATURE
Petition for review of a resolution of the NLRC
FACTS
- Basa was hired by the College as the Academic Dean. He was also appointed President
by the Board of Trustees.
- In a regular meeting, the BoT voted to terminate services of Basa as President. Basa,
without aid of counsel, filed complaint for illegal dismissal not only as President but as
Dean.

A2010

- 300 -

Disini

- Arbiter found dismissal as President meritorious and claim for separation pay without
basis. He ordered the College and the General Baptist Church to pay complainant.
- NLRC said reinstatement as Dean and payment of unpaid salaries is in order.
ISSUES
1. WON Basa has right to be reinstated
2. WON Basa may be reinstated notwithstanding fact that prayer is for payment of unpaid
salaries and separation pay only
HELD
1. YES
- Records are bereft of indication that Basas termination as Dean was proven. College
must inform Basa when, why and how his term as Dean expired, since it was understood
to be indefinite.
- If the intention was to terminate Basas services both as President and Dean, Basa must
be heard. The College failed here. The letter terminated his services as President.
- That nobody was appointed as Dean when Basa was appointed President means that
Basa acted concurrently as President and Dean. These are different positions, and there
is no prohibition against holding these positions concurrently.
- Basa should have continued as Dean even after his termination as President. His having
been considered terminated as Dean has no legal basis, though his dismissal as
President may have been legal.
2. YES
- Basas failure to specifically pray for reinstatement is a procedural lapse which cannot
put to naught a right which he is entitled under a substantive law.
- But due to animosity and antagonism, College must not be compelled to reinstate Basa
but it must be given option to give separation pay in lieu thereof.

PHESCHEM INDUSTRIAL V MOLDEZ


[PAGE 293]

RULES ON REINSTATEMENT
RATIONALE
ROQUERO V PHILIPPINE AIRLINES INC
401 SCRA 424
PUNO; April 22, 2003
NATURE
CERTIORARI
FACTS
- Roquero, along with Rene Pabayo, were ground equipment mechanics of PAL.
- From the evidence on record, it appears that Roquero and Pabayo were caught redhanded possessing and using shabu in a raid conducted by PAL security officers and
NARCOM personnel.
- The two alleged that they did not voluntarily indulge in the said act but were instigated by
a certain Jojie Alipato who was introduced to them by Joseph Ocul, Manager of the Airport
Maintenance Division of PAL.
- Alipato was unsuccessful, until one day, he was able to persuade Pabayo to join him in
taking the drugs and who in turn persuaded Roquero.
- Inside the company premises, they locked the door and Alipato lost no time in preparing
the drugs to be used. When they started the procedure of taking the drugs, armed men
entered the room, arrested Roquero and Pabayo and seized the drugs and the
paraphernalia used.
- Roquero and Pabayo were subjected to a physical examination where the results
showed that they were positive of drugs.
- They were also brought to the security office of PAL where they executed written
confessions without the benefit of counsel.
- Roquero and Pabayo received a notice of administrative charge for violating the PAL
Code of Discipline. They were required to answer the charges and were placed under
preventive suspension.
- Roquero and Pabayo asserted that they were instigated by PAL to take the drugs and
that Alipato was not really a trainee of PAL but was placed in the premises to instigate the
commission of the crime because Alipato was not arrested and that Alipato has no record
of employment with PAL.
- Roquero and Pabayo were dismissed by PAL. Thus, they filed a case for illegal
dismissal.
- LA: the dismissal of Roquero and Pabayo was upheld. The Labor Arbiter found both
parties at fault PAL for applying means to entice the complainants into committing the

Labor Law 1
infraction and the complainants for giving in to the temptation and eventually indulging in
the prohibited activity. Nonetheless, the Labor Arbiter awarded separation pay and
attorneys fees to the complainants.
- RTC: acquitted the 2 of the criminal case of conspiracy for possession and use of a
regulated drug in violation of Section 16, Article III of Republic Act 6425 on the ground of
instigation.
- NLRC: ruled in favor of complainants as it likewise found PAL guilty of instigation;
ordered reinstatement to their former positions but without backwages.
- Complainants did not appeal from the decision but filed a motion for a writ of execution of
the order of reinstatement.
- LA granted the motion but PAL refused to execute the said order on the ground that they
have this petition pending which was eventually referred to the CA.
- CA: reversed the decision of the NLRC and reinstated the decision of the LA insofar as it
upheld the dismissal of Roquero.
ISSUES
1. WON the instigated employee shall be solely responsible for an action arising from the
instigation perpetrated by the employer
2. WON the executory nature of the decision, more so the reinstatement aspect of a labor
tribunals order can be halted by a petition having been filed in higher courts without any
restraining order or preliminary injunction having been ordered in the meantime
3. WON the employer who refused to reinstate an employee despite a writ duly issued can
be held liable to pay the salary of the subject employee from the time that he was ordered
reinstated up to the time that the reversed decision was handed down
HELD
1. YES
- Instigation is only a defense against criminal liability. It cannot be used as a shield
against dismissal from employment especially when the position involves the safety of
human lives.
- It is of public knowledge that drugs can damage the mental faculties of the user. Roquero
was tasked with the repair and maintenance of PALs airplanes. He cannot discharge that
duty if he is a drug user. His failure to do his job can mean great loss of lives and
properties. Hence, even if he was instigated to take drugs he has no right to be reinstated
to his position. He took the drugs fully knowing that he was on duty and more so that it is
prohibited by company rules
- There is no question that petitioner Roquero is guilty of serious misconduct for
possessing and using shabu. He violated Chapter 2, Article VII, section 4 of the PAL Code
of Discipline which states:
Any employee who, while on company premises or on duty, takes or is under the
influence of prohibited or controlled drugs, or hallucinogenic substances or narcotics
shall be dismissed.
- Philippine Aeolus Automotive United Corporation vs. NLRC (2000): Serious misconduct
is defined as the transgression of some established and definite rule of action, a
forbidden act, a dereliction of duty, willful in character, and implies wrongful intent and not
mere error in judgment. For serious misconduct to warrant the dismissal of an employee,
it (1) must be serious; (2) must relate to the performance of the employees duty; and (3)
must show that the employee has become unfit to continue working for the employer
- PAL complied with the twin-notice requirement before dismissing the petitioner. The twinnotice rule requires (1) the notice which apprises the employee of the particular acts or
omissions for which his dismissal is being sought along with the opportunity for the
employee to air his side, and (2) the subsequent notice of the employers decision to
dismiss him.
2. NO
- The order of reinstatement is immediately executory
- Philippine Rabbit Bus Lines, Inc. vs. NLRC, (1999): The unjustified refusal of the
employer to reinstate a dismissed employee entitles him to payment of his salaries
effective from the time the employer failed to reinstate him despite the issuance of a writ of
execution. Unless there is a restraining order issued, it is ministerial upon the Labor Arbiter
to implement the order of reinstatement. In the case at bar, no restraining order was
granted. Thus, it was mandatory on PAL to actually reinstate Roquero or reinstate him in
the payroll. Having failed to do so, PAL must pay Roquero the salary he is entitled to, as if
he was reinstated, from the time of the decision of the NLRC until the finality of the
decision of this Court.
- Article 223 (3rd paragraph) of the Labor Code, as amended by Section 12 of RA No.
6715, and Sec 2 of the NLRC Interim Rules on Appeals under RA No. 6715, Amending the
Labor Code, provide that an order of reinstatement by the Labor Arbiter is immediately
executory even pending appeal.
- The rationale of the law has been explained in Aris (Phil.) Inc. vs. NLRC:
In authorizing execution pending appeal of the reinstatement aspect of a decision of
the Labor Arbiter reinstating a dismissed or separated employee, the law itself has laid
down a compassionate policy which, once more, vivifies and enhances the provisions
of the 1987 Constitution on labor and the working man.
xxx
xxx
xxx

A2010

- 301 -

Disini

These duties and responsibilities of the State are imposed not so much to express
sympathy for the workingman as to forcefully and meaningfully underscore labor as a
primary social and economic force, which the Constitution also expressly affirms with
equal intensity. Labor is an indispensable partner for the nations progress and stability.
xxx
xxx
xxx
x x x In short, with respect to decisions reinstating employees, the law itself has
determined a sufficiently overwhelming reason for its execution pending appeal.
xxx
xxx
xxx
x x x Then, by and pursuant to the same power (police power), the State may authorize
an immediate implementation, pending appeal, of a decision reinstating a dismissed or
separated employee since that saving act is designed to stop, although temporarily
since the appeal may be decided in favor of the appellant, a continuing threat or
danger to the survival or even the life of the dismissed or separated employee and his
family.
3. YES.
- Even if the order of reinstatement of the LA is reversed on appeal, it is obligatory on the
part of the employer to reinstate and pay the wages of the dismissed employee during the
period of appeal until reversal by the higher court.
- If the employee has been reinstated during the appeal period and such reinstatement
order is reversed with finality, the employee is not required to reimburse whatever salary
he received for he is entitled to such, more so if he actually rendered services during the
period.
Disposition The dismissal of petitioner Roquero is AFFIRMED, but PAL is ordered to pay
the wages to which Roquero is entitled from the time the reinstatement order was issued
until the finality of this decision.

PNOC-EDP V ABELLA
448 SCRA 549
CHICO-NAZARIO; January 17, 2005
NATURE
Petition for review on certiorari
FACTS
- Private respondent Frederick V. Abella started working with herein petitioner Philippine
National Oil Company - Energy Development Corporation (PNOC-EDC) as a probationary
Security Assistant at its SNGP in Ticala, Valencia, Negros Oriental. Subsequently, he
became a regular employee.
- Less than one year later, Abella was informed that his employment with PNOC-EDC
would be terminated effective 21 May 1990, allegedly due to a company-wide
reorganization pursuant to its Manpower Reduction Program, wherein the position of
Security Assistant at PNOC-EDC SNGP had been abolished.
- Aggrieved, Abella filed a case of illegal dismissal, and for actual, moral, and exemplary
damages with the NLRC, Regional Arbitration Branch No. VII at Dumaguete City, against
the PNOC-EDC and its officers. After hearing the parties, Labor Arbiter Villahermosa
rendered a Decision, holding that Abella was illegally dismissed as the company and its
officers failed to show a clear scheme and convincing proof of reorganization.
- An appeal was timely filed with the NLRC. Meanwhile, with said appeal still pending in
the NLRC, the labor arbiter issued an order, directing the company to admit back to work
or reinstate the complainant under the same terms and conditions prevailing prior to his
dismissal or separation or, at the option of the employer, merely reinstated in the payroll.
- Pursuant to the above order, Abella was reinstated in the payroll as a General Services
Assistant (PAL II), his original position of Security Assistant having been abolished by
virtue of the company-wide reorganization. According to the company, the position is of
the same level as Assistant Security and had the same salary rate and benefits.
- Abella, through counsel, wrote Quevenco, Resident Manager at the SNGP, to protest his
assignment in the payroll as General Services Assistant (PAL II). Subsequently, he was
again re-slotted in the payroll as a Pipeline Maintenance Foreman, which, according to the
petitioners, is another position with the same salary and benefits as another Security
Assistant. This change of position was classified as a lateral transfer.
- Abella wrote petitioner Quevenco, to request that he (Abella) be physically reinstated
and allowed to perform security functions. Said request was granted. Abella was
temporarily detailed as Security Assistant at SNGPs PAL II Development Project,
Northern Cotabato. However, he was also concomitantly designated as Acting Security
Officer for the entire SNGP due to the reassignment of the incumbent Security Officer to
the Northern Negros Geothermal Project of the company. Abella wrote a letter, this time
addressed to Vasquez, then Vice-President of the company, to confirm that he had
assumed his security functions; that he was open for negotiations regarding his case;
and, that he hoped that his appointment/work status would be normalized.
- In a telegraphic message, Tongco informed Abella to immediately report to Mindanao I
Geothermal Plant (MIGP), Kidapawan, North Cotabato. Tongco defined private
respondents duties and functions and delimited the duration of his stay at MIGP as
temporary, or for about 3 months only.

Labor Law 1
- Shortly thereafter, Abella and the company agreed to settle. Abella consequently
received the amount of P124,824.31 as settlement of the said case and by virtue of the
said agreement, both parties filed a Joint Motion to Dismiss before the NLRC.
- Acting on the Joint Motion to Dismiss, the NLRC issued a Resolution, granting the above
motion dismissing the appeal earlier filed before it. At this time, while carrying out security
functions at MIGP, Kidapawan, North Cotabato, Abellas official item or position in the
payroll was Maintenance Foreman, SNGP, Valencia, Negros Oriental. Said state of affairs
prompted the late Susas to write Tongco about it and to recommend that proper action be
made in order to harmonize security-related support services at MIGP.
- Abella filed a motion for the issuance of a writ of execution of the decision of the labor
arbiter. Corollary to the said motion, he informed Quevenco of his intention to report back
to SNGP, Ticala, Valencia, Negros Oriental, his original assignment prior to the filing of the
1991 case or illegal dismissal. Abella received a show cause memorandum for his alleged
absence without official leave (AWOL) and insubordination. Responding to the above,
Abella explained in a letter that his position as SGS Maintenance is in complete
contravention of the decision of the labor arbiter.
- Despite the above response, Abella was nevertheless transferred to PNOC-EDC Leyte-A
Geothermal Project, as a Security Assistant, a position that was vacant at that time. Said
transfer was accompanied by a Transfer or Change of Position Form showing Abellas
change of official position from Pipeline Maintenance Foreman to Security Assistant to be
a lateral transfer.
- Tongco sent Abella a radiogram message instructing him to present himself, this time at
the Mt. Labo Geothermal Project, Camarines Norte, as a Security Assistant. A second
message followed emphasizing the need for Abella to report at the said site not later than
25 May 1994. On 01 June 1994, Abella was once more instructed to report to the
petitioner companys Mt. Labo Geothermal Project. Said order was again accompanied by
a Transfer or Change of Position Form stating the transfer of Abella as a Security Assistant
from Leyte to Mt. Labo to be a lateral transfer.
- All the above-mentioned directives were disregarded or ignored.
- In the intervening time, the labor arbiter ruled on the motion for execution filed by the
complainant by issuing a Writ of Execution directing the Sheriff, NLRC, Cebu City, to
proceed to the premises of the company at Ticala, Valencia, Negros Oriental, to effect and
to cause the reinstatement of Abella either by physical or by payroll reinstatement. Sheriff
Cornelio issued a certification that per attached pay slip, private respondent had been
reinstated in the payroll with PNOC-EDC.
- For failing to heed the directives of his supervisors, Abella received another show
cause memorandum from Tongco, ordering him to explain in writing why no disciplinary
action should be taken against him for insubordination and for being AWOL.
- Abella, in his reply countered that he is not guilty of insubordination since he was not
reinstated to his former position as Security Assistant at Ticala, Valencia, Negros Oriental,
per Writ of Execution issued by the labor arbiter.
- Claiming unfair and prejudicial treatment, Abella filed a complaint before the NLRC, SubRegional Arbitration Branch No. VII, Dumaguete City, for unfair labor practice, illegal
suspension, nonpayment of mid-year bonus and 13th month pay for 1990 and 1991, claim
for hazard pay, and annual salary increase against the company and its officers.
- Several months later, Abella received a notice of disciplinary action of Grave Suspension
with Final Warning against him. Abella filed another complaint with the NLRC, against the
company and its officers, for unfair labor practice, illegal suspension, and nonpayment of
wages with damages. Nevertheless, Abella continued working at SNGP, Ticala, Valencia,
Negros Oriental, until he was accordingly notified of his termination for cause. Thereafter,
he filed a third complaint with the NLRC against the company and its officers, this time for
unfair labor practice, illegal dismissal, and nonpayment of wages, with prayer for
reinstatement and payment of moral and exemplary damages as well as attorneys fees
docketed.
- After hearing the parties, Labor Arbiter Villahermosa rendered a consolidated
Decision. In maintaining that Abella was not illegally dismissed, the labor arbiter opined
that the records of the case show that Abella was reassigned from his position in
Ticala, Valencia, Negros Oriental, to that in Cotabato province by virtue of a
memorandum issued by Tongco which Abella readily accepted and agreed to said
transfer, therefore there is no valid basis for the claim that he was not validly
reinstated. Thus, the charges of insubordination and AWOL committed by Abella fall
squarely within the provision of Rule 26 of the petitioner companys rules and
regulations as contained in the PNOC Rules and Regulations on Discipline. Said rules
provide for a penalty ranging up to dismissal even for the first offense.
- On appeal, the NLRC reversed and set aside the Decision of the labor arbiter and
entered a new one. The NLRC found that Abella was illegally dismissed considering that
at bar, the parties had reached a settlement without vacating the decision (of the labor
arbiter dated 27 August 1991), then the decision should be given its full force and effect,
and as the [r]ecords show that he was never reinstated to his former position as admitted
by the correspondence of J.T. Susas dated 25 March 1993, memorandum of complainant
dated 17 September 1993 and letter of complainants counsel to Engr. Quevenco, dated
03 January 1994. The assignments of the complainants (sic) to the various positions
could not equate to full enforcement of the decision of 27 August 1991 considering that
these positions were not his former position and his assumption to these positions were
under protest.

A2010

- 302 -

Disini

- There being a timely motion for reconsideration, the Honorable Commission, in a


Resolution, reversed itself insofar as the order for reinstatement and computation of
backwages were concerned. Instead, the Commission held that since Abella had already
reached the retirement age of 60 years, reinstatement would no longer be possible. Abella
should be given all the benefits due him under the retirement provision of the collective
bargaining agreement of the company.
- With the denial of their motion for reconsideration, the company and its officers came to
the Court of Appeals via a petition for certiorari under Rule 65 of the Revised Rules of
Court and sought to nullify the above-stated NLRC Decision and Resolution.
- The appellate court dismissed the petition for lack of merit. The company and its officers
motion for reconsideration having been denied, the instant petition was filed.
ISSUE
WON the reinstatement of private respondent was valid
HELD
YES
- The issue of reinstatement is addressed by paragraph three of Article 223 of the Labor
Code, to wit:
ART. 223. Appeal . . . .
In any event, the decision of the Labor Arbiter reinstating a dismissed or separated
employee, insofar as the reinstatement aspect is concerned, shall immediately be
executory, even pending appeal. The employee shall either be admitted back to work
under the same terms and conditions prevailing prior to his dismissal or separation or,
at the option of the employer, merely reinstated in the payroll. The posting of a bond by
the employer shall not stay the execution for reinstatement provided herein.
- The above-stated provision of the Labor Code, however, must be read in conjunction
with the implementing rules and regulations of the said law. Sec. 4(a) of Rule 1, Book VI of
the Rules and Regulations Implementing the Labor Code, provides that:
SEC. 4. Reinstatement to former position. (a) An employee who is separated from
work without just cause shall be reinstated to his former position, unless such position
no longer exists at the time of his reinstatement, in which case he shall be given a
substantially equivalent position in the same establishment without loss of seniority
rights. [Emphasis supplied.]
- Reinstatement presupposes that the previous position from which one had been
removed still exists, or that there is an unfilled position more or less of a similar nature as
this previously occupied by the employee.
- Accordingly, an employee who is separated from his employment on a false or
nonexistent cause is entitled to be reinstated to his former position because the separation
is illegal. If the position is no longer available for any other valid and justifiable reason,
however, the reinstatement of the illegally dismissed employee to his former position
would neither be fair nor just. The law itself can not exact compliance with what is
impossible. Ad imposible tenetur. The employers remedy is to reinstate the employee to
a substantially equivalent position without loss of seniority rights as provided for above.
- In the case at bar, strictly applying the rules provided above, private respondent Abella
should have been reinstated back to his old position as a Security Assistant at the SNGP,
Ticala, Valencia, Negros Oriental. Or, at the very least, since the position of Security
Assistant at Ticala, Valencia, Negros Oriental, had been abolished as claimed by the
petitioners, he should have been reinstated to another position that is substantially
equivalent to his former one. In reality, private respondent Abella was first reinstated in
the payroll, as a General Services Assistant and subsequently, as a Pipeline Foreman,
while he was actually discharging the functions of a Security Assistant. As insisted by the
petitioners, this situation was due to the fact that the original position of the private
respondent had already been abolished in the previous company-wide reorganization in
1991.
- But then, the private respondent was reslotted as Security Assistant when he was
transferred to the Leyte Geothermal Project. He was, thus, performing the functions of a
Security Assistant and at the same time occupying the official position of a Security
Assistant though in a geographically different location, when said position became vacant.
- Be that as it may, notwithstanding the above disquisitions, the atypical circumstances in
this case capitulate against the outright application of the said rules. Whether or not the
private respondent was validly reinstated per Order of the Labor Arbiter dated 27 August
1991, is beside the point in view of the fact that the Joint Motion to Dismiss filed by the
parties in the earlier case contained a clause whereby the parties agreed that [a]ll other
claims, damages and causes of action arising out of the instant case are waived.
- Regrettably, the Court of Appeals and the NLRC have overlooked this very important
fact.
- The clause agreed to by the parties in the Joint Motion to Dismiss filed before the NLRC
was in the nature of a compromise agreement, i.e., an agreement between two or more
persons, who for preventing or putting an end to a lawsuit, adjust their difficulties by
mutual consent in the manner which they agree on, and which everyone of them prefers to
the hope of gaining, balanced by the danger of losing. Settlement of disputes by way of
compromise, is an accepted, nay desirable and encouraged practice in courts of law and
administrative tribunals. Generally favored in law, such agreement is a bilateral act or
transaction that is binding on the contracting parties and is expressly acknowledged by the
Civil Code as a juridical agreement between them.

Labor Law 1
- Prevailing case law provides that a compromise once approved by final orders of the
court has the force of res judicata between the parties and should not be disturbed except
for vices of consent or forgery. Hence, a decision on a compromise agreement is final and
executory. Such agreement has the force of law and is conclusive on the parties. It
transcends its identity as a mere contract binding only upon the parties thereto, as it
becomes a judgment that is subject to execution in accordance with the Rules. Judges
therefore have the ministerial and mandatory duty to implement and enforce it.
(Underlining supplied.) Hence, compromise agreements duly approved by the courts are
considered the decisions in the particular cases they involve.
- In the case at bar, when both parties agreed to waive all other claims, damages and
causes of action, a compromise they entered into in good faith absent any allegation
otherwise, they did not only agree to dismiss the appeal pending before the NLRC.
Particularly, the private respondent also agreed to receive P124,824.31, thus,
relinquishing his claim to the Decision dated 27 August 1991, rendered by the labor arbiter
in his favor. In return, the petitioner company, to put an end to the labor dispute,
acquiesced to have its appeal before the NLRC dismissed.
- The waiver was approved and considered by the NLRC when it promulgated its Order
dated 22 February 1993, dismissing the appeal of the petitioners. Conformably, to cite
jurisprudence, the Compromise Agreement approved by the proper authority became the
decision in this particular case. Settlements of this kind not only are recognized to be
proper agreements but so encouraged as well.
- Undoubtedly, the allegations of invalid reinstatement on the part of the petitioners are a
mere afterthought on private respondents part in a fascinating attempt to extricate himself
from an assignment that brought him to a far away place and caused him to be separated
from his family.
Disposition Petition granted

EXCEPTIONS
BUSINESS CONDITIONS
UNION OF SUPERVISORS V SEC OF LABOR
[PAGE]
ESPEJO V NLRC (COOP INSURANCE SYSTEM OF THE
PHILS)
255 SCRA 430
BELLOSILLO; March 29, 1996
FACTS
- Espejo was GM of Cooperative Insurance System of the Philippines (CISP). Aside from
salary, he had certain privileges such as company car with driver.
- They had to raise capital so to satisfy the requirement of the Insurance Commission. The
CISP Board of Directors resolved to sell the company car that Espejo was using to help
raise the money. Espejo didnt like this idea and tendered his resignation. He even said
that if the Board really sells his car, his resignation would be irrevocable.
- The Board did resolve to sell his car; but Espejo said he wont resign anymore. But they
dismissed him anyway.
- He filed with the LA for illegal dismissal. LA ordered reinstatement and payment of
backwages. NLRC ordered payment of backwages, without reinstatement. This was
because by the time the case reached the NLRC, Espejo was already 60 years old, so the
NLRC said he cant be reinstated anymore.
ISSUE
WON (60+ yr old) Espejo can be reinstated
HELD
NO
- The law recognizes as valid any retirement plan, agreement or management policy
regarding retirement at an earlier or older age.
- In the case of petitioner, CISP did not have any retirement plan for its employees. In such
situation, Sec. 13, Book IV, of the Omnibus Rules Implementing the Labor Code provides
that in the absence of a retirement plan, agreement or policy an employee may be retired
upon reaching the age of sixty (60) years. Construing this provision, an employee may
retire, or may be retired by his employer, upon reaching sixty (60). Thus, an employee
held to be illegally dismissed cannot be reinstated if he had already reached the age of
sixty (60) years at the time of his complaint. NLRC therefore did not err in denying the
reinstatement of petitioner.
- Generally, an illegally dismissed employee who cannot be reinstated is granted
separation pay and back wages. However considering that petitioner has already reached

A2010

- 303 -

Disini

the statutory retirement age of sixty (60), NLRC is correct that petitioner is entitled only to
back wages. The payment of back wages is a form of relief that restores the income lost
by reason of the unlawful dismissal; separation pay, in contrast, is oriented towards the
immediate future, the transitional period the dismissed employee must undergo before
locating a replacement job.
Disposition Petition denied. Pay backwages from termination until he turned 60 yrs old.

STRAINED RELATIONS
PEARL S. BUCK FOUNDATION INC V NLRC
[PAGE 235]
COMMERCIAL MOTORS CORP V NLRC (UMLAS)
192 SCRA 191
NARVASA; December 10, 1990
FACTS
- CMC is engaged in the sale, marketing and distribution of Mercedes Benz vehicles and
spare parts, as well as in the servicing, repair and maintenance of said vehicles. It has
branches in various parts of the country, including one in Davao. Pedro Umlas had been
working with CMC, first as Spare Parts Checker at the Manila Office; later, as Warehousein-Charge in the Davao Branch; and still later, Spare Parts Supervisor in the same Branch.
Then there was the discovery of the missing parts valued at around 200,000. Umanlas
was thereafter dismissed. The LA ruled against Umanlas but the NLRC reversed.
ISSUES
1. WON NLRC committed grave abuse of discretion
2. WON reinstatement is the appropriate relief
HELD
1. NO
- The dismissal was not based on substantial facts. There wasnt even due process in
terminating petitioner
2. NO
- It would seem, however, that the circumstances of this case render inapproriate Umlas'
reinstatement to his former position, as an item of relief. A more equitable disposition is
that which this Court has more than once made in other cases of the same nature: the
award, in lieu of reinstatement, of separation pay at the rate of one month's salary for
every year of service, "so that . . . (the employee) can be spared the agony of having to
work anew with . . . (the employer) under an atmosphere of antipathy and antagonism,
and the . . . (latter) does not have to endure the continued service of . . . (the former) in
whom it has lost confidence."
Disposition NLRC decision AFFIRMED.

SENTINEL SECURITY AGENCY INC V NLRC


[PAGE 140]
SIBAL V NOTRE DAME OF GREATER MANILA
182 SCRA 538
PARAS; February 23, 1990
NATURE
Petition for certiorari seeking to reverse a decision of the NLRC
FACTS
- Petitioner Delia R. Sibal was employed as school nurse by private respondent Notre
Dame of Greater Manila starting January 1973.
- She was compensated on a 12-month basis, although she worked only during the tenmonth period of classes.
- On March 10, 1976, respondent's director, Fr. Gonzales, requested her to shorten her
summer vacation, from two weeks after the last day of classes to two weeks before the
first day of classes of the next school year. Petitioner acceded to the request
- In April 1980, Fr. Gonzales required petitioner to report during that summer to help in the
library.

Labor Law 1
- Petitioner contested the order, stating that it will necessitate a change in the terms and
conditions of her employment and that library work is alien to her profession as nurse. Fr.
Gonzales relented.
- Fr. Gonzales was replaced by Fr. Pablo Garcia, an American, as new director.
- Fr. Garcia required petitioner to report for work during the summer before the beginning
of school year 1981-1982.
- Petitioner informed him that her contract does not require her to report for work during
the summer vacation.
- Fr. Garcia promised to verify her allegation. However, he failed to inform petitioner of his
findings. Thus, in order that her failure to report for work may not be misinterpreted,
petitioner filed leaves of absence extending from April 1, 1981 to June 14, 1981. Petitioner
failed to receive her vacation pay.
- During school year 1981-1982, petitioner was assigned to teach health subjects to 900
students (19 sections), without being compensated
- In December 1981, petitioner received her 13th month pay computed on the basis of a
10-month period only.
- On April 5, 1982, Fr. Garcia again required petitioner to work during that summer to
update all the clinical records of the students.
- Petitioner objected and reiterated that her contract does not require her to report for work
during summer. She also reminded Fr. Garcia that she had not received any
compensation for teaching health subjects
- Fr. Garcia replied that it was imperative for her to report for work during the summer
because it is the best time to update the clinical records, and that petitioner was not
entitled to extra compensation for teaching because teaching was allegedly part of her
regular working program as a school nurse.
- Petitioner, apart from reiterating her objection to the order, called the attention of Fr.
Garcia to the school's failure to pay her salary for the summer of 1981 and of the
deficiency in her 13th month pay for that year.
- Fr. Garcia adamantly refused to consider petitioner's demands and threatened to take
drastic measures against her if she remains obstinate in her refusal to follow his order to
report for work that summer
- Petitioner, for the fourth time, informed Fr. Garcia that her contract does not require her
to report for work during summer, and she does not intend to do so that summer of 1982.
- Petitioner filed a complaint for non-payment of the following; (1) vacation pay for four (4)
summer months; (2) compensation for teaching health subjects; and (3) deficiency in the
13th month pay for 1981.
- Summons was served on respondent on the opening day of classes. That very day,
respondent school served petitioner her letter of termination effective immediately and it
also submitted a copy of the termination paper to the Ministry of Labor and Employment.
- The following day, petitioner filed an amended complaint, adding two more charges:
illegal dismissal and unfair labor practice
- For the next four to five weeks, more than 20 teachers and personnel, backed up by the
Faculty Association of respondent school, pressed for the ouster of Fr. Garcia with the
Ministry of Education, Culture, and Sports (MECS) by virtue of PD 176 and the following
charges: oppressive behavior, arrogance, contempt for Filipinos in general and Filipino
teachers in particular; unfairness in dealing with personnel; dictatorial conduct; and use of
abusive Fr. Garcia was eventually replaced on September 8, 1983.
- The Labor Arbiter rendered a decision awarding to petitioner separation pay but denying
her claim (1) for compensation for teaching Health subject to 19 sections; (2) for moral
damages; and (3) negating the existence of unfair labor practice
- Petitioner filed a memorandum of partial appeal
- Petitioner informed the NLRC that Fr. Pablo Garcia had been replaced by Fr. Jose Arong,
a Filipino, as new director effective September 8, 1983.
- On April 11, 1986, public respondent NLRC rendered the questioned decision which
affirmed the decision of the Labor Arbiter
ISSUES
1. WON the award of separation pay instead of reinstatement is the proper remedy under
the circumstances
2. WON petitioner is entitled to compensation for teaching health subjects
3. WON unfair labor practice existed which would entitle petitioner to moral damages
HELD
1. NO
- The LA had found that the termination of petitioner was not supported by any just cause
or reason. Yet, she erroneously ordered separation pay instead of reinstatement with
backwages based on the alleged reason that petitioner's working relations with the former
director, Father Garcia, had become so strained and deteriorated that it became
impossible for them to work harmoniously again. And the NLRC affirmed such finding
which is untrue and merely speculative.
- It should be noted that the alleged conflict between the petitioner and the director was
strictly official in nature, the cause of which was the violation of the terms of employment
by the latter. Petitioner's assertion of her right to unpaid salaries and bonus differential was
not motivated by any personal consideration. Rather, she simply claimed benefits which,

A2010

- 304 -

Disini

under the law, she was entitled to and legally due her. In her act of asserting these money
claims, petitioner observed utmost tact, courtesy and civility.
- Significantly, about a month after petitioner's termination on June 14, 1982, more than
twenty teachers and personnel of respondent school, backed by the Faculty Association,
petitioned for the ouster of Director Fr. Garcia for serious charges under P.D. 176 .
Consequently, Fr. Garcia was replaced on September 8, 1983. Clearly, therefore, when
the assailed NLRC decision was rendered on April 11, 1986, the alleged "strained
relations" or "irritant factors" which the Labor Arbiter capitalized on had been totally
eliminated. Respondent NLRC obviously failed to consider this and thus perpetuated the
error committed by the Labor Arbiter in her prior decision.
- The dissenting NLRC Commissioner aptly observed thus:
- Moreover, it should be emphasized, that no strained relations should arise from a valid
and legal act of asserting ones right, such as in the instant case, for otherwise, an
employee who shall assert his/ her right could be easily separated from the service by
merely paying his/her separation pay on the pretext that his/her relationship with his/her
employer had already become strained.
- To Our mind, strained relations in order that it may justify the award of separation pay in
lieu of reinstatement with backwages, should be such, that they are so compelling and so
serious in character, that the continued employment of an employee is so obnoxious to the
person or business of the employer, and that the continuation of such employment has
become inconsistent with peace and tranquility which is an Ideal atmosphere in every
workplace.
2. YES
- Petitioner is entitled to compensation for teaching health subjects. Although the subject
taught is Health and allied to her profession, and is taught during regular working hours,
petitioner's teaching the subject in the classroom and her administering to the health
needs of students in the clinic involve two different and distinct jobs
- It must be noted that petitioner has established that in several precedents, non-teaching
personnel of respondent school who were made to handle teaching jobs were actually
paid actual compensation. Besides, justice and equity demand that since the principle of
equal work has long been observed in this jurisdiction, then it should follow that an extra
pay for extra work should also be applied.
3. YES
- The records show that when summons with attached complaint of petitioner for money
claims was served on respondent school on June 14, 1982, said respondent, on the very
day, gave petitioner her walking papers. Respondent did not waste any time in dismissing
her in brazen violation of these provisions of the Labor Code
- The series of discriminatory and oppressive acts of respondent school against petitioner
invariably makes respondent liable for moral damages under Art. 1701, which prohibits
acts of capital or labor against each other, and Art. 21 on human relations in relation to Art.
2219 No. 10 and Art. 2220, all of the Civil Code
Disposition The appealed decision of respondent NLRC is set aside. Private respondent
is ordered to REINSTATE petitioner to her former position without loss of seniority rights
and with backwages for three (3) years from the time of her illegal dismissal; to pay her
the regular extra compensation relative to her teaching health subjects; and to pay her
moral damages.

NAGA COLLEGE FOUNDATION EDUCATION


WORKERS ORG V BOSE
289 SCRA 274
MENDOZA; April 20, 1998
NATURE
Special civil action of certiorari
FACTS
- Petitioners were employees of the Naga College Foundation. They filed a complaint with
the Regional Arbitration Branch of the NLRC in Naga City for unfair labor practice,
reinstatement, back wages, and damages against private respondents, alleging illegal
dismissal. On the other hand, private respondents filed a complaint against petitioners for
conducting an illegal strike.
- Executive Labor Arbiter ordered the reinstatement of petitioners and the payment of
back wages to them
- Private respondents appealed. While the appeal was pending, petitioners moved for the
execution of the portion of the decision insofar as it granted reinstatement.
- The parties then entered into a compromise agreement whereby private respondents
agreed to reinstate petitioners in the payroll effective Sept 21, 1992, the date of receipt by
them of the ELAs decision. The agreement was approved by the ELA.
- However, after paying three installments of the accrued salaries, private respondents
failed to make further payments to petitioners. Petitioners asked the ELA for assistance
and, as no action was taken on their request, they filed a motion for execution directly with
the NLRC in Manila.
- Petitioners again moved for the execution of the decision before ELA. Petitioners insisted

Labor Law 1
on execution according to the tenor of the original decision, without need of determining
how much they might have earned during the period of their dismissal.
- ELA denied execution on the ground that by filing a motion for execution in the NLRC in
Manila, petitioners had abandoned their motions in the Legaspi office.
- Petitioners contended that the reinstatement portion of the decision had already been the
subject of a compromise agreement and, therefore, the order deleting the order of
reinstatement and awarding separation pay in lieu thereof in effect amended a final and
executory order. Petitioners claimed that they had been forced to file a motion directly in
the NLRC because of the ELAs failure to act upon their motions, but that they had no
intention at all of abandoning their motion in the ELAs office.
ISSUE
WON petitioners abandoned the agreement when they moved for the execution of the
decision (WON the Executive Labor Arbiter could freely consider supervening events)
HELD
NO
Reasoning
- The Solicitor General contends that by asking for the execution of the decision of the
NLRC, petitioner in effect abandoned or rescinded the compromise agreement, leaving
the ELA free to consider supervening events, such as the strain in relations of the parties,
in the resolution of the motion for execution. There is no basis in the record for supposing
that petitioners gave up their claim for reinstatement. The subject of the compromise
agreement was their reinstatement as ordered in the decision of the ELA. The subject of
the NLRC decision, which they sought to enforce in the motion for execution, was also
their reinstatement. Whichever one it is, no supervening event rendering execution unjust
can be considered:
1) Petitioners did not occupy any managerial or confidential position in the Naga
College Foundation which might be affected by any bad feeling which might have been
engendered as a result of the execution of the decision.
2) It was private respondents who appear to have caused a strain in the relation of the
parties. Any bad feeling was caused by its failure to comply in good faith with their
undertaking under the compromise agreement.
- Principle of Strained Relations: This cannot be applied indiscriminately. Otherwise,
reinstatement can never be possible simply because some hostility is invariably
engendered between the parties as a result of litigation. That is human nature. Besides,
no strained relations should arise from a valid and legal act of asserting ones right;
otherwise an employee who shall assert his right could be easily separated from the
service, by merely paying his separation pay on the pretext that his relationship with his
employer had already become strained. (Globe-Mackay Cable and Radio Corp. v. NLRC)
3) Petitioners were frustrated at the undue delay in the resolution of their motions by the
ELA and they thought of turning to the NLRC in Manila in the hope of obtaining assistance.
Abandonment was far from their intention.
Disposition Petition is granted, and the orders of ELA are hereby annulled and set aside.
Private respondents are hereby ordered to reinstate petitioners to their former positions
without loss of seniority and to pay them full backwages, subject however to any
agreement the parties may have entered into.

BASCON V CA
[PAGE 215]
CABATULAN V BUAT
[PAGE 294]
ACESITE CORP V NLRC
CARPIO-MORALES; January 26, 2005
NATURE
Two consolidated petitions for review on certiorari challenging CA decision
FACTS
- Leo Gonzales was hired as Chief of Security of Holiday Inn Manila. On March 25, 1998,
Gonzales took a 4-day sick leave and took emergency leave on March 30, 1998. On April
16-29, 1998, he again took a 12-day vacation leave. Before the expiration of his 12-day
vacation leave, Gonzales filed an application for emergency leave for 10 days. The
application was not approved. He received a telegram from Acesite advising him that he
was on unauthorized leave and asking him to provide a written explanation within the next
24 hours why he was not reporting for work and was required to report for work the
following day. On May 2, 1998, Gonzales father Anacleto sent a telegram to Acesite
stating that he was still recovering from severe stomach disorder and would report back
for work on May 4, 1998, attaching a medical certificate.

A2010

- 305 -

Disini

- On May 4, 1998, Gonzales reported for work and presented himself to Johann
Angerbauer, then Resident Manager of the hotel. He requested for leave without pay from
May 5-9, 1998 which was provisionally approved on condition that he (Gonzales) would be
sending his explanation re his absences that same day through e-mail, which he did. In
the evening Gonzales left for Abra.
- Also on May 4, 1998 Angerbauer sent an inter-office memo to Gonzales, stating that his
presence was needed at a meeting scheduled on the following day. Gonzales claims that
he got hold of a copy of the above-quoted memo only on May 8, 1998.
- Not having reported for work, Angerbauer sent Gozales a telegram on May 5, 1998
stating that Gozales should report for work upon receipt of notice due to very urgent
matters. Gonzales, who claims to have received the telegram only in the afternoon of May
7, 1998, immediately went back to Manila on May 8, 1998 only to be humiliatingly and
ignominiously barred by the guard from entering the premises. It appears that on May 7,
1998, Angerbauer issued a Notice of Termination through an inter-office memo.
- Gonzales thus filed a complaint against Acesite, Angerbauer and Kennedy for illegal
dismissal with prayer for reinstatement and payment of full backwages, service incentive
leave, 13th month pay, moral and exemplary damages and attorneys fees. Gonzales,
however, failed to appear in 2 consecutive hearings despite notice, meriting the dismissal
by the Labor Arbiter of his complaint.
- Gonzales refiled his complaint for illegal dismissal. The Labor Arbiter dismissed the
complaint for lack of merit. The NLRC reversed the decision. The CA affirmed the NLRC
decision.
ISSUES
1. WON Gonzales was illegally dismissed
2. WON Gonzales should be reinstated
HELD
1. YES
- There appears to have been no just cause to dismiss Gonzales from employment. As
correctly ruled by the Court of Appeals, Gonzales cannot be considered to have willfully
disobeyed his employer. Willful disobedience entails the concurrence of at least 2
requisites: the employees assailed conduct has been willful or intentional, the willfulness
being characterized by a wrongful and perverse attitude; and the order violated must
have been reasonable, lawful, made known to the employee and must pertain to the
duties which he had been engaged to discharge.
- In Gonzales case, his assailed conduct has not been shown to have been characterized
by a perverse attitude, hence, the first requisite is wanting. His receipt of the telegram
disapproving his application for emergency leave starting April 30, 1998 has not been
shown. And it cannot be said that he disobeyed the May 5, 1998 telegram since he
received it only on May 7, 1998. On the contrary, that he immediately went back to Manila
upon receipt thereof negates a perverse attitude.
2. NO
- In illegal dismissal cases, reinstatement to an illegally dismissed employees former
position may be excused on the ground of strained relations. This may be invoked
against employees whose positions demand trust and confidence, or whose differences
with their employer are of such nature or degree as to preclude reinstatement. In the case
at bar, Gonzales was Chief of Security, whose duty was to manage the operation of the
security areas of the hotel to provide and ensure the safety and security of the hotel
guests, visitors, management, staff and their properties according to company policies and
local laws. It cannot be gainsaid that Gonzales position is one of trust and confidence, he
being in charge of the over-all security of said hotel. Thus, reinstatement is no longer
possible. In lieu thereof, Acesite is liable to pay separation pay of 1 month for every year
of service.

BPI EMPLOYEES UNION V BPI


454 SCRA 357
CHICO-NAZARIO; March 31, 2005
NATURE
Petition for review on certiorari under Rule 45, ROC, seeks to partially reverse the
Decision and Resolution of the Court of Appeals which affirmed with modification the
Decision rendered by the Accredited Voluntary Arbitrator dated 31 December 1997, in VA
Case No. 08-001-97.
FACTS
- October 1995: Because Zenaida Uy shouted at her Senior Manager Delfin Santos
(Santos), she was told to report to AVP Carlos Fragante (AVP), who later told her to
transfer from the Escolta Branch to Plaza Cervantes Branch to defuse the tense situation
in their branch. The next day, AVP received the report of Santos regarding the incident and
so he ordered Uy to transfer. Uy resisted, awaiting the result of the grievance meeting
initiated by the BPI Union against BPI, where no agreement was reached.

Labor Law 1
- BPI initially sent 2 letters (before and after the grievance meeting) to Uy ordering her to
explain why no disciplinary action should be taken against her for insubordination, then
later for uttering disrespectful, discourteous, insulting and unbecoming language to her
superior, Santos. In her first reply, she explained that there was no proper turnover of her
accountabilities and she could not enter the bank premises to do so. On the second reply,
Uy reiterated her first reply and she requested to be considered on leave starting
November. BPI wrote another letter directing her to show cause, now also for going
AWOL. Now Uy asked for particulars regarding the alleged highly disrespectful,
discourteous, insulting, threatening, and unbecoming language and behavior towards her
manager and her alleged past quarrels with her co-employees, and alleged that she was
sexually harassed by Santos49. Union asked for investigation on the sexual harassment
charge.
- Late November, 2 meetings were held between the union and the management where no
agreement was reached, and the management advised Union and Uy that they would
terminate Uy, which they later did.
- Uy filed case for illegal transfer and termination.
- LA: Uy was illegally dismissed, reinstatement with full backwages and 10% attorneys
fees BPI appealed to NLRC
- NLRC: set aside LAs Decision for lack of jurisdiction, case under Voluntary Arbitrator
(VA)
- VA: Uy illegally dismissed, reinstatement without loss of seniority rights and with full
backwages from time she was dismissed until she is actually reinstated, including other
benefits under CBA, plus 10% attorneys fees MFR of BPI denied, filed petition for
review in CA
- CA: affirmed deci with modification NO REINSTATEMENT because of strained
relations, instead pay back salaries not exceeding 3 years and separation pay of one
month for every year of service.
- so here, partial reconsideration on both side, but BPIs petition denied because of
technicality so only entertain BPI Unions Petition for review.
ISSUES
1. WON decision to limit award of backwages to 3 years contrary to law and jurisprudence
2. WON UY should be reinstated
HELD
1. YES
- The Mercury Drug Rule used by CA already abandoned with the promulgation of RA
6715 in 1989, amending Article 279, LC
Ratio. Absent any exceptional circumstance, it is now settled that an employee who is
unjustly dismissed from work shall be entitled to full backwages, inclusive of allowances,
and to his other benefits or their monetary equivalent from the time his compensation was
withheld from him up to the time of his actual reinstatement.
Reasoning
- Mercury Drug Rule [Mercury Drug, Co., Inc. vs. CIR]: an employee whose illegal
termination had lasted some years was entitled to backwages for a fixed period without
further qualifications, i.e., without need of taking account of whatever he might have
earned during such period, and deducting it from the amount of recovery, by providing a
base period of three years.
- In the case, the illegal dismissal of Uy took effect on 1995 when RA 6715 already was in
effect.
- Verily, the evident legislative intent as expressed in Rep. Act No. 6715 is that the
backwages to be awarded to an illegally dismissed employee, should not, as a general
rule, be diminished or reduced by the earnings derived by him elsewhere during the period
of his illegal dismissal. The underlying reason for this ruling is that the employee, while
litigating the legality (illegality) of his dismissal, must still earn a living to support himself
and his family. Corollary thereto, full backwages have to be paid by the employer as part
of the price or penalty he has to pay for illegally dismissing his employee. Thus, a closer
adherence to the legislative policy behind Rep. Act. No. 6715 points to full backwages as
meaning exactly that, i.e., without deducting from backwages the earnings derived
elsewhere by the concerned employee during the period of his illegal dismissal. In other
words, the provision calling for full backwages to illegally dismissed employees is clear,
plain and free from ambiguity and, therefore, must be applied without attempted or
strained interpretation
- Consequently, in accordance with Section 34, Rep. Act No. 6715, employees illegally
dismissed after 21 March 1989 are entitled to their full backwages, inclusive of other
benefits or their monetary equivalent, from the time their actual compensation was
withheld from them up to the time of their actual reinstatement.
2. YES
- Mere allegation of strained relations to bar reinstatement is frowned upon. Besides, the
members of the management involved in the case are no longer in the Escolta branch so
no more reason for strained relations.
Ratio. The strained relations doctrine should be strictly applied so as not to deprive an
illegally dismissed employee of his right to reinstatement.
49 Santos allegedly told Uy Dito ka na lang, marami and [ang] lalaki dito, and when she answered Hindi ako mahilig sa lalaki, he retorted, Maski dito ka na lang sa kuwarto ko

A2010

- 306 -

Disini

- Well-entrenched is the rule that an illegally dismissed employee is entitled to


reinstatement as a matter of right. Over the years, however, the case law developed that
where reinstatement is not feasible, expedient or practical, as where reinstatement would
only exacerbate the tension and strained relations between the parties, or where the
relationship between the employer and employee has been unduly strained by reason of
their irreconcilable differences, particularly where the illegally dismissed employee held a
managerial or key position in the company, it would be more prudent to order payment of
separation pay instead of reinstatement. Some unscrupulous employers, however, have
taken advantage of the overgrowth of this doctrine of strained relations by using it as a
cover to get rid of its employees and thus defeat their right to job security.
-To protect labors security of tenure, we emphasize that the doctrine of strained relations
should be strictly applied so as not to deprive an illegally dismissed employee of his right
to reinstatement. Every labor dispute almost always results in strained relations and the
phrase cannot be given an overarching interpretation, otherwise, an unjustly dismissed
employee can never be reinstated.
- This Court is cognizant of managements right to select the people who will manage its
business as well as its right to dismiss them. However, this right cannot be abused. Its
exercise must always be tempered with compassion and understanding.
-Where penalty less severe would suffice, whatever missteps may be committed by labor
ought not to be visited with consequence so severe. It is not only because of the laws
concern for the workingmen. There is, in addition, his family to consider. Unemployment
brings untold hardships and sorrows on those dependent on the wage-earner. The misery
and pain attendant on the loss of jobs then could be avoided if there be acceptance of the
view that under all the circumstances of a case, the workers should not be deprived of
their means of livelihood. Nor is this to condone what has been done by them.
Disposition the instant petition is GRANTED. The assailed 28 October 1998 Decision
and 8 March 1999 Resolution of the Court of Appeals are hereby MODIFIED as follows: 1)
respondent BPI is DIRECTED to pay petitioner Uy backwages from the time of her illegal
dismissal until her actual reinstatement; and 2) respondent BPI is ORDERED to reinstate
petitioner Uy to her former position, or to a substantially equivalent one, without loss of
seniority right and other benefits attendant to the position.

SAGUM V CA (INST OF INTEGRATED ELECTRICAL


ENGINEERS)
459 SCRA 223
PUNO; May 26, 2005
FACTS
- petitioner Sagum was appointed as Officer Manager and OIC for the Executive Director
in concurrent capacity as Executive Secretary of respondent institute. She was a member
of the permanent staff for 16 years and, as executive secretary, served 11 National
Presidents.
- Private respondents allege that Sagum abused her power in consistently awarding
printing jobs to DBR Prints, despite having the lowest bids. Petitioner was preventively
suspended for 30 days and served 2 written notices demanding explanations for the
imputed offenses to which she denied the charges. She was later dismissed for gross
negligence and loss of trust and confidence.
- The Labor Arbiter ruled the dismissal was illegal, ordering the payment of backwages
and separation pay. Petitioner argued that the decision did not show a case of irretrievable
estrangement between her and private respondents, thus seeking reinstatement. NLRC
reversed the decision upon appeal by respondents. Her MFR denied, petitioner filed a
petition for review with the CA which again reversed the decision but still denying
reinstatement due to strained relations.
ISSUE
WON the CA erred in its factual finding of strained relations without citing specific
evidence and denying reinstatement thus contradicting Article 279
HELD
YES
- Article 279 and Sec. 2 of the Omnibus Rules Implementing the Labor Code both provide
that a regular employee may not be terminated without just cause. Article 279 and Sec. 3
of the Omnibus further that an employee unjustly dismissed is entitled to reinstatement.
However, it has been held that where reinstatement would only exacerbate tension and
strained relations between the parties by reason of irreconcilable differences, especially
where the employee held a managerial or key position, it would be more prudent to order
payment of separation pay instead of reinstatement.
- Some employers have abused the doctrine of strained relations to defeat their
employees security of tenure. To protect the latter, the doctrine must be strictly applied and
cannot be given an overarching interpretation. Since almost every labor dispute results in
some strain, a liberal interpretation of the doctrine would mean an unjustly dismissed
employee could never be reinstated.

Labor Law 1

A2010

- The existence of strained relations is a factual finding and should be initially raised,
argued and proven before the Labor Arbiter. In the instant case, the defense was not
raised by respondents before the Labor Arbiter and was not subject of the evidence raised
by either party. There is thus no evidentiary support to the parties strained relations and
NLRCs ruling on the alleged antagonism between them was mere guesswork. There is no
hard evidence to prove that the parties relationship has reached the point where it is best
to sever their employment relationship.
Disposition the assailed Decision of the CA is AFFIRMED, with the MODIFICATION that
petitioner is entitled to REINSTATEMENT

IMPLEMENTATION
RATIONALE

OPTIONS

AND

OPTIONS AND RATIONALE


JARDINE DAVIES V NLRC (SALUTIN)
225 SCRA 757
VITUG; August 31, 1983
NATURE
Petition for review on certiorari resolution of NLRC (declaring Salutin not having
abandoned his work)
FACTS
- Salutin was employed by Jardine Davies as a demonstrator/ agronomist to provide
services WRT to the promotion and use of JDs pesticides and other products.
- This case involves 2 special civil actions for certiorari (instant case is the latter one). The
issue began when Salutin filed a complaint fro illegal dismissal, which the LA ruled in his
favor, ordering JD to reinstate him and pay backwages. JD appealed to the NLRC, and
reinstated Salutin on payroll only, in compliance with the write of execution issued by the
LA. Appeal to NLRC was dismissed.
- JD then filed with the SC the 1st petition for certiorari, assailing the NLRC denial of its
appeal. This was dismissed for failure to comply with the SC Circular on forum shopping.
However, while the appeal to the NLRC was still pending, and shortly after the
reinstatement of Salutin on payroll only, JD directed Salutin to report for work to their
Bacolod Branch Manager. He in fact reported to work, but did not stay long, and was
reported never to have returned to work. JD stopped further payment of wages.
- JD then filed with the NLRC a Manifestation and Motion, stating that Salutin should be
considered as having abandoned his work considering his continuous absence of more
than 3 weeks. JD also contended that Salutin has been gainfully employed in another
company already. Salutin opposed motion, claiming that he was forced to leave because
he was then suffering from a serious ailment (peptic ulcer), and presented a medical
certificate to support his claim. He in turn filed his own motion (that JD be ordered to
release his withheld salary), which the NLRC granted. From this decision the instant
petition for certiorari was filed.
ISSUE
WON Salutin should be considered as having abandoned his work when he failed to
report for work pending JDs appeal, considering that at the time he was on reinstatement
on payroll
HELD
NO
Ratio For abandonment to constitute a valid cause for termination of employment there
must be a deliberate unjustified refusal of the employee to resume his employment.
Reasoning
- When JD filed its first petition for certiorari, it also raised, as an added argument on the
alleged abandonment of work by Salutin, the fact that he was gainfully employed
elsewhere. Considering that this matter was thus already taken up by JD in its first petition
for certiorari, which this Court dismissed with finality, JD should really now be barred from
invoking anew that issue in this present (second) petition.
- JDs basis for the declaration of abandonment of work is the unauthorized absences of
Salutin.
- For abandonment to constitute a valid cause for termination of employment, there must
be a deliberate unjustified refusal of the employee to resume his employment. This refusal
must be clearly shown. Mere absence is not sufficient; it must be accompanied by overt
acts pointing to the fact that the employee does not want to work anymore.

- 307 -

Disini

- Abandonment of position is a matter of intention expressed in dearly certain and


unequivocal acts. In this instance, however, uncontroverted facts show just exactly the
opposite. Hence, Salutin did report, as directed, on 24 September 1991, but that he could
not stay long because he was ailing at that time; he, although perhaps belatedly made, did
seek medical consultation for "peptic ulcer; and later on, he did, in fact, manifest his
desire to assume his work with JD.
- The order of immediate reinstatement pending appeal, in cases of illegal dismissal is an
ancillary relief under R.A. 671550 granted to a dismissed employee to cushion him and his
family against the impact of economic dislocation or abrupt loss of earnings. If the
employee chooses not to report for work pending resolution of the case on appeal, he
foregoes such a temporary relief and is not paid of his salary.
** underlined portion really obiter, but most relevant to our topic.
Disposition Petition dismissed. NLRC resolutions AFFIRMED.

PIONEER TEXTURIZING CORP V NLRC (PTWU & DE


JESUS)
280 SCRA 806
FRANCISCO; October 16, 1997
FACTS
- Lourdes A. de Jesus is Pioneers reviser/trimmer since 1980 and as such she based her
assigned work on a paper note posted by petitioners. The posted paper which contains
the corresponding price for the work to be accomplished by a worker is identified by its
P.O. Number. On August 15, 1992, de Jesus worked on P.O. No. 3853 by trimming the
cloths' ribs. She thereafter submitted tickets corresponding to the work done to her
supervisor. Three days later, de Jesus received from petitioners' personnel manager a
memorandum requiring her to explain why no disciplinary action should be taken against
her for dishonesty and tampering of official records and documents with the intention of
cheating as P.O. No. 3853 allegedly required no trimming. The memorandum also placed
her under preventive suspension for thirty days starting from August 19, 1992, In her
handwritten explanation, de Jesus maintained that she merely committed a mistake in
trimming P.O. No. 3853 as it has the same style and design as P.O. No. 3824 which has
an attached price list for trimming the ribs and admitted that she may have been negligent
in presuming that the same work was to be done with P.O. No. 3853, but not for
dishonesty or tampering. Petitioners' personnel department, nonetheless, terminated her
from employment and sent her a notice of termination dated September 18, 1992.
- De Jesus filed a complaint for illegal dismissal against petitioners. The Labor Arbiter who
heard the case noted that de Jesus was amply accorded procedural due process in her
termination from service. Nevertheless, after observing that de Jesus made some further
trimming on P.O. No. 3853 and that her dismissal was not Justified, the Labor Arbiter held
petitioners guilty of illegal dismissal and ordered her reinstatement without loss of seniority
rights and with full backwages from the time of her suspension.
- Petitioners appealed to the public respondent National Labor Relations Commission
(NLRC). The NLRC declared that the status quo between them Should be maintained and
affirmed the Labor Arbiter's order of reinstatement, but without backwages.
- The NLRC further "directed petitioner to pay de Jesus her back salaries from the date
she filed her motion for execution on September 21, 1993 up to the date of the
promulgation of [the] decision."
- Petitioners filed their motion for reconsideration which the NLRC denied, hence this
petition anchored substantially on the alleged NLRC's error in holding that de Jesus is
entitled to reinstatement and back salaries.
- Petitioners' theory is that an order for reinstatement is not self-executory and that there
must be a writ of execution which may be issued by the NLRC or by the Labor Arbiter
motu proprio or on motion of an interested party. And that even if a writ of execution was
issued, their timely appeal coupled by the posting of appropriate supersedes a bond,
effectively forestalled and stayed execution of the reinstatement order of the Labor Arbiter.
ISSUE
WON an order for reinstatement needs a writ of execution
HELD
NO. An award or order for reinstatement is selfexecutory.
- Article 223 of the Labor Code, as amended by R.A. No. 67 15 which took effect on March
2 1, 1989, pertinently provides:
223. Appeal. - Decisions, awards. or orders of the Labor Arbiter are final and executory
unless appealed to the Commission by any or both parties within ten (10) calendar
days from receipt of such decisions, awards, or orders. Such appeal maybe entertained
only on any of' the following grounds:
"In any event, the decision of' the Labor Arbiter reinstating a dismissed or separated
employee insofar as the reinstatement aspect v. concerned. shall Immediately be
executory, even pending appeal. The employee shall either he admitted back to work
50

No mention as to what this RA does, but it amends certain articles of the LC to extend protection to labor, strengthen
constitutional rights of workers, foster inductrial peace and harmony, xxx and reorganize the NLRC.

Labor Law 1
under the same terms and conditions prevailing prior to his dismissal or separation or
at the option of' the employer, merely reinstated in the payroll. The posting of a bond by
the employer shall not stay the execution for reinstatement provided herein.
- In Zamboanga City Water District v. Buat, the Court construed Article 223 to mean
exactly what it says. We said:
"Under the said provision of law, the decision of the Labor Arbiter reinstating a
dismissed or separated employee insofar as the reinstatement aspect is concerned,
shall be immediately executory, even pending appeal. The employer shall reinstate the
employee concerned either by: (a) actually admitting him back to work under the same
terms and conditions prevailing prior to his dismissal or separation; or (b) at the option
of the employer, merely reinstating him in the payroll. Immediate reinstatement is
mandated and is not stayed by the fact that the employer has appealed, or has posted
a cash or surety bond pending appeal."
- We note that prior to the enactment of R.A. No. 67 15, Article 223 24 of the Labor Code
contains no provision dealing with the reinstatement of' an illegally dismissed employee.
The amendment introduced by R.A. No. 67 15 is an innovation and a far departure from
the old law indicating thereby the legislature's unequivocal intent to insert a new rule that
will govern the reinstatement aspect of a decision or resolution in any given labor dispute,
In fact, the law as now worded employs the phrase "shall immediately be executory"
without qualification emphasizing the need for prompt compliance. As a rule, "shall" in a
statute commonly denotes an imperative obligation and is inconsistent with the idea of
discretion and that the presumption is that the word "shall," when used in a statute, is
mandatory. An appeal or posting of bond, by plain mandate of the law, could not even
forestall nor stay the executory nature of an order of reinstatement. The law, moreover, is
unambiguous and clear, Thus, it must be applied according to its plain and obvious
meaning, according to its express terms.
- And in conformity with the executory nature of the reinstatement order, Rule V, Section
16 (3) of the New Rules of Procedure of the NLRC strictly requires the Labor Arbiter to
direct the employer to immediately reinstate the dismissed employee.
- Thus:"In case the decision includes an order of' reinstatement the Labor Arbiter shall
direct the employer to immediately reinstate the dismissed or separated employee even
pending appeal. The order of reinstatement shall indicate that the employee shall either be
admitted back to work under the same terms and conditions prevailing prior to his
dismissal or separation or, at the option of the employer, merely reinstated in the payroll.
- A closer examination, however, shows that the necessity for a writ of execution under
Article 224 applies only to final and executory decisions which are not within the coverage
of Article 223.
- Article 224 states that the need for a writ of execution applies only within five (5)years
from the date a decision, an order or award becomes final and executory. It can not relate
to an award or order of reinstatement still to be appealed or pending appeal which Article
223 contemplates. The provision of Article 223 is clear that an award for reinstatement
shall be immediately executory even pending appeal and the posting of a bond by the
employer shall not stay the execution for reinstatement.
- The legislative intent is quite obvious, i.e., to make an award of reinstatement
immediately enforceable, even pending appeal. To require the application for and issuance
of a writ of execution as prerequisites for the execution of a reinstatement award would
certainly betray and run counter to the very object and intent of Article 223, i. e., the
immediate execution of a reinstatement order. The reason is simple. An application for a
writ of execution and its issuance Could be delayed for numerous reasons. A mere
continuance or postponement of a scheduled hearing, for instance, or an inaction on the
part of the Labor Arbiter or the NLRC could easily delay the issuance of the writ thereby
setting at naught the strict mandate and noble put-pose envisioned by Article 223. In other
words, if the requirements of Article 224 were to govern, as we so declared in Maranaw,
then the executory nature of a reinstatement order or award contemplated by Article 223
will be unduly circumscribed and rendered ineffectual.

INTERNATIONAL CONTAINER SERVICES V NLRC


(TANPEINGCO)
300 SCRA 335
BELLOSILLO; December 21, 1998
NATURE
Special civil action for certiorari for the modification of the decision of the NLRC deleting
therefrom the portion which orders to pay the private respondent Tanpiengco backwages
despite valid dismissal.
FACTS
- Petitioner ICTSI is a corporation engaged in stevedoring, operating the Manila
International Container Terminal (MICT). It employed private respondent Tanpiengco as a
CFS Priority pursuant to a CBA with private respondent's labor union, the Associated Ports
Checkers Services and Workers Union (APCSWU). Tanpiengco has since then become
a regular employee assigned either to a shipping line or bodega to strip and examine

A2010

- 308 -

Disini

cargoes from abroad with usual working schedule from 8:00 a.m. to 5:00 p.m. although in
actual practice cleaning time would start at 4:30 p.m.
- On 7 March 1990 Tanpiengco was assigned at Bodega I. When it was time for him to
clean himself he took his T-shirt which was hanging from a post, tucked it at his waist and
proceeded to the washroom. He was accosted by a security guard allegedly for behaving
suspiciously. Tanpiengco was haled to the comfort room and frisked but nothing of value
was found in his person. Nonetheless he was taken to the security office for further
questioning; he was accused of taking a T-shirt marked "Gesim Corp." from one of the
balikbayan boxes inside the container yard.
- Tanpiengco was referred to petitioner's personnel department for investigation.
According to petitioner, he admitted to the investigating officer that he took the "Gesim
Corp." T-shirt valued at P100.00, but Tanpiengco insisted on his innocence claiming that
he was coerced at knifepoint into admitting the theft. After a brief suspension, Tanpiengco
was dismissed for pilferage which petitioner considered as breach of trust, dishonesty and
theft of property.
- Tanpiengco sued the stevedoring firm for illegal dismissal, reinstatement and backwages.
The Labor Arbiter found the dismissal to be totally unjustified as neither theft nor pilferage
was committed by Tanpiengco.
Accordingly, petitioner was ordered to reinstate
Tanpiengco with full back wages with temporary adjustments as the need would arise until
his actual reinstatement.
- On appeal, the NLRC reversed the decision of the Labor Arbiter and dismissed the
complaint of Tanpiengco, in effect holding that his termination was legal, but ordered
petitioner to pay him his wages from (date of filing of appeal with the NLRC) up to
(promulgation of NLRC decision) pursuant to Art. 223 of the Labor Code. Both parties
moved for reconsideration but were denied.
ISSUE
WON NLRC committed grave abuse of discretion when awarding backwages in spite of
valid dismissal
HELD
NO
- Art. 224 states that the need for a writ of execution applies only within (5) years from the
date a decision, an order or award becomes final and executory. It cannot relate to an
award or order of reinstatement still to be appealed or pending appeal which Art. 223
contemplates. The provision is clear that an award for reinstatement shall be immediately
executory even pending appeal and the posting of a bond by the employer shall not stay
the execution for reinstatement. The legislative intent is quite obvious, i.e., to make an
award of reinstatement immediately enforceable, even pending appeal. To require the
application for an issuance of a writ of execution as prerequisites for the execution of a
reinstatement award would certainly betray and run counter to the very object and intent of
Art. 223, i.e., the immediate execution of a reinstatement order. An application for a writ of
execution and its issuance could be delayed for numerous reasons. In other words, if the
requirements of Art. 224 were to govern, then the executory nature of a reinstatement
order or order contemplated by Art. 223 will be unduly circumscribed and rendered
ineffectual. In enacting the law, the legislature is presumed to have ordained a valid and
sensible law, one which operates no further than may be necessary to achieve a specific
purpose x x x x In introducing a new rule on the reinstatement aspect of a labor decision
under R. A. No. 6715, Congress should not be considered to be indulging in mere
semantic exercise. On appeal, however, the appellate tribunal concerned may enjoin or
suspend the reinstatement order in the exercise of its sound discretion.
- Furthermore, the rule is that all doubts in the interpretation and implementation of labor
laws should be resolved in favor of labor. In ruling that an order or award for
reinstatement does not require a writ of execution the Court is simply adhering and giving
meaning to this rule. Henceforth, we rule that an award or order for reinstatement is selfexecutory. After receipt of the decision or resolution ordering the employee's
reinstatement, the employer has the right to choose whether to re-admit the employee to
work under the same terms and conditions prevailing prior to his dismissal or to reinstate
the employee in the payroll. In either instance, the employer has to inform the employee
of his choice. The notification is based on practical considerations for without notice, the
employee has no way of knowing if he has to report for work or not.
- Thus, in the light of the Pioneer ruling, we hold that the NLRC did not commit grave
abuse of discretion in ordering petitioner to pay Tanpiengco his wages from 25 January
1991 when he appealed to the NLRC, up to 23 September 1993 when the NLRC decision
finding his dismissal valid was promulgated. That portion of the decision, which is the
sole issue in this petition, should stand. Under Art. 223 as presently construed, the
reinstatement aspect of the Labor Arbiter's decision, albeit under appeal, was immediately
enforceable as a consequence of which, petitioner, as the employer, was duty-bound to
choose forthwith whether to re-admit Tanpiengco or to reinstate him in the payroll and to
inform Tanpiengco of its choice to enable the latter to act accordingly. Failing to exercise
the options in the alternative, petitioner must pay the salary of Tanpiengco which
automatically accrued from notice of the Labor Arbiter's order of reinstatement until its
ultimate reversal by the NLRC.
- In the instant case, the NLRC failed to act on Tanpiengco's motion for the issuance of a
writ to execute the Labor Arbiter's reinstatement order. It did not even mention the fact of

Labor Law 1
its filing in its assailed Decision. The Commission's inaction is a serious oversight for
which it should be admonished. While it is incumbent upon the party to take an active role
in his case and not adopt a wait-and-see attitude, the adjudicating body has the
corresponding obligation to act promptly on all incidents brought before it.
Disposition the instant petition is DENIED. The Decision of the NLRC directing
petitioner to pay private respondent Gabriel Tanpiengco his wages from 25 January 1991
to 23 September 1993 is SUSTAINED.

KIAMCO V NLRC
[PAGE 85]

14.09 BACKWAGES
DEFINITION
EQUITABLE BANKING CORP V SADAC
[PAGE 149]
ST. THERESAS SCHOOL OF NOVALICHES
FOUNDATION V NLRC (ESTHER REYES)
289 SCRA 110
PURISIMA; April 15, 1998
NATURE
Petition to modify the Resolution issued by the NLRC
FACTS
- Adoracion Roxas is the president of St. Theresas School of Novaliches Foundation. She
hired Esther Reyes on a contract basis, for the period from June 1, 1991- March 31, 1992.
But she commenced work on May 2, 1991, during which period of employment, she
became ill. She went on leave (Feb. 17-21 NS 24-28) which were approved y Roxas.
She went back to work on March 2, 1992 but only stayed from 6:48-9:38am and never
returned.
- St. Theresa claims that Reyes abandoned her work, while Reyes maintains that she was
replaced by Annie Roxas, daughter of Adoracion Roxas. She tried to contact her
employer but the latter could not be found within the school premises. St. Theresa then
sent her a mail, telling her that her contract will not be renewed anymore. But a few
weeks before this, Reyes has filed a case against St. Theresas for, unfair labor practice
based on harassment, illegal dismissal, 13th month pay, allowances, removal of desk and
chair form place of work, and refusal to communicate, moral and exemplary damages.
The Labor arbiter ruled in favour of Reyes, ordering St. Theresas to reinstate her and to
pay for full backwages from the time of dismissal to her actual reinstatement in the
amount of P76,701.00. NLRC reversed the decision, declaring the separation of Esther
Reyes from service legal and valid. However, it directed to pay backwages from Nov. 12,
1993 up to the date of the resolution. St. Theresas now questions the award of
backwages for Reyes.
ISSUE
WON NLRC erred in awarding backwages to Reyes in the light of the finding that her
dismissal was valid
HELD
NO
- The term backwages has been defined as that for earnings lost by a worker due to his
illegal dismissal. Backwages are generally granted on grounds of equity. Payment
thereof is a form of relief that restores the income lost by reason of such unlawful
dismissal. It is not private compensation or damages, but is awarded in furtherance and
effectuation of the public objectives of the Labor Code. Nor is it a redress of a private right
but, rather, in the nature of a command to the employer to make public reparation for
dismissing an employee, either due to the formers unlawful act or bad faith.
- Jurisprudence is filled to the brim with cases wherein backwages were awarded to an
employee illegally dismissed. But where, as in this case of a pitiful employee rendered
hapless by her lawyers inaction or ignorance, the dismissal has been adjudged valid and
lawful, the challenged award of backwages is decidedly improper and contrary to law and
jurisprudence.
Disposition Petition is GRANTED; the Decision of the respondent NLRC rendered on
November 29, 1995 in NLRC NCR is hereby MODIFIED by deleting therefrom the award
of backwages in question.

A2010

Disini

- 309 -

GENERAL BAPTIST BIBLE COLLEGE V NLRC


[PAGE 298]
VIERNES V NLRC
[PAGE 94]

NATURE-PURPOSE
CLAUDIO V CA (NATIVIDAD)
423 SCRA 122
CALLEJO; February 16, 2004
NATURE
Petition for review on certiorari under Rule 45 of the Rules of Court
FACTS
- Natividad worked with petitioner Tomas Claudio Memorial College (TCMC) in Morong,
Rizal. In time, he was promoted as Liason Officer of the school with the Department of
Education, Culture and Sports (DECS) and with the Commission on Higher Education
(CHED) with the rank of Assistant Registrar.
- Natividad was arrested by the Morong police authorities, without any warrant therefore,
for violation of the Dangerous Drugs Act (Republic Act NO. 6425). A criminal complaint
was later filed against him. A preliminary investigation was conducted by the Municipal
Court of Morong, Rizal which found probable cause to hold him for trial. The court, on the
said date, issued a warrant for the private respondents arrest. The records were elevated
to the Office of the Provincial Prosecutor of Rizal.
- TCMC sent a Memorandum to Natividad informing him that his employment was
already terminated. He was thenceforth barred from entering the school.
- Natividad posted a bail bond in Criminal Case No. 5137 and was released from
his detention cell. He did not, however, file any complaint against the petitioner with
the NLRC on account of his dismissal.
- State Prosecutor issued a Resolution dismissing the criminal complaint filed
against the Natividad for lack of merit.
- Natividad was arrested anew by police authorities. The Morong Chief of Police filed a
criminal complaint against him for violation of Section 27, Article III of Rep. Act No. 6425,
as amended. An Information therefore was filed with the Regional Trial Court of Morong.
On said date, Natividad posted a bail bond and was released from detention.
- Natividad filed a complaint with the NLRC against the petitioner for illegal
dismissal.
ISSUE
WON CA committed a grave abuse of discretion amounting to excess or lack of jurisdiction
when it modified the decision of the NLRC and ordered the petitioner to pay backwages to
the private respondent
HELD
NO
- The normal consequences of a finding that an employee has been illegally dismissed
are, firstly, that the employee becomes entitled to reinstatement to his former position
without loss of seniority rights and secondly, the payment of backwages corresponding to
the period from his illegal dismissal up to actual reinstatement. The statutory intent on this
matter is clearly discernible. Reinstatement restores the employee who was unjustly
dismissed to the position from which he was removed, that is, to his status quo ante
dismissal, while the grant of backwages allows the same employee to recover from the
employer that which he had lost by way of wages as a result of his dismissal. These twin
remedies-reinstatement and payment of backwages make the dismissed employee
whole who can then look forward to continued employment. Thus do these two remedies
give meaning and substance to the constitutional right of labor to security of tenure. The
two forms of relief are distinct and separate, one from the other. Though the grant of
reinstatement commonly carries with it an award of backwages, the inappropriateness or
non-availability of one does not carry with it the inappropriateness or non-availability of the
other. . . .
- The payment of backwages is generally granted on the ground of equity. It is a form of
relief that restores the income that was lost by reason of the unlawful dismissal; the grant
thereof is intended to restore the earnings that would have accrued to the dismissed
employee during the period of dismissal until it is determined that the termination of
employment is for a just cause. It is not private compensation or damages but is awarded
in furtherance and effectuation of the public objective of the Labor Code. Nor is it a
redress of a private right but rather in the nature of a command to the employer to make

Labor Law 1
public reparation for dismissing an employee either due to the formers unlawful act or bad
faith.
- The award of backwages is not conditioned on the employees ability or inability to, in the
interim, earn any income. While it may be true that on June 11, 1996, Natividad was
detained in Criminal Case, the State Prosecutor found no probable cause for the detention
of him and resolved to dismiss the case. Natividad has not yet been convicted by final
judgment. Indeed, he is presumed innocent until his guilt is proved beyond reasonable
doubt.
Disposition Petition is DISMISSED.

EFFECT FAILURE TO CLAIM


DELA CRUZ V NLRC (LO)
290 SCRA 1
DAVIDE JR; November 20, 1998
NATURE
Special Civil Action.Certiorari
FACTS
- Petitioner started working for Emmanuel Lo as a crew hand in the latters fishing boat. He
was over several years promoted to be the Patron of the boat until he was dismissed.
From the case, it was shown that respondent was the one who hired, paid salary to, and
eventually fired dela Cruz.
- Respondent claims that he was in joint venture with the petitioner and hence there was
no employer-employee relationship between them. As pointed out by the Labor Arbiter,
however, Lo exercised the control in the activities and that therefore there was in fact an
employer-employee relationship.
- The Labor arbiter found the dismissal not to be justified and ordered Lo to pay separation
pay bu not back wages as the same, he opines ws not prayed for. Petitioner was however
found to be a managerial employee and hence was hence excluded from the coverage of
the law as regards conditions of employment.
- Both parties appealed the decision.
ISSUE
WON NLRC committed rave abuse of discretion amounting to lack or excess of jurisdiction
in dismissing petitioners claim for separation pay, back wages, allowances, and damages
HELD
YES
- Article 279 of the Labor Code mandates that petitioner who was unjustly dismissed from
work is entitled to reinstatement without loss of seniority rights and other privileges and to
full back pay, inclusive of allowances, and to other benefits or their monetary equivalent
computed from the time compensation was withheld up to time of actual reinstatement.
The grant of back wages allows the unjustly and illegally dismissed employee to recover
from the employer that which the former lost by way of wages as a result of his dismissal
from employment.
Reasoning
- Apparently, the form used in filing the case did not include a box for back wages and
hence the petitioner had to particular item to tick off. The court ruled that award of back
wages resulting from the illegal dismissal of an employee is a substantial right. Thus, the
failure to claim back wages in a complaint is a mere procedural lapse which cannot defeat
a right granted under substantive law.
Disposition Petition granted with modification as to back wages.

EFFECT FAILURE TO ORDER


AURORA LAND PROJECTS CORP V NLRC (DAGUL)
266 SCRA 48
HERMOSISIMA; January 2, 1997
NATURE
petition for certiorari for reversal of NLRC Resolution dated March 16, 1994 affirming the
modification the Labor Arbiter decision, dated May 25, 1992, finding petitioners liable to
pay private respondent the total amount P195, 624.00 as separation pay and attorney's
fees.
FACTS
- Private respondent Honorio Dagui was hired by Doa Aurora Suntay Tanjangco in 1953
to take charge of the maintenance and repair of the Tanjangco apartments and residential
building. He was to perform carpentry, plumbing electrical and masonry work. Upon the

A2010

- 310 -

Disini

death of Doa Aurora Tan\jangco in 1982, her daughter, petitioner Teresita Tanjangco
Quazon, took over the administration of all Tanjangco properties.
- June 8, 1991 - Dagui received the shock of his life Mrs. Quazon suddenly told him: "Wala
nang trabaho mula ngayon." on the alleged ground that his work was unsatisfactory.
- August 29, 1991,- Dagui, who was then already sixty-two (62) years old, filed a
complainant for illegal dismissal with the Labor Arbiter.
- May 25, 1992, - Labor Arbiter Nora ordered Aurora/Teresita Tanjanco Quazon to pay
P195,624.00 as separation pay and 10% attorneys fees
- NLRC > affirmed, with modification (P88, 920.00 instead of P177, 840.00), the Labor
arbiters decision
ISSUES
1. WON Honorio Dagui was an employee of petitioner
2. If he were, WON he was illegally dismissed
HELD
1. YES
- Aurora insisted that Daqui had never been their employee. Since the establishment of
Aurora Plaza, Dagui served therein as a job contractor. Dagui had control and supervision
of whatever task he would perform. On occasion, Dagui was hired only as a 'tubero" or
plumber as the need arises in order to unclog sewerage pipe. Every time his services
were needed, he was paid accordingly. It was understood that his job was limited to the
specific undertaking of unclogging the pipes. BUT SC IS NOT PERSUADED
- Section 8, Rule VIII, Book III of the Implementing Rules Regulations of the Labor Code
provides in part:
"There is job contracting permissible under the Code if the following conditions are met:
(2) the contractor has substantial capital or investment in the form of tools, equipment
machinries work premises, and other materials which are necessary in the condcut of
his business."
- Honorio Dagui erns a measly sum of P180.00 a day (latest salary).7 Ostensibly, and no
stretch of the imagination can Dagui qualify as a job contractor. No proof was adduced by
the petitioners to show that Dagui was merely a job contractor, and it is absurd to expect
that private respondent, with such humble resources, would have substantial capital or
investment in the form of tools, equipment, and machineries, with which to conduct the
business of supplying Aurora Plaza with manpower and services for the exclusive purpose
of maintaining the apartment houses owned by the petitioners herein.
- Dagui was not compensated in terms of profits for his labor or service like an
independent contactor. Rather, he was paid on a daily wage basis at the rate of P180.00.
Employees are those who are compensated for their labor or services by wage rather than
by profits.
- Finally, the records unmistakably show that the most important requisites of control is
likewise extant in this case. It should be bone in mind that the power of control referees
merely to the existence of the power and not to the actual exercise thereof. It is not
essential for the employer to actually supervise the performance of duties of the
employee. it is enough that the former has a right wield the power.
- Dagui shoulld be considered a regular employee by the mere fact he rendered service
for the Tanjancos for more one year, that is, beginning 1953 until 1982, under Doa
Aurora; and then from 1982 up to June 8, 1991 under the petitioners, for a total of twentynine (29) and nine (9) years respectively.
2. YES
Ratio Jurisprudence abound as to the twin requirements of due process, substantive and
procedural, must be complied with, before a valid dismissal exists. Without which the
dismissal become void.
Reasoning
- The twin requirements of notice and hearing constitute the essential elements of due
process. This simply that the employer shall afford the worker ample opportunity to be
heard and to defend himself with the assistance of his representative, if he so desires.
- Pepsi Cola Bottling Co. v. NLRC
"The law requires that the employer must furnish the worker sought to be dismissed
with two written notice before termination of employee can be legally effected: (1)
notice which apprise the employee of the particular acts or omissions for which his
dismissal is sought; and (2) the subsequent notice which informs the employee of the
employers decision to dismiss him (section 13, BP 130; Sections 2-6, Rule XIV, Book V
Rules and Regulations Implementing the Labor code as amended). failure to comply
with the requirements taints the dismissal with illegality. This procedure is mandatory, in
the absence of which, any judgment by judgment by management is void and
inexistent.
- These mandatory requirements were undeniably absent. Quazon dismissed Dagui on
June 8, 1991, without giving him any written notice informing the worker herein of the
cause for his termination. Neither was there any hearing conducted i order to give Dagui
the opportunity to be heard and defend himself he was simply told: "Wala ka nang trabaho
mula ngayon" allegedly because pf poor workmanship on a previous job. The undignified
manner by which private respondent's services were terminated smacks of absolute denial
of the employee's right to due process and betrays petitioner Quazon's utter lack of
respect for labor. Such an attitude indeed deserves condemnation.

Labor Law 1
- It must be remembered that backwages and reinstatement are two reliefs should be
given to an illegally dismissed employee. They are separate and distinct from each other.
In the event that reinstatement is no longer possible, as in this case, separation pay is
awarded to the employee. The award separation pay is lieu of reinstatement and not of
backwages. In other words, an illegally dismissed employee is entitled to (1) either
reinstatement, if viable, or separation pay if reinstatement is no longer viable, and (2)
backwages. Payment of backwages is specifically designed to restore an employee's that
was lost because of his unjust dismissal. On the other hand, payment of separation pay is
intended to provide the employee money during the period in which he will be looking for
another employment
Disposition instant petition is partly GRANTED and the Resolution of NLRC dated march
16, 1994 is hereby MODIFIED in that the award of separation pay against the petitioners
shall be reckoned from the date Daguis was re-employment by the petitioners in 1982,
until June 8, 1991. In additional to separation pay, full backwages are likewise awarded to
Dagui, inclusive of allowances, and other benefits of their monetary equivalent

PERIOD COMPUTATION
ITOGON-SUYOC MINES INC V
SAGILOITOGON WORKERS UNION
24 SCRA 873
SANCHEZ; August 30, 1968
NATURE
petition for review by certiorari decision of CIR (directing the reinstatement of 15 individual
respondents to their former positions with backwages from time of dismissal to actual
reinstatement)
FACTS
- Itogon-Suyoc Mines, Inc., through its superintendent Fertig, had been dismissing from its
employ members of the Sagilo-Itogon Workers Union. On May 28, 1958, sensing that its
members were being eased out of employment one by one, Sagilo called a strike,
accompanied by picketing carried out at or near petitioner's mine premises in Itogon. Work
was paralyzed.
- Cases were filed with the CIR for unfair labor practice, for the arbitrary dismissal of
employees allegedly because of affiliation with the union and for testifying against the
company in a certification case. CIR then ruled that the 15 employees who pushed
through with the case be reinstated, with full backwages. After denial of its MFR, the
company appealed to the SC.
ISSUE
How should the amount of total back wages be determined?
HELD
- The court restated the guidelines to be observed in ascertainment of back wages,
considering that 10 years had passed since the terminations.
Ratio - First. To be deducted from the back wages accruing to each of the laborers to be
reinstated is the total amount of earnings obtained by him from other employment(s) from
the date of dismissal to the date of reinstatement. Should the laborer decide that it is
preferable not to return to work, the deduction should be made up to the time judgment
becomes final. And these, for the reason that employees should not be permitted to enrich
themselves at the expense of their employer.
- Second. Likewise, in mitigation of the damages that the dismissed respondents are
entitled to, account should be taken of whether in the exercise of due diligence
respondents might have obtained income from suitable remunerative employment. We are
prompted to give out this last reminder because it is really unjust that a discharged
employee should, with folded arms, remain inactive in the expectation that a windfall
would come to him. A contrary view would breed idleness; it is conducive to lack of
initiative on the part of a laborer. Both bear the stamp of undesirability.
**note: the real issue in this case is really WON there was unfair labor practice. The SC
found that there was, and just affirmed the judgment of the CIR. The topic of determination
of back wages was just obiter, with neither party raising such issue. The court just wanted
to say it.
Disposition Judgment AFFIRMED.

FEATI UNIVERSITY FACULTY CLUB V FEATI


UNIVERSITY
85 SCRA 395
TEEHANKEE; August 15, 1974
FACTS

A2010

- 311 -

Disini

- Jan 14 1963 Pres of Faculty Club wrote a letter to the president of the university
informing her that the Faculty club has been registered as a labor union
- Jan 22 1963 Pres of Faculty Club in another letter to the university president gave 26
demands regarding the employment of members of the club and giving the president 10
days to act
- University President replied and asked for at least 30 days to study their demands
- Counsel for the University demanded that that the faculty club submit proof of their
majority status and designation as bargaining unit
- President of Faculty club denied universitys presidents request for extension of time
- Feb 18 1963 Feati club declared a strike for bec of unfair labor practice due to the
universitys refusal to recognize their union and to bargain collectively
- President of the Philippines certified the dispute to the CIR
- Several cases were filed as a result of the dispute and the orders of CIR were elevated
to the SC in three cases.
- SC upheld all challenged orders and held among others that
1.CIR had jurisdiction over the parties and subject matter
2.SC recognized employee status of the Faculty club members as employees and
declared RA 875 applicable to them in their employment relations with their school
3.SC repudiated all contentions of the university questioning the legality of the
industrial courts return to work order
- In 1969 2nd case of Feati University v Feati University Faculty Club
- Faculty club filed motions and petitions to execute and to implement the decision as well
as to hear the dispute on its merits
- motion to execute the march 30 1963 return to work order directing the university
management to permit and effect the return to work of the strikers plus the back salaries
that accrued
- the university opposed
- the CIR issued a writ of execution for the enforcement of the return to work order
- the CIR further ordered that the accrued backwages from july 12 1963 to the date of
readmission be paid to the Faculty club members
- CIR denied reconsideration. SC denied as well (twice).
- Nov 24 1969 CIR made a complete turnabout and issued a decision repudiationg and
turning its back on its former decision declaring the strike as illegal and that the members
are terminated. Holding that the university is not chargeable with unfair labor practice. It
sought to nullify the faculty members finally adjudicated right to reinstatement and
backwages upheld by the SC in two judgments
- Thus the final case of certiorari
ISSUE
WON the Faculty Club members are entitled to reinstatement and payment of backwages
(and how will it be computed)
HELD
YES
- The strike was valid as the petitioners has a valid cause to go on strike because of the
unfair labor practice and discriminatory acts aimed at aborting its union even without a
notice of strike. Thus the implementation of the return to work order with accrued
backwages should proceed without delay
- As to the amount of backwages:
Apply the ruling in Mercury Drug Co. V CIR
fixing the amount of backwages to a just and reasonable level without qualification or
deduction so as to avoid protracted delay in the execution of the award due to extended
hearings and unavoidable delays and difficulties encountered in determining the earnings
of the laid off employees ordered to be reinstated with backwages during the pendency of
the case for purposes of deducting the same from the gross backwages awarded.
this formula relieves the employees from proving or disproving their earnings during
their lay off and the employees from submitting counter proofs and obviates win evil s of
idleness on the part of the employee who would with folded arms remain inactive in the
that a windfall would come to him and attrition and protracted delay in satisfying such
award on the part of the unscrupulous employers who have seized upon the further
proceedings which would practically render nugatory such award and compel the
employees to agree to unconscionable settlements of backwages in order to satisfy their
dire needs.
- In this case-3 school years without qualification and deduction

MERCURY DRUG CO INC V CIR (DAYAO)


56 SCRA 694
MAKASIAR; April 30, 1974
NATURE
Petition for review on certiorari of a decision of CIR.
FACTS

Labor Law 1

A2010

- DAYAO was an assistant chief checker of MERCURY. After criticizing their existing labor
union for failing to secure additional pay for overtime, and campaigning among his coemployees to organize another union, MERCURY gave DAYAO a termination pay after the
latter signed a quitclaim.
- 2 years and 15 days from his separation, DAYAO filed a complaint for unfair labor
practice (ULP) against MERCURY. SC held that the finding of fact of CIR that DAYAO was
dismissed because of his union activities and that, consequently, MERCURY were guilty of
ULP is amply substantiated by credible evidence. SC also held that DAYAOs complaint
has been seasonably filed. It explained that in the absence of specific legal provision, the
statute of limitations prescribed by the Civil Code should apply. ART.1146 directs that the
action upon an injury to the rights of the plaintiff must be instituted within 4 years. All other
actions whose periods are not fixed in the Civil Code or in other laws must be brought
within 5 years from the time the right of action accrues (ART.1149). SC finally concluded
that, whether the ULP charge is based on an injury to the rights of DAYAO or placed under
the category of all other actions for which no law prescribes the time limit for their
institution, the filing DAYAO of the ULP charge was well within either the prescriptive
period. The question remains on the computation of DAYAOs backwages.
ISSUE
How should the backwages to be awarded to DAYAO be computed?
HELD
- As stated, the shortest prescriptive period for the filing of all other actions for which the
statute of limitations does not fix a period, is four years. The period of delay in instituting
this ULP charge with claim for reinstatement and back wages, although within the
prescriptive period, should be deducted from the liability of MERCURY to DAYAO for
backwages. In order that the employee, however, should be relieved from proving his
income during the period he was out of the service and the employer from submitting
counter-proofs, which may delay the execution of the decision, MERCURY is to pay
DAYAO backwages equivalent to 1 year, 11 months, and 15 days without further
disqualifications.51
Disposition Petition DISMISSED.

PINES CITY EDUCATIONAL CENTER V NLRC


(BENTREZ, PICART, ET AL)
227 SCRA 655
NOCON; November 10, 1993
NATURE
PETITION for certiorari to reverse a resolution of NLRC
FACTS
- Private respondents were all employed as teachers on probationary basis by petitioner
Pines City Educational Center
- With the exception of Bentrez who was hired as a grade school teacher, the remaining
private respondents were hired as college instructors. All the private respondents, except
Poland Picart and Lucia Chan, signed contracts of employment with petitioner for a fixed
duration.
- due to the explanation of private respondents' contracts and their poor performance as
teacher they were notified of petitioners' decision not to renew their contracts anymore.
- private respondents filed complaint for illegal dismissal before the Labor Arbiter, alleging
that their dismissals were without cause and in violation of due process.
- Except for private respondent Leila Dominguez who worked with petitioners for one
semester, all the other private respondents were employed for one to two years. They
were never informed in writing by petitioners regarding the standards or criteria of
evaluation so as to enable them to meet the requirements for appointment as regular
employees. They were merely notified in writing by petitioners.
- Petitioners contended that private respondent separation from employment, apart from
their poor performance, was due to the expiration of the periods stipulated in their
respective contracts. These stipulations were the laws that governed their relationships,
and there was nothing in said contracts which was contrary to law, morals, good customs

51

Mathematically speaking,
Prescribed period to file ULP complaint = 4 years
Less: Delay in filing DAYAO complaint = 2 years
Basis for DAYAOs backwage claim
= 1 year

00 months
00 months
11 months

00 days
15 days
15 days

Note that the award is without further disqualifications. As Teehankee, J. explained in his separate opinion, such phrase
means without having to determine and deduct earnings from the general award of back wages from date of unlawful
dismissal until actual reinstatement heretofore customarily made in such ULP and reinstatement cases, [which
determination] led to long delays in the execution of the decision for back wages. [By relieving] the employees from
proving their earnings during their lay-offs and the employer from submitting counter-proofs, [the formula in the ponencia]
obviate the twin evils of idleness of the employees [who may remain inactive during layoff in the expectation of a
windfall backwages award] and undue delay in satisfying the award on the part of the employer[, the new formula] is
a realistic, reasonable and mutually beneficial solution.

Disini

- 312 -

and public policy. They argued further that they cannot be impelled to enter into new
contracts with private respondents.
- Labor Arbiter rendered judgment in favor of private respondents, the dispositive portion
of which reads:
...BACKWAGES NOTE: Computation covers only the period complainants were
terminated up to January 31, 1990 or 10 months and does not include backwages from
January 31, 1990 up to their actual reinstatement.
1)ROLAND PICART a) Latest salary per monthP2,136.00
b) Multiplied by the period covered (March 31, 1989 to January 31, 1990)x 10 months c)
Equals backwages dueP21,360.00
2) LUCIA CHAN a) Latest salary per monthP1,600.00
b) Multiplied by period coveredx 10 months c) Equals backwages dueP16,000.00
3) LEILA DOMINGUEZ a) Latest salary per monthP1,648.24
b) Multiplied by period coveredx 10 months c) Equals backwages dueP16,482.40
4) RUPERTA RIBAYA a) Latest salary per monthP1,856.00
b) Multiplied by period coveredx 10 months c) Equals backwages dueP18,560.00
5) CECILIA EMOCLING a) Latest salary per monthP1,648.00
b) Multiplied by period coveredx 10 months c) Equals backwages dueP16,480.00
6) ROSE ANN BERMUDEZ a) Latest salary per monthP2,600.00
b) Multiplied by period coveredx 10 month c) Equals backwages dueP26,000.00
7) DANGWA BENTREZ a) Latest salary per monthP1,700.00
Multiplied by period coveredx 10 months
c) Equals backwages dueP17,000.00
8) JANE BENTREZ a)Latest salary per monthP1,315.44
b)Multiplied by period coveredx 10 months c)Equals backwages dueP13, 154.40
9) APOLLO RIBAYA a)Latest salary per monthP1,875.00
b)Multiplied by period coveredx 10 months c)Equals backwages dueP18,750.00
10)VIRGINIA BOADO a)Latest salary per monthP1,648.24
b)Multiplied by period coveredx 10 months c)Equals backwages dueP16,482.40...
- NLRC affirmed the decision in toto
ISSUE
WON private respondents were lawfully terminated
HELD
We have to make a distinction:
1. YES
Ratio Insofar as the private respondent who knowingly and voluntarily agreed upon fixed
periods of employment are concerned, their services were lawfully terminated by reason of
the expiration of the periods of their respective contracts. These are Dangwa Bentrez,
Apollo Ribaya,
- Sr., Ruperta Ribaya, Virginia Boado, Cecilia Emocling, Jose Bentrez,
- Leila Dominguez and Rose Ann Bermudez. Thus, public respondent committed grave
abuse of discretion in affirming the decision of the Labor Arbiter ordering their
reinstatement and payment of full backwages and other benefits and privileges.
2. NO
Ratio With respect to private respondents Roland Picart and Lucia Chan, both of whom
did not sign any contract fixing the periods of their employment nor to have knowingly and
voluntarily agreed upon fixed periods of employment, petitioners had the burden of proving
that the termination of their services was legal. As probationary employees, they are
likewise protected by the security of tenure provision of the Constitution. Consequently,
they cannot be remooved from their positions unless for cause.
- There is absolutely nothing in the record which will show that the complainants were
afforded even an iota of chance to refute respondents' allegations that the complainants
did not meet the reasonable standards and criteria set by the school.
- Thus, the order for their reinstatement and payment of full backwages and other benefits
and privileges from the time they were dismissed up to their actual reinstatement is proper.
- On Backwages However, in ascertaining the total amount of backwages payable to
them, we go back to the rule prior to the Mercury Drug rule that the total amount derived
from employment elsewhere by the employee from the date of dismissal up to the date of
reinstatement, if any, should be deducted therefrom. We restate the underlying reason that
employees should not be permitted to enrich themselves at the expense of their
employer. In addition, the law abhors double compensation.
Disposition resolution of NLRC is MODIFIED. Private respondents Roland Picart and
Lucia Chan ordered reinstated without loss of seniority rights and other privileges and
then, backwages paid in full inclusive of allowances, and to their other benefits or their
monetary equivalent subject to deduction of income earned elsewhere during the period of
dismissal, if any, to be computed from the time they were dismissed up to the time of their
actual reinstatement.

SEPARATE OPINION
PADILLA

Labor Law 1
- I concur in the Court's decision penned by Mr. Justice Nocon except that I cannot see my
way clear to allowing deductions from the full backwages prescribed by law, given the
language and evident intention of RA 6715.
- Art. 279 of the Labor Code as amended by Rep. Act No. 6715 states: "...An employee
who is unjustly dismissed from work shall be entitled to reinstatement without loss of
seniority rights and other privileges and to his full backwages inclusive of allowances, and
to his other benefits or the monetary equivalent computed from the time his compensation
was withheld from him up to the time of his actual reinstatement." The amendment to Art.
279 of the Labor Code introduced by Rep. Act No. 6715 inserted the qualification "full" to
the word "backwage". The intent of the law seems to be clear. The plain words, of the
statute provide that an employee who is unjustly dismissed is entitled to FULL backwages
from the time of his dismissal to actual reinstatement. The law provides no qualification
nor does it state that income earned by the employee during the period between his unjust
dismissal and reinstatement should be deducted from such backwages. When the law
does not provide, the Court should not improvise.
- It is further my view that the principle of unjust enrichment (if no deduction is allowed
from backwages) does not apply in this case.

BUSTAMANTE V NLRC (EVERGREEN FARMS)


265 SCRA 1
PADILLA; March 15, 1996
NATURE
Petition for certiorari
FACTS
- Petitioners Bustamante, Bantayan, Sumunod and Lamaran were employed as laborers,
harvesters and sprayers in private respondent companys banana plantation in Davao del
Norte. They all signed contracts of employment for 6 months from Jan.2, 1990 to July 2,
1990 but they had started working in Sept. 1989. They were previously hired to do the
same work for periods lasting a month or more, from 1985 to 1989. Before the
employment contracts expired on July 2, 1990, petitioners employments were terminated
on the ground of poor performance due to age, as none of them was allegedly below 40
years old.
- Petitioners then filed a complaint for illegal dismissal which the Labor Arbiter decided in
their favor. The judgment declared the dismissal illegal, and ordered Evergreen Farms to
reinstate them immediately with 6 months backwages. Private respondent company
appealed to the NLRC but the appeal was dismissed for lack of merit. A subsequent MFR
filed by respondent company was similarly disposed of with the modification that the
award for backwages was deleted, as there was no bad faith on the part of Evergreen
Farms.
- The removal of the award of backwages prompted petitioners to file this case, alleging
that public respondent NLRC gravely abused its discretion.
ISSUE
WON petitioners are entitled to backwages
HELD
YES, because petitioners are regular employees.
Ratio Regular employees dismissed for no valid cause are entitled to full backwages and
other benefits from the time their compensation was withheld from them up to the time of
their actual reinstatement.
Reasoning
- Petitioners were employed at various periods from 1985-1989 for the same work they
were hired to perform. They were engaged to perform activities which are necessary in the
usual business of the employer. The contract for probationary employment was utilized by
respondent company as a chicanery to deny petitioners their status as regular employees
and to evade paying them the benefits attached to such status. They were hired and rehired in a span of from 2-4 years to do the same type of work which conclusively shows
the necessity of petitioners services to the respondent.
- The act of hiring and re-hiring the petitioners over a period of time without considering
them as regular employees evidences bad faith on the part of private respondent. The
public respondent made a finding to this effect when it stated that the subsequent rehiring
of petitioners on a probationary status clearly appears to be a convenient subterfuge on
the part of management to prevent complainants (petitioners) from becoming regular
employees.
Disposition Resolution of NLRC is modified, the deletion of the award for backwages is
set aside. LA decision is AFFIRMED, with modification that backwages shall be paid to
petitioners from the time of their illegal dismissal up to the date of their reinstatement.

TORRES V NLRC (E&R SECURITY AGENCY)


330 SCRA 311

A2010

Disini

- 313 PARDO; April 12, 2000

NATURE
Petition for certiorari
FACTS
- Jan 5, 1989, E & R security agency hired Chona Torres as a security guard.
- On Oct 27, 1989, during a routinary meeting of the security guards of the agency
assigned to the Philippine Aerospace Development Corporation, the issue of granting a
P25.00 pay increase pursuant to RA 6727 was taken up and questions were raised as to
the date of implementation of the increase. Petitioner Torres stood up and uttered aloud at
the presiding officer: "BAKIT ANG SASABIHIN NINYO SA OPISINA AT DITO AY
MAGKAIBA!" to which remark the presiding officer replied: "WALA NAMAN PAGKAKAIBA,
DI BA?". The presiding officer also asked: "BAKIT AYAW MO DOON SA OPISINA?" Then
petitioner shouted: "WALA NA AKONG TIWALA SA INYO AT SA AGENCY KASI
SINUNGALING KAYO. EH, KUNG LALAKI LANG AKO, BAKA KUNG ANO PA ANG
NAGAWA KO SA INYO NGAYON!"
- On the same day, the agency sent Torres a letter saying effective immediately, she is
suspended from duty as Security Guard for fifteen days for discourtesy, disloyalty and
insubordination while in the performance of duty.
- Torres filed with the Labor Arbiter a complaint for illegal suspension and violation of R.A.
6727, and for having been required to sign on a blank payroll.
- On Nov 10, 1989, Torres received a letter from the agency informing her that she was reassigned and required to report at the respondent's Manila office for further instructions.
On Nov 27, the agency terminated her services for abandonment when she failed to report
for work in her new assignment.
- Torres filed with the Labor Arbiter an amended complaint charging respondent with
underpayment of wages under R.A. No. 6640 and harassment.
- Labor Arbiter ordered agency to reinstate Torres to her former position, pay her salary for
Oct 1989, and salary differentials under RA 6640 and RA 6727
Agency appealed to NLRC. The NLRC denied the appeal on the ground of non-perfection
due to lack of appeal bond and that there was no reason to disturb the decision.
- The decision having become final, the Labor Arbiter issued a writ of execution on the
reinstatement aspect, but it was not implemented because the monetary aspect of the
decision remained to be determined.
- On Nov 8, 1991, Torres asked the Labor Arbiter to issue an alias writ of execution based
on the completed computation of back wages of P104,396.00 worked out by NLRC's
Research and Information Unit.
- NLRC Sheriff issued a Notice of Garnishment which was served the agencys deposit
account with PNB, in the amount of P105,296.00 inclusive of the execution fee of
P1,000.00.
- On Nov 27, 1991, Labor Arbiter directed PNB to release the garnished amount and to
make it payable to NLRC cashier for the account of Torres pending ultimate release to her.
PNB issued a Manager's Check dated Dec 6, 1991.
- Meantime, on Dec 3, 1991, the agency filed with the Labor Arbiter an Urgent Ex-Parte
Motion to Quash the Alias Writ of Execution on the ground that there has been a change in
the situation of the parties which makes the execution inequitable. Respondent contended
that Torres accepted employment from another security agency without previously
resigning from it.
- On Feb 2, 1992, Labor Arbiter issued an order for partial execution directing the release
of the uncontested salary differential amounting to P15,523.48, to be deducted from the
amount of P105,396.00, and to withhold the balance, pending resolution of the Motion to
Quash the alias Writ of Execution.
- On March 19, 1992, petitioner filed with NLRC a petition for mandamus and injunction to
compel the Labor Arbiter to issue an order directing the NLRC Cashier to release the
entire amount deposited with the latter to petitioner.
- On Aug 11, 1992, NLRC issued a resolution denying the petition for mandamus and
injunction and ordered Labor Arbiter to immediately resolve respondents Urgent Motion to
Quash Writ of Execution.
- Petitioner contends that the release of the judgment award is purely a ministerial duty of
the Labor Arbiter.
ISSUE
WON NLRC committed grave abuse of discretion in ordering the Labor Arbiter to resolve
the motion to quash alias writ of execution
HELD
YES
- Execution is the final stage of litigation, the end of the suit. It can not be frustrated except
for serious reasons demanded by justice and equity. When a judgment becomes final and
executory, it is the ministerial duty of the court to issue a writ of execution to enforce the
judgment.
- A writ of execution may however be refused on equitable grounds as when there was a
change in the situation of the parties that would make execution inequitable or when

Labor Law 1
certain circumstances, which transpired after judgment became final, rendered execution
of judgment unjust. The fact that the decision has become final does not preclude a
modification or an alteration thereof because even with the finality of judgment, when its
execution becomes impossible or unjust, it may be modified or altered to harmonize the
same with justice and the facts.
- The respondent agency's contention that there has been a change in the situation of the
parties making execution inequitable because petitioner accepted employment from
another agency without resigning from it is patently without merit. The rule now is that
back wages awarded to an illegally dismissed employee shall not be diminished or
reduced by the earnings derived by him elsewhere during the period of his illegal
dismissal.
- In this particular case, the decision is final and, in fact, the amount of the salary
differentials and back wages awarded to Torres has been garnished from the account of
respondent agency with PNB with no opposition or resistance and it is the ministerial duty
of the Labor Arbiter to release the money to petitioner.
Disposition Petition granted. Labor Arbiter directed to release the money award to
petitioner

KAY PRODUCTS INC V CA (KAY PRODUCTS


EMPLOYEES UNION, ABILA)
464 SCRA 544
CALLEJO; July 28, 1995
FACTS
- Abila, Morante, Regidor, Escuadro and del Valle were employed by Kay Products Inc
(KPI) as factory sewers. They joined other employees in planning to form a union. When
KPI learned about the plan, it transferred the employees to an employment agency with
which it had a manpower contract, the Gerrico Resources & Manpower Services, Inc.
(GRMSI). KPI promised that the employees would receive bigger and better benefits as
regular employees of GRMSI.
- KPI directed all employees concerned to sign resignation letters preparatory to their
employment with GRMSI. Thus, the employees submitted handwritten letters of
resignation, and KPI took custody of their identification cards.
- The employees continued to report for work in the KPI but received less wages/salaries.
Then, KPI announced that GRMSI had been dissolved and that there was a need for them
to sign separate contracts with RCVJ, another corporation with which KPI had a
manpower contract.
- This time, Abila, Morante, Regidor, Escuadro and Del Valle, refused to sign any contract
with RCVJ.
- Meanwhile, the employees of KPI were able to organize their union, the Kay Products
Employees Union.
- On Aug 25, 2000, 73 employees, together with the Kay Products Employees UnionPTGWO, filed a Complaint with Regional Arbitration Branch No. IV of the DOLE against
KPI. The employees claimed that the petitioners were guilty of unfair labor practice (ULP),
underpayment of salaries and service incentive leave pay, and failure to classify them as
regular employees.
- On Sept 4, 2000, the employees were ordered to take a two-week leave from work
without pay. The employees complained. When they tried to report for work after the twoweek period, they were refused entry and were told that they had ceased to be KPI
employees since they had resigned upon agreeing to be employed by GRMSI.
- The employees and their union amended their complaint to illegal dismissal. The
complainants (respondents) alleged: 1) that they were illegally dismissed; their dismissal
was not grounded on any just and authorized cause under the law, and they were
deprived of their right to due process; 2) that in interfering with their right to selforganization by deceitfully transferring them to an employment agency, KPI thereby
engaged in ULP; 3) that they were coerced and intimidated into signing letters of
resignation; 4) that they were entitled to money claims (13th month pay, service incentive
leave pay, vacation and sick leave pay) from the time of their illegal dismissal until their
reinstatement.
- KPI: 1) employees voluntarily resigned from their work that aside from the typewritten
resignation letter, the complainants also wrote other letters in their own handwriting
that the complainants voluntarily secured their clearances from KPI, transferred to RCVJ,
and never went back to work; 2) that it never interfered with its workers right to selforganization or any of their plans to form any union; 3) that the complainants were not
entitled to reinstatement, backwages, moral and exemplary damages, and attorneys fees.
ISSUES
1. WON the employees resigned voluntarily
2. WON they were illegally dismissed
3. WON they were entitled to backwages
HELD
1. NO

A2010

- 314 -

Disini

- In Molave Tours v. NLRC, resignation has been defined as the voluntary act of an
employee who finds himself in a situation where he believes that personal reasons cannot
be sacrificed in favor of the exigency of the service, then he has no other choice but to
disassociate himself from his employment. In the context of Section II, Rule XIV, Book V
of the Revised Rules Implementing the Labor Code, resignation is a formal
pronouncement or relinquishment of an office. It must be made with the deliberate animus
or intention of relinquishing the office accompanied by an act of relinquishment. In the
same case, it was also held that the voluntariness of a resignation is negated by the filing
of a complaint for illegal dismissal
- We are inclined to agree with the petitioners that they were coerced, threatened or
intimidated into signing blank sheets of paper which materialized into resignation letters,
the contents of which were dictated by the Director and Personnel Manager of the
respondent company. For to us, it is inconceivable that a worker who has already attained
a regular status in his employment would opt to be transferred to another employment
agency, there to start work anew work that would relegate him to a mere casual laborer
or employee. Obviously, petitioners were not given any other choice by management, but
to agree to their transfer to Gerrico Resources, lest they lose their only means of
livelihood. Considering that petitioners are ordinary sewers of respondent company, the
fear of losing their jobs cannot but be a serious, legitimate concern.
2. YES
- Undeniably, petitioners were regular employees at the time they allegedly voluntarily
resigned on July 14, 2000. As such regular employees, the Labor Code grants petitioners
security of tenure, which essentially means that their employer can not terminate their
services except for just and authorized causes, as provided for under the Labor Code.
Viewed in this light, private respondents act or action in transferring petitioners to a
manpower agency (Gerrico Resources then later on to RCVJ) with the promise that they
would receive the same benefits as regular employees, puts in question private
respondents real motive. If anything, it gives currency to the belief that petitioners had
been illegally terminated or dismissed from employment.
3. YES
- As regular employees, the private respondents are entitled to security of tenure provided
under the labor laws and may only be validly terminated from service upon compliance
with the legal requisites for dismissal and considering that they were illegally dismissed,
the private respondents should be reinstated, in accordance with the provision of the
Labor Code, as amended, particularly Article 279, to wit:
Article 279. Security of Tenure. In cases of regular employment, the employer shall
not terminate the services of an employee except for a just cause or when authorized
by this Title. An employee who is unjustly dismissed from work shall be entitled to
reinstatement without loss of seniority rights and other privileges and to his full
backwages, inclusive of allowances, and to his other benefits or their monetary
equivalent computed from the time his compensation was withheld from him up to the
time of his actual reinstatement
- Thus, the said provision provides that illegally dismissed employees are entitled to
backwages plus other benefits computed from the time compensation was withheld up to
the time of actual reinstatement. An illegally dismissed employee who, in contemplation of
the law, never left his office, should be granted the compensation which rightfully belongs
to him from the moment he was unduly deprived of it up to the time it was restored to him;
the backwages to be awarded should not be diminished or reduced by earnings derived by
the illegally dismissed employee elsewhere during the term of his illegal dismissal.

STANDARD ELECTRIC MANUFACTURING CORP V


STANDARD ELECTRIC EMPLOYEES UNION
[PAGE 227]
BPI EMPLOYEES UNION V BPI
[PAGE 303]
FILIPINO PRE-FABRICATED BUILDING SYSTEMS INC
V PUENTE
[PAGE 92]
INTERCONTINENTAL BROADCASTING CORP V
BENEDICTO
495 SCRA 561
CORONA; July 20, 2006
NATURE
Petition for review on certiorari
FACTS

Labor Law 1
- Intercontinental Broadcasting Corporation is a government-owned and controlled
corporation. It is engaged in the business of mass media communications including,
among others, the operation of television IBC 13.
- In 1993, Reynaldo Benedicto was appointed by Ceferino Basilio, the general manager of
IBC 13, as marketing manager. He received P20,000 with commission of 1% from
advertising collections.
- An October 11, 1994 letter terminated Benedicto. Benedicto then filed a complaint for
illegal termination on December 3, 1996. The Labor Arbiter ruled in his favor and ordered
for reinstatement with payment of backwages from the time of his dismissal up to his
actual reinstatement (amounting to P920,000 at the time of promulgation of the decision).
Petitioners Claim
- IBC 13 complained that the award of backwages was excessive and asked for
reconsideration from NLRC. The NLRC did not grant the motion because non-payment of
appeal bond within the reglementary period.
- Petitioner insists that the award should be limited to what Benedicto was entitled to as of
the compulsory retirement age of 65 years. When the labor arbiter promulgated his
decision (wherein he awarded the amount of P920,000 as backwages), Benedicto was
already 68 years old. In an order dated August 10, 1999, he further increased the
backwages by P180,000.
Respondents Comments
- Benedicto alleged that after his appointment, he was able to increase the televiewing,
listening and audience ratings of petitioner which resulted in its improved competitive
financial strength.
- He claimed that he successfully initiated, pursued and consummated an advertising
contract with VTV Corporation for a period of five years involving the amount of P600
million
ISSUE
WON the computation of the backwages was correct
HELD
NO
Reasoning
- The Labor Arbiter found that Benedictos termination was illegal because of absence of
just cause. The reinstatement of Benedicto has been rendered moot and academic
because of is death in 2002.
- Benedicto was entitled to backwages only up to the time he reached 65 years old, the
compulsory retirement age under the law. When Benedicto was illegally dismissed on
October 11, 1994, he was already 64 years old. He turned 65 years old on December 1,
1994 at which age he was deemed to have retired. Since backwages are granted on
grounds of equity for earnings lost by an employee due to his illegal dismissal, Benedicto
was entitled to backwages only for the period he could have worked had he not been
illegally dismissed, i.e. from October 11, 1994 to December 1, 1994.
- As for the issue of Benedictos 1% commission, the court held that there should be a
recomputation with the following in mind:
1. Because Benedicto was entitled to backwages only from October 11 to December 1,
1994 when he turned 65 years old, petitioner should pay his commission only for this
period.
2. By nature, commissions are given to employees only if the employer receives
income. Employees, as a reward, receive a percentage of the earnings of the
employer, which they, through their efforts, helped produce. Commissions are also
given in the form of incentives or encouragement so that employees will be inspired to
put a little more industry into their tasks. Commissions can also be considered as direct
remunerations for services rendered. All these different concepts of commissions are
incongruent with the claim that an employee can continue to receive them indefinitely
after reaching his mandatory retirement age.
- Benedictos right to the commissions was coterminous with his employment with
petitioner and this ended when he reached the compulsory retirement age.
3. Lastly, the stipulation providing for commissions (which did not specify the period of
entitlement) would be too burdensome if interpreted to mean that Benedicto had a right
to it even after his employment with petitioner. Doubts in contracts should be settled in
favor of the greatest reciprocity of interests. A lopsided and open-minded construction
could not have been the parties contemplation. Had that been their intent, then they
should have spelled it out in no uncertain terms
Disposition The assailed decision of the Court of Appeals is set aside.

TPI PHIL CEMENT CORP V CAJUCOM VII


483 SCRA 494
SANDOVAL-GUTIERREZ; February 28, 2006
NATURE
Review on certiorari

A2010

- 315 -

Disini

FACTS
- Petitioners TP Cement and TP Vinyl employed Atty. Benedicto A. Cajucom VII,
respondent, as Vice-President for Legal Affairs.
- As a result of the economic slowdown then experienced in this country, petitioner TP
Cement, having no viable projects, shortened its corporate term from 50 years to 2 years
and 7 months. It was dissolved on January 27, 1998. With respect to petitioner TP Vinyl,
it shifted its business from production to marketing and trading of Thai Petrochemical
products.
- Thus, petitioners implemented cost-cutting measures resulting in the termination from the
service of their employees, including respondent.
- Petitioners sent respondent a notice terminating his services effective December 30,
1998. Simultaneously, petitioners, on the same day, filed with the Department of Labor
and Employment (DOLE) an Establishment Termination Report of respondents
separation from the service.
- Respondent contested petitioners action, claiming that the termination of his services
was based erroneously on petitioners probable losses, instead of their actual, substantial
and imminent losses.
- Respondent further claimed that petitioners were motivated by revenge in terminating his
services.
- The Labor Arbiter rendered a Decision holding that petitioners failed to adduce sufficient
evidence to show that their alleged losses are substantial and imminent, and concluded
that respondent was illegally dismissed from employment.
- Upon appeal, the National Labor Relations Commission (NLRC) reversed the Labor
Arbiters Decision.
- The Court of Appeals rendered the assailed Decision affirming with modification the
NLRC Decision in the sense that aside from paying respondent separation pay, petitioners
are also ordered to pay him his backwages.
ISSUES
1. WON petitioner was validly terminated (i.e. there was valid retrenchment)
2. WON the termination followed due process
3. WON respondent is entitled to backwages
HELD
1. YES
- Retrenchment, under Article 28352 of the Labor Code, as amended, is recognized as an
authorized cause for the dismissal of an employee from the service.
- For a valid retrenchment, three requisites must concur, as provided in Article 283 of
the Labor Code, as amended, namely: (1) The retrenchment is necessary to prevent
losses and the same is proven; (2) Written notice to the employees and to the DOLE at
least one month prior to the intended date thereof; and (3) Payment of separation
pay equivalent to one month pay or at least month pay for every year of service,
whichever is higher.
- We observe that the Court of Appeals, in finding that petitioners suffered from financial
losses and justifying the separation of respondent from the service, relied on the audited
reports prepared by Sycip Gorres Velayo & Co. Such reliance is in order.
- But respondent insists that actual, not probable losses, justify retrenchment. Article 283
entails, among others, only a situation where there is retrenchment to prevent
losses. The phrase to prevent losses means that retrenchment or termination from the
service of some employees is authorized to be undertaken by the employer sometime
before the losses anticipated are actually sustained or realized. This is the situation in
the case at bar. Evidently, actual losses need not set in prior to retrenchment.
2. NO
- Records show that on December 3, 1998, petitioners sent respondent and the DOLE
separate notices of retrenchment effective December 30, 1998. Following the provision
of Article 283, these notices should have been served one month before, or on
November 30, 1998. Clearly, petitioners failed to comply with the one-month notice
requirement
- Due process was not observed as the required notices were not sent to respondent and
the DOLE one month prior to the effectivity of his termination. Thus, petitioners should be
liable for violation of his right to due process and should pay him indemnity in the form of
nominal damages, pursuant to our ruling in Agabon, which we fix at P20,000.00.
3. YES
Reasoning
- It bears reiterating that under Article 283, in case of retrenchment to prevent losses,
respondent is entitled to an award of separation pay equivalent to one-half (1/2) months
52

Art. 283. Closure of Establishment and Reduction of Personnel. The employer may also terminate the employment
of any employee due to the installation of labor-saving devices, redundancy, retrenchment to prevent losses or the
closing or cessation of operations of the establishment or undertaking unless the closing is for the purpose of
circumventing the provisions of this Title, by serving a written notice on the worker and the Department of Labor and
Employment, at least one (1) month before the intended date thereof. x x x. In case of retrenchment to prevent
losses and in cases of closure or cessation of operations of the establishment or undertaking not due to serious
business losses or financial reverses, the separation pay shall be equivalent to one (1) month pay or at least one-half
(1/2) month pay for every year of service, whichever is higher. A fraction of at least six (6) months shall be considered as
one (1) whole year.

Labor Law 1

A2010

- 316 -

Disini

pay for every year of service.


Disposition AFFIRMED.

EFFECT INFLATION
LANTION V NLRC (MENESES)
181 SCRA 513
MELENCIO-HERRERA; January 29, 1990
NATURE
Petition for certiorari
FACTS
- Filomeno Lantion, Clarita C. Lantion, and Juana C. Fuentes were Acting Vice President
and Executive Officer, Dean of the Institute of Business and Agricultural Administration
and concurrently Head and Professor of the Department of Business and Finance, and
Secretary of the Chief Legal Officer respectively when Gregorio Araneta University
Foundation dismissed them in line with the universitys retrenchment and reorganization
program.
- Petitioners, who had served respondent university from 16 to 32 years, claim to have
been terminated from service discriminatorily, arbitrarily and illegally and had not been
rehired by private respondents contrary to their own guidelines and those set forth by
MOLE in the implementation of GAUF RRR program.
- The labor arbiter and the nlrc found that retrenchment was not illegal but ordered
payment of retirement and other benefits.
ISSUE
WON petitioners were illegally dismissed
HELD
YES
- That retrenchment was proper, therefore, there can be no question. The conditions laid
down, however, were not religiously followed. Petitioners were not rehired although they
fall outside the exception provided. Their positions were not affected by the reorganizational changes envisioned in the retrenchment program. The position of VicePresident continued to exist. And as far as Filomeno and Clarita Lantion are concerned,
their temporary appointment to other positions could not have affected their permanent
status pursuant to the ruling in the First GAUF Case. Clarita's position was neither
abolished. She was replaced by another faculty member.
- It may be that petitioners Filomeno and Clarita Lantion had expressed their conformity to
their termination, while Fuentes had tendered her courtesy resignation. As is obvious,
however, those steps were but in administrative compliance with the Memorandum
Circular of 14 October 1983 of the University, ante. As a matter of fact, courtesy
resignations could have been dispensed with as all personnel were deemed resigned.
Besides, such compliance had placed them in a better position than the Complainants in
the First GAUF Case considering the proviso in the Memo-Circular of the University that
"those who submit courtesy resignations may be re-appointed while those who would fail
to submit may be retrenched."
ON INFLATION
- In respect of the argument that the inflation that has supervened justifies the imposition
of interest, this Court has held that the effects of extraordinary inflation are not to be
applied without an agreement between the parties and without an official declaration
thereof by competent authorities
Disposition Decision of respondent National Labor Relations Commission is REVERSED
in so far as it holds that the dismissal of petitioners was not illegal and hereby ORDERS
respondent Gregorio Araneta University Foundation to REINSTATE petitioners to their
former positions with three (3) years backwages under the new terms and conditions of
employment in the University as reorganized.

14.10 FINANCIAL ASSISTANCE


ALLOWED FINANCIAL ASSISTANCE
PHIL LONG DISTANCE TELEPHONE CO V NLRC
[PAGE 18]

SALAVARRIA V LETRAN COLLEGE


296 SCRA 184
ROMERO; September 25, 1998
NATURE
Appeal from the decision of the NLRC reversing the decision of the labor arbiter and
dismissing the complaint for illegal dismissal.
FACTS
- Petitioner Anita Y. Salavarria was employed by respondent Colegio de San Juan de
Letran as a teacher in its High School Department. She was dismissed from her
employment because she allegedly collected money from her student which is in violation
of the schools rules and regulations. The students of the petitioner explained that the
project consisted in collecting contributions from each of them, which amount shall be
used to purchase religious articles such as bibles, chalice, crucifix and similar items to be
distributed among the several churches in Metro Manila and nearby rural areas.
Furthermore, they claimed that in doing so, it would involve them in charity work in
connection with their lesson on Love of God and Neighbor and that such activity would
entail a much lesser expense than the completion of the term papers. After continuous
prodding, petitioner was finally prevailed upon to accede to her students proposal. This
fact was later on discovered by the Administration.
- She filed a complaint for illegal dismissal. The labor arbiter ruled in her favor but upon
appeal by Letran the said decision of the labor arbiter was reversed by the respondent
NLRC. Although the complaint for illegal termination was dismissed, NLRC ordered
respondent school to pay the complainant severance compensation in the amount of Forty
Five Thousand Pesos (P45,000.00), equivalent to her salary for one month for every year
of her nine (9) years service in consideration of the equities of the case.
ISSUE
WON the award of financial assistance or severance compensation was proper
HELD
YES
- The award is proper based on equity and on the provisions of the Constitution regarding
the promotion of social justice and protection of the rights of the workers.
Ratio We rule that the NLRC correctly awarded to petitioner the amount of P45,000.00 as
severance pay which is synonymous with separation pay. As a general rule, an
employee who is dismissed for cause is not entitled to any financial assistance. However,
equity considerations provide an exception. In PLDT v. NLRC, equity has been defined as
justice outside law, being ethical rather than jural and belonging to the sphere of morals
than of law. It is grounded on the precepts of conscience and not on any sanction of
positive law, for equity finds no room for application where there is law. Further, it was
held that the grant of separation pay is not merely based on equity but on the provisions of
the Constitution regarding the promotion of social justice and protection of the rights of the
workers.
- As stressed in the recent cases of Santos v. NLRC and Camua v. NLRC, the Court
therein laid down the guidelines in the grant of separation pay to lawfully dismissed
employees, thus:
"We hold that henceforth separation pay shall be allowed as a measure of social justice
only in those instances where the employee is validly dismissed for causes other than
serious misconduct or those reflecting on his moral character. Where the reason for
the valid dismissal is, for example, habitual intoxication or an offense involving moral
turpitude, like theft or illicit sexual relations with a fellow worker, the employer may not
be required to give the dismissed employee separation pay, or financial assistance, or
whatever other name it is called, on the ground of social justice.
- As applied to the case at bar, we find that petitioners infraction of the school policy
neither amounted to serious misconduct nor reflected that of a morally depraved person
as may warrant the denial of separation pay to her. While the Court is not oblivious of the
fact that petitioner was already warned in a previous offense, certain circumstances,
however, impel us to award her separation pay equivalent to one month salary for every
year of service in respondent school.

GUSTILO V WYETH PHILIPPINES INC


[PAGE 10]

Labor Law 1
PINERO V NLRC
437 SCRA 112
YNARES-SANTIAGO; August 20, 2004
NATURE
Petition for review on certiorari
FACTS
- Private respondent Dumaguete Cathedral College, Inc., an educational institution, is the
employer of the faculty and staff members comprising the labor union DUCACOFSANAFTEU. On December 19, 1986, DUCACOFSA (then affiliated with the National Alliance
of Teachers and Allied Workers NATAW) and private respondent entered into a
Collective Bargaining Agreement (CBA) effective for 3 years. Upon the expiration of their
CBA in 1989, the parties failed to conclude another CBA which led DUCACOFSA (now
affiliated with NAFTEU) to file a notice of strike with the Department of Labor and
Employment (DOLE) on the ground of refusal to bargain.
- On November 4, 1991, DUCACOFSA-NAFTEU conducted a strike in the premises of
private respondent without submitting to the DOLE the required results of the strike vote
obtained from the members of the union. Consequently, on November 21, 1991, private
respondent filed with the DOLE a complaint to declare the strike illegal and to dismiss
some of its officers. On October 28, 1994, the Labor Arbiter rendered a decision declaring
the strike as illegal and declaring the respondent union officers to have lost their
employment status effective on the date of this decision. The union officers appealed to
the NLRC. Meanwhile, on November 29, 1991, the said officers returned to work by virtue
of a Memorandum of Agreement entered into by DUCACOFSA-NAFTEU and private
respondent allowing them to resume service without prejudice to the outcome of the
instant case then pending appeal with the NLRC.
- On December 19, 1995, the NLRC affirmed the decision of the Labor Arbiter.In addition
to the failure to comply with strike vote requirements, the NLRC ratiocinated that the strike
was illegal because DUCACOFSA-NAFTEU, not being a legitimate labor organization, has
no personality to hold a strike. The union officers filed a Motion for Reconsideration but
the same was denied.
- Petitioner Rosendo U. Piero filed with this Court a petition for certiorari which was
referred to the Court of Appeals. On March 25, 2001, the Court of Appeals affirmed the
decision of the NLRC dismissing the petition. On August 29, 2001, Pieros Motion for
Reconsideration was denied. Hence, the instant petition.
ISSUES
1. WON the strike should be declared illegal
2. WON Pinero is entitled to financial assistance
HELD
1. YES
- the strike staged by DUCACOFSA-NAFTEU is illegal for non-compliance with the strikevote requirements. The relevant provisions of Article 263 of the Labor Code read:
Article 263.
c.) the duly certified or recognized bargaining agent may file a notice of strike or the
employer may file a notice of lockout with the Department at least 30 days before the
intended date thereof. In cases of unfair labor practice, the period of notice shall be 15
days and in the absence of a duly certified or recognized bargaining agent, the notice of
strike may be filed by any legitimate labor organization in behalf of its members.
However, in case of dismissal from employment of union officers duly elected in
accordance with the union constitution and by-laws, which may constitute union busting
where the existence of the union is threatened, the 15-day cooling-off period shall not
apply and the union may take action immediately.
(f) A decision to declare a strike must be approved by a majority of the total union
membership in the bargaining unit concerned, obtained by secret ballot in meetings or
referenda called for that purpose. A decision to declare a lockout must be approved by
a majority of the board of directors of the corporation or association or of the partners in
a partnership, obtained by secret ballot in a meeting called for the purpose. The
decision shall be valid for the duration of the dispute based on substantially the same
grounds considered when the strike or lockout vote was taken. The Department may,
at its own initiative or upon the request of any affected party, supervise the conduct of
the secret balloting. In every case, the union or the employer shall furnish the
Department the results of the voting at least seven days before the intended strike or
lock-out, subject to the cooling-off period herein provided.
- Under the aforequoted provisions, the requisites for a valid strike are as follows: (a) a

A2010

- 317 -

Disini

notice of strike filed with the DOLE thirty days before the intended date thereof or fifteen
days in case of unfair labor practice; (b) strike vote approved by a majority of the total
union membership in the bargaining unit concerned obtained by secret ballot in a meeting
called for that purpose; (c) notice given to the DOLE of the results of the voting at least
seven days before the intended strike. These requirements are mandatory and failure of a
union to comply therewith renders the strike illegal.
- Pursuant to Article 264 of the Labor Code, any union officer who knowingly participates
in an illegal strike and any worker or union officer who knowingly participates in the
commission of illegal acts during a strike may be declared to have lost his employment
status. In the case at bar, DUCACOFSA-NAFTEU failed to prove that it obtained the
required strike-vote among its members and that the results thereof were submitted to the
DOLE. The strike was therefore correctly declared illegal, for non-compliance with the
procedural requirements of Article 263 of the Labor Code, and Piero properly dismissed
from service.
2. YES
- The Court notes that petitioner Piero turned 60 years old and retired on March 1, 1996
after 29 years of service, rendering his dismissal from service moot and academic.
However, in view of the propriety of his termination as a consequence of the illegal strike,
he is no longer entitled to payment of retirement benefits because he lost his employment
status effective as of the date of the decision of the Labor Arbiter October 28, 1994. An
employee who is dismissed for cause is generally not entitled to any financial assistance.
Equity considerations, however, provide an exception. Equity has been defined as justice
outside law, being ethical rather than jural and belonging to the sphere of morals than of
law. It is grounded on the precepts of conscience and not on any sanction of positive law,
for equity finds no room for application where there is law. Although meriting termination of
employment, Pieros infraction is not so reprehensible nor unscrupulous as to warrant
complete disregard of his long years of service. Moreover, he has no previous derogatory
records. Weighed on the scales of justice, conscience and reason tip in favor of granting
financial assistance to support him in the twilight of his life after long years of service.
Under the circumstances, social and compassionate justice dictate that petitioner
Piero be awarded financial assistance equivalent to one-half (1/2) months pay for
every year of service computed from his date of employment up to October 28, 1994
when he was declared to have lost his employment status. Indeed, equities of this
case should be accorded due weight because labor law determinations are not only
secundum rationem but also secundum caritatem.
Disposition The petition is partially granted. As modified, Rosendo U. Piero is awarded
financial assistance equivalent to one-half (1/2) months pay for every year of service
computed from his date of employment up to October 28, 1994.

EASTERN SHIPPING LINES INC V SEDAN


486 SCRA 565
QUISUMBING; April 7, 2006
NATURE
CERTIORARI
FACTS
- EASTERN hired on a per-voyage basis Sedan as 3 rd marine engineer and oiler in one of
the vessels it owned
- IN 1997, Sedan sent a letter to EASTERN applying for optional retirement, citing as
reason the death of his only daughter, hence the retirement benefits he would receive
would ease his financial burden.
- However, EASTERN deferred action on his application for optional retirement since his
services on board ship were still needed.
- Nonetheless, the company expressed intention to extend him a loan in order to defray
the costs incurred for the burial and funeral expenses of his daughter.
- But still Sedan insisted that he no longer wanted to continue working on board a vessel
for reasons of health.
- Sedan sent another letter to petitioners threatening to file a complaint if his application
was not granted.
- EASTERN ,in reply, sent a telegram on December 9, 1997 informing Sedan that his
services were needed on board a vessel and that he should report immediately for work
as there was no available replacement.
- Sedan claims he did not receive the telegram, nor was this fact proved by the company
before the LA or the NLRC.
- Sedan proceeded to file a complaint with the LA against EASTERN, demanding payment
of his retirement benefits, leave pay, 13th month pay and attorneys fees.
- LA ruled in favor of Sedan; NLRC affirmed and dismissed for lack of merit the MR.
- CA ruled that the retirement gratuity and attorneys fees awarded by the LA and the
NLRC had no basis in fact or law since pursuant to the Agreement between the company
and the employees, the granting of optional retirement is the exclusive prerogative of the
employer. Unless such prerogative was exercised arbitrarily or capriciously, private
respondent cannot demand it as a right.

Labor Law 1

A2010

- 318 -

Disini

- CA ordered EASTERN to pay SEDAN P200,000 as financial assistance. Denied Es MR.

- AS REGARDS WON the Php 200k granted him is arbitrary and excessive.

ISSUE
WON the court of appeals erred in giving the respondent php200,000.00 as financial
assistance when in fact it was the respondent who refused to report for work

- Philippine Long Distance Telephone Co. v. NLRC: separation pay shall be allowed as a
measure of social justice only in the instances where the employee is validly dismissed for
causes other than serious misconduct or those reflecting on his moral character. A
contrary rule, we said would have the effect of rewarding rather than punishing an erring
employee.
- Cited doctrines in NLRC cases, the SC found that the grant of P200,000 was neither
arbitrary nor excessive.
- Sedan has no derogatory record in his 23 years of service should be granted equitable
assistance equal to one-half months pay for each of his 23 years of service.
Disposition PETITION is DENIED. CAs DECISION is AFFIRMED.

HELD
NO
- SEDAN IS NOT ENTITLED TO RETIREMENT BENEFITS but is entitled to financial
assistance as an equitable concession under the special circumstances of this case.
EASTERNS CLAIM:
- SEDANs refusing to report for work and insisting on applying for optional retirement->
TANTAMOUNT TO VOLUNTARY RESIGNATION. Therefore, petitioners contend, the
respondent should not be entitled to any financial assistance.
- Granting arguendo that SEDAN was entitled to financial assistance, they protest the
amount of the financial assistance awarded by the CA for being disproportionately
excessive.
Reasoning
- PERTINENT LAW IN LC:
ART. 287. Retirement. Any employee may be retired upon reaching the retirement
age established in the collective bargaining agreement or other applicable employment
contract.
In case of retirement, the employee shall be entitled to receive such retirement benefits as
he may have earned under existing laws and any collective bargaining agreement and
other agreements: Provided, however, That an employees retirement benefits under any
collective bargaining and other agreements shall not be less than those provided herein.
In the absence of a retirement plan or agreement providing for retirement benefits of
employees in the establishment, an employee upon reaching the age of sixty (60)
years or more, but not beyond sixty-five (65) years which is hereby declared the
compulsory retirement age, who has served at least five (5) years in the said
establishment may retire and shall be entitled to retirement pay equivalent to at least
one half (1/2) month salary for every year of service, a fraction of at least six (6)
months being considered as one whole year.
xxx
The age of retirement is primarily determined by the existing agreement between the
employer and the employees. However, in the absence of such agreement, the
retirement age shall be fixed by law. Under the aforecited article of the Labor Code,
the legally mandated age for compulsory retirement is 65 years, while the set minimum
age for optional retirement is 60 years.
- In this case there was an agreement between EASTERN and its employees.
- The agreement states the retirement under the LC and the optional retirement where the
eligibility age for optional retirement is set at 60 years. However, employees of herein
petitioners who are under the age of 60 years, but have rendered at least 3650 days (10
years) on board ship or fifteen (15) years of service for land-based employees may also
avail of optional retirement, subject to the exclusive prerogative and sole option of
petitioner company.
- Records show that private respondent was only 48 years old when he applied for
optional retirement. Thus he cannot claim optional retirement benefits as a matter of right.
- His application for optional retirement was subject to the exclusive prerogative and sole
option of the shipping company pursuant to the abovecited agreement between the
workers and the company
AS REGARDS THE CAS GRANT of FINANCIAL ASSISTANCE
- Telefunken Semiconductors Employees Union-FFW v. Court of Appeals (2000): financial
assistance is allowed only in instances where the employee is validly dismissed for causes
other than serious misconduct or those reflecting on his moral character.
- Arc-Men Food Industries Corporation v. NLRC, and Lemery Savings and Loan Bank v.
NLRC: when there is no dismissal to speak of, an award of financial assistance is not in
order.
- Justice Sabino de Leon, Jr. IN TELEFUNKEN: financial assistance may be allowed as a
measure of social justice and exceptional circumstances, and as an equitable concession.
- The instant case equally calls for balancing the interests of the employer with those of
the worker, if only to approximate what Justice Laurel calls justice in its secular sense.
- In this instance, our attention has been called to the following circumstances:
that Sedan joined the company when he was a young man of 25 years and stayed on
until he was 48 years old; that he had given to the company the best years of his youth,
working on board ship for almost 24 years; that in those years there was not a single
report of him transgressing any of the company rules and regulations; that he applied
for optional retirement under the companys non-contributory plan when his daughter
died and for his own health reasons; and that it would appear that he had served the
company well, since even the company said that the reason it refused his application
for optional retirement was that it still needed his services; that he denies receiving the
telegram asking him to report back to work; but that considering his age and health, he
preferred to stay home rather than risk further working in a ship at sea.
- These circumstances indubitably merit equitable concessions, via the principle of
compassionate justice for the working class.

NOT ALLOWED
PHILIPPINE NATIONAL CONSTRUCTION CORP V
NLRC (MANREZA)
170 SCRA 207
GRINO-AQUINO; February 9, 1989
NATURE
Petition for review on certiorari of the decision of NLRC
FACTS
- Manreza was hired by CDCP. The North Luzon Expressway Security Services
Investigation and Intelligence Unit discovered flexbeams in the house of Eusebio and
Enriquez, who declared that Manreza deposited them there. Memo was sent to Manreza
asking him to explain within 48 hrs. Manreza said they were just temporarily deposited
there.
- Case was referred to Union which concluded that Manreza was merely negligent, and
recommended suspension. Another investigation was conducted. NLE administrative
officer found Manreza guilty of stealing or unauthorized taking of company property.
Manreza was placed under preventive suspension and later terminated. Manreza filed
complaint.
- Labor Arbiter ordered reinstatement. Petitioner appealed to NLRC, which said dismissal
was valid, but ordered payment of 1 month pay for every year of service, in spirit of
compassionate justice.
- Petition for certiorari was filed by PNCC, alleging abuse in discretion in awarding
separation pay despite finding of guilt of Manreza and just cause in dismissal.
ISSUES
WON award of separation pay is proper
HELD
NO
- While it is true that in earlier cases, We held that employees dismissed for cause are
nevertheless entitled to separation pay on the ground of social and compassionate justice,
that doctrine was abandoned in Philippine Long Distance Telephone Co. vs. NLRC and
Marilyn Bucay.
- Separation pay shall be allowed as measure of social justice only in instances where
employee is validly dismissed for causes other than serious misconduct or those reflecting
on his moral character. Where reason for valid dismissal is, for example, habitual
intoxication or offense involving moral turpitude, employer may not be required to give
separation pay or financial assistance, or whatever other name it is called, on the ground
of social justice.
- Social justice is not intended to countenance wrongdoing simply because it is committed
by the underprivileged. At best it may mitigate the penalty but it certainly will not condone
the offense. Social justice cannot be permitted to be the refuge of scoundrels any more
than can equity be an impediment to the punishment of the guilty. Those who invoke
social justice may do so only if their hands are clean and their motives blameless and not
simply because they happen to be poor.
- Since Manreza was found guilty of dishonesty for having stolen company property and
was dismissed for cause, he is not entitled to separation pay.

Labor Law 1
EASTERN PAPER MILLS INC V NLRC (MALABANAN)
170 SCRA 595
GRINO-AQUINO; February 24, 1989
NATURE
Petition for certiorari
FACTS
- Eduardo Malabanan, an accounts payable clerk, was dismissed on Jan 10, 1983 by
Eastern Paper Mills Inc after due notice, investigation, and hearing. He was dismissed for
having physically assaulted and verbally abused, within full view and hearing of the other
employees, Mariano Lopingco, who was then the personnel and administrative manager
of the company.
- The reason for the assault was Malabanan's resentment at being suspected of having
stolen an ash tray from Lopingco's office and being questioned about the matter by the
security office.
- Malabanan filed a complaint for illegal dismissal in the Ministry of Labor and
Employment. Labor Arbiter dismissed Malabanan's complaint and found that his dismissal
from employment was for just and valid cause. However, the Labor Arbiter awarded him
P10,000 as "financial assistance".
- On appeal by petitioner, NLRC affirmed the Labor Arbiter's decision with modification. It
awarded Malabanan separation or termination pay amounting to P10,780, in lieu of
financial assistance.
ISSUE
WON the lower court erred in granting separation pay to Malabanan.
HELD
YES
- Philippine Long Distance Telephone Company vs. NLRC: "Separation pay shall be
allowed as a measure of social justice only in those instances where the employee is
validly dismissed for cause other than serious misconduct or those reflecting on his moral
character. Where the reason for the valid dismissal is, for example habitual intoxication or
an offense involving moral turpitude, like theft or illicit sexual relations with a fellow worker,
the employer may not be required to give the dismissed employee separation pay, or
financial assistance, or whatever other name it is called, on the ground of social justice. x
x x A contrary rule would have the effect of rewarding rather than punishing the erring
employee for his offense."
- An award of separation pay to an employee who was dismissed for a valid cause is
legally indefensible. It contravenes Rule 1, Sec. 7, Book VI of the Omnibus Rules
Implementing the Labor Code:
"Sec. 7. Termination of Employment by Employer. The just causes for
terminating the services of an employee shall be those provided in Article 282 of
this Code. The separation from work of an employee for a just cause does not
entitle him to the termination pay, provided in the Code, without prejudice,
however, to whatever rights, benefits, and privileges he may have under the
applicable individual or collective agreement with the employer or voluntary
employer policy or practice."
- The only cases when separation pay shall be paid, although the employee was lawfully
dismissed, are when the cause of termination was not attributable to the employee's fault
but due to: (1) the installation of labor-saving devices, (2) redundancy, (3) retrenchment,
(4) cessation of the employer's business, or (5) when the employee is suffering from a
disease and his continued employment is prohibited by law or is prejudicial to his health
and to the health of his co-employees. (Articles 283 and 284, Labor Code.) Other than
these cases, an employee who is dismissed for a just and lawful cause is not entitled to
separation pay even if the award were to be called by another name.
Disposition petition for certiorari is granted. NLRC decision dismissing Malabanans
complaint for illegal dismissal is affirmed, but the award of separation pay or financial
assistance is set aside for lack of legal basis.

CHUA V NLRC
[PAGE 242]

A2010

- 319 -

Disini

14.11 SEPARATION PAY


WHEN ALTERNATIVE
COCA-COLA BOTTLERS PHILS V VITAL
438 SCRA 278
SANDOVAL-GUTIERREZ; September 13, 2004
NATURE
Petition for review on certiorari
FACTS
- June 1, 1980 - Dominic Vital was employed by Coca-Cola Bottlers Philippines, Inc. as
route driver/helper at its Antipolo Plant, with a monthly salary of P12,860.00. He was also
assigned to perform the duties of a salesman.
- In October 1995 Coca-Cola implemented a local marketing campaign called Operation
Rurok. This was a campaign that allows its trusted wholesaler outlets to retrieve foreign
empties and/or bottles of petitioners competitors, such as Pepsi Cola and Cosmos, from
regular customer outlets, in exchange for Coca-Cola containers and products.
- February 6, 1996 District Sales Supervisor Lagula authorized Vital to deliver, in
exchange for retrieved Pepsi-Cola and Cosmos empties or bottles, 57 cases of 12 oz.
Coca-Cola products to AMC Viray Store situated in Tambunting Street, Blumentritt. The
same went on in two other instances: to deliver to Coras Store in Quezon City and John
Uy in La Loma.
- October 10, 1996, - Petitioner sent respondent a notice of an investigation of its
complaint against him for forgery, fictitious sales transactions, falsification of company
documents, and unauthorized retrieval of empties. He was then placed on preventive
suspension. He was eventually terminated for lack of trust.
- Vital filed a complaint for illegal dismissal with the Labor Arbiter which was dismissed for
being devoid of merit. An appeal with the NLRC reversed the decision in Vitals favor.
ISSUE
WON Vital is entitled to separation pay
HELD
YES
Ratio Respondent who was illegally dismissed from work is entitled to reinstatement
without loss of seniority rights, full backwages, inclusive of allowances, and other benefits
or their monetary equivalent computed from the time his compensation was withheld from
him up to the time of his actual reinstatement
Reasoning
- The circumstances obtaining in this case do not warrant the reinstatement of
respondent. Antagonism caused a severe strain in the relationship between him and
petitioner company. A more equitable disposition would be an award of separation pay
equivalent to at least one month pay, or one month pay for every year of service,
whichever is higher, (with a fraction of at least six (6) months being considered as one (1)
whole year), in addition to his full backwages, allowances and other benefits.
- Records show that respondent was employed from June 1, 1980 to February 8, 1997, or
for sixteen (16) years and eight (8) months, with a monthly salary of P12,860.00. Hence,
he is entitled to a separation pay of P218,620.00.
Disposition The decision of the Court of Appeals is affirmed with modification.

GUSTILO V WYETH PHILIPPINES INC


[PAGE 10]
NATIONAL FEDERATION OF LABOR V CA
CALLEJO SR; October 19, 2004
NATURE
Petition for review of the Decision of the CA
FACTS
- National Federation of Labor (NFL) was the duly registered bargaining agent of the dailyand-monthly-paid rank-and-file employees of Sime Darby Pilipinas, Inc. (SDPI) in the
Latuan rubber plantation. SDPI and NFL executed a CBA in which they agreed that in
case of permanent or temporary lay-off, workers affected would be entitled to termination
pay as provided by the Labor Code.

Labor Law 1
- RA 6657, otherwise known as the Comprehensive Agrarian Reform Law, took effect.
SDPI decided to cease operation of the rubber plantation, and served formal notices of
termination to all the employees of the plantation. Separation pay for the employees was
computed pursuant to the provisions of the CBA. The NFL informed SDPI that the union
members would refuse to receive the computed separation pay if less than that previously
given to employees whose employment had been terminated by SDPI on prior dates
pursuant to the company policy, more specifically separation pay equivalent to 1 month for
every year of employment of the employees.
- Each of the petitioners received his separation pay equivalent to month pay for every
year of service, and other benefits which were all lumped in one check. They
simultaneously executed individual Release and Quitclaim following the explanation to
them by the Labor Arbiter of the nature and legal effects of the said quitclaims. The Labor
Arbiter also assured that each of the petitioners executed his respected deed of quitclaim
voluntarily.
- Petitioners filed a complaint for illegal dismissal, deficiency in separation pay,
backwages, reinstatement, legal interest, moral damages, exemplary damages, attorneys
fees, and cost of litigation before the NLRC. The Labor Arbiter dismissed the complaints
for lack of merit. The NLRC and CA subsequently affirmed.
ISSUE
WON petitioners are only entitled to separation pay equivalent to month pay for every
year of employment with the private respondent
HELD
YES
Ratio While it is true that quitclaims are frowned upon the in labor claims, this holds true
only when the consideration therefor is unconscionably low. Where the consideration is
substantial, the efficacy and validity thereof has been upheld, more so where the quitclaim
was voluntarily and willingly executed.
Reasoning
- The parties did not incorporate in the CBA a specific provision providing that employees
terminated from employment due to the closure of business operations would be entitled
to separation pay equivalent to 1 month pay for every year of service. The parties opted
to be bound by the provisions of the Labor Code and not by company policy. Had the rankand-file employees deemed the same to be a diminution of their benefits, they should
have rejected the CBA. Art. 283 should apply.
Obiter
- In cases of closures or cessation of operations of establishment or undertaking not due
to serious business losses or financial reverses, the separation pay of employees shall be
equivalent to 1 month pay or to at least month pay for every year of service, whichever
is higher. In no case will an employee get less than 1 month separation pay if the
separation from the service is due to the above stated causes, provided that he has
already served for at least 6 months. Thus, if an employee had been in the service for at
least 6 months, he is entitled to a full months pay as his termination pay if his separation
from the job is due to any of the causes enumerated above. However, if he has to his
credit 10 years of service, he is entitled to 5 months pay, this being higher than one-month
pay. Stated differently, the computation of termination pay should be based on either 1
month or month pay, whichever will yield to the employees higher separation pay,
taking into consideration his length of service.
Disposition The petition is DENIED. The decision and resolution of the CA are
AFFIRMED.

PHESCHEM INDUSTRIAL CORP V MOLDEZ


[PAGE 293]
ETCUBAN V SULPICIO LINES
448 SCRA 516
CALLEJO; January 17, 2005
NATURE
Petition for certiorari
FACTS
- Etcuban was the chief purser of M/V Surigao Princess. It was his responsibility, among
other things, to issue passage tickets and to receive payments from the customers of the
respondent, as well as to issue the corresponding official receipts therefore.
- IN a surprise examination, it was discovered that several yellow passengers duplicate
original of yet to be sold or unissued passage tickets already contained the amount of
P88.00 the fare for adult passengers for the Cagayan de Oro to Jagna, Bohol route. He
noticed that three other original copies which made up the full set did not bear the same
impression, although they were supposed to have been prepared at the same time. Acting
on what appeared to be a strong evidence of short-changing the company, the jefe de

A2010

- 320 -

Disini

viaje dug deeper on what he uncovered. As expected, he found inordinate amount of ticket
issuances for children at half the fare of P44.00 in Voyage 434 of the vessel.
- A memorandum was issued to Etcuban, asking him to explain the alleged anomalies. He
refused to acknowledge the memo. He was put under preventive suspension and an
investigation was conducted. He also was replaced.
- Etcuban filed a complaint for illegal dismissal and payment of benefits before the NLRC.
The latter ruled for the Etcuban. The CA reversed.
ISSUES
1. WON the petitioner was validly dismissed
2. WON he should be granted separation pay
HELD
1. YES
- The degree of proof required in labor cases is not as stringent as in other types of cases.
It must be noted, however, that recent decisions of this Court have distinguished the
treatment of managerial employees from that of rank-and-file personnel, insofar as the
application of the doctrine of loss of trust and confidence is concerned. Thus, with respect
to rank-and-file personnel, loss of trust and confidence as ground for valid dismissal
requires proof of involvement in the alleged events in question, and that mere
uncorroborated assertions and accusations by the employer will not be sufficient. But as
regards a managerial employee, the mere existence of a basis for believing that such
employee has breached the trust of his employer would suffice for his dismissal. Hence, in
the case of managerial employees, proof beyond reasonable doubt is not required, it being
sufficient that there is some basis for such loss of confidence, such as when the employer
has reasonable ground to believe that the employee concerned is responsible for the
purported misconduct, and the nature of his participation therein renders him unworthy of
the trust and confidence demanded by his position.
- In the case at bar, the petitioner is not a rank and file employee. While, indeed, it was not
proved that he was the one who made the irregular entries on the tickets, the fact that he
did not lift a finger at all to determine who it was is a sad reflection of his job. In fact, even
if the petitioner had no actual and direct participation in the alleged anomalies, his failure
to detect any anomaly in the passage tickets amounts to gross negligence and
incompetence, which are, likewise, justifiable grounds for his dismissal. Be that as it may,
to our mind, it is no longer necessary to prove the petitioners direct participation in the
irregularity, for what is material is that his actuations were more than sufficient to sow in
his employer the seed of mistrust and loss of confidence.
2. NO
- Anent the petitioners request for separation pay, the Court is constrained to deny the
same. Well-settled is the rule that separation pay shall be allowed only in those instances
where the employee is validly dismissed for causes other than serious misconduct or
those reflecting on his moral character. Inasmuch as reason for which the petitioner was
validly separated involves his integrity, which is especially required for the position of
purser, he is not worthy of compassion as to deserve at least separation pay for his length
of service
Disposition Petition denied

HANFORD PHIL INC V JOSEPH


454 SCRA 773
SANDOVAL-GUTIERREZ; March 31, 2005
NATURE
Petition for review on certiorari
FACTS
- Hanford Philippines, Inc. (Hanford - petitioner) hired Shirley Joseph as a sewer. Joseph
voluntarily tendered her resignation which petitioner accepted the day after. Petitioner then
paid respondent her last salary, 13th month pay and the cash conversion of her unused
vacation and sick leave.
- Respondent (Joseph) sent a letter to petitioner requesting payment of her separation pay
pursuant to Section 1, Article IV of the Collective Bargaining Agreement (CBA) quoted as
follows:
SECTION 1. Regular employees or workers separated by the COMPANY because of
reduction of personnel and employees or workers who may be separated without
cause, or those whose services are terminated or are separated from work due to
suspension or cessation of operation shall be entitled to a termination pay in
accordance with law. The COMPANY shall give termination pay to those who
voluntarily resign due to the reasons heretofore stated subject to the following terms
and conditions: a)
1 to 30 years of service shall be paid 20 days for every year of
service; b) 16 to 20 years of service to the COMPANY shall be paid 15 days pay for
every year of service; c) 11 to 15 years of service to the COMPANY shall be paid 10
days pay for every year of service; and d) 5 to 10 years of service to the COMPANY
shall be paid 5 days pay for every year of service.

Labor Law 1
- Labor Arbiter granted respondents petition and ordering petitioners to pay her separation
pay. NLRC affirmed. CA affirmed.
ISSUE
WON Joseph should be awarded separation pay
HELD
YES
- due to the stipulation of the CBA. As held in Hinatuan Mining Corporation and/or the
Manager versus National Labor Relations and Margo Batister, we held that while it is true
that under the Labor Code, an employee who voluntarily resigns may not be granted
separation pay, as in fact, the general rule is that an employee who voluntarily resigns is
not entitled to separation pay, however, there is an exception, that is, when it is stipulated
in the employment contract or CBA or such payment is authorized by the employers
practice or policy, as in this case. As aptly held by the Labor Arbiter, the NLRC and the CA,
it is very clear from the CBA that when an employee or worker voluntarily resigns due to,
among others, separation from the company without cause, such as voluntary
resignation, then he is entitled to a separation pay. Moreover, records show that
petitioners granted the employees mentioned earlier their separation pay upon their
separation by reason of their retirement. Under the Labor Code, retirement is not also a
ground for the grant of separation pay. If petitioners could be liberal to those employees
who retired, there is no reason why they should not also extend such liberality to
respondent considering that she served petitioner for twenty one years.
Philippine National Construction vs. NLRC finds application:
In the interpretation of an employers program providing for separation benefits, all
doubts should be construed in favor of labor. After all, workers are the intended
beneficiaries of such program and our Constitution mandates a clear bias in favor of
the working class.
Disposition petition is hereby DENIED

WHEN NOT ALLOWED


NORTH DAVAO MINING CORPORATION V NLRC
254 SCRA 721
PANGANIBAN; March 13, 1996
NATURE
Petition for certiorari
FACTS
- In May 1992, petitioner North Davao completely ceased operations due to serious
business reverses. From 1988 until its closure in 1992, North Davao suffered net losses
averaging P3B per year, for each of the five years prior to its closure. It was found that 5
months prior to its closure, its total liabilities had exceeded its assets by P20.4B, as shown
by its financial statements audited by the COA. When it ceased operations, its remaining
employees were separated and given the equivalent of 12.5 days' pay for every year of
service, computed on their basic monthly pay, in addition to the commutation to cash of
their unused vacation and sick leaves. However, it appears that, during the life of the
petitioner corporation, from the beginning of its operations in 1981 until its closure in 1992,
it had been giving separation pay equivalent to 30 days' pay for every year of service.
Moreover, inasmuch as the region where North Davao operated was plagued by
insurgency and other peace and order problems, the employees had to collect their
salaries at a bank in Tagum, Davao del Norte, some 58 km from their workplace and about
2.5 hours' travel time by public transportation; this arrangement lasted from 1981 to 1990.
- A complaint was filed with respondent Labor Arbiter by respondent Wilfredo Guillema and
271 other separated employees for: (1) additional separation pay of 17.5 days for every
year of service; (2) back wages equivalent to two days a month; (3) transportation
allowance; (4) hazard pay; (5) housing allowance; (6) food allowance; (7) postemployment medical clearance; and (8) future medical allowance, all of which amounted
to P58M as computed by private respondent.
- LA ruled in favor of private respondents. On appeal, respondent NLRC affirmed the
decision in toto. Petitioner North Davao's motion for reconsideration was likewise denied.
Hence, this petition.
Respondents Main Contentions: That the award of separation pay should be given,
based solely on petitioner North Davao's long-standing policy of giving separation pay
benefits equivalent to 30-days' pay, which policy had been in force in the years prior to its
closure. That by denying the same separation benefits to private respondent and the
others similarly situated, petitioners discriminated against them.
ISSUES

A2010

Disini

- 321 -

1. WON an employer whose business operations ceased due to serious business losses
or financial reverses is obliged to pay separation pay to its employees separated by
reason of such closure
2. WON the award of back wages and transportation allowance was proper
HELD
1. NO
Ratio The underscored portion of Art. 283 53 governs the grant of separation benefits "in
case of closures or cessation of operation" of business establishments "NOT due to
serious business losses or financial reverses." Said provision does not obligate an
employer to pay separation benefits when the closure is due to losses.
Reasoning
[a] Where the closure was due to business losses as in the instant case, in which the
aggregate losses amounted to over P20 billion the Labor Code does not impose any
obligation upon the employer to pay separation benefits, for obvious reasons. The
company's practice of giving one month's pay for every year of service could no longer be
continued precisely because the company could not afford it anymore. It was forced to
close down on account of accumulated losses of over P20 billion
[b] In this case, the basis for the claim of the additional separation benefit of 17.5 days is
alleged discrimination, i.e., unequal treatment of employees, which is proscribed as an
unfair labor practice by Art. 248 (e) of said Code. Under the facts and circumstances of the
present case, the grant of a lesser amount of separation pay to private respondent was
done, not by reason of discrimination, but rather, out of sheer financial bankruptcy, a fact
that is not controlled by management prerogatives. Stated differently, the total cessation of
operation due to mind-boggling losses was a supervening fact that prevented the
company from continuing to grant the more generous amount of separation pay. The fact
that North Davao at the point of its forced closure voluntarily paid any separation benefits
at all although not required by law and 12.5-days worth at that, should have elicited
admiration instead of condemnation.
2. YES
- The award of back wages and transportation allowance and the issued connected
therewith are factual, the determination of which is best left to the respondent NLRC. It is
well settled that this Court is bound by the findings of fact of the NLRC, so long as said
findings are supported by substantial evidence
- From the evidence on record, Court finds that the hours spent by complainants in
collecting salaries at a bank in Tagum, Davao del Norte shall be considered compensable
hours worked. Considering further the distance between Amacan, Maco to Tagum which is
2.5 hours by travel and the risks in commuting all the time in collecting complainants'
salaries, would justify the granting of backwages equivalent to 2 days in a month as
prayed for. Corollary to the above findings, and for equitable reasons, we likewise hold
respondents liable for the transportation expenses incurred by complainants at P40 round
trip fare during pay days.
- It is petitioners' burden or duty to present evidence of compliance of the law on labor
standards, rather than for private respondents to prove that they were not paid/provided by
petitioners of their backwages and transportation expenses.
Disposition Judgment MODIFYING assailed Resolution by SETTING ASIDE and deleting
the award for "additional separation pay of 17.5 days for every year of service", and
AFFIRMING it in all other aspects.

COMPUTATION
MILLARES V NLRC
[PAGE 79]

EFFECT OF ACCEPTANCE
ANINO V NLRC
[PAGE 9]

53

Art. 283. Closure of establishment and reduction of personnel. The employer may also terminate the employment of
any employee due to the installation of labor saving devices, redundancy, retrenchment to prevent losses or the closing
or cessation of operation of the establishment or undertaking unless the closing is for the purpose of circumventing the
provisions of this Title, by serving a written notice on the workers and the Ministry of Labor and Employment at least 1
month before the intended date thereof. In case of termination due to the installation of labor saving devices or
redundancy, the worker affected thereby shall be entitled to a separation pay equivalent to at least his 1 month pay or to
at least 1 month pay for every year of service, whichever is higher. In case of retrenchment to prevent losses and in
cases of closures or cessation of operations of establishment or undertaking not due to serious business losses or
financial reverses, the separation pay shall be equivalent to 1 month pay or at least 1/2 month pay for every year of
service, whichever is higher. A fraction of at least 6 months shall be considered 1 whole year.

Labor Law 1
LIABILITY OF CORPORATE OFFICERS

A2010

LIABILITY RULE
BOGO-MEDELLIN SUGARCANE PLANTERS ASSN INC
V NLRC
[PAGE 273]
NYK INDUSTRIAL V NLRC (PUBLICO)
397 SCRA 489
QUISUMBING; February 17, 2003
FACTS
- petitioner NYK hired respondent Virginia Publico as a sewer. Under the terms and
conditions of her employment, Publico was paid on a piece-rate basis, but required to
work from 8:00 A.M. to 12:00 midnight. On the average, she earned P185.00 daily
- Publico requested that she be allowed to leave the work place early, as she was not
feeling well due to a bout of influenza. Permission was refused but nonetheless, Publico
went home.
- The following day, Publico called up her employer and notified management that she was
still recovering from her ailment. When she came back to work, she was prevented from
entering the gates on managers orders. When she was finally allowed to, she sought to
talk to the owner Stephen Ng. she was asked to return the following day
- upon meeting him, she was informed that she was dismissed due to her refusal to do
overtime work. She filed a case for illegal dismissal. Corporation countered by saying that
her husband did not allow her to work at night. As night work is a must in their line of
business, particularly when there are rush orders, petitioners claimed that given Publicos
failure to render overtime work, they were left with no other recourse but to fire her. LA
held dismissal as illegal. This was affirmed by the NLRC.
- Appeal before the CA was outright dismissed. Court of Appeals pointed out that there
was non-compliance with Section 1 of Rule 65 of the 1997 Rules of Civil Procedure as the
petition was merely accompanied by a certified xerox copy of the assailed NLRC decision,
instead of a certified true copy thereof as required by the Rules of Court
ISSUE
1. WON CA commit a reversible error in dismissing appeal on purely technical grounds,
i.e., that the attached copy of the NLRC decision is a mere photocopy of the original
decision
2. WON Cathy Ng should be held liable for payment of backwages
HELD
1. NO
- Section 1 of Rule 65, 1997 Rules of Civil Procedure, requires that the petition shall be
accompanied by a certified true copy of the judgment or order subject thereof, together
with copies of all pleadings and documents relevant and pertinent thereto. In the present
case, the disputed document although stamped as certified true copy is not an
authenticated original of such certified true copy, but only a xerox copy thereof, in
contravention of paragraph 3 of the guidelines
- paragraph 5 of Administrative Circular No. 3-96, which provides:
5. It shall be the duty and responsibility of the party using the documents required by
Paragraph (3) of Circular No. 1-88 to verify and ensure compliance with all the
requirements therefor as detailed in the preceding paragraphs. Failure to do so shall
result in the rejection of such annexes and the dismissal of the case
- Petitioners are hereby reminded that the right to file a special civil action of certiorari is
neither a natural right nor a part of due process. A writ of certiorari is a prerogative writ,
never demandable as a matter of right, never issued except in the exercise of judicial
discretion. Hence, he who seeks a writ of certiorari must apply for it only in the manner
and strictly in accordance with the provisions of the law and the Rules
2. YES
- In A.C. Ransom Labor Union-CCLU v. NLRC, which held that since a corporation is an
artificial person, it must have an officer who can be presumed to be the employer, being
the person acting in the interest of the employer.
- In other words the corporation, in the technical sense only, is the employer. In a
subsequent case, we ordered the corporate officers of the employer corporation to pay
jointly and solidarily the private respondents monetary award. More recently, a
corporation and its president were directed by this Court to jointly and severally reinstate
the illegally dismissed employees to their former positions and to pay the monetary
awards.

- 322 -

Disini

- In this case Cathy Ng, admittedly, is the manager of NYK. Conformably with our ruling in
A. C. Ransom, she falls within the meaning of an employer as contemplated by the
Labor Code, who may be held jointly and severally liable for the obligations of the
corporation to its dismissed employees. Pursuant to prevailing jurisprudence, Cathy Ng, in
her capacity as manager and responsible officer of NYK, cannot be exonerated from her
joint and several liability in the payment of monetary award to private respondent

TAN V TIMBAL, JR.


434 SCRA 381
CALLEJO SR; July 14, 2004
NATURE
CERTIORARI
FACTS
- Restituto Timbal, Jr. and Ernesto Valenciano received a letter from their employer,
Nationwide Steel Corporation (NSC), through Conrado Tan, its general manager, informing
them that they were found to be among those employees who filed a complaint with the
Social Security System (SSS) in which they claimed that NSC was not remitting its
employees SSS premiums.
- Tan required the two to explain their side on the matter within 24 hours.
- After submitting their explanation, Timbal, Jr. and Valenciano were instructed by Tan to
report the following day for the resolution of the matter.
- However, when Timbal, Jr. and Valenciano were then suspended indefinitely.
- Aggrieved, Timbal, Jr. and Valenciano filed a complaint for illegal dismissal with the
NLRC, against NSC, and impleaded Conrado Tan as respondent, in his capacity as
general manager of the said corporation
- NSC claims: both complainants were indefinitely suspended as a result of the criminal
case filed by Benny Sy against them for their false charge.
- LA rendered judgment in favor of the complainants; ordered reinstatement.
- The decision became final and executory as no appeal from the decision was filed by
any of the parties.
- 1990, the LA issued a Writ of Execution directing the sheriff to effect the complainants
reinstatement and to collect from the respondent NSC the accrued backwages, and remit
the same to the complainants.
- The sheriff served a notice of garnishment on the Philippine Banking Corporation.
However, the Bank did not respond to the notice, and the decision of the labor arbiter
remained unsatisfied.
- Timbal et al filed an omnibus motion, praying that they be paid separation pay instead of
being reinstated, as part of the monetary award in their favor.
- They also prayed for the issuance of an alias writ of execution enforceable against the
NSC and its officers/stockholders.
- LA granted the motion ordered Tan et al to pay to the NSC, through the Office of the
Labor Arbiter, their unpaid subscribed capital stock that the same may be applied to satisfy
the complainants backwages, failing which, an alias writ of execution would be issued by
his Office against their assets.
- LA issued an alias writ of execution. Denied NSCs motion to set aside writ.
- Conrado Tan and William Ang filed with the NLRC a petition for the issuance of a writ of
preliminary injunction and a TRO to enjoin the implementation of the alias writ of execution
issued by the Labor Arbiter.
- They alleged that they were never furnished copies of the omnibus motion filed by
Timbal, Jr. and Valenciano; that they were not notified of any hearing on the matter; and,
that the Labor Arbiter acted in excess or lack of jurisdiction when he issued an alias writ of
execution ordering the sheriff to collect from the respondent NSC their unpaid
subscriptions.
- NLRC rendered a Decision granting the motion of Tan and Ang and set aside the
assailed order and alias writ of execution of the Labor Arbiter; denied Timbals MR.
- CA: relied onSt. Martin Funeral Homes vs. NLRC, affirmed the decision of the NLRC as
far as William Ang was concerned,but granted as to Tan; denied Tans MR.
ISSUE
WON TAN is liable, either jointly or severally with the NSC, for the monetary award in favor
of the Timbal
HELD
NO
- Only the NSC was found liable for the monetary awards in favor of the complainants
therein, including the herein respondent. The petitioner, although the general manager of
NSC, was not ordered to pay for the monetary award in favor of the complainants, jointly
or severally, with the NSC.
- The decision of the Labor Arbiter had become final and executory; hence, immutable.
- Industrial Management International Development Corporation vs. NLRC:

Labor Law 1
It is an elementary principle of procedure that the resolution of the court in a given
issue as embodied in the dispositive part of a decision or order is the controlling factor
as to settlement of rights of the parties. Once a decision or order becomes final and
executory, it is removed from the power or jurisdiction of the court which rendered it to
further alter or amend it. It thereby becomes immutable and unalterable and any
amendment or alteration which substantially affects a final and executory judgment is
null and void for lack of jurisdiction, including the entire proceedings held for that
purpose. An order of execution which varies the tenor of the judgment or exceeds the
terms thereof is a nullity.
- None of the parties in the case before the Labor Arbiter appealed the decision; hence the
same became final and executory.
- It was, therefore, removed from the jurisdiction of the Labor Arbiter or the NLRC to
further alter or amend it.
- Thus, the proceedings held for the purpose of amending or altering the dispositive
portion of the said decision are null and void for lack of jurisdiction.
- Not even the NLRC, the Court of Appeals and this Court has any appellate jurisdiction to
alter or reverse the decision of the Labor Arbiter.
- CA correctly cited ruling in MAM Realty Development Corporation vs. NLRC, that in labor
cases, corporate directors and officers are solidarily liable with the corporation for the
termination of employment of corporate employees committed with malice or bad faith.
The ruling applies in a case where a corporate officer acts with malice or bad faith in
suspending an employee. Whether or not the petitioner acted with malice or bad faith in
ordering the suspension of the respondent is a question of fact submitted by the parties to
the Labor Arbiter for resolution.
- In the instant case, the Labor Arbiter did not make any finding in his decision that the
petitioner acted with malice or bad faith in ordering the suspension of the respondent.
- Neither did he hold the petitioner liable, either jointly or severally with the NSC, for the
monetary award in favor of the complainants therein including the respondent herein.
- CA had no jurisdiction to delve into and resolve an issue already passed upon by the
Labor Arbiter with finality.
Disposition Petition is GRANTED. NLRCS DECISION is AFFIRMED.

ACESITE CORP V NLRC


[PAGE 303]

14.12 DAMAGES
MORAL/EXEMPLARY
COLEGIO SAN JUAN DE LETRAN-CALAMBA V VILLAS
[PAGE 285]
ASIA PACIFIC CHARTERING (PHILS) INC V FAROLAN
[PAGE 119]
VIERNES V NLRC
[PAGE 94]
TOLOSA V NLRC (QWANA KAIUN)

401 SCRA 391


PANGANIBAN; April 10, 2003
NATURE
Petition for certiorari assailing the CA decision denying petitioners motion without
prejudice to the right of herein petitioner to file a suit before the proper court, if she so
desires and the resolution denying MFR.

FACTS
- Evelyn Tolosa (hereafter EVELYN), was the widow of Captain Virgilio Tolosa (hereafter
CAPT. TOLOSA) who was hired by Qwana-Kaiun, through its manning agent, Asia Bulk
Transport Phils. Inc., (ASIA BULK for brevity), to be the master of the Vessel named M/V
Lady Dona. CAPT. TOLOSA had a monthly compensation of US$1700, plus US$400.00
monthly overtime allowance. His contract officially began on November 1, 1992, as

A2010

- 323 -

Disini

supported by his contract of employment when he assumed command of the vessel in


Yokohama, Japan. The vessel departed for Long Beach California, passing by Hawaii in
the middle of the voyage. At the time of embarkation, CAPT. TOLOSA was allegedly
shown to be in good health.
- During 'channeling activities' upon the vessel's departure from Yokohama sometime on
November 6, 1992, CAPT. TOLOSA was drenched with rainwater. The following day,
November 7, 1992, he had a slight fever and in the succeeding twelve (12) days, his
health rapidly deteriorated resulting in his death on November 18, 1992.
- Because of the death of CAPT. TOLOSA, his wife, EVELYN, as petitioner, filed a
Complaint/Position Paper before the POEA (POEA Case No. 93-06-1080) against QwanaKaiun, thru its resident-agent, Mr. Fumio Nakagawa, ASIA BULK, Pedro Garate and Mario
Asis, as respondents.
- After initial hearings and submissions of pleadings, the case was however transferred to
the Department of Labor and Employment.

- Labor Arbiter granted all the damages, (plus legal


interest), as prayed for by the petitioner:
1. US$176,400.00 (US$2,100.00 x 12 months x 7 years) or P4,586,400.00 (at P26.00
per US$1.00) by way of lost income;
2. interest at the legal rate of six percent (6%) per annum or P1,238,328.00 (from
November 1992 to May 1997 or 4 years);
3. moral damages of P200,000.00;
4. exemplary damages of P100,000.00; and
5. 10% of the total award, or P612,472.80, as attorney's fees.'
xxx
xxx
xxx
- NLRC set aside the decision and dismissed due to lack of jurisdiction. CA affirmed
saying that the cause of action is based on quasi-delict.
ISSUES
1. WON the NLRC has jurisdiction over the case.
2. WON the monetary awards have reached finality

HELD
1. NO
Ratio Not every dispute between an employer and employee involves matters that only
labor arbiters and the NLRC can resolve in the exercise of their adjudicatory or quasijudicial powers. The jurisdiction of labor arbiters and the NLRC under Article 217 of the
Labor Code is limited to disputes arising from an employer-employee relationship which
can only be resolved by reference to the Labor Code, other labor statutes, or their
collective bargaining agreement. (Georg Grotjahn GMBH & Co. v. Isnani)
Reasoning
- The pivotal question is whether the Labor Code has any relevance to the relief sought by
petitioner. From her paper, it is evident that the primary reliefs she seeks are as follows:
(a) loss of earning capacity denominated therein as "actual damages" or "lost income" and
(b) blacklisting. The loss she claims does not refer to the actual earnings of the deceased,
but to his earning capacity based on a life expectancy of 65 years. This amount is
recoverable if the action is based on a quasi delict as provided for in Article 2206 of the
Civil Code, but not in the Labor Code.
- Claims for damages under paragraph 4 of Article 217 must have a reasonable causal
connection with any of the claims provided for in the article in order to be cognizable by
the labor arbiter. Only if there is such a connection with the other claims can the claim for
damages be considered as arising from employer-employee relations. In the present case,
petitioner's claim for damages is not related to any other claim under Article 217, other
labor statutes, or collective bargaining agreements.
2. NO
Reasoning
- Petitioner contends that the labor arbiter's monetary award has already reached finality,
since private respondents were not able to file a timely appeal before the NLRC.
- Whether respondents were able to appeal on time is a question of fact that cannot be
entertained in a petition for review under Rule 45 of the Rules of Court. In general, the
jurisdiction of this Court in cases brought before it from the Court of Appeals is limited to a
review of errors of law allegedly committed by the court a quo.
Disposition the Petition is hereby DENIED, and the assailed Decision and Resolution
AFFIRMED.

MAQUILING V PHILIPPINE TUBERCULOSIS SOCIETY


INC

Labor Law 1
450 SCRA 465
TINGA; February 4, 2005
NATURE
Petition for Review on Certiorari of CA Decision of March 28, 2000 and its Resolution of
May 22, 2000, which reversed the decision of the NLRC dated December 15, 1997, and
that of Labor Arbiter of September 1993, which both found the dismissal from service of
Dr. Maquiling illegal
FACTS

- April 16, 1968: Dr. Maquiling (Dr. M)


was
employed
by
respondent
Philippine Tuberculosis Society (PTS)
- June 8, 1991- Dr. M was dismissed
from service as Deputy Executive
Director after serving PTS for 23 years.
He was then earning P13,900 monthly
salary
Dr.
M
filed
complaint
for
reinstatement or for payment of full
backwages and separation pay in
accordance with A279 of the LC, plus
moral and exemplary damages
- Records show that Dr. M received a
memo April 2, 1991 from PTS directing
him to submit written explanation for
1. Delayed GSIS remittances; 2.
Reported deficit of P7.3 m appearing in
their financial statement for 1990; 3.
Expenses incurred in connection with
the Dale Carnegie and Silva Mind
Control Seminar; 4. P3.7 m misc.
expenses; 5. reasons for renewing their
service contract with Ultra
- Dr. M submitted his explanatory
letter,
and
had
a
30
minute
conversation
with
OIC-executive
director Soriano.
- No further related proceedings were
undertaken before Dr. M received a
letter on June 8, 1991 calling for his
dismissal
effective
immediately,
without any retirement benefits
Dr. M continued reporting for work
despite his dismissal
- July 17, 1991- Dr. M protested nonpayment of his salary
- September 1991- Dr. M stopped
reporting for work, filed his complaint

A2010

- 324 -

Disini

with Labor Arbiter the next month.


- the Labor Arbiter rendered a decision
ordering PTS to immediately reinstate
Dr. Maquiling to the position of Deputy
Executive Director or its equivalent in
rank and pay,
without loss of
seniority rights inclusive of all benefits
attached to said position at the time of
his dismissal, and to pay Dr. Maquiling
backwages computed from the time of
his dismissal on 7 June 1991 until his
actual reinstatement
- Upon appeal by PTS to the NLRC, the
Commission upheld the decision of the
labor arbiter and dismissed the
appeal.However, PTS appealed the
decision to the Court of Appeals which
reversed the decisions of the NLRC
and Labor Arbiter by ordering the
dismissal of the complaint and
declaring that his dismissal from
employment as legal and valid.
It,
however, ordered PTS to pay Dr.
Maquiling the amount of ten thousand
pesos (P10,000.00) as damages or
indemnity for violation of his right to
procedural
due
process
and
separation pay in the amount of one
hundred fifty-nine thousand eight
hundred fifty pesos (P159,850.00) in
the interest of social justice.
ISSUES
1. WON Dr. M was dismissed for just cause
2. WON procedural due process was observed in effecting Dr. Ms dismissal
HELD
1. YES
Reasoning
- Maquiling was dismissed from employment for just cause consisting of loss of trust and
confidence. The records reveal that he was Deputy Executive Director of PTS, a
responsible position, at the time of his dismissal.
- PTS imputes the delayed GSIS remittances to Dr. Maquilings failure to follow his duties
as prescribed by law. The records disclose that Dr. Maquiling was aware of the problem
but he failed to give priority thereto.
- SC attributes the P7.3 million deficit in PTS 1990 financial statements to Dr. Maquilings
failure to consider the realities of the financial condition of the institution. Dr. Maquiling
even aggravated such omission by insisting on the salary increase of both managerial and
non-managerial personnel despite the financial conundrum that puzzles the future fiscal
stability of PTS. The records show that he made representations during the Board
meeting that sufficient funds existed to meet the salary upgrading despite the presence of
financial strains. Such a course of action falls short of his responsibility to safeguard the
financial stability of the institution he leads.
2. NO

Labor Law 1
Reasoning
- two notices must be sent to the employee who is the subject of an investigation for acts
which may warrant his eventual dismissal from employment. The notices required before
an employee may be validly dismissed are: (a) a written notice served on the employee
specifying the grounds for termination and giving the employee reasonable opportunity to
explain his/her side; (b) a hearing or conference wherein the employee, with the
assistance of counsel if so desired, is given opportunity to respond to the charge, present
his evidence or rebut evidence presented against him/her; and (c) written notice of
termination served on the employee indicating that upon due consideration of all the
circumstances, grounds have been established to justify termination. The requirement of
notice is intended to inform the employee concerned of the employers intent to dismiss
and the reason for the proposed dismissal; upon the other hand the requirement of
hearing affords the employee an opportunity to answer his employers charges against
him and accordingly to defend himself therefrom before dismissal is effected.
- The first notice must inform outright the employee that an investigation will be conducted
on the charges particularized therein which, if proven, will result to his dismissal. Such
notice must not only contain a plain statement of the charges of malfeasance or
misfeasance but must categorically state the effect on his employment if the charges are
proven to be true.
- This notice will afford the employee an opportunity to avail all defenses and exhaust all
remedies to refute the allegations hurled against him for what is at stake is his very life
and limb his employment. Otherwise, the employee may just disregard the notice as a
warning without any disastrous consequence to be anticipated. Absent such statement,
the first notice falls short of the requirement of due process. The Labor Arbiter, the NLRC
and the Court of Appeals all agree in concluding that procedural due process in the instant
case was not observed. As revealed by the evidence on record, a confidential
memorandum FN dated 2 April 1991 was sent to Dr. Maquiling by Soriano requiring him to
explain in writing the matters contained therein. The text of the memorandum reads as
follows:
02 April 1991
CONFIDENTIAL MEMORANDUM FOR: DR. ERNESTO I. MAQUILING
Pursuant to the directive of the Board of Directors issued in its meeting on March 25,
1991, you are hereby instructed to report and explain in writing to this office, within five
(5) days from notice hereof, on the following matters:
The delayed GSIS remittances;
The reported deficit of P7.3 million appearing in our financial statement for 1990;
The expenses you approved and incurred in connection with the Dale Carnegie and
Silva Mind Control Seminar;
The P3.7 million miscellaneous expenses appearing in our financial statement; and
Your reasons for renewing our service contract with Ultra.
For immediate compliance.
(SGD.) ATTY. ANDRES B. SORIANO
OIC-Executive Director

- On 11 April 1991, Dr. Maquiling


submitted his written reply. The second
notice which informs Dr. Maquiling of
the
decision
to
terminate
his
employment was sent to him on 8 June
1991. It must be noted that the first
notice dated 2 April 1991 is a mere
instruction to explain the matters
enumerated therein. It did not apprise
Dr. Maquiling of any investigation to be
conducted or being conducted that will
warrant his dismissal from service if
found guilty of charges specified
therein. Thus, such notice fell short of
the requirement of law that an
employee must be afforded the benefit
of the two-notice rule in dismissal
cases that will allow the employee to
substantiate the charges specified in

A2010

Disini

- 325 -

the notice with full knowledge at the


outset that the investigation to be
conducted may result in his dismissal
or suspension from employment.
Disposition the Decision of the Court
of Appeals dated 28 March 2000 is
hereby MODIFIED pursuant to the
Agabon
ruling
as
the
latest
jurisprudential rule on the matter. For
the dismissal from employment of Dr.
Maquiling with a just cause but
without observing procedural due
process, PTS is ORDERED to pay Dr.
Maquiling nominal damages in the
amount of thirty thousand pesos
(P30,000.00).
KAY PRODUCTS INC V CA
[PAGE 312]
ACUNA V CA
[PAGE 12]
ACESITE CORP V NLRC
[PAGE 303]
SAGUM V CA
[PAGE 304]

NOMINAL DAMAGES
CENTRAL LUZON CONFERENCE V CA
466 SCRA 711
AZCUNA; August 12, 2005
FACTS
- Respondent Federico Cabanit was a sales representative of petitioner Central Luzon
Conference Corporation of Seventh-Day Adventist Church, Inc. After six months, he
became its regular employee, assigned to the accounting department. Promoted, he
became branch manager from 1981 to 1990, and auditor from 1990 to 1996.
- On June 11, 1997, respondent Cabanit was informed that he erred in recording US$40
and was later suspended from July 1-31, 1997 but the suspension was rescinded and he
was assigned as general auditor.
- Subsequently, he was required to appear before petitioner corporations office and its
Executive Committee pursuant to two letters. He was placed under preventive suspension
on October 16, 1997 and required to explain in 15 days why he should not be dismissed
due to irregularities committed. He requested copies of pertinent documents to enable
him to explain his side but petitioner corporation allegedly did not oblige. On November
18, 1997, petitioner corporation, through an EXECOM meeting, adopted a resolution
terminating his employment effective October 16, 1997.
- Federico Cabanit filed a complaint for illegal dismissal with the Arbitration Branch of the
National Labor Relations Commission (NLRC).
- The Labor Arbiter (LA) dismissed the complaint for lack of merit.
- On appeal, the NLRC affirmed the decision of the LA.
- On petition to the Court of Appeals (CA), the CA agreed that the dismissal was for a
just cause but found that therein respondents failed in the second requirement of
due process in that the employee was not given a chance to explain his side.
- The CA ordered payment of backwages from the time of termination to the time the
decision becomes final and remanded the case to the LA for computation of backwages.

Labor Law 1
- Hence, this petition for review under Rule 45 of the Rules of Court, principally to question
the award of backwages.
ISSUE
WON (as in the case of Agabon), Cabanit is entitled only to nominal damages
HELD
YES
- The violation of the petitioners right to statutory due process by the private respondent
warrants the payment of indemnity in the form of nominal damages. The amount of such
damages is addressed to the sound discretion of the court, taking into account the
relevant circumstances (Savellano v. Northwest Airlines, G.R. No. 151783, 8 July 2003,
405 SCRA 416). Considering the prevailing circumstances in the case at bar, we deem it
proper to fix it at P30,000.00. We believe this form of damages would serve to deter
employers from future violations of the statutory due process rights of employees. At the
very least, it provides a vindication or recognition of this fundamental right granted to the
latter under the Labor Code and its Implementing Rules
Disposition petition is GRANTED, and the decision of the Court of Appeals is MODIFIED,
by deleting the award of backwages and granting only P30,000 as nominal damages.

SECTION 15: RETIREMENT


15.01 RETIREMENT
DEFIINITION
ARIOLA V PHILEX MINING CORP
[PAGE 205]

TYPES
GERLACH V REUTERS
448 SCRA 335
SANDOVAL-GUTIERREZ; January 17, 2005
FACTS
- On February 15, 1982, Reuters Limited, Phils. (Reuters), a company engaged in news
dissemination with offices worldwide, hired Marilyn Odchimar Gerlach, petitioner, as its
local correspondent. On October 1, 1983, Reuters implemented a local Retirement Benefit
Plan (Plan) for its Philippine-hired employees. The Plan is funded by the company, but an
employee-participant may volunteer to contribute a percentage of his basic monthly salary
to the fund. Gerlach was automatically covered by the Plan by reason of her age and
length of service. However, she opted not to contribute to the fund. She worked in
Reuters Philippines up to December 23, 1983.
- On January 23, 1984, Gerlach was assigned as a journalist to Reuters Singapore. She
was informed that her home base will continue to be Manila and should she return there at
the end of this assignment or following subsequent assignments the terms and conditions
of employment would revert to those of local staff. She would be paid in Singaporean
dollars. However, she would still be included in the Retirement Benefit Plan based in the
Philippines using a notional salary in Philippine pesos. On March 26 to June 4, 1986,
petitioner was assigned to Reuters Hongkong and her actual salary was paid in HK
dollars. Thereafter, or in July, 1986, she was appointed correspondent in Sri Lanka where
her actual salary was in rupees and her peso notional salary was increased to P12,600
per month.
- On October 12, 1988, she was directed to return to Manila and resume her post by
December 15, 1988. However, she requested to be assigned to the Reuters Office either
in Bonn, West Germany or in London. But due to the worldwide reduction of personnel,
respondent denied her request. She then applied for a 14-month study leave to take up
economic subjects at Bonn University. Reuters approved her request for a 14-month leave
without pay from January 1, 1989 up to March 1, 1990. On May 20, 1990, petitioner
resigned from Reuters
- On March 1, 1991, petitioner received her retirement benefits under the Plan in the
amount of P79,228 as determined by the trustee bank (Bank of the Philippine Island) in
accordance with the provisions of the Plan. The computation was based on her notional
salary. However, she questioned the amount she received as well as her entitlement to a
disturbance grant, contending that her retirement benefits must be computed on the basis
of her actual salary abroad, not on her notional salary.

A2010

- 326 -

Disini

- The Labor Arbiter ordered Reuters to pay Gerlach additional retirement benefits in the
sum of P436,000, which amount was based on her actual salary abroad, not on her
notional salary; a disturbance grant in the sum of Stg 1,750 or its equivalent in pesos; and
attorneys fees. After a series of remands, MFRs and reversals, the NLRC ruled in favor of
Gerlach. The CA ruled for Reuters with the modification that Gerlach be paid her
disturbance and resettlement grant.
ISSUE
WON the computation of petitioners retirement benefits should be based on her basic
annual salary while stationed abroad

HELD
NO
Reasoning There are three kinds of retirement schemes. The first type is compulsory and
contributory in character. The second type is one set- up by agreement between the
employer and the employees in collective bargaining agreements or other agreements
between them. The third type is one that is voluntarily given by the employer, expressly as
in an announced company policy or impliedly as in a failure to contest the employee's
claim for retirement benefits. It is this third type of retirement scheme which covers
respondents Plan.
- Article 287 of the Labor Code reads:
Retirement. Any employee may be retired upon reaching the retirement age
established in the collective bargaining agreement or other applicable employment
contract.
In case of retirement, the employee shall be entitled to receive such retirement benefits
as he may have earned under existing laws and any collective bargaining agreement
and other agreements.
- The first paragraph of the above provisions deals with the retirement age of an employee
established in (a) a collective bargaining agreement or (b) other applicable employment
contract. The second paragraph deals with the retirement benefits to be received by a
retiring employee which he may have earned under (a) an existing law, (b) a collective
bargaining or (c) other agreements.
- Article 287 does not in itself purport to impose any obligation upon employers to set up a
retirement scheme for their employees over and above that already established under
existing laws, like the Social Security Act. Nonetheless, Section 14(a), Rule 1 of the Rules
and Regulations Implementing Book VI of the Labor Code, provides:
Retirement benefits. (a) An employee who is retired pursuant to a bona fide
retirement plan or in accordance with the applicable individual or collective agreement
or established employer policy shall be entitled to all the retirement benefits provided
therein . . ."
- Thus, Reuters based Gerlachs retirement benefits on its Plan and established policy,
which is in accord with the above provision. Consequently, Gerlachs theory that the
computation of her retirement benefits should be based on her basic annual salary while
stationed abroad is untenable. Her retirement benefits must be based on her notional
Philippine salary. It is very clear that from the very start of her first assignment overseas,
Reuters apprised her that the companys contribution to the Plan is based on her notional
Philippine salary. In fact, under the Plan, the companys contribution to the fund is 10% of
the basic monthly salary of each participant. Reuters also informed her of the amount of
her notional Philippine salary whenever she was transferred to her next overseas
assignment or when there were increases in her salary, both actual and
notional. Significantly, Reuters was able to prove that it has been its practice worldwide
that the notional salary of an employee is its basis in computing its contribution to the
retirement plan for a local employee detailed abroad. It follows that the amount of
retirement benefits of a retiring employee assigned abroad is based on his notional salary.
- Besides, it is a basic rule in evidence that the burden of proof is on the part of the party
who makes the allegations ei incumbit probatio, qui dicit, non qui negat.
Disposition Petition is DENIED. Decision of the CA is AFFIRMED.

BASIS
AQUINO V NLRC (OTIS ELEVATOR CO)

Labor Law 1
206 SCRA 118
CRUZ; February 11, 1992
NATURE
Petition for review from the decision of the NLRC
FACTS
- Petitioners services were terminated in line with the need of the company to streamline
its operations, consolidate certain functions, reduce its manpower and cut non-essential
spending. They received due notice thereof.
- They received separation pay based on their CBA and which is double than required by
the Labor Code.
- They demanded retirement benefits, invoking the Retirement Plan of Otis in their CBA (A
participant who is terminated from employment and who has rendered at least 10 years of
service shall be entitled to receive in lump sum all or a apportion of his accrued benefit
credits as of his date of termination). They also cited the case of other co-employees
who were terminated on the ground of redundancy and who received separation pay and
retirement benefits.
- Otis argued that separation pay and retirement benefits were mutually exclusive.
- Labor Arbiter: petitioners were entitled to the retirement plan. He said Otis is now
estopped, having given retirement benefits to other workers.
- NLRC reversed labor arbiters decision.
ISSUE
WON petitioners are still entitled to the retirement benefits
HELD
YES
- The petitioners are covered by the Retirement Plan because they have contributed to the
retirement fund, have been separated by reason of retrenchment, and have served the
company for more than the prescribed minimum period of ten years.
- Separation pay is required in the cases enumerated in Arts. 283 and 284 of the Labor
Code, which include retrenchment, and is computed at at least one month salary or at the
rate of one-half month salary for every year of service, whichever is higher. It is a statutory
right designed to provide the employee with the wherewithal during the period that he is
looking for another employment.
- Retirement benefits, where not mandated by law, may be granted by agreement of the
employees and their employer or as a voluntary act on the part of the employer. They are
intended to help the employee enjoy the remaining years of his life, lessening the burden
of worrying for his financial support, and are a form of reward for his loyalty and service to
the employer.
- The CBA and the Retirement Plan has no specific prohibition against the payment of
both benefits to the employee.
- Otis bases its arguments on Book VI, Sec. 14, Rule 1, of the Omnibus Rules of
Implementing the Labor Code, which provides:
(a) An employee who is retired pursuant to a bonafide retirement plan or in accordance
with the applicable individual or collective agreement or established employer policy
shall be entitled to all the retirement benefits provided therein or to termination pay
equivalent to at least one-half month salary for every year of service, whichever is
higher, a fraction of at least 6 months being considered as one whole year. xxx
(c) This Section shall apply where the employee retires at the age of sixty years or
more.
- However, Otis has not shown that petitioners were 60 years or older at the time of their
separation.
- If the private respondent really intended to make the separation pay and the retirement
benefits mutually exclusive, it should have sought inclusion of the corresponding provision
in the Retirement Plan and CBA so as to remove the possible ambiguity regarding this
matter.
- Bargaining is a process where the parties discuss their demands and counter-demands
and, after haggling, agree on what is essentially a compromise reflecting the concessions
mutually given by the parties to arrive at a common understanding. The resultant contract
provides for demandable rights, not withdrawable doles. When the employer signs a CBA,
it recognizes the rights of the workers and does not merely concede certain privileges to
them out of the goodness of its heart.
- Petitioners are entitled to this amount under the provisions of the CBA and the
Retirement Plan freely entered into by the parties. These instruments are binding
agreements, not being contrary to law, morals, good customs, public order or public policy,
and must therefore be upheld.
Disposition Petition is granted. NLRC decision reversed.

GAMOGAMO V PNOC SHIPPING AND TRANSPORT


CORP
[PAGE 38]

A2010 - 327 INTERPRETATION

Disini

LOPEZ V NATIONAL STEEL CORP


423 SCRA 109
SANDOVAL-GUTIERREZ; February 16, 2004
NATURE
Petition for review on certiorari of the decision and resolution of the Court of Appeals
FACTS
- National Steel did 2 projects, and Lopez was one of those employed and trained.
National Steel suffered substantial losses and adopted streamlining. It issued memo
announcing retrenchment. Respondent terminated petitioners services.
- Lopez, after accepting separation benefits, signed Release and Quitclaim. Eventually,
she filed with Labor Arbiter complaint for payment of retirement benefits.
- LA dismissed complaints. NLRC affirmed. CA affirmed.
ISSUES
1. WON Lopez is entitled to retirement benefits
2. WON the quitclaim is valid
HELD
1. NO
- While retirement laws are liberally construed in favor of persons intended to be benefited,
such interpretation cannot be made in light of clear lack of consensual and statutory basis
of the grant of retirement benefits to petitioner. There is no provision in the CBA
authorizing retirement benefits in addition to retrenchment pay. Also, petitioner has not yet
reached retirement age. Lastly, the companys retirement plan precludes employees
whose services were terminated for cause, from availing retirement benefits.
2. YES
- She was not forced to sign.

SALOMON V ASSOCIATE OF INTERNATIONAL


SHIPPING LINES INC
457 SCRA 254
SANDOVAL-GUTIERREZ; April 26, 2005
NATURE
Certiorari under Rule 45
FACTS
- The Association of International Shipping Lines, Inc. (AISI), is a corporation engaged in
the principal business of shipping and container and/or cargo services.
- As a result of a decline in the volume of cargo measuring activities and shipping
transactions, AISI suffered substantial financial losses equivalent to P213,583.00 in 1996;
P783,935.00 in 1997; and P1,334,729.00 in 1998.
- With this development, AISI adopted an organizational streamlining program that
resulted in the closure of its Measuring Department and retrenchment or termination from
the service of 17 workers. Among them were the abovementioned petitioners who
occupied booking coordinator and measurer positions.
- In separate letters AISI terminated petitioners services. Simultaneously, AISI filed with
the DOLE a Notice of Closure or report of petitioners retrenchment from the service.
- Aggrieved, petitioners filed with the National Conciliation and Mediation Board (NCMB) a
complaint for illegal dismissal and payment of retirement benefits against AISI.
- During the conciliation proceedings, AISI paid petitioners their retirement pay at the
rate of 1 month salary per year of service.
- Additionally, they received their leave credits, and pro-rated 13 th month pay. And after
having been paid their retirement pay, they executed and signed separate Releases and
Quitclaims. Consequently, the above case was considered closed and terminated.
- Surprisingly, petitioners filed with the LA a complaint for payment of retirement benefits,
damages and attorneys fees against AISI. They alleged that what each received was a
separation pay, not retirement benefits.
- LA dismissed the complaint.
- NLRC affirmed the LAs Decision. Denied MR.
- CA affirmed the resolutions of NLRC stating that their services were terminated due to
retrenchment undertaken by AISI and retrenchment is recognized as one of the authorized
causes for termination of employment under the Labor Code and that AISI complied with
the requirements of law with regard to retrenchment; Deneid MR.
PetitionersClaims:

Labor Law 1
- They invoke Sections 1 and 3 of the parties Collective Bargaining Agreement (CBA)
expressly providing that retirement benefits may be granted to them in addition to their
separation pay.
- They likewise call our attention to Aquino vs. NLRC holding that payment of separation
benefits does not exclude payment of retirement benefits.
Respondents Claims:
- AISI maintains that the parties CBA expressly prohibits the payment of retirement
benefits to employees terminated for cause. Thus, petitioners reliance on Aquino vs.
NLRC is misplaced. Moreover, they executed valid quitclaims.
ISSUE
WON the grant of retirement benefits to petitioners as shown in their quitclaims precludes
their availment of retirement benefits pursuant to their CBA
HELD
- While it is axiomatic that retirement laws are liberally construed in favor of the persons
intended to be benefited, however, such interpretation cannot be made in this case in
light of the clear lack of consensual and statutory basis of the grant of retirement
benefits to petitioner. (Philippine Scout Veterans Security & Investigation Agency, Inc.
vs. NLRC)
- The parties CBA provides:
Section 1. In case of termination due to redundancy, retrenchment, dissolution of a
department/conference/section and/or the whole ASSOCIATION, sickness or physical
disability, a regular employee shall be entitled to a separate pay equivalent to his one
(1) month basic pay for every year of service. A fraction of at least six (6) months shall
be considered as one (1) whole year and less than six (6) months shall be prorated
accordingly.
xxx
xxx
Section 3. Optional Retirement An employee shall have the option to retire
regardless of age provided he/she has rendered at least 15 years of continuous service
to the ASSOCIATION. An employee shall be entitled to the following benefits.
a. 15 to less than 20 years of service 50% of the monthly basic salary for every year
of service.
b. 20 years of service 100% of the monthly basic salary for every year of service.
- Obviously, petitioners, as prescribed by the parties CBA, are entitled only to either the
separation pay, if they are terminated for cause, or optional retirement benefits, if they
rendered at least 15 years of continuous services.
- Here, petitioners were separated from the service for cause. Consequently, pursuant
to the CBA, what each actually received is a separation pay. Accordingly and considering
their Releases and Quitclaims, they are no longer entitled to retirement benefits.
- It bears stressing that as held by the Labor Arbiter, the NLRC and the Court of Appeals,
there is no provision in the parties CBA authorizing the grant to petitioners of retirement
benefits in addition to their retrenchment pay; and that there is no indication that they were
forced by respondent to sign the Releases and Quitclaims.
- SC has always accorded respect and finality to the findings of fact of the CA, particularly
if they coincide with those of the LA and the NLRC when supported by substantial
evidence, as in this case.
- The reason for this is that quasi-judicial agencies, like the Arbitration Board and the
NLRC, have acquired a unique expertise because their jurisdictions are confined to
specific matters. (Cosmos bottling Corporation vs. NLRC)
Disposition Petition is DENIED. CAS DECISION AFFIRMED.

AGE
MAI PHILIPPINES INC V NLRC (NOLASCO)
151 SCRA 196
NARVASA; June 18, 1987
NATURE
Peition certiorari and prohibition to review the order and resolution of the NLRC
FACTS
- The decision in question was rendered on November 19, 1979, by Regional Director F.
Estrella. It declared illegal the dismissal by MAI Philippines, Inc. of its Customer
Engineering Manager Nolasco, and decreed his reinstatement with full back wages. The
decision was affirmed by the Labor Minister and in due course became final.
- MAI Philippines, Inc. (hereafter simply referred to as MAI) complied with it by paying
Nolasco P155,025.00 on December 15, 1981. It however declined to reinstate Nolasco
because there was no longer any "substantially equivalent position available." Instead, it
offered to give separation pay at the rate of one month for every year of service, an offer it
communicated to Nolasco in writing on November 24, 1981.

A2010

- 328 -

Disini

- On January 12, 1982, Nolasco brought suit in the Court of First Instance against MAI to
recover damages; 6 but on August 10, 1982, he presented a motion to dismiss his own
case, based on lack of jurisdiction, which the Court granted.
He then filed on August 16, 1982 a complaint in the Arbitration Branch, NLRC, also
grounded like his suit in the Court of First Instance, on MAI's refusal on November 24,
1981 to reinstate him pursuant to Director Estrella's final judgment. In this new complaint,
he sought recovery of P539,837.00, representing salaries and pecuniary benefits from
December 1, 1981 until May 1987 "when . . . (he) reaches the age of retirement of 65
years;" P600,000.00 as moral damages; P300,000.00 as exemplary damages; and
P100,000.00 as attorney's fees.
- It is noteworthy that at the time of the filing of this second complaint on August 16, 1982,
Nolasco was already 60 years old, his 60th birth anniversary having fallen sometime in
May, 1982.
- After appropriate proceedings, Arbiter Lasquite rendered judgment dismissing the case
"for being a duplication of the earlier labor) Case involving the same complainant (Rodolfo
Nolasco) and the same respondent (Mai Philippines, Inc.)."
The Arbiter rejected as
"completely unfounded" Nolasco's claim that "he was dismissed by respondent with malice
and deceit on 24 November (1981)" since in truth he had not yet been reinstated on
that day, and had in fact still been ordered reinstated by an alias writ of execution on
December 7, 1981, and had "still received his backwages up to December 1981." The
Arbiter declared that Nolasco's "appropriate remedy . . . (was) to pursue the
enforcement/implementation of the 19 November 1979 Order of Director Estrella . . .
(which) is very clear, specific and definitive," rather than to institute and prosecute a
"duplicate case."
- Notice of the Arbiter's Decision was served on Nolasco's counsel on September 12,
1984. Twelve days later, or the latter filed an "Appeal" dated September 22, but
verified on September 24, 1984 15 assailing the Arbiter's finding of "duplication" or res
judicata and broadly hinting that the Arbiter bad knowingly rendered an unjust judgment.
- MAI filed an "OPPOSITION TO APPEAL" in which it contended that: (1) Nolasco's
appeal was filed out of time and should on this account be dismissed; (2) the Arbiter's
decision was justified by the facts and applicable law and jurisprudence; (3) the appeal is
pro forma, being "a mere rehash of the arguments already presented" to the Arbiter; and
(4) the fact that Nolasco had in the meanwhile "reached the age of mandatory
retirement . . . (rendered) the order of reinstatement moot and academic."
- While his appeal from the Lasquite decision was pending before the NLRC, Nolasco filed
a motion for an alias writ of execution, to compel payment to him pursuant to the Estrella
Decision of "accrued backwages from December 1981 to the present" as well as "his
yearly Christmas bonus or 13th month pay for 1981, 1982, 1983, 1984 and 1985," in view
of MAI's failure of "compliance to the reinstatement of complainant."
Over MAI's
objections that inter alia the matter was pending appeal, and that reinstatement had been
mooted, Director Severo Pucan who had taken over from Mr. Estrella as Director of
the National Capital Region issued the alias writ prayed for. Pursuant thereto, MAI's
deposit with the Bank of America was garnished to the extent of P239,850.00.
- MAI thereupon filed with Director Pucan's Office a "MOTION TO QUASH AND/OR
MOTION FOR RECONSIDERATION" dated February 12, 1985, 22 and with the Ministry
of Labor itself, an "OMNIBUS MOTION AND/OR APPEAL"
seeking the recall of the
alias writ and the negation of the garnishment of its funds in the bank. Minister Ople
instructed Director Pucan "to stay the Alias Writ of Execution . . . pending final
determination of whether the award has already been fully satisfied or not." MAI's
attorneys were advised by the Bank of America over the telephone that Sheriff Alfonso
Balais, Jr. and Nolasco were able to obtain from it and then encash an uncrossed
manager's check amounting to P239,850.00, which was the amount set forth in the alias
writ of execution dated February 6, 1985.
- Some minutes afterwards, the same attorneys received copies of two (2) orders which
apparently constituted the authority for the encashment of said check. The first, rendered
by Deputy Minister Leogardo, Jr. on May 27, 1985, dismissed MAI's initial appeal and
remanded the case to the National Capital Region for enforcement of the alias writ of
execution in question. 29 The second, issued by Director Pucan under date of May 28,
1985, commanded the Bank of America, under threat of contempt, to convert into cash its
check made out in the name of Sheriff Balais in the amount of P239,850.00; this, in view
of the dismissal of MAI's appeal and the judgment's "having long become final and
executory."
- MAI filed with the Office of the Labor Minister a MOTION FOR RECONSIDERATION of
the Leogardo Order of May 27, 1985, (1) assailing it as "most appalling, nay revolting" in
the premises, being contrary to Minister Ople's order (letter) of April 2, 1985 requiring stay
of execution,
and being unwarranted because of the pendency of MAI's (second)
appeal; and (2) deploring, too, the "undue haste and speed" which attended encashment
of the check for P239,850.00. MAI also (3) pointed out that in May, 1982, Nolasco reached
the compulsory retirement age of 60 years set by the Labor Code; and Nolasco's claim
that under MAI's retirement plans, specifically cited by him, the retirement age is 65 is
wrong, because by the terms of those very same retirement plans invoked by him, the
retirement age of 65 applied only to employees in the U.S.A. and Puerto Rico.
- The motion for reconsideration was resolved by Deputy Minister Leogardo. He declared
that since the order of Director Estrella requiring reinstatement of Nolasco with full back
wages had already become final and executory, attacks against that order "on the merits

Labor Law 1
or in substance can no longer be entertained;" but this notwithstanding, a cause had
supervened rendering the continuous execution of that final Order of November 19, 1979
unjust, (this supervening cause being) . . . (Nolasco's) becoming 60 years of age
sometime in May, 1982," which is the compulsory retirement age prescribed by Section
13, Rule I, Book VI of the Omnibus Rules Implementing the Labor Code (overruling
Nolasco's theory that the retirement age for MAI employees is 65). For this reason,
Leogardo continued, Nolasco was "no longer entitled to back wages after he became 60
years old in May, 1982; . . . (and) back wages paid to him after May, 1982 should be
credited as (full satisfaction of) retirement pay benefits. Hence, according to Leogardo,
"the judgment . . . should be . . . considered fully satisfied and . . . (the case) deemed
closed and terminated."
- The present case is for payment of damages allegedly arising from the respondent's bad
faith in causing the abolition of the complainant's position to render nugatory his
reinstatement under a final judgment.
ISSUE
WON the NLRC gravely abused its discretion in ignoring event rendering reinstatement
moot
HELD
YES
- The NLRC gravely abused its discretion when it refused, or neglected to consider the
fact again quite plain from the record and to which MAI had adverted more than once
that the matter of Nolasco's reinstatement had become moot and academic at the time
that he filed his second action before the labor arbiters' office against MAI on August 16,
1982; for as of that day, he had already reached the age of 60 years, which is the
retirement age fixed by the Labor Code.
Disposition Petition granted.

RATIONALE
PRODUCERS BANK OF THE PHILS V NLRC
(PRODUCERS BANK EMPLOYEES ASSN)
298 SCRA 517
ROMERO; November 16, 1998
FACTS
- Producers Bank was assigned a conservator by the Central Bank for the purposes of
protecting its assets. When the Union sought the implementation of the CBA re retirement
plan, the conservator refused. Union filed with LA for unfair labor practice and for flagrant
violation of CBA.
- LA dismissed. NLRC reversed, holding that the CBA must be enforced.
ISSUE
WON the conservator can ignore the CBA
HELD
NO
- The CBA is the law between the Union and the company.
- The conservator can only preserve assets and reorganize management. He cant revoke
an existing valid contract. Even the legislature cannot infringe on this Constitutional right,
much less can it delegate such non-existent powers to the conservator. The conservator
can assail defective contracts; but cant repudiate valid ones.
- Apart from the non-impairment clause, it is also well-settled that when the conflicting
interests of labor and capital are weighed on the scales of social justice, the dominant
influence of the latter must be counter-balanced by the sympathy and compassion the law
must accord the under-privileged worker.
- The retirement of an employee does not, in itself, affect his employment status especially
when it involves all rights and benefits due to him, since these must be protected as
though there had been no interruption of service. It must be borne in mind that the
retirement scheme was part of the employment package and the benefits to be derived
therefrom constituted, as it were, a continuing consideration for services rendered, as well
as an effective inducement for remaining with the corporation. It is intended to help the
employee enjoy the remaining years of his life, releasing him from the burden of worrying
for his financial support, and are a form of reward for his loyalty.
- When the retired employees were requesting that their retirement benefits be granted,
they were not pleading for generosity but were merely demanding that their rights, as
embodied in the CBA, be recognized. Thus, when an employee has retired but his benefits
under the law or the CBA have not yet been given, he still retains, for the purpose of
prosecuting his claims, the status of an employee entitled to the protection of the Labor
Code, one of which is the protection of the labor union.
Disposition Petition denied. NLRC decision affirmed.

A2010

- 329 -

Disini

ELIGIBILITY
BRION V SOUTH PHILIPPINE UNION MISSION OF THE
SEVENTH DAY ADVENTIST CHURCH
307 SCRA 497
ROMERO; May 19, 1999
FACTS
- Blow, blow, thou winter wind,
Thou art not so unkind
As man's ingratitude . . .
(Shakespeare: As You Like It, Act II, sc. 7, Line 174]
- Petitioner Delfin A. Brion became a member of respondent South Philippine Union
Mission of the Seventh Day Adventist Church (hereafter SDA) and worked his way up the
ladder until he became an ordained minister and president of the Northeastern Mindanao
Mission of the Seventh Day Adventist Church in Butuan City. Petitioner worked until he
retired in 1983. As was the practice of the SDA, petitioner was provided a monthly amount
as a retirement benefit.
- Sometime thereafter, petitioner got into an argument with Samuel Sanes, another pastor
of the SDA. This disagreement degenerated into a rift between petitioner and the SDA,
culminating in the establishment by petitioner of a rival religious group which he called the
"Home Church." Petitioner succeeded in enticing a number of SDA members to become
part of his congregation even as he continued disparaging and criticizing the SDA.
Because of his actions, petitioner was excommunicated by the SDA and, on July 3, 1993,
his name was dropped from the Church Record Book. As a consequence of his
"disfellowship," petitioner's monthly retirement benefit was discontinued by the SDA.
- RTC ruled in favor of petitioner. CA reversed taking into account the disloyalty after
retirement.
ISSUE
WON the conditions for eligibility for retirement be met only at the time of retirement
HELD
YES
- The following provisions on retirement, contained in the General Conference Working
Policy of the SDA, are of primary importance in resolving the issue at hand:
Beneficiaries of Retirement Plan The benefits of the retirement plan are designed for
those who have devoted their lives to the work of the Seventh-day Adventist Church
and are eligible to retire for reasons of old age and/or disability.
xxx
xxx
xxx
- Termination of Benefits The benefits shall terminate with the decease of the
beneficiary, except where there is an eligible surviving spouse and/or children.
- In the case at bar, the words are very clear. Benefits are only to terminate upon death.
The employer and employee are free to stipulate on retirement benefits, as long as these
do not fall below the floor limits provided by law. Furthermore, pension and retirement
plans, in line with the Constitutional mandate of affording full protection to labor, must be
liberally construed in favor of the employee, it being the general rule that pension plans
formulated by an employer are to be construed most strongly against the employer. Again,
while paying retirement benefits to petitioner may be odious and abhorrent to the SDA, in
the absence of any other stipulation for the termination of petitioner's retirement benefits,
the SDA must comply with its contractual obligations, the contract being the law between
the parties.
Disposition CA decision reversed and set aside

GROUND TERMINATION
CAINTA CATHOLIC SCHOOL V CAINTA CATHOLIC
SCHOOL EMPLOYEES UNION
489 SCRA 468

Labor Law 1
TINGA; May 4, 2006
NATURE
Petition for review on certiorari of a decision and resolution of the CA
FACTS
- On 6 March 1986, a Collective Bargaining Agreement (CBA) was entered into between
Cainta Catholic School (School) and the Cainta Catholic School Employees Union (Union)
effective 1 January 1986 to 31 May 1989.
- Msgr. Mariano Balbago (Balbago) was appointed School Director in April 1987. From this
time, the Union became inactive.
- It was only in 10 September 1993 that the Union held an election of officers, with Mrs.
Rosalina Llagas (Llagas) being elected as President; Paz Javier (Javier), Vice-President.
- On 15 October 1993, the School retired Llagas and Javier, who had rendered more than
twenty (20) years of continuous service, pursuant to Section 2, Article X of the CBA, to wit:
An employee may be retired, either upon application by the employee himself or by the
decision of the Director of the School, upon reaching the age of sixty (60) or after having
rendered at least twenty (20) years of service to the School the last three (3) years of
which must be continuous.
- Three (3) days later, the Union filed a notice of strike
- On 8 November 1993, the Union struck and picketed the Schools entrances.
- The labor dispute was certified to the NLRC for compulsory arbitration, pursuant to
Article 263(g) of the Labor Code as amended. Accordingly, all striking teachers and
employees were directed to return to work within 24 hours from receipt of the Order and
the School Administrator to accept all returning employees under the same terms and
conditions prevailing prior to the strike. The effects of the termination of Ms. Rosalinda
Llagas and Paz A. Javier were suspended.
- On 27 July 1994, the Union filed a complaint for unfair labor practice before the NLRC
- On 31 January 1997, the NLRC rendered a Resolution favoring the School.
-The NLRC ruled that the retirement of Llagas and Javier is legal as the School was
merely exercising an option given to it under the CBA. The NLRC dismissed the unfair
labor practice charge against the School for insufficiency of evidence. Furthermore, it was
found that the strike declared by the Union from 8 to 12 November 1993 is illegal, thereby
declaring all union officers to have lost their employment status.
- The Union moved for reconsideration but it was denied
- On 9 July 1997, the Union filed a petition for certiorari before this Court. The Court
issued a temporary restraining order (TRO) against the enforcement of the subject
resolutions
- On 28 July 1997, ten (10) regular teachers, who were declared to have lost their
employment status under the aforesaid NLRC Resolution reported back to work but the
School refused to accept them by reason of its pending motion for clarification. This
prompted the Union to file a petition for contempt against Balbago and his agents before
this Court, which was later on consolidated with the earlier petition.
- The case was referred to the Court of Appeals which rendered a decision giving due
course and granting the petition to annul and set aside the 31 January 1997 and 30 April
1997 Resolutions of the NLRC; while dismissing the petition for contempt for lack of merit.
- The motion for reconsideration subsequently filed by the School was denied
- Thereafter, petitioners filed this petition for review on certiorari
ISSUE
WON the forced retirement of Llagas and Javier was a valid exercise of management
prerogative
HELD
YES
- Pursuant to the existing CBA, the School has the option to retire an employee upon
reaching the age limit of sixty (60) or after having rendered at least twenty (20) years of
service to the School, the last three (3) years of which must be continuous.
- Retirement is a different specie of termination of employment from dismissal for just or
authorized causes under Articles 282 and 283 of the Labor Code.
- While in all three cases, the employee to be terminated may be unwilling to part from
service, there are eminently higher standards to be met by the employer validly exercising
the prerogative to dismiss for just or authorized causes. In those two instances, it is
indispensable that the employer establish the existence of just or authorized causes for
dismissal as spelled out in the Labor Code. Retirement, on the other hand, is the result of
a bilateral act of the parties, a voluntary agreement between the employer and the
employee whereby the latter after reaching a certain age agrees and/or consents to sever
his employment with the former.
- Article 28754 of the Labor Code, as amended, governs retirement of employees.
54

Any employee may be retired upon reaching the retirement age established in the collective bargaining agreement or
other applicable employment contract.
In case of retirement, the employee shall be entitled to receive such retirement benefits as he may have earned under
existing laws and any collective bargaining agreement and other agreements: Provided, however, That an employees
retirement benefits under any collective bargaining agreement and other agreements shall not be less than those

A2010

- 330 -

Disini

- The CBA in the case at bar established 60 as the compulsory retirement age. However, it
is not alleged that either Javier or Llagas had reached the compulsory retirement age of
60 years, but instead that they had rendered at least 20 years of service in the School, the
last three (3) years continuous.
- Clearly, the CBA provision allows the employee to be retired by the School even before
reaching the age of 60, provided that he/she had rendered 20 years of service.
- By their acceptance of the CBA, the Union and its members are obliged to abide by the
commitments and limitations they had agreed to cede to management. The questioned
retirement provisions cannot be deemed as an imposition foisted on the Union, which very
well had the right to have refused to agree to allowing management to retire employees
with at least 20 years of service.
- Twenty years is a more than ideal length of service an employee can render to one
employer. Under ordinary contemplation, a CBA provision entitling an employee to retire
after 20 years of service and accordingly collect retirement benefits is "reward for services
rendered since it enables an employee to reap the fruits of his labor particularly
retirement benefits, whether lump-sum or otherwise at an earlier age, when said
employee, in presumably better physical and mental condition, can enjoy them better and
longer.
- Under Article 287 of the Labor Code, a CBA may validly accord management the
prerogative to optionally retire an employee under the terms and conditions mutually
agreed upon by management and the bargaining union, even if such agreement allows for
retirement at an age lower than the optional retirement age or the compulsory retirement
age.
Disposition petition is granted. The Resolution dated 31 January 1997 of the NLRC is
reinstated.

15.02 ACCRUAL OF BENEFITS


ACCRUAL
CRUZ V PHIL GOLBAL COMMUNICATIONS INC
430 SCRA 184
SANDOVAL-GUTIERREZ; May 28, 2004
NATURE
Petition for review on certiorari of CA decision
FACTS
- Phil Global Communications, Inc. is a corporation engaged in the principal business of
communications through telex and telegram, with various branches nationwide. As a result
of a decline in the volume of recorded messages sent via telex and telegram, it suffered
substantial financial losses. It then adopted an organizational streamlining program that
resulted in the closure of its branches and termination from the service of 42 workers.
- In separate letters, the PGCI terminated petitioners services and eventually paid them
their separation pay at the rate of 1 months salary per year of service. They then
executed and signed a "Release, Waiver and Quitclaim."
- Petitioners filed with the Labor Arbiter a complaint for payment of retirement benefits,
damages and attorneys fees.
- Labor Arbiter: Sustained petitioners claim for retirement benefits under respondents
Retirement Plan.
- NLRC: Reversed the Labor Arbiters decision.
- CA: Affirmed NLRC.
- Petitoners contend that the applicable provision should be Sec 4, Art VI of the
PHILCOMs Retirement Plan which reads:
Section 4 Involuntary Separation "A member whose services may be terminated by
the Company for any reason other than just cause or voluntary resignation shall be
entitled to benefit determined in accordance with the retirement benefit formula
provided in Article V hereof. However, if the termination is due to redundancy, the
employee will be paid one and one-half months pay for every year of service (as
amended on July 1, 1988).
ISSUE
WON the retrenched employees may still claim their retirement benefits after receiving
their separation pay
provided herein.
In the absence of a retirement plan or agreement providing for retirement benefits of employees in the establishment, an
employee upon reaching the age of sixty (60) years or more, but not beyond sixty-five (65) years which is hereby
declared the compulsory retirement age, who has served at least five (5) years in the said establishment, may retire and
shall be entitled to retirement pay equivalent to at least one-half (1/2) month salary for every year of service, a fraction of
at least six (6) months being considered as one whole year.

Labor Law 1
HELD
NO
Ratio The employees right to payment of retirement benefits and/or separation pay is
governed by the Retirement Plan of the parties. Under the Retirement Plan before us,
petitioners are not entitled to both separation pay and retirement benefits.
Reasoning
- Sec.4 should not be interpreted singly but should be read together with the other
provisions of the Retirement Plan in question to determine the intent of the Plan. Section
6(b)55 Article XI, of the Retirement Plan is explicit and leaves no doubt as to the intention
to prohibit the recovery of both separation pay and retirement benefits. NLRC correctly
pointed out that the payment of separation pay is a requirement of the law, i.e. the Labor
Code, which is a social legislation. The Retirement Plan itself clearly sets forth the
intention of the parties to entitle employees only to whatever is greater between the
Retirement Benefits then due and that which the law requires to be given by way of
separation pay. To give way to complainants demands would be to totally ignore the
contractual obligations of the parties in the Retirement Plan, and to distort the clear intent
of the parties as expressed in the terms and conditions contained in such plan.
- Under Art 283 of the LC, affected employees, in case of retrenchment or cessation of
operations, are always given termination or separation pay equivalent to one month pay or
at least month pay for every year of service, whichever is higher. Under Sec 4, Art VI of
respondents Retirement Plan, the employees are entitled to a retirement pay equivalent to
1 months pay for every year of service computed on the basis of their basic monthly
salary at the time of retirement. Here, respondent opted to pay petitioners separation
benefits computed under the Retirement Plan, the same being higher than what Art 283 of
the LC provides.
Disposition Petition is DENIED.

LLORA MOTORS INC V DRILON


179 SCRA 175
FELICIANO; November 7, 1989
NATURE
Petition for certiorari with Preliminary Injunction
FACTS
- Sometime in September of 1968, Primitivo Alviar began his employment with Llora
Motors, Inc. As a truck driver, Mr. Alviar rendered services to the company 8 hours a day
(excluding overtime) seven days a week, and for his labor received a salary computed on
a per trip basis plus emergency cost of living allowance (ECOLA). At the time he stopped
working on 19 April 1985, Mr. Alviar was 65 Years of age.
- On 28 October 1985, Alviar filed with NLRC a complaint for separation pay and nonpayment of daily wages against petitioners Llora Motors and Constantino Carlota, Jr., the
company manager. Alviar claimed entitlement to ECOLA underpayments from November
1982 up to April 1985 in the amount of P4,709.54 as well as retirement benefits, computed
at one-half month's pay for every year of service.
- Petitioners alleged that all of the employment benefits claimed by ALviar had already
been fully paid. On the matter of retirement benefits, it was contended that Alviar had not
been dismissed by Llora Motors, but that sometime in the early part of 1985, Alviar
showed utter lack of interest in his work and was absent for no apparent reason; that the
truck assigned to him laid idle and because of non-use for sometime, it deteriorated so
seriously; that in the last week of April of 1985, Alviar reported for work and was then
informed that while the truck has not been rehabilitated as yet, he could act as relief driver;
that Alviar did not like to be a relief driver in the meantime for since then he did not report
for work; and that it was Alviar who abandoned his work since the last week of April 1985
and never reported since then. Neither had Mr. Alviar been retired, petitioners claimed, for
the simple reason that respondent corporation does not have any retirement plan or any
collective bargaining agreement with the employees for no union exists within the
company because the employees, drivers included, received more than the standard
benefits for their labor. Petitioners contended further that "records will show that Alviar had
received retirement benefits from the Social Security System when he retired therefrom in
1983.

55

Section 6 Effect of Social Legislation


a) Social Security and Workmens Compensation The benefits payable under this Plan shall be in addition to such
benefits which the Member shall be entitled under the Social Security and Workmens Compensation Acts.
b) Adjustment of Benefits payments Except only as provided in paragraph (a) of this Section, in the event the
company is required under the law or by lawful order of competent authority, to pay to the Member benefits or
emoluments similar or analogous to those already provided in the Plan, the member concerned shall not be entitled to
both what the law or the lawful order of competent authority requires the company to give and the benefits provided by
the Plan, but shall only be entitled to whichever is the greatest among them, it being understood however that for the
purpose of determining whichever benefits is greatest, it is the total benefits required to be paid under the law or lawful
order of competent authority or the Plan that shall be reckoned. The benefit provided by this Plan may be reduced or
amended in an equitable manner by the company by the value of any present or future contract such as collective
bargaining, law, e.g. termination pay provisions or lawful order of competent authority.

A2010

- 331 -

Disini

- Alviar did not controvert petitioner's allegations of abandonment and non-dismissal. Mr.
Alviar there simply alleged that in April of 1985, he "retired from the service due to his old
age of 65 years."
- The Labor Abriter decided in favor of Alviar and ordered petitioner to pay unpaid ECOLA
and retirement benefits to him. The NLRC affirmed said decision.
ISSUE
WON Alviar is legally entitled to receive retirement benefits from petitioners
HELD
NO
- Our Labor Code has only one article that deals with the subject of "retirement from the
service." Article 287 of the Code reads as follows:
Article 287. Retirement. Any employee may be retired upon reaching the retirement
age established in the Collective Bargaining Agreement or other applicable
employment contract.
In case of retirement, the employee shall be entitled to receive such retirement benefits
as he may have earned under existing laws and any collective bargaining or other
agreement.
- Examination of Article 287 above shows that entitlement to retirement benefits may
accrue either (a) under existing laws or (b) under a collective bargaining agreement or
other employment contract. It is at once apparent that Article 287 does not itself purport to
impose any obligation upon employers to set up a retirement scheme for their employees
over and above that already established under existing laws. In other words, Article 287
recognizes that existing laws already provide for a scheme by which retirement benefits
may be earned or accrue in favor of employees, as part of a broader social security
system that provides not only for retirement benefits but also death and funeral benefits,
permanent disability benefits, sickness benefits and maternity leave benefits. As is
commonplace knowledge, the Social Security Act provides for retirement benefits which
essentially consist of the right to receive a monthly pension for the rest of the covered
employee's life provided that: (1) such employee had paid at least 120 monthly
contributions prior to retirement; and (2) has reached the age of sixty (60) years (if his
salary is less than P300.00 a month) or 65 years. The retirement scheme here
'established is compulsory and contributory in character on the part of both the employer
and the employee, backed up by criminal sanctions and administered by a large and
elaborate bureaucracy.
- Article 287 of the Labor Code recognizes that employers and employees may, by a
collective bargaining or other agreement, set up a retirement plan in addition to that
established by the Social Security law, but prescribes at the same time that such
consensual additional retirement plan cannot be substituted for or reduce the retirement
benefits available under the compulsory scheme established by the Social Security law.
Such is the thrust of the second paragraph of Article 287 which directs that the employee
shall be entitled to receive retirement benefits earned "under existing laws and any
collective bargaining or other agreement."
- It is also important here to examine Section 13 and 14 of Rule, I, book VI of the Rules
and Regulations Implementing the Labor Code (hereafter, "Implementing rule I").
Implementing Rule I deals with both termination of services and retirement, being entitled
"Termination of Employment and Retirement." But Sections 13 and 14 of Implementing
Rule I are the only provisions which deal with retirement matters. Under Section 13 which
provides as follows:
Sec. 13. Retirement. In the absence of any collective bargaining agreement or other
applicable agreement concerning terms and conditions of employment which provides for
retirement at an older age, an employee may be retired upon reaching the age of sixty
(60) years.
- where an additional retirement plan has been established by a collective bargaining
agreement, or other applicable agreement (or, under Section 14, an "established employer
policy"), but such plan fails to specify another, older, age of retirement, an employee may
retire, and may in turn be retired by his employer, upon reaching age sixty (60).
Sec. 14. Retirement benefits.
(a) An employee who is retired pursuant to a bona-fide retirement plan or a in accordance
with the applicable individual or collective agreement or established employer policy shall
be entitled to all the retirement benefits provided therein or to termination pay equivalent
at least one-half month salary for every year of service, whichever is higher, a fraction of
at least six (6) months being considered as one whole year.
(b) Where both the employer and the employee contribute to the retirement plan,
agreement or policy, the employer's total contribution thereto shall not be less than the
total termination pay to which the employee would have been entitled had there been no
such retirement fund. In case the employer's contribution is less than the termination pay
the employee is entitled to receive, the employer shall pay the deficiency upon the
retirement of the employee.
(c) This Section shall apply where the employee retires at the age of sixty (60) years or
older.
- Section 14 (a) refers to "termination pay equivalent to at least one-half (1/2) month for
every year of service" while Section 14 (b) mentions "termination pay to which the
employee would have been entitled had there been no such retirement fund" as well as

Labor Law 1
"termination pay the employee is entitled to receive." It should be recalled that Sections 13
and 14 are found in Implementing rule I which deals with both "termination of employment"
and "retirement." It is important to keep the two (2) concepts of "termination pay" and
"retirement benefits" separate and distinct from each other. Termination pay or separation
pay is required to be paid by an employer in particular situations Identified by the Labor
Code itself or by Implementing rule I. Termination pay where properly due and payable
under some applicable provision of the Labor Code or under Section 4 (b) of Implementing
Rule 1, must be paid whether or not an additional retirement plan has been set up under
an agreement with the employer or under an "established employer policy."
- Section 14 of Implementing Rule 1, like Article 287 of the Labor Code, does not purport
to require "termination pay" to be paid to an employee who may want to retire but for
whom no additional retirement plan had been set tip by prior agreement with the employer.
Thus, Section 14 itself speaks of an employee "who is retired pursuant to a bona-fide
retirement plan or in accordance with the applicable individual or collective agreement or
established employer policy." What Section 14 of Implementing Rule I may be seen to be
saying is that where termination pay is otherwise payable to an employee under an
applicable provision of the Labor Code, and an additional or consensual retirement plan
exists, then payments under such retirement plan may be credited against the termination
pay that is due, subject, however, to certain conditions. These conditions are: (a) that
payments under the additional retirement plan cannot have the effect of reducing the
amount of termination pay due and payable to less than one-half (1/2) month's salary for
every year of service; and (b) the employee cannot be made to contribute to the
termination pay that he is entitled to receive under some provision of the Labor Code; in
other words, the employee is entitled to the full amount of his termination pay plus at least
the return of his own contributions to the additional retirement plan.
- In the instant, case there is no consensual basis for the required payment of additional
retirement benefits. The Labor Arbiter and the NLRC had not declared Alviar to have been
illegally dismissed by petitioners. Neither was there any pretense on the part of Alviar that
labor-saving devices had been installed, or that redundancy or retrenchment or cessation
of operations had occurred in Llora Motors or that he was afflicted by some disabling
disease, or that, being entitled to reinstatement, he could not be reinstated to this old
position. Under these circumstances, the portion of the Labor Arbiter's award which
required petitioners to pay an amount equivalent to a half month's pay for every year of
service of Mr. Alviar cannot be justified either as (additional) retirement benefits or as
termination pay and hence constituted an act without or in excess of jurisdiction.
- Wherefore, petitioner is required to pay Alviar only P4,709.54 for unpaid ECOLA
differentials.

A2010

- 332 -

Disini

WON a 64-year old employee, who voluntarily resigned, is entitled to retirement benefits
under the Labor Code, in the absence of a company retirement plan or collective
bargaining agreement or an established company policy on such benefits
HELD
NO
- Respondent is not asking for retirement benefits due him under the Social Security Law.
He does not claim that there is a collective bargaining agreement or other applicable,
contract or an established company policy, granting him retirement benefits.
Ratio. Under Article 28756 of the Labor Code, entitlement of employees to retirement
benefits must be specifically granted under existing laws, a collective bargaining
agreement or employment contract or an established employer policy [Llora Motors, Inc.
v. Drilon].
Reasoning
- Article 287 does not in itself purport to impose any obligation upon employers to set up a
retirement scheme for their employees over and above that already established under
existing laws, like the Social Security Act.
- EXPLANATION OF A287: The first paragraph of Article 287 deals with the retirement age of
an employee, which is the age established in (a) a collective bargaining agreement or (b)
other applicable retirement contract. The second paragraph of said Article deals with the
retirement benefits to be received by a retiring employee and which are the retirement
benefits as the employee may have earned under (a) an existing law, (b) a collective
bargaining or (c) other agreements.
- KINDS OF RETIREMENT SCHEMES: (1)compulsory and contributory in character; (2) one set
up by agreement between the employer and the employees in collective bargaining
agreements or other agreements between them (Llora Motors, Inc. v. Drilon, supra); (3)
one that is voluntarily given by the employer, expressly as in an announced company
policy or impliedly as in a failure to contest the employee's claim for retirement benefits
(Allied Investigation Bureau, Inc. v. Ople, 91 SCRA 265 [1979]).
- problema daw ng legislature yan!
Disposition instant petition is GRANTED. The assailed Resolutions of the NLRC dated
August 23, 1991 and December 28, 1990 are SET ASIDE, and the Temporary Restraining
Order issued by the Court on November 4, 1991 is made PERMANENT

15.04 BENEFITS AND GRATUITY


STA. CATALINA COLLEGE V NLRC
[PAGE 285]

15.03 PRIVATE PLAN


EMPLOYER OBLIGATION
GVM SECURITY AND PROTECTIVE AGENCY V NLRC
224 SCRA 734
QUIASON; July 23, 1993
NATURE
Petition for Review for Certiorari
FACTS
- Antonio Dulce was employed as a security guard by petitioners in July 1958. After 28
years in the service, he tendered his resignation at age 64 years old. He was earning a
monthly salary of P2350. Petitioners paid him P6650 as his cash deposit and then Dulce
executed a quitclaim.
- Dulce filed a complaint for monetary claims, including retirement pay. Petitioners deny
liability, saying that they did not have a company policy or CBA on employees retirement
benefits.
- LA: dismissed complaint, lack of sufficient supporting evidence to establish claims, BUT
awarded ex-gratia benefits, or any benefits pursuant to company policy (but theres none)
- NLRC: petitioners liable for P3915 as differential to Dulces separation pay, and P391.50
as attorneys fees, using formula in par(a), Sec 14, Rule I, Book VI of the Omnibus Rules
(provision granting separation pay equivalent to month salary for every year of service
and considering a fraction of at least 6 months as 1 whole year) both parties appealed
- NLRC (on appeal): petitioner pay P27325 as differential of retirement benefits and
P2742.50 as attorneys fees
- SC issued TRO on NLRC from implementing assailed resolutions
ISSUE

PART III SOCIAL LEGISLATION


A. INTEGRATED OUTLINE

56 Article 287.

Retirement. -- Any employee may be retired upon reaching the retirement age established in the collective bargaining agreement or other applicable employment contract.

In case of retirement, the employee shall be entitled to receive such retirement benefits as he may have earned under existing laws and any collective bargaining agreement and other agreements.

Labor Law 1

A2010

- 333 -

Disini

3.01 POLICY OBJECTIVES

substances and gas fumes coming from burning welding electrodes. However, petitioner's
motion for reconsideration was likewise denied, upon claim of the GSIS that Francisco's
job as a welder would instead cause lung disease rather than liver cirrhosis.
- On appeal to the Employees' Compensation Commission (ECC), the Commission
affirmed the denial of the GSIS on petitioner's claim relying on the fact that the diagnosis
on Francisco's illness did not specify the type of cirrhosis which caused his death.
Nevertheless, the Commission took cognizant of the fact that the deceased employee did
not have a previous history of alcoholism, hepatitis or a previous history of biliary condition
which could give a clue to the nature of cirrhosis he had.

LAW CONCEPT

ISSUE
WON liver cirrhosis an illness which is compensable

SOCIAL SECURITY ACT; GOVERNMENT SERVICE


INSURANCE ACT; AND EMPLOYEES COMPENSATION
AND STATE INSURANCE FUND; AND NATIONAL
HEALTH INSURANCE ACT

SULIT V EMPLOYEES COMPENSATION COMMISSION


98 SCRA 483
AQUINO; June 30, 1980
FACTS
- Sulit was employed as a mechanic in the Cavite Naval Shipyard. Due to aches
accompanied by fevers and chills, he was confined in PGH for 6 days up to his death due
to bronchopneumonia and pyelonephritis, lung and kidney infections respectively.
- Mrs. Sulit filed a claim for compensation under the Employees Compensation and State
Insurance Fund (PD 626), contending that his work was postural in nature, kinking his
ureters and stagnating his urine flow, causing the infection. GSIS and the Employees
Compensation Commission (ECC) rejected the claim, arguing that his ailments were not
occupational in nature. ECC claims the aggravation of the disease due to the work is not
covered under said law.
ISSUE
WON petitioner is entitled to compensation under PD 626
HELD
NO
- Mrs. Sulit, in her arguments, was referring to the old law, Act No. 3428 as amended by
Act 3812, which originally restricted compensation to injuries sustained in the course of
employment or resulting form its nature.
- RA 772 amended Act 3812, specifying tuberculosis as a compensable disease and
adopting the rule that aggravation of the disease by the nature of the work is compensable
as well as introducing the presumption of compensability.
- Such innovations paved the way for the expansive application of the Workmens
Compensation Law in favor of the worker, destroying the parity between the competing
interests of employer and employee. Hence, to restore this balance, the old law was
jettisoned and replaced with PD 626 and the Labor Code, which abolished the
presumption of compensability and the rule on aggravation of illness caused by the nature
of employment.
- Her claim was thus decided under the Labor Code and the employees compensation
provisions. Article 166 provides that the State shall promote and develop an employees
compensation program whereby employees and their dependents may receive benefits in
case of work connected disability or death. Such compensable sicknesses are left to be
determined and approved by the ECC, based on the peculiar hazards of the employment.
In the instant case, Sulits cause of death was not deemed work related and was thus not
compensable.
Disposition appeal is DISMISSED

SANTOS V EMPLOYEES' COMPENSATION


COMMISSION
221 SCRA 182
NOCON; April 7, 1993
FACTS
- Francisco Santos was employed as welder at the Philippine Navy and its Naval
Shipyard. He died of liver cirrhosis after 32 years in service.
- Mrs. Carmen A. Santos filed a claim for the death benefit of her husband pursuant to
Presidential Decree No. 626, as amended. However, GSIS denied the claim on the ground
that upon proofs and evidence submitted, Francisco's ailment cannot be considered an
occupational disease as contemplated under P.D. 626, as amended.
- Mrs. Santos then sought the assistance of the Commander of NASCOM, PN, who in
turn wrote the GSIS requesting for a favorable action on her claim. Said letter also
substantiated petitioner's claim that her husband's duties as Senior Welder required him to
perform delicate welding jobs inside compartments of naval vessels, like
compartmentation bulk heads; CIC rooms; officers and PO's quarters; fuel, lube oil and
fresh water tanks, where he was exposed to heat and inhalation of burning chemical

HELD
YES
- Liver cirrhosis, although not one among those listed as compensable ailment, as
considered in the case at bar as covered under the Act, on the ground that the nature of
the work of petitioner's husband, exposed him to the risk of contracting the same.
Reasoning
a. As a welder, Francisco was exposed to heat, gas fumes and chemical substances
coming from the burning electrodes caused by welding. Generally, the metal burned is
iron. In the course thereof, other compounds and oxides, such as carbon monoxide,
carbon dioxide, sulfur and phosphorus, may be emitted in the process of welding,
depending on the kind of material used and extent of corrosion of the metal worked on.
These vaporized metals are inhaled by the welder in the process and significantly in this
case, Francisco had to do welding jobs within enclosed compartments.
b. The deceased experienced untold sufferings in the course of his inspection of barrio
schools and that he became malnourished because of the scarcity of food in the places he
travelled to.
c. Liberal interpretation of the law in favour of labor - While the presumption of
compensability and theory of aggravation under the Workmen's Compensation Act may
have been abandoned under the new Labor Code, the liberality of the law in general in
favor of the working man still prevails. The Employees' Compensation Act is basically a
social legislation designed to afford relief to the working man and woman in our society.
The Employees' Compensation Commission, as the agency tasked with implementing the
social justice mandate guaranteed by the Constitution, should be more liberal in resolving
compensation claims of employees especially where there is some basis in the facts for
inferring a work connection to the cause of death.
Disposition GRANTED and the decision of the Employees' Compensation Commission
is REVERSED.

LATGAN V EMPLOYEES COMPENSATION


COMMISSION
[PAGE]

3.02 DEFINITIONS
1. EMPLOYER
ORATE V CA (EMPLOYEES COMPENSATION
COMMISSION)
399 SCRA 572
YNARES-SANTIAGO; March 26, 2003
NATURE
Petition for review on certiorari under Rule 45 of the ROC assailing the Decision of CA and
Resolution denying petitioners MFR
FACTS
- December 5, 1972 - Norma Orate was employed by Manila Bay Spinning Mills, Inc., as a
regular machine operator.
- On March 22, 1995, she was diagnosed with breast cancer. Consequently, she
underwent modified radical mastectomy. The operation incapacitated her from performing
heavy work, for which reason she was forced to go on leave and, eventually, to retire from
service at the age of 44.
- On November 17, 1995, petitioner applied for employees compensation benefits with the
Social Security System (SSS), but the same was denied on the ground that her illness is
not work-related. On January 22, 1996, she moved for reconsideration contending that
her duties as machine operator which included lifting heavy objects increased the risk of
contracting breast cancer. The SSS, however, reiterated its denial of petitioners claim for

Labor Law 1
benefits under the Employees Compensation Program. Instead, it approved her
application as a sickness benefit claim under the SSS, and classified the same as a
permanent partial disability equivalent to a period of twenty-three (23) months.
- Petitioner requested the elevation of her case to the Employees Compensation
Commission (ECC), which affirmed the decision of the SSS. The ECC ruled that
petitioners disability due to breast cancer is not compensable under the Employees
Compensation Program because said ailment is not included among the occupational
diseases under Annex A of the Rules on Employees Compensation; and it was not
established that the risk of contracting said ailment was increased by the working
conditions at Manila Bay Spinning Mills, Inc. Petitioner filed a petition for review with the
CA. On May 14, 1997, the CA reversed the decision of the ECC, and granted petitioners
claim for compensation benefit under the Workmens Compensation Act (Act No. 3428). It
held that petitioners breast cancer must have intervened before the effectivity of Title II,
Book IV of the Labor Code on Employees Compensation and State Insurance Fund on
January 1, 1975, hence, the governing law on petitioners claim for compensation benefit
is Act No. 3428, which works upon the presumption of compensability, and not the
provisions of the Labor Code on employees compensation. The CA further ruled that
since Manila Bay Spinning Mills, Inc. failed to discharge the burden of proving that
petitioners ailment did not arise out of or in the course of employment, the presumption of
compensability prevails, entitling her to compensation.
- Petitioner filed a MFR arguing that it is the Labor Code which should be applied to her
case inasmuch as there is no evidence that the onset of her breast carcinoma occurred
before January 1, 1975. She claimed that the basis of the computation of her
compensation benefits should be the Labor Code and not the Workmens Compensation
Act. CA denied MFR
ISSUE
1. What is the law applicable to petitioners claim for disability benefits?
2. WON she is entitled under the applicable law to be compensated for disability arising
from breast carcinoma
HELD
1. In the case at bar, petitioner was found to be positive for breast cancer on March 22,
1995. Since no evidence was presented as to when she contracted said ailment. The
presumption is that her illness intervened when P.D. No. 626 was already the governing
law.
- The first law on workmens compensation in the Philippines is Act No. 3428, otherwise
known as the Workmens Compensation Act, which took effect on June 10, 1928. This Act
works upon the presumption of compensability which means that if the injury or disease
arose out of and in the course of employment, it is presumed that the claim for
compensation falls within the provisions of the law. The employee need not present any
proof of causation. It is the employer who should prove that the illness or injury did not
arise out of or in the course of employment.
- On November 1, 1974, the Workmens Compensation Act was repealed by the Labor
Code (Presidential Decree No. 442). On December 27, 1974, Presidential Decree No.
626 (which took effect on January 1, 1975) was issued. It amended the provisions of Title
II, Book IV of the Labor Code on Employees Compensation and State Insurance Fund.
The law as it now stands requires the claimant to prove that the illness was caused by
employment and the risk of contracting the disease is increased by the working conditions.
It discarded, among others, the concepts of presumption of compensability and
aggravation and substituted a system based on social security principles. The present
system is also administered by social insurance agencies the Government Service
Insurance System and Social Security System under the Employees Compensation
Commission. The intent was to restore a sensible equilibrium between the employers
obligation to pay workmens compensation and the employees right to receive reparation
for work-connected death or disability.
- In workmens compensation cases, the governing law is determined by the date when
the claimant contracted the disease. An injury or illness which intervened prior to January
1, 1975, the effectivity date of P.D. No. 626, shall be governed by the provisions of the
Workmen's Compensation Act, while those contracted on or after January 1, 1975 shall
be governed by the Labor Code, as amended by P.D. No. 626. Corollarily, where the claim
for compensation benefit was filed after the effectivity of P.D. No. 626 without any showing
as to when the disease intervened, the presumption is that the disease was contracted
after the effectivity of P.D. No. 626.
2. The instant controversy is not on all fours with the cases where the Court applied the
presumption of compensability and aggravation under the Workmens Compensation
Act, even though the claim for compensation benefit was filed after January 1, 1975. In
the said cases, the symptoms of breast cancer manifested before or too close to the cut
off date January 1, 1975, that it is logical to presume that the breast carcinoma of the
employee concerned must have intervened prior to January 1, 1975.
- In Avendao v. Employees Compensation Commission, the Workmens Compensation
Act was applied to a claim for disability income benefit arising from breast carcinoma,
though the said claim was filed only in 1976, after the effectivity of the Labor Code. Per
certification of the physician of the claimant, her breast cancer was contracted sometime
in 1959, although the clinical manifestations thereof started only in 1969.

A2010

- 334 -

Disini

- The presumption of compensability and aggravation under the Workmens


Compensation Act cannot be applied to petitioners claim for compensation benefit arising
from breast cancer. Hence, the provisions of the Labor Code govern. For breast
carcinoma and resulting disability to be compensable, the claimant must prove, by
substantial evidence, either of two things: (a) that the sickness was the result of an
occupational disease listed under Annex A of the Rules on Employees Compensation;
or (b) if the sickness is not so listed, that the risk of contracting the disease was increased
by the claimants working conditions.
Since cancer of the breast is not listed as an occupational disease under Annex A of the
Rules on Employees Compensation, petitioner has the burden of proving, by substantial
evidence, the causal relationship between her illness and her working conditions.
- Substantial evidence means such relevant evidence as a reasonable mind might accept
as adequate to support a conclusion. In the case at bar, petitioner argued before the SSS
and the ECC that her job as machine operator, which required lifting of heavy objects
increased the risk of her contracting breast carcinoma. In addition, she contended that her
job in the winding department exposed her to cancer-causing dyes used in coloring
threads. However, said bare allegations and vague excerpts on cancer do not constitute
such evidence that a reasonable mind might accept as adequate to support the conclusion
that there is a causal relationship between her illness and her working conditions. Awards
of compensation cannot rest on speculations and presumptions.
- It is not also correct to say that all disability or death resulting from all kinds of cancer are
not compensable. There are certain cancers which are reasonably considered as strongly
induced by specific causes. What the law requires for others is proof. This was not
satisfied in the instant case.
While we sustain petitioners claim that it is the Labor Code that applies to her case, we
are nonetheless constrained to rule that under the same code, her disability is not
compensable. Much as we commiserate with her, our sympathy cannot justify an award
not authorized by law. It is well to remember that if diseases not intended by the law to be
compensated are inadvertently or recklessly included, the integrity of the State Insurance
Fund is endangered. Compassion for the victims of diseases not covered by law ignores
the need to show a greater concern for the trust fund to which the tens of millions of
workers and their families look to for compensation whenever covered accidents, diseases
and deaths occur. This stems from the development in the law that no longer is the poor
employee still arrayed against the might and power of his rich corporate employer, hence
the necessity of affording all kinds of favorable presumptions to the employee. This
reasoning is no longer good policy. It is now the trust fund and not the employer which
suffers if benefits are paid to claimants who are not entitled under the law.
Disposition the decision of the Court of Appeals is REVERSED and SET ASIDE. The
decision of the Employees Compensation Commission, dismissing petitioners claim for
compensation benefits under the Employees Compensation Program is REINSTATED.

2. EMPLOYEE
3. DEPENDENT
4. BENEFICIARIES

3.03 COVERAGE
COVERAGE
PHIL BLOOMING MILLS V SSS
17 SCRA 107
BARRERRA; August 31, 1966
FACTS
- Philippine Blooming Mills Co. Inc. is a domestic corporation employing Japanese
technicians. From April 28, 1957, to October 26, 1958, the corporation had in its employ 6
Japanese technicians. In connection with this, Phil. Blooming sent an inquiry to SSS
whether these employees are subject t compulsory coverage under the System. The First
Deputy Administrator of the SSS answered through a letter:
- SIR:
With reference to your letter of August 24, 1957, hereunder are our answers to your
queries:
Aliens employed in the Philippines:
Aliens who are employed in the Philippines shall also be compulsorily covered. But
aliens who are employed temporarily shall, upon their departure from the Philippines,
be entitled to a rebate of a proportionate amount of their contributions; their employers

Labor Law 1
shall be entitled to the same proportionate rebate of their contributions in behalf of said
aliens employed by them. (Rule I, Sec. 3[d], Rules and Regulations.)
- On October 7, 1958, Phil. Blooming filed a claim with the SSS for the refund of the
premiums paid to the System, on the ground of termination of the members' employment.
The claim was denied so they filed a petition with the Social Security Commission for the
return or refund of the premiums, in the total sum of P2,520.00, paid by the employer
corporation and the 6 Japanese employees, plus attorneys' fees. The claim was
controverted by the SSS, alleging that Rule IX of the Rules and Regulations of the
System, as amended, requires membership in the System for at least 2 years before a
separated or resigned employee may be allowed a return of his personal contributions.
Under the same rule, the employer is not also entitled to a refund of the premium
contributions it had paid.
- After hearing, the Commission denied the petition for the reason that, although under the
original provisions of Section 3 (d) of Rule I of the Rules and Regulations of the SSS,
alien-employees (who are employed temporarily) and their employers are entitled to a
rebate of a proportionate amount of their respective contributions upon the employees'
departure from the Philippines, said rule was amended by eliminating that portion granting
a return of the premium contributions. This amendment became effective on January 14,
1958, or before the employment of the subject aliens terminated. The rights of covered
employees who are separated from employment, under the present Rules, are covered by
Rule IX which allows a return of the premiums only if they have been members for at least
2 years.
ISSUE
WON appellants are bound by the amended Rules requiring membership for two years
before refund of the premium contributions may be allowed
HELD
YES
- These rules and regulations were promulgated to provide guidelines to be observed in
the enforcement of the law. As a matter of fact, Section 3 of Rule I is merely an
enumeration of the "general principles to guide the Commission" in the determination of
the extent or scope of the compulsory coverage of the law. One of these guiding principles
is paragraph (d) relied upon by appellants, on the coverage of temporarily-employed
aliens. It is not pretended, that the amendment of this Section 3(d) of Rule I, as to
eliminate the provision granting to these aliens the right to a refund of part of their
premium contributions upon their departure from the Philippines, is not in implementation
of the law or beyond the authority of the Commission to do.
- However, while the amendment to the Rules may have been lawfully made by the
Commission and duly approved by the President on January 14, 1958, such amendment
was only published in the November 1958 issue of the Official Gazette, and after
appellants' employment had already ceased. Suffice it to say, in this regard, that under
Article 2 of the Civil Code, the date of publication of laws in the Official Gazette is material
for the purpose of determining their effectivity, only if the statutes themselves do not so
provide.
- In the present case, the original Rules and Regulations of the SSS specifically provide
that any amendment thereto subsequently adopted by the Commission, shall take effect
on the date of its approval by the President. Consequently, the delayed publication of the
amended rules in the Official Gazette did not affect the date of their effectivity, which is
January 14, 1958, when they were approved by the President. It follows that when the
Japanese technicians were separated from employment in October, 1958, the rule
governing refund of premiums is Rule IX of the amended Rules and Regulations, which
requires membership for 2 years before such refund of premiums may be allowed.
Disposition finding no error in the resolution of the Commission appealed from, the same
is hereby affirmed, with costs against the appellants.

STA. RITA V CA (PPL OF THE PHILS, SSS)


247 SCRA 487
FELICIANO; August 21, 1995
NATURE
Petition for Review an Certiorari of the Decision of the Court of Appeals
FACTS
- Sta. Rita was charged in the RTC with violating Section 2(a) in relation to Sections 22(d)
and 28(e) of Social Security Law. The Information alleged that petitioner, "as
President/General Manager of B. Sta. Rita Co., Inc. a compulsorily covered employer
under the Social Security Law, as amended, did then and there willfully and unlawfully fail,
neglect and refuse and still fails, neglects and refuses to remit to the Social Security
System contributions for SSS, Medicare and Employees Compensation for its covered
employees."
- RTC sustained petitioner's motion and dismissed the criminal case filed against him. It
ruled that the Memorandum of Agreement entered into between the Department of Labor
and Employment ("DOLE") and the Social Security System ("SSS") extending the

A2010

- 335 -

Disini

coverage of Social Security, Medical Care and Employment Compensation laws to Filipino
seafarers on board foreign vessels was null and void as it was entered into by the
Administrator of the SSS without the sanction of the Commission and approval of the
President of the Philippines, in contravention of Section 4 (a) of R.A. No. 1161, as
amended.
- The People, through the Solicitor General, filed in the Court of Appeals a petition for
certiorari, prohibition and mandamus assailing the order of dismissal issued by the trial
court. Respondent appellate court granted the petition and ordered the Presiding Judge of
the trial court to reinstate the criminal case against petitioner. A motion for reconsideration
thereof was denied by the CA.
- Sta. Ana filed in this Court a motion for extension of thirty days from the expiration of
reglementary period within which to file a petition for review on certiorari. The Court
granted the motion and gave petitioner until 9 June 1995 to file the petition with warning
that no further extension will be given. Despite the warning, the petition was filed only on
13 June 1995 or four (4) days after the due date. Moreover, it failed to comply with
requirement no. 2 of Circular No. 1-88, as amended and Circular No. 19-91 of the Court as
it did not contain an affidavit of service of copies thereof to respondents. It was only on 14
July 1995, through an ex-parte manifestation, that the affidavit of service was belatedly
submitted to this Court.
ISSUE
WON Filipino seafarers recruited by B. Sta. Rita Co. and deployed on board foreign
vessels outside the Philippines are exempt from the coverage of R.A. No. 1161
HELD
NO
- Respondent appellate court correctly upheld the validity of the Memorandum of
Agreement entered into between the DOLE and the SSS. The Memorandum of Agreement
was approved by the Social Security Commission per the Commission's Resolution No.
437, dated 14 July 1988. Upon the other hand, the Memorandum of Agreement is not a
rule or regulation enacted by the Commission in the exercise of the latter's quasilegislative authority Under Section 4 (a) of R.A. No. 1161.
- What the Memorandum of Agreement did was to record the understanding between the
SSS on the one hand and the DOLE on the other hand that the latter would include among
the provisions of the Standard Contract of Employment required in case of overseas
employment, a stipulation providing for coverage of the Filipino seafarer by the SSS. The
Memorandum of Agreement is not an implementing rule or regulation of the Social
Security Commission which, under Section 4 (a), is subject to the approval of the
President. Indeed, as a matter of strict law, the participation of the SSS in the
establishment by the DOLE of a uniform stipulation in the Standard Contract of
Employment for Filipino seafarers was not necessary; the Memorandum of Agreement
related simply to the administrative convenience of the two (2) agencies of government.
- Section 8 (j) (5) simply defines the term "employment" and does not in any way relate to
the scope of coverage of the Social Security System. That coverage is, upon the other
hand, set out in Section 9 of R.A. No. 1161 as amended, which defines the scope of SSS
coverage in the following terms:
Sec. 9 Compulsory Coverage. (a) Coverage in the SSS shall be compulsory upon all
employees not over sixty years of age and their employers; Provided, . . . .
(b) Fillpinos recruited in the Philippines by foreign employers for employment abroad
may be covered by the SSS on a voluntary basis. (As amended by Sec. 2, P.D. No.
177, S-1973 and Sec. 6, P.D. No. 735-S-1975) (Emphasis supplied)
- The Standard Contract of Employment to be entered into between foreign shipowners
and Filipino seafarers is the instrument by which the former express their assent to the
inclusion of the latter in the coverage of the Social Security Act. In other words, the
extension of the coverage of the Social Security System to Filipino seafarers arises by
virtue of the assent given in the contract of employment signed by employer and seafarer;
that same contract binds petitioner Sta. Rita or B. Sta. Rita Company, who is solidarily
liable with the foreign shipowners/employers.
- By extending the benefits of the Social Security Act to Filipino seafarers on board foreign
vessels, the individual employment agreements entered into with the stipulation for such
coverage contemplated in the DOLE-SSS Memorandum of Agreement, merely give effect
to the constitutional mandate to the State to afford protection to labor whether "local or
overseas." Nullification of the SSS stipulation in those individual employment contracts,
through nullification of the Memorandum of Agreement, constituted serious reversible error
on the part of the trial court. That petitioner should seek to deprive his countrymen of
social security protection after his foreign principal had agreed to such protection, is cause
for dismay and is to be deplored.
Disposition The Court Resolved to DENY the Petition for having been filed late, for
failure to comply with applicable Court Circulars and for lack of merit. The assailed
Decision of the Court of Appeals is hereby AFFIRMED. Cost against petitioner.

CORPORAL V NLRC (ENTENG)


341 SCRA 658
QUISUMBING; Octovber 2, 2000

Labor Law 1

A2010

NATURE
Special Civil Action.Certiorari
FACTS
- Petitioners have been working as barbers and manicurists in New Look Barber Shop for
many years. The children of the original owner decided to incorporate the barber shops
owned by their father.
- On April 15, 1995, the petitioners were told that the land and building where the barber
shop was housed will be sold thus their services were not longer required. thus they filed a
complaint for illegal dismissal. The Labor Arbiter and the NLRC dismissed the case on the
ground that there was no employer-employee relationship existing. They deemed the
relationship as one of joint venture.
ISSUE
WON NLRC committed rave abuse of discretion amounting to lack or excess of jurisdiction
considering the petitioners as independent contractors
HELD
YES
- The petitioners were registered with the Social Security System as regular employees of
the respondent. The SSS employment records show the employers ID was 03-0606200-1
and ID No. 03-8740074-7 for Lao Enteng. While it is no longer true that membership to
SSS is predicated on the existence of an employer-employee relationship since the policy
is now to encourage the self employed to become members, the court did not appreciate
the allegation of the respondent that the registration was only as a matter of
accommodation. The court held that it was unlikely that respondents would report certain
persons as their workers, pay the SSS premium as well as their wages if it were not true
that they were indedd their employees.
Reasoning
- The petitioners could not have been independent contractors as they fail to meet the test
of what constitute an independent contractor. they neither control or direct their own
activity not do they have substantial capital.
Disposition Petition granted

CHUA V CA
[PAGE 242]

3.04 EFFECT
EMPLOYMENT

OF

SEPARATION

FORM

3.05 REPORTING REQUIREMENTS


POBLETE CONSTRUCTION CO V ASIAIN
20 SCRA 1143
MAKALINTAL; August 30, 1967
NATURE
ORIGINAL ACTION in the Supreme Court. Certiorari with preliminary injunction.
FACTS
- Miguel Asiain was an employee of the Poblete Construction Company from 1956 until his
death on November 22, 1959, with a monthly salary of P300. Upon his death his widow,
Judith Asiain, for herself - and her minor children, filed a petition before the Social Security
Commission against the company 'and its manager, Domingo Poblete, to recover the
following sums:
(1) P3,600.00 equivalent to one year's salary of the deceased;
(2) P600.00 representing his unpaid salary for two months;
(3) P288.00 "representing the cash received by respondents from their laborers as
contribution to the family of the deceased;" and
(4) P2,000.00 by way of attorney's fees.
- Poblete Construction moved to dismiss the petition on the grounds that the Social
Security Commission, had no jurisdiction over the subject-matter and that the Judith Asiain
had no capacity to sue.
- February 25, 1960 - Commission denied the motion to dismiss and ordered Poblete
Construction to file their answer.
- March 9, 1960 - When no answer was forthcoming, Poblete Construction was declared in
default and the Asiain et al were allowed to present their evidence.

- 336 -

Disini

- September 15, 1960 - In its resolution, Commission declared itself without jurisdiction to
entertain the claims in the petition except the one for the sum of P3,600, which it awarded
on the basis of the evidence adduced at the hearing and pursuant to Section 24 of
Republic Act No. 1161, as amended.
- A subsequent motion for reconsideration filed by the respondents was denied
- Court of Appeals issued a writ of preliminary injunction to stop all further proceedings
below, including execution of the award.
ISSUE
WON the Commission has jurisdiction to entertain the claim of P3,600, which should have
been, presented before the ordinary courts
HELD
YES
- This claim was filed under Section 24 of the Social Security Act (R.A. 1161, as
amended), which provides:
'SEC. 24. Employment records and reports. - (a) each employer shall report
immediately to the System the names, ages, civil status, occupations, salaries and
dependents of all his employees who are in his employ and who are or may later be
subject to compulsory coverage: Provided, That if an employee subject to compulsory
coverage should die or become sick or disabled without the System having previously
received a report about him from his employer, the said employer shall pay to the
employee or his legal heirs damages equivalent to the benefits to which said employee
would have been entitled had his name been reported on time by the employer to the
System."
- There is no question that the deceased Miguel Asiain was subject to compulsory
coverage in the Social Security System, although the deceased's SSS Form E-1
(Employees' Date Record) was never filed with the Social Security System for the reason,
according to the company, that he refused to have his share of the corresponding monthly
contributions deducted from his salary. It was the duty of the employer to "report
immediately to the System" his name, age, civil status, occupation, salary and
dependents. Compliance with this duty did not depend upon the employee's willingness to
give his share of the contribution. Section 24 is mandatory, to such an extent that if the
employee should die or become sick or disabled without the report having been made by
the employer, the latter is liable for an amount equivalent to the benefits to which the
employee would have been entitled had such report been made.
- It is true that Section 24 uses the word "damages" in referring to the amount that may be
claimed. But this fact alone does not mean that the Social Security Commission lacks
jurisdiction to award the same. Section 5(a) of the Social Security Act provides that "the
filing, determination and settlement of claims shall be governed by the rules and
regulations promulgated by the Commission;" and the rules and regulations thus
promulgated state that "the effectivity of membership in the System, as well as the final
determination and settlement of claims, shall be vested in the Commission." The term
"claims" is broad enough to include a claim for damages under Section 24. Otherwise an
employer could nullify the jurisdiction of the Commission by the simple expedient of not
making a report as required by said Section. The collection of the employee's share is a
duty imposed by law, and his unwillingness to have it deducted from his salary does not
excuse the employer's failure to make the report aforesaid. It is precisely in this situation
that the employer is liable, and there is no question as to the amount of such liability in this
case.
Disposition The decision of the Social Security Commission is affirmed, and the writ of
preliminary injunction is dissolved, with costs against herein petitioner.

3.06 FUNDING
FUND OWNERSHIP
CMS ESTATE INC V SOCIAL SECURITY SYSTEM
[PAGE 2]

3.07 EFFECT OF NON-REMITTANCE


SSS FAILURE REMIT

Labor Law 1
SANTIAGO V CA (SSS)
133 SCRA 34
MELENCIO-HERRERA; October 31, 1984
NATURE
Petition to review decision of CA (affirming SSS commission resolution denying the
petition to credit in their favor the salary deductions made by their employer as premium
contributions and loan installment payments)
FACTS
- Santiago et. al. were employed by I-Feng Enamelling Co. They had been paying their
contributions, by way of salary deductions, their personal contributions to the SSS. During
their employment, they availed of salary loan benefits, installment payments to which were
also deducted from their salaries. The employer deducted the required amounts from their
salaries, but failed to remit them to the SSS.
- Petitioners then sought to have the amounts credited in their favor but the SSS
Commission denied this. CA affirmed.
- Petitioners contentions: there existed a contract of agency between SSS and the
employer, thus payment to the agent (employer) is payment to the principal (SSS).
ISSUE
WON premium contributions and loans payments by petitioners, which were deducted and
collected from their salaries by their employer, but not remitted to the SSS, should be
credited in their favor by the SSS
HELD
NO
Ratio As there is no contract of agency between the SSS and the employers, payment
made to the employer by way of salary deduction is in no way payment to SSS
Reasoning
- SSS circular no. 52 provides that it is the borrower who expressly authorizes his
employer and subsequent employers to deduct from his salary the installments due on his
salary loan. The employer then remits the installments due to the SSS in accordance with
rules that the SSS has laid down. The employer, in so deducting the installment payments
from the borrower, does so upon the latter's authorization. The employer is merely the
conduit for remitting the premiums for reasons of administrative convenience and
expediency in order that SSS members may be served efficiently and expeditiously. No
contract of agency, in the legal sense, therefore may be said to exist between the
employer and the SSS.
- as correctly observed by the CA: To rule otherwise would be to open the door for
unscrupulous employers to circumvent the law by not remitting their collections of salary
loans installment payments from employees since, anyway, the SSS would credit them
with what they had paid to the employer even though the latter fails to remit them to the
SSS.
- there is a difference WRT to premium contributions (from loan payments):
Clearly, if the employer neglects to pay the premium contributions, the SSS may proceed
with the collection in the same manner as the BIR in case of unpaid taxes. Plainly, too,
notwithstanding non-remittance by employers of the premium contributions, covered
employees are entitled to the benefits of the coverage, such as death sickness, retirement,
and permanent disability benefits. These benefits continue to be enjoyed by the
employees by operation of law and not, as petitioners allege, because the premium
contributions and salary loan installment payments have already became the money of the
SSS upon payment by the employees to the employer. It should be remembered that
funds contributed to the SSS by compulsion of law are funds belonging to the members,
which are merely held in trust by the government. The mentioned benefits, however, do
not include the salary loan privileges that member-employees apply for. The SSS may or
may not grant those loans pursuant to its rules and regulations. The salary loans are not
covered by law but by contract between the SSS as lender, and the private employee, as
borrower.
Disposition Petition dismissed. CA decision AFFIRMED.

SEPARATE OPINION
PLANA [concur]

A2010

- 337 -

Disini

- Who bears the loss of unremitted SSS premium contributions and salary loan
repayments previously withheld from the salaries of employees in private enterprises in
case the employer who has misappropriated the same fails to make restitution?
- the law as it stands seems inadequate to protect either the interest of the employees or
the SSS. Thus, with respect to unremitted salary loan re-payments, the employees have to
shoulder the loss, if the employer is insolvent. On the other hand, as to premium
contributions, the SSS and ultimately the members of the SSS must suffer the employer's
misconduct and insolvency.

BENEDICTO V ABAD SANTOS


183 SCRA 434
FELICIANO; March 21, 1990
NATURE
Petition for Prohibition and Mandamus with preliminary injunction
FACTS
- Benedicto employed the late Salvador Pillon as truck driver. He however, did not report
Pillon's employment to the Social Security System ("SSS") for compulsory coverage and
did not pay the corresponding SSS contributions.
- This came to the knowledge of the SSS sometime in 1975.
- After Pillon's death, an investigator of the SSS was deputized to conduct an enquiry in
respect of Benedicto's alleged violations of theSocial Security Act (RA 1161).
- The report stated that Benedicto had admitted having failed to report and to register with
the SSS Pillon's employment for the period from March 1971 up to the time of Pillon's
death in 1974, and to pay the corresponding SSS contributions.
- Ten years later, upon complaint of the Legal Department of the SSS, the Assistant City
Fiscal of Bacolod City filed an information charging petitioner Benedicto with violations of
the Social Security Act.
- Benedicto moved to quash the information asserting that his liability had already been
extinguished by prescription.
- SSS opposed the Motion to Quash contending that the offense charged in the
information had not yet prescribed since the applicable prescriptive period was twenty (20)
years and since the information was filed within this 20-year period.
- Respondent Judge denied petitioner Benedicto's Motion to Quash .
(Paragraph B of Section 22 of the Republic Act 1161 as amended by Section 15 of the PD
1636 'The right to institute the necessary action against the employer may be commenced
within Twenty Years (20) from the time the Delinquency is known or the assessment is
made by the SSS, or from the time the Benefit Accrues, as the case may be.')
- Petitioner moved for reconsideration of the above order, without success.
Petitioners claim:
- the 20-year prescriptive period established in Section 22 (b) of the Social Security Act
applies only in respect of administrative and civil actions involving delinquency in the
remittance of contributions to the SSS, and not to a criminal action like that initiated in the
case at bar
- since the Social Security Act had not established a prescriptive period in respect of
criminal liability for violations of the Social Security Act or the Rules and Regulations
promulgated by the Social Security Commission concerning registration of employees and
remittance of contributions to the SSS, the applicable statute of limitations was four (4)
years under Act No. 3326.
"SEC. 22. Remittance of Contributions.
(b) The contribution payable under this Act in cases where an employer refuses or
neglects to pay the same shall be collected by the SSS in the same manner as taxes
are made collectible under the' National Internal Revenue Code, as amended. Failure
or refusal of the employer to pay or remit the contributions herein penalized shall not
prejudice the right of the covered employee to the benefits of the employee.
The right to institute the necessary action against the employer may be commenced
within twenty (20) years from the time the delinquency is known or the assessment is
made by the SSS, or from the time the benefit accrues, as the case may be."
"Sec. 28. Penal Clause-x x x
(e) Whoever fails or refuses to comply with the provisions of this Act or with the rules
and regulations promulgated by the Commission, shall be punished by a fine of not
less than five hundred pesos nor more than five thousand pesos, or imprisonment for
not less than six months nor more than one year, or both, at the discretion of the court:
Provided, That, where the violation consists in failure or refusal to register employees,
or to deduct contributions from employee's compensation and remit the same to the
SSS. the penalty shall be a fine of not less than five hundred pesos nor mom than five
thousand pesos and imprisonment for not less than six months nor more than one
year."
- Act No. 3326, as amended by Act No. 3763 provides as follows:
"SECTION 1. Violations penalized by special acts shall, unless otherwise provided in
such acts, prescribe in accordance with the following rules: (a) after a year for offenses
punished only by a fine or by imprisonment for not more than one month, or both; (b)

Labor Law 1
after four years for those punished by imprisonment for more than one month, but less
than two years;. Provided, however, That all offenses against any law or part of law
administered by the Bureau of Internal Revenue shall prescribe after five years.
Violations penalized by municipal ordinances shall prescribe after two months.
SSS argument:
- Act No. 3326, as amended, was not applicable to the instant case for the reason that the
Social Security Act itself had established a statute of limitations in section 22 (b) of the
Social Security Act as amended by P.D. No. 1636,
- the 20-year statute of limitations embodied in Section 22 (b) is applicable in respect of all
kinds of actions against a delinquent employer, whether criminal or civil.
ISSUE
WON the offense with which petitioner Benedicto is charged has prescribed
HELD
YES
- The statutory crime here charged had prescribed by then, the prescriptive period here
applicable being four (4) years.
Reasoning
- The 20-year prescriptive period is found in Section 22 of the Social Security Act which
deals generally with the remittance of contributions to the SSS. Section 22 (b) does not
deal with penal sanctions for violations of provisions of the Social Security Act nor of the
rules and regulations promulgated by the Social Security Commission.
- a 20-year statute of limitation would constitute a disproportionately long statute of
limitations if one compared it with either the applicable prescriptive periods in respect of
crimes punishable under the Revised Penal Code or the prescriptive periods applicable to
crimes punished by special statutes under Act No. 3326.
- to apply a single, uniform 20-year prescriptive period to both civil actions for collection of
unremitted SSS premiums and to criminal prosecutions for violations of provisions of the
Social Security Act and of rules and regulations promulgated by the Social Security
Commission, would be both grossly disproportional and too simplistic.
- This is concerned only with the extinguishment of the criminal liability for the offense
charged in the information and does not carry with it the extinction of the civil action to
enforce civil liability arising from the offense charged.
- The information against petitioner Benedicto itself specified the amount of the civil
liability-P6,381.00, being the amount equivalent to the benefits which the heirs of the
deceased Salvador Pillon would have been entitled to had Pillon been reported and
registered with the SSS on the time and the necessary contributions remitted to the SSS.
That amount will, upon collection, be payable to Salvador Pillon's heirs.

CHUA V CA
[PAGE 242]

3.08 BENEFITS
GSIS RETIREMENT BENEFITS
CONTE V COMMISSION ON AUDIT
264 SCRA 19
PANGANIBAN; November 4, 1996
NATURE
Petition for certiorari of a decision of COA.
FACTS
- SSS Resolution No. 56 [RES.56, for brevity], approved on 1971, provides incentive to
retiring SSS employees to avail of the retirement benefits under RA 660 as amended, 57
rather than those under RA 1616 as amended, 58 by giving them financial assistance
equivalent to the difference between what a retiree would have received under RA 1616,
less what he was entitled to under RA 660. The benefits under RA 660 are entirely
payable by GSIS while those under RA 1616 are primarily shouldered by SSS. RES.56
came about upon observation that qualified SSS employees have invariably opted to retire
under RA 1616 instead of RA 660 because the benefit under the former is much greater
than that under the latter. As a consequence, the SSS usually ended up virtually paying
the entire retirement benefit, instead of GSIS which is the main insurance carrier for
57

An Act to amend [CA No. 186] entitled An Act to create and establish a Government Service Insurance System,
and to provide retirement insurance and for other purposes. (1951)
58
An Act further amending [CA No. 186], as amended, by prescribing two other modes of retirement and for other
purposes. (1957)

A2010

- 338 -

Disini

government employees. In other words, RES.56 was issued not only to equalize the
benefits received under both RAs, but also as a cost-cutting device for SSS.
- PETITIONERS were former SSS employees who, upon retirement, availed of the
benefits under RA 660, and as expected, they sought payment from SSS of the benefits
under RES.56. SSS denied their request pursuant to a ruling by COA disallowing all
claims under RES.56. The ruling stated that SSS scheme of financial assistance is similar
to the separate retirement plans by other government agencies which results in the
increase of benefits beyond what is allowed under existing retirement laws. The law
referred to is RA 4968 59 which amended CA No.186 by providing that no retirement
plan for employees shall be created by employer. All supplementary retirement
plans in force in any government office, are hereby declared inoperative.
- Consequently, PETITIONERS sought reconsideration of the COA ruling. COA denied the
request; hence, this petition.
ISSUE
WON COA abuse its discretion when it disallowed PETITIONERS claims for benefits
under RES.56
HELD
NO
Ratio RES.56 itself states: Resolved, further, that SSS employees who availed
themselves of the said life annuity (under RA 660), in recognition of their long and
faithful service, be granted financial assistance. It is plain to see that the grant of said
financial assistance is inextricably linked with the application for and approval of retirement
benefits under RA 660 availment of said assistance under RES.56 may be done only in
conjunction with the availment of benefits under RA 660, and the former (RES.56) is in
supplementation of the latter benefits (RA 660). CA 186, as amended by RA 4968, in no
uncertain terms bars the creation of any insurance or retirement plan - other than the
GSIS - for government employees, in order to prevent the undue and inequitous
proliferation of such plans. It is beyond cavil that RES.56 contravenes the provision of law
and is, therefore, void.
- We are not unmindful of the laudable purposes for promulgating RES.56, and the
positive results it must have had, not only in reducing costs of the SSS in connection with
the pay-out of retirement benefits and gratuities, but also in improving the quality of life for
scores of retirees. But it is simply beyond dispute that the SSS had no authority to
maintain and implement such retirement plan, particularly in the face of the statutory
prohibition. The SSS cannot, in the guise of rule-making, legislate or amend laws or
worse, render them nugatory.
- The scheme cannot also be likened to honoraria received by employees of DOJ and of
the Office of the Solicitor-General. As clarified by the Solicitor-General himself, the
honoraria are travelling and/or representation expenses incurred by them in the course
of handling cases, attending hearings, or performing other field work. These monetary
benefits are given upon rendition of service while the financial benefits under [RES.56]
are given upon retirement from service.
- Finally as equity is applied only in the absence of statutory law or rules of procedure, it
cannot be applied to give effect to RES. 56, which directly contravenes the clear mandate
of RA 4968. But it can be still be invoked for it is clear appears that PETITIONERS are
simply innocent bystanders in this whole bureaucratic rule-making drama; to the extent
possible, they ought not be penalized as a result of the subsequently determined invalidity
of RES. 56, the promulgation and implementation of which they had nothing to do with.
PETITIONERS, being qualified to avail of benefits under RA 660, may also readily qualify
under RA 1616. It would therefore not be misplaced to enjoin the SSS to render all
possible assistance to PETITIONERS for the prompt processing and approval of their
applications under RA 1616, and in the meantime, unless barred by existing regulations, to
advance to petitioners the difference between the amounts due under RA 1616, and the
amounts they already obtained, if any, under RA 660.
Disposition Petition DISMISSED.

ECSIF CASES

DEFINITION DISABILITY
EMPLOYEES COMPENSATION COMMISSION (SSS) V
SANICO
59

The Teves Retirement Law, which took effect in 1967.

Labor Law 1
34 SCRA 269
KAPUNAN; December 17, 1999
NATURE
Petition for review of the decision of the CA
FACTS
- Private respondent was a former employee of John Gotamco and Sons. He worked in
said company as wood filer from 1986 until he was separated from employment on 31
December 1991 due to his illness.
- His medical evaluation report, dated 31 September 1991, showed that he was suffering
from pulmonary tuberculosis (PTB). Subsequent chest x-rays taken on 9 October 1994
and 3 May 1995 diagnostically confirmed his illness.
- On 9 November 1994, private respondent filed with SSS a claim for compensation
benefits under P.D. No. 626, as amended.
- SSS denied private respondents claim on the ground of prescription. The SSS ruled
that under Article 201 of the Labor Code, a claim for compensation shall be given due
course only when the same is filed with the System three (3) years for the time the cause
of action accrued.
- In private respondents case, the SSS reckoned the three-year prescriptive period on 21
September 1991 when his PTB first became manifest. When he filed his claim on 9
November 1994, the claim had allegedly already prescribed.
- CA reversed petitioners decision and granted private respondents claim for
compensation benefits. In ruling that private respondents claim was filed well within the
prescriptive period under the law, the CA reconciled Article 201 of the Labor Code with
Article 1144(2) of the Civil Code.
ISSUE
WON private respondents claim for compensation benefit had already prescribed when
he filed his claim on 9 November 1994
HELD
NO
Ratio Disability should not be understood more on its medical significance but on the loss
of earning capacity. Permanent total disability means disablement of an employee to earn
wages in the same kind of work, or work of similar nature that [he] was trained for or
accustomed to perform, or any kind of work which a person of [his] mentality and
attainment could do. It does not mean absolute helplessness. In disability compensation,
it is not the injury which is compensated, but rather it is the incapacity to work resulting in
the impairment of ones earning capacity.
Reasoning
- In determining whether or not private respondents claim was filed within the three-year
prescriptive period under Article 201, petitioner and the SSS reckoned the accrual of
private respondents cause of action on 31 September 1991, when his PTB became
known. This is erroneous.
- the prescriptive period for filing compensation claims should be reckoned from the time
the employee lost his earning capacity, i.e., terminated from employment, due to his illness
and not when the same first became manifest. Indeed, a persons disability might not
emerge at one precise moment in time but rather over a period of time.
- the Court takes this opportunity to once again remind petitioner that P.D. No. 626, as
amended, is a social legislation whose primordial purpose is to provide meaningful
protection to the working class against the hazards of disability, illness and other
contingencies resulting in the loss of income.
Disposition instant petition is hereby DISMISSED.

SSS V CA (RAGO)
439 SCRA 239
DAVIDE; September 27, 2004
NATURE
Petition for review of CA decision and resolution which reversed the resolution of the
Social Security Commission denying respondent Ragos request to convert his monthly
pension from permanent partial disability to permanent total disability
FACTS
- Private respondent Rago, an electrician for Legend Engineering in Cebu, sustained an
injury while working on the ceiling of a building. He stepped on a weak ceiling joist which
gave way and caused him to crash into a corridor 12 feet below. X-rays revealed that he
had a marked compression fracture without signs of dislocation and bone destruction as
well as slight kyphosis (curvature or deformity of the upper spine), with the alignment of
the spine still normal. He was confined in the hospital for almost a month and thereafter
confined in his home for 8 months from Dec.25, 1993 to Aug. 25, 1993.

A2010

- 339 -

Disini

- On Nov. 1995, Rago filed a claim for Employees Compensation (EC) sickness benefit
which was approved for a maximum of 120 days to cover the period from Dec 1993 to
March 1994. He then filed another claim to convert his SSS disability to EC disability
which was again resolved in his favor. A claim for the extension of his EC partial disability
was likewise granted. Rago, thereafter, filed several requests for the adjustment of his
partial disability to total disability but they were denied by the SSS based on medical
findings that he was not totally prevented from engaging in any gainful occupation. Rago,
undaunted, filed with the SSC a petition for total permanent disability benefits on the
grounds that he had been confined both in the hospital and at home for 268 days (under
SSC guidelines, injury persisting for more than 240 days is considered permanent total
disability), x-rays show a deteriorating condition, and he had lost his original capacity to
work as an electrician and has been unemployed since the accident. On the other hand,
the SSS argued that Rago had already been granted the maximum partial disability
benefits. A physical examination showed that he was more than capable of physically
engaging in any gainful occupation thus, it recommended the denial of the Ragos petition.
- The SSC later denied Ragos petition for lack of merit. It ruled that he was not entitled to
permanent partial disability more than what was already granted, more so to permanent
total disability benefits since he was already granted the maximum allowable benefit for
his injury.
- Rago did not file a MFR but instead appealed to the CA by filing a petition for review and
reiterating his claim for permanent disability benefits. The CA reversed the SSCs
resolution. MFR was filed by SSC but the same was denied by the CA. Thus, in this
petition, the SSS and the SSC pray to set aside the CA decision and resolution and to
have the case remanded to the SSC for further proceedings.
***Procedural matters: The SSC challenged the immediate appeal by Rago to the CA
and likewise the CAs decision to take cognizance of the case. On this issue, the SC held:
Rago disregarded procedural and curative rules in taking immediate recourse to the
appellate court. The CA similarly erred in taking cognizance of the appeal... Nevertheless,
to require Rago to comply with the principle of exhaustion of administrative remedies at
this stage of the proceedings would be unreasonable, unjust and inequitable. It would
prolong needlessly and uselessly the resolution of this claim. The principle of exhaustion
of remedies is not an ironclad rule. It may be disregarded when, as in this case, requiring
the exhaustion of administrative remedies would be unreasonable.
ISSUE
WON Rago is entitled to permanent total disability
HELD
YES
Ratio An employee who is unable to perform his customary job for more than 120 days
because of the injury or sickness he sustained and does not come within the coverage of
Rule 10 of the Amended rules on EC, suffers from permanent total disability regardless
of WON he loses the use of any part of his body.
- A temporary total disability lasting continuously for more than 120 days, shall be
considered permanent.
- The test of WON an employee suffers from permanent total disability is a showing of
the capacity of the employee to continue performing his work notwithstanding the disability
he incurred.
Reasoning
- The CA correctly observed that Ragos injury made him unable to perform any gainful
occupation for a continuous period exceeding 120 days. The SSS had granted Ragos
sickness benefit for 120 days and, thereafter, permanent partial disability for 38 months.
Such grant is an apparent recognition by the SSS that his injury is permanent and total as
we have pronounced in several cases. This is in conformity with Section 2(b), Rule VII of
the Amended Rules on EC which defines a disability to be total and permanent if, as a
result of the injury or sickness, the employee is unable to perform any gainful occupation
for a continuous period exceeding 120 days, and Section 1, b(1) of Rule XI of the same
Amended Rules which provides that a temporary total disability lasting continuously for
more than 120 days, shall be considered permanent.
- In Vicente v Employees Compensation Commission, we laid down the litmus test and
distinction between Permanent Total Disability and Permanent Partial Disability:
While permanent total disability invariably results in an employees loss of work or
inability to perform his usual work, permanent partial disability, on the other hand,
occurs when an employee loses the use of any particular anatomical part of his body
which disables him to continue with his former work. Stated otherwise, the test of
whether or not an employee suffers from permanent total disability is a
showing of the capacity of the employee to continue performing his work
notwithstanding the disability he incurred. Thus, if by reason of the injury or
sickness he sustained, the employee is unable to perform his customary job for more
than 120 days and he does not come within the coverage of Rule X of the Amended
Rules on EC (which describes what constitutes temporary total disability), then the said
employee undoubtedly suffers from permanent total disability regardless of WON he
loses the use of any part of his body.
- Disability should be understood less on its medical significance than on the loss of
earning capacity. Permanent total disability means disablement of an employee to earn

Labor Law 1
wages in the same kind of work, or work of similar nature that he was trained for or
accustomed to perform, or any kind of work which a person of his mentality and attainment
could do. It does not mean absolute helplessness. Moreover, a persons disability may not
manifest fully at one precise moment in time but rather over a period of time. It is possible
that an injury which at first was considered to be temporary may later on become
permanent or one who suffers a partial disability becomes totally and permanently
disabled from the same cause.
Disposition CAs decision and resolution are AFFIRMED

MANIFESTATION
GSIS V CA (LIWANAG)
285 SCRA 430
DAVIDE; September 28, 1998
NATURE
Appeal from decision of CA
FACTS
- Jaime Liwanag was a Senior Superintendent of the PNP at the time of his death on Sept
14, 1994. He was 48 years old and had served the police force for 27 years. Zenaida
Liwanag, private respondent, is his surviving spouse.
- On Aug 28, 1994, P/Sr. Supt. Jaime Liwanag was admitted at the Medical Center of
Manila. CT Scan showed Cirrhosis with probable Hepatocellular CA, HB 5A3 positive.
Despite medical intervention, Jaime Liwanag succumbed to Upper GI Bleeding, Cirrhosis
Secondary to Hepatitis B; Hepatocellular Carcinoma on Sept 14, 1994.
- Zenaida filed a claim with GSIS for compensation benefits. The claim was denied for not
being an occupational disease under the law, neither was the risk of contracting the
ailment of the deceased increased by his employment as a member of the police force.
- On appeal the Employees Compensation Commission affirmed the GSIS ruling and
dismissed the appeal for lack of merit, ruling that Sec 1 (B), Rule III of the Amended Rules
on Employees' Compensation clearly provides: For the sickness and the resulting
disability or death to be compensable, the sickness must be the result of an occupational
disease listed under Annex "A" of these rules with the conditions set therein satisfied;
otherwise proof must be shown that the risk of contracting the disease is increased by the
working conditions. It has not been shown that Jaimes ailments were among those listed
as occupational diseases, nor has proof been shown that risk of contracting the diseases
were increased by his work.
CA proceedings
a. Private respondent Zenaida
Zenaida relied heavily on two documents as proof of the causal relation between
Jaimes ailments and his employment and working conditions:
"Investigation Report Re Death of the Late P/SSUPT. JAIME M. LIWANAG" dated
14 Sept 1994 submitted by Cristeto Rey R. Gonzalodo, Police Chief Inspector,
Investigator on Case. In essence, the report says:
Jaime had constantly undergone physical/medical examination throughout the
course of his employment and all the while was found to be physically and
mentally fit for the service.
It is highly believed that Jaime acquired his illness in the course of his
employment with PNP considering that there are some personnel in his office
who were positive with Hepatitis B (Reactive) virus.
The report concluded that the death of Jaime was in Line of Duty and not
attributable to his own misconduct or negligence.
REPORT OF PROCEEDINGS OF LINE OF DUTY (LOD) BOARD TO DETERMINE
THE LINE OF DUTY STATUS OF THE LATE P/SSUPT. JAIME J. LIWANAG PNP."
In essence, the report says:
Based on the record, the immediate cause of death of Jaime was CardioRespiratory Arrest Secondary to Gastro-Intestinal bleeding as a result of
fulminating Hepatitis.
It is highly possible that he got infected in the Directorate for Plans since there
were 5 other ODPL personnel out 45 who are reactive to Hepatitis B Antigen
Test. Modes of transmittal are through body fluids and secretion. All the
immediate members of his family are negative [for the] Hepatitis B Virus.
It
was strongly believed that he got this while working at the Headquarters since
this is was already endemic in the camp.
The type of Hepatitis B acquired by Jaime was of Acute Fulminant Type. The
effect is so immediate that one out of ten usually dies.
It was raised that under Ministry of National Defense DO No 162 dated 15 Jan
65, a military personnel who died, while in the Active Service is presumed to
have died in Line of Duty and not as a result of his own misconduct unless there
is substantial evidence to rebut such presumption. This was still binding upon
PNP personnel.

A2010

- 340 -

Disini

It was concluded that Jaime died in the line of duty.


Zenaida also argued that the requirement of proof of a causal relation between a
claimant's ailments and his employment and working conditions "admits of exceptions
and must yield to the higher interests of justice."
b. GSIS
the burden then lay on Zenaida to prove that the risk of contracting the disease was
increased by her late husband's working conditions and employment as a member of
the PNP.
Investigation Report fallaciously concluded that Jaime contracted Hepatitis B in the
course of his employment as some of his co-workers tested positive. The ailments of
Jaime were not inherent among policemen and everybody was susceptible to the
disease regardless of one's work.
There was no substantial evidence pointing to a reasonable connection, much less, a
direct causal relation, between Jaimes ailments and the nature of his employment
c. ECC
echoed the arguments of GSIS.
- CA ruled in favor of Zenaida, citing the reports relied upon by Zenaida, and adding that
under the law, it is not required that the employment [is] the sole factor in the growth,
development and acceleration of his illness. It is enough if his employment had
contributed, even in small degree, to the development or acceleration of the disease.
Petitioners Claims:
- CA has forgotten that PNP investigation was only for purpose of determining the line of
duty status of the [deceased] and if his ailment was work connected.
Hepatitis B cannot be acquired by mere mingling with other people who test positive for
the illness. Hepatitis B is not just acquired by simple association. There was no medical
proof/evidence presented how the [deceased] could have acquired his illness.
ISSUE
WON CA committed an error of law by misappreciating the legal standard of what
constitutes substantial evidence; and in according full credence to the proceedings before
the PNP Board and thus shifting the burden of evidence to petitioner to rebut private
respondent's claim, when private respondent's evidence was sorely wanting to justify the
award of compensation benefits under

P.D. No. 626, as amended

HELD
YES
- there is no dispute that Hepatitis B is not listed as an occupational disease under Annex
"A" of the Rules on Employees' Compensation. As such, private respondent's burden of
evidence before the ECC was to prove, by substantial evidence, the causal relationship
between her deceased husband's illness and his working conditions. This she failed to do.
- Zenaida merely relied on the PNP Reports, and nothing more, to substantiate her claim.
However, the PNP reports merely contained sweeping statements and conclusions and
treated the matter in a most perfunctory manner. There was merely the general averment
that five out of 45 ODPL personnel contracted Hepatitis B. There was no showing
whatsoever as to the degree of contact, if any, between the deceased and his office mates
who contracted Hepatitis B.
- Ang Tibay v CIR: mere uncorroborated hearsay or rumor does not constitute substantial
evidence. Substantial evidence is more than a mere scintilla. It means such relevant
evidence as a reasonable mind might accept as adequate to support a conclusion.
On PD 626 governing employees compensation benefits
- PD 626 abandoned the presumption of compensability and the theory of aggravation
under the Workmen's Compensation Act.
- for the sickness and resulting disability or death to be compensable, the claimant must
prove either of two (2) things: (a) that the sickness was the result of an occupational
disease listed under Annex "A" of the Rules on Employees' Compensation; or (b) if the
sickness is not so listed, that the risk of contracting the disease was increased by the
claimant's working conditions.
- claimant must prove this causal relation between the ailment and working conditions by
substantial evidence, since the proceeding is taken before the ECC, an administrative or
quasi-judicial body. Within the field of administrative law, while strict rules of evidence are
not applicable to quasi-judicial proceedings, nevertheless, in adducing evidence
constitutive of substantial evidence, the basic rule that mere allegation is not evidence
cannot be disregarded.
- in case of doubt in construction and interpretation of social legislation statutes, the
liberality of the law in favor of the working man and woman prevails in light of the
Constitution's social justice policy.
On undeserving claims for compensation
If diseases not intended by the law to be compensated are inadvertently or
recklessly included, the integrity of the State Insurance Fund is endangered. Compassion
for the victims of diseases not covered by the law ignores the need to show a greater
concern for the trust fund to which the tens of millions of workers and their families look to
for compensation whenever covered accidents, diseases and deaths occur.

Labor Law 1
- This stems from the development in the law that no longer is the poor employee still
arrayed against the might and power of his rich corporate employer, hence the necessity
of affording all kinds of favorable presumptions to the employee. This reasoning is no
longer good policy. It is now the trust fund and not the employer which suffers if benefits
are paid to claimants who are not entitled under the law. The employer joins the employee
in trying to have their claims approved. The employer is spared the problem of proving a
negative proposition that the disease was not caused by employment.
- the new system instituted by the new law has discarded, among others, the concept of
"presumption of compensability and aggravation" and substituted one based on social
security principles. The new system is administered by social insurance agencies the
GSIS and the SSS under the ECC. The purpose of this innovation was to restore a
sensible equilibrium between the employer's obligation to pay workmen's compensation
and the employee's right to receive reparation for work-connected death or disability.
Disposition Petition granted. Decision of CA reversed and set aside. Decision of ECC
reinstated.

DISTINCTION DISABILITY
GSIS V CA (BELLA)
304 SCRA 243
PURISIMA; March 4, 1999
FACTS
- Romeo S. Bella was employed by the Bureau of Animal Industry as a livestock inspector
for 22 yrs. After that, he was employed by the Department of Agriculture as Agriculturist II.
- On July 1995, Bella, who was then 56 years old, filed a terminal leave of absence due to
physical disability.
- His medical records revealed that he was suffering from Acute Myocardial Infraction (a
disease characterized by on- and- off shortness of breath, and effort related chest pains)
and was confined at the Notre Dame Hospital from Sept 13, 1988 to Sept 19, 1988 and at
the Philippine Heart Center from Sept 6, 1994 to Sept 26, 1994
- For this, Bella filed with the GSIS a claim for compensation benefits under P.D. 626
where the GSIS awarded him a Temporary Total Disability income benefit during the
periods of July 16 to July 21, 1994 and Aug 24 to Aug 29, 1994, as well as reimbursement
for medical expenses.
- He was also granted a Permanent Partial Disability income benefit equivalent to thirtyeight months for his Ischemic Cardiomayopathy.
- Bella requested for the conversion of his benefits from Permanent Partial Disability to
Permanent Total Disability, reasoning out that his ailments of Ischemic Cardiomayopathy
(characterized by recurring attacks of fainting accompanied by headache, occassional loss
of consciousness and amnesia) and Chronic Obstructure Pulmonary Disease (patients
typically have chronic productive cough for many years, followed by slowly progressive
breathlessness with decreasing amounts of exertion) rendered him unable to engage in
any gainful occupation for a continuous period exceeding 120 days, as certified to by his
five attending physicians.
- GSIS denied his request for Permanent Total Disability on the ground that the degree of
private respondents disability as evaluated by petitioners medical officers, did not satisfy
the criteria for Permanent Total Disability. His MFR was also denied. On appeal, the
Employees Compensation Commission (ECC) affirmed the Decision of the GSIS.
- Bella filed Petition for Review with the CA. CA reversed the decision of the ECC.
ISSUE
WON the private respondent is entitled to permanent total disability benefits
HELD
YES
-The Labor Code classifies employees disability into three distinct categories, namely:
a) temporary total disability; [Art 191] b) permanent total disability; [Art 192] and c)
permanent partial disability. [Art 193]
- Section 2, Rule VII, of the Rules and Regulation Implementing Title II, Book IV of the
Labor Code defines and clarifies these categories, as follows:
"SEC. 2. Disability. - (a) A total disability is temporary if as a result of the injury or
sickness the employee is unable to perform any gainful occupation for a continuous
period not exceeding 120 days, except as otherwise provided for in Rule X of these
Rules.
(b) A disability is total and permanent if as a result of the injury or sickness the
employee is unable to perform any gainful occupation for a continuous period
exceeding 120 days except as otherwise provided for in Rule X
(c) A disability is partial and permanent if as a result of the injury or sickness the
employee suffers a permanent partial loss of the use of any part of his body."
- In Vicente vs. Employees Compensation Commission, the Court laid down the litmus test
and distinction between Permanent Total Disability and Permanent Partial Disability, to wit:

A2010

- 341 -

Disini

"x x x while permanent total disability invariably results in an employees loss of


work or inability to perform his usual work, permanent partial disability, on the other
hand, occurs when an employee loses the use of any particular anatomical part of his
body which disables him to continue with his former work. Stated otherwise, the test of
whether or not an employee suffers from permanent total disability is a showing
of the capacity of the employee to continue performing his work notwithstanding the
disability he incurred. Thus, if by reason of the injury or sickness he sustained, the
employee is unable to perform his customary job for more than 120 days and he does
not come within the coverage of Rule X of the Amended Rules on Employees
Compensability (which, in a more detailed manner, describes what constitutes
temporary total disability), then the said employee undoubtedly suffers from permanent
total disability regardless of whether or not he loses the use of any part of his body.
"total disability does not mean a state of absolute helplessness, but disablement of an
employee to earn wages in the same kind of work or a work of similar nature, that he
was trained or accustomed to perform, or any kind of work which a person of his
mentality and attachments could do."
- The fact that he was forced to retire at the early age of 56, due to a sickness disabling
him from performing his job as Agriculturist II, qualifies his disability as a Permanent Total
Disability, though he lost no use of any particular anatomical part of his body.
- Also, no less than five doctors certified that private respondents illness disabled him
from performing any gainful occupation for a continuous period exceeding 120 days.
- It is then beyond cavil that the sickness of the private respondent made him unable to
perform any gainful occupation for a continuous period exceeding 120 days, thus entitling
him to permanent total disability benefits.
Disposition petition DENIED, the Decision of CA affirmed

PERMANENT TOTAL DISABILITY


EMPLOYEES COMPENSATION COMMISSION V
SANICO
[PAGE 336]
GOVERNMENT SERVICE INSURANCE SYSTEM V CA
[PAGE 338]
SANTOS V EMPLOYEES COMPENSATION
COMMISSION
[PAGE 330]
NARAZO V EMPLOYEES COMPENSATION
COMMISSION
181 SCRA 874
PADILLA; February 6, 1990
NATURE
Petition for review of decision of Employees Compensation Commission
FACTS
- Geronimo Narazo worked for 38 years as Budget Examiner in the Office of the Governor
in Negros Occidental whrere he prepared the budget of the Province, financial reports and
reviewed or examined of the budget of some provincial and municipal offices.
- May 14, 1984 Geronimo died at age of 57. Before his death, he had been confined in
the hospital for urinary retention, abdominal pain and anemia. He was later diagnosed to
have uremia. One of the causes of the disease is obstruction in the flow of urinary waste
products.
- Geronimos widow Amalia filed a claim with GSIS for death benefits for the death of her
husband under the Employees' Compensation Law. The claim was, however, denied
because the cause of death of Narazo is not listed as an occupational disease. There was
also no showing that Geronimos job had increased the risk of contracting uremia.
- Amalia asked GSIS to reconsider but her request was denied. The Employees
Compensation Commissin affirmed the GSIS ruling on the ground that the ailments of the
deceased could not be attributed to employment factors. Medical opinion also indicates
that uremia is quite common among men over 50 regardless of occupation.
Petitioners Claim
- Amalia claimed that although uremia is not considered an occupational disease,
Geronimos job as Budget Examiner required long hours of work and he had to deal with
stress and pressure which caused him many times to delay urination and this eventually
led to the development of illnesses.
Respondents Comments:

Labor Law 1
- The ECC argues that Amalia did not show proof that uremia is work-connected and that
there was no medical basis for Amalia to allege that it was her husbands delayed
urination which led to his sickness.
ISSUE
WON Amalias claim for death benefits should be granted
HELD
YES
Ratio Compensable sickness is defined as any illness definitely accepted as an
occupational disease listed by the ECC or any illness caused by employment subject to
proof by the ernployee that the risk of contracting the same is increased by working
conditions.
Reasoning
- A sickness or death caused by said sickness is compensable if the same is listed as an
occupational disease. If it is not so listed, compensation may still be recovered if the
illness was aggravated by employment. However, it is incumbent upon the claimant to
show proof that the risk of contracting the illness was increased by his working conditions.
- Uremia is not among the listed occupational diseases. To establish the right to be
compensated, Amalia must show proof of reasonable work-connection, not necessarily
direct causal relation.
- The degree of proof required is merely substantial evidence which means such relevant
evidence as will support a decision, or clear and convincing evidence.
- Geronimos work required him to delay his urination often because he had to forego
going to the restroom so as not to interrupt the concentration required in his work.
- In the case of Ceniza v. ECC the Court held that prolonged sitting down and putting off
urination result in stagnation of the urine which promote the growth of micro organisms
which could infect the urinary tract. Thus even if these illnesses are not directly caused
by the nature of the dutiesthe risk of contracting the same is certainly aggravated by
their working habits necessitated by demands of job efficiency.
Disposition petition is GRANTED. The decision of the Employees' Compensation
Commission denying petitioner's claim for benefits arising from the death of her husband,
is hereby REVERSED and SET ASIDE.

UNKNOWN CAUSE
RARO V EMPLOYEES COMPENSATION COMMISSION
172 SCRA 845
GUTIERREZ JR; April 27, 1989
NATURE
Petition for review
FACTS
- The petitioner states that she was in perfect health when employed by the Bureau of
Mines and Geo-Sciences.
- About four (4) years later, she began suffering from severe and recurrent headaches
coupled with blurring of vision.
- Petitioner was diagnosed to be suffering from brain tumor. By that time, her memory,
sense of time, vision, and reasoning power had been lost.
- A claim for disability benefits filed by her husband with the Government Service
Insurance System (GSIS) was denied.
- An appeal to the Employees Compensation Commission resulted in the Commissions
affirming of the GSIS decision.
ISSUES
1. WON brain tumor which causes are unknown but contracted during employment is
compensable under the present compensation laws
2. WON the presumption of compensability is absolutely inapplicable under the present
compensation laws when a disease is not listed as occupational disease
HELD
1. NO
- The key argument of the petitioner is based on the fact that medical science cannot, as
yet, positively identify the causes of the various types of cancer.
- The list of occupational diseases prepared by the Commission includes some cancers as
compensable. What the law requires for others is proof. Petitioner questions this listing.
- The law, as it now stands requires the claimant to prove a positive thing that the
illness was caused by employment and the risk of contracting the disease is increased by
the working conditions.
- To say that since proof is not available, therefore, the trust fund has the obligation to pay
is contrary to the legal requirement that proof must be adduced. The existence of
otherwise non-existent proof cannot be presumed.

A2010

- 342 -

Disini

2. INAPPLICABLE.
- The Workmens Compensation Act was replaced by a novel scheme under the new
Labor Code. The new law discarded, among others, the concepts of presumption of
compensability and aggravation.
- Instead of an adversarial contest by the worker or his family against the employer, we
now have a social insurance scheme where regular premiums are paid by employers to a
trust fund and claims are paid from the trust fund to those who can prove entitlement.
- It is now the trust fund and not the employer which suffers if benefits are paid to
claimants who are not entitled under the law. The employer joins the employee in trying to
have their claims approved. The employer is spared the problem of proving negative
proposition that the disease was not caused by employment.
- The employee this time, assisted by his employer, is required to prove a positive
proposition, that the risk of contracting the disease is increased by the working conditions
Compassion for the victims of diseases not covered by the law ignores the need to show a
greater concern for the trust fund to which tens of millions of workers and their families
look for compensation whenever covered accidents, diseases, and deaths occur.
Disposition AFFIRMED.

PERMANENT TOTAL
BEJERANO V EMPLOYEES' COMPENSATION
COMMISSION
205 SCRA 598
PADILLA; January 20, 1992
NATURE
Petition for review on certiorari
FACTS
- Petitioner Jose Bejerano was a cash supervisor of the Development Bank of the
Philippines, Zamboanga City Branch Office. Medical records disclose that sometime in
1985, petitioner became ill with chronic obstructive lung disease. A certain Dr. Salazar, in a
medical certificate, classified petitioner's disability as permanent total.
- Due to his disability, petitioner was forced to retire at the age of sixty-two (62) on 31
December 1985 and received the sum of P60,890.57 corresponding to five (5) years lump
sum of his annuity.
- GSIS awarded petitioner Bejerano benefits for temporary total disability and, eventually,
permanent partial disability. Not satisfied with the award, petitioner requested the GSIS for
a change of the classification of his disability benefits from permanent partial to permanent
total. Such request was denied by the GSIS. The request for reconsideration was denied
by GSIS and also by ECC because the appellant's disability could not be considered
permanent total disability in the sense that he is not completely incapable of engaging in
gainful occupation. Hence, this petition.
ISSUE
WON the petitioners disability is permanent total
HELD
YES
- It has been repeatedly held by this Court that "permanent total disability means
disablement of an employee to earn wages in the same kind of work, or work of a similar
nature that she was trained for or accustomed to perform, or any kind of work which a
person of her mentality and attainment could do." It does not mean state of absolute
helplessness, but inability to do substantially all material acts necessary to prosecution of
an occupation for remuneration or profit in substantially customary and usual manner.
Permanent total disability is the lack of ability to follow continuously some substantially
gainful occupation without serious discomfort or pain and without material injury or danger
to life.
- It is therefore clear from the aforecited rulings that the loss of one's earning capacity
determines the disability compensation one is entitled to. Thus, this Court ruled:
"In disability compensation, it is not the injury which is compensated, but rather it is the
incapacity to work resulting in the impairment of one's earning capacity (Ulibas vs.
Republic, 83 SCRA 819 Roma vs. WCC, 80 SCRA 1270)."
- A thorough examination of the records convinces us that petitioner's claim is
substantiated with enough evidence to show that his disability is permanent and total.
First, the attending physician during the petitioner's treatment diagnosed petitioner's
condition as Chronic Obstructive Lung Disease and classified petitioner s disability as
permanent and total. Second, the Medical Examiner's report on Claim for Disability
Benefits prepared by a Medical Examiner also classified petitioner's disability as total and
permanent. It is evident from the foregoing that, as per his physician's opinion, petitioner is
totally incapacitated from engaging in any gainful occupation, and that therefore his

Labor Law 1
disability is permanent and total. In earlier cases, this Court ruled that the physician's
report of sickness or accident substantiates the disability claim.
- It is also of importance to note that petitioner was forced to retire at the age of 62
because of his physical condition. This, again, is another indication that petitioner's
disability is permanent and total. As held by this Court, "the fact of an employee's disability
is placed beyond question with the approval of the employee's optional retirement, for
such is authorized only when the employee is 'physically incapable to render sound and
efficient service' . . ."
Disposition Decision of the Employees' Compensation Commission is MODIFIED and
the GSIS is hereby ordered to pay petitioner compensation benefits for permanent total
disability effective January 1986, which is the start of the period when his earning capacity
was impaired due to his disability

OCCUPATIONAL DISEASE
GOVERNMENT SERVICE INSURANCE SYSTEM V CA
[PAGE 338]

3.09 BASIS OF CLAIM


BASIS INCREASED RISKS
RIOV EMPLOYEES COMPENSATION COMMISSION
331SCRA 596
PANGANIBAN; May 9, 2000
NATURE
Petition for review the decision of the ECC
FACTS
- Virgilio T. Rio Sr., husband of herein petitioner, was employed by Allied Port Services
Inc. as stevedore since July, 1982. His duties included: (1) handling of steel cargoes; (2)
loading and unloading of silica sand; (3) handling, loading and unloading of lumber
products; (4) supervising other stevedores; and (5) performing other related work.
- On July 19, 1992, Virgilio Rio collapsed while working at the South Harbor, Manila. He
was rushed to the Philippine General Hospital (PGH) because of "melena, fever, chills and
abdominal pains 8 days [prior to confinement]. He died three days later. According to the
Medical Certificate issued by Fe B. Bais, chief of the PGH Medical Records Division, the
cause of death was "uremia [secondary] to chronic renal failure. Chronic
glomerulonephritis.
- Petitioner Beberisa Rio, his spouse, filed a claim for death benefits before the Social
Security System (SSS). However, the SSS denied the claim because according to them
the cause of death is not work related.
- On appeal, the ECC affirmed the findings of the SSS. Ruling that petitioner failed to
present relevant evidence to establish the causal connection between the deceaseds
ailment and his work as stevedore.
ISSUE
WON petitioners claim for death benefits under P.D. No. 626, as amended, shall prosper
under the increased risk theory
HELD
NO
- The beneficiaries of an employee are only entitled to death benefits if the cause of death
is a sickness listed as occupational disease by the ECC; or any other illness caused by
employment. In this case the employees ailment is not connected with his work and not
listed as occupational diseases.
Ratio The primary and antecedent causes of Virgilio Rios death are not listed as
occupational diseases. Hence, petitioner should have presented substantial evidence, or
such relevant evidence which a reasonable mind might accept as adequate to justify a
conclusion, showing that the nature of her husbands employment or working conditions
increased the risk of uremia, chronic renal failure or chronic glomerulonephritis. This the
petitioner failed to do.
- Petitioner did not adduce any proof of a reasonable connection between the work of the
deceased and the cause of his death. There was no showing that the progression of the
disease was brought about largely by the conditions in Virgilios job
- As we ruled in Sante v. Employees Compensation Commission, a claimant must submit
such proof as would constitute a reasonable basis for concluding either that the conditions
of employment of the claimant caused the ailment or that such working conditions had
aggravated the risk of contracting that ailment. What kind and quantum of evidence would

A2010

- 343 -

Disini

constitute an adequate basis for a reasonable man (not necessarily a medical scientist) to
reach one or the other conclusion, can obviously be determined only on a case-to-case
basis. That evidence must, however, be real and substantial, and not merely apparent; for
the duty to prove work-causation or work-aggravation imposed by existing law is real not
merely apparent.
- Such bare allegation does not ipso facto make Virgilios death compensable. Awards of
compensation cannot rest on speculations or presumptions. The beneficiaries must
present evidence to prove a positive proposition.

COMING AND GOING RULE


LENTEJAS VS EMPLOYEES COMPENSATION
COMMISSION
300 SCRA 484
FELICIANO; May 14, 1991
NATURE
Petition for review on certiorari
FACTS
- Victorio Lentejas, the husband of petitioner Rosa Lentejas, entered the government
service on 13 January 1968 as Maintenance "Capataz" at the Bureau of Public Highways
in Calbayog City, Western Samar. He rose from the ranks until he became a maintenance
foreman in 1978, a construction foreman in 1979, and eventually general foreman at the
City Engineer's Office of Calbayog City. His official working hours were from 8:00 A.M. to
5:00 P.M. LLphil
- On 25 July 1984, Victorio as general foreman of the City Engineer's Office, went to
Barangay Banti, Tinambacan District, Calbayog City, there to inspect work being done on
a damaged seawall protecting the shoreline against encroachment by the sea. At around
4:30 P.M., being then (according to the police report) on his way home from that place,
Victorio was suddenly attacked and stabbed with a knife by Arnulfo Luaton who inflicted
upon him multiple stab wounds on different parts of the body causing his instantaneous
death.
- Police investigation showed that the killing was brought about by a personal grudge. The
deceased Victorio and the killer Arnulfo were owners of adjoining lots situated in San
Vicente St., Tinambacan District, Calbayog City and they were in disagreement as to the
correct boundary between their respective lots. About six (6) months earlier, petitioner and
Arnulfo's father had a heated argument regarding this boundary dispute.
- Because of Victorio's death, petitioner as the surviving spouse filed with the Government
Service Insurance System (GSIS) a claim for compensation benefits under the provisions
of Presidential Decree No. 626, as amended. The GSIS denied her claim upon the ground
that the killing was not work-connected since the motive of the assailant in slaying her
husband was a personal grudge. Petitioner filed a motion for reconsideration, which
motion was denied by the GSIS. This denial was elevated by the petitioner to public
respondent Employees' Compensation Commission (ECC). The ECC, however, in its
decision dated 24 May 1989, affirmed the decision of the GSIS denying petitioner's claim
for compensation benefits on the same basis that the cause of Victorio's death was not
work-connected.
ISSUE
WON petitioners death under the aforesaid circumstances is compensable
HELD
- The Court resolved to grant due course to the Petition for Review, and to REVERSE and
SET ASIDE the decision of the Employees' Compensation Commission dated 24 May
1989. The case is REMANDED to the ECC and the GSIS for disposition conformably with
this Resolution
Ratio Under the Amended Rules on Employees' Compensation promulgated by the ECC
and relating to both government and private sector employees, more particularly Section 1
of Rule III, the requirements for compensability of an injury and the resulting death are as
follows:
"a) For the injury and the resulting death to be compensable, the injury must be the
result of an employment accident satisfying all of the following conditions:
1. the employee must have been injured at the place where his work requires him to
be;
2. the employee must have been performing his official functions:
3. if the injury is sustained elsewhere, the employee must have been executing an
order for the employer.
- It is not disputed that on 24 July 1984, Victorio Lentejas, a general foreman at the City
Engineer's Office, Calbayog City, was assigned the task of inspecting the construction or
rehabilitation work then in process on the damaged seawall along the shoreline of
Barangay Banti. While he was on his way from Barangay Banti, Arnulfo Luaton attacked

Labor Law 1
and stabbed him with deadly effect. He was dead when the police reached the scene of
the crime; the circumstances of Victorio's death were related by an eyewitness to the
police officers. Thus, there was no definite evidence to show that the deceased was
actually on his way home at the time of the attack upon him. The killing took place at 4:30
P.M., that is, during the deceased's official hours of work and hence, Victorio might well
have been on his way back to the City Engineer's Office when he was set upon and killed.
He was, in other words, on official time and in the course of performing his official
functions when he was attacked. The deceased was at a place where his work required
him to be, that is, at Barangay Banti and there is no evidence to show that the route he
took leaving the situs of the damaged seawall was not a usual or convenient route from
that place. He was not, to borrow a phrase from the common law of torts, on "a frolic of his
own."
In a line of cases, the Court has held that an injury sustained by the employee while on
his way to or from his place of work, and which is otherwise compensable, is deemed
to have arisen out of and in the course of his employment. Under the foregoing cases,
it is quite clear that although Victorio might have been on his way home from Barangay
Banti at the time he was attacked and killed by Arnulfo Luaton, that circumstance did
not by itself operate to render his death non-compensable. We note, at the same time,
that in all the cases noted above from Vda. de Torbela to Lazo, the events which
caused or precipitated injury or death did not involve the intentional inflicting of harm or
injury or death upon the employee concerned. In the instant case, however, as noted
earlier, Victorio's death was the result of a murderous assault upon him. Thus, the
precise technical issue here is whether or not the circumstance that Victorio's death
was the result of a criminal attack by another person, not a member of the staff of the
Calbayog City Engineer's Office, had the effect of rendering his death noncompensable although such death had occurred in the course of performance of official
functions. There is no question that the deceased in the instant case did not foresee,
and could not have foreseen, the attack on himself when he undertook to go to
Barangay Banti to inspect and oversee the municipal construction work then on-going
in that place. In so far as the mind and will of the victim were concerned, the homicidal
intent on the part of Arnulfo Luaton was as external and fortuitous an event as a
speeding mini-bus or a recklessly negligent jeepney driver. In other words, The Court
does not think that the third person's criminal intent should be regarded as a
supervening cause having the effect of nullifying the circumstance that, when Victorio
was attacked and killed, he was where his work required him to be and that he was
then in the course of performing his official duties.

3.10 PRESCRIPTIVE PERIOD


REMITTANCE
BENEDICTO V ABAD SANTOS
[PAGE 334]

REGISTRATION
PEOPLE V MONTEIRO
192 SCRA 549
CRUZ; December 21, 1990
NATURE
Appeal from the order of the Court of First Instance of Laguna
FACTS
- The appellant is questioning the order of the Court of First Instance of Laguna granting
the motion to quash the information filed against Salvador Monteiro alleging him to have

A2010

Disini

- 344 -

willfully, unlawfully, and feloniously failed to register to the Social Security System,
Elizabeth Collantes as member.
- The reason for this motion to quash was prescription.
- The trial court agreed with the appellee that the crime charged could no longer be
prosecuted because the offense prescribed after four years.
The trial court reckoned the four-year prescriptive period from the time the offense was
committed.
- According to the Solicitor General, the prescriptive period should have commenced from
the time when the offense was discovered.
- The appellee argues that the appellant cannot change its theory on appeal from its
original position that the offense was a continuing one. He insists that the prescriptive
period should be counted from the date the alleged violation as this was not concealed
and did not have to be discovered. Furthermore, the statements of Collantes were not
admissible evidence because they had not been formally offered. He also suggests that
the subsequent registration of Collantes with the SSS had the effect of extinguishing the
offense and that in any case the appeal would put him in double jeopardy.
ISSUE
WON the prescriptive period should be counted from the time the alleged offense was
committed
HELD
NO
- The Court affirms the conclusion in the case of Benedicto v Abad Santos, that the period
of prescription for the offense of failure to register with the SSS shall begin from the day of
discovery of the violation if this was not shown at the time of its commission.
- The court also rejects the appellees claim that his subsequent registration of Collantes
with the SSS extinguished his current liability. If subsequent compliance with the law
were sufficient to condone past violation, then the penal clause might as well be deleted
from the statute (okay mataray ang court!) The court held that such interpretation would
nullify the purpose of the law, which is precisely to protect the members of the working
class.
Disposition Challenged order is set aside and the case is remanded.

STATUTE OF LIMITATION
BUENA OBRA V SSS
401 SCRA 206
PUNO; April 9, 2003
NATURE
Appeal from the Decision of the CA as well as its subsequent Resolution denying
petitioners MFR
FACTS
- Juanito Buena Obra, husband of petitioner was employed at Jollar Industrial Sales and
Services Inc. as a dump truck driver. He suffered a heart attack while driving a dump truck
inside the work compound, and died shortly thereafter. Petitioner Maria M. Buena Obra
immediately filed her claim for death benefits under the SSS law. Petitioner was, however,
unaware of the other compensation benefits due her under PD 626, or the Law on
Employees Compensation. More than ten (10) years after the death of her husband, she
learned of the benefits under PD 626 through the television program of then broadcaster
Ted Failon who informed that one may claim for Employees Compensation Commission
(ECC) benefits if the spouse died while working for the company. Petitioner filed with the
SSS her claim for funeral benefits under PD 626.
- The SSS denied the claim for funeral benefits ruling that the cause of death was not
work-connected, absent a causal relationship between the illness and the job. Petitioner
wrote to the Executive Director of the ECC, appealing the denial of her claim. The
Commission declared that petitioners claim had prescribed. The CA dismissed petitioners
appeal, ruling that petitioner's filing of her claim for SSS benefits shortly after Juanitos
death did not suspend the running of the prescriptive period for filing EC claims.
ISSUES
1. WON petitioners claim had prescribed
2. WON the cause of death was work-related
HELD
1. NO
Ratio When petitioner filed her claim for death benefits under the SSS law, her claim for
the same benefits under the Employees Compensation Law should be considered as
filed.
Reasoning

Labor Law 1
- Under the law, the employees compensation claim shall be filed with the GSIS/SSS
within a reasonable time as provided by law. It should be noted that neither statute nor
jurisprudence has defined the limits of reasonable time. Thus, what is reasonable time
depends upon the peculiar facts and circumstances of each case. The evidence shows
that the System failed to process her compensation claim. Under the circumstances, the
petitioner cannot be made to suffer for the lapse committed by the System. It is the
avowed policy of the State to construe social legislations liberally in favor of the
beneficiaries.
2. YES
Ratio Petitioners husband was driving a dump truck within the company premises when
he suffered the heart attack. This is a clear indication that severe strain of work brought
about the acute attack that caused his death.
Reasoning
- As the official agents charged by law to implement social justice guaranteed by the
Constitution, the ECC and the SSS should adopt a liberal attitude in favor of the employee
in deciding claims for compensability especially where there is some basis in the facts for
inferring a work connection with the illness or injury. It is only this kind of interpretation that
can give meaning and substance to the compassionate spirit of the law.
Disposition petition is GRANTED. The Decision of the CA and its Resolution are hereby
SET ASIDE. The SSS is directed to pay petitioner the death/funeral benefits due her under
the existing law.

3.11 EXCLUSIVITY OF BENEFITS


EXCLUSIVITY
YSMAEL MARITIME CORP V AVELINO
397 SCRA 489
FERNAN; June 30, 1987
NATURE
Petition for certiorari
FACTS:
- Rolando Lim was a licensed second mate on a vessel owned by petitioner Ysmael
Maritime Corp. The vessel sank. Lim died.
- Lims parents filed an action against petitioner corp, alleging that the death was due to
petitioners negligence. Petitioner filed a motion to dismiss the case, alleging that [1] that
the complaint stated no cause of action; [2] that respondent-plaintiffs had received P4,160
from petitioner and had signed release papers discharging petitioner from any liability
arising from the death of their son, and [3] that most significantly, the respondents had
already been compensated by the Workmen's Compensation Commission [NCC] for the
same incident, for which reason they are now precluded from seeking other remedies
against the same employer under the Civil Code. Judge Avelino denied the motion.
ISSUE:
WON respondent judge erred in not dismissing the case
HELD:
YES. At issue is the exclusory provision of Section 5 of the Workmen's Compensation Act
reiterated in Article 173 of the Labor Code 7:
Sec. 5 Exclusive right to compensation. The rights and remedies granted by this Act to
an employee by reason of a personal injury entitling him to compensation shall exclude all
other rights and remedies accruing to the employee, his personal representatives,
dependents or nearest of kin against the employer under the Civil Code and other laws,
because of said injury,
Art. 173 Exclusive of liability. Unless other wise provided, the liability of the State
Insurance Fund under this Title shall be exclusive and in place of all other liabilities of the
employer to the employee, his dependents or anyone otherwise entitled to receive
damages on behalf of the employee or his dependents. The payment of compensation
under this Title shall bar the recovery of benefits as provided for in Section 699 of the
Revised Administrative Code, Republic Act No. 1161, as amended, Commonwealth Act
No. 186, as amended, Republic Act No. 610, as amended, Republic Act No. 4864, as
amended, and other laws whose benefits are administered by the System, during the
period of such payment for the same disability or death, and conversely.
The Court adopted the ruling in Floresca vs. Philex Mining Company. It ruled that the
action is selective and the employee or his heirs have a choice of availing themselves of
the benefits under the WCA or of suing in the regular courts under the Civil Code for
higher damages from the employer by reason of his negligence. But once the election has

A2010

- 345 -

Disini

been exercised, the employee or his heirs are no longer free to opt for the other remedy.
In other words, the employee cannot pursue both actions simultaneously.
DISPOSITIVE
Petition granted; civil case dismissed

OFFICIAL FUNCTIONS
HINOGUIN V ECC
170 SCRA 350
FELICIANO; April 17 1989
NATURE
This Petition for Review against the Decision of the Employees' Compensation
Commission ("ECC")
FACTS
- Sgt. Hinoguin and two (2) members of his Detachment, Cpl. Rogelio Clavo and Dft.
Nicomedes Alibuyog, sought permission from Captain Frankie Z. Besas, Commanding
Officer of "A" Company to go on overnight pass to Aritao, Nueva Viscaya, "to settle
important matter thereat." Captain Besas orally granted them permission to go to Aritao
and to take their issued firearms with them, considering that Aritao was regarded as "a
critical place," 2 that is, it had peace and order problems due to the presence of elements
of the New People's Army ("NPA") in or in the vicinity of Aritao.
- Hinoguin was accidentally shot by Alibuyog after mistakenly thinking that his guns safety
lever was on safety. An investigation conducted by H.Q. concluded that the shooting of
Sgt. Hinoguin was "purely accidental in nature." The recommendation written by the
Chairman who investigated suggested that all benefits due the legal dependents of the
late Sgt. Lemick Hinogiun be given."
- Sometime in March 1986, petitioner filed his claim for compensation benefits under P.D.
No. 626 (as amended), claiming that the death of his son was work-connected and
therefore compensable. This was denied 6 by the GSIS on the ground that petitioner's son
was not at his work place nor performing his duty as a soldier of the Philippine Army at the
time of his death. GSIS denied it.
ISSUE
WON the death of Sgt. Hinoguin is compensable under the applicable statute and
regulations
HELD
YES
- It will be seen that because the Amended (Implementing) Rules are intended to apply to
all kinds of employment, such rules must be read and applied with reasonable flexibility
and comprehensiveness. The concept of a "work place" referred to in Ground 1 (See Art.
167 below), cannot always be literally applied to a soldier on active duty status, as if he
were a machine operator or a worker in an assembly line in a factory or a clerk in a
particular fixed office. Obviously, a soldier must go where his company is stationed. In the
instant case, Aritao, Nueva Viscaya was not, of course, Carranglan, Nueva Ecija. Aritao
being approximately 1-1/2 hours away from the latter by public transportation. But Sgt.
Hinoguin, Cpl. Clavo and Dft. Alibuyog had permission from their Commanding Officer to
proceed to Aritao, and it appears to us that a place which soldiers have secured lawful
permission to be at cannot be very different, legally speaking, from a place where they are
required to go by their commanding officer. We note that the three (3) soldiers were on an
overnight pass which, notably, they did not utilize in full. They were not on vacation leave.
Morever, they were required or authorized to carry their firearms with which presumably
they were to defend themselves if NPA elements happened to attack them while en route
to and from Aritao or with which to attack and seek to capture such NPA elements as they
might encounter. Indeed, if the three (3) soldiers had in fact encountered NPAs while on
their way to or from Aritao and been fired upon by them and if Sgt. Hinoguin had been
killed by an NPA bullet, we do not believe that respondent GSIS would have had any
difficulty in holding the death a compensable one.
- It has already been pointed out above that the Line of Duty Board of Officers of the 14th
Infantry Battalion Headquarters had already determined that the death of Sgt. Hinoguin
had occurred "in line of duty." It may be noted in this connection that a soldier on active
duty status is really on 24 hours a day official duty status and is subject to military
discipline and military law 24 hours a day. He is subject to call and to the orders of his
superior officers at all times, 7 days a week, except, of course, when he is on vacation
leave status (which Sgt. Hinoguin was not). Thus, we think that the work-connected
character of Sgt. Hinoguin's injury and death was not effectively precluded by the simple
circumstance that he was on an overnight pass to go to the home of Dft. Alibuyog, a
soldier under his own command. Sgt. Hinoguin did not effectively cease performing
"official functions" because he was granted a pass. While going to a fellow soldier's home
for a few hours for a meal and some drinks was not a specific military duty, he was

Labor Law 1
nonetheless in the course of performance of official functions. Indeed, it appears to us that
a soldier should be presumed to be on official duty unless he is shown to have clearly and
unequivocally put aside that status or condition temporarily by, e.g., going on an approved
vacation leave. Even vacation leave may, it should be remembered, be preterminated by
superior orders.
- A soldier must also assume the risk of being accidentally fired upon by his fellow soldiers.
This is reasonably regarded as a hazard or risk inherent in his employment as a soldier.
"SECTION 1. Conditions to Entitlement.
(a) The beneficiaries of a deceased
employee shall be entitled to an income benefit if all of the following conditions are
satisfied:
(1) The employee had been duly reported to the System;
(2) He died as a result of injury or sickness; and
(3) The System has been duly notified of his death, as well as the injury or sickness
which caused his death.
- His employer shall be liable for the benefit if such death occurred before the employee is
duly reported for coverage of the System.
- Article 167 (k) of the Labor Code as amended defines a compensable "injury" quite
simply as "any harmful change in the human organism from any accident arising out of
and in the course of the employment." The Amended (Implementing) Rules have,
however, elaborated considerably on the simple and succinct statutory provision. Rule III,
Section 1 (a) reads:
"SECTION 1. Grounds. (a) For the injury and the resulting disability or death to be
compensable, the injury must be the result of an employment accident satisfying all of
the following grounds:
(1) The employee must have been injured at the place where his work requires him to
be;
(2) The employee must have been performing his official functions; and
(3) If the injury is sustained elsewhere, the employee must have been executing an
order for the employer.
Disposition Decision of the GSIS taken through its Claim Review Committee dated 20
November 1986 and the Decision dated 24 May 1988 of the Employees' Compensation
Commission in ECC Case No. 3275, are hereby REVERSED and the GSIS is hereby
DIRECTED to award all applicable benefits in respect of the death of Sgt. Lemick G.
Hinogiun, to petitioner.

3.12 BENEFIT PROTECTION


3.13 DISPUTE SETTLEMENT

A2010

- 346 -

Disini

S-ar putea să vă placă și